You are on page 1of 349

DERMATOLOGA

Preparacin Examen de Seleccin 05/06 1 Vuelta Seguimiento a distancia

1. Si en una biopsia cutnea se aprecia una epidermis con Con respecto a su enfermedad, seale la FALSA:
clulas queratinizadas en estratos inferiores, acompaada
de acantlisis, usted sospecha: 1) Est ocasionado por una espiroqueta.
2) En el 10% de los pacientes aparece una artritis crnica de
1) Pnfigo. aparicin tarda.
2) Liquen plano. 3) Sin tratamiento, las lesiones cutneas desaparecen, aun-
3) Eczema numular. que pueden presentar recadas.
4) Enfermedad de Darier. 4) El diagnstico se realiza mediante serologas.
5) Psoriasis. 5) La aparicin de neuropatas perifricas son excepcionales
y tardas.
2. Qu vehculo empleara para el tratamiento de lesiones de
tipo descamativo en el cuero cabelludo?: 7. Acude a su consulta un nio de 7 aos de edad con una lesin
en la frente papulonodular, firme, infiltrada, de 1 cm de
1) Pasta al agua. dimetro y 5 semanas de evolucin, cubierta de una costra
2) Crema. superficial, asintomtica, que al ser retirada deja ver una
3) Pomada. lcera con bordes sobreelevados e indurados. En el estudio
4) Ungento. histolgico se aprecia una lcera epidrmica con un infiltra-
5) Solucin. do inflamatorio compuesto de linfocitos, histiocitos y neu-
trfilos, junto con presencia de grupos de 2-4 partculas azl-
3. El signo de Darier en una mastocitosis consiste en: grisceas en el citoplasma de los macrfagos drmicos. Cul
es el diagnstico ms probable?:
1) Insensibilidad de las lesiones frente al calor.
2) Aparicin de fenmenos congestivos generalizados. 1) Pioderma gangrenoso.
3) Picor y urticacin de los elementos a causa del frotamiento. 2) ntrax.
4) Aumento de la sensibilidad de las lesiones tras el rascado. 3) Imptigo contagioso.
5) Desaparicin temporal de los elementos explorados. 4) Botn de oriente.
5) Lupus vulgar.
4. Un varn de 16 aos presenta un intenso acn conglobata
desde hace 2 aos. Su dermatlogo le ha recetado isotretino- 8. Varn de 17 aos de edad acude a la urgencia por presentar
na oral en dosis de 0,5 mg/kg/da durante 6 meses. Seale el fiebre de 38,5 C y cefalea intensa desde hace 4 das. Refiere
efecto secundario ms grave de este tratamiento: presentar una lesin costrosa negruzca, rodeada de un halo
rojizo en un tobillo desde hace 8 das, en que se fue de
1) Teratogenia. acampada con unos amigos. A la exploracin, usted aprecia
2) Xerosis. lesiones maculosas eritematosas de pequeo tamao, salpi-
3) Hipercolesterolemia. cadas por tronco y extremidades en escasa cantidad. Seale
4) Calcificaciones osteoligamentosas. la FALSA:
5) Hepatotoxicidad.
1) Probablemente encontremos lesiones cutneas palmo-
5. Varn de 30 aos consulta por presentar desde hace 3 plantares.
semanas una erupcin en tronco y extremidades de forma 2) Se puede observar alteracin de las pruebas funcionales
progresiva muy pruriginosa, ms intensamente por las no- hepticas.
ches. A la exploracin presenta lesiones eritematosas leve- 3) La lesin costrosa negruzca aparece en todos los pacien-
mente descamativas, de pequeo tamao, con excoriaciones tes.
en tronco y races de extremidades. En las caras laterales de 4) El diagnstico se establece mediante serologas.
los dedos se aprecian lesiones vesiculosas sueltas, de peque- 5) El tratamiento de eleccin son las tetraciclinas.
Preguntas TEST

o tamao y contenido claro, y alguna lesin lineal de 2-3 mm


de longitud. Tambin se aprecian ndulos rojizos en escroto 9. Varn de 7 aos de edad, con placa de alopecia en cuero
y pene. Seale la FALSA: cabelludo, de 3 cm, eritema, descamacin blanquecina y
pelos rotos a distintos niveles, leve prurito, de 15 das de
1) Probablemente existan personas convivientes tambin evolucin. La causa ms probable ser:
afectadas.
2) El diagnstico se confirma con el raspado de las lesiones 1) Liquen plano pilar.
lineales. 2) Lupus discoide.
3) Las lesiones nodulares son las que ms tardan en resolver- 3) Tia tonsurante.
se. 4) Alopecia areata.
4) El tratamiento con permetrina 5% es muy til. 5) Tricotilomana.
5) En las embarazadas, el tratamiento de eleccin es el
lindano. 10. Ante un paciente que acude a consulta por presentar, desde
hace una semana, una placa en la pierna derecha, de creci-
6. Mujer de 35 aos, propietaria de una peluquera canina, miento ascendente progresivo, caliente, brillante, rojiza,
consulta por la presencia de una ppula eritematosa que se edematosa, indurada, y con un borde elevado, bien diferen-
sigui de un eritema de crecimiento perifrico y migratorio. ciada de la piel adyacente, dolorosa y acompaada de fiebre
y leucocitosis, seale la FALSA:

M exico A rgentina
C hile U ruguay
CTO Medicina C/ Nez de Balboa, 115 28006 MADRID (Espaa) Tfno.: (91) 782 43 32 / Fax: (91) 782 43 27
E-mail: secretaria@ctomedicina.com; iberocto@ctomedicina.com WEB: www.ctomedicina.com; www.iberocto.com
DM Pg. 1
DERMATOLOGA
Seguimiento a distancia Preparacin Examen de Seleccin 05/06 1 Vuelta
1) El diagnstico se realiza siempre mediante cultivo de una 3) Granuloma anular.
biopsia profunda. 4) Lepra tuberculoide.
2) Esta misma lesin, si aparece en la cara, debe diferenciarse 5) Morfea en placas.
en ocasiones de un herpes zoster.
3) Para diferenciarlo de una celulitis aguda, es til el borde 15. Sobre el sarcoma de Kaposi en el VIH, seale la respuesta
mal definido que deja reas adyacentes sanas en esta FALSA, de entre las siguientes:
ltima.
4) Ocasionalmente puede confundirse en la cara tambin 1) Existen datos epidemiolgicos que hablan a favor de un
con un edema angioneurtico. vector infeccioso como etiologa.
5) De no tratarse, pueden aparecer complicaciones no 2) La piel suele ser la localizacin de presentacin.
supurativas posteriormente. 3) El sarcoma de Kaposi no suele ser causa de muerte.
4) Una mnima parte de los pacientes llega a tener afectacin
11. Mujer de 27 aos, que consulta por presentar desde hace ms gastrointestinal.
de 2 aos brotes repetidos de lesiones nodulares, dolorosas 5) El sarcoma de Kaposi en el VIH tiene el mismo patrn
y abscesificadas, que terminan drenando material purulento histolgico que el africano o el de los trasplantados.
en ambas ingles. Ha sido tratada con diversos ciclos de
antibiticos va oral y drenajes quirrgicos, pero slo ha 16. Mujer de 74 aos consulta por presentar desde hace 8 aos
conseguido mejoras parciales, sin verse completamente una placa bien delimitada en abdomen, de crecimiento
libre de lesiones. Qu tratamiento cree que podra ofrecerle excntrico lentamente progresivo, que mide 4 cm de dime-
mejores expectativas a esta paciente?: tro, y en cuyo interior se aprecia descamacin, eritema y
lesiones costrosas sin infiltracin. La biopsia demuestra
1) Rifampicina va oral, durante 3 meses. prdida de polaridad atpica y mitosis en los queratinocitos,
2) Antispticos orales potentes, asociados a esteroides intra- ocupando toda la epidermis, pero respetando la integridad
lesionales en las lesiones. de la membrana basal. Cul es su diagnstico?:
3) Ciclo de esteroides orales, durante 2 meses.
4) Cefuroxima oral en cada brote con drenaje quirrgico, si 1) Psoriasis en placas.
as lo precisara. 2) Enfermedad de Bowen.
5) Isotretinona oral, durante 6 meses. 3) Enfermedad de Paget.
4) Herpes circinado.
12. Un varn de 23 aos presenta fiebre y un rash compuesto por 5) Queratosis seborreica.
vesculas, pstulas, costras y ppulas eritematosas, de 6 das
de evolucin, diseminadas por la piel y muy pruriginosas, sin 17. Mujer de 77 aos, con hiperglucemias mantenidas desde
otra sintomatologa. Cul sera su actitud?: hace 2 meses, presenta una placa blanco-amarillenta, escle-
rosa, con bordes papulosos de brillo blanquecino, localizada
1) No tratar. en la mejilla derecha. La paciente refiere aparicin frecuente
2) Dar corticoides orales en dosis bajas. de costras sobre la lesin. Seale la actitud correcta:
3) Administrar antibiticos orales.
4) Tratamiento tpico astringente, antihistamnicos orales y 1) Extirpacin quirrgica de la lesin.
paracetamol. 2) Realizacin de TC abdominal.
5) Antivricos orales y antitrmicos. 3) Radioterapia.
4) Infiltracin con interfern alfa.
13. Nia de 6 aos de edad, con un estado general bueno, 5) Gluconato de antimonio intralesional.
presenta lesiones papulosas, eritematosas, algunas cubiertas
por una vescula de contenido claro, suelto y distribuidas en 18. Respecto al carcinoma basocelular, sealar la FALSA:
brazos, piernas, glteos y mejillas, de 8 das de evolucin. Una

Preguntas TEST
analtica demuestra elevacin de las enzimas hepticas. Cul 1) Es el tumor maligno cutneo ms frecuente.
es el diagnstico ms probable?: 2) Cuando aparecen en labio, suelen asentarse sobre una
leucoplasia.
1) Acrodermatitis papulosa infantil. 3) Est constituido por queratinocitos de la capa germinativa
2) Dermatitis herpetiforme. dispuestos en empalizada.
3) Picaduras mltiples de insecto. 4) En su etiologa se ha implicado la exposicin solar prolon-
4) Varicela. gada.
5) Enfermedad pie-mano-boca. 5) Los pacientes que han padecido un epitelioma basocelu-
lar tienen ms riesgo de presentar otros en distintas loca-
14. Una mujer de 35 aos presenta una placa nica seca, de lizaciones.
textura irregular, hipopigmentada, con disminucin de la
sensibilidad, de 3 x 3 cm, en la espalda. Biopsia: granulomas 19. Nia de 2 aos de edad que presenta desde el nacimiento
de clulas epitelioides y clulas gigantes tipo Langhans, que placa amarillenta y alopcica en el cuero cabelludo. Con
tambin afectan a estructuras nerviosas. El diagnstico ser: respecto a esta enfermedad, seale la correcta:

1) Sarcoidosis. 1) Se denomina nevus sebceo de Pringle.


2) Lupus vulgar. 2) La actitud correcta es la observacin de la lesin a lo largo
de la vida.

M exico A rgentina
Pg. 2 DM C hile U ruguay
CTO Medicina C/ Nez de Balboa, 115 28006 MADRID (Espaa) Tfno.: (91) 782 43 32 / Fax: (91) 782 43 27
E-mail: secretaria@ctomedicina.com; iberocto@ctomedicina.com WEB: www.ctomedicina.com; www.iberocto.com
DERMATOLOGA
Preparacin Examen de Seleccin 05/06 1 Vuelta Seguimiento a distancia

3) Son ms frecuentes en el tronco, aunque le sigue en 1) Extirpacin quirrgica, en la medida de lo posible.


frecuencia el cuero cabelludo. 2) Observacin a partir de la pubertad.
4) Se deben extirpar antes de la pubertad. 3) Tratamiento con lser de la lesin, previa toma histolgica
5) Es un cmulo de histiocitos con alto riesgo de degenera- que confirme su benignidad.
cin maligna. 4) Biopsias semestrales de la lesin.
5) Electrocoagulacin.
20. Qu tumor desarrollar con ms frecuencia un paciente con
mltiples lesiones de morfologa similar a las verrugas pla- 25. En el melanoma maligno en estadio I, sealar cul es el factor
nas?: pronstico ms importante:

1) Se trata de una epidermodisplasia verruciforme y desarro- 1) Fototipo cutneo.


llar epiteliomas basocelulares. 2) Sexo.
2) Desarrollar carcinomas epidermoides. 3) Localizacin (reas BANS).
3) Se trata de la enfermedad de Lewandowsky, y desarrollar 4) Invasin en profundidad (grosor en mm).
queratosis seborreica. 5) Tipo clnico.
4) Presentar carcinoma de crvix dorsal, asociado con la
infeccin por VPH. 26. Mujer de 32 aos de edad, que consulta a su mdico por
5) Angiofibromas. presentar lesin pigmentada asintomtica en la pierna dere-
cha, cara posterior. Segn la paciente, la lesin ha crecido
21. Varn de 50 aos, que consult por una lesin localizada en progresivamente en los ltimos 3 aos. En la histologa se
la espalda de 3 aos de evolucin. La lesin era de aspecto aprecian melanocitos atpicos en dermis con grandes n-
verrucoso-papilomatoso, de color marrn oscuro. A veces cleos y mitosis. Seale la FALSA, con respecto a esta enferme-
produca prurito, y haba crecido desde su aparicin. La dad:
biopsia mostr una proliferacin exoftica de clulas epidr-
micas de aspecto basalioide y globos crneos. Qu diagns- 1) Estas lesiones suelen ser inicialmente planas.
tico sospechara?: 2) El color suele ser irregular.
3) La fase radial puede durar hasta dcadas.
1) Nevus adquirido. 4) En ocasiones se aprecia un infiltrado inflamatorio perile-
2) Epitelioma basocelular pigmentado. sional.
3) Mancha caf con leche. 5) La extirpacin precoz de la lesin es el mejor tratamiento.
4) Melanoma.
5) Queratosis seborreica. 27. Seale cundo est indicado el tratamiento con interfern
sistmico en un melanoma:
22. Varn de 27 aos, minero de profesin, presenta desde hace
aos mltiples lesiones papulosas, perladas, alguna erosio- 1) En los melanomas de ms de 4 mm con enfermedad
nada y otras pigmentadas, localizadas en cara, tronco y localizada cutnea.
extremidades. Seale la respuesta FALSA: 2) En los melanomas nodulares.
3) En melanomas con metstasis pulmonares solitarias.
1) Presentar disqueratosis en palmas y plantas. 4) En los melanomas en nivel III de Clark y superiores.
2) El hipertelorismo es llamativo. 5) En los melanomas de las extremidades.
3) La afectacin ocular en forma de fotofobia es casi cons-
tante. 28. Con respecto a la micosis fungoide, NO es cierto que:
4) Se hereda de manera autosmico dominante.
5) Es frecuente la presencia de quistes seos mandibulares en 1) Es un proceso linfoproliferativo maligno T inductor/cola-
este cuadro. borador.
Preguntas TEST

2) Se manifiesta inicialmente en la piel.


23. Varn de 65 aos, agricultor y fumador habitual, presenta 3) Las lesiones en la fase premictica pueden ser confundi-
desde hace 2-3 aos lesiones maculosas blanquecinas que das con mltiples dermatosis.
ocupan prcticamente la totalidad de la semimucosa del labio 4) En ocasiones puede comenzar en clnica como una
inferior, con atrofia del epitelio. No le produce ninguna eritrodermia.
molestia significativa, salvo sensacin de tirantez. Qu po- 5) Las manifestaciones ganglionares y viscerales suelen apa-
sibilidad diagnstica le parece la ms adecuada?: recer al poco tiempo de evolucin.

1) Liquen plano oral. 29. Qu tratamiento instaurara en una micosis fungoide con
2) Queilitis actnica. lesiones eccematosas por el tronco?:
3) Aftas orales.
4) Candidiasis oral. 1) Corticoides tpicos.
5) Exantema fijo. 2) UVB.
3) Radioterapia.
24. Seale la actitud correcta ante una nia de 6 aos de edad, 4) Acitretina.
con una lesin melanoctica presente desde el nacimiento, 5) Interfern alfa.
que ocupa la cara externa del muslo izquierdo:

M exico A rgentina
C hile U ruguay
CTO Medicina C/ Nez de Balboa, 115 28006 MADRID (Espaa) Tfno.: (91) 782 43 32 / Fax: (91) 782 43 27
E-mail: secretaria@ctomedicina.com; iberocto@ctomedicina.com WEB: www.ctomedicina.com; www.iberocto.com
DM Pg. 3
DERMATOLOGA
Seguimiento a distancia Preparacin Examen de Seleccin 05/06 1 Vuelta
30. Varn de 58 aos, con astenia y anorexia de 3 meses de 3) Microabscesos de Munro-Sabouraud.
evolucin. Desde hace 2 meses presenta eritrodermia, as 4) Pstulas de Von Zumbusch.
como adenopatas generalizadas. En sangre perifrica se 5) Pstulas de Kogoj-Lapiere.
detectan un 20% de clulas linfoides atpicas. En relacin con
este cuadro, es cierto que: 36. Varn de 46 aos, con hepatopata enlica conocida e
infeccin crnica por VHC, presenta una psoriasis generali-
1) El prurito no suele ser un sntoma. zada, intensa, que no responde a la medicacin tpica
2) Es de larga evolucin. habitual ni a la fototerapia. Cul sera su siguiente actitud
3) La hepatoesplenomegalia suele ser evidente. teraputica?:
4) El uso de retinoides con interfern se ha empleado como
tratamiento de las fases premicticas de la enfermedad. 1) Metotrexate.
5) En esta fase de la enfermedad no se objetivan linfocitos en 2) Corticoides orales.
dermis. 3) Acitretina.
4) Ciclosporina.
31. Mujer de 50 aos de edad, que consulta por presentar 5) Calcipotriol.
lesiones papulopustulosas en regin facial, as como un
eritema persistente en ambas mejillas. Al parecer, empeora en 37. Mujer de 31 aos, con psoriasis en placas extensas, afectando
verano. Seale la respuesta FALSA en relacin con esta a ms de un 40% de la superficie corporal total. Ni el
enfermedad: tratamiento con medicacin tpica ni la PUVA han consegui-
do controlar la enfermedad. Seale la FALSA:
1) Las tetraciclinas orales son muy efectivas.
2) En los casos rebeldes, pueden usarse retinoides orales. 1) El acitretino no debe emplearse sin acompaarse simul-
3) Es frecuente la afectacin de la mucosa conjuntival. tneamente de anticoncepcin, debido a su prolongada
4) Puede desencadenarse con el caf. teratogenicidad.
5) La afectacin renal puede verse en periodos tardos. 2) En estos casos, la isotretinona oral es igual de efectiva que
el acitretino.
32. Mujer de 50 aos de edad, con anorexia de 2 meses de 3) El metotrexate puede ser una buena opcin teraputica,
evolucin. Desde hace 3 semanas, refiere lesin pruriginosa si su funcin heptica es normal.
en una de las areolas mamarias. En la exploracin se observa 4) La ciclosporina ser el frmaco que ms rpidamente
una placa eccematosa bien delimitada. Seale la FALSA: consiga el control del brote.
5) El metotrexate obliga a mantener los anticonceptivos hasta
1) En este cuadro, las clulas de Paget se hallan desprovistas 12 semanas despus de terminar el tratamiento.
de desmosomas.
2) Las clulas de Paget contienen mucina y antgeno carci- 38. Cul de las siguientes patologas tiene una mala respuesta a
noembrionario. los corticoides?:
3) La lesin termina siendo bilateral.
4) A menudo se palpan ndulos en la mama afecta. 1) Psoriasis.
5) Suele aparecer en mujeres de ms de 40 aos. 2) Liquen plano.
3) Pnfigo.
33. Las opacidades corneales puntiformes son caractersticas de 4) Pitiriasis rubra pilaris.
una de las siguientes ictiosis: 5) Eczema seborreico.

1) Ictiosis vulgar. 39. Mujer de 30 aos de edad consulta por presentar desde hace
2) Hiperqueratosis epidermoltica. 10 das una erupcin eritematoescamosa, fundamentalmen-
3) Ictiosis lamelar. te en tronco, asintomtica. A la exploracin, existe en la

Preguntas TEST
4) Ictiosis adquirida. espalda una distribucin de las lesiones en rbol de navidad,
5) Ictiosis X. siendo una de ellas de mayor tamao que el resto. Seale la
FALSA:
34. Ante una nia con mltiples placas queratsicas, de pequeo
tamao y de reciente aparicin, que presentan una descama- 1) Las recurrencias de este cuadro son raras.
cin nacarada con el rascado, seale la respuesta correcta: 2) Se cree que est producido por un virus.
3) El tratamiento de eleccin es la prednisona oral.
1) Probablemente tenga afectacin articular. 4) Las lesiones pueden salir durante 6-8 semanas.
2) Las lesiones en mucosas son frecuentes. 5) Las lesiones curarn sin dejar cicatriz ni alteraciones de la
3) No suelen existir factores desencadenantes. pigmentacin.
4) Tiene muy buen pronstico.
5) Debe tratarse con teraputica oral. 40. Varn de 14 aos de edad, con lesiones papulosas, hiperque-
ratsicas, de localizacin predominantemente folicular, de
35. La presencia en la psoriasis de pstulas subcrneas se deno- varios meses de evolucin, distribuidas de forma simtrica
mina: por cara anterior de tronco y cuello, con lesiones en V en el
borde libre ungueal. Empeora mucho en verano. Su padre
1) Microabscesos de Pautrier. padece la misma enfermedad. Qu entidad sospecha us-
2) Pstulas de Auspitz. ted?:

M exico A rgentina
Pg. 4 DM C hile U ruguay
CTO Medicina C/ Nez de Balboa, 115 28006 MADRID (Espaa) Tfno.: (91) 782 43 32 / Fax: (91) 782 43 27
E-mail: secretaria@ctomedicina.com; iberocto@ctomedicina.com WEB: www.ctomedicina.com; www.iberocto.com
DERMATOLOGA
Preparacin Examen de Seleccin 05/06 1 Vuelta Seguimiento a distancia

1) Pitiriasis rubra pilar. 45. Paciente con lesiones nodulares, subcutneas, dolorosas,
2) Dermatitis seborreica. presentando alguna de ellas licuefaccin con eliminacin de
3) Enfermedad de Hailey-Hailey. material, distribuidas por tronco y extremidades, y que
4) Enfermedad de Darier. muestran en el estudio histolgico clulas fantasmas en el
5) Enfermedad de Grover. tejido adiposo. Usted sospechar que padece:

41. Varn de 52 aos, con fiebre, mal estado general y lesiones 1) Sndrome de Weber-Christian.
ampollosas, flccidas, que se distribuyen por toda la super- 2) Adenocarcinoma de pncreas.
ficie corporal, dando lugar a grandes erosiones. Adems se 3) Tuberculosis.
aprecian importantes erosiones en mucosa oral, genital y 4) Lupus eritematoso sistmico.
conjuntival. El tratamiento ms indicado ser: 5) Forunculosis miliar.

1) Acitretina. 46. Mujer de 19 aos, sin antecedentes personales de inters,


2) Azatioprina. que toma anticonceptivos desde hace 2 aos. Consulta por
3) Corticoides tpicos. presentar fiebre, artralgias en rodillas y tobillos de una
4) Fluconazol. semana de evolucin, junto con edema de tobillos y lesiones
5) Sueroterapia y antibioterapia profilctica de amplio es- nodulares dolorosas en la cara anterior de ambas piernas.
pectro. Cul sera la actitud ms adecuada?:

42. Varn de 74 aos, que consulta por presentar en los ltimos 1) Biopsia de una lesin nodular, retirar anticonceptivos y
2 meses lesiones eritematoedematosas, con aparicin de reposo.
ampollas tensas en tronco y extremidades muy pruriginosas, 2) Reposo y esteroides orales.
que no han desaparecido tras tratamiento con esteroides 3) Antiinflamatorios no esteroideos.
tpicos y antihistamnicos orales. La biopsia de una lesin 4) Retirada de anticonceptivos y dar esteroides orales.
demuestra una ampolla subepidrmica, con infiltrado infla- 5) Administrar yoduro potsico y reposo.
matorio donde existen numerosos eosinfilos. Cul sera lo
ms indicado para confirmar su diagnstico?: 47. Mujer de 67 aos, hipertensa y con bronquitis crnica.
Presenta desde hace 4 das lesiones asintomticas, bilatera-
1) Hacer una IFI en suero. les, en ambos miembros inferiores. A la exploracin se
2) La respuesta positiva a tratamiento esteroideo es diagns- aprecian lesiones maculo-papulosas, eritematosas, que no
tica. desaparecen con la digitopresin. Seale la correcta respec-
3) Realizar una IFD en piel perilesional. to a esta enfermedad:
4) Realizar un TC para descartar neoplasia subyacente.
5) La respuesta positiva a tratamiento con sulfonas es diagns- 1) Se produce por un depsito de inmunocomplejos pre-
tica. capilar.
2) Las lesiones pueden llegar a hacerse necrticas.
43. Varn de 5 aos de edad, que presenta desde hace meses 3) El tratamiento de eleccin de este cuadro se hace con
lesiones papulosas y ampollosas con intenso prurito en ciclosporina.
tronco y extremidades. Se le realiza una biopsia que muestra 4) La imagen histolgica caracterstica es la linfocitosis peri-
un depsito de IgA lineal a lo largo de la membrana basal. vascular.
Seale la opcin INCORRECTA: 5) Es excepcional la etiologa vrica de este cuadro.

1) Responde a sulfonas. 48. Con respecto a la pregunta anterior, cul de los siguientes
2) Debe establecerse el diagnstico diferencial con la enfer- tratamientos NO empleara nunca?:
medad de Duhring-Brocq.
Preguntas TEST

3) Los brotes aparecen de manera brusca. 1) Tratamiento del factor desencadenante en caso de iden-
4) En el 90% de los casos presentan cambios histolgicos tificarse.
similares a la enfermedad celaca. 2) AINES.
5) Para el diagnstico es esencial la inmunofluorescencia 3) Corticoides orales.
directa. 4) Inmunosupresores.
5) PUVA.
44. Mujer de 26 aos en su primera gestacin, de 24 semanas.
Comienza con placas urticarianas muy pruriginosas, sobre 49. Nio de 8 aos de edad, con dolor articular y abdominal,
las que brotan lesiones vesiculosas de contenido claro. lesiones purpricas en extremidades inferiores, hematuria y
Comenzaron en el abdomen y se extendieron al resto del proteinuria. Seale la correcta con respecto a la enfermedad
tronco. Cmo tratara a esta paciente?: que sospecha:

1) Dieta sin gluten. 1) Es una vasculitis linfocitaria.


2) Sulfona. 2) Cursa con depsito de IgA en vasos drmicos.
3) Prednisona en dosis bajas. 3) Es ms frecuente en mujeres.
4) No requiere tratamiento. 4) En el 90%, se asocia a alteraciones neurolgicas.
5) Azatioprina. 5) Suele comenzar en las primeras 48 horas desde el inicio
de una infeccin estreptoccica de vas respiratorias altas.

M exico A rgentina
C hile U ruguay
CTO Medicina C/ Nez de Balboa, 115 28006 MADRID (Espaa) Tfno.: (91) 782 43 32 / Fax: (91) 782 43 27
E-mail: secretaria@ctomedicina.com; iberocto@ctomedicina.com WEB: www.ctomedicina.com; www.iberocto.com
DM Pg. 5
DERMATOLOGA
Seguimiento a distancia Preparacin Examen de Seleccin 05/06 1 Vuelta
50. Mujer de 27 aos, que presenta en 1/3 posterior de mucosa ulcerada, con crecimiento perifrico y localizada en pierna
geniana un fino reticulado blanquecino, que le ocasiona derecha. Seale el tratamiento correcto de su enfermedad
molestias espordicas. Adems, presenta alguna ppula vio- cutnea:
lcea, pruriginosa en superficies de flexin de extremidades.
Seale la FALSA en relacin a su patologa: 1) Ceftacidina.
2) Cloroquina.
1) La afectacin mucosa es frecuente. 3) Desbridamiento quirrgico.
2) Pueden afectarse en un 10% de los casos las uas. 4) Corticoides.
3) Se trata con corticoides. 5) Acitretina.
4) Los retinoides no se muestran eficaces en este proceso.
5) Debe solicitarse una serologa heptica. 56. Varn de 48 aos, con presencia de placas rosadas, infiltra-
das con dilatacin de orificios foliculares en la cara anterior
51. Qu tratamiento empleara usted ante una mujer de 18 aos de ambas piernas. La biopsia de una de ellas demuestra el
con acn papulopustuloso moderado, leve alopecia difusa e depsito de mucina en grandes cantidades en la dermis.
hirsutismo facial moderado?: Seale la FALSA:

1) Isotretinona oral + anticonceptivos orales. 1) Probablemente tenga una hiperfuncin tiroidea.


2) Minociclina oral. 2) La correccin del trastorno de base no influye en la
3) Adapaleno + eritromicina tpicos. evolucin de las lesiones cutneas.
4) Acetato de ciproterona + anticonceptivos orales. 3) El tratamiento con corticoides potentes es bastante satis-
5) Doxiciclina oral + anticonceptivos orales. factorio.
4) Las lesiones pueden adoptar aspecto elefantisico en
52. Seale la FALSA sobre el tratamiento con finasteride de la ocasiones.
alopecia andrognica masculina: 5) Se debe descartar tambin afectacin ocular.

1) Se emplea en dosis de 1 mg/da. 57. Mujer de 53 aos, bebedora en exceso, consulta por enro-
2) Los efectos secundarios ms frecuentes se refieren a la jecimiento de cara y cuello en el ltimo mes, cada vez con
esfera sexual. mayor intensidad. En las dos ltimas semanas han brotado
3) Acta inhibiendo la 5-alfa-reductasa a nivel folicular. ampollas de contenido claro en dorso de antebrazos y
4) Se logra un recrecimiento capilar evidente hasta en el 90% manos. En la ltima semana se ha aadido un incremento en
de los casos. el nmero de deposiciones, pero sin productos patolgicos.
5) Una vez suspendido el tratamiento, se pierde el beneficio Cul es su primera sospecha diagnstica?:
transcurridos unos meses.
1) Dficit de vitamina C.
53. Varn de 27 aos, opositor, que desde hace 30 das presenta 2) Dficit de cido nicotnico.
5 placas alopcicas en cuero cabelludo, sin otras alteraciones 3) Dficit de zinc.
cutneas. Seale la correcta, respecto a esta enfermedad: 4) Glucagonoma.
5) Dficit de vitamina B6.
1) Este caso debe tratarse con corticoides orales.
2) El curso es progresivo e invariable. 58. Varn de 48 aos, alcohlico crnico, que presenta en dorso
3) Se asocia a la presencia de dermatitis seborreica. de manos, cara y orejas lesiones ampollosas, tensas, de
4) Existe atrofia cutnea en los mrgenes de la placa, lo que contenido seroso. En la inmunofluorescencia no se detecta
origina la presencia de pelos peldicos. depsito de anticuerpo alguno. Seala la FALSA respecto a la
5) La produccin de melanina a nivel del pelo afectado no enfermedad que padece este paciente:
se altera.

Preguntas TEST
1) Puede encontrarse melanosis e hipertricosis.
54. Varn de 43 aos de edad, que acude a consulta por fiebre, 2) Est ocasionada por un dficit de uro III descarboxilasa.
artralgias y ppulas y placas eritematosas, dolorosas indura- 3) Su orina ser de color marrn.
das distribuidas por el tronco. En el hemograma destaca 4) Es frecuente la afectacin heptica.
leucocitosis con neutrofilia. Seale la FALSA, con respecto a 5) Debe tratarse retirando el alcohol y administrando corti-
esta enfermedad: coides.

1) Es frecuente que venga precedida de una infeccin del 59. Un nio de 15 meses que presenta placas erosivas eritema-
tracto respiratorio alto. tosas, periorificiales y acras, diarrea, fragilidad ungueal y
2) En ocasiones se asocia a malignidad, fundamentalmente alopecia, debe tratarse con:
carcinomas pulmonares.
3) Los antibiticos no alteran el curso de la enfermedad. 1) Vitamina C.
4) El tratamiento de eleccin son los corticoides orales. 2) Acido nicotnico.
5) Histolgicamente aparece edema en dermis papilar y un 3) Sulfato de zinc.
intenso infiltrado en dermis de neutrfilos, sin vasculitis. 4) Esteroides orales.
5) Dieta exenta en gluten.
55. Varn de 53 aos, diagnosticado de enfermedad de Crohn
desde hace 10 aos. Presenta desde hace 2 meses una lesin 60. Sealar la correcta sobre la necrobiosis lipodica:

M exico A rgentina
Pg. 6 DM C hile U ruguay
CTO Medicina C/ Nez de Balboa, 115 28006 MADRID (Espaa) Tfno.: (91) 782 43 32 / Fax: (91) 782 43 27
E-mail: secretaria@ctomedicina.com; iberocto@ctomedicina.com WEB: www.ctomedicina.com; www.iberocto.com
DERMATOLOGA
Preparacin Examen de Seleccin 05/06 1 Vuelta Seguimiento a distancia

1) Las zonas de aparicin ms frecuentes son los brazos. 1) Predominio de lesiones de prrigo.
2) Son placas atrficas, eritematosas y amarillentas con telan- 2) Aparecer inmediatamente tras el nacimiento.
giectasias. 3) Ausencia de prurito.
3) No dejan lesiones residuales cuando curan. 4) Afectacin flexural.
4) Responden muy bien a los corticoides tpicos. 5) Respetar el tringulo nasogeniano.
5) Siguen un curso paralelo a la evolucin de la glucemia.
67. Un varn de 27 aos consulta por la aparicin de hinchazn
61. Mujer de 68 aos, con lesiones papulosas y eritematosas de y dolor en dorso de pies, que aparece entre 3-6 horas despus
localizacin periorificial y en superficies flexoras, que desca- de caminar. El dolor y la hinchazn se mantienen varias horas,
man y van creciendo por la periferia, de varias semanas de incluso en alguna ocasin hasta 1 da. Cmo lo tratara?:
evolucin, acompaadas de diarrea, anemia, malabsorcin,
prdida de peso e hiperglucemia. Cul es su diagnstico?: 1) Loratadina.
2) Hidroxicina.
1) Acantosis nigricans. 3) Corticoides orales.
2) Sndrome de Bazex. 4) Dexclorfeniramina.
3) Eritema necroltico migratorio. 5) Anti-H1 + anti-H2.
4) Acrodermatitis enteroptica.
5) Sndrome carcinoide. 68. Una de las siguientes caractersticas de la urticaria colinrgi-
ca es FALSA:
62. La lesin ms precoz de la esclerosis tuberosa es:
1) Suele comenzar entre los 20 y 30 aos.
1) Angiofibromas faciales. 2) Cursa con habones de gran tamao.
2) Adenomas sebceos. 3) Suele respetar palmas y plantas.
3) Tumores de Koenen. 4) Puede cursar con hipotensin y dolor abdominal.
4) Ndulos de Lisch. 5) El desencadenante puede ser un aumento de la tempera-
5) Mancha hipocrmica en hoja de fresno. tura corporal por la sudacin.

63. NO es cierto que, en la neurofibromatosis: 69. Paciente de 63 aos, cardipata e hipertenso, en tratamiento
desde hace 3 semanas con captopril. Desde hace 7 das
1) Las manchas caf con leche son el signo ms precoz. presenta un exantema pruriginoso por toda la superficie
2) La presencia de "pecas" axilares puede considerarse casi corporal, que en las ltimas 48 horas ha evolucionado en
patognomnico. forma de grandes erosiones y despegamientos de mucosas.
3) El tumor intracraneal solitario ms frecuente es un glioma Usted considera ms adecuado el tratamiento con:
del nervio ptico.
4) Se ha demostrado que tiene una herencia autosmica 1) Cloxacilina.
recesiva. 2) Ciclosporina.
5) Los neurofibromas pueden experimentar una transforma- 3) Hidrocortisona.
cin sarcomatosa. 4) Budesonida en fomento.
5) Acitretina.
64. Mujer de 35 aos consulta por lesiones eritematosas, vesicu-
losas, exudativas, muy pruriginosas en los lbulos de los 70. Varn de 47 aos de edad, que consulta por presentar desde
pabellones auriculares, de varios meses de evolucin. Cul hace 5 das una placa violcea, de bordes bien definidos, no
de las siguientes actitudes NO considera adecuada?: infiltrada, pruriginosa, centrada por una vescula en la cara.
Refiere la ingesta de un medicamento que no recuerda el da
1) Fomentos con sulfato de zinc. de la aparicin de la lesin. Haba presentado algn episodio
Preguntas TEST

2) Corticoides tpicos. previo. Seale la FALSA con respecto a esta patologa:


3) Batera estndar de pruebas epicutneas.
4) Seguramente deber evitar contactos con sustancias que 1) Frecuentemente se afectan las mucosas.
contengan cromo. 2) Al curar, no deja alteraciones de la pigmentacin.
5) Los antihistamnicos tpicos son poco tiles para tratar el 3) Pueden existir varias lesiones a la vez.
cuadro. 4) Al tomar de nuevo el frmaco responsable, la lesin
reaparece en la misma localizacin.
65. La forma infantil de la dermatitis atpica suele localizarse 5) Sulfamidas, barbitricos y AINEs son frmacos habitual-
tpicamente en: mente responsables.

1) Cara. 71. En el vitligo, es cierto que:


2) Porcin extensora de extremidades.
3) Tronco. 1) No existe predisposicin gentica.
4) Palmas y plantas. 2) Casi siempre se encuentran factores desencadenantes.
5) Flexuras de codos y rodillas. 3) Las lesiones del vitligo generalizado tienden a la simetra.
4) Histolgicamente existe disminucin de melanosomas.
66. Cul de las siguientes respuestas es un dato caracterstico 5) El tratamiento habitual es con corticoides orales.
del eccema atpico del lactante?:

M exico A rgentina
C hile U ruguay
CTO Medicina C/ Nez de Balboa, 115 28006 MADRID (Espaa) Tfno.: (91) 782 43 32 / Fax: (91) 782 43 27
E-mail: secretaria@ctomedicina.com; iberocto@ctomedicina.com WEB: www.ctomedicina.com; www.iberocto.com
DM Pg. 7
DERMATOLOGA
Seguimiento a distancia Preparacin Examen de Seleccin 05/06 1 Vuelta
72. Varn de 62 aos de edad, consulta por aparicin de lesiones
en tronco y extremidades que han brotado en los ltimos 2
meses, pigmentadas, sobreelevadas, de aspecto aterciopela-
do, con comedones en superficie y asintomticas. Usted
sospecha:

1) Acantosis nigricans maligna.


2) Signo de Darier.
3) Signo de Leser-Trlat.
4) Sndrome de Gryzybowsky.
5) Sndrome de Gorlin.

73. Con respecto a las siguientes dermatosis paraneoplsicas, es


cierto que:

1) El sndrome carcinoide puede cursar con lesiones seme-


jantes al escorbuto.
2) El eritema necroltico migratorio es un cuadro que se
asocia a glucagonoma.
3) La acantosis nigricans maligna respeta casi siempre las
mucosas.
4) La ictiosis adquirida es tpica de los tumores slidos.
5) La alopecia mucinosa del adulto se asocia a cncer de
estmago.

74. Sealar la afirmacin FALSA respecto a la telangiectasia


macular eruptiva perstans:

1) Son lesiones maculosas telangiectsicas.


2) Un 25% tienen afectacin sistmica.
3) Evoluciona en brotes.
4) Se presenta en adultos.
5) Presentan signo de Darier.

75. Se conoce como sndrome de Sturge-Weber:

1) Hipertrofia hemiangiectsica de un miembro.


2) Aparicin de mltiples verrugas seborreicas.
3) Angiomatosis encefalotrigeminal.
4) Malignizacin de un nevus organoide.
5) Angiocondromatosis.

Preguntas TEST

M exico A rgentina
Pg. 8 DM C hile U ruguay
CTO Medicina C/ Nez de Balboa, 115 28006 MADRID (Espaa) Tfno.: (91) 782 43 32 / Fax: (91) 782 43 27
E-mail: secretaria@ctomedicina.com; iberocto@ctomedicina.com WEB: www.ctomedicina.com; www.iberocto.com
DERMATOLOGA
Preparacin Examen de Seleccin 05/06 1 Vuelta Seguimiento a distancia
Pregunta 1.- R: 4 crnea es un proceso pasivo que sigue las leyes de difusin de Fick; es
La queratinizacin precoz y anmala que ocurre en capas profun- directamente proporcional al coeficiente de particin de la droga
das de la epidermis puede ocurrir en procesos benignos como la entre su vehculo y la crnea e inversamente proporcional al espesor
enfermedad de Darier o malignos como la enfermedad de Bowen o el de esta capa. Por tanto la liberacin del principio activo de su vehcu-
carcinoma espinocelular. En las primeras fases de la enfermedad de lo y la difusin en la capa crnea es tanto mayor cuanto menos sea la
Darier se observa prdida de adhesin de las clulas epidrmicas, con afinidad de la droga por el vehculo.
acantlisis. Adems presentan clulas en el estrato espinoso que se
denominan cuerpos redondos. Los cuerpos redondos son clulas
disqueratsicas que muestran una parcial y prematura queratiniza-
cin y se caracterizan por poseer un ncleo central oscuro y picntico
con citoplasma claro eosinfilo.
En el liquen plano se aprecia un denso infiltrado linfocitario que se
dispone en banda por la dermis superior. En la epidermis se aprecia
una acantosis irregular, en dientes de sierra, y queratinocitos degene-
rados, disqueratsicos, que se denominan cuerpos slidos o cuerpos
de Civatte.
En el pnfigo se aprecia formacin de una ampolla intraepidrmi-
ca por acantlisis. Ver figura a pie de pgina.

Pregunta 2.- R: 5
Al ser lesiones en zonas pilosas ser preferible emplear frmulas
galnicas como una solucin, locin, gel o una espuma. Cuando las
lesiones son queratsicas, descamativas, es preferible emplear vehcu-
los grasos o muy grasos como cremas, pomadas y ungentos. En los
casos de lesiones agudas, con componente exudativo, se deben em-
plear vehculos con alto contenido en agua como los fomentos, pastas
al agua y soluciones. La difusin de una sustancia a travs de la capa Pregunta 2. Tratamiento tpico en dermatologa.
Comentarios TEST

Pregunta 1. Alteraciones histolgicas cutneas.

M exico A rgentina
C hile U ruguay
CTO Medicina C/ Nez de Balboa, 115 28006 MADRID (Espaa) Tfno.: (91) 782 43 32 / Fax: (91) 782 43 27
E-mail: secretaria@ctomedicina.com; iberocto@ctomedicina.com WEB: www.ctomedicina.com; www.iberocto.com
DM Pg. 1
DERMATOLOGA
Seguimiento a distancia Preparacin Examen de Seleccin 05/06 1 Vuelta
Pregunta 3.- R: 3 teratogenia, que hace que sea imprescindible la anticoncepcin has-
El signo de Darier consiste en la aparicin de habones sobre una ta despus de 1 mes en el caso de la isotretinona, y hasta dos aos
lesin, cuando sta es traumatizada por friccin o presin. La presen- despus en el caso de la acitretina. Otro efecto secundario importante
cia de un signo de Darier es patognomnico de mastocitosis. Las es la hepatotoxicidad, siendo el ms grave en los varones. La aparicin
mastocitosis son proliferaciones benignas de mastocitos. Estos pueden de xerosis cutnea y de queilitis son los efectos secundarios ms fre-
proliferar por todos los rganos pero la mayora de las ocasiones lo cuentes (95% de los casos).
hacen por la piel, en especial en dermis. Ah forman unos acmulos En un 12% de los casos se producen modificaciones seas o
llamados mastocitomas. Estos mastocitomas le otorgan a la piel una ligamentosas, con presencia de calcificaciones. Por este motivo los
coloracin griscea justo en la zona donde se encuentran. Cuando retinoides estn contraindicados en la infancia. Adems no deben
rascamos estas mculas aparece un habn, ya que los mastocitos asociarse tratamientos con tetraciclinas por el peligro de pseudotu-
subyacentes degranulan la histamina. mor cerebral.
De ah que la forma clnica ms frecuente de mastocitosis se deno- Los retinoides tpicos tienen efectos secundarios leves, y por lo
mina urticaria pigmentosa. No se debe confundir un signo de Darier, general no requieren cuidados especiales.
con un dermografismo. En el dermografismo los habones aparecen
en cualquier zona de la piel sobre la que aplicamos un traumatismo, Pregunta 5.- R: 5
mientras que el signo de Darier slo se presenta cuando esa presin la En la anamnesis destaca que las lesiones son pruriginosas y que el
ejercemos sobre una zona de piel con lesin, es decir sobre un prurito es de predominio nocturno. Adems, la localizacin de las
mastocitoma. mismas es en lugares caractersticos como las caras laterales de los
dedos, y con una morfologa papulosa y en ocasiones lineales. Todo
Pregunta 4.- R: 5 esto debe hacernos pensar que el paciente presenta una escabiosis.
Los retinoides son derivados de la vitamina A y se emplean en Las escabiosis se caracterizan por prurito intenso de predominio noc-
muchas patologas dermatolgicas. Fundamentalmente empleamos turno y prurito familiar, ya que la sarna es extremadamente contagio-
la isotretinona y la acitretina. Los retinoides ejercen una variedad de sa, cuando existe un contacto personal. El diagnstico de presuncin
efectos biolgicos, como disminuir la capacidad proliferativa celular, nos lo da por tanto la historia clnica y la exploracin fsica, aunque la
regular la diferenciacin celular tanto a tejidos adultos como embrio- certeza slo la tendremos cuando observemos el caro en la piel. Esto
narios, son capaces de evitar o retardar la carcinognesis y son inhibi- se logra con el raspado de las lesiones por lo que la respuesta 2 es
dores de la queratinizacin. La isotretinona se emplea sobre todo en correcta. Ver figura a pie de pgina.
el acn severo y la acitretina o etretinato en las formas graves de psoriasis. La aparicin de las lesiones nodulares se deben a una hipersensibi-
Los efectos secundarios son muy parecidos en ambos frmacos con lidad del caro. A este tipo de sarna se le denomina sarna nodular.
alguna diferencia. El efecto ms grave es el que concierne a la Para la desaparicin de las lesiones nodulares no slo es necesario

Comentarios TEST

Pregunta 5. Sarna.

M exico A rgentina
Pg. 2 DM C hile U ruguay
CTO Medicina C/ Nez de Balboa, 115 28006 MADRID (Espaa) Tfno.: (91) 782 43 32 / Fax: (91) 782 43 27
E-mail: secretaria@ctomedicina.com; iberocto@ctomedicina.com WEB: www.ctomedicina.com; www.iberocto.com
DERMATOLOGA
Preparacin Examen de Seleccin 05/06 1 Vuelta Seguimiento a distancia
matar los caros sino que es necesario que se eliminen fsicamente los cida por la Rickettsia Conorii, y transmitida por una garrapata del
restos de los caros. La sarna nodular no se trata nicamente con gnero Riphicephalus. La respuesta 1 es correcta ya que el exantema
escabicidas sino que precisa el uso de antihistamnicos y de corticoi- aparece por todo el cuerpo, incluyendo palmas y plantas. En algunos
des. El tratamiento de eleccin de la sarna es la permetrina al 5% que casos se puede apreciar una pequea afectacin heptica. La res-
se emplea de forma tpica. Se puede emplear tanto en embarazadas puesta 3 es falsa ya que la mancha negra slo se aprecia en el 75% de
como en nios pequeos. El lindano es una alternativa de uso tpico, los casos. La mancha negra no es ni ms ni menos que el lugar de la
pero es neurotxico y muy irritante, por lo que no se debe emplear en picadura de la garrapata. Despus aparece fiebre y cefalea que se
embarazadas ni en nios. En las sarnas muy extendidas o en puede prolongar durante das o incluso semanas. Por eso la fiebre
inmunodeprimidos puede usarse la ivermectina oral. botonosa mediterrnea es una de las causas que hay que buscar en
una fiebre de origen desconocido.
Pregunta 6.-R: 5 El exantema es lo ltimo en aparecer, y no suele ser pruriginoso. El
La paciente presenta una lesin eritematosa y migratoria. Adems diagnstico de una fiebre botonosa mediterrnea se fundamenta en
entre sus antecedentes destaca que tiene un contacto habitual con los la realizacin de serologas y en la clnica.
perros. Todo sto debe llevarnos a pensar que la paciente presenta Suele ser una enfermedad que evoluciona de forma satisfactoria
una enfermedad de Lyme. Esta enfermedad est causada por una con una respuesta espectacular a las tetraciclinas.
espiroqueta, Borrelia burgdorferi, y es transmitida por la picadura de
una garrapata del gnero Ixodes. Despus de la picadura, y tras un Pregunta 9.- R: 3
perodo de incubacin de entre 7 y 20 das, aparece desde ese lugar La respuesta correcta es la 3 por distintos motivos.
un eritema homogneo, anular, que se va extendiendo. Esta lesin En primer lugar tanto el liquen plano pilar como el lupus discoide
puede persistir meses o semanas y acompaarse en la mitad de los no son patologas tpicas de la infancia. Adems ambas patologas
casos de fiebre o cefalea. Como dice la respuesta 2 la afectacin producen una destruccin del folculo piloso y por tanto dejan como
articular es tarda y aparece en una minora de los pacientes. La afec- secuela alopecia cicatricial. En la descripcin clnica no se nos da
tacin sistmica tambin incluye afectacin cardaca en forma de ningn dato que nos oriente hacia la existencia de tejido cicatricial.
bloqueo aurculo-ventricular y afectacin neurolgica. Tanto la afec- La tricotilomana consiste en arrancarse el pelo de forma consciente o
tacin cardaca como la neurolgica suelen ser afectaciones preco- inconsciente. Eso podra justificar la presencia de pelos rotos pero no
ces y no tardas como se expone en la respuesta 5. el eritema, la descamacin ni el prurito. El cuanto a la alopecia areata,
El diagnstico de esta enfermedad se realiza mediante serologas y este proceso s es bastante frecuente en la infancia, en especial en
el tratamiento debe incluir tetraciclinas cuando no existe afectacin nios que presentan dermatitis atpica. La alopecia areata podra dar
visceral y slo es cutnea, y s debe incluir en el caso de afectacin pelos rotos pero nunca veramos la sintomatologa aadida de este
visceral tratamiento con penicilina. caso como el eritema y la descamacin. En la alopecia areata se
aprecia un infiltrado linfocitario perifolicular pero la piel est intacta.
Por tanto, la nica posibilidad era la respuesta 3. La tia tonsurante
Pregunta 6. Enfermedades cutneas transmitidas por artrpodos. se ve sobre todo antes de la adolescencia y produce una alopecia
reversible. En este caso, una buena alternativa teraputica ser la
*DUUDSDWDV)LHEUHERWRQRVDPHGLWHUUiQHD 5LFNHWVVLDFRQRULL  griseofulvina, dada la edad del paciente.
(QIHUPHGDGGH/\PH %RUUHOLDEXJGRUIHUL  Adems el tratamiento ser oral, ya que trataremos de forma oral
0RVTXLWRV%RWyQGH2ULHQWH /HLVKPDQLD  aquellas micosis con afectacin de pelo, uas, inflamatorias o presen-
cia de varias placas.
Pregunta 7.-R: 4
En el enunciado llama la atencin de una histologa con histiocitos
que contienen en el citoplasma una serie de microorganismos. Esta
histologa es tpica de la leishmaniasis. Un pioderma gangrenoso no
puede ser ya que no suele aparecer en la cara, sino ms frecuente-
mente en las extremidades. Adems no es propio de la infancia. El
ntrax y el imptigo contagioso hubiesen evolucionado a mejor o a
Comentarios TEST

peor durante esas 5 semanas, pero no hubiera permanecido con la


misma morfologa durante tanto tiempo. El lupus vulgar es la forma
ms frecuente de tuberculosis cutnea, pero su localizacin suele ser
en la regin preauricular. El paciente de esta pregunta presenta una
leishmaniasis cutnea del viejo mundo o botn de Oriente. Es una
enfermedad producida por protozoos, en nuestro medio por la
Leishmania donovani, variedad infantum y por la Leishmania trpica. El
vector de transmisin de la enfermedad es una mosca Phlebotomus,
siendo el reservorio de la enfermedad los perros. El botn de Oriente
deja inmunidad y debemos pensar en l cuando nos encontremos
ante una lesin papulosa, que tarda en curar, localizada en la cara de
un nio o nia de pocos aos de edad.
Adems de la clnica nos orienta en el diagnstico la histologa y el
diagnstico inmunolgico que se efecta mediante una intradermo-
rreaccin, la prueba de Montenegro, que resulta positiva en el 75%
de los casos.
En este paciente el tratamiento se realizara mediante inyecciones
intralesionales de gluconato de antimonio.

Pregunta 8.-R: 3
Tenemos un paciente con fiebre, cefalea y lesiones cutneas. Entre
sus antecedentes tenemos el hecho de que ha estado en el campo
unos das antes de que se iniciara el cuadro clnico. Adems el pa-
ciente tiene un exantema, que junto a la fiebre y sus antecedentes
debe hacernos pensar en fiebre botonosa mediterrnea. Est produ- Pregunta 9. Tia tonsurante.

M exico A rgentina
C hile U ruguay
CTO Medicina C/ Nez de Balboa, 115 28006 MADRID (Espaa) Tfno.: (91) 782 43 32 / Fax: (91) 782 43 27
E-mail: secretaria@ctomedicina.com; iberocto@ctomedicina.com WEB: www.ctomedicina.com; www.iberocto.com
DM Pg. 3
DERMATOLOGA
Seguimiento a distancia Preparacin Examen de Seleccin 05/06 1 Vuelta
Pregunta 10.- R: 1 a la enfermedad celaca pero no a alteraciones hepticas. Por la
Clnicamente estamos ante una placa bien definida, caliente, do- clnica podramos pensar en las picaduras de insecto pero es excep-
lorosa, eritematosa, y que se acompaa de fiebre y leucocitosis. Por cional que se acompaen de alteraciones en las enzimas hepticas.
tanto estamos ante una erisipela. En cuanto a la enfermedad de pie-mano-boca la distribucin de las
La respuesta falsa es la 1, ya que el diagnstico se realiza bien lesiones no se corresponde con la descripcin de la pregunta. La
mediante hemocultivos o por la clnica. La localizacin ms frecuente respuesta correcta es la 1. Esta enfermedad, autolimitada, representa
de las erisipelas es en los miembros inferiores, en especial en mujeres un patrn de respuesta, propia de la edad infantil frente a la primo-
con antecedentes de insuficiencia venosa, diabetes mellitus u obesi- infeccin por el virus de la hepatitis B. La clnica tiene una instaura-
dad. El herpes zoster no siempre da vesculas, con lo que en ocasiones cin brusca, distribuyndose por extremidades, cara, nalgas, palmas
puede cursar con la presencia de placas eritematosas y dolorosas, que y plantas. La hepatitis suele iniciarse de manera simultnea. No tiene
pueden llevarnos a confundirlo con una erisipela. Esto sucede de un tratamiento especfico, pues es un cuadro autoinvolutivo. Cuan-
manera frecuente en la cara, en especial con el herpes zoster trigeminal. do se produce por otros virus distintos al VHB hablamos del Sndro-
La diferencia entre una erisipela y una celulitis es que en esta ltima, me de Gianotti-Crosti en vez de enfermedad de Gianotti-Crosti.
se llega a afectar el tejido celular subcutneo, por tanto tiene una
afectacin ms profunda. Este hecho hace que los lmites de una Pregunta 14. R: 4
celulitis estn mal definidos. Tanto la erisipela como la celulitis, van a La lepra tuberculoide afecta a la piel y a los nervios perifricos. En
tener afectacin del estado general. la piel se aprecian una o pocas mculas, hipocrmicas, asimtricas
El tratamiento tanto de las celulitis como de la erisipela debe incluir y que por afectacin de los anejos son alopcicas y secas. Llama la
penicilina administrada de forma sistmica. La evolucin con el trata- atencin que estas mculas son anestsicas o hipoestsicas debido a
miento adecuado suele ser positiva y sin dejar secuelas. la precoz afectacin sensitiva que se produce en la lepra tuberculoide.
La prueba de la lepromina es muy positiva. Slo el 1% de las lepras
Pregunta 11.- R: 5 tuberculoides acaban siendo lepras lepromatosas. Los bordes de las
En este caso llama la atencin que la paciente presenta lesiones lesiones cutneas de la lepra tuberculoide son ligeramente elevados,
que se abscesifican, con una evolucin muy larga pues llega a los dos muy bien delimitados y en ocasiones con morfologa anular.
aos. Adems la localizacin es especfica de ambas regiones inguina- Las lesiones cutneas de la lepra lepromatosa son mltiples mcu-
les y parece que con los tratamientos intentados se logra mejorar el las amplia y simtricamente distribuidas, con bordes muy mal defini-
proceso, pero acaba recidivando. Un chancro blando produce fistu- dos y que con el paso del tiempo se engrosarn formando placas.
lizacin y abscesificacin inguinal, pero no es bilateral. Adems en los Estas placas en la cara dan origen a la facies leonina, con prdida del
antecedentes no aparece el chancro. El linfogranuloma venreo tam- pelo de cejas y pestaas (madarosis). En la lepra lepromatosa la afec-
bin da abscesificacin y fistulizacin bilateral. Lo que ocurre es que tacin neurolgica es mas tarda que en la tuberculoide.
hubiera respondido a algunos de los ciclos de antibiticos orales, que
suponemos habran incluido las tetraciclinas. Tampoco figuran ante-
cedentes que nos hagan sospechar una enfermedad de transmisin
sexual. La enfermedad de Crohn puede en algunos casos dar fstulas Pregunta 14. Diagnstico diferencial de LT y LL.
perianales pero no cuadra con el enunciado de la pregunta. En este
caso estamos ante una hidrosadenitis crnica supurativa. 7XEHUFXORLGH /HSURPDWRVD
La hidrosadenitis supurativa aparece en zonas apocrinas. El trata- 0iFXODV 1yGXORVOHSURPDV
miento es con cloxacilina, o con ciclos largos de tetraciclinas. OHVLRQHV 0XFKDVOHVLRQHV
En algunos casos es necesario recurrir a la exresis quirrgica de la
$VLPpWULFDV 6LPpWULFDV
glndula o al drenaje quirrgico de los abscesos.
En los casos ms recidivantes se pueden emplear los retinoides (ULWHPDQRGRVR (1 DXVHQWH (1SUHVHQWHFRQIUHFXHQFLD
orales, la isotretinona, que mejoran sustancialmente el cuadro. (QJURVDPLHQWRDVRFLDGRGH (QJURVDPLHQWRGLIXVR
QHUYLRVSHULIpULFRV $IHFWDFLyQWDUGtD
Pregunta 12.- R: 4 $IHFWDQHUYLRV $QHVWHVLDOLJHUDRDXVHQWH
La varicela se caracteriza por la presencia de lesiones en diferentes $QHVWHVLDLQWHQVD
estados, es decir, en fase de ppula, vescula y costra. Cada vez que
vuelven a aparecer lesiones papulosas significa que existe una nueva %LRSVLDSLHO

Comentarios TEST
vivencia. Adems la varicela se acompaa de un gran quebrantamien- *UDQXORPDVWXEHUFXORLGHV ,QILOWUDGRGHKLVWLRFLWRVLV
to del estado general, con fiebre alta y cefalea. La varicela no debe ser &pOVWLSR/DQJKDQV PDFUyIDJRVFRQJORELV
tratada salvo que sea una varicela en un inmunodeprimido, neonato, $QHVWHVLDLQWHQVD %DQGDGH8QQD
que exista alguna complicacin o que la afectacin cutnea sea muy 5HDFFLRQHV
intensa. Por eso en este caso la respuesta correcta es la 4. 0LWVXGD 0LWVXGD
La complicacin ms frecuente de la varicela es la sobreinfeccin
%DFLORVFRSLD %DFLORVFRSLD
bacteriana de las lesiones cutneas. Una de las complicaciones graves
de la varicela es la neumona por el virus varicela zoster. Esta neumo- 7UDWDPLHQWR
na puede cursar con escasas alteraciones en las imgenes radiolgi- 5LIDPSLFLQDPJPHV 5)3PJPHV
cas, por lo que en ocasiones, en especial si sospechamos que el pa- 6XOIRQDPJGtD 6)PJGtD
ciente presenta esta complicacin es recomendable la realizacin de &ORIDFLPLQDPJPHV
una gasometra arterial. 7UDWDPLHQWRPHVHV 7UDWDPLHQWRDxRV
Cuando la varicela precisa un tratamiento especfico se realiza con
6HJXLPLHQWRDxRV DxRV
aciclovir a dosis de 800mg/5h durante 7 das. La varicela no compli-
cada slo precisa un tratamiento sintomtico para el prurito y para la /HSURUUHDFFLRQHVWLSR, 7LSR
fiebre. )HQyPHQRGH/XFLR

Pregunta 13.- R: 1 Pregunta 15.- R: 4


Vemos en esta nia que las lesiones que presenta son ppulas y El sarcoma de Kaposi es una proliferacin vascular multifocal que
vesculas, pero el estado general es bueno, con lo que descartamos puede afectar tanto a la piel como a las mucosas. Existen cuatro varian-
que se trate de una varicela. La dermatitis herpetiforme cursa con tes, clsico, endmico, asociado a inmunosupresores y asociado a SIDA.
lesiones papulosas, vesiculosas, que se distribuyen por las zonas de En el Kaposi asociado a SIDA existen datos epidemiolgicos que indi-
extensin de manera simtrica. La dermatitis herpetiforme no suele can la etiologa vrica del cuadro, especficamente por un virus herpes
comenzar en la infancia y adems se asocia con mucha frecuencia 8. Las lesiones se inician como pequeas ppulas violceas que a dife-

M exico A rgentina
Pg. 4 DM C hile U ruguay
CTO Medicina C/ Nez de Balboa, 115 28006 MADRID (Espaa) Tfno.: (91) 782 43 32 / Fax: (91) 782 43 27
E-mail: secretaria@ctomedicina.com; iberocto@ctomedicina.com WEB: www.ctomedicina.com; www.iberocto.com
DERMATOLOGA
Preparacin Examen de Seleccin 05/06 1 Vuelta Seguimiento a distancia
rencia del clsico suelen localizarse ms en la cara y en el tronco. No paciente que ya ha presentado un carcinoma basocelular es posible
suele ser la causa de muerte del paciente ya que cuando se inician las que a lo largo de su vida tenga otros. sto se debe a que se supone que
terapias antirretrovirales el cuadro suele estabilizarse e incluso ese paciente ha estado expuesto de forma crnica e intensa a las
involucionar. La afectacin de la mucosa oral acontece en un 25% de radiaciones solares.
los pacientes. Adems, cuando existe afectacin extracutnea, sta sue-
le localizarse en ganglios linfticos, pulmn y mucosa gastrointestinal. Pregunta 19.- R: 4
Las alteraciones histolgicas son idnticas en las cuatro variantes de El nevus sebceo de Jadasshon es una proliferacin localizada de
Kaposi. lbulos de glndulas sebceas. Es muy caracterstico que se localice
En cuanto al tratamiento del Kaposi asociado al VIH se emplea la en el cuero cabelludo, como una placa congnita, con frecuencia
radioterapia, interfern alfa y ltimamente se estn realizando estu- con ausencia de pelo. La placa es de consistencia elstica, coloracin
dios clnicos con derivados de los retinoides. amarillenta y superficie ligeramente rugosa. Es frecuente que con los
aos aparezcan sobre el nevus sebceo otras lesiones de naturaleza
Pregunta 16.- R: 2 tumoral. La lesin tumoral benigna que aparece con mayor frecuen-
En la histologa de la lesin ya nos habla de la presencia de mitosis y cia es el siringocistoadenoma papilfero, y de los malignos el epitelio-
de mltiples atipias celulares. Esto ya nos debe inducir a pensar en que ma basocelular, que aparece en el 30% de los casos. Su imagen clni-
la lesin de la paciente se trata de un tumor. Una psoriasis en placas no ca y localizacin le hacen inconfundible aunque en ocasiones se
puede ser, ya que adems de que no presenta una histologa especfica, puede confundir con una aplasia cutis o un xantogranuloma juvenil.
no es lgico pensar que una placa de psoriasis va a crecer de forma El problema de degeneracin maligna acontece a partir de la puber-
progresiva y se va a localizar en el mismo emplazamiento durante un tad. Por eso el tratamiento consiste en la extirpacin quirrgica antes
perodo tan largo de tiempo. Un herpes circinado, tia corporis, s de la pubertad. En el caso de que no se pudiera extirpar por ser muy
puede persistir muchos aos, incluso con crecimiento progresivo, pero amplia la lesin o por otros factores, entonces se debern seguir revi-
la biopsia tampoco es como nos la exponen. En cuanto a las queratosis siones peridicas. La respuesta 1 trataba de confundirnos pues lo que
seborreicas, son lesiones benignas, con proliferacin epidrmica, sin existe es el adenoma sebceo de Pringle, que en realidad se trata de
llegar a traspasar la membrana basal. Pero por supuesto una queratosis angiofibroma que se localiza en la regin centrofacial y es muy espe-
seborreica no tendra atipias. La enfermedad de Paget se localizar en la cfica de la esclerosis tuberosa.
areola mamaria. En su histologa es muy caracterstico que las clulas no
establezcan puentes intercelulares entre ellas. Pregunta 20.- R: 2
La enfermedad de Bowen es un carcinoma epidermoide in situ La epidermodisplasia verruciforme o tambin llamada enferme-
que ocurre con preferencia en el tronco. La aparicin de lesiones dad de Lewandwosky-Lutz es una enfermedad hereditaria con carc-
mltiples hacen necesario descartar intoxicaciones por arsnico. En ter autosmico recesivo que determina una respuesta anormal frente
este caso, al ser un carcinoma in situ, en vez de ciruga podramos a la infeccin por el VIH. La clnica consiste en la aparicin de mlti-
realizar una electrocoagulacin o radioterapia. ples lesiones, bien verrugas planas o bien verrugas vulgares. En un 30%
de los pacientes existe transformacin maligna de las lesiones, el tu-
Pregunta 17.- R: 1 mor al cual degeneran con mayor frecuencia es al carcinoma
La lesin que presenta la paciente, es una lesin papulosa, brillan- epidermoide.
te, blanquecina. Esto nos indica que se trata de un epitelioma El carcinoma epidermoide o epitelioma espinocelular es el tumor
basocelular. El tratamiento de eleccin de un epitelioma basocelular maligno ms frecuente en mucosas. Es excepcional que aparezca
es la extirpacin quirrgica. No son necesarios ms que escasos mr- sobre piel sana ya que tiene tendencia a localizarse sobre lesiones
genes ya que no existe el riesgo de metstasis. La presencia de preexistentes.
hiperglucemias mantenidas en una mujer de 77 aos no debe ser una Los factores que predisponen a la aparicin de un carcinoma epider-
contraindicacin quirrgica. La respuesta 2, realizacin de un TAC moide son la gentica (personas de piel clara), radiacin (exposicin a la
abdominal, no tiene sentido, ya que el carcinoma basocelular no radiacin ultravioleta), lesiones cutneas inflamatorias y degenerativas
metastatiza. La realizacin de radioterapia se reserva para aquellos crnicas (lupus vulgar, lupus crnico, cicatrices, radiodermitis, quemadu-
epiteliomas basocelulares, de gran tamao, en los que la ciruga sera ras, etc.), inmunosupresin, infecciones virales (en especial las ocasiona-
una mala solucin teraputica. La radioterapia no est indicada en das por VPH), lesiones precancerosas (queratosis cutneas). Las localiza-
los carcinomas basocelulares esclerodermiformes, ya que tiene malos ciones ms comunes del carcinoma espinocelular son aquellas que es-
Comentarios TEST

resultados. En cuanto al interfern y al imiquimod son alternativas tn ms expuestas a las radiaciones ultravioletas.
teraputicas en epiteliomas basocelulares en los que tampoco sera
factible la ciruga. Tanto el interfern alfa, como el imiquimod tienen Pregunta 21.- R: 5
efectos inmunomoduladores. Tambin se estn empleando ambas Bien, nos hablan de una lesin de 3 aos de evolucin, que le
sustancias en el tratamiento de algunas formas clnicas de melanoma. ocasiona al paciente cierto prurito. La lesin era verrucosa, de colora-
cin marrn oscura. Como vemos en ningn momento nos hablan
Pregunta 18.-R: 2 de ppulas perladas, brillo blanquecino o superficie brillante. Por
El carcinoma basocelular es el tumor maligno ms frecuente. Lo es tanto no podemos pensar que se trate de un carcinoma basocelular.
an ms que el de la prstata, mama o pulmn. Su nombre se debe a Por la descripcin clnica podra tratarse de un melanoma o de un
estar formado por clulas de la capa basal y de la epidermis y de los nevus melanoctico. Lo que ocurre es que en la histologa nos estn
anejos cutneos. Histolgicamente el tumor est formado por clulas hablando de proliferacin de clulas epidrmicas y presencia de glo-
epiteliales de la capa basal, que se distribuyen formando nidos y bos crneos. Un melanoma o un nevus melanoctico presentaran en
cordones, y se disponen en empalizada. En ocasiones los carcinomas la histologa melanocitos, que en el caso de melanoma seran atpicos.
basocelulares pueden contener elevadas cantidades de melanina y La queratosis seborreica es un tumor benigno de una incidencia
mostrar un aspecto pigmentoso. El carcinoma basocelular tambin se muy alta en la raza blanca. Su nmero ha aumentado con el paso de
denomina epitelioma basocelular porque a diferencia de otros los aos. Clnicamente son lesiones papulosas, blandas, untuosas o
carcinomas slo de forma excepcional produce metstasis. El carci- grasientas al tacto. No se ha descrito transformacin maligna. Debe
noma basocelular se distribuye por zonas en las que existen folculos plantearse el diagnstico diferencial en algunos casos, en especial con
pilosebceos. Es ms frecuente sobre las lneas de cierre embrionario el epitelioma basocelular pigmentoso y con el melanoma. El trata-
y sobre lesiones hamartomatosas anexiales, no coincidiendo en cam- miento de las queratosis seborreicas es la extirpacin mediante
bio con las zonas de mayor incidencia de radiacin solar. Se piensa curetaje, electrocoagulacin o crioterapia.
por tanto que existiran factores locales, como la presencia de clulas
pluripotenciales del germen epitelial primario, que participaran en la Pregunta 22.- R: 3
etiopatogenia de este tumor. En esta pregunta llama la atencin que el paciente presente unas
El carcinoma basocelular no se localiza nunca en las mucosas. Un lesiones papulosas, perladas, localizadas por toda la superficie corpo-

M exico A rgentina
C hile U ruguay
CTO Medicina C/ Nez de Balboa, 115 28006 MADRID (Espaa) Tfno.: (91) 782 43 32 / Fax: (91) 782 43 27
E-mail: secretaria@ctomedicina.com; iberocto@ctomedicina.com WEB: www.ctomedicina.com; www.iberocto.com
DM Pg. 5
DERMATOLOGA
Seguimiento a distancia Preparacin Examen de Seleccin 05/06 1 Vuelta
ral. Clnicamente las lesiones se corresponden con la descripcin gan un pronstico ms favorable que los varones. Adems las lesiones
morfolgica de carcinomas basocelulares. Ahora bien, no parece l- en las extremidades tienen mejor pronstico que las localizadas en el
gico encontrar carcinomas basocelulares en una persona joven. Ade- tronco, cabeza y cuello. No sucede lo mismo con las localizadas en
ms el carcinoma basocelular se relaciona con la exposicin solar las palmas, plantas y subungueales. En cuanto al riesgo de tener
crnica, y un minero no est expuesto, en condiciones normales, a melanoma, es ms alto cuanto ms bajo el fenotipo cutneo, es decir,
las radiaciones solares. El cuadro que presenta el paciente es un sn- cuanto ms clara es la piel del paciente. El melanoma es ms frecuen-
drome de Gorlin. El sndrome del Nevus basocelular o sndrome de te en mujeres que en varones con una proporcin de 2 a 1. Es un
Gorlin es una enfermedad gentica que sigue un patrn de herencia tumor muy raro antes de la pubertad, pero cuando lo hace antes de la
autosmica dominante. Afecta por igual a ambos sexos aunque es pubertad, el 50% de las ocasiones aparece sobre un nevus congnito
ms frecuente en personas de piel clara. Adems de cursar con la gigante.
aparicin de mltiples epiteliomas basocelulares presenta mltiples El melanoma es actualmente el tumor con mayor tasa de inciden-
lesiones pigmentadas en palmas y plantas, anomalas esquelticas, en cia en los adultos jvenes.
especial queratoquistes odontognicos maxilares y cierto retraso men-
tal. No incluye como se cita en la respuesta 3 afectacin ocular. En el
tratamiento del sndrome de Gorlin se han utilizado los retinoides por
va oral para prevenir el desarrollo de nuevas lesiones e impedir la
progresin de las ya existentes.

Pregunta 23.- R: 2
El que las lesiones expuestas en la pregunta se encuentren en las
zonas del labio inferior, nos debe hacer pensar que el sol tiene un
papel en el desarrollo de estas lesiones. Adems existen antecedentes
de exposicin solar como es el hecho de que se trate de un agricultor
y adems existe un factor carcinognico como es el hbito tabquico.
El liquen plano oral suele aparecer en la mucosa lingual o geniana
pero es raro que aparezca en la mucosa labial. Las aftas orales son
lesiones erosivas, ulceradas, no aparecen en la semimucosa ni se rela-
cionan con la exposicin solar. El exantema fijo medicamentoso tiene Pregunta 25. Clasificacin de Clark.
predileccin por aparecer en mucosas, aunque nunca persiste la le-
sin tanto tiempo, 3 aos, y no se relaciona con el sol. La queilitis Pregunta 26.- R: 3
actnica es el equivalente de la queratosis actnica pero en este ltimo La paciente presenta una lesin de pocos aos de evolucin, locali-
caso localizada en la semimucosa del labio inferior. Su diagnstico y zada en una extremidad. En la histologa de la lesin se aprecian
tratamiento son importantes debido a la alta tasa de metstasis del melanocitos atpicos con muchas mitosis. Por lo tanto debemos pensar
carcinoma epidermoide de labio. Siempre que existan dudas debe en un melanoma. El melanoma ms frecuente de todos es el melanoma
realizarse una biopsia para tener un diagnstico de certeza. El trata- de extensin superficial. Adems esta forma clnica de melanoma es la
miento de las queratosis actnicas se basa en la crioterapia, el lser ms frecuente en los adultos y jvenes. El melanoma de extensin su-
CO2 y el uso del 5-fluoracilo. perficial se presenta inicialmente como una lesin plana, que paulati-
namente se va elevando. El color oscila entre el marrn y el negro.
Pregunta 24.- R: 1 Cuando al principio vemos que la lesin es plana, se corresponde con
Los dos grupos de nevus melanocticos que parecen asociarse la fase radial del crecimiento. Posteriormente la superficie se va hacien-
significativamente con un mayor riesgo de transformacin maligna a do irregular con zonas papulosas e incluso nodulares. Esto significa que
melanoma son los nevus congnitos de gran tamao y los nevus el melanoma de extensin superficial ha entrado en fase de crecimien-
displsicos. Un nevus congnito grande o gigante, tiene un riesgo de to vertical o invasiva. La respuesta 3 es la falsa ya que el crecimiento
degeneracin a melanoma que oscila entre el 5% y el 20% segn las radial del melanoma de extensin superficial se prolonga durante un
series publicadas. En cualquier caso, el riesgo es muy alto. La pa- mximo de 5 aos. El lntigo maligno melanoma, es el melanoma ms
ciente de la pregunta presenta un nevus congnito de gran tamao frecuente en sujetos de edad avanzada. Su fase de crecimiento radial s
pues le ocupa la cara externa del muslo izquierdo. Por esa razn la que puede durar hasta dcadas. La extirpacin precoz de la lesin es el

Comentarios TEST
respuesta mejor sera la de extirparlo quirrgicamente en la medida mejor tratamiento. Debe realizarse una extirpacin con mrgenes, que
de lo posible. Si el nevus congnito fuera inoperable deberamos se determinan en funcin del ndice de Breslow.
entonces observarlo y en el momento que tuviramos dudas acerca
de si existe transformacin maligna de la lesin en algn lugar debe- Pregunta 27.- R: 1
ramos extirpar esa zona y someterla a un estudio histolgico. El El tratamiento del melanoma cutneo primario se realiza mediante
hacer biopsias semestrales de la lesin es un absurdo ya que siempre extirpacin quirrgica con mrgenes de seguridad. Actualmente se reco-
quedaran zonas sin biopsiar. mienda en los melanomas con un ndice de Breslow menor de 1 ampliar
En cuanto a la eliminacin con lser de la lesin no sera una mrgenes hasta en 1 cm. Cuando el ndice de Breslow es mayor de 1 mm
buena alternativa teraputica. El lser lo que hace es quemar. En un se recomienda extirpar 2 cms de margen, hasta llegar a la fascia. La reali-
nevus congnito existen melanocitos hasta en la dermis profunda, zacin de la tcnica del ganglio centinela se realiza en aquellos melano-
con lo que la quemadura tendra que ser muy agresiva. Adems luego mas con una profundidad o ndice de Breslow mayor a 1. Esta tcnica
quedara una cicatriz que podra enmascarar la degeneracin malig- consiste en estudiar si el ganglio ms cercano al tumor se encuentra
na de alguno de los melanocitos que hubieran quedado. Adems el afectado o no. La tcnica del ganglio centinela no parece que mejore el
resultado esttico no sera bueno en absoluto. pronstico del paciente, slo logra diagnosticar mejor la enfermedad. El
uso de interfern alfa en el melanoma se reserva para los casos con un
Pregunta 25.-R: 4 elevado ndice de Breslow y por tanto con altas posibilidades de desarro-
El factor pronstico ms importante en el melanoma en estadio I es llar metstasis. Esto es as porque el interfern alfa parece disminuir o
la profundidad medida en milmetros desde la capa granulosa de la retrasar el desarrollo de metstasis. Por eso en la pregunta la respuesta
epidermis. A esto se le llama ndice de Breslow. Este factor pronstico correcta es la 1. Cuando ya existen metstasis desarrolladas, el interfern
ha sustituido a los niveles de invasin de Clark que relacionan el nivel alfa no parece jugar ningn papel.
de penetracin del tumor con el pronstico. Otros factores histolgicos
que otorgan un factor pronstico desfavorable son un alto ndice Pregunta 28.- R: 5
mittico y la existencia de satelitosis microscpica. Existen adems La micosis fungoide es un proceso linfoproliferativo maligno T
factores pronsticos clnicos como el hecho de que las mujeres ten- inductor/colaborador de causa desconocida. Se origina primaria-

M exico A rgentina
Pg. 6 DM C hile U ruguay
CTO Medicina C/ Nez de Balboa, 115 28006 MADRID (Espaa) Tfno.: (91) 782 43 32 / Fax: (91) 782 43 27
E-mail: secretaria@ctomedicina.com; iberocto@ctomedicina.com WEB: www.ctomedicina.com; www.iberocto.com
DERMATOLOGA
Preparacin Examen de Seleccin 05/06 1 Vuelta Seguimiento a distancia

Pregunta 28. Micosis fungoide.


mente en la piel pudiendo permanecer en ella hasta dcadas. A
partir de ah puede afectar ganglios linfticos, rganos internos y
acarrear la muerte del paciente. Las fases premicticas son aquellas
fases previas al diagnstico histolgico de la enfermedad. Las lesio-
nes en la fase premictica pueden ser confundidas con mltiples
dermatosis como los eccemas, psoriasis o tias. En ocasiones la en-
fermedad puede comenzar como una eritrodermia como el hom-
bre rojo de Hallopeau o con la forma agresiva de linfoma cutneo
que es el Sndrome de Szary. En la tercera fase de la micosis
fungoide, fase tumoral, aparecen lesiones tumorales sobre la piel
normal o sobre las lesiones previas. A partir de esta fase comienzan a
producirse manifestaciones extracutneas, afectndose con mucha
frecuencia los ganglios linfticos, hgado y bazo. La afectacin de la
Comentarios TEST

mdula sea es muy infrecuente.


La respuesta 5 es falsa ya que en la micosis fungoide las manifesta-
ciones ganglionares y viscerales aparecen al cabo de aos de evolu-
cin. Ver dibujo en la pgina siguiente.

Pregunta 29.- R: 1
Pregunta 29. Tratamiento de la micosis fungoide segn estadios.
No existen tratamientos curativos de la micosis fungoide, sino
que los tratamientos que se emplean lo que logran es retrasar la
Pregunta 30.- R: 3
enfermedad y prolongar la vida de los pacientes. La eleccin del
El sndrome de Szary viene definido por la trada de eritrodermia,
tratamiento va a depender del estadio evolutivo de la enfermedad.
aparicin en sangre perifrica de ms de un 10% de clulas de Szary,
En la fase de eccema y de placas recurrimos a los tratamientos
que son clulas linfoides atpicas, y linfadenopatas generalizadas. La
tpicos. Los que ms se emplean son los corticoides tpicos y la
respuesta 1 es falsa ya que el sndrome de Szary ocasiona mucho
mostaza nitrogenada. La mostaza nitrogenada tiene un efecto se-
prurito, lgico al presentar eritrodermia. El sndrome de Szary, a dife-
cundario que es la alta frecuencia con la que induce hipersensibi-
rencia de la micosis fungoide, tiene una evolucin corta. Existe discu-
lidad retardada. Este fenmeno ha sido propuesto como uno de
sin an sobre si la eritrodermia del sndrome de Szary es diferente o
los mecanismos por los que frenara a las clulas neoplsicas. Tam-
no de la que puede presentar una micosis fungoide. Para algunos
bin en las primeras fases de la enfermedad se han empleado la
autores ambos procesos seran la misma entidad patolgica. La res-
fototerapia (UVB) y la fotoquimioterapia (PUVA). En la fase tumoral
puesta 3 es correcta ya que al existir linfadenopatas generalizadas es
se emplea el tratamiento con radioterapia, ya que las lesiones cu-
frecuente que aparezca hepatomegalia o esplenomegalia.
tneas de micosis fungoide son muy sensibles al efecto de las radia-
ciones ionizantes.
Pregunta 31.- R: 5
En la fase tumoral tambin se emplean el interfern alfa, la acitreti-
Se trata de una mujer con lesiones eritematosas en ambas mejillas, que
na y la fotofresis extracorprea con la que se han obtenido muy
claramente presentan historia de fotosensibilidad. Dado el sexo femeni-
buenos resultados, con remisiones de hasta el 80%.
no y la edad de la paciente deberamos pensar en dos patologas: roscea
o lupus eritematoso. En la roscea son tpicas la presencia, en algunos

M exico A rgentina
C hile U ruguay
CTO Medicina C/ Nez de Balboa, 115 28006 MADRID (Espaa) Tfno.: (91) 782 43 32 / Fax: (91) 782 43 27
E-mail: secretaria@ctomedicina.com; iberocto@ctomedicina.com WEB: www.ctomedicina.com; www.iberocto.com
DM Pg. 7
DERMATOLOGA
Seguimiento a distancia Preparacin Examen de Seleccin 05/06 1 Vuelta
estados, de lesiones papulosas o pustulosas. Este tipo de lesiones no se Clnicamente cursa con la aparicin, casi desde el nacimiento, de
observan en el lupus eritematoso. Adems las respuestas de la pregunta mltiples lesiones de coloracin oscura, grandes, que se distribuyen
nos orientan ms hacia la roscea. El tratamiento de la roscea es difcil. por el tronco y extremidades, afectando con frecuencia a las flexuras.
Un punto primordial es evitar los estmulos que facilitan la vasodilatacin En el 50% de los casos aparecen opacidades a nivel corneal. La enfer-
como las bebidas calientes, el sol, picantes, cambios bruscos de tempera- medad se transmite de manera autosmica recesiva ligada al cromo-
tura, etc. El tratamiento de eleccin son las tetraciclinas orales, que se soma X. La histopatologa no es especfica y el tratamiento consiste en
emplean a las mismas dosis que en el acn vulgar. Como alternativa se el empleo de emolientes y en los casos severos el empleo de retinoides
pueden emplear el metronidazol bien en formulacin tpica o de forma orales.
oral. En los casos resistentes o muy intensos se pueden utilizar los retinoides
(isotretinona) de forma oral. El tratamiento del rinofima requiere correc-
cin quirrgica. La respuesta falsa es la 5 ya que en la roscea no existe Pregunta 33. Tipos de ictiosis.
compromiso de rganos internos, cosa que s hubiera sido factible en el
caso de tratarse de un lupus eritematoso. ,FWLRVLVOHYHV ,FWLRVLVJUDYHV

,FWLRVLVOLJDGDDO ,FWLRVLV
,FWLRVLVYXOJDU ,FWLRVLVODPHODU
Pregunta 31. Dermatosis agravadas por el sol. ; HSLGpUPLFD

+HUHQFLD $' /LJDGDDO; $5 $'


/(6 +HUSHV
3RUILULDV 3UHFDQFHURVLV [S FiQFHU 3UHVHQWHDO
5DUR 5DUR 6t 6t
'DULHU )RWRDOHUJRIRWRWR[LFRGHUPLDV QDFLPLHQWR

5RViFHD 3HODJUDFDUFLQRLGH $XVHQFLDGH


3pQILJRIROLiFHR $OELQLVPRRFXORFXWiQHR 3DWRJHQLD 'HVFRQRFLGD DU\OVXOIDWDVD 'HVFRQRFLGD 'HVFRQRFLGD
&
Pregunta 32.- R: 3 +LSHUTXHUDWRVLV
La enfermedad de Paget representa un carcinoma epidermotropo PLWRVLV
+LSHUTXHUDWRVLV +LSHUTXHUDWRVLV +LSHUTXHUDWRVLV 9DFXROL]DFLyQ
de los conductos glandulares de la mama. Ante cualquier lesin ecce- $3
+LSHUJUDQXORVLV PLWRVLV
matosa en una mama siempre debemos establecer el diagnstico dife- GHFpOXODV
rencial con la enfermedad de Paget del pezn. Clnicamente se carac- JUDQXORVDV
teriza por ser lesiones generalmente unilaterales y de lento crecimiento. 2SDFLGDGHV 2SDFLGDGHV
Las lesiones suelen tener bordes muy bien delimitados y son unilatera- $VRFLDFLRQHV $WRSLD FRUQHDOHVVLQ FRUQHDOHVFRQ 'HIRUPLGDGHV
les por lo que la respuesta 3 es falsa. Normalmente afecta a mujeres FHJXHUD FHJXHUD
mayores de 40 aos, siendo excepcional en varones.
Las clulas de Paget son clulas de gran tamao, con grandes ncleos /RPiV (VFDPDV 3UHVHQWDQ
y sin desmosomas o tonofibrillas. La mastectoma es el tratamiento ms
1RWDV
IUHFXHQWH RVFXUDV DPSROODV
adecuado con linfadenectoma regional si precisa. No est recomenda- 7UDWDPLHQWR (PROLHQWHV\TXHUDWROtWLFRV 5HWLQRLGHVRUDOHV
da la extirpacin cutnea local o la irradiacin de las lesiones cutneas.
La enfermedad de Paget extramamaria clnica e histolgicamente se pa-
rece a la de la mama aunque la mayora de las veces no asocia un Pregunta 34.- R: 4
carcinoma subyacente. Se localiza con ms frecuencia en la regin Destaca en el enunciado la aparicin de lesiones descamativas,
anogenital y regiones axilares. En cerca del 20% de los casos localizados nacaradas. Una de las enfermedades que cursa con descamacin
en regin anogenital se encuentra un carcinoma primario en otros rga- nacarada es la psoriasis. Adems el hecho de que las lesiones sean ppulas
nos, principalmente recto, uretra, vejiga y cuello uterino. y que aparezcan en una paciente de pocos aos nos debe hacer pensar
en una psoriasis en gotas. La psoriasis en gotas cursa con numerosas
Pregunta 33.- R: 5 lesiones pequeas en gotas que se distribuyen en especial por el tronco.
La ictiosis X afecta a los varones aunque portadores femeninos Es una enfermedad que aparece fundamentalmente en la infancia. Sue-
pueden mostrar rasgos fenotpicos aislados. La enfermedad se produ- le aparecer despus de infecciones del tracto respiratorio superior, espe-
ce debido a que existe una ausencia de la enzima microsomal sulfatasa cialmente despus de infecciones estreptoccicas. La psoriasis en gotas

Comentarios TEST
esteroidea. La ausencia de la enzima microsomal sulfatasa esteroidea tiene buen pronstico ya que se resuelve espontneamente despus de
determina un aumento de colesterol sulfato en el estrato crneo a unas semanas tras la infeccin estreptoccica. La psoriasis en gotas no
cuyo nivel aparece un aumento de la extensin intracorneal caracte- suele por tanto tener afectacin articular. Respecto a la respuesta 2, la
rstica de la enfermedad. aparicin de lesiones de psoriasis en mucosas es muy infrecuente. Cuan-
do aparece en mucosas lo hace en menos del 1% de los pacientes. La
respuesta 5 es falsa ya que al tener buen pronstico el tratamiento se
fundamenta en el uso de productos tpicos.

Pregunta 35.- R: 3
La respuesta correcta es la 3. El aspecto histolgico de una biopsia
est relacionado con el estadio evolutivo de la lesin. La primera
alteracin objetivable consiste en afectacin vascular con dilatacin
capilar, edema papilar e infiltrado mononuclear perivascular. En esta
fase puede existir paraqueratosis. Adems las crestas papilares se alar-
gan con exudacin de neutrfilos hacia la epidermis. Estos neutrfilos
emigran hacia las zonas paraqueratsicas constituyendo los micro-
abscesos de Munro. Tambin aparecen neutrfilos en la porcin su-
perior del estrato espinoso, as se constituyen las pstulas espongiformes
de Kogoj. Otros hallazgos caractersticos de la psoriasis son la ausencia
de capa granulosa, acantosis e hiperqueratosis. El fenmeno del roco
hemorrgico de Auspitz se debe a la papilomatosis, al estar elongadas
las papilas drmicas los capilares suben hasta niveles muy altos con lo
Pregunta 33. Histologa de la ictiosis. que fcilmente se rompen al desprender escamas de una placa de
psoriasis.

M exico A rgentina
Pg. 8 DM C hile U ruguay
CTO Medicina C/ Nez de Balboa, 115 28006 MADRID (Espaa) Tfno.: (91) 782 43 32 / Fax: (91) 782 43 27
E-mail: secretaria@ctomedicina.com; iberocto@ctomedicina.com WEB: www.ctomedicina.com; www.iberocto.com
DERMATOLOGA
Preparacin Examen de Seleccin 05/06 1 Vuelta Seguimiento a distancia
Pregunta 36.- R: 4 que sean muy raras las recidivas, se ha postulado que tuviera una
El paciente padece una psoriasis, aparentemente grave porque no etiologa vrica, probablemente por virus herpes. El tratamiento de la
responde a la teraputica tpica y tambin debido a que afecta a una pitiriasis rosada es con emolientes. La exposicin solar mejora el cua-
gran extensin de la superficie corporal. dro. Ver dibujo en la siguiente pgina.
Dentro de los antecedentes del paciente vemos que es un hepat-
pata por el alcohol y por la infeccin por el VHC. Si furamos a Pregunta 40.- R: 4
instaurarle un tratamiento la respuesta 1 la descartaramos debido a La pitiriasis rubra pilar cursa con la aparicin de lesiones papulosas
que el Metotrexate es hepatotxico. En cuanto a la respuesta 2 es una queratsicas, foliculares, que confluyen formando placas que de for-
mala opcin ya que los corticoides orales o sistmicos no deben ma caracterstica contienen islotes de piel sana respetada. En cuanto a
usarse en la psoriasis debido a que cuando se suspenden los enfermos la dermatitis seborreica es una patologa que no cursa con ppulas y
presentan un rebrote en su enfermedad. En la prctica clnica los que adems mejora con la exposicin solar. La enfermedad de Hailey-
corticoides sistmicos en la psoriasis llegamos a usarlos en casos extre- Hailey es una enfermedad gentica, autosmica dominante, que tam-
mos. La respuesta 3, acitretina, sera el tratamiento de eleccin en bin se denomina pnfigo benigno familiar. Cursa con lesiones
cualquier psoriasis grave. El problema es que en el presente caso no vesiculosas por axilas, cuello y regiones inguinales. Se han obtenido
podemos emplearla ya que uno de los efectos secundarios de los buenos resultados con el tratamiento de esta enfermedad con el em-
retinoides es la hepatotoxicidad. pleo de antibiticos o antifngicos tpicos. La respuesta correcta es la
Como alternativa a los retinoides en la psoriasis grave, empleamos 4 ya que la enfermedad de Darier tiene unos hallazgos clnicos muy
la ciclosporina. El principal efecto secundario de la ciclosporina es tpicos, como son la localizacin de las lesiones en zonas seborreicas,
que es nefrotxica. En cuanto a la ltima respuesta, el calcipotriol, queratosis puntata en palmas y plantas y la presencia de tonalidades
es falsa ya que es un producto tpico que se emplea en psoriasis blanquecinas y rojizas con una morfologa en V en el borde libre
leves. ungueal. Los pacientes con enfermedad de Darier deben protegerse
del sol ya que este empeora mucho la enfermedad. En formas exten-
Pregunta 37.- R: 2 sas se emplean los retinoides ya que los corticoides no mejoran e
De nuevo nos encontramos ante una psoriasis grave, con afecta- incluso pueden exacerbar la enfermedad.
cin de mas del 40% de la superficie corporal total. El tratamiento de
eleccin sera la acitretina. El problema que nos encontramos es que
la pacientes es una mujer en edad frtil con lo que tendramos que
mantener anticoncepcin durante al menos 2 aos. Adems los trata-
mientos con retinoides son tratamientos largos. La respuesta 2 es la
falsa ya que la acitretina no tiene las mismas acciones que la isotreti-
nona. El principal efecto de la isotretinona es la disminucin de la
secrecin sebcea por atrofia de las glndulas, adems disminuye la
extensin de los queratinocitos del infundibular folicular, favorecien-
do la eliminacin de comedones.
La alternativa a los retinoides es la ciclosporina, y la alternativa a
esta ltima es el uso de metotrexate. El metotrexate es de eleccin en
la artropata psorisica. La PUVA est indicada principalmente en las
psoriasis en placas que abarcan el 30% de la superficie corporal.
Tambin puede emplearse en las psoriasis graves el RePUVA que es la
combinacin de los retinoides y el PUVA.

Pregunta 38.- R: 4
El tratamiento de eleccin de una psoriasis leve son los corticoides
tpicos. Los tratamientos de la psoriasis varan en funcin de la loca-
lizacin, severidad, tratamientos previos, sexo y edad del paciente. En
la psoriasis pustulosa y eritrodrmica el tratamiento de eleccin es la
Comentarios TEST

acitretina que en ocasiones se puede asociar a la PUVA, siendo muy Pregunta 39. Pitiriasis rosada. Recordad que es ms frecuente en mujeres.
efectivo. Como alternativa a la acitretina podemos emplear la
ciclosporina. El tratamiento de eleccin del liquen plano son los
Pregunta 41.- R: 2
corticoides, que se emplearn de forma tpica o sistmica en funcin
de la extensin. En los casos severos de liquen plano podemos em- Nos encontramos ante un varn, adulto, que presenta lesiones
plear la acitretina. En el caso de las enfermedades ampollosas ampollosas y erosivas en la cavidad oral, mucosa genital y lesiones
autoinmunes el tratamiento de eleccin son los corticoides, salvo en ampollosas por toda la superficie corporal.
la dermatitis herpetiforme en la que emplearemos sulfonas. Los corti- Ante este caso nos podemos plantear que el paciente tiene un
coides se emplean en el pnfigo a dosis altas. La respuesta correcta es imptigo ampolloso, ya que las ampollas del imptigo por estafiloco-
la 4 pues la pitiriasis rubra pilaris se trata con queratolticos y acitretina co suelen ser flccidas. El problema es que el imptigo ampolloso no
ya que los corticoides no se muestran eficaces. afecta mucosas y adems no suele acompaarse de mal estado gene-
ral, fiebre etc. Otra patologa que nos podemos plantear como res-
ponsable del cuadro es el pnfigo vulgar. El pnfigo vulgar aparece en
Pregunta 39.- R: 3 estados medios de la vida, cursa con ampollas flccidas que aparecen
Destaca en el enunciado que existe una lesin de mayor tamao, sobre piel aparentemente sana. La afectacin mucosa en el pnfigo
y otras que siguen una distribucin en rbol de Navidad. Los hallazgos vulgar es prcticamente constante tanto de la oral como de la genital.
clnicos son caractersticos constituyendo la clave para el diagnstico. El pnfigo vulgar es una enfermedad grave que puede comenzar len-
La lesin de gran tamao suele ser la lesin inicial, placa herldica, tamente, con un escaso nmero de lesiones y permanecer as largo
que habitualmente se localiza en el tronco. El resto de las lesiones tiempo. Aproximadamente en el 50% de los casos, las lesiones se
tienen una morfologa ovalada, con descamacin perifrica, seme- inician en las mucosas. Al ser una enfermedad grave el tratamiento
jantes a la placa herldica pero con un menor tamao. La distribu- debe ser agresivo, empleando para ello inmunosupresores. De elec-
cin por el tronco, en rbol de Navidad, es muy caracterstica. Las cin se emplean corticoides a dosis altas y administrados de forma
lesiones son asintomticas, sin producir prurito. Las lesiones persisten sistmica. Cuando existe contraindicacin en el uso de corticoides o
4 - 8 semanas, llegando a regresar espontneamente. Por este motivo falta de respuesta, se emplean otros inmunosupresores como la aza-
la respuesta falsa es la 3. La pitiriasis rosada de Gibert es de causa tioprina, ciclofosfamida o micofenolato.
desconocida, aunque por el hecho de la variabilidad estacional y de

M exico A rgentina
C hile U ruguay
CTO Medicina C/ Nez de Balboa, 115 28006 MADRID (Espaa) Tfno.: (91) 782 43 32 / Fax: (91) 782 43 27
E-mail: secretaria@ctomedicina.com; iberocto@ctomedicina.com WEB: www.ctomedicina.com; www.iberocto.com
DM Pg. 9
DERMATOLOGA
Seguimiento a distancia Preparacin Examen de Seleccin 05/06 1 Vuelta

Pregunta 44. Diagnstico diferencial de las enfermedades ampollosas.

3pQILJR 3HQILJRLGH +HUSHV 'HUPDWLWLV (SLGHPyOLVLV 'HUPDWRVLV,J$


*HVWDWLRQLV KHUSHWLIRUPH DPSROORVDDGTXLULGD OLQHDO
(WLRSDWRJHQLD $XWRLQPXQH $XWRLQPXQH $XWRLQPXQH *OXWHQ $XWRLQPXQH
9XOJDU )ROLiFHR $PSROODVWHQVDV /HVLRQHV /HVLRQHVKHUSHWLIRUPHV +LSHUIUDJLOLGDG
&OtQLFD DPSROODV 5DUR VREUHSLHO KHUSHWLIRUPHVHQ PX\SUXULJLQRVDVHQ (QWUHSHQILJRLGH
IOiFFLGDV DPSROODV HULWHPDWRVD DEGRPHQPX\ iUHDVGHH[WHQVLyQ FXWiQHD \'KHUSHWLIRUPH
(URVLRQHV 3UXULWRLQWHQVR SUXULJLQRVDV
$IHFWDFLyQ 6t 1R 6t  1R 1R
PXFRVD
$PSROOD $PSROOD $PSROOD $PSROOD $PSROOD $PSROOD
LQWUDHSLGpU LQWUDHSLGpU VXEHSLGpUPLFD VXEHSLGpUPLFD VXEHSLGpUPLFDFRQ
$3 PLFD PLFDHQ FRQLQILOWUDGR PLFURDEVFHVRVGHSPQ VXEHSLGpUPLFD
VXSUDEDVDO JUDQXORVD GpUPLFRHRVLQyILOR FRQHRVLQyILORV HQODVSDSLODVGpUPLFDV FROiJHQR9,,
1LNROVN\    
,)' ,J*HQWUHORVTXHUDWLQRFLWRV ,J*OLQHDOHQPE ,J*OLQHDOHQPE ,J$JUDQXODUHQYpUWLFH
GHSDSLODV ,J$OLQHDOHQPE

,), HQHO HQHO HQHO ,J$JUDQXODUHQYpUWLFH ,J$OLQHDOHQPE


GHSDSLODV

&RUWLFRLGHVVLVWpPLFRV &RUWLFRLGHV &RUWLFRLGHV 6XOIRQDGLHWDVLQ &RUWLFRLGHV


772 ,QPXQRVXSUHVRUHV VLVWpPLFRV VLVWpPLFRV JOXWHQ VLVWpPLFRV 6XOIRQD
,QPXQRVXSUHVRUHV

(OSHQILJRLGH SUHVHQWDQ
1RWDV (OSpQILJRYXOJDUHVHOPiV FLFDWUL]DOHVXQD LQWROHUDQFLDDOJOXWHQ
JUDYH YDULDQWH SHURVyORHOFRQ
FOtQLFD

Pregunta 42.- R: 3
Pregunta 41. Diagnstico diferencial de lesiones en mucosa oral. Nos encontramos ahora ante un paciente con aparente buen esta-
do general, de edad avanzada, con lesiones ampollosas tensas por el
/LTXHQSODQRGHPXFRVDRUDO 5HWLFXODGRSDSXODUEODQTXHFLQRHQWHUFLR tronco y extremidades. Con ampollas tensas cursan aquellas patolo-
WDPELpQHQRWUDV SRVWHULRUGHPXFRVD\XJDOGHODUJD gas que tienen despegamiento subepidrmico. El herpes gestationis
ORFDOL]DFLRQHV  HYROXFLyQ0ROHVWLDVYDULDEOHV cursa con despegamiento subepidrmico pero como es natural, en
un varn, y encima de 74 aos, es "difcil" que llegue a tener un herpes
/HVLRQHVHURVLYDVURGHDGDVGHiUHDV
/LTXHQHURVLYRGHPXFRVDV gestationis.
EODQTXHFLQDVSDSXODUHVPX\GRORURVDV
IRUPDSRFRIUHFXHQWHGH La dermatitis herpetiforme tambin tiene despegamiento subepi-
HQPXFRVD\XJDO$YHFHVTXHLOLWLV
OLTXHQSODQR  drmico y por tanto ampollas tensas, pero esta opcin la descartamos
DGKHUHQWH
ya que la dermatitis herpetiforme no suele aparecer a esas edades tan
$IWRVLVUHFXUUHQWH GHVFDUWDU OFHUDVRUDOHVGRORURVDVGHGLiPHWUR avanzadas y menos an sin su distribucin herpetiforme. Por lo tanto

Comentarios TEST
5HLWHUQRGXHOHQ%HKoHW(76 YDULDEOHTXHFXUDQ estamos ante un penfigoide ampolloso. Lo ms adecuado para reali-
FHOLDTXtDHQILQIODPDWRULD HQPHQRVGHGtDVVLQUHODFLyQFRQ zar un diagnstico de certeza en cualquier enfermedad ampollosa de
LQWHVWLQDO  RWUDSDWRORJtD aparente etiologa inmunolgica es la realizacin de una biopsia de la
/(6 ODV~OFHUDVVRQXQ piel perilesional. Debe ser de la zona perilesional porque si se recoge
OFHUDVRUDOHVFRQHULWHPDFLUFXQGDQWH del centro de la lesin veremos que existe una ampolla pero no sere-
FULWHULRGLDJQyVWLFR 
JLQJLYLWLVSHWHTXLDVTXHLOLWLVGHVFDPDWLYD
0X\LQHVSHFtILFDVGHMDQ mos capaces de ver el depsito de anticuerpos. Por eso realizaremos
DGKHUHQWHSODFDVUHWLFXODUHVSODWHDGDV
FLFDWUL] una biopsia perilesional y tambin mediante tcnicas de
inmunofluorescencia directa determinaremos el tipo de anticuerpo
$PSROODJUDQGHIOiFFLGDVHGHVSUHQGH\
3pQILJR (QHOHVOD
GD~OFHUDVDPSOLDVTXHSURJUHVDQKDVWD
depositado y dnde lo ha hecho. La inmunofluorescencia indirecta
PDQLIHVWDFLyQ 
ODELRV(YROUiSLGDPDOHVWDUJHQHUDO
en el penfigoide no aporta mucha informacin y apenas tiene valor.
En cuanto a la respuesta a los corticoides, el penfigoide suele res-
3HQILJRLGH 1RHVOD (URVLRQHVRUDOHVFRQHULWHPDSHULIpULFR ponder bien a dosis moderadas de los mismos, pero por supuesto eso
PDQLIHVWDFLyQ  5HVSHWDQHOODELR)UHFTXHLOLWLV no es diagnstico como insina la respuesta 2.
'HVFDUWDUHSLGHUPyOLVLV GHVFDPDWLYD7LHQGHQDFXUDU\QR
DPSROORVD SURJUHVDQ Pregunta 43.- R: 4
6G6WHYHQV-RKQVRQ HULWHPD Se trata de un paciente de corta edad, con lesiones ampollosas
(ULWHPDHGHPDJUDOL]DGRDPSROODV~OFHUDV pruriginosas. En la biopsia existe un depsito de IgA lineal. Una de las
PXOWLIRUPHPD\RU 
FRQLQIODPDFLyQSHULIpULFD\0(*HQ
'HVFDUWDUQHFUyOLVLVHSLGpUPLFD enfermedades que cursa con depsito de IgA en la dermis es la dermatitis
MyYHQHVWUDVLQIHFFLyQSRU0\FRSODVPD
Wy[LFD SRVWIiUPDFRV  herpetiforme. Pero la dermatitis herpetiforme tiene un depsito granular,
*LQJLYRHVWRPDWLWLVSRUKHUSHV
en las papilas drmicas, y no de forma lineal. La respuesta 1 es correcta
VLPSOH ,GHP HULWHPDHGHPD~OFHUDVPDOHVWDU
pues si bien la sulfona es el tratamiento de eleccin de la dermatitis
6LYHVtFXODVHQRURIDULQJH FRQDGHQRSDWtDV\SDUWLFLSDFLyQGHHQFtDV herpetiforme, tambin es el tratamiento de la dermatosis IgA lineal de la
KHUSDQJLQD &R[VDFNLH$  infancia. Debe establecerse el diagnstico diferencial con la dermatitis
herpetiforme por su similitud clnica, tanto es as que hasta 1970 no se

M exico A rgentina
Pg. 10 DM C hile U ruguay
CTO Medicina C/ Nez de Balboa, 115 28006 MADRID (Espaa) Tfno.: (91) 782 43 32 / Fax: (91) 782 43 27
E-mail: secretaria@ctomedicina.com; iberocto@ctomedicina.com WEB: www.ctomedicina.com; www.iberocto.com
DERMATOLOGA
Preparacin Examen de Seleccin 05/06 1 Vuelta Seguimiento a distancia
desligaron ambas enfermedades. Las lesiones aparecen de forma brusca, Vemos entonces que todas las respuesta de la pregunta son correc-
se acompaan de prurito intenso, en forma de vesculo-ampollas, tensas tas. Pero la ms correcta es la respuesta 1, que adems de tratamiento
de componente claro que despus se convierten en hemorrgico. La realiza una biopsia, y por lo tanto tenemos un diagnstico de certeza.
respuesta 4 es falsa pues al contrario que la dermatitis herpetiforme que se
asocia a enfermedad celaca en el 90% de las ocasiones no ocurre lo Pregunta 47.- R: 2
mismo con la dermatosis IgA lineal de la infancia. La base diagnstica ms Estamos ante una paciente, probablemente polimedicada ya que
importante, ya que es la nica que nos permite diferenciarla de otros tiene bronquitis crnica e hipertensin. Desde el punto de vista cut-
procesos ampollosos como dato positivo es la presencia de IgA en la zona neo apreciamos lesiones de pocos das de evolucin que le aparecen
de la membrana basal y de forma lineal. en miembros inferiores. En la morfologa de las lesiones es llamativo el
hecho de que son rojas, eritematosas, y no blanquean a la presin. Es
Pregunta 44.- R: 3 la tpica lesin de vasculitis. La forma ms frecuente de vasculitis cut-
La pregunta es muy sencilla pues ante una mujer embarazada con nea es la vasculitis por hipersensibilidad. La etiologa de las vasculitis
lesiones vesiculares periumbilicales debemos pensar inmediatamente cutneas es muy variada, pudiendo estar causada por infecciones,
en la penfigoide gestacional o herpes gestationis. Las lesiones pueden tumores, medicamentos, enfermedades autoinmunes... etc. El meca-
aparecer en cualquier momento del embarazo o del puerperio. Pue- nismo de produccin del dao vascular se explica por el depsito de
den verse tambin en otras situaciones como el coriocarcinoma, men- inmunocomplejos antgeno-anticuerpo. Estos complejos se deposi-
struacin o incluso con la ingesta de anticonceptivos. La morfologa tan en la pared de los vasos pequeos, en especial en las vnulas. Por
de las lesiones en el herpes gestationis vara desde las papulovesculas este motivo la respuesta 1 es falsa. Las vasculitis por hipersensibilidad
a las placas urticariformes. Suelen comenzar alrededor del ombligo suelen localizarse, clnicamente, en las zonas ms declives, debido a
para extenderse despus sobre todo por el tronco. Una vez que apa- que el depsito de los inmunocomplejos se produce por gravedad. La
recen las lesiones tienen tendencia a persistir a lo largo del embarazo. forma clnica ms frecuente de presentacin es la prpura palpable.
Suele recurrir en nuevos embarazos y cuando as sucede suele iniciar- La vasculitis por hipersensibilidad se caracteriza por la degeneracin y
se de forma ms precoz. Los corticoides tpicos y las cremas hidratantes necrosis fibrinoide de los vasos. Por eso tambin se denomina vasculitis
pueden eliminar el picor pero son insuficientes para el control de la necrotizante y leucocitoclstica. Por lo tanto la repuesta correcta es la
enfermedad. En la mayor parte de los casos es necesaria la administra- 2. El tratamiento consiste en retirar la causa desencadenante si se
cin de corticoides a dosis moderada y de forma oral. La respuesta 3 conoce. El uso de los inmunosupresores est indicado cuando hay
por tanto es la correcta. El tratamiento se instaura principalmente para afectacin sistmica o progresin de la enfermedad.
disminuir la incidencia de prematuridad y de bajo peso al nacimiento
del feto. La histologa del herpes gestationis es similar a la del penfigoide Pregunta 48.- R: 5
ampolloso pero en la inmunofluorescencia directa se demuestra el Ver comentario de la pregunta anterior.
depsito de C3 a lo largo de la membrana basal.
Pregunta 49.- R: 2
Pregunta 45.- R: 2 Un tipo de vasculitis por hipersensibilidad caracterstica es la pr-
El paciente tiene lesiones nodulares, que llegan a eliminar material. pura de Schnlein-Henoch, que se caracteriza por tener depsito de
La enfermedad que tiene como lesiones elementales los ndulos sub- IgA en las vasos drmicos. Clnicamente cursa con la aparicin de una
cutneos son las paniculitis. Por eso deberamos pensar en la respues- prpura palpable, que puede llegar a ulcerarse. Es muy tpico que se
ta 1 o 2 como respuestas correctas. El Sndrome de Weber-Christian acompae de un fuerte dolor abdominal existiendo incluso casos de
puede ser una paniculitis traumtica o un sndrome idioptico, recu- invaginacin intestinal e infartos con perforacin. La histologa es la
rrente y febril. En la pregunta nos estn dando datos muy caractersti- misma que la de las vasculitis por hipersensibilidad, con lo que la
cos de la paniculitis pancretica como son la histologa y la morfolo- respuesta 1 es falsa. Se produce ms frecuentemente en nios, y nor-
ga de las lesiones. As la imagen histolgica caracterstica es la presen- malmente menores de 10 aos. La afectacin del sistema nervioso
cia de focos de necrosis grasa en el panculo adiposo con presencia central es una complicacin grave pero que aparece en menos del
de clulas grasas necrotizantes o clulas fantasmas. Las lesiones suelen 1% de los pacientes. Por lo tanto la respuesta 4 es falsa. La respuesta 5
acompaarse de elevacin en sangre y orina de la amilasa. La morfo- tambin es incorrecta ya que si bien es cierto que se ha asociado a
loga clnica es la de ndulos de localizacin diferente, que se suelen infecciones estreptoccicas, la aparicin de las lesiones cutneas sue-
ulcerar eliminando un material aceitoso. La paniculitis grasa le comenzar al cabo de 10 das y no a las 48 h. El curso normal de la
Comentarios TEST

pancretica est asociada a un cncer de pncreas en el 40% de los prpura Schnlein-Henoch es a la curacin, teniendo en general
casos. Por estos motivos la respuesta correcta es la 2. Las lesiones de la buen pronstico. La afectacin renal aparece en un 20% de los casos
paniculitis pancretica pueden preceder en el tiempo al adenocarci- en forma de glomerulonefritis. De este porcentaje un 1% llegan a
noma de pncreas. desarrollar una insuficiencia renal crnica. El tratamiento es el mismo
que el de otras vasculitis, es decir retirar la causa, y el uso de
Pregunta 46.- R: 1 imnunosupresores en el caso de complicaciones.
De nuevo un caso sencillo ya que nos aportan todos los datos para
saber de qu enfermedad se trata. Sin embargo la respuesta no es fcil. Artralgias
Vemos que se trata de una mujer joven con lesiones nodulares en cara
(Glomerulonefritis sin IR)
anterior de ambas piernas. Adems presenta fiebre, artralgias y las
lesiones la ocasionan dolor. Se trata pues de una paniculitis septal sin
vasculitis, es decir del eritema nodoso. El eritema nodoso es la paniculitis
ms frecuente, aparece con mayor frecuencia en mujeres, jvenes o
de edad media y de forma caracterstica cursa con lesiones nodulares,
contusiformes, en cara anterior de ambas piernas. La etiologa del
eritema nodoso es desconocida, pensndose en mecanismos
inmunolgicos mltiples. Cuando el eritema nodoso es recidivante la
causa ms frecuente son las infecciones estreptoccicas de vas altas.
En las mujeres es frecuente encontrar la ingesta de anticonceptivos
como responsable del cuadro. Es importante pensar tambin en las Dolor clico abdominal
siguientes enfermedades como responsables del cuadro: infecciones
por yersinia, sarcoidosis, tuberculosis y enfermedad inflamatoria in- Prpura palpable
testinal. El tratamiento del eritema nodoso se fundamenta en la retira- MMII y glteos
da de las causas, el reposo, administracin de antiinflamatorios y/o
corticoides. Pregunta 49. Prpura de Schnlein-Henoch.

M exico A rgentina
C hile U ruguay
CTO Medicina C/ Nez de Balboa, 115 28006 MADRID (Espaa) Tfno.: (91) 782 43 32 / Fax: (91) 782 43 27
E-mail: secretaria@ctomedicina.com; iberocto@ctomedicina.com WEB: www.ctomedicina.com; www.iberocto.com
DM Pg. 11
DERMATOLOGA
Seguimiento a distancia Preparacin Examen de Seleccin 05/06 1 Vuelta
Pregunta 50.- R: 4 descartamos la posibilidad de lupus eritematoso crnico, liquen plano
El liquen plano es una enfermedad de causa desconocida que pilar, neoplasia, esclerodermia... etc. Tampoco puede tratarse de una
aparece con mayor frecuencia entre los 25 y los 65 aos. Es muy raro tia, pues debera tener descamacin y eritema. El antecedente de ser
verlo en la infancia o en la vejez. Los hallazgos clnicos caractersticos opositor es clave para el diagnstico pues una de las alopecias no
consisten en ppulas poligonales, brillantes, violceas y que son muy cicatriciales que se ha relacionado con la tensin emocional es la alo-
pruriginosas. Las ppulas no son de gran tamao y se distribuyen por pecia areata. La etiologa de la alopecia areata es desconocida aunque
la parte distal de las extremidades y en especial por las superficies se piensa que alteraciones inmunolgicas juegan un papel importante.
flexoras de las muecas, tobillos y antebrazos. Alrededor del 50% de Es frecuente su asociacin con otros procesos autoinmunes: el vitligo,
los pacientes presentan lesiones en mucosas, especialmente en mu- tiroiditis, dermatitis atpica etc. En la histologa es tpico el hallazgo de
cosa oral. Incluso puede cursar nicamente con lesiones en mucosas un infiltrado linfocitario que aparece alrededor de los bulbos pilosos.
donde la morfologa de las lesiones es distinta a la que vemos en la La respuesta 2 es falsa ya que el curso es variable, con remisiones espon-
piel. En las mucosas se ven estriaciones blanquecinas aunque en oca- tneas. La respuesta 5 tambin es falsa ya que es frecuente la alteracin
siones se ven formas erosivas que producen grandes molestias a los de la sntesis de melanina y que aparezcan pelos blancos. Existen ml-
enfermos. Adems estas formas, cuando son crnicas pueden acabar tiples tratamientos para la alopecia areata. Cuando son pocas placas y
siendo carcinomas. La alteracin ungueal es una caracterstica menor de pequeo tamao se utilizan corticoides tpicos o intralesionales;
del liquen plano que se ve en el 10% de los casos. Se ha encontrado cuando son varias y de gran tamao se prefieren corticoides sistmicos.
asociacin entre el liquen plano y el VHC, sobre todo las formas con Alternativas son la puvaterapia, difenciprona y dinitrobenceno.
afectacin mucosa. El tratamiento del liquen plano es muy variado en
funcin de la intensidad del proceso. Los casos leves se tratan con Pregunta 54.- R: 2
corticoides tpicos mientras que en los casos muy intensos se usan En principio la pregunta parece difcil, pero existen ya datos im-
corticoides orales o acitretina. Incluso en los casos graves se llegan a portantes que deben hacernos pensar en una patologa. Destacan la
emplear inmunosupresores potentes como la ciclosporina. fiebre, la leucocitosis con neutrofilia y un cuadro con lesiones cut-
neas. Se trata de la antiguamente llamada dermatosis neutroflica fe-
Pregunta 51.- R: 4 bril o Sndrome de Sweet. La enfermedad es de causa desconocida
El tratamiento de un acn leve debe ser en principio tpico. Dentro aunque en el 80% de los casos se asocia con un cuadro infeccioso. El
de los tratamientos tpicos no existe uno que sea de eleccin. Si las otro 20% se asocia a procesos linfoproliferativos, en especial a
lesiones son inflamatorias vamos a emplear los antibiticos tpicos. leucemias. Por esta razn la respuesta 2 es incorrecta. Las lesiones
Dentro de stos los que ms se emplean son la clindamicina y la cutneas habitualmente son precedidas de cuadros infecciosos, con
eritromicina. Cuando lo que predomina son lesiones comedonianas mayor frecuencia respiratorios. Las lesiones suelen ser papulopstulas
preferiremos entonces o el perxido de benzoilo o los retinoides tpi- que predominan en superficie de extensin. La histologa no es
cos. El perxido de benzoilo tiene accin queratoltica y como efectos diagnstica pero s muy sugestiva del cuadro, pues se aprecia un infil-
secundarios puede producir irritacin y descamacin cutnea. Los trado neutroflico en dermis. El sndrome de Sweet es una dermatosis
retinoides tpicos tienen una potente accin comedoltica y exfoliante. benigna. El problema surge de esa asociacin con procesos linfopro-
Se emplea la isotretinona, adapaleno y el tazaroteno. Cuando el acn liferativos. Lo ms habitual es que el diagnstico del Sndrome de
es grave o un acn moderado que no responde a tratamiento tpico, Sweet anteceda al diagnostico del tumor. El tratamiento de esta der-
entonces vamos a emplear los tratamientos sistmicos. El de eleccin matosis neutroflica febril es con corticoides sistmicos en pauta de-
sern los antibiticos orales, en especial las tetraciclinas. Una complica- creciente. Tambin pueden emplearse el yoduro potsico y la
cin con el tratamiento prolongado con antibiticos es la foliculitis por colchicina.
gram -. Si no responde con antibiticos entonces emplearemos los
retinoides, en forma de isotretinona. Existen unas excepciones a sto, y Pregunta 55.- R: 4
una de ellas es la que nos exponen en la pregunta. Cuando existen Nos encontramos ante un paciente con una lesin ulcerada y como
evidencias de alopecia andrognica, hirsutismo y acn, entonces prefe- antecedentes de inters el que padezca una enfermedad inflamatoria
riremos el empleo de hormonas antiandrognicas como el acetato de intestinal. El pioderma gangrenoso se caracteriza clnicamente por una
ciproterona. Debe emplearse asociado con un estrgeno como el lcera irregular con un borde inflamatorio y una base necrtica. Las
etinilestradiol para conseguir una accin anticonceptiva y para minimi- lesiones comienzan como ndulos dolorosos que se abren y ulceran
zar los efectos secundarios sobre el ciclo menstrual. secretando un exudado purulento o hemorrgico. El borde es irregular,
dentado y tiene una caracterstica coloracin prpura o violcea. Los

Comentarios TEST
Pregunta 52.- R: 4 lugares donde se localiza con mayor frecuencia el pioderma gangreno-
La alopecia androgentica es la calvicie comn. En cierta forma es so son las extremidades inferiores, nalgas y abdomen. Hemos pensado
un proceso fisiolgico en el cual los folculos producen cada vez pelos en un pioderma gangrenoso ya que una de las asociaciones frecuentes
ms pequeos y de menos dimetro. Con el paso del tiempo el folculo del mismo es con la enfermedad inflamatoria intestinal. Es ms frecuen-
se va miniaturizando hasta que se convierte en folculos atrficos. La te su asociacin con la colitis ulcerosa que con el Crohn. El curso de la
etiologa de este proceso es gentica y hormonal. Hormonal debido a enfermedad intestinal es independiente de la enfermedad cutnea. Otras
que algunos metabolitos de la testosterona inhiben el metabolismo de enfermedades con las que se ha asociado son con las artritis, gammapata
los folculos. Este proceso afecta a la mayora de los varones y a un monoclonal, leucemia mieloide y Enfermedad de Takayasu. El trata-
porcentaje elevado de mujeres. En la mujer la alopecia androgentica miento del pioderma gangrenoso es con corticoides administrados de
se trata con acetato de ciproterona, que es un potente antiandrgeno. forma sistmica. En los pacientes con enfermedad subyacente el trata-
En el varn el tratamiento, nunca curativo sino que lo que hace es el miento no slo se debe dirigir al pioderma gangrenoso sino tambin al
enlentecimiento del proceso, es con finasteride a dosis de 1 mg. Produ- trastorno sistmico. La respuesta 3 sera una opcin mala debido al
ce un recrecimiento temporal en el 50% de los pacientes. Al suspender fenmeno de patergia que consiste en que un traumatismo trivial pro-
el tratamiento sigue la cada. El finasteride inhibe la 5-alfa-reductasa y voca nuevas lesiones o agrava las preexistentes. Este fenmeno tambin
como efecto secundario a destacar, la impotencia sexual que aparece puede ser la causa del rechazo de injertos cutneos autlogos o del
en el 1% de los pacientes. Otra alternativa al finasteride, aunque menos desarrollo de nuevas lesiones en los sitios donantes del injerto.
eficaz, es el empleo de minoxidil al 2% de forma tpica. El minoxidil es
un vasodilatador y como efectos secundarios destaca la cefalea y pro- Pregunta 56.- R: 3
bables complicaciones cardacas en pacientes con patologa de base. Con la descripcin de la lesin nicamente podemos tener una
Por todo lo dicho la respuesta falsa es la 4. aproximacin diagnstica. Se nos describen lesiones en cara anterior
de las piernas.
Pregunta 53.- R: 1 En esa localizacin nos encontramos el eritema nodoso, pioderma
Varias placas alopcicas, con piel normal. Con esta descripcin no gangrenoso, mixedema pretibial y necrbiosis lipodica. En el mixe-
nos debemos plantear que se trate de una alopecia cicatricial. Por tanto dema pretibial es caracterstico el depsito importante de mucina a

M exico A rgentina
Pg. 12 DM C hile U ruguay
CTO Medicina C/ Nez de Balboa, 115 28006 MADRID (Espaa) Tfno.: (91) 782 43 32 / Fax: (91) 782 43 27
E-mail: secretaria@ctomedicina.com; iberocto@ctomedicina.com WEB: www.ctomedicina.com; www.iberocto.com
DERMATOLOGA
Preparacin Examen de Seleccin 05/06 1 Vuelta Seguimiento a distancia
nivel de dermis media y profunda. El mixedema pretibial forma parte enteropata pero adems es caracterstica la hiperglucemia, que no
de la enfermedad de Graves, acompaada de la oftalmoplejia e hi- figura en el cuadro clnico de la paciente. La pelagra est producida
pertiroidismo. Aunque estas caractersticas pueden presentarse de for- por el dficit de niacina, cido nicotnico o vitamina B3. La mayora
ma independiente, el mixedema pretibial suele aparecer en presen- de los casos aparecen en alcohlicos crnicos. Tambin pueden
cia de oftalmopata. La correccin de la alteracin hormonal carece aparecer en el sndrome carcinoide. Clnicamente se caracteriza
de efectos sobre las lesiones cutneas. De tal forma que el 50% de los por una trada en la que destaca una erupcin eritemato-vesicular,
mixedemas pretibiales aparecen despus de que el paciente se ha que dejan una hiperpigmentacin caracterstica. Las lesiones suelen
vuelto eutiroideo. Esta dermopata infiltrativa no es patognomnica aparecer en zonas fotoexpuestas. Cuando aparecen en el cuello
del hipertiroidismo ya que se han comunicado lesiones similares en dejan una erupcin simtrica que se denomina "collar de Casal". El
pacientes con hipotiroidismo. Los factores patognicos del mixedema tratamiento consiste en administrar nicotinamida, pues la adminis-
pretibial no estn claros. El tratamiento del mixedema pretibial no es tracin del cido nicotnico produce un cuadro de enrojecimiento
muy satisfactorio. Pueden brindar cierto alivio los corticoides sistmi- facial.
cos o intralesionales pero en ningn caso lo que dice la respuesta 3.
Pregunta 58.- R: 5
Volvemos a encontrarnos con un paciente alcohlico, y con lesio-
Pregunta 56. Hipotiroidismo vs. hipertiroidismo. nes cutneas. En este caso las lesiones cutneas se distribuyen por
dorso de manos, cara, es decir por zonas claramente fotoexpuestas.
+LSRWLURLGLVPR +LSHUWLURLGLVPR Pensaremos entonces en dos patologas: porfiria cutnea tarda y pela-
gra. Esta ltima se acompaa de demencia y diarrea que aparece en el
+~PHGD 50% de los casos, por lo que no puede ser la patologa del paciente.
3,(/ 6HFDSiOLGD\IUtD HULWHPDWRVD\ La porfiria hereditaria puede aparecer a cualquier edad pero la forma
FDOLHQWH adquirida suele aparecer en la edad adulta. La respuesta falsa es la 5
pues el tratamiento se fundamenta en evitar los factores desencade-
0L[HGHPD nantes as como la exposicin solar. Las sangras peridicas son el
/HVLyQFXWiQHDPiV 0L[HGHPD tratamiento de eleccin de la porfiria cutnea tarda. Un tratamiento
JHQHUDOL]DGR RMRV
FDUDFWHUtVWLFD SUHWLELDO alternativo es el empleo de cloroquina. Existe un cuadro idntico
PDQRV\SLHV 
clnicamente a la porfiria cutnea tarda, que se presenta en pacientes
1RGHMDIyYHD 1RGHMDIyYHD en hemodilisis y se denomina pseudoporfiria. La nica diferencia es
que no poseen alteraciones bioqumicas.
'HSyVLWRiFLGR
KLDOXUyQLFRHQ
'HSyVLWRiFLGR GHUPLVSURPRYLGR
KLDOXUyQLFRHQ SRUIRUPDFLyQGH
GHUPLV FRPSOHMRV$J$F
 HQIHUPHGDGGH
*UDYHV 
1RUHODFLRQDGR
5HODFLRQDGRFRQHO FRQHOH[FHVRGH+
GpILFLWGH+ WLURLGHDV/D
WLURLGHDV QRUPDOL]DFLyQQR
UHJUHVDHOSURFHVR

7UDWDPLHQWR 7UDWDPLHQWRFXUD
KRUPRQDO RFOXVLYD
VXVWLWXWLYR FRUWLFRLGHD
Comentarios TEST

+LSHUFDURWLQHPLDHQ 3UXULWR
FDSDFyUQHD GHUPRJUDILVPR
2WUDV
3~USXUD\ HOHYDGRIOXVKLQJ
HTXLPRVLV WHODQJLHFWDVLDV
6HFR\GHVOXVWUDGR Pregunta 58. Manifestaciones clnicas de la porfiria cutnea tarda.
$ORSHFLDH[WGH 6XDYHIULDEOH\GH
3(/2 Pregunta 59.- R: 3
ODVFHMDV FUHFLPLHQWRUiSLGR
 PDGDULRVLV  Destacan en el caso actual la escasa edad del paciente y la distribu-
cin de las lesiones cutneas, que es periorificial. Enfermedades con
'HVOXVWUDGDV 8xDVGH3OXPPHU clara distribucin periorificial tenemos el vitligo, que por supuesto
FUHFLPLHQWROHQWR FyQFDYDV\FRQ clnicamente no tiene ninguna relacin con el caso actual. Otra de las
8f$6 (VWULDV RQLFyOLVLVGLVWDO patologas claramente caracterizada por distribucin periorificial es la
ORQJLWXGLQDOHV\ $FURSDTXLD lentiginosis de Peutz-Jeghers. Tambin en el eritema necroltico migra-
WUDQVYHUVDOHV WLURLGHD torio es posible ver esa distribucin, pero suele aparecer asociado a
un glucagonoma. La acrodermatitis enteroptica cursa con lesiones
Pregunta 57.- R: 2 en regiones periorificiales, y en zonas acras. En la cara suele aparecer
Ante un paciente con antecedentes de ingesta alcohlica y lesio- un cuadro similar a la dermatitis seborreica. En las uas aparecen
nes cutneas, debemos pensar en el MIR en dos patologas, la porfiria lesiones transversales de Beau, debido a interrupciones temporales en
cutnea tarda y la pelagra. La respuesta 1 es falsa debido a que el el crecimiento de las mismas. Existe tambin cada de pelo. Adems es
escorbuto cursa con hemorragias gingivales. La respuesta 3 es falsa frecuente que esta clnica cutnea se acompae de diarrea. La acro-
pues la acrodermatitis enteroptica suele aparecer al suspenderse la dermatitis enteroptica es un cuadro infrecuente, transmitido de for-
lactancia. Adems no es esa la distribucin caracterstica de presen- ma autosmica recesiva, y su etiopatogenia es la deficiente absorcin
tacin de las lesiones cutneas. El glucagonoma puede cursar con intestinal del zinc. Por eso la respuesta correcta es la 3 ya que el

M exico A rgentina
C hile U ruguay
CTO Medicina C/ Nez de Balboa, 115 28006 MADRID (Espaa) Tfno.: (91) 782 43 32 / Fax: (91) 782 43 27
E-mail: secretaria@ctomedicina.com; iberocto@ctomedicina.com WEB: www.ctomedicina.com; www.iberocto.com
DM Pg. 13
DERMATOLOGA
Seguimiento a distancia Preparacin Examen de Seleccin 05/06 1 Vuelta
tratamiento es con sulfato de zinc oral. Adems de esta enfermedad frecuente verlas alrededor de la boca, genitales y pliegues. En la
hereditaria por dficit de zinc tambin se ha descrito un cuadro ad- histologa llama la atencin la necrosis de las capas epidrmicas
quirido en pacientes con malnutricin asociada a cirrosis alcohlica superiores y en algunos casos con acantlisis, por lo que puede
o pancreatitis alcohlica. parecer un pnfigo pero la inmunofluorescencia directa e indirecta
es negativa. Se han descrito tambin casos de eritema necroltico
Pregunta 60.- R: 2 migratorio en pacientes con cirrosis y pancreatitis crnica.
Las necrobiosis lipodica es una enfermedad degenerativa del co-
lgeno que clnicamente se caracteriza por placas amarillentas, con Pregunta 62.- R: 5
aspecto atrfico, que pueden llegar a ulcerarse. La causa de la necro- La esclerosis tuberosa o enfermedad de Pringle Bourneville se
biosis lipodica es desconocida. Es un hallazgo tpico en los diabticos transmite por herencia autosmica dominante. La expresin clnica
pero tan slo aparece en el 0,3% de los mismos. Cuando se asocia a de la enfermedad puede ser muy variable, incluso en pacientes de la
diabetes mellitus el curso evolutivo de las lesiones es independiente misma familia. La trada clsica que caracteriza a la enfermedad slo
del de la diabetes. Aunque en la mayora de los casos aparece des- se encuentra en el 30% de los pacientes. Esta trada comprende el
pus de establecerse la diabetes es posible incluso que la anteceda. La adenoma sebceo, retraso mental y epilepsia. La primera lesin cu-
localizacin clsica es en la cara anterior de los miembros inferiores. tnea en aparecer es la mcula hipocrmica con aspecto lanceola-
Este hecho hace que tengamos que establecer diagnstico diferencial do o en hoja de fresno. Aparecen en la infancia estando con fre-
con otras patologas que clsicamente aparecen en la cara anterior de cuencia presentes al nacimiento. Son el marcador ms temprano de
los miembros inferiores como el mixedema pretibial, eritema nodoso, la enfermedad pero no son patognomnicas. Los angiofibromas o
pioderma gangrenoso, vasculitis cutnea, etc. Adems un diagnstico adenomas sebceos de Pringle estn presentes en el 80% de los
diferencial frecuente es con el granuloma anular, pues ambas patolo- casos. Aparecen a partir de los 2 aos de edad. Clnicamente son
gas tienen una histologa similar y son lesiones cutneas que se aso- ppulas rojizas que se distribuyen de forma caracterstica, simtrica,
cian a la diabetes. El tratamiento de la necrobiosis lipodica se funda- por las zonas malares, regin perioral y paranasal. Otra de las lesio-
menta en el uso de corticoides tpicos o intralesionales obtenindose nes cutneas tpicas son los fibromas periungueales o tumores de
buenos resultados. Otras lesiones cutneas que se aprecian en los Koenen, que aparecen en el 50% de los pacientes, comienzan a
diabticos son la dermopata diabtica que se aprecia en el 50% de surgir despus de la pubertad. Las eflides no son lesiones de la
los mismos, los xantomas eruptivos, el escleredema y la lipodistrofia esclerosis tuberosa sino de la neurofibromatosis. Otras anomalas
generalizada. asociadas a la esclerosis tuberosa son los rabdomiomas cardacos,
alteraciones radiolgicas mostrando pseudoquistes en las falanges,
Pregunta 61.- R: 3 epilepsia en el 80% de los pacientes y hamartomas en la sustancia
Lesiones cutneas periorificiales, acompaadas de diarrea y prdi- gris del cerebro.
da de peso. Esa descripcin del cuadro concuerda con una acroder-
matitis enteroptica, aunque con 68 aos es evidente que la lactancia
materna le queda un poco alejada. Adems el dato de la anemia y en
especial de hiperglucemia, junto a que las lesiones cutneas van cam-
biando de localizacin nos debe hacer pensar en el eritema necroltico
migratorio.

Lesin anular perioral


caracterstica

Comentarios TEST
Glucagonoma in situ

Lesin en piegue inguinal


caracterstica

Pregunta 61. Glucagonoma y manifestaciones cutneas asociadas.

Esta dermatosis se encuentra dentro del sndrome del glucagono-


ma caracterizado por prdida de peso, diabetes, anemia y tumor
pancretico de clulas alfa secretoras de glucagn. Esta enfermedad
aparece con ms frecuencia en mujeres postmenopusicas. La en-
fermedad cutnea es el resultado de la glucagonemia. Las lesiones Pregunta 62. Esclerosis tuberosa.
cutneas desaparecen despus de extirpar el tumor productor de
glucagn. La clnica de las lesiones cutneas es muy caracterstica Pregunta 63.- R: 4
con presencia de eritema de aspecto anular, vesculas y ampollas en La neurofibromatosis tiene una herencia autosmica dominante,
el borde externo. Las lesiones tienen tendencia a diseminarse y es con penetrancia variable. El gen responsable de la neurofibromatosis

M exico A rgentina
Pg. 14 DM C hile U ruguay
CTO Medicina C/ Nez de Balboa, 115 28006 MADRID (Espaa) Tfno.: (91) 782 43 32 / Fax: (91) 782 43 27
E-mail: secretaria@ctomedicina.com; iberocto@ctomedicina.com WEB: www.ctomedicina.com; www.iberocto.com
DERMATOLOGA
Preparacin Examen de Seleccin 05/06 1 Vuelta Seguimiento a distancia
tipo I ha sido aislada en el cromosoma 17. Es una de las enfermeda- desconocida pero probablemente inmunolgica, con alteraciones
des genticas ms frecuente. Se distinguen 2 tipos siendo la clsica la tanto de la inmunidad humoral como celular.
tipo I o neurofibromatosis perifrica. La tipo II o central se caracteri- La dermatitis atpica es una enfermedad crnica que cursa en
za por tener neurofibromas que afectan de forma bilateral al acsti- brotes de lesiones muy pruriginosas y se caracteriza por la presencia
co. La manifestacin ms temprana de la neurofibromatosis son las de lesiones eccematosas. La atpica del lactante no suele aparecer
manchas caf con leche que pueden estar presentes en el nacimien- hasta despus del segundo mes de vida. En esta fase predominan las
to. No son patognomnicas pues pueden verse en otras enfermeda- lesiones eccematosas exudativas, distribuyndose por el cuero cabe-
des como el Sndrome de McCune-Albright, Sndrome de Bloom, lludo y la cara, aunque respetando el tringulo nasogeniano. La atopia
esclerosis tuberosa y en el 20% de las personas sanas. La presencia infantil puede ser el progreso de la del lactante o aparecer por primera
de 6 o ms manchas caf con leche indican alta probabilidad de vez. Son lesiones mucho ms secas y con tendencia a distribuirse por
tener neurofibromatosis. Sin embargo la lesin ms caracterstica de las flexuras. Uno de los fallos ms habituales en el examen MIR es
la neurofibromatosis son los neurofibromas cutneos que es muy considerar a la atopia infantil como si fuera una atopia del lactante. La
raro que aparezcan antes de los 6 meses de edad. La presencia de mayora de las atopias aparecen en la infancia, siendo raras las que
eflides en los pliegues (signo de Crowe) es un signo patognomnico aparecen en edad adulta. Esto sucede en el 10% de los casos. La
de la enfermedad. Otras anomalas y enfermedades asociadas son atopia del adulto aparece en la cara, el tronco y dorso de las manos.
alteraciones seas que se observan en el 50% de los pacientes, gliomas El sntoma comn en todos los eccemas atpicos es el prurito.
del nervio ptico que se aprecian en el 15% de los pacientes y la
aparicin de tumores como el rabdomiosarcoma o el tumor de Pregunta 66.- R: 5
Wilms. Ver comentario de pregunta anterior.

Pregunta 67.- R: 3
Pregunta 63. Caractersticas de las facomatosis. Son lesiones eritematosas, dolorosas, que claramente comienzan o
se desencadenan con la presin de los zapatos al caminar. La urticaria
)DFRPDWRVLV $OWHUDFLRQHVFXWiQHDVFDUDFWHUtVWLFDV por presin es una urticaria poco frecuente y representa el 2,5% de las
urticarias fsicas. La lesin que produce es similar al angioedema y se
0DQFKDVFDIpFRQOHFKH traduce por un edema profundo en el tejido celular subcutneo,
1HXURILEURPDWRVLV, 1HXURILEURPDV acompaado de linfocitos, neutrfilos y eosinfilos. Las lesiones apa-
(IpOLGHVD[LODUHV recen entre 3-6 horas despus del estmulo por presin, con una
1yGXORVGH/LVFK duracin aproximada de 24-48h. Las lesiones pueden dar prurito
1HXURILEURPDWRVLV,, (VFDVDVRQXODV pero lo ms llamativo es que resulten dolorosas. Estas lesiones pueden
aparecer en cualquier localizacin donde exista presin, por eso son
0DQFKDVKLSRFUyPLFDVODQFHRODGDV zonas frecuentes los pies, glteos y manos.
$QJLRILEURPDVIDFLDOHV El diagnstico viene dado por la historia clnica aunque pueden
(VFOHURVLVWXEHURVD )LEURPDVSHULXQJXHDOHV\VXEXQJXHDOHV emplearse diversas tcnicas para su confirmacin, mediante la aplica-
 WXPRUHVGH.RHQHQ  cin de un peso sobre la piel. Los tratamientos no suelen ser muy
3ODFDGHSLHOGH&KDJUHQ SLHOGH=DSD  eficaces. Los antihistamnicos son de nula utilidad. Pueden responder
al uso de los corticoides orales. Por eso la respuesta correcta es la 3.
6tQGURPHGH6WXUJH:HEHU 0DQFKDHQYLQRGHRSRUWR
Pregunta 68.- R: 2
(QIHUPHGDGGH 3RFRIUHFXHQWHVHLQHVSHFtILFDV La urticaria colinrgica es una urticaria muy caracterstica que apa-
9RQ+LSSHO/LQGDX rece cuando se producen aumentos bruscos de la temperatura cor-
6tQGURPHGH poral y que cursa con la aparicin de lesiones habonosas de peque-
%ORFK6XO]EHUJHU +LSHUSLJPHQWDFLyQ o tamao. Por ese motivo la respuesta falsa es la 2. Representa el 30%
de las urticarias fsicas siendo ms frecuente en adolescentes y adultos
jvenes. Es frecuente su asociacin con el dermografismo y la urticaria
Pregunta 64.- R: 4 por fro. La denominacin de colinrgica le viene por tener una sus-
Comentarios TEST

La respuesta 1 es correcta ya que al tratarse de una lesin exuda- ceptibilidad aumentada a la acetilcolina ya que la inyeccin
tiva estar indicada la aplicacin de fomentos. La respuesta 2 tambin intracutnea de acetilcolina o de pilocarpina inducen la aparicin de
es correcta pues el tratamiento de todos los eccemas se fundamenta lesiones habonosas. El cuadro tpico es el de un paciente que al poco
en la administracin de corticoides tpicos u orales en funcin de la de tiempo de haber comenzado a hacer ejercicio, que conlleve un
gravedad e intensidad de las lesiones. En cuanto a la respuesta 3, la aumento de la temperatura corporal y de la sudoracin, comienza
realizacin de pruebas epicutneas estar indicada pues si sospecha- con lesiones habonosas, de pequeo tamao, redondeadas, que se
mos un eczema de contacto alrgico ser til saber cul es el o los distribuyen sobre todo por el tercio superior del tronco. En ocasiones
alergenos implicados. La respuesta 5 tambin es correcta pues los el brote de lesiones pueden acompaarse de nuseas, cefalea o tras-
antihistamnicos tpicos no son muy eficaces y pueden producir sen- tornos gastrointestinales. Otras situaciones que pueden desencadenar
sibilizaciones con lo que agravaran o enmascararan el proceso. La las crisis son los alimentos ricos en especias, baos o duchas con agua
respuesta falsa es la 4 pues el cromo no es un alergeno frecuente en los muy caliente y en situaciones de tensin emocional. Responde bien a
eccemas de contacto alrgico en mujeres. El alergeno ms frecuente- la hidroxicina.
mente implicado a mujeres es el nquel. Su fuente de exposicin ms
frecuente es la bisutera. Hemos pensado que la paciente tena un Pregunta 69.- R: 2
eccema de contacto alrgico porque las lesiones son pruriginosas. En Estamos ante un paciente que presenta desde hace una semana
las orejas la causa ms frecuente son el empleo de gotas ticas y de lesiones muy pruriginosas distribuidas por toda la superficie corporal.
bisutera. En los hombres la fuente principal de sensibilizacin son los El antecedente que claramente relacionamos con el proceso, es la
cementos que contienen sales de cromo en su composicin. En mu- ingesta de captopril 3 semanas antes. Bien, la reaccin cutnea ms
chas ocasiones se va a acompaar de sensibilizacin concomitante a frecuente ocasionada por medicamentos es el exantema, que tiende
sales de cobalto y de nquel. a confluir y a ser muy pruriginoso. Los medicamentos ms frecuente-
mente implicados son los betalactmicos, sulfamidas, captopril, AINES
Pregunta 65.- R: 5 y las hidantonas.
La dermatitis atpica es un proceso muy frecuente que afecta al Por tanto nos encontramos al principio con un exantema medica-
10% de la poblacin general. Suele asociarse a otras patologas como mentoso por captopril. Lo que sucede es que en las ltimas 48 horas
el asma alrgico y la rinitis alrgica. Es una enfermedad de causa ha empeorado y entonces comienza con despegamientos cutneos y

M exico A rgentina
C hile U ruguay
CTO Medicina C/ Nez de Balboa, 115 28006 MADRID (Espaa) Tfno.: (91) 782 43 32 / Fax: (91) 782 43 27
E-mail: secretaria@ctomedicina.com; iberocto@ctomedicina.com WEB: www.ctomedicina.com; www.iberocto.com
DM Pg. 15
DERMATOLOGA
Seguimiento a distancia Preparacin Examen de Seleccin 05/06 1 Vuelta
mucosos. Es decir el cuadro ha evolucionado hacia una necrlisis mos aos est en discusin la verdadera existencia de este sndrome
epidrmica txica. dado que las queratosis seborreicas son muy frecuentes en personas
Este proceso es la reaccin cutnea ms grave ocasionada por mayores.
medicamentos. Se caracteriza por la aparicin de erosiones y ampo-
llas flccidas, con prdida de la epidermis de extensas zonas de la Pregunta 73.- R: 2
superficie corporal. Adems aparecen despegamientos mucosos ex- La acantosis nigricans cursa con la aparicin de placas hiperquera-
tensos. Aunque no se conoce exactamente el por qu aparece el tsicas de coloracin griscea o azulada, que aparecen por zonas de
cuadro, existen evidencias indirectas que abogan por un mecanismo flexin, especialmente en regin cervical y axilas. Podemos distinguir
inmunolgico. Por eso el tratamiento se fundamenta en medidas de una acantosis nigricans benigna puede aparecer de forma familiar,
soporte y en el uso de inmunodepresores como corticoides, ciclofos- autosmica dominante o asociada a patologas como endocrinopat-
famida, ciclosporina o azatioprina. as, obesidad, uso prolongado de corticoides, enfermedad de Wilson,
etc. La acantosis nigricans maligna se denomina as porque en la ma-
Pregunta 70.- R: 2 yora de los casos se asocia a un carcinoma interno. Las lesiones
Nuevamente tenemos como antecedente el que el paciente hu- cutneas pueden coexistir con el tumor o incluso antecederlo en
biera ingerido un frmaco 5 das antes. En este caso estamos ante una aos. En el 60% de los casos se trata de adenocarcinomas gstricos. La
lesin nica y que el paciente refiere que tuvo episodios similares clnica de todas las acantosis nigricans es muy parecida diferencin-
tiempo atrs. El exantema fijo medicamentoso se caracteriza por la dose nicamente en la intensidad y extensin de las lesiones. Las
presencia de lesiones en piel y/o mucosas. Suelen ser lesiones macu- lesiones de la acantosis nigricans maligna se desarrollan rpidamente,
losas, eritematosas y ocasionalmente ampollosas. Cuando la adminis- siendo ms severas que las benignas. Con mucha frecuencia presen-
tracin del medicamento se interrumpe las lesiones pierden actividad tan una queratodermia amarillenta en palmas y plantas y en el 50%
dejando una lesin residual pigmentada. Por eso decimos exantema de los casos existe afectacin de mucosas con lesiones hipertrficas
fijo medicamentoso, fijo que deja marca. Cuando se vuelve a ingerir el en mucosa oral. La acantosis nigricans maligna regresa al extirpar el
medicamento las lesiones vuelven a aparecer en los mismos sitios que tumor asociado.
la vez anterior, es decir donde quedaron las marcas. Tambin pueden
aparecer en nuevas localizaciones. Para ayudarnos a recordarlo po- Pregunta 74.- R: 2
demos decir entonces que el exantema fijo sale en los mismos sitios. La telangiectasia macular eruptiva perstans es una rara variante de
Normalmente aparecen como lesin nica o en nmero escaso. La mastocitosis que aparece limitada a la piel. Por ese motivo la respuesta
localizacin de las lesiones es muy variada pero son lugares tpicos las falsa es la 2. Esta variante de mastocitosis es caracterstica de los adul-
mucosas genitales. tos. Clnicamente se caracteriza por la aparicin de mculas de unos
Por eso antes lesiones ampollosas, o erosivas en las mucosas debe- 5 mm, eritematosas, difusas, con mltiples telangiectasias, que a dife-
mos incluir siempre al exantema fijo medicamentoso como diagns- rencia de las araas vasculares no poseen el vaso central.
tico diferencial. Por todo lo comentado, la respuesta falsa es la 2. Las mastocitosis sistmicas se caracterizan por la proliferacin de
mastocitos a nivel de la mdula sea, bazo e hgado. Casi siempre
Pregunta 71.- R: 3 aparecen en mastocitosis de aparicin tarda, aunque tambin aque-
El vitligo es una enfermedad bastante frecuente que se caracteriza llas mastocitosis infantiles que persisten en la vida adulta presentan
por la desaparicin de los melanocitos en algunas zonas o en toda la una alta probabilidad de desarrollo de enfermedad sistmica.
piel.
Afecta al 1% de la poblacin y el 25% de los pacientes tienen Pregunta 75.- R: 3
antecedentes familiares. La etiologa del vitligo es desconocida aun- El Sndrome de Sturge-Weber se caracteriza por la asociacin de
que existen dos teoras, una basada en que un mediador neuroqumico un nevus en mancha de Oporto asociado a una angiomatosis Lepto-
destruira los melanocitos (recordad que el melanocito deriva de la menngea occipital. El nevus "flammeus" aparece de forma unilateral
cresta neural) y otra teora implicara el sistema inmune. El comienzo en las ramas oftlmicas y maxilar del trigmino, pudiendo extenderse
del vitligo puede ser brusco o gradual, puede existir con factor hasta la lnea media. Aparece asociado a una angiomatosis leptome-
desencadenante a agravante como los traumatismos ya que es una nngea homolateral que cursa con epilepsia en el 90% de los casos.
enfermedad con fenmeno de isomorfismo de Koebner. La despig- Pueden tener tambin afectacin ocular y un cierto retraso mental. La
mentacin puede ocurrir en cualquier parte del cuerpo pero hay angiomatosis leptomenngea puede tambin manifestarse como he-
reas de despigmentacin caractersticas como las prominencias seas miplejias transitorias o permanentes. El diagnstico es sencillo pues se

Comentarios TEST
(codos y rodillas). Tambin es frecuente su distribucin periorificial. A establece ante la asociacin de Nevus en mancha de Oporto y epilep-
menudo el vitligo, en especial el generalizado, presenta lesiones sim- sia. En los estudios radiolgicos se aprecian calcificaciones corticales.
tricas como afirma la respuesta 3. Los pacientes con vitligo es ms Otra enfermedad en la que aparece el Nevus Flammeus o en mancha
frecuente que tengan otras enfermedades autoinmunes, como las de Oporto es en el Sndrome de Klippel-Trenaunay. Este sndrome se
patologas tiroideas que son las ms frecuentes. Los pacientes con caracteriza por la asociacin de nevus flammeus que suele aparecer
melanoma, en especial los metastsicos, pueden sufrir vitligos incluso en un miembro con hipertrofia de partes blandas, habitualmente miem-
a distancia de la localizacin del melanoma. El tratamiento del vitligo bros inferiores. Puede existir tambin una insuficiencia venosa crni-
no es satisfactorio, emplendose corticoides tpicos y PUVA. ca en el miembro afecto.

Pregunta 72.- R: 3
La queratosis seborreica es un tumor benigno muy frecuente en la
raza blanca. Aparece a partir de los 50 aos y a lo largo de la vida
puede sufrir regresiones espontneas. Aparecen sobre piel previamente
sana, en tronco, cara y cuello. Suelen ser lesiones papilomatosas,
marronceas e incluso negras. Al tacto es caracterstica la sensacin
grasienta o untuosa. No se han descrito transformaciones malignas de
esta patologa. Su histologa se caracteriza por ser proliferaciones
epiteliales con hiperqueratosis y formacin de quistes crneos. El tra-
tamiento es el curetaje, crioterapia o electrocoagulacin de las mis-
mas cuando plantean problemas mecnicos o estticos. Cuando apa-
recen mltiples queratosis seborreicas, podemos encontrarnos ante el
signo de Leser-Trelat que consiste en la aparicin sbita de queratosis
seborreicas acompaando a una neoplasia. En el 60% de los casos el
tumor maligno encontrado es una neoplasia abdominal. En los lti-

M exico A rgentina
Pg. 16 DM C hile U ruguay
CTO Medicina C/ Nez de Balboa, 115 28006 MADRID (Espaa) Tfno.: (91) 782 43 32 / Fax: (91) 782 43 27
E-mail: secretaria@ctomedicina.com; iberocto@ctomedicina.com WEB: www.ctomedicina.com; www.iberocto.com
DIGESTIVO
Preparacin Examen de Seleccin 05/06 1 Vuelta Seguimiento a distancia

1. En cul de los siguientes trastornos esperara encontrar una 4) Sndrome de Sjgren.


disfagia orofarngea?: 5) Esclerodermia.

1) Enfermedad de Parkinson. 7. Varn de 62 aos, con antecedente de pirosis de larga


2) Esfago de Barrett. evolucin y, ocasionalmente, regurgitacin de alimentos.
3) Acalasia. Consulta porque, desde hace tres meses, nota dolor retroes-
4) Anillo de Schatzki. ternal con la deglucin y disfagia progresiva. Cul de las
5) Hernia hiatal. siguientes pruebas diagnsticas debera solicitar a continua-
cin?:
2. Seale cul es el tumor que con ms frecuencia da lugar a una
acalasia secundaria: 1) Ecografa abdominal.
2) Esofagograma con bario.
1) Carcinoma epidermoide de esfago. 3) Endoscopia.
2) Adenocarcinoma gstrico. 4) pHmetra.
3) Carcinoma de pulmn. 5) Test de Bernstein.
4) Linfoma.
5) Carcinoma de pncreas. 8. Un paciente de 63 aos ha sido estudiado en un hospital por
dolor torcico, y en una endoscopia digestiva alta con toma
3. Una de las siguientes NO suele verse como complicacin de de biopsias le han diagnosticado de esfago de Barrett, sin
la acalasia. Selela: observarse displasia. Le consulta a usted para que le aconseje
sobre su enfermedad; cul de las siguientes consideraciones
1) Disminucin de peso. NO sera correcta en este paciente?:
2) Dficit de vitamina B12.
3) Carcinoma esofgico. 1) Debera hacerse usted endoscopia y biopsias cada 1,5-2
4) Aspiracin pulmonar. aos.
5) Esofagitis. 2) Dado el cuadro clnico que presenta el paciente, debera
usted tomar omeprazol.
4. Mujer de 48 aos con diagnstico de acalasia que, tras 3) Su enfermedad se debe probablemente al reflujo gastro-
someterse a dos dilataciones neumticas, presenta recu- esofgico.
rrencia de su disfagia. Cul de los siguientes tratamientos 4) Tiene usted riesgo de adenocarcinoma de esfago, slo si
sera considerado de eleccin?: existe colonizacin por H. pylori.
5) El omeprazol probablemente no le revertir el esfago de
1) Miotoma de Heller. Barrett.
2) Reseccin esofgica.
3) Toxina botulnica. 9. Seale la afirmacin correcta en relacin al carcinoma de
4) Nitritos orales. esfago:
5) Funduplicatura laparoscpica.
1) Histolgicamente es un adenocarcinoma en menos del
5. Una mujer de 42 aos consult por dolor precordial irradia- 10% de los casos.
do a espalda. Aportaba un extenso estudio cardiolgico, 2) La incidencia es similar en todas las regiones del mundo.
incluyendo una coronariografa que era normal. Un estudio 3) Suelen diagnosticarse precozmente por la disfagia.
radiolgico con bario y una esofagogastroscopia fueron 4) La presencia de una lcera en el esofagograma descarta el
normales. Se realiz una manometra que demostr, tras la diagnstico de carcinoma.
deglucin, contracciones simultneas de gran amplitud en la 5) Actualmente la supervivencia a los 5 aos es del 50%.
parte inferior del esfago; la presin basal del esfnter eso-
Preguntas TEST

fgico inferior estaba ligeramente aumentada, pero se rela- 10. Seale cul es el mejor mtodo para la estadificacin T de los
jaba durante la deglucin. Qu diagnstico inicial hara a tumores esofgicos:
esta paciente?:
1) Endoscopia.
1) Acalasia cricofarngea. 2) Radiologa con contraste.
2) Acalasia vigorosa. 3) Ultrasonografa endoscpica.
3) Espasmo esofgico difuso. 4) TC.
4) Probable amiloidosis del esfago. 5) Laparoscopia.
5) Anillo esofgico tipo B.
11. Es cierto que, en la infeccin por H. pylori:
6. Si en el estudio de un paciente se encuentra con aperistalsis
en los dos tercios inferiores del esfago con hipotensin del 1) La bacteria se identifica en la lmina propia de la mucosa.
esfnter esofgico inferior, qu enfermedad explicara me- 2) Se asocia siempre a colonizacin de la mucosa duodenal.
jor el cuadro?: 3) El cultivo es la prueba diagnstica ms sensible.
4) La tasa de portadores sanos es muy elevada.
1) Acalasia clsica. 5) Se visualiza al microscopio ptico como Gram negativo y
2) Hernia hiatal paraesofgica. flagelado.
3) Sndrome de Boerhaave.

M exico A rgentina CTO Medicina C/ Nez de Balboa, 115 28006 MADRID (Espaa) Tfno.: (91) 782 43 32 / Fax: (91) 782 43 27
C hile U ruguay E-mail: secretaria@ctomedicina.com; iberocto@ctomedicina.com WEB: www.ctomedicina.com; www.iberocto.com
DG Pg. 1
DIGESTIVO
Seguimiento a distancia Preparacin Examen de Seleccin 05/06 1 Vuelta
12. El aumento plasmtico de la gastrina-17 en un paciente con 2) Un 10% de las lceras duodenales recidivan.
gastritis crnica atrfica suele deberse en primer lugar a: 3) Las lceras duodenales suelen ser ms grandes que las
gstricas.
1) Asociacin a carcinoma gstrico. 4) Las lceras duodenales aparecen sobre todo en la primera
2) Gastritis por alcohol. porcin del duodeno.
3) Tratamiento con sucralfato. 5) Las lceras del fundus siempre son benignas.
4) Desarrollo de un tumor carcinoide.
5) Presencia de hipoclorhidria. 18. En el tratamiento de un paciente con ulcus duodenal sangran-
te y test de la ureasa positivo, son correctas las siguientes
13. Mujer de 46 aos, sin antecedentes de inters, que consulta decisiones, EXCEPTO:
por un episodio mensual de migraa que cede en 24 horas
con 2 dosis de ibuprofeno. No presenta sntomas disppticos 1) Test del aliento para H. pylori, para confirmar la erra-
y se decide continuar con el mismo tratamiento analgsico. dicacin.
En relacin con la profilaxis de la lcera pptica por AINEs, 2) Tratar con metronidazol, si hay alergia a amoxicilina.
sera adecuado recomendar: 3) Tratamiento con bismuto coloidal, si presenta intoleran-
cia a claritromicina.
1) Sucralfato. 4) Cultivo de la mucosa antral para H. pylori, si hay fracasos
2) Misoprostol. teraputicos.
3) Famotidina. 5) Evitar biopsias de los bordes de la lcera.
4) Lansoprazol.
5) Ninguna medida farmacolgica. 19. Para confirmar la erradicacin de H. pylori, es correcto
afirmar que:
14. Mujer de 52 aos, recientemente diagnosticada de Addison,
que es remitida a consulta por presentar ttulos muy elevados 1) Debe suspenderse previamente la toma de omeprazol.
de anticuerpos anticlula parietal gstrica. Actualmente se 2) Es de eleccin el cultivo de la mucosa antral.
encuentra asintomtica. La Hb es de 9,6 g/dl y el volumen 3) El test del aliento es poco sensible.
corpuscular es de 113 fl. Con estos datos, qu aconsejara 4) Slo est indicada ante la sospecha de resistencia bacte-
a la paciente?: riana.
5) Debe realizarse siempre en las lceras duodenales.
1) Realizar una gastroscopia y solicitar B12.
2) Un estudio baritado gastroduodenal. 20. En caso de ulcus gstrico, es correcto afirmar que:
3) Repetir la determinacin de los Ac anticlula parietal.
4) Tratamiento inmediato con cido flico. 1) Se debe solicitar gastrina basal.
5) Revisiones peridicas, ya que la anemia se corregir con 2) La biopsia de las zonas ms atrficas es til para demostrar
el tratamiento para el Addison. H. pylori.
3) La erradicacin de H. pylori precisa 14 das de trata-
15. A un varn de 46 aos se le remite a consultas para realizar miento .
una gastroscopia por dispepsia refractaria a tratamiento con 4) Las localizadas en fundus suelen necesitar ciruga.
ranitidina. En la analtica se observa Hto normal y ligera 5) Debe realizarse mantenimiento con un frmaco antise-
hipoalbuminemia con VSG normal. El estudio endoscpico cretor.
demuestra en fundus gstrico pliegues muy engrosados. Con
estos datos, cul sera el diagnstico ms probable?: 21. Un paciente de 76 aos, con antecedente de ulcus gstrico
sangrante por H. pylori al que se le erradic con xito la
1) Adenocarcinoma gstrico tipo intestinal. infeccin, consulta porque se le ha prescrito piroxicam por
2) Gastritis crnica tipo A. una artritis traumtica de rodilla. Cul de las siguientes

Preguntas TEST
3) Gastritis eosinoflica. medidas de gastroproteccin recomendara a continuacin?:
4) Mntrier.
5) Gastritis aguda por H. pylori. 1) Acexamato de Zinc.
2) Omeprazol oral.
16. Cul de los siguientes AINEs tiene menor potencial gas- 3) No precisa ninguna.
troerosivo?: 4) Ranitidina-bismuto al menor sntoma.
5) Sucralfato, y si hay sntomas, aadir misoprostol.
1) Meloxicam.
2) Diclofenaco. 22. A un varn de 56 aos, sin antecedentes de inters, excepto
3) Rofecoxib. tabaquismo moderado, se le ha prescrito ibuprofeno por
4) Ibuprofeno. dolor articular. Cul de las siguientes medidas es la ms
5) Nabumetona. adecuada?:

17. Seale la correcta: 1) Parches de nicotina, para evitar riesgo de ulcus.


2) Misoprostol, 3-4 veces al da.
1) La lcera gstrica se perfora con ms frecuencia que la 3) Administrar rofecoxib en vez de ibuprofeno.
duodenal. 4) Recomendar dieta rica en lcteos.
5) Ranitidina y magaldrato, si tiene dispepsia.

M exico A rgentina CTO Medicina C/ Nez de Balboa, 115 28006 MADRID (Espaa) Tfno.: (91) 782 43 32 / Fax: (91) 782 43 27
Pg. 2 DG C hile U ruguay E-mail: secretaria@ctomedicina.com; iberocto@ctomedicina.com WEB: www.ctomedicina.com; www.iberocto.com
DIGESTIVO
Preparacin Examen de Seleccin 05/06 1 Vuelta Seguimiento a distancia

23. Seale cul de los siguientes NO es un factor predisponente 5) Es excepcional en los gastrectomizados.
de cncer gstrico:
30. Un varn de 14 aos consulta por diarrea crnica. Presenta
1) Esfago de Barrett. ascitis y edemas en miembros inferiores, ms intensos sobre
2) Infeccin por H. pylori. todo en el miembro inferior derecho. En los anlisis aparece
3) Sndrome de Lynch tipo II. hipoalbuminemia y linfopenia. La serologa de VIH es nega-
4) Anemia perniciosa. tiva, no hay proteinuria y el aclaramiento de alfa-1-antitrip-
5) Alcoholismo crnico. sina est aumentado. La paracentesis muestra una ascitis
quilosa. Se realiza una biopsia intestinal que confirma el
24. En un enfermo con diarrea y fiebre de 7 das de evolucin, diagnstico clnico de:
cul de las siguientes pruebas diagnsticas indicara en
primer lugar?: 1) Iletis tuberculosa.
2) Enfermedad de Whipple.
1) Sigmoidocolonoscopia. 3) Esprue refractario.
2) Leucocitos fecales. 4) Giardiasis.
3) Grasas fecales. 5) Linfangiectasia intestinal.
4) Coprocultivo.
5) Trnsito intestinal. 31. Varn de 80 aos, con antecedente de dos episodios de
hemorragia digestiva alta por lcera duodenal hace 30 aos,
25. En la enfermedad celaca, NO es un hallazgo caracterstico: y estenosis artica con insuficiencia cardaca grado II. Con-
sulta por presentar, hace diez das, rectorragias que le
1) Los anticuerpos IgA antitransglutaminasa. duraron dos das. Actualmente contina asintomtico. Una
2) Asociacin a HLA B51. sigmoidocolonoscopia hasta ngulo esplnico no demuestra
3) Hiperplasia crptica. lesiones. Qu medida diagnstica recomendara a continua-
4) Infiltracin linfoplasmocitaria de la lmina propia. cin?:
5) Linfocitos intraepiteliales.
1) Nueva colonoscopia hasta ciego.
26. Cul le parece la causa ms frecuente de que un paciente con 2) Arteriografa.
esprue celaco contine con sntomas tras iniciar tratamien- 3) Gastroduodenoscopia.
to?: 4) Estudio baritado gastroduodenal.
5) Enema opaco.
1) Linfoma.
2) Mala realizacin de la dieta. 32. Cul de las siguientes pruebas realizara en primer lugar tras
3) Esprue colgeno. una hematoquecia?:
4) Neoplasia esofgica.
5) Error diagnstico. 1) Sigmoidoscopia.
2) Colonoscopia.
27. Entre las siguientes, es una causa de diarrea secretora: 3) Enema opaco.
4) Gastroduodenoscopia.
1) Ingesta de hidrxido de magnesio. 5) Arteriografa.
2) Sndrome del intestino irritable.
3) Administracin de lactulosa. 33. La presencia de anemia ferropnica y melenas en una mujer
4) Sndrome carcinoide. que tiene 23 aos, sugiere fundamentalmente el diagnstico
5) Diarrea postvagotoma con piloroplastia. de:
Preguntas TEST

28. Las siguientes enfermedades pueden cursar con entero-pata 1) Ulcus duodenal.
y prdida de protenas, EXCEPTO: 2) Divertculo de Meckel.
3) Cncer de colon.
1) Carcinoma esofgico. 4) Poliposis rectal.
2) Pericarditis constrictiva. 5) Adenocarcinoma gstrico.
3) Enfermedad de Whipple.
4) Colitis granulomatosa. 34. Varn de 16 aos, con antecedentes de 2 3 episodios
5) Hipertiroidismo. anuales de dolor en fosa ilaca derecha, que duran entre
2 y 5 das. Habitualmente se acompaan de diarrea y se
29. En el sobrecrecimiento bacteriano, es cierto todo lo si- resuelven de forma espontnea. Consulta porque, en el
guiente, EXCEPTO: ltimo episodio, el dolor es ms intenso, se acompaa de
fiebre y tres deposiciones al da. En la exploracin fsica
1) La esteatorrea se debe a desconjugacin de las sales se encuentra con buen estado general. En la analtica
biliares. destaca leucocitosis. El coprocultivo es negativo. En una
2) Los test del aliento son tiles para el diagnstico. sigmoidocolonoscopia se aprecian lceras aftoides en
3) Suele haber recurrencias. ciego y colon derecho. En las biopsias se aprecia un
4) Puede haber atrofia intestinal leve. infiltrado inflamatorio inespecfico. Cul, de entre las

M exico A rgentina CTO Medicina C/ Nez de Balboa, 115 28006 MADRID (Espaa) Tfno.: (91) 782 43 32 / Fax: (91) 782 43 27
C hile U ruguay E-mail: secretaria@ctomedicina.com; iberocto@ctomedicina.com WEB: www.ctomedicina.com; www.iberocto.com
DG Pg. 3
DIGESTIVO
Seguimiento a distancia Preparacin Examen de Seleccin 05/06 1 Vuelta
siguientes opciones teraputicas, considera la ms apro- 2) Ecografa abdominal.
piada para este paciente?: 3) Enema opaco.
4) Radiografa de abdomen.
1) Antagonistas de TNF. 5) TC abdominal.
2) Esteroides.
3) Metronidazol y mesalazina. 40. En la colitis isqumica subaguda, es cierto que:
4) Azatioprina.
5) 6-Mercaptopurina. 1) Se produce por la obstruccin proximal de la arteria
clica izquierda.
35. Seale la correcta: 2) El recto est afectado en el 80% de los casos.
3) La angiografa es la tcnica diagnstica de eleccin.
1) Puede haber ANCA positivos en la colitis ulcerosa. 4) Las imgenes en huella dactilar la diferencian de la
2) La colitis ulcerosa es ms frecuente en fumadores. forma aguda.
3) En el 50% de las personas con colitis ulcerosa aparece una 5) Los sntomas remiten de forma habitual en 3 4 semanas
colangitis esclerosante. y no recidivan.
4) La colitis ulcerosa afecta siempre y de forma uniforme a
todo el colon. 41. Ante la sospecha de una diverticulitis complicada con un
5) Es caracterstico de la colitis ulcerosa la presencia de absceso de pared, solicitara como prueba diagnstica ms
hemorragias sin moco. rentable:

36. Una paciente de 32 aos consulta por presentar, desde 2 1) TC.


semanas antes, diarrea sanguinolenta de 6 deposiciones dia- 2) Eco abdominal.
rias, dolor abdominal y febrcula. Dos coprocultivos fueron 3) Rx lateral de abdomen.
negativos. Una colonoscopia demostr una mucosa eritema- 4) Enema opaco.
tosa y granular con hemorragias puntuales desde recto a 5) Ecografa hidroclica.
ngulo esplnico del colon. La biopsia fue compatible con el
diagnstico de enfermedad inflamatoria crnica intestinal. 42. En relacin a la hemorragia diverticular, es FALSO:
Cul de las siguientes actitudes sera ms correcta?:
1) Suele originarse en el colon derecho.
1) Mesalazina y metronidazol. 2) Es una complicacin poco frecuente de la enfermedad
2) Sulfasalazina sola. diverticular del colon.
3) Esteroides y metronidazol. 3) En la mayora de los casos, la hemorragia recidiva.
4) Mesalazina y esteroides. 4) Suele presentarse como hematoquecia indolora.
5) Ciprofloxacino y mesalazina. 5) Suele dejar de sangrar espontneamente.

37. Uno de los siguientes hallazgos es ms frecuente en la colitis 43. Seale cul de entre las siguientes caractersticas de los
ulcerosa que en la enfermedad de Crohn. Selelo: plipos de colon NO se correlaciona con el potencial de
malignizacin:
1) Dolor abdominal.
2) Prdida de peso. 1) El tamao.
3) Masa abdominal. 2) El tipo histolgico.
4) Friabilidad al estudio endoscpico. 3) El grado de displasia.
5) Fstulas. 4) La localizacin.
5) La multiplicidad.
38. En el tratamiento de la enfermedad de Crohn, las siguientes

Preguntas TEST
pautas son adecuadas, EXCEPTO en el caso de: 44. El sntoma ms comn de los plipos colorrectales es:

1) Esteroides, como mantenimiento tras brote leve-mo-dera- 1) Intususcepcin.


do, ya que garantizan una remisin prolongada. 2) Prolapso.
2) Azatioprina, ante enfermedad crnicamente activa. 3) Rectorragia.
3) En brotes graves, puede indicarse ciclosporina i.v. 4) Estreimiento.
4) Colestiramina puede usarse en casos de afectacin ileal 5) Diarrea.
severa, para control de la diarrea.
5) Sulfasalazina es eficaz en la colitis activa. 45. Un varn de 66 aos, sin antecedentes personales ni familia-
res de inters, consulta por hematoquecia. En una colonos-
39. Una mujer de 35 aos, diagnosticada de colitis ulcerosa 4 copia se le extirpan 3 plipos adenomatosos sin signos de
aos antes, ingresa con fiebre, distensin abdominal doloro- displasia en sigma y colon izquierdo. En relacin con el
sa, diarrea de mltiples deposiciones y signos de deshi- cribado del cncer de colon, sera lo ms apropiado someter
dratacin. Presenta leucocitosis y anemia. Qu prueba peridicamente al paciente a:
diagnstica realizara en primer lugar?:
1) Test de hemorragias ocultas.
1) Colonoscopia. 2) Colonoscopia.

M exico A rgentina CTO Medicina C/ Nez de Balboa, 115 28006 MADRID (Espaa) Tfno.: (91) 782 43 32 / Fax: (91) 782 43 27
Pg. 4 DG C hile U ruguay E-mail: secretaria@ctomedicina.com; iberocto@ctomedicina.com WEB: www.ctomedicina.com; www.iberocto.com
DIGESTIVO
Preparacin Examen de Seleccin 05/06 1 Vuelta Seguimiento a distancia

3) Eco hidroclica. 51. Un paciente de 44 aos consult por rectorragias. Una


4) Niveles de antgeno CEA. colonoscopia demostr un cncer de sigma y mltiples
5) Si est asintomtico, solamente revisiones en consulta. plipos adenomatosos en todo el colon. A su hermana de 47
aos se le realiz una sigmoidoscopia que fue normal. Qu
46. En la aparicin de un carcinoma colorrectal, NO se considera tipo de seguimiento habra que hacerle a la hermana?:
un factor importante la existencia previa de:
1) Rectosigmoidoscopia cada 2 aos.
1) Antecedentes familiares. 2) Colonoscopia cada 3 aos.
2) Sndrome del colon irritable. 3) Enema opaco anual.
3) Colitis ulcerosa. 4) Ningn seguimiento.
4) Poliposis colorrectal. 5) Proponerle colectoma, aunque no tenga plipos, y en-
5) Sndrome de los adenomas planos. doscopia alta cada 2 aos, posteriormente.

47. En relacin a la asociacin entre el adenocarcinoma de colon 52. Cul de las siguientes alteraciones NO se asocia con un
y la colitis ulcerosa, seale la que es INCORRECTA: aumento aislado de la bilirrubina indirecta srica?:

1) El riesgo depende de la extensin de la enfermedad. 1) Ictericia fisiolgica del recin nacido.


2) El riesgo se relaciona con la duracin de la enfermedad. 2) Ictericia por lactancia materna.
3) El riesgo se relaciona con el grado de displasia. 3) Ictericia hemoltica.
4) Se recomienda colectoma profilctica a los 15 aos de 4) Sndrome de Gilbert.
evolucin de la enfermedad. 5) Sndrome de Dubin-Johnson.
5) Existe mayor riesgo cuando la enfermedad se presenta a
edades ms jvenes. 53. Una paciente de 29 aos consulta por padecer astenia leve.
En la exploracin fsica se aprecia una dudosa ictericia y el
48. La presencia de cncer de colon en varios miembros de una resto es normal. Se realiza una analtica de orina urgente que
familia, a edades inferiores a los 50 aos y sin evidencia de demuestra dos cruces de bilirrubina. Cul de los siguientes
poliposis asociada, le sugiere un sndrome de: trastornos sera MENOS probable?:

1) Lynch tipo I. 1) Coledocolitiasis.


2) Gardner. 2) Ampuloma.
3) Muir-Torre. 3) Hepatitis.
4) Cowden. 4) Colestasis benigna recurrente.
5) Peutz-Jeghers. 5) Sndrome de Gilbert.

49. En el seguimiento de un paciente intervenido de carcinoma 54. Cul es la primera exploracin instrumental que debe rea-
de colon estadio C, la elevacin del antgeno CEA significa: lizarse en un paciente con colestasis?:

1) Recidiva tumoral. 1) Colecistografa oral.


2) Indicacin de test de hemorragias ocultas. 2) Colangiografa intravenosa.
3) Error inicial en el estadiaje. 3) Colangiografa retrgrada endoscpica.
4) Sndrome de Lynch. 4) Ecografa.
5) Una segunda neoplasia a nivel rectal. 5) TC.

50. Mujer de 46 aos, con antecedente de enfermedad celaca en 55. Seale cul de las siguientes NO es una caracterstica del
la adolescencia, que consult por presentar diarrea acuosa sndrome alcohol-paracetamol:
Preguntas TEST

de 3 meses de evolucin. En ocasiones se acompaaba de


moco, pero no de sangre ni pus. Un test de grasas fecales fue 1) Puede producir un fallo heptico agudo.
de 3 g/da. Los coprocultivos y estudios de parsitos fecales 2) Est descrito incluso con el empleo de dosis teraputicas
fueron negativos. Los anticuerpos antigliadina y antiendomi- de paracetamol.
sio y los ANCA fueron negativos. Las biopsias de duodeno y 3) Tiene un buen pronstico, en relacin al fallo heptico
yeyuno mostraron una mucosa normal. En una posterior agudo en general.
colonoscopia se observ una mucosa ligeramente enrojeci- 4) Es una de las causas ms frecuentes de fallo heptico
da en sigma y colon izquierdo. En el estudio histolgico se agudo.
demostr un infiltrado predominantemente crnico con 5) Las transaminasas no suelen estar muy elevadas.
linfocitos y plasmticas, sin microabscesos ni granulomas.
Cul de los siguientes diagnsticos es el ms probable?: 56. El tratamiento de eleccin de una ascitis a tensin es:

1) Sndrome de intestino irritable. 1) Dieta hiposdica y de restriccin hdrica.


2) Colitis ulcerosa. 2) Espironolactona 100 mg/da.
3) Celaca colgena. 3) Paracentesis evacuadora y reposicin con albmina.
4) Hipertiroidismo. 4) Espironolactona asociado a furosemida.
5) Colitis linfoctica. 5) Colocacin de TIPS.

M exico A rgentina CTO Medicina C/ Nez de Balboa, 115 28006 MADRID (Espaa) Tfno.: (91) 782 43 32 / Fax: (91) 782 43 27
C hile U ruguay E-mail: secretaria@ctomedicina.com; iberocto@ctomedicina.com WEB: www.ctomedicina.com; www.iberocto.com
DG Pg. 5
DIGESTIVO
Seguimiento a distancia Preparacin Examen de Seleccin 05/06 1 Vuelta
57. Un paciente de 47 aos consult por molestias abdominales episodio de hematemesis. En una endoscopia se observan
inespecficas y aumento del permetro abdominal. En la varices grado IV con puntos sangrantes. Tras 48 horas de
exploracin fsica se detect matidez en ambos flancos, y la iniciar el tratamiento con somatostatina y esclerosis de las
ecografa confirm la presencia de lquido libre peritoneal. varices, contina sangrando con descenso notable de la TA
El estudio del lquido asctico demostr un gradiente albmi- y del hematcrito. Las siguientes medidas podran estar
na srica/albmina en ascitis de 3,1. Cul de estos diagns- indicadas, EXCEPTO:
ticos sera MENOS probable?:
1) Quinolonas orales.
1) Sndrome de Budd-Chiari. 2) Colocacin de TIPS.
2) Cirrosis biliar primaria. 3) Administrar lactulosa en enemas.
3) Tuberculosis peritoneal. 4) Transfundir concentrados de hemates.
4) Enfermedad venooclusiva heptica. 5) Aumentar la dosis del diurtico.
5) Trombosis de la porta.
63. A un varn de 44 aos, asintomtico, se le realiz una biopsia
58. En cul de los siguientes casos de ascitis maligna suele heptica por hepatitis VHC. El estudio histolgico revel
encontrarse citologa positiva en lquido asctico?: como hallazgo una cirrosis. Una ecografa abdominal y la
endoscopia oral fueron normales. En relacin con la profila-
1) Carcinomatosis peritoneal, sin metstasis hepticas. xis de la hemorragia por varices esofgicas, qu cree lo ms
2) Metstasis masivas hepticas, sin carcinomatosis perito- adecuado recomendar?:
neal asociada.
3) Ascitis quilosa de origen tumoral. 1) Nitritos en dosis bajas.
4) Metstasis masivas hepticas, con gran aumento de la 2) Propranolol.
fosfatasa alcalina. 3) Ligadura endoscpica.
5) Pacientes con aumento de la alfafetoprotena en lquido 4) nicamente vigilancia peridica.
asctico. 5) Losartn.

59. Seale la respuesta FALSA en relacin a la peritonitis bacte- 64. Seale la respuesta INCORRECTA en relacin a la infeccin
riana espontnea: por el VHC:

1) Est producida generalmente por gramnegativos. 1) El ARN-VHC es el mejor marcador para el diagnstico.
2) Cursa con peritonismo en el 80% de los casos. 2) Tiene un riesgo elevado de evolucin a la cronicidad.
3) En el tratamiento emprico, no es necesario cubrir anae- 3) Es frecuente en drogadictos parenterales.
robios. 4) La mayora tienen historia de transfusin.
4) Su tratamiento no incluye la ciruga. 5) Un porcentaje importante de casos no tienen un antece-
5) El parmetro ms utilizado para el diagnstico de perito- dente epidemiolgico de inters.
nitis bacteriana espontnea es la cifra de polimorfonu-
cleares. 65. Uno de los siguientes marcadores serolgicos es impres-
cindible para diagnosticar una infeccin aguda por VHB:
60. Indicara la profilaxis antibitica con quinolonas de la peri-
tonitis bacteriana espontnea en un cirrtico con ascitis: 1) HBsAg.
2) ADN-VHB.
1) Si asocia antecedentes de alcoholismo. 3) Anti-HBc de tipo IgM.
2) En caso de hemorragia digestiva alta. 4) HBeAg.
3) nicamente a partir de haber padecido un segundo 5) Actividad ADN-polimerasa.
episodio de peritonitis.

Preguntas TEST
4) Cuando las protenas en lquido asctico son >1 g/dl. 66. Seale cul de las siguientes manifestaciones extrahepticas
5) Cuando los neutrfilos en lquido asctico son <100/ de la infeccin por el virus de la hepatitis C es ms frecuente:
mm3.
1) Crioglobulinemia mixta esencial.
61. Cul de los siguientes datos histolgicos renales es tpico del 2) Glomerulonefritis membranoproliferativa.
sndrome hepatorrenal?: 3) Glomerulonefritis membranosa.
4) Anemia aplsica.
1) Proliferacin mesangial. 5) Polineuropata perifrica.
2) Trombos de bilirrubina en las asas de Henle.
3) Infiltracin intersticial aguda de linfocitos. 67. En cul de las siguientes situaciones existe mayor riesgo de
4) Trombosis de la vena renal. cronificacin de la infeccin por VHB?:
5) Histologa normal.
1) Adquisicin por va parenteral.
62. Paciente de 45 aos, en tratamiento con espironolactona por 2) Tras contagio sexual.
ascitis secundaria a cirrosis en estadio C, y con varios episo- 3) Adquisicin en la infancia.
dios de sangrado por varices esofgicas, por lo que se 4) Contagio en la edad adulta.
encuentra a la espera de trasplante. Reingresa por un nuevo 5) Tras contagio concomitante con infeccin por VIH.

M exico A rgentina CTO Medicina C/ Nez de Balboa, 115 28006 MADRID (Espaa) Tfno.: (91) 782 43 32 / Fax: (91) 782 43 27
Pg. 6 DG C hile U ruguay E-mail: secretaria@ctomedicina.com; iberocto@ctomedicina.com WEB: www.ctomedicina.com; www.iberocto.com
DIGESTIVO
Preparacin Examen de Seleccin 05/06 1 Vuelta Seguimiento a distancia

68. Un paciente de 28 aos, diagnosticado de infeccin crnica 73. En relacin con el empleo de lamivudina en el tratamiento de
por VHB en fase no replicativa, sufre un cuadro de hepatitis la hepatitis crnica VHB, es cierto que:
aguda. Los datos de laboratorio confirman el diagnstico
clnico. Los marcadores vricos dan los siguientes resultados: 1) Est contraindicada en VIH positivos.
HBsAg+, HbeAg-, anti-HBc+, ADN VHB-, anti-VHC-, HD- 2) Est indicado en la mutante Ag e negativo.
Ag+, anti-HD IgM-, anti-HD IgG- e IgM anti-VHA-. Qu 3) Su efecto adverso ms frecuente es el hipotiroidismo.
diagnstico hara?: 4) La va de administracin es intramuscular semanal.
5) Debe usarse asociado a interfern.
1) Reactivacin de la infeccin por VHB.
2) Seroconversin anti-e. 74. Cul es el objetivo principal del tratamiento de la hepatitis
3) Sobreinfeccin delta. crnica por VHB?:
4) Coinfeccin delta.
5) No hay datos para poder asegurar un diagnstico con 1) La eliminacin del virus del organismo.
certeza. 2) La seroconversin anti-e.
3) Que el virus, aunque se replique, no produzca lesin
69. Un paciente de 23 aos, ADVP activo, consulta por un cuadro heptica.
clnico y bioqumico compatible con una hepatitis aguda. 4) Evitar la aparicin de mutantes precore y de escape.
Presenta los siguientes marcadores: HBsAg-, anti-HBs-, anti- 5) Mejorar la cirrosis heptica.
HBc+, anti-HBe+, anti-VHC-, IgM anti-VHA+. Qu diagns-
tico hara?: 75. A una mujer de 37 aos, ex ADVP, se le descubren anticuerpos
anti-VHC positivos. Durante seis meses de seguimiento, las
1) Infeccin aguda por VHB. transaminasas y el resto de la bioqumica heptica fueron
2) Infeccin crnica por VHB. normales. El RNA VHC fue +. En el manejo de la paciente,
3) Infeccin aguda por VHA. recomendara:
4) Probable sobreinfeccin delta.
5) Hepatitis no vrica. 1) Biopsia heptica.
2) Interfern.
70. Un paciente vacunado frente al VHB hace 2 aos y con 3) Solicitar anticuerpos antimitocondriales.
respuesta desconocida a la vacuna sufre un contacto acci- 4) Interfern + ribavirina.
dental con el VHB. Su ttulo de anti-HBs es <10 mUI/ml. Qu 5) Seguimiento evolutivo.
le recomendara?:
76. En relacin con la terapia antiviral de la hepatitis crnica por
1) Vacunacin completa de nuevo. VHB, es cierto todo lo siguiente, EXCEPTO:
2) Una dosis de Ig y una dosis de recuerdo de vacuna.
3) Dos dosis de Ig. 1) El interfern gamma es el frmaco ms eficaz.
4) Ningn tratamiento. 2) La respuesta es inferior en los VIH positivos.
5) Una dosis de recuerdo de vacuna. 3) La respuesta es superior en las mujeres.
4) Los niveles elevados de ALT predicen una buena respuesta.
71. Seale el dato histolgico mnimo para poder hacer el diag- 5) Los pacientes con niveles bajos de ADN-VHB responden
nstico de hepatitis crnica activa: mejor.

1) Necrosis heptica con puentes. 77. Respecto a un paciente de 62 aos, con cirrosis posthe-patitis
2) Colapso multilobular. por VHC, al que se va a someter a un trasplante heptico, es
3) Ndulos de regeneracin. correcto afirmar que:
4) Necrosis parcelar perifrica.
Preguntas TEST

5) Infiltrado mononuclear del espacio porta. 1) Es muy probable que el nuevo hgado se infecte por VHC.
2) En un alto porcentaje, se detecta anti-VHC.
72. Un paciente de 43 aos consulta por molestias abdomina-les 3) La presencia de crioglobulinas es una indicacin incluso
inespecficas. En la exploracin fsica presenta nicamente en estadio A de Child.
un discreto aumento del tamao heptico. Los datos de 4) No puede ser trasplantado por la edad.
laboratorio ofrecen los siguientes resultados: GPT 100 UI/l, 5) Si el genotipo es Ib, es menor el riesgo de recidiva precoz
GOT 87 UI/l, HBsAg+, HBeAg-, anti-HBc+ de tipo IgG, anti- postrasplante.
HBe+, ADN-VHB+. El resto de los marcadores vricos dieron
resultado negativo. Qu diagnstico hara?: 78. Una paciente de 18 aos, sin antecedentes de inters, consulta
por hipertransaminasemia persistente. En el estudio inmuno-
1) Infeccin crnica por VHB en fase no replicativa. lgico se detectan ANA y ANCA positivos con marcadores
2) Infeccin aguda por VHB. vricos negativos. Ante esta situacin, es correcto afirmar que:
3) Infeccin crnica por la cepa mutante precore.
4) Infeccin pasada por VHB. 1) Presenta una colangitis esclerosante.
5) Probable hepatopata txica o metablica. Se puede 2) Se trata de una hepatitis crnica autoinmune tipo 2.
descartar enfermedad vrica. 3) La biopsia heptica no establecer el diagnstico de
certeza.

M exico A rgentina CTO Medicina C/ Nez de Balboa, 115 28006 MADRID (Espaa) Tfno.: (91) 782 43 32 / Fax: (91) 782 43 27
C hile U ruguay E-mail: secretaria@ctomedicina.com; iberocto@ctomedicina.com WEB: www.ctomedicina.com; www.iberocto.com
DG Pg. 7
DIGESTIVO
Seguimiento a distancia Preparacin Examen de Seleccin 05/06 1 Vuelta
4) Hay que descartar una enfermedad de Wilson. 1) El hipogonadismo puede ser el cuadro de presentacin en
5) Debe recibir tratamiento con tacrolimus. jvenes.
2) La intolerancia hidrocarbonada mejora con las fleboto-
79. A un paciente de 51 aos con cirrosis grado C y frecuentes mas.
episodios de encefalopata y ascitis, se le aprecia una lesin 3) El gen anmalo est en el cromosoma 12.
de 3 cm en lbulo heptico derecho. Una PAAF confirma que 4) El hipogonadismo se debe al acmulo de hierro en las
se trata de un hepatocarcinoma. Cul de las siguientes gnadas.
opciones estara indicada?: 5) Cursa con herencia autosmica dominante.

1) Trasplante. 85. Cul de las siguientes entidades NO se asocia de forma


2) Embolizacin. habitual con esteatosis heptica?:
3) Alcoholizacin percutnea.
4) Ciruga. 1) Cirrosis alcohlica.
5) Cisplatino y adriamicina. 2) Hepatitis crnica VHB mutante precore.
3) Obesidad.
80. Una paciente de 38 aos consult por prurito y astenia. En 4) Diabetes mellitus.
la exploracin fsica presentaba una hepatomegalia lisa a 2 5) Tratamiento con corticoides.
centmetros del reborde costal. Los datos de laboratorio
fueron: bilirrubina 1,6 mg/dl; fosfatasa alcalina 1.040 U/L 86. En cul de las siguientes enfermedades existe mayor riesgo
(normal <280 U/L); el resto de la bioqumica heptica era de desarrollar clculos pigmentarios en la vescula biliar?:
normal; los anticuerpos antimitocondriales fueron positi-
vos. Una ecografa abdominal fue normal. Cul de los 1) Insuficiencia renal crnica.
siguientes frmacos sera ms til para la paciente?: 2) Estado de portador crnico del VHB.
3) Enfermedad de Crohn.
1) Interfern. 4) Estados hemolticos crnicos.
2) Penicilamina. 5) Obesidad.
3) Trientine.
4) Esteroides. 87. Seale la causa por la que la bilis se vuelve ms litognica tras
5) cido ursodesoxiclico. las resecciones ileales:

81. Cul de estos parmetros tendra MENOS valor a la hora de 1) Se reabsorbe menos agua.
evaluar a un paciente con cirrosis biliar primaria?: 2) Se pierde colesterol por la diarrea.
3) Disminuye la reabsorcin de sales biliares.
1) Niveles de IgM. 4) Aumenta la reabsorcin de colesterol.
2) Anticuerpos antimitocondriales. 5) El flujo biliar es ms lento.
3) Hormonas tiroideas.
4) Niveles de bilirrubina. 88. La ecografa abdominal en una pancreatitis aguda ofrece
5) Niveles de triglicridos. particularmente informacin sobre:

82. Seale la opcin FALSA en relacin a la colangitis esclerosante: 1) Confirmacin del diagnstico de pancreatitis.
2) La etiologa.
1) Existe asociacin con el HLA-B8. 3) El pronstico.
2) La mayora de los casos aparecen en pacientes que tienen 4) La evolucin.
colitis ulcerosa. 5) Modificaciones del tratamiento.
3) La colectoma en la colitis ulcerosa protege del desarrollo

Preguntas TEST
de colangitis esclerosante. 89. Cul es la causa ms frecuente de las pancreatitis agudas?:
4) El diagnstico se establece basndose en los hallazgos
colangiogrficos. 1) Alcoholismo.
5) Es ms frecuente en varones. 2) Litiasis biliar.
3) Microlitiasis.
83. El hijo de 26 aos de un paciente afecto de hemocromatosis 4) Esfnter de Oddi hipertensivo.
presenta un ndice de saturacin de transferrina muy elevado 5) Fibrosis qustica.
(65 %) y ferritina srica de 550 (N < 200). A continuacin,
se debe aconsejar: 90. Cul de las siguientes pruebas es la de mayor sensibilidad
para el diagnstico de pancreatitis crnica?:
1) Biopsia heptica.
2) Estudio mutaciones gen HFE. 1) Ecografa abdominal.
3) Flebotomas. 2) Rx simple de abdomen.
4) D-penicilamina. 3) TC.
5) TC abdominal. 4) CPRE.
5) Test de secretina.
84. Seale cul de las siguientes es correcta, en la hemocroma-
tosis primaria:

M exico A rgentina CTO Medicina C/ Nez de Balboa, 115 28006 MADRID (Espaa) Tfno.: (91) 782 43 32 / Fax: (91) 782 43 27
Pg. 8 DG C hile U ruguay E-mail: secretaria@ctomedicina.com; iberocto@ctomedicina.com WEB: www.ctomedicina.com; www.iberocto.com
DIGESTIVO
Preparacin Examen de Seleccin 05/06 1 Vuelta Seguimiento a distancia

91. Todos los siguientes son criterios de Ransom, EXCEPTO uno 3) Dficit de B12.
de ellos. Selelo: 4) Ictericia.
5) Ascitis pancretica.
1) Calcio < 8 mg/dl.
2) pO2 < 60 mmHg. 97. Paciente alcohlico, de 49 aos, que consulta por dolor
3) GOT>250 UI. epigstrico postprandial de 3 meses de evolucin. En una
4) Leucocitosis >16.000. ecografa se observa una masa pancretica que se confirma
5) PCR > 2 veces superior al lmite normal. en la TC. Una CPRE muestra una estenosis nica en el
Wirsung. Con estos datos, el diagnstico ms probable es:
92. Para evitar complicaciones en el curso de la pancreatitis
aguda moderada, es necesario: 1) Pancreatitis aguda ceflica.
2) Pancreatitis crnica etlica.
1) Diferir el inicio de la alimentacin oral. 3) Carcinoma de pncreas.
2) Iniciar inmediatamente tratamiento antibitico. 4) Pancreatitis crnica obstructiva.
3) Tratar precozmente con glucagn. 5) Nesidioblastosis.
4) Ciruga precoz.
5) Tratar con esteroides. 98. El parmetro ms apropiado para indicar el tratamiento con
enzimas pancreticas como substitucin es:
93. Un paciente de 33 aos, alcohlico, ingresa por una pancrea-
titis aguda sin evidencia de litiasis biliar. A las 24 horas, 1) Test de la secretina.
presenta fiebre, hipotensin, oliguria e hipoxemia. Un TC con 2) Quimiotripsina fecal.
contraste urgente demuestra un rea de necrosis pancreti- 3) Tripsina srica.
ca del 70%, sin evidencia de absceso. Una puncin aspiracin 4) Grasas fecales.
con aguja fina de la zona afectada demuestra bacilos gram- 5) Test del pancreolauryl.
negativos. Entre las siguientes, qu medida teraputica
propondra para mejorar la evolucin?: 99. En el manejo de la ascitis pancretica, cul de entre las
siguientes medidas diagnsticas es la ms importante?:
1) Necrosectoma pancretica.
2) Somatostatina i.v. 1) CPRE.
3) Imipenem i.v. 2) TAC.
4) Esteroides i.v. 3) Ecografa.
5) Drenaje percutneo dirigido por TC. 4) Gammagrafa.
5) Laparoscopia.
94. Una paciente de 70 aos ingresa en el hospital por dolor
abdominal y vmitos. Los estudios analticos demuestran una 100. Las siguientes medidas pueden ser tiles en el tratamiento del
amilasa muy elevada. Una ecografa demuestra una ligera dolor en la pancreatitis crnica, EXCEPTO:
dilatacin de la va biliar y colelitiasis. A las 48 horas de ingreso
contina con fuerte dolor abdominal, el Hto ha disminuido 1) Pancreatectoma ceflica.
un 15%, el calcio a 7,8 mg/dl, el BUN ha aumentado 15 mg/ 2) Suplementos de enzimas pancreticos.
dl y la albmina es 3,1 g/dl. Cul de los siguientes tratamien- 3) AINEs.
tos sera ms adecuado?: 4) Antagonistas del TNF.
5) Dilatacin y endoprtesis del Wirsung.
1) Papilotoma endoscpica.
2) Pancreatectoma.
3) Administracin de antibiticos, ms el tratamiento habi-
Preguntas TEST

tual.
4) Colecistectoma urgente.
5) Colecistectoma laparoscpica.

95. Cul es la causa ms frecuente de pancreatitis crnica?:

1) Alcoholismo crnico.
2) Malnutricin calrico-proteca.
3) Hiperlipemia.
4) Mucoviscidosis.
5) Poliovirus.

96. En el curso de la pancreatitis crnica, es EXCEPCIONAL que


suceda:

1) Aparicin de calcificaciones pancreticas.


2) Cetoacidosis diabtica.

M exico A rgentina CTO Medicina C/ Nez de Balboa, 115 28006 MADRID (Espaa) Tfno.: (91) 782 43 32 / Fax: (91) 782 43 27
C hile U ruguay E-mail: secretaria@ctomedicina.com; iberocto@ctomedicina.com WEB: www.ctomedicina.com; www.iberocto.com
DG Pg. 9
DIGESTIVO Y CIRUGA GENERAL
Preparacin Examen de Seleccin 05/06 1 Vuelta Seguimiento a distancia
Pregunta 1.-R: 1 nes o divertculos en el esfago distal, ya que existira riesgo de
El trmino disfagia orofarngea incluye a las enfermedades farnge- hemorragia o perforacin.
as y a las del esfago superior. Las enfermedades involucradas con 3. Cuando deseamos plantear al paciente un tratamiento definitivo.
mayor frecuencia producen disfagia por: La miotoma de Heller es ms eficaz: presenta menos fracasos que
1. Debilidad de los msculos farngeos, como en las miopatas. las dilataciones. Por eso en adolescentes y jvenes se prefiere de
2. Rigidez muscular, como en el Parkinson que es la respuesta correcta. primera eleccin el tratamiento quirrgico. ste se recomienda
3. Alteraciones en la motilidad secundarias a origen neurgeno, bien por va laparoscpica y con ciruga antirreflujo asociada.
de nervio perifrico o de SNC como es el caso de los ictus.
La toxina botulnica administrada va endoscpica en esfago distal
Estas enfermedades suelen comportarse como disfagias motoras y se reserva para los casos de imposibilidad de someterse al tratamiento
por tanto tienen sntomas tanto con la ingesta de slidos como de dilatador o la miotoma. La toxina acta paralizando a las nicas
lquidos. El sntoma que orienta con mayor precisin al origen orofa- fibras que permanecen funcionantes en el esfnter (las que lo mantie-
rngeo de una disfagia es la presencia de episodios de aspiracin rela- nen contrado permanentemente) y, por tanto, como consecuencia
cionados con la deglucin (atragantamiento). de su accin el esfnter quedar constantemente relajado. Su desven-
La acalasia es tambin una enfermedad motora, por tanto con taja es que requiere repetidas administraciones. Los nitritos orales
disfagia a slidos y lquidos, pero afecta al tercio inferior del esfago nicamente alivian el dolor torcico, pero no son resolutivos del pro-
fundamentalmente. El esfago de Barrett causa disfagia, si se asocia a blema esencial de la enfermedad.
una esofagitis con estenosis pptica. El anillo de Schatzki, o anillo B, es
una protrusin en la zona inferior del esfago que causa disfagia recu- Pregunta 5.-R: 3
rrente e intermitente. La presencia en una manometra de contracciones simultneas de
gran amplitud y duracin en el esfago distal puede deberse a dos
Pregunta 2.-R: 2 entidades: el espasmo esofgico difuso, que es la respuesta correcta y
El diagnstico de certeza de la acalasia es manomtrico. Sin em- la acalasia vigorosa. Se distinguen porque esta ltima presenta adems
bargo siempre debe realizarse una endoscopia con biopsias del seg- dificultad para relajar el esfnter durante la deglucin. En realidad la
mento distal del esfago para excluir neoplasias que simulen una acalasia vigorosa no es una enfermedad o entidad clnica como tal,
acalasia, y que se conocen como acalasias secundarias. Estos tumores sino que se trata generalmente de una acalasia en fases iniciales de su
cursan con una disfagia semejante a la acalasia primaria, porque infil- evolucin y que por la denervacin de la pared esofgica presenta
tran la pared del esfago alterando la motilidad de su zona distal y contracciones de gran amplitud, por lo que adems de disfagia, pue-
esfnter. Los tumores mucosos esofgicos, como el carcinoma epider- de tener episodios de intenso dolor torcico, retroesternal, y en oca-
moide o el adenocarcinoma, al ser mucosos se comportan como una siones odinofagia.
obstruccin y, por tanto, inicialmente con disfagia a slidos y en los La amiloidosis esofgica tiene un comportamiento clnico y ma-
estados muy avanzados, a lquidos. Entre los tumores que causan nomtrico muy parecido a la esclerodermia: disfagia motora y reflu-
acalasia secundaria destacan: jo.
1. El ms frecuente de todos: el adenocarcinoma gstrico, que origi- El anillo B o de Schatzki causa disfagia intermitente a slidos y
na acalasia cuando progresa infiltrando el espesor de la pared y menos frecuentemente dolor.
llega a la zona distal del esfago y al esfnter esofgico inferior. Por La acalasia cricofarngea o barra farngea es una enfermedad mo-
eso la respuesta correcta es adenocarcinoma gstrico e incluso, si tora que provoca disfagia orofarngea. Debe diagnosticarse por
se hubiera expuesto, de origen fndico o de la unin gastroesof- manometra y, en ocasiones, se aprecia en la Rx cervical lateral una
gica, an hubiera sido ms correcta la respuesta. protrusin en la regin prevertebral anterior, que corresponde al
2. Los situados en mediastino posterior: extensin de un microctico msculo cricofarngeo contrado.
de pulmn y los linfomas.
Pregunta 6.-R: 5
Pregunta 3.-R: 2 Los trastornos motores del esfago distal son fundamentalmente el
Entre las complicaciones de la acalasia cabe citar: espasmo esofgico difuso, la acalasia y la esclerodermia. Estos dos
1. Disminucin de peso por la desnutricin, debida a que por la ltimos presentan ondas sugestivas de un peristaltismo reducido e
disfagia el paciente ingiere menor cantidad de alimento. incluso en ocasiones falta absoluta de movimiento o aperistalsis. Fun-
Comentarios TEST

2. La esofagitis que se produce por el contacto repetido del alimento damentalmente la esclerodermia se diferencia de la acalasia en la
con la pared esofgica. En ocasiones se debe a una infeccin aa- manometra en que el esfnter esofgico inferior est hipotnico (por
dida por Cndida; pero no es posible que sea por reflujo, ya que eso la respuesta correcta es esclerodermia) y se relaja bien con la
en esta enfermedad el esfnter est hipertnico en reposo y no deglucin (ver tabla en pgina siguiente).
permite el reflujo gstrico al esfago. El sndrome de Sjgren puede dar lugar, en ocasiones, a disfagia
Es de inters destacar que: alta por la intensa sequedad de la mucosa.
Los pacientes tratados con dilatacin endoscpica tienen pos- El sndrome de Boerhaave es la perforacin espontnea del esfa-
teriormente reflujo al quedar el esfnter incompetente. go, generalmente tras una maniobra intempestiva, como un vmito o
Si un paciente con clnica de pirosis por reflujo gastroesofgico un valsalva. Presenta la clnica de dolor retroesternal, semejante a la
presenta de forma simultnea desaparicin de su pirosis (es de- perforacin iatrgena y, adems, derrame pleural con lquido rico en
cir, mejora del reflujo) y a la vez aparicin de disfagia se debe amilasa.
sospechar que se ha establecido una acalasia.
3. Tienen ms riesgo de cncer de esfago. Pregunta 7.-R: 3
4. El dficit de vitamina B12, respuesta correcta, no se puede atribuir El antecedente de pirosis y regurgitacin es altamente sugestivo de
a la acalasia, ya que los problemas para su absorcin no se deben enfermedad por reflujo gastroesofgico cido. Con estos sntomas no
a patologa esofgica y s a gstrica (gastritis autoinmune), insufi- se precisan pruebas diagnsticas y la actitud adecuada es instaurar
ciencia exocrina pancretica, sobrecrecimiento bacteriano o alte- tratamiento con inhibidores de la bomba de protones (no con anti-
raciones en la pared del ileon distal. H2) y asumir el diagnstico si desaparece la sintomatologa. La apari-
cin de dolor o disfagia plantea que ha surgido una complicacin:
Pregunta 4.-R: 1 bien una estenosis pptica o un carcinoma, por lo que es necesario la
El tratamiento de eleccin de la acalasia es la dilatacin neumtica realizacin de una endoscopia (la respuesta correcta). La pH-metra,
endoscpica, por ser menos cruenta que la ciruga; pero cuatro cir- prueba de certeza de la enfermedad por reflujo gastroesofgico, tiene
cunstancias obligan a plantear como primera opcin la ciruga: sus indicaciones:
1. El fracaso de las dilataciones endoscpicas (respuesta correcta la 1). 1. Refractariedad al tratamiento mdico.
2. La imposibilidad de someterse a la dilatacin por presentar erosio- 2. Valoracin pre y postquirrgica.

M exico A rgentina CTO Medicina C/ Nez de Balboa, 115 28006 MADRID (Espaa) Tfno.: (91) 782 43 32 / Fax: (91) 782 43 27
C hile U ruguay E-mail: secretaria@ctomedicina.com; iberocto@ctomedicina.com WEB: www.ctomedicina.com; www.iberocto.com
HM Pg. 1
DIGESTIVO Y CIRUGA GENERAL
Seguimiento a distancia Preparacin Examen de Seleccin 05/06 1 Vuelta

Pregunta 6. Caractersticas clnicas de las principales enfermedades esofgicas que cursan con disfagia.

12131451 6423678967
51 64 14
8 648 528 217258
1
95 48

6 123456728672956597
2 2 2647
123456728672956597
 2647

9562 2647

4 92249
92249
49229

 4

4
9 9689 392 4!236"4#427
"45#926#68676279# 566

%6 542&62973'26(6 )65429


9 4
4
$"#4
 5#62*9 9689 39
979#68565

$6 342"6627,85472#44
2  +4*97(6!2 #689 392"6627,8547 0716*6236 342"6627,8547
&29-7392".5562592"974
2/"946 #4428254726#4"6656 
"66282547297
3939 392&2 4297 0716*62"4*97(62 342#4
  #4 2982973.72&2#45672'"567
42 425929184
"4*97(62 29-7392".556 ".5562592"974

592"974

2092989##, 28626 4936 26 4936623"#67


2254 5922792479(622"97, 2598 22#4 36##4 97 2/ 547#4"62&24"76

22//32 46824269 3656


2/ 26 4936286 229"935672592*6 22/72'7219#9 3929 298
22462"97, 2676825982#9"4 22"97, 25982//326768 226"8352&2783' 967 739#42954!
9 ! 22798929736269 3656
22973'257 562"94 222922#49 )6 29 286 789*4239#42 194
2206 39286259*8#, 26"69#9 228629866#, 2598 22"6392 1942598 7&289*4227"94

22#4 36##4 972783' 967


22971 392972 468
2297,16*4

256462679 #62592#'66226.96
2564623"#626*9 29
736*9 29 28"#425928448
2276#6#4#947

3. Sntomas predominantes extraesofgicos, por ejemplo. tos noctur- En ocasiones se aprecian ulceraciones visibles en el estudio barita-
na, asma o laringitis. do. La respuesta correcta es la 1, porque el adenocarcinoma supone
aproximadamente un 10% o menos, aunque en los ltimos aos ha
El test de Bernstein, o de provocacin con una ligera cantidad de aumentado su incidencia, posiblemente en relacin con el esfago
cido, se utiliza para aquellos casos en los que predomina una clnica de Barrett.
con episodios aislados y limitados de intenso dolor retroesternal pare-
cido al angor y con pH-metra no diagnstica. Pregunta 10.-R: 3
El esofagograma con bario tiene poca utilidad, nicamente como Para los tumores esofgicos malignos podemos concretar que
prueba complementaria a la endoscopia en el estudio de las estenosis. para:
El diagnstico es de eleccin la toma de biopsias por endoscopia.
Pregunta 8.-R: 4 Evaluar metstasis en pulmn, hgado u otras localizaciones: el
El esfago de Barrett es una complicacin del reflujo. Se considera a TAC y la resonancia magntica son las mejores pruebas.
su vez una forma de peor pronstico si se acompaa de displasia en Conocer si hay adenopatas tumorales, estadiaje N, es de eleccin
alguna de las biopsias. Se considera displasia grave si la displasia apare- el TAC torcico.

Comentarios TEST
ce en todas las muestras y se comprueba por dos anatomopatlogos y Determinar el grado de extensin intramural en el esfago es la
se asume que el riesgo de un adenocarcinoma injertado en el esfago ecografa endoscpica (respuesta correcta).
de Barret es muy elevado, por tanto se indica la esofaguectoma de la
zona afecta. A diferencia de una esofagitis por reflujo no complicada, el Pregunta 11.-R: 5
esfago de Barrett requiere controles endoscpicos para valorar el ries- Es un gram negativo y flagelado (respuesta correcta la 5), que afecta
go de progresin a adenocarcinoma aunque el paciente se encuentre a la mucosa antral y, en ocasiones, a la duodenal prxima al ploro;
asintomtico. Si se observaron displasias se recomienda endoscopia por tanto no se asocia siempre a colonizacin duodenal. Suele en-
cada seis meses. Si no hay displasia, como en el presente caso, cada ao contrarse en las capas profundas del moco o entre ste y el epitelio,
y medio o dos aos. pero no invade la lmina propia. Su infeccin siempre produce le-
El Barrett est provocado por el reflujo, sin embargo, puede persis- sin histolgica, por tanto no hay portadores, slo enfermos, aunque
tir aunque mejore o cure la esofagitis. El paciente debe tomar omepra- frecuentemente asintomticos.
zol porque es muy probable que en el momento del diagnstico
tenga una esofagitis acompaante, al menos moderada. En relacin con el diagnstico de la infeccin debemos recordar
La respuesta a sealar es la 4, pues es muy controvertida en el que (ver tabla):
esfago de Barret la relacin entre adenocarcinoma de esfago y 1. El cultivo de la mucosa antral es la prueba de certeza, pero es poco
colonizacin por H.pylori; actualmente se piensa que no guardan sensible y difcil de realizar.
relacin. 2. El estudio histolgico de la mucosa con tincin de Giemsa es muy
eficaz para el diagnstico.
Pregunta 9.-R: 1 3. La prueba de la ureasa sobre histologa o el test del aliento con urea
El carcinoma epidermoide sigue siendo el grupo histolgico ms marcada con istopo C13 son muy rentables. Esta ltima tiene una
frecuente. Desgraciadamente se diagnostica cuando aparece disfagia sensibilidad y especificidad superior al 95%, siempre que el pa-
y significa que la enfermedad est muy avanzada y, por ello, la super- ciente no est tomando antibitico o inhibidor de la bomba de
vivencia a los 5 aos oscila entre el 15 al 20%. protones al realizarse la prueba.

M exico A rgentina CTO Medicina C/ Nez de Balboa, 115 28006 MADRID (Espaa) Tfno.: (91) 782 43 32 / Fax: (91) 782 43 27
Pg. 2 HM C hile U ruguay E-mail: secretaria@ctomedicina.com; iberocto@ctomedicina.com WEB: www.ctomedicina.com; www.iberocto.com
DIGESTIVO Y CIRUGA GENERAL
Preparacin Examen de Seleccin 05/06 1 Vuelta Seguimiento a distancia
1. Sfilis.
Pregunta 11. Fiabilidad de las pruebas diagnsticas del H. pylori. 2. Tuberculosis.
3. Sarcoidosis.
123456 7487959
9 6  626427967 4. Linfoma.
5. Carcinoma gstrico, sobre todo tipo difuso.
12345672879
4 7 
76727 44
 6. La enfermedad de Mntrier. La asociacin de engrosamiento en
fundus, dispepsia e hipoalbuminemia, sin que se observen reac-
22222222
844  

2 72
72 2 7 tantes bioqumicos de enfermedad aguda, sugiere este diagnstico.
!"68
3866 La confirmacin sera histolgica.
&7
2
2677 Pregunta 16.-R: 3
22222222#8
$4 % 67
87
224 El mecanismo por el cual los AINES producen gastrotoxicidad es
8
5
8
4 por la reduccin de las prostaglandinas gstricas como consecuencia
22222222'787
2 7 ( )68
de la inhibicin de la COX-1. A ms poder de inhibicin de la COX-
222222228
$
 267 1 ms gastroerosivo es el frmaco. En la escala de gastroerosividad son
piroxicam y ketorolaco los de mayor capacidad erosiva e ibuprofeno
*62464562 el de menor. Entre los nuevos AINES inhibidores selectivos de la COX-
12&2
784 % 766

2876+8
 2 se encuentran los menos gastroerosivos de todos: rofecoxib (res-
,-272 7+. puesta correcta) y celecoxib. stos reducen un 50% el riesgo
/42
8
727867 gastroerosivo frente a frmacos tradicionales como diclofenaco o na-

07
28
$2 2 1 proxeno.
1237644 % 28
27278
4
7
7
4
41 Pregunta 17.-R: 4
328+
24277  La causa ms frecuente de lcera es la infeccin por Helicobacter
7272&35361 pylori. Sus lceras aparecen fundamentalmente en antro gstrico y,
sobre todo, en la primera porcin del duodeno.
Las lceras gstricas son ms grandes que las duodenales, por eso
Pregunta 12.-R: 5 tienen ms riesgo relativo de sangrado digestivo. Aunque son ms gran-
La gastrina aumenta, en general, como consecuencia de la reduc- des, las gstricas se perforan menos, dado el mayor espesor de la
cin de la acidez gstrica, por tanto la hipoclorhidria es la causa ms
pared gstrica. Se calcula que entre el 6 y el 8% de las lceras gstricas
frecuente (respuesta correcta la 5). Por tanto las situaciones que cau-
pueden ser malignas. Cuando una lcera aparece en fundus, la pri-
san hipoclorhidria provocan aumento de la gastrina, como son las
mera posibilidad es que siga siendo benigna, pero respecto a lo que
gastritis atrficas y el uso de anti-H2 o inhibidores de la bomba de
sucede en otras regiones gstricas aumentan las probabilidades de
protones. No lo causan el sucralfato, que es un protector de la super-
ficie ulcerada o erosionada pero no un antisecretor. que sea maligna. Las diferencias entre duodenales y gstricas se obser-
Es cierto que algunos carcinomas gstricos se asocian a hipoclor- van en la tabla.
hidria y, por tanto, a aumento de gastrina. Esta hipoclorhidria de los
tumores se debe a que coexiste una gastritis atrfica severa, no es Pregunta 17. Diferencias entre lcera duodenal y gstrica.
por tanto un efecto tumoral. Otras causas de hipergastrinemia son
los gastrinomas y menos frecuentemente los tumores carcinoides.
123456789
84 62 1234567 536
Pregunta 13.-R: 5 8
6 27
La profilaxis de la lcera por AINES, o gastroproteccin, se realiza 1234256789
4 6
9
4 68
con omeprazol o misoprostol. No se ha demostrado utilidad de los 
56 8 262 2
6
8492
6
anti-H2, sucralfato o anticidos. Aunque el riesgo de padecer una 266
lesin gstrica, incluso una lcera complicada, es superior por el mero
6
22

Comentarios TEST

hecho de tomar AINES, se considera que slo procede administrar 89



omeprazol o misoprostol cuando coexisten factores de riesgo que 
36263 4 2 6
4
64
682

aumentan ste de forma intolerable. Son factores de riesgo: edad


superior a 70 aos, antecedente de lcera, estar anticoagulado, altas 868  865  !865 
dosis de AINES, combinaciones de los mismos, uso concomitante de
corticoides y una enfermedad grave (no necesariamente estar hospita- 72
5 " # $ % #
lizado). En el presente caso la paciente no tiene ningn factor de 43543 7 38
&
68 8 &
68 8
riesgo, por lo que la respuesta adecuada es no recomendar ninguna
medida farmacolgica.
662
' 
829(
 ' 
829(

Pregunta 14.-R: 1 2 36 )9 2 8)9
Los anticuerpos anticlula parietal gstrica se usan para el diagns-
tico de la gastritis autoinmune (tipo A). Aunque son muy sensibles, son 12 2
6 7
poco especficos y pueden aparecer ttulos, en general moderados, *2

66 -2

668./ #07
en otras enfermedades autoinmunes o en sujetos sanos. Los antifactor 
2363
4 
967 92

6 89 
intrnseco son ms especficos; sin embargo para el diagnstico es 92

6 7
242 2,
obligado una biopsia del fundus gstrico, donde se objetive la atrofia. +
4 ,
La anemia macroctica sugiere dficit de B12, por todo ello la respues-
ta correcta es la primera. 128 
627
Un estudio baritado no permite hacer el diagnstico de gastritis, ni 3   96829
27
2   968.
aguda ni crnica. 6 3
9
26078 8
.26
6

6 60
46
Pregunta 15.-R: 4
Entre las causas de pliegues gstricos engrosados cabe destacar: 438!6 %"#82 8/865  4
242 2

M exico A rgentina CTO Medicina C/ Nez de Balboa, 115 28006 MADRID (Espaa) Tfno.: (91) 782 43 32 / Fax: (91) 782 43 27
C hile U ruguay E-mail: secretaria@ctomedicina.com; iberocto@ctomedicina.com WEB: www.ctomedicina.com; www.iberocto.com
HM Pg. 3
DIGESTIVO Y CIRUGA GENERAL
Seguimiento a distancia Preparacin Examen de Seleccin 05/06 1 Vuelta
Pregunta 18.-R: 3 Pregunta 23.-R: 5
Cuando una lcera gastroduodenal presenta una hemorragia di- H.pylori es un reconocido agente carcingeno debido a la atro-
gestiva y se confirma la infeccin por H.pylori, la erradicacin del fia gstrica y metaplasia que induce. La gastritis tipo A, frecuente-
microorganismo es una prioridad. Se inicia el tratamiento oral en mente asociada a anemia perniciosa, es tambin un factor de riesgo
cuanto el paciente comienza a tener una ingesta de alimentos. Se para el cncer. El adenocarcinoma en el Barrett puede, a veces,
usa la triple terapia con omeprazol, amoxicilina y claritromicina, localizarse dentro de lo que se considera anatmicamente el est-
cuya eficacia es del 95%; pero en caso de alergia o intolerancia a mago. El Lynch tipo II, asocia cncer colorrectal hereditario sin
alguno de los antibiticos se usa metronidazol y timidazol, no bismuto plipos con cncer gstrico. El alcoholismo crnico puede facilitar
coloidal, por eso la respuesta correcta es la 3. ste se utiliza en com- una gastritis, pero no hay evidencia de que sea un factor causal de
binacin con tetraciclina, metronidazol y omeprazol en caso de lcera o cncer gstrico.
que la triple terapia previa no consiga la erradicacin de H.pylori.
Si, an con esta triple terapia, no se consiguiera erradicar, se debe- Pregunta 24.-R: 2
ran tomar cultivos de la mucosa para obtener antibiograma y as Habitualmente en una diarrea aguda, de menos de 14 das de evolu-
seleccionar la terapia. cin, no se requiere obtener el diagnstico etiolgico, pues en general
Despus de una lcera con hemorragia es obligado confirmar la son procesos leves y autolimitados. En caso de deshidratacin, sepsis,
erradicacin de H.pylori con un test del aliento. En el momento del estado de inmunosupresin o edad muy avanzada, al igual que en las
sangrado de una lcera se deben evitar las biopsias de los bordes, duraciones prolongadas del cuadro diarreico de ms de 10 das es nece-
pero es que adems en esta pregunta se trata de una lcera duodenal, sario obtener el diagnstico. La primera prueba, y por eso es la respuesta
por lo que de todas formas es innecesaria la biopsia. correcta, es el estudio de los leucocitos fecales. Si es positivo sugiere un
origen inflamatorio y hay que esperar al resultado del coprocultivo para
Pregunta 19.-R: 1 el diagnstico. En caso de que las pruebas bacteriolgicas no consiguie-
En las lceras gstricas debe confirmarse siempre la erradicacin ran el diagnstico, se realizara una colonoscopia. Si no hay leucocitos en
de H.pylori tras completar tratamiento erradicador siendo altamente las heces, se entiende que es una diarrea acuosa aguda, en general debi-
recomendable en las duodenales (por tanto no deben realizarse siem- da a toxinas bacterianas, que suelen ser autolimitadas y precisan diagns-
pre de forma obligada, aunque sto no es contradictorio con el con- ticos microbiolgicos y epidemiolgicos especficos.
cepto de altamente recomendable), pasando a ser obligatorio si las
duodenales se complicaran con hemorragia. Pregunta 25.-R: 2
La prueba de eleccin es el test del aliento y para evitar falsos Para el diagnstico de enfermedad celaca se precisa una biopsia
negativos debe realizarse entre 15 das y un mes despus de haber que demuestre atrofia de las vellosidades, hiperplasia de las criptas e
suspendido omeprazol y antibiticos. infiltracin linfoplasmocitaria de la lmina propia. Si, adems, el
estudio inmunolgico es positivo, la actitud diagnstica pasa por
Pregunta 20.-R: 5 retirar el gluten de la dieta, observar la mejora clnica y la negativiza-
Las dos pruebas ms utilizadas para el diagnstico de la infeccin cin de los anticuerpos. Si sto ocurriera no es obligada una segun-
por H.pylori en ulcus gstrico son el test de la ureasa y el estudio con da biopsia dos aos despus de la primera que confirme la recupe-
tinciones de la mucosa biopsiada. Su sensibilidad depende de la ca- racin histolgica.
lidad de la muestra, por eso se debe evitar tomar muestras de las zonas Los linfocitos intraepiteliales estn aumentados en nmero en esta
con mayor atrofia o metaplasia, pues en estas localizaciones es muy enfermedad y desempean un papel patgeno importante. Se consi-
difcil demostrar la presencia de la bacteria. dera celaca latente a la positividad de los anticuerpos y aumento de
El tratamiento erradicador con la triple terapia requiere entre 7 y 10 linfocitos intraepiteliales en un familiar de un celiaco. La asociacin al
das y, en principio, no es necesario llegar hasta 14 das, pues el benefi- HLA es fundamentalmente al DQ2 y no al B51. En relacin con los
cio adicional es mnimo. Las lceras gstricas una vez concluido el anticuerpos se utilizan los antigliadina IgG e IgA, los antiendomisio IgA
tratamiento erradicador requieren un mantenimiento durante un pe- y, recientemente, los antitransglutaminasa IgA, que son muy sencillos
rodo no concretado de semanas (oscila entre 4 y 8) con un frmaco de realizar, sensibles y especficos.
antisecretor, preferentemente un anti-H2 (esta es la respuesta correcta) y
no es necesario en las duodenales, a no ser que debutaran sangrando o Pregunta 26.-R: 2
fueran gigantes. La ciruga del ulcus est fundamentalmente motivada Entre las causas de refractariedad al tratamiento sin gluten se en-
por sus complicaciones, no por su localizacin. En un ulcus no refrac- cuentran por orden de probabilidad:

Comentarios TEST
tario y sin otros sntomas ms que los habituales de la enfermedad 1. Incumplimiento diettico, que es la respuesta correcta.
ulcerosa, no es necesario solicitar gastrina. 2. Linfoma tipo T intestinal.
3. Esprue colgeno.
Pregunta 21.-R: 2 4. Otros diagnsticos distintos a la propia enfemedad celiaca, como
El paciente presenta un factor de riesgo para la gastropata y lcera el esprue autoinmune o el tropical.
complicada por AINES, como es la edad mayor de 70 aos, por lo
que requiere omeprazol oral (la respuesta correcta). Pregunta 27.-R: 4
Parece que erradicar H.pylori reducira el riesgo de lcera compli- En general secretora y acuosa suele ser un trmino equivalente.
cada al tomar un AINE, pero no se ha demostrado que evite tomar el Son causas los tumores endocrinos, como el vipoma, pepoma, o tu-
gastroprotector. En relacin a la ranitidina bismuto coloidal es un mor carcinoide (respuesta correcta), que liberan sustancias estimula-
frmaco con actividad frente a H.pylori y buena alternativa al ome- doras de la liberacin de lquido a la luz intestinal o que alteran la
prazol en la triple terapia. permeabilidad del epitelio intestinal. Otras causas son los plipos ve-
llosos grandes y mltiples. En los casos agudos, las diarreas por toxinas
Pregunta 22.-R: 5 bacterianas, como Staph.aureus o E.Coli.
No tiene factores de riesgo para asociar gastroprotector al consumo La vagotoma y el intestino irritable son diarreas motoras y el hidr-
de ibuprofeno, por lo que no necesita misoprostol y no es necesario xido de magnesio y la lactulosa osmticas.
sustituir ibuprofeno por rofecoxib.
El tabaquismo tiene un efecto deletreo comprobado bsicamen- Pregunta 28.-R: 5
te como elemento que dificulta la cicatrizacin de las lceras. La dieta La enteropata pierdeprotenas no es una enfermedad, sino una
no tiene influencia. modalidad de expresin clnica en la que predominan la prdida de
Una dispepsia inducida por antiinflamatorios debe tratarse reti- protenas por la luz intestinal frente a la maldigestin o malabsorcin
rando si es posible el antiinflamatorio y aadiendo un anticico o proteica de otros procesos. Son causa la propia enfermedad celaca,
un anti-H2 durante un tiempo limitado, por eso la respuesta correc- las infiltraciones tumorales de la pared intestinal, la linfangiectasia o
ta es la 5. las obstrucciones al drenaje del conducto torcico, como el carcino-

M exico A rgentina CTO Medicina C/ Nez de Balboa, 115 28006 MADRID (Espaa) Tfno.: (91) 782 43 32 / Fax: (91) 782 43 27
Pg. 4 HM C hile U ruguay E-mail: secretaria@ctomedicina.com; iberocto@ctomedicina.com WEB: www.ctomedicina.com; www.iberocto.com
DIGESTIVO Y CIRUGA GENERAL
Preparacin Examen de Seleccin 05/06 1 Vuelta Seguimiento a distancia
ma de esfago o la pericarditis constrictiva. La prueba diagnstica
utilizada con ms frecuencia es el aclaramiento entre las cifras fecales
y sricas de alfa1-antitripsina.
El hipertiroidismo provoca una diarrea por hipermotilidad (res-
puesta correcta) y no causa una enteropata pierdeprotenas.

Pregunta 29.-R: 5
En general siempre existe un factor o antecedente personal asocia-
do que justifica el sobrecrecimiento bacteriano. En los gastrectomiza-
dos es muy frecuente (respuesta correcta) sobre todo tipo Billroth II,
tambin en los by-pass intestinales, sndrome de intestino corto,
divertculos yeyunales, fstulas y asas ciegas que son los ejemplos ms
frecuentes.
Produce maldigestin por alteracin de las sales biliares. A veces
hay ligera atrofia intestinal, que no es responsable de los trastornos
nutricionales. Si no se resuelve la enfermedad subyacente, es frecuen-
te la recurrencia. La prueba diagnstica de certeza es el recuento de
aerobios y anaerobios en jugo intestinal aspirado superior a 100.000
UFC; pero la dificultad de esta prueba obliga a utilizar los test del
aliento, siendo el ms rentable el de xilosa con carbono isotpico.

Pregunta 30.-R: 5
Esta pregunta hace referencia al diagnstico diferencial de la ascitis
quilosa:
1. Los traumatismos abdominales con fistulas linftico-peritoneales.
2. Los linfomas.
3. La cirrosis heptica ocasionalmente.
4. La linfangiectasia primaria (respuesta correcta). Se acompaa de
edemas en MMII, linfopenia y aclaramiento de alfa-1-antitripsina
elevado pues es una enteropata pierde protenas. Pregunta 32. Hemorragia digestiva baja.

Pregunta 31.-R: 1 Pregunta 34.-R: 3


La causa ms frecuente de hemorragia digestiva baja en un ancia- La presencia de aftas en colon sugiere una colitis de Crohn y en
no es el sangrado por divertculos de colon, sobre todo los situados en este caso leve-moderada. Su tratamiento incluye:
el hemicolon derecho. Sin embargo, suele ser autolimitado y rara- 1. Por afectar el colon se recomienda, aunque no es obligado, metro-
mente recurrente. nidazol a dosis no muy elevadas como inmunomodulador.
La angiodisplasia aparece como segunda causa, pasando a ser la 2. 5-aminosalicilatos, p.e. mesalazina, aunque su eficacia es menor
primera en caso de sangrado recurrente. Se observa con ms frecuen- que en la colitis ulcerosa y frecuentemente se necesitan aadir
cia si coexiste estenosis artica o insuficiencia renal y para su valora- dosis bajas de esteroides orales o en enema.
cin la colonoscopia no debe limitarse hasta ngulo esplnico como
en este caso, sino realizarse completa hasta ciego. Esta prueba es la de Si hubiera sido severa los corticoides se pautan desde el inicio y a
mayor rentabilidad diagnstica superior incluso a la propia arteriografa dosis altas (ver figura).
permitiendo el tratamiento con fotocoagulacin en los casos en que
el sangrado se limite a un nmero reducido de lesiones.

Pregunta 32.-R: 1
Comentarios TEST

Las causas ms frecuentes de hemorragia digestiva baja varan se-


gn la edad. En los mayores de 60 aos, que es el grupo con mayor
incidencia, el orden por frecuencia es:
1. Divertculos.
2. Angiodisplasia.
3. Plipos.
4. Neoplasia.

Tericamente la prueba de eleccin es una colonoscopia completa,


pero esta prueba requiere una preparacin. Una sigmoidoscopia se
puede realizar casi de forma inmediata por lo que puede ser la primera
prueba a realizar sobretodo si el sangrado es relevante. El algoritmo
diagnstico se observa en la figura. La gastroduodenoscopia no aporta Pregunta 34. Tratamiento de un brote de enfermedad inflamatoria intestinal.
nada de inters en una HDB y la arteriografa es til slo si el sangrado es
cuantioso. Pregunta 35.-R: 1
Los ANCA patrn perinuclear aparecen en el 70% de las colitis
Pregunta 33.-R: 1 ulcerosas, pero es tambin conveniente que recordemos que en el
La presencia de melenas es indicativo de una hemorragia digestiva, MIR aparecen en la poliangetis microscpica, hepatitis autoinmune,
en el 80% de los casos alta. La causa ms frecuente de hemorragia colangitis esclerosante, y el patrn citoplasmtico en la enfermedad
digestiva en un paciente previamente sano es el ulcus duodenal, (res- de Wegener.
puesta correcta) en segundo lugar el ulcus gstrico, en tercero las Actualmente se han descrito en algunos casos de Crohn anticuer-
erosiones y gastritis de stress. En este caso coexiste anemia ferropnica pos antisacharomyces (ASCA).
que en una mujer joven es secundaria al propio ulcus o a prdidas Son caractersticas de la colitis ulcerosa:
menstruales. Es de inters recordar que en un anciano la anemia 1. Es muy infrecuente en fumadores.
ferropnica obliga a excluir la neoplasia de colon. 2. Afecta siempre al recto y slo en un 20% a todo el colon.

M exico A rgentina CTO Medicina C/ Nez de Balboa, 115 28006 MADRID (Espaa) Tfno.: (91) 782 43 32 / Fax: (91) 782 43 27
C hile U ruguay E-mail: secretaria@ctomedicina.com; iberocto@ctomedicina.com WEB: www.ctomedicina.com; www.iberocto.com
HM Pg. 5
DIGESTIVO Y CIRUGA GENERAL
Seguimiento a distancia Preparacin Examen de Seleccin 05/06 1 Vuelta
3. El sntoma clnico ms frecuente es la rectorragia, pero casi siempre Pregunta 38.-R: 1
con gran emisin de moco, incluso en las remisiones. Los esteroides son tiles para lograr la remisin. Pero tanto en la
4. Aproximadamente entre el 6 y el 10% de los que presentan colitis colitis ulcerosa como en la enfermedad de Crohn, su uso como man-
ulcerosa, y no el 50%, asocian una colangitis esclerosante. tenimiento no garantiza una remisin prolongada (respuesta correcta
la nmero 1). Cuando se suspenden o se reduce la dosis no se modi-
Pregunta 36.-R: 4 fica el riesgo de padecer un nuevo brote. Por tanto, en el manteni-
La descripcin endoscpica de una mucosa eritematosa y granular miento de la enfermedad de Crohn se opta ante un primer brote leve
en colon o recto es muy sugestiva de colitis ulcerosa, ms an si se o moderado por no tomar ninguna medida, salvo si la afectacin en el
describieran lesiones petequiales, erosiones o lceras. No obstante, el brote fue una colitis que mejor con salicilatos, en cuyo caso se pue-
diagnstico definitivo no es endoscpico sino que se debe disponer de de optar por probar a continuar con este tratamiento como manteni-
histologa compatible aunque no sea patognomnica. La presencia de miento. En caso de brote inicial muy grave o brotes repetidos, se pasa
sepsis, deshidratacin severa, o, como en este caso, seis ms deposi- directamente al mantenimiento con un inmunosupresor como la
ciones al da, supone que el episodio es severo y, por tanto, el tratamien- azatioprina.
to es con 5-aminosalicilatos ms esteroides (respuesta n 4). Los salicilatos En los brotes graves refractarios a dosis altas de corticoides puede
como nico tratamiento se emplean en las formas leves. En la colitis emplearse ciclosporina i.v.
ulcerosa el metronidazol slo se utiliza si se asocia megacolon txico. En algunos casos los enfermos de Crohn tienen diarrea no por
actividad de la enfermedad sino por sus secuelas:
Pregunta 37.-R: 4 1. Sobrecrecimiento bacteriano.
En general, tanto la colitis ulcerosa como la enfermedad de Crohn 2. Intestino corto postquirrgico.
pueden presentar los mismos sntomas, pero son ms tpicas de colitis 3. Ante extensa afectacin ileal pueden no absorberse bien los cidos
ulcerosa la rectorragia y la friabilidad al estudio endoscpico (res- biliares que al pasar a colon provocaran una diarrea secretora
puesta correcta) y ms propio de la enfermedad de Crohn: dolor cuyo tratamiento es la colestiramina.
abdominal sobre todo en fosa ilaca derecha, prdida de peso, fiebre,
presencia de masa abdominal, fstulas y cuadros obstructivos o Pregunta 39.-R: 4
pseudoobstructivos. Es conveniente recordar que aunque la enferme- La presencia de diarrea con mltiples deposiciones y deshidrata-
dad de Crohn pueda afectar a capas profundas de la pared intestinal, cin en una enferma con colitis ulcerosa sugiere un brote severo. La
es ms frecuente la presencia de perforaciones en la colitis ulcerosa, distensin abdominal no es propia del brote y sugiere un megacolon
posiblemente por su asociacin a megacolon. txico, que se diagnostica en una Rx simple de abdomen con dimetro

Comentarios TEST

Pregunta 46. Caractersticas ms relevantes de las poliposis familiares.

M exico A rgentina CTO Medicina C/ Nez de Balboa, 115 28006 MADRID (Espaa) Tfno.: (91) 782 43 32 / Fax: (91) 782 43 27
Pg. 6 HM C hile U ruguay E-mail: secretaria@ctomedicina.com; iberocto@ctomedicina.com WEB: www.ctomedicina.com; www.iberocto.com
DIGESTIVO Y CIRUGA GENERAL
Preparacin Examen de Seleccin 05/06 1 Vuelta Seguimiento a distancia
luminal de colon transverso superior a 6 cm. Esta situacin es muy grave ocasiones su cuadro clnico puede ser parecido al de un cncer de
y obliga al tratamiento propio de las formas severas ms metronidazol. Si colon incipiente y, por eso, cuando la clnica de colon irritable
no responde en 48 h o se perfora, est indicada la colectoma urgente. aparece por primera vez en sujetos de ms de 50 aos se debe
excluir la presencia de un cncer de colon mediante la realizacin
Pregunta 40.-R: 5 de pruebas diagnsticas como la colonoscopia o el enema opaco.
Es una lesin por alteracin de la microvasculatura arterial y, por
tanto, la arteriografa no es diagnstica y s lo es la histologa del colon; Pregunta 47.-R: 4
por esto, tanto las respuestas 1 como 3 son falsas. Afecta al colon, sobre La colitis ulcerosa y la enfermedad de Crohn son enfermedades en
todo al ngulo esplnico, pero respeta al recto. Las imgenes en la Rx en las que aumenta el riesgo de padecer cncer de colon. Este aumento
huella dactilar se observan tambin en las formas agudas. es ms notorio en la colitis ulcerosa.
Los sntomas, generalmente diarrea, suelen remitir de forma habi- El riesgo depende fundamentalmente del grado de actividad de la
tual en unas pocas semanas y no suele haber recurrencias, por eso la enfermedad y de la duracin de la misma. Aumenta a partir de los 10
respuesta correcta es la 5. aos en enfermos con formas crnicamente activas o con multibrotes;
por eso si la enfermedad debuta en edad juvenil hay un mayor poten-
Pregunta 41.-R: 1 cial de padecer ms brotes a lo largo de la vida y por tanto ms riesgo
Para el diagnstico de la enfermedad diverticular del sigma o colon de desarrollar un carcinoma.
la prueba de eleccin es el enema opaco. Est contraindicado su Otro factor adicional es la extensin de la enfermedad, por ejem-
prctica si se sospecha una complicacin como la diverticulitis, por el plo: no hay ms riesgo de cancer cuando nicamente present
riesgo de perforacin y peritonitis clnica. El diagnstico de diverticu- proctitis.
litis es clnico y bioqumico: anciano con fiebre, dolor en fosa ilaca En los casos en que su severidad as lo indique y a partir de los 10
izquierda y leucocitosis. La sospecha de una diverticulitis complicada aos de evolucin, se recomiendan colonoscopias cada dos aos
con un absceso de pared por persistencia de la fiebre o una palpa- para tomar biopsias y valorar la presencia de neoplasias precoces. Si
cin muy sugestiva de peritonismo, se comprueba mediante un TAC. se demuestran zonas displsicas se asume tal riesgo de neoplasia y se
Esta prueba es la ms rentable para el diagnstico de masas, tumores y recomienda la colectoma; pero sta no se indica por el mero hecho
abscesos del espesor de la pared intestinal. de llevar 15 aos de evolucin (por eso la respuesta incorrecta es la 4).

Pregunta 42.-R: 3 Pregunta 48.-R: 1


La enfermedad diverticular del colon es muy frecuente, general- La causa ms frecuente de cancer de colon hereditario, a edad
mente es asintomtica. En caso de producirse clnica es ms frecuente inferior a los 40 aos, es el carcinoma colorrectal hereditario no aso-
la diverticulitis que la hemorragia; pero es tan prevalente la enferme- ciado a plipos (respuesta correcta la nmero 1). Esta entidad es cinco
dad que la hemorragia diverticular, sobre todo localizada en colon veces ms frecuente que la poliposis adenomatosa familiar del colon.
derecho, es la causa ms frecuente de hemorragia digestiva baja en
mayores de 60 aos. Como no suele ocurrir sobre un divertculo Pregunta 49.-R: 1
inflamado: es indolora, autolimitada y no recidiva; por eso la falsa es Un paciente con cncer de colon necesita para su seguimiento:
la nmero 3. 1. Colonoscopias peridicas.
2. Ecografa abdominal para observar posibles lesiones metastsicas
Pregunta 43.-R: 4 hepticas.
El punto fundamental que da mal pronstico en cuanto al poten- 3. La determinacin del antgeno carcinoembrionario (CEA). Su ele-
cial de malignizacin de un plipo es la presencia y grado de displa- vacin significa recidiva tumoral (respuesta 1). El test de hemorra-
sia en su interior. Lgicamente a ms plipos y de mayor tamao, gias ocultas no est indicado por su baja rentabilidad para el segui-
ms riesgo de neoplasia. El tipo histolgico velloso de los adenomas miento de las neoplasias de colon.
tiene ms riesgo que el tubular. El hiperplsico no es una lesin
premaligna. El rea de intestino grueso que est afectada no se aso- Las segundas neoplasias pueden surgir en otra localizacin en
cia a un mayor riesgo de transformacin neoplsica (respuesta co- un enfermo con cncer de colon despus de la ciruga. Por ejemplo
rrecta la 4). en un enfermo con colectoma puede aparecer una segunda neo-
plasia en recto; pero esta situacin, adems de infrecuente, no tiene
Comentarios TEST

Pregunta 44.-R: 3 por qu cursar necesariamente con CEA elevado, puesto que este
Los plipos, tanto hiperplsicos como adenomatosos, suelen ser segundo tumor puede encontrarse en un estado poco avanzado.
asintomticos; pero en caso de producir sntomas suelen presentar
rectorragia o hematoquecia. Unicamente producen diarrea cuando Pregunta 50.-R: 5
son de tipo velloso y adems muy indiferenciados, mltiples y de gran El que los anticuerpos antigliadina sean negativos hace muy impro-
tamao. La intususcepcin es menos frecuente y puede provocar bable que se trate de una celaca o su variante espre colgeno.
dolor e leo. En el sndrome de intestino irritable, tanto la visin colonoscpica
como las biopsias son normales. El hipertiroidismo da una diarrea
Pregunta 45.-R: 2 motora.
En poblacin sana con edad superior a los 50 aos y sin factores La respuesta correcta es la colitis linfoctica. Esta entidad se asocia
de riesgo asociados se recomienda un screening con un test de hemo- con cierta frecuencia a la celiaca. En general o no se aprecian altera-
rragias ocultas cada dos aos. Pero siempre que exista un factor de ciones endoscpicas en el colon o son leves e inespecficas. El diag-
riesgo personal (antecedente de colitis ulcerosa o como en este caso nstico se establece al obtener muestras por biopsia para estudio
plipos adenomatosos previos) o familiar por antecedente de neopla- histologico (hay que resaltar que en un protocolo de diarrea se toman
sia de colon, el cribado debe hacerse con colonoscopia cada ao y biopsias incluso con un aspecto endoscpico normal). Se aprecia un
medio o dos. Este plan es indefinido, pues es la nica forma de detec- infiltrado linfoplasmocitario y en ocasiones una banda de colgeno
tar precozmente un cncer de colon. en cuyo caso recibe el nombre de colitis colgena. El tipo de diarrea
que produce es secretora.
Pregunta 46.-R: 2
Las caractersticas ms relevantes de las poliposis familiares se Pregunta 51.-R: 4
ilustran en la figura. Son factores de riesgo bien reconocidos para el En un paciente de edad media la presencia de mltiples plipos
cncer de colon: la presencia de cualquier poliposis familiar, ex- adenomatosos y un cncer en sigma o colon sugiere fuertemente que
cepto en el sndrome de Cowden; los antecedentes familiares de se trate de una poliposis adenomatosa familiar del colon. Esta entidad,
cncer de colon y los antecedentes de enfermedades como la coli- dada su herencia autosmica dominante y alto grado de penetrancia,
tis ulcerosa o el Crohn. No as el sndrome del intestino irritable (por obliga a los familiares a someterse a un cribado del cncer de colon.
eso es la respuesta correcta). Otro tema bien distinto es que en Este debe iniciarse antes de los 20 aos y si se demuestran plipos
M exico A rgentina CTO Medicina C/ Nez de Balboa, 115 28006 MADRID (Espaa) Tfno.: (91) 782 43 32 / Fax: (91) 782 43 27
C hile U ruguay E-mail: secretaria@ctomedicina.com; iberocto@ctomedicina.com WEB: www.ctomedicina.com; www.iberocto.com
HM Pg. 7
DIGESTIVO Y CIRUGA GENERAL
Seguimiento a distancia Preparacin Examen de Seleccin 05/06 1 Vuelta
extirpar el colon. Si con edad superior a 40 aos no se han demostra- Ambas opciones son vlidas, pero es ms eficiente, pues ahorra
do plipos, es que ese familiar no va a padecer la enfermedad y por costes, la paracentesis.
tanto no tiene sentido someterle a colonoscopias de seguimiento (res- 3. Si la ascitis es a tensin, la paracentesis evacuadora con reposicin
puesta correcta es la nmero 4). de albmina es el tratamiento de eleccin (respuesta correcta).
4. En los casos refractarios, surge la indicacin de transplante hepti-
Pregunta 52.-R: 5 co. Se ha comprobado que en esta situacin la colocacin de TIPS
El aumento aislado de bilirrubina indirecta, o no conjugada, es resuelve la ascitis refractaria en el 30% de los casos.
generalmente leve o moderado, excepto en el recin nacido con
ictericia fisiolgica que puede llegar a producir Kernicterus. Causas
habituales son las anemias hemolticas y el sndrome de Gilbert. En
ambos casos la bilirrubina no suele exceder de 4-5 mg/dl. Tambin
se observa en una forma o variante del Gilbert, como es el Crigler-
Najar, donde alcanza cifras ms importantes y puede ser incluso
mortal en nios con el tipo I. Otras causas son el hipotiroidismo, la
lactancia materna o los estados de hipoalbuminemia severa. Todas
estas entidades no presentan bilirrubinuria. El sndrome de Dubin-
Johnson se caracteriza porque la bilirrubina ya conjugada no pue-
de pasar a los canalculos biliares y, por tanto, la bilirrubinemia es
de tipo conjugado o directo, por eso es la respuesta correcta. Muy
parecido a este cuadro es el sndrome de Rotor, por tanto tambin
con elevacin predominante de la bilirrubina conjugada. Estos dos
ltimos diagnsticos s se acompaan de bilirrubinuria.

Pregunta 53.-R: 5
Siempre que hay colestasis, un Dubin-Johnson o un sndrome de
Rotor, predomina la bilirrubina conjugada y se encuentra bilirrubina
en orina, puesto que sta s puede filtrarse por el rin. El sndrome de
Gilbert (respuesta correcta) causa aumento de bilirrubina indirecta y,
por tanto, no cursa con hiperbilirrubinuria.

Pregunta 54.-R: 4
En un sndrome colesttico la ecografa (respuesta correcta) es la
primera prueba que debe realizarse. Si se demuestra dilatacin de la
va biliar principal supone probablemente que se trate de una coles-
tasis extraheptica y el siguiente paso diagnstico es un TAC. En caso
de no obtener el diagnstico, se realiza una colangiografa retrgrada
endoscpica. Esta tcnica es cruenta: puede producir un 5% de pan-
creatitis y un 1% de colangitis, por eso actualmente estn cobrando
importancia otras tcnicas:
1. La ecografa endoscpica de la va biliar, situando el endoscopio Pregunta 56. Manejo de la ascitis.
en duodeno se realiza un barrido de pncreas y va biliar. Tiene
una alta rentabilidad para las enfermedades del coldoco, sobre Pregunta 57.-R: 3
todo la litiasis. Para determinar la presencia de hipertensin portal se debe cons-
2. La colangiografa por resonancia magntica, que es incruenta y no tatar:
precisa contraste. 1. Que las presiones estn aumentadas en vena porta, determinadas
3. La colangiografa i.v. y la oral no tienen utilidad, salvo en los casos por mtodo cruento, o
previos a la litotricia biliar para evaluar si la vescula es funcionan- 2. De forma indirecta, visualizando varices esofgicas o gstricas en la

Comentarios TEST
te. endoscopia.

Pregunta 55.-R: 5 Otro tema bien distinto es evaluar si el origen de un lquido asc-
La intoxicacin por paracetamol produce un fallo heptico agu- tico es atribuible o no a la hipertensin portal. Para ello se utiliza el
do por un efecto txico directo por consumir los niveles de glutatin gradiente, o resta, entre la albmina en suero y la albmina en
del hepatocito. Se produce con dosis superiores a los 8 gramos al ascitis. Cuando esta resta da un resultado superior a 1,1 gr/dl se
da, excepto en alcohlicos, en los que incluso se puede producir presume que la ascitis es por hipertensin portal, como sucede en
con dosis dentro del rango teraputico habitual. Tiene buen pro- cualquier tipo de cirrosis, BuddChiari, enfermedad venooclusiva
nstico si se consigue administrar precozmente N-acetilcistena i.v. heptica, esquistosomiasis o trombosis de la porta. Si el gradiente es
Las transaminasas estn muy elevadas y se acompaa de alteracio- menor de 1,1 sugiere otros diagnsticos distintos al de hipertensin
nes en la coagulacin y si hay fracaso heptico establecido, de portal, como la tuberculosis peritoneal (es la respuesta correcta, ya
encefalopata (por eso la 5 es la respuesta correcta). que el gradiente es de 3,1) o la carcinomatosis peritoneal.

Pregunta 56.-R: 3 Pregunta 58.-R: 1


El tratamiento de la ascitis por descompensacin hidrpica de un La presencia de clulas tumorales malignas en el lquido asctico
cirrtico es (ver figura de la pgina siguiente): sucede porque el peritoneo est infiltrado por un tumor. En general
1. En los casos leves-moderados: restriccin diettica de sal y espiro- suele ser metastsico fundamentalmente de colon o estmago (res-
nolactona con dosis progresivas segn resultado si no fuera sufi- puesta correcta 1) y es independiente el que se asocie o no a metsta-
ciente la de 100 mg/da. En ocasiones puede ser necesario aadir sis hepticas.
un segundo diurtico. Esta situacin confiere muy mal pronstico, ya que la sobrevida de
2. En los casos severos se puede optar por la combinacin de espiro- una neoplasia con carcinomatosis peritoneal oscila, como mediana,
nolactona y otro diurtico como furosemida o torasemida, o bien en los 6 meses. Si hubiera sido quilosa se nos describira un aumento
por la paracentesis evacuadora con reposicin de albmina i.v. notable de los triglicridos en el lquido.

M exico A rgentina CTO Medicina C/ Nez de Balboa, 115 28006 MADRID (Espaa) Tfno.: (91) 782 43 32 / Fax: (91) 782 43 27
Pg. 8 HM C hile U ruguay E-mail: secretaria@ctomedicina.com; iberocto@ctomedicina.com WEB: www.ctomedicina.com; www.iberocto.com
DIGESTIVO Y CIRUGA GENERAL
Preparacin Examen de Seleccin 05/06 1 Vuelta Seguimiento a distancia
Pregunta 59.-R: 2 mita justificar la infeccin, debi ser, pues, inaparente. Por tanto es
Se produce por la colonizacin y posterior infeccin del perito- una respuesta falsa que la mayora tengan historia de transfusin, esto
neo por microorganismos propios de la flora intestinal del intestino s ocurri en los primeros aos de descripcin de la enfermedad, pero
delgado: por eso se produce en primer lugar por gramnegativos, como no en la actualidad.
E.coli, y en segundo lugar por cocos gram positivos, incluido el neu- Se ha calculado que el 80% de los casos evolucionan a la cronici-
mococo. dad. El RNA es el mejor marcador para el diagnstico. Inicialmente se
La clnica dolorosa es poco relevante. Es muy infrecuente el pe- hace un cribado con los anticuerpos anti-VHC y, si son positivos, la
ritonismo, por eso la respuesta a sealar es la nmero 2. A diferencia infeccin se confirma solicitando amplificacin genmica viral, que
de las secundarias la presencia de anaerobios es excepcional. El demuestra si hay positividad para el RNA viral.
tratamiento es mdico, con cefalosporinas de tercera generacin.
La prueba diagnstica de certeza es el cultivo del lquido asctico, Pregunta 65.-R: 3
pero, dada la gravedad de la enfermedad, se precisa iniciar un trata- Para diagnosticar una hepatitis aguda se precisa, tanto en los casos
miento emprico y, para ello, es suficiente con demostrar un recuento que tengan clnica como en los asintomticos, demostrar un incre-
de neutrfilos superior a 250/mm3 en lquido asctico. mento notable y agudo de las transaminasas. Una vez que tenemos el
diagnstico probable clnico-bioqumico o slo bioqumico es cuan-
Pregunta 60.-R: 2 do se solicitan los marcadores virales y, en concreto, la serologa IgM
Las indicaciones de la profilaxis antibitica con quinolonas orales es la que va a ser positiva en los casos agudos (respuesta correcta).
en la peritonitis bacteriana espontnea de un cirrtico son: En la infeccin aguda por VHB los 5 marcadores de esta pregunta
1. Sin antecedentes de haber padecido antes una peritonitis: si se son positivos, pero el que es especfico de hepatitis aguda es el anti-
presenta una hemorragia digestiva (respuesta correcta) o las prote- cuerpo IgM.
nas en lquido asctico son inferiores a 1 gr/dl.
2. Como profilaxis secundaria despus de un episodio de peritonitis. Pregunta 66.-R: 1
Se mantiene la profilaxis con quinolonas de forma indefinida hasta La infeccin por el virus de la hepatitis C afecta tambin a rganos
que el paciente deje de tener ascitis. extrahepticos, sobre todo a los ganglios linfticos, generando riesgo
de linfomas y de enfermedades de naturaleza inmune como la aplasia
Pregunta 61.-R: 5 medular, eritroblastopenia, glomerulonefritis, liquen plano, sndrome
El sndrome hepatorrenal se produce como una forma de evolu- seco, y, sobre todo la ms frecuente, la crioglobulinemia mixta esen-
cin terminal de una hepatopata crnica avanzada. Pero tambin se cial. Esta entidad es una vasculitis que cursa con crioglobulinas positi-
reconocen factores precipitantes del sndrome hepatorrenal: la peri- vas en sangre, polineuropata, afectacin renal y prpura. La etiologa
tonitis bacteriana espontnea, la hepatitis alcohlica aguda y cada vez es, prcticamente siempre, debida a la infeccin por el virus de la
con menor frecuencia el uso intempestivo de diurticos. hepatitis C.
Se trata de una vasoconstriccin funcional irreversible, que afecta
bsicamente a las arteriolas de filtracin, pero en escasa cuanta a las Pregunta 67.-R: 3
arterias de las cuales se obtiene la perfusin de la nefrona, por lo cual En general la hepatitis por virus B tiene una tasa de cronificacin
sta no resulta daada desde el punto de vista morfolgico y as se baja que oscila alrededor del 5% en adultos. La va de adquisicin no
entiende que exista un fallo en la filtracin renal, y, por tanto, insufi- influye en la cronicidad, salvo en los nios que lo suelen adquirir en
ciencia renal progresiva y severa con un rin histolgicamente nor- el canal del parto y tienen un riesgo mucho ms elevado de padecer
mal (respuesta correcta la 5). una forma crnica, llegar a cirrosis o quedar como portadores. Por
eso la respuesta correcta es la 3.
Pregunta 62.-R: 5
El tratamiento de eleccin del sangrado por varices esofgicas es la Pregunta 68.-R: 3
combinacin de un tratamiento endoscpico como la esclerosis o, si Los pacientes con VHB y hepatitis crnica o portadores crnicos,
es posible, la ligadura endoscpica de las varices, ms el farmacolgi- cuando sufren un episodio de hepatitis aguda debe pensarse en las
co con somatostatina o un anlogo de la vasopresina. Si este trata- siguientes posibilidades:
miento no es eficaz y contina con sangrado, se indica la colocacin 1. Que est sucediendo el proceso de seroconversin: la prdida de
de un TIPS, excepto en el estado A de Child, en donde se indica la la capacidad replicativa del virus. Esta es la causa ms frecuente de
Comentarios TEST

ciruga urgente derivativa. Si esta recidiva es severa y no permite espe- elevacin de las transaminasas; en general es poco intensa y no
rar a estos procedimientos, hay que colocar una sonda de Sengstaken suele cursar con complicaciones, excepto ligera ictericia. Se diag-
hasta que se pueda realizar el tratamiento intervencionista. nostica porque el DNA viral se negativiza.
Se deben transfundir hemates si hay sndrome anmico, prescribir 2. Sobreinfeccin por un virus de la hepatitis A. Esta situacin es de
quinolonas orales para la profilaxis de la peritonitis y lactulosa para la alto riesgo para desarrollar un fracaso heptico fulminante. Se re-
profilaxis de la encefalopata heptica. comienda que estos enfermos estn vacunados para el virus A, si no
Est contraindicado el empleo de diurticos o subir las dosis, pues- han padecido previamente la infeccin. Esta situacin se diagnos-
to que provocaran un mayor deterioro hemodinmico (respuesta tica porque la IgM anti-VHA es positiva.
correcta). 3. Sobreinfeccin por el virus Delta. Esta situacin es hoy infrecuente,
slo se observa en hemoflicos y ADVP. Conlleva un riesgo elevado
Pregunta 63.-R: 4 de hepatitis fulminante y, tambin, favorece que el virus Delta quede
La profilaxis primaria de la hemorragia digestiva por varices debe en situacin de cronicidad. Se diagnostica porque se observan, como
hacerse con propanolol o nadolol. Ha demostrado reducir el n- en el presente caso, marcadores de infeccin aguda por virus D. El
mero de primeros episodios de sangrado y la mortalidad atribuible ms importante y precoz de ellos es el antgeno Delta, que es positi-
al mismo. Se debe iniciar siempre que se observen varices y que stas vo. En ningn caso puede ser una coinfeccin, ya nos lo aclara el
sean de riesgo, es decir, grandes (grado III IV). Si no se observan, enunciado, en el que se describe a un paciente diagnosticado de
como ocurre en este caso, o son pequeas, nicamente se debe infeccin crnica por el VHB y la coinfeccin es la situacin de dos
someter a vigilancia endoscpica peridica para ver cmo evolu- infecciones agudas simultneas B y D (ver figuras).
cionan las varices y por tanto no se precisa un tratamiento farmaco-
lgico. Pregunta 69.-R: 3
La positividad IgM para el virus A en una hepatitis aguda ofrece el
Pregunta 64.-R: 4 diagnstico de seguridad de hepatitis aguda por VHA. Por eso la res-
La infeccin por el virus de la hepatitis C sucede por va parenteral puesta correcta es la nmero 3. En relacin con la serologa del virus
y por eso se observa en drogadictos, pero cada vez con mayor fre- B, es probable que el enfermo se encuentre en una fase previa a la
cuencia hay casos en que no se demuestra un antecedente que per- aparicin de la respuesta inmune: todava no la tiene porque es anti-

M exico A rgentina CTO Medicina C/ Nez de Balboa, 115 28006 MADRID (Espaa) Tfno.: (91) 782 43 32 / Fax: (91) 782 43 27
C hile U ruguay E-mail: secretaria@ctomedicina.com; iberocto@ctomedicina.com WEB: www.ctomedicina.com; www.iberocto.com
HM Pg. 9
DIGESTIVO Y CIRUGA GENERAL
Seguimiento a distancia Preparacin Examen de Seleccin 05/06 1 Vuelta
S negativo. La asociacin de: anti-E positivo, anti-Core positivo, aun- gativo y anti-e positivo. Esto es lo contrario de lo que sucede habitual-
que no explican si es IgG o IgM, ms un antgeno de superficie nega- mente con la cepa salvaje, que cuando replica es: DNA virus B positi-
tivo sugiere que el paciente est atravesando el perodo ventana pre- vo, pero con antgeno e positivo y anti-e negativo.
vio a la resolucin virolgica definitiva.
Pregunta 73.-R: 2
Lamivudina es un antiviral muy eficaz para el tratamiento del VHB.
Sus indicaciones son:
1. En la cepa mutante precore (por eso la respuesta correcta es la 2).
2. En VIH positivos.
3. En intolerancia o contraindicacin al Interfern.

Tiene como ventajas que la administracin es oral y tiene escasos


efectos adversos frente a Interfern; sin embargo se debe prolongar su
uso alrededor de un ao, a diferencia del Interfern, que oscila alre-
dedor de los cuatro meses.
Lamivudina es una alternativa a Interfern, pero no est indicado
su uso combinado.

Pregunta 74.-R: 2
El objetivo primordial es lograr que deje de replicar el virus B, es
decir que se haga DNA negativo. Este proceso tambin se conoce
como seroconversin anti-e, ya que los pacientes pierden el antgeno
e y desarrollan anticuerpos anti-e. El fundamento de este objetivo es
que, una vez pasado a no replicante, la progresin histolgica de la
hepatitis queda detenida y, por tanto, se reduce casi por completo el
riesgo de desarrollar cirrosis heptica. Este objetivo se consigue entre
el 50-60% de los casos, pero slo en un 15% se consigue negativizar el
antgeno de superficie y conseguir la inmunidad, siendo anti-s positi-
vo. La mayora de los pacientes no consiguen eliminar por completo
el virus y, aunque no sean replicantes, quedan como portadores.

Pregunta 75.-R: 5
La enferma tiene, sin duda, una infeccin por virus C, puesto que
es anti-VHC positivo y el RNA es tambin positivo. Pero con normali-
dad de las transaminasas no se indica la biopsia heptica, porque a
pesar de que est documentada la infeccin, no se va a prescribir
tratamiento. Slo se realiza biopsia a los pacientes con hipertransami-
nasemia prolongada. Slo reciben tratamiento los pacientes en los
Pregunta 68. Coinfeccin y sobreinfeccin por hepatitis D. que la biopsia demuestre hepatitis crnica activa, no se trata de entra-
da a los que tienen hepatitis persistente en la histologa. Por eso este
Pregunta 70.-R: 2 paciente precisa nicamente seguimiento evolutivo, que es la res-
En general un ttulo de anti S menor de 10 es muy bajo y, aunque puesta correcta.
no excluye que el paciente tenga una respuesta inmune porque no es El tratamiento, en caso de necesitarse, es actualmente la combina-
un inmunodeprimido, lo adecuado es plantearlo como un fallo par- cin de Interfern y Ribavirina, que consigue eficacia en prctica-
cial en la respuesta y administrar inmunoglobulina especfica y una mente el 60% de los sujetos. Es un factor predictivo de eficacia tera-
dosis de recuerdo de la vacuna, siendo innecesaria la vacunacin putica comprobar que a las 12 semanas de tratamiento el RNA viral
completa. Es ms seguro esta pauta que administrar nicamente una se ha negativizado. Se considera xito teraputico cuando se cumple

Comentarios TEST
dosis de recuerdo. el concepto de respuesta teraputica sostenida: negativizacin persis-
tente del RNA seis meses despus de concluir la terapia.
Pregunta 71.-R: 4
El colapso multilobular es propio de las situaciones de fracaso Pregunta 76.-R: 1
heptico fulminante. La presencia de un infiltrado mononuclear slo En el tratamiento de la hepatitis crnica por VHB se utiliza Interfe-
nos permitir establecer el diagnstico de hepatitis persistente y no el rn alfa, no gamma (respuesta correcta la 1). Este ltimo se usa cuando
de crnica activa. Es muy importante, porque en las situaciones de hay enfermedades con deficiencia del sistema inmunolgico, como
hepatitis persistente no est indicado el tratamiento antiviral, pues se la enfermedad granulomatosa crnica. En relacin con el tratamiento
da un margen a la evolucin espontnea a la curacin. Para estable- con Interfern en la infeccin crnica por el VHB, se han objetivado
cer el diagnstico de hepatitis crnica activa se necesita, como mni- factores de buen pronstico:
mo, demostrar cierto grado de necrosis (por eso es la respuesta correc- 1. El de mayor impacto predictivo es el de niveles bajos de DNA.
ta), que puede llegar a establecer puentes de tejido colgeno en las 2. Niveles entre 100 y 300 de transaminasas predicen una mejor
formas ms avanzadas. La presencia de fibrosis masiva y ndulos de respuesta que cuando oscilan entre 50 y 100.
regeneracin superan el diagnstico de hepatitis crnica: ya se trata 3. La respuesta es superior en mujeres e inferior en VIH positivos.
de una cirrosis heptica.
Pregunta 77.-R: 1
Pregunta 72.-R: 3 Actualmente las indicaciones de trasplante heptico han aumenta-
Se trata de una infeccin por el VHB, que indudablemente est en do y la edad slo es un lmite y no absoluto por encima de los 70
fase replicativa, ya que el DNA, marcador especfico de esta situacin, aos. Son factores de mal pronstico para la recidiva: el genotipo viral
es positivo, por lo que la respuesta 1 es falsa. No hay una situacin de cuando es I, sobre todo si es b. Es muy probable que el hgado se
hepatitis aguda y el anti-Core es de tipo IgG nicamente. La respuesta reinfecte por el VHC, pero la historia natural de esta infeccin, si no
correcta es la nmero 3: se trata de una infeccin por la mutante existe otro factor asociado como el alcohol, suele ser la de una evolu-
precore. Es fcil detectarla en el enunciado de la pregunta, porque es cin lenta a la cirrosis en un perodo de tiempo de alrededor de 20
un paciente con replicacin, DNA positivo, pero con antgeno e ne- aos y, adems, si fuese necesario, se administra tratamiento antiviral.

M exico A rgentina CTO Medicina C/ Nez de Balboa, 115 28006 MADRID (Espaa) Tfno.: (91) 782 43 32 / Fax: (91) 782 43 27
Pg. 10 HM C hile U ruguay E-mail: secretaria@ctomedicina.com; iberocto@ctomedicina.com WEB: www.ctomedicina.com; www.iberocto.com
DIGESTIVO Y CIRUGA GENERAL
Preparacin Examen de Seleccin 05/06 1 Vuelta Seguimiento a distancia

Pregunta 82. Diferencias entre CBP y CEP.

CIRROSIS BIL IAR PRIMARIA COL ANGITIS ESCL EROSANTE PRIMARIA


Epidemiologa M ujer. Va rn.
- Alt. inm unida d hum ora l y celula r. - Desconocida .
Patogenia
- Ac. a ntim itocondria les M 2. - Asocia cin HLA B8.
- I Cola ngitis destructiva .
Anatoma - II Gra nulom a s e infla m a cin periporta l.
- Destruccin y fibrosis de los conductos bilia res intra y extra hep ticos.
Patolgica - III Fibrosis septa l sin gra nulom a s.
- IV Cirrosis.
- Lo m s frecuente ASTENIA Y PRURITO.
- Asintom ticos dura nte m uchos a os.
Clnica - Hepa toesplenom ega lia .
- Luego prurito, a stenia , ictericia , prdida de peso.
- Lesiones por ra sca do.
- Lo m s frecuente Sd. SECO.
Enfermedades - EII sobre todo COLITIS ULCEROSA.
- Escleroderm ia , hipotiroidism o.
asociadas - Fibrosis retroperitonea l o m edia stnica .
- ATR.
- Aum ento de FOSFATASA ALCALINA y resto de - Colesta sis crnica .
L aboratorio enzim a s de colesta sis con tra nsa m ina sa s norm a les. - 1/3 hiperga m m a globulinem ia .
- Aum enta el colesterol e IgM . - 1/2 a um enta IgM , 65 % p-ANCA+.
- Sospecha clnica y la bora torio. - CPRE: DE ELECCIN.
Diagnstico
- Confirm a cin: BIOPSIA. - Estenosis y dila ta cin de los conductos intra y extra hep ticos.
Sintom tico: Sintom tico: el m ism o.
- Colestira m ina .
- Vita m ina s liposolubles.
Tratamiento Especfico: Especfico:
- Ac. ursodesoxiclico. - M etotrexa te.
- Colchicina , m etotrexa te. - Tra spla nte
- Tra spla nte.
- M ejor correla cin con el nivel de - 10 a os de supervivencia tra s dia gnstico.
Pronstico
BILIRRUBINA. - Aum enta la incidencia de COLANGIOCARCINOM A.

Por eso la respuesta correcta es la nmero 1. Pregunta 80.-R: 5


La presencia de crioglobulinas en sangre no establece ni excluye Presenta elevacin predominante de la fosfatasa alcalina, supone-
indicaciones de trasplante. nicamente si se asocia a clnica de crio- mos que de origen heptico, por lo que presenta una colestasis. Como
globulinemia est indicado el tratamiento secuencial con corticoides la ecografa abdominal fue normal, no demostrndose por tanto dila-
y, posteriormente, Interfern y Ribavirina. tacin de la va biliar, es que la colestasis es intraheptica. En este
contexto los anticuerpos antimitocondriales positivos sugieren fuerte-
Pregunta 78.-R: 3 mente el diagnstico de cirrosis biliar primaria. A continuacin estara
La presencia de ANA y ANCA en un enfermo con hipertransamina- indicado una biopsia heptica para confirmarlo.
Comentarios TEST

semia persistente es muy sugestivo de hepatitis autoinmune. Esta situa- El tratamiento es con cido ursodesoxiclico, que mejora el pro-
cin obliga a excluir una infeccin viral y a practicar una biopsia nstico de la enfermedad y apenas tiene efectos adversos por lo que
heptica que demuestre una hepatitis, aunque no hay criterios es muy bien tolerado. Los esteroides no tienen utilidad y, en las fases
histolgicos patognomnicos, por eso la respuesta 3 es la verdadera. avanzadas de la enfermedad, cuando ya se ha desarrollado hiperten-
Se trata de una hepatitis crnica autoinmnune tipo 1, ya que esta sin portal est indicado el trasplante heptico.
presenta habitualmente ANA y/o antimsculo liso positivos. La tipo 2
se caracteriza por presentar estos anticuerpos negativos y ser anti-LKM Pregunta 81.-R: 5
positivo. Los ANCA, variedad perinuclear, pueden ser positivos en En la valoracin pronstica de la cirrosis biliar primaria es funda-
ambos tipos de hepatitis autoinmune. El tratamiento es con corticoi- mental conocer el nivel de bilirrubina, porque cuanto mayor sea sta,
des y casi todos requieren un largo mantenimiento con azatioprina es decir ms colestasis, se presume mayor territorio biliar afecto. Los
una vez conseguida la remisin. El tacrolimus empleado en el tras- anticuerpos antimitocondriales son tiles, sobre todo, porque sugie-
plante heptico se est utilizando en algunos casos de hepatitis auto- ren el diagnstico. Igual sucede con el aumento policlonal de IgM.
inmune a nivel experimental. Las hormonas tiroideas se solicitan porque es muy frecuente la
asociacin a tiroiditis y, sobre todo, el desarrollo de hipotiroidismo en
Pregunta 79.-R: 1 estos enfermos. Los niveles de colesterol pueden ser tiles, puesto que
El tratamiento de eleccin de un hepatocarcinoma es la extirpacin estn elevados en los enfermos con colestasis, no as los niveles de
quirrgica siempre que se pueda. Peores resultados ofrece el trasplante triglicridos, por eso es la respuesta que hay que sealar.
heptico, pero pasa a ser de eleccin como en este caso si el paciente
tiene un grado funcional heptico avanzado, como es el grado C de Pregunta 82.-R: 3
Child, ya que en esta situacin no tolerara la ciruga, pues le dejara en Esta enfermedad es infrecuente. Casi siempre suele ser ANCA pa-
una situacin de insuficiencia heptica grave. En las situaciones de trn perinuclear positivo. Con mucha frecuencia existe el anteceden-
lesiones grandes y mltiples no se puede realizar ciruga y se intenta te previo de colitis ulcerosa o se asocia a ella a lo largo de su evolu-
como tratamiento paliativo para mejorar la supervivencia la cin. Esta asociacin es independiente y por lo tanto el desarrollo de
alcoholizacin percutnea. Cuando hay metstasis o trombosis de la la colitis ulcerosa no influye en el desarrollo y complicaciones de la
porta extraheptica nicamente se utiliza el tratamiento sintomtico. colangitis esclerosante. Es ms frecuente en varones, se asocia al HLA-

M exico A rgentina CTO Medicina C/ Nez de Balboa, 115 28006 MADRID (Espaa) Tfno.: (91) 782 43 32 / Fax: (91) 782 43 27
C hile U ruguay E-mail: secretaria@ctomedicina.com; iberocto@ctomedicina.com WEB: www.ctomedicina.com; www.iberocto.com
HM Pg. 11
DIGESTIVO Y CIRUGA GENERAL
Seguimiento a distancia Preparacin Examen de Seleccin 05/06 1 Vuelta

Pregunta 91. Actitud diagnstica y teraputica en la pancreatitis aguda. Comentarios TEST


B8 y su diagnstico se establece mostrando en la colangiografa retr- alterados, el siguiente paso es evaluar las mutaciones del gen HFE y, si
grada endoscpica un patrn de estenosis irregulares, con dilatacio- se encuentra homozigoto o heterozigoto se establece el diagnstico
nes postestenticas por todo el rbol biliar (ver tabla en pgina si- de hemocromatosis sin necesidad de una biopsia heptica y se indi-
guiente). Adems la biopsia heptica, aunque no ofrece el diagnstico can a continuacin las flebotomas peridicas.
es de inters, pues permite conocer si se ha establecido ya una cirrosis La biopsia heptica es til para el diagnstico de aquellos casos
biliar secundaria, con lo que tiene de valor pronstico este dato. con sospecha clnica o bioqumica y estudio gentico negativo. Ade-
ms tambin se realiza en los casos con elevacin persistente de las
Pregunta 83.-R: 2 transaminasas para evaluar si hay fibrosis o cirrosis heptica.
En el screening de los familiares de un paciente con hemocroma-
tosis se valora el ndice de saturacin de transferrina, que se considera Pregunta 84.-R: 1
patolgico cuando es superior al 45% y la ferritina. El primero de ellos El gen anmalo est en el cromosoma 6. La herencia es autosmi-
es el ms precoz en alterarse. Si estos marcadores bioqumicos estn ca recesiva.

M exico A rgentina CTO Medicina C/ Nez de Balboa, 115 28006 MADRID (Espaa) Tfno.: (91) 782 43 32 / Fax: (91) 782 43 27
Pg. 12 HM C hile U ruguay E-mail: secretaria@ctomedicina.com; iberocto@ctomedicina.com WEB: www.ctomedicina.com; www.iberocto.com
DIGESTIVO Y CIRUGA GENERAL
Preparacin Examen de Seleccin 05/06 1 Vuelta Seguimiento a distancia
Entre el cuadro clnico destacan: cirrosis y sus complicaciones; Pregunta 90.-R: 4
miocardiopata; pigmentacin bronceada de la piel; diabetes melli- El diagnstico de pancreatitis crnica se establece, si el paciente
tus que una vez establecida no mejora con las flebotomas; insuficien- presenta sntomas, solicitando en primer lugar una Rx simple de ab-
cia exocrina del pncreas; artropata generalmente con condrocalci- domen para evaluar la presencia de calcificaciones. Si stas se de-
nosis que no mejora con el tratamiento e hipogonadismo, que puede muestran el diagnstico queda establecido. Si no se observan, enton-
ser el cuadro de presentacin en jvenes por eso es la respuesta co- ces se solicita una TAC, que es la prueba ms sensible para observar las
rrecta y que se debe no a una lesin genital sino hipofisaria. calcificaciones pancreticas. Si el TAC no es diagnstico, entonces se
recurre a la CPRE, que es la prueba ms sensible y especfica, la que
Pregunta 85.-R: 2 detecta los casos ms precoces, pero que no se utiliza de entrada
La macroesteatosis heptica es una situacin en la que predomina porque es cruenta y tiene efectos secundarios. El test de la secretina
el acmulo de triglicridos en el interior del hepatocito, con lo que diagnostica insuficiencia pancretica exocrina, pero es de escasa uti-
stos adquieren un aspecto hinchado y el hgado ecogrficamente lidad para el diagnstico de pancreatitis crnica.
presenta una ecogenicidad brillante. Suele cursar con un ligero
aumento de GOT o sin ninguna alteracin. Suele ser reversible, aun- Pregunta 91.-R: 5
que recientemente se estn describiendo algunos casos de cirrosis Entre los indicadores de gravedad el ms precoz es la elevacin de
idioptica atribuibles a la evolucin de esta entidad. los niveles sricos de la elastasa de los neutrfilos. Menos inters tiene
Las causas ms frecuentes son la obesidad, diabetes mellitus tipo 2, determinar la PCR (protena C reactiva) o el pptido activador del
el alcoholismo y el uso de corticoides. No debemos confundir esta tripsingeno urinario. Entre los indicadores radiolgicos de gravedad
entidad con la esteatohepatitis microvesicular, que tambin se puede destacan los que se obtienen con un TAC con contraste, ya que ms
producir en el alcohol y que se observa en el sndrome de Reye o en de un 50% de tejido necrtico confiere mal pronstico.
el consumo de cido valproico. Este cuadro es ms grave y cursa Clsicamente se han utilizado los criterios de Ransom (ver esquema
como una hepatitis aguda severa. en pgina siguiente), entre los que no est la determinacin de PCR .

Pregunta 86.-R: 4 Pregunta 92.-R: 1


Los clculos de bilirrubina se producen porque pasa a la va biliar Una pancreatitis aguda leve o moderada requiere como tratamiento:
una cantidad de bilirrubina no conjugada superior a la tolerable. Esto 1. Analgesia, excepto mrficos porque contraen el esfnter de Oddi.
sucede cuando se saturan sus mecanismos por: 2. Fluidoterapia i.v.
1) Hiperaporte, como ocurre en los estados hemolticos crnicos, 3. Dieta absoluta a slidos y lquidos, al menos 3 das. Es muy impor-
respuesta correcta, o en el sndrome de Gilbert. Los clculos son tante para evitar complicaciones diferir el inicio de la alimentacin
negros. oral al menos hasta que el paciente est asintomtico, por eso la
2) Lesiones de la va biliar, como en las colangitis crnicas o en las respuesta correcta es la nmero 1. Slo est indicado colocar una
cirrosis, siendo en esta ocasin clculos de bilirrubina marrones. sonda nasogstrica de aspiracin si hay vmitos secundarios a un
leo paraltico asociado.
Los clculos de colesterol se producen porque se reduce la absor- No est indicado administrar antibiticos.
cin de sales biliares, como sucede en la enfermedad de Crohn, o
aumenta la excrecin heptica de colesterol, o hay una situacin de Pregunta 93.-R: 1
hipomotilidad biliar como sucede en la obesidad y en la gestacin. En el manejo de una pancreatitis aguda grave es muy importante
evaluar mediante una puncin si presenta infeccin del tejido necr-
Pregunta 87.-R: 3 tico, porque en ese caso est indicada la necrosectoma quirrgica. Si
La bilis se vuelve ms litognica despus de resecar el leon o por no se demuestra infeccin, nicamente se toman medidas de soporte,
una enfermedad ileal extensa porque no se reabsorben las sales bilia- antibiticos i.v. (preferentemente que perfundan bien en tejido pan-
res y como consecuencia aumenta la proporcin en la va biliar del cretico, como imipenem o las quinolonas). La somatostatina no ha
componente de colesterol sobre el de cidos biliares. Esto provoca demostrado su utilidad en el manejo de estos enfermos, que, adems,
que el colesterol cristalice. En una bilis con cristales de colesterol, ante requieren nutricin parenteral o, si fuera posible, nutricin enteral
cualquier situacin de estasis en la vescula, se formarn clculos por yeyunal, porque esta ltima parece reducir las tasas de infeccin de la
aposicin de calcio sobre los cristales de colesterol. Por eso en la necrosis pancretica.
Comentarios TEST

enfermedad de Crohn los clculos son de colesterol.


Pregunta 94-R: 1
Pregunta 88.-R: 2 Se trata de una pancreatitis aguda de origen biliar que cumple
El diagnstico de pancreatitis aguda se establece en un enfermo criterios de gravedad. En este caso, adems del manejo general de una
con dolor abdominal agudo por la elevacin de la amilasa o la lipasa pancreatitis aguda grave, se ha comprobado que la papilotoma
por encima de tres veces el lmite superior de la normalidad. En oca- endoscpica realizada en las primeras 72 h. mejora el pronstico de
siones se sospecha el cuadro, pero las enzimas no son diagnsticas y, la enfermedad, por eso es la respuesta correcta. En este caso la colecis-
entonces, se requiere la realizacin de un TAC, que observando las tectoma estara indicada, pero en un segundo paso cuando se resuel-
alteraciones radiolgicas inflamatorias del pncreas, permite estable- va la pancreatitis aguda.
cer el diagnstico de pancreatitis aguda.
El inters de la ecografa en el manejo de la pancreatitis aguda se Pregunta 95.-R: 1
dirige ms a evaluar su etiologa. Si se demuestra litiasis o microlitiasis La causa ms frecuente de pancreatitis crnica en nuestro medio
biliar se establece en la ecografa que el diagnstico etiolgico de la es el alcoholismo crnico, oscilando alrededor del 70% de los casos.
pancreatitis aguda es biliar. Si fuese necesario evaluar las complicacio- Generalmente se inicia manifestndose como episodios de pancrea-
nes de una pancreatitis o su pronstico, el TAC ofrece ms informa- titis aguda recurrente y, finalmente, como dolor crnico. Otras causas
cin que la ecografa. como hipertrigliceridemia, hipercalcemia, fibrosis qustica o secun-
daria a obstruccin de los conductos pancreticos son menos fre-
Pregunta 89.-R: 2 cuentes.
La litiasis biliar es la causa ms frecuente de pancreatitis aguda,
representando alrededor del 60%. El alcoholismo es la segunda causa Pregunta 96.-R: 2
oscilando entre el 30 y 40% de los casos dependiendo de las reas Adems del dolor puede aparecer diabetes mellitus en los estados
geogrficas. En tercer lugar con menor frecuencia est involucrada la finales de la enfermedad. Esta diabetes tiene parecidas complicacio-
microlitiasis y por ltimo las causas idiopticas u otras inhabituales nes a las de la idioptica, excepto:
entre las que se incluyen: el esfnter de Oddi hipertensivo, la fibrosis 1. Menor tendencia a la cetoacidosis
qustica y frmacos como pentamidina, DDI o azatioprina. 2. Como tiene poco glucagn, ms hipoglucemias.

M exico A rgentina CTO Medicina C/ Nez de Balboa, 115 28006 MADRID (Espaa) Tfno.: (91) 782 43 32 / Fax: (91) 782 43 27
C hile U ruguay E-mail: secretaria@ctomedicina.com; iberocto@ctomedicina.com WEB: www.ctomedicina.com; www.iberocto.com
HM Pg. 13
DIGESTIVO Y CIRUGA GENERAL
Seguimiento a distancia Preparacin Examen de Seleccin 05/06 1 Vuelta
Adems se desarrolla maldigestin, con esteatorrea, prdida de
peso y malabsorcin de las vitaminas liposolubles, pero tambin dfi-
cit de B12, ya que la lipasa es necesaria para la absorcin de esta
vitamina.
En ocasiones cursa con masas pancreticas que al obstruir la va
biliar causan ictericia.
Una complicacin infrecuente es la fuga de lquido de un pseudo-
quiste o del propio Wirsung al interior del peritoneo, provocando el
cuadro de ascitis pancretica. Se diagnostica por una amilasa muy
elevada en lquido asctico.

Pregunta 97.-R: 3
En una CPRE la presencia de una estenosis nica en un enfermo
con dolor sugiere mucho ms un carcinoma de pncreas que pan-
creatitis crnica. Las pancreatitis crnicas de cualquier tipo suelen
tener irregularidades mltiples, con estenosis y reas de dilatacin
postestenticas.

Pregunta 98.-R: 4
Una vez conocido que un paciente tiene insuficiencia pancreti-
ca exocrina, bien mediante el test ms fiable, como el de la secretina,
o con los de menor rentabilidad como el de pancreolauryl o elastasa
fecal, queda por determinar si hay que administrar enzimas pan-
creticas y, para ello, es suficiente con conocer el valor de las grasas
fecales (respuesta correcta), porque este es el marcador que estable-
ce la indicacin del tratamiento sustitutivo. De igual forma la valora-
cin del beneficio teraputico se establece: con los cambios en la
clnica y la cuantificacin de las grasas fecales que demuestre des-
aparicin o reduccin de la esteatorrea.

Pregunta 99.-R: 1
Para el manejo de la ascitis pancretica es fundamental detectar el
punto de fuga del jugo pancretico entre los conductos pancreticos
y el peritoneo. Para ello, es de eleccin la CPRE. Aunque la TAC
pueda mostrar un pseudoquiste es ms difcil que detecte el punto de
fuga, algo crucial para establecer las indicaciones quirrgicas.

Pregunta 100.-R: 4
El manejo del dolor en la pancreatitis crnica se realiza de forma
progresiva:
1. Analgesia convencional: paracetamol, metamizol, AINES.
2. Suplementos con enzimas pancreticos en ocasiones alivian el dolor.
3. Analgsicos de tercer nivel, excepto agonistas mrficos puros.
4. Dilataciones o colocacin de prtesis por CPRE si hay obstruccio-
nes del Wirsung responsables del dolor.
5. Ciruga derivativa o incluso pancreatectomas parciales.

Comentarios TEST
Los antagonistas del TNF no tiene utilidad en la pancreatitis crni-
ca, recordemos que para el examen MIR pueden estar indicados en
las fstulas refractarias de la enfermedad de Crohn y en casos de artritis
reumatoide refractarios al tratamiento convencional.

M exico A rgentina CTO Medicina C/ Nez de Balboa, 115 28006 MADRID (Espaa) Tfno.: (91) 782 43 32 / Fax: (91) 782 43 27
Pg. 14 HM C hile U ruguay E-mail: secretaria@ctomedicina.com; iberocto@ctomedicina.com WEB: www.ctomedicina.com; www.iberocto.com
CIRUGA GENERAL
Preparacin Examen de Seleccin 05/06 1 Vuelta Seguimiento a distancia

GENERALIDADES. 1) El diagnstico de la diverticulitis ha de ser fundamental-


mente clnico, siendo la TC la mejor prueba de imagen.
1. El procedimiento consistente en colocar un tubo en el est- 2) En los pacientes con diverticulitis, la intervencin quirr-
mago que salga a la piel, con la intencin de utilizarlo para gica se debe realizar en aquellos enfermos que no respon-
alimentar a un paciente, se llama: dan al tratamiento mdico.
3) La mayora de los enfermos con diverticulosis son sintom-
1) Gastrotoma. ticos y precisan intervencin.
2) Gastrostoma. 4) La intervencin de Hartmann es la operacin ms comn
3) Gastrectoma. para el tratamiento de la diverticulitis con peritonitis
4) Tubulizacin gstrica.
generalizada.
5) Gastroplastia.
5) El signo fsico ms comn en la diverticulitis es la hipersen-
sibilidad a la palpacin en fosa ilaca izquierda.
2. La fiebre que se produce tras la fuga de una anastomosis
gastrointestinal aparece con mayor frecuencia:
7. Seale la opcin FALSA sobre la fstula colovesical:
1) A partir del 12 da postoperatorio.
2) Nunca. 1) Es ms frecuente en el hombre.
3) Alrededor del primer mes. 2) La colonoscopia es el mejor mtodo diagnstico.
4) A las 48-72 horas de la intervencin. 3) Es el tipo ms frecuente de fstula de la enfermedad
5) En la segunda semana del postoperatorio. diverticular.
4) Es caracterstica la fecaluria y la neumaturia.
3. En relacin a las complicaciones de la ciruga gastrointestinal, 5) No es obsttrica.
seale la respuesta INCORRECTA:
8. Paciente varn de 29 aos, diagnosticado hace dos aos de
1) La sepsis es la causa ms comn de muerte en pacientes colitis ulcerosa; acude a Urgencias por presentar fiebre,
con fstulas gastrointestinales. diarrea sanguinolenta, y dolor y distensin abdominales, con
2) En las fstulas gastrointestinales est indicada la ciruga tras signos de hipersensibilidad generalizada. Qu exploracin
6-8 semanas de tratamiento conservador (antibiticos y nos acercar al diagnstico?:
reposo digestivo con NPT).
3) Las anastomosis colorrectales estn ms predispuestas a 1) Enema opaco.
la disrupcin que las del estmago e intestino delgado. 2) Rx simple de abdomen.
4) Suele ser habitual la existencia de un leo paraltico, de
3) Colonoscopia.
duracin de alrededor de una semana, tras la ciruga.
4) TC.
5) La hernia paraestomal es la complicacin ms frecuente,
5) Ecografa abdominal.
que requiere intervencin quirrgica tras colocar una
colostoma o ileostoma.
9. El binomio diagnstico - teraputico en este paciente ser:
4. El tratamiento de las heridas quirrgicas infectadas que dan
fiebre en las primeras 24 horas suele ser: 1) Brote agudo de colitis ulcerosa - salazopirina.
2) Cncer de colon sobreinfectado - ciruga.
1) Innecesario. 3) Sndrome de Ogilvie - aspiracin gstrica.
2) Ciruga urgente. 4) Megacolon txico - corticoterapia y antibioterapia.
3) Antispticos locales. 5) Diverticulitis perforada - intervencin de Hartmann.
4) Antibioterapia tpica.
5) Sintomtico. 10. Pasadas 24 h, el enfermo no evoluciona favorablemente;
ahora la actitud ser:
Preguntas TEST

5. Una de las siguientes afirmaciones en relacin a las quema-


duras NO es correcta: 1) Asociar antibiticos al tratamiento.
2) Descompresin con enemas.
1) Los quemados drmicos superficiales superiores al 50% 3) Colectoma total + ileostoma.
son crticos. 4) Continuar observacin 48 h.
2) La traccin del folculo piloso diferencia las quemaduras 5) Intervencin de Hartmann.
de 1 y 2 grado.
3) La profundidad de la quemadura aumenta con la edad.
11. Cul de entre las siguientes opciones es FALSA con respecto
4) No est indicada de forma sistemtica la profilaxis antibi-
al carcinoma de colon asociado a la enfermedad inflamatoria
tica en los quemados.
intestinal?:
5) En la fase tarda del sndrome de inhalacin la mortalidad
se debe a neumona.
1) El riesgo es mayor en pacientes con colitis ulcerosa.
2) El riesgo aumenta con el tiempo de evolucin.
INTESTINO. 3) Tiene mejor pronstico que en pacientes sin enfermedad
inflamatoria intestinal.
6. Respecto de la enfermedad diverticular del intestino grueso, 4) Con mayor frecuencia son mltiples.
seale la respuesta INCORRECTA: 5) Tienen mayor tendencia a surgir en colon ascendente.

M exico A rgentina CTO Medicina C/ Nez de Balboa, 115 28006 MADRID (Espaa) Tfno.: (91) 782 43 32 / Fax: (91) 782 43 27
C hile U ruguay E-mail: secretaria@ctomedicina.com; iberocto@ctomedicina.com WEB: www.ctomedicina.com; www.iberocto.com
CG Pg. 1
CIRUGA GENERAL
Seguimiento a distancia Preparacin Examen de Seleccin 05/06 1 Vuelta
12. El antibitico que suele utilizarse habitualmente para el biopsia, y sta es informada intraoperatoriamente como
manejo de la afectacin perianal en la enfermedad de Crohn, metstasis de un adenocarcinoma de colon. Ante ello, la
pero que tiene el inconveniente de poder producir neuropa- actitud quirrgica ser:
ta en tratamientos prolongados, es:
1) Se decide que es un caso inoperable.
1) Cefotaxima i.v. 2) Reseccin segmentaria del tumor.
2) Mupirocina tpica. 3) Colostoma de descarga proximal.
3) Amoxicilina + clavulnico oral. 4) Reseccin reglada del colon y exresis completa de la
4) Gentamicina i.v. metstasis.
5) Metronidazol oral. 5) Reseccin reglada del colon y hepatectoma derecha
reglada.
13. Paciente mujer, que acude a Urgencias a sus 66 aos, por
deposiciones sanguinolentas desde hace diez das, sin dolor. 18. Un paciente varn, de 60 aos, con astenia de meses de
Refiere notarse ms estreida de lo que era previamente, con evolucin, acude a Urgencias por cuadro de ausencia de
ocasionales deposiciones lquidas. Se nota cansada y con deposicin desde hace 7 das, mal estado general, distensin
menos apetito, y dice que ha perdido tres Kg en dos meses. abdominal y dolor clico infraumbilical. Su actitud inicial NO
La primera prueba que realizara usted sera: incluir:

1) Tacto rectal. 1) Estudio radiolgico.


2) Tacto vaginal. 2) Estudio analtico.
3) Sistemtica de sangre. 3) Reposicin hidroelectroltica.
4) Endoscopia. 4) Historia detallada de antecedentes.
5) Enema opaco. 5) Ciruga urgente.

14. Es diagnosticada por biopsia de adenocarcinoma de recto, a 19. Si entre los antecedentes del paciente slo consigue reflejar
10 cm del margen anal. El estudio de extensin lo hace usted debilidad de 6 meses de evolucin y una anemia microctica
con: como hallazgo analtico, su primera sospecha diagnstica ser:

1) TC abdominal. 1) Cncer obstructivo de antro gstrico.


2) RM. 2) Cncer obstructivo de colon.
3) Gammagrafa sea. 3) Vlvulo de sigma.
4) Rx simple de abdomen. 4) Obstruccin adherencial.
5) TC torcico. 5) leo biliar.

15. Es operada por reseccin anterior baja. La anatoma patol- 20. Es diagnosticado por TC de cncer de colon derecho obstruc-
gica muestra tumor de 4 cm, que alcanza capa muscular, tivo. Ante esta situacin clnica, el tratamiento de eleccin ser:
invadindola totalmente, sin ganglios infiltrados. El trata-
miento coadyuvante ser: 1) Ileostoma proximal ms reseccin.
2) Hartmann.
1) Slo radioterapia. 3) Hemicolectoma derecha con anastomosis ileotransversa.
2) QT con ciclofosfamida y radioterapia opcional. 4) Colostoma de descarga proximal.
3) QT con 5-FU y radioterapia opcional. 5) Colectoma total con anastomosis ileoanal.
4) Ampliar mrgenes con amputacin abdominoperineal
(Miles). 21. Mujer que historiamos en la consulta de ciruga por rectorra-
5) Braquiterapia. gias repetidas y abundantes, sin repercusin hemodinmica

Preguntas TEST
actual. La exploracin abdominal resulta anodina. Entre las
16. Respecto del cncer de colon, es FALSO que: peticiones del estudio programado, NO incluira:

1) La reseccin de metstasis hepticas aisladas puede ofre- 1) Arteriografa de AMS.


cer un aumento de la supervivencia. 2) Enema opaco.
2) Las lesiones del colon derecho requieren hemicolecto- 3) Colonoscopia.
ma derecha. 4) TC.
3) En lesiones obstructivas del lado derecho se realiza resec- 5) Analtica rutinaria.
cin y anastomosis primaria.
4) En el estadio B2 de la clasificacin de Astler y Coller, estn 22. En el diagnstico diferencial del cuadro NO estamos inclu-
afectos los ganglios linfticos. yendo:
5) En el estudio del cncer de recto distal debe realizarse
radiografa de trax. 1) Cncer de colon.
2) Angiodisplasia de colon.
17. Paciente diagnosticado de adenocarcinoma de colon. En la 3) Hemorroides.
ciruga se descubre una lesin de 2 cm de dimetro en la 4) Isquemia mesentrica.
superficie anterior del lbulo heptico derecho. Se toma una 5) Diverticulosis de sigma.

M exico A rgentina CTO Medicina C/ Nez de Balboa, 115 28006 MADRID (Espaa) Tfno.: (91) 782 43 32 / Fax: (91) 782 43 27
Pg. 2 CG C hile U ruguay E-mail: secretaria@ctomedicina.com; iberocto@ctomedicina.com WEB: www.ctomedicina.com; www.iberocto.com
CIRUGA GENERAL
Preparacin Examen de Seleccin 05/06 1 Vuelta Seguimiento a distancia

23. Durante el estudio, la paciente acude a Urgencias con dolor 4) lceras por hipersecrecin en los quemados.
en FII, fiebre, defensa abdominal e irritacin peritoneal. 5) lceras tpicas de pacientes psiquitricos.
Solicitaremos:

1) Analtica y colonoscopia. HEPATOBILIOPNCREAS.


2) Colonoscopia y TC.
3) Enema opaco y analtica. 29. Paciente mujer, de 52 aos, que acude a Urgencias por dolor
4) Analtica, radiografa de trax y abdomen. en epigastrio e hipocondrio derecho, que aumenta y dismi-
5) Arteriografa y TC. nuye, pero no desaparece, desde hace 2 horas, acompaado
de nuseas y vmitos. El dolor se irradia hacia la espalda. La
paciente est afebril, la exploracin fsica es normal y la
ESTMAGO. analtica con amilasa y bilirrubina no presenta alteraciones.
Qu esperara encontrar si hace una ecografa?:
24. Respecto a las complicaciones de la lcera pptica, seale la
respuesta correcta: 1) Dilatacin de va biliar extraheptica.
2) Dilatacin de va biliar extra e intraheptica.
1) La perforacin afecta a la cara posterior duodenal, produ- 3) Tumor en cabeza de pncreas.
ciendo un aumento del nivel de amilasa srica. 4) Colelitiasis simplemente.
2) La hemorragia de la lcera gstrica es ms frecuente en 5) Coledocolitiasis.
cara posterior, afectando a la arteria gastroduodenal.
3) El tratamiento definitivo de la estenosis es quirrgico, tras 30. Qu sospecha diagnstica tiene Vd?:
restablecer la correcta nutricin del paciente.
4) Es tpico de la perforacin los vmitos, acompaados de 1) Clico biliar.
"abdomen en tabla". 2) Colecistitis aguda.
5) El tratamiento de entrada de la HDA es quirrgico. 3) leo biliar.
4) UGD.
25. Paciente varn, de 40 aos, que presenta un cuadro sbito 5) Pancreatitis.
de dolor "en pualada" en epigastrio, con sudoracin y
taquicardia. La Rx de trax en bipedestacin confirmar: 31. El tratamiento de la paciente referida ser:

1) Neumotrax. 1) Ciruga urgente.


2) lcera antral sangrante. 2) CPRE + esfinterotoma.
3) Diseccin artica. 3) Antibiticos.
4) Perforacin de lcera duodenal. 4) Espasmolticos y analgsicos.
5) Pancreatitis aguda. 5) Dieta absoluta + SNG + repetir analtica en 4 horas.

26. El tratamiento que considera de eleccin sera: 32. Si la paciente anterior fuera sometida a colecistectoma por
clicos repetidos, digestiones pesadas y sensacin de disten-
1) Sutura de la lcera + vagotoma troncular + piloroplastia. sin abdominal con meteorismo postprandial, cul sera el
2) Antrectoma + anastomosis tipo Billroth I. probable diagnstico anatomopatolgico?:
3) Gastrectoma 2/3 + vagotoma + Y de Roux.
4) Vagotoma supraselectiva. 1) Vescula sin alteraciones.
5) Sutura de la lcera + epiploplastia. 2) Simplemente colelitiasis.
3) Clico biliar.
27. Un paciente es diagnosticado por endoscopia de carcinoma 4) Colecistitis crnica + colelitiasis.
gstrico antral de tipo enteroide. Durante la laparotoma
Preguntas TEST

5) Adenomiosis de vescula.
exploradora se objetiva un tumor que afecta al antro y
cuerpo, extenso, con algunos ganglios positivos infrapilri-
33. Paciente varn, de 83 aos, con doble lesin valvular y
cos y en curvadura menor. El tratamiento ser:
fibrilacin auricular crnica, en tratamiento con anticoagu-
lantes orales, que acude a Urgencias por un cuadro de dolor
1) Gastrectoma subtotal paliativa.
en epigastrio e hipocondrio derecho, irradiado a la espalda,
2) Gastrectoma total + linfadenectoma + omentectoma
de 7 horas de evolucin. Ha presentado nuseas y vmitos,
asociada.
sin otras alteraciones del trnsito. Temperatura: 37,9C. TA:
3) Gastroenterostoma + pancreatectoma parcial.
140/90 mmHg. Coloracin normal de piel y mucosas. A la
4) Antrectoma + linfadenectoma.
palpacin, el enfermo se queja y defiende cuando se toca en
5) Gastrectoma subtotal + linfadenectoma.
hipocondrio derecho. En la analtica de sangre hay 15.700
leucocitos (83% neutrfilos, 4% cayados), amilasa: 90 U/l,
28. Qu son las lceras de Cushing?:
bilirrubina total: 1,1 mg/dl, y resto sin alteraciones significa-
tivas. Cul sera la secuencia ms correcta de diagnstico y
1) Las UGD tpicas de politraumatizados con afectacin del
tratamiento?:
SNC (por hipersecrecin).
2) lceras por isquemia crnica del tronco celaco.
1) Gammagrafa con HIDA / analgesia + espasmolticos.
3) lceras por isquemia aguda en quemados.
2) CPRE / analgesia + espasmolticos.

M exico A rgentina CTO Medicina C/ Nez de Balboa, 115 28006 MADRID (Espaa) Tfno.: (91) 782 43 32 / Fax: (91) 782 43 27
C hile U ruguay E-mail: secretaria@ctomedicina.com; iberocto@ctomedicina.com WEB: www.ctomedicina.com; www.iberocto.com
CG Pg. 3
CIRUGA GENERAL
Seguimiento a distancia Preparacin Examen de Seleccin 05/06 1 Vuelta
3) Ecografa abdominal / ciruga urgente. 38. Cualquiera que fuera su actitud, el paciente acude a revisin
4) Ecografa abdominal / analgesia + antibiticos. a las 6 semanas, para control. Tras repetir la prueba, que fue
5) TC abdominal / ciruga urgente. diagnstica, se decide continuar observacin. El hallazgo
ms probable habr sido:
34. Respecto de las enfermedades del sistema biliar, seale la
respuesta correcta: 1) Pseudoquiste pancretico no complicado, de 5 cm de
dimetro.
1) La coledocolitiasis, tras colecistectoma previa, se trata con 2) Colangitis esclerosante, que no ha respondido a corticoides.
esfinterotoma endoscpica. 3) Pancreatitis crnica con dilatacin del Wirsung.
2) La incidencia de la colecistitis calculosa aguda es ms 4) Clculo en leon terminal, no resuelto por litotoma en-
frecuente en varones. doscpica.
3) La colecistitis enfisematosa es ms frecuente en pacientes 5) Recidiva del cncer de pncreas.
con hipertensin.
4) Est indicada la colecistectoma en pacientes asintomti- 39. El paciente anterior acude a Urgencias por presentar dolor
cos. abdominal en mesogastrio, acompaado de fiebre en picos
5) El carcinoma de clulas escamosas es el tipo ms frecuente >38,5C con escalofros, objetivndose en la analtica de urgen-
de cncer de vescula. cias leucocitosis de 22.000 con neutrofilia y desviacin izquier-
da, sin foco infeccioso aparente. Cul sera su actitud ahora?:
35. Cul de las siguientes es indicacin de ciruga urgente en un
paciente de 58 aos, con antecedentes de herniorrafia, 1) Pancreaticoyeyunostoma latero-lateral (operacin de
colelitiasis e HTA, ingresado por dolor abdominal en meso- Puestow).
gastrio, sin signos de irritacin peritoneal, que no cambia con 2) Observacin.
posturas, coloracin azulada en los flancos, 12.000 leucoci- 3) Quistoyeyunostoma urgente.
tos/mm3, hipocalcemia, amilasa de 3.940 U/l, bilirrubina 4) Puncin percutnea bajo control radiolgico.
total: 1,9 mg/dl y derrame pleural derecho?: 5) Intervencin de Whipple.

1) No control del dolor con AINEs en dosis plenas. 40. Los motivos que nos empujan a operar los adenomas hep-
2) Dilatacin de asas intestinales en mesogastrio en la Rx de ticos son sus potenciales complicaciones, a saber:
abdomen y distensin abdominal.
3) Aparicin de dolor, que aumenta con la palpacin y 1) Malignizacin y colestasis.
vmitos repetidos, con aumento de la bilirrubina y apa- 2) Colestasis y aparicin de "lesiones satlite".
ricin de ictericia. 3) Colestasis por compresin y hemorragia.
4) Imagen de borramiento de la grasa en rea pancretica en 4) Hemorragia y malignizacin.
el TC, con posibles zonas de necrosis. 5) Malignizacin y sndromes paraneoplsicos.
5) Existencia de necrosis peripancretica en el TC, con
cultivo positivo tras puncin. 41. Cul de las siguientes afirmaciones es cierta con respecto al
hemangioma heptico?:
36. Paciente varn, de 50 aos, bebedor habitual, tratado hace
tres semanas por un cuadro de dolor epigstrico irradiado en 1) Se relaciona con el consumo de esteroides.
cinturn (no aporta informes), que requiri ingreso. Acude 2) Se diagnostica por RM o ecografa.
ahora con dolor persistente en hemiabdomen superior, 3) Suelen ocasionar ictericia por compresin biliar.
sensacin de saciedad temprana, nuseas y vmitos; se 4) Pueden malignizarse.
evidencia masa palpable en hipocondrio derecho. La anal- 5) Su tratamiento es quirrgico por el peligro de complica-
tica es inespecfica. La exploracin que ms datos nos apor- ciones.
tar sobre la patologa que refiere el enfermo ser:

Preguntas TEST
42. Sobre los tumores hepticos, seale la opcin FALSA:
1) Ecografa abdominal.
2) CPRE. 1) El tratamiento curativo del hepatocarcinoma es la ciruga.
3) TC abdominal. 2) En los hepatocarcinomas irresecables sin enfermedad
4) Rx simple abdomen. extraheptica est indicado el trasplante heptico.
5) Gastroscopia. 3) El cncer de coldoco se denomina tumor de Klatskin.
4) El angiosarcoma heptico se relaciona con exposicin a
37. La informacin de dicha prueba es concluyente, demostran- cloruro de vinilo.
do un pseudoquiste pancretico de aproximadamente 7 cm 5) El carcinoma fibrolamelar es ms frecuente en adultos
de dimetro. Su actitud ser?: jvenes y adolescentes.

1) Reseccin pancretica, incluyendo el pseudoquiste. 43. Paciente varn de 70 aos, fumador y diabtico. Deprimido
2) Observacin y seguimiento mediante la realizacin de desde hace 2 meses. Estudiado en la consulta de digestivo por
ecografas peridicas. prdida de peso y dolor epigstrico opresivo que se irradia
3) Puncin percutnea del pseudoquiste. a la espalda. El enfermo est ictrico y la exploracin abdo-
4) Intervencin de Whipple. minal es anodina. Qu prueba diagnstica NO nos ayudar
5) Cistogastrostoma quirrgica. en este caso?:

M exico A rgentina CTO Medicina C/ Nez de Balboa, 115 28006 MADRID (Espaa) Tfno.: (91) 782 43 32 / Fax: (91) 782 43 27
Pg. 4 CG C hile U ruguay E-mail: secretaria@ctomedicina.com; iberocto@ctomedicina.com WEB: www.ctomedicina.com; www.iberocto.com
CIRUGA GENERAL
Preparacin Examen de Seleccin 05/06 1 Vuelta Seguimiento a distancia

1) TC. hemiabdomen inferior. Ahora refiere dolor intenso, constante


2) CPRE. y generalizado; ha presentado dos episodios de rectorragia. En
3) Ecografa abdominal. la exploracin fsica, impresiona de gravedad, con distensin
4) Estudio gastroduodenal. abdominal, signos de peritonismo y ausencia de ruidos hi-
5) Gammagrafa con Tc99. droareos. Rx de abdomen: leo paraltico con distensin de
intestino grueso hasta el ngulo esplnico. La actitud ser:
44. Antes de obtener los resultados, y en orden de frecuencia,
esperamos encontrar: 1) Realizar intervencin quirrgica urgente.
2) Realizar enema opaco.
1) Cncer de cabeza de pncreas - adenocarcinoma ductal. 3) Realizar arteriografa de AMI.
2) Cncer de cola de pncreas - adenocarcinoma lobulillar. 4) Realizar colonoscopia.
3) Cncer de cabeza de pncreas - adenocarcinoma lobu- 5) Esperar evolucin y repetir estudios radiolgicos.
lillar.
4) Colangiocarcinoma distal. 50. Al final, la enferma es intervenida quirrgicamente. El diag-
5) Ampuloma. nstico ms probable ser:

45. Por tanto, el nico tratamiento curativo ser: 1) Isquemia mesentrica aguda.
2) Carcinoma obstructivo de ngulo esplnico.
1) Pancreatectoma total. 3) Angiodisplasia de ngulo esplnico.
2) Intervencin de Whipple. 4) Colitis isqumica gangrenada de ngulo esplnico.
3) Pancreatectoma corporocaudal. 5) Diverticulitis aguda perforada.
4) Trasplante hepatopancretico.
5) Quimiorradioterapia. 51. El tratamiento quirrgico empleado ser:

1) Reseccin del segmento clico afecto y anastomosis pri-


ABDOMEN AGUDO. maria trmino-terminal (T-T).
2) Reseccin del segmento de intestino delgado afecto y
46. Cul de las siguientes patologas "mdicas" NO cuenta entre anastomosis T-T.
sus complicaciones tpicas con una perforacin digestiva, 3) Reseccin del segmento clico afecto y exteriorizacin de
con la consiguiente necesidad de ciruga?: su extremo proximal (Hartmann).
4) Laparotoma exploradora.
1) Fiebre tifoidea. 5) Cierre de la perforacin y colostoma de descarga proximal.
2) Enteritis neutropnica.
3) Porfiria aguda intermitente. 52. Paciente mujer, de 40 aos, que vive en un pueblo, acude a
4) Carbunco digestivo. Urgencias con cuadro evolucionado de abdomen agudo,
5) lcera duodenal. ms focalizado en fosa ilaca derecha, estado sptico en pre-
shock, y que en las ltimas horas se acompaa de escalofros
47. En la apendicitis aguda, seale la correcta: y fiebre en picos, ictericia y dolor en hipocondrio derecho.
Su actitud ser:
1) Si el vmito precede al dolor, debe ponerse en duda el
diagnstico. 1) Antibioterapia i.v. y ver evolucin.
2) La perforacin del apndice suele producir un alivio de 2) Laparotoma urgente.
la sintomatologa. 3) Arteriografa mesentrica superior.
3) El signo del psoas es bastante especfico. 4) Batera de pruebas radiolgicas (eco, Rx, TC).
4) El dolor se localiza en fosa ilaca derecha desde el inicio 5) Fasciotoma urgente.
Preguntas TEST

del cuadro.
5) En nios menores de dos aos, es ms frecuente la 53. Ya que su primera sospecha diagnstica es:
apendicitis que la invaginacin intestinal.
1) Gangrena de Fournier.
48. Seale la respuesta INCORRECTA sobre la apendicitis aguda: 2) Absceso tubo-ovrico.
3) leo biliar.
1) Es el abdomen agudo que ms a menudo necesita ciruga. 4) Pileflebitis.
2) Tras la perforacin, comienza a aparecer distensin abdo- 5) Cncer de colon perforado con metstasis hepticas.
minal por leo.
3) En pacientes ancianos, se asemeja a una obstruccin 54. El tratamiento de un paciente con dolor abdominal genera-
mecnica de intestino delgado. lizado, hipersensibilidad a la palpacin, cuadro obstructivo,
4) Es frecuente en lactantes. fiebre, 17.000 leucocitos/mm3, signos de hipovolemia, y que
5) La incidencia de perforacin se acerca al 100% antes del en la radiografa simple de abdomen muestra dilatacin de
primer ao de vida. colon con "imagen en omega", ser:

49. Paciente mujer, de 72 aos, DMID, que present dolor abdo- 1) Enema opaco teraputico.
minal de inicio sbito, 10 horas antes de su ingreso, en 2) Devolvulacin endoscpica.

M exico A rgentina CTO Medicina C/ Nez de Balboa, 115 28006 MADRID (Espaa) Tfno.: (91) 782 43 32 / Fax: (91) 782 43 27
C hile U ruguay E-mail: secretaria@ctomedicina.com; iberocto@ctomedicina.com WEB: www.ctomedicina.com; www.iberocto.com
CG Pg. 5
CIRUGA GENERAL
Seguimiento a distancia Preparacin Examen de Seleccin 05/06 1 Vuelta
3) Reseccin del vlvulo de sigma + anastomosis T-T. 61. En el caso de que el enfermo fuera intervenido quirrgica-
4) Solucin evacuante. mente, los hallazgos quirrgicos que encontrara ms proba-
5) Intervencin de Hartmann con reseccin del segmento blemente seran:
afecto.
1) Apendicitis perforada.
55. El tratamiento inicial de la obstruccin intestinal de cualquier 2) lcera duodenal perforada encubierta con epipln.
causa es: 3) Diverticulitis perforada.
4) Isquemia mesentrica.
1) Laparotoma exploradora. 5) Diverticulitis de Meckel.
2) Adhesilisis.
3) Descompresin endoscpica. 62. Con respecto a su tratamiento, cul de estas opciones le
4) Reposicin hidroelectroltica y aspiracin gstrica. parece FALSA?:
5) Analgesia.
1) Habr que cerrar la perforacin.
56. Varn de 38 aos, que presenta dolor sbito epigstrico, de 2) Probablemente habr que dar tratamiento erradicador
1 hora de evolucin, con nuseas, sudoracin y taquicardia. del H. pylori.
En la exploracin fsica presenta defensa abdominal involun- 3) El cierre simple puede ser una opcin correcta.
taria, con abdomen "en tabla". La primera prueba que usted 4) El tratamiento debe incluir reseccin de la lcera.
debe emplear en la orientacin diagnstica sera: 5) Si el test de ureasa intraoperatorio es negativo, debe
realizarse vagotoma troncular + piloroplastia.
1) TC abdominal.
2) ECG. 63. De todo lo anterior y de sus amplios conocimientos sobre la
3) Rx de abdomen. patologa ulcerosa, Vd. deduce que es cierto que:
4) Rx de trax en bipedestacin.
5) Sedimento de orina. 1) El tratamiento de la lcera complicada es siempre resectivo.
2) Es frecuente que una perforacin cause un shock hipovo-
57. La primera sospecha diagnstica ser: lmico.
3) La perforacin es una complicacin ms frecuente que la
1) Infarto agudo de miocardio. hemorragia.
2) Clico nefrtico. 4) Un enfermo puede estar perforado y no existir neumope-
3) Perforacin de ulcus duodenal. ritoneo.
4) Diverticulitis perforada. 5) La mayora de las complicaciones actuales por lcera se
5) Apendicitis aguda. deben a fracaso del tratamiento farmacolgico.

58. La radiografa en bipedestacin no demostr neumoperito- 64. Mujer de 70 aos, con antecedentes de clicos biliares;
neo. La clnica del paciente se exacerb. Ante ello, la sospecha acude a Urgencias con un cuadro de dolor abdominal difuso
diagnstica sera ahora: clico, de 24 h de evolucin, nuseas, vmitos biliosos y
alimentarios. Actualmente asintomtica y afebril. Se solicitan
1) Infarto agudo de miocardio. analtica sangunea y Rx de abdomen. Se pautan espasmol-
2) Clico nefrtico. ticos etiquetando el cuadro de clico biliar. Con la evolucin,
3) Perforacin de ulcus duodenal. la enferma empeora, predominando clnica y semiologa de
4) Diverticulitis perforada. obstruccin intestinal. En la Rx de abdomen NO esperaremos
5) Apendicitis aguda. encontrar:

59. Por ello, el paso siguiente sera: 1) Aerobilia.

Preguntas TEST
2) Dilatacin de asa de intestino delgado.
1) Laparotoma urgente con el diagnstico de perforacin de 3) Clculo en leon terminal.
lcera duodenal. 4) Niveles hidroareos.
2) Endoscopia digestiva alta. 5) Dilatacin de colon sigmoide.
3) Estudio con contraste hidrosoluble.
4) Colocar SNG, introducir aire y repetir Rx trax en bipedes- 65. Su sospecha diagnstica ser:
tacin.
5) Lavado peritoneal diagnstico. 1) Colecistitis.
2) Coledocolitiasis.
60. Con ello, objetivamos la aparicin de una imagen de densidad 3) leo biliar.
area bajo ambas cpulas diafragmticas. La actitud terapu- 4) Pancreatitis litisica.
tica debera ser ahora: 5) Cncer obstructivo de intestino delgado.

1) Intervencin quirrgica. 66. El tratamiento de urgencias NO incluir:


2) Antibiticos.
3) Espasmolticos. 1) Reposo digestivo.
4) Analgsicos y evolucin. 2) Aspiracin digestiva.
5) Trombolticos. 3) Fluidoterapia intravenosa.

M exico A rgentina CTO Medicina C/ Nez de Balboa, 115 28006 MADRID (Espaa) Tfno.: (91) 782 43 32 / Fax: (91) 782 43 27
Pg. 6 CG C hile U ruguay E-mail: secretaria@ctomedicina.com; iberocto@ctomedicina.com WEB: www.ctomedicina.com; www.iberocto.com
CIRUGA GENERAL
Preparacin Examen de Seleccin 05/06 1 Vuelta Seguimiento a distancia

4) CPRE. ticamente presenta 11.000 leucocitos (79 % neutrfilos). La


5) Enterolitotoma. Rx de abdomen es normal. Cul de las siguientes actitudes
le parece MENOS adecuada?:
67. Mujer de 58 aos, que fue colecistectomizada de forma
programada por colelitiasis sintomtica. Tras la ciruga, la 1) Pedir valoracin ginecolgica.
paciente present episodios de dolor en epigastrio e hipo- 2) Realizar ecografa abdominal.
condrio derecho, clico, con nuseas y vmitos y sin altera- 3) Realizar ecografa vaginal.
ciones analticas. Este cuadro lo podemos etiquetar como: 4) Dejar en observacin 12 horas y hacer nueva valoracin
clnica y analtica.
1) Coledocolitiasis. 5) Intervenir quirrgicamente.
2) Reflujo gastroesofgico.
3) Dispepsia.
4) Sndrome postcolecistectoma. TRAUMATISMOS ABDOMINALES.
5) Sndrome depresivo.
73. Paciente varn de 35 aos, que presenta, tras accidente de
68. Mientras espera los estudios, acude a urgencias por dolor en trfico, traumatismo abdominal contuso con shock hipovo-
hipocondrio derecho y fiebre en picos. En el hemograma lmico, dolor y distensin abdominales. En el lavado perito-
destaca 17.000 leucocitos/mm3 con desviacin izquierda y neal aparece hemoperitoneo. El diagnstico ms probable
6,7 mg% de bilirrubina total; fosfatasa alcalina elevada. ser:
Probablemente la enferma padece:
1) Rotura esplnica.
1) Coledocolitiasis y colangitis. 2) Rotura heptica.
2) Coledocolitiasis y pancreatitis. 3) Laceracin heptica.
3) Coledocolitiasis y hepatitis. 4) Rotura diafragmtica.
4) Mun cstico calculoso. 5) Rotura de vena cava.
5) Cncer de pncreas.
74. Ante un paciente que presenta una herida por arma blanca
69. La actitud de urgencia debe ser: penetrante en abdomen, con evisceracin de varias asas de
intestino delgado, qu actitud tomara?:
1) Ciruga.
2) Antibioterapia. 1) Observacin durante 24-48 h.
3) Hepatoprotectores. 2) Laparotoma.
4) Analgsicos. 3) Realizar TC abdominal.
5) Seriar bilirrubina y ver evolucin. 4) Laparotoma, slo si el enfermo presenta shock.
5) Lavado peritoneal diagnstico.
70. El tratamiento diferido ser:
75. Un paciente varn es trado a urgencias por la UVI del 061,
1) Antagonistas del calcio. tras sufrir un accidente de trfico. El mdico os cuenta que
2) Reseccin del mun cstico. el paciente estaba con 90/60 mmHg de tensin cuando lo
3) Esfinterotoma transduodenal. recogi, hace 15 minutos, y que le ha administrado 700 cc de
4) CPRE + esfinterotoma endoscpica. lquidos i.v., 10 mg de midazolam y bloqueante muscular para
5) Trasplante heptico. la intubacin. El paciente, a su llegada, tiene Glasgow 6/15, TA
120/70 mmHg, 98 lpm, una herida incisa en la frente, mlti-
71. Cules son las tres exploraciones fundamentales que deben ples escoriaciones y contusiones, y pupilas algo dilatadas y
buscarse al valorar inicialmente a un paciente de 50 aos con reactivas. La placa de trax porttil es normal. La palpacin
Preguntas TEST

un cuadro clnico sugerente de obstruccin intestinal?: abdominal parece dolorosa, aunque no hay marcada disten-
sin del abdomen. La hemoglobina es de 11,8 g/dl, 12.800
1) Bsqueda de hernias, existencia de laparotomas previas leucocitos/mm3 (79% neutrfilos) y saturacin 96%. Cul
y tacto rectal. sera su actitud en esta situacin?:
2) Percusin abdominal, hemograma y Rx abdomen.
3) Palpacin abdominal, ionograma y Rx abdomen. 1) Laparotoma urgente.
4) Inspeccin abdominal, ionograma y tacto rectal. 2) Realizar TC craneal y abdominal.
5) Bsqueda de hernias, hemograma y Rx abdomen. 3) Realizar puncin lavado peritoneal.
4) Realizar ecografa abdominal.
72. Paciente de 29 aos, operada por endometriosis (anexecto- 5) Realizar Rx simple de abdomen.
ma izquierda) hace 2 aos. Acude a urgencias presentando
un cuadro de dolor abdominal que se ha ido localizando en
fosa ilaca derecha, nuseas, 2 vmitos y febrcula. No refiere
diarrea. Est en la mitad del ciclo. A la exploracin, parece
existir ms dolor en fosa ilaca derecha, sin contractura local,
pero con signo de Blumberg positivo. En el tacto vaginal,
parece que hay ms dolor al palpar el anejo derecho. Anal-

M exico A rgentina CTO Medicina C/ Nez de Balboa, 115 28006 MADRID (Espaa) Tfno.: (91) 782 43 32 / Fax: (91) 782 43 27
C hile U ruguay E-mail: secretaria@ctomedicina.com; iberocto@ctomedicina.com WEB: www.ctomedicina.com; www.iberocto.com
CG Pg. 7
CIRUGA GENERAL
Preparacin Examen de Seleccin 05/06 1 Vuelta Seguimiento a distancia
Pregunta 1.- R: 2 rias: la gangrena gaseosa (causada por Clostridium perfringens) y la
El sufijo "-toma" significa cortar. En ciruga se refiere al hecho de fascitis necrotizante (causada por algunas cepas de estreptococos del
abrir un rgano hueco para mirar en su interior. Por ejemplo, la grupo A). En estos casos la cobertura antibitica sistmica es necesaria
gastrotoma, en la que se abre el estmago para controlar un punto pero la piedra angular del tratamiento es el desbridamiento quirrgi-
sangrante o extraer un cuerpo extrao; la colotoma, en la que se co urgente de todo el tejido afectado, con la mxima clsica de la
abre el colon para extraer un plipo (polipectoma); la coledoco- gangrena: "cortar por lo sano".
toma, en la que se abre el coldoco para extraer un clculo biliar
retenido en l. El cierre de una "-toma" suele ser una "-rrafia" Pregunta 5.- R: 2
(gastrorrafia, colonorrafia, coledocorrafia). La traccin del folculo piloso permite distinguir si est afectada o
El sufijo "-ectoma" significa cortar y extirpar. Por ejemplo, gastrec- no la raz del pelo, esto es, si hay quemadura en dermis profunda
toma, colectoma, mastectoma o histerectoma. (distingue el 2 grado superficial del profundo).
El sufijo "-stoma" significa hacer una apertura o comunicacin. Los criterios de gravedad en funcin de la superficie y de la profun-
Cuando va precedido de dos rganos se refiere a comunicarlos didad se exponen claramente en el Manual. Ante un mismo estmulo
entre s (por ejemplo, gastroyeyunostoma que equivale a anasto- gneo, la profundidad de las quemaduras aumenta con la edad. Ello
mosis gastroyeyunal), mientras que si va precedido de un solo rga- se debe a que los procesos de envejecimiento conllevan una progre-
no se entiende que es comunicar ese rgano con la piel, directa- siva deshidratacin tisular, por lo que en tejidos con menos agua la
mente o mediante un tubo, como es el caso que nos ocupa (por quemadura es mayor.
ejemplo, gastrostoma, colostoma, ileostoma o colecistostoma), La antibioterapia sistmica profilctica no est indicada en los que-
estableciendo un estoma. mados, salvo en 3 situaciones bien concretas:
El sufijo "-pexia" significa pegar o fijar. Por ejemplo, la mastopexia Cuando existe inhalacin de humos, dado que se asocia a una neu-
(donde se fijan las mamas excesivamente colgantes), la cecopexia mona severa con frecuencia y conlleva importante mortalidad.
(donde se fija un ciego redundante para evitar que se volvue) o la Como profilaxis periquirrgica.
gastropexia posterior (donde se fija el estmago a los planos prever- En quemaduras por alto voltaje, que conllevan grandes zonas de
tebrales para evitar su deslizamiento hacia el trax, tcnica que se necrosis tisular.
puede utilizar en las hernias de hiato por deslizamiento).
El sufijo "-plastia" significa reconstruir o modificar la forma. Por Pregunta 6.- R: 3
ejemplo, la gastroplastia tubular, en la que se construye un tubo La diverticulosis del colon (localizada con mayor frecuencia en
con el estmago de modo que pueda sustituir a un esfago que ha sigma) es una condicin de muy alta prevalencia en la poblacin
sido extirpado; o la mamoplastia (de aumento o de reduccin del occidental y no asocia ningn tipo de sintomatologa, por lo que no
tamao mamario); o la gran variedad de gastroplastias empleadas precisa tratamiento alguno. Suele ser un hallazgo casual en enema
en la ciruga de la obesidad (ciruga baritrica). opaco o colonoscopia.
Cuando se complica es cuando da clnica, hablndose entonces de
Pregunta 2.- R: 5 enfermedad diverticular del colon: sus dos formas son la diverticulitis
Como se expone en el Manual, el diagnstico de la fiebre postope- (la complicacin ms frecuente de la diverticulosis) y la hemorragia
ratoria se enfoca en funcin del momento en el que aparece. Las diverticular. En estas situaciones s que es preciso un tratamiento, que en
causas de fiebre postoperatoria que tienen una relacin ms clara la mayora de los casos de diverticulitis es conservador, con antibiticos.
con la ciruga suelen manifestarse a partir del 3er da postoperatorio: la El cuadro de la diverticulitis es muy similar al de la apendicitis, pero
infeccin de la herida quirrgica a partir del 5 da y la fiebre en localizado en fosa iliaca izquierda (generalmente en sigma) y puede ir
relacin con fstulas, fugas y abscesos perianastomticos (que consti- desde un dolor suave hasta un cuadro de peritonitis. La mejor prueba
tuyen en realidad un mismo proceso con distintos nombres), suele de imagen en su diagnstico es la TC abdominal, estando contraindica-
aparecer al inicio de la segunda semana postoperatoria. dos el enema opaco y la colonoscopia. Slo en caso de formas asocia-
das a peritonitis o con mala evolucin existe una indicacin quirrgica
Pregunta 3.- R: 4 urgente. En estos casos se suele realizar una intervencin de Hartmann,
Las zonas del tubo digestivo ms predispuestas a complicaciones dejando la reconstruccin del trnsito para un segundo tiempo.
anastomticas son sus extremos: esfago y recto; ello explica la mayor
morbilidad con que estn gravadas estas cirugas. Cuando aparece Pregunta 7.- R: 2
Comentarios TEST

una fstula anastomtica, el tratamiento es conservador durante 4-8 La fstula colovesical tiene un origen esencialmente inflamatorio:
semanas y consiste en administrar antibiticos para controlar la infec- bien sea por diverticulitis (donde un divertculo inflamado se pone en
cin y dejar en reposo digestivo para minimizar el dbito a travs de la contacto con la vejiga), bien sea por enfermedad inflamatoria intesti-
fstula y facilitar su cierre espontneo (ello suele implicar la necesidad nal. La fstula vesicovaginal, en cambio, suele tener un origen obsttri-
de nutricin parenteral, dado que un buen estado nutricional es ne- co, mientras que la rectovaginal puede deber su origen a patologa
cesario para una correcta cicatrizacin). Para que la infeccin se man- perianal o ser un complicacin de ciruga oncolgica rectal o de
tenga controlada, es importante evitar que se formen grandes abscesos radioterapia.
intraabdominales, para lo cual es fundamental que las colecciones Clnicamente, la fstula colovesical se manifiesta como fecaluria,
tengan un camino de salida al exterior (un drenaje que se pudo dejar neumaturia e infecciones urinarias de repeticin.
en el momento de la ciruga o que se coloca posteriormente con Para el diagnstico de una fstula colovesical, el mejor abordaje diag-
control radiolgico para dar salida a una coleccin). Si la infeccin nstico es la va urinaria, bien sea por cistoscopia, bien por cistografa.
no se controla de esta manera, sobreviene una sepsis, que es la causa
ms frecuente de muerte en estas circunstancias. Pregunta 8.- R: 2
El leo paraltico es una respuesta fisiolgica a la agresin quirrgica En este caso se nos presenta un paciente con una colitis ulcerosa
(es un leo reflejo) y dura 24-48 horas en la ciruga gastrointestinal y que viene con un cuadro agudo que parece una reagudizacin (brote
hasta 5 das en la ciruga del colon. Un leo que se prolonga ms all severo), en el que llama la atencin la existencia de distensin abdo-
de lo esperado puede alertarnos de una complicacin anastomtica, minal con hipersensibilidad generalizada. Parece imperativo hacerse
mxime si se asocia a la aparicin de fiebre. una idea de la distensin colnica subyacente, porque si es superior
a 9 cm el riesgo de perforacin es importante, y si es superior a 12 cm
Pregunta 4.- R: 2 hay una indicacin absoluta de ciruga urgente; para ello lo ms til
Como veamos previamente, la fiebre en relacin con la infeccin ser la radiologa simple de abdomen.
de herida suele aparecer a partir del 5 da, precedida de dolor local
y eritema. Sin embargo, existen dos infecciones severas de partes blan- Pregunta 9.- R: 4
das que pueden aparecer en el contexto de una ciruga y que origi- Con la clnica antes mencionada, parece que este brote severo est
nan un cuadro sptico muy grave en las primeras horas postoperato- presentndose como un megacolon txico, por lo que habr que

M exico A rgentina CTO Medicina C/ Nez de Balboa, 115 28006 MADRID (Espaa) Tfno.: (91) 782 43 32 / Fax: (91) 782 43 27
C hile U ruguay E-mail: secretaria@ctomedicina.com; iberocto@ctomedicina.com WEB: www.ctomedicina.com; www.iberocto.com
EP Pg. CG
CIRUGA GENERAL
Seguimiento a distancia Preparacin Examen de Seleccin 05/06 1 Vuelta
instaurar un tratamiento inicialmente conservador (pero muy vigilan- pia coadyuvante al tratamiento quirrgico (esta quimioterapia suele
te) del mismo: dieta absoluta, fluidoterapia, antibiticos y corticoides ser a base de 5-FU y leucovorn, aunque ahora hay nuevos
parenterales. quimioterpicos en juego como el oxaliplatino y el irinotecn).

Pregunta 10.- R: 3 Pregunta 16.- R: 4


Dada su evolucin insatisfactoria a las 24 h, hay una indicacin La afectacin linftica define al estadio C en la clasificacin de
quirrgica evidente, de urgencia. Dado que se trata de un colon no Dukes y en todas sus modificaciones posteriores.
preparado, severamente dilatado y con una enfermedad de base que Las lesiones del colon derecho se manejan con hemicolectoma
afecta a toda su extensin, lo que habr que realizar es una colectoma derecha (ampliada a parte del transverso si la lesin est en el ngulo
total sin anastomosis (haciendo una ileostoma terminal temporal y heptico) y en la mayora de los casos se puede realizar anastomosis
difiriendo a un segundo tiempo la realizacin de una anastomosis y la primaria, incluso si son lesiones obstructivas (salvo en casos de perfo-
extirpacin del recto). Todas las otras opciones ofrecidas en esta pre- racin con gran contaminacin local que pondra en riesgo la anasto-
gunta son insuficientes y algunas (la 2 y la 4) entraan un riesgo in- mosis).
aceptable. Por el peculiar drenaje venoso del recto distal (a vasos ilacos y no
al sistema portal), es posible en esta localizacin la diseminacin al
Pregunta 11.- R: 3 pulmn sin existir enfermedad heptica. Por eso en esta situacin es
Los pacientes con EII tienen riesgo aumentado de cncer de colon importante la Rx de trax.
con respecto a la poblacin general (sobre todo los afectados por La reseccin de metstasis hepticas aisladas de cncer colorrectal
colitis ulcerosa). Este riesgo aumenta con el tiempo de evolucin y ofrece una supervivencia de hasta el 45 % a los 3 aos.
con la extensin de la enfermedad, y estos tumores presentan como
peculiaridad que los diferencia del cncer en otro contexto, el hecho Pregunta 17.- R: 4
de que con cierta frecuencia son tumores multifocales (mltiples) y se Nos encontramos ante una metstasis sincrnica de un cncer
presentan en el colon ascendente con una frecuencia superior a la de colorrectal (se descubre a la vez que el tumor primario, y no en el
la poblacin general (donde la localizacin ms habitual es recto y seguimiento posterior, en cuyo caso se llamara metstasis metacrnica).
sigma). Las sincrnicas tienen peor pronstico, pero ello no contraindica su
Su pronstico y tratamiento son iguales que los de un cncer de reseccin si cumplen los criterios actualmente aceptados para ello:
iguales caractersticas que aparece fuera del contexto de la EII. Enfermedad primaria controlada y sin recidiva ni metstasis a otro
nivel.
Pregunta 12.- R: 5 Enfermo capaz de tolerar una reseccin heptica.
El Metronidazol oral es el antibitico que se suele utilizar en trata- Metstasis en nmero de 3 menos.
miento prolongado para los pacientes con enfermedad de Crohn Deseable que las metstasis estn anatmicamente cerca, aunque
perianal que presentan fstulas, con la intencin de que no se formen no imprescindible.
abscesos. El Metronidazol tiene el inconveniente de producir neuro- Estos son los factores que se acepta que marcan la indicacin de
pata perifrica hasta en un 50% de los pacientes cuando el tratamien- resecar las metstasis, con un margen sano de 1 cm (no es necesario
to dura 6 meses o ms. hacer resecciones mayores si se pueden quitar las metstasis con
Como alternativas en el tratamiento de la enfermedad perianal (de resecciones menores). Sin embargo, estos criterios en realidad no
menos a ms agresiva en funcin de los fracasos teraputicos previos), son tan estrictos actualmente y se interpretan ms bien como marca-
se puede intentar la fistulectoma y el cierre de fstulas con colgajos de dores de mejor pronstico, extendindose cada vez ms las indica-
avance, la colostoma de derivacin para evitar que las heces sean ciones para resecar las metstasis que pueden aparecer en el cncer
fuente de infeccin para las fstulas y, hasta en el 30% de los pacientes colorrectal.
con afectacin perianal severa, la amputacin abdominoperineal. Un tema en discusin es si resecar las metstasis sincrnicas en el
mismo acto operatorio o diferir la metastasectoma, sin embargo en
Pregunta 13.- R: 1 este caso parece una lesin pequea y accesible fcilmente, por lo
Ante todo sndrome abdominal, tacto rectal... y vaginal. Este es un que puede hacerse en el mismo acto.
aforismo clsico de la ciruga francesa y debera de ser parte integrante
de nuestra prctica diaria. Estas dos exploraciones pertenecen a la Pregunta 18.- R: 5
exploracin fsica completa, que debe preceder a toda prueba com- Nos encontramos ante un paciente que consulta por un cuadro de

Comentarios TEST
plementaria. tipo obstructivo, progresivo, con aparente sndrome constitucional de
En el caso concreto de una paciente que refiere rectorragia, inde- fondo. Desde luego, el paciente no est clnicamente en situacin de
pendientemente del contexto clnico de la misma, el tacto rectal es gravedad que nos obligue a una laparotoma urgente (no nos cuentan
absolutamente imperativo e inexcusable. siquiera un cuadro de abdomen agudo). Parece lgico iniciar el estu-
dio con una analtica general y una radiologa simple de abdomen
Pregunta 14.- R: 1 para valorar dilatacin de asas y nivel de la obstruccin, adems de
Todos los tumores digestivos, a excepcin del esfago superior y dejarle en dieta absoluta, con sonda nasogstrica y fluidoterapia (como
del recto distal-canal anal tienen un drenaje venoso portal, por lo que a todo cuadro obstructivo).
su lugar habitual de metstasis es el hgado. Aunque el hgado puede
valorarse convenientemente con la ecografa, sin embargo sta no nos Pregunta 19.- R: 2
permite hacernos idea de la masa tumoral en el colon, sus relaciones Ya con los datos de la pregunta anterior, ante un cuadro obstructivo
con otras estructuras anatmicas, la existencia de adenopatas, etc. progresivo asociado a un sndrome constitucional, sospechamos un
igual de bien que lo hace la TAC, adems de que la ecografa tiene cncer de colon. Si adems nos encontramos ahora una anemia aso-
unas imgenes de difcil interpretacin por quien no la est llevando ciada, nuestra primera opcin diagnstica ser esa. Un cncer gstri-
a cabo. co obstructivo se manifestara de forma ms rpida (no tan larvada) y
con un cuadro predominante de vmitos (como corresponde a una
Pregunta 15.- R: 3 obstruccin alta) y no de estreimiento. Por otra parte, no hay que
Dado que no hay adenopatas ni metstasis, podemos descartar los olvidar que el cncer de colon izquierdo es la causa ms frecuente de
estadios C y D, respectivamente. El tumor se extiende a TODA la capa obstruccin del colon.
muscular, lo cual implica necesariamente que alcanza la serosa (la
afectacin de serosa es la que marca el 1 el 2, tanto en el B como en Pregunta 20.- R: 3
el C). Por ello se trata de un estadio B2, que justamente es el estadio en Sin embargo, pese a ser ms frecuente la obstruccin por cncer
el cual est indicado en el cncer colorrectal aadir una quimiotera- de colon izquierdo, la existencia de anemia asociada es bastante ca-

M exico A rgentina CTO Medicina C/ Nez de Balboa, 115 28006 MADRID (Espaa) Tfno.: (91) 782 43 32 / Fax: (91) 782 43 27
Pg. 2 CG C hile U ruguay E-mail: secretaria@ctomedicina.com; iberocto@ctomedicina.com WEB: www.ctomedicina.com; www.iberocto.com
CIRUGA GENERAL
Preparacin Examen de Seleccin 05/06 1 Vuelta Seguimiento a distancia
racterstica de la patologa del colon derecho (no olvidemos que el cin, dado que la mayora de las hemorragias se manejan con xito
colon derecho sangra y el colon izquierdo se obstruye) y si el tumor de forma conservadora.
es voluminoso se presenta igualmente como obstruccin. Dado que La hemorragia suele suceder en la cara posterior, afectando ra-
este es el diagnstico de nuestro paciente y que la localizacin dere- mas de la pared, y no a los troncos de la arteria gastroduodenal o de
cha es favorable a una anastomosis primaria salvo situaciones catas- la pilrica, que no se encuentran incluidas en el espesor de la pared.
trficas, el tratamiento de eleccin ser la reseccin segmentaria y La perforacin, por el contrario, suele suceder en cara anterior.
anastomosis. La penetracin es el nombre especial que recibe la perforacin
que sucede en cara posterior, generalmente hacia el pncreas, de
Pregunta 21.- R: 4 modo que queda contenida y no aparece una comunicacin libre
Nos encontramos ante una paciente que consulta de forma ambu- a cavidad peritoneal, con lo cual no aparece la peritonitis. En lo
latoria (no urgente, ojo!) por un cuadro de rectorragias. En este con- tocante a la presentacin de la perforacin ulcerosa, suele ini-
texto entran en juego muchas patologas: enfermedad diverticular, EII, ciarse como un dolor intenso, brusco en su aparicin, bien locali-
cncer de colon, hemorroides y otra patologa anorrectal benigna, zado en epigastrio, a veces irradiado a espalda, hombro o fosa
angiodisplasia intestinal y otras patologas vasculares. Habr que ha- iliaca (esto ltimo por acumulacin del cido a ese nivel). Rara-
cer una analtica bsica para ver la repercusin de estas rectorragias y mente aparecen vmitos (el perforado no es vomitador, se afirma
detectar una posible anemia. Por otra parte, una colonoscopia o un clsicamente). La exploracin muestra un paciente afectado, bas-
enema opaco nos darn informacin preciosa sobre la causa del tante inmvil, con la forma extrema de la irritacin peritoneal: el
sangrado y su localizacin. La arteriografa, aunque no se pedira de abdomen en tabla (o peritonitis), por el cido libre. Es clsico, pero
entrada, puede ser necesaria ms adelante para evaluar una patologa de escasa utilidad, el signo de la prdida de la matidez heptica,
vascular intestinal. Sin embargo, la TC no es una prueba de imagen causado por el neumoperitoneo. Se considera que la peritonitis
que nos vaya a dar gran informacin sobre un punto de sangrado en por perforacin ulcerosa es estril hasta que pasan unas 6 horas
la luz del tubo digestivo. del suceso. Ante este cuadro clnico, se debe buscar confirmar el
neumoperitoneo con la realizacin de una radiografa de trax en
Pregunta 22.- R: 4 bipedestacin (mostrar aire libre bajo las cpulas diafragmticas)
En el contexto clnico en que nos encontramos (consulta no ur- o, si el paciente no tolera la bipedestacin, una radiografa de
gente) y en un cuadro donde predomina la rectorragia esencialmente abdomen en decbito lateral con rayo horizontal.
(no se nos habla del dolor para nada), podemos descartar de entrada La estenosis pilrica es una urgencia diferible en el sentido de
la isquemia mesentrica (aguda o crnica), dado que su rasgo que su correccin no precisa realizarse de forma inmediata y es
prototpico y elemental es el dolor (como en todo cuadro isqumico conveniente rehidratar al paciente y restaurar una nutricin acepta-
a excepcin de los ACVAs). ble y una homeostasis correcta antes de ir a la solucin definitiva,
que es quirrgica (en ocasiones es resectiva y en otras la piloroplastia
basta).
Pregunta 22. Causas de hemorragia digestiva baja segn la edad.
Pregunta 25.- R: 4
Menos de 55 aos Ms de 55 aos Ver comentario anterior y el grfico siguiente.

1. Enfermedad anorrectal. 1. Enfermedad anorrectal. Rx de trax


2. Colitis (EII, infecciosas). 2. Diverticulosis. Da el diagnstico
3. Diverticulosis. 3. Angiodisplasia.
S No
4. Plipos, cncer (hiperplasia, 4. Plipos, cncer.
hamartomas). 5. Enterocolitis (isqumica,
5. Angiodisplasia. infecciosa, EII, radiacin). SNG
+
introducir aire
Comentarios TEST

Pregunta 23.- R: 4 y repetir Rx de trax


La paciente presenta un abdomen agudo (dolor abdominal de
inicio agudo y signos de irritacin peritoneal), focalizado en fosa iliaca da el diagnstico
izquierda. Ante esta clnica, nuestra primera sospecha diagnstica es
S No
siempre una diverticulitis.
Inicialmente, se har la valoracin inicial de todo abdomen agu-
do, es decir una anamnesis y exploracin, con una analtica ele- TC abdomen
mental. En el abdomen agudo es conveniente realizar una RX de
trax, puesto que hay patologa torcica que se presenta como ab- Pregunta 25. Diagnstico de la lcera gastroduodenal perforada.
domen agudo (tpicamente las neumonas de lbulo inferior) y, ade-
ms, podemos detectar un neumoperitoneo en caso de perfora- Pregunta 26.- R: 5
cin; tambin la radiologa de abdomen puede mostrar hallazgos Hasta hace unos pocos aos se habra dado como opcin correcta
interesantes. Y en esta pregunta nos piden lo que solicitaremos en en esta pregunta la 1. Sin embargo, las cosas han cambiado mucho en
primer lugar, y no cul sera la mejor prueba de imagen ante una la ciruga de la lcera con la llegada del Helicobacter pylori y los
sospecha de diverticulitis, que ya hemos dejado claro que es la TAC. inhibidores de la bomba de protones.
Toda opcin que incluya endoscopia o pruebas con contraste Actualmente se recomienda en las complicaciones quirrgicas
debe rechazarse por aumentar el riesgo de perforacin (al paciente de la lcera (hemorragia y perforacin), limitarse a solucionar la
obstruido o con abdomen agudo, por arriba slo la SNG y por abajo complicacion aguda (es decir, hacer hemostasia en el caso de la
slo el dedo: nicas excepciones, el vlvulo de sigma y la colonos- hemorragia y cerrar la perforacin en caso de que exista) SIN asociar
copia descompresiva del Ogilvie). ningn gesto definitivo de los que se preconizaban clsicamente
(que eran la vagotoma troncular + piloroplastia en la mayora de
Pregunta 24.- R: 3 los casos). En los casos de lceras gstricas de muy gran tamao y
En relacin con las complicaciones de la lcera gastroduodenal, con intenso componente fibrtico-inflamatorio est indicada toda-
son por orden de frecuencia: hemorragia, perforacin y estenosis. va la reseccin gstrica, pero esta situacin es poco frecuente en la
Actualmente, la que con ms frecuencia requiere ciruga es la perfora- actualidad.

M exico A rgentina CTO Medicina C/ Nez de Balboa, 115 28006 MADRID (Espaa) Tfno.: (91) 782 43 32 / Fax: (91) 782 43 27
C hile U ruguay E-mail: secretaria@ctomedicina.com; iberocto@ctomedicina.com WEB: www.ctomedicina.com; www.iberocto.com
EP Pg. CG
CIRUGA GENERAL
Seguimiento a distancia Preparacin Examen de Seleccin 05/06 1 Vuelta
Todava quedara una indicacin (ms terica que prctica) para tc- afectacin localmente irresecable, no hay carcinomatosis peritoneal),
nicas definitivas (vagotomas o antrectomas) en pacientes con lcera aunque la existencia de ganglios ensombrece algo el pronstico.
con Helicobacter negativo o en aquellos con muchos factores para En estas circunstancias est indicada una intervencin quirrgica
recidivar (recidivas previas tras tratamiento correcto de una afectacin con intencin curativa (oncolgica), que en el cncer gstrico se
ulcerosa crnica), pero estas situaciones son realmente excepcionales. basa en la realizacin de una reseccin gstrica (total o subtotal, en
este caso total porque hay afectacin del cuerpo y no slo del antro)
+ linfadenectoma + omentectoma. En ocasiones se asocia a esta
reseccin la esplenectoma o la pancreatectoma distal, bien por-
que haya afectacin focal del pncreas o, ms frecuentemente, para
poder asegurar una buena linfadenectoma.
En el dibujo de la pgina siguiente puedes ver la tcnica quirrgica
de la gastrectoma y linfadenectoma del adenocarcinoma gstrico.

Pregunta 28.- R: 1
Las lceras de Cushing son lceras gstricas por hipersecrecin en
pacientes con patologa del sistema nervioso central (traumtica o
tumoral). Se deben distinguir de cara al MIR de las lceras de Curling,
que aparecen en situaciones de hipovolemia aguda (sobre todo en
quemados) y cuya patogenia parece ms relacionada con una hipo-
perfusin de la pared gstrica por la hipovolemia.
Para acordarse de quin es quin puede sernos de utilidad recor-
dar a Harvey Cushing, padre de la neurociruga, que dio su nombre al
sndrome y enfermedad de Cushing por produccin hipofisaria excesi-
va de ACTH. De esta manera podremos relacionar su nombre con la
neurociruga y, por lo tanto, con patologa del sistema nervioso central.

Pregunta 29.- R: 4
Ver comentario de la pregunta 32.

Pregunta 30.- R: 1
Ver comentario de la pregunta 32.

Pregunta 26. Intervencin de Billroth I Pregunta 31.- R: 4


Ver comentario de la pregunta 32.

Pregunta 32.- R: 4
Nos encontramos con un caso clnico en el que nos presentan la
complicacin ms frecuente de la colelitiasis: el clico biliar. Aparece
cuando un clculo intenta salir de la vescula biliar, induciendo una
respuesta vagal refleja: espasmo de musculatura lisa (con el consi-
guiente dolor clico), nuseas, vmitos, sudor fro, malestar general. El
paciente presenta exclusivamente sntomas (destaca el dolor clico
en hipocondrio derecho), siendo la exploracin anodina (no hay
irritacin peritoneal ni otros signos) y la analtica rigurosamente nor-
mal. En este contexto de exploracin y analtica normales, no hay
indicacin de solicitar prueba de imagen urgente.
El manejo de esta situacin consiste en un tratamiento sintomtico

Comentarios TEST
durante la fase aguda (espasmolticos y analgsicos), con lo cual el
episodio cede y el paciente es dado de alta. De forma programada se
realizar una ecografa para comprobar la existencia de colelitiasis y,
dado que ha presentado sntomas, existe indicacin de colecistecto-
ma electiva (preferentemente laparoscpica).
La pieza de colecistectoma que enviemos al patlogo ser infor-
mada probablemente como colelitiasis (algo que ya suponamos) y
colecistitis crnica. Este ltimo hallazgo es el correlato anatomopato-
lgico del clico biliar, de manera que cualquier persona que es
portadora de una colelitiasis (muchas veces aunque sea asintomtica)
desarrolla habitualmente un infiltrado inflamatorio crnico a nivel de
la vescula. Ello explica que en algunos textos aparezca como compli-
cacin ms frecuente de la colelitiasis el clico biliar y en otros la
colecistitis crnica: son las dos caras de la misma moneda, una vista
por el clnico y otra por el patlogo. (Ver tabla).

Pregunta 33.- R: 4
Nos encontramos aqu con otra complicacin de la colelitiasis: la
colecistitis aguda. En este caso el paciente presenta un abdomen agu-
Pregunta 26. Intervencin de Billroth II. do (dolor abdominal de inicio agudo acompaado de signos de irri-
tacin peritoneal, como es la defensa). Un abdomen agudo focalizado
Pregunta 27.- R: 2 en hipocondrio derecho nos sugiere como primer diagnstico una
Nos encontramos ante un cncer gstrico que afecta cuerpo y antro colecistitis aguda. El dolor irradiado a espalda, la febrcula y el dolor al
y que no presenta criterios de irresecabilidad (no hay metstasis, no hay palpar en hipocondrio derecho y epigastrio son caractersticos. Aun-

M exico A rgentina CTO Medicina C/ Nez de Balboa, 115 28006 MADRID (Espaa) Tfno.: (91) 782 43 32 / Fax: (91) 782 43 27
Pg. 4 CG C hile U ruguay E-mail: secretaria@ctomedicina.com; iberocto@ctomedicina.com WEB: www.ctomedicina.com; www.iberocto.com
CIRUGA GENERAL
Preparacin Examen de Seleccin 05/06 1 Vuelta Seguimiento a distancia
Comentarios TEST

Pregunta 27. Gastrectoma: tcnica quirrgica.Linfadenectoma segn el territorio afectado.

que no se refiera en este caso, el signo de Murphy es tpico de la (leucocitosis, neutrofilia, en casos graves desviacin izquierda), sin otros
colecistitis, y consiste en que la inspiracin del paciente se corta cuan- hallazgos particulares (ntese que NO hay colestasis significativa ni
do presionamos en hipocondrio derecho sobre la localizacin hipo- citlisis, luego la bilirrubina y los enzimas hepticos no estn
ttica de la vescula (el signo de Murphy ecogrfico es lo mismo, pero significativamente alterados).
hecho por el ecografista, y tiene ms valor diagnstico por el correlato El diagnstico de colecistitis aguda se confirma con la ecografa,
de la imagen). Analticamente aparecen hallazgos de infeccin que mostrar signos indirectos de colecistitis: distensin, edema peri-

M exico A rgentina CTO Medicina C/ Nez de Balboa, 115 28006 MADRID (Espaa) Tfno.: (91) 782 43 32 / Fax: (91) 782 43 27
C hile U ruguay E-mail: secretaria@ctomedicina.com; iberocto@ctomedicina.com WEB: www.ctomedicina.com; www.iberocto.com
EP Pg. CG
CIRUGA GENERAL
Seguimiento a distancia Preparacin Examen de Seleccin 05/06 1 Vuelta
vesicular, pared engrosada, colelitiasis en el 90% de los casos (existe la traumatizados, SIDA, enfermedad inflamatoria intestinal severa, tras
colecistitis alitisica, sobre todo en pacientes crticos), a veces signo de circulacin extracorprea, tras nutricin parenteral prolongada...).
la doble pared. La colecistitis enfisematosa es un tipo de colecistitis aguda que pue-
El tratamiento de la colecistitis, como de todas las complicaciones de ser litisica o alitisica, que aparece tpicamente en diabticos y es
de la colelitiasis es, a la postre, quirrgico. Sin embargo, se ha discuti- originada por anaerobios (ello explica la presencia de gas en las vas
do mucho sobre el momento ideal para la ciruga. Parece claro que biliares o aerobilia). Se presenta como un cuadro sptico muy grave y
hay que evitar realizar la colecistectoma entre el da 3 y el 30 del con alta mortalidad.
episodio inflamatorio, por lo que se proponen dos actitudes igual- Como norma general (las excepciones se indican en el libro), NO
mente vlidas: est indicada la colecistectoma en pacientes asintomticos o con
Operar en el momento de hacer el diagnstico, sin demorar la inter- sntomas no atribuibles a su colelitiasis.
vencin. El epitelio de la vescula es glandular, por lo que la neoplasia ms
Realizar un tratamiento antibitico inicial al hacer el diagnstico y frecuente a este nivel es el adenocarcinoma. Ver dibujos en pginas 6 y 7.
reevaluar a las 48 h si est siendo eficaz. Si la evolucin es favora-
ble, completar el tratamiento antibitico (enfriar la colecistitis)
para luego hacer ciruga electiva pasados 2 3 meses; si la evolu-
cin es desfavorable, indicar ciruga urgente antes de que entremos
en la fase indeseable que se inicia hacia el da 3.
En este caso, para que no tengamos dudas, nos ponen un paciente
anciano, cardipata y anticoagulado, de manera que la opcin con-
servadora aparece como la ms adecuada.

Pregunta 34.- R: 1
La coledocolitiasis aparece cuando un clculo sale de la vescula y
obstruye la va biliar principal, provocando una colestasis (y si se localiza
cerca de la papila, tambin una pancreatitis litisica). Muchas veces el
clculo pasa solo al tubo digestivo, pero en ocasiones se hace necesario
extraerlo, siendo para ello de utilidad la CPRE con extraccin del clculo
y esfinterotoma asociada (antes de la aparicin de la CPRE, esta situacin
se solucionaba con ciruga abierta: coledocotoma y extraccin de cl-
culos, opcin que sigue siendo vlida si la CPRE no es resolutiva).
Normalmente, una vez resuelta la fase aguda de colestasis median-
te CPRE, se indica colecistectoma electiva. Sin embargo, cuando la
coledocolitiasis aparece en pacientes ya colecistectomizados, la CPRE
se convierte en la solucin definitiva, siendo sta su indicacin ms
clsica y resolutiva.
La colecistitis aguda calculosa (supone el 90% de las colecistitis,
como vimos en la pregunta anterior) aparece con ms frecuencia en
mujeres, lo cual es lgico si tenemos en cuenta que la prevalencia de
colelitiasis es mucho mayor en el sexo femenino. La colecistitis ali-
tisica (acalculosa), sin embargo, es ms frecuente en varones y en Pregunta 34. Lugares ms frecuentes de impactacin de clculos biliares.
pacientes crticos o con patologa de base grave (quemados, poli-

Comentarios TEST

Pregunta 34. Esfinterotoma endoscpica.

M exico A rgentina CTO Medicina C/ Nez de Balboa, 115 28006 MADRID (Espaa) Tfno.: (91) 782 43 32 / Fax: (91) 782 43 27
Pg. 6 CG C hile U ruguay E-mail: secretaria@ctomedicina.com; iberocto@ctomedicina.com WEB: www.ctomedicina.com; www.iberocto.com
CIRUGA GENERAL
Preparacin Examen de Seleccin 05/06 1 Vuelta Seguimiento a distancia
Pregunta 37.- R: 2
La actitud inicial tras diagnosticar un pseudoquiste es realizar una
vigilancia peridica con ecografas seriadas, durante un perodo aproxi-
mado de 6 semanas. Si durante este tiempo disminuye de tamao por
debajo de los 6 cm, la actitud ser conservadora (ello sucede ms fre-
cuentemente con los pseudoquistes de la pancreatitis aguda); si persis-
ten en su tamao por encima de 6 cm (lo cual es ms frecuente en la
pancreatitis crnica), el riesgo de complicaciones es mayor (rotura, he-
morragia) y por lo tanto se indica drenaje quirrgico a un asa de intes-
tino delgado (cistoyeyunostoma en Y) o al estmago (cistogastrostoma).

Pregunta 38.- R: 1
Vase comentario anterior.

Pregunta 39.- R: 4
El paciente presenta un cuadro de dolor abdominal y sepsis. La
existencia de un nivel hidroareo en el pseudoquiste confirmar la
sobreinfeccin del mismo, por lo que nos hallamos ante una grave
complicacin del quiste: la sepsis pancretica.
En condiciones normales, como indicamos en la pregunta 37, el
tratamiento de los pseudoquistes es el drenaje interno, evitndose las
punciones percutneas y derivaciones externas, ya que generan fstulas
de muy difcil manejo. Sin embargo, en condiciones crticas (como lo
es la sepsis pancretica), el riesgo vital del enfermo es grave y el peligro
de la fstula pancretica se convierte en un mal menor, de manera
que se trata de drenar el foco sptico lo antes posible de la forma menos
agresiva. Por ello elegimos en esta situacin el drenaje percutneo.
Por otra parte, el hecho de que la lesin se haya abscesificado nos
habla a favor de su independencia fsica (ser una coleccin cerrada,
probablemente sin comunicacin con el Wirsung), lo cual nos tran-
quiliza algo en relacin con el riesgo de fstula pancretica.

Pregunta 40.- R: 4
Pregunta 34. Colangitis por coledocolitiasis. Los adenomas son neoplasias benignas del hgado. Clsicamente
se relaciona su aparicin con el uso de esteroides (anticonceptivos
en mujeres, que constituyen el grupo ms numeroso, o testosterona
Pregunta 35.- R: 5 en varones), disminuyendo o a veces incluso desapareciendo cuan-
Se nos presenta un cuadro clnico caracterstico de una pancreatitis, do se abandona el consumo de estos frmacos.
con algunos signos de especial gravedad, como es el hecho de que
presente coloracin azulada en flancos (sugiere pancreatitis necrohe-
morrgica), hipocalcemia y derrame pleural. Sin embargo, por mucha Pregunta 40. Tumores hepticos benignos.
que sea la gravedad, los pacientes con pancreatitis aguda slo parecen
beneficiarse de la ciruga (segn la evidencia actual) cuando existe una Adenoma Hiperplasia Hemangioma (es
necrosis que se demuestra infectada mediante cultivo de material obte- hepatocelular nodular focal el ms frecuente)
nido por puncin, situacin en la cual se indica realizar una
Comentarios TEST

necrosectoma. Fuera de este supuesto, la ciruga slo aade morbilidad M u je r +


Paciente tipo M u je r M u je r
y dispara la mortalidad de estos enfermos, que ya de por s es elevada. a nticonceptivos
Ha lla zgo
Dolor a bdom ina l
Clnica
inespecfico
M a sa pa lpa ble
Hem orra gia
Shock
Complicaciones hipovolm ico
M a ligniza cin (si
m s de 6 cm )
Pregunta 35. Criterios de Ransom a las 48 horas. No (si ha sido No (slo si es
Tratamiento Ciruga correcta m ente gra nde y/o da n
Pregunta 36.- R: 3 dia gnostica do) s n t o m a s )
El cuadro que se refiere que present el paciente en su anterior
ingreso es, evidentemente, una pancreatitis aguda. Ahora regresa con
clnica, lo cual nos debe hacer sospechar una complicacin de aquel Tambin aparecen en el contexto de las enfermedades por dep-
episodio reciente. En concreto, presenta masa palpable en mesogas- sito de glucgeno, situacin mucho ms rara. Hasta el 80% de los
trio junto con clnica de efecto masa (saciedad, vmitos). Indepen- pacientes presentan algn tipo de molestia abdominal, si bien son
dientemente de lo que sea el cuadro en cuestin, su origen ser pro- sntomas muy inespecficos. Sus dos potenciales complicaciones son
bablemente pancretico: dado que sabemos la localizacin, la mejor la hemorragia en el seno del tumor, que puede llegar a romperse a la
prueba de imagen en el pncreas es el TC abdominal. superficie heptica y causar un hemoperitoneo grave (esto sucede

M exico A rgentina CTO Medicina C/ Nez de Balboa, 115 28006 MADRID (Espaa) Tfno.: (91) 782 43 32 / Fax: (91) 782 43 27
C hile U ruguay E-mail: secretaria@ctomedicina.com; iberocto@ctomedicina.com WEB: www.ctomedicina.com; www.iberocto.com
EP Pg. CG
CIRUGA GENERAL
Seguimiento a distancia Preparacin Examen de Seleccin 05/06 1 Vuelta
aproximadamente en un 30% de los adenomas), y la posibilidad de el estudio gastroduodenal puede ser de utilidad en el adenocarcino-
malignizacin (hasta un 10% de los adenomas extirpados contienen ma duodenal o en el ampuloma, permitiendo ver la masa. Sin embar-
un hepatocarcinoma). Por ello, se recomienda una actitud quirrgi- go, las pruebas de medicina nuclear no aportan nada en el diagnsti-
ca ante adenomas que no regresan al interrumpir el tratamiento co de los tumores periampulares.
hormonal, as como en los de mayor tamao. El pronstico de los tumores periampulares es globalmente malo,
sobre todo por la gran preponderancia del cncer de pncreas (el
Pregunta 41.- R: 2 ampuloma y el colangiocarcinoma tienen mejor pronstico, sobre todo
Los hemangiomas son el tumor benigno ms frecuente del hga- el primero). El tratamiento con intencin curativa de todos ellos (si el
do, siendo francamente prevalentes (aparecen entre el 2% y el 7% estudio de extensin no lo contraindica), es la duodenopancreatecto-
de las autopsias). Su aparicin no se relaciona especialmente con ma ceflica (intervencin de Whipple, descrita en 1935 por vez prime-
ningn consumo farmacolgico. Habitualmente son asintomticos, ra), al cual llegan slo un 20 % de los cnceres de pncreas. Tras un
no se complican y no causan ningn tipo de problemas, salvo que Whipple (ciruga que tiene una mortalidad de hasta el 15 % en algunas
sean de gran tamao (se llaman gigantes a partir de los 4 cm). El series), el pronstico es an as malo en el caso del pncreas, sobrevi-
diagnstico puede establecerse a menudo por su apariencia ecogr- viendo a los 5 aos menos del 20% de los enfermos resecados con
fica y con ayuda del Doppler, si bien la RM es de utilidad para intencin curativa.
caracterizarlos en caso de duda por su alta sensibilidad y especifici- La quimioterapia y la radioterapia tienen en el cncer de pncreas
dad. Dada su evolucin benigna, no est indicado ningn tipo de un papel esencialmente paliativo, aunque hay ensayos recientes que
tratamiento en estos tumores. apuntan a una indicacin neoadyuvante (es decir, preoperatoria para
Sin embargo, en algunos casos en los que el tumor es voluminoso y reducir el tamao tumoral).
aparece clnica de ocupacin abdominal o molestia, est indicada la
reseccin del tumor como tratamiento sintomtico. Ms raramente Pregunta 46.- R: 3
aparecen en el contexto de cuadros angiomatosos sistmicos (sndrome El trmino abdomen agudo se refiere al sndrome compuesto por
de Kasabach-Merritt), asociados a coagulopata de consumo y trombo- la aparicin de dolor abdominal de inicio agudo y signos de irritacin
citopenia, siendo incluso necesario el trasplante de forma excepcional. peritoneal (hipersensibilidad, contractura, rebote). Sin embargo, el
hecho de que un paciente presente un abdomen agudo NO indica la
Pregunta 42.- R: 3 necesidad de ciruga. Hay muchas causas de abdomen agudo que no
El trmino tumor de Klatskin se refiere exclusivamente al colangio- son subsidiarias de ciruga (ver tabla).
carcinoma que aparece en la unin de ambos conductos hepticos, En esta pregunta se nos presentan patologas mdicas que se perfo-
en la vecindad de la placa hiliar. ran (la fiebre tifoidea lo hace en el 4% de los casos, la colitis por CMV
El trasplante es, sin duda, el tratamiento que ofrece mejores resulta- lo hace ms raramente, el carbunco digestivo puede perforarse y tam-
dos al hepatocarcinoma. Sin embargo, dada la escasez de rganos y bin lo hacen hasta el 8% de las lceras duodenales). Sin embargo, la
la altsima probabilidad de recidiva en los casos que presentan mu- PAI causa una neuropata que manifiesta crisis severas de dolor abdo-
chos factores de mal pronstico, actualmente se restringen las indica- minal, pero sin afectacin del tubo digestivo (se afectan los plexos
ciones a pacientes con enfermedad de buen pronstico (no metsta- nerviosos, pero no las vsceras).
sis, un tumor nico menor de 5 cm o hasta tres tumores de menos de
3 cm) y cuya situacin funcional heptica impide la reseccin (esta- Pregunta 46.Causas no quirrgicas de dolor abdominal.
dio B C de Child). En los estadios A de Child en los que la reseccin
es factible, habitualmente se recurre a ella.
El angiosarcoma es una neoplasia de muy mal pronstico, cuya IAM.
Cardacas.
relacin con la exposicin al cloruro de vinilo es bien conocida. Pericarditis aguda.
El carcinoma fibrolamelar es una neoplasia heptica de mejor
pronstico que el carcinoma hepatocelular. Sin embargo, hay va- Neumona.
Pulmonares.
rios trabajos recientes que afirman que el mejor pronstico obede- Infarto pulmonar.
ce exclusivamente al hecho de que aparece en pacientes jvenes y
Pancreatitis aguda.
sobre un hgado sano, sin que histopatolgicamente pueda distin-
guirse del carcinoma hepatocelular del que en definitiva es parte. Enfermedad de Crohn.
Adenitis mesentrica.
Gastrointestinales.

Comentarios TEST
Pregunta 43.- R: 5 Diverticulitis no complicada.
Ver pregunta 45. Gastroenteritis.
Hepatitis aguda.
Pregunta 44.- R: 1
Ver pregunta 45. Insuficiencia SR aguda.
Endocrinas.
Cetoacidosis diabtica.
Pregunta 45.- R: 2
El caso es el de un paciente de 70 aos que presenta dolor abdominal Tabes dorsal
SNC y perifrico.
y prdida de peso, as como ictericia. En este contexto hay que investigar Compresin races nerviosas.
una neoplasia abdominal que afecte a la va biliar (bien sea primaria del
rea hepatobiliopancretica, o bien sea primaria de otro rgano y ha Porfiria aguda.
afectado en su crecimiento a la va biliar, como podra ser un cncer Metablicas. Fiebre mediterrnea familiar.
gstrico). El hecho de que el paciente no presente otra clnica digestiva Hiperlipemia.
hace poco probable el cncer gstrico, que raramente debuta con icte-
ricia. En este contexto, lo ms probable es un tumor periampular Hematolgicas. Crisis drepanoctica.
(adenocarcinoma de cabeza pancretica, ampuloma, colangiocarcinoma EIP.
distal o adenocarcinoma duodenal), de los cuales el ms frecuente es el
cncer de la cabeza del pncreas (adenocarcinoma ductal). Embarazo ectpico.
Ginecolgicas.
La ecografa abdominal constituye el estudio inicial para cualquier Endometriosis.
ictericia obstructiva. La CPRE tambin puede ser de utilidad en este Quiste ovrico torsionado.
contexto, para diagnstico diferencial con una coledocolitiasis, para
tomar muestras para estudio citolgico y para colocar una prtesis Pregunta 47.- R: 1
paliativa biliar en caso de enfermedad irresecable. La TC es impres- Ver comentario de la pregunta siguiente.
cindible como estudio de extensin (local y a distancia). Finalmente,

M exico A rgentina CTO Medicina C/ Nez de Balboa, 115 28006 MADRID (Espaa) Tfno.: (91) 782 43 32 / Fax: (91) 782 43 27
Pg. 8 CG C hile U ruguay E-mail: secretaria@ctomedicina.com; iberocto@ctomedicina.com WEB: www.ctomedicina.com; www.iberocto.com
CIRUGA GENERAL
Preparacin Examen de Seleccin 05/06 1 Vuelta Seguimiento a distancia
Pregunta 48.- R: 4 Pregunta 52.- R: 2
La apendicitis aguda es la causa ms comn de abdomen agudo, Otra vez nos encontramos ante un abdomen agudo, localizado
cifrndose en un 7% el riesgo de padecerla a lo largo de la vida. Es ms inicialmente en fosa ilaca derecha (la causa ms frecuente en esta
rara en los extremos de la vida, siendo excepcional en nios menores localizacin es la apendicitis aguda), en un contexto clnico de grave-
de 2 aos. dad. Por ello, la decisin debe de ser quirrgica y urgente en cual-
Clnicamente aparece un dolor periumbilical o epigstrico con quier caso por lo previamente expuesto en la pregunta 49.
anorexia, que en las horas siguientes se sita en fosa iliaca derecha,
apareciendo entonces los vmitos y la febrcula o fiebre (raramente Pregunta 53.- R: 4
mayor de 38 C si no est complicada). Por ello, cuando los vmitos La pileflebitis es una complicacin infrecuente de la apendicitis agu-
preceden al dolor se deben de considerar otros diagnsticos (espe- da, consistente en la suelta de mbolos spticos al sistema venoso portal,
cialmente una gastroenteritis aguda). con lo que se provocan abscesos hepticos por siembra hematgena.
Como en todo abdomen agudo, son imprescindibles los signos En esta situacin est indicada la intervencin quirrgica urgente y
de irritacin peritoneal, que en este caso son variopintos. Sin embar- la antibioterapia intravenosa. Con ello se suelen resolver los abscesos
go, el que tiene mayor valor de todos ellos de cara al diagnstico es hepticos sin necesidad de drenaje percutneo.
el signo de Blumberg (rebote positivo en fosa ilaca derecha). El diag-
nstico es clnico en la mayora de los casos (la experiencia del Pregunta 54.- R: 5
cirujano es fundamental), pero hasta en un 25% existen dudas diag- Nuevamente se nos presenta un caso de abdomen agudo (dolor
nsticas y se puede recurrir a la ecografa o la TC. An as, el 20% de agudo + irritacin peritoneal) con signos de gravedad (hipovolemia).
las laparotomas por apendicitis NO presentan apendicitis aguda. En este contexto la ciruga es imperativa, salvo que tengamos un diag-
En los lactantes y en los ancianos, el cuadro puede ser menos nstico que nos descarte la ciruga (si se tratara de una neumona,
manifiesto, y sin embargo evoluciona ms rpidamente a la perfora- cetoacidosis, IAM, Crohn, etc... lo cual no es el caso). Antes bien, la
cin. En el caso de los ancianos es caracterstico que el cuadro se existencia de una imagen en omega nos indica que se trata de un
presente con caractersticas obstructivas, debido al leo local o al vlvulo de sigma. Dados los datos de gravedad, el intento de trata-
atrapamiento de algn asa en el plastrn apendicular. Tanto en unos miento conservador mediante devolvulacin est contraindicado, ya
como en otros puede existir ausencia de leucocitosis en la analtica. que probablemente nos encontramos ya con un severo compromiso
La perforacin no debera de llegar al 10% en el caso de los vascular (irreversible) si no es ya una perforacin por gangrena del
adultos, y suele acompaarse de un empeoramiento del dolor, que sigma. En este contexto el Hartmann es de eleccin.
se hace ms difuso, aparece mayor irritacin peritoneal, mal estado
general y fiebre ms alta. Hay un mnimo porcentaje de casos en los Pregunta 55.- R: 4
que se alivia la sintomatologa transitoriamente al perforarse. Cualquier cuadro de detencin del trnsito digestivo (que reci-
ben el nombre genrico de leo independientemente de su causa,
Pregunta 49.- R: 1 obstructiva o paraltica) se maneja inicialmente de forma comn:
Cuando nos encontramos con un dolor abdominal (tanto en la dieta absoluta dado que est paralizado el trnsito y existen proba-
clnica como en el MIR), debemos de proceder siempre por orden y blemente vmitos, sonda nasogstrica para aliviar los vmitos y
hacernos estas 3 preguntas por este orden: extraer todo el lquido retenido en el tubo digestivo y sueroterapia
1) Tiene el/la paciente un abdomen agudo?. intravenosa (porque el paciente est en dieta absoluta y porque el
2) Hay datos objetivos de gravedad? (de sepsis o peritonitis generalizada). secuestro de lquidos en el tubo digestivo facilita su deshidratacin,
3) Tengo un diagnstico claro?. amn de que los trastornos hidroelectrolticos estn en la base de
Las respuestas, aunque en apariencia no aporten mucho, nos van muchos leos paralticos). La analgesia tambin est indicada, aun-
a determinar fcilmente la actitud a seguir en cada caso. que en el contexto de un abdomen agudo no diagnosticado puede
En la paciente que nos ocupa, s tiene un abdomen agudo (dolor enmascarar la clnica. Luego, en funcin de la evolucin o de la
abdominal agudo + irritacin peritoneal) y s que hay ahora datos causa (si es conocida y mecnica) se indica el tratamiento etiolgi-
objetivos de gravedad (de hecho, en este caso nos lo dicen explcita- co (como sera la ciruga en caso de obstruccin por cncer) o
mente). Por otra parte, no tenemos un diagnstico claro que contrain- paliativo adecuado (como sera una colonoscopia descompresiva
dique la ciruga. En un abdomen agudo con datos de gravedad y en un sndrome de Ogilvie o pseudoobstruccin aguda del colon).
causa desconocida es mandatoria la laparotoma exploradora.
Pregunta 56.- R: 4
Comentarios TEST

Pregunta 50.- R: 4 Vase siguiente comentario.


La paciente es una diabtica que present un dolor de inicio sbi-
to (tpico de cuadros isqumicos o perforativos), en hemiabdomen Pregunta 57.- R: 3
inferior (no ms localizado, como suele suceder en las perforaciones, Se nos cuenta un abdomen agudo de aparicin sbita (recorde-
pero tambin es una anciana y diabtica, que siempre localizan peor mos que en este contexto hay que considerar sobre todo cuadros
el dolor). Ha presentado rectorragia y tiene un abdomen con disten- isqumicos y perforativos), en un paciente joven (ya podemos ir des-
sin y pocos ruidos (probablemente es un leo reflejo, dado que el cartando los isqumicos), muy bien localizado en epigastrio (la buena
dolor ha precedido a la distensin, a diferencia de lo que suele ocu- localizacin es tpica de los cuadros perforativos) y seguido de la apa-
rrir en la obstruccin mecnica). La Rx de abdomen nos muestra un ricin de un abdomen en tabla (que suele ser reflejo de peritonitis
leo con fin abrupto en ngulo esplnico, lo cual sugiere un proceso difusa por existencia de un lquido libre irritante en la cavidad
a ese nivel. El ngulo esplnico es el asiento tpico de cnceres de peritoneal: bilis, sangre, cido, pus...). La sospecha evidente es de
colon izquierdo (pero la clnica es excesivamente rpida y florida perforacin ulcerosa y para ver el neumoperitoneo solicitaremos una
para una obstruccin por cncer, que suele ser subaguda y ms larvada) radiografa de trax en bipedestacin. Si el dolor impide la bipedesta-
y de colitis isqumica (que es lo que tenemos que pensar en este caso cin al enfermo, se realizar una radiografa de abdomen en decbito
ante el antecedente de diabetes, el inicio sbito, la rectorragia acom- lateral y con rayo horizontal.
paante y la radiologa).
Pregunta 58.- R: 3
Pregunta 51.- R: 3 Ver pregunta 60.
Dado que tenemos un segmento gangrenado del colon, su resec-
cin es imperativa. Una vez realizada, teniendo en cuenta que se trata Pregunta 59.- R: 4
de una enfermedad vascular y de una paciente de riesgo (anciana y Ver pregunta 60.
diabtica) a la que hemos operado en un contexto de gravedad, la
anastomosis primaria queda totalmente descartada, siendo de elec- Pregunta 60.- R: 1
cin un Hartmann (colostoma proximal tras la reseccin). Hasta un 25% de las perforaciones ulcerosas no presentan neumo-
peritoneo en la radiologa simple. Por ello, la ausencia de neumope-

M exico A rgentina CTO Medicina C/ Nez de Balboa, 115 28006 MADRID (Espaa) Tfno.: (91) 782 43 32 / Fax: (91) 782 43 27
C hile U ruguay E-mail: secretaria@ctomedicina.com; iberocto@ctomedicina.com WEB: www.ctomedicina.com; www.iberocto.com
EP Pg. CG
CIRUGA GENERAL
Seguimiento a distancia Preparacin Examen de Seleccin 05/06 1 Vuelta
ritoneo no debe de modificar nuestra sospecha clnica, sino hacernos El tratamiento del leo biliar es inicialmente el de cualquier cua-
buscar otro medio para confirmarla, que segn el algoritmo que vi- dro obstructivo (SNG + dieta + sueros), seguido de la enterolitoto-
mos en la pregunta 25, ser la colocacin de SNG e introduccin de ma (extraccin del clculo que obstruye el intestino). En el acto
aire repitiendo a continuacin la radiologa simple. Si an as segui- quirrgico urgente no es necesario actuar sobre la vescula ni sobre
mos sin encontrar neumoperitoneo, la TC es el medio ms sensible la fstula, lo cual se har slo si las condiciones son muy favorables.
para la deteccin del mismo. La CPRE carece por completo de sentido porque no hay patologa a
nivel de la va biliar y, desde luego, no tiene ninguna utilidad ver el
Pregunta 61.- R: 2 rbol biliar.
La ausencia de neumoperitoneo puede obedecer a tres causas:
La lcera se ha tapado con epipln, evitando la salida de ms aire. Pregunta 67.- R: 4
La perforacin ha sucedido en cara posterior (penetracin), con lo Ver pregunta 70.
que no existe salida para el aire a cavidad libre.
La cantidad de aire que ha salido es muy pequea y la radiologa Pregunta 68.- R: 1
simple no la detecta. Ver pregunta 70.

Pregunta 62.- R: 4 Pregunta 69.- R: 2


El cierre de la perforacin es imperativo y slo puede ser quirrgi- Ver pregunta 70.
co (abierto o laparoscpico). Hasta la llegada de la era del Helicobacter,
se asociaba una medida definitiva que era la vagotoma troncular + Pregunta 70.- R: 4
piloroplastia en casos como este. Sin embargo, actualmente est esta- Se nos presenta el caso de una paciente colecistectomizada que
blecido que no se precisa asociar ningn gesto definitivo al cierre presenta cuadros de clicos biliares tras la ciruga. Ello define lo que se
simple, ya que lo que ms disminuye el riesgo de recidiva ulcerosa es conoce como sndrome postcolecistectoma.
la erradicacin del Helicobacter pylori. Sin embargo, aunque la ma- Este cuadro obedece frecuentemente a que la colecistectoma no
yora de las lceras duodenales estn relacionadas con Helicobacter controla la sintomatologa previa por no ser la colelitiasis culpable del
pylori, si se descarta su implicacin est indicado realizar una tcnica cuadro. Sin embargo, hay otras causas de sndrome postcolecistecto-
definitiva (vagotoma troncular y piloroplastia) para prevenir la recidi- ma, morfolgicas o funcionales, que precisan tratamientos especficos.
va, aunque esta situacin es excepcional. En concreto, nuestra paciente presenta posteriormente un cuadro
Las lceras slo se resecan en caso de sospecha de malignidad, lo de dolor en hipocondrio derecho, fiebre en picos e ictericia, caracte-
cual sucede en lceras gstricas (en este paciente hablamos esencial- rstica trada de Charcot de la colangitis. En este contexto, la causa ms
mente de una duodenal) de evolucin trpida. probable es una coledocolitiasis residual (aunque a veces se puede
formar una litiasis coledociana de novo) que obstruye la va biliar y
Pregunta 63.- R: 4 sta se ha sobreinfectado.
Ya hemos comentado que la cuarta parte de los perforados no El tratamiento de urgencia de la colangitis, en principio es siempre
presentan neumoperitoneo en la radiologa simple. conservador: antibiticos i.v. y control sintomtico, programando una
Las complicaciones de la lcera pptica son, por este orden, he- CPRE para intentar extraer el clculo obstructivo y hacer una esfinte-
morragia, perforacin y estenosis pilrica. Suelen aparecer aislada- rotoma en previsin de episodios futuros. En el caso del paciente
mente, de modo que es poco probable que una perforacin sangre y colecistectomizado, la CPRE encuentra su mejor y ms brillante indi-
excepcional que cause un shock hipovolmico. cacin, dado que es por s misma totalmente resolutiva del cuadro, sin
La ltima opcin es falsa porque el tratamiento mdico actual para precisar ningn gesto adicional (dado que ya no hay vescula que
la lcera pptica (inhibidores de la bomba y terapia erradicadora) es extirpar).
altamente efectivo, por lo que la ciruga programada de la lcera (por
lcera refractaria a tratamiento) se ha convertido en una situacin Pregunta 71.- R: 1
prcticamente inexistente. La casi totalidad de las cirugas por lcera La causa ms frecuente de obstruccin intestinal es la obstruccin
que se realizan desde los aos 90 son por complicaciones agudas de delgado, sobre todo por bridas o adherencias postquirrgicas.
(sobre todo perforaciones, dado que las hemorragias, aunque ms Cuando se nos especifique que un paciente no ha sido operado
frecuentes, se suelen controlar bien endoscpicamente). nunca del abdomen, deberemos buscar como causa ms frecuente
de obstruccin una hernia complicada. Por lo tanto, la sola explora-

Comentarios TEST
Pregunta 64.- R: 5 cin abdominal rutinaria (inspeccin del abdomen en busca de lapa-
Ver comentario nmero 66. rotomas y exploracin de los orificios herniarios) nos puede poner
en la pista de las 2 causas ms frecuentes de obstruccin intestinal.
Pregunta 65.- R: 3 Por otra parte, la obstruccin ms frecuente del colon es debida a
Ver comentario nmero 66. cncer de colon, especialmente en localizacin recto-sigma. Una pro-
porcin respetable de estos tumores son lo suficientemente bajos como
Pregunta 66.- R: 4 para ser accesibles al tacto rectal.
Se nos presenta el caso de una paciente con antecedentes de cli- Por ello, con estas tres exploraciones baratas y sencillas, podemos
cos de repeticin, que presenta lo que inicialmente parece un nuevo ponernos en la pista de la causa de la obstruccin. Por otra parte, no
clico. Sin embargo, la enferma no mejora con el tratamiento sinto- debemos olvidar que todas ellas pertenecen al examen fsico, y que
mtico (lo cual debe hacernos sospechar una complicacin), apare- ste siempre debe preceder a las pruebas complementarias, de modo
ciendo clnica de obstruccin intestinal. Esta secuencia de aconteci- que en buena praxis las dems opciones deben ser rechazadas.
mientos (clico biliar seguido de obstruccin intestinal) es caracters-
tica del leo biliar. Pregunta 72.- R: 5
Normalmente el leo biliar aparece en el contexto de una colecis- Ya hemos comentado previamente cules deben ser las preguntas
titis, aunque no siempre es as. Se forma una fstula biliodigestiva (lo que nos haremos ante un dolor abdominal para decidir la actitud a
ms frecuente es que sea colecistoduodenal), a travs de la cual pasan seguir (pregunta 49). Vemoslo aplicado a este caso:
uno o varios clculos al tubo digestivo, obstruyndolo si son de gran 1) Tiene el/la paciente un abdomen agudo? Evidentemente s, pues-
volumen; esta obstruccin sucede ms frecuentemente en la unin to que hay un dolor abdominal de inicio agudo acompaado de
ileocecal, por ser zona de paso ms estrecho. signos de irritacin peritoneal (Blumberg en este caso).
Radiolgicamente, la aerobilia aparece por pasar aire del tubo 2) Hay datos objetivos de gravedad? (de sepsis o peritonitis generali-
digestivo a las vas biliares, la dilatacin de asas de intestino delgado y zada) NO hay datos de gravedad en este caso que apremien a una
los niveles hidroareos son signos de obstruccin, y el clculo puede ciruga urgente.
verse en leon terminal si es radioopaco.

M exico A rgentina CTO Medicina C/ Nez de Balboa, 115 28006 MADRID (Espaa) Tfno.: (91) 782 43 32 / Fax: (91) 782 43 27
Pg. 10 CG C hile U ruguay E-mail: secretaria@ctomedicina.com; iberocto@ctomedicina.com WEB: www.ctomedicina.com; www.iberocto.com
CIRUGA GENERAL
Preparacin Examen de Seleccin 05/06 1 Vuelta Seguimiento a distancia
3) Tengo un diagnstico claro? Tengo un dolor en fosa ilaca dere-
cha, pero ningn diagnstico claro. Hay una historia de endome-
triosis previa, est justo a la mitad del ciclo... Puede ser nuevamente
endometriosis, simple rotura folicular o folculo hemorrgico, apen-
dicitis, EIP, embarazo ectpico....
Dado que tengo una paciente sin un diagnstico claro y sin datos
de gravedad, parece muy precipitado someterla a una ciruga de la
que va a obtener un muy dudoso beneficio, por lo cual la actitud
menos adecuada es sin duda esa. Cualquiera de las otras opciones es
correcta, dado que busca aclarar un poco el cuadro de la paciente
(mediante pruebas complementarias u observacin clnica).

Pregunta 73.- R: 1
La vscera ms frecuentemente lesionada en los traumatismos ab-
dominales cerrados es el bazo. Su rotura origina un hemoperitoneo
que con frecuencia est en el origen de la inestabilidad hemodinmi-
ca que pueden presentar estos pacientes. Sin embargo, la inestabili-
dad hemodinmica en el paciente con traumatismo cerrado no es
indicacin de ciruga: siempre se debe comprobar mediante una
prueba (ecografa o lavado peritoneal) que el foco de sangrado es Pregunta 74. A la izquierda: laparoscopia; a la derecha laparotoma.
intraperitoneal, porque un paciente puede presentarse tras traumatis-
mo cerrado con un shock hipovolmico secundario a una fractura de Pregunta 75.- R: 2
pelvis o fmur o a una lesin torcica. Como norma general, el accidente de trfico es el prototipo del
La siguiente vscera lesionada por orden de frecuencia en el trau- traumatismo abdominal cerrado, pero por ser politraumatizado se
ma cerrado es el hgado. debe de seguir el protocolo ABCDE antes de hacer el balance de
lesiones. El paciente estaba algo hipotenso al recogerlo, pero ha que-
dado estable con los 700 cc del transporte. Por otra parte, la A y la B
estn aseguradas, dado que el paciente viene intubado. De manera
que tenemos un ABC resuelto y nos centramos en D y E. La explora-
cin neurolgica no es valorable en un paciente que ha sido sedado
y relajado para la intubacin, por lo que no podemos afirmar nada
en relacin con lesiones del SNC. Sin embargo, en el contexto de un
politraumatismo importante, la TC craneal es imperativa. El hecho de
que exista dolor abdominal justifica en s mismo la realizacin de una
prueba de imagen en abdomen, que dada la estabilidad del paciente
ser una TC.
Comentarios TEST

Pregunta 73. Manejo de un traumatismo abdominal.

Pregunta 74.- R: 2
La actitud en la herida por arma blanca abdominal no viene deter-
minada tanto por la estabilidad del enfermo (como sucede en el trauma
cerrado) como por la penetracin en peritoneo. Si hay datos directos o
indirectos de penetracin en peritoneo (y, ciertamente, la inestabilidad
es un dato indirecto de penetracin, aunque no el nico posible),
entonces hay indicacin de exploracin quirrgica. Es el caso que nos
ocupa, la evisceracin nos indica la penetracin de forma inequvoca.
Si hay certeza de no penetracin, lo indicado es la observacin.
Sin embargo, en muchos casos no tenemos evidencia de penetra-
cin, pero no podemos descartarla. En estas situaciones se puede
realizar un lavado peritoneal o una laparoscopia diagnstica para
intentar aclarar la existencia de lesiones intraabdominales. Algunos
autores defienden la observacin clnica estrecha con exploracin
repetida por un mismo cirujano, en busca de signos de irritacin
peritoneal, pero esta postura no ha sido an aceptada de forma uni-
versal.

M exico A rgentina CTO Medicina C/ Nez de Balboa, 115 28006 MADRID (Espaa) Tfno.: (91) 782 43 32 / Fax: (91) 782 43 27
C hile U ruguay E-mail: secretaria@ctomedicina.com; iberocto@ctomedicina.com WEB: www.ctomedicina.com; www.iberocto.com
EP Pg. CG
ENDOCRINOLOGA
Preparacin Examen de Seleccin 05/06 1 Vuelta Seguimiento a distancia

HIPOTLAMO-HIPFISIS. 2) Se puede asociar a HTA, diabetes mellitus e hiperprolac-


tinemia.
1. En todas las siguientes patologas hipotlamo-hipofisarias 3) La prueba de laboratorio diagnstica es la determinacin
existir hiperprolactinemia, EXCEPTO en: basal de GH en suero.
4) La hipercalcemia y la hipercalciuria, con litiasis renal, son
1) Sndrome de Nelson. frecuentes.
2) Hipofisitis linfocitaria. 5) La hipercalcemia asociada sugiere un MEN 2a.
3) Craneofaringioma.
4) Adenoma cromfobo no funcionante, de 3 cm de dimetro. 6. En un paciente acromeglico portador de un macroadenoma,
5) Sndrome de Sheehan. tras ser operado no se produce curacin. La resonancia mag-
ntica realizada despus de la intervencin no muestra la
2. Todos los siguientes datos nos deben hacer pensar en el existencia de restos de adenoma; el paciente se niega a la
diagnstico de prolactinoma, EXCEPTO uno: radioterapia. Usted qu actitud teraputica le recomendara?:

1) Mujer con amenorrea secundaria y prolactina basal mayor 1) Bromocriptina.


de 300 microg/L. 2) Octretide.
2) Embarazada con prolactinemia de 400 microg/L. 3) Lisuride.
3) Mujer con amenorrea secundaria, test de embarazo ne- 4) Reintervencin con hipofisectoma, aunque no se visua-
gativo y prolactinemia de 160 microg/L. lice lesin.
4) Mujer con alteraciones menstruales, prolactina de 75 5) Cateterismo de los senos petrosos para determinar el
microg/L y adenoma hipofisario de 23 mm. origen hipofisario y reintervencin.
5) Amenorrea primaria, prolactinemia de 200 microg/L y RM
selar sin alteraciones. 7. En relacin a la acromegalia, es FALSO:

3. Varn de 40 aos, sin antecedentes clnicos ni farmacolgi- 1) Se habla de curacin cuando las cifras de IGF-1 son
cos de inters. Acude por disfuncin sexual, sin otra manifes- normales para sexo y edad y la GH es inferior a 2 ng/dl tras
tacin clnica. Niveles sricos de prolactina: 330 microg/L. En SOG por RIA.
la TC craneal se observa un macroadenoma de 1,5 cm, 2) Cuando se realiza el diagnstico de la enfermedad, se
limitado a la silla turca. La actitud teraputica a seguir es: debe descartar la existencia de poliposis colnica.
3) Cuando se alcanzan los criterios de curacin, la morbi-
1) No precisa tratamiento. mortalidad se equipara a la de la poblacin general.
2) Ciruga transesfenoidal. 4) Puede existir elevacin de GH, tras estmulo con TRH.
3) Ciruga combinada con bromocriptina, con o sin radio- 5) Aunque la dopamina en condiciones normales produzca
terapia. elevacin en las cifras de GH, la bromocriptina puede
4) Cabergolina, que es un agonista dopaminrgico. resultar til en el tratamiento.
5) Antagonistas de DA, como bromocriptina.
8. Paciente de 48 aos, que acude a nuestra consulta por dismi-
4. Una paciente de 25 aos, diagnosticada de hiperprolactine- nucin de la libido, impotencia, astenia, nuseas y vmitos,
mia idioptica, sin tratamiento ni sintomatologa en la actua- letargia, piel seca, estreimiento. En la analtica de rutina desta-
lidad, salvo la existencia de menstruaciones irregulares de ca: anemia (Hb: 11 g/dl), colesterol normal, iones normales. En
forma ocasional, expresa en una visita rutinaria deseo de estudio de imagen: RM se observa una lesin hipofisaria peque-
embarazo. Cul de las siguientes actitudes teraputicas sera a (<1cm). Cul ser el diagnstico MENOS probable?:
la ms apropiada?:
1) Microprolactinoma.
1) No tratar, y cuando se produzca el embarazo, administrar 2) Sarcoidosis.
dosis bajas de bromocriptina, para mantener cifras de 3) Infarto hipofisario.
Preguntas TEST

prolactina dentro de la normalidad. 4) Tuberculosis.


2) Iniciar tratamiento con dosis bajas de bromocriptina, y 5) Carcinoma metastsico.
cuando se produzca el embarazo, aumentar la dosis hasta
mantener cifras de prolactina dentro de la normalidad. 9. Varn de 30 aos, que acude al servicio de urgencias por
3) Iniciar tratamiento con dosis bajas de bromocriptina, y cefalea de inicio brusco, muy dolorosa, con oftalmoparesia,
cuando se produzca el embarazo, suspender la medica- pupilas midriticas, TC normal y LCR sin alteraciones. A usted
cin, realizando controles analticos y campimtricos le parece correcto:
peridicamente.
4) Iniciar tratamiento con cabergolina, y cuando se produz- 1) Podra tratarse de una apopleja hipofisaria y estara
ca el embarazo, mantener dosis mnimas hasta mantener indicada una RM urgente.
cifras de prolactina dentro de la normalidad. 2) Este cuadro es muy frecuente en la clnica de debut de un
5) Mantener sin ningn tipo de tratamiento, antes y durante adenoma no funcionante de hipfisis.
el embarazo. 3) Probablemente exista otra patologa neurolgica concomi-
tante, ya que la afectacin de oculomotores nunca se produce.
5. Referente a la acromegalia, es cierto que: 4) Se denomina tambin sndrome de Sheehan.
5) En la histologa se demostrara un infiltrado de clulas
1) El estudio anatomopatolgico postquirrgico determina mononucleares en la hipfisis, y por ello se encuadra
si se trata de adenoma o carcinoma. dentro del sndrome de hipofisitis linfoctica.

M exico A rgentina CTO Medicina C/ Nez de Balboa, 115 28006 MADRID (Espaa) Tfno.: (91) 782 43 32 / Fax: (91) 782 43 27
C hile U ruguay E-mail: secretaria@ctomedicina.com; iberocto@ctomedicina.com WEB: www.ctomedicina.com; www.iberocto.com
ED Pg. 1
ENDOCRINOLOGA
Seguimiento a distancia Preparacin Examen de Seleccin 05/06 1 Vuelta
10. Cul de las siguientes afirmaciones, en relacin a un paciente 14. Cul de estas situaciones NO justifica una diabetes inspida
intervenido de un macroadenoma hipofisario no secretor, es nefrognica?:
FALSA?:
1) Hipotiroidismo.
1) Si presentara dficit de TSH y ACTH, se debera iniciar 2) Tratamiento con litio.
tratamiento sustitutivo, primero con glucocorticoides y 3) Uropata obstructiva.
despus con levotiroxina. 4) Aldosteronismo primario.
2) Para descartar inmediatamente despus de la ciruga el 5) Nefritis pierde sal.
dficit de ACTH, la prueba de eleccin sera administrar
ACTH i.v. y medir la respuesta del cortisol plasmtico. 15. Cul de los siguientes datos NO orienta hacia un SIADH en
3) Si persistieran restos tumorales tras la ciruga, se planteara un paciente con disminucin del nivel de conciencia?:
la posibilidad de administrar radioterapia.
4) Si existiera dficit de TSH, el paciente no presentara 1) Agitacin.
hipercolesterolemia. 2) Osm (orina): 350.
5) Inmediatamente despus de la ciruga se debe controlar 3) Edemas maleolares.
4) Natremia 120.
de forma estricta la diuresis.
5) Ac. rico: 2,5.
11. Una paciente de 30 aos acude a nuestra consulta por
16. Varn diagnosticado de ca. pulmonar microctico, asintom-
amenorrea, piel seca, intolerancia al fro, ganancia de peso,
tico. Exploracin, salvo la auscultacin pulmonar, normal.
astenia. Como antecedentes destacan: DM tipo 1, incapaci-
Natremia: 130. Natriuria: 25. Osm (sangre): 270. Osm (ori-
dad para la lactancia tras un parto, hace 6 meses. Seale la
na): 300. Qu tratamiento inicial est indicado?:
respuesta FALSA:
1) Restriccin hdrica a 500-1.000 cc/da.
1) Sera til realizar una prueba de hipoglucemia insulnica 2) Furosemida y reposicin hidroelectroltica con salino al
para valorar posible dficit de GH y ACTH. 0,9% y ClK.
2) Es probable que presente un panhipopituitarismo secun- 3) Demeclociclina.
dario a un infarto hipofisario postparto. 4) No precisa tratamiento.
3) En la exploracin fsica debe presentar hiperpigmentacin. 5) Desmopresina s.c. o intranasal.
4) Debe de tratarse con levotiroxina y con hidrocortisona.
5) El dficit de prolactina suele ser la manifestacin inicial tras
el parto. PATOLOGA TIROIDEA.

12. Un paciente de 68 aos, diagnosticado hace 25 aos de 17. Respecto a las acciones del yodo sobre el tiroides, la nica
sndrome de Cushing e intervenido mediante suprarrenalec- afirmacin correcta es:
toma bilateral, se encuentra en tratamiento con hidrocorti-
sona (30 mg/da). En la revisin se objetiva hiperpigmenta- 1) Inhibe de forma aguda y transitoria la peroxidasa tiroidea
cin cutnea intensa. Cul sera su sospecha diagnstica (efecto Jod-Basedow).
ms probable?: 2) Administrado en dosis elevadas de forma aguda, puede
producir en pacientes con bocio simple un hipertiroidis-
1) No seguimiento del tratamiento con hidrocortisona. mo con gammagrafa hipocaptante.
2) Sndrome de Nelson. 3) El bocio inducido por yoduro se acompaa siempre de
3) Falta de absorcin de los glucocorticoides. hipotiroidismo.
4) Porfiria cutnea. 4) Los pacientes con enfermedad autoinmune son especial-
5) Recidiva del sndrome de Cushing, posiblemente ectpico. mente resistentes al efecto bloqueante del yodo.
5) La utilizacin preoperatoria de yodo en los bocios hiper-

Preguntas TEST
13. Paciente de 43 aos, sin antecedentes de inters, que funcionantes est contraindicada, ya que puede aumen-
consulta por polidipsia y poliuria (6-7 litros/da) en las seis tar la vascularizacin de la glndula y producir graves
semanas previas. El hemograma y una bioqumica bsica hemorragias intraoperatorias.
son normales. Osm (sangre): 290; osm(orina): 100. La
osmolaridad urinaria no se modifica significativamente tras 18. Respecto a la fisiologa tiroidea, sealar la respuesta correcta:
deshidratacin. Se administran 2 microg. de DDAVP, obser-
1) La tiroglobulina es la protena que transporta mayoritaria-
vndose que la osmolaridad en orina aumenta a 370. Seale
mente T3 y T4.
la verdadera:
2) El transporte del yoduro desde el plasma hasta el interior de
las clulas foliculares es un transporte de membrana pasivo.
1) El diagnstico de diabetes inspida no queda demostrado
3) La oxidacin del yoduro se realiza por la yodotirosina
con estos datos.
deshalogenasa tiroidea.
2) La misma respuesta se obtiene en la polidipsia primaria. 4) Las sustancias precursoras monoyodotirosina (MYT) y
3) Est indicada la medicin de ADH en plasma u orina. diyodotirosina (DYT) son liberadas de la tiroglobulina y
4) Podra tratarse con sueroterapia con 2.500 cc de salino sufren una reaccin de acoplamiento por la peroxidasa
0,9% i.v. al da, dieta absoluta y ADH nativa. dentro del coloide folicular.
5) Podra tratarse con desmopresina. 5) El tiroides es la nica fuente de T4 endgena.

M exico A rgentina CTO Medicina C/ Nez de Balboa, 115 28006 MADRID (Espaa) Tfno.: (91) 782 43 32 / Fax: (91) 782 43 27
Pg. 2 ED C hile U ruguay E-mail: secretaria@ctomedicina.com; iberocto@ctomedicina.com WEB: www.ctomedicina.com; www.iberocto.com
ENDOCRINOLOGA
Preparacin Examen de Seleccin 05/06 1 Vuelta Seguimiento a distancia

19. La conversin perifrica de T4 a T3 NO disminuye en caso de: hipotiroidismo primario idioptico. Est en tratamiento con
100 microg/da de levotiroxina y aporta una analtica reciente
1) Ayuno y desnutricin. con los siguientes datos hormonales: T4 libre 1,8 ng/dl (N:
2) Gran traumatismo. 0,5-2,4) y TSH 0,01 mcU/ml (N: 0,5-4). Sealar la respuesta
3) Tratamiento con carbimazol. correcta:
4) Tratamiento con propranolol.
5) Contrastes yodados. 1) Habr que solicitar anticuerpos antimicrosomales para
descartar una tiroiditis crnica autoinmune.
20. Los anticuerpos antimicrosomales y antitiroglobulina: 2) Debemos disminuir la dosis de tiroxina.
3) Se puede mantener el mismo tratamiento y reevaluar cada
1) Son especficos de la enfermedad de Graves. 6 meses.
2) Son especficos de la enfermedad de Hashimoto. 4) Debemos aumentar la dosis sustitutiva hasta que T4 libre
3) Pueden existir en el hipotiroidismo primario, en la enfer- alcance niveles superiores al nivel mximo, para asegurar-
medad de Graves y en la enfermedad de Hashimoto. nos una adecuada respuesta tisular.
4) Inhiben la unin de TSH al receptor. 5) Es necesario asociar esteroides, por si se tratara de un
5) Producen enfermedad de Graves neonatal. sndrome poliglandular autoinmune.

21. Paciente que acude por aumento de tamao tiroideo, disnea 26. Una mujer de 54 aos, sin antecedentes de inters, consulta
de esfuerzo y pltora facial con sncope al elevar los brazos. por ganancia de 5 kg de peso en el ltimo ao. No refiere otros
Presenta hormonas tiroideas y TSH normales. La actitud ms sntomas. Sus anlisis generales son normales y sus hormonas
adecuada es la siguiente: tiroideas: T4 libre 1,0 ng/dl, TSH 6,1 mcU/ml. Seale la
correcta:
1) Radioyodo.
2) Antitiroideos. 1) Se trata de un hipotiroidismo primario, se debe iniciar
3) Tiroidectoma. tratamiento con levotiroxina.
4) Actitud expectante. 2) Se trata de un hipotiroidismo secundario, se debe iniciar
5) Aumento de ingesta de yodo en la dieta. tratamiento con levotiroxina.
3) Se trata de un hipotiroidismo subclnico, se debe iniciar
22. Sealar la respuesta correcta respecto a la patologa tiroidea: tratamiento con levotiroxina.
4) Se trata de un hipotiroidismo subclnico, se puede vigilar
1) El hipotiroidismo se asocia siempre a disminucin del sin ningn tratamiento.
tamao glandular tiroideo. 5) No presenta ninguna patologa tiroidea, no precisa ningn
2) El hipertiroidismo se asocia siempre a aumento del tama- tratamiento.
o glandular tiroideo.
3) El bocio se asocia con aumento, normalidad o disminu- 27. Sobre el tratamiento del coma mixedematoso, seale la
cin de la secrecin hormonal. FALSA:
4) El cncer de tiroides (variante folicular) se asocia con
frecuencia a tirotoxicosis. 1) El tratamiento de eleccin es T4 va i.v.
5) Cuando el tiroides lingual es el nico tejido tiroideo 2) Debe administrarse tambin hidrocortisona.
funcionante, su secrecin suele ser suficiente para man- 3) Si no se dispone de T4-i.v. se puede emplear T3 en SNG.
tener un estado eutiroideo. 4) Tiene buen pronstico.
5) Se inicia el tratamiento, a pesar de no tener confirmacin
23. Son pruebas diagnsticas tiles en el bocio simple todas, analtica del diagnstico.
MENOS una:
28. En una mujer con tirotoxicosis por el consumo de ciertos
Preguntas TEST

1) Ecografa tiroidea. productos para adelgazar que contienen T3, qu NO espe-


2) Exploracin cervical. rara encontrar?:
3) Radiografa trax.
4) Determinacin de tiroglobulina, seguida de rastreo cor- 1) Tiroglobulina normal o disminuida.
poral total con I-131. 2) Captacin de yodo disminuida.
5) TC cervicotorcico. 3) T3 y T4 elevadas.
4) Signos clnicos de tirotoxicosis.
24. NO se encuentra entre las causas de bocio: 5) Ausencia de bocio.

1) Amiodarona. 29. De las siguientes, seale cul NO produce un hipertiroidismo


2) Litio. con gammagrafa silente:
3) Tiroiditis de Hashimoto.
4) Yoduro de forma continuada en pacientes susceptibles. 1) Tirotoxicosis facticia.
5) Propranolol. 2) Coriocarcinoma.
3) Struma ovarii.
25. Una mujer de 34 aos acude a nuestra consulta para revisin 4) Tirotoxicosis por hamburguesas.
con un informe mdico en el que aparece el diagnstico de 5) Tiroiditis con tirotoxicosis transitoria.

M exico A rgentina CTO Medicina C/ Nez de Balboa, 115 28006 MADRID (Espaa) Tfno.: (91) 782 43 32 / Fax: (91) 782 43 27
C hile U ruguay E-mail: secretaria@ctomedicina.com; iberocto@ctomedicina.com WEB: www.ctomedicina.com; www.iberocto.com
ED Pg. 3
ENDOCRINOLOGA
Seguimiento a distancia Preparacin Examen de Seleccin 05/06 1 Vuelta
30. Mujer de 28 aos, que acude a su consulta por presentar, 1) Carcinoma papilar.
desde hace tres meses, aumento del cuello, sudacin profusa, 2) Carcinoma folicular.
prdida de peso, diarrea, hiperfagia, palpitaciones y "ojos 3) Adenoma o quiste.
saltones". Usted NO hara: 4) Bocio txico multinodular.
5) Metstasis.
1) Pensar como primera opcin en una enfermedad de
Graves y solicitar TSH, T4 libre y TSI. 36. Mujer de 40 aos, sin antecedentes de inters, que presenta
2) Tambin pedira una gammagrafa tiroidea como parte ndulo tiroideo. PAAF: no concluyente. Cul es la actitud
del estudio. ms indicada?:
3) Le comentar a la paciente que, si es una enfermedad de
Graves, existe un tratamiento que se debe mantener unos 1) Hormona tiroidea en dosis supresoras y repeticin de la
2 aos. PAAF a los 3-6 meses.
4) Tambin le comentar que, a medida que mejore la 2) Biopsia tiroidea.
enfermedad del tiroides, tambin lo har el problema 3) Tiroidectoma subtotal.
ocular. 4) Antitiroideos y repeticin de la PAAF a los 3-6 meses.
5) Tambin esperara encontrar anticuerpos anti-TG y anti- 5) Yodo radiactivo en altas dosis.
TPO.
37. En el tratamiento del cncer diferenciado de tiroides, cul
31. NO es contraindicacin al tratamiento de la enfermedad de de los siguientes NO est indicado?:
Graves con yodo radiactivo:
1) Tiroidectoma total.
1) Embarazo. 2) Linfadenectoma cervical radical, aunque no exista afec-
2) Nio.
tacin ganglionar macroscpica.
3) Anciano.
3) Tratamiento supresor con hormona tiroidea.
4) Bocio muy voluminoso retroesternal.
4) Tratamiento con yodo-131, para ablacin de restos tiroi-
5) Ndulo fro en gammagrafa tiroidea.
deos tras la ciruga.
5) Tratamiento con yodo-131 de las metstasis funcionantes.
32. Mujer embarazada, diagnosticada de enfermedad de Graves.
Seale la respuesta FALSA:
38. Sobre el carcinoma medular de tiroides, es FALSO:
1) Los frmacos antitiroideos pueden producir hipotiroidis-
1) Puede no ser palpable.
mo fetal.
2) Los IgG estimuladores del tiroides pueden provocar hiper- 2) Se trata con tiroidectoma total.
tiroidismo fetal. 3) Secreta calcitonina y, a veces, otras sustancias.
3) El yodo radiactivo est siempre contraindicado. 4) Puede transmitirse de forma autosmica dominante.
4) La ciruga est siempre contraindicada. 5) Las metstasis se tratan con yodo-131.
5) El propranolol est contraindicado.
39. Una paciente de 42 aos presenta desde hace 3 un ndulo
33. Mujer de 75 aos, diagnosticada de enfermedad de Graves, tiroideo derecho fro. La PAAF inicial fue benigna. Desde
insuficiencia cardaca con disnea de reposo y angor inesta- entonces realiza tratamiento supresor con levotiroxina. En la
ble. Qu tratamiento inicial NO est indicado?: revisin actual, el ndulo ha crecido significativamente. Una
nueva PAAF sigue reflejando benignidad. Cul le parece la
1) Yodo radiactivo. actitud ms correcta?:
2) Propiltiouracilo.
3) Yoduro. 1) Aumentar la dosis de levotiroxina.

Preguntas TEST
4) Dexametasona. 2) Remitir a ciruga para hemitiroidectoma.
5) Ipodato sdico. 3) Remitir a ciruga para tiroidectoma total.
4) Suspender la levotiroxina.
5) Continuar el tratamiento supresor y revisar anualmente
NDULO Y CNCER DE TIROIDES. con ecografa.

34. Ante un ndulo tiroideo, cul es la primera prueba que se 40. En un paciente con un ndulo tiroideo, la actitud a seguir
debe realizar?: segn el resultado de la PAAF es la siguiente:

1) Gammagrafa tiroidea, para clasificar el ndulo en fro o 1) Con citologa maligna, intentar tratamiento con levotiro-
caliente. xina para reducir el tamao tumoral antes de la ciruga.
2) Puncin-aspiracin con aguja fina. 2) Ciruga siempre, si la citologa es no concluyente.
3) TSH, T3 y T4. 3) Iniciar tratamiento con levotiroxina, si la citologa es
4) Biopsia. benigna, y vigilar.
5) Captacin de yodo radiactivo. 4) Repetir la puncin si se objetiva abundante proliferacin
folicular.
35. Ante un ndulo tiroideo fro, lo ms probable es: 5) Ciruga siempre, aunque sea benigno.

M exico A rgentina CTO Medicina C/ Nez de Balboa, 115 28006 MADRID (Espaa) Tfno.: (91) 782 43 32 / Fax: (91) 782 43 27
Pg. 4 ED C hile U ruguay E-mail: secretaria@ctomedicina.com; iberocto@ctomedicina.com WEB: www.ctomedicina.com; www.iberocto.com
ENDOCRINOLOGA
Preparacin Examen de Seleccin 05/06 1 Vuelta Seguimiento a distancia

41. Una paciente, diagnosticada de carcinoma papilar de tiroi- 46. En un paciente con sndrome de Cushing por administracin
des, fue tratada con tiroidectoma total y una dosis postope- exgena de esteroides, NO es esperable encontrar:
ratoria de 50 mCi de I-131 para ablacin de restos ectpicos,
sin evidenciarse metstasis en el rastreo corporal. A los 6 1) Hbito cushingoide.
meses, el rastreo de seguridad es negativo. A los 12 meses de 2) Cortisol en sangre y orina disminuido.
la operacin, sin embargo, se detectan cifras elevadas de 3) Hipopotasemia.
tiroglobulina. La actitud teraputica ser: 4) Hiperpigmentacin.
5) Hiperlipoproteinemia (LDL).
1) Aumentar la dosis supresora de tiroxina.
2) Esperar 6 meses y repetir determinacin de tiroglobulina. 47. La causa ms frecuente de sndrome de Cushing por exceso
3) Realizar nuevo rastreo corporal, para detectar la existencia de ACTH es:
de restos tiroideos o extratiroideos hipercaptantes.
4) Realizar una TC de cuerpo entero, como primera medida 1) Secrecin de ACTH por oat-cell.
para detectar metstasis. 2) Hiperplasia nodular suprarrenal.
5) La elevacin de la tiroglobulina en los pacientes que 3) Microadenoma hipofisario (<1cm).
siguen tratamiento supresor con L-T4 no indica, en la 4) Macroadenoma hipofisario (>1cm).
mayora de los casos, la existencia de metstasis. 5) Hiperplasia difusa de clulas corticotropas.

42. Son datos de sospecha de malignidad de un ndulo tiroideo 48. Varn de 57 aos que consulta por hemoptisis. En urgencias
todos, MENOS uno de los siguientes: se encuentra afebril. TA: 120/60. Hb: 14,5, Hto: 54,5, leuco-
citos 10.000 (frmula normal), plaquetas: 250.000. La GAB
1) Varn. muestra pH: 7,55, HCO3: 38, pCO2: 40, pO2: 80. Na: 140,
2) Disfona. K: 2,8, Glucosa: 170. La radiografa de trax muestra ensan-
3) Desplazamiento del tiroides con la deglucin. chamiento hiliar e infiltrado en lbulo superior derecho. Qu
4) Ausencia de dolor. otro dato clnico esperara encontrar si sospechase un sndro-
5) Ndulo fro gammagrficamente. me de Cushing por secrecin ectpica de ACTH?:

43. Seale la INCORRECTA: 1) HTA.


2) Cara de luna llena.
1) Tiroiditis subaguda: patogenia probablemente viral, fie- 3) Obesidad troncular.
bre, dolor cervical irradiado a odos, tiroglobulina y VSG 4) Hiperpigmentacin.
altas, bocio nodular. 5) Hirsutismo.
2) Tiroiditis indolora: fase de hipertiroidismo, seguida de fase
de hipotiroidismo, y finalmente normofuncin. 49. Cul de las siguientes pruebas diagnsticas puede ser equi-
3) Tiroiditis de Riedel: tratamiento con antitiroideos en la fase valente a la prueba de despistaje con 1 mg de dexametasona
de hipertiroidismo. nocturna, en el sndrome de Cushing?:
4) Tiroiditis de Hashimoto: tratamiento con levotiroxina.
5) Tiroiditis aguda: antibiticos y drenaje quirrgico. 1) Test de ACTH.
2) Cortisol plasmtico.
44. Paciente con aumento del tamao tiroideo, indoloro, de 3) Cortisol libre urinario (24 horas).
consistencia ptrea, con disnea y disfagia. Seala la FALSA: 4) Ritmo de cortisol (8:00-23:00).
5) ACTH.
1) Si se evidencia fibrosis en la histologa, puede asociarse a
hipotiroidismo en un 25% de los casos.
50. Con respecto a la prueba definitiva con dexametasona (DXM)
2) Est indicada la ciruga para descomprimir.
Preguntas TEST

para el diagnstico de sndrome de Cushing, seale la correcta:


3) Puede asociar fibrosis mediastnica y retroperitoneal.
4) Habra que descartar un carcinoma anaplsico de tiroi-
1) No tiene falsos positivos.
des.
2) DXM, 0,5 mg/6 h, durante 2 das.
5) El diagnstico ms probable es una tiroiditis subaguda, si
3) Permite el diagnstico diferencial entre microadenoma
la VSG est aumentada.
hipofisario y neoplasia suprarrenal.
4) DXM, 8 mg/da, durante 2 das.
5) Consigue suprimir el cortisol al 50% en caso de microade-
PATOLOGA SUPRARRENAL.
noma hipofisario.
45. La causa ms frecuente de sndrome de Cushing es:
51. Mujer de 35 aos, con obesidad troncular, cara de luna
llena, estras rojo-vinosas en abdomen, hirsutismo, HTA y
1) Microadenoma hipofisario (enfermedad de Cushing).
diabetes. Presenta cortisoluria de 24 horas alta. El cortisol no
2) Adenoma suprarrenal.
suprime con 1 mg de DXM ni con 0,5mg/6h durante 2 das.
3) Adrenalitis autoinmune.
ACTH est alta. Cortisoluria suprime ms de un 90% con 8 mg/
4) Paraneoplsico.
da de DXM. Cul es el diagnstico ms probable y qu
5) Esteroides exgenos.
prueba de imagen haras en primer lugar?:

M exico A rgentina CTO Medicina C/ Nez de Balboa, 115 28006 MADRID (Espaa) Tfno.: (91) 782 43 32 / Fax: (91) 782 43 27
C hile U ruguay E-mail: secretaria@ctomedicina.com; iberocto@ctomedicina.com WEB: www.ctomedicina.com; www.iberocto.com
ED Pg. 5
ENDOCRINOLOGA
Seguimiento a distancia Preparacin Examen de Seleccin 05/06 1 Vuelta
1) Cushing ectpico - gammagrafa con octretide. 1) Embarazo.
2) Enfermedad de Cushing - resonancia de las glndulas 2) Intolerancia al calcio oral.
suprarrenales. 3) Sndrome pluriglandular autoinmune tipo 1.
3) Cushing ectpico - TC toracoabdominal. 4) Sndrome pluriglandular autoinmune tipo 2.
4) Enfermedad de Cushing - resonancia hipofisaria. 5) Iniciara tratamiento con T4 intravenosa antes de plantear
5) Carcinoma suprarrenal - resonancia abdominal. cualquier diagnstico, puesto que es una urgencia endo-
crinolgica.
52. En el tratamiento del sndrome de Cushing, se incluyen todos,
MENOS uno de los siguientes: 58. Paciente que acude por astenia, poliuria, polidipsia y cefalea.
Exploracin normal, salvo TA: 165/110. Na:155, K:2,8,
1) Extirpacin del adenoma productor de glucocorticoides. Glu:110. Alcalosis metablica. ARP disminuida basal y tras
2) Suprarrenalectoma bilateral. restriccin de sodio. Aldosterona aumentada, tras infusin
3) Extirpacin de un tumor carcinoide productor de ACTH. de suero salino. Diagnstico ms probable:
4) Ciruga transesfenoidal.
5) Ketoconazol como tratamiento definitivo. 1) Sndrome de Liddle.
2) Aldosteronoma o sndrome de Conn.
3) Defecto de 11-beta-OH-deshidrogenasa.
53. La causa ms frecuente en la actualidad de enfermedad de
4) Adenoma productor de DOCA.
Addison es:
5) Hiperaldosteronismo secundario.
1) Tuberculosis.
59. El sndrome de hiperaldosteronismo remediable con corti-
2) Adrenalitis autoinmune. coides consiste en:
3) Asociada al VIH.
4) Hemorragia adrenal. 1) Alteracin de la 11-OH-esteroide-DH.
5) Yatrgena. 2) Un tipo de aldosteronismo secundario.
3) Exceso de consumo de regaliz.
54. La interrupcin del tratamiento crnico con corticoides es la 4) Se denomina as a los tipos de hiperplasia adrenal cong-
causa ms frecuente de insuficiencia suprarrenal aguda. nita que cursan con hipermineralcorticismo.
Qu dato NO esperara encontrar?: 5) Una alteracin gentica por la cual la aldosterona se
sintetiza en la capa fascicular regulada por el promotor de
1) Ausencia de hiperpigmentacin mucocutnea. glucocorticoides.
2) Natremia: 155 mEq/L.
3) Obesidad troncular y estras rojo-vinosas. 60. Qu sustancia se ha relacionado con el sndrome de exceso
4) Glucemia: 50 mg/dl. aparente de mineralocorticoides?:
5) Eosinofilia.
1) Alcohol.
55. Pueden ser causa de insuficiencia suprarrenal aguda todas 2) Regaliz.
las siguientes, EXCEPTO una: 3) Tabaco.
4) Herona.
1) Septicemia por meningococo. 5) Alquitrn.
2) Tratamiento con anticoagulantes.
3) Supresin tras tratamiento prolongado con AINEs. 61. NO es causa de exceso de mineralocorticoides:
4) Episodio de cardiopata isqumica en un paciente addi-
soniano. 1) Sndrome de Conn.
5) Fractura de fmur en un paciente que ha recibido trata- 2) Defecto de 21-hidroxilasa.
miento con dosis altas de esteroides en los ltimos seis meses. 3) Defecto de 11-hidroxilasa.

Preguntas TEST
4) Defecto de 17-hidroxilasa.
56. Seala la asociacin INCORRECTA: 5) Defecto de 11-beta-OH-deshidrogenasa.

1) Sndrome de Cushing: hipopotasemia, alcalosis, poliglo- 62. Ante un paciente con HTA, debilidad y calambres musculares,
bulia y linfopenia. en cuya analtica encontramos un potasio de 3,1 mEq/l, la
primera posibilidad diagnstica es:
2) Addison: hiperkaliemia, acidosis, anemia y linfocitosis.
3) Hiperaldosteronismo: hipopotasemia, alcalosis y diabe-
1) Hiperplasia suprarrenal bilateral.
tes inspida nefrognica.
2) Adenoma suprarrenal productor de aldosterona.
4) Hipoaldosteronismo: hiperpotasemia, acidosis.
3) Sndrome de Cushing ectpico.
5) Insuficiencia suprarrenal secundaria: hiperpotasemia, aci-
4) Tratamiento antihipertensivo con diurticos.
dosis, anemia, linfocitosis y ausencia de hiperpigmentacin. 5) Ingesta de regaliz.
57. Mujer de 20 aos, en tratamiento con calcio por hipocalcemia 63. En el tratamiento del hiperaldosteronismo NO suele em-
desde hace aos, de la que no recuerda la causa. Refiere astenia, plearse:
anorexia, debilidad, mareo, malestar general e hiperpigmenta-
cin de varias semanas de evolucin. Acude a Urgencias por 1) Suprarrenalectoma.
vmitos y dolor abdominal. En qu diagnostico pensara?: 2) Espironolactona.

M exico A rgentina CTO Medicina C/ Nez de Balboa, 115 28006 MADRID (Espaa) Tfno.: (91) 782 43 32 / Fax: (91) 782 43 27
Pg. 6 ED C hile U ruguay E-mail: secretaria@ctomedicina.com; iberocto@ctomedicina.com WEB: www.ctomedicina.com; www.iberocto.com
ENDOCRINOLOGA
Preparacin Examen de Seleccin 05/06 1 Vuelta Seguimiento a distancia

3) Dexametasona. 3) Feocromocitoma.
4) Antihipertensivos. 4) Adenoma suprarrenal no funcionante.
5) Furosemida. 5) Metstasis.

64. Si, en una mujer con hirsutismo usted detecta niveles muy
elevados de testosterona, lo ms probable es: METABOLISMO DEL CALCIO.

1) Hiperplasia suprarrenal congnita (inicio tardo). 71. En relacin con la hipercalcemia, una de las siguientes afir-
2) Neoplasia ovrica. maciones es FALSA:
3) Ovario poliqustico.
4) Neoplasia suprarrenal. 1) La causa ms frecuente de hipercalcemia en pacientes
5) Idioptico. ambulatorios es el hiperparatiroidismo primario.
2) La causa ms frecuente de hiperparatiroidismo primario
65. Cul es el trastorno suprarrenal ms frecuente en la infancia?: es la existencia de hiperplasia paratiroidea.
3) En la mayora de los casos, el diagnstico de hiperparati-
1) Enfermedad de Addison autoinmune. roidismo primario se realiza de forma casual al detectar en
2) Sndrome de Cushing. una analtica hipercalcemia con hipofosfatemia en perso-
3) Carcinoma suprarrenal. nas asintomticas.
4) Tumores virilizantes. 4) En el diagnstico diferencial se deben descartar frmacos
5) Hiperplasia suprarrenal congnita. y neoplasias.
5) Las neoplasias, son la causa ms frecuente de hipercalce-
66. Para descartar feocromocitoma, solicitara inicialmente: mia en pacientes hospitalizados.

1) Catecolaminas en plasma. 72. En un paciente con hiperparatiroidismo primario, cul NO


2) Prueba con glucagn. es criterio de ciruga?:
3) Prueba con fentolamina.
4) Catecolaminas libres y metanefrinas en orina de 24 h. 1) Ms de 50 aos.
5) Gammagrafa con metayodobencilguanidina - I-131. 2) Calcio total en sangre >12 mg/dl.
3) Densidad sea menor de -2DS.
67. Un enfermo diagnosticado de feocromocitoma va a ser 4) Litiasis renal recidivante.
operado. Qu tratamiento preoperatorio indicara usted?: 5) Calciuria > 400 mg/d.

1) Prednisona. 73. Mujer diagnosticada de hiperplasia paratiroidea con calce-


2) Fenoxibenzamina. mia de 12 mg/dl, a pesar del tratamiento. Antecedentes
3) Propranolol. personales de HTA mal controlada y asma. Antecedentes
4) Prazosn. familiares de cncer de tiroides. Qu actitud tomara a
5) Fentolamina. continuacin?:
68. Cul de las siguientes pruebas de imagen NO es til en el 1) Paratiroidectoma subtotal.
diagnstico de feocromocitoma?: 2) Continuar con el mismo tratamiento.
3) Asociar propranolol.
1) TAC abdominal. 4) Medir catecolaminas en plasma y orina.
2) Resonancia magntica abdominal. 5) Medir PTH.
3) Gammagrafa con MIBG.
4) Gammagrafa con I-colesterol. 74. Paciente diagnosticado de carcinoma epidermoide de pul-
5) Arteriografa abdominal.
Preguntas TEST

mn, que acude al servicio de urgencias por cuadro de


poliuria y deshidratacin leve, presentando en la analtica
69. Paciente con antecedentes de HTA (en tratamiento con
hipercalcemia de 12,5 mg/dl. Resto de parmetros bioqumi-
captopril y diurticos, manteniendo TA alrededor de 170/
cos dentro de la normalidad. Seale la respuesta FALSA:
110 mmHg). Casualmente, se encuentra una masa suprarre-
nal en una ecografa abdominal. Qu actitud NO tomara?:
1) Debe recibir tratamiento con suero salino isotnico.
2) Los niveles de vitamina D sern normales o algo bajos.
1) PAAF de la masa con control ecogrfico.
3) La eficacia del tratamiento ser menor que en las hipercal-
2) Cortisol libre urinario.
cemias secundarias a adenoma paratiroideo.
3) Catecolaminas y sus metabolitos en orina.
4) Las cifras de PTH intacta estarn elevadas.
4) 17-cetosteroides en orina.
5) En el tratamiento pueden ser de utilidad bifosfonatos,
5) TC abdominal.
calcitonina y glucocorticoides.
70. La mayora de los incidentalomas suprarrenales se corres-
75. Paciente diagnosticado de mieloma mltiple e insuficiencia
ponden con:
renal crnica, que acude al servicio de urgencias por cuadro
progresivo de astenia, debilidad de la musculatura proximal,
1) Carcinoma suprarrenal.
letargia y oligoanuria. En la analtica presenta: creatinina 4
2) Adenoma suprarrenal productor de aldosterona.

M exico A rgentina CTO Medicina C/ Nez de Balboa, 115 28006 MADRID (Espaa) Tfno.: (91) 782 43 32 / Fax: (91) 782 43 27
C hile U ruguay E-mail: secretaria@ctomedicina.com; iberocto@ctomedicina.com WEB: www.ctomedicina.com; www.iberocto.com
ED Pg. 7
ENDOCRINOLOGA
Seguimiento a distancia Preparacin Examen de Seleccin 05/06 1 Vuelta
mg/dl, urea 180 mg/dl, sodio 130 mEq/l, potasio 6 mEq/l, 1) Suero salino 0,9%.
calcio 14,5 mg/dl. En la exploracin fsica presenta edemas 2) Cuando glucemia sea de 300, suero glucosado 5%.
maleolares. Cul de los siguientes sera el tratamiento inicial 3) Insulina i.v.
de eleccin?: 4) Potasio i.v.
5) Bicarbonato i.v.
1) Hidratacin con suero salino isotnico.
2) Furosemida i.v. 80. Seale cul es la FALSA sobre el tratamiento de la descompen-
3) Hemodilisis. sacin hiperosmolar:
4) Calcitonina.
5) Bifosfonatos. 1) Es tpico de la DM autoinmune.
2) Los aportes de potasio son necesarios ms precozmente
que en la cetoacidosis.
DIABETES MELLITUS. 3) Los aportes de bicarbonato son necesarios cuando existe
acidosis lctica.
76. Un paciente presenta, en una analtica realizada en ayunas, 4) La insulina permite el control de la hiperglucemia.
una glucemia basal de 120 mg/dl. Utilizando los criterios 5) La mortalidad es mayor, en general, que en la cetoacidosis.
diagnsticos de diabetes mellitus de la ADA de 1997, usted le
podra diagnosticar de: 81. En el control de un paciente diabtico, es POCO importante
medir:
1) La cifra es normal. No se debe realizar ninguna medida.
2) Si la cifra en ayunas est entre 110 y 126 mg/dl, se 1) Glucosuria.
etiquetar como alteracin de la glucosa en ayunas. 2) Cetonuria.
3) La cifra indica alteracin de la glucosa en ayunas. Si la 3) Glucemia.
glucemia, 2 horas despus de 75 g de glucosa, es inferior a 4) Hemoglobina glicada (HbA1c).
140 mg/dl, se diagnosticar de intolerancia hidrocarbonada. 5) Fructosamina.
4) Si se realiza una sobrecarga de glucosa y la glucemia a las 2
horas est entre 140 y 200 mg/dl, se le etiquetar de diabetes. 82. Se llama efecto Somogyi a:
5) Si se repite 1 sola vez y la cifra en ayunas es superior o igual
a 126 mg/dl, se le puede etiquetar de diabetes. 1) Hiperglucemia matutina, como consecuencia de hipoglu-
cemia nocturna, con estmulo de hormonas contrainsulares.
77. Sobre la diabetes mellitus insulinodependiente, seale la 2) Hiperglucemia matutina por hipersecrecin de GH noc-
correcta: turna.
3) Hipoglucemia matutina, como consecuencia de hiper-
1) Se hereda de forma autosmica recesiva ligada al cromo- glucemia nocturna, con secrecin de insulina.
soma 6. 4) Hipoglucemia matutina por exceso de insulina adminis-
2) Es una enfermedad que se desarrolla siempre sobre una trada.
base exclusivamente gentica. 5) Hiperglucemia a las 3 a.m., que causa hipoglucemia
3) Existe una activacin de la inmunidad humoral (anticuer- nocturna.
pos antiislote, antiinsulina), permaneciendo inactiva la
inmunidad celular. 83. Un paciente de 50 aos, obeso, hipertenso y fumador,
4) Existen anticuerpos contra la descarboxilasa del cido consulta por el hallazgo casual de una glucemia en ayunas de
glutmico (GAD) que pueden reaccionar con los islotes 256 mg/dl, no acompaada de sntomas cardinales. En con-
pancreticos. sulta presenta una glucemia capilar en ayunas de 240 mg/dl.
5) Su etiopatogenia es equiparable a la de la DM no insulin- Tras iniciar tratamiento con dieta hipocalrica y metformina
dependiente. en dosis crecientes hasta alcanzar dosis mximas, el paciente

Preguntas TEST
presenta glucemias basales de 210, 200 y 189 mg/dl y no ha
78. Sobre la diabetes mellitus tipo MODY, seale la FALSA: perdido peso. Cmo modificara su tratamiento?:

1) Se hereda de forma autosmica dominante. 1) Insistir en la dieta. Aceptable control glucmico, no hay
2) Para su diagnstico se exige la presencia de la enfermedad que modificar el tratamiento.
en al menos dos miembros de la familia. 2) Insistir en la dieta. Sustituir biguanidas por acarbosa.
3) Se presenta en edades generalmente inferiores a los 25 aos. 3) Insistir en la dieta. Asociar acarbosa al tratamiento con
4) Existe una alteracin de la secrecin de insulina y una metformina.
resistencia a la accin de la insulina en los tejidos. 4) Insistir en la dieta. Suspender biguanidas y comenzar con
5) El tipo 2 se debe a una alteracin del gen de la glucoqui- sulfonilureas.
nasa que se encuentra en el cromosoma 7p. 5) Insistir en la dieta. Asociar insulina nocturna.

79. Varn de 20 aos, sin antecedentes conocidos, que acude 84. Slo una de las siguientes afirmaciones es cierta, en relacin
por vmitos, dolor abdominal y poliuria. GAB: pH 7,21, con el tratamiento farmacolgico de la diabetes mellitus:
HCO3: 16, pCO2: 20, pO2: 90. Na 120, K 4,5, Cl 90, glucosa
500, cetonuria positiva, creatinina 1,5. Cul de las siguientes 1) Glimepirida - nuevo y potente inhibidor de las alfagluco-
medidas NO est indicada?: sidasas intestinales, que reduce la glucemia postprandial.

M exico A rgentina CTO Medicina C/ Nez de Balboa, 115 28006 MADRID (Espaa) Tfno.: (91) 782 43 32 / Fax: (91) 782 43 27
Pg. 8 ED C hile U ruguay E-mail: secretaria@ctomedicina.com; iberocto@ctomedicina.com WEB: www.ctomedicina.com; www.iberocto.com
ENDOCRINOLOGA
Preparacin Examen de Seleccin 05/06 1 Vuelta Seguimiento a distancia

2) Miglitol - biguanida de ltima generacin. 1) Los pacientes con insulinoma tienen una concentracin
3) Metformina - sulfonilurea, hipoglucemiante oral. de pptido C aumentada paralela a los valores de insulina.
4) Repaglinida - sulfonilurea de 4 generacin, de accin 2) La proinsulina se eleva cuando se administra insulina
rpida. exgena.
5) Rosiglitazona - acta a nivel del receptor nuclear PPAR, 3) El pptido C se eleva cuando se administra insulina
reduciendo la resistencia insulnica. exgena.
4) Los antidiabticos orales elevan la concentracin de
85. Paciente de 55 aos, con antecedentes personales de HTA insulina, pero no de pptido C.
controlada con dieta, obesidad, hipercolesterolemia en tra- 5) El cociente insulina/glucosa es normal en los insulinomas.
tamiento con estatinas y diabetes mellitus tipo 2, tratada con
metformina. Acude a la consulta presentando cifras de glu- 89. La causa ms frecuente de hipoglucemia en un paciente
cemia capilar en ayunas entre 180-250 mg/dL, con cifras hospitalizado es:
postprandiales aceptables. En la analtica presenta HbA1c de
8%, LDL:120, TGs: 150, y cifras habituales de presin arterial 1) Tumores.
de 145/80 mmHg. Cul sera la actitud ms correcta en 2) Desnutricin.
cuanto a la modificacin del tratamiento?: 3) Hepatopata.
4) Frmacos.
1) Aadir IECAs, suspender metformina e iniciar tratamiento 5) Septicemia.
con insulina en pauta intensiva, dados los factores de
riesgo cardiovascular que presenta el paciente. 90. En el tratamiento del insulinoma se emplean todos los siguien-
2) Insistir en dieta hiposdica para el control de la presin tes, EXCEPTO:
arterial, suspender metformina e iniciar tratamiento con
sulfonilureas o repaglinida. 1) Pancreatectoma.
3) Aadir IECAs, mantener metformina y aadir rosiglitazona 2) Diazxido.
o pioglitazona, valorando la respuesta en visitas sucesivas. 3) Octretide.
4) Aadir IECAs, mantener metformina y aadir al tratamien- 4) Estreptozocina.
to sulfonilureas y acarbosa, valorando la respuesta al 5) Fenoxibenzamina.
tratamiento en visitas sucesivas.
5) Aadir IECAs, iniciar tratamiento con insulina en pauta
convencional, suspendiendo metformina, al igual que la DISLIPEMIA, OBESIDAD Y NUTRICIN.
estatina, ya que las cifras de LDL y TGs son excelentes.
91. En un paciente varn de 50 aos, con un infarto agudo de
86. Seale la asociacin INCORRECTA en cuanto al tipo de miocardio hace 2 aos y que presenta cifras elevadas de
insulina: colesterol, cul sera su objetivo de control de la hipercoles-
terolemia?:
1) Lispro - anlogo de insulina de accin ultrarrpida.
2) Glargina - insulina de accin muy prolongada, ms de 24 1) LDL < 200.
horas. 2) LDL < 160.
3) NPL - insulina de accin intermedia con perfil farmaco- 3) LDL < 130.
cintico similar a la NPH. 4) LDL < 100.
4) Insulina inhalada - farmacocintica similar a la insulina 5) LDL indetectable.
regular recombinante humana.
5) Insulina asprtica - insulina ultratard. 92. Dentro de las indicaciones de tratamiento hipolipemiante en
los diabticos, seale la verdadera:
87. NO indicara el tratamiento intensivo con insulina en uno de
Preguntas TEST

los siguientes: 1) Nunca se deben usar estatinas, pues alteran las cifras de
glucemia.
1) Debut de diabetes mellitus en un paciente de 14 aos con 2) Es siempre de eleccin el uso de cido nicotnico.
cuerpos cetnicos en orina. 3) Se recomienda mantener los niveles de LDL por debajo de
2) Diabetes en una paciente de 38 aos que ha debutado 100 mg/dl en los pacientes diabticos con cardiopata
con una cetoacidosis diabtica. isqumica.
3) Nio de 10 aos con DM tipo 1. 4) El miglitol es el hipolipemiante ms potente que se conoce
4) Diabtico tipo 2 de 53 aos, en mal control crnico con hoy en da.
pauta de insulinoterapia convencional. 5) La cerivastatina es un fibrato que disminuye slo los niveles
5) Diabtico tipo 2 de 85 aos de edad, con complicaciones de triglicridos.
microvasculares evolucionadas y en mal control crnico
con pauta convencional. 93. Seale la asociacin frmaco-mecanismo de accin INCO-
RRECTA:

HIPOGLUCEMIAS. 1) Atorvastatina - inhibicin de la HMG CoA reductasa.


2) Colestiramina - interrupcin de la circulacin enterohe-
88. Seala la respuesta correcta respecto al diagnstico diferen- ptica.
cial de la hipoglucemia:

M exico A rgentina CTO Medicina C/ Nez de Balboa, 115 28006 MADRID (Espaa) Tfno.: (91) 782 43 32 / Fax: (91) 782 43 27
C hile U ruguay E-mail: secretaria@ctomedicina.com; iberocto@ctomedicina.com WEB: www.ctomedicina.com; www.iberocto.com
ED Pg. 9
ENDOCRINOLOGA
Seguimiento a distancia Preparacin Examen de Seleccin 05/06 1 Vuelta
3) Gemfibrocil - aumento de hidrlisis de triglicridos y 100. En cul de los siguientes casos NO indicara la realizacin de
catabolismo de LDL. ciruga baritrica?:
4) cido nicotnico - disminuye sntesis de VLDL y LDL.
5) Lovastatina - aumenta la sntesis de cidos biliares. 1) IMC > 40, sin respuesta a dieta, ejercicio y frmacos.
2) IMC > 35, con diabetes mellitus.
94. Seale el efecto ADVERSO fundamental de los inhibidores de 3) IMC > 35, con sndrome de apnea del sueo.
la HMG CoA reductasa: 4) Paciente con trastorno de la conducta alimentaria de tipo
bulimia.
1) Arritmias auriculares. 5) IMC > 35, con cardiopata isqumica.
2) Estreimiento.
3) Diarrea y nuseas.
4) Miositis y afectacin heptica.
5) Colelitiasis.

95. Cul de los siguientes parmetros NO es til en la valoracin


del estado nutricional de un paciente?:

1) Protenas plasmticas de vida media corta-intermedia, como


prealbmina, transferrina y protena ligada a retinol.
2) Pliegue tricipital.
3) Recuento de linfocitos.
4) Niveles de vitaminas y oligoelementos.
5) Albmina en un paciente en UCI y situacin crtica.

96. Indicara una nutricin enteral por sonda nasogstrica en


todas las situaciones siguientes, EXCEPTO una:

1) Disfagia secundaria a ACVA.


2) Paciente joven en coma por traumatismo craneoencef-
lico.
3) Anorexia nerviosa con incapacidad para mantener un
peso mnimo mediante alimentacin oral.
4) Cuadro de obstruccin intestinal.
5) Fibrosis qustica con requerimientos nutricionales muy
altos.

97. Son posibles complicaciones asociadas a la nutricin paren-


teral, todas MENOS una:

1) Hiperglucemia y alteraciones hidroelectrolticas.


2) Vmitos.
3) Sepsis por catter.
4) Neumotrax.
5) Esteatosis heptica.

Preguntas TEST
98. Cul de las siguientes es FALSA respecto a la clasificacin de
la poblacin segn su ndice de masa corporal?:

1) IMC de 18,5 a 24,9 normalidad.


2) IMC de 25 a 29,9 sobrepeso.
3) IMC de 30 a 34,9 obesidad grado 1.
4) IMC de 35 a 39,9 obesidad grado 2.
5) IMC mayor de 40 superobesidad o extrema.

99. En un paciente con un IMC de 38 y con una ingesta habitual


de 3.000 Kcal, qu dieta prescribira?:

1) 500-700 Kcal.
2) 1.000-1.500 Kcal.
3) 1.500-2.500 Kcal.
4) 3.000-3.500 Kcal.
5) No precisa dieta.

M exico A rgentina CTO Medicina C/ Nez de Balboa, 115 28006 MADRID (Espaa) Tfno.: (91) 782 43 32 / Fax: (91) 782 43 27
Pg. 10 ED C hile U ruguay E-mail: secretaria@ctomedicina.com; iberocto@ctomedicina.com WEB: www.ctomedicina.com; www.iberocto.com
ENDOCRINOLOGA
Preparacin Examen de Seleccin 05/06 1 Vuelta Seguimiento a distancia
HIPFISIS. Pregunta 3.- R: 4

Pregunta 1.- R: 5
La hiperprolactinemia se puede producir en muchas situaciones, sien-
do la ms frecuente la secundaria a frmacos como estrgenos, opiceos,
neurolpticos, metildopa, etc. El s. de Nelson es un adenoma hipofisario
que aparece aos despus de una suprarrenalectoma bilateral realizada a
pacientes con Cushing. Puede producir hiperprolactinemia por compre-
sin del tallo hipofisario. Este es el mismo mecanismo de los macroadenomas
no secretores de hipfisis. El craneofaringioma es el tumor hipotalmico
ms frecuente en nios, y en su crecimiento compromete las funciones
hipotalmicas, incluyendo la secrecin de dopamina, motivo por el que
eleva las cifras de prolactina. Las formas de hipofisitis linfocitarias que
afectan al tallo tambin elevan las cifras de prolactina. Sin embargo, el s. de
Sheehan consiste en una necrosis isqumica postparto hipofisaria que se
manifiesta por incapacidad para la lactancia, al disminuir la prolactina.

Stress

Succin del Neurolpticos


pezn 5HT VIP, TRH Frmacos Opiceos
Alfa-metildopa
Reserpina
Prolactina Dopamina

Secrecin
Estrgenos
lctea

Pregunta 1. Regulacin de la prolactina.

Pregunta 2.- R: 4
Existen cifras orientativas para el diagnstico diferencial de la hiper-
prolactinemia. Aquellas superiores a 300 microgramos/L son caracte-
rsticas del prolactinoma. Igualmente, descartado un embarazo, cifras
mayores a 150 tambin lo son. Las cifras entre 50-100 suelen ser
causas 2 como compresin del tallo (respuesta 4), lesiones hipotal-
micas, frmacos, etc. Ver figura a pie de pgina. Pregunta 3. Manejo de la hiperprolactinemia.
Comentarios TEST

Pregunta 2. Patologas hipofisarias-hipotalmicas.

M exico A rgentina CTO Medicina C/ Nez de Balboa, 115 28006 MADRID (Espaa) Tfno.: (91) 782 43 32 / Fax: (91) 782 43 27
C hile U ruguay E-mail: secretaria@ctomedicina.com; iberocto@ctomedicina.com WEB: www.ctomedicina.com; www.iberocto.com
ED Pg. 1
ENDOCRINOLOGA
Seguimiento a distancia Preparacin Examen de Seleccin 05/06 1 Vuelta
El tratamiento de primera eleccin en los prolactinomas, indepen- gos de somatostatina (octretide), a los que se pueden asociar agonistas
dientemente del tamao, son los agonistas dopaminrgicos (bromo- dopaminrgicos, es la ltima lnea. No est claro todava el papel del
criptina, quinagolida y cabergolina). En caso de fracaso teraputico, tratamiento con anlogos de somatostatina antes de la ciruga.
la siguiente opcin es la ciruga transesfenoidal. La radioterapia es
poco eficaz en los prolactinomas y se reserva para los macroprolacti- Pregunta 7.- R: 2
nomas resistentes a agonistas dopaminrgicos con resto tumoral tras Se habla de curacin bioqumica en la acromegalia cuando se
ciruga. Todos los macroprolactinomas se tratan siempre, mientras normalizan los niveles de IGF-1 y se suprime la GH a menos de 1 tras
que los micros o la causa idioptica se tratan si dan sntomas. SOG. Ello evita el exceso de mortalidad de estos pacientes. Las res-
puestas paradjicas a GnRH y TRH son menos usadas en la actualidad
Pregunta 4.- R: 3 como criterios pronsticos. Las indicaciones de colonoscopia en la
La infertilidad es una manifestacin clnica de los prolactinomas. acromegalia son: AP o familiares de plipos en colon, clnica intesti-
Aunque los estrgenos aumentan los niveles de prolactina, la mayora nal, ms de 10 aos de enfermedad, sobre todo si activa y en pacien-
de los prolactinomas no crecen en el embarazo. Por ello la actitud tes con ms de 3-6 acrocordomas en cuello o dorso superior.
correcta es la 3. Ninguno de los agonistas dopaminrgicos es teratog-
nico, por lo que ante unos niveles de prolactina mayores de 400 o Pregunta 8.- R: 1
alteraciones campimtricas se reinicia el tratamiento. En el enunciado nos dan la clnica de varios dficit hormonales
asociados: alteracin de libido e impotencia (hipogonadismo), astenia
Pregunta 5.- R: 2 y nuseas (insuficiencia suprarrenal), piel seca y letargia (hipotiroidis-
La acromegalia se debe casi siempre a tumores hipofisarios producto- mo). Los iones y colesterol normales orientan hacia un origen hipofisa-
res de GH y los pacientes tienen un aumento de morbimortalidad por rio. Por tanto, de las respuestas, la patologa que no produce hipopitui-
causas cardiovasculares y aumento de tumores. Los criterios diagnsticos tarismo es el microprolactinoma. Recordar que la sensibilidad de la RM
actuales son niveles de IGF-1 elevados para sexo y edad y GH mayor de no es del 100% en la deteccin de toda la patologa selar.
1 microg/L tras SOG. El MEN 1 asocia tumores de hipfisis, paratiroides y
pncreas y se debe sospechar en un acromeglico con hipercalcemia, ya Pregunta 9.- R: 1
que sta no se produce en la acromegalia aislada. La apopleja hipofisaria es un cuadro grave con clnica neurolgi-
ca en su presentacin, por necrosis de tumores hipofisarios. El diag-
nstico diferencial con otras entidades tales como la hemorragia
Pregunta 5. Criterios de acromegalia. subaracnoidea o rotura de aneurismas cerebrales, etc, es lo primor-
dial. Su tratamiento debe ser urgente, con dosis altas de corticoides y
CRITERIOS DIAGNSTICOS neurociruga descompresiva urgente.
SOG para GH (120 min.): GH>1 con IRMA (> 2g/l con RIA). Pregunta 10.- R: 2
Aumento de IGF-1 para valor normal segn edad y sex o. En endocrinologa, los dficits hormonales se diagnostican con
CRITERIOS DE CURACIN determinaciones basales si no hay solapamiento con valores norma-
les, en cuyo caso se necesitan realizar pruebas de estimulacin. La
SOG para GH (120 min.): GH<1 con IRMA (<2 ug/l con RIA). mejor de ellas para valorar el dficit de ACTH es la hipoglucemia con
IGF-1: normales (para edad y sex o). insulina. Ver el resto en la tabla.

Pregunta 11.- R: 3
Pregunta 6.- R: 2 El enunciado nos presenta un caso de s. de Sheehan, que consiste
El tratamiento de 1 eleccin de la acromegalia es en todos los casos la en una necrosis isqumica postparto hipofisaria que se manifiesta por
ciruga transesfenoidal, seguida por la radioterapia asociada a anlogos incapacidad para la lactancia, al disminuir la prolactina. Dado que
de somatostatina hasta que la radio acte en caso de persistencia de puede producir hipopituitarismo, no habr hiperpigmentacin, ya
actividad y resto tumoral tras la ciruga. El tratamiento mdico con anlo- que los niveles de ACTH estarn disminuidos.

Pregunta 10. Diagnstico y tratamiento del hipopituitarismo.

Comentarios TEST
Hormona Determinaciones basales Pruebas funcionales Tratamiento

1. Hipoglucemia insulnica (la de mayor


IGF-1 (puede tener falsos negativos en el utilidad) para GH. 1. Nios: dficit aislado o combinado.
GH
diagnstico de dficit de GH) 2. Otras: arginina, ornitina, clonidina, 2. Adultos: dficit GH con hipopituitarismo.
GHR H, hexarelin).

1. Hipoglucemia insulnica (la de mayor


Cortisol basal <3,5 es diagnstico, Hidrocortisona oral o IV en caso de crisis.
ACTH utilidad) para cortisol.
>18 mg/dL excluye. Primer dficit a sustituir.
2. Test de ACTH con 1 ug para cortisol.
Prolactina basal (a veces no excluye el dficit). Si Estimulacin con TR H o metoclopramida No se sustituye.
PRL
alta indica lesin en hipotlamo o tallo. para excluir el dficit. Lactancia artificial.
Test de TR H (en desuso porque no
TSH y T4L basales (un 30% de pacientes con TSH Levotiroxina oral despus de corticoides si
TSH discrimina bien entre lesin hipotalmica e
basal normal) dficit de ACTH.
hipofisaria)

1. Esteroides gonadales si no deseo de


Testosterona varones. fertilidad.
1. Test de estimulacin con GnR H.
LH/FSH Menstruacin mujeres. 2. Estimulacin con clomifeno.
2. LH y FSH si deseo de fertilidad.
LH y FSH basales si las anteriores alteradas. 3. GnR H en bomba con pulsos en
algunos casos.

M exico A rgentina CTO Medicina C/ Nez de Balboa, 115 28006 MADRID (Espaa) Tfno.: (91) 782 43 32 / Fax: (91) 782 43 27
Pg. 2 ED C hile U ruguay E-mail: secretaria@ctomedicina.com; iberocto@ctomedicina.com WEB: www.ctomedicina.com; www.iberocto.com
ENDOCRINOLOGA
Preparacin Examen de Seleccin 05/06 1 Vuelta Seguimiento a distancia
Pregunta 12.- R: 2 to de que la yoduria refleja la cantidad de yodo de que dispone una
El enunciado nos presenta un caso de s. de Nelson que es un persona. Sin embargo, el yodo en situaciones y dosis determinadas
adenoma hipofisario que aparece aos tras una suprarrenalectoma puede producir efectos distintos a los fisiolgicos. Un ejemplo es el
bilateral realizada a pacientes con Cushing. Puede producir hiper- fenmeno de Jod-Basedow que ocurre en pacientes con bocio ante
prolactinemia por compresin del tallo hipofisario, hiperpigmenta- la administracin de dosis elevadas de yodo, y cuyo efecto es la
cin por secrecin de ACTH y clnica compresiva local como cefalea, aparicin de un hipertiroidismo que en el momento del diagnstico
alteraciones visuales, etc. tiene una gammagrafa hipocaptante. Otro ejemplo es el Wolff-
Chaikoff, que consiste en la inhibicin de la organificacin ante la
Pregunta 13.- R: 5 administracin de dosis altas de yodo en pacientes con hipertiroi-
Ante un s. polirico y polidpsico, lo primero es descartar por su dismo. Otros efectos son la induccin de bocio o hipotiroidismo en
prevalencia una diabetes mellitus. El siguiente paso es determinar pacientes con enfermedades autoinmunes de tiroides o la fibrosis
osmolaridad simultnea en plasma y orina. Aquellos con osm-p nor- preoperatoria, que se consigue con la administracin de lugol.
mal y osm-o disminuida son candidatos a un test de deshidratacin.
En caso de estar deshidratados, inicialmente no se debe realizar esta Pregunta 18.- R: 5
prueba y bastara con la inyeccin de vasopresina de entrada. El transporte de yodo al interior de la clula folicular se realiza me-
diante un proceso activo en el que se acopla la actividad de la Na/K
Pregunta 13. Diagnstico de diabetes inspida. ATPasa a un simporte I/Na (NIS). El yodo es organificado en el coloide
(por la peroxidasa tiroidea) y se une a los residuos de tirosina de la
tiroglobulina (protena producida por la clula folicular). Una vez for-
DI Central DI nefrognica PP mados, los complejos monoyodotirosina y diyodotirosina se acoplan
Osm. ur. tras para formar las hormonas tiroideas, que son liberadas al torrente san-
No cambia No cambia Aumenta guneo y el yodo no utilizado se recupera por una dehalogenasa del
deshidratacin
Osm. tras ADH Su b e > 1 0 % Su b e < 9 % Su b e < 9 %
citosol de la clula folicular. La nica fuente de T4 es el tiroides, que
tambin produce una pequea cantidad de T3. Sin embargo, los teji-
dos perifricos son capaces de producir T3 a partir de la desyodacin
La diabetes inspida central se trata con desmopresina (DDAVP) oral de la T4.
o nasal en pacientes conscientes, y parenteral si estn inconscientes.

Pregunta 14.- R: 1
La DI nefrognica tiene un comienzo ms insidioso que la central
(comienzo brusco). Las causas ms frecuentes de DI nefrognica son
las 2, debidas a enfermedades tubulointersticiales renales o por fr-
macos como el litio o la demeclociclina. La hipocaliemia o hipercal-
cemia crnicas tambin pueden producirla. El hipotiroidismo, sin
embargo, es causa de hiponatremia, incluso de SIADH.

Pregunta 15.- R: 3
Los criterios diagnsticos de SIADH incluyen la hiponatremia con
hiposmolaridad plasmtica, volumen circulante normal (no edemas ni
hipotensin), osm en orina inapropiadamente elevada (mayor de 100-
300) y la exclusin de hipotiroidismo e insuficiencia suprarrenal.

Pregunta 15. Criterios diagnsticos de SIADH.

Mayores Menores
Comentarios TEST

1. Hiponatremia. Pregunta 18. Sntesis de hormonas tiroideas.


2. Hiposmolaridad plasmtica.
3. No edemas. Pregunta 19.- R: 3
4. No deplecin de volumen 1. Sobrecarga hdrica Como hemos dicho antes, los tejidos perifricos son capaces de
(T.A. normal). patolgica. producir T3 a partir de la desyodacin de la T4. La inhibicin de las
5. Falta de dilucin mx ima de orina 2. Niveles de AVP plasma y
desyodasas perifricas se puede producir en muchas situaciones tales
(osm orina >100 mOsm/kg). orina elevados.
como enfermedades graves, desnutricin o ayuno severos, politrau-
6. Ex clusin de hipotiroidismo e
insuficiencia suprarrenal.
matismos (sndrome de enfermedad sistmica no tiroidea) o por la
accin de frmacos tales como la amiodarona, dexametasona, pro-
*Se precisan todos los criterios mayores para el diagnstico. Los menores son pranolol, contrastes yodados y propiltiouracilo (el nico antitiroideo
opcionales. que posee este efecto).

Pregunta 16.- R: 1 Pregunta 20.- R: 3


El tratamiento del SIADH es urgente en caso de Na menor de 125 Las enfermedades tiroideas autoinmunes constituyen un grupo en el
o clnica grave del SNC, en cuyo caso se realiza con una infusin de que se encuentran la tiroiditis linfocitaria crnica (de Hashimoto y la
suero salino hipertnico con o sin furosemida IV. El tratamiento del variante atrfica), la enfermedad de Graves-Basedow y la tiroiditis indolo-
SIADH es conservador con Na mayor de 125 y sin clnica. Para ello ra o linfocitaria transitoria. En todas ellas pueden aparecer con frecuencia
comenzamos con restriccin hdrica (500-1.000cc/d), y si no se con- variable anticuerpos antitiroideos (antitiroglobulina y antiperoxidasa). Hay
trola la hipoNa, usaremos frmacos como el litio o la demeclociclina. que diferenciarlos de los anticuerpos frente al receptor de TSH, que son
tpicos, aunque no exclusivos de la enfermedad de Graves.
TIROIDES.
Pregunta 21.- R: 3
Pregunta 17.- R: 2 El bocio simple se define como un aumento del tamao de la
El yodo es necesario para la formacin de las hormonas tiroideas, glndula tiroidea con funcin tiroidea normal, es decir, todas la hor-
y su metabolismo est muy estrechamente controlado, hasta el pun- monas normales en plasma. El tratamiento, en el caso de que existan

M exico A rgentina CTO Medicina C/ Nez de Balboa, 115 28006 MADRID (Espaa) Tfno.: (91) 782 43 32 / Fax: (91) 782 43 27
C hile U ruguay E-mail: secretaria@ctomedicina.com; iberocto@ctomedicina.com WEB: www.ctomedicina.com; www.iberocto.com
ED Pg. 3
ENDOCRINOLOGA
Seguimiento a distancia Preparacin Examen de Seleccin 05/06 1 Vuelta
fenmenos compresivos tales como disfagia, disnea, compresin de Pregunta 29.- R: 2
vasos con pltora facial y sncope (signo de Pemberton) es la ciruga La gammagrafa es una prueba til para aproximarse al diagnstico
mediante tiroidectoma subtotal. El resto de los casos se debe indivi- de hipertiroidismo, ya que nos separa las causas producidas por
dualizar, aunque el tratamiento ms empleado es la administracin hiperfuncin autnoma del tiroides (gammagrafa hipercaptante) de
de dosis supresoras de tiroxina. las que tienen otra fisiopatologa (gammagrafa silente). En este sentido
cabe recordar que el coriocarcinoma produce un hipertiroidismo en
Pregunta 22.- R: 3 el que la gammagrafa es de hipercaptacin difusa, como en la enfer-
Bocio significa aumento del tamao de la glndula tiroidea y no medad de Graves. En el caso del coriocarcinoma es la beta-hCG la
define su funcionalidad. Podemos tener patologas con bocio e hipo- que estimula el receptor de TSH.
tiroidismo como la tiroiditis de Hashimoto, bocio con hipertiroidismo
como la enfermedad de Graves o bocio con normofuncin tiroidea
(bocio simple). El cncer diferenciado de tiroides es raramente Pregunta 29. Clasificacin de los estados de tirotoxicosis
funcionante. segn la captacin gammagrfica.

Pregunta 23.- R: 4 Captacin aumentada:


Como se ha dicho anteriormente, el bocio simple consiste en au- - Enfermedad de Graves.
mento del tamao de la glndula tiroidea con normofuncin tiroi- - Tumores productores de TSH y hCG.
dea. En su diagnstico se pueden realizar pruebas de imagen para - BMNT y adenoma txico.
descartar los fenmenos compresivos en otros rganos o para el diag- Captacin disminuida:
nstico diferencial con otras entidades (bocio multinodular, ndulos - Tiroiditis y fenmeno de Jod-Basedow.
solitarios, etc.). La determinacin de tiroglobulina y la realizacin de - Tirotoxicosis facticia.
rastreos con I-131 forman parte del seguimiento de los pacientes ope- - Struma ovarii.
rados de cncer diferenciado de tiroides y no juegan ningn papel en - Metstasis funcionantes de carcinoma folicular.
el diagnstico del bocio simple.

Pregunta 24.- R: 5 Pregunta 30.- R: 4


La causa ms frecuente de bocio, simple o con hipotiroidismo, es La enfermedad de Graves es autoinmune y el hipertiroidismo no
el dficit de yodo. Muchas de las causas de bocio simple pueden sigue un curso paralelo al de la oftalmopata en muchos casos. El
producir con el tiempo o la dosis hipotiroidismo. Bocigenos como la tratamiento debe individualizarse dependiendo del grupo de edad
amiodarona, el litio, etc. son un ejemplo. Igualmente el yodo en pa- (nios y jvenes, adultos o ancianos) y ante la presencia de embarazo
cientes con patologa autoinmune de tiroides puede producir bocio, en una mujer. En general, en nuestro medio se recomienda tratamien-
incluso hipotiroidismo. to con antitiroideos prolongado en nios, jvenes y adultos (entre 12
y 24 meses) para conseguir un efecto inmunomodulador y as evitar
Pregunta 25.- R: 2 brotes de hipertiroidismo futuros.
El tratamiento sustitutivo del hipotiroidismo se realiza en la actuali-
dad con tiroxina (T4) oral. El objetivo del tratamiento es la normaliza- Pregunta 31.- R: 3
cin de los niveles de TSH en caso de hipotiroidismo primario y de los La administracin de yodo-131 es una herramienta til en el trata-
niveles de T4 libre en el caso de hipotiroidismo central. En la pregunta miento de la enfermedad de Graves, tanto por su eficacia como por su
nos encontramos ante un caso de hipotiroidismo primario en el que seguridad. Est contraindicado en embarazadas, nios y jvenes por
la TSH del plasma est por debajo del lmite normal, lo que indica debajo de los 20 aos y no debe emplearse ante la existencia de
que la dosis empleada es elevada para ese paciente, y la actitud es ndulos tiroideos sin antes investigar la naturaleza de los mismos. En
disminuir la dosis y reevaluar al paciente pasadas 4-6 semanas con los adultos mayores y ancianos es de eleccin tras la normalizacin
una nueva determinacin de TSH. de la funcin tiroidea con antitiroideos.

Pregunta 26.- R: 4 Pregunta 32.- R: 4


El hipotiroidismo subclnico es la situacin definida por unos niveles En el embarazo, la enfermedad de Graves debe controlarse con
de TSH plasmtica elevados con una T4 libre normal en plasma. Hoy en antitiroideos (especialmente con propiltiouracilo) en dosis suficientes
para mantener las hormonas maternas en el lmite alto de la normalidad,

Comentarios TEST
da se conoce que no es una situacin verdaderamente subclnica y se
recomienda su tratamiento siempre que la TSH est por encima de 10 y as evitar el hipotiroidismo fetal. En general, la enfermedad de Graves
mUI/mL. Cuando est entre 5 y 10, se recomienda tratar en la mayora de mejora a lo largo del embarazo, pero en casos refractarios, dado que el
las situaciones, excepto en ancianos o pacientes con cardiopata. radioyodo est contraindicado, se debe recurrir a la ciruga.

Pregunta 27.- R: 4 Pregunta 33.- R: 1


El coma mixedematoso es una urgencia rara hoy en da que ocurre Las urgencias en el hipertiroidismo son la crisis tirotxica o tormen-
en pacientes con hipotiroidismo de larga duracin o mal tratados ta tiroidea y las crisis cardacas, como la del enunciado. En estos casos
cuando se exponen a un fro intenso, ciruga, sepsis y otros factores el tratamiento est dirigido a controlar de forma rpida los sntomas y
desencadenantes. El pronstico es malo y conlleva una elevada mor- la disminucin de la cantidad de hormonas tiroideas en el organismo.
talidad. El tratamiento debe iniciarse ante la sospecha y confirmarse Ello es posible de forma rpida, sobre todo mediante la inhibicin de
mediante analtica de hormonas urgente. Inicialmente se administran la conversin perifrica de T4 a T3. El tratamiento definitivo (en este
glucocorticoides intravenosos, y despus hormona tiroidea, bien T4 caso el yodo radiactivo) debe postponerse hasta la normalizacin de
intravenosa o T3 por sonda nasogstrica. la situacin de tirotoxicosis.

Pregunta 28.- R: 3 Pregunta 34.- R: 2


La tirotoxicosis facticia consiste en el hipertiroidismo por ingesta de La mayora de los ndulos tiroideos solitarios son benignos (el 95%),
hormonas tiroideas. En el caso de la toma de preparados de T3, el incluso el ndulo solitario, que es fro en la gammagrafa, tambin
perfil hormonal es una TSH suprimida con T3 libre elevada y T4 tiene alta probabilidad de ser benigno (el 80%). Existen una serie de
disminuida, lo que lleva al diagnstico. En el caso de ingesta de T4, el factores que aumentan el riesgo de que un ndulo sea maligno, tales
diagnstico diferencial debe hacerse con otras situaciones de hiperti- como la existencia de adenopatas, la fijacin a estructuras adyacen-
roidismo con gammagrafa hipocaptante, ya que en este caso el perfil tes, el crecimiento rpido indoloro del ndulo, el tamao mayor de 4
hormonal es el de una TSH suprimida con niveles tanto de T3 libre cms, sexo varn, edades extremas de la vida (ancianos o nios), ante-
como de T4 libre elevados. cedentes familiares de cncer de tiroides o personales de radiacin en
la infancia. La prueba que aporta hoy en da una mayor razn de

M exico A rgentina CTO Medicina C/ Nez de Balboa, 115 28006 MADRID (Espaa) Tfno.: (91) 782 43 32 / Fax: (91) 782 43 27
Pg. 4 ED C hile U ruguay E-mail: secretaria@ctomedicina.com; iberocto@ctomedicina.com WEB: www.ctomedicina.com; www.iberocto.com
ENDOCRINOLOGA
Preparacin Examen de Seleccin 05/06 1 Vuelta Seguimiento a distancia
probabilidades positiva (sensibilidad/1-especificidad) es la puncin metstasis incluso tras este perodo, lo que obliga a un seguimiento de por
aspiracin con aguja fina PAAF, y por tanto, la que se recomienda vida de estos pacientes. El tratamiento de eleccin hoy en da es la
como prueba inicial para decidir la actitud. tiroidectoma total con linfadenectoma en caso de afectacin ganglionar.
Los pacientes reciben tratamiento supresor con tiroxina de por vida, y en
Pregunta 35.- R: 3 caso de elevacin de tiroglobulina, se someten a un rastreo con yodo
La mayora de los ndulos tiroideos solitarios son benignos (el 95%), radiactivo para posterior ablacin de los restos o metstasis captantes.
incluso el ndulo solitario, que es fro en la gammagrafa, tambin tiene
alta probabilidad de ser benigno (el 80%), siendo histolgicamente quistes Pregunta 38.- R: 5
coloidales o adenomas foliculares no funcionantes. El carcinoma medular de tiroides es espordico en la mayora de los
casos, pero puede ser familiar y asociarse a la neoplasia endocrina mltiple
Pregunta 36.- R: 1 tipo 2 (carcinoma medular, hiperparatiroidismo y feocromocitoma en el
Aunque la PAAF es la mejor opcin diagnstica ante el ndulo 2a y carcinoma medular, feocromocitoma y neuromatosis en el 2b), de
tiroideo, tambin tiene falsos negativos y positivos. En caso de PAAF carcter autosmico dominante con penetrancia variable. En estos casos se
maligna se debe intervenir al paciente, y en caso de ser benigna, se ha descrito en la mayora de las familias afectadas, mutaciones en el
realiza un seguimiento bajo tratamiento supresor con tiroxina. En caso protooncogen RET, que debe investigarse en dicha situacin. El carcinoma
de PAAF folicular o no concluyente es importante tener en cuenta otros medular secreta calcitonina, la que se usa para el diagnstico y el segui-
datos de riesgo para cncer de tiroides ya comentados antes. miento de los pacientes. La terapia ms eficaz y la nica que ha demostrado
modificar la supervivencia es la ciruga. Este cncer no capta I-131 y no
responde de forma eficaz a quimioterapia ni radioterapia externa.
Ndulo tiroideo
Pregunta 39.- R: 2
PAAF
Los ndulos tiroideos sospechosos de malignidad deben remitirse
a ciruga para biopsia de la pieza, lo que aporta el diagnstico defini-
tivo. Por tanto, los ndulos que crecen con tratamiento supresor con
Maligno Folicular No concluyente Benigno
levotiroxina, ndulos con PAAF no concluyente en pacientes con
factores de riesgo o PAAF folicular con gammagrafa fra, deben remi-
Tiene No tirse para hemitiroidectoma y biopsia de la pieza, tras cuyo resultado
Gammagrafa factores se proceder o no segn el caso a ampliar la reseccin con tiroidecto-
de riesgo
ma total y exploracin ganglionar.
para
ca tiroides
Pregunta 40.- R: 3
Caliente Fro Aunque la PAAF es la mejor opcin diagnstica ante el ndulo
Supresin tiroideo, tambin tiene falsos negativos y positivos. En caso de PAAF
con L-T4 3 meses maligna se debe intervenir al paciente, y en caso de ser benigna, se
y repetir PAAF realiza un seguimiento bajo tratamiento supresor con tiroxina. En caso
de PAAF folicular o no concluyente, es importante tener en cuenta
Aumento de Sin cambio
que una serie de factores aumentan el riesgo de que un ndulo sea
tamao o reduccin
de tamao
maligno, tales como la existencia de adenopatas, la fijacin a estruc-
Tiroidectoma Estudio Hemitiroidectoma e turas adyacentes, compresin del larngeo recurrente, el crecimiento
total hipertiroidismo istmectoma para estudio A.P rpido indoloro del ndulo, el tamao mayor de 4 cms, sexo varn,
edades extremas de la vida (ancianos o nios), antecedentes familiares
Seguimiento
de cncer de tiroides o personales de radiacin en la infancia, as
Maligno Benigno
como el aumento de riesgo de cncer de tiroides en los ndulos fros.
Pregunta 36. Actitud ante el ndulo tiroideo solitario.
Pregunta 41.- R: 3
Se trata de una paciente con carcinoma diferenciado de tiroides con
Comentarios TEST

tiroidectoma que recibi dosis ablativa de restos tiroideos con radioyodo.


Pregunta 37.- R: 2 Ante la elevacin de las cifras de tiroglobulina se sospecha recidiva o
metstasis de la enfermedad y procedemos a un rastreo corporal con
yodo-131 para localizar la enfermedad residual, y en caso de captacin
positiva, la administracin de una dosis ablativa del istopo.

Pregunta 42.- R: 3

Pregunta 42. Factores de riesgo de carcinoma de tiroides


ante un ndulo tiroideo.

Familiares de cncer de tiroides o MEN 2.


Antecedentes Personales de radiacin en cabeza o cuello, sobre
todo en infancia.
E da d > 4 5 a o s o < 1 6 .
Sexo masculino.
Anamnesis Ndulo palpable reciente, de crecimiento rpido
e indoloro.
Pregunta 37. Seguimiento del paciente con carcinoma diferenciado de R onquera por parlisis del larngeo recurrente.
tiroides.TG: tiroglobulina, RCT: rastreo con I-131.
Tamao > 4 cms.
El carcinoma diferenciado de tiroides (papilar y folicular) tiene una Adenopatas palpables.
Exploracin
supervivencia a los 20 aos de casi un 95% en el papilar y 85% en el Fijacin a estructuras profundas (no desplazable
folicular, aunque por otro lado, estn descritas las recidivas tumorales y con deglucin).

M exico A rgentina CTO Medicina C/ Nez de Balboa, 115 28006 MADRID (Espaa) Tfno.: (91) 782 43 32 / Fax: (91) 782 43 27
C hile U ruguay E-mail: secretaria@ctomedicina.com; iberocto@ctomedicina.com WEB: www.ctomedicina.com; www.iberocto.com
ED Pg. 5
ENDOCRINOLOGA
Seguimiento a distancia Preparacin Examen de Seleccin 05/06 1 Vuelta

Pregunta 44. Caractersticas de las principales tiroiditis.

ETIOLOGA CLNICA DIAGNSTICO TRATAMIENTO


Dolor, calor, rubor y tum efaccin en cara
Fiebre, leucocitosis con Antibiticos.
Aguda Bacteriana anterior del cuello, sntom as sistm icos de
desviacin izquierda Drenaje quirrgico.
infeccin
Bocio doloroso nodular. VSG aum entada, captacin Aspirina.
Subaguda Viral Dolor cervical anterior, febrcula, n u l a e n g a m m a g r a f a , Corticoides.
hipertiroidism o. autoanticuerpos negativos. Betabloqueantes.
VSG norm al, captacin nula
Bocio no doloroso, hipertiroidism o. Betabloqueantes.
Linfocitaria transitoria Autoinm une? e n g a m m a g r a f a , t t u l o s b a jo s
Posible hipotiroidism o transitorio. Levotiroxina.
de antiperoxidasa.
Bocio indoloro, hipotiroidism o.
Linfocitaria crnica Autoinm une A veces hipertiroidism o inicial Altos ttulos de antiperoxidasa Tiroxina
autolim itado (Hashitoxicosis).
Bocio m uy duro, sntom as de Captacin nula en
Ciruga si produce
Fibrosante Desconocida com presin cervical gam m agrafa, anticuerpos
com presin
Hipotiroidism o 25%. n e g a t iv o s

Una serie de factores aumentan el riesgo de que un ndulo sea SUPRARRENAL.


maligno, tales como la existencia de adenopatas, la fijacin a estruc-
turas adyacentes, compresin del larngeo recurrente, el crecimiento Pregunta 45.- R: 5
rpido indoloro del ndulo, el tamao mayor de 4 cms, sexo varn, La causa ms frecuente de sndrome de Cushing es el yatrgeno o
edades extremas de la vida (ancianos o nios), antecedentes familiares exgeno. Se debe a la administracin de esteroides de forma crnica
de cncer de tiroides o personales de radiacin en la infancia, as y es la causa de s. Cushing ms frecuente en la prctica clnica. Su
como el aumento de riesgo de cncer de tiroides en los ndulos fros. diagnstico se realiza con la demostracin de la supresin del cortisol
y ACTH del paciente que acude con clnica compatible e historia de
Pregunta 43.- R: 3 tratamiento con esteroides.
Las tiroiditis son procesos en los que existe una inflamacin de la
glndula. En muchas de ellas el patrn funcional es de una primera Pregunta 46.- R: 4
fase de hipertiroidismo por liberacin de hormonas tiroideas prefor- Dado que hemos dicho que la ACTH estar suprimida, no puede
madas al torrente sanguneo y una segunda fase de hipotiroidismo haber hiperpigmentacin. El resto son todas compatibles con el exce-
que puede ser autolimitada. La tiroiditis fibrosante o de Riedel con- so de glucocorticoides y pueden verse en el Cushing exgeno.
siste en una fibrosis del cuello que puede afectar al mediastino y al
retroperitoneo de forma concomitante, en la que la funcin tiroidea Pregunta 47.- R: 3
suele ser normal, existiendo hipotiroidismo hasta en el 25% de los Dentro del Cushing endgeno nos encontramos con: el adeno-
casos, pero el hipertiroidismo no est descrito en esta entidad. ma hipofisario productor ACTH (70%): Microadenoma (80-90%) y
macroadenoma (10-20%); tumor ectpico productor ACTH (15-
Pregunta 44.- R: 5 20%), adenoma suprarrenal (10%) e hiperplasia nodular suprarre-
La tiroiditis granulomatosa subaguda (de Quervain) se debe a cau- nal (<1%).
sas virales que producen una inflamacin del tiroides que produce

Comentarios TEST
dolor e hipertiroidismo con febrcula y sntomas de catarro de vas Pregunta 48.- R: 4
altas. El diagnstico se basa en la comprobacin de VSG elevada, El enunciado hace referencia a un Cushing ectpico agresivo en el
junto con una gammagrafa hipocaptante o silente. El enunciado nos que predomina la hiperpigmentacin, las alteraciones metablicas del
muestra una tiroiditis fibrosante, que no debe confundirse con la exceso de cortisol y la clnica del cncer de base. En estos casos puede
tiroiditis subaguda. Si bien la VSG puede estar elevada en la fibrosante, no existir hbito Cushingoide en el paciente, ya que el cambio en el
no suele ser del rango de la de Quervain. Ver tabla en pgina siguiente. fenotipo por efecto del hipercortisolismo puede llevar su tiempo.

Pregunta 51. Diagnstico diferencial del sndrome de Cushing.

123456789  5 35 9



7 3749 7 4244782
756789   492 7892 2492 7892 647 9 39
943 9


 39 7 7!7 12 12 34 12534

" 9 739 4982
7 3 # 7 7437
$ 12 34 34 12534
6 4 7

M exico A rgentina CTO Medicina C/ Nez de Balboa, 115 28006 MADRID (Espaa) Tfno.: (91) 782 43 32 / Fax: (91) 782 43 27
Pg. 6 ED C hile U ruguay E-mail: secretaria@ctomedicina.com; iberocto@ctomedicina.com WEB: www.ctomedicina.com; www.iberocto.com
ENDOCRINOLOGA
Preparacin Examen de Seleccin 05/06 1 Vuelta Seguimiento a distancia

Pregunta 48. Clnica del Cushing ectpico. Pregunta 54. Diagnstico diferencial de la insuficiencia suprarrenal.

Tumores agresivos Tumores no agresivos


Ej: microctico de pulmn. Ej: carcinoide.

Hiperpigmentacin.
Alteraciones metablicas:
Fenotipo cushingoide (parecido a
Hiperglucemia.
la clnica del Cushing hipofisario)
Alcalosis metablica.
Hipokalemia.

Pregunta 49.- R: 3
El primer paso en el diagnstico de sndrome de Cushing es en
todos los casos la determinacin del hipercortisolismo. Para ello se
usan varias pruebas, pero se suelen emplear las llamadas de screening
por la comodidad en la realizacin y la posibilidad de hacerlo ambu-
latoriamente. Son prcticamente equivalentes y se usan: determina-
cin de cortisol en orina de 24h y cortisol en plasma tras la adminis-
tracin nocturna de 1 mg de dexametasona.

Pregunta 50.- R: 2
Clsicamente se consideraba como prueba definitiva de confirma-
cin del hipercortisolismo a la supresin dbil con 0,5 mg de dexa-
metasona cada 6 horas durante dos das. Ms recientemente otras
pruebas tales como la determinacin de cortisol en plasma o en saliva
entre las 23 y 24 horas de la noche parecen incluso ms eficaces
aunque su prctica no es generalizada an.
Pregunta 55.- R: 3
Pregunta 51.- R: 4 Aparte de lo comentado en la pregunta anterior, otras causas de
Una vez diagnosticado el sndrome de Cushing , el siguiente paso crisis suprarrenal son el debut de una enfermedad de Addison o los
es el diagnstico de la etiologa del mismo. Para ello, la determina- pacientes con dicha patologa que ante el estrs fsico no doblan o
cin de ACTH es el paso siguiente que permite separar el ACTH triplican la dosis de glucocorticoides. Causas ms raras son la hemo-
independiente (suprarrenal) del dependiente (central o ectpico). rragia suprarrenal, en nios por meningococemia diseminada (s.
Otras pruebas tiles son la supresin fuerte con dexametasona y la Waterhouse-Friderichsen) y en adultos, asociada a anticoagulacin.
estimulacin del eje con CRH o metopirona. Ver tabla. La suspensin brusca de AINEs no produce crisis suprarrenal.

Pregunta 52.- R: 5 Pregunta 56.- R: 5


Conviene distinguir las alteraciones metablicas de distintas patolo-
El tratamiento del Cushing es etiolgico, siendo la ciruga transesfe-
gas suprarrenales, ya que pueden ser confundidas en el examen MIR,
noidal de eleccin en los tumores hipofisarios y la extirpacin del
sobre todo si no se tienen los conceptos claros. Para ello recordar que
tumor ectpico o la suprarrenalectoma en caso de adenoma o hiper-
el exceso de mineralcorticoides produce alcalosis e hipopotasemia, ya
plasia suprarrenal. Cuando no se localiza la lesin se procede a blo- que la aldosterona reabsorbe sodio en el tbulo distal intercambindolo
queo qumico de la secrecin de cortisol con ketoconazol, y en caso por potasio e hidrogeniones. No existe hipernatremia en esta situacin
de fracaso del mismo o de aparicin de efectos secundarios se realiza debido al fenmeno de escape que ocurre con el tiempo por la accin
suprarrenalectoma bilateral. de los pptidos natriurticos. Al revs, en los casos de hipoaldosteronismo
Comentarios TEST

(lo que puede ser aislado o formar parte de un Addison) existe


Pregunta 53.- R: 2 hiperpotasemia y acidosis metablica. Hay que recordar que, adems
La causa ms frecuente en la actualidad de enfermedad de Addi- del papel de aclaramiento de agua libre que tienen los glucocorticoides
son o insuficiencia suprarrenal primaria es la adrenalitis autoinmune, en la nefrona, su exceso importante puede activar los receptores tipo 1
en la que una inflamacin crnica de la glndula produce su destruc- renales y producir efecto mineralcorticoide, de ah la posibilidad de
cin y atrofia. Pueden aparecer en estos pacientes anticuerpos alcalosis e hipopotasemia en el Cushing. Dentro de las causas de diabe-
antiadrenales y asociarse a otras patologas autoinmunes. Existen dos tes inspida nefrognica recordar que la hipopotasemia crnica (ej.: en
asociaciones dentro de sndromes poliglandulares autoinmunes es- el hiperaldosteronismo primario) y la hipercalcemia crnica (ej.: en el
pecficos: el tipo 1, en el que existe una mutacin del gen AIRE, hiperparatiroidismo primario) pueden producirla.
autosmica recesiva en el que se asocia adrenalitis autoinmune con
hipoparatiroidismo autoinmune y candidiasis, y el tipo 2 o s. de Pregunta 57.- R: 3
Schmidt, en el que existe una predisposicin gentica asociada a Como se dijo anteriormente, la adrenalitis autoinmune puede aso-
determinados haplotipos del HLA y asocia, adems de adrenalitis ciarse a otras patologas autoinmunes, como en los sndromes
autoinmune, diabetes mellitus autoinmune y enfermedad autoinmu- poliglandulares autoinmunes: el tipo 1, en el que existe una mutacin
ne tiroidea. del gen AIRE, autosmica recesiva, en el que se asocia adrenalitis
autoinmune con hipoparatiroidismo autoinmune y candidiasis y el
Pregunta 54.- R: 2 tipo 2 o s. de Schmidt, en el que existe una predisposicin gentica
La insuficiencia suprarrenal aguda o crisis suprarrenal puede de- asociada a determinados haplotipos del HLA y asocia, adems de
berse a varias causas, siendo la ms frecuente la suspensin brusca de adrenalitis autoinmune, diabetes mellitus autoinmune y enfermedad
un tratamiento glucocorticoideo prolongado. En este caso el paciente autoinmune tiroidea.
tiene clnica de insuficiencia suprarrenal secundaria, en la que existe
hiponatremia por prdida de la accin de los glucocorticoides en la Pregunta 58.- R: 2
nefrona, que fisiolgicamente favorecen el aclaramiento de agua li- En el enunciado se describe un paciente con HTA, hipopotase-
bre. Ver tabla en la pgina siguiente. mia y alcalosis y clnica de diabetes inspida nefrognica. Adems

M exico A rgentina CTO Medicina C/ Nez de Balboa, 115 28006 MADRID (Espaa) Tfno.: (91) 782 43 32 / Fax: (91) 782 43 27
C hile U ruguay E-mail: secretaria@ctomedicina.com; iberocto@ctomedicina.com WEB: www.ctomedicina.com; www.iberocto.com
ED Pg. 7
ENDOCRINOLOGA
Seguimiento a distancia Preparacin Examen de Seleccin 05/06 1 Vuelta
nos indican cifras de aldosterona elevadas que no se suprime (prue- te, el hiperaldosteronismo primario es responsable de un 1-2% de
ba de supresin con infusin de suero salino, administracin de todos los casos de HTA. Por ello, la actitud correcta en estos casos es
fludrocortisona o de captopril) y de renina plasmtica (ARP) dismi- suspender el diurtico, administrar suplementos de potasio y reeva-
nuida que no se estimula (prueba de estimulacin con ortostatismo luar al paciente a los 15 das. Si presenta entonces todava hipopota-
o furosemida). Ello es compatible con un hiperaldosteronismo pri- semia, es preciso un estudio de aldosterona y ARP.
mario, entre cuyas causas estn el aldosteronoma, la hiperplasia
nodular bilateral, el carcinoma suprarrenal (raro) y el hiperaldoste- Pregunta 63.- R: 5
ronismo remediable con glucocorticoides (s. de Sutherland). Las res- La furosemida est contraindicada en el hiperaldosteronismo pri-
puestas 1, 3 y 4 corresponden a excesos de otros mineralcorticoides mario ya que agrava la hipopotasemia y la alcalosis metablica. La
o alteraciones funcionales que aumentan la actividad mineral- ciruga se reserva para los tumores productores de aldosterona, la
corticoidea y en estos casos tanto la aldosterona como la ARP estn espironolactona para el tratamiento preoperatorio de stos o el trata-
disminuidas. En el hiperaldosteronismo secundario, la ARP est ele- miento crnico de la hiperplasia nodular, y los glucocorticoides en el
vada, lo que aumenta la aldosterona. caso del sndrome de Sutherland.

Pregunta 59.- R: 5 Pregunta 64.- R: 2


En circunstancias normales, la regulacin de la aldosterona se rea- Denominamos hirsutismo a la aparicin de vello en la mujer en las
liza por el eje renina-angiotensina y los niveles de potasio en plasma. zonas terminales, es decir, aquellas que son de distribucin de vello
Aunque la ACTH es un estimulador de la produccin de aldosterona, tpicas de varn. El hirsutismo se valora clnicamente por la escala de
fisiolgicamente tiene mucha menos importancia que los dos anterio- Ferriman y se considera que existe cuando la puntuacin es superior
res. En el s. Sutherland o hiperaldosteronismo remediable con a 7. Cuando el hirsutismo es muy rpido en aparicin y se acompaa
glucocorticoides, se produce una translocacin gentica en la que la de virilizacin, hay que sospechar un cncer productor de andrgenos
aldosterona pasa a producirse en las capas internas de la corteza (ovrico si existe exceso grave de testosterona o suprarrenal ante exce-
suprarrenal con el promotor donde se une la ACTH. Ello produce un so severo de DHEA-S). Afortunadamente las causas ms frecuentes son
hiperaldosteronismo que se trata con glucocorticoides para reducir las idiopticas (hormonas normales) y funcionales (ovario poliqustico
los niveles de ACTH plasmticos. o defectos enzimticos de la suprarrenal).
Pregunta 60.- R: 2 Pregunta 65.- R: 5
El cortisol se degrada a cortisona en el rin para evitar su unin a La hiperplasia suprarrenal congnita agrupa las alteraciones genti-
los receptores mineralcorticoideos. La enzima responsable de este cas de la esteroidognesis suprarrenal. La ms frecuente es el dficit de
paso es la 11-OH-esteroide deshidrogenasa. Existen sustancias que 21-hidroxilasa, del que existen dos formas: la forma clsica (neonatal o
inhiben la actividad de esta enzima tales como el regaliz o la carbeno- de aparicin en la niez) y no clsica o tarda (de aparicin peripuberal
xolona, o bien puede estar alterada de forma gentica. En estos casos en la mujer en forma de clnica de hiperandrogenismo).
el cortisol ejerce en concentraciones fisiolgicas efecto
mineralcorticoideo y se produce un sndrome denominado de exce- Pregunta 66.- R: 4
so aparente de mineralcorticoides. El diagnstico de feocromocitoma es hormonal, y para ello realiza-
mos una determinacin conjunta en orina de 24 horas de catecola-
Pregunta 61.- R: 2 minas y metanefrinas. Su elevacin patolgica nos da el diagnstico, y
Dentro de los tipos de hiperplasia suprarrenal congnita ms fre- el paso siguiente sera proceder a la localizacin del tumor, la mayora
cuentes, los que cursan con HTA son el dficit de 11-hidroxilasa y el de de los cuales estn localizados en la suprarrenal (>90%). Para ello
17-hidroxilasa. Ambos producen un exceso de mineralcorticoides que indicamos un TC o RM suprarrenal. La realizacin de una gammagrafa
es la DOCA (deoxicorticosterona), un precursor de la aldosterona con con meta-yodo-bencil-guanidina (MIBG) se recomienda, ya que es la
efecto mineralcorticoide menos intenso. El s. de Conn hace referencia tcnica que mejor localiza metstasis de feocromocitoma o los casos
al aldosteronoma y la respuesta 5 al exceso aparente de de tumor extrasuprarrenal. La prueba de provocacin con glucagn
mineralcorticoides. no se recomienda y la de supresin con fentolamina suele realizarse
cuando es necesario administrar el frmaco ante crisis HTA de los
pacientes.

Comentarios TEST
Pregunta 67.- R: 2
El tratamiento preoperatorio del feocromocitoma consiste en el
bloqueo progresivo y completo de los receptores alfa adrenrgicos,
para lo que se emplea fenoxibenzamina por va oral unos 15 a 21
das antes de la ciruga. El uso de betabloqueantes puede ser necesa-
rio ante la taquicardia refleja de los pacientes o arritmias, sobre todo
en el momento de la induccin anestsica, pero nunca se deben
administrar antes que el alfa ya que se podran inducir crisis
hipertensivas por bloqueo de los receptores beta vasculares que son
vasodilatadores.

Pregunta 68.- R: 4
La gammagrafa con yodo-colesterol es til para la visualizacin de
la patologa de la corteza suprarrenal, pero no tiene ningn papel en
el diagnstico del feocromocitoma. La arteriografa rara vez es necesa-
ria en la actualidad para el diagnstico del feocromocitoma, y debe
realizarse con precaucin.
Pregunta 61. Esteroidognesis suprarrenal.
Pregunta 69.- R: 1
Pregunta 62.- R: 4 La realizacin de una PAAF en un feocromocitoma est contrain-
Desde un punto de vista estadstico, ante un paciente con HTA e dicada; por ello, ante un ndulo o masa suprarrenal, antes de cual-
hipopotasemia, dado que los diurticos tiacdicos y de asa son muy quier otra actitud, debe descartarse la existencia de un feocromocito-
empleados para el tratamiento de la HTA, lo ms probable es que la ma, incluso en el paciente asintomtico, ya que hasta el 10% de los
hipopotasemia sea un efecto secundario de los diurticos. No obstan- feocromocitomas son diagnosticados como incidentales.

M exico A rgentina CTO Medicina C/ Nez de Balboa, 115 28006 MADRID (Espaa) Tfno.: (91) 782 43 32 / Fax: (91) 782 43 27
Pg. 8 ED C hile U ruguay E-mail: secretaria@ctomedicina.com; iberocto@ctomedicina.com WEB: www.ctomedicina.com; www.iberocto.com
ENDOCRINOLOGA
Preparacin Examen de Seleccin 05/06 1 Vuelta Seguimiento a distancia
Pregunta 73.- R: 4
El caso clnico nos muestra la asociacin de hiperplasia de parati-
roides con HTA y cncer de tiroides (medular), es decir un MEN 2a. La
actitud correcta en estos casos, al igual que siempre antes de intervenir
un CMT, es la exclusin de feocromocitoma mediante determinacin
de catecolaminas y metanefrinas en orina de 24h.

Pregunta 73. Asociaciones del MEN 2.

Tipo Sndrome Caractersticas


Carcinoma medular tiroides (90%).
Feocromocitoma (50%).
MEN 2A
Hiperplasia/adenomas paratiroides
(30%).

MEN 2A CMF Carcinoma medular familiar aislado.


Con amilodosis Lesin pruriginosa en dorso
liquenoide cutnea
Con enfermedad de MEN 2A o CMF con agangliosis
Hirschsprung colnica.
CMT, feocromocitoma,
Pregunta 69. Actitud ante un incidentaloma suprarrenal. MEN 2B ganglioneuromatosis intestinal y
MEN 2B mucosa.
Pregunta 70.- R: 4
Tras el estudio de un incidentaloma suprarrenal, segn indica la Marfanoide Idem con hbito marfanoide.
figura de la pregunta anterior, la mayora sern adenomas no funcio-
nantes, en los que la actitud es vigilancia con pruebas de imagen cada Pregunta 74.- R: 4
6-12 meses, y en caso de crecimiento, proceder a la reseccin. Los mecanismos de hipercalcemia tumoral son variados. En los
casos de ostelisis intensa, como las metstasis del cncer de mama o
PATOLOGA DEL CALCIO. el mieloma mltiple, tanto el calcio como el fsforo suelen estar eleva-
dos, al igual que otros marcadores de resorcin sea (fosfatasa alcalina,
Pregunta 71.- R: 2 etc). La produccin ectpica de hormonas por parte del tumor es otra
La causa ms frecuente de hipercalcemia en pacientes ambulatorios causa (hipercalcemia paraneoplsica). La produccin en exceso de
es el hiperparatiroidismo primario que produce una hipercalcemia cr- vit D ocurre en algunos linfomas y la de PTHrp (protena relacionada
nica y asintomtica en la mayora de los pacientes. El diagnstico se con la PTH) en muchos tumores, especialmente epidermoides. En
realiza demostrando niveles de PTHi elevados ante cifras elevadas de este ltimo caso, la PTHi (fisiolgica) est suprimida.
calcio en plasma. La causa ms frecuente es el adenoma nico de
paratiroides. Por otro lado, la hipercalcemia tumoral es aguda y sintomtica, Pregunta 75.- R: 3
y es la causa ms frecuente de hipercalcemia en pacientes ingresados. El tratamiento de la hipercalcemia aguda debe ser urgente. La me-
dida inicial ms eficaz es forzar la diuresis con suero salino IV al que se
Pregunta 72.- R: 1 puede aadir furosemida IV. La calcitonina sc o im tambin es eficaz,
Dado que el hiperparatiroidismo primario produce una hipercal- pero sufre efecto de taquifilaxia en 24-48h. En caso de insuficiencia
cemia crnica y asintomtica en la mayora de los pacientes, y aun- renal grave o riesgo alto de descompensacin cardiovascular por so-
Comentarios TEST

que la extirpacin quirrgica cura la enfermedad, hay que valorar su brecarga de volumen se debe dializar al paciente. Los bifosfonatos IV
realizacin. Las indicaciones actuales son: todos los menores de 50 son muy eficaces, aunque su accin ocurre a las 48-72h y estn
aos, y en los mayores de 50 cuando alguna de las siguientes est contraindicados en la insuficiencia renal. Los corticoides son eficaces
presente: complicaciones, calcio plasma mayor de 12, calcio en ori- en la ostelisis, el mieloma y el exceso de vit D, pero su comienzo de
na de 24h mayor de 400 mg, prdida de masa sea en densitometra accin es a los 3-5 das de su administracin.
mayor de 2 DS. La realizacin de pruebas de localizacin
preoperatorias no es necesaria, pues la exploracin quirrgica es lo DIABETES MELLITUS.
ms eficaz para localizar el adenoma o identificar hiperplasia.
Pregunta 76.- R: 2
Pregunta 72. Indicaciones de ciruga en el Los criterios diagnsticos de DM actuales son los siguientes: 1)
glucemia mayor o igual a 200 en cualquier momento del da, junto
hiperparatiroidismo primario.
con clnica cardinal, 2) glucemia mayor o igual a 126 en ayunas y 3)
Menores de 50 aos. glucemia mayor o igual a 200 a las 2 horas de una sobrecarga oral con
Mayores de 50 aos con alguna de las siguientes: 75 g de glucosa. El diagnstico se establece confirmando cualquiera
- Calcio srico 1,6 mg/dl por encima del nivel mximo del labora- de estos criterios otro da. Existen dos categoras que aumentan el
torio, es decir, mayor de 12 mg/dl en plasma. riesgo de desarrollar DM con el tiempo y son: 1) glucosa en ayunas
- Historia de un episodio de hipercalcemia potencialmente letal. alterada, cuando la glucemia en ayunas est entre 110-125 y 2) into-
- Reduccin del aclaramiento de creatinina en ms del 30%. lerancia a la glucosa, cuando a las 2 horas de una sobrecarga oral con
- Existencia de clculos renales en las Rx abdominales o nefrolitiasis 75 g de glucosa los niveles estn entre 140-199. Estas categoras no
recidivante. requieren confirmacin.
- Calciuria superior a 400 mg/da.
- Reduccin de la masa sea ms de 2 desviaciones tpicas en la Pregunta 77.- R: 4
densitometra. La DM tipo 1 es una enfermedad autoinmune en la que existe una
predisposicin gentica a padecerla asociada a determinados

M exico A rgentina CTO Medicina C/ Nez de Balboa, 115 28006 MADRID (Espaa) Tfno.: (91) 782 43 32 / Fax: (91) 782 43 27
C hile U ruguay E-mail: secretaria@ctomedicina.com; iberocto@ctomedicina.com WEB: www.ctomedicina.com; www.iberocto.com
ED Pg. 9
ENDOCRINOLOGA
Seguimiento a distancia Preparacin Examen de Seleccin 05/06 1 Vuelta
haplotipos del HLA (cromosoma 6). Factores diversos tales como in-
fecciones virales o consumo precoz de leche de vaca, as como otros Pregunta 80. Diferencias entre cetoacidosis diabtica y
no conocidos, inducen el proceso autoinmune. Tanto la inmunidad descompensacin hiperglucmica hiperosmolar.
celular, que produce un infiltrado de clulas mononucleares en los
islotes pancreticos (insulitis), como la inmunidad humoral Caractersticas CAD DHH
(anticuerpos) se activan. Los anticuerpos producidos son varios, aun-
que la mayor rentabilidad diagnstica se consigue con la determina- Depende de la ca usa
Mortalidad Actua lm ente 1- 2%. desenca dena nte. En
cin conjunta de anti-IA2 y GAD.
gener a l un 15%.
No necesa r ia pa r a
Pregunta 77. Autoanticuerpos en la diabetes mellitus tipo I. Siem pr e necesa r ia pa r a
r ever tir el cua dr o per o
Requerimient- r ever tir el cua dr o.
a cor ta el tr a ta m iento.
os de ins ulina 0,1 UI/K g/h en
Nomenclatura Antgeno Utilidad Se suele indica r a
per fusin continua .
dosis m enor.
Molcula intacta de insulina Baja rentabilidad
INS-Ab Inicia lm ente SS M s ca ntida d de
del paciente diagnstica fisiolgico, cua ndo volum en (ha sta 10%
Sueroterapia
glucem ia 250- 300 de pr dida s), glucosa do
Anticuerpos frente a clulas Sensibilidad moderada en
ICAs a por ta r SG 5%. e n f a s e s f in a l e s .
de los islotes pancreticos el diagnstico
Cua ndo el pota sio en
Decarboxilasa del cido M s pr ecozm ente sa lvo
GAD Suplementos pla sm a nor m a l o
glutmico Conjuntamente ofrecen la en ca sos de FRA
de pot a s io dism inuido.
mayor rentabilidad pr er r ena l.
Esper a r 2 h si K >5,5- 6.
Anticuerpos frente a diagnstica. Los usados
IA2 fosfatasa de tirosina-kinasa actualmente Slo si a cidosis l ctica
Bicarbonato Cua ndo pH<7,0.
de clula beta concom ita nte.

Pregunta 78.- R: 4 Pregunta 81.- R: 1


La DM tipo MODY (Maturity Onset Diabetes of the Young) se debe El objetivo en el tratamiento de la diabetes es evitar las complica-
a un conjunto de alteraciones enzimticas de la clula beta pancre- ciones agudas, y sobre todo evitar las complicaciones crnicas micro
tica que altera la secrecin de insulina. Produce una DM leve de y macrovasculares. Diversos estudios han relacionado claramente
curso ms benigno que otros tipos de DM, aunque vara segn el tipo los niveles elevados de glucemia con las complicaciones crnicas,
de enzima alterada. Es una enfermedad autosmica dominante, de sobre todo microangiopticas (DCCT, UKPDS, etc.). Por ello monito-
comienzo antes de los 25 aos de edad. El tipo ms frecuente en rizamos a los pacientes con niveles de glucemia y de otros parme-
Espaa es el MODY 2, por alteracin en el gen de la glucoquinasa. El tros que valoran el control a medio-largo plazo (HbA1c o glicada
tratamiento inicial es con dieta y ejercicio, y si no hay buen control, nos da el control de los 2-3 meses previos, fructosamina de las 2-3
con sulfonilureas y eventualmente insulina. sem previas). La glucosuria no es til, ya que el dintel plasmtico
para que aparezca glucosa en orina es de 180-200, y adems trans-
Pregunta 79.- R: 5 gresiones dietticas aisladas pueden producirla sin efecto en el con-
La cetoacidosis diabtica (CAD) es la descompensacin aguda t- trol crnico.
pica de la DMID. Se define por la presencia de cetoacidosis (pH<7,30
con cetosis) e hiperglucemia (generalmente >300). Tiene una morta- Pregunta 81. Objetivos de control metablico de la diabetes.
lidad actual del 1-2%. La fisiopatologa consiste en un dficit de insu-
lina, junto a un aumento de hormonas contrarreguladoras, sobre
Valores deseables Valores aceptables
todo glucagn. Ello aumenta la cetognesis heptica, la hipergluce-
mia y la acidosis metablica. La clnica suele ser inespecfica: nu- HbA1c < 6,5% < 7,5%
seas, dolor abdominal, obnubilacin que se correlaciona con osmo-

Comentarios TEST
laridad elevada y la clnica cardinal de diabetes previa y de la causa Glucemias en ayunas 80-110 110-140
precipitante suelen estar presentes. La CAD no se puede revertir sin Glucemia postpandrial
insulina, siendo el objetivo la saturacin completa y rpida de los < 140 140-180
a las 2h
receptores de insulina del organismo, manteniendo dicho tratamien-
to hasta revertir el cuadro, para lo que usamos insulina rpida IV. La
reposicin de lquidos se realiza comenzando con SS fisiolgico, y Pregunta 82.- R: 1
cuando la glucemia sea de 250-300 se aporta suero glucosado al 5% Ante una hiperglucemia matutina en un diabtico en tratamiento
para evitar hipoglucemia y edema cerebral. El potasio debe aportarse con insulina caben bsicamente dos posibilidades: 1) fenmeno del
con cifras bajas o normales de entrada y el bicarbonato slo cuando alba debido a los picos de secrecin nocturna de GH y 2) efecto Somogyi
el pH es inferior a 7,0. por aumento de hormonas contrarreguladoras tras una hipoglucemia
de madrugada. El diagnstico diferencial se realiza con una determina-
Pregunta 80.- R: 1 cin glucmica a las 3 a.m. (normal en el alba y disminuida en Somogyi).
La descompensacin hiperglucmica hiperosmolar es la descom- El primer caso se trata aumentando la insulina NPH de la cena y el
pensacin aguda tpica de la DMNID. La mortalidad es todava eleva- segundo disminuyendo la insulina de la cena.
da, alrededor del 15%. Se define como una situacin de hipergluce-
mia mayor de 600 mg/dL, junto con una osmolaridad superior a 320 Pregunta 83.- R: 3
mosm/L. Buscar siempre una causa precipitante. La clnica suele ser El tratamiento de la diabetes tipo 2 se basa en una actitud escalona-
inespecfica: estupor, obnubilacin, incluso coma, junto con da que comienza por la dieta y el ejercicio. El segundo paso es la
monoterapia con antidiabticos orales (ADO) hasta la dosis mxima del
hipotensin y signos de deshidratacin. Lo ms importante para re-
frmaco. Si no se controlara bien el paciente el tercer escaln es la
vertir la situacin de DHH es la administracin de lquidos intraveno-
asociacin de ADOs. En caso de no controlarse as, llegara la insuliniza-
sos, ya que los pacientes pueden tener una prdida hdrica de hasta el
cin, empezando por monodosis nocturna de insulina NPH y mante-
10% de su peso. Por lo dems, se usa un protocolo similar al de la
niendo ADOs durante el da. Es importante tener en cuenta la existen-
cetoacidosis diabtica. cia o no de obesidad a la hora de elegir los frmacos.

M exico A rgentina CTO Medicina C/ Nez de Balboa, 115 28006 MADRID (Espaa) Tfno.: (91) 782 43 32 / Fax: (91) 782 43 27
Pg. 10 ED C hile U ruguay E-mail: secretaria@ctomedicina.com; iberocto@ctomedicina.com WEB: www.ctomedicina.com; www.iberocto.com
ENDOCRINOLOGA
Preparacin Examen de Seleccin 05/06 1 Vuelta Seguimiento a distancia
Pregunta 84.- R: 5
1er escaln DIETA Y EJERCICIO
La glimepirida es una sulfonilurea de vida media larga que se toma
en monodosis diaria. El miglitol, como la acarbosa, es un inhibidor de
las disacaridasas intestinales. La metformina es una biguanida que dis-
Obesidad Peso normal o delgados minuye la resistencia insulnica. La repaglinida es una meglitinida que
produce secrecin rpida de insulina. Ver tabla en pgina siguiente.

Pregunta 85.- R: 3
2 escaln ADO MONOTERAPIA Otros factores adems de la hiperglucemia se han demostrado inclu-
so ms importantes en la produccin de las complicaciones macroan-
giopticas: dislipemia, hipertensin y tabaquismo. Las recomendacio-
Metformina Sulfonilureas repaglinida nes actuales incluyen mantener la TA inferior a 130/80 mmHg, LDL
inferior a 130 -incluso recientemente se aconseja inferior a 100- y
niveles de triglicridos inferiores a 150 y HDL superior a 45, junto con
la abstinencia de fumar. Los IECAs se consideran la primera lnea de
3er escaln ASOCIACIN ADOS
tratamiento de la HTA en los diabticos por su efecto nefroprotector
(aunque recientemente los ARA-2 tambin lo han demostrado).

SU glinidas glitazonas I.D.I Metformina I.D.I Pregunta 86.- R: 5

Pregunta 86. Tipos de insulina.


4 escaln INSULINIZAR
Tipo Inicio Pico Duracin

Prolongada 1,5-3 h 10-16 h 24-30 h


Inicio con monodosis Idem.
nocturna de insulina Si no hay control pasar NP H o
1-1,5 horas 4-6 h 10-12 h
manteniendo ADO . a dos dosis de insulina. intermedia
Si no hay control pasar No necesario mantener SU . R egular 30 minutos 2-3h 5-6 h
a pauta de dos dosis
manteniendo M etformina. Anlogos 10 minutos 45 minutos 2-3 horas

Pregunta 83. Intervencin teraputica en la diabetes mellitus tipo 2. SU: sulfo-


nilureas; IDI: inhibidores de las disacaridasas intestinales. La insulina asprtica, ya disponible en nuestro pas, es un anlogo de
insulina que, al igual que la insulina lispro, no forma hexmeros en el

Pregunta 84. Antidiabticos orales.

Inhibidore s de la
Sulfonilure as Me glitinidas Biguanidas Tiazolidine dionas
alfaglucosidasa
1 Generacin:
Clorpropamida.
Tolazamida.
Acetohexamida
2 Generacin:
R epaglinida. Acarbosa. Pioglitazona.
Comentarios TEST

Antidiab tico oral Glibenclamida. Metformina


Nateglinida. Miglitol. R osiglitazona.
Glipizida.
Gliburide.
G l iq u ido n a .
3 Generacin:
Glimepiride.
Regula la secrecin de Ins Disminuye la insulino-
Estimulan la
Me canismo de actuando en receptores Disminuye la insulino- Inhiben las alfa- rresistencia perifrica
secrecin de
accin diferentes a los de las rresistencia central (hgado) glucosidasas (msculo, grasa) actuando
insulina (Ins)
sulfonilureas en receptores PPAR -gamma
Gastrointestinal
Hipoglucemia (es lo ms frecuente). Hepatotoxicidad.
Efe ctos adve rsos Hipoglucemia (escasa) Gastrointestinal
(severa y duradera) Acidosis lctica R etencin hdrica.
(es lo ms grave).
Favorecen acidosis lctica
Alcohol.
Embarazo. Embarazo. Enfermedad aguda Qx.
Embarazadas.
Contraindicado Hepatopata. Hepatopata. Insuficiencia cardaca. Hepatopata
N i o s .
Nefropata. IR C severa. Insuficiencia respiratoria.
Hepatopata.
Nefropata.
1. Monoterapia
De eleccin en DM con De eleccin en
Uso DM2 no obesos (poco eficaz). Asociar a Metformina
hiperglucemia postprandial DM-2 obeso
2. Asociar a otros ADO.

M exico A rgentina CTO Medicina C/ Nez de Balboa, 115 28006 MADRID (Espaa) Tfno.: (91) 782 43 32 / Fax: (91) 782 43 27
C hile U ruguay E-mail: secretaria@ctomedicina.com; iberocto@ctomedicina.com WEB: www.ctomedicina.com; www.iberocto.com
ED Pg. 11
ENDOCRINOLOGA
Seguimiento a distancia Preparacin Examen de Seleccin 05/06 1 Vuelta
tejido subcutneo tras su inyeccin, y por esta razn, se absorbe ms o anlogos de la somatostatina (octretide) en algunos pacientes. En
rpidamente. Tiene las ventajas de producir menos hipoglucemias que la caso de insulinoma maligno, la quimioterapia con estreptozocina y
insulina rpida y se inyecta en el preciso momento de empezar a comer. doxorrubicina parece la ms eficaz. En casos de metstasis hepticas
exclusivamente es posible la quimioembolizacin de las mismas.
Pregunta 87.- R: 5
El tratamiento intensivo con insulina ha demostrado ser claramente
superior al convencional para la prevencin de las complicaciones DISLIPEMIA, OBESIDAD Y NUTRICIN.
crnicas de la DM. El lmite al tratamiento intensivo es el problema de
las hipoglucemias. Por ello no se recomienda el tratamiento intensivo Pregunta 91.- R: 4
en los casos en los que las hipoglucemias son menos deseables por sus Los objetivos de tratamiento en una dislipemia dependen de los
graves consecuencias que un control menos perfecto. factores de riesgo cardiovascular que tenga el paciente, y la prioridad
son los niveles de colesterol LDL. Los factores de riesgo cardiovascular
principales son: edad > 45 aos en varones, climaterio en mujeres,
Pregunta 87. El tratamiento insulnico intensivo no debe ser HTA, tabaco, diabetes mellitus, enfermedad cardiovascular precoz en
recomendado en los siguientes casos: familiares de primer grado. El objetivo de LDL es: 1) menor de 100 en
cardiopata isqumica, diabetes y otras formas de aterosclerosis seve-
Nios menores de 8 aos, por el efecto deletreo que la hipoglu-
ra, 2) menor de 130 con 2 o ms factores de riesgo cardiovascular y 3)
cemia puede tener sobre el desarrollo cognitivo.
menor de 160 con 1 o ningn factor de riesgo cardiovascular.
Diabticos con neuropata autonmica severa, por el riesgo de
sufrir hipoglucemias inadvertidas.
Pregunta 92.- R: 3
Pacientes con trastornos mentales graves, que no pueden respon-
La dislipemia diabtica se caracteriza por HDL disminuida con
sabilizarse de un tratamiento intensivo.
LDL normal o levemente elevada con predominio de las formas pe-
Ancianos.
queas y densas (ms aterognicas) y triglicridos elevados, sobre todo
Cardipatas o pacientes con antecedentes de accidentes cerebrovas-
en caso de mal control glucmico. En el tratamiento, la prioridad es la
culares, en los que la hipoglucemia puede tener consecuencias serias.
correccin del nivel de LDL (menor de 100). La cerivastatina ha sido
retirada debido a la alta incidencia de rabdomilisis en combinacin
con gemfibrocilo.
HIPOGLUCEMIAS.
Pregunta 93.- R: 5
Pregunta 88.- R: 1 Las estatinas tienen como mecanismo de accin la inhibicin de
Las hipoglucemias se dividen en su estudio en las de ayunas y las la HMG CoA reductasa y disminuyen las concentraciones de LDL.
reactivas o postprandiales. Las primeras pueden ser a su vez hiper o Los fibratos actan sobre los receptores PPAR alfanucleares y au-
hipoinsulinmicas. Esta pregunta hace referencia a las hipoglucemias mentan la oxidacin de cidos grasos en el hgado y msculo, por lo
de ayuno hiperinsulinmicas, en las que el cociente insulina/glucemia que disminuyen la secrecin heptica de lipoprotenas ricas en TGs.
est elevado. Para su diagnstico diferencial es til la determinacin Aumentan la actividad de la lipoproteinlipasa (LPL) muscular, favo-
de pptido C (indica exceso de insulina endgena) y de proinsulina reciendo el metabolismo de los triglicridos. Son los frmacos
(elevada en el insulinoma). hipotrigliceridemiantes ms eficaces. Las resinas disminuyen la ab-
sorcin de colesterol y reducen la LDL, pero aumentan los niveles
de TGs. El cido nicotnico inhibe la movilizacin de cidos grasos
Pregunta 88. Diagnstico diferencial de las de los tejidos. Tambin aumenta la HDL de forma importante.
hipoglucemias de ayuno hiperinsulinmicas.
Pregunta 93. Hipolipemiantes.
PEPTIDO C PROINSULINA SU EN ORINA
Resinas
Insulina cido
- Estatinas Fibratos fijadoras de
nicotnico
ex gena cidos biliares

Comentarios TEST
Sulfonilureas Normal + Atorvastatina.
Lovastatina.
Principios Fenofibrato. Colestiramina.
Insulinoma - activos
Simvastatina.
Gemfibrocilo. Colestipol.
Niacina
Pravastatina.
Fluvastatina.
Pregunta 89.- R: 4
La causa ms frecuente de hipoglucemia en pacientes ingresados Efe cto
Colesterol Triglicridos Colesterol Triglicridos
principal
son los frmacos, la mayora insulina y ADOs (sulfonilureas). Otros
frmacos posibles son: pentamidina, aspirina, betabloqueantes, etc. sntesis de lipoprotein- Circulacin
Algunos tumores (mesenquimales, retroperitoneales) pueden produ- colesterol lipasa (aumenta enteroheptica
Mecanismo sntesis de
cir hipoglucemias de ayuno hipoinsulinmicas por produccin ect- de accin
(por inhibicin hidrlisis de de ac. biliares:
VLDL y LDL
pica de sustancias como la IGF-2. En otras ocasiones, las hipogluce- de la HMG TGs) sntesis la absorcin de
mias de ayuno hipoinsulinmicas son por deplecin de glucgeno, CoA reductasa) VLDL colesterol
como en las hepatopatas severas, desnutricin, sepsis, enfermedades Gastrointestinal Gastrointestinal Sofocos
del metabolismo (glucogenosis, etc). transaminasas L it ia s is b il ia r (nuseas, Prurito
Efe ctos
Miopata Miopata vmitos, Got a
Pregunta 90.- R: 5 adve rsos
Cataratas metabolismo estreimiento, Diabetes
El insulinoma es, tras el gastrinoma, el tumor neuroendocrino de vitamina K esteatorrea) mellitus
gastroenteropancretico ms frecuente. La mayora son solitarios y
benignos y se curan con ciruga. En los insulinomas localizados en
cabeza o cuerpo de pncreas, el procedimiento empleado es la enu- Pregunta 94.- R: 4
cleacin, mientras que los de la cola de pncreas se resecan con Los efectos adversos ms frecuentes son las molestias digestivas, los
pancreatectoma caudal. Para el control de las hipoglucemias antes dolores musculares y la hepatitis (sta en < 1%). La miopata, la
de la ciruga o en caso de recidiva o malignidad se emplea diazxido neuropata y el exantema son raros. La incidencia de miopata au-

M exico A rgentina CTO Medicina C/ Nez de Balboa, 115 28006 MADRID (Espaa) Tfno.: (91) 782 43 32 / Fax: (91) 782 43 27
Pg. 12 ED C hile U ruguay E-mail: secretaria@ctomedicina.com; iberocto@ctomedicina.com WEB: www.ctomedicina.com; www.iberocto.com
ENDOCRINOLOGA
Preparacin Examen de Seleccin 05/06 1 Vuelta Seguimiento a distancia
menta de forma importante al asociarse con fibratos. Las estatinas nutricionalmente y aceptada por el paciente. El objetivo es la prdida
interactan con muchos frmacos que son metabolizados por el cit P- progresiva, moderada y mantenida del peso de un 10%.
450 (excepto la pravastatina), lo que debe tenerse en cuenta. No se
pueden usar en nios ni embarazo. Pregunta 100.- R: 4
Las indicaciones de la ciruga de la obesidad son: obesidad mrbi-
Pregunta 95.- R: 5 da que ha fracasado en tratamientos no quirrgicos, sobrepeso del
La desnutricin se clasifica en 3 tipos en los que se alteran determi- 200% mantenido ms de 5 aos, ausencia de trastornos psiquitricos,
nados parmetros: 1) Calrica o marasmo. Alteracin de variables riesgo quirrgico y anestsico aceptable. Tambin ante la presencia
antropomtricas y no de protenas. 2) Proteica o Kwashiorkor. Altera- de complicaciones graves de la obesidad mrbida en la de grado 2.
cin de protenas viscerales, con variables antropomtricas en lmites
normales. 3) Mixta, la ms frecuente en el hospital en la que se alteran
ambos parmetros. A veces la valoracin nutricional est limitada por
la enfermedad del paciente (el peso no sirve en un asctico, la albmi-
na en un politrauma, etc).

Pregunta 95. Valoracin de la desnutricin.

Calrica o marasmo Proteica o Kwashiorkor Mixta

Alteraciones Alteraciones de proteinas


Ambas
antropomtricas plasmticas
1. Peso: 1. Compartimento 1. Combinacin de
- % disminucin del muscular. las anteriores.
peso previo. - B alance nitrogenado. 2. Disminucin de
- % disminucin del - ndice creatinina-altura. los linfocitos
peso ideal. 2. Compartimento visceral. totales*.
- Velocidad de a. Proteinas vida media 3. Anergia cutnea*.
prdida de peso. intermedia.
- IMC <18,5 (severa - Albmina (20 das).
si <16). - Transferrina (10 d).
2. Pliegues cutneos b. Proteinas vida media
(tricipital, etc). corta.
3. Circunferencia - Prealbmina (2 das).
media braquial. - Proteina ligadora de
retinol (10 h).
* Se alteran en cualquier tipo de desnutricin e indican que es de grado severo.
Si linfos <800 y anergia cutnea.

Pregunta 96.- R: 4
Las indicaciones de la nutricin enteral son: alimentacin oral
menor del 50% en ms de 7 das con tracto GI funcionante, nutricin
precoz en estrs grave con tracto GI funcionante, intestino corto
funcionante de 100-200 cms, disfagia alta severa, fstula enterocut-
nea de bajo dbito. Contraindicada en obstruccin intestinal, reposo
intestinal absoluto, tracto GI no funcionante.
Comentarios TEST

Pregunta 97.- R: 2
Las complicaciones de la nutricin parenteral son las alteraciones
hidroelectrolticas que obligan a la realizacin de monitorizacin
analtica cada 24-48h, la posibilidad de inducir hiperglucemia o este-
atosis heptica por el contenido de glucosa de las soluciones, la
sobrealimentacin con alta mortalidad, si comenzamos con exceso
de caloras, y sobre todo el riesgo de infeccin de la va central, que es
lo que suele limitar su uso en la mayora de los casos. Los vmitos slo
aparecen cuando se dan alteraciones hidroelectrolticas graves.

Pregunta 98.- R: 5
La clasificacin actual segn el IMC deja el lmite inferior del peso
normal rebajado de 20 a 18,5 kg/m2, de acuerdo con las recientes
recomendaciones internacionales. La amplia gama del sobrepeso (IMC
= 25-29,9 kg/m2) en el que est incluida una gran parte de la pobla-
cin adulta, se divide en 2 categoras (grado 1 de 25 a 26,9 y grado 2
de 27 a 29,9). Se define obesidad a partir de IMC de 30 en cuatro
categoras: 1) obesidad grado 1 entre 30-34,9, 2) obesidad grado 2
entre 35-39,9, 3) obesidad mrbida o severa entre 40-49,9 y 4) obe-
sidad extrema si ms de 50.

Pregunta 99.- R: 3
La dieta es el primer escaln de tratamiento de la obesidad, junto
con el ejercicio fsico moderado. Debe ser hipocalrica, equilibrada

M exico A rgentina CTO Medicina C/ Nez de Balboa, 115 28006 MADRID (Espaa) Tfno.: (91) 782 43 32 / Fax: (91) 782 43 27
C hile U ruguay E-mail: secretaria@ctomedicina.com; iberocto@ctomedicina.com WEB: www.ctomedicina.com; www.iberocto.com
ED Pg. 13
ESTADSTICA
Preparacin Examen de Seleccin 05/06 1 Vuelta Seguimiento a distancia

ESTADSTICA. 1) La media es igual a la moda, y sta, igual a la mediana.


2) El 50% de las observaciones son mayores que la moda.
1. Seale cul de las siguientes es una variable cuantitativa 3) Aproximadamente el 68% de las observaciones caen
discreta: dentro de 1 desviacin estndar de la media.
4) El nmero de observaciones entre 0 y 1 desviacin estn-
1) Glucemia basal. dar de la media es el mismo que entre 1 y 2 desviaciones
2) Sexo. estndar de la media.
3) Nmero de camas. 5) El valor que ms se repite es la media.
4) Color de pelo.
5) Respuesta a tratamiento analgsico. 8. Se sabe que el 50% de la poblacin fuma, y que el 10% fuma
y es hipertensa. Cul es la probabilidad de que un fumador
2. Qu tipo de variable es la graduacin del coma de la escala sea hipertenso?:
de Glasgow?:
1) 15%.
1) Cualitativa dicotmica. 2) 80%.
2) Cuantitativa discreta. 3) 40%.
3) Cualitativa ordinal. 4) 20%.
4) Cuantitativa ordinal. 5) 25%.
5) Cuantitativa continua.
9. Sabemos que la enfermedad X causa la muerte al 20% de los
3. En nueve familias estudiadas, el nmero de hijos por familia afectados en el primer ao. Si tenemos dos pacientes con esa
era de 4, 6, 2, 2, 4, 3, 2, 1, 7. La media, la mediana y moda de enfermedad, cul es la probabilidad de que mueran ambos
hijos por familia es: en el primer ao?:

1) 3,4; 2; 3. 1) 2%.
2) 3; 3,4; 2. 2) 4%.
3) 3; 3; 2. 3) 10%.
4) 2; 3,5; 3. 4) 40%.
5) 3,4; 3; 2. 5) 52%.

4. En la siguiente distribucin: 16, 12, 13, 14, 50, qu medida 10. En un colegio de 1.000 alumnos, 200 nios padecen asma y
de centralizacin elegira?: 100 padecen dermatitis atpica. De los pacientes asmticos,
150 no padecen dermatitis. Cul es la probabilidad de que,
1) Recorrido. escogido un individuo al azar, sea asmtico o atpico?:
2) Media aritmtica.
3) Desviacin tpica. 1) 20%.
4) Mediana. 2) 25%.
5) Coeficiente de variacin. 3) 30%.
4) 40%.
5. Cul de los siguientes parmetros es de dispersin?: 5) 45%.

1) Media aritmtica. 11. La inferencia estadstica es:


2) Moda.
3) Percentil 50. 1) La descripcin de los resultados obtenidos en la muestra.
4) Rango. 2) La descripcin de los resultados obtenidos en la poblacin.
Preguntas TEST

5) Mediana. 3) La generalizacin de los resultados obtenidos en la mues-


tra.
6. Respecto al coeficiente de variacin, una de las siguientes 4) La generalizacin de los resultados obtenidos en la pobla-
afirmaciones es FALSA: cin.
5) La estimacin de la media muestral a partir de la media
1) Se emplea para comparar la variabilidad relativa de dife- poblacional.
rentes distribuciones.
2) Se aplica partiendo del problema de que las desviaciones 12. Se ha estudiado una muestra de 100 personas, obtenindose
tpicas no son comparables directamente. una glucemia basal media de 80 mg/ml, con una desviacin
3) Puede decirse que los valores no son homogneos si la estndar de 10 mg/ml. La glucemia basal media de la pobla-
desviacin tpica supone ms de un tercio de la media. cin ser:
4) El coeficiente de variacin permite comparar la variabili-
dad relativa de dos muestras. 1) 80 2 mg/ml.
5) No puede utilizarse para comparar dos muestras expresa- 2) 80 20 mg/ml.
das en distintas unidades. 3) 80 2 mg/ml, p <0,05.
4) 80 2 mg/ml, p <0,01.
7. Todas las siguientes afirmaciones con respecto a la distribu- 5) 80 3 mg/ml, p <0,01.
cin normal son ciertas, EXCEPTO:

M exico A rgentina CTO Medicina C/ Nez de Balboa, 115 28006 MADRID (Espaa) Tfno.: (91) 782 43 32 / Fax: (91) 782 43 27
C hile U ruguay E-mail: secretaria@ctomedicina.com; iberocto@ctomedicina.com WEB: www.ctomedicina.com; www.iberocto.com
ET Pg. 1
ESTADSTICA
Seguimiento a distancia Preparacin Examen de Seleccin 05/06 1 Vuelta
13. Leemos en un artculo publicado que la frecuencia cardaca 2) 195-205.
media de los varones fumadores es de 70 5 lpm, con p 3) 150-250.
<0,05. Esto significa: 4) 199,5-200,5.
5) 175-225.
1) La probabilidad de que la frecuencia cardaca media
obtenida en la muestra no sea 70 lpm es menor de 0,01. 18. En una muestra de 100 mujeres, la media de peso fue de 65
2) El 95% de los sujetos muestreados est entre 60 y 80 lpm. kg con una desviacin tpica de 5 kg. Cul es el intervalo de
3) La probabilidad de que la frecuencia cardaca media de confianza del 95 % para la media poblacional?:
los varones fumadores est entre 65 y 75 lpm es menor de
0,05. 1) 60 - 70.
4) La probabilidad de que la frecuencia cardaca media de 2) 55 - 75.
los varones fumadores no est entre 65 y 75 lpm es menor 3) 64 - 66.
de 0,05. 4) 63 - 67.
5) Se tiene un 95% de confianza de que el verdadero valor 5) 62 - 68.
poblacional est entre 60 y 80 lpm.
19. En una revista mdica se publica un artculo en el que se afirma
14. En una muestra de 100 nios se encuentra una colesterole- que el colesterol medio de los pacientes obesos mayores de 45
mia media de 200 mg/dl. Usted ha ledo que en la poblacin aos es 260 10 mg/dl, p<0,05. Esto significa que:
peditrica la colesterolemia media es 196 - 204, p<0,05.
Seale qu intervalo incluye al 95% de los nios de su 1) La probabilidad de que el colesterol medio obtenido en
muestra: la muestra no sea 260 es menor de 0,05.
2) El 95% de los sujetos muestreados tiene una colesterole-
1) 196 - 204. mia entre 240 y 280 mg/dl.
2) 180 - 220. 3) La probabilidad de que el colesterol medio de los pacien-
3) 160 - 240. tes obesos mayores de 45 aos no est entre 250 y 270 es
4) 198 - 202. menor del 5%.
5) 170 - 230. 4) La probabilidad de que el verdadero valor medio pobla-
cional est entre 240 y 280 es del 95%.
15. En una muestra de 100 pacientes se ha observado que un 5) El 95% de los sujetos muestreados tiene un colesterol entre
tratamiento produce una disminucin media de la colestero- 250 y 270.
lemia de 2 mmol/l, con un error estndar de la media de 0,5
mmol/l. Cul de las siguientes afirmaciones es correcta?: 20. En una muestra de 37 pacientes se realiza determinacin de
la uricemia, obtenindose una media de 7 y una desviacin
1) El 95% de los sujetos disminuyeron entre 1,5 y 2,5 mmol/ estndar de 1. La estimacin de la media poblacional (p
l. <0,05) ser:
2) El 95% de los sujetos disminuyeron entre 1 y 3 mmol/l.
3) Se tiene un 95% de confianza de que el verdadero 1) 7 2.
descenso se site entre 1,5 y 2,5 mmol/l. 2) 7 1.
4) Se tiene un 95% de confianza de que el verdadero 3) 7 0,16.
4) 7 0,32.
descenso se site entre 1 y 3 mmol/l.
5) 7.
5) El 95 % de los pacientes con hipercolesterolemia conse-
guirn descensos entre 1 y 3 mmol/l a ese grado de
21. El error alfa o tipo I es:
significacin.
1) No se rechaza la hiptesis alternativa, siendo cierta.
16. El error estndar de la media se puede reducir a la mitad:
2) Se rechaza la hiptesis alternativa, siendo cierta.

Preguntas TEST
3) No se rechaza la hiptesis nula, siendo cierta.
1) No es posible estudiando la misma variable, pero s
4) Se rechaza la hiptesis nula, siendo cierta.
podemos reducir a la mitad el intervalo de confianza con
5) Se clasifica a un enfermo como tal.
un tamao muestral ms pequeo.
2) Reduciendo el tamao muestral a la mitad. 22. En un contraste de hiptesis, la probabilidad de aceptar la
3) Seleccionando muestras con una desviacin tpica 2 hiptesis nula, siendo falsa es:
veces superior.
4) Aumentando el tamao muestral 4 veces. 1) Es la decisin correcta.
5) Reduciendo la probabilidad de error tipo II a la mitad. 2) Es el error tipo II o beta.
3) Es la potencia del test.
17. Se ha obtenido que en el intervalo (175-225), est incluido 4) Es el error tipo I o alfa.
el 68% central de una muestra de diabticos. Si el tamao 5) Es nula siempre.
muestral es 100 y la media 200, calcule un intervalo en el que
se incluya el verdadero valor medio poblacional con una 23. La capacidad de un test de encontrar diferencias, cuando en
probabilidad del 95%: realidad las hay:

1) 197,5-202,5. 1) Es imprescindible.

M exico A rgentina CTO Medicina C/ Nez de Balboa, 115 28006 MADRID (Espaa) Tfno.: (91) 782 43 32 / Fax: (91) 782 43 27
Pg. 2 ET C hile U ruguay E-mail: secretaria@ctomedicina.com; iberocto@ctomedicina.com WEB: www.ctomedicina.com; www.iberocto.com
ESTADSTICA
Preparacin Examen de Seleccin 05/06 1 Vuelta Seguimiento a distancia

2) Es el error tipo I. 2) 1%.


3) Es el error tipo II. 3) 95%.
4) Es el poder o potencia del test. 4) 68%.
5) Es generalmente de 0,05. 5) 99%.

24. Al comparar la diferencia entre dos medias, se encuentra un 29. El resultado de un contraste de hiptesis no es estadstica-
valor de la p=0,6. La interpretacin correcta del resultado es: mente significativo. Indique la FALSA:

1) Se rechaza la hiptesis nula. 1) No puede rechazarse la hiptesis nula.


2) La diferencia es estadsticamente significativa. 2) La probabilidad de que el resultado sea explicable por el
3) La diferencia no se debe al azar. azar es mayor del 5%.
4) La diferencia es compatible con la hiptesis nula. 3) Podra haberse conseguido un resultado estadsticamente
5) La variacin en el muestreo no es una explicacin proba- significativo con un tamao muestral mayor.
ble de la diferencia. 4) No hay diferencias.
5) Las diferencias no exceden lo atribuible al azar.
25. Al tratar una artritis psorisica con azatioprina, mejora el
60% de los pacientes; la 6-mercaptopurina mejora a un 55%. 30. Se quiere comparar un nuevo tratamiento con otro convencio-
La diferencia es significativa, p<0,05, lo cual se debe inter- nal, para lo que se realiza un contraste de hiptesis, fijando un
pretar como que: error alfa del 1% y un error beta del 3%. Cul es la probabilidad
de que, existiendo las diferencias, stas se detecten?:
1) Con azatioprina mejorarn el 60% de tus pacientes.
2) La azatioprina es en un 95% mejor que la 6-mercaptopu- 1) 90%.
rina. 2) 95%.
3) El nivel de significacin es del 1%. 3) 97%.
4) Hay diferencias entre los dos tratamientos, a ese nivel de 4) 98%.
significacin. 5) 99%.
5) No se puede concluir nada.
31. Un estudio epidemiolgico longitudinal concluye que existe
26. Se efectu un estudio para valorar un nuevo procedimiento una asociacin estadsticamente significativa entre un factor
quirrgico planeado para disminuir la frecuencia de compli- de riesgo y determinada enfermedad con un nivel de signifi-
caciones postoperatorias. La frecuencia de complicaciones cacin p <0,05. Cul es la conclusin correcta?:
fue del 40% en 25 pacientes sometidos al nuevo procedi-
miento, y del 60% en 20 pacientes en quienes se practic el 1) En caso de no existir diferencias entre expuestos y no
procedimiento antiguo. La diferencia NO es estadsticamen- expuestos al factor de riesgo, la probabilidad de observar
te significativa. Cabe deducir: por azar los resultados obtenidos es menor del 5%.
2) Concluimos que existe asociacin real, ya que se ha
1) El nuevo procedimiento es eficaz para disminuir las com- demostrado asociacin estadsticamente significativa.
plicaciones postoperatorias. 3) Hasta el 95% de los expuestos al factor de riesgo desarro-
2) El nuevo procedimiento es ineficaz para disminuir las llar la enfermedad.
complicaciones postoperatorias. 4) La asociacin no es estadsticamente significativa.
3) La muestra est sesgada. 5) Se confirma relacin de causalidad entre el factor de riesgo
4) El resultado carece de relevancia clnica. y la enfermedad estudiada.
5) No existe la suficiente evidencia para decir que ambos
procedimientos son diferentes. 32. Una de las siguientes afirmaciones sobre el contraste de
hiptesis es FALSA:
27. Al comparar dos frmacos hipoglucemiantes, observamos
Preguntas TEST

que el nuevo es significativamente mejor que el usual (p 1) El valor p es el nivel de significacin estadstica especifica-
<0,05). Indique la correcta: do para el estudio.
2) El valor p corresponde a la probabilidad alfa de cometer
1) El nuevo tratamiento es, sin ninguna duda, mejor que el un error tipo I.
antiguo. 3) La capacidad del test de detectar diferencias cuando estas
2) Si p <0,05, el resultado no es significativo. existen aumenta con el aumento del tamao muestral.
3) El nuevo tratamiento es un 95% ms eficaz que el usual.
4) Un error tipo I consiste en aceptar la hiptesis nula cuando
4) Si los dos tratamientos fueran iguales, resultados como los
es falsa.
obtenidos o diferencias an mayores ocurriran con una
5) Los valores de p elegidos habitualmente son 0,05 y 0,01.
probabilidad menor del 5%.
5) No existe la suficiente evidencia como para decir que
33. Si en un contraste de hiptesis obtenemos una diferencia
ambos tratamientos son diferentes.
estadsticamente significativa con un nivel de significacin p
<0,05, indique el enunciado FALSO:
28. Indique cul es el valor mximo admitido de error alfa para
establecer diferencias mediante contraste de hiptesis en
1) Demostramos la falsedad de la hiptesis nula, ya que p
trabajos cientficos:
<0,05 indica que existe una diferencia estadsticamente
significativa.
1) 5%.

M exico A rgentina CTO Medicina C/ Nez de Balboa, 115 28006 MADRID (Espaa) Tfno.: (91) 782 43 32 / Fax: (91) 782 43 27
C hile U ruguay E-mail: secretaria@ctomedicina.com; iberocto@ctomedicina.com WEB: www.ctomedicina.com; www.iberocto.com
ET Pg. 3
ESTADSTICA
Seguimiento a distancia Preparacin Examen de Seleccin 05/06 1 Vuelta
2) Si la hiptesis nula es verdadera, diferencias iguales o 5) Coeficiente de correlacin de Pearson.
mayores que las observadas pueden ocurrir con una
probabilidad demasiado pequea como para que sea 38. Qu test estadstico debemos utilizar para comparar la
atribuible exclusivamente al azar. presencia cualitativa del antgeno de superficie de la hepatitis
3) Un tamao muestral menor puede hacernos encontrar un B en estudiantes de Medicina y Odontologa?:
nivel de significacin p >0,05 tras el anlisis de los
resultados. 1) t de Student.
4) La asociacin estadstica significativa no implica asocia- 2) Test de McNemar.
cin causal. 3) ANOVA.
5) La significacin estadstica se refiere slo a la existencia de 4) Chi-cuadrado.
una diferencia, no a su magnitud. 5) Test de Friedman.

34. Se someten a un estudio comparativo la ticlopidina y la 39. Se comparan tres tratamientos en cuanto a eficacia en la
aspirina en cuanto a capacidad para disminuir el riesgo de reduccin del nmero de lesiones de acn en pacientes
sufrir ACVAs; la ticlopidina parece mostrarse ms eficaz que adolescentes de 14 a 16 aos. Qu tipo de test estadstico
la aspirina. Sin embargo, la diferencia no resulta estadstica- aplicaremos?:
mente significativa en el contraste de hiptesis para p <0,05.
Usted afirma que: 1) t de Student.
2) ANOVA.
1) No hay diferencia de eficacia entre aspirina y ticlopidina. 3) Regresin lineal.
2) La ticlopidina es un 5% ms eficaz que la aspirina. 4) Chi-cuadrado.
3) La probabilidad de que las diferencias observadas se 5) Coeficiente de correlacin de Spearman.
deban al azar es menor del 5%.
4) Es posible que aumentando el tamao de la muestra 40. Para comparar la tensin arterial diastlica en dos muestras
obtengamos diferencias significativas para el mismo nivel de mujeres embarazadas y no embarazadas, el test ms
de significacin. apropiado es:
5) Las diferencias entre ticlopidina y aspirina exceden lo
atribuible al azar. 1) Chi-cuadrado.
2) Regresin lineal.
35. Se ha comparado el peso de dos grupos de recin nacidos, 3) Test exacto de Fisher.
uno formado por hijos de no fumadoras y otro formado por 4) t de Student.
hijos de fumadoras, encontrndose un peso menor en el 5) Test de Wilcoxon.
segundo grupo (p <0,05). Esto significa que:
41. Se pretende establecer una ecuacin, mediante la cual pueda
1) Fumar durante la gestacin retarda el crecimiento fetal. predecirse el tiempo de reproduccin, en base al conoci-
2) Es muy probable que las diferencias en el peso se deban miento del fotoperodo (nmero de horas de luz/da). Qu
al azar. tipo de test utilizaremos?:
3) El nmero de individuos estudiado fue insuficiente para
llegar a un resultado concluyente. 1) Coeficiente de correlacin.
4) Si no existieran diferencias entre los dos grupos, la proba- 2) Anlisis de Regresin.
bilidad de encontrar nuestros resultados es menor del 5%. 3) Test exacto de Fisher.
5) No hay diferencias entre ambos grupos. 4) t de Student para muestras independientes.
5) Test de Friedman.
36. Seale la FALSA:
42. Un coeficiente de correlacin de Pearson de -1 indica:

Preguntas TEST
1) Si aumenta el tamao muestral, disminuye el error tipo I.
2) Cuando estamos ante un error beta del 3%, la potencia del 1) No hay correlacin.
test es alta. 2) Existe una dbil correlacin.
3) La potencia del test es alta si son grandes las diferencias 3) Existe una fuerte correlacin.
reales. 4) Al aumentar los valores de la variable X, aumentan los
4) Si las diferencias entre dos tratamientos son muy grandes, valores de la variable Y.
es preciso un tamao muestral grande. 5) Al disminuir los valores de la variable X, disminuyen los
5) Un error tipo I del 5% no equivale a una potencia del 95%. valores de la variable Y.

37. La prueba estadstica adecuada para comparar las medias de 43. Son datos apareados:
tensin arterial entre un grupo de fumadores, uno de ex-
fumadores, y otro de personas que nunca han fumado es: 1) Tensin arterial en mujeres, frente a tensin arterial en
hombres.
1) t de Student para datos independientes. 2) Tiroxina en una muestra pequea, frente a tiroxina en una
2) t de Student para datos apareados. muestra grande.
3) Anlisis de la varianza. 3) Aclaramiento de creatinina en una muestra, frente a
4) Chi-cuadrado. aclaramiento de creatinina en una poblacin.

M exico A rgentina CTO Medicina C/ Nez de Balboa, 115 28006 MADRID (Espaa) Tfno.: (91) 782 43 32 / Fax: (91) 782 43 27
Pg. 4 ET C hile U ruguay E-mail: secretaria@ctomedicina.com; iberocto@ctomedicina.com WEB: www.ctomedicina.com; www.iberocto.com
ESTADSTICA
Preparacin Examen de Seleccin 05/06 1 Vuelta Seguimiento a distancia

4) Glucemia antes de comer, frente a glucemia despus de 50. Es una prueba de contraste de hiptesis til para establecer
comer. relacin entre una variable cualitativa y otra cuantitativa:
5) Talla en hombres frente a talla en mujeres.
1) Distribucin t de Student.
44. Para comparar el descenso medio en la carga viral del VIH que 2) Anlisis de regresin.
se observa al emplear indinavir y saquinavir, se utiliza: 3) Correccin de Yates.
4) Test de Mann-Whitney.
1) Chi-cuadrado. 5) Prueba de Spearman.
2) t de Student.
3) Correlacin de Pearson.
4) Test de Spearman.
5) ANOVA.

45. Cul de las siguientes afirmaciones sobre las pruebas esta-


dsticas no paramtricas es FALSA?:

1) Son menos potentes que las paramtricas.


2) Son tiles en muestras pequeas.
3) Son tiles en el caso de variables ordinales.
4) Requieren ciertas asunciones sobre la distribucin de las
variables en la poblacin.
5) Pueden utilizarse para comparar variables de tipo cuan-
titativo.

46. Qu prueba estadstica usara para comparar el resultado


(curacin/no curacin) de tres tratamientos distintos?:

1) ANOVA.
2) Test de Spearman.
3) t de Student.
4) Chi-cuadrado.
5) Test de Wilcoxon.

47. Una muestra de 25 pacientes nefrpatas se someten a deter-


minacin cuantitativa de ADN viral (VHB), antes y despus de
ser sometidos a hemodilisis. Qu test empleara para com-
parar los resultados obtenidos?:

1) t de Student para datos apareados.


2) t de Student para datos independientes.
3) Correlacin de Pearson.
4) Test de la U de Mann-Whitney.
5) Test de Wilcoxon.
Preguntas TEST

48. Qu test estadstico utilizara para comparar la relacin


entre la tensin arterial y la colesterolemia?:

1) Correlacin de Pearson.
2) ANOVA.
3) Chi-cuadrado.
4) t de Student.
5) Test de Wilcoxon.

49. Para comparar el resultado de un IECA en la reduccin de la


TAS con el de un betabloqueante, empleamos el test:

1) U de Mann-Whitney.
2) Test de Wilcoxon.
3) Test de Friedman.
4) Rho de Spearman.
5) t de Student.

M exico A rgentina CTO Medicina C/ Nez de Balboa, 115 28006 MADRID (Espaa) Tfno.: (91) 782 43 32 / Fax: (91) 782 43 27
C hile U ruguay E-mail: secretaria@ctomedicina.com; iberocto@ctomedicina.com WEB: www.ctomedicina.com; www.iberocto.com
ET Pg. 5
ESTADSTICA
Preparacin Examen de Seleccin 05/06 1 Vuelta Seguimiento a distancia
Pregunta 1.- R: 3 el 50), el clculo de la media aritmtica puede resultar sesgado, por lo
Los estudios estadsticos pretenden llegar a conocer a la poblacin que es preferible utilizar la mediana, la cual es estable ante estas medi-
a partir de un anlisis de una muestra de individuos. ciones atpicas o errores de medicin. Resumiendo, es preferible el
Lo que se estudia acerca de cada individuo de la muestra es lo que uso de la mediana como medida de centralizacin en los siguientes
se llama carcter o variable; es decir, se trata de una caracterstica casos:
observable en los individuos. El color de los ojos, el estado civil, el Mediciones atpicas.
nmero de hijos, las pulsaciones por minuto, la talla o la velocidad de Errores de medicin.
sedimentacin constituyen ejemplos de variables. Muestra no homognea.
Las variables cuantitativas son aquellas que pueden medirse, Distribucin asimtrica.
cuantificarse o expresarse de forma numrica; las variables que no
pueden cuantificarse numricamente reciben el nombre de varia- Pregunta 5.- R: 4
bles cualitativas; estas variables, por lo tanto, toman valores no nu- El concepto de dispersin implica "cunto estn de separados los
mricos. datos con respecto al dato central (a la media)", por tanto, se puede
Dentro de las opciones, hay dos variables cuantitativas: la glucemia calcular como la suma de las diferencias con la media de los datos.
y el nmero de camas. Cuando entre dos categoras cualquiera de Hay que tomar los valores absolutos, ya que, puesto que hay unos
una variable, podemos interponer otra (ej. entre 80 y 81 mg/dl de datos mayores que la media y otros menores, el sumatorio de las
glucemia, tenemos 80,5) la variable es continua. El nmero de camas diferencias dara cero si no se emplease esta argucia matemtica. Pos-
por el contrario es discreta. teriormente se divide la suma total entre el nmero de individuos de la
muestra, para que sea comparable con otras dispersiones en otras
series con distinto nmero de individuos. Si te fijas, lo que hemos
Pregunta 1. Tipos de variables. hecho es obtener la desviacin media de los datos con respecto a su
media aritmtica.
VARIABLE SUBTIPO EJEMPLO Avanzando un poco ms, podemos, a partir de la desviacin me-
dia, obtener un mejor ndice de dispersin: si en vez de trabajar con
Cualitativa Nominal dicotmica Sexo valores absolutos, elevamos al cuadrado las diferencias, tambin nos
Nominal no dicotmica R aza
deshacemos de los nmeros negativos, y pasamos a trabajar con "re-
as" en vez de segmentos (las diferencias se elevan al cuadrado); hemos
Ordinal Nivel socioeconmico obtenido la varianza, que es una excelente medida de dispersin.
El problema de la varianza es que sus unidades son las mismas que
Cuantitativa Discreta N episodicos de asma/sem
las de los datos pero elevadas al cuadrado, lo cual es matemticamen-
Continua Uricemia te incmodo, por ello se obtiene la raz cuadrada positiva del resulta-
do de la varianza para manejarse ms cmodamente; a este valor se le
llama desviacin tpica, y es el mejor ndice de dispersin.
Pregunta 2.- R: 3 El rango o recorrido, que tienes definido en el enunciado, es la
Si recuerdas la exploracin del coma o del traumatismo craneoen- forma ms elemental. Su inters es muy relativo, ya que utiliza sola-
ceflico, se haca referencia a la escala de Glasgow; en ella, se consi- mente dos datos de la serie (el primero y el ltimo), que son, precisa-
deraban tres parmetros: respuesta motora, ocular y verbal. Segn el mente, los menos representativos, puesto que son los valores extremos
paciente, se asignaban unos puntos a cada parmetro, que iban des- o ms raros de la variable.
de 3, que se considera una situacin muy grave a 15, que sera una
situacin de normalidad.
Sin embargo, en dicha escala el nmero asignado NO tiene valor Pregunta 5. Formas de medida.
matemtico (7 no significa "doble" peor pronstico que 14), por lo
que la variable NO se considera cuantitativa sino cualitativa. MEDIDA DE MEDIDA DE
Las categoras de esta variable (cualitativa ordinal) tienen una ma- CENTRALIZACIN DISPERSIN
nera lgica de ordenarse (de 3 a 15), a diferencia de las variables
cualitativas nominales (por ej. el color de pelo). DISTRIBUCIN Desviacin tpica
Media
Comentarios TEST

HOMOGNEA o estndar
Pregunta 3.- R: 5
El primer escaln dentro de la estadstica es el descriptivo. En esta DISTRIBUCIN R ango intercuartlico
Mediana
fase se realiza la recogida de la informacin, que debe ser posterior- ASIMTRICA R ango
mente ordenada, resumida y presentada de una forma comprensible
que permita pasar posteriormente a la segunda fase, que es el anlisis
Pregunta 6.- R: 5
de los datos.
El coeficiente de variacin es adimensional, debido a que repre-
Dentro del apartado de estadstica descriptiva nos encontramos
senta el porcentaje de la desviacin estndar sobre la media.
con las medidas de centralizacin que se mencionan en esta pregun-
Su utilidad est en que sirve para comparar la variabilidad de dos o
ta: la media, la mediana y la moda.
ms distribuciones, estn expresadas en las mismas o en diferentes
La media se obtiene sumando todos los valores numricos ob-
unidades. Por tanto, la opcin 5 es la falsa.
servados y dividiendo el resultado por el nmero de observaciones.
La forma de comparar dos desviaciones tpicas de muestras dife-
Es la medida de centralizacin ms empleada. Recuerda que es
rentes se hace mediante el coeficiente de variacin, que considera
muy sensible a las observaciones atpicas. En este caso: 31, que es la
cada desviacin en relacin a la muestra de la que procede, es decir,
suma total, entre 9, que es el nmero de observaciones, da 3,4.
en relacin (divisin) a la media de la muestra. Mediante la relacin,
La mediana slo emplea el valor central de la serie (es decir, el que
se obtiene una medida adimensional que puede compararse con la
deja a cada lado, la misma proporcin de la muestra); sin embargo, se
de otra muestra.
afecta menos por observaciones atpicas (por ejemplo, errores de
Despus de hablar de las medidas de dispersin surge la pregunta
medida). En este caso, es 3.
"Cundo se puede decir que una distribucin es concentrada o dis-
La moda es el dato que ms se repite en una serie, puede no ser un
persa?, a partir de qu valor de la desviacin tpica?". En general, para
valor nico, a diferencia de los otros dos parmetros. En este caso, es 2.
distribuciones biolgicas de datos, sin valores negativos, se pueden
considerar concentradas aquellas cuya desviacin tpica no excede
Pregunta 4.- R: 4
1/3 del valor de la media. Desviaciones superiores a la mitad de la
La medida de centralizacin ms utilizada es la media aritmtica, si
media se traducen en dispersiones excesivas.
bien cuando en una muestra hay valores atpicos (como en este caso,

M exico A rgentina CTO Medicina C/ Nez de Balboa, 115 28006 MADRID (Espaa) Tfno.: (91) 782 43 32 / Fax: (91) 782 43 27
C hile U ruguay E-mail: secretaria@ctomedicina.com; iberocto@ctomedicina.com WEB: www.ctomedicina.com; www.iberocto.com
ET Pg. 1
ESTADSTICA
Seguimiento a distancia Preparacin Examen de Seleccin 05/06 1 Vuelta
Pregunta 7.- R: 4 dad de ser ambas cosas a la vez es P(AB)= (200-150)/1.000 = 0,05
La distribucin normal sigue una forma de campana en la que el (si 150 asmticos de los 200 no son atpicos, significa que 50 s lo
valor central es la media y tambin la mediana (puesto que la campa- son). Por tanto, la probabilidad de que un individuo escogido al azar
na es simtrica) y la moda (puesto que en ese punto la curva tiene su sea asmtico o atpico es:
mxima altura).
Entre 1 desviacin estndar se encuentra el 68% de las observa- B) = 0,2 + 0,1 - 0,05 = 25%
P(AUB) = P(A) + P(B) - P(A
ciones y entre 2 desviaciones estndar estn el 95% de las observa-
ciones. Por tanto, entre 1 y 2 desviaciones estndar se encuentran el Para calcular la probabilidad de A o B, cuando A y B no son
27% (95-68), por lo que la opcin 4 es incorrecta. excluyentes, hay que restar la interseccin de A y B (la probabilidad
El resto de opciones resumen una serie de caractersticas que de- de que se den los dos sucesos simultneamente), porque de no hacer-
bes conocer sobre la curva de Gauss. lo estaramos contando esos casos (en este ejemplo, los nios con
asma y dermatitis atpica) dos veces.

Pregunta 11.- R: 3
La razn de existir de la estadstica es la imposibilidad de poder tener
acceso a toda la poblacin. Basndose en esta limitacin, los cientficos
eligen una muestra pequea de dicha poblacin y realizan todas las
mediciones sobre ella, una vez obtenidas las conclusiones, que son
ciertas para el 100% de la muestra, se deben generalizar o inferir los
resultados a la poblacin; durante este proceso hay que aceptar que la
muestra, por muy representativa que sea, nunca va a ser idntica a la
poblacin y, por tanto, la generalizacin va a tener que asociar una
probabilidad de que en el intervalo de confianza estimado NO se en-
cuentre la verdadera medida del parmetro estimado (el valor que se
obtendra si se calculase con toda la poblacin).

Pregunta 12.- R: 3
En esta pregunta nos estn pidiendo un intervalo en el que con una
confianza determinada (95% o 99%) est la media poblacional de
glucemia, por lo que las dos primeras opciones NO pueden ser co-
Pregunta 7. Distribucin normal. rrectas, ya que al estimar un parmetro en la poblacin (en este caso
una media) hay que aportar la probabilidad de que en ese intervalo
Pregunta 8.- R: 4 no est el valor poblacional (la "p").
El 50% de la poblacin fuma y el 10% de la poblacin total fuma Como dos de las opciones tienen una p<0,01, calcularemos en
y es hipertensa, es decir, hay 50 fumadores por cada 100 habitantes, primer lugar el intervalo de confianza del 99% y, en caso de que NO
de los que 10 son hipertensos y 40 no. coincida con estas dos opciones, marcaremos la opcin 3.
La proporcin de hipertensos entre los fumadores es, por tanto, de
1 entre 5, que es el 20%.
Se trata de una probabilidad condicionada, consideramos que DS
P(AB) es la probabilidad de fumar y ser hipertenso y P(B) la probabili- IC 99%= X +
2,6
dad de ser fumador. Nos piden la probabilidad de ser hipertenso una n-1
vez que se es fumador (P (A/B)). La frmula es:
Aplicando la frmula:
B)/P(B)
P (A/B)= P (A
10
Pregunta 9.- R: 2 IC 99%= 80 +
2,6
Es un caso de probabilidades independientes (el hecho de que un 100

Comentarios TEST
individuo muera no condiciona la muerte del otro), por tanto, la
probabilidad de que un sujeto A fallezca es 0,2 y la probabilidad de IC 99%= 80 +
que el sujeto B fallezca es tambin 0,2 (no se modifica por el hecho de 2,6
que el sujeto A muera o no). Como vemos, el resultado NO coincide con las dos ltimas opcio-
La probabilidad de que ambos fallezcan es: nes, por lo que la correcta es la tercera:
B)= 0,2 x 0,2= 0,04 = 4%
P(A
10
IC 95%= 80 +
2
100

De cara al clculo del intervalo de confianza, en el MIR, si la raz


cuadrada del tamao muestral es un nmero entero (por ejemplo,
raz de 100), se suele redondear y no se resta 1.

Pregunta 13.- R: 4
La clave de esta pregunta es tener claro qu es un intervalo de
Pregunta 9. Reglas de probabilidad. confianza y cmo se expresa.
Un intervalo de confianza es una horquilla de dos valores entre los
Pregunta 10.- R: 2 que hay una probabilidad dada de que se encuentre el valor
Es un caso de probabilidad con sucesos no excluyentes (el hecho poblacional de determinada medida (una media, una prevalencia).
de tener asma no protege contra la atopia y viceversa). Los intervalos de confianza se pueden dar desarrollados (por ejem-
La probabilidad de ser asmtico es P(A)= 200/1.000= 0,2 y la plo, 65-75 lpm, p<0,05) o sintticos (por ejemplo, 70 5 lpm,
probabilidad de ser atpico es P(B)= 100/1.000= 0,1; la probabili- p<0,05).

M exico A rgentina CTO Medicina C/ Nez de Balboa, 115 28006 MADRID (Espaa) Tfno.: (91) 782 43 32 / Fax: (91) 782 43 27
Pg. 2 ET C hile U ruguay E-mail: secretaria@ctomedicina.com; iberocto@ctomedicina.com WEB: www.ctomedicina.com; www.iberocto.com
ESTADSTICA
Preparacin Examen de Seleccin 05/06 1 Vuelta Seguimiento a distancia
Lo ms importante es darse cuenta de que en el enunciado de esta Ya que nos dan el error estndar de la media, lo ms fcil es calcu-
pregunta YA nos dan el intervalo de confianza (en el 5 est compren- lar primero el IC 95 % y, en caso de que no coincida con las opciones
dido 2 x error estndar de la media), por lo que la probabilidad de que hablan de la poblacin (la 3 y la 4), desarrollar despus el inter-
que la media de frecuencia cardaca en la poblacin NO est en l es valo que incluye el 95 % de la muestra y comprobar con cul de las
menor del 5%, que es lo que dice la opcin 4. opciones que se refieren a la muestra (opciones 1 y 2) coincide.

Pregunta 14.- R: 3 IC 95%= X 2 x EEM


Es importante darse cuenta de que en esta pregunta, aunque nos
dan algunos datos poblacionales, lo que se pide es un intervalo que Aplicacin de la frmula:
comprenda al 95 % de una muestra. Este intervalo (que NO se llama
de confianza), si la variable sigue una distribucin normal (en caso de IC 95%= 2 2 x 0,5
que este dato no lo aporten, como ocurre casi siempre en el MIR, se IC 95%= 2 1
asume que es as) se calcula:
Es decir, que se tiene un 95 % de confianza de que el verdadero
X 2 x DS descenso medio se site entre 1 y 3 mmol/l, que es lo que dice la
opcin 4).
Del enunciado, extraemos que la media es 200 mg/dl y el dato que
nos falta es la desviacin tpica. Pregunta 16.- R: 4
Para determinar la desviacin tpica, es necesario darse cuenta de Esta pregunta tiene importancia, ya que de su comprensin deriva
que en el clculo del dato poblacional que nos aporta el enunciado el entender cmo se aumenta la precisin en una estimacin de
(el intervalo de confianza del 95 %) est la desviacin tpica. parmetros.
En la frmula del Error Estndar de la Media, la desviacin tpica
est en el numerador y el tamao muestral en el denominador. Para
DS reducir el EEM, no podemos actuar sobre el numerador, ya que la
IC 95%= X +
2 desviacin tpica es una caracterstica que depende de la variable que
n-1 estamos midiendo. Lo que s podemos hacer es aumentar el denomi-
nador, es decir el tamao muestral, de modo que al aumentar el
Aplicando la frmula: tamao muestral cuatro veces (como est dentro de una raz cuadra-
da), conseguimos disminuir el EEM a la mitad.
DS De modo que si un intervalo de confianza es demasiado amplio y
IC 95%= 200 +
2 queremos reducirlo (esto es, ganar en precisin), deberemos aumen-
100 tar el tamao muestral.

(Date cuenta de que generalmente al tamao muestral NO se le Pregunta 17.- R: 2


resta uno, ya que la diferencia del resultado es mnima y el clculo de En esta pregunta, nos piden un intervalo que incluya con una
la raz cuadrada es mucho ms sencillo). probabilidad del 95 % la verdadera media poblacional, es decir, el
Intervalo de Confianza del 95 %:
Sustituimos el IC 95 % por los valores que nos dan en el enunciado:
DS
IC 95%= X +
2
DS
196 - 204 = 200 + n-1
2
100 Aplicacin de la frmula:
Si desarrollamos esta ecuacin (eligiendo el lmite superior o el
inferior del IC), tenemos: DS
IC 95%= 200 +
2
Comentarios TEST

100
IC 95%= 200 2 (DS/10)

Para su clculo nos falta la desviacin estndar, que no nos la dan


204 - 200 = 2 (DS/10) directamente, pero que la podemos buscar en un dato de la muestra.
4 = 2 DS / 10 Nos dicen que el 68 % de la muestra est comprendido entre 175
40 = 2 DS; DS = 20 y 225, por lo que:
Ahora que ya tenemos la desviacin tpica, podemos calcular el X DS = 175 - 225
intervalo de la muestra por el que nos preguntaban: 200 DS = 175 - 225
X 2 x DS Desarrollando esta ecuacin, obtendremos que la DS es 25.
Ahora slo tenemos que retomar la frmula del IC 95%:
Aplicacin de la frmula:
IC 95%= 200 2 (DS/10)
200 2 x 20 = 200 40 = 160 - 240 IC 95%= 200 2 (25/10)
IC 95%= 200 2 x 2,5
Pregunta 15.- R: 4 IC 95%= 200 5
Con los datos que aporta el enunciado (tamao muestral, media y IC 95%= 195 - 205
error estndar de la media), podemos calcular la desviacin tpica,
por lo que, de todas la opciones, puede ser cierta alguna que hable de Pregunta 18.- R: 3
la muestra (cuyos intervalos se calculan con la media y la desviacin En esta pregunta, nos piden el Intervalo de Confianza del 95 % y
tpica) o de la poblacin (cuyos intervalos de confianza se calculan nos aportan todos los datos necesarios para su clculo:
con la media y el error estndar de la media).

M exico A rgentina CTO Medicina C/ Nez de Balboa, 115 28006 MADRID (Espaa) Tfno.: (91) 782 43 32 / Fax: (91) 782 43 27
C hile U ruguay E-mail: secretaria@ctomedicina.com; iberocto@ctomedicina.com WEB: www.ctomedicina.com; www.iberocto.com
ET Pg. 3
ESTADSTICA
Seguimiento a distancia Preparacin Examen de Seleccin 05/06 1 Vuelta
mente debidas al azar (a la variabilidad en el muestreo), y por lo tanto
DS
IC 95%= X + no podremos refutar la hiptesis nula o si, por el contrario, las diferen-
2 cias obtenidas son demasiado grandes y la probabilidad de que se
n-1 deban al azar es pequea, y por tanto rechazaremos la Ho y diremos
que las diferencias son compatibles con la Hiptesis Alternativa (H1),
Aplicacin de la frmula: que es aquella que sostiene que ambos grupos son distintos.
Lo ms importante de todo es darse cuenta de que, cuando recha-
5 zamos la Ho, no estamos completamente seguros de que las diferen-
IC 95%= 65 +
2 cias encontradas no se deban al azar. Lo que ocurre es que, como la
100 probabilidad de que se deban al azar es tan pequea, nos inclinamos
a pensar que la que es cierta es la H1.
IC 95%= 65 1 Y qu parmetro estadstico es el que determina cul es la proba-
IC 95%= 64 - 66 bilidad de que la diferencia obtenida se deba al azar? Pues el grado
de significacin (la "p").
Pregunta 19.- R: 3 Proponemos un ejemplo para la comprensin de uno de los con-
Para contestar a esta pregunta hay que tener claro que 260 10 ceptos ms importantes de la Estadstica y del bloque de Estadstica en
con una p<0,05 no puede ser otra cosa que el intervalo de confianza el examen MIR:
del 95 % (recuerda que en la muestra no hablamos de confianza y no Para comparar la TAD de los hombres y las mujeres, se recluta a 10
se aporta probabilidad de equivocarnos, es decir, la "p") y que en el 10 mujeres y 10 hombres sin HTA y se les toma la TAD. Se calcula la
ya est incluido 2 x EEM. media aritmtica de ambos grupos, y el resultado es:
Si nos ha quedado claro lo anterior, la probabilidad de que en ese
intervalo NO est la verdadera media poblacional de colesterol ser X de TAD en hombres: 80
del 5%, que es lo que dice la opcin correcta (la 3). X de TAD en mujeres: 63
Si te das cuenta con los datos que aporta el enunciado (NO nos
dan tamao muestral, por lo que NO podemos calcular la desviacin Segn estos resultados, estaramos tentados a decir que la TAD es
tpica a partir del intervalo de confianza) NO podemos calcular inter- distinta en hombres y en mujeres, pero ANTES de afirmar dicha dife-
valos de la muestra, por lo que aquellas opciones que hablen de la rencia, deberemos preguntarnos: estas diferencias encontradas no se
muestra (1,2 y 5) sern falsas. debern a la variabilidad que deriva de trabajar con muestras en vez
de con toda la poblacin de mujeres y hombres?.
Pregunta 20.- R: 4 Imaginamos que se calcula el grado de significacin y se obtiene
En esta pregunta, nos piden el Intervalo de Confianza del 95 % y un 3%, es decir, la probabilidad de que estos resultados (la diferencia
nos aportan todos los datos necesarios para su clculo: obtenida en nuestro estudio) se deba al azar es muy pequea, por lo
que tendremos suficiente evidencia para rechazar la hiptesis nula y
decir que entre los dos grupos hay diferencias.
DS
IC 95%= X + Pero, es posible que habiendo dicho que la TAD en hombres es
2 distinta a la de mujeres nos hayamos equivocado?.
n-1 Efectivamente, la probabilidad de que hayamos rechazado la Ho y
fuese cierta (lo que se denomina error tipo I), coincide con la proba-
Aplicacin de la frmula: bilidad de que las diferencias encontradas se deban al azar (es decir,
que NO haya diferencias), esto es, el grado de significacin.
1 Por lo que, recapitulando: siempre que rechazamos la hiptesis
IC 95%= 7 + nula, deberemos aportar la probabilidad de que nos equivoquemos,
2 es decir, de cometer un error tipo I: la "p".
36

(Date cuenta de cmo en esta pregunta s hemos restado uno al Pregunta 22.- R: 2
tamao muestral, ya que en este caso facilita el clculo). En el caso de que la hiptesis nula fuese falsa (esto es, si en realidad
hubiese diferencias) y el grado de significacin obtenido no fuera tan

Comentarios TEST
IC 95%= 7 2 (1/6) pequeo como el nivel mnimo exigido en Estadstica (5%), NO po-
IC 95%= 7 0,32 dremos rechazar esa hiptesis nula, aunque sea falsa.
A esto, en Contraste de Hiptesis, se le denomina error tipo II.
Pregunta 21.- R: 4 Es importante insistir en que cuando no se tiene suficiente eviden-
Cuando se comparan dos muestras o grupos (por ejemplo, dos grupos cia para rechazar la Ho, no por ello se dice que sea cierta.
de personas sometidos a dos tratamientos distintos), incluso aunque entre
estos dos grupos no hubiera diferencia, al medir en ellos una variable Pregunta 22. Contraste de hiptesis.
resultado (por ejemplo: respuesta o no a un analgsico o disminucin de
la TA con una antihipertensivo) es muy poco probable que el promedio
de la variable resultado de ambos grupos sea exactamente el mismo. REALIDAD
Para entender en qu consiste el Contraste de Hiptesis, debere- EXISTE NO EXISTE
mos partir del anterior razonamiento: "en caso de que ambos grupos DIFERENCIA DIFERENCIA
sean iguales, se pueden obtener pequeas diferencias en la variable Ho falsa Ho cierta
resultado que se deben a la variabilidad del muestreo".
En Contraste de Hiptesis, al supuesto de que ambos grupos o HAY 1-b
muestras provienen de una poblacin homognea (es decir, de que DIFERENCIAS Poder estadstico Error tipo I o
son iguales o de que no hay diferencias entre ellos), lo llamamos SIGNIFICATIVAS. o potencia del error alfa
hiptesis nula (Ho) y las diferencias que se deben al muestreo son RESULTADOS Rechazo Ho. test
diferencias que se pueden explicar por el azar. DEL TEST NO HAY
Por lo tanto, podramos repetir el anterior argumento con otros
DIFERENCIAS Error tipo II o
trminos: "en caso de que la Ho sea cierta, se pueden obtener peque- 1- a
SIGNIFICATIVAS. error beta
as diferencias en la variable resultado que se deben al azar".
No rechazo Ho.
Y en qu consiste el Contraste de Hiptesis?. Pues en establecer si las
diferencias que se han obtenido entre los dos grupos son probable-

M exico A rgentina CTO Medicina C/ Nez de Balboa, 115 28006 MADRID (Espaa) Tfno.: (91) 782 43 32 / Fax: (91) 782 43 27
Pg. 4 ET C hile U ruguay E-mail: secretaria@ctomedicina.com; iberocto@ctomedicina.com WEB: www.ctomedicina.com; www.iberocto.com
ESTADSTICA
Preparacin Examen de Seleccin 05/06 1 Vuelta Seguimiento a distancia
Pregunta 23.- R: 4 rencias observadas sean debidas al azar es demasiado alta (fjate que
El poder o potencia estadstica es la capacidad de demostrar las "demasiado alta" puede ser un 6 %), por lo que no tenemos suficiente
diferencias en caso de ser ciertas (esto es, rechazar la hiptesis nula evidencia para rechazar la hiptesis nula.
siendo falsa sta y siendo cierta la alternativa). No rechazar la hiptesis nula NO significa que sea cierta (es ms,
En el caso de que la hiptesis nula fuese falsa (esto es, si en realidad un grado de significacin cercano al 5 % sugiere que hay diferencias,
hubiese diferencias), nos podemos encontrar con dos situaciones: pero que stas pueden deberse al azar con una probabilidad no tan
No rechazamos la Ho: error tipo II. baja como la exigida), por lo que NO podremos decir que no haya
Rechazamos la Ho: poder estadstico. diferencias (opcin 4 falsa).
Como puedes ver, el poder estadstico es el complementario del
error tipo II, es decir, si antes de arrancar un estudio, aceptamos un Pregunta 30.- R: 3
error tipo II del 10 %, sabremos que la probabilidad de demostrar La probabilidad de que existiendo diferencias (es decir, de que
diferencias, en caso de haberlas, ser del 90 %. siendo falsa la hiptesis nula), se detecten, o lo que es lo mismo, se
obtenga suficiente evidencia para rechazar la hiptesis nula, es la
Pregunta 24.- R: 4 potencia estadstica.
Al comparar dos medias nos encontramos que la probabilidad de La probabilidad de que habiendo diferencias NO se demuestren
que las diferencias halladas puedan deberse al azar es del 60 % es el error tipo II, por lo que:
(p=0,6), es decir, la probabilidad de que esos resultados (esas diferen-
cias) puedan deberse a variaciones en el muestreo es bastante alta, por Error tipo II + Poder Estadstico = 1
lo que posiblemente NO haya verdaderas diferencias, es decir, los
resultados son compatibles con la hiptesis nula. Como el Error tipo II y el Poder Estadstico son complementarios, si
la probabilidad de cometer un error beta es del 3 %, la probabilidad
Pregunta 25.- R: 4 de detectar las diferencias ser del 97% (como dice la opcin 3).
Incluso si la eficacia de la azatioprina y la 6-mercaptopurina fuese
igual (Ho cierta), es rarsimo que el porcentaje de respuesta sea exacta- Pregunta 31.- R: 1
mente igual en dos grupos de personas; lo que tenemos que decidir es si Pongamos un ejemplo para su mejor comprensin:
las diferencias observadas (60 % de mejora con azatioprina y 55% con 6- Se realiza un estudio longitudinal en el que se selecciona un grupo
mercaptopurina) se pueden deber al azar; la respuesta a esta pregunta es de personas expuestas a HTA y otro grupo de personas que no presen-
el grado de significacin, que en este caso es menos del 5 %, por lo que tan este factor de estudio; se siguen durante 2 aos a todas las perso-
podremos decir que hay diferencias entre esos dos tratamientos (rechaza- nas participantes y se detecta una Incidencia de ACV entre las perso-
remos la Ho) con una probabilidad de equivocarnos (es decir, de que se nas que padecen HTA de 80 por cada 1.000 personas y entre las que
deban al azar) menor del 5%, que es lo que dice la opcin 4. no tienen HTA de 40 por cada 1.000 personas.
Estas diferencias encontradas se pueden deber al azar, a la variabi-
Pregunta 26.- R: 5 lidad en el muestreo?.
Nos encontramos con dos grupos de pacientes que, sometidos a La respuesta hay que buscarla en el grado de significacin, que en
distintos procedimientos quirrgicos, presentan un porcentaje de com- este caso es menor al 5%, por lo que en caso de que no hubiese una
plicaciones distinto, que desde el punto de vista clnico es importante. asociacin entre el factor estudiado (la HTA) y la enfermedad (los
Lo que hay que establecer es con qu probabilidad estas diferen- ACVs) la probabilidad de haber encontrado estos resultados sera menor
cias pueden ser explicadas por el azar; en el enunciado no nos lo dan del 5% (opcin 1 correcta).
directamente, pero la probabilidad de que el azar explique las dife- NO se confirma relacin de causalidad entre el factor de riesgo y la
rencias debe de ser mayor del 5 %, ya que nos dicen que la diferencia enfermedad estudiada, ya que para ello se tienen que cumplir una serie
no es estadsticamente significativa. de criterios de causalidad que en esta pregunta NO se mencionan.
Es importante recalcar que, cuando no se tiene suficiente eviden-
cia para rechazar la Ho, no por ello se dice que sea cierta, por eso lo Pregunta 32.- R: 4
nico que podremos decir es que no existe evidencia para decir que Un Error Tipo I consiste en rechazar la Hiptesis Nula cuando en
ambos tratamientos son diferentes (opcin 5 correcta), pero NO po- realidad es cierta (opcin 4 falsa).
dremos aseverar que los dos tratamientos son iguales. La probabilidad de que rechacemos la Ho y sea cierta coincide con la
Comentarios TEST

probabilidad de que las diferencias encontradas se deban al azar (es decir


Pregunta 27.- R: 4 que NO haya diferencias), sto es, el grado de significacin.
Pongamos un ejemplo que podra ser el del enunciado de esta pre- En otras palabras, decimos que la probabilidad de que las diferen-
gunta: en 35 personas tratadas con el hipoglucemiante A se consigue cias observadas se deban al azar es tan baja como para creer que hay
como media una reduccin de la glucemia de 45 mg/dl y en 35 perso- verdaderas diferencias y en realidad, aunque era muy poco proba-
nas tratadas con el hipoglucemiante B se consigue como media una ble, las diferencias encontradas son debidas al azar.
reduccin de la glucemia de 60 mg/dl. En el contraste de hiptesis se
calcula cul es la probabilidad de encontrar estos resultados si los dos Pregunta 33.- R: 1
tratamientos fuesen iguales y se obtiene un grado de significacin del 2 %. No demostramos la falsedad de la hiptesis nula, ya que aunque p
Trasladado a cmo est formulada la pregunta, si los dos tratamien- <0,05 (lo que indica que existe una diferencia estadsticamente signi-
tos fuesen iguales (si la Ho fuese cierta), resultados como los obtenidos ficativa) siempre existe una pequea probabilidad de que la diferen-
o diferencias an mayores (en nuestro ejemplo, diferencias mayores a cia encontrada se deba al azar (Error Tipo I), con lo que rechazamos
60 - 45), ocurriran con una probabilidad menor del 5 % (en nuestro (no demostramos que es falsa) la Hiptesis Nula a ese nivel de significa-
caso con una probabilidad del 2%), que es lo que dice la opcin 4. cin (<5%).

Pregunta 28.- R: 1 Pregunta 34.- R: 4


El valor mximo admitido de error alfa para establecer diferencias Aunque la p haya resultado no significativa (para el nivel crtico del 5
mediante contraste de hiptesis en trabajos cientficos es del 5%; el %), no podemos decir que no haya diferencia de eficacia entre aspirina
otro valor habitualmente utilizado (ms estricto) es el 1%. y ticlopidina (no sugerimos que la hiptesis nula sea cierta, simplemente
Como vemos, el mximo error tipo I aceptado en trabajos cientfi- no tenemos suficiente evidencia para decir que es falsa).
cos es el 5 %. La p no tiene nada que ver con la magnitud de la eficacia por lo
que la opcin que dice "la ticlopidina es un 5% ms eficaz que la
Pregunta 29.- R: 4 aspirina" es FALSA.
Cuando el grado de significacin obtenido es mayor que el nivel La probabilidad de que las diferencias observadas se deban al azar
crtico fijado (habitualmente el 5%), la probabilidad de que las dife- NO es menor del 5% ya que la p no ha sido significativa.

M exico A rgentina CTO Medicina C/ Nez de Balboa, 115 28006 MADRID (Espaa) Tfno.: (91) 782 43 32 / Fax: (91) 782 43 27
C hile U ruguay E-mail: secretaria@ctomedicina.com; iberocto@ctomedicina.com WEB: www.ctomedicina.com; www.iberocto.com
ET Pg. 5
ESTADSTICA
Seguimiento a distancia Preparacin Examen de Seleccin 05/06 1 Vuelta
Las diferencias entre ticlopidina y aspirina NO exceden lo atribui- El test de contraste de hiptesis adecuado para determinar si existe
ble al azar, ya que sto se puede decir cuando la p es menor al 5 %. asociacin o no (es decir, que las diferencias encontradas entre los dos
De todas las opciones es cierta la que dice que "es posible que aumen- grupos de estudiantes se puedan deber al azar) entre dos variables cua-
tando el tamao de la muestra obtengamos diferencias significativas para litativas (independientemente de que sean dicotmicas o no, aunque
el mismo nivel de significacin", es decir, que aumentando el tamao en este ejemplo lo sean ambas) es el test de Chi-cuadrado.
muestral (factor ms importante de la potencia estadstica) aumentamos La opcin correcta es la 4.
la probabilidad de que se encuentren diferencias si las hay.
Pregunta 39.- R: 2
Pregunta 35.- R: 4 Al buscar las dos variables de este estudio, tienes que darte cuenta de
En caso de que no existan diferencias entre dos grupos compara- que la edad de las pacientes NO es una variable, simplemente nos estn
dos, la probabilidad de encontrar cierta diferencia (por azar o variabi- definiendo qu personas participan en el estudio, es decir, nos estn
lidad en el muestreo) es el grado de significacin. dando los criterios de seleccin. Mediante los criterios de seleccin se
En esta pregunta, en caso de que entre el peso de hijos de fumado- define la poblacin a la que se podrn extrapolar los resultados de
ras y no fumadoras no hubiese diferencias, la probabilidad de encon- nuestro estudio, que en este caso ser a adolescentes de 14 a 16 aos.
trar los resultados obtenidos ser menor del 5 %, por lo que la opcin El objetivo de este estudio es determinar si existen diferencias entre
correcta es la 4. tres tratamientos en cuanto a la reduccin del nmero de lesiones de
acn, por lo que las variables sern:
Pregunta 36.- R: 4 Variable predictora: tratamiento utilizado: variable cualitativa de
Si las diferencias entre dos tratamientos son muy grandes, NO es tres categoras.
preciso un tamao muestral grande. Si con un antihipertensivo A se Variable resultado: el nmero de lesiones que desaparecen: varia-
consigue como media un descenso de TAS de 40 mm de Hg y con ble cuantitativa; el resultado lo podan haber expresado como
otro antihipertensivo B un descenso de TAS de 12 mm de Hg, aunque variable dicotmica (mejora o no), pero en el enunciado nos de-
tengas un tamao muestral pequeo, es poco probable que esta dife- jan claro que van a cuantificar el resultado ("nmero de lesiones de
rencia se pueda explicar por el azar. acn").
Otra situacin sera que antes de empezar el estudio, se establecie- Con una variable predictora cualitativa de ms de dos categoras y
se como diferencia mnima con significacin clnica 30 mm de Hg; una variable resultado cuantitativa, el test de contraste de hiptesis
aqu s que para demostrar diferencias de, al menos, esta magnitud adecuado es el Anlisis de la Varianza (ANOVA).
(relativamente grande) es posible que se necesitase un tamao muestral La opcin correcta es la 2.
grande.
Pregunta 40.- R: 4
Pregunta 37.- R: 3 Imaginemos que tenemos un grupo de mujeres embarazadas a las
Para contestar este tipo de preguntas lo ms importante es encon- que se les toma la TAD y se obtiene como resultado una media aritm-
trar las dos variables del estudio que nos propongan. tica de 72 mm de Hg, y otro grupo de mujeres no gestantes a las que
En este caso, la variable predictora (V1) ser: no haber fumado se les toma la TAD y se obtiene como resultado una media aritmtica
nunca, ser ex-fumador o fumador; vemos que se trata de una variable de 74 mm de Hg.
cualitativa que tiene ms de dos categoras (variable cualitativa no Posiblemente esta diferencia se deba a la variabilidad en el mues-
dicotmica). La variable resultado (V2) ser: la tensin arterial medida treo (al azar), pero para poder hacer esta afirmacin tendremos que
cuantitativamente. hacer el test de contraste de hiptesis que corresponda con las varia-
Con una variable predictora cualitativa de ms de dos categoras y una bles de este estudio:
variable resultado cuantitativa el test de contraste de hiptesis adecuado V1: estar o no embarazada (variable dicotmica).
es el Anlisis de la Varianza (ANOVA). La opcin correcta es la 3. V2: TAD (variable cuantitativa).
Con una variable predictora cualitativa de dos categoras y una
Pregunta 37. Test de contraste de hiptesis. variable resultado cuantitativa, el test de contraste de hiptesis ade-
cuado es la t de Student.
La opcin correcta es la 4.
VARIABLE 1 VARIABLE 2 TEST DE HIPTESIS
Chi-cuadrado. Pregunta 41.- R: 2

Comentarios TEST
Dicotmica Dicotmica Aunque el enunciado de esta pregunta nos quede un poco
Test exacto de Fisher.
confuso, debemos ser capaces de distinguir las siguientes variables:
Cualitativa Cualitativa tiempo de reproduccin y fotoperodo. La primera es una variable
Chi-cuadrado
(> 2 categoras) (>= 2 categoras) cuantitativa (el tiempo) y la segunda tambin (nmero de horas de
luz), por lo que disponemos de dos posibles test para trabajar:
Dicotmica Cuantitativa t de Student Anlisis de Regresin.
Cualitativa Test de correlacin.
Cuantitativa Anlisis de la Varianza Cul es la clave para elegir uno o el otro?
(> 2 categoras)
Cuando lo que nos estn pidiendo es la ecuacin que relaciona a
Coeficiente de correlacin las dos variables, dicho de otro modo, la frmula que me permite
Cuantitativa Cuantitativa de Pearson. decir cunto valdr la y en funcin de la x (por ejemplo, cunto ser
Regresin. el tiempo de reproduccin si el fotoperodo vale 6 horas) hay que
hacer un Anlisis de Regresin.
Pregunta 38.- R: 4 La opcin correcta es la 2.
Y cundo se utiliza el test de correlacin?
Se pretende determinar si existe asociacin entre el tipo de carre-
ra que se cursa y la presencia o no de antgeno de la superficie del El test de correlacin de Pearson sirve para establecer, en dos varia-
bles que entre ellas siguen una relacin lineal (la ecuacin obtenida
virus de la hepatitis B, por lo que se aprecian las siguientes variables:
V1: Carrera universitaria que se estudia: variable cualitativa que en con el anlisis de regresin es lineal, del tipo y=A+Bx), si la relacin
nuestro estudio es dicotmica, ya que slo tiene dos posibilidades lineal es buena (la ecuacin obtenida se ajusta bien a la realidad) o no
(no es una buena ecuacin para predecir y en funcin de x).
(Medicina u Odontologa).
V2: Presencia o no del antgeno de superficie del VHB: tambin
Pregunta 42.- R: 3
variable cualitativa dicotmica, ya que slo tiene dos posibilidades
(dar positivo para Ag VHB o dar negativo). Un coeficiente de correlacin de Pearson de -1 indica que existe
una fuerte correlacin lineal entre las dos variables y que esta relacin

M exico A rgentina CTO Medicina C/ Nez de Balboa, 115 28006 MADRID (Espaa) Tfno.: (91) 782 43 32 / Fax: (91) 782 43 27
Pg. 6 ET C hile U ruguay E-mail: secretaria@ctomedicina.com; iberocto@ctomedicina.com WEB: www.ctomedicina.com; www.iberocto.com
ESTADSTICA
Preparacin Examen de Seleccin 05/06 1 Vuelta Seguimiento a distancia
es negativa, es decir, que cuando aumenta una variable la otra dismi- Vemos que la primera variable (dicotmica) es pareada, ya que la
nuye y viceversa. V2 se va a medir en los mismos pacientes (antes y despus); la segunda
La opcin correcta es la 3. variable es cuantitativa (nivel de ADN viral), por lo que el test param-
Recuerda que los valores del coeficiente de correlacin oscilan de trico que le correspondera es el de la t de Student.
-1 a 1, y que cuanto ms se acerca a -1 a 1 la relacin lineal ser ms Pero como el tamao muestral es menor de 30, no nos valen los
fuerte, mientras que un coeficiente de correlacin de 0 indica ausen- test paramtricos, por los que tendremos que elegir el equivalente a la
cia de relacin lineal. t de Student para datos apareados: el test de Wilcoxon.
La opcin correcta es la 5.
Pregunta 43.- R: 4
Si se pretende estudiar si hay diferencia o no entre los niveles de
glucosa en sangre en personas que no han comido y personas que ya Pregunta 47. Test no paramtricos.
han comido, el estudio se puede plantear de dos maneras:
a) Cogemos a una muestra de personas y les extraemos una muestra VARIABLE 1 VARIABLE 2 TEST DE HIPTESIS
de sangre antes de comer, y a otro grupo de personas distintas y les
extraemos sangre despus de que hayan comido. Dicotmica Cuantitativa (n <30) U de Mann-Whitney.
b) Cogemos a una muestra de personas y les extraemos una muestra independiente u ordinal Test de Wilcoxon.
de sangre antes de comer, y a ese mismo grupo de personas les
extraemos sangre despus de que hayan comido. Dicotmica pareada Cuantitativa (n <30)
Test de Wilcoxon
En ambos casos, las variables de estudio seran: o dependiente u ordinal
V1: No haber comido o haber comido (variable dicotmica).
V2: Niveles de glucosa en sangre: variable cuantitativa. Cualitativa Cuantitativa (n <30)
Test de Kruskal-Wallis
La nica diferencia es que en el primer caso las muestras son inde- (>2 categoras) u ordinal
pendientes (dos grupos de personas distintas) y en el segundo hay una
muestra pareada (se mide la V2 en la mismas personas antes y despus Cualitativa pareada Cuantitativa (n <30)
Test de Friedman
de comer), por lo que utilizaramos un test de t de Student para datos (>2 categoras) u ordinal
apareados, mientras que en el primer caso sera para datos indepen-
Rho de Spearman.
dientes. Ordinal Ordinal
Tau de Kendall.
La opcin correcta es la 4.

Pregunta 44.- R: 2 Pregunta 48.- R: 1


Se quiere establecer si hay diferencia entre el descenso de carga Si se quiere saber la manera de relacionarse de dos variables cuan-
viral que se consigue con indinavir frente al obtenido con saquinavir, titativas (la ecuacin que informa del tipo de relacin), se debe reali-
por lo que las variables de este estudio sern: zar un Anlisis de Regresin y, en caso de que la relacin entre dos
V1: tratamiento con indinavir o saquinavir: variable cualitativa di- variables cuantitativas sea lineal, el coeficiente de correlacin informa
cotmica. de cmo de buena es esa relacin lineal.
V2: descenso en la carga viral: variable cuantitativa. La formulacin de esta pregunta es ciertamente ambigua (y nos
Con una variable predictora cualitativa de dos categoras y una tenemos que acostumbrar a este tipo de preguntas en el MIR) y no nos
variable resultado cuantitativa, el test de contraste de hiptesis ade- permite saber por cul de los dos test tiles para dos variables cuanti-
cuado es la t de Student. tativas nos preguntan: de suerte que entre las opciones slo hay uno,
La opcin correcta es la 2. por lo que marcaremos la opcin 1 como correcta.

Pregunta 45.- R: 4 Pregunta 49.- R: 5


La mayora de las preguntas de test de contraste de hiptesis en el Si se comparan dos frmacos (como es el caso de esta pregunta), la
MIR se refieren a test paramtricos, si bien debemos conocer cundo primera variable ser cualitativa dicotmica (IECA frente a betablo-
no se pueden utilizar y es necesario elegir un test no paramtrico: queante).
Cuando el tamao muestral es pequeo (n<30). La variable resultado podra ser tambin dicotmica (disminucin
Comentarios TEST

Cuando V1 y/o V2 sean variables ordinales. o no de la TAS), pero si no lo especifican y la variable se puede medir,
Los test no paramtricos son menos potentes que los paramtricos, debemos sobreentender que la variable resultado ser cuantitativa
por ello siempre que sea posible se prefiere utilizar pruebas paramtri- (media de descenso de TAS en ambos grupos de tratamiento).
cas. Con una variable predictora cualitativa de dos categoras y una
Las pruebas no paramtricas, a diferencia de la paramtricas, no variable resultado cuantitativa, el test de contraste de hiptesis ade-
requieren que las variables sigan una distribucin normal, pueden cuado es la t de Student.
usarse independientemente de cmo sea la distribucin de las varia- La opcin correcta es la 5.
bles en la poblacin (normal, binomial, etc.). La opcin falsa es la 4. En caso de que se interpretara la V2 como una variable cualitativa
(respuesta o no al tratamiento), el test que se debe utilizar es el Chi-
Pregunta 46.- R: 4 cuadrado, que, como ves, no est entre las opciones.
Si se estn comparando tres tratamientos distintos, la variable
predictora ser el tipo de tratamiento (variable cualitativa de tres Pregunta 50.- R: 4
categoras). De todas las opciones citadas, la nica prueba til para establecer
En este estudio, la variable resultado tambin es cualitativa (cura- relacin entre una variable cualitativa y otra cuantitativa es el Test de
cin o no curacin), por lo que el test de contraste de hiptesis que se Mann-Whitney, que se usa cuando la V1 es una variable dicotmica,
debe utilizar ser el de Chi-cuadrado. la V2 es cuantitativa, las muestras son independientes y el tamao
La opcin correcta es la 4. muestral es menor de 30.
La opcin correcta es la 4.
Pregunta 47.- R: 5 El test de la t de Student tambin es til para establecer relacin
Se pretende ver si hay una diferencia entre los niveles de ADN viral entre una variable cualitativa y otra cuantitativa, pero si lees con
del VHB en pacientes nefrpatas antes de ser sometidos a hemodilisis cuidado las opciones, te dars cuenta de que no est (la opcin 1 es
y despus de esta intervencin, por lo que las variables de este estudio la distribucin t de Student, no es test de contraste de hiptesis).
son:
V1: no haber recibido todava la hemodilisis o haberla recibido ya.
V2: ttulo de ADN viral del VHB.

M exico A rgentina CTO Medicina C/ Nez de Balboa, 115 28006 MADRID (Espaa) Tfno.: (91) 782 43 32 / Fax: (91) 782 43 27
C hile U ruguay E-mail: secretaria@ctomedicina.com; iberocto@ctomedicina.com WEB: www.ctomedicina.com; www.iberocto.com
ET Pg. 7
MEDICINA PREVENTIVA
Preparacin Examen de Seleccin 05/06 1 Vuelta Seguimiento a distancia
1. Seale la opcin correcta: 7. Aislamiento se define como:

1) Vacunacin frente a meningococo-prevencin secunda- 1) Separar de la comunidad a un enfermo cortando las vas
ria. de trasmisin de la enfermedad a la poblacin sana.
2) Citologa vaginal en pacientes con antecedentes de displa- 2) Separar de la comunidad a una persona aparentemente
sia-prevencin secundaria. sana que ha estado expuesta al contagio.
3) Mamografa en familiares de pacientes con cncer de 3) Identificar el microorganismo patgeno y aislarlo para
mama-prevencin primaria. instaurar un tratamiento correcto.
4) Autopalpacin mamaria-prevencin primaria. 4) Utilizacin de material desechable.
5) Abandono de tabaco en EPOC-prevencin secundaria. 5) Separacin indefinida de sanos y enfermos.

2. Una de las siguientes afirmaciones sobre factores de riesgo 8. Sobre la situacin epidemiolgica del SIDA en Espaa, una es
NO es correcta: cierta:

1) El tabaco es causa de hipertensin arterial. 1) La tendencia de casos diagnosticados de SIDA es ascen-


2) El tabaco es la principal causa de muerte evitable. dente en los ltimos aos.
3) La obesidad se relaciona con los estados de hiperinsulinis- 2) El nmero de nuevos casos producidos en el 2001 ha sido
mo y resistencia perifrica a la insulina. mayor en mujeres que en hombres.
4) Dosis moderadas de alcohol se han relacionado con 3) Aproximadamente un 1% de los casos de SIDA que se
disminucin del riesgo de cardiopatas. produjeron en Espaa fueron debidos a transfusiones.
5) La edad es un marcador de riesgo para la hipertensin 4) La forma de transmisin ms frecuente en mujeres ha sido
arterial. la heterosexual.
5) De los casos producidos de SIDA en 2001, un 10%
3. Seale la INCORRECTA sobre la intoxicacin por plomo: corresponde a la trasmisin madre-hijo.

1) La forma ms habitual de intoxicacin en nuestro medio 9. Seale la INCORRECTA sobre la evolucin del SIDA en
es la profesional. Espaa:
2) El saturnismo es la intoxicacin crnica por plomo y es
clnicamente evidente a dosis bajas (contaminacin). 1) El mayor nmero de infecciones de SIDA en Espaa se
3) La clnica neuropsiquitrica es inespecfica. produjo entre 1985-88.
4) En el sndrome hematolgico debe hacerse diagnstico 2) La prevalencia de SIDA mxima en Espaa se alcanz en
diferencial con talasemias y anemia ferropnica. 1988.
5) La determinacin de coproporfirina III en orina no es 3) El nmero de muertes por SIDA ha disminuido en los
exclusiva del saturnismo. ltimos aos.
4) La mayora de los infectados por VIH no tienen SIDA.
5) Aunque ha disminuido el nmero de nuevas infecciones
4. La capacidad del agente infeccioso de instalarse y multiplicar-
por VIH en los ltimos aos, el nmero total de infectados
se en los tejidos, produciendo o no enfermedad, se denomi-
ha aumentado.
na:
10. Todos los siguientes son factores de riesgo en la trasmisin
1) Patogenicidad.
sexual en la infeccin por VIH, EXCEPTO:
2) Prdromos.
3) Contagiosidad.
1) Promiscuidad.
4) Infectividad.
2) Estadio avanzado de la enfermedad.
5) Virulencia.
3) lceras genitales y relaciones traumticas.
4) Otras ETS coexistentes.
Preguntas TEST

5. Aquella enfermedad que se presenta con una incidencia 5) Algunos espermicidas.


mayor del 75% en una poblacin se denomina:
11. Las siguientes medidas reducen el riesgo de trasmisin ver-
1) Mesoendemia. tical del VIH de forma significativa, EXCEPTO:
2) Endoepidemia.
3) Holoendemia. 1) Administracin de AZT a la madre desde el principio del
4) Hiperendemia. segundo trimestre hasta el final del embarazo.
5) Hipoendemia. 2) Administracin AZT al RN durante 6 meses.
3) Realizacin cesrea.
6. Aquella persona que elimina el microorganismo antes de que 4) Supresin lactancia materna.
aparezca la enfermedad que est incubando se denomina: 5) Acortar el expulsivo.

1) Portador paradjico o pseudoportador. 12. Sobre la prevalencia de la hepatitis B, una es INCORRECTA:


2) Portador precoz o incubacionario.
3) Portador convaleciente. 1) Existen aproximadamente 300 millones de personas in-
4) Portador sano. fectadas con el VHB en todo el mundo.
5) Portador pasivo o contacto. 2) En Espaa se producen ms de 10000 casos nuevos cada
ao.

M exico A rgentina CTO Medicina C/ Nez de Balboa, 115 28006 MADRID (Espaa) Tfno.: (91) 782 43 32 / Fax: (91) 782 43 27
C hile U ruguay E-mail: secretaria@ctomedicina.com; iberocto@ctomedicina.com WEB: www.ctomedicina.com; www.iberocto.com
ED Pg. 1
MEDICINA PREVENTIVA
Seguimiento a distancia Preparacin Examen de Seleccin 05/06 1 Vuelta
3) En nuestro pas, hasta un 10% de las infecciones tienen un 18. Respecto al MANTOUX, es FALSO:
curso hacia la cronicidad.
4) Asia y frica son reas de mayor prevalencia. 1) Pone de manifiesto el contacto con el bacilo (infeccin).
5) En las zonas de mayor prevalencia, la frecuencia de 2) Es una reaccin de inmunidad celular.
marcador HbsAc es baja. 3) Consiste en inyectar 5 U.I. de tuberculina subcutneas.
4) El dimetro que se mide es el de la induracin y no el del
13. En cuanto a la vacuna frente al VHB, una NO es correcta: eritema.
5) Es til para identificar candidatos para la quimioprofilaxis.
1) La eficacia de la vacuna es mayor en nios y adultos
jvenes. 19. Seale cul de las siguientes NO es indicacin para la quimio-
2) La obesidad o el hbito tabquico reducen la eficacia de profilaxis de tuberculosis con isoniacida:
la vacuna.
3) En pacientes inmunocomprometidos, la eficacia de la 1) Joven de 20 aos con Mantoux de 16 mm.
vacuna es aproximadamente del 50%. 2) Nio de 9 aos, con Mantoux negativo, y cuyo padre
4) En los pacientes inmunocompetentes que no desarrollan acaba de ser diagnosticado de tuberculosis pulmonar.
anticuerpos est indicada la revacunacin completa. 3) Hombre de 45 aos, Mantoux positivo, que va a ser
5) La duracin de la proteccin de la pauta completa de sometido a trasplante.
vacunacin parece ser menor de 5 aos. 4) Mujer de 60 aos, Mantoux positivo, a la que se le va a
realizar una histerectoma.
14. Sobre la prevencin de la hepatitis B, es FALSO: 5) Mujer de 35 aos, con Mantoux de 10 mm., que acaba
de ser diagnosticada de LES.
1) En el hijo de madre VHB debe administrarse IGHB en las
primeras horas asociando la vacunacin completa. 20. Un paciente de 60 aos acude a nuestra consulta por tos seca
2) Cuando se sospecha contacto sexual en un sujeto no y fiebre vespertina. La enfermera le pincha una tuberculina
vacunado, debe administrarse IGHB y vacuna, en un que resulta negativa. A la semana se repite la prueba que esta
plazo menor a dos semanas. vez es positiva. Todo lo siguiente es cierto, EXCEPTO:
3) La pauta de vacunacin de referencia es 0-1-6 meses.
4) No se realiza verificacin de anticuerpos de forma sistem- 1) Efecto Booster.
tica en los sujetos vacunados. 2) Debera realizarse un nuevo Mantoux.
5) Si se interrumpe la pauta de vacunacin, sta debe ser 3) La primera tuberculina pudo estar expuesta a la luz.
reiniciada. 4) La tcnica de la primera intradermorreaccin pudo ser
incorrecta.
15. En cuanto al VHA, podremos afirmar todo lo siguiente, 5) Debera hacerse a continuacin una placa de trax.
EXCEPTO:
21. En una chabola conviven 10 personas. Un varn de 34 aos
1) Se inactiva por cloracin del agua, ebullicin y luz ultra- con tos pertinaz, expectoracin escasa y fiebre persistente es
violeta. diagnosticado de TBC. En esa chabola vive una nia de 5 aos.
2) Hay portadores asintomticos que trasmiten la enferme- La actuacin ms correcta con la nia sera:
dad.
3) En pases subdesarrollados es ms frecuente en nios y 1) Inicialmente hacer Mantoux, y si es negativo, dar quimio-
jvenes. profilaxis primaria con isoniacida.
4) La inmunoprofilaxis pasiva se realiza con IgG no especfica. 2) Hacer un Mantoux, y si es positivo, vacunar con BCG.
5) La inmunoprofilaxis activa se realiza con vacuna inactiva- 3) Hacer un Mantoux, y si es negativo, repetirlo a las 8
da frente VHA. semanas.

Preguntas TEST
4) Hacer placa de trax. Si es negativa, iniciar quimioprofi-
16. El agente ms frecuentemente causal de la hepatitis postrans-
laxis secundaria.
fusional es:
5) Dar triple terapia a la nia.
1) VHA.
22. En la situacin anterior, cul sera la pauta de actuacin si
2) VHB.
viviera una joven de 23 aos?:
3) VHC
4) VHD.
1) Inicialmente hacer Mantoux, y si es negativo, dar quimio-
5) VHE.
profilaxis primaria con isoniacida.
2) Hacer un Mantoux, y si es negativo, repetirlo a las 8
17. Cul de las formas de TBC es MENOS contagiosa?:
semanas. Si ste segundo es positivo, iniciar tratamiento.
3) Si dos Mantoux son negativos y la placa de trax es
1) Larngea.
negativa, debera iniciarse quimioprofilaxis secundaria.
2) Traqueal.
4) Si el segundo Mantoux es positivo y la placa negativa,
3) Endobronquial.
debera iniciarse quimioprofilaxis secundaria.
4) Cavitacin extensa.
5) Dar triple terapia a la joven.
5) Miliar.

M exico A rgentina CTO Medicina C/ Nez de Balboa, 115 28006 MADRID (Espaa) Tfno.: (91) 782 43 32 / Fax: (91) 782 43 27
Pg. 2 ED C hile U ruguay E-mail: secretaria@ctomedicina.com; iberocto@ctomedicina.com WEB: www.ctomedicina.com; www.iberocto.com
MEDICINA PREVENTIVA
Preparacin Examen de Seleccin 05/06 1 Vuelta Seguimiento a distancia

23. En la situacin anterior, cul sera la pauta de actuacin si 29. Con respecto a la vacunacin de la meningitis meningocci-
viviera un varn de 45 aos?: ca, una NO es correcta:

1) Inicialmente hacer Mantoux, y si es negativo, dar quimio- 1) La inmunizacin conseguida con la vacuna de polisac-
profilaxis primaria con isoniacida. ridos es pequea en nios menores de 4 aos.
2) Hacer un Mantoux, y si es negativo, repetirlo a las 8 2) La vacuna de polisacridos previene frente a los serogru-
semanas. Si ste segundo es positivo, iniciar tratamiento. pos ms frecuentes en nuestro medio.
3) Si dos Mantoux son negativos y la placa de trax es 3) La vacuna conjugada es ms inmungena en nios meno-
negativa, debera iniciarse quimioprofilaxis secundaria. res de 6 aos.
4) Si el segundo Mantoux es positivo y la placa negativa, 4) La vacuna conjugada puede administrarse en nios me-
debera iniciarse quimioprofilaxis secundaria. nores de 1 ao (2-4-6 meses).
5) Si el segundo Mantoux es positivo y la placa negativa, no 5) Es una indicacin de vacunacin la asplenia.
estara indicado continuar el estudio.
30. Sobre el ttanos, seale la INCORRECTA:
24. En la situacin anterior, cul sera la pauta de actuacin si
viviera un varn de 70 aos?: 1) La prevalencia del ttanos en Espaa est en descenso.
2) La forma ms frecuente de ttanos en los pases en vas de
1) Inicialmente hacer Mantoux, y si es negativo, dar quimio- desarrollo es el ttanos neonatal.
profilaxis primaria con isoniacida. 3) El ttanos neonatal tiene una gran mortalidad.
2) Hacer un Mantoux, y si es negativo, repetirlo a las 8 4) La proteccin frente al ttanos es mayoritaria en los sujetos
semanas. Si el segundo es positivo, iniciar tratamiento. mayores de 60 aos en nuestro medio.
3) Si el Mantoux es negativo, se realiza otro a la semana y si 5) La espora del ttanos es resistente a la ebullicin.
es negativo, a las 8 semanas del primero.
4) Si el segundo Mantoux es positivo y la placa negativa, 31. En caso de vacunacin incompleta frente al ttanos, seale la
debera iniciarse quimioprofilaxis secundaria. FALSA:
5) Si el segundo Mantoux es positivo y la placa negativa, no
estara indicado continuar el estudio. 1) Un sujeto que recibi 3 dosis hace 8 aos, no debe
administrarse toxoide.
25. El agente causal ms frecuente de meningitis bacteriana entre 2) Un sujeto bien inmunizado que se vacun hace 14 aos
los 30-65 aos es: debe iniciar la pauta de vacunacin 0-1-6.
3) Un individuo que recibi 2 dosis hace 4 aos debe com-
1) Gram (-). pletar la pauta de inmunizacin con otra dosis de toxoide.
2) Meningococo. 4) Un individuo que ha recibido dos dosis hace 7 aos debe
3) H. influenzae. recibir dos dosis con un mes de intervalo.
4) Neumococo. 5) Un sujeto que recibi 1 dosis hace 6 aos, debe recibir la
5) E. coli. pauta de vacunacin completa.

26. La infeccin meningoccica se transmite por: 32. Acude a urgencias un individuo con una herida incisocontusa
de bordes anfractuosos en la pierna producida por un
1) Gotas de Pflugge. cortador de csped. Vd. interroga al paciente, quien le
2) Directamente. informa de que no est vacunado frente al ttanos, aunque
3) Por la piel. tuvo dicha enfermedad hace 4 aos. Su actitud ser:
4) En relacin con los fmites.
5) A travs del agua o los alimentos. 1) Administrar 250 U.I. de gammaglobulina antitetnica.
2) Administrar toxoide tetnico.
Preguntas TEST

27. Respecto a los serogrupos de N. meningitidis, es FALSO: 3) No hacer nada, ya que el riesgo tetangeno de la herida es
bajo.
1) El B es el ms prevalente en Espaa. 4) Administrar toxoide ms gammaglobulina.
2) El C est en aumento en Espaa. 5) No hacer nada, ya que tiene inmunidad debida a la
3) El B produce brotes limitados o casos espordicos. enfermedad pasada.
4) El A es el que tiene mayor capacidad para producir
epidemias. 33. Sobre la gripe, seale la INCORRECTA:
5) Los serogrupos A, B y C dan cuenta del 40% de los casos.
1) La enfermedad se presenta en brotes anuales durante los
28. La quimioprofilaxis de la infeccin meningoccica en emba- meses fros.
razadas se realiza con: 2) La incidencia en el grupo de nios de corta edad es muy
elevada.
1) Penicilina 1 dosis v.o. 3) Los contagios se producen habitualmente a partir del
2) Ceftriaxona 1 dosis i.m. sujeto portador asintomtico o convaleciente.
3) Rifampicina 1 dosis i.m. 4) La vacunacin se realiza de forma sistemtica en grupos de
4) Rifampicina 600 mg / 12 h. durante 2d. alto riesgo con las cepas que circularon en la temporada
5) Sulfamidas 1 dosis v.o. anterior.

M exico A rgentina CTO Medicina C/ Nez de Balboa, 115 28006 MADRID (Espaa) Tfno.: (91) 782 43 32 / Fax: (91) 782 43 27
C hile U ruguay E-mail: secretaria@ctomedicina.com; iberocto@ctomedicina.com WEB: www.ctomedicina.com; www.iberocto.com
ED Pg. 3
MEDICINA PREVENTIVA
Seguimiento a distancia Preparacin Examen de Seleccin 05/06 1 Vuelta
5) Las glicoprotenas de superficie H y N son las que se tienen 39. Seale la correcta:
en cuenta para la realizacin de las vacunas del virus A.
1) En los nios inmunocomprometidos debe administrarse
34. Con respecto a la vacunacin frente a la gripe, una es correcta: la DTPa en vez de DTP.
2) A partir de los 7 aos no se incluye componente diftrico
1) En sujetos inmunocomprometidos como el SIDA, est en la vacuna frente al ttanos.
contraindicada. 3) La eficacia de la DTP es superior al 90%.
2) No debe administrarse durante el embarazo. 4) Cuando se asocia el antgeno Hib a al DTP, es preciso
3) Puede administrarse en nios con fibrosis qustica a los 3 continuar con la pauta de vacunacin hasta los 12 aos.
meses de edad. 5) Es importante reiniciar la pauta de vacunacin si el nio
4) En algunos casos de vacunacin con componentes de H ha saltado una de las dosis.
humana, se han descrito casos de Guillain- Barr.
5) Puede administrarse aunque exista anafilaxia documen- 40. Con respecto a la vacuna triple vrica, es FALSO:
tada a la neomicina.
1) La eficacia desde la primera dosis es muy elevada.
35. Con respecto a las vacunas, una de las siguientes es INCO- 2) Puede indicarse en menores de 15 meses si la situacin
RRECTA: epidemiolgica lo aconseja, no evitando la dosis a los 12-
15 meses.
1) Las vacunas vivas suelen precisar de varias administracio- 3) Si debe realizarse el test de Mantoux, debern esperarse
nes para conseguir buena respuesta inmunitaria. varias semanas tras la vacunacin TV.
2) Las vacunas inactivas rara vez son enterales. 4) En mujeres embarazadas no inmunizadas puede vacunar-
3) Las vacunas inactivas suelen precisar de adyuvantes. se para evitar el sarampin neonatal.
4) La presencia de un sujeto inmunodeprimido en un domi- 5) Debe evitarse su administracin en alrgicos al huevo.
cilio no contraindica la vacunacin con virus tipo Salk a
los otros sujetos de la familia. 41. Con respecto al calendario vacunal del adulto, la opcin
5) La produccin de IgA es frecuente tras la administracin INCORRECTA es:
de la vacuna Sabin.
1) Deben asegurarse inmunizacin frente a neumococo y
36. Con respecto a las reacciones adversas de las vacunas, una gripe los mayores de 65 aos.
es FALSA: 2) A partir de los 45 aos no se aconseja la inmunizacin de
rubola.
1) Ndulo cutneo Td. 3) No es necesaria la vacunacin frente al ttanos en mayores
2) Adenitis locorregional BCG. de 65 aos.
3) Vesculas varicela. 4) El componente de la vacuna del ttanos para adulto
4) Exantema generalizado primera semana tras la vacuna- contiene tambin una pequea dosis de toxoide frente a
cin con rubola. la difteria.
5) Artralgias tercera semana tras la vacunacin anti-rubola. 5) No se aconseja la vacunacin frente a la polio en los
adultos.
37. Frente a uno de los siguientes microorganismos NO existe un
suero especfico con eficacia probada para la inmunizacin 42. Con respecto a la inmunizacin del nio VIH, una es FALSA:
pasiva:
1) Debe evitarse la vacunacin con microorganismos vivos si
1) Hepatitis A. es posible.
2) Hepatitis B. 2) Se aconseja la vacunacin frente a varicela y TV en nios
3) Varicela Zoster. no muy inmunodeprimidos.
4) Ttanos. 3) Si un nio se realiza una herida sucia y ha seguido el

Preguntas TEST
5) Difteria. calendario vacunal correctamente, no se aconseja admi-
nistrar gammaglobulina.
38. Seale la INCORRECTA respecto a la inmunizacin pasiva: 4) Si en una guardera un nio desarrolla sarampin y asiste
un nio VIH oligosintomtico, bien vacunado frente a
1) La inmunoglobulina hiperinmune frente a VHB se utiliza TV, debe recibir inmunoglobulina frente al sarampin.
en la profilaxis postexposicin. 5) Se aconseja la vacunacin con polio tipo Salk si el nio
2) Est indicada la gammaglobulina inespecfica frente VHA VIH + no padece SIDA.
si se va a estar en riesgo o se ha estado en contacto con un
caso en un periodo de tiempo menor de 2 semanas. 43. Una de las siguientes aseveraciones es FALSA respecto al
3) Un nio VIH que ha seguido correctamente el calendario cncer:
vacunal y que est en contacto con un caso de sarampin,
debe recibir gammaglobulina.
1) El 50% de los cnceres se diagnostican en el mundo
4) La inmunoglobulina frente VHZ debe administrarse a un
desarrollado.
sujeto inmunocomprometido en las primeras 96 horas
2) El cncer ms frecuente en los pases occidentales es el
tras el contacto con un caso.
cncer de pulmn.
5) En un nio de 6 meses en contacto con un caso de
3) El cncer ms frecuente en las mujeres es el cncer de
sarampin hay que dar inmunoglobulina y vacunar a las
mama.
4 semanas de recibir la inmunoglobulina.

M exico A rgentina CTO Medicina C/ Nez de Balboa, 115 28006 MADRID (Espaa) Tfno.: (91) 782 43 32 / Fax: (91) 782 43 27
Pg. 4 ED C hile U ruguay E-mail: secretaria@ctomedicina.com; iberocto@ctomedicina.com WEB: www.ctomedicina.com; www.iberocto.com
MEDICINA PREVENTIVA
Preparacin Examen de Seleccin 05/06 1 Vuelta Seguimiento a distancia

4) La primera causa de aos potenciales de vida perdidos en 1) El principal marcador de riesgo es el sexo masculino.
la mujer es el cncer de mama. 2) El principal predictor para cardiopata isqumica en los
5) El cncer de crvix es, como el de mama, un cncer que varones de edades medias es el colesterol.
aparece con mayor frecuencia en los pases desarrollados. 3) El aumento de la mortalidad por cardiopata isqumica es
ms evidente con niveles sricos de colesterol superiores
44. Con respecto al tabaco, seale la FALSA: a los 240 mg/dl.
4) Se observa un aumento de los niveles plasmticos de
1) Se considera que la forma ms oncognica es el consumo colesterol y triglicridos con la edad.
de cigarrillos. 5) La obesidad parece ser un factor determinante en la
2) El cncer de labio es especialmente frecuente en los elevacin de colesterol y triglicridos con la edad.
individuos que fuman en pipa.
3) Est implicado en un 10% de los cnceres que afectan a 49. Cul de las siguientes es FALSA respecto a la influencia del
la mujer. tabaco en el desarrollo de enfermedad cardiovascular?:
4) El riesgo de cncer es mayor cuanto mayor es el consumo
de tabaco. 1) Existe una clara relacin dosis - respuesta entre el tabaco
5) Los fumadores pasivos no tienen mayor riesgo de cncer. y el riesgo de cardiopata isqumica.
2) El tabaco es el ms importante de los factores de riesgo
45. Indique cul de las siguientes afirmaciones sobre factores modificables.
hormonales y cncer NO es correcta: 3) El riesgo de cardiopata isqumica es menor en mujeres
fumadoras que en varones fumadores.
1) Los andrgenos estn implicados en la gnesis del cncer 4) El riesgo de cardiopata isqumica tarda unos 10 aos en
de prstata. descender de forma importante tras el abandono del
2) Los anticonceptivos orales disminuyen el riesgo de pade- tabaco.
cer cncer de ovario. 5) El tabaquismo no es factor de riesgo para el desarrollo de
3) La nuliparidad es un factor protector para el cncer de hipertensin arterial.
ovario.
4) La terapia sustitutiva sin progestgenos en la menopausia 50. Con respecto a las hiperlipidemias y a la enfermedad cardio-
aumenta el riesgo de cncer de endometrio. vascular, una NO es correcta:
5) El embarazo que se produce a edades tempranas dismi-
nuye el riesgo de padecer cncer de mama. 1) Son el principal indicador de cardiopata isqumica en el
varn de mediana edad.
46. Indique cul de las siguientes NO es correcta: 2) Cursan frecuentemente con aterosclerosis del sector aor-
toilaco.
1) El riesgo de cncer de pulmn disminuye al dejar de 3) Las dietas ricas en grasas saturadas elevan el c-LDL.
fumar, regresando al nivel del no fumador a los 10 aos. 4) No es posible frenar la progresin de la placa de ateroma
2) La placa de trax realizada a pacientes de alto riesgo ya formada.
(varones mayores de 50 aos) no ha demostrado disminuir 5) El ejercicio fsico produce una elevacin del c-HDL.
la mortalidad por cncer de pulmn.
3) La elevacin del PSA es la prueba ms sensible para el
diagnstico precoz del cncer de prstata.
4) Ni la fosfatasa cida ni el PSA son suficientemente espec-
ficos como para realizar screening poblacional.
5) La citologa de Papanicolaou es poco sensible en el
diagnstico de adenocarcinoma de endometrio.
Preguntas TEST

47. Una de las siguientes es FALSA respecto a la epidemiologa de


las enfermedades cardiovasculares:

1) Los factores socioculturales parecen ms importantes que


la determinacin gentica en el desarrollo de cardiopata
isqumica.
2) Las enfermedades cardiovasculares son la principal causa
de muerte en los pases desarrollados.
3) En Espaa ha disminuido en los ltimos aos la mortalidad
debida a cardiopata isqumica.
4) La tercera parte de las muertes por cardiopata isqumica
ocurren antes de los 65 aos (son prematuras).
5) La enfermedad cerebrovascular es causa de un mayor
porcentaje de las muertes que la cardiopata isqumica.

48. Sobre los factores y/o marcadores de riesgo para las enferme-
dades cardiovasculares, una es FALSA:

M exico A rgentina CTO Medicina C/ Nez de Balboa, 115 28006 MADRID (Espaa) Tfno.: (91) 782 43 32 / Fax: (91) 782 43 27
C hile U ruguay E-mail: secretaria@ctomedicina.com; iberocto@ctomedicina.com WEB: www.ctomedicina.com; www.iberocto.com
ED Pg. 5
MEDICINA PREVENTIVA
Preparacin Examen de Seleccin 05/06 1 Vuelta Seguimiento a distancia
Pregunta 1.- R: 2 Pregunta 4.- R: 4
Los niveles de prevencin en Medicina son tres. La prevencin pri- Las enfermedades transmisibles son las producidas por un agente
maria corresponde a las medidas de prevencin que se realizan antes vivo. La cadena epidemiolgica se compone de reservorio-mecanis-
de que aparezca la enfermedad; es decir, se llevan a cabo cuando el mo de transmisin-husped susceptible. Desde el punto de vista epi-
sujeto susceptible est en contacto con el factor de riesgo pero todava demiolgico, la forma de reservorio de mayor importancia son los
no ha aparecido la enfermedad. Un ejemplo de esto seran las opcio- portadores. stos pueden ser agudos si eliminan el germen durante un
nes 1 y 3. Con respecto a la opcin 3, aunque la mamografa se realice periodo inferior a 6 meses; crnicos, cuando lo eliminan durante ms
en familiares de cncer de mama, no se va a evitar la enfermedad, sino de 6 meses; y permanentes, cuando se eliminan de por vida. Las
que se va a hacer un diagnstico precoz. Las medidas de prevencin enfermedades transmisibles tienen habitualmente dos periodos clni-
secundaria se realizan cuando ya ha aparecido la enfermedad, pero se cos claramente diferenciados. Uno inicial, caracterizado por la apari-
diagnostica en una fase precoz (habitualmente en fase asintomtica). cin de clnica inespecfica y que se denomina prdromos (respuesta
Son medidas de prevencin secundaria, por tanto, todas las pruebas de 2), y otro posterior, en el que ya son evidentes los signos y sntomas
screening tales como la opcin 2 y 3. La autopalpacin mamaria, si propios de la enfermedad. Segn sea esta interrelacin entre el hus-
bien no ha demostrado disminuir la mortalidad por cncer de mama, ped y el microorganismo, se definen los trminos de:
puede considerarse prevencin secundaria al permitir diagnosticar Contagiosidad: es la capacidad de propagacin. Se mide segn la
precozmente la enfermedad. La prevencin terciaria va encaminada a tasa de ataque (que no es una tasa realmente sino la incidencia
tratar las secuelas de la enfermedad as como las recurrencias, tal y acumulada de una enfermedad infecciosa aguda).
como se refiere en la opcin 5. Infectividad: es la capacidad del microorganismo de instalarse en
el husped. Puede estacionarse aqu o evolucionar produciendo
Pregunta 2.- R: 1 enfermedad.
El tabaco es el factor de riesgo ms importante de enfermedades Patogenicidad: es la capacidad de invasin. Por tanto, es la capaci-
cardiovasculares y de cncer, suponiendo la principal causa de muerte dad de producir enfermedad. Cuando esta capacidad es muy gran-
evitable en los pases desarrollados. Aun cuando las personas ya ha- de se dice que la infeccin es muy virulenta.
yan cumplido los 65 aos, se benefician de la interrupcin del consu-
mo de tabaco. No se ha demostrado sin embargo que sea causa direc- En el caso de la tuberculosis, el diagnstico de infectividad se rea-
ta de hipertensin arterial (opcin 1). lizara mediante la prueba de la tuberculina, mientras que el de enfer-
Otro factor de riesgo importante en los pases desarrollados funda- medad tuberculosa por el aislamiento del germen.
mentalmente es la obesidad. El exceso de grasas saturadas en la dieta
se ha relacionado con el desarrollo de aterosclerosis, as como con Pregunta 5.- R: 3
Las enfermedades trasmisibles pueden presentarse de dos maneras
diversos tipos de cncer como el de mama, tero, colon, prstata y
muy distintas. En primer lugar, pueden manifestarse como caso aislado
pulmn. Adems, la obesidad per se, sobre todo la de perfil
o espordico, sin repercusin desde el punto de vista epidemiolgico,
andrognico, se ha relacionado con los estados de resistencia a la o de forma ms o menos constante (prevalente) en la poblacin. Segn
insulina e hiperinsulinismo. este nivel de prevalencia se habla de hipoendemia si afecta a menos de
Tambin son importantes otros factores tales como la ingesta de un 10% de la poblacin; mesoendemia, si afecta hasta un 50%;
alcohol, que si bien en dosis excesivas puede desencadenar miocar- hiperendemia, si hasta un 75%, y holoendemia si ms del 75% (res-
diopatas o enfermedades hepticas, en dosis bajas se ha relacionado puesta 3). Adems de estos casos prevalentes pueden aparecer en la
con menor riesgo de cardiopata isqumica. poblacin casos nuevos de enfermedad (casos incidentes) que dan
La edad avanzada y el sexo masculino son los principales marca- lugar a epidemias de gran impacto en la poblacin. En Espaa, el
dores de riesgo para el desarrollo de aterosclerosis. Obsrvese que se sistema de vigilancia y recogida de datos ms importante es el sistema
define marcador de riesgo cuando no puede evitarse, mientras que EDO o enfermedad de declaracin obligatoria. En este sistema EDO se
factor de riesgo cuando ste es evitable. calcula el ndice epidmico, que es la razn entre la incidencia de la
semana en curso y la incidencia de la mediana del quinquenio ante-
Pregunta 3.- R: 2 rior para la misma semana. Si el brote no alcanza el grado de epidemia
La intoxicacin crnica por plomo se denomina saturnismo y tie- pero la prevalencia es mayor de la habitual, se dice que se est ante
ne que ver fundamentalmente con la ingesta en determinados grupos una endoepidemia. El trmino pandemia hace referencia no a la pre-
profesionales de riesgo tales como empleados de la industria siderr- valencia de la enfermedad sino a su extensin, refirindose a la enfer-
Comentarios TEST

gica. Inicialmente la ingestin de plomo da lugar a una fase asintomtica medad que est presente a nivel mundial.
que se define como contaminacin (por eso es incorrecta la opcin
2) y posteriormente, a una fase clnicamente evidente que se denomi-
Pregunta 5. Presentacin de las enfermedades transmisibles.
na intoxicacin. Esta fase de intoxicacin puede manifestarse
clnicamente de 4 formas distintas: Tipos de endemia.
Sndrome neuropsiquitrico: se caracteriza por la aparicin de
irritabilidad, convulsiones, neuritis perifricas o hipertensin intra- 1234562347 89
73429763757
craneal.
Sndrome gastrointestinal: tambin muy inespecfico, puede pre- 4 625647 1232 456
sentarse como estreimiento, dolor abdominal tipo clico, mal 6 625647 789
2 56 6
sabor de boca y por un tpico ribete gingival (de Burton).
Sndrome hematolgico: es muy caracterstica la anemia microc- 46625647 2 6 
tica e hipocroma con punteado basfilo en los hemates.
Sndrome urinario: aparecen aminoaciduria o glucosuria. Puede  625647 722 
detectarse de manera muy tpica coproporfirina III en orina, si bien
no es patognomnico, ya que puede aparecer en la intoxicacin Pregunta 6.- R: 2
por talio (matarratas). Si se nos pregunta cul es la definicin correcta de portador, lo
definiremos como la persona que, sin presentar signos o sntomas de
Adems de estos cuadros sindrmicos, la intoxicacin por plomo se la infeccin, es capaz de eliminar microorganismos (MO). Correspon-
manifiesta de forma muy caracterstica como clico saturnino, que es de a la definicin de portador sano. Estos portadores se llaman agudos
un dolor abdominal tipo clico que no cede a las medidas habituales, cuando eliminan grmenes durante un tiempo inferior a 6 meses,
pero s de forma muy caracterstica a la administracin de una sal clci- crnicos si lo eliminan durante ms tiempo y permanentes cuando lo
ca. Esta forma de intoxicacin suele aparecer en los adultos. Tambin es eliminan de por vida. Sin embargo, hay circunstancias que definen
caracterstica del saturnismo la lesin del nervio radial. tipos concretos de portadores. Definimos como portador paradjico

M exico A rgentina CTO Medicina C/ Nez de Balboa, 115 28006 MADRID (Espaa) Tfno.: (91) 782 43 32 / Fax: (91) 782 43 27
C hile U ruguay E-mail: secretaria@ctomedicina.com; iberocto@ctomedicina.com WEB: www.ctomedicina.com; www.iberocto.com
ED Pg. 1
MEDICINA PREVENTIVA
Seguimiento a distancia Preparacin Examen de Seleccin 05/06 1 Vuelta
o pseudoportador al que elimina MO que no son patgenos, como en constante aumento, alcanzando en 2000 la cifra de 140000 per-
por ejemplo, la transmisin que se produce en la infancia de N. sonas vivas con VIH y libres de SIDA en Espaa.
lactamica (que confiere inmunidad cruzada mediada por el comple- Si tenemos en cuenta que el SIDA es la causa ms frecuente de
mento frente a N. meningitidis). El portador precoz o incubacionario muerte en muchas zonas de Espaa entre los 25 y los 35 aos, esta
corresponde al enunciado de la pregunta. En este caso los MO se enfermedad supone una de las causas ms importantes de aos de
eliminan antes de la fase clnica, como ocurre en el caso de la VHA, vida perdidos, de forma que en estas edades la expectativa de vida no
que se elimina en heces antes de que aparezca la fase ictrica de la ha aumentado en los ltimos 25-30 aos.
infeccin. Por el contrario, un portador convaleciente lo es cuando
elimina el MO despus del acm de la enfermedad y antes de que sta Pregunta 10.- R: 5
se haya resuelto completamente. El portador pasivo es un eslabn La forma de transmisin sexual es la ms frecuente a nivel mundial
entre el verdadero portador que elimina MO y el husped suscepti- (pases con patrn tipo I de distribucin de la OMS) y se est convir-
ble. Esto ocurre, por ejemplo, cuando al explorar un nio con diarrea tiendo en una forma cada vez menos frecuente de contagio en los
por rotavirus, por descuido no nos lavamos las manos y a continua- pases desarrollados debido a la concienciacin de los grupos tradi-
cin se explora a un nio susceptible de infeccin por este virus. cionales de riesgo (ADVP, homosexuales).
Cualquier forma de relacin sexual en la que estn involucrados el
Pregunta 7.- R: 1 semen o las secreciones vaginales es contagiosa. El riesgo heterosexual
Se define cuarentena como el aislamiento que se produce de una es mayor del hombre a la mujer que viceversa, lo que tiene que ver
persona sana que ha estado expuesta al contagio de una enfermedad. con la cantidad de vehculo infeccioso. El sexo oral es una forma ms
Las enfermedades cuarentenables son el clera, la fiebre amarilla (am- difcil de transmisin porque la ptialina (maltasa) salival tiene la capa-
bos durante 5 das) y la peste (6 das). El aislamiento consiste en cidad de inactivar el VIH. Por el contrario, la relacin anal es de ms
apartar de la comunidad a un enfermo para que as no transmita la fcil contagio debido a que la mucosa del recto no est preparada
enfermedad a la poblacin sana. Segn la forma de transmisin de la para el traumatismo del coito.
enfermedad se habla de un aislamiento estricto (el mayor nivel de Como en cualquier forma de transmisin, sta es ms frecuente en
aislamiento), respiratorio, entrico, de piel y mucosas y protector. To- fases avanzadas de la enfermedad, debido a una mayor viremia. Por
das las formas de aislamiento tienen en comn el lavado de manos otra parte la presencia de lesiones genitales inflamatorias o lceras
como barrera en la transmisin de la enfermedad. En el aislamiento (recuerda que la causa ms frecuente de lcera genital en nuestro
respiratorio no es necesario el empleo de guantes, aunque s mascari- medio es el VHS) aumenta el riesgo de transmisin sexual.
lla. Son ejemplos de este tipo de aislamiento la meningitis o la tuber- Los mtodos barrera son los nicos que pueden disminuir la trans-
culosis. En el aislamiento entrico es especialmente importante el misin sexual. Adems de los preservativos es aconsejable el empleo
empleo de guantes, siendo ejemplo de este tipo de aislamiento la del espermicida nanoxinol 9, que inhibe el VIH. No obstante este
infeccin por rotavirus o el clera. El aislamiento de piel y mucosas se espermicida nunca deber emplearse como mtodo profilctico sin
hace en enfermedades como las estafilococias o el zoster diseminado. condn.
El aislamiento protector hace referencia al aislamiento del sujeto
inmunosuprimido, para evitar que ste se contagie a partir de Pregunta 11.- R: 2
microorganismos ubicuos en el medio ambiente. La transmisin materno-infantil se ha reducido en pases como
Espaa (1% de los casos nuevos de SIDA en 2001), pero supone una
Pregunta 8.- R: 3 forma muy frecuente de contagio en los pases desarrollados. El mxi-
El nmero de casos de SIDA diagnosticados en 2001 ha sido de mo riesgo de infeccin corresponde a fases de viremia maternas eleva-
2300. Esto supone un descenso en el nmero de casos con respecto das como son el embarazo en etapas tardas de la enfermedad o la
al ao anterior y refleja una tendencia iniciada en 1995. El hecho primoinfeccin que coincide con la gestacin. Menos de un 20% de
determinante de que esto se est produciendo es la aparicin de los los recin nacidos vivos de madre con VIH se infectan y, con la intro-
retrovirales. Por tanto, si tenemos en cuenta que el SIDA es la conse- duccin del tratamiento antirretroviral, la modificacin de la prctica
cuencia ltima de la infeccin del VIH, la reduccin del nmero de obsttrica (parto por cesrea) y la supresin de la lactancia materna
casos de SIDA no implica obligatoriamente la disminucin del nme- (pases desarrollados), este riesgo ha descendido hasta un 5%. Si bien
ro de infecciones por VIH. En Espaa, tradicionalmente el grupo de la transmisin puede realizarse en cualquier momento del embarazo,
riesgo ms castigado por el SIDA lo han sido los UDVP. Este grupo sigue es en el momento del parto cuando tiene lugar el mayor riesgo, por lo
siendo el ms numeroso de forma global seguido del grupo de que se recomienda la cesrea cuando sea factible realizarla. La segu-

Comentarios TEST
heterosexuales. No obstante, los casos de SIDA en mujeres en el grupo ridad y eficacia de la mayora de los antirretrovirales es desconocida,
de heterosexuales se acercan mucho al de los UDVP. salvo para el AZT. Se recomienda el tratamiento con este frmaco a la
Pese a los controles de laboratorio, todava existe riesgo de SIDA a madre desde el segundo trimestre de embarazo y hasta 6 semanas en
partir de hemoderivados. Se estima que 1 de cada 400000 concen- el neonato. Sin embargo parece lgico pensar que la asociacin de
trados de hemates de un banco de sangre estn infectados por el VIH. frmacos pueda ser ms eficaz. Otros antirretrovirales que parecen
Esta forma de transmisin del SIDA supuso el ao 2001 el 1% de los eficaces y seguros son el ddI, el 3TC y la nepiravina.
casos.
Aunque el riesgo de transmisin madre-hijo ha disminuido con las Pregunta 12.- R: 5
nuevas terapias y con el diagnstico prenatal, ste todava existe, su- Se estima que hasta un 5% de la poblacin mundial es portadora
poniendo tambin el 1% de los casos diagnosticados en el 2001. del virus de la hepatitis B, manteniendo una endemicidad ms o me-
nos constante segn los distintos pases. Esta elevada prevalencia oca-
Pregunta 9.- R: 2 siona unos 2 millones de muertes anuales causadas por el VHB.
Si tenemos en cuenta que el SIDA es la ltima etapa de la infeccin Puede hablarse de 3 niveles de endemicidad. Obsrvese que, cuanto
por VIH y que sta se produce a los 10 aos aproximadamente de la mayor es la prevalencia de la enfermedad, tambin es mayor la preva-
infeccin, podremos entender cul ha sido la evolucin del SIDA en lencia de anticuerpos protectores anti-HBs (respuesta 5 incorrecta).
Espaa en los ltimos 15-20 aos. La mayor incidencia de infeccin Mientras que en los pases de prevalencia baja (<1%) la presencia de
por VIH en nuestro pas tuvo lugar 3-4 aos despus de que fueran anticuerpos protectores es aproximadamente del 5%, en los pases de
descritos los primeros casos en los homosexuales de Estados Unidos. prevalencia media (aprox. 5%), los niveles serolgicos de proteccin
El pico de incidencia de VIH ocurri en 1985, por lo que el pico de estn entre un 20-50%. Asia y frica son las reas con mayor prevalen-
casos nuevos de SIDA y de mortalidad por SIDA se produjo en 1995. cia (10-20%) con unos niveles de anti-HBs en torno a un 70-95%.
Desde que la poblacin tiene ms conciencia de la enfermedad y por En Espaa, la incidencia es de 12000 casos al ao. De ellas slo el
la llegada de la nuevas terapias, el nmero de infecciones nuevas ha 10% evoluciona a la cronicidad y un 2% desarrolla hepatocarcinoma
descendido, pero no as la prevalencia de VIH, que se ha mantenido (recuerda que slo evolucionan a la cronicidad las hepatitis de trans-

M exico A rgentina CTO Medicina C/ Nez de Balboa, 115 28006 MADRID (Espaa) Tfno.: (91) 782 43 32 / Fax: (91) 782 43 27
Pg. 2 ED C hile U ruguay E-mail: secretaria@ctomedicina.com; iberocto@ctomedicina.com WEB: www.ctomedicina.com; www.iberocto.com
MEDICINA PREVENTIVA
Preparacin Examen de Seleccin 05/06 1 Vuelta Seguimiento a distancia
misin no enteral). En Espaa, la evolucin a la cronicidad no es tan desde un sujeto enfermo. Es importante recordar que el periodo de
elevada como en Asia o frica debido a la forma de transmisin pre- mxima contagiosidad no ocurre en la fase ictrica sino en el periodo
dominante de la enfermedad. En las reas con mayor prevalencia la preclnico (el aislamiento por tanto en esta enfermedad no tiene nin-
transmisin es fundamentalmente de madre a hijo, lo que se acompa- gn sentido ya que cuando es clnicamente relevante el sujeto ya no
a de desarrollo de cronicidad, hepatopata crnica y cncer. Sin es contagioso).
embargo, en Espaa el grupo de mayor riesgo lo suponen los conta- Al transmitirse fecal-oral, es muy importante el agua como vehculo
gios por va parenteral que suelen producirse por tanto en adultos y se contaminante por lo que hay que recordar que mediante la ebulli-
acompaan habitualmente de menor riesgo de cronicidad. cin, la cloracin y con la radiacin U.V. el virus se inactiva.
Los grupos de riesgo son distintos segn la prevalencia de la enfer-
Pregunta 13.- R: 5 medad. En los pases donde la hepatitis A es endmica el grupo funda-
La vacuna que se emplea en la actualidad frente a la hepatitis B es mental de riesgo lo suponen los nios que frecuentemente presentan
la de segunda generacin o de ADN recombinante. La pauta habi- formas subclnicas de la enfermedad, mientras que en los pases desa-
tual de aplicacin consiste en 3 dosis (recuerda que slo suelen rrollados en los que la prevalencia es baja, el grupo fundamental de
precisar una dosis las vacunas de microorganismos vivos atenuados). riesgo lo suponen los individuos que emigran a un pas de elevada
La pauta es 0-1-6 meses con una elevada eficacia. Un 90% de los prevalencia.
adultos sanos y un 95% de nios y jvenes alcanzan niveles protec-
tores (anti-HBs mayor de 10 mUI/ml). La edad mayor de 40 aos, los Pregunta 16.- R: 3
fumadores y los obesos presentan una respuesta a la vacuna ms El virus de la hepatitis que es causa ms frecuentemente de hepa-
reducida. Asimismo, en inmunosuprimidos, la proteccin es alcan- titis postrasfusional es el virus VHC. El riesgo de transmisin percut-
zada por un 40-60% de los sujetos. No se conoce el hecho de por nea de este virus es muy elevada, por lo que ante un contacto percu-
qu algunos sujetos sanos no responden a la vacunacin. En stos se tneo accidental del personal sanitario el riesgo de transmisin de
recomienda volver a vacunar con la pauta completa, y si de nuevo este virus es mayor que para el VHB o el VIH. La nica forma de
no alcanzan ttulos protectores, no se vuelve a intentar la vacuna- transmisin conocida es la percutnea por lo que este virus es muy
cin. prevalente en Espaa dentro del grupo de los ADVP. Sin embargo,
La vacuna frente a VHB no slo es una vacuna eficaz, sino que hay que recordar que el grupo ms importante de sujetos que pre-
adems induce la formacin de anticuerpos neutralizantes a largo sentan infeccin por el VHC lo constituyen sujetos que no pertene-
plazo. Aun con ttulos bajos, el contacto con el VHB produce una cen a ninguno de los grupos de riesgo tpicos. No se conoce ninguna
respuesta inmunolgica eficaz incluso trascurridos ms de 10 aos. medida de profilaxis ni pre-exposicin ni post-exposicin, por lo
Debes recordar que como la vacuna no es del virus completo sino que cualquier personal sanitario debe considerar a todo paciente
de una fraccin (HBs-Ag), el nico anticuerpo que puede aparecer al como potencial transmisor de la enfermedad. Recurdese que los
hacer la serologa frente a VHB en un vacunado es el anti-HBs. Por otra virus A y E son de transmisin fecal-oral, por lo que no pueden
parte, tambin es importante que se recuerde que la vacuna es una considerarse como agentes etiolgicos de la hepatitis postrasfusional.
profilaxis pre-exposicin, por lo que no est indicada cuando un
sujeto presente ya en plasma cualquier tipo de antgeno o anticuerpo. Pregunta 17.- R: 5
La tuberculosis (TBC) es una enfermedad crnica infecciosa muy
Pregunta 14.- R: 5 prevalente a nivel mundial. Se produce por el contacto del husped
La profilaxis frente a la hepatitis B puede ser: con el M. tuberculosis o M. bovis. Este microorganismo es un bacilo
1) Profilaxis pre-exposicin: Vacuna. cido-alcohol resistente que produce el granuloma con necrosis
La pauta de referencia es 0-1-6 meses. En Espaa forma parte del caseosa como lesin histopatolgica especfica. Recuerda que la ma-
calendario vacunal a los 0-2-6 meses de vida o las 3 dosis a los 11 yora de las micobacterias son muy sensibles al medio ambiente por
aos en aquellos nios que no estuvieran vacunados previamente. lo que la tuberculosis o la lepra se transmiten fundamentalmente en
Cuando se precise una inmunizacin rpida se pueden aproximar situaciones de hacinamiento. El bacilo tuberculoso (BT) es sensible a
las 3 dosis, asegurando la inmunidad mediante una dosis al ao (0- la luz UV por lo que ventilar el hogar donde vive un tuberculoso es
1-2-12). El hecho de interrumpir el programa de vacunacin no una forma muy eficaz de disminuir el riesgo de transmisin a otros
implica reanudarlo nuevamente sino continuarlo donde se aban- convivientes. La transmisin es de persona a persona con contacto
don (recuerda que esto es lo habitual para cualquier tipo de estrecho y se produce a travs de las gotas de Pflugge que se producen
Comentarios TEST

vacuna). durante la tos. La primoinfeccin suele cursar de forma asintomtica,


No se recomienda determinacin de ttulos frente a VHB de forma quedando el BT acantonado en los macrfagos. El riesgo mayor de
sistemtica salvo a personas especialmente en riesgo (trabajadores infeccin lo presentan los nios y los adultos jvenes que desarrollan
sanitarios, requerimiento de politrasfusin, en lista de trasplante) o la clnica en forma de tuberculosis secundaria habitualmente 2-3
el inmunosuprimidos (VIH, trasplantados, hemodilisis). aos tras la primoinfeccin. Como el contagio es fundamentalmente
2) Profilaxis post-exposicin: Vacuna + Inmunoglobulina. va respiratoria las formas de mayor riesgo de transmisin son las que
Exposicin perinatal: IGHB especfica hiperinmune en las 12 pri- afectan a la va area (larngea, traqueal, endobronquial o de cavita-
meras horas y vacunacin (la proteccin para el RN con esta pauta cin pulmonar extensa).
es superior al 95%).
Exposicin en el lactante que convive con infectado de forma Pregunta 18.- R: 3
aguda: IGHB + vacuna lo antes posible. La prueba tuberculnica (PT) consiste en la intradermorreaccin de
Contacto sexual: vacuna + IGHB en las 2 primeras semanas. Mantoux que, como su nombre indica, consiste en la inoculacin
Contacto percutneo. Si el sujeto expuesto no est vacunado o si intradrmica (la inoculacin subcutnea puede inducir a error en la
lo est pero desconoce su serologa o es negativa: vacuna + IGHB interpretacin del resultado) del PPD (antgeno purificado). La lectura
lo antes posible. Si presenta ttulos protectores de anticuerpos no es del resultado debe hacerse a las 48 horas y no antes ya que la inmuni-
necesario ni vacuna ni IGHB. dad frente a la TBC es celular (mediada por linfocitos y por tanto ms
lenta). La inoculacin se realiza en el antebrazo del sujeto y lo que se
Pregunta 15.- R: 2 mide es el dimetro transverso de la ppula (se mide el dimetro palpa-
El virus VHA es un picornavirus (ARN virus) monocatenario de trans- ble y no el eritema). Si la PT es positiva, esto slo indica que el sujeto ha
misin fecal-oral. Recuerda que los virus de la hepatitis que se trans- estado en contacto con el bacilo (infeccin), lo que permitir diagnos-
miten de esta forma (VHA y VHE) nunca cronifican por lo que no ticar a los sujetos que son candidatos a la quimioprofilaxis. Un Mantoux
pueden desarrollar cirrosis y hepatocarcinoma. Si no tienen la capa- positivo por tanto slo indica infeccin pero no nos informa de si
cidad de cronificar esto quiere decir que la opcin 2 es incorrecta: no adems el sujeto es un individuo enfermo. Para esto ltimo debera
hay portadores asintomticos y la infeccin siempre se va a transmitir realizarse una placa de trax cuando se est realizando bsqueda de

M exico A rgentina CTO Medicina C/ Nez de Balboa, 115 28006 MADRID (Espaa) Tfno.: (91) 782 43 32 / Fax: (91) 782 43 27
C hile U ruguay E-mail: secretaria@ctomedicina.com; iberocto@ctomedicina.com WEB: www.ctomedicina.com; www.iberocto.com
ED Pg. 3
MEDICINA PREVENTIVA
Seguimiento a distancia Preparacin Examen de Seleccin 05/06 1 Vuelta
casos secundarios o, de forma definitiva, mediante el cultivo. Recuerda un sujeto infectado va a presentar un M negativo (perodo ventana). Si,
que la baciloscopia puede ser positiva en sujetos que siguen tratamien- como en el caso de la pregunta, el primer M fue negativo y a la semana
to tuberculosttico sin que ello implique resistencias tuberculosas ya se positiviza, hay que pensar como primera posibilidad en un efecto
que esos bacilos no suelen ser viables. Booster, que consiste en que, en los pacientes de edad avanzada, la
inmunidad frente al bacilo TBC se encuentra atenuada y el primer M
Pregunta 19.- R: 4 sirve para reactivar esta memoria inmunolgica, de forma que ser po-
La quimioprofilaxis (QP) antituberculosa se realiza por definicin sitivo el segundo M si el sujeto estuvo infectado en el pasado. Si ambos
en sujetos no enfermos. M realizados en el plazo de una semana son negativos, se descarta
Esta quimioprofilaxis puede ser de dos tipos: efecto Booster y concluimos definitivamente que el sujeto nunca estuvo
QP primaria: se realiza en sujetos que an no tienen evidencia de infectado y ya no habra que realizar nuevos M. Si el 2 M hubiera
infeccin tuberculosa (Mantoux negativo). resultado positivo, indicara infeccin y habra que realizar una placa
QP secundaria: se realiza en sujetos con evidencia de infeccin de trax para descartar enfermedad tuberculosa.
(Mantoux positivo) pero sin enfermedad. Tambin podra pensarse (menos frecuente) que el primer M que
se realiz a este individuo fue un falso negativo debido a la tcnica
En ambos casos la quimioprofilaxis se realiza en sujetos con espe- errnea de aplicacin o a que el PPD fuera inactivo por exposicin
cial riesgo de enfermedad (habitualmente convivientes estrechos de prolongada a la luz.
un sujeto bacilfero), aunque tambin estara indicada en aquellos Pregunta 21.- R: 1
sujetos con la inmunidad comprometida como son un sujeto que va Esta es una situacin de hacinamiento donde, por tanto, el riesgo
a ser sometido a trasplante o bien un individuo que va a recibir de TBC es elevado. Los nios y adultos jvenes son los que presentan
corticoides (diagnstico de LES). No influye en el estado inmunitario principal riesgo de infeccin y enfermedad, y debe ser en este grupo,
la realizacin de una histerectoma por lo que es en este supuesto en por tanto, en el que ms importante sea la quimioprofilaxis (QP).
el que no estara indicado realizar quimioprofilaxis. Adems, es el grupo donde menos probable es la toxicidad por iso-
El frmaco empleado es la isoniacida. El efecto secundario ms niacida, y por tanto el que se beneficia ms de la QP.
importante de este frmaco es la hepatitis txica que guarda estrecha En el caso de nios y adultos jvenes (habitualmente en los menores
relacin con la edad de los sujetos (a ms edad, mayor riesgo), por lo de 15 aos) hay que realizar un Mantoux (M). Si el M es negativo, no
que la QP se restringe a partir de los 35 aos salvo en sujetos VIH. Hay quiere decir que el nio no est infectado, sino que puede encontrarse
que recordar que el mayor riesgo de infeccin y enfermedad tuber- en el periodo ventana (hay que recordar que la respuesta inmune tarda
culosa lo tienen nios y adultos jvenes, siendo en este grupo en el unas 8 semanas desde el momento de la infeccin). Es tan alto el riesgo
que la hepatoxicidad es menor por isoniacida, por lo que estos suje- de infeccin en los nios y tan bajo el riesgo de hepatotoxicidad que se
tos son los que ms se benefician de la QP. inicia la QP incluso con M negativo (QP 1). Para confirmar definitiva-
mente si el nio est infectado, habr que repetir el M a las 8 semanas
Pregunta 20.- R: 2 (habr terminado el periodo ventana). Si el M es negativo en esta segun-
Si un adulto con edad avanzada tiene un Mantoux (M) que resulta da ocasin, se suspender la QP 1 (nunca estuvo infectado). Si el segun-
negativo, se deber realizar una nueva prueba tuberculnica a la se- do M es positivo, entonces habr que descartar la enfermedad mediante
mana. Esto no indica seroconversin reciente. placa de trax. Si la placa es negativa, entonces el nio est infectado,
pero no enfermo. En este ltimo caso, el nio deber finalizar la QP que
inici y prolongarla hasta los 6 meses (QP 2).
Pregunta 20. Causas de FP y FN en el Mantoux.
Pregunta 22.- R: 4
1234546754898
54 Si el sujeto es mayor de 15 aos, entonces lo primero que habr que
realizar es un Mantoux (como en cualquier contacto). Si ste es negati-
123456745897

58548
854
48647
7
74
8 vo, no tendremos la certeza de no infeccin (periodo ventana), por lo
12
7
4
8745897
4
5
8
4
78884
4

2 que habr que repetir el Mantoux a las 8 semanas. La diferencia con el
22577

 ! grupo de nios es que en los mayores de 15 aos no est indicada la
12"44
#$
87%55897
54! QP 1, por lo que el sujeto no recibe ningn frmaco hasta la realiza-
12&7%5587

854584
4874! cin del 2 Mantoux. Si ste 2 es negativo se suspende el seguimiento
(nunca habr estado en contacto con el bacilo). Sin embargo, si el 2 es

Comentarios TEST
1234546 298
54
positivo lo primero que habr que hacer es una placa de trax para
'45
45874
57

87886( descartar la presencia de enfermedad. Si finalmente nos encontramos
12)6568
44744*
88744
#$
57
4%545897

4! con 2 Mantoux positivo y placa negativa est indicada la QP 2, que se
12
7474

+85897
4
8845897
,6
6


-./ realiza como la QP 1 con isoniacida. La QP 2 se prolonga durante 6
22474! meses salvo en dos situaciones: infectados por VIH y portadores de
123&".0& 1! lesiones fibrticas extensas, en los cuales la QP se prolonga durante 12
12 8744
87%5587
854

458474! meses. La opcin 2 es falsa porque si el Mantoux es positivo en cual-
12345674587
57
86
8
4764! quier situacin, antes de iniciar QP 2 o tratamiento hay que descartar
12)484
876764(
5858
#
! la presencia de enfermedad.
127%4
72854*
4588*
876%8587584
74
597854*
2255897
854
4*
3
58644*
,6464! Pregunta 23.- R: 5
1255837
7458
#
475847! En los mayores de 35 aos se restringen las quimioprofilaxis (QP),
'45
45874
57
4
656874
#
57
4
357854( salvo en sujetos de especial riesgo como los VIH. En este grupo de
121457487
874564! edad, la hepatoxicidad por isoniacida es mayor cuanto ms avanzada
12 86587
8745644! sea la edad del sujeto. El manejo apropiado comienza por realizar un
121747
54654! Mantoux. Si ste resulta negativo, hay que realizar como siempre otro
a las 8 semanas. Si el 2 es negativo se acabar el estudio. Sin embargo,
12&7#55897

5
4747!
si el segundo es positivo se descartar la presencia de la enfermedad
12&7#55897
484
%674! mediante una placa de trax. Si la placa resulta negativa y el Mantoux
1265
87+! positivo, no se iniciar ningn tipo de QP, slo se realizar seguimiento
clnico. Podra realizarse QP 2 en mayores de 35 aos si el riesgo de
Desde que el bacilo tuberculoso entra en contacto con el husped infeccin es muy grande, como ocurre en el caso de convivientes de
y ste desarrolla la respuesta inmune va a pasar un intervalo de aproxi- sujetos bacilferos.
madamente 8 semanas, por lo que hasta que no transcurra este tiempo

M exico A rgentina CTO Medicina C/ Nez de Balboa, 115 28006 MADRID (Espaa) Tfno.: (91) 782 43 32 / Fax: (91) 782 43 27
Pg. 4 ED C hile U ruguay E-mail: secretaria@ctomedicina.com; iberocto@ctomedicina.com WEB: www.ctomedicina.com; www.iberocto.com
MEDICINA PREVENTIVA
Preparacin Examen de Seleccin 05/06 1 Vuelta Seguimiento a distancia
En ancianos, alcohlicos y hepatpatas el riesgo de hepatitis txica que las penicilinas, que forman parte del arsenal teraputico, pue-
por isoniacida es muy alto. En estos grupos se recomienda la realiza- den suprimir de forma temporal la Neisseria de la rinofaringe, pero
cin seriada de analticas. Debera interrumpirse la QP si se no la erradican. El perodo de incubacin de la enfermedad es de 3
quintuplican los valores basales de transaminasas o aparece clnica de a 4 das.
hepatitis. Si no se pertenece a ninguno de los grupos antes sealados,
se recomienda nicamente una analtica al mes del inicio de la QP y Pregunta 27.- R: 5
un seguimiento clnico. La enfermedad meningoccica est producida por N. meningitidis,
que contiene una cpsula de polisacridos que permite la divisin en
Pregunta 24.- R: 3 serogrupos (A, B y C los ms importantes). La enfermedad meningoc-
En los infectados mayores de 65 aos y en los vacunados con BCG, cica tiene distribucin universal, presentndose habitualmente de
la respuesta inmunitaria frente al bacilo tuberculoso puede estar ate- modo individual con aumento de las tasas de incidencia cada 10
nuada, de forma que un primer Mantoux puede resultar negativo, aos aproximadamente. Los serogrupos A, B y C originan un 90% del
aunque el sujeto hubiera estado expuesto a la infeccin. Para evitar total de casos. El serogrupo B es el ms frecuente en Espaa y suele
este falso negativo de la prueba debe hacerse un nuevo Mantoux a la originar casos espordicos. El C se manifiesta habitualmente con pe-
semana del primero. En este caso, si el Mantoux a la semana es posi- queos brotes y est creciendo su prevalencia en Espaa en los lti-
tivo, indicar que el sujeto ha estado en contacto con el bacilo. A esta mos aos (por esto se ha introducido recientemente en el calendario
reactivacin de la inmunidad frente a la TBC se le llama efecto Boos- vacunal). El serogrupo A habitualmente produce epidemias y es el
ter. Si el 2 Mantoux es tambin negativo entonces hay que descartar ms frecuente en el llamado cinturn de la meningitis.
que no se est en periodo ventana por lo que se realizar un nuevo En Espaa se producen habitualmente unos 3 casos/100000 habs.
Mantoux a las 8 semanas del primero. Si este ltimo Mantoux resulta y ao, con una letalidad de un 6-10%, algo mayor en las que se
negativo, se habr finalizado el estudio. Si fuera positivo, habra que presentan en forma de sepsis fulminante y cuando aparecen en me-
descartar enfermedad tuberculosa mediante placa de trax. Como nores de 14 aos. Sin embargo, el ltimo brote epidmico ocurrido
vemos, el nico grupo en el que es necesario realizar hasta 3 Mantoux en nuestro pas ha sido debido al serogrupo C, alcanzando inciden-
para concluir el estudio es en el de los ancianos y vacunados con cias de 6 casos/100000 habs/ao. con una tasa de letalidad especfica
BCG. para este serogrupo mayor que para el serogrupo B.
El efecto Booster no es la nica causa de falso negativo de la prue-
ba. Tambin pueden verse falsos negativos en recin nacidos, VIH, Pregunta 28.- R: 2
vacunados con triple vrica, enfermedades graves como el cncer, la Si tenemos en cuenta que el serogrupo ms prevalente de N.
sarcoidosis o la deplecin proteica, tuberculosis diseminadas como la meningitidis en nuestro medio es el B, para el cual no existe vacuna ni
TBC miliar o la que afecta a serosas. de polisacridos ni conjugada, entenderemos que en Espaa es esen-
cial la quimioprofilaxis para evitar los contagios. Salvo que se demues-
Pregunta 25.- R: 4 tre que el brote que aparece en nuestro medio no est producido por
El agente etiolgico ms frecuente de meningitis en nuestro medio es el serogrupo B, se debe administrar a los contactos estrechos
el meningococo. Es la causa ms frecuente en los dos grupos de edad quimioprofilaxis. El frmaco de eleccin es la rifampicina, que se
en los que habitualmente se producen las meningitis: nios pequeos administra durante 2 das. Alternativas son la ceftriaxona y las quino-
y nios y adultos jvenes. Sin embargo, rara vez es la causa de las lonas, administradas en dosis nica. Cuando se sabe que el caso no
meningitis del recin nacido (aparecidas en el primer mes de vida: est producido por el serogrupo B, adems debe administrarse la
Listeria, Streptococcus agalactie y E. coli). En los nios hasta los 4 aos de vacuna, ya que sta es ms eficaz que la quimioprofilaxis.
edad, despus del meningococo el segundo agente etiolgico es el H. En la embarazada debe evitarse la rifampicina, ya que hay alterna-
influenzae, que se est haciendo cada vez ms prevalente en nuestro tivas eficaces. No debe tampoco administrarse quinolonas (recuerda
medio. H influenzae es la causa ms frecuente de meningitis en ese que tampoco se deben dar a nios, porque en ambos casos deforman
grupo de edad en EEUU. En los nios mayores y en adultos jvenes, la el cartlago de crecimiento). En la embarazada, por tanto, se indica la
causa ms frecuente en nuestro medio sigue siendo el meningococo, administracin de ceftriaxona i.m. en dosis nica.
pero el Haemophilus ya no ocupa el segundo lugar, sino que pasa a ser Es importante recordar que la quimioprofilaxis debe realizarse en
el neumococo. En mayores de 30 aos, el meningococo pasa a un los contactos estrechos, sean stos nios o adultos (a diferencia de la
segundo lugar, quedando como agente etiolgico ms frecuente en ese quimioprofilaxis contra H. influenzae, que se realiza slo en nios).
Comentarios TEST

grupo de edad el neumococo. Algunos agentes etiolgicos son frecuen-


tes en determinados grupos de riesgo. Por ejemplo, la Listeria vuelve a Pregunta 29.- R: 2
ser una causa frecuente en ancianos y la Klebsiella es habitual causa de Existen dos tipos de vacunas distintas frente al meningococo:
meningitis en alcohlicos (recuerda que la Klebsiella es tambin la cau- Vacuna de polisacridos: contiene antgenos de varios serogrupos,
sa ms frecuente de neumona en alcohlicos). entre los que estn el A y el C. No contiene, sin embargo, el serogru-
po B, que es el ms frecuente en nuestro medio. Una desventaja
Pregunta 26.- R: 1 importante es que la capacidad de desarrollar respuesta inmune
La transmisin de la meningitis meningoccica se realiza a travs en menores de 2 aos es nula y muy baja entre 2 y 4 aos (40%). En
de las secreciones respiratorias ms frecuentemente de un portador adultos jvenes, sin embargo, el nivel protector se alcanza hasta en
asintomtico y ms rara vez de un enfermo. Los portadores suelen un 90%.
ser resistentes a la infeccin porque en la infancia tuvieron en la Vacuna conjugada: la ventaja frente a la anterior es que es muy
rinofaringe N. lactamica, que no es patgena e induce inmunidad inmungena incluso en menores de un ao, por eso es la vacuna
cruzada con otras neiserias mediada por el complemento. La que se ha introducido en el calendario vacunal en la pauta 2-4-6
nasofaringe humana es el nico reservorio conocido, y dado que el meses y 1 dosis nica en mayores de 1 ao no vacunados previa-
meningococo difcilmente sobrevive en el medio ambiente, para mente. El inconveniente frente a la anterior es que slo protege
que se produzca el contagio es necesario un contacto estrecho entre frente al serogrupo C. ste es un inconveniente relativo, ya que el
el reservorio y el husped susceptible. Los contactos estrechos de los serogrupo C es el 2 ms frecuente en nuestro medio y la vacuna
sujetos portadores tienen un riesgo hasta 800 veces mayor de desa- de polisacridos tampoco es eficaz frente al serogrupo B.
rrollar la enfermedad (recuerda que el riesgo es an mucho mayor
para los sujetos que presentan dficit del factor final del comple- Recuerda las indicaciones de vacunacin:
mento). El perodo de contagio es mximo en los 10 das posteriores Dficit de fraccin final del complemento.
al contacto con el portador. El sujeto portador o el individuo enfer- Asplenia.
mo dejan de ser contagiosos a las 24 horas de iniciar la Inmunodeficiencias humorales.
quimioprofilaxis o el tratamiento adecuado, teniendo en cuenta Contactos de casos debidos al serogrupo C.

M exico A rgentina CTO Medicina C/ Nez de Balboa, 115 28006 MADRID (Espaa) Tfno.: (91) 782 43 32 / Fax: (91) 782 43 27
C hile U ruguay E-mail: secretaria@ctomedicina.com; iberocto@ctomedicina.com WEB: www.ctomedicina.com; www.iberocto.com
ED Pg. 5
MEDICINA PREVENTIVA
Seguimiento a distancia Preparacin Examen de Seleccin 05/06 1 Vuelta
Pregunta 30.- R: 4
El ttanos est producido por C. tetani, que es un bacilo anaerobio
estricto Gram positivo, esporulado. Esta bacteria es ubicua en forma
de espora, resistiendo condiciones tan adversas como la ebullicin.
En contacto con un medio red-ox bajo (heridas sucias con
coinfecciones por otros grmenes), el bacilo toma su forma activa y es
capaz de liberar la toxina tetnica. Esta toxina migra por los axones
motores hasta llegar al asta lateral de la mdula, donde inhibe a la
neurona encargada de inhibir el tono motor (quitara el freno a las
neuronas motoras, de ah la clnica de espasticidad, rigidez y opistto-
nos). Como en Espaa la vacunacin frente al ttanos est incluida en
el calendario vacunal, la prevalencia de anticuerpos protectores es
superior al 99% en los menores de 14 aos. Sin embargo, en los pases
subdesarrollados donde la vacunacin no es universal, es frecuente el
ttanos neonatal, forma especialmente grave.
En Espaa se producen unos 0,1-0,2 casos/100000 habs/ao (en
descenso en los ltimos aos). Habitualmente ocurren en mayores de
60 aos porque se abandona la vacunacin recomendada cada 10 Pregunta 32. Profilaxis del ttanos.
aos. En nuestro medio, lo ms frecuente es que no se recuerde el
antecedente de herida tetangena. Pregunta 33.- R: 3
La gripe es una enfermedad producida por un virus ARN monoca-
Pregunta 31.- R: 2 tenario muy prevalente en la poblacin general, afectando a un 10%
Aunque la eficacia de la vacuna frente al ttanos es casi del 100%, los de sta. Cursa en brotes anuales que se producen en los meses fros y
niveles de anticuerpos suelen disminuir con el paso del tiempo, por lo la transmisin es casi exclusivamente a partir del sujeto enfermo. Para
que se recomienda la vacunacin cada 10 aos a partir de la ltima que se produzca el contagio, el contacto debe ser muy estrecho, ya
dosis del calendario vacunal (14 aos). La vacunacin correcta consiste que el virus de la gripe es muy lbil en el medio ambiente. Un papel
en la administracin de 3 dosis de toxoide adsorvido en sal de aluminio muy importante en la morbi-mortalidad de los sujetos adultos es el
con dosis de toxoide diftrico Td. Si la vacunacin es incompleta, no es contagio a partir de los nios. En los meses de mayor contagiosidad
necesario reanudar la vacunacin en todos los casos: (meses fros), la gripe puede afectar incluso a un 40% de los nios. El
Si se recibieron 3 dosis (vacunacin completa), slo se recomienda perodo de incubacin de la enfermedad es de 2 das de media,
una nica dosis de recuerdo si las 3 dosis se recibieron hace ms empezando la clnica de forma abrupta sin prdromos.
de 10 aos. Las glicoprotenas superficiales N (neuraminidasa) y H (hemagluti-
Si se recibieron menos de 3 dosis hace menos de 5 aos slo hay nina) estn en la envoltura lipdica y varan en la gripe tipo A, condi-
que completar la vacunacin hasta alcanzar las 3 dosis. cionando la inmunidad de la poblacin, de forma que la vacuna
Si se recibieron menos de 3 dosis hace ms de 5 aos debe admi- frente a la gripe se hace teniendo en cuenta las cepas del ao anterior.
nistrarse toxoide hasta un total de 4 dosis. Esta variabilidad de las glicoprotenas no ocurre en la gripe tipo B, por
lo que la importancia desde el punto de vista epidemiolgico es me-
Es importante recordar que, para evitar el ttanos neonatal, la medida nor (poca capacidad de producir infecciones). No obstante debe
ms eficaz es vacunar a la mujer embarazada que no estuviera previa- recordarse que el mayor riesgo de la gripe B es la capacidad de produ-
mente inmunizada. En este caso se considera suficiente con dos dosis de cir sndrome de Reye en nios que toman salicilatos.
toxoide, debiendo administrarse la segunda dosis antes del parto.
Pregunta 34.- R: 5
Pregunta 32.- R: 4 La vacuna frente a la gripe que se emplea de manera habitual es una
Para contestar correctamente este tipo de preguntas, hay que valo- vacuna trivalente inactiva que contiene dos subtipos de vacuna A que
rar dos aspectos: circularon la temporada anterior (H1N1 y H3N2) y frente al grupo B.
Cunto hace que se vacun? Cuando las cepas del ao en curso coinciden con las de la vacuna,
De qu tipo de herida se trata? la eficacia para sujetos menores de 65 aos inmunocompetentes es

Comentarios TEST
de un 70-80%. Esta eficacia es sensiblemente inferior en ancianos,
Se considera que una herida es tetangena cuando sta es sucia trasplantados, hemodilisis, y en todos aquellos en los que la respues-
(heridas anfractuosas, contaminadas con polvo o tierra, por morde- ta inmune est alterada. Los ttulos de anticuerpos protectores apare-
dura de animales...). Si la herida es sucia y el sujeto no est correcta- cen a los 10-14 das y protegen hasta 6 meses tras la vacunacin. El
mente inmunizado o hace ms de 10 aos que recibi las 3 dosis, momento de la vacunacin es en los meses de otoo en nuestro
deber administrarse, adems del toxoide, gammaglobulina especfi- medio. Con vacunas que contienen la hemaglutinina porcina se han
ca antitetnica (IGT). Por otra parte, si el sujeto recibi las 3 dosis hace descrito casos de Guillain-Barr.
menos de 5 aos, no deber administrarse ni vacuna ni IGT. Recuerda las indicaciones:
La situacin particular de esta pregunta es un sujeto que ha pasado Mayores de 65 aos.
la infeccin. Dado que la cantidad de toxoide que produce la enfer- Portadores de enfermedades crnicas (no se incluyen hipertensos).
medad es muy pequea y que la toxina no entra a la circulacin Nios mayores de 6 meses que toman salicilatos de forma prolon-
(migra por los axones de neurona motora), el haber pasado la enfer- gada.
medad no asegura anticuerpos, por lo que este sujeto, teniendo en Mujeres que se encuentran en el tercer trimestre en la poca gripal.
cuenta que no est vacunado, deber recibir, adems de la pauta Y tambin las contraindicaciones:
vacunal (3 dosis de toxoide), IGT especfica. Nios menores de 6 meses.
Recuerda este esquema muy general: Anafilaxia a las protenas del huevo.
Sujeto inmunizado (3 dosis hace menos de 5 aos): nada, inde-
pendientemente de la herida. Pregunta 35.- R: 1
Sujeto no inmunizado (resto de situaciones): asegurar inmunidad Las vacunas pueden clasificarse segn el tipo de antgeno en vivas
mediante toxoide en heridas limpias y, adems, en heridas sucias, (atenuadas) y muertas (inactivas). Las vacunas vivas estn formadas por
asegurar inmunidad inmediata mediante IGT. microorganismos viables, pero que presentan prcticamente nula ca-
pacidad patgena en sujetos inmunocompetentes. Como es una for-
ma poco virulenta de un microorganismo vivo, la capacidad de gene-

M exico A rgentina CTO Medicina C/ Nez de Balboa, 115 28006 MADRID (Espaa) Tfno.: (91) 782 43 32 / Fax: (91) 782 43 27
Pg. 6 ED C hile U ruguay E-mail: secretaria@ctomedicina.com; iberocto@ctomedicina.com WEB: www.ctomedicina.com; www.iberocto.com
MEDICINA PREVENTIVA
Preparacin Examen de Seleccin 05/06 1 Vuelta Seguimiento a distancia
rar respuesta inmune es similar o incluso mayor a la de la infeccin Actualmente se dispone de una vacuna frente a VHA (virus muer-
natural. El nivel de inmunidad por tanto que se consigue con las tos) que es muy eficaz y se aplica si se va a estar en riesgo de infeccin
vacunas vivas atenuadas es muy bueno desde la primera dosis (nica (por ejemplo, un viaje a un pas endmico). Si el plazo de tiempo en
excepcin es la polio oral o tipo Sabin). Como son microorganismos el que se va a producir este riesgo es inferior a 2 semanas o ya se ha
vivos, se pueden administrar va enteral, como la vacuna frente a la producido el contacto, la inmunoglobulina inespecfica frente a VHA
polio Sabin, condicionando respuesta inmune tambin local (IgA es tambin muy eficaz.
secretora). Precisamente en esta vacuna hay que vigilar que no existan Como la vacuna del sarampin est incluida en el calendario
sujetos convivientes inmunocomprometidos, ya que pueden elimi- vacunal, la necesidad de administracin de inmunoglobulina poliva-
narse virus con las heces que causen enfermedad en estos sujetos. lente es escasa. No obstante est indicada la profilaxis pasiva ante un
Sin embargo, las vacunas de microorganismos inactivos suelen pre- contacto (en un plazo inferior a 6 das) si:
cisar de varias dosis que se administran de forma percutnea. Como Nio menor de 1 ao y contacto estrecho.
no son microorganismos viables, la enfermedad no puede transmitirse Inmunodeprimidos no vacunados.
a sujetos convivientes inmunocomprometidos. Las vacunas muertas VIH, independientemente del estado vacunal.
pueden ser de microorganismos enteros como la antipertussis de c- Embarazadas no vacunadas.
lulas enteras, la polio Salk, la antigripal o la de VHA, o fracciones
como antihepatitis B, de toxoide como antidifteria o antitetnica, de Tras haber recibido la profilaxis pasiva hay que vacunar, transcurri-
polisacridos como la del meningococo, neumococo o Haemophilus das 6 semanas desde la aplicacin de la gammaglobulina. Si no han
o acelulares como la antipertussis acelular. pasado 3 das desde el contacto, puede realizarse vacunacin como
nica profilaxis postexposicin.
Pregunta 36.- R: 4 La profilaxis frente a la VVZ se realiza con gammaglobulina hiperin-
Esta pregunta hace referencia a las reacciones adversas producidas mune. Si recordamos que la vacuna no entra dentro del calendario
por las vacunas. Habitualmente se produce una reaccin inespecfica vacunal y que est formada por virus vivos, entenderemos que es
en el lugar de inoculacin en las primeras 48 horas de la vacunacin, relativamente frecuente la administracin de profilaxis pasiva frente al
y que consiste en edema, eritema, dolor o induracin. Algunas formas VVZ. Sus indicaciones son:
de reaccin local son ms especficas: Sujetos inmunodeprimidos con contacto estrecho.
Td (vacunas adsorbidas en aluminio): ndulo cutneo que puede RN con varicela materna en el periparto.
persistir durante semanas. Prematuros <28 semanas, independientemente de si la madre tuvo
Varicela: vesculas localizadas. varicela.
BCG: adenopatas locorregionales o adenitis supurada. Prematuros >28 semanas, si la madre no tuvo varicela.
Rubola: Exantema generalizado polimorfo que aparece a las dos
semanas de la vacunacin, en el 10% de los sujetos. Pregunta 39.- R: 3
Rubola: Artralgias en el 10-20% de las mujeres jvenes vacuna- La vacuna conjugada DTP est introducida en el calendario vacu-
das, 2-3 semanas tras la vacunacin. nal. Como est formada a partir de microorganismos vivos, puede
Rubola o Sarampin: 20% de los vacunados, adenopatas gene- administrarse incluso en sujetos inmunocomprometidos.
ralizadas 2-3 semanas tras la vacunacin. En el calendario vacunal infantil se administran 3 dosis en el primer
Antitfica-paratfica: afectacin severa del estado general. ao de vida con intervalos de 8 semanas entre cada dosis. Es importan-
Sarampin y parotiditis: encefalitis. te recordar que la vacuna Pertussis puede ser celular (DTP) o acelular
Triple vrica: Convulsiones no febriles, 15 das tras la administra- (DTPa). La forma celular est contraindicada en mayores de 1 ao de
cin de la vacuna. edad. Es necesaria una cuarta dosis de esta vacuna que se suele admi-
DTP: hipotona y disminucin del estado de alerta. nistrar a los 18 meses (DTPa) y otra ms a los 4-6 aos (DTPa /DT). El
Ttanos: polineuropata 14 das siguientes a la vacunacin. calendario vacunal se completa con una 6 dosis a los 14 aos (Td)
Pertussis celular: Sndrome de llanto persistente horas tras la vacu- con una menor dosis de toxoide diftrico. Recientemente se ha intro-
nacin. ducido tambin el componente Hib frente a H. influenzae, que se
administra en 4 dosis junto con la vacuna DTP. Si recordamos que H.
Pregunta 37.- R: 1 influenzae suele afectar exclusivamente a nios pequeos, entendere-
La inmunidad pasiva consiste en la administracin de anticuerpos mos que no es necesaria ninguna dosis por encima de los 18 meses.
Comentarios TEST

preformados. La nica ventaja sobre la inmunizacin activa (vacunas) Tambin es nueva la incorporacin de la vacuna C conjugada frente a
es la inmediatez de los anticuerpos. La desventaja fundamental es que meningococo C. Se administra en el calendario vacunal a los 2-4-6
la inmunidad es menos duradera y menos fuerte. meses de edad. Si el nio no fue vacunado en el primer ao de vida, se
Los sueros se dividen en homlogos o heterlogos. Los primeros administra una dosis nica en nios mayores.
son los ms frecuentes y son los que se obtienen de individuos huma- Como ocurre con la mayora de las vacunas, si se ha interrumpido
nos. Los heterlogos sin embargo se obtienen del suero de animales. el calendario vacunal no hace falta reiniciarlo, sino completarlo.
De estos ltimos slo se emplea el suero antibotulismo. Los sueros
heterlogos pueden producir enfermedad del suero (reaccin de Pregunta 40.- R: 4
depsito de complejos antgeno-anticuerpo). La vacuna TV se administra en 2 dosis en el calendario vacunal. En
Otra clasificacin de los sueros es la que los divide en especficos o menores de 1 ao no est contraindicada, si bien no se aconseja su
hiperinmunes e inespecficos. Dentro de los primeros estn el de la administracin porque la vacuna podra inactivarse por la presencia
hepatitis B, el de varicela, el de ttanos o el de la difteria. Sin embargo, de anticuerpos maternos, siendo este el motivo por el cual se aconseja
el suero frente a VHA es inespecfico. la primera dosis a los 15 meses. Esta primera dosis podra adelantarse
La inmunoglobulina frente al VHA se emplea en la profilaxis pre- si fuese necesario por motivos epidemiolgicos (recuerda que en
exposicin. Est indicada cuando se necesita inmunidad de forma menores de 1 ao con contacto muy estrecho se podra administrar
inmediata (menos de 2 semanas), ya que de otra forma debera apli- inmunoglobulina). Si se adelanta la primera dosis, como posiblemen-
carse la vacuna frente al VHA, que es ms eficaz a largo plazo. Tam- te se inactive por los anticuerpos maternos, debe administrarse siem-
bin se emplea en la profilaxis post-exposicin. En este caso, la efica- pre en cualquier caso la dosis de los 15 meses. La segunda dosis se
cia es alta (90%), previniendo la enfermedad o atenundola. administra a los 4 aos para asegurar la eficacia de la vacunacin. En
aquellos nios que no han recibido esta segunda dosis, se les adminis-
Pregunta 38.- R: 5 tra a los 11 aos.
La inmunoglobulina hiperinmune frente al VHB se aplica junto Como esta vacuna est compuesta por virus vivos est contraindicada
con la vacunacin (administracin simultnea, aunque en distinta en situaciones de inmunosupresin como el embarazo (en este caso se
localizacin) en la profilaxis postexposicin. podra administrar inmunoglobulina polivalente frente al ttanos).

M exico A rgentina CTO Medicina C/ Nez de Balboa, 115 28006 MADRID (Espaa) Tfno.: (91) 782 43 32 / Fax: (91) 782 43 27
C hile U ruguay E-mail: secretaria@ctomedicina.com; iberocto@ctomedicina.com WEB: www.ctomedicina.com; www.iberocto.com
ED Pg. 7
MEDICINA PREVENTIVA
Seguimiento a distancia Preparacin Examen de Seleccin 05/06 1 Vuelta
La vacuna TV no debe administrarse en alrgicos a las protenas del frente a VHB (fraccin antignica) o antineumoccica, antihaemophilus
huevo, si bien para estos sujetos existe la posibilidad de vacunacin o antimeningococo (polisacridos o conjugadas).
con la presentacin cultivada en clulas diploides humanas.
Un hecho particular de la TV es la hipoergia tuberculnica (res- Pregunta 43.- R: 5
puesta disminuida al Mantoux), que puede dar lugar a un falso nega- El cncer es una de las causas ms importante de mortalidad en los
tivo de la prueba tuberculnica, por lo que si se ha vacunado frente a pases desarrollados. La mitad de los tumores aparece en los pases
la TV, debe posponerse la realizacin del Mantoux varias semanas. desarrollados y la otra mitad en los pases subdesarrollados. Sin em-
bargo, los distintos tipos de tumor que afectan en una y otra localiza-
Pregunta 41.- R: 3 cin son distintos. En los pases desarrollados son frecuentes los tumo-
Si bien el calendario vacunal infantil se cumple en gran medida en res de pulmn, colon y mama, mientras que en los subdesarrollados,
nuestro medio, no ocurre lo mismo con el calendario vacunal del son ms frecuentes los de estmago, esfago y crvix. En algunos tipos
adulto. Las vacunas recomendadas en los adultos son: Td, TV, gripe y concretos de tumor est observndose un cambio significativo, de
neumoccica. La vacuna frente al ttanos debe continuarse desde la forma que en los pases desarrollados ha disminuido claramente la
ltima dosis del calendario vacunal (14 aos), cada 10 aos, de forma incidencia de cnceres de esfago y estmago, probablemente por la
indefinida. Precisamente en nuestro medio la forma ms frecuente de relacin de estos tumores con el consumo de productos ahumados.
ttanos es el ttanos en mayores de 65 aos porque en este grupo de Otro cambio que se est observando es que el cncer de mama, que
edad los pacientes no siguen ningn tipo de pauta de vacunacin. En es el tumor ms frecuente en la mujer en los pases desarrollados, est
la mayora de los casos de ttanos en nuestro medio no est clara cul siendo sobrepasado en las mujeres por el cncer de pulmn, lo que
fue la puerta de entrada para la toxina. La vacuna contiene una pe- tiene que ver en gran medida con el hbito tabquico en las mujeres.
quea dosis de toxoide diftrico. El cncer de mama no obstante es en nuestro medio el tipo de neo-
Desde los 15 a los 65 aos se recomienda la vacunacin contra TV plasia ms frecuente en las mujeres, suponiendo la principal causa de
en aquellos sujetos no inmunizados previamente. Esto es de especial aos de vida perdidos en este sexo.
importancia en las mujeres en edad frtil no vacunadas con la TV, por
el riesgo de rubola fetal. La vacunacin frente a la rubola est Pregunta 44.- R: 5
contraindicada si la mujer est embarazada. El principal carcingeno ambiental es el humo del tabaco, que
En mayores de 65 aos se recomienda la vacunacin anual frente constituye la principal causa de muerte en nuestro medio al ser un
a la gripe que protege a un 70% aproximadamente de los vacunados factor etiolgico clave tanto en las enfermedades cardiovasculares
y deben recibir tambin la vacunacin frente al neumococo, con una como en los tumores ms frecuentes. El tabaco se ha relacionado con
eficacia muy similar a la de la gripe. uno de cada tres tumores que se producen en el varn y en un 10%
de los que se producen en mujeres. Se han relacionado muchos tipos
Pregunta 42.- R: 3 de tumor con el tabaco. Aparte de todos los de la esfera ORL y el de
El calendario vacunal en el nio VIH es en todo idntico al del pulmn, tambin ha sido implicado en la gnesis de tumores de est-
nio sano, excepto porque se recomienda el empleo de la vacuna mago y esfago, pncreas, pulmn, vejiga y crvix. Los cnceres de la
parenteral tipo Salk en los nios VIH y en todos los sujetos que con l cavidad oral tambin se asocian muy frecuentemente al tabaquismo.
conviven, en vez de la tipo Sabin. La vacuna TV, aunque de virus vivos, Si bien se considera que la forma menos perniciosa de tabaco es el
s se aconseja en estos nios VIH porque el riego de contraer el saram- fumado en pipa, el cncer de labio se ha relacionado especialmente
pin y la mortalidad que presenta en estos pacientes es muy elevada. con el tabaco en pipa. Cuanto mayor es el consumo de tabaco, ma-
Tambin se recomienda la vacunacin en estos nios frente a VVZ, yor es el riesgo de cncer. Este riesgo existe tambin para los fumado-
por el riesgo de desarrollar zster diseminado. Estas dos ltimas vacu- res pasivos. Se ha estudiado especialmente el riesgo de cncer de
nas, por ser de virus vivos, no deben administrase si: pulmn y los fumadores pasivos, encontrndose un 25% de riesgo
VVZ: no administrarse si CD4<25%. superior en este tipo de fumadores que en los controles no expuestos.
TV: si CD4< 15%. Hay que recordar que nunca es tarde para dejar de fumar. Si bien el
riesgo de cncer de pulmn no se iguala nunca al del no fumador,
Debe recordarse tambin que si un sujeto VIH est expuesto a una este riesgo s se reduce a lo largo del tiempo, siendo muy parecido al
enfermedad para la que exista una forma de profilaxis pasiva eficaz, del no fumador a los 10 - 15 aos de abandonar el consumo de
sta debe realizarse independientemente del estado vacunal. Esto es tabaco.
aplicable especialmente para el ttanos, el sarampin y la varicela.

Comentarios TEST
No existe ningn problema por administrar a estos nios la vacuna Pregunta 45.- R: 3
DTP (por ser de microorganismos inactivos), al igual que la vacuna Para que se produzca cncer de prstata es absolutamente necesa-

Pregunta 42. Recomendaciones para la vacunacin sistemtica en los pacientes infectados por VIH.

123456789
8675 7947  88  767
48 4 2 363 86  363 86
122324 56 56 5789
7  74
 
9 47
9

7
   97 47
78 47  
7 
 47
9 47  9 ! 47 
7

   "

# 56 56 $ 797  
97  9747 7  7 4 7 
77 
%

2
 7 "
 56 56 5789
7  74
 
9 47
9

7
&
 '984 56 56 5789
7  74
 
9 47
9

7
&( 56 56 9747  7  7   
 47 
97 
79 56 56 177 4 7  4  
) 47 *  % ) 47 744
+
7 56 56 9747 7  7 4   
 47 
97  7  
77 
%


7     7  794
4 7 9 47  9 7 7  
%


M exico A rgentina CTO Medicina C/ Nez de Balboa, 115 28006 MADRID (Espaa) Tfno.: (91) 782 43 32 / Fax: (91) 782 43 27
Pg. 8 ED C hile U ruguay E-mail: secretaria@ctomedicina.com; iberocto@ctomedicina.com WEB: www.ctomedicina.com; www.iberocto.com
MEDICINA PREVENTIVA
Preparacin Examen de Seleccin 05/06 1 Vuelta Seguimiento a distancia
ria la presencia de andrgenos. Basta recordar que una forma de producen en menores de 65 aos. El segundo marcador de riesgo
tratamiento de este tumor es la terapia antiandrognica. ms importante es el sexo masculino.
Tambin est asociado al cncer de mama la presencia de estrge- La hiperlipidemia es un factor de riesgo muy importante. El princi-
nos, por lo que slo uno de cada cien tumores aparece en el varn. El pal predictor de riesgo de cardiopata isqumica en los varones de
embarazo que se produce a edades tempranas (menores de 30 aos), mediana edad es la hipercolesterolemia. Con la edad se observa un
se ha relacionado con menor riesgo de cncer de mama. No se ha aumento de los niveles plasmticos de colesterol y triglicridos que
encontrado relacin entre el tipo de lactancia y este tumor. parece mediado por la obesidad. Si bien se evidencia un aumento de
La terapia sustitutiva empleada en la menopausia a base de estrge- la cardiopata isqumica prematura con unos niveles de colesterol
nos se ha relacionado de forma clara con la aparicin de cncer de total superiores a 200 mg/dl, este riesgo es especialmente elevado
endometrio, por lo que esta forma de teraputica no debe realizarse si cuando se superan los 240 mg/dl. La forma de colesterol que se ha
no se asocian progestgenos. relacionado con el desarrollo de cardiopata isqumica es la LDL-c y
Para el cncer de ovario se puede decir que favorecen este cncer la VLDL-c. Sin embargo, diversos estudios han relacionado con la
todos aquellos factores que se relacionan con la ovulacin. Tendran cardiopata isqumica, ms que los niveles elevados de estas dos for-
por tanto efecto protector tanto el embarazo como el empleo de mas de colesterol, los niveles disminuidos de HDL-c. Los factores que
anticonceptivos orales, mientras que la nuliparidad, la menarquia producen elevacin del HDL-c son el ejercicio fsico, el sexo femeni-
precoz y la menopausia tarda favoreceran la aparicin de este tipo no y la dieta hipolipemiante, mientras que se relacionan con la dismi-
de tumor. Hay que recordar otra asociacin muy tpica y es el consu- nucin, la obesidad, el sedentarismo y el consumo de tabaco. Se
mo por parte de la madre durante la gestacin de dietilestilbestrol es recomienda que el aporte de caloras de la dieta que procede de los
causa de carcinoma de clulas claras vaginal en las hijas. lpidos no supere el 30% del total de caloras.

Pregunta 46.- R: 1 Pregunta 49.- R: 4


Aunque se ha tratado de buscar alguna forma de deteccin precoz El principal marcador de riesgo modificable para las enfermedades
para el cncer de pulmn, no puede decirse que ninguna prueba por cardiovasculares es el tabaco. Este factor de riesgo es causa por s solo
s sola haya conseguido disminuir la incidencia de este cncer. Ni de ms del 20% de las muertes por cardiopata isqumica. El riesgo de
siquiera la realizacin de placa de trax o la citologa de esputo, aun cardiopata isqumica puede llegar a ser tres o cuatro veces mayor en
en pacientes fumadores mayores de 50 aos, han conseguido este los fumadores que en los no fumadores, existiendo una clara relacin
beneficio. Si bien el riesgo de cncer nunca alcanzar el del no fuma- dosis-respuesta. Si el abandono del tabaco se asocia a disminucin
dor, este riesgo s se disminuye sensiblemente a partir de los 10 aos de del riesgo de cncer de pulmn a los 10-15 aos de dejar el tabaco,
la supresin del tabaco. el beneficio es mucho ms rpido para la cardiopata isqumica, ya
El antgeno prosttico especfico (PSA) es el marcador ms sensible que este riesgo aumentado desaparece a los 2-3 aos de dejar el
que hay en el cncer de prstata, pero tiene el inconveniente de que tabaco. Hay que recordar adems que el tabaco se asocia a un des-
es muy poco especfico, ya que tambin se encuentra muy elevado censo de las cifras de HDL-c. No se asocia, sin embargo, el tabaquismo
en patologas benignas como la hiperplasia prosttica benigna o en las con un aumento en las cifras de tensin arterial. El sexo femenino
prostatitis. Otro marcador que se ha utilizado en el screening de este parece proteger de la accin deletrea del tabaquismo, si bien la
tumor ha sido la fosfatasa cida, si bien es poco sensible puesto que asociacin de tabaquismo en mujeres mayores de 35 aos con la
cuando est elevado, suele indicar ya afectacin extraganglionar. Aun- ingesta de anticonceptivos se asocia a mayor riesgo de cardiopata
que el tacto rectal tampoco es eficaz como prueba de screening, se isqumica. Tambin el tabaco es un factor de riesgo para la aparicin
recomienda su realizacin fundamentalmente en varones, al ser una de enfermedades cerebrovasculares y para el desarrollo de arteriopa-
maniobra sencilla y poco costosa. ta perifrica bien oclusiva, bien aneurismtica.
Si bien la citologa de Papanicolau s es eficaz como screening del
cncer de crvix, no lo es para el adenocarcinoma de endometrio. Pregunta 50.- R: 4
La prevencin de las enfermedades cardiovasculares se realiza
Pregunta 47.- R: 5 mediante el control de los factores de riesgo. El primer paso en el
Las enfermedades cardiovasculares (CV) son la principal causa de tratamiento de las hiperlipidemias es la dieta. Es recomendable una
mortalidad tanto en los pases desarrollados como en los subdesarro- dieta que contenga menos de un 30% de caloras provenientes de las
llados, por encima de los 45 aos de edad. En Espaa, el 45% de las grasas. Tambin es importante el tipo de grasa consumida, ya que las
Comentarios TEST

muertes se producen por enfermedades CV, manifestndose como grasas saturadas producen un aumento de la LDL-c, mientras que lo
cardiopata isqumica, accidente cerebrovascular y arteriopata peri- contrario ocurre con las grasas monoinsaturadas. Asimismo, el consu-
frica. En los ltimos aos se est evidenciando una disminucin de la mo moderado de alcohol tambin se ha relacionado con disminu-
mortalidad por esta causa tras el aumento importante que se observ cin del riesgo de mortalidad por cardiopata isqumica. Es importan-
en los aos 70, aunque la repercusin de esta patologa sigue siendo te conseguir el control del peso, ya que la obesidad (fundamental-
dramtica, ya que hasta un tercio de las muertes que se producen mente la central o andrgina) se relaciona con hipercolesterolemia,
ocurren en menores de 65 aos (muertes prematuras). hipertrigliceridemia y con diabetes y resistencia a la insulina, mientras
Los factores ms importantes en el desarrollo de las enfermedades que el ejercicio fsico aerbico produce un aumento de los niveles de
CV son el sexo masculino, la edad (principal marcador de riesgo), la HDL-c. Si con la dieta no se consigue disminuir los niveles de colesterol
hiperlipemia (tanto hipercolesterolemia como hipertrigliceridemia), srico por debajo de 200 mg/dl, debe asociarse un frmaco
la diabetes (insulinodependiente o no), la hipertensin y el tabaquis- hipolipemiante. Para hacer el diagnstico de las hiperlipemias no se
mo. aconseja en nuestro medio su deteccin como prueba de screening,
De las tres formas de EV comentadas (cardiopata isqumica, acci- sino la bsqueda oportunista aprovechando la extraccin por otros
dente cerebrovascular y arteriopata perifrica), la ms prevalente es motivos. Para su determinacin no es necesario que el sujeto est en
la cardiopata isqumica, si bien la asociacin de ms de una de las ayunas. El estudio de las hiperlipidemias est especialmente indicado
anteriores es frecuente. Lo ms habitual es que las EV se manifiesten en los varones jvenes con historia personal o familiar de cardiopata
como cardiopata isqumica, suponiendo adems la forma de pre- isqumica.
sentacin que se asocia a una mayor mortalidad.

Pregunta 48.- R: 1
El principal marcador de riesgo para el desarrollo de enfermeda-
des cardiovasculares es la edad, de forma que son tanto ms prevalen-
tes cuanto mayor sea la edad de los individuos, si bien hay que recor-
dar que hasta un tercio de las muertes por cardiopata isqumica se

M exico A rgentina CTO Medicina C/ Nez de Balboa, 115 28006 MADRID (Espaa) Tfno.: (91) 782 43 32 / Fax: (91) 782 43 27
C hile U ruguay E-mail: secretaria@ctomedicina.com; iberocto@ctomedicina.com WEB: www.ctomedicina.com; www.iberocto.com
ED Pg. 9
HEMATOLOGA
Preparacin Examen de Seleccin 05/06 1 Vuelta Seguimiento a distancia

1. Indique cul de las siguientes anemias cursa con elevacin de 1) Palidez.


reticulocitos en sangre perifrica: 2) Hemorragias mucocutneas.
3) Esplenomegalia.
1) Aplasia. 4) Fiebre.
2) Sndrome mielodisplsico. 5) Ausencia de signos inflamatorios, a pesar de infeccin
3) Hemorragia aguda. activa.
4) Ferropenia.
5) Sideroblstica. 9. En referencia al tratamiento de la aplasia de mdula sea, NO
es correcto:
2. Cul de las siguientes anemias NO suele ser microctica?:
1) La mejora tras la administracin de globulina antitimo-
1) Por hipotiroidismo. cito (GAT) ocurre en las primeras semanas de tratamien-
2) Talasemias. to.
3) Ferropnica. 2) Asociando ciclosporina a GAT, se aumenta el nmero de
4) Sideroblstica hereditaria. respuestas, pero no la supervivencia.
5) Saturnismo. 3) Los esteroides en dosis altas pueden ser beneficiosos, pero
no se utilizan aisladamente.
3. De las siguientes enfermedades que producen anemia, cul 4) El principal efecto del GAT a largo plazo es la aparicin de
es la ms frecuente como anemia normoctica?: mielodisplasia.
5) El riesgo de rechazo del TMO es muy inferior en los
1) Aplasia. pacientes que no han recibido previamente transfusiones
2) Mieloptisis. sanguneas.
3) Anemia de enfermedad crnica.
4) Hemlisis congnita. 10. En el frotis sanguneo de un enfermo con anemia y trombo-
5) Hemlisis adquirida. penia se aprecian clulas en lgrima, normoblastos y clulas
blancas inmaduras. Cul de las siguientes causas le parece
4. Cul de las siguientes circunstancias NO es causa de macro- la MENOS probable?:
citosis eritrocitaria?:
1) Tuberculosis.
1) Tratamiento quimioteraputico. 2) Enfermedad de Gaucher.
2) Deficiencia de folato. 3) Timoma.
3) Hemorragia crnica. 4) Mielofibrosis agnognica.
4) Mielodisplasia. 5) Carcinoma de prstata.
5) Alcoholismo.
11. El incremento en la CHCM eritrocitaria es un dato caracte-
5. Los esquistocitos son caractersticos de: rstico de:

1) Talasemias. 1) Saturnismo.
2) Mieloptisis. 2) Ferropenia.
3) Hemlisis traumtica. 3) Microesferocitosis hereditaria.
4) Mielodisplasia. 4) Anemia sideroblstica.
5) Hipoesplenismo. 5) Talasemias.

6. En la aplasia de mdula sea, NO debe existir: 12. Seale la respuesta FALSA respecto al metabolismo del hie-
rro:
Preguntas TEST

1) Pancitopenia.
2) Presencia de blastos incrementados en M.O. 1) La mayor parte del hierro del organismo se encuentra en
3) Reticulocitopenia. los depsitos.
4) Hemorragias. 2) La mayor parte del hierro necesario para la eritropoyesis
5) Infecciones recidivantes. basal procede de la destruccin de los hemates viejos.
3) La absorcin intestinal es superior en forma de hierro hem
7. En el diagnstico diferencial de la pancitopenia, cabran las de la dieta.
siguientes posibilidades, con la EXCEPCIN de: 4) Las prdidas de hierro en la mujer suelen ser el doble que
en el hombre.
1) Infeccin por micobacterias atpicas en SIDA. 5) Las situaciones de eritropoyesis ineficaz incrementan la
2) Hemoglobinuria paroxstica nocturna. absorcin intestinal.
3) Dficit de vitamina B12.
4) Mielodisplasia. 13. NO ocurre en la anemia ferropnica:
5) Ferropenia.
1) Bajo VCM.
8. En los enfermos con anemia aplsica, usted NO esperara 2) Baja saturacin de transferrina.
encontrar uno de los siguientes signos de la exploracin fsica: 3) Baja concentracin de transferrina.

M exico A rgentina CTO Medicina C/ Nez de Balboa, 115 28006 MADRID (Espaa) Tfno.: (91) 782 43 32 / Fax: (91) 782 43 27
C hile U ruguay E-mail: secretaria@ctomedicina.com; iberocto@ctomedicina.com WEB: www.ctomedicina.com; www.iberocto.com
HM Pg. 1
HEMATOLOGA
Seguimiento a distancia Preparacin Examen de Seleccin 05/06 1 Vuelta
4) Baja HCM. 2) Incremento de LDH srica.
5) Baja CHCM. 3) Incremento de bilirrubina srica.
4) Incremento de homocistena srica.
14. Cul es el primer parmetro de laboratorio que se altera en 5) Incremento de metilmalnico srico.
la ferropenia?:
21. Respecto a los anticuerpos anti-factor intrnseco, seale la
1) Saturacin de transferrina. opcin INCORRECTA:
2) Ferritina.
3) Concentracin de transferrina. 1) Son ms especficos que los anticuerpos anti-clula parie-
4) Reticulocitos. tal.
5) VCM. 2) Su ausencia permite excluir el diagnstico de anemia
perniciosa.
15. El tratamiento con hierro en la anemia ferropnica debe 3) Su importancia patognica es incierta.
mantenerse hasta la normalizacin de: 4) Su positividad puede aparecer en diferentes enfermeda-
des autoinmunes.
1) Nmero de hemates. 5) Pueden verse en algunos casos de mieloma.
2) Valor de hemoglobina.
3) Sideremia. 22. En la diferenciacin entre las hemlisis intravasculares y
4) Transferrina srica. extravasculares, el parmetro ms fiable es:
5) Ferritina srica.
1) Elevacin de LDH srica.
16. Cul de los siguientes parmetros diferencia mejor ferrope- 2) Elevacin de bilirrubina indirecta en suero.
nia de anemia de enfermedad crnica?: 3) Descenso de haptoglobina srica libre.
4) Hemoglobinuria.
1) Sideremia. 5) Reticulocitosis.
2) Saturacin de transferrina.
3) Concentracin de transferrina. 23. Entre los hallazgos clnicos que son caractersticos de las
4) VCM disminuido. anemias hemolticas estn los siguientes, EXCEPTO:
5) HCM.
1) Coluria.
17. En relacin a la anemia asociada a enfermedades crnicas, es 2) Esplenomegalia.
INCORRECTO: 3) Ictericia.
4) lceras maleolares.
1) La produccin de eritropoyetina es inadecuada para el 5) Litiasis biliar.
grado de anemia.
2) El interfern inhibe la eritropoyesis. 24. Una de las siguientes anemias hemolticas NO tiene causa
3) El nmero de sideroblastos medulares est aumentado. extracorpuscular:
4) El grado de anemia no suele ser intenso.
5) La saturacin de transferrina suele estar menos descendi- 1) Esplenomegalia.
da que en la ferropenia. 2) Inmunohemlisis.
3) Microangioptica.
18. La presencia de anemia con incremento de sideremia y 4) Hemoglobinuria paroxstica nocturna.
saturacin de transferrina debe hacer pensar en: 5) Sepsis clostridial.

1) Ferropenia. 25. Respecto a la esferocitosis hereditaria, es FALSO:

Preguntas TEST
2) Anemia sideroblstica.
3) Dficit de folato. 1) La herencia suele ser autosmica dominante.
4) Dficit de B12. 2) Puede no manifestarse hasta la edad adulta.
5) Sndrome de Imerslund. 3) La prueba de Coombs es positiva.
4) La autohemlisis disminuye al aadir glucosa.
19. NO es propio de la hemopoyesis megaloblstica la presencia 5) Los hemates tienen una menor superficie.
de:
26. Acerca de la deficiencia de glucosa- 6-P deshidrogenasa, una
1) Precursores eritroides gigantes. de las siguientes respuestas NO es correcta:
2) Metamielocitos gigantes.
3) Macroovalocitos. 1) Es la causa ms frecuente de anemia hemoltica enzimo-
4) Indice de Herbert <3. ptica.
5) Neutrfilos hipersegmentados. 2) La herencia es ligada al X.
3) Puede ser desencadenada por sulfamidas.
20. En la deficiencia de cido flico, NO se encuentra: 4) La dosificacin enzimtica correcta debe hacerse en
reticulocitosis.
1) Hiperplasia de mdula sea. 5) El favismo es una variedad etiolgica.

M exico A rgentina CTO Medicina C/ Nez de Balboa, 115 28006 MADRID (Espaa) Tfno.: (91) 782 43 32 / Fax: (91) 782 43 27
Pg. 2 HM C hile U ruguay E-mail: secretaria@ctomedicina.com; iberocto@ctomedicina.com WEB: www.ctomedicina.com; www.iberocto.com
HEMATOLOGA
Preparacin Examen de Seleccin 05/06 1 Vuelta Seguimiento a distancia

27. Qu es INCORRECTO respecto a la anemia de Cooley?: 4) Coagulacin intravascular diseminada.


5) Sndrome HELLP.
1) Cursa con incremento de HbF.
2) Deben evitarse las transfusiones sanguneas. 34. Cul de las siguientes NO es componente caracterstico de
3) Cursa con malformaciones seas por incremento de la hemoglobinuria paroxstica nocturna?:
eritropoyetina.
4) Se asocia a hemocromatosis secundaria. 1) Trombocitosis.
5) El factor limitante del pronstico es la siderosis cardaca. 2) Preleucemia.
3) Hemlisis intravascular.
28. El rasgo talasmico se caracteriza por: 4) Trombosis venosa.
5) Episodios vasooclusivos dolorosos.
1) Anemia.
2) Esplenomegalia. 35. La prueba ms sensible y especfica para el diagnstico de
3) Malformaciones seas. hemoglobinuria paroxstica nocturna es:
4) Hemocromatosis.
5) Microcitosis con presencia de nmero normal o aumen- 1) Prueba de Ham.
tado de hemates. 2) Prueba de lisis con sacarosa.
3) Citometra de flujo sangunea.
29. Qu es FALSO en la hemoglobinopata S?: 4) Estudio de mdula sea.
5) Cariotipo de precursores hemopoyticos.
1) Se produce por polimerizacin y precipitacin de Hb.
2) Se debe realizar esplenectoma. 36. En el tratamiento de la hemoglobinuria paroxstica nocturna,
3) Hay isostenuria. es til todo lo siguiente, con EXCEPCIN de:
4) Cursa con lceras maleolares.
5) Debe hacerse profilaxis de infeccin neumoccica. 1) Hierro.
2) Esteroides.
30. Qu es FALSO sobre las anemias inmunohemolticas?: 3) Piridoxina.
4) Transfusiones.
1) La activacin del complemento ocurre sobre todo por IgM 5) Globulina antitimocito.
y causa hemlisis intravascular.
2) La hemlisis extravascular tiene lugar por clulas con 37. Cul de las siguientes caractersticas NO es propia de
receptores Fc de IgG. sndrome mielodisplsico?:
3) El Coombs directo es prueba inequvoca de hemlisis
inmune. 1) Panmielopata clonal.
4) La prueba de Coombs indirecta no siempre es positiva. 2) Eritropoyesis ineficaz.
5) La prueba de Coombs directa siempre es positiva. 3) Incremento de clulas sanguneas en sangre perifrica.
4) Metamorfosis blstica.
31. Una de las siguientes respuestas, respecto a la anemia por 5) Curso inicial indolente.
autoanticuerpos calientes, es INCORRECTA:
38. Entre los factores pronsticos desfavorables de los sndro-
1) Es ms frecuente en el sexo femenino. mes mielodisplsicos aparecen todos los siguientes, EXCEP-
2) Puede asociarse a trombopenia. TO:
3) La especificidad de Ac es frente a Ag Rh.
4) Es de predominio intravascular. 1) Anomalas en cariotipo.
5) Siempre hay que descartar como causas LES y LLC-B. 2) Presencia de sideroblastos en anillo en mdula.
Preguntas TEST

3) Grado de citopenias en sangre perifrica.


32. Una respuesta es FALSA respecto a la enfermedad por aglu- 4) Presencia de ms de un 5% de blastos en sangre.
tininas fras: 5) Edad avanzada.

1) Casi siempre es por IgM. 39. El tratamiento ms utilizado para los pacientes con mielodis-
2) La forma idioptica puede ser monoclonal. plasias es:
3) Los Ac estn dirigidos frente a Ag I/i.
4) Cursa con fenmeno de Raynaud. 1) Transfusiones sanguneas.
5) El inicio de la hemlisis es intravascular. 2) Trasplante de mdula sea.
3) Piridoxina.
33. La asociacin de anemia con frecuentes esquistocitos en el 4) Factores de crecimiento hemopoytico.
frotis y trombopenia es tpica de las siguientes enfermedades, 5) Quimioterapia.
EXCEPTO:
40. Seale la respuesta que mejor caracteriza a un sndrome
1) Sndrome de Evans. mieloproliferativo crnico:
2) Prpura trombopnica trombtica.
3) Sndrome hemoltico-urmico. 1) Aumento celular en mdula sea y aumento de clulas
maduras en sangre.

M exico A rgentina CTO Medicina C/ Nez de Balboa, 115 28006 MADRID (Espaa) Tfno.: (91) 782 43 32 / Fax: (91) 782 43 27
C hile U ruguay E-mail: secretaria@ctomedicina.com; iberocto@ctomedicina.com WEB: www.ctomedicina.com; www.iberocto.com
HM Pg. 3
HEMATOLOGA
Seguimiento a distancia Preparacin Examen de Seleccin 05/06 1 Vuelta
2) Aumento celular en mdula sea y aumento de clulas 2) Congestin venosa retiniana.
inmaduras en sangre. 3) Esplenomegalia.
3) Aumento celular en mdula sea y descenso de clulas 4) Aspecto pletrico.
maduras en sangre. 5) Hipertensin arterial.
4) Aumento de porcentaje de blastos en mdula sea.
5) Aumento de porcentaje de blastos en medula sea y 47. Uno de los siguientes parmetros NO es caracterstico de la
sangre. fase de metamorfosis de la policitemia vera:

41. Seale la respuesta que mejor define a un sndrome mielopro- 1) Incremento de hematocrito.
liferativo agudo: 2) Incremento de hepatoesplenomegalia.
3) Mielofibrosis.
1) Aumento celular en mdula sea y aumento de clulas 4) Metaplasia mieloide.
maduras en sangre. 5) Transformacin leucmica.
2) Aumento celular en mdula sea y aumento de clulas
inmaduras en sangre. 48. Respecto a la trombocitemia esencial, es FALSO:
3) Aumento de blastos en mdula sea y descenso de clulas
maduras en sangre. 1) La eritromelalgia puede ser una manifestacin clnica.
4) Aumento de blastos en mdula sea y aumento de clulas 2) En personas jvenes, puede no ocasionar sntomas.
maduras en sangre. 3) Una forma de tratamiento es la esplenectoma.
5) Aumento de clulas maduras en mdula sea y presencia 4) Puede dar accidentes isqumicos transitorios cerebrales.
de blastos en sangre. 5) No siempre existe correlacin entre la clnica y el nmero
de plaquetas.
42. De qu enfermedad es un hemograma con 150.000 leuco-
citos por mm3 con un 1% de blastos en sangre perifrica?: 49. Respecto a la crisis blstica de la leucemia mieloide crnica,
es INCORRECTO:
1) Leucemia mieloide crnica.
2) Leucemia mieloide aguda. 1) Se define por la existencia de ms de un 30% de blastos
3) Reaccin leucemoide en mieloptisis por cncer. en sangre o mdula sea.
4) Anemia refractaria con exceso de blastos. 2) Es ms frecuentemente mieloblstica.
5) Anemia refractaria con exceso de blastos en transforma- 3) El trasplante de mdula sea consigue un elevado porcen-
cin. taje de remisiones.
4) Puede presentarse sin fase crnica previa.
43. A qu enfermedad corresponde este hemograma: leucoci- 5) Puede asociarse a la existencia de cloromas.
tos 5.500/mm3 con 90% de blastos en sangre?:
50. Seale la respuesta INCORRECTA respecto a la incidencia de
1) Leucemia mieloide crnica. las diversas formas de leucemia:
2) Leucemia linftica crnica.
3) Leucemia mieloide aguda. 1) El pico de incidencia de la leucemia aguda linfoide es de
4) Anemia refractaria con exceso de blastos. 2 a 5 aos de edad.
5) Sepsis con reaccin leucoeritroblstica. 2) La leucemia mieloide crnica es ms frecuente por enci-
ma de 65 aos.
44. A qu enfermedad NO corresponde un hemograma con 3) La leucemia mieloide aguda es ms frecuente en adultos.
2.500 leucocitos/mm3 con un 3% de blastos en sangre?: 4) La leucemia linftica crnica es la leucemia ms frecuente
en ancianos.
1) Leucemia mieloide aguda. 5) La tricoleucemia predomina en adultos no ancianos.

Preguntas TEST
2) Leucemia linfoide aguda.
3) Leucemia mieloide crnica. 51. El factor pronstico ms importante, respecto al aumento de
4) Anemia refractaria con exceso de blastos. supervivencia en leucemia aguda mieloide, es:
5) Leucemia aguda en tratamiento.
1) Obtencin de remisin completa.
45. Clulas blsticas en sangre perifrica NO aparecen en: 2) Edad superior a 60 aos.
3) Grado de leucocitosis.
1) Leucemia mieloide crnica. 4) Subtipo de la FAB.
2) Leucemia linftica crnica. 5) Alteraciones citogenticas.
3) Mielodisplasia.
4) Mieloptisis. 52. Seale cul de los siguientes NO es un factor pronstico
5) Leucemia linftica aguda. adverso en leucemias agudas linfoblsticas:

46. Uno de los siguientes hallazgos de la exploracin fsica es 1) Nios mayores de 10 aos.
fundamental para diferenciar a los enfermos con policitemia 2) Adultos mayores de 35 aos.
vera de otras formas de poliglobulia: 3) Inmunofenotipo comn.
4) Leucocitosis superior a 50.000/mm3.
1) Cianosis. 5) Afectacin del sistema nervioso central.

M exico A rgentina CTO Medicina C/ Nez de Balboa, 115 28006 MADRID (Espaa) Tfno.: (91) 782 43 32 / Fax: (91) 782 43 27
Pg. 4 HM C hile U ruguay E-mail: secretaria@ctomedicina.com; iberocto@ctomedicina.com WEB: www.ctomedicina.com; www.iberocto.com
HEMATOLOGA
Preparacin Examen de Seleccin 05/06 1 Vuelta Seguimiento a distancia

53. Respecto a la infiltracin de las leucemias agudas, una res- 60. Un enfermo con afectacin por enfermedad de Hodgkin en
puesta es INCORRECTA: ganglios cervicales, axilares, mediastnicos y celacos, que no
presenta ningn sntoma B, se encuentra en estadio:
1) La afectacin de encas es ms frecuente en la forma
promieloctica. 1) IIA.
2) La CID es ms frecuente en la forma M3. 2) IIIA1.
3) La afectacin del SNC es ms frecuente en la LAL. 3) IIIA2.
4) La masa mediastnica es propia de LAL-T. 4) IIIAS.
5) La afectacin testicular es ms frecuente en LAL. 5) IVA.

54. Cul es el rgimen habitual de tratamiento de la LAM?: 61. El tratamiento inicial de la enfermedad de Hodgkin tiene
finalidad:
1) Vincristina + prednisona.
2) Vincristina + prednisona + L-asparaginasa. 1) Curativa en estadios I y II.
3) Vincristina + prednisona + L-asparaginasa + daunoblas- 2) Paliativa en estadios III y IV.
tina. 3) Paliativa en todos los estadios.
4) Arabinsido de citosina + daunoblastina. 4) Curativa en estadio I.
5) 6-mercaptopurina + metotrexate. 5) Curativa en todos los estadios.

55. El cido todo-transretinoico (tretinona) se ha usado como 62. En cul de las siguientes situaciones cree que es correcta la
inductor de remisin leucmica en una de las siguientes: laparotoma de estadiaje en la enfermedad de Hodgkin?:

1) Leucemia mieloide crnica. 1) Presencia de clulas de Sternberg en mdula sea.


2) Leucemia linfoblstica T. 2) Varn de 70 aos, en estadio clnico IIIB.
3) Tricoleucosis. 3) Presencia de lesiones ocupacionales en bazo, en la TC, y
4) Leucemia aguda promieloctica. biopsia heptica positiva.
5) Leucemia megacarioblstica. 4) Mujer joven, sin sntomas B y con adenopatas cervicales
y mediastnicas.
56. NO es correcto que la LLC: 5) Enfermedad IA cervical alta.

1) Habitualmente es de fenotipo B. 63. El tratamiento ms adecuado en un paciente con enferme-


2) Es una forma de linfoma de bajo grado. dad de Hodgkin estadio IIA con gran masa mediastnica es:
3) Es la forma ms frecuente de leucemia crnica en Occi-
dente. 1) RT en manto.
4) Casi nunca se acompaa de paraprotena srica. 2) MOPP.
5) Puede ser secundaria a quimioterpicos. 3) MOPP + ABVD.
4) ABVD.
57. En el sistema de estadiaje de RAI para la LLC, el PEOR 5) ABVD + RT mediastnica.
pronstico corresponde a:
64. Entre las complicaciones producidas por el tratamiento de la
1) Nmero de linfocitos. enfermedad de Hodgkin, seale la relacin FALSA:
2) Presencia de adenopatas.
3) Presencia de hepatomegalia. 1) Adriamicina - cardiotoxicidad.
4) Presencia de anemia. 2) MOPP - mielitis transversa.
5) Presencia de trombopenia. 3) RT - sarcoma de partes blandas.
Preguntas TEST

4) Bleomicina - neumonitis.
58. NO es propio de tricoleucosis: 5) Vincristina - neuropatas.

1) Resistencia de fosfatasa cida a tartrato. 65. Son factores pronsticos desfavorables en la enfermedad de
2) Pancitopenia. Hodgkin todos los siguientes, EXCEPTO:
3) Esplenomegalia progresiva.
4) Inmunofenotipo T. 1) Aumento de velocidad de sedimentacin globular.
5) Respuesta a interfern. 2) Hipogammaglobulinemia.
3) Edad avanzada.
59. Respecto de la histologa de la enfermedad de Hodgkin, cul 4) Respuesta lenta al tratamiento.
NO es correcta?: 5) Hipoalbuminemia.

1) La clula de Sternberg es patognomnica. 66. NO se ha objetivado relacin del VEB con uno de los siguien-
2) La clula de Hodgkin es la variante uninuclear. tes linfomas:
3) La variante de la esclerosis nodular es la clula lacunar.
4) La esclerosis nodular recidiva con la misma histologa. 1) Burkitt endmico.
5) El mejor pronstico corresponde a la forma de predomi- 2) Burkitt no endmico.
nio linfoctico.

M exico A rgentina CTO Medicina C/ Nez de Balboa, 115 28006 MADRID (Espaa) Tfno.: (91) 782 43 32 / Fax: (91) 782 43 27
C hile U ruguay E-mail: secretaria@ctomedicina.com; iberocto@ctomedicina.com WEB: www.ctomedicina.com; www.iberocto.com
HM Pg. 5
HEMATOLOGA
Seguimiento a distancia Preparacin Examen de Seleccin 05/06 1 Vuelta
3) Linfoma T del adulto. 2) El estudio de extensin debe incluir laparotoma.
4) Linfoma en enfermos de SIDA. 3) La base del tratamiento es la poliquimioterapia.
5) Enfermedad de Hodgkin. 4) Pueden ser B o T.
5) Tienen corta supervivencia sin tratamiento.
67. Cul de los siguientes linfomas no Hodgkin se considera de
alto grado en la clasificacin de Kiel?: 74. Respecto a los linfomas no hodgkinianos de bajo grado,
seale la respuesta INCORRECTA:
1) Inmunocitoma.
2) Centroctico. 1) Son poco frecuentes en la infancia.
3) Folicular. 2) Es frecuente la afeccin retroperitoneal.
4) Inmunoblstico. 3) Presentan elevadas remisiones completas con quimiote-
5) Leucemia linftica crnica. rapia.
4) No suelen invadir el sistema nervioso central.
68. Cul de las siguientes es la manifestacin ms frecuente del 5) El paciente puede estar asintomtico.
mieloma mltiple?:
75. El sndrome de Richter corresponde a la fase de transforma-
1) Adenopatas. cin de:
2) Dolores seos.
3) Hipercalcemia. 1) Linfoma de Burkitt.
4) Insuficiencia renal. 2) Linfoma folicular.
5) Neumona neumoccica. 3) Leucemia linftica crnica.
4) Linfoma histioctico.
69. Todos los siguientes criterios son propios del estadio I del 5) Linfoma inmunoblstico.
mieloma mltiple segn Durie-Salmon, con la EXCEPCIN
de: 76. Cul de los siguientes linfomas est relacionado con una
infeccin bacteriana crnica?:
1) Radiologa sea normal.
2) Lesin sea solitaria. 1) Linfoma de Burkitt.
3) Hemoglobina mayor de 10 g/dl. 2) Linfoma folicular.
4) Calcemia mayor de 12 mg/dl. 3) Linfoma MALT gstrico.
5) Cadenas ligeras en orina menor de 4 g/da. 4) Linfoma cerebral del SIDA.
5) Linfoma postrasplante renal.
70. En el mieloma mltiple, qu parmetro bioqumico se corre-
laciona mejor con la masa tumoral?: 77. Entre los factores pronsticos incluidos en el ndice interna-
cional para linfomas no hodgkinianos, aparecen todos los
1) Beta-2-microglobulina. siguientes, EXCEPTO:
2) Calcemia.
3) Creatinina srica. 1) Edad.
4) Viscosidad srica. 2) Estado de rendimiento del paciente.
5) Uricemia. 3) Inmunofenotipo de la clula tumoral.
4) LDH srica.
71. El tratamiento ms adecuado del plasmocitoma extramedu- 5) Afeccin extraganglionar.
lar sera:
78. En relacin con los linfomas asociados al SIDA, seale la
1) Melfaln y prednisona. respuesta INCORRECTA:

Preguntas TEST
2) Ciclofosfamida y prednisona.
3) Radioterapia. 1) Pueden manifestarse con derrames en cavidades.
4) Esteroides en dosis altas. 2) Predominantemente son de clula T.
5) Interfern. 3) Presentan frecuente afeccin de tubo digestivo.
4) Son de alto grado de agresividad.
72. Respecto a la enfermedad de Waldenstrm, NO es propio: 5) Suelen estar diseminados en el diagnstico.

1) Histologa de linfoma inmunoblstico. 79. Uno de los siguientes datos es ms frecuente en el linfoma-
2) Sndrome de hiperviscosidad. leucemia T asociado al HTLV-I que en otros linfomas:
3) Trastornos neurolgicos.
4) Presencia de crioaglutininas. 1) Elevacin de transaminasas.
5) Invasin neoplsica de la mdula sea. 2) Hipercalcemia.
3) Coagulacin intravascular.
73. Cul de las siguientes afirmaciones es FALSA en los linfomas 4) Hiperamoniemia.
no hodgkinianos de alto grado?: 5) Inmunodeficiencia humoral.

1) Con frecuencia tienen sntomas. 80. Uno de los siguientes factores de coagulacin NO es depen-
diente de vitamina K:

M exico A rgentina CTO Medicina C/ Nez de Balboa, 115 28006 MADRID (Espaa) Tfno.: (91) 782 43 32 / Fax: (91) 782 43 27
Pg. 6 HM C hile U ruguay E-mail: secretaria@ctomedicina.com; iberocto@ctomedicina.com WEB: www.ctomedicina.com; www.iberocto.com
HEMATOLOGA
Preparacin Examen de Seleccin 05/06 1 Vuelta Seguimiento a distancia

1) Protrombina. 2) Afectacin miocrdica.


2) Fibringeno. 3) Esquistocitos.
3) VII. 4) Fiebre.
4) IX. 5) Alteracin neurolgica.
5) X.
87. Cul es el tratamiento de eleccin de la prpura trombop-
81. Respecto a las pruebas de hemostasia, es INCORRECTO: nica trombtica?:

1) La trombopenia es la causa ms frecuente de prolonga- 1) Esplenectoma.


cin del tiempo de hemorragia. 2) Esteroides.
2) La causa ms frecuente de prolongacin del tiempo de 3) Inmunosupresores.
hemorragia con nmero normal de plaquetas es la enfer- 4) Plasmafresis.
medad de von Willebrand. 5) Antiagregantes.
3) El tiempo de protrombina controla la coagulacin extrn-
seca. 88. Qu respuesta es FALSA, respecto a la enfermedad de Von
4) El control del tratamiento anticoagulante oral se realiza Willebrand?:
mediante el tiempo de tromboplastina.
5) El tiempo de tromboplastina controla la coagulacin 1) Es la trombocitopata ms frecuente.
intrnseca. 2) El factor VW se sintetiza en megacariocitos y endotelio
vascular.
82. La deficiencia congnita de uno de los siguientes factores de 3) La forma ms grave de la enfermedad es la autosmica
coagulacin NO prolonga los tiempos de protrombina ni dominante.
tromboplastina: 4) El sangrado cutneo y ORL es el ms frecuente.
5) En el defecto cuantitativo, puede utilizarse como trata-
1) II. miento el acetato de desmopresina.
2) VII.
3) X. 89. En cul de las siguientes enfermedades NO es tpica la
4) XII. trombosis vascular?:
5) XIII.
1) Hemoglobinuria paroxstica nocturna.
83. NO es una trombopenia de mecanismo perifrico la asociada 2) Policitemia vera.
a: 3) Trombocitemia esencial.
4) Enfermedad de Rendu-Osler.
1) Mielodisplasia. 5) Enfermedad de Moschcowitz.
2) Coagulacin intravascular diseminada.
3) Prpura trombopnica idioptica. 90. Cul de los siguientes parmetros hemostticos NO es
4) Prpura trombopnica trombtica. propio de la coagulacin intravascular diseminada?:
5) Esplenomegalia.
1) Trombopenia.
2) Antitrombina III elevada.
84. Cul de los siguientes NO es criterio diagnstico de prpura
3) Tiempo de protrombina prolongado.
trombopnica idioptica?:
4) Hemlisis microangioptica.
5) Productos de degradacin de la fibrina.
1) Trombopenia.
2) Aumento del nmero y tamao de megacariocitos en
91. La alteracin de laboratorio que guarda una correlacin ms
M.O.
estrecha en la coagulacin intravascular diseminada con el
Preguntas TEST

3) Esplenomegalia.
sangrado es el nivel de:
4) Anticuerpos antiplaquetarios.
5) Descartar otras causas de trombopenia.
1) Trombopenia.
2) Prolongacin del tiempo de protrombina.
85. Cul es el tratamiento inicial de la prpura trombopnica 3) Prolongacin del TTPa.
idioptica crnica sintomtica?: 4) Fibringeno plasmtico.
5) PDF.
1) Esteroides.
2) Vigilancia sin tratamiento. 92. Entre los trastornos protrombticos hereditarios se encuen-
3) Esplenectoma. tran todos los siguientes, con EXCEPCIN de:
4) Inmunosupresores.
5) Gammaglobulina. 1) Factor V Leiden.
2) Dficit de plasmingeno.
86. Cul NO es componente propio de la prpura trombopni- 3) Dficit de PAI-I.
ca trombtica?: 4) Disfibrinogenemia.
5) Dficit de antitrombina III.
1) Trombopenia.

M exico A rgentina CTO Medicina C/ Nez de Balboa, 115 28006 MADRID (Espaa) Tfno.: (91) 782 43 32 / Fax: (91) 782 43 27
C hile U ruguay E-mail: secretaria@ctomedicina.com; iberocto@ctomedicina.com WEB: www.ctomedicina.com; www.iberocto.com
HM Pg. 7
HEMATOLOGA
Seguimiento a distancia Preparacin Examen de Seleccin 05/06 1 Vuelta
93. Respecto a los trastornos protrombticos hereditarios, sea- 99. La necrosis cutnea relacionada con anticoagulantes orales
le la respuesta FALSA: est en relacin con:

1) Suelen tener herencia autosmica dominante. 1) Hipersensibilidad medicamentosa.


2) No existen buenas pruebas de laboratorio para realizar 2) Dosis insuficiente.
despistaje. 3) Dosis excesiva.
3) Se deben sospechar en casos de tromboembolismo reci- 4) Asociacin con heparina de bajo peso molecular.
divante. 5) Deficiencia de protena C.
4) Ocasionan trombosis, sobre todo en la infancia.
5) Las mujeres deben evitar el uso de anovulatorios orales. 100. Los niveles de INR para control de anticoagulacin oral
deben mantenerse entre 2 y 3, con la EXCEPCIN de una de
94. En las cirrosis hepticas, NO suele encontrarse: las siguientes indicaciones:

1) Descenso de fibringeno. 1) Trombosis venosa profunda.


2) Descenso de factor V. 2) Prtesis valvular cardaca mecnica.
3) Tiempo de trombina alargado. 3) Fibrilacin auricular en estenosis mitral.
4) Dficit de factor VIII. 4) Disfuncin severa de ventrculo izquierdo.
5) Trombopenia. 5) Tromboembolismo pulmonar.
95. Cul de las siguientes situaciones puede producir hipocoa-
gulabilidad adquirida?:

1) Policitemia vera.
2) Dficit de hierro.
3) Ictericia obstructiva.
4) Dficit de cido flico.
5) Anemia hemoltica.

96. Las alteraciones de la hemostasia son caractersticas de los


siguientes procesos, con la EXCEPCIN de:

1) Adenocarcinoma de prstata.
2) Leucemia promieloctica.
3) Mieloma mltiple.
4) Enfermedad de Hodgkin.
5) Enfermedad de Waldenstrm.

97. Un paciente portador de una prtesis valvular cardaca


precisa intervencin quirrgica programada. En relacin a su
tratamiento anticoagulante, lo ms correcto ser:

1) Suspender la anticoagulacin de todo tipo 24 horas antes


de la intervencin.
2) Suspender anticoagulacin de todo tipo 48 h antes.
3) Suspender anticoagulacin oral 24 h antes y mantener
heparina intravenosa hasta el momento de la interven-

Preguntas TEST
cin.
4) Suspender anticoagulacin oral unos das antes y mante-
ner heparina intravenosa hasta unas horas antes de la
intervencin.
5) Mantener anticoagulacin oral hasta unas horas antes y
administrar entonces vitamina K y plasma.

98. La trombocitopenia secundaria a heparina:

1) Como otras trombopenias secundarias a frmacos, la


clnica hemorrgica es frecuente.
2) En esta trombopenia, caractersticamente son ms frecuen-
tes los fenmenos trombticos que los hemorrgicos.
3) Es ms frecuente con las heparinas de bajo peso molecular.
4) Se cree que su origen no es inmunolgico.
5) No existe tal asociacin.

M exico A rgentina CTO Medicina C/ Nez de Balboa, 115 28006 MADRID (Espaa) Tfno.: (91) 782 43 32 / Fax: (91) 782 43 27
Pg. 8 HM C hile U ruguay E-mail: secretaria@ctomedicina.com; iberocto@ctomedicina.com WEB: www.ctomedicina.com; www.iberocto.com
HEMATOLOGA
Preparacin Examen de Seleccin 05/06 1 Vuelta Seguimiento a distancia
Pregunta 1.- R: 3 El hemograma tpico de la aplasia se caracteriza por pancitopenia
Los reticulocitos traducen el grado de produccin de la mdula con reticulocitopenia. Esta disminucin de clulas sanguneas es con-
sea. La elevacin de reticulocitos es propia de las anemias secuencia de escasez de tejido hematopoytico, no de la invasin de
regenerativas, como la hemorragia aguda o los procesos hemolticos. la medula sea por tejidos anormales, incluyendo blastos. El incre-
Por contra, los fallos medulares (aplasias, sndrome mielodisplsico, mento de blastos en medula sea es propio de las leucemias agudas y
anemias sideroblsticas), o los trastornos carenciales (ferropenia) son de las mielodisplasias.
anemias hiporregenerativas donde no se elevan los reticulocitos.
Pregunta 7.- R: 5
Pregunta 2.- R: 1 La pancitopenia es propia de la aplasia, pero puede aparecer en
El hipotiroidismo suele producir anemias macrocticas, siendo ne- diversas patologas, como son la hemoglobinuria paroxstica noctur-
cesario introducir dicha enfermedad en el diagnostico diferencial de na, la deficiencia de vitamina B12 y la mielodisplasia. Estos trastornos
anemias macrocticas. Procesos que produzcan disminucin de la tienen en comn la posibilidad de producir simplemente anemia, si
membrana del hemate, como la microesferocitosis hereditaria, o tras- bien en formas ms avanzadas los tres trastornos pueden producir
tornos en los componentes de la hemoglobina (hierro, HEM, inmuno- pancitopenia. La invasin de la mdula sea por micobacterias pue-
globulina) producen anemias microcticas como son la ferropnica, la de cursar tambin con pancitopenia. En la ferropenia, adems de la
anemia de los trastornos crnicos, sideroblsticas y talasemias. anemia puede ser caracterstica la elevacin de la cifra de plaquetas.

Pregunta 3.- R: 3 Pregunta 8.- R: 3


La anemia de los trastornos crnicos es la causa ms frecuente de Las manifestaciones clnicas del paciente con aplasia derivan de la
anemia normoctica. Otras enfermedades que cursan con anemias de escasez de clulas sanguneas, por lo que los pacientes tiene sndrome
tamao normal de hemate son enfermedades medulares como la apla- anmico, infecciones si presentan neutropenias severas y hemorragias
sia y las mieloptisis y la mayor parte de los procesos hemolticos. principalmente mucocutneas si hay una cifra muy baja de plaquetas.
En neutropenias muy intensas pueden faltar los signos inflamatorias
Pregunta 4.- R: 3 acompaantes de procesos infecciosos. La aplasia no cursa con
La hemorragia crnica genera ferropenia, y por tanto, anemia mi- organomegalias, incluida esplenomegalia.
croctica. Las anemias macrocticas pueden ser megaloblsticas (pre-
cursores gigantes en la deficiencia de folato y B12), y en general, las Pregunta 9.- R: 1
enfermedades de precursores medulares como mielodisplasia pue- El tratamiento de referencia de la aplasia es el trasplante de mdula
den producir anemias macrocticas. En la prctica habitual, la causa sea. La posibilidad de rechazo de dicho trasplante es inferior en
ms frecuente de macrocitosis es el alcoholismo. pacientes que no han sido sensibilizados por transfusiones sanguneas
previas. Como alternativa al trasplante, la eleccin consiste en la aso-
Pregunta 5.- R: 3 ciacin de globulina antitimocito y ciclosporina. Ocasionalmente se
El concepto de enfermedad hemoltica es aquella en la que la vida pueden asociar esteroides. Estos tratamientos producen mejora del
media del hemate se encuentra acortada. Si este acortamiento de la pacientes a lo largo de varias semanas, no de forma rpida. Una com-
vida eritrocitaria se debe a traumatismo, aparecen caractersticamente plicacin del tratamiento con GAT puede ser la aparicin de displasia.
los esquistocitos, que por tanto no son caractersticos de todas las
enfermedades hemolticas, sino de los trastornos traumticos. Pregunta 10.- R: 3
La disociacin de clulas rojas en lgrimas (dacriocitos) y la reac-
Pregunta 6.- R: 2 cin leucoeritroblstica (clulas inmaduras en sangre perifrica de la
seria blanca y roja) es muy caracterstica, asociada a las citopenias del
paciente, a la mieloptisis. La mieloptisis consiste en la ocupacin de la
medula sea por tejidos anormales, que por proceso ocupacional
disminuyen la produccin de clulas sanguneas normales, al mismo
tiempo que pueden desplazar hacia la sangre perifrica clulas inma-
duras. Causas frecuentes de mieloptisis son las metstasis de cncer de
mdula sea. El timoma produce una lesin medular distinta de la
Comentarios TEST

mieloptisis, consistente en la eritroblastopenia o aplasia pura de la


serie roja.

Pregunta 11.- R: 3
Las anemias microcticas se producen fundamentalmente por dos
Pregunta 6. Caractersticas hematolgicas de la aplasia. mecanismos. En el primero de ellos, ms frecuente, se produce una

Pregunta 10. Anemia mieloptsica: reaccin leucoeritroblstica y clulas en lgrima.

M exico A rgentina CTO Medicina C/ Nez de Balboa, 115 28006 MADRID (Espaa) Tfno.: (91) 782 43 32 / Fax: (91) 782 43 27
C hile U ruguay E-mail: secretaria@ctomedicina.com; iberocto@ctomedicina.com WEB: www.ctomedicina.com; www.iberocto.com
HM Pg. 1
HEMATOLOGA
Seguimiento a distancia Preparacin Examen de Seleccin 05/06 1 Vuelta
disminucin de la cantidad de hemoglobina del hemate. Estas pato- anemia sideroblstica, que ocasionalmente precisa tratamiento con
logas estaran representadas por problemas en los componentes de la quelantes del hierro para evitar sobrecarga frrica.
hemoglobina (hierro, HEM, y cadenas de globina). Con este mecanis-
mo, no se produce slo disminucin del tamao del hemate, sino Pregunta 19.- R: 4
tambin hipocroma eritrocitaria (representada en el hemograma por Los hallazgos del hemograma en anemia megaloblstica suelen ser
el parmetro CHCM). El segundo mecanismo consiste en la disminu- tpicos: macrocitosis, ocasionalmente muy intensa, hemates de for-
cin de la superficie de membrana, como es el caso de la esferocitosis ma ovalada en vez de redonda (macroovalocitos), presencia de hiper-
hereditaria, si bien este mecanismo no produce disminucin del con- segmentacin de los neutrfilos, con existencia de anemia exclusiva-
tenido de hemoglobina, y por tanto, no ocasiona hipocroma, sino mente o diversas citopenias, hasta la pancitopenia. A nivel medular se
aumento de la CHCM. objetivan precursores de tamao gigante. El ndice de Herbert es una
media de la segmentacin del ncleo de los neutrfilos que, como se
Pregunta 12.- R: 1 ha dicho, est hipersegmentado en las anemias megaloblsticas. Un
La mayor parte del hierro del organismo se encuentra en la mol- ndice de Herbert inferior a 3 producira una situacin de
cula de hemoglobina. La mayora del hierro necesario para la eritro- hiposegmentacin de los neutrfilos, rasgo que es ms caracterstico
poyesis basal procede de la destruccin de los hemates viejos de las mielodisplasias que de las anemias megaloblsticas.
(hemocateresis). La absorcin intestinal de hierro es mejor en forma
de hierro orgnico, como es el hiero HEM. Las situaciones de eritro- Pregunta 20.- R: 5
poyesis ineficaz incrementan la absorcin intestinal de hierro, pu- La anemia megaloblstica se caracteriza por citopenias en sangre
diendo ocasionar a largo plazo situaciones de hemosiderosis. perifrica, pero mdulas hipercelulares, en las que existe una intensa
hemopoyesis ineficaz, lo que puede ocasionar a nivel srico datos de
Pregunta 13.- R: 3 destruccin celular excesiva (elevacin de LDH y de bilirrubina).
La situaciones de ferropenia se caracterizan por un patrn bioqumi- Ocasionalmente en la anemia megaloblstica los niveles sricos
co consistente en disminucin de la sideremia, aumento de la concentra- de folato y B12 pueden ser normales, siendo ms sensibles para de-
cin de transferrina, disminucin de la saturacin de transferrina y dismi- tectar carencia de esta sustancia los niveles de homocistena, que se
nucin de la ferritina srica. En el hemograma, adems de microcitosis se incrementan tanto en la carencia de folato como en la de B12 y los
ocasiona hipocroma, objetivada por descenso de HCM y CHCM. de metilmalnico, que slo se incrementan en la deficiencia de B 12.

Pregunta 14.- R: 2 Pregunta 21.- R: 2


En la situacin de ferropenia inicial no se produce ni anemia ni La anemia perniciosa suele ser la forma ms severa de anemia
tienen porque aparecer alteraciones en el hemograma. Dado que para megaloblstica, caracterizndose histolgicamente por una atrofia de
mantener la eritropoyesis el organismo utiliza las reservas de hierro, la la mucosa gstrica y por la aparicin de anticuerpos contra clula
primera alteracin en el laboratorio consiste en disminucin de la ferri- parietal (poco especficos) y contra el factor intrnseco, si bien estos
tina srica, parmetro que se correlaciona con el depsito del hierro. ltimos slo aparecen en dos tercios de los casos, por lo que la ausen-
cia de anticuerpos contra el factor intrnseco no puede descartar una
Pregunta 15.- R: 5 anemia perniciosa.
Durante el desarrollo de ferropenia se produce inicialmente el
consumo de los depsitos de hierro. Durante el tratamiento de la Pregunta 22.- R: 4
ferropenia se produce inicialmente un incremento de la eritropoye- El hemograma de las anemias hemolticas se suele caracterizar por
sis, reconocida en el hemograma por el aumento de los reticulocitos anemias normocticas con reticulocitosis sangunea y frotis caracters-
sanguneos y una sucesiva normalizacin del resto de parmetros, tico. En gran parte de los pacientes con enfermedades hemolticas se
siendo lo ltimo en normalizarse los depsitos de hierro, y por tanto produce destruccin de hemates tanto intra como extravascular, por
los niveles de ferritina srica. lo que datos como la elevacin de LDH o de bilirrubina indirecta en
suero o el descenso de haptoglobina srica libre se pueden ver tanto
Pregunta 16.- R: 3 en los procesos hemolticos intra como extravasculares. La existencia
La hiposideremia es un hallazgo propio tanto de la ferropenia como de hemoglobinuria o hemosiderinuria es, sin embargo, caracterstica
de la anemia de los trastornos crnicos, dado que en esta ltima existe de la lisis intravascular.
un bloqueo en la salida del hierro de los macrfagos hacia la sangre.

Comentarios TEST
Por tanto, el concepto de hiposideremia no puede hacerse igual a Pregunta 23.- R: 1
ferropenia. La saturacin de transferrina est generalmente disminui- Entre las caractersticas clnicas de los pacientes con enfermedades
da en ambos procesos, mientras que la concentracin de transferrina hemolticas puede aparecer la existencia de orinas oscuras. Dicha
est elevada en la ferropenia y suele ser normal, o ms frecuentemen- alteracin es consecuencia de hemoglobinuria y no de bilirrubinuria
te disminuida, en la anemia de la enfermedad crnica. Ambos proce- o coluria. En ausencia de obstruccin de la va biliar, la bilirrubina
sos producen anemias que pueden ser microcticas e hipocromas. generada en el proceso de destruccin de hemates es conjugada en
el hgado y eliminada por bilis, no por orina. La litiasis biliar puede ser
Pregunta 17.- R: 3 una complicacin de los pacientes con enfermedades hemolticas
Los mecanismos de produccin de anemia en las enfermedades crnicas, por el exceso de produccin de bilirrubina. La
crnicas son mltiples. Por una parte est la inadecuada produccin esplenomegalia es un dato caracterstico de muchos pacientes con
de la mdula sea como consecuencia de una inadecuada produc- enfermedades hemolticas.
cin de eritropoyetina o de la presencia de interfern o factor de necrosis
tumoral alfa, que son inhibidores de la eritropoyesis. Por otra parte, en Pregunta 24.- R: 4
lo que se refiere al uso del hierro, existe un bloqueo en la salida del La destruccin de hemates en las anemias hemolticas puede ser
hierro de depsito, tanto a la sangre como hacia los precursores de la consecuencia de alteraciones a nivel celular intracorpuscular (como
serie roja a la mdula, por lo que no existe el incremento de precursores es el caso de la hemoglobinuria paroxstica nocturna) o extracorpus-
con exceso de hierro (sideroblastos). La intensidad de la anemia en los cular, en cuyo caso el hemate es normal, pudiendo ser destruido por
trastornos crnicos suele ser moderada. una hiperfuncin esplnica, anticuerpos antihemates, alteraciones
de la microcirculacin o toxinas como ocurre en la sepsis clostridial a
Pregunta 18.- R: 2 travs de una hemolisina.
Si bien el descenso de sideremia y de saturacin de transferrina son
hallazgos propios tanto de la anemia ferropnica como la de los tras- Pregunta 25.- R: 3
tornos crnicos, el incremento en ambos parmetros es caracterstico La existencia de esferocitos en sangre perifrica es caracterstica
de situaciones de exceso orgnico de hierro, como es el caso de la de los procesos hemolticos, fundamentalmente la esferocitosis he-

M exico A rgentina CTO Medicina C/ Nez de Balboa, 115 28006 MADRID (Espaa) Tfno.: (91) 782 43 32 / Fax: (91) 782 43 27
Pg. 2 HM C hile U ruguay E-mail: secretaria@ctomedicina.com; iberocto@ctomedicina.com WEB: www.ctomedicina.com; www.iberocto.com
HEMATOLOGA
Preparacin Examen de Seleccin 05/06 1 Vuelta Seguimiento a distancia
reditaria y las inmunohemlisis. Este ltimo grupo se caracteriza
por positividad en la prueba de Coombs, dado que la destruccin Pregunta 28.- R: 5
de los hemates es por efectos de anticuerpos. La aparicin de
esferocitos en la esferocitosis hereditaria no es consecuencia de
ataque de la clula roja por anticuerpos, sino del atrapamiento
esplnico del hemate como consecuencia de alteraciones en la
adaptabilidad celular por fallo en la membrana. La esferocitosis
hereditaria suele ser autosmica dominante, aunque hay formas
recesivas ms graves. La clnica de la esferocitosis hereditaria es
muy variable, pudiendo no manifestarse hasta la edad adulta en
sus formas leves.

Pregunta 26.- R: 4
La deficiencia de glucosa-6-fosfato deshidrogenasa es la enzi-
mopata ms frecuente eritrocitaria y est ligada al cromosoma X.
Caractersticamente el proceso hemoltico aparece en brotes des-
encadenados por infecciones, situaciones de acidosis, ingestin
de habas o utilizacin de frmacos como las sulfamidas. En los
reticulocitos, la cantidad de glucosa-6-fosfato deshidrogenasa es
ms elevada que en los hemates ms viejos, por lo que no se
aconseja la cuantificacin enzimtica en situaciones de reticuloci-
tosis, como ocurre durante las crisis hemolticas o inmediatamente
despus de las mismas.

Pregunta 26. Frmacos que producen hemlisis en personas con dficit


de G-6PD

123456768569
2 6 2465
 45
429
4469
2
4
2226
4696296
24669

123456645
6
2495642 6965

12345669
29

 642 646426 64
125
2
7496265642 Pregunta 28. Fisiopatologa de la -talasemia.

En el rasgo talasmico, el hemograma se caracteriza por la apari-


Pregunta 27.- R: 2 cin de un nmero incrementado de hemates de pequeo tamao.
La anemia de Cooley es la forma grave de la talasemia beta y ac- Hay que tener presente que dicha situacin puede corresponder tam-
tualmente su nico tratamiento curativo es el trasplante de progenito- bin ocasionalmente a situaciones de ferropenia no intensa, por lo
res hematopoyticos. En ausencia de dicho tratamiento se puede rea- que el hemograma no permite diferenciar entre ambas patologas,
lizar paliacin de la anemia administrando transfusiones sanguneas, siendo necesarios estudios de las hemoglobinas eritrocitarias para diag-
tratamiento que tambin es beneficioso por disminuir el grado de nosticar rasgo talasmico y de ferritina srica para diagnosticar ferro-
hiperplasia medular y su alteracin sea consiguiente. Las talasemias penia. Hay que tener presente tambin que diferenciar entre ambas
graves se caracterizan por una eritropoyesis ineficaz intensa que faci- situaciones es fundamental, dado que la ferropenia requiere trata-
lita la absorcin intestinal de hierro y puede dar lugar a sobrecarga miento con hierro y el rasgo talasmico es una situacin en la que el
frrica, siendo la afeccin cardaca la forma ms grave. Por dicho hierro est contraindicado.
motivo es necesario tratamiento tambin con quelantes del hierro.
Comentarios TEST

Pregunta 27. Anemia de Cooley. Alteraciones craneofaciales.

M exico A rgentina CTO Medicina C/ Nez de Balboa, 115 28006 MADRID (Espaa) Tfno.: (91) 782 43 32 / Fax: (91) 782 43 27
C hile U ruguay E-mail: secretaria@ctomedicina.com; iberocto@ctomedicina.com WEB: www.ctomedicina.com; www.iberocto.com
HM Pg. 3
HEMATOLOGA
Seguimiento a distancia Preparacin Examen de Seleccin 05/06 1 Vuelta
Pregunta 29.- R: 2 sensibles al complemento. Este hecho es consecuencia de la escasez
La hemoglobinopata S, anemia de clulas falciformes o drepano- en la membrana celular de sustancias que neutralizan la actividad del
citosis, es un proceso en el que la hemoglobina inestable precipita complemento (CD59 y CD55). La forma ms exacta de diagnosticar la
formando clulas en forma de hoz que, adems de ser destruidas, enfermedad consiste en el estudio de la membrana celular mediante
ocluyen los vasos sanguneos, produciendo las denominadas crisis citometra de flujo, objetivando la escasez de dichos marcadores.
vasooclusivas. La isquemia generada por estos episodios vasooclusivos Formas indirectas, menos sensibles y especficas son las prueba de
puede desencadenar infartos, de los que los ms frecuentes son los Ham y la lisis con sacarosa.
renales, con la consiguiente alteracin en la concentracin urinaria
(isostenuria). Como consecuencia de infartos esplnicos repetidos se Pregunta 36.- R: 3
puede llegar a una situacin de hipoesplenismo con inmunodefi- Como otros procesos hemolticos, la hemoglobinuria paroxsti-
ciencia asociada. La esplenectoma, por tanto, no es un tratamiento ca nocturna puede responder ocasionalmente a esteroides. En si-
para esta enfermedad. tuaciones de hemosiderinuria crnica puede desencadenarse una
carencia de hierro, por lo que el tratamiento con hierro puede ser
Pregunta 30.- R: 5 beneficioso en ciertas circunstancias. Las transfusiones forman parte
En las anemias inmunohemolticas, la destruccin del hemate se del tratamiento paliativo de todo tipo de anemias. Ocasionalmente
produce por la accin de anticuerpos, complemento o ambas sustan- la hemoglobinuria paroxstica nocturna puede coexistir con hipo-
cias sobre la membrana del hemate. Cuando tiene lugar una intensa plasia de medula sea, siendo entonces beneficioso el uso de tra-
activacin del complemento, como es el caso de la producida por tamientos para este proceso, como es el caso de la globulina anti-
IgM, se produce una destruccin intravascular y cuando la activacin timocito.
del complemento es escasa, como es el caso con anticuerpos del tipo
IgG, la destruccin es predominantemente esplnica. La base del diag- Pregunta 37.- R: 3
nstico de las enfermedades inmunohemolticas es la prueba de Los sndromes mielodisplsicos son mielopatas clonales que se
Coombs para detectar anticuerpos antieritrocitarios o la accin del caracterizan por eritropoyesis ineficaz con la consiguiente anemia.
complemento. La prueba de Coombs directa detecta anticuerpo y/o En formas severas de mielodisplasia puede existir tambin leucopenia
complemento sobre la membrana del hemate y la de Coombs indi- y trombopenia, caracterizndose por tanto estos procesos por
recta anticuerpo en plasma. Ambas pruebas pueden salir negativas citopenias sanguneas diversas aisladas o en combinacin. Como to-
(anemia inmunohemoltica: Coombs negativo). das las mielopatas clonales, estos procesos pueden evolucionar hacia
leucemia aguda.
Respuesta 31.- R: 4
Un anticuerpo caliente se fija en la membrana del hemate a tem- Pregunta 38.- R: 2
peratura fisiolgica y produce una destruccin principalmente La existencia de sideroblastos en anillo en mdula sea es caracte-
esplnica, tras unin de la fraccin constante de la IgG a los recepto- rstica de una forma de sndrome mielodisplsico denominado ane-
res de los macrfagos esplnicos. Ocasionalmente puede asociarse mia refractaria con sideroblastos en anillo, que es el trastorno menos
trombopenia de mecanismo inmune (sndrome de Evans). Aunque grave. En general, las alteraciones citogenticas en enfermedades de
hay formas idiopticas siempre hay que descartar etiologas conoci- precursores medulares suelen implicar mal pronstico. Al tratarse de
das (frmacos, infecciones, enfermedades autoinmunes o procesos enfermedades que ocasionan citopenias sanguneas es lgico que la
linfoproliferativos). intensidad de las citopenias implique importancia pronstica, al igual
que la edad avanzada y el incremento de blastos sanguneos, situa-
Pregunta 32.- R: 4 cin que se da con la evolucin hacia la leucemia aguda.
La enfermedad de las aglutininas fras est causada por anticuerpos
que se fijan al hemate a temperatura inferior a la fisiolgica, son Pregunta 39.- R: 1
habitualmente de tipo IgM y activan intensamente complemento pro- Se han intentado mltiples tratamientos en los pacientes con mie-
duciendo lisis intravascular. El trmino aglutinina implica que es un lodisplasias, lo que implica que no se ha encontrado todava un trata-
anticuerpo que aglutina hemates, pudiendo producirse atasco en la miento bueno para la mayor parte de los pacientes. El trasplante de
microcirculacin con fenmenos isqumicos desencadenados por mdula sea sera un tratamiento definitivo de un grupo minoritario
exposicin al fro. En este caso se produce una acrocianosis con la de pacientes (dada la edad avanzada en gran parte de casos). Quimio-
exposicin al fro sin verdadero vasoespasmo o fenmeno de Raynaud. terapia con intento de erradicacin del clon premaligno, o frmacos

Comentarios TEST
como los factores de crecimiento hemopoytico son tratamientos
Pregunta 33.- R: 1 paliativos. La piridoxina ocasionalmente produce un incremento en
La asociacin de anemia y trombopenia se puede dar en los cinco la sntesis HEM y mejora de la anemia de los pacientes (anemia refrac-
procesos especificados en las respuestas, si bien el sndrome de Evans, taria sensible a la piridoxina).
por ser un proceso hemoltico inmune, se asocia caractersticamente
con esferocitos en vez de con esquistocitos. Los otros cuatro procesos Pregunta 40.- R: 1
ocasionan hemlisis traumtica, y por tanto la existencia de esquisto-
citos es caracterstica.

Pregunta 34.- R: 1
La hemoglobinuria paroxstica nocturna se caracteriza por la exis-
tencia de citopenias hematolgicas, pudiendo existir slo anemia o,
en sus formas ms severas, incluso pancitopenia. Como todos los tras-
tornos clonales de la mdula sea, puede evolucionar hacia leuce-
mia aguda.
El trastorno es consecuencia de un exceso de sensibilidad de los
hemates al complemento, por lo que la lisis es intravascular. Otra
caracterstica de la enfermedad es la existencia de fenmenos trom-
bticos de localizaciones inhabituales, como es el caso de una trom-
bosis mesentrica.

Pregunta 35.- R: 3
La hemoglobinuria paroxstica nocturna es una mielopata clonal Pregunta 40. Sndrome mieloproliferativo crnico.
donde las clulas sanguneas del clon anmalo son especialmente

M exico A rgentina CTO Medicina C/ Nez de Balboa, 115 28006 MADRID (Espaa) Tfno.: (91) 782 43 32 / Fax: (91) 782 43 27
Pg. 4 HM C hile U ruguay E-mail: secretaria@ctomedicina.com; iberocto@ctomedicina.com WEB: www.ctomedicina.com; www.iberocto.com
HEMATOLOGA
Preparacin Examen de Seleccin 05/06 1 Vuelta Seguimiento a distancia
Los sndromes mieloproliferativos crnicos son mielopatas clona- sarios otros estudios para el diagnstico diferencial con patologas que
les caracterizadas por un exceso de proliferacin en las clulas del presentan blastos en escasa proporcin en la sangre, como pueden
clon anormal, que conserva la capacidad de maduracin, por lo que ser la leucemia mieloide crnica, los sndromes mielodisplsicos, y
lo caracterstico desde el punto de vista hematolgico en estos proce- muy excepcionalmente, procesos mieloptsicos. En la leucemia linf-
sos es una hipercelularidad medular y el incremento de clulas san- tica crnica las clulas sanguneas son de apariencia madura, no
guneas, pero de tipo maduro. blsticas.

Pregunta 41.- R: 3 Pregunta 46.- R: 3


Un sndrome mieloproliferativo agudo (leucemia aguda) es un pro- La existencia de esplenomegalia es un hecho caracterstico de los
ceso clonal de la mdula sea donde las clulas malignas no son sndromes mieloproliferativos crnicos, pudiendo aparecer en todos
capaces de madurar mas all de la fase de blasto, condicionando una ellos. Se ha considerado adems un criterio mayor de policitemia
ocupacin medular por blastos, que a su vez disminuye el espacio vera, dada la gran importancia de este hallazgo, teniendo presente
para la fabricacin de clulas normales, con la consiguiente disminu- adems que la existencia de esplenomegalia en una persona suele ir
cin de clulas maduras en sangre perifrica. aadida a cifras bajas de clulas sanguneas como consecuencia de
hiperesplenismo. Cuando en un paciente se dan a la vez la existencia
de esplenomegalia y elevacin de clulas sanguneas, siempre ha de
considerarse como diagnstico un sndrome proliferativo que llena la
sangre de clulas e infiltra el bazo, ocasionando una asociacin apa-
rentemente paradjica. En muchas manifestaciones, la policitemia
vera coincide con otras formas de poliglobulia, como consecuencia
de los trastornos circulatorios a los que da lugar el incremento de la
viscosidad sangunea. La existencia de eritrocianosis es una caracters-
tica que puede existir en la policitemia vera.

Pregunta 47.- R: 1
La denominada fase de metamorfosis de policitemia vera o fase
gastada es consecuencia de un proceso fibrtico de la mdula sea
en el que se va reduciendo progresivamente la formacin de clulas
sanguneas, incluida la serie roja, con la aparicin incluso de anemia.
En esta fase, la policitemia vera es indistinguible de la denominada
Pregunta 41. Sndrome mieloproliferativo agudo. mielofibrosis con metaplasia mieloide, pudiendo aparecer en ambos
procesos hepatoesplenomegalia como consecuencia de la
hemopoyesis extramedular en hgado y bazo. Excepcionalmente
Pregunta 42.- R: 1 puede haber una evolucin a la leucemia aguda.
Una leucocitosis de la intensidad del paciente es propia de una
leucemia. Las llamadas reacciones leucemoides son situaciones de Pregunta 48.- R: 3
leucocitosis que pueden confundirse con leucemias en el hemogra- La trombocitosis o trombocitemia esencial se considera el menos
ma, si bien generalmente no alcanzan cifras tan intensas. Los sndro- grave de los sndromes mieloproliferativos crnicos clsicos, pudiendo
mes mielodisplsicos se caracterizan por leucopenia en vez de leuco- incluso ser asintomtico en fases iniciales. La clnica propia de la enfer-
citosis. Se entiende que si el 1% de los leucocitos sanguneos son medad consiste en trastornos isqumicos por el exceso de plaquetas
blastos, el 99% restante son clulas que estn en fases ms maduras, sanguneas, no existiendo una necesaria correlacin entre la gravedad
situacin que es incompatible con la leucemia aguda. En la leucemia de la clnica y la cifra plaquetaria en sangre. Como el resto de sndromes
mieloide crnica, a pesar de que existe predominio de clulas madu- mieloproliferativos crnicos, puede cursar con esplenomegalia. Sin
ras, puede haber ocasionalmente una pequea cuanta de blastos en embargo, la esplenectoma no seria un tratamiento de la enfermedad,
sangre perifrica. ya que la eliminacin del bazo favorecera el incremento en la cifra de
plaquetas. En el tratamiento de esta enfermedad se suele utilizar
Comentarios TEST

Pregunta 43.- R: 3 hidroxiurea para disminuir la formacin de clulas a nivel medular.


La existencia de ms de un 20% de blastos en sangre perifrica es
un criterio definitivo de leucemia aguda. En el hemograma del pa- Pregunta 49.- R: 3
ciente no se establecen caractersticas de la celularidad blstica, por
lo que no podemos asegurar el tipo de leucemia aguda que tiene este
paciente, de ah que si la respuesta 3 dijese leucemia linfoide aguda,
tambin valdra como contestacin de la pregunta.

Pregunta 44.- R: 3
Las leucemias crnicas (leucemia mieloide crnica, leucemia mie-
lomonoctica crnica, leucemia linfoide crnica) se caracterizan siem-
pre por leucocitosis sangunea, por lo que la existencia de leucopenia
va en contra del diagnstico de leucemia crnica. En las leucemias
agudas, cuando la cantidad de blastos sanguneos es muy escasa,
puede condicionarse leucopenia como consecuencia de la disminu-
cin de produccin de clulas normales por la medula sea. Los
sndromes mielodisplsicos pueden tener tambin leucopenia con
blastos en sangre.

Pregunta 45.- R: 2
La existencia de blastos en sangre siempre es patolgica, siendo
caracterstica de las leucemias agudas. Si el porcentaje de blastos es
superior al 20%, el diagnstico de leucemia aguda es seguro. Ocasio-
nalmente los pacientes con leucemias agudas pueden tener un por-
Pregunta 49. Fases en la evolucin de la LMC.
centaje de blastos en sangre inferior al citado, siendo entonces nece-

M exico A rgentina CTO Medicina C/ Nez de Balboa, 115 28006 MADRID (Espaa) Tfno.: (91) 782 43 32 / Fax: (91) 782 43 27
C hile U ruguay E-mail: secretaria@ctomedicina.com; iberocto@ctomedicina.com WEB: www.ctomedicina.com; www.iberocto.com
HM Pg. 5
HEMATOLOGA
Seguimiento a distancia Preparacin Examen de Seleccin 05/06 1 Vuelta
La leucemia mieloide crnica es un sndrome mieloproliferativo Pregunta 52.- R: 3
crnico en el que la evolucin a leucemia aguda (crisis blstica) es El denominado inmunofenotipo comn, o preB, recibe este nom-
ms frecuente. Un dato de existencia de ms de un 20% de blastos en bre por ser la variedad de leucemia aguda linfoblstica ms frecuente
sangre perifrica o mdula sea puede definir la crisis blstica, que es en nios. Afortunadamente es una leucemia de pronstico favorable
ms frecuente de tipo mieloide que linfoide. Ocasionalmente esta que presenta como marcador tpico el CALLA. El mejor pronstico en
fase se puede presentar sin una fase crnica previa. El tratamiento de nios corresponde entre las edades de 1 y 9 aos, y en adultos, antes
esta fase blstica es infructuoso en la mayora de las ocasiones y ni de los 35 aos. La existencia de leucocitosis, que se traduce en una
siquiera un trasplante de mdula consigue porcentajes elevados de elevada masa tumoral, al igual que la afectacin del sistema nervioso
curaciones. Los cloromas son masas slidas de mieloblastos leucmi- central, son factores pronsticos desfavorables.
cos, que se pueden ver tanto en leucemias agudas mieloides como en
fase blstica de leucemia mieloide crnica. Pregunta 53.- R: 1
La clnica de las leucemias agudas se caracteriza por dos grandes
Pregunta 50.- R: 2 grupos de manifestaciones. El primero de ellos corresponde a la exis-
Las leucemias linfoides son caractersticas de las edades extremas tencia de escasas clulas maduras en sangre perifrica en las tres series
de la vida, siendo la forma aguda propia de nios y la forma crnica hematolgicas con los consiguientes: sndrome anmico, hemorra-
de ancianos. En la edad intermedia de la vida son ms frecuentes las gias por trombopenia e infecciones por neutropenia grave. El segundo
leucemias mieloides que las linfoides, predominando las formas agu- grupo de manifestaciones corresponde a la infiltracin por blastos de
das sobre la crnica. diferentes rganos y tejidos. Aunque puede haber afectacin de todo
tipo de rganos y tejidos en las leucemias agudas linfoides es tpica la
Pregunta 51.- R: 1 afectacin de rganos linfticos (adenopatas, esplenomegalia, masa
Aunque existen diversos factores pronsticos en pacientes con mediastnica por crecimiento del timo) y la infiltracin del sistema
leucemia aguda mieloide, edad avanzada, alteraciones citogenticas nervioso y en nios, el testculo. En leucemias agudas mieloides, los
desfavorables, existencia de leucemias agudas secundarias, suelen ser monoblastos caractersticamente infiltran sistema nervioso, piel o en-
los factores de peor pronstico. cas. La caracterstica clnica de la CID es tpica fundamentalmente en
la variedad M3.
Pregunta 51. Factores de mal pronstico de las leucemias agudas. Pregunta 54.- R: 4
El rgimen de induccin a la remisin en la leucemia aguda mie-
123  1 loide suele estar constituido por arabinsido de citosina y derivados
456789
6 de la adriamicina. Los regmenes que incluyen vincristina, prednisona
y derivados de la adriamicina son caractersticos del tratamiento de
2 123456786934646
8 induccin de la leucemia aguda linfoblstica. Mercaptopurina y me-
456
 totrexate son frmacos empleados en el tratamiento de mantenimien-
6 9524 9524 to de la leucemia aguda linfoblstica.
 6
69 9

8 Pregunta 55.- R: 4
Los derivados del cido retinoico (tetrinona) son frmacos que
624 242 9 6 6 6!4"454"956#6$6%

67
2 66 6
facilitan la maduracin de clulas tumorales en la leucemia aguda
promieloctica (M3). Este frmaco, asociado a quimioterapia, produ-
688 ce altos porcentajes de remisin en la leucemia aguda mieloide M3
6 5 296 6! "252& con la translocacin 15;17.

569 4" 96 665 "995 6 ) 92&6 6*1+
6
'6 6(95456 665 6,  "2965  9!29 Pregunta 56.- R: 5
6!99"2 4 La leucemia linftica crnica es un proceso neoplsico de linfoci-
tos de inmunofenotipo B y apariencia madura, que es el equivalente
leucmico del linfoma linfoctico bien diferenciado. Es una tipologa
Dada la extraordinaria agresividad de las leucemias agudas, con relativamente frecuente en pacientes sobre todo de edad avanzada. A
la consiguiente velocidad evolutiva, la nica forma de conseguir que

Comentarios TEST
pesar de que las clulas tumorales forman inmunoglobulina, en la
un paciente tenga una prolongacin en su supervivencia es que mayora de los casos esta inmunoglubulina queda en la membrana
exista remisin completa con tratamiento. No obstante, la remisin celular y no es segregada a la sangre, por lo que la mayora de los
completa no indica curacin, dado que estos pacientes pueden pacientes no presentan paraprotena. La enfermedad es de origen
tener recidivas. idioptico.

Pregunta 59. Clasificacin de Rye de la enfermedad de Hodgkin

123456789
3 893 3
2 238455 23 9 238459
16 5859 123454673894
46 894
46  74  

6
748
74 6

849
7525 7 4
6745946 
67666 
67666
748
7864 776
67666
776
67666
878939 2
5 
 6 !44 745
"45# 56

5
7$
67 47"56675 4
7
8 2672546 77%&7'() 7!48
7
7+ $ 77 474575
6
7!4 9764547
7767 6# 7*5424
46754 96

494 6 67, 6
454  7 64
 

M exico A rgentina CTO Medicina C/ Nez de Balboa, 115 28006 MADRID (Espaa) Tfno.: (91) 782 43 32 / Fax: (91) 782 43 27
Pg. 6 HM C hile U ruguay E-mail: secretaria@ctomedicina.com; iberocto@ctomedicina.com WEB: www.ctomedicina.com; www.iberocto.com
HEMATOLOGA
Preparacin Examen de Seleccin 05/06 1 Vuelta Seguimiento a distancia
Pregunta 57.- R: 5 respuesta 5, aun recibiendo radioterapia como nico tratamiento no
Los sistemas de estadificacin de la leucemia linftica crnica se necesitase laparotoma, se debe a la peculiar forma de diseminacin
basan en la historia natural de la evolucin de la enfermedad. Dado por va linftica de la enfermedad de Hodgkin, que sigue un orden
que es una enfermedad de nacimiento medular y de escasa agresivi- estricto en la afectacin de territorios linfticos. En dicho paciente, la
dad, su fase inicial corresponde a existencia tumoral en mdula sea existencia de enfermedad aparentemente slo en el cuello hace alta-
y sangre perifrica, siendo en esta fase los pacientes diagnosticados de mente improbable la presencia de enfermedad abdominal, pudiendo
forma casual al realizar hemogramas, donde aparece linfocitosis de evitar la laparotoma en esta circunstancia.
apariencia madura. En fases ms avanzadas se produce crecimiento
de organomegalias (adenopatas, esplenomegalia, hepatomegalia) y Pregunta 63.- R: 5
en fases avanzadas, como consecuencia del proceso mieloptsico, la Como resumen teraputico: la radioterapia como nico tratamiento
aparicin de anemia y trombopenia, que en el sistema de Rai implica se puede aplicar en estadios localizados sin sntomas B ni masa volu-
la fase ms avanzada de la enfermedad. Hay que recordar, sin embar- minosa. Para todos los estadios es posible la administracin de qui-
go, que la leucemia linftica crnica puede producir anemia y/o mioterapia, independientemente de que presenten sntomas B o no,
trombopenia de mecanismo inmunolgico. Cuando la anemia o la si bien la existencia de masa voluminosa aconseja tratamiento combi-
trombopenia son de dicho mecanismo, este hecho no est necesaria- nado.
mente ligado a la masa tumoral.
Pregunta 64.- R: 2
Pregunta 58.- R: 4 El protocolo MOPP es leucemognico, y entre otras complicacio-
La tricoleucemia o leucemia de clulas peludas es una neoplasia de nes, produce elevados ndices de esterilidad definitiva, por lo que no es
clulas linfoides de inmunofenotipo B casi siempre. Clnicamente se un tratamiento aconsejable en pacientes jvenes. Por otra parte, el pro-
caracteriza por la existencia de pancitopenia y esplenomegalia. Las tocolo ABVD se considera en la actualidad como el protocolo de qui-
clulas tumorales presentan una caracterstica citoqumica (fosfatasa mioterapia de referencia para esta enfermedad. Este ltimo tratamiento
cida resistente a tartrato) adems de las caractersticas prolongaciones puede producir como complicaciones cardiotoxicidad por la
de la membrana citoplasmtica. La tricoleucemia presenta buena res- adriamicina o patologa pulmonar por la bleomicina, como dicen las
puesta a diversos frmacos, fundamentalmente la cladribina. respuestas 1 y 4. La mielitis transversa es un efecto de la radioterapia.

Pregunta 59.- R: 1 Pregunta 65.- R: 2


La clula de tipo Sternberg es caracterstica de la enfermedad de La enfermedad de Hodgkin se caracteriza por producir inmuno-
Hodgkin, y en la actualidad se considera que es una clula linfoide deficiencia celular ms que humoral. La edad avanzada, lentitud en
activada. Morfolgicamente son clulas de gran tamao con dos o la respuesta teraputica y la hipoalbuminemia son caractersticas
mas ncleos y nuclolos muy prominentes. Esta imagen morfolgica oncolgicas generales de mal pronstico. El aumento de la velocidad
puede aparecer en otras patologas distintas de la enfermedad de de sedimentacin globular es un parmetro sensible para definir acti-
Hodgkin, como son linfomas no Hodgkinianos o procesos linfoproli- vidad en la enfermedad de Hodgkin.
ferativos reactivos, por ejemplo en el seno de viriasis. Dependiendo
de la proporcin de estas clulas malignas en relacin con clulas
reactivas acompaantes en las biopsias tisulares, se reconocen cuatro Pregunta 65. Factores de mal pronstico de la enfermedad de
histologas en la enfermedad de Hodgkin, de las que la esclerosis Hodgkin.
nodular se caracteriza por clulas llamadas lacunares. El predominio
linfocitario y esclerosis nodular se han considerado histologas favora- 1234567879
2 5 7 8 2272
bles, y la celularidad mixta y deplecin linfocitaria, histologas desfa- 128648 2  42 6 8464
vorables. 12 2 !8752 58"4 5
12#!8$%
Pregunta 60.- R: 2 12&
67' 52#
La existencia de adenopatas cervicales, axilares, mediastnicas y 12()2 *75
celacas, es decir, a ambos lados del diafragma, define un estadio clnico 12& 72' 5!847
III, con la letra A por la ausencia de sntomas B y el subndice 1, dado 12347 8!'4 '4
Comentarios TEST

que la localizacin adenoptica abdominal est en la zona alta del 122+&,


abdomen. La letra S, inicial de la palabra bazo en ingls (Spleen) se 12 8 2 5! 56 2 826 6 '4 67
coloca cuando existe esplenomegalia y se traduce en una mayor posi-
bilidad de diseminacin hematgena de la enfermedad. Un estadio IV
tambin se traduce en enfermedad diseminada hematgenamente. Pregunta 66.- R: 3
El virus de EPSTEIN-BARR se ha visto implicado en la patogenia de
Pregunta 61.- R: 5 diferentes linfomas, como es el tipo Burkitt tanto endmico como no
Para la mayor parte de las neoplasias malignas, la existencia de endmico, los linfomas de los inmunodeprimidos (SIDA y postras-
diseminacin tumoral implica la imposibilidad de curacin. Esta nor- plantados) y enfermedad de Hodgkin. El linfoma o leucemia T del
ma oncolgica no es aplicable a tumores hematolgicos como adulto est relacionado con otro virus, el retrovirus HTLV-I.
leucemias y linfomas, enfermedades que, estando extendidas, pue-
den ser curadas. Pregunta 67.- R: 4
Los linfomas no hodgkinianos de alto grado de agresividad se ca-
Pregunta 62.- R: 4 racterizan citolgicamente por presentar clulas de gran tamao o
La laparotoma como tcnica de estadificacin de la enfermedad clulas inmaduras. Las clulas inmaduras o precursoras se denomi-
de Hodgkin se realiza cada vez con menor frecuencia y est ligada a la nan obviamente blastos (linfoblastos) y algunos esquemas de clasifica-
necesidad de asegurar la ausencia o presencia de enfermedad abdo- cin de linfomas denominan tambin blastos a las clulas grandes.
minal en pacientes que podran ser tratados con radioterapia (estadio Como consecuencia se pueden considerar linfomas de alta agresivi-
IA y IIA sin masa voluminosa). Dichos estadios pueden tambin tratarse dad todos aquellos que terminan en la denominacin blstico, es
con quimioterapia, evitndose en este caso la laparotoma, situacin decir, presentan clulas precursoras o clulas de gran tamao. La
que es la habitual en la actualidad. No se aconseja utilizar radioterapia excepcin es un peculiar linfoma que presenta dos clulas tumorales,
como nico tratamiento en enfermedad diseminada (respuestas 1, 2 y una clula de ncleo hundido y otra clula grande, que en esta clasi-
3). Los pacientes de las respuestas 4 y 5 podran ser tratados con radio- ficacin recibe el nombre de linfoma centroctico-centroblstico y es
terapia o con quimioterapia. Si se elige radioterapia, se necesitara de escasa agresividad. El termino inmunocitoma es equivalente al de
laparotoma slo en la respuesta 4. El hecho de que el paciente de la linfoma linfoplasmocitoide.

M exico A rgentina CTO Medicina C/ Nez de Balboa, 115 28006 MADRID (Espaa) Tfno.: (91) 782 43 32 / Fax: (91) 782 43 27
C hile U ruguay E-mail: secretaria@ctomedicina.com; iberocto@ctomedicina.com WEB: www.ctomedicina.com; www.iberocto.com
HM Pg. 7
HEMATOLOGA
Seguimiento a distancia Preparacin Examen de Seleccin 05/06 1 Vuelta
Pregunta 68.- R: 2 Pregunta 74.- R: 3
A diferencia de los linfomas, donde las adenopatas suelen ser las Los linfomas no hodgkinianos de escasa agresividad presentan
manifestaciones clnicas ms frecuentes, en el mieloma mltiple las fases iniciales asintomticas y en estas fases asintomticas los linfo-
manifestaciones clnicas predominantes corresponden al hueso. De mas de escasa agresividad pueden no requerir tratamiento. A dife-
hecho, el mieloma mltiple forma parte de los procesos considerados rencia de la enfermedad de Hodgkin, que puede presentar enfer-
como tumores seos. Todo paciente hematolgico con dolores seos medad localizada fundamentalmente supradiafragmtica, los linfo-
o lesiones osteolticas, en preguntas de MIR, debemos considerar que mas no hodgkinianos de baja agresividad se presentan ocasional-
presenta un mieloma hasta que se demuestre lo contrario. mente en el retroperitoneo. A diferencia de los linfomas altamente
Hipercalcemia, anemia, proteinuria, insuficiencia renal, infecciones agresivos no suelen invadir el sistema nervioso central y son tumores
fundamentalmente respiratorias y urinarias son otras manifestaciones de pacientes adultos. En estos linfomas, la proporcin de clulas
tpicas del mieloma, asociadas estas ltimas a la inmunodeficiencia tumorales en fase proliferativa es escasa, por lo que la respuesta a
humoral. los tratamientos es menor que en los linfomas de alta agresividad.

Pregunta 69.- R: 4
A diferencia del sistema de estadificacin de ANN-ARBOR de los Pregunta 74. Diferencias entre linfomas por su agresividad.
linfomas, el sistema de Durie-Salmon del mieloma mltiple no esta-
blece extensin tumoral, sino masa. La existencia de una intensa 123456728948

4567
hipercalcemia es lgicamente propia de mielomas de alta masa tu- 12 872567 
moral (estadio III). El mieloma en estadio I ha recibido ocasional- 12
988
528492452
64 2
56
87
mente el nombre de mieloma quiescente y no suele precisar trata- 12345676 12 6
7257
miento.
689
6 1249
458
2927
12 82898
5
Pregunta 70.- R: 1
Todos los parmetros bioqumicos citados en las respuestas estn 12 96
456724549 45642
6
72 757
relacionados con la masa tumoral, pero el que mejor lo hace es la 22727
2! 
beta-2-microglobulina srica. Dicha sustancia no es un marcador pa-
tognomnico de la enfermedad, dado que se encuentra tambin ele- 12"
728948

45672  23#$2447

vada en otros procesos linfoproliferativos e incluso situaciones distin- 12567 29484564


tas como el SIDA o la insuficiencia renal. Por dicho motivo no vale 12#
4
58
5287 72 6 24252898
57 
para asegurar un diagnstico de mieloma mltiple, pero una vez he-
123466
229 24%69
56
87
cho el diagnstico de la enfermedad s sirve para valorar masa tumoral, 689
6
1249
458
2879624 5272 44
existiendo un sistema de estadificacin exclusivamente basado en el 12&452944622!
valor de dicha sustancia en suero.
12'7
2! 2$(')22*+('1&

Pregunta 71.- R: 3
A diferencia del mieloma mltiple, que como su nombre indica es Pregunta 75.- R: 3
un tumor diseminado que afecta a la mdula sea hemopoytica de Todos los procesos linfoproliferativos malignos de escasa agresivi-
todos los huesos, y que por tanto, si precisa tratamiento, ste debe ser dad pueden, con el paso del tiempo, evolucionar a formas histolgi-
sistmico, el plasmocitoma suele ser un tumor solitario que cuando ca y clnicamente ms agresivas. Cuando esta situacin ocurre en la
precisa tratamiento puede ser de tipo local, como la radioterapia. leucemia linftica crnica, este hecho recibe el nombre de sndro-
Melfaln o ciclofosfamida con prednisona suelen ser el tratamiento me de Richter, complicacin que, aunque tpica, es infrecuente.
clsico del mieloma mltiple que permite mantener en buen control
a muchos pacientes, a pesar de no ser tratamientos curativos. Esteroi- Pregunta 76.- R: 3
des, interfern, poliquimioterapia, son otros tipos de tratamiento de Los linfomas tipo MALT aparecen en el tejido linfoide asociado a
la enfermedad, aunque el trasplante de progenitores hematopoyti- las mucosas, siendo el ms frecuente de localizacin gstrica. Dicho
cos es el nico tratamiento curativo. linfoma est asociado a la infeccin por Helicobacter pylori. Los linfo-
mas gstricos tipo MALT son tumores generalmente de escasa agresivi-

Comentarios TEST
Pregunta 72.- R: 1 dad que pueden desaparecer tras la erradicacin del Helicobacter.
La macroglobulinemia de Waldenstrm suele tener como base Como otros linfomas indolentes, pueden tener evolucin a linfoma
histolgica un linfoma de tipo linfoplasmocitoide, es decir una agresivo. El linfoma de las respuestas 1, 4 y 5 puede estar relacionado
neoplasia de escasa agresividad a diferencia del linfoma inmuno- con el EBV.
blstico citado en la respuesta 1. Es caracterstica de esta enferme-
dad la existencia de sndrome de hiperviscosidad en relacin con Pregunta 77.- R: 3
la paraprotena de tipo IgM plasmtica. En este sndrome se inclu- Existen muchos factores de importancia pronstica en linfomas no
yen trastornos neurolgicos, hemorragias y otras alteraciones cir- Hodgkinianos, y entre ellos se encuentran todas las respuestas de la
culatorias. La IgM monoclonal puede tener actividad de pregunta. De hecho, en general se considera que el inmunofenotipo
crioaglutinina. T en ciertas circunstancias puede tener peor pronstico que el B. Sin
embargo, el llamado ndice internacional de factores pronsticos para
Pregunta 73.- R: 2 linfomas no Hodgkinianos est basado en factores puramente clni-
Los linfomas no hodgkinianos de alta agresividad pueden tener cos, como la edad, el estado de rendimiento, el estadio de la enferme-
inmunofenotipo B o T, aunque ms frecuentemente son B en nues- dad, la afeccin extraganglionar y como parmetros analticos la LDH
tro medio. Al tratarse de tumores altamente agresivos presentan srica como medida de masa tumoral. En dicho sistema de factores
corta supervivencia sin tratamiento, aunque con tratamientos qui- pronsticos no aparecen datos relacionados con citogentica ni ca-
mioterpicos agresivos o trasplante de progenitores hematopoyticos ractersticas inmunofenotpicas.
se pueden producir curaciones en muchos casos. La laparotoma
no es una tcnica de estadificacin de linfomas no hodgkinianos. Pregunta 78.- R: 2
No debe confundirse la laparotoma de estadificacin con la Los linfomas asociados al SIDA son linfomas no Hodgkinianos y
laparotoma diagnstica, que ocasionalmente es necesaria para dentro de ellos predominantemente de clulas B y de alta agresividad.
conocer la enfermedad que un paciente con una masa abdominal Como linfomas tpicos se encuentran los de clula pequea
presenta. (linfoblsticos) de tipo Burkitt o no Burkitt, de clula grande (como el
linfoma cerebral primario) y los linfomas de cavidades o serosas. Es

M exico A rgentina CTO Medicina C/ Nez de Balboa, 115 28006 MADRID (Espaa) Tfno.: (91) 782 43 32 / Fax: (91) 782 43 27
Pg. 8 HM C hile U ruguay E-mail: secretaria@ctomedicina.com; iberocto@ctomedicina.com WEB: www.ctomedicina.com; www.iberocto.com
HEMATOLOGA
Preparacin Examen de Seleccin 05/06 1 Vuelta Seguimiento a distancia

Pregunta 80. Hemostasia secundaria.


Comentarios TEST

frecuente la afectacin extralinftica y la diseminacin en el momen- el tiempo de tromboplastina la va intrnseca. El tratamiento anticoa-
to del diagnstico. Con frecuencia se encuentran asociados al EBV. gulante oral se controla fundamentalmente con el tiempo de pro-
trombina, no con el de tromboplastina, a diferencia de la heparina
Pregunta 79.- R: 2 intravenosa.
La leucemia-linfoma T del adulto se caracteriza por presentar tanto
enfermedad sangunea (leucemia) como tisular (linfoma). Suele ser un Pregunta 82.- R: 5
tumor de alta agresividad caracterstico de zonas geogrficas asociadas Para que el tiempo de protrombina y el de tromboplastina sean
al retrovirus HTLV-I, como es el caso de Japn o Caribe,y presenta con normales no debe haber alteraciones en los factores especficos de la
relativa frecuencia caractersticas que se ven frecuentemente en los va extrnseca y de la va intrnseca, y tampoco en los factores comu-
mielomas, como es el caso de la hipercalcemia o las lesiones seas. nes a ambas vas, es decir factor X, factor V, protrombina y fibringeno.
Por tanto, para cumplir las caractersticas del enunciado de la pregun-
Pregunta 80.- R: 2 ta, la alteracin debe corresponder a un factor que no est incluido
Los factores de coagulacin dependientes de la vitamina K de en ninguna de las vas, como es el factor XIII, un factor que no acta
sntesis heptica son la protombina, factor II, y los factores VII, IX y antes de la formacin de la fibrina, sino despus, para impedir su
X (ver esquema de pgina siguiente). degradacin. Si queremos conocer el grado de funcionamiento de
dicho factor, no debemos solicitar tiempos de coagulacin, sino tiem-
Pregunta 81.- R: 4 po de lisis del cogulo, que se acorta cuando este factor no funciona
El tiempo de hemorragia es una tcnica de estudio de hemostasia adecuadamente o est en cantidad insuficiente.
primaria, alterndose en patologas de la plaqueta o del vaso sangu-
neo. La patologa primaria de la hemostasia primaria ms frecuente es Pregunta 83.- R: 1
la trombopenia. En ausencia de trombopenia y de frmacos que in- Las trombopenias pueden clasificarse en dos tipos segn su meca-
terfieran con la hemostasia primaria, hay que considerar la enferme- nismo: trombopenias centrales, cuando la medula sea no produce
dad de Von Willebrand como causa ms probable. El tiempo de suficiente cantidad de plaquetas, donde estn patologas como la
protrombina valora bsicamente la va extrnseca de la coagulacin y aplasia, sndromes mielodisplsicos o leucemias agudas y trombope-

M exico A rgentina CTO Medicina C/ Nez de Balboa, 115 28006 MADRID (Espaa) Tfno.: (91) 782 43 32 / Fax: (91) 782 43 27
C hile U ruguay E-mail: secretaria@ctomedicina.com; iberocto@ctomedicina.com WEB: www.ctomedicina.com; www.iberocto.com
HM Pg. 9
HEMATOLOGA
Seguimiento a distancia Preparacin Examen de Seleccin 05/06 1 Vuelta
nias perifricas, donde la mdula sea es ajena a la trombopenia. A su Pregunta 89.- R: 4
vez, la trombopenia perifrica puede ser consecuencia de destruc- La trombosis vascular es una manifestacin tpica de ciertas enfer-
cin acelerada de plaquetas, como ocurre con las trombopenias medades clonales de la medula sea, como son la hemoglobinuria
autoinmunes o en el hiperesplenismo; secuestro plaquetario, como paroxstica nocturna, los sndromes mieloproliferativos crnicos, fun-
ocurre en la esplenomegalia, o consumo plaquetario en situaciones damentalmente policitemia vera y trombocitosis esencial de algunas
de trombosis excesiva, como es la prpura trombtica trombocitop- leucemias agudas, fundamentalmente la LAM-M3. La enfermedad
nica (PTT) o la coagulacin intravascular diseminada (CID). de Moschcowitz es el nombre de la PTT. La enfermedad de Rendu-
Osler es la telangiectasia hemorrgica hereditaria, que se caracteriza,
Pregunta 84.- R: 3 como su nombre indica, por hemorragias y no por trombosis.
La PTI es una trombopenia perifrica, por tanto con buen funciona-
miento medular, de mecanismo inmunolgico (mediado por anticuer- Pregunta 90.- R: 2
pos antiplaquetarios) cuyo diagnstico exige descartar otras causas de La CID se caracteriza por el consumo de plaquetas y de factores de
trombopenia como pueden ser frmacos, infecciones como el VIH, coagulacin para la produccin de cogulos de forma generalizada
trastornos autoinmunes o procesos linfoproliferativos. Aunque la des- en la microcirculacin. Como consecuencia de ello se produce
truccin plaquetaria en esta enfermedad tiene lugar en el bazo y la trombopenia y prolongacin de todos los tiempos de coagulacin.
esplenectoma es la forma de curacin definitiva en gran parte de los Asociado a este hecho aparece la destruccin traumtica de hemat-
pacientes, paradjicamente no se produce crecimiento esplnico. es, el consumo de los inhibidores de la coagulacin (por tanto, des-
censo de antitrombina III) y la aparicin de PDF consiguientes a la lisis
Pregunta 85.- R: 1 del cogulo por el sistema fibrinoltico.
La PTI crnica puede no producir sntomas si la cifra de plaquetas
no est muy disminuida. En dicha situacin se aconseja la abstencin Pregunta 91.- R: 4
teraputica y la vigilancia del paciente. En formas sintomticas el trata- Todas las alteraciones citadas en las respuestas son tpicas de la
miento inicial, como en otros procesos autoinmunes, suelen ser los CID. Dado el consumo plaquetario de factores de coagulacin, en la
esteroides, que permiten el control de parte de los pacientes. En CID se produce trombopenia y prolongacin de todos los tiempos de
pacientes que presenten recidivas tras el tratamiento esteroideo o que coagulacin, al mismo tiempo que disminucin de la cantidad de
necesiten prolongadamente esteroides, hay que valorar la utilizacin todos los factores de coagulacin y aparicin de PDF. La CID es una
de esplenectoma, que suele ser un tratamiento definitivo en la mayo- enfermedad bifsica en la que en la primera fase se producen
ra de ellos. Para pacientes que no mejoran con los anteriores trata- indiscriminadamente cogulos y en la fase final se produce sangrado
mientos se aconseja medicacin inmunosupresora. La gammaglobu- generalizado. Dicho sangrado tiene fundamentalmente relacin con
lina y la transfusin plaquetaria son tratamientos de emergencia para la desaparicin de los factores de coagulacin, principalmente fibri-
formas graves sintomticas. ngeno.

Pregunta 86.- R: 2 Pregunta 92.- R: 3


La PTT es una trombopenia perifrica por consumo plaqueta- Las denominadas trombofilias primarias hereditarias son trastor-
ria, en la que la disminucin de la cifra de plaquetas es consecuen- nos en el sistema de coagulacin-fibrinolisis que promueven un exce-
cia de la produccin de trombos generalizados en la microcircula- so de actividad coagulante por varios mecanismos. El primer mecanis-
cin, lo cual se suele acompaar de hemlisis de tipo traumtico mo puede ser la aparicin de factores de coagulacin en cantidad
con la consiguiente aparicin de esquistocitos. Entre las manifesta- excesiva o con caractersticas anormales (disfibrinogenemia, muta-
ciones clnicas predominan los trastornos isqumicos neurolgicos cin del gen de la protrombina y factor V Leiden). El segundo meca-
y la insuficiencia renal, pudiendo aparecer ocasionalmente fiebre. nismo puede ser la deficiencia de los inhibidores de la coagulacin
como son la deficiencia de antritrombina-III, de protena C o de
protena S. El tercer mecanismo es consecuencia de fallos en el siste-
Pregunta 86. Diagnstico diferencial entre PTT y SHU. ma fibrinoltico, bien por deficiencia en su actividad como conse-
cuencia de deficiencia de plasmingeno o por inhibicin de dicho
123 455 sistema por un exceso de inhibidores.
6787 12345 6789
45 Pregunta 93.- R: 4

Comentarios TEST
49
7  7
8
8  9 1 Las trombofilias primarias hereditarias son procesos congnitos,
generalmente autosmicos dominantes, que ocasionan tendencia
59  
 8 2  52
5 tromboemblica principalmente en pacientes adultos jvenes. Se

8  
9   8  14 debe sospechar ante situaciones de tromboembolismo de causa in-
cierta en adultos jvenes, o mujeres tras uso de anovulatorios orales
y su diagnstico suele hacerse tras la aparicin de clnica, ya que
Pregunta 87.- R:4 efectivamente no existen buenas pruebas de laboratorio para des-
Las cinco respuestas de la pregunta son formas de terapia de la PTT, pistaje.
aunque el tratamiento ms eficaz y por tanto de eleccin es la
plasmafresis. El motivo es que, en esta enfermedad, existen en el Pregunta 94.- R: 4
suero multmeros del factor Von Willebrand que favorecen la agresin En las cirrosis hepticas graves puede producirse un trastorno gene-
plaquetaria al endotelio vascular, como consecuencia de la falta de ralizado de la hemostasia. Por una parte puede haber trombopenia
degradacin del factor. como consecuencia de hiperesplenismo por hipertensin portal. Por
otra parte, y dado que el hgado sintetiza factores de coagulacin que
Pregunta 88.- R: 3 estn tanto en la va intrnseca como en la va extrnseca pueden
La enfermedad de Von Willebrand es la trombocitopata ms prolongarse el tiempo de protrombina y de tromboplastina. Adems
frecuente. Esta enfermedad puede transmitirse de forma autosmica la sntesis defectuosa de fibringeno puede producir prolongacin
dominante (tipos I y II) o recesiva (tipo III), siendo esta ltima forma la del tiempo de trombina. Dichas alteraciones son similares a las que
ms grave. Este factor se sintetiza en megacariocitos, endotelio vascu- aparecen en la CID diferencindose, entre otras cosas, en el hecho de
lar y lo presentan las plaquetas en sus grnulos. Como otros defectos que en la CID se produce un consumo de todos los factores de la
de la hemostasia primaria se caracteriza por hemorragias mucocut- coagulacin y en la hepatopata disminuyen los factores relacionados
neas. El acetato de desmopresina es un tratamiento que permite con la sntesis heptica, fundamentalmente dependientes de la vita-
incrementar la cantidad de factor, y es til bsicamente en el tipo I. mina K, fibringeno y factor V, mantenindose el factor VIII en valores
normales.

M exico A rgentina CTO Medicina C/ Nez de Balboa, 115 28006 MADRID (Espaa) Tfno.: (91) 782 43 32 / Fax: (91) 782 43 27
Pg. 10 HM C hile U ruguay E-mail: secretaria@ctomedicina.com; iberocto@ctomedicina.com WEB: www.ctomedicina.com; www.iberocto.com
HEMATOLOGA
Preparacin Examen de Seleccin 05/06 1 Vuelta Seguimiento a distancia
Pregunta 95.- R: 3
En la ictericia obstructiva severa se produce disminucin de aporte
de bilis al duodeno, con la consiguiente malabsorcin grasa. Dado
que la vitamina K se absorbe con las grasas, en estos pacientes se
puede producir una malabsorcin de vitamina K, con los consiguien-
tes problemas hemorrgicos, fundamentalmente tras tratamientos o
medidas diagnsticas invasivas. Para evitar estos problemas, se acon-
seja la administracin de vitamina K por va parenteral. La deficiencia
de cido flico puede ocasionalmente producir trombopenia, pero
la trombopenia produce un trastorno de la hemostasia primaria, no
de la coagulacin.

Pregunta 96.- R: 4
Las alteraciones de la hemostasia son caractersticas de la leucemia
promieloctica (CID aguda), las neoplasias diseminadas, fundamental-
mente adenocarcinomas (CID crnica), o situaciones de hiperviscosi-
dad srica como ocurre en el mieloma y en la enfermedad de
Waldenstrm.

Pregunta 97.- R: 4
El tratamiento con heparina intravenosa produce un efecto anti-
coagulante, prcticamente inmediato, dado que la heparina, a tra-
vs de la antitrombina-III, produce inhibicin de factores de coagu-
lacin ya existentes. Por contra, el tratamiento anticoagulante oral,
produce anticoagulacin diferida, dado que dicha medicacin no
produce inhibicin de la actividad de factores ya existentes sino de
la sntesis de nuevos factores. Por otra parte, el efecto de la heparina
es de breve duracin, por lo que la suspensin de la heparina pro-
voca la desaparicin del efecto anticoagulante en pocas horas, mien-
tras la anticoagulacin oral puede mantener su efecto varios das. En
la situacin que se establece en la pregunta, dado que el paciente
precisa estar anticoagulado por tener una prtesis valvular cardaca
la actitud ms correcta es la que se sugiere en la respuesta 4.

Pregunta 98.- R: 2
El efecto secundario ms frecuente de la medicacin anticoagu-
lante es el sangrado. Un efecto infrecuente, pero que hay que cono-
cer por su gravedad, es la trombocitopenia secundaria a la heparina.
Dicha situacin, paradjicamente, produce trombosis, y es motivada
por la aparicin de anticuerpos antiheparina que atacan la membra-
na plaquetaria, produciendo trombocitopenia inmunolgica y po-
niendo, por mecanismos no del todo conocidos en marcha la coagu-
lacin sangunea. Esta situacin exige la suspensin del tratamiento
con heparina y la administracin de medicacin anticoagulante
antignicamente no relacionada con la heparina.
Comentarios TEST

Pregunta 99.- R: 5
La necrosis cutnea por anticoagulantes orales, est mediada por
un fenmeno trombtico de la microcirculacin cutnea en relacin
con deficiencia de inhibidores de la coagulacin, como protena C o
protena S. La explicacin de este fenmeno paradjico tiene que ver
con el hecho de que la vitamina K es necesaria, no slo para la sntesis
de factores de coagulacin sino tambin para la sntesis de inhibidores
de coagulacin como los citados.

Pregunta 100.- R: 2
El INR es la forma habitual de control de medicacin anticoagu-
lante oral. Bsicamente consiste en una relacin entre el tiempo de
protrombina del paciente anticoagulado por va oral y el tiempo de
protrombina normal para el laboratorio, por lo que un INR de 1
implicara una situacin de no anticoagulacin. Cuanto mayor sea el
resultado numricamente del INR ms intensa es la actividad anticoa-
gulante. En la mayor parte de las situaciones, se precisa un INR en el
intervalo de 2-3 para mantener una adecuada anticoagulacin. Hay
situaciones de elevado riesgo procoagulante como la que el paciente
tiene, en donde el INR debera estar entre 3-4.

M exico A rgentina CTO Medicina C/ Nez de Balboa, 115 28006 MADRID (Espaa) Tfno.: (91) 782 43 32 / Fax: (91) 782 43 27
C hile U ruguay E-mail: secretaria@ctomedicina.com; iberocto@ctomedicina.com WEB: www.ctomedicina.com; www.iberocto.com
HM Pg. 11
INMUNOLOGA Y GENTICA
Preparacin Examen de Seleccin 05/06 1 Vuelta Seguimiento a distancia

1. La inmunoglobulina predominante en superficies mucosas y 2) La naturaleza dimrica de la IgA secretora.


secreciones externas en la superficie humana es: 3) Actividad antiproteasa del componente secretor.
4) Presencia de la cadena J.
1) La forma monomrica de la IgM, unida al componente S 5) Actividad antiproteasa de la cadena J.
(secretor).
2) La forma polimrica de la IgM, unida a la cadena J. 8. Seale cul de las siguientes anomalas es la causa de la forma
3) Un dmero IgA, unido a la cadena J y al componente S. adquirida del angioedema:
4) Un dmero IgA unido al componente S, sin cadena J.
5) IgA mono o polimrica, indistintamente, unida al compo- 1) C1 inhibidor deficiente cuantitativa o cualitativamente.
nente S. 2) Niveles bajos de IL-10.
3) Dficit de C1.
2. Para activarse el linfocito T CD4+, necesita la presentacin del 4) Deficiencia de IgE.
antgeno en el HLA de clase II de la clula presentadora de 5) Aumento de los niveles de IgE.
antgeno (CPA), y la coestimulacin de otras molculas como la
CD28. Seale cul es el ligando de sta en la CPA: 9. La respuesta de anticuerpos independiente de clulas T (timoin-
dependiente) es de un slo tipo de inmunoglobulina. Selela:
1) CD44.
2) CD7. 1) Ig G.
3) B7 (CD80). 2) Ig M.
4) ICAM-1. 3) Ig A.
5) LFA-2. 4) Ig D.
5) Ig E.
3. La secuencia por la cual las diferentes inmunoglobulinas alcan-
zan en los nios niveles iguales a los del adulto es: 10. Cul es el dficit de factores del complemento ms frecuente
en la poblacin mundial?:
1) IgA, IgG, IgM.
2) IgA, IgM, IgG. 1) C1.
3) IgM, IgA, IgG. 2) C2.
4) IgM, IgG, IgA. 3) C3.
5) IgG, IgM, IgA. 4) C5.
5) C9.
4. Cul de las siguientes clulas produce IgE?:
11. Seale cul de estos tipos celulares NO expresa molculas de
1) Eosinfilo. HLA DR:
2) Basfilo.
3) Linfocito T. 1) Osteoclasto.
4) Clula plasmtica. 2) Clulas de Kupffer.
5) Mastocito. 3) Clulas de Langerhans.
4) Linfocitos B.
5. El marcador ms caracterstico de los linfocitos T en el hombre 5) Neutrfilos.
es:
12. Cmo se denominan las inmunoglobulinas que precipitan a
1) Inmunoglobulina de superficie. 4C y se redisuelven a 37C?:
2) Marcador CD4.
3) Receptor para interleuquina 2. 1) Factor reumatoide.
Preguntas TEST

4) Antgeno HLA-DR. 2) Haptenos.


5) Marcador CD3. 3) Crioglobulinas.
4) Piroglobulinas.
6. La estimulacin antignica de un linfocito Th (helper) en ausen- 5) Epsilon.
cia de coestimulacin, provoca en el mismo:
13. Un nio de 4 aos presenta una historia de infecciones pigenas
1) Hipersecrecin de citoquinas. de repeticin cuya causa fue siempre bacterias encapsuladas
2) Hipoestimulacin. ricas en polisacridos. Se sospecha deficiencia de un tipo de
3) Anergia clonal. inmunoglobulina. Cul de las siguientes sera compatible con
4) Delecin clonal. ese cuadro?:
5) Supresin activa.
1) IgE.
7. La resistencia de la IgA secretora a la protelisis enzimtica es 2) IgG1.
consecuencia de: 3) IgG (todas las subclases).
4) IgG2.
1) La unin del componente secretor alrededor de la regin 5) IgA2.
bisagra.

M exico A rgentina
C hile U ruguay
CTO Medicina C/ Nez de Balboa, 115 28006 MADRID (Espaa) Tfno.: (91) 782 43 32 / Fax: (91) 782 43 27
E-mail: secretaria@ctomedicina.com; iberocto@ctomedicina.com WEB: www.ctomedicina.com; www.iberocto.com
IG Pg. 1
INMUNOLOGA Y GENTICA
Seguimiento a distancia Preparacin Examen de Seleccin 05/06 1 Vuelta
14. Cules son las molculas de membrana que definen el fenotipo 1) Se conocen tres tipos de mecanismo efector, denominados
de la poblacin mayoritaria de los linfocitos T cooperadores?: respectivamente Th1, Th2 y Th3.
2) Cada mecanismo helper lo realiza un tipo de clulas diferen-
1) TcR+, CD3-, CD4+. te, por lo que, en realidad, hay dos clases de linfocitos T: los
2) TcR-, CD3+, CD4-. CD4 y los CD8.
3) TcR+, CD3+, CD8+. 3) Los linfocitos Th1 potencian la actividad de los macrfagos.
4) TcR+, CD3+, CD4+. 4) Los Th2 potencian la actividad de los linfocitos B.
5) TcR-, CD3+, CD4+. 5) El tipo concreto de mecanismo efector T que intervenga en
la respuesta inmune ante una bacteria puede modificar el
15. La zona efectora de la molcula de IgG que media el proceso de cuadro clnico que sta produzca.
opsonizacin es:
21. Un nio de 15 meses es ingresado por un cuadro muy grave de
1) F(ab)2. sarampin varios das despus de ser vacunado. En la analtica
2) Fab. llama la atencin la ausencia de linfocitos T en sangre. Este
3) Fc. cuadro es muy sugerente de:
4) Regin bisagra.
5) Componente secretor. 1) Inmunodeficiencia variable comn.
2) Dficit de adenosindeaminasa (ADA).
16. Cul es el mecanismo de herencia de los alelos del complejo 3) Hipogammaglobulinemia transitoria de la infancia.
principal de histocompatibilidad?: 4) Inmunodeficiencia combinada ligada al cromosoma X.
5) Sndrome de Di George.
1) Ligado al X.
2) Autosmico dominante. 22. Una adolescente de 17 aos es ingresada por una neumona
3) Autosmico codominante. neumoccica severa. En la historia destaca que, pese a haber
4) Autosmico recesivo. tenido una infancia normal, en los ltimos 2 aos ha tenido
5) No mendeliano. cuadros de diarreas intermitentes, otitis supurada crnica y
cuatro neumonas. En la exploracin destaca una moderada
17. Un nio con inmunodeficiencia combinada severa es sometido esplenomegalia. Cul de los siguientes diagnsticos encaja
a un trasplante de mdula sea. Cul es la complicacin ms perfectamente con su cuadro clnico?:
importante que puede aparecer tras este tratamiento?:
1) Inmunodeficiencia combinada severa.
1) Regresin tmica precoz. 2) Dficit de adenosindeaminasa.
2) Retraso en la produccin de anticuerpos tras ser reinmuni- 3) Inmunodeficiencia variable comn.
zado. 4) Aplasia tmica (sndrome de DiGeorge).
3) Reaccin de injerto contra husped. 5) Sndrome de Wiskott-Aldrich.
4) Rechazo de tejido trasplantado.
5) Ausencia de clulas B maduras en la mdula sea trasplan- 23. En cul de los siguientes procesos es imprescindible el papel
tada. de la inmunidad humoral?:

18. Con respecto a la inmunodeficiencia variable comn, es FALSO 1) Eliminacin de clulas infectadas por virus.
que: 2) Bloqueo de toxinas bacterianas.
3) Bacterilisis.
1) Es un cuadro de inmunodeficiencia primaria por dficit de 4) Fagocitosis.
anticuerpos. 5) Estimulacin del receptor de la clula T.
2) Cursa con infecciones bacterianas de repeticin.

Preguntas TEST
3) Es tpico encontrar en el laboratorio panhipogammaglobu- 24. Seale cul de las siguientes alteraciones analticas se observa
linemia. de modo caracterstico en los pacientes con sndrome de
4) Debuta en el perodo neonatal inmediato. Bruton:
5) Puede asociarse a neoplasias, sobre todo a linfomas o a
cncer gstrico. 1) Neutropenia marcada.
2) Ausencia de IgG.
19. Una caracterstica del sndrome de DiGeorge es: 3) Niveles muy bajos de IgM.
4) Linfocitos B indetectables.
1) Anomalas del desarrollo de la 1 y 2 bolsas farngeas. 5) Linfocitos T indetectables.
2) Hiperplasia de las glndulas paratiroideas.
3) Hipercalcemia. 25. Slo con una de las siguientes molculas es posible la presen-
4) Anomalas cardiovasculares. tacin del antgeno, por parte de los macrfagos, a los linfocitos
5) Evidencia clara de herencia. T helper 2 CD4+. Indquela:

20. Indique la afirmacin FALSA acerca de los linfocitos T colabo- 1) Inmunoglobulina de superficie.
radores (T helper): 2) HLA-A
3) HLA- DR.

M exico A rgentina
Pg. 2 IG C hile U ruguay
CTO Medicina C/ Nez de Balboa, 115 28006 MADRID (Espaa) Tfno.: (91) 782 43 32 / Fax: (91) 782 43 27
E-mail: secretaria@ctomedicina.com; iberocto@ctomedicina.com WEB: www.ctomedicina.com; www.iberocto.com
INMUNOLOGA Y GENTICA
Preparacin Examen de Seleccin 05/06 1 Vuelta Seguimiento a distancia

4) HLA-C. 32. En cul de los siguientes trasplantes es MENOS probable que


5) Receptor de interleuquina 2. aparezca enfermedad injerto contra husped?:

26. Cul de los siguientes NO es un rgano linfoide secundario?: 1) Mdula.


2) Rin.
1) Bazo. 3) Corazn.
2) Timo. 4) Crnea.
3) Amgdala palatina. 5) Hgado.
4) Placas de Peyer.
5) Ganglios linfticos. 33. Cul es el defecto citogentico que suelen presentar los
pacientes con sndrome de Di George?:
27. A las molculas que son incapaces de inducir una respuesta
inmune, a menos que se asocien a otra molcula portadora, se 1) Trisoma X.
les denomina: 2) Trisoma 13.
3) Microdelecin en el brazo largo del cromosoma X.
1) Alergenos. 4) Microdelecin en el brazo largo del cromosoma 5.
2) Paratopos. 5) Microdelecin en el brazo largo del cromosoma 22.
3) Haptenos.
4) Haplotipos. 34. Una reaccin de hipersensibilidad retardada a un antgeno se
5) Idiotipos. caracteriza por:

28. Un nio de 7 meses con historia de infecciones pigenas fue 1) Aparecer en las primeras 24 horas.
hospitalizado por una infeccin severa por hongos. El paciente 2) Un rea de induracin de al menos 10 mm de dimetro a las
tena ausencia de sombra tmica en Rx, hipogammaglobuline- 48-72 horas.
mia y falta de linfocitos B. Esta historia sugiere: 3) Una zona eritematosa de al menos 10 mm de dimetro a las
48-72 horas.
1) Sndrome de inmunodeficiencia adquirida. 4) Estar mediada por complejos antgeno-anticuerpo.
2) Mieloma mltiple. 5) Estar mediada por anticuerpos de la clase IgE.
3) Enfermedad granulomatosa crnica.
4) Hipogammaglobulinemia ligada al sexo. 35. Para distinguir entre linfocitos B, linfocitos T y macrfagos se
5) Inmunodeficiencia combinada severa. recurre a ciertas propiedades de cada poblacin celular. Cul
de las siguientes es FALSA?:
29. Cul de las siguientes inmunodeficiencias enumeradas a con-
tinuacin presenta mejor pronstico?: 1) Los linfocitos T son CD3+, los B y macrfagos no.
2) Los linfocitos B tienen Ig de superficie, los T no.
1) Disgenesia reticular. 3) Los macrfagos y algunos linfocitos T son CD4+.
2) Deficiencia de HLA de clase II. 4) Los macrfagos tienen inmunoglobulina de superficie.
3) Inmunodeficiencia combinada severa ligada a X. 5) Los T se estimulan con concanavalina A y los B no.
4) Inmunodeficiencia combinada severa.
5) Inmunodeficiencia variable comn.
GENTICA
30. El tratamiento de la molcula de IgG con papana rompe la
molcula en: 36 En cul de las siguientes enfermedades se ha observado
heterogeneidad de locus?:
1) Un fragmento F(ab)2.
Preguntas TEST

2) Dos fragmentos Fab y un fragmento Fc. 1) Fibrosis qustica.


3) Dos fragmentos Fd. 2) Inmunodeficiencia combinada severa.
4) Un fragmento F(ab)2 y un fragmento Fc. 3) Fenilcetonuria.
5) Un fragmento Fab y un fragmento Fc. 4) Corea de Huntington.
5) Distrofia de Duchenne.
31. Antes de realizar un trasplante renal se realiza la prueba
cruzada donante-receptor, siendo esta positiva. Seale cul 37. En una ecografa de rutina se diagnostica a un paciente poliquis-
debe ser la actitud a tomar: tosis renal del adulto. El paciente es varn y tiene cuatro hijos,
tambin todos ellos varones; estadsticamente, cuntos de
1) Proceder a realizar normalmente el trasplante. ellos sera lgico que sufriesen la enfermedad?:
2) La prueba cruzada positiva invalida la posibilidad de trasplan-
te, por lo que hay que buscar otro receptor para ese rgano. 1) 0.
3) Administracin de un bolo suplementario de corticoides 2) 1.
inmediatamente despus de la reperfusin del rgano. 3) 2.
4) Plasmafresis pre-trasplante. 4) 3.
5) Realizar la inmunosupresin de base con Tacrolimus (FK- 5) 4.
506) en vez de ciclosporina.

M exico A rgentina
C hile U ruguay
CTO Medicina C/ Nez de Balboa, 115 28006 MADRID (Espaa) Tfno.: (91) 782 43 32 / Fax: (91) 782 43 27
E-mail: secretaria@ctomedicina.com; iberocto@ctomedicina.com WEB: www.ctomedicina.com; www.iberocto.com
IG Pg. 3
INMUNOLOGA Y GENTICA
Seguimiento a distancia Preparacin Examen de Seleccin 05/06 1 Vuelta
38. A un segmento de ADN cromosmico que contiene toda la 4) Sndrome de Prader-Willi.
informacin necesaria para sintetizar una cadena polipeptdica 5) Linfoma de Burkitt.
se le denomina:
44. En el gen responsable de la fibrosis qustica pueden aparecer
1) Opern. diversas mutaciones diferentes que van traducindose todas
2) Gen. ellas en un mismo fenotipo de enfermedad. A esta situacin se
3) Intrn. la denomina:
4) Exn.
5) Trasposn. 1) Heterogeneidad clnica.
2) Heterogeneidad de locus.
39. Seale la proposicin correcta acerca del patrn de herencia 3) Heterogeneidad allica.
mitocondrial: 4) Imprinting gnico.
5) Fenocopia.
1) La mitad de los hijos varones de una mujer afecta sufrirn la
enfermedad. 45. Cul de los siguientes patrones de herencia de enfermedad es
2) Todas las hijas de un varn afecto son portadoras de la el ms frecuente en humanos?:
enfermedad.
3) Las mujeres afectas solo trasmiten la enfermedad a sus hijas. 1) Autosmica dominante.
4) Ninguno de los hijos de un varn afecto sufrir la enfermedad. 2) Autosmica recesiva.
5) Tanto los varones como las mujeres trasmiten la enfermedad 3) Ligada al X.
a la totalidad de sus hijos con independencia de su sexo. 4) Ligada al Y.
5) Mitocondrial.
40. Un nio fallece a las pocas horas del nacimiento por una crisis
de apnea; en la exploracin realizada en el momento de nacer, 46. Un carcter est ligado totalmente al sexo si su locus gnico est
destaca labio leporino bilateral, polidactilia, microftalma y en:
dextrocardia. Cul de los siguientes sndromes encaja con el
fenotipo de este nio?: 1) Slo en el cromosoma X.
2) Slo en el cromosoma Y.
1) Sndrome de Edwards (trisoma 18). 3) En el segmento diferencial del X solamente.
2) Sndrome de ojo de gato (trisoma parcial 22). 4) Un cromosoma localizado muy prximo a los cromosomas
3) Sndrome de Patau (trisoma 13). sexuales.
4) Sndrome de Down (trisoma 21). 5) En el segmento diferencial de los cromosomas sexuales.
5) Sndrome del maullido de gato (monosoma parcial del
cromosoma 5). 47. La inactivacin de un cromosoma X en la mujer:

41. Cul es la aneuploida que se observa ms frecuentemente en 1) Es un fenmeno que se produce en los primeros estadios de
la especie humana?: la gametognesis.
2) La glucosilacin del ADN desempea un importante papel
1) Sndrome de Down. en dicha inactivacin.
2) Trisoma 16. 3) Se considera un mecanismo de compensacin de la dosis
3) Trisoma 13. gnica.
4) Trisoma X. 4) Es visible citolgicamente en el ncleo de las clulas feme-
5) Monosoma X. ninas como cuerpo heterocromtico o corpsculo de Ta-
ylor.
42. Seale la afirmacin cierta acerca de la traslocacin robertso- 5) La inactivacin se produce al azar, por lo que todas las clulas

Preguntas TEST
niana: tienen inactivados el mismo cromosoma X, bien el de origen
paterno o el de origen materno.
1) No origina patologas.
2) Es un fenmeno que se produce por la fusin de dos 48. Indique la relacin correcta entre las siguientes enfermedades
cromosomas acrocntricos. y sus mecanismos de herencia:
3) La traslocacin afecta a los brazos largos de dos cromosomas.
4) Un ejemplo es la alteracin cromosmica del sndrome del 1) Neuropata ptica de Leber: ligada al X recesiva.
maullido de gato. 2) Raquitismo resistente a vitamina D: ligada al X recesiva.
5) Aunque se la denomine as, no es una traslocacin sino una 3) Enfermedad de Huntington: autosmica recesiva.
duplicacin de un segmento gnico. 4) Enfermedad de Hunter: autosmica recesiva.
5) Sndrome del cromosoma X frgil: expansin de secuencias.
43. Cul de las siguientes patologas puede originarse por la
presencia de una disoma uniparental?: 49. Seale la respuesta FALSA acerca de las traslocaciones Robert-
sonianas:
1) Talasemia.
2) Corea de Huntington. 1) Se trata de la fusin de dos cromosomas acrocntricos.
3) Sndrome de Down. 2) Explican un porcentaje de casos de sndrome de Down.

M exico A rgentina
Pg. 4 IG C hile U ruguay
CTO Medicina C/ Nez de Balboa, 115 28006 MADRID (Espaa) Tfno.: (91) 782 43 32 / Fax: (91) 782 43 27
E-mail: secretaria@ctomedicina.com; iberocto@ctomedicina.com WEB: www.ctomedicina.com; www.iberocto.com
INMUNOLOGA Y GENTICA
Preparacin Examen de Seleccin 05/06 1 Vuelta Seguimiento a distancia

3) Al ser balanceadas, no originan alteraciones en la descenden-


cia.
4) Son importantes para realizar consejo gentico.
5) Influyen en el riesgo de recurrencia de cromosomopatas.

50. En un paciente afecto de una enfermedad de Marfan, cul es


la probabilidad de que al menos uno de sus progenitores tenga
la enfermedad?:

1) Prcticamente un 100%.
2) 50%.
3) Los dos padres deben de ser portadores asintomticos.
4) La probabilidad es baja, dada la alta incidencia de mutacio-
nes espontneas que aparecen en este tipo de herencia.
5) 25%.
Preguntas TEST

M exico A rgentina
C hile U ruguay
CTO Medicina C/ Nez de Balboa, 115 28006 MADRID (Espaa) Tfno.: (91) 782 43 32 / Fax: (91) 782 43 27
E-mail: secretaria@ctomedicina.com; iberocto@ctomedicina.com WEB: www.ctomedicina.com; www.iberocto.com
IG Pg. 5
INMUNOLOGA Y GENTICA
Preparacin Examen de Seleccin 05/06 1 Vuelta Seguimiento a distancia
Pregunta 1.- R: 3 Pregunta 3.- R: 4
La IgA es una IG ampliamente distribuida, la podemos encontrar La primera inmunoglobulina elaborada por el propio organismo
tanto en el suero como en las secreciones. Es la predominante en las es la IgM, se empieza a producir a partir de los 6 meses y medio de
secreciones externas: tubo digestivo y bilis, epitelios respiratorios, le- gestacin. La segunda en aparecer es la IgG, que lo hace ya despus
che y calostro, saliva, lgrimas, y flujo vaginal. del nacimiento, en torno a los 7-15 das de vida. IgA aparece ms
Aunque la IgA srica es mayoritariamente monomrica, la IgA de tardamente en torno al segundo mes de vida. Tngase en cuenta que
las secreciones es fundamentalmente dimrica, est formada por dos durante los primeros meses de vida la inmunoglobulina mayoritaria es
molculas de IgA unidas covalentemente (puente disulfuro) con la la IgG de origen materno.
cadena J y asociada a un polipptido conocido por componente
secretor (CS). Pregunta 4.- R: 4
El CS se sintetiza en las clulas epiteliales de la mucosa correspon- Las nicas clulas capaces de producir inmunoglobulinas son los
diente y se sita en el polo basal de la membrana celular, donde acta linfocitos B, y las clulas que derivan de la maduracin de los mismos
como un receptor para IgA dimrica sintetizada por las clulas B de la (plasmticas). Cuando leemos en una pregunta las siglas IgE, inmedia-
submucosa. La IgA se une al receptor e inmediatamente es endocitada tamente la asociamos con los cuadros clnicos de la alergia, al igual
y transportada por el interior de la clula (transcitosis). Cuando llega a que los eosinfilos, basfilos o clulas cebadas, pero estas clulas NO
la zona de la membrana celular de la luz del epitelio, el receptor se producen inmunoglobulinas.
escinde y queda libre la IgA unida a un fragmento del mismo (el CS).
La respuesta 1 es falsa porque la IgM no es estable en las secrecio- Pregunta 5.- R: 5
nes: se degrada inmediatamente. Adems la IgM humana soluble es Durante el proceso de maduracin de los linfocitos en el timo
pentamrica y slo predominan las formas monomricas en situacio- adquieren el receptor antignico especfico de la clula T que les va a
nes patolgicas, como la cirrosis biliar primaria. permitir identificar su antgeno. Varias molculas se asocian a dicho
La respuesta 2 es falsa porque aunque la cadena J se asocia a IgM, receptor fundamentalmente la denominada CD3 (presente en todos
como vimos en el comentario anterior, esta molcula prcticamente los linfocitos T).
no se ve en las secreciones. Adems de CD3 y el receptor antignico, los linfocitos T perifricos se
La respuesta 4 y 5 son falsas porque, como ya se ha comentado, la caracterizan por expresar las siguientes molculas de superficie: CD2 (la
IgA de secreciones es dimrica y para serlo necesita tener cadena J. responsable de la formacin de rosetas con hemates de carnero), CD5,
CD7 y adems uno de los siguientes (pero no los dos a la vez):
Pregunta 2.- R: 3 CD4: los linfocitos T CD4+ son los que reconocen antgenos pre-
El linfocito T slo puede reconocer su antgeno especfico si le es sentados junto con el HLA de clase II. La mayor parte de los CD4+
presentado por otra clula en el interior de una molcula del comple- desarrollan funciones colaboradoras (HELPER), aunque tambin
jo principal de histocompatibilidad (HLA). Cuando lo reconoce, se existen T CD4+ con actividad citotxica (el 10%).
establece un dilogo molecular entre ambas clulas: presentadora CD8: los linfocitos T CD8+ reconocen antgenos presentados jun-
de antgeno y linfocito T, que puede acabar en la activacin del linfo- to con el HLA de clase I. La mayora son citotxicos, pero tambin
cito o la anergia clonal. existen T8 colaboradores. Los CD4 predominan sobre los CD8 en
De todas las interacciones que tienen lugar en el proceso de activa- una relacin 2:1.
cin del linfocito T, la ms importante es la B7-CD28. Se produce La inmunoglobulina de superficie (Respuesta1) slo se expresa en
entre las molculas B7 (CD80, CD86) situadas en la clula presenta- los linfocitos B. EL HLA-DR (Respuesta 4) se expresa en monocitos-
dora de antgeno y la CD28 del linfocito T CD4+. Regla mnemotc- macrfagos y linfocitos B. En los T slo se expresa, por un perodo de
nica: CD4 x B7 = CD28 (4x7=28). tiempo muy corto, durante la activacin del linfocito.
Si se produce esta coestimulacin, que se comporta como una El receptor para interleuquina 2 (Respuesta 3) no es exclusivo de
segunda seal de activacin, el proceso de la respuesta inmune con- los linfocitos T.
tinua, pero si la interaccin no se lleva a cabo, por ejemplo, al no estar
presente la molcula B7 en la superficie de la clula presentadora, el Pregunta 6.- R: 3
linfocito T no slo no se activar, sino que acabar entrando en La teora de la segunda seal en la presentacin antignica a los
apoptosis y se desarrollar un estado de tolerancia a ese antgeno que linfocitos T es uno de los temas preferidos en el MIR en los ltimos
se denomina anergia clonal. aos, y por eso existen en este test varias preguntas con un cierto
Comentarios TEST

La nica respuesta que podra plantear alguna confusin es la 2 parecido.


(CD7), pero la interaccin B7-CD28 es tan importante (la han pregun- Recordad que el reconocimiento de un antgeno por parte de un
tado varias veces en el MIR) que slo con leerla debemos identificarla linfocito T necesita que le sea presentado por otra clula en el interior
a la primera. de una molcula del complejo principal de histocompatibilidad (HLA)
y adems otra serie de seales complementarias.
Si se produce coestimulacin, la segunda seal de activacin, el
proceso sigue su curso y el linfocito acabar desarrollando una res-
puesta inmune frente a ese antgeno. No obstante, si la segunda seal
est ausente, por ejemplo al no estar presente la molcula B7 en la
superficie de la clula presentadora, en vez de una respuesta inmune,
se desarrolla un estado de tolerancia a ese antgeno que se denomina
anergia clonal.
La delecin clonal es el proceso que tiene lugar en el timo para
eliminar los linfocitos autorreactivos.

Pregunta 7.- R: 1
La nica inmunoglobulina que es capaz de mantenerse estable en
las secreciones externas es la IgA debido a su resistencia a la degrada-
cin enzimtica . La IgA dimrica de las secreciones tiene unido el
componente secretor (CS), que oculta las zonas de la inmunoglobuli-
na ms sensibles a la accin de las enzimas (regin bisagra), actuando
como un escudo protector.
Recordad que el CS es una protena de unos 80 kD que es la parte
Pregunta 2. Restriccin histocompatible. extracelular (que se liber por protelisis) de un receptor de membra-
na situado en clulas epiteliales, con gran afinidad para la Fc de la IgA

M exico A rgentina
C hile U ruguay
CTO Medicina C/ Nez de Balboa, 115 28006 MADRID (Espaa) Tfno.: (91) 782 43 32 / Fax: (91) 782 43 27
E-mail: secretaria@ctomedicina.com; iberocto@ctomedicina.com WEB: www.ctomedicina.com; www.iberocto.com
IG Pg. 1
INMUNOLOGA Y GENTICA
Seguimiento a distancia Preparacin Examen de Seleccin 05/06 1 Vuelta
. El receptor original tiene un peso molecular de 100 kD y es sintetiza- B responsables de la misma, al no tomar contacto con los TH2, no
do y expresado abundantemente en la membrana de las clulas pueden cambiar de clase de inmunoglobulina (este proceso es
epiteliales de las mucosas. regulado por los linfocitos TH2). Los principales antgenos son
lipopolisacrido, flagelina y polmeros de d- aminocidos.
Pregunta 8.- R: 1
El cuadro clnico denominado angioedema o edema angioneur- Pregunta 10.- R: 5
tico se debe a la presencia de niveles muy bajos de C1-inhibidor. Es la La deficiencia de factores de complemento ms frecuente a nivel
deficiencia de factores del complemento ms frecuente en los pases mundial es la de C9. Es especialmente abundante en Japn, China y
desarrollados. Aunque generalmente, se trata de una enfermedad de otros pases de Extremo Oriente; no obstante dicha deficiencia es
herencia autosmica dominante, tambin existen formas adquiridas, asintomtica. En los pases desarrollados, la deficiencia ms frecuente
entre las que destacan dos: la asociada a sndromes linfoproliferativos es la de C1 inhibidor. Entre el resto de las deficiencias de componen-
B y la autoinmune debida a un autoanticuerpo contra el C1-inh que tes de la VA CLSICA, la ms frecuente es la deficiencia de C2 y en
impide su accin biolgica. algunos libros hacen especial hincapi en la misma, por lo que con-
La clnica consiste en la aparicin de edemas de forma aguda y viene recordar que, en general, en las deficiencias de la fase inicial de
recurrente. El edema est circunscrito al tejido subcutneo y las la va clsica la sintomatologa ms frecuente es la aparicin de infec-
mucosas y las localizaciones ms frecuentes son cara y cuello, extre- ciones pigenas y cuadros pseudoautoinmunes, como artritis reuma-
midades, genitales, laringe y tracto gastrointestinal. El tratamiento es toide-like o lupus-like (cuadro similar al LES, generalmente con
preventivo con andrgenos atenuados (danazol) que aumentan la negatividad de los anticuerpos antinucleares y anti-ADN, pero con
sntesis intraheptica de C1-inh. El C1-inh purificado slo se debe anticuerpos contra el antgeno Ro).
emplear en crisis agudas y graves. La deficiencia de los componentes terminales (complejo de ata-
que a membrana), del C5 al C9, se asocia a infecciones de repeticin
Pregunta 9.- R: 2 por neisserias, tales como las septicemias gonoccicas o la meningitis
La respuesta inmune humoral (secrecin de anticuerpos) puede ser: meningoccica de repeticin y excepcionalmente enfermedades por
1. Timodependiente (depende de clulas T). En ella todo el proceso inmunocomplejos.
es controlado por lo linfocitos T colaboradores tipo 2 (Helper 2:
TH2). Es la mayoritaria, y por ello cuando se nos hace referencia a Pregunta 11.- R: 5
la respuesta de anticuerpos, lo habitual es que se hable slo de la Las molculas HLA de clase I, se encuentran en la membrana de
dependiente de clulas T. Existen dos formas de respuesta: prcticamente todas las clulas nucleadas y plaquetas. No se expresa
A) Primaria: aparece la primera vez que se entra en contacto con el HLA de clase I clsico (HLA-A, HLA-B y HLA-C) en hemates, sincitio-
un antgeno y como caractersticas principales destacan que: trofoblasto y algunos escasos timocitos.
- Tarda unos cuatro das en comenzar. Las molculas HLA de clase II (HLA-DR, HLA-DP y HLA-DQ) estn
- La inmunoglobulina secretada es IgM. presentes solamente en la superficie de las clulas presentadoras de
B) Secundaria: es la que aparece cuando ha habido previamente antgenos (monocitos, macrfagos, Von-Kupfer, Langerhans, microgl-
un contacto con el antgeno (das, meses o aos antes). a, etc.) y los linfocitos B.
- La inmunoglobulina secretada puede ser IgG, IgA, IgD y/o IgE. Comentario a las respuestas falsas:
- Es rpida debido a que la desarrollan linfocitos de memoria. Los neutrfilos solo tienen HLA de clase I (respuesta 5).
La vacunacin se basa en generar linfocitos de memoria para Los osteoclastos, como macrfagos que son, s expresan HLA de
antgenos concretos. clase 2 (respuesta 1).
2. Timoindependiente (independiente de clulas T). No es necesaria
la colaboracin de los linfocitos TH2. Se trata de una respuesta Pregunta 12.- R: 3
frente a productos polimricos de difcil degradacin metablica, Las crioglobulinas son inmunoglobulinas que precipitan reversi-
que no existen en el organismo humano en condiciones fisiolgi- blemente con el fro (4C) y se redisuelven a 37C. Es posible detectar
cas. Esta respuesta siempre es de clase IgM, puesto que los linfocitos en el suero de los enfermos con estados de hipocoagulabilidad (trata-

Comentarios TEST

Pregunta 10. Complejo de ataque a membrana.

M exico A rgentina
Pg. 2 IG C hile U ruguay
CTO Medicina C/ Nez de Balboa, 115 28006 MADRID (Espaa) Tfno.: (91) 782 43 32 / Fax: (91) 782 43 27
E-mail: secretaria@ctomedicina.com; iberocto@ctomedicina.com WEB: www.ctomedicina.com; www.iberocto.com
INMUNOLOGA Y GENTICA
Preparacin Examen de Seleccin 05/06 1 Vuelta Seguimiento a distancia
dos con cumarinas), un precipitado similar al colocar el suero a 4C, lizadas en esta fraccin comn. Por tanto la respuesta correcta es la
pero estos ltimos se diferencian de la crioglobulinemia real porque 3.
el precipitado no se redisuelve a 37C. Dos idnticos llamados Fab (respuesta 2), iniciales de las palabras
Las crioglobulinas suelen aparecer en enfermos que padecen inglesas Fraction - Anti Body. Cada uno de ellos contiene la zona de
mielomas o linfomas, infecciones y cuadros autoinmunes, aunque la molcula responsable de la unin al antgeno. En realidad una
algunas veces aparecen sin patologa asociada y carecen de relevan- Fab est formada por una cadena ligera y la mitad aminoterminal
cia. Pueden desencadenar cuadros graves, entre los que destacan el de una cadena pesada.
fenmeno de Raynaud, la prpura vascular, la urticaria inducida por Digiriendo con pepsina obtenemos la F(ab)2 (respuesta 1), que se
el fro y la trombosis arterial distal con gangrena. trata de una fraccin antibody bivalente, es decir con capacidad de
Comentario a las respuestas falsas: unir dos antgenos, y diversos restos peptdicos.
Las piroglobulinas son inmunoglobulinas monoclonales que preci- La regin bisagra, situada aproximadamente es una zona tremen-
pitan irreversiblemente a 56C. El 50 % corresponden a pacientes de damente mvil de la cadena pesada que contiene los puentes disulfu-
mieloma mltiple. El resto son pacientes lpicos o con linfomas. ro que unen entre s las dos cadenas pesadas.
El factor reumatoide es un anticuerpo que reconoce a la IgG del
propio paciente, generalmente son IgM, pero puede ser cualquier Pregunta 16.- R: 3
clase. Aunque la presencia en pequeas cantidades no es un dato Esta pregunta es mixta de Inmunologa y gentica. Conviene recor-
relevante y puede darse en personas normales, unos niveles altos dar algunos conceptos para entender perfectamente el significado del
de FR es tpico que aparezca en la artritis reumatoide. enunciado:
Los haptenos son los antgenos incompletos incapaces de desper- Alelos son las distintas formas en las que nos podemos encontrar la
tar, por s solos, una respuesta inmune. Slo se detectan si previa- expresin de un gen polimrfico. Por ejemplo, en el caso de los gru-
mente se asocian a otra molcula portadora. pos sanguneos humanos, podemos observar tres posibles alelos dis-
tintos que son A, B y O. El 80 % de los genes no presentan variabilidad
Pregunta 13.- R: 4 de una persona a otra, es decir son idnticos para todas las personas.
Los antgenos polisacridos pueden ser neutralizados por la res- El 20 % restante varan de unos a otros pero generalmente el producto
puesta de anticuerpos T independiente (IgM) o T dependiente, en que codifica cada alelo vara de los otros slo en uno o dos
cuyo caso el anticuerpo ms importante es la IgG2. La IgM no est aminocidos.
dentro de las respuestas posibles, por lo que la nica opcin que nos Si las dos copias de un gen determinado, situadas en un cromoso-
queda es elegir la respuesta 4 (IgG2). ma procedente de cada progenitor, son iguales, el individuo es HO-
MOCIGOTO (homo=igual), si son distintos el individuo es HETERO-
Pregunta 14.- R: 4 CIGOTO (heteros=distinto).
Todos los linfocitos T, por definicin, tienen en su membrana el Comportamiento de los alelos: el responsable del fenotipo del
receptor antignico de la clula T (TcR) asociado a la molcula CD3. individuo es el grado de expresin de cada alelo respecto a su compa-
Por tanto las respuestas 1, 2 y 5 se descartan inmediatamente. Queda ero. Los alelos se definen como dominantes, recesivos o codomi-
por decidir cual de las dos que restan, 3 y 4, es la verdadera. Si el nantes.
enunciado de la pregunta no incluyese el trmino poblacin mayo- Dominante: para que aparezca el fenotipo es necesario que el
ritaria, ambas seran ciertas porque existen linfocitos T colaborado- alelo responsable del mismo est presente en uno solo de los 2
res tanto CD4+ como CD8+. Pero al incluirlo y puesto que la inmen- cromosomas homlogos. En los heterocigotos, el alelo recesivo
sa mayora de los linfocitos colaboradores son CD4+, queda perfec- queda silenciado por el dominante. Ejemplo de alelo dominante
tamente claro que la correcta es la respuesta 4. es el grupo sanguneo A respecto del O.
Recesivo: slo se expresa el fenotipo de un alelo si est en los dos
Pregunta 15.- R: 3 cromosomas. Si el otro alelo es dominante, el heterocigoto para un
La diseccin enzimtica de la IgG en la regin bisagra (la ms sen- alelo recesivo no lo expresa. As el genotipo de grupo sanguneo
sible a la accin enzimtica) nos permite obtener diversos fragmentos AO expresa el fenotipo A (O es recesivo, A es dominante).
cada uno con una funcin concreta. Codominantes. Los dos alelos tienen la misma fuerza para expre-
sarse, y se expresan a la vez. El heterocigoto para los dos alelos, por
tanto, tendr un fenotipo mezcla de los fenotipos de cada alelo
Comentarios TEST

por separado. Ejemplo: fenotipo AB de grupo sanguneo.


El sistema HLA es completamente codominante, y puesto que ade-
ms es muy polimrfico, el estudio de la herencia del mismo es muy
empleado en las pruebas de paternidad. En un individuo siempre
deben expresarse la mitad de los alelos HLA del padre, y si no lo
hacen, dado que es imposible que un HLA se convierta en recesivo, se
puede afirmar la no paternidad.
Conviene recordar que las pruebas de paternidad, ya sean con
HLA o DNA, slo sirven para excluir paternidad; la afirmacin de la
paternidad nunca se puede hacer con una certeza del 100 %.

Pregunta 17.- R: 3
La respuesta 1 (regresin tmica) no tiene nada que ver con un
trasplante de mdula sea. La respuesta 2 no constituye en s ninguna
complicacin. Las respuestas 4 y 5 indicaran que el injerto no ha sido
satisfactorio, pero desde luego no son complicaciones tan graves como
Pregunta 15. Digestin enzimtica de inmunoglobulina. la reaccin de injerto contra husped, que es, sin duda, la peor de las
complicaciones posibles (respuesta 3).
Si digerimos con papana (enzima extrada de la papaya) se obtie- La alorreactividad consiste en la identificacin como extraas de
nen 3 fragmentos: las clulas de otro individuo de la misma especie y la eliminacin
Un fragmento Fc (Fraccin Comn o cristalizable), formado por las posterior de las mismas. Ocurre cuando los linfocitos T de un indivi-
dos mitades carboxiterminales de las cadenas pesadas. Las funcio- duo se ponen en contacto con antgenos de histocompatibilidad dife-
nes comunes a todas las inmunoglobulinas, independientemente rentes a los propios, caso de un injerto, detectando las diferencias y
de a qu antgeno estn dirigidas, tales como la opsonizacin, el respondiendo como si stas se debieran a la modificacin que resulta
paso a travs de placenta o la fijacin del complemento, estn loca- de la incorporacin de un antgeno extrao a las molculas de

M exico A rgentina
C hile U ruguay
CTO Medicina C/ Nez de Balboa, 115 28006 MADRID (Espaa) Tfno.: (91) 782 43 32 / Fax: (91) 782 43 27
E-mail: secretaria@ctomedicina.com; iberocto@ctomedicina.com WEB: www.ctomedicina.com; www.iberocto.com
IG Pg. 3
INMUNOLOGA Y GENTICA
Seguimiento a distancia Preparacin Examen de Seleccin 05/06 1 Vuelta
histocompatibilidad propias, considerando pues a la clula portado- TH2 producen IL4, e IL6 y colaboran en las reacciones de inmuni-
ra como si estuviese infectada por un virus. dad humoral: linfocitos B y clulas plasmticas (respuesta 4 correc-
Debido a la alorreactividad puede producirse tanto el rechazo de ta).
rganos como la enfermedad de injerto contra husped. Esta ltima TH3 producen IL10 y TGF Beta y se les considera linfocitos regula-
consiste en un rechazo del injerto al cuerpo que lo acoje, y para que dores.
se desencadene, son necesarias tres condiciones simultneas: Del tipo de respuesta de linfocitos colaboradores que se desarrolle
1. Trasplante de rganos entre personas no histocompatibles. frente a un antgeno depende que dicha respuesta concluya en el
2. El receptor debe ser una persona inmunocomprometida. desenlace de la enfermedad o en aparicin de formas ms severas de
3. En el rgano a trasplantar debe haber linfocitos T viables. la misma (respuesta 5 correcta).
Los poqusimos linfocitos T que estn localizados en el rgano La respuesta 2 es falsa, ya que en ella se nos reduce el tipo de
trasplantado, aunque numricamente inferiores, son funcionalmente linfocitos T solamente a dos, aunque es cierto que los dos tipos bsi-
muy superiores al conjunto de todo el sistema inmune del organismo cos son los CD4+ y los CD8 positivos, pero stos a su vez se subdivi-
receptor y acaban destruyndolo. den en otros subgrupos (colaboradores, citotxicos) y estos otros tam-
bin se subdividen.... Adems, la pregunta nos est dando pistas para
Pregunta 18.- R: 4 que sepamos que la respuesta 2 es incorrecta: la propia respuesta 1
La inmunodeficiencia variable comn es un sndrome de inmuno- nos est diciendo que, slo entre los T colaboradores, hay tres tipos
deficiencia humoral adquirido que suele debutar en la adolescencia (existe una gran variedad de fenotipos de linfocitos T).
y que est caracterizado por una panhipogammaglobulinemia (las En definitiva, la respuesta al decir que solo existen dos tipos de
inmunoglobulinas totales suelen estar por debajo de 500 mg/dL). En linfocitos T simplifica excesivamente la clasificacin de un conjunto
esta enfermedad existe una respuesta de anticuerpos muy dbil lo de clulas tan heterogneo como los linfocitos T.
que se manifiesta en infecciones bacterianas de repeticin, especial-
mente respiratorias y digestivas. Existe un nmero normal de linfocitos Pregunta 21.- R: 4
B que, in vitro, la mayora de ellos son incapaces de secretar Las respuestas 1 (inmunodeficiencia variable comn) y 3 (hipo-
inmunoglobulinas o lo hacen de forma deficitaria. gammaglobulinemia transitoria de la infancia), las descartamos inme-
Son, por tanto, completamente verdaderas las respuestas 1, 2 y 3. diatamente. Estas enfermedades consisten en una deficiencia de anti-
Asimismo, al igual que el resto de las inmunodeficiencias primarias, se cuerpos y nunca dan una clnica tan severa como la descrita en el
asocia a una mayor incidencia de tumores, siendo el linfoma no enunciado, destacando, por el contrario, las infecciones respiratorias
Hodgking el ms frecuente (respuesta 5 cierta). de repeticin con un nmero de linfocitos T normales.
Lo que es falso es la edad de aparicin: no aparece en los primeros Las otras tres respuestas s pueden dar una clnica como la descrita,
meses de vida, sino que debuta a partir de la adolescencia. de inmunodeficiencia combinada severa (ICS). La clave para distin-
guir una de otra y por tanto acertar la correcta est en la ausencia de
Pregunta 19.- R: 4 linfocitos T. Descartamos la deficiencia de ADA (respuesta 2) al tratarse
El sndrome de Di George consiste en una serie de malformaciones de un problema metablico de la clula que difcilmente acaba en
congnitas mltiples que se deben a un anormal desarrollo embrio- ausencia de clulas.
nario de los rganos derivados del tercero y el cuarto arcos farngeos Ahora se nos ocurrira que la respuesta es el sndrome de Di George
(la respuesta 1 es falsa) y que afectan fundamentalmente a timo, gran- por la atrofia del timo, que impedira la formacin de clulas T, pero
des vasos (la respuesta 4 es verdadera) y glndulas paratiroideas (la no lo es porque, como vimos en la pregunta 19, es muy raro que en
respuesta 2 es falsa). este cuadro haya ausencia total de linfocitos T.
En la gran mayora de los pacientes se detecta una microdelecin Sin embargo, en la inmunodeficiencia combinada severa ligada al
en el brazo largo del cromosoma 22 (22q11) sin que se conozcan cromosoma X, la ms frecuente de las ICS, s que es tpico encontrar la
formas heredadas (la respuesta 5 es falsa). ausencia de linfocitos T, por lo que sta es la respuesta verdadera (la 4).
La clnica de estos enfermos est relacionada con la aplasia o hipo-
plasia de los rganos afectados: la primera manifestacin a poco de Pregunta 22.- R: 3
nacer es la tetania debida a la hipocalcemia (la respuesta 3 es falsa), Las infecciones respiratorias de repeticin, junto con el sndrome
que se instaura por la disfuncin de las glndulas paratiroideas (hipo- diarreico intermitente, son las dos principales sintomatologas de los
paratiroidismo). sndromes de deficiencia de anticuerpos. Otras sintomatologas son
Las anomalas del sistema inmune son variables. La ausencia total las otitis y sinusitis crnicas. Queda claro, por tanto, que nuestra enfer-

Comentarios TEST
de linfocitos T es rara, y en estos casos, la clnica es la de una inmuno- ma tiene una deficiencia de anticuerpos, por tanto descartamos in-
deficiencia combinada severa. Lo habitual es que haya un nmero mediatamente las respuestas 1, 2 y 4, que se tratan de sndromes de
bajo de linfocitos T (linfopenia T) y la clnica est relacionada con el inmunodeficiencia combinada severa, que se caracterizan por infec-
grado de disminucin de la respuesta inmune celular. La presencia de ciones de repeticin por hongos y virus, adems de la sintomatologa
linfocitos T en la mayora de los enfermos se debe a que la aplasia descrita anteriormente.
tmica total es muy poco frecuente; en la biopsia de estos enfermos es La edad de aparicin (15 aos) nos lleva a descartar el sndrome de
comn encontrar restos tmicos o tejido tmico ectpico, por lo que Wiskott-Aldrich, que aparece en el primer ao de vida. La inmunode-
siempre queda algo de funcin inmune celular. ficiencia variable comn (respuesta correcta), como ya vimos en otra
En los casos de ausencia total de timo, las manifestaciones clnicas pregunta, es un sndrome de inmunodeficiencia humoral adquirido
son las de gentica. que suele debutar a partir de la adolescencia.
Otros datos clnicos: hipertelorismo, micrognatia, anomalas en los
grandes vasos e implantacin baja de los pabellones auriculares. Pregunta 23.- R: 2
En todas las situaciones descritas en las respuestas, las inmunoglo-
Pregunta 20.- R: 2 bulinas no tienen ninguna funcin (respuesta 5) o bien realizan la
Actualmente se acepta que existen tres tipos de linfocitos T colabo- funcin sealada de modo paralelo a otros mecanismos del sistema
radores TH (Helper en ingls): TH1, TH2 y TH3 (respuesta 1 verdade- inmune, siendo, en cierta manera, prescindibles.
ra). As es posible eliminar clulas infectadas por virus (respuesta 1) con
Estos linfocitos, ya sean CD4+ o CD8+, regulan la respuesta inmu- clulas T citotxicas o NK adems de con anticuerpos. La bacterilisis
ne ofreciendo su colaboracin, en forma de citoquinas, a otras clu- (respuesta 3) puede realizarse fijando el complemento por la va clsi-
las del sistema inmune. El criterio para clasificarlos son las citoquinas ca (con anticuerpos) o directamente por la va alterna. La fagocitosis
que secretan cada uno de ellos; las fundamentales son: (respuesta 4) se puede realizar directamente o gracias a la opsonizacin
TH1 producen IL2 e interfern gamma y controlan las respuestas con anticuerpos.
de inmunidad celular estimulando a macrfagos y otras clulas T La nica funcin que realizan los anticuerpos sin que se conozca
(respuesta 3 correcta). por el momento ningn otro mecanismo inmune paralelo, es la neu-

M exico A rgentina
Pg. 4 IG C hile U ruguay
CTO Medicina C/ Nez de Balboa, 115 28006 MADRID (Espaa) Tfno.: (91) 782 43 32 / Fax: (91) 782 43 27
E-mail: secretaria@ctomedicina.com; iberocto@ctomedicina.com WEB: www.ctomedicina.com; www.iberocto.com
INMUNOLOGA Y GENTICA
Preparacin Examen de Seleccin 05/06 1 Vuelta Seguimiento a distancia
tralizacin de toxinas (botulnica, tetnica, colrica, pertussis, etc.) Por sera difcil que cayese en un examen; aqu est entre las respuestas
tanto, la nica respuesta correcta es la 2. falsas con la nica intencin de hacer bulto para rellenar las 4
falsas.
Pregunta 24.- R: 4 Un haplotipo es un conjunto de genes que estn muy juntos en un
La agammaglobulinemia ligada al sexo (sndrome de Bruton) fue la cromosoma y que, por lo general, se heredan como una unidad.
primera inmunodeficiencia primaria descrita. Se trata de una defi- As el conjunto de genes del HLA se considera un haplotipo.
ciencia pura de anticuerpos, por lo que las respuestas 1 y 5 quedan
automticamente descartadas. Pregunta 28.- R: 5
El gen causante de la enfermedad, situado en el cromosoma X La edad del nio y la gravedad de la sintomatologa nos hace
(como la hemofilia, slo los varones tienen la enfermedad) codifica sospechar una inmunodeficiencia primaria y por tanto descartar el
una tirosinquinasa (tirosinquinasa de Bruton) expresada en clulas B y SIDA (ID secundaria a infeccin por VIH). Esta decisin de descartarlo
que es necesaria para la maduracin de los linfocitos B. viene avalada porque en ningn momento se nos habla de positivi-
Caractersticamente, los enfermos carecen de linfocitos B (respues- dad de ninguna prueba ni de antecedentes de la madre.
ta 4 verdadera) y de clulas plasmticas. Las dems poblaciones linfo- La respuesta 2 la descartamos por los mismos motivos, la edad y
citarias son normales. una clnica no compatible.
Al no haber linfocitos B, los enfermos no tienen nada de IgM (res- La respuesta 3 la descartamos porque en ningn sitio nos hacen
puesta 3 es falsa, para ser cierto deber decir: IgM indetectable). referencia a las infecciones por microorganismos catalasa positivos,
Paradjicamente, estos enfermos que carecen de linfocitos B s caractersticas de esta enfermedad.
tienen IgG (paradoja de Bruton), por tanto la respuesta 2 es falsa. La respuesta 4 (sndrome de Bruton) tambin la descartamos por-
que en ella habra infecciones respiratorias de repeticin y diarreas
Pregunta 25.- R: 3 intermitentes como sintomatologa ms marcada. Quiz el dato de la
Como ya se ha visto en otras preguntas, los linfocitos T slo son hipogammaglobulinemia y la ausencia de linfocitos B nos hace sospe-
capaces de reconocer antgenos si stos le son presentados por otras charla, pero las infecciones severas por hongos, como es el caso, no
clulas junto con el complejo principal de histocompatibilidad (mo- aparecen en el Bruton.
lculas HLA). Existen dos tipos de linfocitos, atendiendo a su modo de Se trata de una inmunodeficiencia combinada severa (respuesta 5
detectar los antgenos: los CD4+ reconocen antgenos presentados correcta), ya que fallan tanto la inmunidad celular (ausencia de timo
junto con el HLA de clase II y los CD8+ reconocen antgenos en el e infecciones severas por hongos) como la humoral (ausencia de
contexto de molculas HLA clase I. linfocitos B e hipogammaglobulinemia).
En el enunciado de la pregunta se nos habla de los CD4+, por
tanto debemos buscar entre las respuestas aquella que contenga un Pregunta 29.- R: 5
HLA de clase II. Como regla mnemotcnica, los HLA de clase I tienen La disgenesia reticular es la ms grave inmunodeficiencia que se
una letra (A, B, C) y los de clase II, dos letras (DR, DP, DQ). conoce (respuesta 1 falsa). Los nios que la padecen no tienen ningn
Se descartan las respuestas 1 y 5 por no ser molculas HLA y las 2 leucocito en la sangre y mueren a las pocas horas de vida, a no ser que
y 4 por describir HLA de clase I (una letra). La nica respuesta posible se realice trasplante de mdula sea intratero.
es, por tanto, la 3. La deficiencia de HLA de clase II constituye una de las formas de la
enfermedad denominada inmunodeficiencia combinada severa (ICS),
Pregunta 26.- R: 2 de la que hablamos ms adelante.
Se denominan rganos linfoides primarios a aquellos en los que se Las respuestas 3 y 4 hacen referencia a la ICS ligada al X (la forma
originan y maduran las clulas del sistema inmune y rganos linfoides ms frecuente) y al sndrome de ICS en general. Se trata de uno de los
secundarios a aquellos donde se disponen los linfocitos maduros, e cuadros clnicos de inmunodeficiencia ms graves al fallar tanto la
inmunolgicamente competentes. Es en stos ltimos donde se produ- inmunidad humoral como la celular. El pronstico de estos enfermos
cen las respuestas inmunitarias frente a los estmulos antignicos. era infausto hasta que apareci el trasplante de mdula sea. Actual-
Son rganos linfoides primarios la mdula sea y el timo y secun- mente con dicho trasplante recuperamos la funcin inmune, pero no
darios los ganglios linfticos, el bazo y el tejido linfoide asociado a las siempre se llega a la curacin al 100 % porque este tipo de enferme-
mucosas (placas de Peyer, anillo linftico de Waldeyer, etc.). dades suelen deberse a deficiencias metablicas que inciden espe-
Las respuestas 1, 3, 4 y 5 hacen referencia, sin ninguna duda, a cialmente en el sistema inmune, pero que afectan tambin a otros
Comentarios TEST

rganos linfoides secundarios. Sin embargo, la respuesta 2 (Falsa) hace rganos, a los que el trasplante de mdula sea no les es til.
referencia al timo, que es un rgano linfoide primario. La inmunodeficiencia variable comn (respuesta 5 cierta) es una
deficiencia de anticuerpos que se puede controlar muy bien con
Pregunta 27.- R: 3 administracin parenteral de inmunoglobulinas, lo que permite que
Esta es una pregunta en la que debemos tener muy claros los princi- los pacientes tengan una esperanza de vida slo algo menor que la de
pales conceptos de inmunologa bsica y que vamos a recordar: otra persona sana de su edad. Este descenso en la esperanza de vida se
Epitopo: tambin llamado determinante antignico, es la regin debe, como en todas las inmunodeficiencias, a una mayor incidencia
concreta de un antgeno donde se une el anticuerpo. Un antgeno de tumores que la poblacin general. Por tanto, la menos mala de
suele tener varios epitopos distintos. las enfermedades aportadas en las respuestas es la ID variable comn.
Haptenos: son sustancias qumicamente sencillas y de bajo peso
molecular, que por s solas no son inmungenas, pero que pueden Pregunta 30.- R: 2
comportarse como tales si se unen covalentemente a otra molcu- Ya hicimos una pregunta en la que abordamos la diseccin enzi-
la ms grande (portador o carrier), por lo que algunos autores los mtica de la IgG, pero debido a su importancia y a que ltimamente
denominan antgenos incompletos. Esta es la respuesta correcta parece que se ha vuelto a poner de moda, hemos abordado este
(3). tema con esta otra pregunta con un enfoque diferente a la anterior.
Idiotipo: definindolo de una manera grfica y fcilmente Recordemos que la digestin enzimtica con papana se realiza en
recordable, es la zona del anticuerpo que determina su actividad, la regin bisagra y nos da varios fragmentos:
la que interacta directamente con el epitopo del antgeno. En una Un fragmento Fc (Fraccin Comn), formado por las dos mitades
definicin ms formal (pero ms fcil de olvidar), es el conjunto de carboxiterminales de las cadenas pesadas. Aqu estn las zonas de
determinantes antignicos (Idiotopos) situados en las regiones va- la molcula responsables de la opsonizacin, el paso de placenta
riables de las cadenas pesada y ligera de una inmunoglobulina. y la fijacin del complemento.
Paratopo: son las zonas concretas del idiotipo responsables del Dos fragmentos idnticos llamados Fab, que quieren decir Fraction-
reconocimiento del determinante antignico, los aminocidos que Anti Body. Cada uno de ellos es capaz de unirse por s solo a un
interactan con la molcula del antgeno existen. Este ltimo con- antgeno.
cepto ya no se considera tan importante como hace unos aos y Lo que acabamos de recordar est reflejado en la respuesta 2.

M exico A rgentina
C hile U ruguay
CTO Medicina C/ Nez de Balboa, 115 28006 MADRID (Espaa) Tfno.: (91) 782 43 32 / Fax: (91) 782 43 27
E-mail: secretaria@ctomedicina.com; iberocto@ctomedicina.com WEB: www.ctomedicina.com; www.iberocto.com
IG Pg. 5
INMUNOLOGA Y GENTICA
Seguimiento a distancia Preparacin Examen de Seleccin 05/06 1 Vuelta
Del resto de las respuestas, la nica con sentido es la 1, que refleja Reaccin de tipo I: anafilctica, basada en anticuerpos IgE.
el producto de la diseccin con pepsina. El resto de las respuestas se Reaccin de tipo II: citotxica. Anticuerpos que fijan el comple-
han rellenado como respuestas falsas inventadas, para hacer bulto. mento sobre clulas.
Reaccin de tipo III: lesin por depsito de inmunocomplejos
Pregunta 31.- R: 2 antgeno-anticuerpo.
Las pruebas cruzadas inmunolgicas se realizan para detectar, en Reaccin de tipo IV: retardada, de tipo tuberculnico, mediada por
la sangre de una persona a la que se plantea trasplantar un rgano, clulas. A su vez se subdivide en cuatro tipos:
anticuerpos contra antgenos de histocompatibilidad situados en la - Reaccin de Jones-Mote. Comienza a las 12-24 h y dura 24 h.
membrana de las clulas del rgano del donante. Si estos anticuerpos - Dermatitis por contacto, aparece en torno a las 24 h de la expo-
existen, est contraindicado el trasplante porque, de hacerse, estos sicin y dura 48-72 h.
anticuerpos se fijaran en las primeras clulas que tomaran contacto - Tipo tuberculina, comienza a las 24 h y dura 72-96 h (es la tipo
con la sangre (endoteliales) y fijaran sobre ellas el complemento des- IV ms caracterstica).
truyndolas. Al destruirse el endotelio, quedara la membrana basal - Granuloma. Comienza a los 7-14 das y dura varias semanas.
de los vasos sanguneos en contacto directo con el plasma, desenca- Las pruebas cutneas de hipersensibilidad retardada son el medio
denando progresivamente: agregacin plaquetaria, coagulacin ms simple y eficaz para valorar clnicamente la respuesta inmune
intravascular, hipoxia del rgano y coagulopata de consumo. celular frente a un antgeno. Para poder evaluar adecuadamente la
El nico tratamiento posible, por tanto, es el preventivo: no tras- respuesta inmune celular de un individuo mediante estas pruebas, se
plantar. deben utilizar, al menos, seis antgenos diferentes (p. e.: candidina,
La presencia de los citados anticuerpos se debe a inmunizaciones coccidiocidina, antgeno de estafilococo, tuberculina, estreptoquina-
previas frente a antgenos de histocompatibilidad alognicos, debidas sa-estreptodornasa y dermatofitina).
a transfusiones de sangre completa o plaquetas, y embarazos (slo en Dependiendo de la cantidad de antgeno inoculada, ser tanto
mujeres). mayor el grado de reaccin, pero por lo general, la dosis de antgeno
se ajusta para que en una persona normal se considere positiva si se
Pregunta 32.- R: 4 observa una induracin de ms de 9 mm, 48 h despus de la inyec-
Ya tratamos en otra pregunta el importante tema de la reaccin de cin del antgeno.
injerto contra husped (EICH) tras un trasplante. Recordemos que son La negatividad de las pruebas debe ser tratada con cautela, y en
necesarias tres circunstancias: caso de darse, se deben repetir con una mayor concentracin de
Trasplante de rganos entre personas no histocompatibles. antgeno antes de poder afirmar que existe anergia cutnea.
Receptor del trasplante seriamente inmunodeprimido. Las pruebas cutneas en nios de corta edad son de interpretacin
Que existan linfocitos T inmunocompetentes en el rgano tras- dudosa.
plantado. Por tanto, en nuestra pregunta, la respuesta 1 es falsa (sera a partir
En los trasplantes de mdula, el receptor es una persona de las 48 h). La 3 es falsa porque lo que se mide es la induracin y no
inmunocomprometida; es el trasplante donde la probabilidad de que el eritema. Las respuestas 4 y 5 son falsas porque este tipo de reaccin
aparezca la EICH es mayor (falsa). En los de rganos macizos, como est mediada por clulas y no por anticuerpos.
rin, hgado o corazn, aunque el receptor no sea una persona
inmunodeprimida, siempre quedan linfocitos viables del donante en Pregunta 35.- R: 4
el rgano y existe la posibilidad, aunque sea remota, de aparicin de Esta pregunta es un magnfico recordatorio de las caractersticas
la EICH (respuestas 2, 3 y 5 falsas) . Pero en el trasplante de crnea, que diferencian entre s a las principales clulas del sistema inmune.
nunca hay linfocitos del donante y, por tanto, es imposible que se La respuesta 1 es cierta, slo los linfocitos T tienen receptor de la
desencadene dicha enfermedad. clula T (TcR), el cual se expresa en la membrana de estas clulas
asociado a la molcula CD3, cuya funcin es transmitir la seal de
Pregunta 33.- R: 5 activacin al interior de la clula cuando el TcR reconoce al antgeno.
El sndrome de Di George es un cuadro clnico consistente en un Todos los linfocitos T, independientemente de que su receptor sea
anormal desarrollo embrionario de los rganos derivados del tercer y alfa-beta (el 95 % de los de sangre perifrica) o gamma-delta, tienen en
el cuarto arcos farngeos que da lugar a malformaciones congnitas su membrana CD3.
mltiples, entre las que destacan la atrofia o hipotrofia del timo, las La respuesta 2 es cierta, los linfocitos B tienen inmunoglobulina en
glndulas paratiroideas y los grandes vasos de cuello. En la mayor la membrana celular.
parte de los enfermos, el cuadro se asocia a una microdelecin en el La respuesta 3 es cierta, efectivamente, adems de un subgrupo de

Comentarios TEST
brazo largo del cromosoma 22 (22q11). los linfocitos T (el mayoritario), tambin los monocitos y macrfagos
La clnica que presentan los enfermos est relacionada con el gra- son CD4+, y por eso son infectados por el VIH.
do de aplasia o hipoplasia de estos rganos, siendo la tetania, debida La respuesta 4 es FALSA, solamente tienen inmunoglobulina de
a la hipocalcemia por la disfuncin de las glndulas paratiroideas superficie los linfocitos B. Los macrfagos no tienen IG de superficie,
(hipoparatiroidismo), la primera manifestacin clnica que general- ni siquiera las tienen las clulas plasmticas (las productoras de gran-
mente presentan estos enfermos. Otros signos clnicos asociados al des cantidades de inmunoglobulinas solubles).
sndrome son el hipertelorismo, la micrognatia y la implantacin baja La respuesta 5 es cierta, es un poco difcil, pero el que haya una
de los pabellones auriculares. nica respuesta con una dificultad mayor no hace compleja el
Sistema inmune: no es frecuente que aparezcan pacientes con resto de la pregunta. La falsedad de la respuesta 4 es evidente. El
sndrome de Di George que presenten una ausencia total de linfocitos objeto de la pregunta es dejar bien asentados conceptos funda-
T, ya que la aplasia tmica total es muy poco frecuente. En estos mentales de inmunologa: todas las clulas con CD3 son linfocitos
escassimos casos las manifestaciones son las de una inmunodeficien- T y todas las clulas con inmunoglobulina de superficie son
cia combinada severa. linfocitos B.
Lo habitual es que estos pacientes presenten una hipotrofia del
timo ms o menos marcada o bien tengan tejido tmico ectpico, lo GENTICA
que supone que casi siempre va a haber funcin tmica en mayor o
menor grado. La mayora de las pacientes, por tanto, presentan una Pregunta 36.- R: 2
linfopenia T, ms o menos marcada, y la clnica est relacionada con Hace aos, cuando se descubri el origen gentico de algunas
el grado de disminucin de la respuesta inmune celular. enfermedades, se pens que cada gen, si se alteraba, poda dar lugar
a una enfermedad y que, a su vez, cada enfermedad estara originada
Pregunta 34.- R: 2 por un solo gen. Segn se fue estudiando el genoma, se comprob
Coombs y Gell realizaron en 1963 una clasificacin de los meca- que la gentica humana no era tan sencilla y surgi el concepto de
nismos inmunopatognicos que luego fue modificada en 1975 y que heterogeneidad gentica para tratar de explicar anomalas como, por
establece las denominadas reacciones de hipersensibilidad: ejemplo, que una enfermedad puede estar originada por alteraciones

M exico A rgentina
Pg. 6 IG C hile U ruguay
CTO Medicina C/ Nez de Balboa, 115 28006 MADRID (Espaa) Tfno.: (91) 782 43 32 / Fax: (91) 782 43 27
E-mail: secretaria@ctomedicina.com; iberocto@ctomedicina.com WEB: www.ctomedicina.com; www.iberocto.com
INMUNOLOGA Y GENTICA
Preparacin Examen de Seleccin 05/06 1 Vuelta Seguimiento a distancia
en genes diferentes. El concepto de heterogeneidad gentica ha ido lidad de que un caso concreto se deba a mutacin espontnea es
evolucionando con los aos y por ello, en preguntas de MIR de los muy baja (por debajo de 0,00001).
aos 80, las respuestas que fueron correctas en su momento, ahora 2. No hay portadores sanos, slo existen enfermos (aunque no ten-
no lo seran. gan clnica en el momento del diagnstico) y sanos.
Actualmente, el concepto de heterogeneidad gentica (HG) tiene 3. Afecta a ambos sexos por igual y un enfermo tendr un 50 % de
un sentido ms amplio: se dice que existe HG cuando una enferme- hijos afectados y un 50 % de hijos sanos.
dad se puede originar por diferentes mecanismos genticos tales como 4. Los hijos sanos de un enfermo slo tendrn hijos sanos.
mutaciones, deleciones, interacciones gnicas, heterogeneidad de
locus, allicas, etc. Es decir, viene a significar algo as como enferme- Pregunta 38.- R: 2
dades con gentica compleja y dentro de ella se engloban tres tipos Se llama GEN o CISTRON al segmento de ADN que contiene la
de heterogeneidad. informacin para sintetizar una cadena polipeptdica. El genoma (el
La heterogeneidad de locus se aplica a las situaciones en las que conjunto de genes de un individuo) se reparte en 46 molculas linea-
alteraciones en genes distintos (locus diferentes) pueden dar un cua- les de ADN, organizadas en torno a un esqueleto proteico y que se
dro clnico idntico. Un ejemplo lo son la retinitis pigmentaria o la denominan cromosomas. Los distintos genes de un cromosoma se
inmunodeficiencia combinada severa (a esto era a lo que en los aos sitan linealmente en el ADN, uno detrs de otro. Solemos hablar
80 se le llamaba heterogeneidad gentica). habitualmente de la protena como producto de la expresin de un
Son falsas las respuestas 1, 3, 4 y 5 porque slo se conoce un locus gen, pero en realidad deberamos hablar del pptido producto de ese
(gen) responsable de esas enfermedades, pero es cierta la respuesta 2 gen, puesto que una protena puede estar formada por varias cadenas
porque la inmunodeficiencia combinada severa se puede deber a peptdicas (polipptido).
mutaciones en genes diferentes, incluso en cromosomas distintos (for- Un opern (respuesta 1, falsa) es un segmento de ADN que contie-
mas autosmicas y ligadas al X). ne la informacin para producir varias protenas distintas (relaciona-
das funcionalmente), se transcribe a un nico ARNm, ste se traduce
Pregunta 37.- R: 3 a una protena que luego es escindida en varias protenas que tienen
La poliquistosis renal del adulto es una de las enfermedades here- acciones independientes (por ejemplo, enzimas con diferentes
ditarias ms frecuentes. Se estima que, en Espaa, su frecuencia es substratos). Un ejemplo de opern es el Lac de la bacteria E. coli, que
mayor que la de la hipercolesterolemia familiar. codifica tres enzimas de la ruta metablica de la lactosa.
El mecanismo de herencia es autosmico dominante, es decir, que Respuestas 3 y 4 (falsas): la informacin contenida en los genes
la presencia de una de las dos copias gnicas alterada va a dar lugar a eucariotas no es continua, sino que se encuentra repartida en varios
la enfermedad. segmentos de ADN codificante (exones), interrumpidos por segmen-
Conviene recordar que, en las enfermedades con herencia autos- tos de ADN no codificante (intrones). El nmero de exones e intrones
mica recesiva, es preciso que estn alteradas las dos copias para que se vara de un gen a otro. Conviene recordar que los genes procariotas
manifieste el cuadro clnico. Al ser autosmicas (autosomas son todos no contienen intrones.
los cromosomas con excepcin de los sexuales), los dos sexos tienen la
misma probabilidad de padecer y transmitir la enfermedad. Pregunta 39.- R: 4
La mayora de las enfermedades dominantes muestran dos carac- La herencia mitocondrial es un tema que ha sido preguntado fre-
tersticas: edad tarda de aparicin y expresin clnica variable. cuentemente en el MIR y que, por tanto, debemos tener muy claro.
Patrn de herencia: Las mitocondrias son orgnulos que tienen su propio ADN (16,5
1. Transmisin vertical: todos los individuos afectados han heredado kb) con un cdigo gentico distinto del que utiliza el ADN nuclear.
el cuadro de un progenitor tambin afectado, excepto los Se considera que las mitocondrias son endosimbiontes: bacterias
escassimos casos debidos a mutaciones espontneas. La probabi- primitivas que se fusionaron simbiticamente con una clula pri-
Comentarios TEST

Pregunta 42. Translocacin robertsoniana, un portador asintomtico puede tener hijos con trisoma.

M exico A rgentina
C hile U ruguay
CTO Medicina C/ Nez de Balboa, 115 28006 MADRID (Espaa) Tfno.: (91) 782 43 32 / Fax: (91) 782 43 27
E-mail: secretaria@ctomedicina.com; iberocto@ctomedicina.com WEB: www.ctomedicina.com; www.iberocto.com
IG Pg. 7
INMUNOLOGA Y GENTICA
Seguimiento a distancia Preparacin Examen de Seleccin 05/06 1 Vuelta
mordial, tienen ribosomas (70S como los bacterianos) y sntesis Pregunta 42.- R: 2
proteica propia. La traslocacin robertsoniana es una anomala cromosmica es-
En la fecundacin, durante la formacin del cigoto, el ovocito tructural. Puesto que esta patologa ha sido preguntada bastantes ve-
aporta el proncleo femenino y todo el citoplasma de la nueva ces en el MIR, conviene que antes de seguir hagamos una introduc-
clula. Por el otro lado, el espermatozoide slo aporta el proncleo cin que nos sirva de recordatorio.
masculino, por tanto las mitocondrias se heredan siempre de la Las anomalas cromosmicas pueden ser de dos tipos: numricas
madre. (las vimos en la pregunta anterior) y estructurales.
Las alteraciones en el ADN mitocondrial (ejemplo, la neuropata Estas ltimas consisten en una reordenacin de la posicin de los
ptica atrfica de Leber) dan lugar a enfermedades genticas que se genes sobre los cromosomas y las dos variedades ms frecuentes, y por
heredan en lnea directa materna, es decir, una madre enferma tras- tanto preguntables en el MIR, son las deleciones y traslocaciones.
mitir la enfermedad a todos sus hijos e hijas, y un padre enfermo no Delecin: es la prdida de un segmento cromosmico, y por tanto,
se la trasmitir a ninguno (respuesta 4 correcta). de los genes situados (se trata de una monosoma parcial). Si la
delecin es grande o los genes en ella situados son importantes, es
Pregunta 40.- R: 3 incompatible con la vida.
Las trisomas consisten en la existencia de 2n+1 cromosomas, exis- Traslocacin: se produce una delecin en dos cromosomas, y la
tiendo por tanto uno de ms. Cuando afectan a los cromosomas reparacin se hace de modo ineficiente, intercambindose los seg-
sexuales se traducen en retraso mental y alteraciones en la conducta, mentos. Se la denomina tambin traslocacin balanceada o rec-
y cuando afectan a los autosomas producen gravsimas malformacio- proca.
nes congnitas que suelen impedir el normal desarrollo embrionario. La traslocacin robertsoniana es una situacin intermedia entre las
Se han observado trisomas de todos los pares excepto del 1. Slo anomalas numricas y estructurales, se produce por la fusin de dos
pueden llegar al final de la gestacin y sobrevivir al parto una pequea cromosomas acrocntricos (se trasloca el cromosoma entero).
proporcin de los afectos de trisomas de los pares 13 y 18 y una La respuesta 1 es falsa porque, al fusionarse los dos cromosomas, el
proporcin algo mayor de los del par 21. No obstante, los primeros portador de la traslocacin tendr 45 (en realidad, un cromosoma es
sobreviven slo algunas horas o das y a la edad adulta slo llegan los doble). El portador ser fenotpicamente normal (no falta ningn gen),
pacientes del Sndrome de Down (21). pero tras la meiosis sus gametos tendrn la informacin gentica de 22
Como sntomas ms destacados de las principales trisomas, com- o 24 cromosomas, dependiendo de si lleven o no el cromosoma
pletas o parciales, citemos: doble, pudiendo dar lugar, tras la fecundacin, a embriones
Sndrome de Edwards: el pie en mecedora y el occipucio promi- monosmicos o trismicos.
nente (respuesta 1, falsa). La respuesta 3 es falsa porque afecta a la prctica totalidad del
Trisoma parcial del 22: coloboma del iris y atresia anal (respuesta cromosoma. La respuesta 4 es falsa porque la enfermedad del maulli-
2, falsa). do de gato es una microdelecin. La 5 es falsa porque no es una
Trisoma del 13: polidactilia y labio leporino (respuesta 3, verda- duplicacin, sino una fusin.
dera) y tambin dextrocardia y microftalma.
La Trisoma del 21 (sndrome de Down): no necesita descripcin Pregunta 43.- R: 4
(respuesta 4, falsa). Conviene recordar el concepto de disoma uniparental: la per-
La respuesta 5 (sndrome del maullido de gato) no corresponde a sona que tiene la disoma ha heredado las dos copias de un cromo-
una trisoma sino a una monosoma (falta de un cromosoma) parcial. soma concreto solamente de uno de sus progenitores, y no hereda el
Las monosomas puras, con excepcin del sndrome de Turner (cro- cromosoma homlogo del otro padre. Tendr, por tanto, un carioti-
mosoma X), son incompatibles con la vida. Entre las trisomas parcia- po normal: 46 cromosomas, 24 procedentes de un progenitor y 22
les preguntables en el MIR estn la enfermedad del maullido de gato del otro. Si en estos cromosomas no existen genes con imprinting no
(cromosoma 5) y el sndrome de Di George (cromosoma 22). habr ningn problema: existen casos en los que el padre le trasmite
al hijo varn los cromosomas X e Y y la madre no trasmite el cromoso-
Pregunta 41.- R: 5 ma X. Estos casos slo se detectan en las muy poco frecuentes situacio-
El nmero euploide de cromosomas de la especie humana es 46 nes en las que el padre padece una enfermedad ligada al X, como la
(diploide). Las anomalas cromosmicas numricas suponen una va- hemofilia A, y que, debido a la disoma uniparental, se trasmite de
riacin (ganancia o prdida) del nmero euploide: padres enfermos a hijos varones.
Se dice que hay poliploida cuando la clula tiene un nmero de Pero si algunos de los genes situados en el cromosoma estn

Comentarios TEST
cromosomas diferente de 46, pero mltiplo de 23 (triploide, 69; sujetos a imprinting gnico, la cosa se complica.
tetraploide, 92). El imprinting consiste en que, de determinados genes, situados en
Por otro lado, se dice que existe aneuploida cuando una clula cualquier cromosoma, solamente se usa la copia procedente de uno
tiene un nmero de cromosomas distinto del euploide, pero que de los dos progenitores y la otra copia se inactiva.
no es mltiplo de 23. Son pues aneuploidas las trisomas (ganan- Las dos enfermedades ms caractersticas (y ms preguntables) de-
cia de un cromosoma) y las monosomas (prdida de un cromoso- bidas a la alteracin de un gen con imprinting son el sndrome de
ma). Prader Willi y el sndrome de Angelman.
Las trisomas, consideradas en conjunto, son las aneuploidas ms En el gen responsable del sndrome de Prader-Willi slo se expresa la
frecuentes en la especie humana; no obstante, y si se considera por copia paterna; si los dos cromosomas son de origen materno, no se
separado a todas las trisomas y monosomas, la aneuploida ms fre- expresar, y por tanto, no aparecer el cuadro clnico. En el sndrome de
cuente es la monosoma del cromosoma X (sndrome de Turner), y en Angelman, por el contrario, slo se expresa la copia materna del gen.
segundo lugar, la trisoma del 16. La respuesta 1 es falsa porque la talasemia es una enfermedad con
A veces nos cuesta entender cmo el Turner, menos frecuente en herencia mendeliana (aunque compleja) y no se debe a disomas. Lo
clnica que el sndrome de Down, sea la aneuploida ms frecuente mismo ocurre con el Huntington (respuesta 2 falsa). El sndrome de
en humanos. Debe tenerse en cuenta que cuando se habla de la Down es una trisoma, no una disoma uniparental (respuesta 3 falsa).
incidencia de estas patologas en la especie humana se considera EL linfoma de Burkitt es una enfermedad compleja en la que se detec-
tambin a los embriones afectados que mueren intratero: la in- tan traslocaciones cromosmicas, no disomas (respuesta 5 falsa).
mensa mayora de los afectados por el Turner no llegan a trmino.
Entendiendo esto tambin nos explicaremos por qu la trisoma ms Pregunta 44.- R: 3
frecuente en la especie humana es la del cromosoma 16, y sin em- En otra pregunta anterior hablamos de la heterogeneidad gentica,
bargo nunca hayamos visto un enfermo con dicha patologa. La en concreto de la heterogeneidad de locus, y quiz nos supo a poco
trisoma del 16 produce tan graves malformaciones que los afecta- porque no qued claro el resto de las heterogeneidades.
dos mueren a las pocas semanas de desarrollo embrionario y ningu- Se denomina heterogeneidad allica (heterogeneidad molecular
no llega a trmino. en algunos textos) a la situacin en la que se producen diferentes

M exico A rgentina
Pg. 8 IG C hile U ruguay
CTO Medicina C/ Nez de Balboa, 115 28006 MADRID (Espaa) Tfno.: (91) 782 43 32 / Fax: (91) 782 43 27
E-mail: secretaria@ctomedicina.com; iberocto@ctomedicina.com WEB: www.ctomedicina.com; www.iberocto.com
INMUNOLOGA Y GENTICA
Preparacin Examen de Seleccin 05/06 1 Vuelta Seguimiento a distancia
mutaciones en un mismo gen (locus) que producen todas ellas un El raquitismo resistente a la vitamina D se hereda ligado al X domi-
mismo fenotipo enfermo, aunque la mutacin sea en zonas distintas nante (respuesta 2 falsa).
del gen (como en la fibrosis qustica) o bien varios fenotipos distintos La corea de Huntington se hereda de modo autosmico dominan-
de la misma enfermedad (ejemplo: las betatalasemias). te (respuesta 3 falsa).
La heterogeneidad clnica (respuesta 1 falsa) consiste en que dife- La enfermedad de Hunter tiene un patrn de herencia ligado al
rentes mutaciones en un mismo gen (locus) se traducen en enfermeda- cromosoma X recesivo (respuesta 4 falsa).
des completamente distintas (ejemplo: gen de la distrofina y las distro-
fias musculares de Duchenne y Becker). Pregunta 49.- R: 3
La heterogeneidad de locus (respuesta 2 falsa) consiste en que Una traslocacin robertsoniana es una anomala cromosmica
mutaciones en genes diferentes dan lugar a un cuadro clnico igual, se estructural que puede originar una anomala numrica. Por tanto se
explic anteriormente. El imprinting gnico (respuesta 4 falsa) da pa- trata de una situacin mixta entre ambos tipos de anomalas.
tologas slo en las disomas uniparentales (no es este el caso). Se produce por la fusin de dos cromosomas acrocntricos en la
La fenocopia (respuesta 5 falsa) es una enfermedad ambiental que que los brazos largos de ambos cromosomas quedan fusionados (res-
es clnicamente muy parecida a otra de origen gentico. puesta 1 cierta). Esto quiere decir que, tras la fusin, el cromosoma,
originalmente con forma de V, pasa a tener forma de X (dos V
Pregunta 45.- R: 1 fusionadas).
De los patrones de herencia de enfermedad, el ms frecuente es el Los sujetos portadores asintomticos de la traslocacin tienen 45
autosmico dominante. Tngase en cuenta que la probabilidad de cromosomas (uno de ellos en realidad es doble).
que un enfermo con un cuadro autosmico dominante transmita la Los gametos que producen los portadores de la translocacin tras
enfermedad a un hijo es del 50 %. la fecundacin pueden dan lugar a trisomas o monosomas de un
Sin embargo, en las autosmicas recesivas, la probabilidad de trans- cromosoma completo (respuesta 3 FALSA). En torno al 5 % de los
mitir la enfermedad es muy baja, el enfermo transmite el gen alterado casos de sndrome de Down se producen por este mecanismo (res-
pero para que aparezca la enfermedad, el hijo debe recibir tambin puesta 2 cierta), los enfermos presentan 46 cromosomas, uno de ellos
una copia alterada del otro progenitor (un enfermo solo tendr hijos doble, y es muy importante detectar esos casos de cara al consejo
sanos si su pareja es sana). gentico: la probabilidad de recurrencia de un hijo con sndrome de
Down en una pareja no portadora de la traslocacin es alrededor de
Pregunta 46.- R: 5 1:700 (similar a la de tener un hijo con Down en la poblacin gene-
Un gen tiene una herencia ligada al sexo cuando su locus (el lugar ral) sin embargo, en los portadores la probabilidad es bastante mayor
que ocupa en el genoma) est situado en uno de los cromosomas (respuestas 4 y 5 ciertas) y se puede fijar en cerca del 10 % (vara segn
sexuales, el X o el Y. los estudios).
En los dos cromosomas sexuales existe un segmento homlogo,
igual para ambos y que se recombina en la meiosis, y un segmento Pregunta 50.- R: 1
diferencial o heterlogo (no se recombina). La enfermedad de Marfan, como casi todas las que se deben a
Si el gen est localizado en la regin homloga, tambin denomi- alteracin de una protena estructural, se hereda de modo autosmi-
nada pseudoautosmica, el carcter por l codificado est ligado co dominante. En la mayor parte de los casos observados, el alelo
parcialmente al sexo porque en la meiosis puede pasar de un cromo- causante de la enfermedad se hereda de padre o madre enfermos
soma a otro. (respuesta 1 cierta; 2 y 5 falsas) y slo una pequesima proporcin se
Si el gen se sita en la regin diferencial o heterloga, el carcter deben a mutaciones de novo (respuesta 4 falsa).
que codifica estar totalmente ligado al sexo, pues no hay posibilidad Muy importante: en las enfermedades autosmicas dominantes,
de intercambio de material gentico en la meiosis. slo existen enfermos y sanos, no existen los portadores asintomticos
Las respuestas 1, 2 y 3 son falsas porque puede situarse en ambos (respuesta 3 falsa).
cromosomas. La respuesta 4 es un sinsentido. Un nio de 3 aos que ha heredado un gen mutado de un proge-
nitor enfermo y que no tiene clnica, no es un portador, se trata de un
Pregunta 47.- R: 3 enfermo en fase preclnica o silente.
La inactivacin de un cromosoma X en la mujer (EFECTO LYON)
consiste en la inactivacin en las hembras de uno de los dos cromoso-
Comentarios TEST

mas X, el cual se organiza como cromatina sexual o corpsculo de Barr


(respuesta 4 falsa), que consiste en un cuerpo heteropicntico en el
ncleo celular de las clulas femeninas. Se considera que es un meca-
nismo de compensacin de la dosis gnica, en el hombre y la mujer.
La inactivacin no ocurre en la gametognesis (respuesta 1 falsa),
sino en los primeros estadios de la embriognesis, sobre el da 16 de
gestacin. Solo se inactiva el segmento diferencial y el proceso es
irreversible por metilacin del ADN (respuesta 2 falsa) y se mantiene
durante toda la vida.
La inactivacin se realiza al azar en todas las clulas, es decir, que
en unas clulas se inactiva el cromosoma X de origen paterno y en
otras el materno (respuesta 5 falsa). Una vez establecida dicha inacti-
vacin, dando origen a que la hembra sea funcionalmente un mosai-
co para los genes correspondientes al cromosoma X.
El nmero de corpsculos de Barr de una clula es igual al nmero
de cromosomas X inactivados, o lo que es lo mismo, es igual al nme-
ro total de cromosomas X que posea la clula menos uno. As en una
superhembra (XXX) habr dos corpsculos.

Pregunta 48.- R: 5
El sndrome del cromosoma X frgil es una enfermedad en la que el
mecanismo patognico es la inestabilidad en la longitud de un seg-
mento gnico (expansin de secuencias).
La neuritis de Leber presenta un patrn de herencia mitocondrial
(respuesta 1 falsa).

M exico A rgentina
C hile U ruguay
CTO Medicina C/ Nez de Balboa, 115 28006 MADRID (Espaa) Tfno.: (91) 782 43 32 / Fax: (91) 782 43 27
E-mail: secretaria@ctomedicina.com; iberocto@ctomedicina.com WEB: www.ctomedicina.com; www.iberocto.com
IG Pg. 9
NEFROLOGA
Preparacin Examen de Seleccin 05/06 1 Vuelta Seguimiento a distancia

FISIOLOGA. 4) Si es irreversible.
5) Se refiere a la nefropata lpica.
1. Cul es el procedimiento ms adecuado para saber si existe
una disminucin de la funcin renal?: 8. El hallazgo distintivo de la nefropata IgA es:

1) Urografa intravenosa. 1) Aparicin de semilunas.


2) Cifra de urea en sangre. 2) Proliferacin mesangial difusa.
3) Determinar el aclaramiento de creatinina. 3) Depsito mesangial de IgA en la inmunofluorescencia.
4) Determinar la concentracin de creatinina en sangre. 4) ptico normal.
5) Determinar el aclaramiento de urea. 5) Desdoblamiento de membrana basal.

2. Qu sustancia es fundamental en el mecanismo de la 9. El mecanismo ms frecuente de produccin de glomerulo-


autorregulacin glomerular?: nefritis es:

1) IL I. 1) AMBG.
2) IL 6. 2) IC formados in situ.
3) VMA. 3) Activacin del complemento.
4) AG II. 4) IC circulantes.
5) Endotelina. 5) Inmunoglobulinas circulantes.

3. De qu estructura de la nefrona depende la concentracin 10. En el tratamiento del edema del sndrome nefrtico, cul se
de la orina?: emplea en el caso de edema refractario?:

1) Tbulo proximal. 1) Albmina intravenosa.


2) Asa de Henle. 2) Dieta hiperproteica.
3) Tbulo distal. 3) Dieta pobre en calcio.
4) Tubo colector. 4) Diurticos tiacdicos.
5) Vasa recta. 5) Diurticos ahorradores de potasio.

4. La renina se produce en: 11. Qu cuadro infeccioso puede producir la remisin de la GN


de cambios mnimos?:
1) Tbulo proximal.
2) Tbulo distal. 1) TBC.
3) Aparato yuxtaglomerular. 2) Sfilis.
4) Clulas mesangiales. 3) Sarampin.
5) Capilares peritubulares. 4) Brucelosis.
5) HIV.
5. De las siguientes sustancias, cul se utiliza para medir el
flujo plasmtico renal?: 12. Una mujer de 16 aos de edad tiene diuresis escasas y de color
"coca cola". Nueve das antes, tuvo un cuadro de faringoamig-
1) Urea. dalitis. En el sistemtico de orina se objetiva proteinuria de
2) Insulina. 2 g/da. La TA es de 180/120 mmHg. En la exploracin hay
3) cido PAH. edemas. Qu esperara encontrar en la sangre?:
4) Creatinina.
5) Fenolsulfoftalena. 1) Alcalosis metablica.
Preguntas TEST

2) Aumento de LDH.
6. La reabsorcin del magnesio ocurre fundamentalmente en: 3) Hipocomplementemia.
4) Hipopotasemia .
1) Tbulo Proximal. 5) Aumento del CH5O.
2) Tbulo Colector.
3) Asa de Henle. 13. La GN extracapilar Tipo II se caracteriza por:
4) Tbulo Distal.
5) Vasa recta. 1) Cursar con poliuria.
2) Ig E elevada.
3) Llevar un curso doloroso.
GLOMERULONEFRITIS. 4) Ser reversible.
5) Ser hipocomplementmica.
7. Qu entendemos por glomerulonefritis aguda?:
14. El dao originado en los glomrulos en la endocarditis
1) La que cursa con hematuria. infecciosa es?:
2) Si debuta con oligoanuria.
3) Si cursa con acidosis metablica. 1) Mediado por AMBG.
2) Mediado por P-ANCA.

M exico A rgentina CTO Medicina C/ Nez de Balboa, 115 28006 MADRID (Espaa) Tfno.: (91) 782 43 32 / Fax: (91) 782 43 27
C hile U ruguay E-mail: secretaria@ctomedicina.com; iberocto@ctomedicina.com WEB: www.ctomedicina.com; www.iberocto.com
HM Pg. 1
NEFROLOGA
Seguimiento a distancia Preparacin Examen de Seleccin 05/06 1 Vuelta
3) Mediado por C-ANCA. 3) Biopsia renal.
4) Mediado por inmunocomplejos circulantes. 4) Urografa i.v.
5) No se conoce el mecanismo. 5) TC renal.

15. Cuando un proceso glomerular desarrolla un cuadro de 22. En cul de las siguientes entidades es MENOS frecuente la
oligoanuria, en qu pensaramos en primer lugar?: glomerulonefritis rpidamente progresiva?:

1) NTA. 1) Sndrome de Goodpasture.


2) Necrosis tbulointersticial aguda. 2) Enfermedad de Berger.
3) Aparicin de G.N.R.P. 3) Crioglobulinemia mixta esencial.
4) Vasculitis asociada. 4) Sepsis visceral oculta.
5) Amiloidosis. 5) Granulomatosis de Wegener.

16. Ante un sndrome nefrtico que no responde a corticoides 23. En un paciente con carcinoma de pulmn y sndrome
en un nio, qu pensaramos?: nefrtico, qu cuadro glomerular considera usted ms
probable?:
1) Es lo habitual.
2) Hay que abandonar los corticoides. 1) Glomerulonefritis endocapilar.
3) Hay que hacer dilisis. 2) Sndrome nefrtico idioptico.
4) Que puede tener una hialinosis focal y segmentaria. 3) Glomerulonefritis membranosa.
5) Nunca ocurre. 4) Nefropata lpica.
5) Glomerulonefritis de la endocarditis infecciosa.
17. La glomerulonefritis inmunotactoide:
24. Un varn acude al Servicio de Urgencias por cuadro de
1) No existe. orinas oscuras. Tiene faringitis aguda. Interrogado, refiere
2) Es propia de la infancia. un episodio similar hace 1 ao. La diuresis es de cuanta
3) Se asocia a hipopotasemia. normal, y la cifra de creatinina en sangre es de 1,3 mg/dl. En
4) Suele cursar con proteinuria. qu entidad pensara en primer lugar?:
5) Tiene buen pronstico.
1) GN endocapilar difusa.
18. Una afectacin glomerular focal quiere decir: 2) Nefropata mesangial IgA.
3) GN rpidamente progresiva.
1) Se afectan unos glomrulos s y otros no. 4) GN membranoproliferativa.
2) Se afecta una parte de glomrulos s y otra no. 5) GN membranosa.
3) Se afectan los glomrulos corticales.
4) Se afectan los glomrulos yuxtamedulares. 25. La lipodistrofia es ms frecuente en?:
5) Se afectan los glomrulos ms alejados de la corteza.
1) Glomerulonefritis membranosa.
19. En la GN rpidamente progresiva por anticuerpos antimem- 2) Glomerulonefritis endocapilar difusa.
brana basal, la inmunofluorescencia nos presenta: 3) Glomerulonefritis membranoproliferativa tipo I.
4) Glomerulonefritis membranoproliferativa tipo II.
1) Depsitos lineales de IgG. 5) Glomerulonefritis lpica.
2) Depsitos granulosos de IgG.
3) Depsitos granulosos de IgA. 26. En un varn de 20 aos sin evidencia de enfermedad
4) Depsitos lineales de IgM. sistmica, que presenta sndrome nefrtico, microhematu-

Preguntas TEST
5) Depsito de C3. ria, leve insuficiencia renal y niveles bajos de complemento
(C3) en suero, el diagnstico histolgico ms probable es:
20. El pronstico de una GN aguda postestreptoccica es:
1) Glomerulonefritis mesangiocapilar o membranopro-
1) Peor en varones que en mujeres. liferativa.
2) Peor en nios que en adultos. 2) Glomerulonefritis mesangial IgA.
3) Mejor en nios que en adultos. 3) Sndrome nefrtico por mnimos cambios.
4) No vara con la edad. 4) Glomeruloesclerosis focal.
5) Peor en mujeres que en varones. 5) Glomerulonefritis membranosa.

21. Un varn de 40 aos acude por presentar edemas en piernas 27. La glomerulonefritis esclerosante focal y segmentaria con
y astenia. En la analtica, se objetiva un aclaramiento de hialinosis puede aparecer sobre todo en la evolucin de qu
creatinina del 20%. La proteinuria es rica en albmina y de proceso:
3,5 g/da. Qu hara en primer lugar en este paciente?:
1) Sndrome de Bartter.
1) Inmunosupresin. 2) Acidosis tubular renal tipo I.
2) Corticoterapia. 3) Nefropata tubulointersticial crnica, asociada a reflujo
vesicoureteral.

M exico A rgentina CTO Medicina C/ Nez de Balboa, 115 28006 MADRID (Espaa) Tfno.: (91) 782 43 32 / Fax: (91) 782 43 27
Pg. 2 HM C hile U ruguay E-mail: secretaria@ctomedicina.com; iberocto@ctomedicina.com WEB: www.ctomedicina.com; www.iberocto.com
NEFROLOGA
Preparacin Examen de Seleccin 05/06 1 Vuelta Seguimiento a distancia

4) Cristalizacin intratubular de cido rico. 3) Hay depsitos aislados de IgA en el glomrulo.


5) Tratamiento con captopril. 4) Se trata con bolos de ciclofosfamida.
5) Se suele presentar con sndrome nefrtico recidivante.
28. El sndrome nefrtico de la nefropata membranosa se
caracteriza tpicamente: 35. Mujer de 16 aos, con edemas maleolares y periorbitarios,
proteinuria de 4 g/da, numerosos hemates y cilindros en el
1) Por el depsito granular de IgE en el examen de inmu- sedimento, C3 40 mg/dl, C4 5 mg/dl. La nefropata ms
nofluorescencia. probable es:
2) Por responder frecuentemente al tratamiento con esteroides.
3) Por ser ms frecuente a partir de los 50 aos de edad. 1) Glomerulonefritis rpidamente progresiva.
4) Por evolucionar rpidamente a la insuficiencia renal, en 2) Glomerulonefritis membranosa.
la mayora de los casos. 3) Nefropata lpica.
5) Por ser reversible en el 50 % de los casos. 4) Amiloidosis renal.
5) Nefritis intersticial por drogas.
29. La presencia de cilindros hemticos en el sedimento urinario
sugiere: 36. Una mujer de 28 aos presenta, de forma aguda, nuseas,
vmitos, oliguria y macrohematuria. Tiene edemas, TA 180/
1) Coagulopata. 110, urea 80 mg/dl, proteinuria y cilindros hemticos en la
2) Sndrome nefrtico. orina. Esta paciente tiene:
3) Glomerulonefritis aguda.
4) Pielonefritis crnica. 1) Sndrome nefrtico.
5) Fracaso renal agudo. 2) Nefritis lpica.
3) Pielonefritis.
4) Necrosis tubular aguda.
30. Un varn con sndrome nefrtico e hipocomplementemia
5) Sndrome nefrtico.
tendr, como primera posibilidad:
37. De las siguientes glomerulonefritis, cul puede recurrir en
1) GN mesangial.
horas en el rin trasplantado?:
2) GN membranosa.
3) GN membranoproliferativa.
1) Sndrome nefrtico por lesiones mnimas.
4) GN mesangial IgA.
2) GN proliferativa mesangial.
5) Hialinosis focal y segmentaria. 3) GN esclerosante focal y segmentaria.
4) GN membranosa.
31. Son caractersticas de la glomerulonefritis postestreptoc- 5) GN membranoproliferativa.
cica todas, EXCEPTO:
38. En la glomerulonefritis aguda, la anemia de la fase aguda es
1) Depsitos granulares glomerulares de IgG y C3 en IF. secundaria a:
2) Est mediada por inmunocomplejos.
3) Proliferacin endocapilar en los glomrulos. 1) Hemlisis.
4) La hematuria microscpica puede persistir durante meses. 2) Dilucional.
5) Hipocomplementemia persistente. 3) Ausencia de eritropoyetina.
4) Hemorragia.
32. La nefropata membranosa puede remitir espontneamente: 5) Hiperesplenismo.

1) Pero las remisiones no son nunca completas. 39. La enfermedad de Hodgkin se asocia frecuentemente a:
2) En nios.
3) En el 20-30% de los casos. 1) Nefropata mesangial.
Preguntas TEST

4) En ms del 80% de los casos. 2) GN membranosa.


5) Prcticamente nunca. 3) GN lpica.
4) Nefrosis lipoidea.
33. Un varn de 60 aos de edad tiene edemas con hipoalbu- 5) Enfermedad de Wegener.
minemia. El aclaramiento de creatinina es del 30%. Las cifras
de complemento son normales. De las etiologas secunda- 40. Cul es el mecanismo de produccin ms frecuente en la
rias propuestas, en qu entidad pensara probablemente?: glomerulonefritis aguda?:

1) Colagenosis. 1) Directa por virus.


2) Vasculitis. 2) Directa por inmunoglobulinas.
3) Neoplasias. 3) Inmunocomplejos circulantes.
4) Mieloma mltiple. 4) Por anticuerpos antimembrana basal glomerular.
5) Enfermedad inflamatoria intersticial. 5) Inmunocomplejos formados "in situ".

34. Respecto a la nefropata IgA, es cierto que: 41. Cul es la glomerulopata en la que los glomrulos afecta-
dos son los de las nefronas yuxtamedulares?:
1) Cursa con hipocomplementemia.
2) Ocasionalmente, tiene incidencia familiar. 1) Extracapilar difusa.

M exico A rgentina CTO Medicina C/ Nez de Balboa, 115 28006 MADRID (Espaa) Tfno.: (91) 782 43 32 / Fax: (91) 782 43 27
C hile U ruguay E-mail: secretaria@ctomedicina.com; iberocto@ctomedicina.com WEB: www.ctomedicina.com; www.iberocto.com
HM Pg. 3
NEFROLOGA
Seguimiento a distancia Preparacin Examen de Seleccin 05/06 1 Vuelta
2) Hialinosis segmentaria y focal. 4) Oliguria.
3) Berger. 5) Eosinofiluria.
4) Epimembranosa.
5) Exudativa difusa. 48. El modo ms habitual de presentarse clnicamente una
nefropata intersticial es:
42. Una mujer de 30 aos de edad tiene unas cifras de creatinina
srica de 3 mg/dl. En el sedimento de orina aparece un sedimen- 1) HTA maligna.
to rico en todo tipo de cilindros, con una proteinuria rica en 2) Hematuria macroscpica recidivante.
albmina y microhematuria. La cifra de C3 en sangre es de 3 mg/ 3) Sndrome nefrtico.
dl sin oliguria. Qu esperara encontrar en la biopsia renal?: 4) Poliuria y acidosis metablica.
5) Proteinuria mayor de 5 g/da.
1) Nefropata de cambios mnimos.
2) Glomerulonefritis membranoproliferativa. 49. En la nefropata intersticial crnica, la infiltracin intersticial
3) Glomerulonefritis mesangial IgA. es fundamentalmente a base de:
4) Glomerulonefritis endocapilar difusa.
5) Glomerulonefritis extracapilar. 1) Linfocitos y clulas plasmticas.
2) Eosinfilos.
43. Una mujer de 30 aos de edad acude al mdico por cuadro 3) Monocitos.
de orinas oscuras. En la exploracin, se objetiva una asime- 4) Fibroblastos.
tra corporal, con prdida de panculo adiposo en el miem- 5) Polimorfonucleares.
bro inferior izquierdo. En el sedimento hay hematuria y
proteinuria, y las cifras de complemento son bajas. Cul 50. La nefropata por analgsicos crnica es ms frecuente en:
sera su diagnstico?:
1) En nios menores de 2 aos.
1) Glomerulonefritis endocapilar difusa. 2) En varones adolescentes.
2) Glomerulonefritis membranoproliferativa tipo I. 3) En adultos que consumen cocana.
3) Glomerulonefritis membranoproliferativa tipo II. 4) Slo en adultos con isquemia renal.
4) Glomerulonefritis extracapilar. 5) En mujeres entre los 20 y los 40 aos.
5) Glomerulonefritis mesangial IgA.
51. El primer dato patolgico detectable en la pielonefritis
crnica es:
NEFROPATAS INTERSTICIALES.
1) Disminucin en la capacidad de concentracin.
44. En el dao intersticial?: 2) Proteinuria superior a 3 g/da.
3) Glucosuria.
1) Hay hematuria. 4) Fosfaturia.
2) Hay proteinuria rica en albmina. 5) Presencia de un aumento de urea mayor que el aumento
3) Hay hipocalciuria. de creatinina.
4) Hay poliuria.
5) Hay hipomagnesemia. 52. La nefropata por analgsicos es una enfermedad:

45. Cul de las siguientes etiologas de nefropata tbulointers- 1) Reversible.


ticial desarrolla ms precozmente HTA?: 2) Idiosincrsica.
3) De etiologa inmunitaria.
1) Rin hipocalimico. 4) Que produce necrosis de papila.
2) Rin qustico medular. 5) Que nunca lleva al trasplante.

Preguntas TEST
3) Rin gotoso.
4) Rin poliqustico. 53. Un varn de 60 aos de edad est diagnosticado de nefropata
5) Nefropata de Bartter. intersticial por plomo. Acude a urgencias por estar edematoso,
y tiene proteinuria en rango nefrtico. Qu pensaramos?:
46. En cul de las siguientes patologas hay mayor incidencia de
uroteliomas?: 1) Tiene amiloide renal.
2) Hialinosis focal y segmentaria.
1) Nefropata por analgsicos crnica. 3) Nefropata membranosa.
2) Hialinosis focal. 4) Adenocarcinoma renal.
3) Nefropata hipopotasmica. 5) Cambios mnimos.
4) Nefropata membranosa.
5) Cambios mnimos. 54. Un varn de 30 aos de edad presenta unas cifras de
creatinina en sangre de 3 mg/dl. Refiere poliuria y nicturia.
47. La pielonefritis crnica produce principalmente: Qu esperara encontrar en la analtica sangunea?:

1) Glomerulopata. 1) Hiperpotasemia.
2) Afectacin intersticial. 2) Hipercalcemia.
3) Tubulopata proximal. 3) Hipernatremia.

M exico A rgentina CTO Medicina C/ Nez de Balboa, 115 28006 MADRID (Espaa) Tfno.: (91) 782 43 32 / Fax: (91) 782 43 27
Pg. 4 HM C hile U ruguay E-mail: secretaria@ctomedicina.com; iberocto@ctomedicina.com WEB: www.ctomedicina.com; www.iberocto.com
NEFROLOGA
Preparacin Examen de Seleccin 05/06 1 Vuelta Seguimiento a distancia

4) Acidosis metablica. 61. En las nefropatas intersticiales crnicas:


5) Hipocloremia.
1) No hay proteinuria.
55. Cul de los siguientes cuadros se asocia con la presencia de 2) No hay nunca HTA.
uroteliomas?: 3) Hay hipernatremia.
4) Puede haber hipopotasemia.
1) Nefropata por plomo. 5) Hay alcalosis metablica.
2) Nefropata por cido rico.
3) L.E.S. 62. La pielonefritis xantogranulomatosa:
4) Nefronoptisis.
5) Nefropata de los Balcanes. 1) No existe.
2) Es de etiologa autoinmune.
56. Cul de estas patologas se asocia ms frecuentemente con 3) Puede confundirse con una neoplasia renal.
HTA?: 4) Nunca produce fiebre.
5) Nunca requiere ciruga.
1) Sndrome de Liddle.
2) Sndrome de Bartter.
3) Acidosis tubular tipo II. ENFERMEDADES SISTMICAS.
4) Nefropata de los Balcanes.
5) Nefropata intersticial por cido rico. 63. Uno de los siguientes cuadros suele cursar con hematuria
macroscpica recurrente:
57. Un varn de 40 aos, de profesin fontanero, refiere un
cuadro consistente en poliuria y astenia. En la analtica, hay
1) Sndrome de Alport.
acidosis metablica hiperclormica. Qu esperara encon-
2) Nefronoptisis.
trar en el sedimento?:
3) Bartter.
4) Glomerulonefritis diabtica.
1) Cilindros hemticos.
5) Amiloidosis.
2) Sedimento telescopado.
3) Cilindros hialinos.
64. En cul de los siguientes procesos puede haber una protei-
4) Cristales de estruvita.
nuria mayor de 4 g/da?:
5) Cilindros anchos y creos.

58. Cul es el microorganismo causal ms frecuente de las 1) Nefropata por analgsicos.


pielonefritis?: 2) Intoxicacin por DDT.
3) Sndrome de Bartter.
1) Yersinia spp. 4) Nefropata de los Balcanes.
2) Proteus mirabilis. 5) Mieloma mltiple.
3) Klebsiella pneumoniae.
4) E. coli. 65. La enfermedad de Alport-Perkoff cursa con:
5) Enterococo.
1) Sordera y alteraciones oculares y renales.
59. Un paciente con antecedentes de podagra tiene un cuadro 2) Distrofia de uas y displasia de rodilla.
de acidosis metablica, acompaado de intensa poliuria y 3) Riones poliqusticos, HTA e insuficiencia renal.
sed. En el sedimento de orina aparecen gran cantidad de 4) Sndrome de Fanconi y cataratas.
cristales de cido rico. Qu esperara encontrar en la 5) Amiloidosis y polineuropata.
exploracin de este paciente?:
Preguntas TEST

66. La nefropata por depsito intratubular de cido rico se


1) Dolor abdominal. puede ver en:
2) Poliglobulia.
3) HTA. 1) Gota primaria.
4) Orinas oscuras. 2) Sndrome de Lesch-Nyhan.
5) Masa abdominal. 3) Hipercalcemia.
4) Linfoma tratado con antiblsticos.
60. Un varn de 40 aos de edad tiene un LES con escasa 5) Hiperprolactinemia.
actividad desde hace varios aos. En el sedimento de orina
aparecen cilindros mixtos. Consulta, porque en los ltimos 67. La manifestacin clnica cardinal de la nefropata diabtica
2 meses, refiere abundante poliuria y nicturia. Qu espe- es la presencia de:
rara encontrar en el sedimento de orina de este paciente?:
1) Hematuria.
1) Eosinofiluria. 2) Proteinuria.
2) Sndrome nefrtico. 3) Piuria.
3) Leucocituria. 4) Sndrome nefrtico.
4) Cadenas ligeras. 5) Insuficiencia renal.
5) pH urinario de 9.

M exico A rgentina CTO Medicina C/ Nez de Balboa, 115 28006 MADRID (Espaa) Tfno.: (91) 782 43 32 / Fax: (91) 782 43 27
C hile U ruguay E-mail: secretaria@ctomedicina.com; iberocto@ctomedicina.com WEB: www.ctomedicina.com; www.iberocto.com
HM Pg. 5
NEFROLOGA
Seguimiento a distancia Preparacin Examen de Seleccin 05/06 1 Vuelta
68. La lesin ms frecuentemente encontrada en la diabetes es: 2) Cadmio.
3) Flor.
1) Glomeruloesclerosis intercapilar difusa. 4) Metano.
2) Glomeruloesclerosis nodular. 5) Etanol.
3) Necrosis papilar.
4) Infeccin urinaria. 75. La lesin glomerular ms frecuentemente encontrada en el
5) Nefritis intersticial crnica. SIDA es:

69. Un varn de 50 aos de edad es estudiado por haber 1) Cambios mnimos.


presentado un episodio de hemoptisis y hematuria autolimi- 2) Glomerulonefritis membranoproliferativa.
tada. La cifra de complemento es normal. Hay disminucin de 3) Glomerulonefritis membranosa.
la funcin renal y anticuerpos antimieloperoxidasa en plasma 4) Glomerulonefritis segmentaria y focal.
(con patrn p-ANCA). Cul es la primera posibilidad?: 5) Glomerulonefritis proliferativa difusa.

1) Crioglobulinemia mixta esencial. 76. Un hombre de 56 aos ingresa para realizar el estudio de un
sndrome nefrtico, presentando durante su estancia dolor
2) LES.
sbito en regin lumbar, con aumento claro de la proteinu-
3) Enfermedad de Schnlein-Henoch.
ria, hematuria y clnica asociada de dolor torcico y disnea.
4) PAN microscpica.
Cul sera el diagnstico que habra que sospechar?:
5) Linfangioleiomiomatosis.
1) Embolia de la arteria renal.
70. Una enferma, con artritis reumatoide de veinte aos de 2) Aneurisma disecante artico, a nivel de arterias renales.
evolucin, que nunca ha recibido sales de oro, desarrolla 3) Trombosis de la vena renal.
proteinuria y sndrome nefrtico. Su diagnstico de sospe- 4) Clico renal, con bacteriemia y afectacin pulmonar
cha sera: secundaria.
5) Sndrome de Goodpasture.
1) Sndrome nefrtico idioptico.
2) GN membranoproliferativa. 77. Cul es la vasculitis que afecta a arterias de mediano calibre?:
3) Vasculitis.
4) Amiloidosis. 1) Schnlein-Henoch.
5) Nefropata diabtica por esteroides. 2) Angetis por hipersensibilidad.
3) Crioglobulinemia mixta esencial.
71. Un varn de 50 aos tiene un episodio de hemoptisis. En el 4) Panarteritis nodosa.
sedimento de orina hay hematuria. Se realiza una Rx de trax, 5) Enfermedad de Wegener.
encontrndose infiltrados cavitados. Entre sus antecedentes
figura rinitis de repeticin. Qu prueba pedira en primer lugar?: 78. En el tratamiento de la proteinuria de la nefropata diabtica:

1) Crioglobulinas. 1) Hay que dar betabloqueantes.


2) AMBG. 2) Los diurticos son de eleccin.
3) ANA. 3) Los IECAs pueden revertir la microalbuminuria.
4) C-ANCA. 4) No tiene tratamiento.
5) P-ANCA. 5) Se dan bloqueantes de los canales del calcio.

72. Si un paciente presenta dolor lumbar intenso, hematuria 79. Un hallazgo casi patognomnico de nefropata lpica es:
microscpica y aumento de las cifras de LDH y GOT en plasma.
Hay que pensar como primera posibilidad diagnstica?: 1) Asas de alambre.

Preguntas TEST
2) Depsitos subendoteliales de inmunoglobulinas.
1) Diseccin artica abdominal. 3) Depsitos intramembranosos de anti-ADN nativo.
2) Uropata obstructiva. 4) Engrosamiento de la membrana basal por depsitos de ADN.
3) Embolismo de la arteria renal. 5) Cuerpos de hematoxilina.
4) Glomerulonefritis evolucionada.
80. Qu cuadro histopatolgico lpico renal tiene PEOR pro-
5) Trombosis aguda de la vena renal.
nstico?:
73. Qu es correcto en la enfermedad de Alport?:
1) GN proliferativa focal lpica.
2) GN membranosa lpica.
1) Es adquirida.
3) GN cambios mnimos lpica.
2) Hay un dficit de colgeno tipo III. 4) GN proliferativa difusa.
3) El varn no padece la enfermedad. 5) GN mesangial lpica.
4) Puede ser causa de sndrome nefrtico.
5) Recidiva en el trasplante renal. 81. Un varn de 60 aos tiene antecedentes de bronquiectasias
desde su juventud. Es estudiado por sndrome constitucio-
74. Cul de estas sustancias se asocia al adenocarcinoma renal?: nal, y presenta proteinuria de 4 g/ da, rica en albmina. En
qu proceso pensara inicialmente?:
1) Mercurio.

M exico A rgentina CTO Medicina C/ Nez de Balboa, 115 28006 MADRID (Espaa) Tfno.: (91) 782 43 32 / Fax: (91) 782 43 27
Pg. 6 HM C hile U ruguay E-mail: secretaria@ctomedicina.com; iberocto@ctomedicina.com WEB: www.ctomedicina.com; www.iberocto.com
NEFROLOGA
Preparacin Examen de Seleccin 05/06 1 Vuelta Seguimiento a distancia
1) Nefropata de cambios mnimos. 1) Ac anti-MBG.
2) Glomerulonefritis mesangiocapilar. 2) HLA-A8.
3) Amiloidosis. 3) C-ANCA.
4) Mieloma mltiple. 4) P-ANCA.
5) PAN. 5) HLA-B12.

82. Cul es el tratamiento de eleccin en la afectacin grave y 89. Qu es caracterstico en la afectacin renal de la enferme-
rpida del rin?: dad de Wegener?:

1) Corticoides. 1) Eosinofiluria.
2) Ciclosporina A. 2) Granulomas en el sedimento de orina.
3) Plasmafresis. 3) Piuria intensa.
4) Dilisis. 4) Hematuria y proteinuria asintomticas.
5) Interfern. 5) Clulas lipoideas en el sedimento.

83. Cul de los siguientes sntomas o signos mejora con la 90. La manifestacin ms frecuente de la afectacin renal de la
hemodilisis?: amiloidosis es:

1) Prurito. 1) Hematuria.
2) Hipertrigliceridemia. 2) Beta2-microglobulinas en orina.
3) Hipercolesterolemia. 3) Albuminuria.
4) Anorexia. 4) Acidosis metablica.
5) Calcifilaxia. 5) Hipercalciuria.

84. Qu hematocrito mnimo es el ideal en pacientes con 91. En el sedimento del lupus, es caracterstico:
insuficiencia renal crnica?:
1) Cilindros anchos.
1) 45%. 2) Cilindros piricos.
2) 33%. 3) Eosinofiluria.
3) 20%. 4) Sedimento telescopado.
4) 25%. 5) Cilindros hialinos.
5) 50%.
92. Un paciente acude a consulta por cuadro de eliminacin de
85. Qu manifestaciones extrarrenales son propias de la granu- orinas oscuras. Tiene febrcula y eliminacin de esputos
lomatosis de Wegener?: hemoptoicos. En la Rx de trax aparecen infiltrados pulmo-
nares no cavitados. En la anamnesis, refiere aparicin de
1) Hemorragias digestivas. fenmeno de Raynaud, y lesiones purpricas en piernas de
2) Rinitis. forma ocasional. En qu proceso pensara?:
3) Cardiopata.
4) Lesiones ulceradas en dedos. 1) LES.
5) Fenmeno de Raynaud. 2) Crioglobulinemia mixta esencial.
3) PAN.
4) Enfermedad de Wegener.
86. La asociacin con trombosis de la vena renal es un dato
5) Sndrome de Goodpasture.
frecuente en:

1) Esclerodermia.
INSUFICIENCIA RENAL.
Preguntas TEST

2) Toxemia gravdica.
3) Panarteritis nodosa. 93. La causa ms frecuente de insuficiencia renal aguda es?:
4) Amiloidosis.
5) Enfermedad de Wegener. 1) Deshidratacin.
2) Necrosis cortical bilateral.
87. Las imgenes aneurismticas renales son propias de: 3) Colagenosis.
4) Nefropata por analgsicos aguda.
1) Lupus eritematoso diseminado. 5) NTA.
2) Sndrome de Goodpasture.
3) Amiloidosis renal. 94. En una hiperpotasemia aguda, cul es la primera medida
4) Panarteritis nodosa. teraputica?:
5) Ninguna de las anteriores.
1) Mg++ oral.
88. Un varn de 35 aos de edad acude a Urgencias por 2) Resn calcio rectal.
hemoptisis. La Rx de trax revela un patrn reticulonodular 3) Fsforo oral.
en bases pulmonares. El enfermo tiene febrcula y artromial- 4) Calcio intravenoso.
gias. En la orina hay microhematuria. Qu prueba diagns- 5) Dilisis.
tica pedira en primer lugar?:

M exico A rgentina CTO Medicina C/ Nez de Balboa, 115 28006 MADRID (Espaa) Tfno.: (91) 782 43 32 / Fax: (91) 782 43 27
C hile U ruguay E-mail: secretaria@ctomedicina.com; iberocto@ctomedicina.com WEB: www.ctomedicina.com; www.iberocto.com
HM Pg. 7
NEFROLOGA
Seguimiento a distancia Preparacin Examen de Seleccin 05/06 1 Vuelta
95. En enfermos agudamente oligricos, lo fundamental en el 102. El sndrome de Bartter presenta todo, EXCEPTO:
tratamiento es:
1) Hipopotasemia crnica.
1) Corregir la hipovolemia. 2) Hipertrofia del aparato yuxtaglomerular.
2) Digitalizar. 3) Aumento de aldosterona.
3) Restringir el aporte proteico. 4) Acidosis hiperclormica.
4) Administrar furosemida. 5) Alcalosis hipoclormica.
5) Restringir el aporte lquido.
103. El sndrome de Gitelman:
96. Es causa de fracaso renal agudo, con excrecin fraccional
de sodio alta: 1) Cursa con acidosis metablica.
2) Afecta a los glomrulos.
1) GN postestreptoccica. 3) No produce nefrocalcinosis.
2) NTA. 4) Produce HTA.
3) PAN. 5) Cursa con hiperpotasemia.
4) Shock hipovolmico.
5) Primeras horas del FRA por obstruccin. 104. En el sndrome de Bartter:

97. Los frmacos ms frecuentemente implicados en la nefro- 1) Hay hipoaldosteronismo.


pata tubulointersticial por frmacos, son: 2) Hay hipermagnesemia.
3) Puede haber hipernatremia.
1) Macrlidos. 4) Hay aumento de PGE circulante.
2) Tetraciclinas. 5) Siempre hay HTA.
3) Betalactmicos.
4) Sulfamidas.
5) Lincosaminas.

98. La causa ms frecuente de FR agudo intrnseco es:

1) La necrosis tubular aguda.


2) LES.
3) HTA.
4) Pielonefritis.
5) La necrosis cortical.

NEFROPATAS HEREDITARIAS.

99. Cul es el sntoma ms frecuente de la poliquistosis renal?:

1) Dolor en flanco.
2) Hematuria macroscpica.
3) Nicturia.
4) Expulsin de clculos.
5) Disuria.

Preguntas TEST
100. El sndrome de Liddle:

1) Es reversible.
2) Cursa con hiperpotasemia.
3) Tiene acidosis metablica.
4) Es causa de HTA secundaria.
5) Tiene asociado aumento de renina.

101. Si en varios miembros de una familia se detectan casos de


nefritis, la exploracin diagnstica ms til es:

1) Ecografa renal.
2) TC.
3) UIV.
4) Electromiograma.
5) Audiometra.

M exico A rgentina CTO Medicina C/ Nez de Balboa, 115 28006 MADRID (Espaa) Tfno.: (91) 782 43 32 / Fax: (91) 782 43 27
Pg. 8 HM C hile U ruguay E-mail: secretaria@ctomedicina.com; iberocto@ctomedicina.com WEB: www.ctomedicina.com; www.iberocto.com
NEFROLOGA
Preparacin Examen de Seleccin 05/06 1 Vuelta Seguimiento a distancia
Pregunta 1.- R: 3
Se define el aclaramiento de creatinina como el volumen de plas- Pregunta 8. Distribucin etaria de las glomerulonefritis.
ma que se limpia de creatinina al pasar por el rin en un minuto.
Se determina mediante la ecuacin: 7829
2694 26 7829
2694 26
123456
64 3  
13 
Volo x [Cr]o
CCr= x 100 9   12345
[Cr]p  ! "#!$#%& 12345 2315

Su valor normal es de Ccv =100-120ml/min. '273 15 15345


Es el mejor parmetro para determinar el filtrado glomerular, es '6 62 ()
decir, la funcin renal.
'6* %" 4 7 62
Pregunta 2.- R: 4 '6* %" 44 8 62
El filtrado glomerular tiene un mecanismo de autorregulacin muy
complejo. La sustancia fundamental que interviene es la angiotensina II
(Ag II), que acta sobre todo en la arteriola eferente.
Cuando la perfusin del glomrulo disminuye, la Ag II constrie la Pregunta 9.- R: 4
arteriola eferente, aumentando la filtracin al disminuir el drenaje de Hay 3 mecanismos de formacin de GN (ver tabla en pgina siguiente):
sangre por esta arteriola. Anticuerpos antimembrana basal del glomrulo (AMBG): 2% del
total. Caracterstico de la enfermedad de Goodpasture. Produce
Pregunta 3.- R: 2 un patrn de IF lineal.
El asa de Henle se introduce en la mdula renal, es decir en el gradiente Inmunocomplejos: constituye el 97% del total. Hay 2 tipos:
de concentracin. En la porcin descendente se concentra la orina y en - I.C. circulantes, el ms frecuente.
la rama ascendente se vuelve a diluir, ello permite la formacin del gradiente - I.C. formados "in situ" caracterstico de la GN membranosa.
y su aprovechamiento para concentrar la orina. Patrn IF granular.
Las nefronas se diferencian por la longitud de su asa de Henle. Cuanto Activacin del complemento: 1% del total. No hay inmunoglobuli-
mas larga es, ms profundiza en el gradiente y puede concentrar ms. nas y produce un patrn granular en la I.F.
La concentracin final de la orina depende del tbulo colector.
Pregunta 10.- R: 1
El sndrome nefrtico se trata con dieta normoproteica y sin sal.
Pregunta 4.- R: 3
Hay que aadir diurticos al tratamiento. Se comienza con tiacidas y
La renina es una enzima producida y almacenada en los grnulos de
espirinolactona, y si no es suficiente, se aade tratamiento con diur-
las clulas yuxtaglomerulares del intersticio medular renal. Actua sobre el
ticos de asa. Es preciso un control estricto para evitar una deplecin
angiotensingeno, que es una globulina producida por el hgado, trans-
de volumen y que aparezca una N.T.A.
formndolo en angiotensina I, que a su vez es convertida por la enzima Cuando el edema es refractario a estas medidas, se utiliza la alb-
conversora de angiotensina (ECA), especialmente abundante en endotelio mina intravenosa.
vascular pulmonar, en angiotensina II, que estimula la produccin de
aldosterona en la zona glomerular de la corteza suprarrenal. Pregunta 11.- R: 3
El sarampin puede producir la remisin de G.N. de cambios m-
Pregunta 5.- R: 3 nimos por mecanismos no bien aclarados (posiblemente por modifi-
Aproximadamente por los dos riones pasa 1 litro de sangre por minuto cacin de la inmunidad celular).
(1/5 gasto cardaco), es decir aproximadamente 550 ml de plasma.
Para su determinacin se utiliza el aclaramiento de PAH (cido Pregunta 12.- R: 3
paraaminohiprico). Esta sustancia se filtra en los glomrulos y difun- Este cuadro clnico es compatible con un sndrome nefrtico. El
de desde los vasos a los tbulos, siendo nula la cantidad que sale por antecedente de faringoamigdalitis nos debe hacer pensar en la apari-
Comentarios TEST

la vena renal; por ello permite conocer el volumen plasmtico renal. cin de una glomerulonefritis postinfecciosa como causa probable
de este cuadro.
Pregunta 6.- R: 3 Esta glomerulonefritis cursa con hipocomplementemia durante un
Hay dos sustancias que tienen su mxima reabsorcin en el asa de perodo no mayor de 8 semanas. Si se prolonga, es preciso realizar
Henle: el Iodo y el Mg. biopsia renal para descartar L.E.S. o una glomerulonefritis membra-
El resto de sustancias se reabsorben fundamentalmente en el tbu- noproliferativa.
lo proximal.
Pregunta 13.- R: 5
Pregunta 7.- R: 2 Hay 3 tipos de glomerulonefritis rpidamente progresivas (GNRP)
La glomerulonefritis se define como aquel proceso de etiologa segn el depsito inmunitario:
inmunitaria en los que hay inflamacin de los glomrulos. Hablamos Tipo I: Anticuerpos antimembrana basal glomerular.
de glomerulonefritis aguda cuando existe oligoanuria. Son agudas: Tipo II: Por inmunocomplejos (puede ser hipocomplementmica).
Las que cursan con sndrome nefrtico. Tipo III: La ms frecuente, puede tener o no depsito de ANCA.
Las GN rpidamente progresivas primarias o secundarias.
Cuando aparece trombosis de la vena renal bilateral. Pregunta 14.- R: 4
En la endocarditis infecciosa tambin puede haber dao glomerular.
Pregunta 8.- R: 3 El mecanismo est mediado por inmunocomplejos circulantes (ICC).
La nefropata mesangial IgA es la ms frecuente en Espaa. En el glomrulo se puede encontrar una GN membranoprolifera-
Aunque el 50% de enfermos tiene elevacin de IgA en plasma, el tiva, aparicin de GNRP o una GN postinfecciosa.
100% de pacientes tiene depsito mesangial de IgA difuso en la inmu-
nofluorescencia. Pregunta 15.- R: 3
Es la nica GN que necesita ser diagnosticada mediante la realiza- Cuando en el curso evolutivo de una GN aparece un cuadro de
cin de inmunofluorescencia, para demostrar la IgA depositada. oligoanuria, hay que plantearse fundamentalmente la aparicin de
una GNRP.

M exico A rgentina CTO Medicina C/ Nez de Balboa, 115 28006 MADRID (Espaa) Tfno.: (91) 782 43 32 / Fax: (91) 782 43 27
C hile U ruguay E-mail: secretaria@ctomedicina.com; iberocto@ctomedicina.com WEB: www.ctomedicina.com; www.iberocto.com
IG Pg. 1
NEFROLOGA
Seguimiento a distancia Preparacin Examen de Seleccin 05/06 1 Vuelta

Pregunta 9. Principales caractersticas de las glomerulonefritis.

Cambios
GN A GN R P GN membranosa GNMP GN mesangial GEFS
mnimos

I: Proliferacin
Proliferacin Proliferacin mesangial Esclerosis
endocapilar difusa ex tracapilar Engrosamiento de la ("ral de tren"). Proliferacin glomerular
MO
ex udativa semilunas pared capilar. II: engrosamiento difusa. segmentaria y
(PMN). >50%. membrana basal focal.
(depsitos densos).

I: Lineal.
Granulares Granulares I: Subendoteliales. Mesangiales.
IF II: Granulares. Granulares.
subepiteliales. subepiteliales. II: Intramembranosos. Si IgA-Berger.
III:No depsitos.
Fusin de Spikes
ME Humps (jorobas).
podocitos. (espigas).
Sd. nefrtico, A veces
Insuficiencia Hematuria;
Nios (80%) Adulto (35%) hematuria, proteinuria
renal Berger slo
Clnica Sd. nefrtico. pronstico trombosis vena Sd. nefrtico. que no llega a
rpidamente hematuria
ex celente. renal. II: la de peor rango
progresiva. recidivante.
pronstico. nefrtico.
Infeccin por
Carcinomas, I: Hepatitis crnica
estreptococo - I: Goodpasture. Berger: infeccin VIH, herona,
Hodgkin; hepatitis B, LES activa.
hemoltico del II: Final de en resolucin reflujo,
Asociaciones atopia (HLA- D -penicilamina, II: Lipodistrofia
grupo A. muchos (latencia de unos rechazo
B12). sales de oro, parcial, hemlisis.
Latencia 1-3 procesos. das). trasplante.
captopril.
semanas.

Si el enfermo est diagnosticado de un sndrome nefrtico previo, Pregunta 21.- R: 3


hay que pensar antes en un cuadro de trombosis venosa renal (TVR), Ante un sndrome nefrtico en un adulto hay que realizar una
y si se descarta esta ltima, pensamos en una GNRP. biopsia de entrada. No se realiza si conocemos una causa en cuya
evolucin clnica pueda aparecer dicho sndrome, por ejemplo, si el
Pregunta 16.- R: 4 paciente es diabtico.
El tratamiento ante la sospecha de un sndrome nefrtico en un Sin embargo, si a pesar de conocer una muy probable causa el
nio es el esteroideo, ya que la causa ms frecuente de este sndrome curso del sndrome nefrtico es atpico (muy rpido, tienen hematu-
en la infancia es la nefropata de cambios mnimos, y responde en un ria, etc), hay que realizar biopsia renal.
90% a dicho tratamiento.
Cuando no hay respuesta, hay que plantearse la presencia de un Pregunta 22.- R: 2
cuadro de hialinosis focal y segmentaria y realizar una biopsia profun- En la GN mesangial IgA es muy rara la aparicin de un sndrome
da para coger tejido yuxtamedular, ya que este cuadro suele comen- extracapilar.
zar en esta zona.
Pregunta 23.- R: 3

Comentarios TEST
Pregunta 17.- R: 4 Aunque la etiologa ms frecuente de la nefropata membranosa es
La GN inmunotactoide es un cuadro de afectacin glomerular de idioptica, siempre hay que tener en cuenta la presencia de una
causa desconocida que afecta fundamentalmente a varones adultos, neoplasia slida (cncer de estmago, pulmn, colon), frmacos (Cap-
cursando con proteinuria y fracaso renal progresivo. topril, Penicilamina y sales de oro, etc), metales (Pb, Hg y Cd), los
Hay depsito de material proteico no amiloide en el glomrulo. procesos colgeno-vasculares y la hepatitis B.

Pregunta 18.- R: 1 Pregunta 24.- R: 2


Cuando nos referimos a una afectacin focal, significa que el dao En este caso clnico, la presencia de hematuria asociada a faringitis
afecta a unos glomrulos s y a otros no. aguda y la existencia de un episodio similar hace un ao, sin oliguria
Cuando hablamos de afectacin segmentaria, nos referimos a que acompaante, nos habla de la presencia de una nefropata mesangial
hay dao de una parte del glomrulo, no de todo ste. IgA probable. Si el paciente tuviese un cuadro de hematuria con
oliguria, nos orientara a una GN endocapilar difusa.
Pregunta 19.- R: 1
La afectacin por anticuerpos antimembrana basal produce un Pregunta 25.- R: 4
patrn de inmunofluorescencia lineal para IgG, y a veces para C3. Una caracterstica de la GN membranoproliferativa es la probable
asociacin con un cuadro de lipodistrofia, sobre todo asociado al
Pregunta 20.- R: 3 tipo II (enfermedad por depsitos densos).
La glomerulonefritis postinfecciosa tiene mejor pronstico en ni- A pesar de que haya sospecha de lipodistrofia, hay que realizar
os que en adultos, no habiendo clara diferencia en cuanto al sexo. biopsia cutnea y confirmarla.
Hay que realizar biopsia renal en las siguientes situaciones:
La hematuria dura ms de 6 meses. Pregunta 26.- R: 1
La hipocomplementemia dura ms de 8 semanas. En este caso, el nico cuadro que tiene descenso en el comple-
La oligoanuria dura ms de 21 das. mento es la GN membranoproliferativa o mesangiocapilar.

M exico A rgentina CTO Medicina C/ Nez de Balboa, 115 28006 MADRID (Espaa) Tfno.: (91) 782 43 32 / Fax: (91) 782 43 27
Pg. 2 IG C hile U ruguay E-mail: secretaria@ctomedicina.com; iberocto@ctomedicina.com WEB: www.ctomedicina.com; www.iberocto.com
NEFROLOGA
Preparacin Examen de Seleccin 05/06 1 Vuelta Seguimiento a distancia
Pregunta 27.- R: 3
La etiologa ms frecuente de la GN focal y segmentaria con hiali-
nosis es la idioptica. Causas secundarias que hay que recordar sobre
todo para el MIR son:
El reflujo vesicoureteral. Afecta ms a la mujer
Las nefropatas tubulointersticiales crnicas. (25-30 aos)
La evolucin en ocasiones de la nefropata de cambios mnimos.
Nefropata glomerular del paciente con infeccin por HIV, SIDA o
A.D.V.P.

Pregunta 28.- R: 3
La causa ms frecuente de sndrome nefrtico a partir de los 50
aos es la nefropata membranosa. El 20% debuta con proteinuria y Hipocomplementemia
hematuria asintomticas.Este proceso es el ejemplo de dao glomerular Aumento de la creatinina
por formacin de IC "in situ". Aumento de la urea
Con microscopa electrnica en fases iniciales nos encontramos una Hipertensin hiperreninmica
imagen caracterstica, los "spikes" o picos, que ayudan al diagnstico.

Pregunta 29.- R: 3
La ausencia de cilindros hemticos no excluye un cuadro de
glomerulonefritis.
Es preciso buscar la presencia de hemates deformados en la orina
ante todo cuadro de hematuria, ya que indican origen glomerular. El
nico falso positivo es la presencia de orinas hipotnicas. Hematuria La afectacin renal en el lupus
Posibles causas de cilindros hemticos son: Sd. nefrtico ensombrece el pronstico
Glomerulonefritis agudas y crnicas. (sedimento telescopado)
Vasculitis.
Crioglobulinemia.
Lesiones inflamatorias Lesiones irreversibles

Pregunta 30.- R: 3 - Proliferacin endocapilar. - Glomeruloesclerosis.


Aunque el cuadro caracterstico de las GN membranoproliferati- - Infiltrado leucocitario. - Semilunas fibrosas.
- Depsitos en asa de alambre. - Atrofia tubular.
vas es la presencia de hematuria y proteinuria asintomticas, tambin - Semilunas epiteliales. - Fibrosis intersticial.
pueden cursar con sndrome nefrtico, sndrome nefrtico o un cua- - Inflamacin del intersticio.
dro de oligoanuria por la presencia de GNRP.
Pregunta 35. Afectacin renal en el LES.
Pregunta 31.- R: 5
La hipocomplementemia asociada a la GN endocapilar o postin- Pregunta 36.- R: 5
fecciosa no puede durar mas de ocho semanas, ya que ello induce a En este caso clnico, la presencia de oliguria, hematuria y proteinu-
pensar en otros procesos hipocomplementmicos y que pueden cur- ria con cilindros hemticos nos orienta al diagnstico de sndrome
sar con sndrome nefrtico, como son el LES y las GN membranopro- nefrtico. Este proceso se produce cuando hay una inflamacin agu-
liferativas. Por ello, si el complemento bajo dura ms de ocho sema- da del glomrulo. La causa ms frecuente es postinfecciosa, pero pro-
nas, est indicada la biopsia renal. cesos inflamatorios que afectan al glomrulo tambin lo pueden pro-
ducir (LES, etc).
Pregunta 32.- R: 3
Las remisiones espontneas son frecuentes en los nios, mientras Pregunta 37.- R: 3
que en los adultos se dan entre un 20 y un 40%. La glomerulonefritis La nica GN que puede recurrir en 24/48h es la hialinosis focal y
Comentarios TEST

membranosa progresa a insuficiencia renal progresiva en un 20- 30% segmentaria.


de los pacientes, siendo este hecho ms frecuente en adultos. Si este
deterioro de la funcin renal es rpido debe pensarse en la existencia Pregunta 38.- R: 2
de una trombosis de la arteria renal o una glomerulonefritis con La anemia que se produce en el fracaso renal agudo y crnico es
semilunas asociadas. de etiologa multifactorial:
Esta nefropata puede recidivar en el trasplante. Disminucin de Epo.
Hemorragia digestiva.
Pregunta 33.- R: 3 Hemlisis.
Ante un cuadro de sndrome nefrtico en un adulto mayor de 50 Otros.
aos hay que pensar como primera posibilidad en una GN membra- En la glomerulonefritis aguda, la presencia de oligoanuria con re-
nosa. La etiologa secundaria ms frecuente es una neoplasia. tencin de volumen de agua y sal en el plasma produce mecanismo
dilucional como causa de anemia.
Pregunta 34.- R: 2
La nefropata IgA puede tener ocasionalmente incidencia familiar Pregunta 39.- R: 4
y asociar al HLA Bw35. La nefrosis lipoidea o nefropata de cambios mnimos se asocia en
Es la GN ms frecuente en Espaa, y por tanto la que ms recidiva ocasiones a linfoma de Hodgking. No se conoce el motivo de esta
en el trasplante (en nuestro pas). asociacin. Podra obedecer a un trastorno de inmunidad celular.

Pregunta 35.- R: 3 Pregunta 40.- R: 3


Una mujer joven con cuadro de afectacin renal y complemento De los 3 mecanismos de produccin de GN el ms frecuente im-
bajo nos debe hacer pensar en un cuadro de nefropata lpica. plicado en la produccin de GN aguda son los IC circulantes.
Recordad que el cuadro glomerular ms benigno es la GN mesan-
gial lpica y el cuadro ms agresivo y de peor pronstico es la GN Pregunta 41.- R: 2
proliferativa difusa lpica. La hialinosis focal y segmentaria suele comenzar afectando a los
glomrulos yuxtamedulares.

M exico A rgentina CTO Medicina C/ Nez de Balboa, 115 28006 MADRID (Espaa) Tfno.: (91) 782 43 32 / Fax: (91) 782 43 27
C hile U ruguay E-mail: secretaria@ctomedicina.com; iberocto@ctomedicina.com WEB: www.ctomedicina.com; www.iberocto.com
IG Pg. 3
NEFROLOGA
Seguimiento a distancia Preparacin Examen de Seleccin 05/06 1 Vuelta
Suele manifestarse con hematuria y proteinuria asintomticas o Nefropata por analgsicos.
sndrome nefrtico. Nefropata de los Balcanes.
Ambas son ms frecuentes en mujeres (por ello aumentan la inci-
Pregunta 42.- R: 2 dencia de uroteliomas en el sexo femenino).
En este caso, la cifra de C3 descendido puede estar asociado a GN
membranosa proliferativa o bien una GN endocapilar difusa. La au- Respuesta 47.- R: 2
sencia de oligoanuria nos hace pensar en primer lugar en una GN La infeccin del intersticio se denomina pielonefritis. Puede ser
membranoproliferativa. aguda o crnica. La causa ms frecuente es el reflujo de orina y el
germen mas habitual el E. coli.
Pregunta 43.- R: 3
La presencia de lipodistrofia asociada a un cuadro de GN nos orienta Pregunta 48.- R: 4
a pensar en una GN membranoproliferativa (sobre todo un tipo II). Las nefropatas intersticiales se presentan con dificultad de con-
centracin y acidificacin de la orina.
Pregunta 44.- R: 4
La afectacin del intersticio se caracteriza por la presencia de tras- Pregunta 49.- R: 1
tornos de concentracin y acidificacin de la orina. El paciente tiene En las nefropatas intersticiales agudas, el infiltrado es fundamental-
poliuria, nicturia y un cuadro de acidosis metablica hiperclormica mente de polimorfonucleares. Sin embargo, en los procesos crnicos
(anin gap normal). est formado por linfocitos, monocitos y fibroblastos, con formacin
En la orina hay leucocituria que indica la presencia de inflama- de fibrosis y retraccin del intersticio.
cin en el intersticio. La protena intersticial es la B2 microglobulina.
Pregunta 50.- R: 5
Pregunta 45.- R: 3 La nefropata por analgsicos crnica es ms frecuente en mujeres,
Es rara la hipertensin precoz en el dao intersticial, ya que existe sobre todo en aquellas que padecen cuadros de depresin con somatizacin
un fenmeno de prdida de agua y sal, por la dificultad de concentra- y sndrome fibromilgico (ver figura a pie de pgina). Se requiere la toma de
cin de la orina. por lo menos 1 Kg de analgsicos en 1 ao para que se produzca. Es ms
La hipertensin puede aparecer en estadios avanzados debido a la frecuente en pases centroeuropeos. En Espaa es rara.
fibrosis intersticial que libera renina.
Sin embargo, el rin gotoso produce hipertensin arterial ms Pregunta 51.- R: 1
precoz que otros procesos de intersticio crnico debido a la asocia- Pregunta frecuente en el examen MIR. El primer dato de la clnica
cin de hiperplasia de la ntima arteriolar motivada por los cristales de del proceso intersticial es el trastorno de concentracin de la orina.
cido rico.
Pregunta 52.- R: 4
Pregunta 46.- R: 1 La segunda causa de necrosis de papila es la nefropata por analg-
Hay 2 etiologas de dao intersticial que pueden producir urote- sicos crnica. La causa ms frecuente es la microangiopata diabtica.
liomas:

Aumento de la
incidencia de carcinomas
transicionales de
urter y pelvis renal

Comentarios TEST
Lesin tubulointersticial
difusa en la corteza

Hipertensin arterial
Piuria con cultivos
negativos

Anemia
desproporcionada
con el grado de
Insuficiencia renal

Signo del anillo


en la pielografa

Necrosis crnica de papila Mujer (ms frecuente anglosajona) que Gastritis o lcera pptica
abusa de los AINES (fenatecinas,...)

Pregunta 50. Nefropata por analgsicos.

M exico A rgentina CTO Medicina C/ Nez de Balboa, 115 28006 MADRID (Espaa) Tfno.: (91) 782 43 32 / Fax: (91) 782 43 27
Pg. 4 IG C hile U ruguay E-mail: secretaria@ctomedicina.com; iberocto@ctomedicina.com WEB: www.ctomedicina.com; www.iberocto.com
NEFROLOGA
Preparacin Examen de Seleccin 05/06 1 Vuelta Seguimiento a distancia
Pregunta 53.- R: 2 Proteinuria de Bence-Jones.
Cuando un paciente tiene diagnosticada una nefropata intersticial Sndrome nefrtico por amiloidosis AL asociada.
por Pb y tiene en su evolucin un sndrome nefrtico, hay que pensar
en un cuadro de hialinosis focal y segmentaria como causa de dicho Pregunta 65.- R: 1
sndrome. En su patogenia est implicada la formacin de IC circulan- En la enfermedad de Alport-Perkoff existe afectacin de distintos
tes con el concurso de antgenos intersticiales. rganos:
Rin.
Pregunta 54.- R: 4 Ojos: malformaciones en el cristalino (esferofaquia, lenticono).
Cuando un paciente tiene un fracaso renal y clnica de poliuria y Odos: sordera o hipoacusia de percepcin.
nicturia tenemos que pensar en un cuadro intersticial. Existe asociado
un cuadro de acidosis metablica hiperclormica por trastorno de Pregunta 66.- R: 4
acidificacin de la orina. Cuando se trata con quimioterapia una neoplasia hematolgica se
libera gran cantidad de purinas que dan lugar a cido rico que
Pregunta 55.- R: 5 puede filtrarse y obstruir los tbulos de la nefrona.
La nefropata de los Balcanes est producida por un virus RNA Es preciso prevenir este proceso con abundantes lquidos, bicar-
(coronavirus) que infecta el intersticio. Este germen produce cambios bonato y alopurinol.
en el urotelio con aparicin de metaplasia, displasia y cncer urote-
lial. Por ello es preciso realizar citologas de control en estos pacientes
Pregunta 67.- R: 2
en las revisiones.
Cuando la diabetes afecta al glomrulo, aparece microalbuminuria
Pregunta 56.- R: 5 por aparicin de un cuadro de glomeruloesclerosis difusa intercapilar.
Ya comentado con anterioridad (pregunta 45), el cido rico en el Es preciso confirmar esta proteinuria dos veces ms en 6 meses, ya
intersticio produce dao en la pared vascular por hiperplasia endote- que su confirmacin es indicacin de tratamiento con IECAS.
lial, produciendo liberacin de renina.
EVOLUCIN NEFROPATA TERAPUTICA
Pregunta 57.- R: 5
La fibrosis intersticial produce traccin y ensancha la luz tubular,
haciendo mayor el tamao de los cilindros. Son creos por su colora-
cin (formados por material celular degenerado). Inicio Estadio I
hiperglucemia + tamao renal
Pregunta 58.- R: 4
Las pielonefritis se producen frecuentemente por la existencia de 10-15 aos

reflujo vesicoureteral. El germen ms implicado es el E. coli, ya que es Estadio II


el microorganismo ms frecuente en las infecciones de orina. Microalbuminuria Control metablico
intermitente Disminuir HTA (eleccin IECA)
Pregunta 59.- R: 3 Estadio III Restriccin proteica
Ya explicado anteriormente (pregunta 56). M. Persistente

3-7 aos
Pregunta 60.- R: 3
La leucocituria en orina es indicativo de la existencia de procesos Proteinuria manifiesta
inflamatorios, pero no necesariamente de etiologa infecciosa; es por Control estricto
(Macroproteinuria >300 mg/d) Estadio IV
de la HTA (eleccin IECA)
ello que, sea cual sea la causa de la inflamacin del intersticio, se pro- Nefropata clnica
duce leucocituria, apareciendo en ocasiones cilindros leucocitarios.
La piuria es la leucocituria de origen pigeno. 5-7 aos

Pregunta 61.- R: 4 Estadio V


Dilisis
I.R. Terminal
Comentarios TEST

En los procesos de intersticio crnico, la dificultad de reabsorcin Trasplante


a nivel proximal junto con el mayor aporte de sodio al tbulo distal y
mayor intercambio de Na+ por K+ conlleva la aparicin en ocasiones Pregunta 67. Dao renal en la DM.
de deplecin de potasio.
Pregunta 68.- R: 1
Pregunta 62.- R: 3 La lesin ms frecuente en el glomrulo del rin diabtico es la
La pielonefritis xantogranulomatosa es una patologa muy poco glomeruloesclerosis difusa intercapilar. Sobre sta puede aparecer en
frecuente. Ocurre en enfermos con infecciones de orina por grme- el 15% de los pacientes un cuadro de glomeruloesclerosis nodular
nes urealticos (como por ejemplo el Proteus mirabilis) y con mecanis- que suele manifestarse como sndrome nefrtico.
mo de reflujo, junto con una dificultad de los macrfagos del intersti-
cio renal para eliminar lpidos que hacen que se llenen de grasa. Pregunta 69.- R: 4
Como consecuencia, el parnquima renal se deforma dando mor- La asociacin hematuria-hemoptisis se denomina sndrome de
fologa pseudotumoral. El cuadro clnico a veces se confunde con Goodpasture. La causa ms frecuente de este cuadro es la Enferme-
una neoplasia (fiebre, leucocitosis, VSG , dolor). dad de Goodpasture.
Su tratamiento es la nefrectoma.
El segundo cuadro en frecuencia es la vasculitis de Wegener (c-
Pregunta 63.- R: 1 ANCA positivo en el 97% de los casos).
La enfermedad de Alport es gentica. Suele estar ligada la herencia Cuando el paciente presenta en la analtica p-ANCA pensamos en
al cromosoma X. Hay defecto del colgeno tipo IV. La membrana un cuadro de PAN microscpica.
basal esta laminada como un "hojaldre". Por ello es frecuente la hema-
turia y la proteinuria rica en albmina. Pregunta 70.- R: 4
Algunos pacientes con procesos inflamatorios crnicos (AR, EA,
Pregunta 64.- R: 5 etc.) y portadores de procesos infecciosos crnicos pueden desarro-
El mieloma mltiple puede dar lugar a proteinuria abundante por llar un sndrome nefrtico secundario a la presencia de amiloidosis
2 mecanismos: con afectacin glomerular.

M exico A rgentina CTO Medicina C/ Nez de Balboa, 115 28006 MADRID (Espaa) Tfno.: (91) 782 43 32 / Fax: (91) 782 43 27
C hile U ruguay E-mail: secretaria@ctomedicina.com; iberocto@ctomedicina.com WEB: www.ctomedicina.com; www.iberocto.com
IG Pg. 5
NEFROLOGA
Seguimiento a distancia Preparacin Examen de Seleccin 05/06 1 Vuelta
Pregunta 77.- R: 4
La PAN clsica afecta en el 90% de pacientes al rin y suele debutar
con HTA vasculorrenal. Es frecuente la aparicin de microaneurismas a
nivel renal que pueden ayudar al diagnstico (arteriografa).

Pregunta 78.- R: 3
Cuando aparece proteinuria en la nefropata diabtica hay que
iniciar tratamiento con IECA, ya que han demostrado que pueden
remitir la proteinuria.
Cuando existen contraindicaciones para su uso (tos severa, angio-
edema, artritis por IECAs), se deben usar ARA II.

Pregunta 79.- R: 5
Los cuerpos de hematoxilina son restos nucleares que se tien, (con
tcnicas de hematoxilina-eosina) de rojo. Son patognomnicos de LES.

Pregunta 80.- R: 4
El cuadro que tiene peor pronstico es la GN proliferativa difusa
lpica. El cuadro con mejor pronstico es la GN mesangial lpica.

Pregunta 81.- R: 3
Pacientes con cuadros de osteomielitis crnica, bronquiectasias o
portadores de sonda vesical permanente, o enfermos parapljicos
con escaras tienen mayor probabilidad de desarrollar un cuadro de
sndrome nefrtico secundario a una amiloidosis AA.

Pregunta 82.- R: 3
La plasmafresis con recambio del plasma es de mucha utilidad en
procesos como:
Enfermedad de Goodpasture.
Crioglobulinemia.
Rin de mieloma.

Pregunta 83.- R: 4
Pregunta 70. Amiloidosis en la AR de larga evolucin. El sntoma que siempre mejora con dilisis es la anorexia. Sin em-
bargo no se modifica el prurito, al estar mediado por PTH y calcifica-
Pregunta 71.- R: 4 ciones subcutneas.
Se trata de una enfermedad de Wegener por la presencia del
cuadro de hematuria y hemoptisis, asociado a infiltrados cavitados Pregunta 84.- R: 2
y rinitis de repeticin (afectacin ORL en el 97% de los pacientes). El enfermo con I.R. crnica tiene aterosclerosis generalizada, y por
El c-ANCA est presente en un 97% de los pacientes y es un ello el riesgo de eventos cardiovasculares (IAM y ACV) es ms frecuen-
marcador diagnstico y pronstico de la enfermedad. No es causa te. Por esta razn, hay que asegurarse una adecuada oxigenacin
de la enfermedad. Sirve para el seguimiento y pronstico del enfer- mediante una adecuada cifra de hemoglobina 10-11g. Para ello uti-
mo. lizaremos E.P.O. y/o transfusiones.
Pregunta 72.- R: 3 Pregunta 85.- R: 2
La presencia de LDH elevada en sangre y/o orina indica necrosis

Comentarios TEST
La vasculitis de Wegener afecta en el 100% de los pacientes al
tisular. Cuando ocurre unido a un cuadro de lumbalgia y hematuria pulmn, aunque la Rx de trax sea normal.
nos plantearemos el diagnstico de oclusin arterial trombtica o El sistema ORL se afecta en el 97% de los casos, con rinitis, sinusitis,
emblica con infarto isqumico del parnquima. En la trombosis de mastoiditis, etc.
la vena renal no aumenta la LDH. El rin aparece afectado en un 70% de los casos. La lesin predo-
minante es la GN membranoproliferativa, apareciendo hematuria y
Pregunta 73.- R: 4 proteinuria en el sedimento de orina.
La enfermedad de Alport raramente produce un sndrome nefrti-
co. El resto de las opciones son incorrectas. Pregunta 86.- R: 4
La causa principal de TVR presenta diferencias etarias:
Pregunta 74.- R: 2 En los neonatos, la causa ms frecuente es la deshidratacin hiper-
El cadmio puede producir adenocarcinoma renal y uroteliomas.
tnica.
A nivel glomerular puede dar un cuadro de nefropata membranosa
En la infancia, la causa ms frecuente es la GN de cambios mnimos.
y tambin puede producir una nefropata intersticial crnica.
En el adulto, las causas ms frecuentes son la GN membranosa y la
Pregunta 75.- R: 4 amiloidosis.
La afectacin glomerular ms frecuente encontrada en el SIDA es
la hialinosis focal y segmentaria. Pregunta 87.- R: 4
Los microaneurismas suelen ser caractersticos de la panarteritis
Pregunta 76.- R: 3 nodosa.
La aparicin de un cuadro de lumbalgia y hematuria en el seno de
un sndrome nefrtico nos har pensar en la aparicin de trombosis Pregunta 88.- R: 1
de la vena renal (TVR). El 15% se asocia a TEP, y por ello la TVR es La causa ms frecuente de sndrome de Goodpasture es la Enfer-
causa de sndrome hematuria-hemoptisis. medad de Goodpasture, por ello la primera prueba que hay que
pedir es Ac anti-MBG. Son positivos en el 90% de los enfermos.

M exico A rgentina CTO Medicina C/ Nez de Balboa, 115 28006 MADRID (Espaa) Tfno.: (91) 782 43 32 / Fax: (91) 782 43 27
Pg. 6 IG C hile U ruguay E-mail: secretaria@ctomedicina.com; iberocto@ctomedicina.com WEB: www.ctomedicina.com; www.iberocto.com
NEFROLOGA
Preparacin Examen de Seleccin 05/06 1 Vuelta Seguimiento a distancia

1) Insuficiencia prerrenal (descenso de perfusin renal).


1
Disminucin absoluta del volumen de sangre efectivo.
- Hemorragia.
3
- Prdidas cutneas: quemaduras, sudor.
- Prdidas gastrointestinales: diarrea, vmitos.
- Prdidas renales: diurticos.
- Tercer espacio: peritonitis, quemaduras.
Disminucin relativa del volumen de sangre efectivo.
- Insuficiencia cardaca congestiva.
- Arritmias.
- Cirrosis heptica: ascitis, sndrome hepatorrenal.
- Sndrome nefrtico. 2
- Hipotensin de cualquier etiologa.
- Fracaso hemodinmico intraglomerular:
Antiinflamatorios no esteroideos.
Inhibidores del enzima de conversin.
Hipercalcemia.
Aminas vasoconstrictoras. 2) Insuficiencia parenquimatosa.
Oclusin arterial.
- Tromboembolismo bilateral. Alteraciones vasculares.
- Tromboembolismo de un rin solitario. - Vasculitis.
- Aneurisma de la arteria renal o aorta. - Hipertensin arterial maligna.
- Sustancias vasoconstrictoras.
- Eclampsia.
- Microangiopata trombtica.
3) Insuficiencia postrenal. - Estados de hiperviscosidad.
- Antiinflamatorios no esteroideos.
Intraluminal. - Hipercalcemia.
- Litiasis: rica, clcica, infecciosa. Alteraciones glomerulares.
- Cogulos. - Glomerulonefritis agudas.
- Necrosis papilar. Alteraciones tubulares.
- Tumores: hipernefroma, urotelioma. - Necrosis tubular isqumica.
Intraparietal. Hipotensin profunda: hemorragia, shock, ICC.
- Malacoplaquia. Rin trasplantado.
- Estenosis: congnita, postinfecciosa, Frmacos vasoconstrictores.
postraumtica. - Necrosis tubular txica.
- Tumores. Nefrotxicos.
- Anomalas congnitas. Antibiticos (aminoglucsidos, anfotericina B, etc.)
Compresin extrnseca. Metales (mercurio, bismuto, arsnico, plata, cadmio,
- Malformacin congnita: urter retrocavo, rin hierro, antimonio).
en herradura, bridas vasculares. Disolventes (tetracloruro de carbono, glicol,
- Hiperplasia prosttica. tetracloruro de etileno).
- Adenocarcinoma prosttico. Contrastes radiolgicos yodados.
- Fibrosis retroperitoneal: metisergida, neoplasias. Pigmentos intratubulares.
- Tumores: prstata, pelvis congelada. Hemoglobinuria.
- Ligadura yatrgena de urter. Mioglobinuria.
Disfuncin neurgena. Bilirrubinuria.
- Vejiga neurgena. Protenas intratubulares.
Lesin
Lesinsobre
sobreL1-L3:vejiga
L1-L3: vejiga tona ooflccida.
espstica automtica Mieloma mltiple.
Comentarios TEST

Lesin bajo L1-L3: vejiga tona o flccida. Cristales intratubulares.


- Disfuncin de la unin pieloureteral. Acido rico.
- Reflujo vesicoureteral: con el mismo efecto Oxalatos.
hidrodinmico que una obstruccin real. Alteraciones tubulointersticiales.
Obstruccin venosa. - Por frmacos.
- Trombosis venosa renal. - Por infecciones.
- Neoplasia. - Por radiacin.

Pregunta 93. Clasificacin del fracaso renal agudo.

Pregunta 89.- R: 4 ciada en el 8% de los casos a hepatitis por virus C).


Los hallazgos ms frecuentes en la orina de la enfermedad de Se manifiesta en el sistemtico de orina por hematuria y proteinu-
Wegener son la hematuria y proteinuria asintomticas. ria asintomticas.
El cuadro ms caracterstico de afectacin glomerular es la GN
Pregunta 90.- R: 3 membranoproliferativa.
La afectacin glomerular por amiloide condiciona la aparicin de Al microscopio da una imagen caracterstica de pseudotrombos de
albuminuria. globulinas.

Pregunta 91.- R: 4 Pregunta 93.- R: 1


El sedimento telescopado es caracterstico del LES y algunas vascu- La causa ms frecuente de azoemia prerrenal es la deshidratacin.
litis. Es un sedimento con todo tipo de cilindros. La insuficiencia renal prerrenal es la causa ms frecuente de insufi-
ciencia renal aguda.
Pregunta 92.- R: 2 En la tabla puedes ver otras causas de fracaso renal segn el nivel de
La crioglobulinemia afecta al rin en ms del 85% de los pacien- afectacin de la nefrona.
tes (principalmente el tipo II o crioglobulinemia mixta esencial, aso-

M exico A rgentina CTO Medicina C/ Nez de Balboa, 115 28006 MADRID (Espaa) Tfno.: (91) 782 43 32 / Fax: (91) 782 43 27
C hile U ruguay E-mail: secretaria@ctomedicina.com; iberocto@ctomedicina.com WEB: www.ctomedicina.com; www.iberocto.com
IG Pg. 7
NEFROLOGA
Seguimiento a distancia Preparacin Examen de Seleccin 05/06 1 Vuelta
Pregunta 94.- R: 4 Pregunta 100.- R: 4
Ante un cuadro de hiperpotasemia aguda, el nico frmaco que La enfermedad de Liddle es una tubulopata distal; es una causa de
puede antagonizar el efecto txico del K+ a nivel cardaco es el HTA secundaria, sobre todo en varones jvenes. En esta patologa el
calcio i.v (cloruro o gluconato). tbulo distal retiene sodio en exceso y elimina K+ en la orina, de tal forma
Para disminuir las cifras de K+ hay que utilizar glucosado ms que siempre cursa con alcalosis metablica y cifras de potasio bajas en el
insulina, salbutamol, bicarbonato 1/6 M intravenoso (para corregir la plasma. Su tratamiento es con Triamtirene y en su defecto amiloride.
acidosis), furosemida y/o dilisis, si es preciso.
Pregunta 101.- R: 5
Pregunta 95.- R: 1 La enfermedad de Alport tambin se denomina nefritis familiar y es
En los enfermos agudamente oligricos, lo fundamental en el un cuadro de herencia ligada al cromosoma X.
tratamiento es corregir la hipovolemia mediante expansin de volu- Existe una alteracin del colgeno tipo IV y se manifiesta por altera-
men. ciones renales, oculares y auditivas.
En el caso de insuficiencia cardaca, no se puede expandir y hay que En el ojo puede aparecer lenticono o esferofaquia y en el odo sordera
tratar al paciente con dobutamina para aumentar el gasto cardaco. de percepcin, por ello una prueba muy til es la audiometra.
La nica indicacin de expansin de volumen en enfermos olig- En el rin lo caracterstico es la aparicin de una membrana basal
ricos con clnica de ICC es el infarto masivo del VD. en "capa de hojaldre" con microscopio electrnico.
Suele manifestarse con proteinuria y raramente con sndrome
Pregunta 96.- R: 2 nefrtico.
La excrecin fraccional de sodio es el mejor parmetro para dife-
renciar un fracaso agudo renal prerrenal (<1) de un fracaso paren- Pregunta 102.- R: 4
quimatoso (>1). La enfermedad de Bartter es un proceso autosmico recesivo que
Es un cociente de aclaramiento de sodio y creatina; relaciona el afecta al funcionamiento del asa de Henle. Hay disfuncin del
filtrado de sodio con respecto a la funcin renal. intercambiador Na+: 2 Cl: K+. Ello origina grandes prdidas de pota-
sio por la orina y aparicin de hipopotasemia y alcalosis metablica.
Pregunta 97.- R: 3 Los pacientes tienen poliuria y pueden tener nefrocalcinosis debi-
Los frmacos ms implicados en la nefropata tubulointersticial do a la presencia de hipercalciuria.
aguda por frmacos son los Betalactmicos y la Rifampicina.
Es un cuadro idiosincrsico y reversible que cursa con fracaso
Sndrome de Bartter
renal agudo polirico con clnica alrgica asociada.
El 80% de los enfermos tienen eosinofilia y/o eosinofiluria. Porcin ascendente gruesa del asa de Henle

Pregunta 98.- R: 1 Aparato Mcula densa


yuxtaglomerular
La causa ms frecuente de insuficiencia renal aguda parenquima-
tosa es la NTA de etiologa isqumica y/o txica.
La excrecin fraccional de sodio en este cuadro clnico es mayor
de 1 y el sodio en orina > 30 mEq/L.
Na+
Pregunta 98. Diagnstico diferencial de la IR prerrenal vs intrnseca. Renina
Cl
-

Mg++
Angiotensina II
NDICE DIAGNSTICO FRA PRERRENAL FRA INTRNSECA Prostaglandinas
F.E. Na <1 >1 Aldosterona
Concentracin de sodio en
<10 >20
orina (mmol/l) Tensin arterial Alcalosis
Proporcin de creatinina en normal hipopotasmica
>40 <20
orina y plasma
Pregunta 102.Fisiopatologa del sndrome de Bartter.

Comentarios TEST
Proporcin de nitrgeno
>8 <3
ureico en orina y plasma
Pregunta 103.- R: 3
Densidad urinaria >1,018 <1,012 La enfermedad de Gitelman es muy parecida a la enfermedad de
Osmolalidad urinaria >500 <250 Bartter. En ella no hay hipercalciuria, y por lo tanto, no hay nefrocal-
Proporcin entre BUN y cinosis.
>20 <10-15
creatinina en plasma
Pregunta 104.- R: 4
ndice de insuficiencia renal >1 <1
La hipopotasemia induce la sntesis de prostaglandinas E y F por las
Cilindros granulosos clulas del intersticio medular. Son las responsables de que estos
Sedimento urinario Cilindros hialinos.
marrn turbios. enfermos tengan normotensin a pesar del hiperaldosteronismo se-
cundario que padecen y que no tengan respuesta hipertensiva al
Pregunta 99.- R: 1 darles agentes vasoconstrictores.
La poliquistosis renal o enfermedad qustica hepatorrenal es la
quinta causa ms frecuente de I.R. crnica.
Tienen 2 tipos de herencia: autosmica dominante y autosmica
recesiva.
La forma del adulto es autosmica dominante y empieza a mani-
festarse aproximadamente a la edad de 40 aos.
El sntoma ms frecuente es el dolor en flancos y el que sigue ms
frecuente es la presencia de hematuria.
Es importante recordar la asociacin de esta enfermedad con insu-
ficiencias valvulares cardacas, aneurismas intracraneales, divertculos
intestinales y hernias inguinales.

M exico A rgentina CTO Medicina C/ Nez de Balboa, 115 28006 MADRID (Espaa) Tfno.: (91) 782 43 32 / Fax: (91) 782 43 27
Pg. 8 IG C hile U ruguay E-mail: secretaria@ctomedicina.com; iberocto@ctomedicina.com WEB: www.ctomedicina.com; www.iberocto.com
NEUMOLOGA Y CIRUGA TORCICA
Preparacin Examen de Seleccin 05/06 1 Vuelta Seguimiento a distancia

FISIOLOGA. 4) La administracin de oxgeno corrige la hipoxemia produci-


da por trastornos de ventilacin/perfusin.
1. Seale la respuesta FALSA: 5) En un mismo paciente pueden existir varios mecanismos
causantes.
1) La zona de transicin est constituida por los bronquiolos
6. De las siguientes afirmaciones indique la INCORRECTA:
respiratorios.
2) El espacio muerto anatmico se denomina tambin zona de
1) Los vrtices pulmonares estn mejor perfundidos que las
conduccin. bases.
3) El acino es la unidad anatmica situada distal al bronquiolo 2) Las bases pulmonares estn mejor ventiladas que los vrtices.
terminal. 3) La relacin ventilacin/perfusin es algo mayor en los vrtices
4) El lobulillo secundario est formado por sacos alveolares y pulmonares.
ductus alveolares. 4) De todas las causas de hipoxemia, los trastornos de ventila-
5) La cantidad de moco bronquial producido por el rbol cin/perfusin intervienen en la mayora de los casos.
bronquial es de 100 ml/da. 5) La sangre que procede de los vrtices tiene una PaO2 algo
mayor que la procedente de las bases.
2. Seale la relacin FALSA:
7. Respecto a la curva de saturacin de la hemoglobina (Hb),
1) Capacidad vital (CV): volumen espirado mximo despus de seale lo FALSO:
una inspiracin mxima.
2) Capacidad pulmonar total (CPT): volumen que contienen 1) La PaO2 60 mmHg corresponde a una saturacin de Hb del 90%.
los pulmones en la posicin de inspiracin mxima. 2) Cuando la Hb alcanza una saturacin del 90%, la curva
3) Volumen espiratorio de reserva (VER): volumen espirado adquiere un aspecto de meseta, de tal forma que el aumento
despus de una espiracin normal. de PaO2 modifica escasamente la saturacin.
4) Volumen residual (VR): volumen inspirado y/o espirado en 3) El desplazamiento de la curva a la derecha implica una mayor
cada movimiento respiratorio. cesin de O2 a los tejidos por parte de la Hb.
5) Capacidad funcional residual (CFR): volumen de gas que 4) El aumento de hidrogeniones desplaza la curva a la izquierda.
contienen los pulmones tras una espiracin normal. 5) La acidosis respiratoria y las situaciones febriles desplazan la
curva a la derecha.
3. Con relacin al patrn ventilatorio obstructivo, todo es cierto, 8. Uno de los siguientes NO constituye un mecanismo compen-
EXCEPTO: sador de hipoxemia:

1) El volumen residual se eleva, aunque en algunas enfermeda- 1) Aumento del gasto del VD.
des la elevacin es ms marcada. 2) Aumento de la eritropoyesis.
2) La CVF suele estar disminuida. 3) Aumento de bicarbonato en orina.
3) El patrn obstructivo se acompaa en la mayora de los casos 4) Aumento de PCO2 en sangre arterial.
de una disminucin en la DLCO. 5) Aumento de la frecuencia respiratoria.
4) El VEF1 es menor del 80% del terico.
5) La CPT puede aumentar. 9. La hipoxemia que aparece en un embolismo pulmonar agudo
se relaciona con:
4. La ventilacin alveolar es:
1) Aumento del espacio muerto anatmico, en relacin con la
presencia de reas mal perfundidas.
1) El volumen de gas contenido en la zona comprendida entre 2) Aumento de las reas de cortocircuito.
la trquea y los bronquiolos terminales. 3) Es secundaria a una hiperventilacin perifrica.
Preguntas TEST

2) El volumen de gas comprendido en la zona entre bronquiolos 4) Aumento del espacio muerto fisiolgico.
respiratorios y alveolos. 5) No se corrige con O2 al 100% ya que se debe a un shunt
3) El volumen de gas que no participa del intercambio de gases intrapulmonar.
por unidad de tiempo.
4) El volumen de gas intrapulmonar en la posicin de inspira- 10. De las siguientes situaciones clnicas, indique en cul NO est
cin mxima. disminuida la DLCO:
5) El volumen de gas que participa del intercambio de gases por
unidad de tiempo. 1) Fibrosis pulmonar.
2) Enfisema.
5. NO es correcto en relacin a la hipoxemia: 3) Bronquitis crnica.
4) Hipertensin pulmonar primaria.
1) Las alteraciones en la difusin son una causa de hipoxemia 5) Embolismos pulmonares recurrentes.
frecuente.
2) La hipoxemia de las grandes alturas se acompaa de normo 11. Respecto al control de la ventilacin, es FALSO:
o hipocapnia.
3) Son causa de hipoxemia las anomalas vasculares con corto- 1) El principal regulador de la ventilacin es la PCO2, que
circuito arteriovenoso, que pueden ser hereditarias o adqui- estimula a los quimiorreceptores centrales y tambin a los
ridas. perifricos.

M exico A rgentina CTO Medicina C/ Nez de Balboa, 115 28006 MADRID (Espaa) Tfno.: (91) 782 43 32 / Fax: (91) 782 43 27
C hile U ruguay E-mail: secretaria@ctomedicina.com; iberocto@ctomedicina.com WEB: www.ctomedicina.com; www.iberocto.com
NM Pg. 1
NEUMOLOGA Y CIRUGA TORCICA
Seguimiento a distancia Preparacin Examen de Seleccin 05/06 1 Vuelta
2) La respuesta de PCO2 disminuye con la edad y durante el 16. Uno de los siguientes datos NO es tpico de los pacientes EPOC
sueo. tipo bronquitis crnica:
3) El nivel de PO2 altera la sensibilidad al CO2. As, si hay
hipoxemia se es ms sensible al nivel de PCO2. 1) Frecuentes reagudizaciones infecciosas.
4) La ingesta de barbitricos baja la sensibilidad tanto al CO2 2) Tendencia al desarrollo de poliglobulia.
como al O2. 3) Tendencia al desarrollo de cor pulmonale crnico.
5) Las variaciones de PO2 y pH estimulan a los receptores 4) Hiperinsuflacin pulmonar.
centrales y perifricos, con la misma intensidad. 5) Aspecto abotargado, cianosis labial.

17. Uno de los siguientes datos NO aparece en la Rx de trax de un


12. Un paciente con insuficiencia respiratoria muestra una gaso-
paciente con enfisema:
metra arterial con PO2 59 mmHg, PCO2 38 mmHg, pH 7,36 y
una diferencia alveolo-arterial de oxgeno de 35 mmHg. Se
1) Aplanamiento diafragmtico.
administra O2 al 100% y no se corrigen los valores de los gases 2) Oligoemia vascular.
arteriales. El cuadro subyacente MENOS probable es: 3) Aumento de la trama broncovascular.
4) Corazn en gota.
1) Enfermedad vascular pulmonar. 5) Imgenes qusticas de pared fina.
2) Atelectasia.
3) Edema agudo de pulmn. 18. Un paciente de 65 aos, fumador de 25 cigarrillos diarios desde
4) Shunt intracardaco. su juventud, presenta tos y expectoracin crnicas, que a veces
5) Shunt intrapulmonar. ha sido ligeramente hemoptoica, sobre todo coincidiendo con
agudizaciones infecciosas. En los ltimos 6 meses nota disnea
13. Un paciente de 50 aos acude a Urgencias por disnea de curso progresiva, que es en la actualidad de medianos esfuerzos. De los
progresivo. La gasometra arterial muestra los siguientes datos: siguientes enunciados, indique el que NO esperara encontrar:
pH 7,47, PaO2 62 mmHg, PaCO2 30 mmHg, bicarbonato 26
mEq/l. Indique, de las siguientes posibilidades, la que mejor 1) Indice de Tiffeneau del 60%.
explicara su situacin: 2) Hipocapnia.
3) Frecuentes descompensaciones infecciosas.
1) Hipoventilacin alveolar primaria. 4) PaO2 55 mmHg.
2) Poliomielitis bulbar. 5) Aumento del ndice cardiotorcico.
3) Cifoescoliosis.
19. Respecto al tratamiento de la EPOC, es FALSO:
4) Enfermedad pulmonar obstructiva crnica.
5) Insuficiencia cardaca.
1) Los antitusgenos estn contraindicados.
2) La hidratacin del esputo, no ha demostrado ningn beneficio.
14. Un paciente con EPOC presenta en situacin de estabilidad una 3) La vacuna anti-influenza reduce la morbimortalidad de los
gasometra arterial con los siguientes valores: pH 7,40, PaO2 58 EPOC durante las epidemias gripales, y es eficaz en 3/4 partes
mmHg, PaCO2 46 mmHg. Ante una descompensacin presumi- de los pacientes.
blemente infecciosa acude a Urgencias y se obtiene una gaso- 4) La rehabilitacin mejora la tolerancia al ejercicio.
metra que demuestra: pH 7,36, PaO2 50 mmHg, PaCO2 60 5) Los anticolinrgicos son los broncodilatadores de eleccin,
mmHg, bicarbonato 35 mEq/l. Una de las siguientes respuestas presentando una accin ms duradera que los adrenrgicos
es correcta: beta 2 de accin corta.

1) La gasometra presenta una acidosis respiratoria aguda. 20. Indique lo que NO es cierto en relacin al enfisema por dficit
2) La gasometra demuestra una acidosis mixta. de alfa-1-antitripsina:
3) En este caso, existe insuficiencia ventilatoria crnica con

Preguntas TEST
hipoxemia. 1) La enfermedad se hereda de modo autosmico codominante.
4) El gradiente (A-a) de O2 probablemente es de 10 mmHg. 2) Los homocigotos Z y S tienen concentraciones reducidas del
5) En este caso est indicada la ventilacin mecnica. enzima, pero los ltimos suelen tener cantidad suficiente para
conseguir la proteccin.
3) El enfisema predomina en lbulos inferiores.
ENFERMEDAD PULMONAR OBSTRUCTIVA CRNICA. 4) En el mismo paciente no se ha descrito la existencia de
enfisema y bronquiectasias.
5) Hay controversia sobre si el heterocigoto se asocia a altera-
15. Qu dato caracteriza, de entre los siguientes, a los pacientes
cin de la funcin pulmonar.
con enfermedad pulmonar obstructiva crnica (EPOC)?:
21. Seale lo que NO es cierto en relacin a la oxigenoterapia
1) Exposicin al tabaco durante al menos 10 aos, ms de 20 continua domiciliaria:
cig/da.
2) Presencia de disnea de moderados esfuerzos de un ao de 1) Mejora la tolerancia al ejercicio.
evolucin. 2) Mejora la funcin neuropsicolgica.
3) Hipersecrecin bronquial. 3) Alivia la hipertensin pulmonar.
4) Limitacin al 50% del flujo areo reversible. 4) Una vez conseguida la estabilizacin clnica puede suspenderse.
5) Obstruccin bronquial crnica no reversible con tratamiento. 5) El tratamiento incluye un mnimo de 16 horas al da.

M exico A rgentina CTO Medicina C/ Nez de Balboa, 115 28006 MADRID (Espaa) Tfno.: (91) 782 43 32 / Fax: (91) 782 43 27
Pg. 2 NM C hile U ruguay E-mail: secretaria@ctomedicina.com; iberocto@ctomedicina.com WEB: www.ctomedicina.com; www.iberocto.com
NEUMOLOGA Y CIRUGA TORCICA
Preparacin Examen de Seleccin 05/06 1 Vuelta Seguimiento a distancia

22. Respecto al cor pulmonale y su tratamiento, es FALSO de entre 2) Si la espirometra es normal, podemos descartar el diagns-
las siguientes opciones: tico de asma.
3) En un paciente con tos crnica y espirometra normal hemos
1) La correccin de la hipoxemia es el mejor tratamiento del cor de realizar un test de metacolina.
pulmonale. 4) En un paciente con tos y opresin torcica tras el ejercicio,
2) Los diurticos se emplean para tratamiento del edema, aunque en el que se haya descartado una cardiopata como causa de
tienen poco efecto sobre el problema primario. su opresin, hemos de realizar un test de metacolina o un test
3) Si se instaura tratamiento diurtico es necesario un control de ejercicio.
riguroso de iones sricos, ya que la aparicin de alcalosis 5) El test de metacolina y el test de histamina son test de
metablica disminuira el estmulo del centro respiratorio. provocacin que se realizan ante la sospecha clnica de asma
4) Los vasodilatadores muestran una mejora de la relacin V/Q. con espirometra basal normal.
5) La digoxina slo est indicada si existe fracaso del ventrculo
27. Respecto al diagnstico diferencial del asma, es FALSO entre las
izquierdo.
siguientes opciones:
23. Paciente varn de 60 aos, fumador desde hace 30 aos,
1) En el asma de predominio nocturno evidente, es necesario
diagnosticado de EPOC tipo enfisema, acude a urgencias descartar episodios de aspiraciones digestivas.
porque, tras un episodio de infeccin de va area superior, 2) En los lactantes es de enorme importancia plantearse como
presenta empeoramiento de su disnea habitual, agitacin e diagnstico diferencial la aspiracin de cuerpo extrao.
insomnio. Se le realiza gasometra arterial con PO2 50 mmHg, 3) En personas ansiosas hay que descartar que se trate de una
PCO2 55 mmHg, pH 7,25 y bicarbonato de 31 mEq/l. Ante la disfuncin gltica. Estos casos suelen presentar una mala
situacin de insuficiencia respiratoria se le administra oxgeno, respuesta al tratamiento convencional.
y a las pocas horas comienza con cuadro de estupor y coma. 4) Conviene examinar si existen datos cutneos, neurolgicos
En la gasometra arterial con O2 se objetiva pH 7,09, PO2 70 o de otros rganos, que nos hiciesen pensar que se trata de
mmHg y PCO2 75 mmHg. El paso siguiente es: una vasculitis.
5) Si no hay antecedentes familiares de asma o eosinofilia, es
1) Intubacin orotraqueal y ventilacin mecnica. raro que sea realmente un caso de asma por lo que hay que
2) Retirar el oxgeno, porque es la causa del aumento de CO2. insistir en el diagnstico diferencial.
3) Aumento del oxgeno, pues el estupor es secundario a la
hipoxia cerebral. 28. De los siguientes hallazgos en el paciente con crisis de asma,
4) Iniciar tratamiento antibitico. seale el que se relaciona con obstruccin grave:
5) Teofilinas para estimular el centro respiratorio.
1) Sibilancias difusas.
24. Un paciente con EPOC presenta disnea de medianos esfuerzos. 2) Hiperinsuflacin acentuada.
La espirometra muestra un VEF1 del 60%. De los siguientes 3) Tos con expectoracin.
tratamientos, indique el ms adecuado: 4) Taquicardia.
5) Disnea.
1) Bromuro de ipratropio y betaadrenrgicos.
29. Indique lo FALSO en relacin a una crisis asmtica:
2) Teofilina.
3) Corticoides inhalados y bromuro de ipratropio.
1) Uno de los indicadores de gravedad de la obstruccin es el
4) Oxigenoterapia domiciliaria y betaadrenrgicos.
pulso paradjico.
5) Betaadrenrgicos orales.
2) El pico de flujo es un importante indicador de la gravedad.
3) En situaciones extremas las sibilancias disminuyen de forma
considerable.
ASMA. 4) La aparicin de neumomediastino es una complicacin muy
frecuente.
Preguntas TEST

25. A propsito de las diferencias entre el asma intrnseco y extrn- 5) Pueden producirse atelectasias.
seco, seale la afirmacin correcta:
30. De los siguientes frmacos, indique cules son de eleccin en
1) El asma extrnseco est mediado generalmente por una el tratamiento de los episodios agudos de asma:
reaccin inmunolgica semirretardada.
2) El asma intrnseco presenta en la mayora de los casos una 1) Glucocorticoides inhalados.
elevacin importante de la IgE. 2) Metilxantinas.
3) El asma extrnseco no suele tener en la familia miembros con 3) Simpaticomimticos subcutneos.
alguna enfermedad alrgica. 4) Simpaticomimticos inhalados.
4) En el asma intrnseco podemos encontrar algunos pacientes 5) Glucocorticoides sistmicos.
que tengan intolerancia a la aspirina.
5) La eosinofilia es mayor en el asma extrnseco. 31. En la evolucin de un paciente con crisis de asma se decide
trasladarle a una UVI. Seale de los siguientes el parmetro que
26. Respecto al diagnstico del asma bronquial, es FALSO: ha hecho con mayor probabilidad que esta actuacin sea la
recomendada:
1) Ante una historia clnica compatible y una espirometra
obstructiva, hemos de realizar un test broncodilatador para 1) PEF 60%
demostrar la reversibilidad de la obstruccin bronquial. 2) Taquipnea.

M exico A rgentina CTO Medicina C/ Nez de Balboa, 115 28006 MADRID (Espaa) Tfno.: (91) 782 43 32 / Fax: (91) 782 43 27
C hile U ruguay E-mail: secretaria@ctomedicina.com; iberocto@ctomedicina.com WEB: www.ctomedicina.com; www.iberocto.com
NM Pg. 3
NEUMOLOGA Y CIRUGA TORCICA
Seguimiento a distancia Preparacin Examen de Seleccin 05/06 1 Vuelta
3) Hiperinsuflacin torcica. 3) Las inmunodeficiencias con dficit de produccin de anticuer-
4) PaO2 61 mmHg. pos suelen asociar sinusitis a la presencia de bronquiectasias.
5) PaCO2 46 mmHg. 4) El dficit de IgA es la inmunodeficiencia que ms comn-
mente presenta bronquiectasias.
32. Un paciente presenta clnica de disnea con sibilancias con una 5) Las bronquiectasias post-tuberculosis son causantes de hasta
frecuencia aproximada de 3 veces/semana. De los siguientes el 40% de los casos de hemoptisis masivas.
tratamientos, cul sera el ms apropiado?:
36. Una de las siguientes enfermedades que se asocian con bron-
1) Corticoides sistmicos de base y betaadrenrgicos durante los quiectasias tiene tambin asma mediado por una reaccin
episodios de clnica. inmunolgica tipo I. Indique cul:
2) Cromoglicato de base y betaadrenrgicos durante los episo-
dios de clnica. 1) Sndrome de Williams-Campbell.
3) Corticoides inhalados de base y betaadrenrgicos cuando 2) Sndrome de Mounier-Kuhn (traqueobroncomegalia).
tenga clnica. 3) Aspergilosis broncopulmonar alrgica.
4) Betaadrenrgicos de modo continuo. 4) Sndrome de Kartagener.
5) Nedocromil sdico y corticoides sistmicos. 5) Panhipogammaglobulinemia.

33. Paciente con diagnstico de asma desde hace dos aos y que 37. De los siguientes grmenes, indique cul coloniza a ms del 90%
realiza tratamiento de forma estable con corticoides inhalados de los pacientes con fibrosis qustica (FQ):
y betaadrenrgicos inhalados de corta duracin a demanda.
Acude a Urgencias porque en los ltimos dos das presenta 1) Staphylococcus aureus.
aumento de su disnea, y en la ltimas 24 horas se ha adminis- 2) Haemophilus influenzae.
trado 12 pulsaciones del betaadrenrgico. Su pico de flujo (PEF) 3) Pseudomonas aeruginosa variedad mucoide.
nada ms llegar es 90 l/min (<50%). Esto indicara: 4) Mycoplasma.
5) Legionella.
1) Que la crisis es moderada (PEF <50% del terico), y deben
utilizarse nicamente 4 pulsaciones del betaadrenrgico 38. Respecto a la fibrosis qustica, es FALSO:
inhalado.
2) Que en este caso existe indicacin de ventilacin mecnica. 1) Es una enfermedad de herencia autosmica recesiva.
3) Que la crisis es severa, y deben utilizarse betaadrenrgicos
2) Es la enfermedad letal ms frecuente en la raza blanca.
inhalados de accin corta y corticoides sistmicos.
3) El 90% debuta al nacimiento con leo meconial.
4) Que no precisa oxigenoterapia.
4) El cromosoma afecto es el 7 en su brazo largo.
5) Que no precisar aminofilina intravenosa.
5) La causa ms frecuente de muerte son las complicaciones
pulmonares.
34. Un paciente con historia de asma de 5 aos de evolucin tiene
sntomas continuos con limitacin de su actividad fsica habi-
39. Ante las repetidas sinusitis, otitis y bronquitis que padece un
tual. En perodos de aparente estabilidad su PEF y/o VEF1 est
paciente de 16 aos, su mdico de familia le solicita un TC
entre el 60-80%. En situacin de estabilidad, el tratamiento ms
torcico que demuestra la presencia de espacios areos qus-
adecuado sera:
ticos en toda la va area, junto con ubicacin cardaca en
hemitrax derecho. Por todo ello, usted piensa en:
1) Bromuro de ipratropio pautado y betaadrenrgicos de ac-
cin corta a demanda.
1) Fibrosis qustica.
2) Prednisona oral en dosis de 10 mg/da y betaadrenrgicos de
2) Sndrome de Young.
accin corta a demanda.
3) Sndrome de Kartagener.
3) Corticoides inhalados (budesonida o beclometasona) a dosis
4) Dficit de alfa-1-antitripsina.
>800-1.000 microg/da y betaadrenrgicos inhalados de

Preguntas TEST
5) Aspergilosis broncopulmonar alrgica.
accin larga a demanda.
4) Corticoides inhalados en la dosis del punto anterior, betaa-
drenrgicos inhalados de accin larga y betaadrenrgicos
inhalados de accin corta a demanda. BRONQUIOLITIS.
5) Cromoglicato disdico, teofilinas de accin retardada y
betaadrenrgicos inhalados de accin corta empleados a 40. Respecto a la bronquiolitis, es FALSO:
demanda.
1) La bronquiolitis simple presenta un infiltrado inflamatorio
en la pared bronquiolar, mientras que la obliterante asocia,
BRONQUIECTASIAS. adems, mltiples tapones fibrosos que "obliteran" la luz.
2) La bronquiolitis constrictiva muestra fibrosis en la muscular
35. Es FALSO, respecto a las bronquiectasias: con epitelio intacto.
3) Si en la bronquiolitis obliterante se asocia fibrosis en los
1) En Espaa, una causa frecuente es la post-tuberculosis. alveolos cercanos, el proceso se denomina B.O.N.O. (bron-
2) La clnica ms frecuente es tos y expectoracin purulenta quiolitis obliterante con neumona organizada).
aunque en ocasiones los pacientes se encuentran asintom- 4) La bronquiolitis infecciosa suele deberse al VRS.
ticos. 5) La bronquiolitis infecciosa suele afectar a adultos y ser
irreversible.

M exico A rgentina CTO Medicina C/ Nez de Balboa, 115 28006 MADRID (Espaa) Tfno.: (91) 782 43 32 / Fax: (91) 782 43 27
Pg. 4 NM C hile U ruguay E-mail: secretaria@ctomedicina.com; iberocto@ctomedicina.com WEB: www.ctomedicina.com; www.iberocto.com
NEUMOLOGA Y CIRUGA TORCICA
Preparacin Examen de Seleccin 05/06 1 Vuelta Seguimiento a distancia
41. Respecto a las bronquiolitis, una de las siguientes afirmaciones 3) Carcinoma epidermoide - tumor que puede cavitarse.
NO es correcta: 4) Carcinoma epidermoide - hipercalcemia por secrecin ec-
tpica de PTH.
1) La bronquiolitis obliterante (B.O.) idioptica se sospecha 5) Carcinoma microctico - variedad histolgica de carcinoma de
ante una obstruccin no reversible rpidamente progresiva pulmn que ms frecuentemente produce derrame pleural.
con ms de un 25% de neutrfilos en el LBA y sin causa clara.
2) La B.O.N.O. idioptica suele tener una clnica subaguda 46. En el estudio de extensin del cncer de pulmn, una de las
consistente en malestar general, fiebre, tos, expectoracin y siguientes exploraciones NO est indicada:
prdida de peso.
3) La B.O. idioptica generalmente presenta Rx normal o con 1) Gammagrafa sea si hay sntomas o aumento de la fosfatasa
discreta hiperinsuflacin, siendo rara la existencia de infiltrados. alcalina.
4) La B.ON.O. idioptica suele presentar una Rx tpica con 2) Toracoscopia en pacientes con derrame pleural con citolo-
infiltrados perifricos bilaterales. ga negativa.
5) La B.O.N.O. idioptica suele manifestarse con crepitantes y 3) Mediastinoscopia si hay parlisis completa del nervio recu-
acropaquias. rrente larngeo.
4) TAC abdominal para valorar hgado y las suprarrenales si hay
signos de afectacin.
TUMORES PULMONARES.
5) TAC craneal en el adenocarcinoma.
42. Respecto al tumor carcinoide, es FALSO:
47. Uno de los siguientes NO es criterio de irresecabilidad en el
1) Comprenden el mayor porcentaje en el grupo de adenomas cncer de pulmn:
bronquiales.
2) Pueden comportarse como tumores malignos metastatizan- 1) Sndrome de vena cava superior.
do por va hematgena y ganglionar. 2) Metstasis heptica nica.
3) Suelen ser tumores centrales que presentan sintomatologa 3) Adenopatas en mediastino contralateral.
durante meses o aos. 4) Invasin de cuerpos vertebrales.
4) El diagnstico de confirmacin se realiza con puncin aspi- 5) Carcinoma microctico en estadio I.
racin con aguja fina.
5) Su tratamiento es la reseccin quirrgica. 48. NO se considera criterio de inoperabilidad en el cncer de
pulmn:
43. Es cierto, respecto del tumor de Pancoast:
1) VEMS preoperatorio <1000 cc.
1) El tipo histolgico que se asocia con ms frecuencia es el 2) IAM en los tres meses previos a la ciruga.
carcinoma de clulas pequeas. 3) PCO2 > 45 mmHg.
2) No est indicada la mediastinoscopia como procedimiento 4) CV > 45%.
de estadificacin ganglionar. 5) VEMS postoperatorio predicho 670 ml.
3) La radioterapia preoperatoria est indicada dentro de la
planificacin teraputica. 49. Enfermo de 48 aos con carcinoma epidermoide de 4 cm de
4) Aparece sndrome de Horner en todos los pacientes. dimetro en LID. En la FBC se observa lesin endobronquial en
5) Es tpico el dolor escapular y en regin radial de miembros el bronquio del LID situada a 4 cm de la carina principal y en el
superiores. TAC se observan adenopatas hiliares ipsilaterales mayores de
1cm. Cul es el estadio TNM del paciente?:
44. En relacin a las manifestaciones clnicas locales y a distancia
del carcinoma pulmonar son ciertos todos los enunciados, 1) IIa.
EXCEPTO: 2) IIb.
Preguntas TEST

3) IIIa.
1) El carcinoma escamoso y el oat cell, al ser preferentemente 4) IIIb.
centrales, suelen manifestarse con tos y expectoracin he- 5) IV.
moptoica.
2) Cuando el tumor central alcanza un tamao capaz de 50. Un paciente de 60 aos, ex fumador desde hace 10, presenta
obstruir un bronquio lobar, puede producirse neumonitis
en una Rx de trax realizada por una cada casual, un ndulo
obstructiva.
pulmonar de 2 cm de dimetro con bordes muy ntidos. La
3) El adenocarcinoma pulmonar, al ser perifrico, a veces es
actitud ms correcta sera:
descubierto de modo accidental, antes de producir sntomas.
4) El pulmn contralateral es metastatizado con mucha fre-
1) Ya que es ex fumador, se dar de alta al paciente, no sin antes
cuencia.
5) Cerebro, hueso, hgado y glndulas suprarrenales son rga- indicarle que consulte en el caso de presentar sintomatologa
nos metastatizados con frecuencia. respiratoria.
2) Indicar una Rx de trax a los 3 meses para controlar el tamao,
45. Indique la asociacin INCORRECTA: y si hubiera algn cambio, realizar entonces broncoscopia.
3) Realizar un TC torcico, y si el ndulo tuviera calcificacin,
1) Carcinoma microctico - hiponatremia. dar el alta al paciente, ya que cuando los ndulos tienen
2) Adenocarcinoma - tumor que asienta sobre zonas cicatrizales. calcio son benignos.

M exico A rgentina CTO Medicina C/ Nez de Balboa, 115 28006 MADRID (Espaa) Tfno.: (91) 782 43 32 / Fax: (91) 782 43 27
C hile U ruguay E-mail: secretaria@ctomedicina.com; iberocto@ctomedicina.com WEB: www.ctomedicina.com; www.iberocto.com
NM Pg. 5
NEUMOLOGA Y CIRUGA TORCICA
Seguimiento a distancia Preparacin Examen de Seleccin 05/06 1 Vuelta
4) Realizar broncoscopia con biopsia transbronquial. 4) QT neoadyuvante y posterior intervencin quirrgica.
5) Realizar de entrada toracotoma diagnstica, ya que a un 5) QT de induccin, seguida de RT.
ndulo de ese tamao es prcticamente imposible acceder
con biopsia transbronquial.
ENFERMEDADES DE LA PLEURA, EL MEDIASTINO Y EL DIA-
51. Est usted valorando la operabilidad de un paciente con carci- FRAGMA.
noma anaplsico de clulas grandes y se encuentra con un
VEMS de 1.500 ml. Esto indicara que: 56. En relacin a la etiologa del derrame pleural, son ciertos todos
los enunciados, EXCEPTO:
1) Ya es inoperable el paciente.
2) Se debe mirar ahora el valor de la CPT. 1) La causa ms frecuente de derrame pleural es la insuficiencia
3) Se debe mirar el valor de la DLCO. cardaca.
4) Se debe realizar una gammagrafa de perfusin y calcular el 2) El derrame paraneumnico es la causa ms frecuente de
VEMS predicho postoperatorio. exudado.
5) No hay problema para la operabilidad. 3) El derrame pleural producido por el mesotelioma est en
clara relacin con exposicin al asbesto.
52. Con respecto a la clasificacin TNM del cncer de pulmn, 4) El derrame tuberculoso es una causa frecuente de derrame
seale cul de las siguientes opciones le parece correcta: pleural, sobre todo en mayores de 65 aos.
5) El embolismo pulmonar puede producir derrame pleural.
1) Tumor de 2 cm de dimetro: T2.
2) Derrame pleural tumoral: M1. 57. Indique el signo radiolgico ms frecuente en Rx de trax en
3) Metstasis ganglionares subcarnicas: N1. bipedestacin en un paciente con derrame pleural:
4) Invasin del nervio frnico: T4.
5) Parlisis recurrencial: T4. 1) Aumento de distancia entre cmara gstrica y margen pulmo-
nar izquierdo inferior.
53. Paciente diagnosticado de carcinoma broncognico tipo oat 2) Menisco en borde lateral de uno de los hemitrax.
cell que presenta hinchazn de cara, cuello y circulacin 3) Ensanchamiento cisural.
colateral desde hace varias semanas, con adenopatas sub- 4) Opacificacin de un hemitrax.
carnicas tumorales y sin evidencia clnica de metstasis a 5) Borramiento de ngulo costofrnico.
distancia. Seale el estadio correcto:
58. El derrame pleural en relacin con la artritis reumatoide presen-
1) Ia ( T1N0M0 ). ta todas las siguientes caractersticas, EXCEPTO:
2) IIIb ( T4N3M0 ).
3) IIIb ( T4N2M0 ). 1) Predominio por el sexo masculino.
4) IIb ( T2N1M0 ). 2) Es generalmente unilateral y de predominio derecho.
5) IV ( T4N0M1). 3) Puede aparecer antes que las manifestaciones articulares de
la enfermedad.
54. Paciente de 61 aos de edad, diagnosticado de carcinoma 4) Es un exudado con niveles de glucosa <30 mg/dl en la
epidermoide en LSI. En la broncoscopia se identific que la mayora de los casos.
tumoracin se originaba en un bronquio subsegmentario del 5) El nivel del pH es generalmente >7,35.
culmen. Rx y TC: masa de 2 cm de dimetro en LSI, sin
evidenciarse adenopatas mediastnicas de tamao significa- 59. En relacin al derrame pleural del lupus eritematoso sistmico,
tivo. Analtica: Ca 9,8, FA 105, Na 130, K 3,9, bilirrubina 1,4, todas las caractersticas son ciertas, EXCEPTO:
GOT 258, GPT 450. Seale cul de las siguientes exploracio-
nes estara indicada en este caso: 1) En la mitad de los casos el derrame es bilateral.

Preguntas TEST
2) Cuando es unilateral, predomina en el lado izquierdo.
1) Gammagrafa sea. 3) La combinacin de derrame pleural bilateral y aumento del
2) ECO-TC abdominal. tamao de la silueta cardaca sugiere el diagnstico, sobre
3) TC cerebral. todo en mujeres jvenes.
4) RM torcica. 4) El pH en lquido pleural es tpicamente <7,2.
5) Broncoscopia con cepillado bronquial y citologa del aspirado. 5) La glucosa en lquido pleural es aproximadamente igual al
valor en sangre.
55. Seale cul es el tratamiento de eleccin de un paciente con
carcinoma epidermoide en LID, de 7 cm de dimetro mayor, 60. Un paciente con derrame pleural unilateral presenta en el estudio
con invasin de diafragma derecho y sin afectacin de ganglios del lquido de toracocentesis un hematocrito del 2%. De los
linfticos locorregionales ni evidencia clnica de metstasis a siguientes diagnsticos, indique el ms probable:
distancia:
1) Derrame pleural tuberculoso.
1) RT torcica, por tratarse de un caso irresecable. 2) Derrame pleural paraneumnico.
2) QT, pues se trata de una enfermedad T4. 3) Derrame pleural por embolismo pulmonar.
3) Ciruga con reseccin pulmonar ampliada a diafragma y 4) Insuficiencia cardaca.
linfadenectoma. 5) Linfangioleiomiomatosis.

M exico A rgentina CTO Medicina C/ Nez de Balboa, 115 28006 MADRID (Espaa) Tfno.: (91) 782 43 32 / Fax: (91) 782 43 27
Pg. 6 NM C hile U ruguay E-mail: secretaria@ctomedicina.com; iberocto@ctomedicina.com WEB: www.ctomedicina.com; www.iberocto.com
NEUMOLOGA Y CIRUGA TORCICA
Preparacin Examen de Seleccin 05/06 1 Vuelta Seguimiento a distancia

61. Un paciente presenta un derrame pleural de las siguientes 3) Teratoma.


caractersticas protenas LP/P 0,65, glucosa 60, pH 7,4, linfoci- 4) Neurofibroma.
tos 60% y ADA 15 U/L. NO es probable: 5) Paraganglioma.

1) Derrame pleural maligno. 67. Cul de las siguientes NO es indicacin de intervencin quirr-
2) TBC Pleural. gica en un neumotrax?:
3) Lupus eritematoso sistmico.
4) Artritis reumatoide. 1) Neumotrax espontneo bilateral simultneo.
5) Empiema. 2) Primer episodio de neumotrax con fuga area durante ms
de 7 das.
62. Seale lo FALSO en relacin al mesotelioma maligno: 3) Neumotrax espontneo contralateral a uno previo.
4) Segundo episodio de neumotrax espontneo ipsilateral.
1) Es el tumor pleural ms frecuente. 5) Neumotrax yatrgeno.
2) En la mayora de los casos se relaciona con exposicin previa
al asbesto. 68. Mujer de 52 aos de edad, con antecedente de mastectoma
3) Uno de los sntomas es el dolor torcico. derecha por adenocarcinoma de mama. Acude a Urgencias
4) En muchos casos, el diagnstico no se confirma hasta la por presentar disnea progresiva hasta hacerse de reposo. La
autopsia. Rx de trax muestra la presencia de derrame pleural izquierdo
5) Con frecuencia hay derrame pleural hemorrgico. en cantidad moderada. Se realiza una toracocentesis diagns-
tica, obteniendo un exudado mononuclear con citologa po-
63. Todos los enunciados siguientes referidos al neumotrax son sitiva para adenocarcinoma. Seale la respuesta correcta:
correctos, EXCEPTO:
1) El derrame pleural maligno es una contraindicacin de
1) El neumotrax espontneo primario ocurre sin traumatismo drenaje pleural con tubo, dado el mal pronstico que tienen
previo. estos pacientes.
2) El neumotrax espontneo secundario ocurre con trauma- 2) Si no existe atelectasia asociada, debera colocarse tubo de
tismo previo. trax para realizar pleurodesis.
3) El neumotrax espontneo primario suele ocurrir por ruptu- 3) Abstencin teraputica, pues la disnea suele desaparecer con
ra de bullas apicales. el tiempo.
4) El anterior neumotrax puede ocurrir en fumadores. 4) Se realizar biopsia pleural percutnea, pues la citologa
5) La causa ms frecuente de neumotrax espontneo secun- positiva no es diagnstica de derrame maligno.
dario es la EPOC. 5) Debe realizarse toracotoma con extirpacin de las metstasis
pleurales.
64. Con relacin a la parlisis diafragmtica, seale lo FALSO:
69. Un paciente de 50 aos, fumador de 30 cigarrillos diarios desde
1) La parlisis unilateral puede deberse a infiltracin tumoral los 20 aos, presenta tos desde hace 2 meses y sintomatologa
por carcinoma broncognico. constitucional. En el ltimo mes observa disnea que se ha hecho
2) La parlisis unilateral suele tener intensa disnea. progresiva, siendo en el momento de la consulta de medianos
3) El diagnstico de lo anterior se confirma mediante fluorosco- esfuerzos. Se realiza una Rx de trax, donde se aprecia un
pia o radioscopia dinmica. derrame pleural izquierdo. La espirometra muestra una CVF
4) En la parlisis bilateral, la capacidad vital est marcadamente del 60%. Se realiza una toracocentesis, donde se aprecia un
reducida. lquido serohemtico, con una relacin de protenas lquido
5) En la parlisis bilateral, el tratamiento de eleccin es la pleural/suero 0,7 y una glucosa 70 mg/dl. Se realiza una biopsia
respiracin asistida. pleural cerrada donde no se objetivan granulomas, y la citologa
es negativa para proceso tumoral. La actitud ms correcta en
Preguntas TEST

65. Con relacin a las hernias diafragmticas, seale lo FALSO: este caso sera:

1) Las del agujero de Bochdalek se localizan en la parte poste- 1) Iniciar tratamiento con un antibitico de amplio espectro, ya que
rolateral de trax. se ha descartado de modo razonable un proceso neoplsico.
2) Generalmente aparecen en el lado izquierdo. 2) Ya que la causa ms frecuente de derrame pleural es la
3) Son ms frecuentes en pacientes obesos las de Bochdalek. insuficiencia cardaca, deberamos iniciar tratamiento con
4) La hernia de Morgagni aparece con ms frecuencia en la un diurtico y repetir la Rx de trax a los 7 das.
parte anterior del trax. 3) Ya que el lquido es serohemtico, esto hace que la causa ms
5) Si dan sntomas, se deben intervenir. probable sea un tromboembolismo pulmonar.
4) No se ha descartado un proceso tumoral, por lo tanto el
66. Un paciente se realiza una Rx de trax para incorporarse al siguiente proceder debera ser la toracoscopia y biopsia con
servicio militar, y se aprecia una masa en el mediastino anterior control visual.
con calcificaciones. El paciente est asintomtico. El diagnsti- 5) Debera ponerse de inmediato un tubo para drenar la
co ms probable ser: cavidad pleural.

1) Ganglioneuroma. 70. Ante el diagnstico de carcinoma epidermoide de esfago, se


2) Linfoma. realiza una reseccin del tercio superior del esfago a un varn

M exico A rgentina CTO Medicina C/ Nez de Balboa, 115 28006 MADRID (Espaa) Tfno.: (91) 782 43 32 / Fax: (91) 782 43 27
C hile U ruguay E-mail: secretaria@ctomedicina.com; iberocto@ctomedicina.com WEB: www.ctomedicina.com; www.iberocto.com
NM Pg. 7
NEUMOLOGA Y CIRUGA TORCICA
Seguimiento a distancia Preparacin Examen de Seleccin 05/06 1 Vuelta
de 62 aos, transcurriendo el postoperatorio sin incidencias, 3) Ante un cuadro clnico compatible es preciso una historia
y siendo dado de alta a los 6 das de la intervencin. Una semana detallada para buscar factores de riesgo.
despus, el paciente acude al hospital por disnea de moderados 4) El derrame pleural unilateral es la manifestacin radiolgica
esfuerzos. Se realiza una Rx trax, objetivndose un derrame ms frecuente.
pleural izquierdo. Seale la cierta: 5) El sncope suele ser un sntoma de embolismo masivo.

1) Probablemente se trata de un hemotrax, dado el antece- 74. Con respecto al TEP, una de las siguientes afirmaciones NO es
dente quirrgico reciente. correcta. Cul?:
2) Debe tratarse de un derrame tumoral metastsico, ya que los
tumores pleurales suelen ser de origen metastsico. 1) La sospecha clnica alta es muy valiosa para tomar decisiones.
3) Se debe sospechar un quilotrax. Un nivel de triglicridos 2) La radiografa de trax puede mostrar elevacin de un
>110 mg/dl apoyar este diagnstico. hemidiafragma.
4) Se trata de un pseudoquilotrax de larga evolucin. 3) La pletismografa de impedancia en pacientes con sntomas
5) El tratamiento es quirrgico de entrada. de trombosis venosa profunda tiene una sensibilidad del
90% aproximadamente cuando se evalan las pantorrillas.
4) La ultrasonografa es en la actualidad una de las tcnicas ms
EMBOLISMO PULMONAR E HIPERTENSIN PULMONAR utilizadas entre las no invasivas para evaluar las extremidades
PRIMARIA. inferiores.
5) En algunos casos es preciso realizar una arteriografa pulmonar
71. Es FALSO sobre la hipertensin pulmonar primaria (HPP): para asegurar o descartar el tromboembolismo pulmonar.

1) Para su diagnstico se requiere una presin media de la 75. Con relacin al tratamiento anticoagulante, seale lo FALSO:
arteria pulmonar mayor de 25 mmHg en reposo y mayor de
30 mmHg con el esfuerzo. 1) La heparina sdica por va intravenosa es el anticoagulante
2) Para diagnosticar una hipertensin pulmonar como prima- de eleccin.
ria, es preciso haber descartado patologa cardaca o pulmo- 2) Para conseguir el efecto deseado, el tiempo parcial de
nar que pueda producir una forma secundaria, as como tromboplastina activado (TTPA) se establece entre 1,5-2,5 el
enfermedad del tejido conectivo. tiempo control.
3) La infeccin por VIH, la inhalacin de cocana y la ingesta de 3) Las HBPM han demostrado ser tan eficaces y seguras en el
frmacos supresores del apetito, as como la hipertensin tratamiento del TEP como las HNF, por lo que cada vez son
portal, pueden producir una enfermedad vascular pulmonar ms utilizadas.
con caractersticas clnicas y patolgicas similares a la HPP. 4) La principal complicacin del tratamiento con heparina es
4) Es una enfermedad rara que predomina en mujeres en la 2 la hemorragia, que aparece aproximadamente en un tercio
y 4 dcada. de los pacientes.
5) Una forma anatmica llamada venooclusiva predomina sin 5) Son tambin efectos secundarios de la heparina la plaque-
embargo a partir de los 60 aos. topenia y osteoporosis.

72. Indique lo FALSO en relacin al tratamiento de la HPP: 76. Paciente varn de 75 aos, con antecedentes de fumador y
bebedor habitual. Ha sido intervenido hace 5 das de una
1) La enfermedad es de mal pronstico, con una supervivencia tumoracin en el sistema nervioso central y presenta disnea
media sin tratamiento de 2,5 aos. brusca. A la exploracin, el paciente est taquicrdico y taquip-
2) Tienen ms probabilidad de responder a los vasodilatado- neico, y en la GAB presenta como nico dato relevante hipoxe-
res a largo plazo por va oral los pacientes que respondieron mia moderada. Se realiza una gammagrafa ventilacin-perfu-
al test de vasorreactividad aguda con vasodilatadores de sin con hallazgo de alta probabilidad de TEP. La actitud ms
accin corta y potente (prostaciclina i.v.) con una disminu- correcta es:

Preguntas TEST
cin de la presin de la arteria pulmonar, un aumento del
gasto cardaco y pocos cambios en la presin sistmica. 1) Evaluar las EEII con tcnicas invasivas para valorar la etiologa
3) Cuando el test de vasorreactividad es como el descrito del TEP.
anteriormente, el uso de vasodilatadores como los bloque- 2) Iniciar anticoagulacin con heparina i.v. los primeros das y
antes del calcio producen una mejora sostenida en la posteriormente cambiar el tratamiento por ACO durante 6
mayora de los pacientes. meses.
4) Cuando los pacientes dejan de responder a los vasodilatado- 3) Colocarle un filtro de cava.
res orales, se podra utilizar la prostaciclina i.v. de modo 4) Iniciar tratamiento con terapia tromboltica.
crnico como puente al trasplante. 5) Realizar tratamiento con HBPM durante 3 meses.
5) El trasplante sera la opcin teraputica cuando los pacientes
tienen la enfermedad ms avanzada. 77. En relacin a las formas clnicas de la neumonitis por hipersen-
sibilidad, todo es cierto, EXCEPTO:
73. Seale lo FALSO en relacin al embolismo pulmonar:
1) La forma aguda aparece a las 4-8 horas de la exposicin, lo cual
1) La disnea de instauracin brusca es generalmente el sntoma est en relacin con un mecanismo inmunolgico tipo III.
ms frecuente. 2) La clnica aguda se caracteriza por fiebre, escalofros, tos no
2) Cuando el embolismo se acompaa de una zona de infarto, productiva, disnea, mialgias, malestar general.
puede haber dolor pleurtico.

M exico A rgentina CTO Medicina C/ Nez de Balboa, 115 28006 MADRID (Espaa) Tfno.: (91) 782 43 32 / Fax: (91) 782 43 27
Pg. 8 NM C hile U ruguay E-mail: secretaria@ctomedicina.com; iberocto@ctomedicina.com WEB: www.ctomedicina.com; www.iberocto.com
NEUMOLOGA Y CIRUGA TORCICA
Preparacin Examen de Seleccin 05/06 1 Vuelta Seguimiento a distancia

3) Los sntomas pueden durar hasta 3 das, tras cesar la exposi- 1) La neumona eosinfila crnica presenta infiltrados migratorios.
cin, pero generalmente duran 1 da. 2) La neumona eosinfila crnica puede asociar asma.
4) El diagnstico puede confundirse con una neumona viral o 3) La toxicidad aguda por nitrofurantona presenta infiltrados
bacteriana. basales.
5) La forma aguda y subaguda se caracterizan por un patrn 4) La eosinofilia pulmonar simple se trata en ocasiones con
intersticial nodular, a veces acompaado de zonas de con- mebendazol.
densacin del espacio areo, con un claro predominio en 5) El sndrome hipereosinfilo afecta sobre todo a varones de
lbulos superiores. edad media.

78. Una de las siguientes afirmaciones en relacin con las enferme- 83. La aspergilosis broncopulmonar se caracteriza por todo lo
dades producidas por polvos inorgnicos NO es verdadera. siguiente, EXCEPTO:
Seale cul:
1) Al depositarse el hongo en el rbol bronquial, prolifera y
1) Las placas pleurales, que son lesiones localizadas sobre todo provoca la produccin de Ig G y E especficas, as como Ig E total.
en la pleura parietal, son especficas de la exposicin a asbesto. 2) Los anticuerpos especficos se unen a antgenos del hongo y se
2) La atelectasia redonda es una complicacin rara de la producen inmunocomplejos, los cuales daan las paredes
exposicin a asbesto. bronquiales producindose bronquiectasias centrales.
3) Las formas de silicosis que ms predisponen a tuberculosis 3) En la patogenia se implica una reaccin inmunolgica tipo
son la aguda y la acelerada. I y tipo III.
4) La enfermedad intersticial producida por exposicin a asbes- 4) El cultivo del Aspergillus fumigatus es un criterio principal de
to es de predominio en lbulos inferiores. enfermedad.
5) El derrame pleural benigno en relacin con la exposicin a 5) La eosinofilia sangunea (generalmente ms de 1.000 eosin-
asbesto tiene un perodo de latencia menor que el mesote- filos/mm3) es un criterio principal.
lioma.
84. Con respecto a la sarcoidosis, cul de las siguientes afirmacio-
79. Con respecto a la silicosis, es FALSO: nes es cierta?:

1) La silicosis simple predomina en lbulos superiores, presen- 1) El sndrome de Lfgren presenta tpicamente adenopatas
tando lesiones micronodulares menores de 1 cm. hiliares unilaterales y patrn intersticial nodular.
2) La silicosis complicada presenta conglomerados mayores de 2) El sndrome de Heerfordt presenta entre sus caractersticas
1 cm en lbulos superiores. uvetis posterior.
3) La silicosis acelerada predomina en campos medios. 3) La sarcoidosis es algo ms frecuente en mujeres, y la mayor
4) La silicosis aguda presenta un patrn alveolar en lbulos incidencia ocurre entre los 20-40 aos.
inferiores. 4) Es tpica de pacientes fumadores.
5) Se han descrito adenopatas hiliares calcificadas en la 5) El derrame pleural aparece aproximadamente en la mitad de
silicosis, pero no son patognomnicas de esta enfermedad. los casos.

80. Uno de los siguientes datos NO esperara encontrarlo en un 85. Indique lo verdadero en relacin a la sarcoidosis:
paciente en el que se sospecha una fibrosis pulmonar idioptica
(alveolitis fibrosante criptogentica): 1) El eje hipotlamo-hipofisario es la parte del sistema endocri-
no que se afecta con ms frecuencia.
1) Predominio de la enfermedad alrededor de los 50 aos. 2) Los rganos reproductores, cuando se afectan, suelen produ-
2) Crepitantes teleinspiratorios en las bases pulmonares. cir esterilidad.
3) Patrn reticular en la radiografa de trax de predominio en 3) Con el embarazo, las manifestaciones de sarcoidosis suelen
campos inferiores junto a la existencia de placas pleurales. empeorar.
Preguntas TEST

4) Disminucin de la CPT. 4) Cuando hay afectacin de la glndula partida, suele ser


5) Disminucin de la difusin. unilateral.
5) La afectacin de la partida no es caracterstica de la sarcoi-
81. En la afectacin pulmonar en las enfermedades del colgeno dosis.
todo es cierto, EXCEPTO:
86. Indique la relacin INCORRECTA en la sarcoidosis:
1) Es ms frecuente en el lupus la enfermedad intersticial que la
pleural. 1) Estadio I: adenopatas hiliares.
2) El lupus puede presentar hemorragia pulmonar. 2) Estadio II: adenopatas hiliares y paratraqueales.
3) La afectacin pulmonar por la artritis reumatoide es ms 3) Estadio III: patrn intersticial reticulonodular.
frecuente en hombres. 4) Estadio IV: patrn en panal.
4) El Sndrome de Sjgren puede comportarse como linfoma. 5) Sndrome de Lfgren: eritema nodoso ms adenopatas
5) La esclerodermia es la que con ms frecuencia desarrolla hiliares bilaterales.
enfermedad intersticial.
87. La histiocitosis X primaria pulmonar o granuloma eosinfilo
82. Con respecto a las eosinofilias pulmonares, es FALSO: pulmonar se caracteriza por todo lo siguiente, EXCEPTO:

M exico A rgentina CTO Medicina C/ Nez de Balboa, 115 28006 MADRID (Espaa) Tfno.: (91) 782 43 32 / Fax: (91) 782 43 27
C hile U ruguay E-mail: secretaria@ctomedicina.com; iberocto@ctomedicina.com WEB: www.ctomedicina.com; www.iberocto.com
NM Pg. 9
NEUMOLOGA Y CIRUGA TORCICA
Seguimiento a distancia Preparacin Examen de Seleccin 05/06 1 Vuelta
1) Es una enfermedad infrecuente que produce una enferme- 4) La radiografa de las formas tpicas suele cursar con patrn
dad pulmonar intersticial y que puede acompaarse de alveolar y broncograma.
lesiones osteolticas solitarias. 5) El derrame pleural acompaa con ms frecuencia a las
2) Se afectan sobre todo varones fumadores con edad funda- neumonas bacterianas.
mentalmente entre 20-40 aos.
3) El pulmn presenta una infiltracin por clulas de Langer- 92 Respecto a la clnica de las neumonas atpicas, es FALSO:
hans (clulas muy diferenciadas), que son histiocitos dendr-
ticos que pertenecen a la serie monocito-macrfago. 1) La Legionella spp en ocasiones cursa con hiponatremia (que
4) La proliferacin de clulas de Langerhans es patognomnica puede ser causa de confusin mental), diarrea o hematuria.
de esta enfermedad. 2) La Chlamydia pneumoniae ocasiona broncoespasmo, con
5) Las clulas de Langerhans se distinguen de otros monocitos sibilancias en la auscultacin pulmonar.
porque al microscopio electrnico tienen unos grnulos 3) La Chlamydia psittaci (causante de la psitacosis) se relaciona
intracitoplsmicos que se llaman de Birbeck. con la exposicin a pjaros y cursa con esplenomegalia.
4) La Coxiella burnetti (causante de la fiebre Q), al ser una
88. La linfangioleiomiomatosis se caracteriza por todo lo siguiente, rickettsiosis, cursa con exantema.
EXCEPTO: 5) En la fiebre Q, la manifestacin extrapulmonar ms frecuente
es la hepatitis.
1) Proliferacin de clulas musculares inmaduras en los pulmo-
nes y en los linfticos del trax y abdomen. 93. Paciente de 47 aos de edad, no fumador, con antecedente de
2) Formacin de quistes de pared fina en pulmones. contacto profesional con ganado, clnicamente asintomtico,
3) Adenopatas retroperitoneales. en el que en estudio radiolgico rutinario mediante Rx y TAC
4) Angiomiolipomas renales. torcico se detecta una lesin qustica de contenido lquido de
5) Derrame pleural hemtico. 6 cm de dimetro en LID. En la analtica sangunea destaca una
leve eosinofilia y serologa positiva para hidatidosis. Cul de las
89. Con relacin a la granulomatosis de Wegener, todos los siguien- siguientes opciones le parece adecuada:
tes enunciados son correctos, EXCEPTO:
1) Seguimiento radiolgico del paciente, ya que el quiste hida-
1) Es una vasculitis necrotizante granulomatosa que afecta funda- tdico pulmonar no suele presentar complicaciones durante
mentalmente a vasos pequeos (arteriolas, vnulas y capilares), su evolucin.
aunque a veces puede afectar tambin a arterias y venas. 2) Iniciar tratamiento con albendazol, ya que la hidatidosis
2) Los granulomas pueden ser intra y extravasculares. pulmonar rara vez requiere ciruga y con tratamiento mdico
3) Afecta generalmente a personas alrededor de los 50-60 aos suele controlarse.
de edad. 3) Realizar broncoscopia y biopsia de la lesin para descartar
4) En la biopsia renal es tpico encontrar tanto la vasculitis la posibilidad de carcinoma broncognico.
4) PAAF de la lesin, si no es accesible mediante broncoscopia.
caracterstica como los granulomas.
5) Toracotoma y extirpacin.
5) La biopsia de senos paranasales y nasofarngeos suele mostrar
granulomas pero no vasculitis.
94. Un paciente de 35 aos, sano previamente y fumador ocasional,
presenta despus de una gripe un cuadro de fiebre de 38C, tos,
90. En relacin al sndrome de Goodpasture, es FALSO que:
expectoracin herrumbrosa y dolor pleurtico derecho. En la
analtica hay leucocitosis y la Rx de trax muestra una conden-
1) El aumento de la DLCO refleja la ocupacin alveolar por la
sacin alveolar en el lbulo superior derecho. Se realiza una
hemorragia.
gasometra arterial, que es normal. Indique la respuesta correcta:
2) La biopsia renal es diagnstica.
3) Los corticoides mejoran la glomerulonefritis.
1) Como se trata de una neumona severa, hay que ingresar al
4) La causa principal de muerte en estos pacientes es la hemo-

Preguntas TEST
paciente y tratarle con penicilina i.v.
rragia pulmonar.
2) Como es una neumona probablemente por Legionella, le
5) La biopsia renal tiene implicacin pronstica.
ingresaramos y pautaramos tratamiento con eritromicina
intravenosa.
3) Como se trata de una neumona severa, ingresaramos al
NEUMONAS.
paciente y le administraramos una cefalosporina de 3
generacin intravenosa.
91. Seale, de todos los enunciados, el que NO es correcto en
4) Como lo ms probable es que la etiologa sea neumoccica,
relacin a las neumonas:
el tratamiento sera con penicilina oral durante 7-10 das.
5) La sospecha de C. psittacci es alta, por lo que le trataramos
1) Las formas tpicas suelen cursar con tos, expectoracin, fiebre con tetraciclina.
alta y dolor pleurtico.
2) El aumento de las vibraciones vocales y el soplo tubrico en 95. Una de las siguientes situaciones en un paciente con neumona
la auscultacin pulmonar es tpico sobre todo de las neumo- NO rene criterios de ingreso hospitalario:
nas virales.
3) Las neumonas atpicas suelen tener manifestaciones genera- 1) Paciente de 26 aos, previamente sano, con 3.500 leucoci-
les y poca focalidad pulmonar. tos/mm3.

M exico A rgentina CTO Medicina C/ Nez de Balboa, 115 28006 MADRID (Espaa) Tfno.: (91) 782 43 32 / Fax: (91) 782 43 27
Pg. 10 NM C hile U ruguay E-mail: secretaria@ctomedicina.com; iberocto@ctomedicina.com WEB: www.ctomedicina.com; www.iberocto.com
NEUMOLOGA Y CIRUGA TORCICA
Preparacin Examen de Seleccin 05/06 1 Vuelta Seguimiento a distancia

2) Paciente de 45 aos, con diabetes mellitus tipo II bien 100. Con respecto al trasplante de pulmn, seale la opcin FALSA:
controlada.
3) Paciente de 23 aos, sin antecedentes de inters, con TA 70/ 1) La mayora de los trasplantados sufre al menos un episodio
40 mmHg, 115 lpm y ms de 28 rpm. de rechazo agudo en los primeros tres meses.
4) Paciente de 62 aos, sin antecedentes, con 14.000 leucoci- 2) En el rechazo agudo, los pacientes pueden presentar leuco-
tos/mm3, 16 rpm y PO2 64 mmHg. citosis, febrcula e infiltrados pulmonares.
5) Paciente de 31 aos, diagnosticado hace 3 das. Se inici 3) El rechazo agudo es la primera causa de mortalidad.
tratamiento antibitico, pero acude a Urgencias nuevamente 4) En el rechazo agudo la sensibilidad y especificidad de la
por persistencia de la clnica (fiebre y disnea de moderados biopsia transbronquial es alta (80-100% respectivamente).
esfuerzos). 5) La manifestacin principal del rechazo crnico es la bron-
quiolitis obliterante, que ocurre generalmente entre el 1-2
96. En los sndromes de hipoventilacin, NO es cierto que: aos postrasplante.

1) Los pacientes con defecto a nivel neuromuscular y del


aparato ventilatorio son capaces de hiperventilar voluntaria-
mente.
2) Los pacientes con defecto a nivel del sistema de control
metablico tienen mediciones normales de resistencia y
distensibilidad.
3) La presin inspiratoria y espiratoria mximas estn conserva-
das cuando hay una alteracin a nivel del sistema de control
metablico.
4) Cuando el defecto es a nivel neuromuscular, las presiones
que se pueden generar en la va area estn disminuidas.
5) Cuando hay un defecto a nivel del aparato ventilatorio, los
volmenes y flujos son tpicamente anormales y est deterio-
rada la respuesta ventilatoria a los estmulos qumicos.

97. Con relacin a las apneas, NO es cierto:

1) En la apnea central cesa el impulso a los msculos respiratorios.


2) En gente sana puede haber episodios de apnea central.
3) Un ndice apnea-hipopnea de 10 o ms sugiere apnea
obstructiva del sueo.
4) En la apnea obstructiva no cesa el impulso respiratorio.
5) El mtodo definitivo que confirma la apnea obstructiva es la
oximetra nocturna.

98. En el sndrome de dificultad respiratoria aguda, NO esperara


encontrar:

1) Compliance pulmonar disminuida.


2) Presin capilar enclavamiento alta.
3) Shunt intrapulmonar.
4) Membranas hialinas intraalveolares.
Preguntas TEST

5) Infiltrados alveolointersticiales.

MALFORMACIONES Y TRASPLANTE PULMONAR.

99. Es FALSO, con relacin al secuestro broncopulmonar:

1) No presenta comunicacin con la va area.


2) Habitualmente su aporte sanguneo procede de una arteria
intercostal.
3) El secuestro extralobar tiene un drenaje venoso anmalo
hacia la circulacin sistmica.
4) El secuestro intralobar suele asentar en el ngulo costofrnico
posterior izquierdo.
5) El tratamiento es la extirpacin quirrgica, aun cuando se
trate de un hallazgo casual.

M exico A rgentina CTO Medicina C/ Nez de Balboa, 115 28006 MADRID (Espaa) Tfno.: (91) 782 43 32 / Fax: (91) 782 43 27
C hile U ruguay E-mail: secretaria@ctomedicina.com; iberocto@ctomedicina.com WEB: www.ctomedicina.com; www.iberocto.com
NM Pg. 11
NEUMOLOGA Y CIRUGA TORCICA
Preparacin Examen de Seleccin 05/06 1 Vuelta Seguimiento a distancia
Pregunta 1.- R: 4 Pregunta 3.- R: 3
La pregunta hace referencia a la divisin de la va area y sus unida- Los patrones ventilatorios anormales vienen definidos en funcin
des anatmicas. La va area se divide en tracto respiratorio superior, de las alteraciones en los volmenes pulmonares estticos y dinmi-
cuya funcin fundamental es acondicionar el aire inspirado para que cos. Hay dos alteraciones ventilatorias fundamentales, las restrictivas y
llegue en perfectas condiciones a la zona de intercambio gaseoso. Este las obstructivas.
ltimo se divide en:
Zona de conduccin: desde la trquea hasta los bronquiolos ter-
minales. Como dice la respuesta 2, se denomina tambin espacio
muerto anatmico y contiene un volumen aproximado de 150ml.
Zona de transicin: formada por los bronquiolos respiratorios
(Resp. 1).
Zona respiratoria: constituida por los ductus y sacos alveolares.
Realizan la funcin fundamental del aparato respiratorio, el inter-
cambio de gases.

Las unidades anatmicas son:


Acino: unidad anatmica distal al bronquiolo terminal, es decir
ductus y sacos alveolares.
Lobulillo primario: sacos alveolares dependientes de un ductus
alveolar (por ello es falsa la respuesta 4, ya que define al lobulillo
primario).
Lobulillo secundario: mnima porcin de parnquima pulmonar
independiente de los lobulillos vecinos y rodeada de tejido con-
juntivo.

La ltima respuesta hace referencia a la cantidad de moco bron-


quial formado por da en el aparato respiratorio. En la trquea y
bronquios es producido en su mayor parte por las glndulas submu-
cosas y en los bronquiolos por las clulas de Clara. Pregunta 3. Alteraciones ventilatorias.

Pregunta 2.- R: 4 El patrn ventilatorio obstructivo se caracteriza por una dificultad


En esta pregunta se repasan los parmetros que evalan la funcin en la espiracin, lo que se pone de manifiesto en las pruebas de
ventilatoria. funcin respiratoria con una disminucin de los flujos espiratorios y
Hay dos tipos de volmenes, los estticos y los dinmicos. un aumento del volumen residual. El primer parmetro que se altera
La mayora de los volmenes y capacidades pulmonares (suma de es el FEF 25-75%, y a ste le sigue la disminucin del FEV1 y del ndice
distintos volmenes) se determinan mediante una espirometra simple de Tiffeneau, junto con el aumento progresivo del volumen residual.
en el caso de los estticos, y forzada en los dinmicos. Sin embargo, La CPT puede mantenerse normal o aumentar a medida que aumen-
tanto el volumen residual como las capacidades que dependen de l ta el VR. Nunca disminuye, dato caracterstico de las enfermedades
(capacidad pulmonar total y capacidad funcional residual) requieren restrictivas. La CV, sin embargo, permanece normal o disminuida, al
otras tcnicas como la pletismografa o la dilucin con Helio. igual que en las enfermedades restrictivas, motivo por el que no es un
El volumen corriente es el volumen inspirado o espirado que se parmetro muy til en el diagnstico diferencial. La CVF es menor
moviliza en cada movimiento respiratorio (aproximadamente 500ml). que la CV puesto que encontramos un colapso precoz de la va area
Esto es lo que define la respuesta 4, haciendo referencia al volumen durante la espiracin en estos enfermos.
residual, por lo que es la respuesta falsa. El volumen residual es el La DLCO slo estara disminuida si hubiese una destruccin acom-
volumen de gas que contienen los pulmones despus de una espira-
paante de la zona de intercambio gaseoso, como ocurre en el enfi-
cin mxima. Es muy importante para que sea posible el intercambio
gaseoso en todas las fases de la ventilacin. La capacidad vital, la sema, pero no en un gran grupo de enfermos con patrn obstructivo,
como son los bronquticos crnicos.
Comentarios TEST

capacidad pulmonar total, el volumen de reserva espiratorio y la ca-


pacidad funcional residual son volmenes estticos correctamente
definidos en esta pregunta. Pregunta 4.- R: 5
La ventilacin es el volumen de aire que se moviliza en la respira-
cin por unidad de tiempo. De todo ese volumen de aire, hay una
parte que no interviene en el intercambio gaseoso. Por ello, cuando
hablamos de ventilacin, podemos hacer referencia a:

Ventilacin Total (VT): volumen total de aire movilizado por uni-


dad de tiempo (minuto).
- VT = Volumen corriente (VC) x Frecuencia resp reposo.
- VT = 500 ml x 12-16 rpm (7L/m).

Ventilacin Alveolar (VA): volumen de aire que interviene en el


intercambio gaseoso por unidad de tiempo (minuto).
- VA = (volumen corriente - Volumen espacio muerto) x
frecuencia respiratoria en reposo.
- VA=(500 ml - 150 ml) x 12-16 rpm (5L/m).

Ventilacin del Espacio Muerto (VEM): volumen de gas que no


interviene en el intercambio gaseoso (volumen del espacio muerto)
por unidad de tiempo.
- VEM =Volumen Espacio Muerto x F.R. reposo.
- VEM = 150 x 12-16 rpm (2L/m).
Pregunta 2. Volmenes pulmonares estticos.

M exico A rgentina CTO Medicina C/ Nez de Balboa, 115 28006 MADRID (Espaa) Tfno.: (91) 782 43 32 / Fax: (91) 782 43 27
C hile U ruguay E-mail: secretaria@ctomedicina.com; iberocto@ctomedicina.com WEB: www.ctomedicina.com; www.iberocto.com
NM Pg. 1
NEUMOLOGA Y CIRUGA TORCICA
Seguimiento a distancia Preparacin Examen de Seleccin 05/06 1 Vuelta
Pregunta 5.- R: 1 correcta de la tcnica denominada pulsioximetra, ya que mediante
Se define hipoxemia como una PaO2 menor de 80mmHg. Hay su utilizacin obtenemos valores de saturacin de la hemoglobina
varios mecanismos causantes de hipoxemia: por el oxgeno. La curva de disociacin de la hemoglobina tiene
forma sigmoidea y nos interesa diferenciar en ella dos partes en las
Disminucin de la PO2 en el aire inspirado. Es la hipoxemia que que el comportamiento es completamente diferente. Una zona de
aparece en las grandes alturas. Segn la intensidad de la hipoxe- pendiente en la que pequeas variaciones en la PaO2 condicionan
mia, el individuo aumentar su frecuencia respiratoria ms o me- grandes cambios en la saturacin de hemoglobina, y una zona de
nos, por lo que puede acompaarse de normocapnia o hipocap- meseta donde grandes cambios en la PaO2 casi no condicionan varia-
nia por hiperventilacin compensadora (respuesta 2). ciones en la saturacin de la hemoglobina. En condiciones normales,
Hipoventilacin. Cursa de modo caracterstico con un aumento el punto donde cambia la curva de pendiente a meseta es lo que
de la PaCO2 y mejora con la administracin de oxgeno. General- corresponde con una PO2 de 60 mmHg y una saturacin aproxima-
mente el gradiente alvolo arterial de oxgeno es normal, salvo que damente del 90%. Este valor y todos los valores superiores aseguran
haya sobreaadido otro mecanismo causante de hipoxemia como una oxigenacin adecuada.
el shunt o las alteraciones ventilacin-perfusin. Esta curva puede ser desplazada hacia la derecha o hacia la izquier-
Alteraciones de la relacin V/Q. Son las ms frecuentes (resp 1 es da en funcin de distintos factores que modifican la afinidad de la
incorrecta). La PaCO2 es variable, depende de la enfermedad sub- hemoglobina por el oxgeno. La disminucin del pH (o aumento del
yacente, y el gradiente de O2 suele estar aumentado, ya que la hidrogeniones), el aumento de la PaCO2, el aumento del 2-3 DPG y el
lesin est en el pulmn, y es entonces cuando se altera este par- aumento de la temperatura desplazan la curva a la derecha, con lo que
metro. Es caracterstica la respuesta a la administracin de oxgeno la afinidad de la Hb es menor y se libera el O2 a los tejidos con mayor
a alto flujo. facilidad. Situaciones inversas desplazan la curva hacia la izquierda.
Efecto Shunt. Los alveolos estn perfundidos, pero no ventilados.
Como en el caso anterior, hay una alteracin del gradiente de
% SATURACIN Sangre Sangre
oxgeno, pero sin respuesta a la administracin de O2 , ya que al
estar lesionada la zona ventilatoria, ste no llega a la zona de inter- HEMOGLOBINA venosa arterial
cambio gaseoso.
Alteraciones de la difusin. Produce hipoxemia en ejercicio. Si
aparece en reposo, hay sobreaadido otro mecanismo como las
alteraciones V/Q.
80

60
AUMENTO HIDROGENIONES
(disminuye pH)
AUMENTO CO2
40 AUMENTO TEMPERATURA
AUMENTO DPG

20

0 20 40 50 60 80 100 120
PO2 (mmHg)
Muestra A Muestra B

Comentarios TEST
Pregunta 7. Curva de disociacin de la hemoglobina y factores modificadores.

Pregunta 8.- R: 4
La hipoxemia condiciona la activacin de mecanismos compen-
Pregunta 5. Algoritmo diagnstico de la hipoxemia. sadores que son tanto ms eficaces cuanto ms lenta y progresiva es su
instauracin. Sus mecanismos compensadores son los siguientes:
Pregunta 6.- R: 1 Aumento del gasto cardaco.
Para asegurar un correcto intercambio gaseoso debe existir una Aumento de la eritropoyesis. Se estimula la secrecin de eritropo-
adecuada relacin entre la ventilacin y la perfusin. La situacin yetina y aparece poliglobulia.
ideal es la concordancia completa entre la ventilacin y la perfusin, Aumento de la ventilacin. El estmulo hipoxmico acta sobre los
con lo que el cociente V/Q tiende a 1. Sin embargo, en bipedestacin quimiorreceptores articos y carotdeos condicionando un au-
y por efecto de la gravedad, se establece tanto un gradiente de ventila- mento de la frecuencia respiratoria que induce la aparicin de
cin como un gradiente de perfusin. En ambos casos, las bases estn alcalosis respiratoria por una disminucin de la PaCO2 secundaria
mejor ventiladas y perfundidas que los vrtices, pero el gradiente cre- a la hiperventilacin (la respuesta 4 es incorrecta). Posteriormente
ciente que se establece vrtice-base es mayor en caso de la perfusin se intenta compensar la alcalosis respiratoria perdiendo bicarbo-
que en el de la ventilacin, motivo por el que la relacin V/Q es mayor nato a nivel renal.
en los vrtices. Aumento del 2-3 DPG para desviar la curva de disociacin de la
La mayor relacin V/Q de los vrtices conlleva que la sangre proce- hemoglobina a la derecha y liberar ms oxgeno a los tejidos.
dente de esa zona tenga una mayor PO2 y una menor PCO2, puesto Vasodilatacin tisular.
que el intercambio es casi perfecto.
Pregunta 9.- R: 4
Pregunta 7.- R: 4 Para asegurar un correcto intercambio gaseoso se necesita una
En esta pregunta repasamos la curva de disociacin de la oxihemo- adecuada relacin ventilacin perfusin. Cuando hablamos de trom-
globina, que es indispensable conocer para hacer una interpretacin

M exico A rgentina CTO Medicina C/ Nez de Balboa, 115 28006 MADRID (Espaa) Tfno.: (91) 782 43 32 / Fax: (91) 782 43 27
Pg. 2 NM C hile U ruguay E-mail: secretaria@ctomedicina.com; iberocto@ctomedicina.com WEB: www.ctomedicina.com; www.iberocto.com
NEUMOLOGA Y CIRUGA TORCICA
Preparacin Examen de Seleccin 05/06 1 Vuelta Seguimiento a distancia
boembolismo pulmonar hablamos de una alteracin ventilacin-per- insuficiencia respiratoria, sin hipercapnia (PaCO2 normal). Adems
fusin tipo espacio muerto. Lo que ocurre es que la unidad est co- hacen mencin al GA-a de O2, que est alterado (>15mmg), lo que
rrectamente ventilada y pobremente perfundida porque hay mbo- indica una alteracin a nivel del parnquima pulmonar o de la va
los en el lecho vascular. Esto condiciona que la relacin V/Q tienda a area, puesto que en las alteraciones de la regulacin de la ventila-
infinito a expensas de un aumento del espacio muerto alveolar (des- cin y de caja torcica est en rango de normalidad. Por ltimo, nos
preciable en condiciones de normalidad) que conlleva un aumento dicen que no hay respuesta a la administracin de O2 a alto flujo para
del espacio muerto fisiolgico. La respuesta correcta es por tanto la 4. corregir la hipoxemia. Una vez excluidas las causas de hipoventila-
La respuesta 1 es incorrecta, porque el espacio muerto anatmico cin, al tener la PCO2 normal, el diagnstico diferencial se va a reali-
es el volumen de aire contenido en la va area que no participa en el zar en funcin de la respuesta al O2 de la hipoxemia:
intercambio de gases, y por tanto no se modifica cuando se lesionan Hay respuesta alteracin V/Q efecto espacio muerto. Por ello la
las zonas de intercambio gaseoso. respuesta 1 es la menos probable.
La respuesta 2 hace referencia al otro tipo de alteracin de la No hay respuesta shunt o cortocircuito (resp. 2-5).
ventilacin-perfusin, el efecto shunt o cortocircuito, en el que la
unidad de intercambio est bien perfundida e incorrectamente venti- Pregunta 13.- R: 5
lada, por lo que la relacin V/Q tiende a cero y la mezcla de sangre En este caso estamos ante un paciente que presenta en la GAB una
que llega a la unidad es similar a la que sale al no realizarse el inter- alcalosis respiratoria, es decir, est hiperventilando, por lo que las dos
cambio de gases. En este caso, la administracin de O2 no corrige la primeras respuestas son falsas, ya que hacen referencia a enfermos
hipoxemia mientras que en el efecto espacio muerto s. incapaces de hiperventilar. La alcalosis respiratoria aparece cuando el
enfermo aumenta su frecuencia respiratoria y elimina ms CO2 de lo
Pregunta 10.- R: 3 habitual. La prdida de CO2 condiciona que el pH aumente por en-
La determinacin de la capacidad de difusin de monxido de cima de 7,45 y d lugar a una situacin de alcalosis de origen respira-
carbono (DLCO) hace una distincin del estado funcional de la mem- torio. En el caso de las respuestas 3 y 4, estamos hablando de enfer-
brana alveolocapilar. La DLCO slo aumenta en 2 situaciones: mos cuya situacin basal es de acidosis respiratoria compensada, es
Fases iniciales de la ICC por un aumento del volumen de sangre decir, pacientes que retienen CO2 y compensan el pH reteniendo
que llegue al capilar de la zona de intercambio, uno de los factores bicarbonato por va renal. Sera necesario una hiperventilacin exce-
que condicionan la DLCO. siva para eliminar ese exceso de PCO2 por debajo del rango de nor-
Hemorragia alveolar, pues la hemoglobina de los hemates vertidos malidad, y adems en ningn caso encontraramos unas cifras de
al alveolo tambin capta monxido de carbono. bicarbonato inferiores a lo normal, ya que existe el mecanismo de
compensacin en estas enfermedades.
Generalmente la DLCO disminuye de forma tpica en todas las En resumen: pH PCO2 HCO3-
situaciones en las que hay alteracin de alguno de los factores que Alcalosis Resp >7,45 <35 Normal
condicionan la DLCO, como la superficie de intercambio, la concen- Acidosis Resp
tracin de hemoglobina en la sangre, la relacin V/Q y el grosor de la - Aguda <7,45 >45 Normal
membrana alveolocapilar. Esto ocurre tpicamente en el enfisema, las - CR/Compensada normal >45 Aumentada
enfermedades intersticiales, el TEP recurrente y la HT Pulmonar.
En el caso de la bronquitis crnica, no se producen alteraciones de Pregunta 14.- R: 3
la DLCO hasta fases muy avanzadas de la enfermedad. En esta pregunta tenemos que valorar 2 gasometras en un mismo
paciente con EPOC como enfermedad de base.
Pregunta 11.- R: 5 La primera gasometra hace referencia a la situacin habitual del
Hay 2 sistemas de control de la ventilacin: el control voluntario, enfermo en fase estable y la segunda al momento de la reagudizacin.
localizado en la corteza cerebral y responsable de estimular o inhibir En ambas hay que valorar 2 cosas:
el impulso cerebral, y el control involuntario. En el control involunta- Oxigenacin: en ambas hay una situacin de hipoxemia, puesto
rio intervienen varios factores: que la PO2 es inferior a 80 mmHg. Adems, estn en rango de
insuficiencia respiratoria al ser la PO2 <60 mmHg.
Quimiorreceptores (QR): hay 2 tipos: los centrales y los perifri-
cos. Los QR centrales estn localizados fundamentalmente a nivel Equilibrio cido-base: la situacin es diferente segn el momento.
En fase estable, el paciente tiene una acidosis respiratoria crnica,
bulbar y son sensibles a las variaciones de la PaCO2 , que es el
es decir, tiene una hipercapnia que compensa reteniendo bicar-
Comentarios TEST

principal regulador de la ventilacin. Su aumento condiciona un


bonato por va renal para mantener el pH compensado. En el
aumento de la frecuencia respiratoria (hiperventilacin). Las varia-
momento de la reagudizacin la hipercapnia aumenta y el bicar-
ciones de PaO2 no actan como estmulo directo a este nivel. bonato retenido por va renal no es suficiente para compensar el
Los quimiorreceptores perifricos estn localizados en el seno
pH, por lo que los encontramos en un valor prximo a la acidosis.
carotdeo y cuerpo artico. Estos son sensibles a las variaciones Es lo que se denomina acidosis respiratoria parcialmente compen-
tanto de PaCO2 como de PaO2. La respuesta a las variaciones de sada o crnica descompensada, si el pH es < a 7,35.
PaCO2 disminuye con la edad, durante el sueo y con la ingesta de Las respuestas 1 y 2 son incorrectas, ya que el paciente tiene datos
barbitricos. La hipoxemia, sin embargo, aumenta la sensibilidad a de insuficiencia respiratoria crnica en ambas gasometras, como ex-
las variaciones de la PaCO2. presa la respuesta 3.
Mecanorreceptores: protegen al pulmn de la sobredistensin La respuesta 4 es incorrecta, ya que la lesin est en la va area y
mediante el reflejo de Hering-Breuer. Cuando se activan inhiben la el parnquima pulmonar, por lo que el GA-a O2 debe ser patolgico
inspiracin. (>15 mmHg) y la 5 tambin, porque no hay datos que indiquen la
Centros respiratorios: generan el impulso ventilatorio y son regu- ventilacin mecnica.
lados tanto por las aferencias que provienen desde los QR centra-
les como las que provienen de los QR perifricos. Pregunta 15.- R: 5
En condiciones normales, el principal regulador de la ventilacin La EPOC es una patologa que se caracteriza por una obstruccin
es la PaCO2; pero en situaciones de hipercapnia crnica es la PaO2 el crnica al flujo areo que generalmente es progresiva y no revierte
principal estmulo respiratorio. Por este motivo, en los pacientes con completamente con tratamiento (respuesta 5). El principal factor de
EPOC no se debe corregir bruscamente la hipoxemia. riesgo es el tabaco. El efecto depende del nmero de cigarrillos, de la
edad de inicio y del tiempo que se fuma. Esto se valora con un
Pregunta 12.- R: 1 parmetro que se denomina paquetes/ao.
Estamos ante el manejo habitual de un paciente en insuficiencia Paquetes/ao = n de paquetes/da por aos fumando.
respiratoria. En los casos clnicos es importante conocer exactamente Si se superan los 20 paquetes/ao, hay mayor riesgo para desarro-
a qu hace referencia la gasometra. En este caso, lo que tenemos es llar una EPOC, y si se superan los 40 paquetes/ao, mayor riesgo de
un paciente con una PaO2 < 60, es decir, hipoxemia en rango de cncer de pulmn.

M exico A rgentina CTO Medicina C/ Nez de Balboa, 115 28006 MADRID (Espaa) Tfno.: (91) 782 43 32 / Fax: (91) 782 43 27
C hile U ruguay E-mail: secretaria@ctomedicina.com; iberocto@ctomedicina.com WEB: www.ctomedicina.com; www.iberocto.com
NM Pg. 3
NEUMOLOGA Y CIRUGA TORCICA
Seguimiento a distancia Preparacin Examen de Seleccin 05/06 1 Vuelta
La disnea suele aparecer alrededor de los 60 aos y tiene un desa- En la radiologa de la bronquitis crnica no hay un patrn caracte-
rrollo progresivo a lo largo de aos. Generalmente antes de la apari- rstico, aunque podemos encontrar habitualmente:
cin de la disnea suelen referir tos y expectoracin. Aumento de la trama bronquial con imgenes en ral de tranva por
El estudio de la funcin pulmonar es til para establecer tanto el engrosamiento y fibrosis de las paredes bronquiales.
diagnstico de obstruccin bronquial (FEV1 < 80% respecto al valor Signos de hipertensin pulmonar precapilar con aumento de la
terico) como la gravedad de la enfermedad (EPOC grave: FEV1 < trama vascular perihiliar y cardiomegalia por aumento del tamao
40% del valor terico). de la aurcula derecha y del ventrculo derecho.
En casos avanzados aparecern datos radiolgicos de enfisema
Pregunta 16.- R: 4 centroacinar, con aumento de los espacios retroesternal y
Cuando hablamos de EPOC, hablamos de 2 grandes grupos de retrocardaco y aplanamiento diafragmtico. Es ms raro el hallaz-
enfermos que se comportan de forma diferente: go de imgenes compatibles con bullas.
Bronquitis crnica: son enfermos con un hbito pcnico (obesos y
con cuello corto) y presentan una clnica habitual de disnea progresi- Pregunta 18.- R: 2
va no muy importante que toleran bien, acompaada de tos con Estamos ante un paciente fumador desde su juventud, con clnica
expectoracin habitual. Tienen infecciones bronquiales con frecuen- caracterstica de EPOC tipo bronquitis crnica. Refiere tos con expec-
cia, lo que condiciona episodios repetidos de insuficiencia respirato- toracin que a veces es hemoptoica y frecuentes reagudizaciones
ria. Son enfermos que en situacin basal tienen una buena tolerancia infecciosas. En este momento ya tiene disnea de moderados esfuer-
a la hipercapnia, por lo que no hiperventilan y mantienen una insufi- zos, lo que nos tiene que hacer pensar que es un paciente que al
ciencia respiratoria crnica con acidosis respiratoria compensada. La menos tiene una EPOC moderada.
presencia de hipoxemia crnica conlleva la aparicin de poliglobulia En sus pruebas de funcin respiratoria lo que vamos a encontrar es
e hipertensin pulmonar (HTP). Esto se manifiesta clnicamente en su un patrn ventilatorio obstructivo con un VEMs y un ndice de
aspecto ciantico abotargado. Todas estas caractersticas se definen en Tiffeneau disminuidos, junto con un aumento del volumen residual.
las respuestas 1, 2, 3 y 5. La capacidad de difusin de monxido de carbono, sin embargo,
Enfisema: en estos enfermos la manifestacin clnica mas importan- estar poco afectada. Como en este momento de la enfermedad la
te es la disnea. No toleran bien la hipercapnia, por lo que hiperven- insuficiencia respiratoria ya est consolidada, lo habitual es encontrar
tilan para compensarla. Por esta razn en su GAB presentan una hipoxemia asociada a hipercapnia (no hipocapnia como dice la res-
hipoxemia leve sin hipercapnia, lo que hace que la aparicin de puesta 2), encontrando datos en la gasometra de acidosis respiratoria
HTP y poliglobulia sea rara. Lo caracterstico en esta alteracin es la crnica con pH normal, PCO2 y HCO3- altos.
destruccin de los septos alveolares y el aumento de los espacios En la radiografa de trax encontraremos un aumento de la trama
alveolares con la hiperinsuflacin consiguiente (respuesta 4). bronquial como consecuencia del engrosamiento y fibrosis de las pare-
des de los bronquios, y un aumento del ndice cardiotorcico y de las
Pregunta 17.- R: 3 ramas vasculares perihiliares, ya que en este momento el paciente ya
La radiologa del enfisema es bastante expresiva en casos evolucio- tendr asociada a la hipoxemia crnica una hipertensin pulmonar.
nados, pero puede ser normal en estadios iniciales. Los datos ms
relevantes son: Pregunta 19.- R: 2
Hiperclaridad pulmonar por un aumento del volumen de aire y
ausencia de vasos en las zonas ms perifricas al perderse los septos
alveolares (resp. 2).
Hiperinsuflacin que se manifiesta por un corazn pequeo y
alargado (corazn en gota, resp. 4), horizontalizacin de las costi-
llas con aumento de los espacios intercostales, aplanamiento y
descenso diafragmtico (resp. 1) y aumento de los espacios retroes-
ternal y retrocardaco.
Presencia de bullas, evidenciables como imgenes qusticas de
pared fina (resp 5).
Datos de hipertensin pulmonar. Solamente se van a apreciar en
casos muy avanzados.

Comentarios TEST

Pregunta 17. Radiografa de trax en un paciente con EPOC tipo enfisema. Pregunta 19. Algoritmo teraputico del EPOC.

M exico A rgentina CTO Medicina C/ Nez de Balboa, 115 28006 MADRID (Espaa) Tfno.: (91) 782 43 32 / Fax: (91) 782 43 27
Pg. 4 NM C hile U ruguay E-mail: secretaria@ctomedicina.com; iberocto@ctomedicina.com WEB: www.ctomedicina.com; www.iberocto.com
NEUMOLOGA Y CIRUGA TORCICA
Preparacin Examen de Seleccin 05/06 1 Vuelta Seguimiento a distancia
El tratamiento de la EPOC comprende varios aspectos: El objetivo de la OCD es corregir la hipoxemia y mantener una
Tratamiento especfico: tiene tres objetivos fundamentales, supre- PaO2 aproximada de 65 mmHg, lo que se consigue generalmente con
sin del hbito tabquico, evitar la contaminacin ambiental y flujos de 1,5-2 lpm administrados mediante gafas nasales un mnimo
control de la infeccin. El tabaco es el factor de riesgo fundamental de 16 horas al da, incluyendo el perodo del sueo.
en el desarrollo de EPOC. Su abandono, junto con oxigenoterapia La OCD siempre se plantea con el enfermo en fase estable (resp 4.
domiciliaria cuando est indicada, son las nicas medidas que incorrecta), una vez abandonado el hbito tabquico y recibiendo
mejoran la supervivencia. Para conseguir la deshabituacin un tratamiento correcto y completo. Los criterios de OCD son una
tabquica se han creado unidades de apoyo en las que se hace un PO2 persistente inferior a 55mmHg o una PaO2 entre 55mmHg y
tratamiento psicolgico y mdico, con terapia sustitutiva y otros 60mmHg con evidencia de hipertensin pulmonar, Cor Pulmonale
frmacos ms recientes como el bupropin. En el control de la crnico, insuficiencia cardaca congestiva, arritmias o poliglobulia. La
infeccin se van a emplear las vacunas antigripal y antineumocci- indicacin se considera definitiva tras 3 meses de tratamiento. Una
ca. La vacuna antigripal disminuye el riesgo de infeccin en aproxi- indicacin no plenamente aceptada es la presencia de una PaO2 >
madamente 2/3 de los pacientes. 60mmHg que presente importante descenso durante el ejercicio o
Tratamiento sintomtico: va dirigido a controlar el trastorno ven- durante el sueo.
tilatorio y la insuficiencia respiratoria y a conseguir una readapta- Nos podemos replantear el tratamiento en aquellos enfermos en
cin al ejercicio y una reinsercin laboral. El tratamiento del tras- los que en la evolucin encontramos cifras de PaO2 superiores a
torno ventilatorio comprende una serie de medidas destinadas a 60mmHg respirando aire ambiental de forma persistente.
facilitar la eliminacin de secreciones; son la estimulacin de la tos
voluntaria y la hidratacin correcta del esputo. El tratamiento Pregunta 22.- R: 4
broncodilatador es un pilar fundamental. Los ms utilizados en El tratamiento del Cor Pulmonale (CP) supone actuar sobre la en-
estos enfermos son los anticolinrgicos, ya que actan tanto a nivel fermedad de base y la insuficiencia respiratoria que provoca, y sobre
de la pequea va area como en las zonas de mayor calibre. Hasta la hipertensin pulmonar. El tratamiento de la enfermedad de base
el momento slo exista un compuesto, el bromuro de ipratropio. supondr en la mayora de los casos actuar sobre la EPOC.
En la actualidad existe el bromuro de tiotropio, cuya ventaja prin- La correccin de la hipoxemia, que es la causa ms importante de
cipal es la administracin cada 24h frente a la administracin cada hipertensin pulmonar, es el tratamiento ms adecuado del cor pulmo-
8h del bromuro de ipratropio. Los frmacos utilizados para dismi- nale. Si iniciamos el tratamiento con oxgeno en la fase reversible de la
nuir la inflamacin son los corticoides, cuya indicacin funda- circulacin pulmonar, obtenemos grandes mejoras hemodinmicas
mental son las reagudizaciones. Los programas de rehabilitacin como la disminucin de la resistencia vascular pulmonar (resp. 1).
han demostrado mejorar la tolerancia al ejercicio y la calidad de Los diurticos tienen un papel en el paciente con cor pulmonale y
vida, aunque no mejoran los parmetros de funcin pulmonar. edema perifrico. Pero hay que recordar que la expansin de volu-
men permite al corazn derecho mantener el G.C. cuando las RVP
Pregunta 20.- R: 4 estn aumentadas. Una diuresis excesiva puede provocar una mala
El dficit de 1 antitripsina (AAT) es una enfermedad hereditaria perfusin por disminucin del G.C. y una alcalosis metablica que a
que se caracteriza por una disminucin de las concentraciones plas- su vez inducir de forma compensadora una mayor retencin de
mticas de AAT. CO2. Por ello es necesario un control riguroso de los iones sricos
La AAT es una protena sintetizada en el hgado y localizada funda- (resp. 2 y 3). La digoxina no est indicada en estos pacientes, salvo si
mentalmente en los pulmones. Inactiva a la elastasa de los neutrfilos hay asociada una taquiarritmia o una insuficiencia ventricular izquier-
contrarrestando as su accin destructora sobre la matriz estructural del da. Este frmaco, aunque mejora la contractilidad del VD, produce
pulmn. Es codificada por un nico gen en el cromosoma 14. La una vasoconstriccin pulmonar, lo que empeora la hipertensin
enfermedad se hereda de forma autosmica codominante, ya que el pulmonar (resp. 5).
fenotipo se determina genticamente por la expresin independiente Los vasodilatadores no han demostrado beneficios a largo plazo.
de los 2 alelos paternos. La mayora de los enfermos tienen un fenotipo La utilizacin de estos frmacos produce fundamentalmente dos efectos
PiZZ con niveles de enzima por debajo del 16% de lo normal (se esta- adversos: la hipotensin sistmica, derivada de que su accin es fun-
blece como nivel protector el 35%). El fenotipo de la poblacin normal damentalmente sistmica, y empeoramiento de la relacin V/Q, al
es el PiMM y el de aquellos pacientes con dficit de la enzima (AAT mejorar el flujo sanguneo de zonas mal ventiladas (por ello la resp. 4
54%) sin manifestaciones clnicas es el Pizz. es la incorrecta).
Comentarios TEST

Existe controversia sobre si la heterocigosis asocia manifestaciones.


El dficit de AAT se asocia fundamentalmente a enfisema panaci- Pregunta 23.- R: 1
nar, aunque tambin podamos encontrar otras lesiones del parnqui- Estamos ante un paciente diagnosticado de EPOC. El enfermo acu-
ma pulmonar como las bronquiectasias (la respuesta 4 es incorrecta). de a urgencias por empeoramiento de su situacin clnica y con datos
La afectacin heptica es menos frecuente que la pulmonar y raras clnicos de encefalopata hipercpnica. En la GAB, el paciente mues-
veces coexisten. En nios se ha descrito hepatitis neonatal con marca- tra una marcada acidosis respiratoria (pH 7,25) y una hipoxemia im-
da colestasis y en adultos cirrosis heptica. portante (pO2 50mmHg). Se instaura tratamiento con oxigenoterapia
Las manifestaciones de la enfermedad pulmonar son las del enfer- que consigue mejorar la oxigenacin. Sin embargo, el paciente no
mo EPOC tipo enfisema cuyo sntoma fundamental es la disnea. La slo no mejora clnicamente sino que progresa a un cuadro de estu-
principal caracterstica es la localizacin radiolgica en las bases (en por y coma. En este momento, la GAB muestra una acidosis an ms
el enfisema del fumador es en los vrtices) y la precoz aparicin, en la importante con unas cifras de PCO2 muy altas.
4 dcada de la vida, si el paciente es adems fumador. El tratamiento La respuesta 4 es correcta, hay que tratar la infeccin que ha des-
de la enfermedad heptica es el trasplante. Para la afectacin pulmonar encadenado la reagudizacin, pero no es el objetivo prioritario en
inicialmente se intenta el tratamiento sustitutivo junto con las medidas este momento. Las teofilinas, como dice la respuesta 5, son estimulan-
generales y si no va bien, se recurre al trasplante. tes respiratorios, pero no lo suficientemente potentes como para re-
solver la situacin en la que se encuentra el paciente.
Pregunta 21:- R: 4 Son incorrectas tanto la respuesta 2 como la 3. No es posible retirar
En estadios avanzados de la enfermedad, cuando existe insufi- el O2, porque se producir una hipoxemia tisular brusca sobre la
ciencia respiratoria clnica, se ha demostrado que la OCD aumen- situacin de acidosis que ya tiene el paciente que podra provocarle
ta la supervivencia de los enfermos. Adems mejora las funciones la muerte y tampoco es necesario aumentar el flujo, porque el estupor
neuropsicolgicas, reduce los perodos de hospitalizacin y consi- es secundario a la hipercapnia y no a la hipoxemia.
gue una mayor tolerancia al ejercicio. Estas acciones se obtienen La actitud adecuada, como expresa la respuesta 1, es la instaura-
como resultado de la correcin de la hipoxia tisular que conlleva cin de la ventilacin mecnica. El deterioro del nivel de conciencia
una disminucin de la hipertensin pulmonar y de la poliglobulia. es el dato clnico fundamental para indicar la ventilacin mecnica
tanto en la insuficiencia respiratoria aguda como en la crnica.

M exico A rgentina CTO Medicina C/ Nez de Balboa, 115 28006 MADRID (Espaa) Tfno.: (91) 782 43 32 / Fax: (91) 782 43 27
C hile U ruguay E-mail: secretaria@ctomedicina.com; iberocto@ctomedicina.com WEB: www.ctomedicina.com; www.iberocto.com
NM Pg. 5
NEUMOLOGA Y CIRUGA TORCICA
Seguimiento a distancia Preparacin Examen de Seleccin 05/06 1 Vuelta
Pregunta 24.- R: 1 del FEV1 un 20% o ms de su lnea basal tras la realizacin de una
El tratamiento de la EPOC en fase estable se establece en funcin prueba de provocacin con agentes inhalados (metacolina o histamina)
de la gravedad de la enfermedad. La gravedad de la enfermedad de- o estmulos fsicos (ejercicio o hiperventilacin). Esto se debe a que hay
pende a su vez del FEV1: una respuesta exagerada a estos agentes por parte de la va area. Las
EPOC leve (FEV1 60-80% del valor terico). respuestas 3, 4 y 5 son correctas. Generalmente, despus de realizar el
El dato clnico ms frecuente es la "tos del fumador" y la disnea o no test de provocacin se realiza el test de broncodilatacin para demos-
existe o es mnima. El tratamiento depende fundamentalmente del trar la reversibilidad de la obstruccin.
tipo de disnea que presente el paciente. Si es continua, se instaurar
tratamiento con anticolinrgicos y se reservarn los agonistas 2 Pregunta 27.- R: 5
para las crisis como medicacin de rescate. Si la disnea es intermi- En el diagnstico diferencial del asma hay que tener en cuenta
tente, se utilizarn los agonistas 2 a demanda. mltiples procesos. La respuesta 1 hace referencia al reflujo gastroeso-
EPOC moderada (FEV1 40-59% del valor terico). fgico. Durante la noche, con el decbito, el reflujo aumenta y son
Hay disnea moderada, tos acompaada o no de expectoracin y ms fciles las microaspiraciones digestivas. Esto produce una irrita-
ruidos patolgicos en la auscultacin pulmonar. Este grupo de en- cin sostenida en la va area que conduce a una situacin de hipe-
fermos recibe tratamiento con anticolinrgicos y agonistas 2 de rreactividad fundamentalmente de predominio nocturno. El trata-
larga duracin para mantener una broncodilatacin sostenida. Los miento con inhibidores de la bomba de protones y medidas postura-
agonistas 2 de corta duracin permanecen como medicacin de les suprime las manifestaciones. En nios de corta edad hay que tener
rescate. Si no hay control sintomtico, se aaden las teofilinas y si en cuenta, como dice la respuesta 2, la posibilidad de aspiracin de
hay insuficiencia respiratoria se instaura la OCD. En la pregunta un cuerpo extrao. En ese caso las manifestaciones suelen ser locali-
estamos en este grupo de enfermos, por lo que la respuesta correc- zadas en la zona del pulmn donde se encuentra el cuerpo extrao.
ta es la 1. La respuesta tres hace referencia a las crisis de ansiedad. El diagnstico
EPOC grave (FEV1 del 40% del valor terico). diferencial se basa en la ausencia de alteraciones en el intercambio de
Todos reciben tratamiento con anticolinrgicos, agonistas 2 de gases y la mala respuesta al tratamiento convencional. Otros procesos
corta y larga duracin y teofilinas. Si no hay buena respuesta se con los que se puede plantear el diagnstico diferencial son la EPOC,
plantea la utilidad del tratamiento con corticoides inhalados. Al el asma cardial, el tromboembolismo pulmonar, fibrosis qustica y
igual que en el grupo anterior, si hay insuficiencia respiratoria se bronquiolitis. Tambin hay que descartar estenosis en las VAS. El diag-
instaura la OCD. nstico diferencial se basa fundamentalmente en los ruidos respirato-
rios. En el caso del asma aparecen sibilancias, que son ruidos respira-
Pregunta 25.- R: 4 torios de predominio espiratorio. En las estenosis de las VAS aparece el
Hay distintas clasificaciones de asma. En esta pregunta se hace estridor, ruido de predominio inspiratorio.
referencia a la clasificacin etiolgica. Segn la etiologa, distinguimos Por ltimo, al valorar un a paciente asmtico hay que considerar
entre asma extrnseco o atpico y asma intrnseco. El asma extrnseco aquellos procesos que pueden cursar con asma, como la neumona
es la que se desencadena por un agente inmunolgico determinado. eosinfila crnica, la aspergilosis broncopulmonar alrgica, las vascu-
La reaccin inmune est mediada por la IgE. Los linfocitos B segregan litis (respuesta 4) y el sndrome carcinoide.
IgE especfica cuando son activados por los linfocitos Th2, los cuales Para terminar, la ltima respuesta es incorrecta, puesto que los
se activan al contacto con el antgeno. Los alergenos se unen a la IgE, antecedentes familiares aparecen en el caso del asma extrnseca, pero
que est unida a la membrana de clulas productoras de sustancias no son caractersticos de la intrnseca, y la eosinofilia puede no apare-
proinflamatorias que van a desencadenar la sintomatologa. La reex- cer en enfermos diagnosticados de asma.
posicin aumenta la sntesis de IgE especfica. Suele iniciarse en la
infancia o en la juventud y cursa de forma intermitente en relacin a Pregunta 28.- R: 2
la exposicin a los antgenos. Es frecuente la historia familiar o perso- Las crisis de asma se clasifican en funcin del grado de obstruc-
nal de alergia, por lo que son tiles las pruebas cutneas que son cin. A mayor obstruccin bronquial, ms grave es la crisis. No obs-
positivas. Por todo lo anterior podemos descartar las respuestas 1 y 3. tante en el momento de la crisis la valoracin se basa, adems de en
En el asma intrnseco no se puede demostrar la existencia de un los datos de pico flujo, en la situacin clnica del enfermo.
agente etiolgico ni un mecanismo inmunolgico responsable. Los Son signos de gravedad la presencia de disnea de reposo, la utiliza-
linfocitos pueden liberar citoquinas, que activan los leucocitos. Esto se cin de la musculatura accesoria, las sibilancias intensas, la diaforesis,
produce tambin en respuesta a algn antgeno, pero sin intervencin taquicardia mayor a 120 lpm, taquipnea mayor a 30 rpm y el pulso

Comentarios TEST
de la IgE. Los eosinfilos son las clulas caractersticas de la reaccin paradjico mayor a 25mmHg.
asmtica activados por la interleuquina 5 y liberan un gran nmero de Existen tambin unos signos llamados de extrema gravedad, que son
sustancias inflamatorias responsables de la sintomatologa. La eosinofi- a los que hace referencia el enunciado de la pregunta. En estos enfer-
lia es mayor en este tipo de pacientes (las respuestas 2 y 5 son incorrec- mos el signo que ms se relaciona con la gravedad de la obstruccin es
tas). De forma caracterstica, comienza en la edad adulta, se asocia a la hiperinsuflacin, ya que a medida que la broncoconstriccin au-
veces a poliposis nasal e intolerancia a los salicilatos y tiene peor evolu- menta, el aire va quedando atrapado en el interior del pulmn. Al no
cin, con sntomas ms persistentes y peor respuesta al tratamiento. Por salir el aire a travs de una va cerrada, no encontraremos sibilancias
lo dicho, la respuesta 4 es la respuesta correcta. (ruido que se produce al salir el aire por una va estenosada) sino silen-
cio auscultatorio, otro de los datos de extrema gravedad. En este mo-
Pregunta 26.- R: 2 mento aparecen signos de compromiso hemodinmico como la
La sospecha de asma bronquial se basa siempre en la historia clni- bradicardia y la hipotensin. Por ltimo, la caracterstica en el patrn
ca y en los hallazgos de la exploracin fsica. La trada clsica es la respiratorio es la disnea intensa acompaada de una incapacidad para
disnea, sibilancias y tos, y a veces opresin torcica. Pueden aparecer hablar. La tos y la expectoracin no son posibles en este momento
de forma simultnea o aislada y son muy importantes las situaciones porque el paciente es incapaz de generar un flujo espiratorio suficiente.
ante las que aparecen estos sntomas.
Para confirmar el diagnstico se realizan unas pruebas de funcin Pregunta 29.- R: 4
respiratoria. Si demostramos un patrn obstructivo, el paso siguiente es La crisis asmtica se manifiesta por la aparicin aguda de disnea,
realizar un test de broncodilatacin para demostrar la reversibilidad de sibilancias, opresin torcica, tos, etc. Estos sntomas y signos pueden
la obstruccin. Se considera que el test es positivo cuando hay una aparecer de forma aislada o combinada. Adems hay que conocer
mejora del FEV1 igual o mayor al 15%. Si la espirometra es normal, no los signos de gravedad y los de extrema gravedad para tomar una
se puede excluir el diagnstico de asma y hay que continuar el estudio actitud teraputica adecuada.
realizando un test de provocacin bronquial. Por ello, la respuesta Son signos de gravedad la disnea de reposo, las sibilancias intensas,
incorrecta es la 2. Los test de provocacin demuestran la hiperreactivi- la diaforesis, la taquicardia >120 lpm, la taquipnea >30 rpm y el
dad de la va area. Se define como hiperreactividad a la disminucin pulso paradjico > 25mmHg.

M exico A rgentina CTO Medicina C/ Nez de Balboa, 115 28006 MADRID (Espaa) Tfno.: (91) 782 43 32 / Fax: (91) 782 43 27
Pg. 6 NM C hile U ruguay E-mail: secretaria@ctomedicina.com; iberocto@ctomedicina.com WEB: www.ctomedicina.com; www.iberocto.com
NEUMOLOGA Y CIRUGA TORCICA
Preparacin Examen de Seleccin 05/06 1 Vuelta Seguimiento a distancia
A medida que la crisis es ms grave, cambia la auscultacin pulmo- esquema de tratamiento. Utiliza criterios clnicos y funcionales para
nar. En las fases de menor obstruccin encontramos una disminucin definir cuatro escalones:
del murmullo vesicular y sibilancias difusas. Cuando progresa la obs- Asma intermitente: los sntomas diurnos aparecen menos de una
truccin, las sibilancias van disminuyendo hasta llegarse al silencio vez por semana con crisis nocturnas muy infrecuentes (menos de
auscultatorio en crisis muy graves. Acompaan al silencio auscultatorio dos veces por mes). Su funcin pulmonar es normal (PEF, o FEV1
otros signos de extrema gravedad como la cianosis, bradicardia, hipo- >80%) y la variabilidad del PEF es menor del 20%.
tensin y deterioro del nivel de conciencia. Asma persistente leve: hay ms de una crisis semanal, pero menos
La medicin del pico flujo espiratorio (PEF) es el parmetro de de una diaria. Los sntomas nocturnos aparecen ms de 2 veces al
mayor valor para evaluar el grado de obstruccin al flujo areo y la mes. La funcin pulmonar es normal, pero a diferencia del grupo
respuesta al tratamiento. El PEF corresponde al flujo mximo obteni- anterior, la variabilidad est entre el 20-30%.
do en una espiracin forzada y tiene una buena correlacin con el Este es el escaln en el que se sita el enfermo del enunciado de la
volumen espirado en el primer segundo (FEV1). Idealmente debe valo- pregunta. En el tratamiento se utilizan corticoides inhalados en
rarse con un espirmetro, pero en el entorno de las urgencias se dosis bajas asociados a un agonista 2 de accin corta de rescate. Si
utilizan medidores de flujo espiratorio ambulatorios (peak flow meter, aparecen sntomas nocturnos se pueden aadir agonistas 2 de
o PFM). accin prolongada. En los nios se pueden sustituir los corticoides
Es importante tambin identificar las posibles complicaciones en el inhalados por inhibidores de las clulas cebadas. Este tratamiento
contexto de una crisis asmtica como el neumotrax, neumomedias- es el que aparece en la respuesta 3.
tino y atelectasias por impactacin de tapones mucosos. Son manifes- Asma persistente moderada: sntomas continuos durante el da
taciones infrecuentes que aparecen generalmente en crisis graves em- con crisis nocturnas semanales. Su PEF / FEV1 est entre el 60-80%
peorando la situacin del enfermo (la resp. 4 es la incorrecta). del terico con una variabilidad mayor del 30%.
Asma persistente grave: sntomas diurnos y nocturnos continuos con
Pregunta 30.- R: 4 crisis graves que le limiten su actividad normal y el sueo. PEF o FEV1
Los agonistas 2 de accin rpida inhalados representan el trata- <60% del valor terico con una variabilidad mayor del 30%.
miento inicial de eleccin en la crisis asmtica. Proporciona una bron-
codilatacin inmediata pero breve (menos de tres horas). Estos frmacos
tambin pueden administrarse por va parenteral, subcutnea o intra- Pregunta 32. Tratamiento del asma.
venosa, aunque no se consigue una mejor respuesta. Por ello, esta va de
administracin es para aquellos pacientes que no responden a la va 567689
7 9286 176962
94
176962
94

1234
inhalada, o cuando, por algn motivo, sta no es posible.
92694726 6


882
Los corticoides sistmicos se utilizan en pacientes que no respon-
den al tratamiento inicial con agonistas 2. Esto generalmente ocurre 14652742
1234
  564424
en crisis moderadas y graves. El efecto beneficioso de estos frmacos 567893578678
4954554
ocurre aproximadamente a las 6-12 horas de su administracin. La 83464
eficacia de los corticoides administrados por va intravenosa y por va
 14652742
oral es similar. 1234 975 582
Los corticoides inhalados no tienen una utilidad demostrada en el 892527867888
4954554
 56442 564424
83464
momento agudo, pero su uso debe iniciarse antes de suspenderse la
terapia sistmica, si el paciente lo tena como tratamiento de base. Esto 975 582
se debe a que el inicio de su accin es gradual. 1234
! 56442 14652742
Las metilxantinas se utilizan como coadyuvantes de los agonistas 8925278678
4954554 " 564424
2. Como monoterapia es un tratamiento menos eficaz. Se utilizan 3894 156442 83464
como tercera lnea de tratamiento en las crisis graves. Tras el momento 4 5#6494
inicial en el que se administran por va intravenosa se aportan por va
oral. 975 582 146527424

! 56442 83464$256
Pregunta 31.- R: 5 12342889
4954554 " 88%949
156442 6 975 582
Comentarios TEST

En los pacientes asmticos, los criterios de ingreso en UCI en una


reagudizacin son clnicos, gasomtricos y de funcin pulmonar. 4 5#6494 9482&
Entre los datos clnicos est la necesidad de ventilacin mecnica
por deterioro del nivel de conciencia o parada cardiorrespiratoria y el Pregunta 33.- R: 3
agotamiento fsico. A todo enfermo, en cualquier fase de la evolucin, se le deben
Los criterios gasomtricos son la hipoxemia <60 mmHg refractaria suministrar instrucciones para iniciar tratamiento inmediatamente, ante
al tratamiento y la hipercapnia refractaria al tratamiento. Nunca se una reagudizacin y sobre cundo y cmo debe solicitar atencin
indicar intubacin orotraqueal y ventilacin mecnica antes de com- mdica. La valoracin de la gravedad y la respuesta al tratamiento se
probar la respuesta al tratamiento, ya que ms del 80% mejoran con basan en los sntomas y en la cada del PEF. En funcin de la gravedad
la terapia broncodilatadora y no requieren esta tcnica cruenta. Pero de la crisis, el tratamiento comprende agonistas 2 nebulizados,
hay que tener en cuenta que una PaCO2 dentro de los valores de oxigenoterapia y corticoides sistmicos fundamentalmente.
normalidad indica que el paciente se est fatigando, ya que lo habi- Crisis leve: los sntomas slo aparecen con el ejercicio. Es capaz de
tual en el contexto de una crisis asmtica es que la PaCO2 est dismi- hablar normalmente y hay buena respuesta al tratamiento inicial.
nuida por hiperventilacin. El FEV1 o PEF es > 70-80%. La PaO2 es > 80 mmHg y la PaCO2 <35
Por ltimo, el criterio de funcin pulmonar que indica ingreso en mmHg. Generalmente es suficiente el tratamiento con agonistas 2
UCI es el PEF menor del 33% a pesar del tratamiento. en nebulizacin o mediante inhalacin con cmara espaciadora.
La taquipnea y la hiperinsuflacin torcica son signos de crisis La reevaluacin inicial se hace a los 30 minutos, y si hay mejora
asmtica grave, pero no de ingreso en UCI. completa, puede darse de alta a las 4-6 horas.
Por tanto, la respuesta 5, que nos habla de una PCO2>45mmHg, Crisis moderada: el paciente refiere disnea de reposo y es capaz
es la opcin adecuada. de hablar con frases cortas. Hay respiracin sibilante y la frecuen-
cia cardaca supera los 100 lpm. La respuesta al tratamiento habi-
Pregunta 32.- R: 3 tual es slo parcial. El FEV1 o PEF se sita entre el 50-80%. La PO2
Segn el estado basal del paciente y la frecuencia de las crisis, est entre 60-80 mmHg y la PCO2 < 42mmHg. Se iniciar trata-
podemos establecer la clasificacin clnica del asma bronquial. Es la miento con oxigenoterapia y agonistas 2 mediante nebulizacin o
ms til desde un punto de vista teraputico, ya que en ella se basa el inhalacin con cmara. Se aadir tambin tratamiento con

M exico A rgentina CTO Medicina C/ Nez de Balboa, 115 28006 MADRID (Espaa) Tfno.: (91) 782 43 32 / Fax: (91) 782 43 27
C hile U ruguay E-mail: secretaria@ctomedicina.com; iberocto@ctomedicina.com WEB: www.ctomedicina.com; www.iberocto.com
NM Pg. 7
NEUMOLOGA Y CIRUGA TORCICA
Seguimiento a distancia Preparacin Examen de Seleccin 05/06 1 Vuelta
corticoides sistmicos. Deben ingresar al menos unas horas, y si es La tos y la expectoracin mucopurulenta son los sntomas princi-
preciso, se pueden aadir otros broncodilatadores. pales, aunque algunos pacientes pueden estar asintomticos. La dis-
Crisis grave: el paciente sufre una importante disnea y tiene un nea, la anorexia y la prdida de peso son sntomas tardos que indi-
habla entrecortada. Presenta signos de gravedad como diaforesis, can enfermedad evolucionada. La hemoptisis moderada es una com-
agitacin, uso de la musculatura accesoria, frecuencia respiratoria plicacin relativamente frecuente que habitualmente se asocia a
>30 rpm, frecuencia cardaca >120 lpm, pulso paradjico > 25 infecciones del tracto respiratorio inferior. La hemoptisis masiva no
mmHg y respiracin sibilante. El FEV1 o PEF es < 50% o es imposi- es frecuente, y en nuestro medio las bronquiectasias secundarias a
ble de realizar. La PaO2 es < 60 mmHg y la PaCO2 >42 mmHg. Se tuberculosis son responsables de esta complicacin en el 40% de los
considera muy grave con criterios de ingreso en UCI cuando apa- casos. La sinusitis se asocia con frecuencia a bronquiectasias de
rece deterioro del nivel de conciencia, cianosis, silencio ausculta- predisposicin gentica como la F.Q., sndrome de Young, sndrome
torio, bradicardia e incapacidad para hablar, la PaO2 es <60mmHg de Kartagener y las inmunodeficiencias con dficit de produccin
y la PCO2 >45 mmHg, su PEF es <33% y puede complicarse con de anticuerpos.
PCR. En este momento estara indicada la intubacin orotraqueal
y la ventilacin mecnica. En la pregunta se hace referencia a un Pregunta 36.- R: 3
enfermo con una crisis grave pero sin criterios de UVI. Su trata- Hay mltiples enfermedades que se asocian a bronquiectasias.
miento es con agonistas 2 inhalados o nebulizados ms corticoi- El sndrome de Williams-Campbell y el sndrome de Mounier-
des sistmicos. Si no hay mejora se pueden aadir teofilinas i.v. y Khun son entidades que cursan con bronquiectasias debido a una
sulfato de magnesio. La respuesta correcta por tanto es la 3. alteracin congnita del rbol traqueobronquial. El sndrome de
Kartagener es una alteracin congnita donde hay una reduccin o
Pregunta 34.- R: 4 ausencia del trasporte mucociliar debido a un mal movimiento de los
Para establecer el tratamiento de asma bronquial estable, lo ms til cilios de las clulas respiratorias. Es por tanto una discinesia ciliar
es utilizar la clasificacin clnica. Se utilizan para establecer dicha clasi- constituida por bronquiectasias, sinusitis y situs inverso.
ficacin criterios clnicos y funcionales. Se establecen 4 escalones. La ausencia de los brazos de denena es la responsable ultraestructu-
Asma intermitente: la sintomatologa es infrecuente y la funcin ral de esta anomala, que afecta al epitelio ciliado del aparato respirato-
pulmonar es normal. rio, al de los senos paranasales, trompas auditivas, conducto deferente
Asma persistente leve: los pacientes refieren ms de una crisis y flagelo de los espermatozoides. Junto con la F.Q. y el sndrome de
semanal, pero menos de una al da. Hay sntomas durante la no- Young son las 3 entidades que cursan con esterilidad y bronquiectasias.
che ms de 2 veces al mes. La funcin pulmonar es normal, pero la La panhipogammaglobulinemia, tambin denominada inmunodefi-
variabilidad se sita entre el 20-30%. ciencia variable comn, puede pasar desapercibida en la infancia y
Asma persistente moderada: tienen sntomas continuos que limi- debutar en la edad adulta con infecciones pulmonares recurrentes. Es
tan su actividad fsica, y durante la noche aparecen sntomas con importante establecer su diagnstico, ya que es susceptible de trata-
mucha frecuencia, lo que altera la calidad del sueo. La funcin miento sustitutivo con gammaglobulinas, con buena respuesta en el
pulmonar (VEF1 o PEF) se sita entre el 60-80% y la variabilidad es control de las infecciones. La Aspergilosis brocopulmonar alrgica es un
>30%. En el tratamiento se utilizan corticoides inhalados en altas sndrome asmtico secundario a una reaccin de hipersensibilidad
dosis (800-1600 mg/d) ms agonistas 2 inhalados de larga dura- tipo I y tipo III frente a antgenos del Aspergillus. Las bronquiectasias en
cin. Los de corta duracin se utilizan como medicacin de resca- estos enfermos tienen un aspecto caracterstico, ya que la dilatacin
te. tiene lugar en la porcin proximal del bronquio (bronquiectasias cen-
El paciente del enunciado est situado en este escaln diagnstico trales). La respuesta 3 es la correcta.
y el tratamiento correcto es el descrito en la respuesta 4.
Asma persistente grave: sntomas diurnos y nocturnos continuos Pregunta 37.- R: 3
con crisis graves que pueden condicionar ingresos hospitalarios de En el adulto, el grado de afectacin respiratoria secundaria a las
repeticin. El FEV1 o PEF es < 60% con una variabilidad >30%. bronquiectasias y a la fibrosis pulmonar es el responsable de la eleva-
da mortalidad. La morbimortalidad de estos pacientes est condicio-
Pregunta 35.- R: 4 nada por las infecciones bacterianas broncopulmonares. Al princi-
Las bronquiectasias se definen como la dilatacin anormal e irre- pio, los grmenes habituales son el Staphylococcus aureus y el
versible del rbol bronquial causada por la destruccin de los com- Haemophilus influenzae. La prevalencia de S. aureus es mucho ms
ponentes elsticos y musculares de las paredes bronquiales. Aunque acentuada durante los primeros aos de vida (30-40%) y disminuye

Comentarios TEST
nuestro pas debe considerarse como un pas desarrollado, la tuber- con la edad. Con el transcurso del tiempo es la Pseudomonas aeruginosa
culosis sigue siendo el factor predisponente que se asocia a bron- el germen que las coloniza crnicamente, siendo muy difcil su erra-
quiectasias con mayor frecuencia. A ello contribuyen la prescripcin dicacin. Su prevalencia aumenta con la edad alcanzando a ms del
de tratamientos incorrectos, el retraso en el diagnstico y la alta inci- 90% de los pacientes adultos. Inicialmente la colonizacin corres-
dencia en pacientes VIH seropositivos. ponde a cepas no mucoides y ms adelante a cepas mucoides (la
Causas de bronquiectasias son: resp. 3 es la correcta). La presencia de B. cepacia se asocia general-
Infecciones: sarampin, tosferina, neumonas necrotizantes, in- mente a un rpido progreso de la enfermedad respiratoria. A veces, se
fecciones virales. encuentra A. fumigatus en los cultivos de esputo. En general, no impli-
Obstruccin bronquial: cuerpo extrao, estenosis postuberculo- ca la aparicin de sntomas y no es necesario el tratamiento.
sas, adenopatas, tumoraciones.
Neumonitis inflamatorias por aspiracin de contenido gstrico o Pregunta 38.- R: 3
inhalacin de gases. La fibrosis qustica (F.Q.) es la enfermedad hereditaria letal ms
EPID como sarcoidosis. frecuente en la raza blanca (1/2500 RN vivos) y es una de las entidades
Alteraciones de la inmunidad: que ms casos est aportando como causa de bronquiectasias en la
- Por hiperrespuesta: ABPA, post-trasplante pulmonar. edad adulta. El defecto gentico produce una alteracin en el regula-
- Inmunodeficiencias: variable comn, enfermedad Bruton, en- dor de conductancia transmembrana (CFTR). El gen del CFTR se loca-
fermedad granulomatosa crnica, VIH, dficit de inmunoglo- liza en el brazo largo del cromosoma 7 y se transmite por una heren-
bulinas (dficit de subclases de IgG, con o sin dficit de IgA). La cia autosmica recesiva. Se han descrito ms de 600 mutaciones que
IgA deficitaria de forma aislada no condiciona la aparicin de pueden producir el fenotipo F.Q.
bronquiectasias (respuesta 4 incorrecta). Las manifestaciones ms frecuentes en el adulto y el adolescente
Malformaciones congnitas traqueobronquiales: sndrome de son la enfermedad pulmonar secundaria a bronquiectasias y fibrosis
Williams-Campbell y sndrome Mounier-Khun. pulmonar, la insuficiencia pancretica con esteatorrea, prdida de
Alteraciones de la escalera mucociliar: FQ, discinesia ciliar pri- peso y, en menor frecuencia, diabetes, azoospermia obstructiva. Una
maria, sndrome de Young. forma de presentacin clsica en el recin nacido es el leo meconial.

M exico A rgentina CTO Medicina C/ Nez de Balboa, 115 28006 MADRID (Espaa) Tfno.: (91) 782 43 32 / Fax: (91) 782 43 27
Pg. 8 NM C hile U ruguay E-mail: secretaria@ctomedicina.com; iberocto@ctomedicina.com WEB: www.ctomedicina.com; www.iberocto.com
NEUMOLOGA Y CIRUGA TORCICA
Preparacin Examen de Seleccin 05/06 1 Vuelta Seguimiento a distancia
Aparece inmediatamente despus del nacimiento y cursa como una la mayora de estas formas de bronquiolitis son errneamente in-
obstruccin intestinal. Su incidencia se sita aproximadamente entre cluidas entre otras formas ms comunes de obstruccin crnica al
el 10-15% (la respuesta incorrecta es, por tanto, la 3). El tratamiento es flujo areo. La radiografa de trax puede ser normal o estar discre-
quirrgico, aunque ocasionalmente responden al enema con tamente hiperinsuflada. La exploracin funcional respiratoria mues-
gastrografn, sobre todo si se utiliza precozmente. tra una disfuncin obstructiva con una DLCO conservada. En el
TACAR se observan datos indirectos de obstruccin bronquiolar
Pregunta 39.- R: 3 como dilatacin de bronquios y bronquiolos terminales, areas de
Estamos ante un paciente joven que presenta manifestaciones res- mosaico (distinta atenuacin) e hiperinsuflacin. En el LBA es ca-
piratorias tanto de va area superior (sinusitis y otitis) como de va racterstica la presencia de neutrfilos en un porcentaje superior al
area inferior (bronquitis). El diagnstico diferencial en este enfermo 25%. Se tratan con corticoides sistmicos en altas dosis, siendo
es muy amplio, aunque podemos reducirlo a aquellas entidades que pobre la respuesta.
cursan con una alteracin en la motilidad ciliar. Esto condiciona una Bronquiolitis obliterante con neumona organizada: afecta por
reduccin o ausencia del transporte mucociliar en distintos epitelios, igual a ambos sexos, preferentemente entre los 40-60 aos. Los
como el del aparato respiratorio, los senos paranasales y las trompas pacientes suelen referir los sntomas a partir de un cuadro pseudo-
auditivas. En este grupo de enfermos podemos incluir la F.Q., el sn- gripal que se contina con tos persistente, fiebre, malestar general,
drome de Young y el sndrome de Kartagener. disnea y prdida de peso. Suele haber crepitantes en la explora-
El dficit de -1-antitripsina es una enfermedad hereditaria que cin fsica y son raras las acropaquias (resp. 5 incorrecta). Desde un
como manifestaciones respiratorias tiene fundamentalmente enfise- punto de vista funcional presenta un trastorno restrictivo con dis-
ma y bronquiectasias. La ABPA cursa con sintomatologa asmtica y minucin de la DLCO. La Rx trax muestra en la mayora de los
bronquiectasias. En ninguna de las dos suele existir afectacin de los pacientes infiltrados alveolares mltiples, parcheados y bilaterales.
senos paranasales o los conductos auditivos, por lo que se pueden En el LBA se ha descrito un aumento de linfocitos con disminucin
descartar las respuestas 4 y 5. del cociente CD4/CD8. Una BTB de buena calidad permite el diag-
El diagnstico definitivo nos lo dan los hallazgos del TAC torcico. nstico por un patlogo experimentado. La mayor parte de los
Adems de imgenes compatibles con bronquiectasias (stas apare- pacientes responde a la corticoterapia.
cen en las 3 entidades enunciadas en las 3 primeras respuestas) nos
hablan de la existencia de un situs inversus. Esto, junto con la presen- Pregunta 42.- R: 4
cia de bronquiectasias y sinusitis, nos tiene siempre que hacer pensar El tumor carcinoide representa el 80-90% de los adenomas bron-
en el sndrome de Kartagener (respuesta 3). quiales. Aunque son los tumores primitivos distintos al cncer de
pulmn ms frecuentes, slo representan el 1-2% de todos los tu-
Pregunta 40.- R: 5 mores pulmonares. Son tumores neuroendocrinos que se originan
La bronquiolitis simple es la inflamacin de la pared bronquiolar. en las clulas del sistema APUD. Desde el punto de vista histolgico
Esa inflamacin inicial puede conducir a un proceso de reparacin se dividen en carcinoide tpico y atpico. El tpico representa el
en el que se produce una proliferacin excesiva de tejido de granula- 90% de todos los carcinoides bronquiales. Son tumores de baja
cin y posteriormente una fibrosis, apareciendo a nivel histolgico malignidad. El atpico representa el 10% de estos tumores y tienen
mltiples tapones fibrosos que "obliteran" la luz bronquiolar. Esta en- un comportamiento agresivo con metstasis por va ganglionar y
tidad es la que se denomina bronquiolitis obliterante. hematgena.
La bronquiolitis obliterante se divide en dos grupos: El 80% de los carcinoides bronquiales son centrales. Su sintomato-
Bronquiolitis obliterante, sin neumona organizada. Afecta de for- loga se relaciona con la ocupacin y/o la obstruccin bronquial. Sus
ma exclusiva o predominante al bronquiolo terminal. La afecta- manifestaciones ms frecuentes son la disnea, fiebre, dolor torcico y
cin puede ser constrictiva o proliferativa segn la afectacin histo- hemoptisis (son tumores muy vascularizados). Es frecuente que por su
lgica. La proliferativa muestra lesiones de tejido fibroso ocupando lento crecimiento la sintomatologa aparezca durante meses o aos.
la luz y la constrictiva fibrosis concntrica en la muscular acompa- Los tumores perifricos (20%) son, en general, asintomticos. El sn-
ada de infiltrado inflamatorio en la pared bronquiolar. drome carcinoide es una entidad clnica constituida por manifesta-
Bronquiolitis obliterante con neumona organizada. En ella la pro- ciones cutneas (enrojecimiento facial), gastrointestinales (diarrea, hi-
liferacin ocurre tambin a nivel de los bronquiolos respiratorios o perperistaltismo), respiratorias (broncoespasmo) y cardiovasculares (co-
a nivel alveolar. Si no hay afectacin del bronquiolo terminal, se lapso vasomotor). Se asocia con mayor frecuencia a los carcinoides
Comentarios TEST

denomina neumona organizada. intestinales. Slo aparece en el 2% de los carcinoides bronquiales, y


Esta clasificacin tiene una traduccin clnica, ya que desde un es caracterstico que aparezca sin presencia de metstasis distales. El
punto de vista funcional, la primera se expresa como un trastorno diagnstico de confirmacin se realiza mediante fibrobroncoscopia.
obstructivo y la segunda como uno restrictivo o con menor frecuencia Su aspecto muy vascularizado es caracterstico, al igual que el sangra-
mixto. do con la toma de biopsia. La puncin aspiracin aguja fina se reserva
La bronquiolitis infecciosa es una de las enfermedades pulmona- para aquellos que escapan al diagnstico por fibrobroncoscopia.
res que con mayor frecuencia requiere hospitalizacin durante la El tratamiento de eleccin es la reseccin quirrgica. Se realiza
infancia. Durante los dos primeros aos de vida, el VRS es el responsa- ciruga conservadora en los tpicos y se reserva la ciruga agresiva para
ble de la mayora de los casos. Pasada esta edad, los virus siguen los atpicos. En aquellos pacientes que presentan una enfermedad
siendo los agentes ms frecuentemente implicados, aunque otros gr- ms all de las posibilidades de la ciruga se utiliza la quimioterapia.
menes como el Mycoplasma pneumoniae son ocasionalmente res- Para el tratamiento del sndrome carcinoide se utilizan los anlogos de
ponsables. Los adenovirus tipos 3, 7 y 21 ocasionan las formas ms la somatostatina.
graves, pudiendo desencadenar la aparicin de una bronquiolitis
obliterante en el 60% de los casos que se recuperan. La respuesta 5 es Pregunta 43.- R: 3
la incorrecta, porque en adultos el nmero de casos es escaso. Por Los tumores del sulcus superior (tumor de Pancoast) son tumores
tanto, la bronquiolitis infecciosa es una enfermedad tpicamente in- que por su localizacin e invasin de estructuras vecinas producen
fantil. manifestaciones caractersticas. El tipo histolgico que se asocia con
ms frecuencia es el carcinoma epidermoide. Suele comenzar con
Pregunta 41.- R: 5 dolor en el hombro o en la regin vertebroescapular por invasin
La bronquiolitis obliterante se clasifica fundamentalmente en: directa de las costillas y msculos. El tumor progresa produciendo
Bronquiolitis obliterante sin neumona organizada: debe sospe- una afectacin de las races C8, T1 y T2 del plexo braquial, lo que
charse en pacientes con tos seca, disnea de esfuerzo y obstruccin conlleva un dolor irradiado por el brazo en su regin cubital y una
bronquial no reversible rpidamente progresiva. La sospecha diag- atrofia de la musculatura de la mano. Cuando invade el ganglio
nostica es ms fcil si no hay otros factores de riesgo para la obs- estrellado y afecta a la cadena simptica causa un sndrome de Horner
truccin bronquial como el tabaquismo. Hoy en da se admite que (ptosis, miosis, enoftalmos y anhidrosis facial). El diagnstico se basa

M exico A rgentina CTO Medicina C/ Nez de Balboa, 115 28006 MADRID (Espaa) Tfno.: (91) 782 43 32 / Fax: (91) 782 43 27
C hile U ruguay E-mail: secretaria@ctomedicina.com; iberocto@ctomedicina.com WEB: www.ctomedicina.com; www.iberocto.com
NM Pg. 9
NEUMOLOGA Y CIRUGA TORCICA
Seguimiento a distancia Preparacin Examen de Seleccin 05/06 1 Vuelta
en la historia clnica, la radiologa de trax y la broncoscopia o la de ACTH), y con menor frecuencia, sndrome de Lambert-Eaton y ce-
puncin aspiracin aguja fina. En el estudio de extensin tumoral se guera retiniana (resp. 1). El carcinoma epidermoide puede asociar hi-
utilizarn el TAC toracoabdominal, la mediastinoscopia y la medias- percalcemia (secrecin inadecuada de PTH) (resp 4) y el carcinoma de
tinotoma. El TAC cerebral y la gammagrafa sea se realizan cuando clulas grandes ginecomastia (produccin de gonadotropinas).
se sospecha una posible afectacin en estos rganos. La radioterapia Adems de los sndromes paraneoplsicos, hay determinadas for-
no aumenta la supervivencia en pacientes con cncer de pulmn mas de presentacin que se asocian ms a una estirpe histolgica que
pero est indicada, de forma preoperatoria, en el tumor de Pancoast a otra. La cavitacin es ms frecuente en el carcinoma epidermoide
para incrementar las posibilidades de reseccin. La ciruga que se (>20%) y en el carcinoma de clulas grandes (<20%), mientras que el
realiza es una reseccin en bloque (la respuesta 3 es la correcta). La derrame pleural y la asociacin a cicatrices previas es caracterstico
supervivencia a los 5 aos flucta entre el 25-50%. del adenocarcinoma (resp 2 correcta y 5 incorrecta), que es adems
la extirpe histolgica que se diagnostica con mayor frecuencia en
Pregunta 44.- R: 4 pacientes no fumadores.
Las manifestaciones clnicas locales estn en funcin de la localiza-
cin tumoral. Los tumores de localizacin preferentemente central, Pregunta 46.- R: 3
como el carcinoma epidermoide y el oat cell, presentan sntomas como Una vez establecido el diagnstico, tipificacin histlogica del tu-
consecuencia de la irritacin, ulceracin y obstruccin bronquial. Tam- mor y situacin endoscpica, es preciso realizar un estudio de exten-
bin pueden aparecer manifestaciones como resultado de la infeccin sin tumoral.
del parnquima pulmonar distal al tumor. La tos con expectoracin La realizacin de una TAC torcica y de la porcin superior del
hemoptoica es el sntoma ms frecuente. El dolor torcico y la disnea abdomen es en la actualidad una exploracin rutinaria. La TAC tor-
aparecen con menos frecuencia. La neumonitis obstructiva o las neu- cica nos aporta informacin sobre la lesin primaria y su relacin con
monas de repeticin en la misma localizacin deben hacer sospechar estructuras vecinas. Es el mtodo no invasivo ms eficaz para la evalua-
la existencia de una obstruccin bronquial que favorece las complica- cin ganglionar mediastnica. La TAC abdominal sirve para evaluar el
ciones infecciosas en el parnquima distal. Los tumores de localizacin hgado y las glndulas suprarrenales (resp 4 correcta).
perifrica, como el adenocarcinoma, pueden ser diagnosticados de La mediastinoscopia es la tcnica ms precisa para evaluar la
forma casual en un examen radiolgico. Dan sntomas ms tardamen- afectacin mediastnica de forma bilateral. Unos autores la indican
te y suelen hacerlo por la afectacin pleural. en todo paciente que vaya a ser sometido a ciruga y otros slo en
Respecto a las manifestaciones a distancia por las metstasis, depen- aquellos cuyas adenopatas sean mayores de 1cm en el TAC o sean
den del rgano afecto. Ocurren con mayor frecuencia en el sistema tumores hiliares de gran tamao. La mediastinotoma se inicia cuan-
nervioso central (SNC), huesos, hgado y glndulas suprarrenales. El do hay sospecha de afectacin ganglionar en la ventana aortopul-
pulmn contralateral es una afectacin poco frecuente (resp. 4 inco- monar, es decir, en tumores de lbulo superior izquierdo y bron-
rrecta). En el momento del diagnstico existen metstasis en el SNC en quio principal izquierdo. Estas dos tcnicas son invasivas y no se
aproximadamente el 10% de los casos, dando lugar a nuseas, vmitos, realizan en pacientes con criterios de irresecabilidad (parlisis del
cambios de conducta, cefalea y focalidad neurolgica. Las metstasis nervio recurrente larngeo). Por este motivo, la respuesta 3 es inco-
seas ocurren con preferencia en la columna vertebral, pelvis y fmur, rrecta. La toracoscopia es til en el estudio del derrame pleural con
produciendo dolor intenso con mala respuesta a los analgsicos. Las citologa negativa para comprobar realmente la ausencia de afecta-
metstasis hepticas y suprarrenales suelen ser asintomticas. cin pleural.
Ver tabla en la parte inferior. La gammagrafa sea est indica ante la presencia de dolores seos,
hipercalcemia o aumento de la fosfatasa alcalina. La TAC craneal
Pregunta 45.- R: 5 debe hacerse de rutina en el adenocarcinoma o carcinoma de clu-
Aproximadamente el 2% de los pacientes tienen sntomas y signos las grandes por el alto riesgo de metstasis en SNC. En el resto de
sistmicos no relacionados con la propagacin tumoral. Son los sn- estirpes histolgicas slo se realizar si aparece clnica sugestiva de
dromes paraneoplsicos. La mayor parte de las manifestaciones estn lesin subyacente.
en relacin con liberacin de sustancias.
El tumor que asocia con mayor frecuencia la presentacin de Pregunta 47.- R: 5
sndromes paraneoplsicos es el carcinoma de clulas pequeas. La resecabilidad hace referencia a la posibilidad de que el tumor
Pueden ser la primera manifestacin, aparecer de forma simultnea a sea oncolgicamente resecado, es decir, que el cirujano sea capaz de
otros signos y sntomas o ser el dato de recidiva tumoral en pacientes quitar toda la masa tumoral.

Comentarios TEST
ya tratados. No son pacientes quirrgicos los pacientes en estadio IIb y IV en la
El carcinoma microctico puede asociar hiponatremia (por secre- clasificacin TNM para el cncer de pulmn de clulas no pequeas
cin inadecuada de ADH), hipopotasemia (por secrecin inadecuada y el carcinoma microctico con estadio TNM superior a I (la respuesta

Pregunta 44. Caractersticas diferenciales de los carcinomas pulmonares.

Relacin con tabaco L ocalizacin y cavitacin Sndromes paraneoplsicos Caractersticas especficas


Centr a l - Sndr om e de Pa ncoa st
Epidermoide +++
++ - Aum enta PTH
- Aum enta ADH
Centr a l - Aum enta ACTH
Microctico ++ Tum or m a s m eta st tico
- - Dedos pa lillo ta m bor
- Sndr om es neur olgicos m iop ticos
- Der r a m e pleur a l m a ligno
Adenocarcinoma - Segundo tum or m s
Per ifr ico
(incluy e - Osteoa r tr opa ta hiper tr fica m eta st tico
-
bronquioloalveolar) - Cr ecim iento sobr e cica tr ices
- Aum ento secr eciones
Per ifr ico
Clulas grandes - - Ginecom a stia
+

M exico A rgentina CTO Medicina C/ Nez de Balboa, 115 28006 MADRID (Espaa) Tfno.: (91) 782 43 32 / Fax: (91) 782 43 27
Pg. 10 NM C hile U ruguay E-mail: secretaria@ctomedicina.com; iberocto@ctomedicina.com WEB: www.ctomedicina.com; www.iberocto.com
NEUMOLOGA Y CIRUGA TORCICA
Preparacin Examen de Seleccin 05/06 1 Vuelta Seguimiento a distancia
5 es la incorrecta). Por lo tanto, son irresecables los tumores que Pregunta 49.- R: 2
invaden el mediastino, corazn, grandes vasos, trquea o carina En esta pregunta hacen referencia a un carcinoma epidermoide y
traqueal, esfago y cuerpos vertebrales, y los que ocasionan derrame su clasificacin TNM. Nos dan datos de tamao y su localizacin en
pleural maligno, metstasis en los ganglios mediastnicos contralaterales funcin de los hallazgos de fibrobroncoscopia. Segn estos datos es
o supraclaviculares y metstasis a distancia. un T2 , ya que su tamao supera los 3cms, est situado a ms de 2cms
de la carina traqueal y no hay aparente invasin de estructuras veci-
nas. En la TAC tenemos hallazgos de adenopatas hiliares homolatera-
les, lo que corresponde a un N1. Finalmente la ausencia de metstasis
es un M0.
El estadio tumoral T2N1M0 corresponde a un estadio TNM IIb (res-
Afectacin
NLR puesta 2). Podemos resumir los estadios TNM como aparecen en el
siguiente esquema:
Sdme. VCS
Afectacin
Traqueal extensa
o < 2 cm de la carina
Pregunta 49. Estadiaje TNM del cncer pulmonar no microctico.

11 12 13 14
25 12 13 113 1113

21 112 113 1112 1113


Cncer microctico
estadio > 1 22 1112 1112 1112 1113

23 1113 1113 1113 1113

4567862898
1

Pregunta 50.- R: 4
En esta pregunta valoramos la actitud ante el Ndulo Pulmonar
Solitario (NPS). Se define as el hallazgo de una densidad radiolgica
Derrame
rodeada de parnquima pulmonar sano, con margen circunscrito
Metstasis
pleural maligno a distancia que mide de 1 a 6 cms en un paciente asintomtico. Lo primero que
hay que hacer es determinar si hay criterios de malignidad o de benig-
Afectacin del
nervio frnico nidad. Son criterios de benignidad: edad menor de 35 aos y no
fumador, ausencia de crecimiento en 2 aos, duplicacin en menos
Pregunta 47. Criterios de irresecabilidad. de un mes, margen ntido, lesiones satlites, calcificacin en "ojo de
buey" o en "palomitas de maz".
No obstante, y a pesar de darse las circunstancias enunciadas, hay En estos casos se hace un seguimiento radiolgico cada 3 meses
un grupo de pacientes estrictamente seleccionados que pueden ser durante 1 ao, y luego anualmente. Cuando no hay criterios de
sometidos a ciruga. En algunos tumores que invaden el frnico infe- benignidad se considera un NPS con criterios de malignidad. En
rior puede hacerse una exresis completa. Pacientes jvenes con tu- estos casos, el primer paso es la realizacin de una fibrobroncosco-
mores localizados que invaden carina pueden ser subsidiarios de pia (FBC) con biopsia. Si se llega al diagnstico de malignidad, hay
carinoplastia y las metstasis nicas en SNC pueden ser extirpadas que hacer el estudio de extensin para indicar el tratamiento co-
junto al tumor primario en su totalidad. rrecto. Cuando no se llega al diagnstico con la FBC se valora la
En relacin con el estadio IIIa, la invasin ganglionar mediastnica localizacin. Si es central, se repite la fibrobroncoscopia. Si es peri-
N2 es un dato de mal pronstico. Por ello, antes de considerar los frico, se pasa al siguiente escaln diagnstico, puncin aspiracin
resecables hay que someterlos a quimioterapia de induccin. Si hay aguja fina (PAAF) guiada por TAC. Esta misma prueba diagnstica se
buena respuesta se operan, y si no, se tratan con quimioterapia.
Comentarios TEST

utiliza cuando hay un NPS central con 2 fibrobroncoscopias nega-


tivas. Cuando la PAAF es negativa hay que realizar una toracotoma
Pregunta 48.- R: 4 diagnstica. En este caso tenemos un paciente mayor de 35 aos
La operabilidad se refiere a la situacin funcional y fisiolgica del con antecedentes de tabaquismo, por tanto tiene criterios de malig-
paciente que le har resistir la ciruga dejando suficiente parnquima nidad. Lo primero que hay que realizar entonces, es una fibrobron-
como para mantener un adecuado intercambio gaseoso. La edad por coscopia con biopsia (resp. 4).
s misma no es una contraindicacin quirrgica, aunque es conocida
la peor evolucin de los pacientes mayores de 70 aos a los que se les Pregunta 51.- R: 4
realiza una neumonectoma. Cuando se habla de operabilidad se hace referencia a la situacin
Los criterios de inoperabilidad se dividen en: funcional y fisiolgica del enfermo. Se valora si podr resistir la ciruga,
Globales: el estado clnico (Karnofsky < 40%) y las enfermedades si quedar suficiente parnquima sano para poder realizar correcta-
sistmicas graves e incontrolables (insuficiencia heptica o renal mente el intercambio de gases.
severas, diabetes, coagulopatas, malnutricin, alteraciones psqui- Se valoran distintas facetas:
cas o cerebrales). Situacin global del paciente: donde hay que tener en cuenta su
Cardacas: arritmias incontrolables, insuficiencia cardaca grave estado general y las enfermedades de base.
refractaria al tratamiento e IAM en los 3 meses previos (en las lti- Si el Karnofsky es menor del 40% o hay enfermedades graves aso-
mas revisiones se reduce a 4-6 semanas). ciadas, no ser subsidiario de tratamiento quirrgico.
Pulmonares: el principal criterio es el FEV1. Si es superior a 2l, el Situacin cardaca: las arritmias mal controladas o los anteceden-
paciente es operable, y si es inferior a 1l, inoperable. Si se sita tes de infarto agudo de miocardio en las semanas previas tambin
entre 1 y 2l, hay que calcular el FEV1 postoperatorio predicho contraindican la ciruga.
mediante una gammagrafa de perfusin. Si es mayor de 800cc, es Situacin pulmonar: el principal criterio de inoperabilidad se basa
operable, si es menor de 08l, no. La CV <45%, la PaCO2 >45mmHg en el FEV1 del paciente, que se evala de la siguiente manera:
y la hipertensin pulmonar grave son tambin criterios de inopera- - FEV1 < 1l: inoperable.
bilidad. (La respuesta incorrecta es la 4). - FEV1 1-2l: es necesario valorar la situacin en la que el paciente
quedar tras la ciruga. El FEV1 postoperatorio predicho se reali-

M exico A rgentina CTO Medicina C/ Nez de Balboa, 115 28006 MADRID (Espaa) Tfno.: (91) 782 43 32 / Fax: (91) 782 43 27
C hile U ruguay E-mail: secretaria@ctomedicina.com; iberocto@ctomedicina.com WEB: www.ctomedicina.com; www.iberocto.com
NM Pg. 11
NEUMOLOGA Y CIRUGA TORCICA
Seguimiento a distancia Preparacin Examen de Seleccin 05/06 1 Vuelta
za mediante una gammagrafa de perfusin. Esta es la situacin ECO-TAC abdominal: se plantea ante la sospecha de metstasis
en la que se encuentra nuestro paciente, por lo que la respuesta heptica o suprarrenal. Este es nuestro caso; encontramos aumen-
correcta es la 4. Si en este clculo obtenemos un FEV1 >08l, el tada tanto la bilirrubina como las enzimas hepticas, lo que nos
paciente podr operarse. obliga a descartar la existencia de dao heptico secundario a
- FEV1 >2l: operable. metstasis hepticas (resp. 2 correcta).

Pregunta 52.- R: 5 Pregunta 55.- R: 3


Cuando hay situaciones especiales debemos conocer a qu cate- Estamos ante un paciente con un carcinoma de clulas no peque-
gora TNM pertenecen. as. Para poder decidir cul es el tratamiento de eleccin hay que
Podemos definir las siguientes situaciones: conocer el estadio TNM, ya que en funcin de ste decidimos la
Invasin del nervio frnico T3. actitud teraputica.
Parlisis de la cuerda vocal (resultante de la invasin del nervio En este caso, tenemos un tumor de 7cms que afecta a estructuras
recurrente larngeo) T4. de la pared inferior de la caja torcica. Por ello se corresponde con un
Afectacin grandes vasos T4. tumor T3. No hay afectacin ganglionar locorregional linftica ni me-
Sndrome de vena cava superior T4. tstasis a distancia. Es por tanto un N0, M0. Un tumor T3 N0 M0 se
Extensin pericardio visceral T4. clasifica como estado TNM IIb. La reseccin quirrgica es el trata-
Extensin pericardio parietal T3. miento indicado en los pacientes en estadio I, II, y IIIA, siempre que el
Tumor de Pancoast T3. tumor y su drenaje linftico sea complementariamente extirpado. La
Tumor de Pancoast con afectacin de cuerpos vertebrales T4. lobectoma y la neumonectoma son los procedimientos de eleccin.
Afectacin traqueal o esofgica T4. Cuando hay afectacin de estructuras de la pared torcica, como es
Derrame pleural o pericrdico con citologa positiva T4. nuestro caso, hay que ampliar la ciruga haciendo reseccin de la
Cncer bronquioloalveolar difuso bilateral M1. pared torcica, o el diafragma. Si hablamos de un tumor de sulcus,
Segn esto, la respuesta 5 es correcta; adems, los tumores < 3 cms estara indicada la radioterapia preoperatoria. Otra situacin especial
se clasifican como T1, el derrame pleural maligno como T4, las mets- son los tumores IIIb con N2. En estos pacientes es necesario realizar
tasis ganglionares subcarinales como N2 y la invasin del nervio frni- poliquimioterapia previa para controlar la enfermedad ganglionar antes
co como T3, por lo que las otras respuestas no son ciertas. de la ciruga. Nuestro paciente tiene un tumor IIb con extensin
diafragmtica, por tanto la respuesta correcta es la que incluye la
Pregunta 53.- R: 3 ciruga. Hay que elegir entre la 3 y la 4. La respuesta 4 no es correcta,
Esta pregunta hace referencia a un carcinoma microctico. Habi- ya que nuestro paciente no tiene afectacin ganglionar, por lo que no
tualmente en esta histologa tumoral se diferencian dos estadios: estara indicada la quimioterapia neoadyuvante. La respuesta 3 per-
Enfermedad localizada: indica que la enfermedad puede ser abar- mite la reseccin completa del tumor y su drenaje linftico, es la
cada por un campo de radioterapia torcica, es decir est limitada correcta. No son candidatos a ciruga los pacientes en estadio IIIb y IV.
al trax, ganglios linfticos regionales (mediastnicos, hiliares y Estos pacientes recibirn tratamiento con poliquimioterapia y radio-
supraclaviculares, ipsi y contralaterales), afectacin del larngeo terapia.
recurrente y sndrome de vena cava superior (SVCS). CIRUGA:
Enfermedad avanzada: no abarcable por radioterapia. - IA.
Sin embargo, a veces tambin se emplea la clasificacin TNM en - IB.
estos pacientes. Esto es importante sobre todo si hablamos de estadios - IIA.
tumorales Ia Ib, ya que son resecables. - IIB.
Nuestro paciente tiene clnica de SVCS, por tanto corresponde a - IIIA.
una clasificacin tumoral T4; tiene adenopatas mediastnicas, lo que - (T3 N1 M0).
corresponde a un estadio ganglionar N2 , y no tiene metstasis (M0). El
estadio TNM que corresponde al T4N2M0 es el IIIb (respuesta 3). En la TRATAMIENTO NEOADYUVANTE + CIRUGA:
clasificacin habitual sera un tumor de clulas pequeas en estadio - IIIA.
localizado no subsidiario de ciruga. Recuerda que el carcinoma - T 1-3 N2 M0.
microctico con estadio TNM>1 es un criterio de irresecabilidad.
QUIMIOTERAPIA-RADIOTERAPIA:

Comentarios TEST
Pregunta 54.- R: 2 - IIIb.
Estamos ante un paciente con 61 aos, diagnosticado de carcino- - IV.
ma epidermoide en LSI. Por los hallazgos radiolgicos y de fibrobron-
coscopia nos encontramos con una masa tumoral <3 cm que est a Pregunta 56.- R: 4
> 2 cm de la carina traqueal. Es, por tanto, un estadio tumoral T1. Las causas de derrame pleural son mltiples y el mecanismo de
Adems no hay evidencia radiolgica de existencia de metstasis gan- produccin vara de unas a otras. Precisamente, atendiendo a estos
glionares (N0). La respuesta 5 no tiene ningn sentido, ya que la clasi- mecanismos, los derrames pleurales pueden dividirse en dos grandes
ficacin histolgica del tumor ya la conocemos. La respuesta 4 tam- grupos exudados y trasudados.
bin es incorrecta ya que la RMN rara vez es necesaria, al no aportar En general, cada patologa produce uno u otro tipo de derrame,
informacin adicional al TAC. Salvo en los tumores de Pancoast o en aunque algunas enfermedades, como el tromboembolismo pulmo-
los ntimamente relacionados con el mediastino (no es nuestro caso), nar o el mixedema, pueden originar ambos (resp. 5).
donde la RMN torcica nos ayuda a dilucidar la relacin del tumor Los exudados se producen cuando hay enfermedad pleural. Los
con el plexo braquial y los grandes vasos mediastnicos, esta prueba trasudados cuando los factores sistmicos que influyen en la forma-
no est indicada. La bsqueda exhaustiva de metstasis en pacientes cin o absorcin de lquido pleural se alteran (aumento de la presin
asintomticos y sin anormalidades analticas no est indicada ante la hidrosttica o disminucin de la presin onctica).
baja probabilidad de deteccin de metstasis silentes. Sin embargo, La separacin entre exudados y trasudados se establece en fun-
cuando existen sntomas o signos sugestivos hay que indicar las explo- cin de los criterios de Light:
raciones pertinentes. Protenas en LP / Protenas plasmticas > 0,5.
TAC cerebral: se realiza cuando hay sntomas neurolgicos, en el LDH en LP / LDH plasmtica > 0,6.
oat cell (hay un 10% de metstasis asintomticas) y en algunos LDH superior 2/3 del lmite superior plasmtico.
casos de adenocarcinoma y carcinoma de clulas grandes. Si no se cumple ningn criterio, se considera trasudado. La causa
Gammagrafa sea: est indicada cuando hay un aumento de la ms frecuente es la insuficiencia cardaca congestiva (resp. 1). Otras
fosfatasa alcalina, hipercalcemia o sntomas articulares. causas son el tromboembolismo pulmonar y la cirrosis heptica.

M exico A rgentina CTO Medicina C/ Nez de Balboa, 115 28006 MADRID (Espaa) Tfno.: (91) 782 43 32 / Fax: (91) 782 43 27
Pg. 12 NM C hile U ruguay E-mail: secretaria@ctomedicina.com; iberocto@ctomedicina.com WEB: www.ctomedicina.com; www.iberocto.com
NEUMOLOGA Y CIRUGA TORCICA
Preparacin Examen de Seleccin 05/06 1 Vuelta Seguimiento a distancia
Si se cumple alguno de los criterios, hablamos de exudados. Son Pregunta 58.- R: 5
causa de exudado: La afectacin pleural es posiblemente la afectacin pleuropulmo-
Derrame pleural paraneumnico: es la causa ms frecuente de nar ms frecuente de la artritis reumatoide. Su prevalencia se ha esti-
derrame tipo exudado (resp. 2). La etiologa ms frecuente son los mado en menos del 5%. El derrame pleural puede aparecer antes de
anaerobios, neumococo y gram negativos. El tratamiento es el de la las manifestaciones articulares o coincidir con las fases de artritis acti-
infeccin de base. va. Suele aparecer en pacientes varones mayores de 40 aos y con
Tuberculosis pleural: suele ser una primoinfeccin. Los pacientes ndulos subcutneos. Puede cursar asintomtico y descubrirse en
afectados suelen ser jvenes y con Mantoux negativo (resp. 4 inco- una Rx de trax, o debutar con dolor pleurtico acompaado o no de
rrecta). disnea. Puede acompaarse de enfermedad parenquimatosa
Derrame neoplsico: la causa ms frecuente son los tumores pul- pulmonar y con menos frecuencia de pericarditis. Suele ser de escasa
monares. Los tumores pleurales malignos son poco frecuentes y o moderada cuanta, generalmente unilateral y de predominio dere-
estn en relacin con el asbesto (resp. 3). cho. Suele persistir durante semanas o meses y, finalmente, resolverse
Derrame asociado a enfermedades gastrointestinales: pancreatitis, de forma espontnea. En algunos casos persiste durante aos, evolu-
rotura esofgica, enfermedad de Whipple. cionando a pseudoquilotrax. Es un lquido con caractersticas de
Otras: colagenosis, neumotrax, quilotrax, mixedema, embolis- exudado, con niveles altos de protenas y LDH, y tpicamente presen-
mo pulmonar, sndrome de Meigs, etc. ta cifras bajas de glucosa (<30mg/dl), pH (por debajo de 7,20, la
respuesta 5 es incorrecta) y complemento. El recuento celular suele
mostrar predominio linfoctico. Un dato interesante es la presencia de
cristales de colesterol cuando es de larga evolucin. Otro hallazgo son
ttulos altos de FR, aunque hay que tener en cuenta la relativa inespe-
cificidad de este dato, ya que ttulos altos de FR pueden encontrarse
en derrames neoplsicos, tuberculosos o lpicos.
El tratamiento no est estandarizado; el uso de frmacos inducto-
res de remisin de la enfermedad de base tambin induce a la resolu-
cin de la afectacin pleural. Si evolucionan a fibrotrax extenso, es
necesaria la ciruga.

Pregunta 58. Diferencias entre exudado tuberculoso y reumatoide.

123245637859428
4 68 123245637859428
963649268
12343567 3
 89
 8 3

  343
3 3
  8 3
23

Pregunta 59.- R: 4
Las manifestaciones pleuropulmonares en el lupus eritematoso
sistmico (LES) aparecen en alrededor del 50% de los pacientes en
algn momento de su evolucin. La afectacin pleural es el cuadro
ms frecuente (45% de los pacientes). Aunque es habitual que apa-
rezca en fases iniciales de la enfermedad, slo en aproximadamente
Pregunta 56. Causas de trasudado pleural. el 2% de los casos es la forma de presentacin. Suele manifestarse
como dolor pleurtico acompaado de roce pleural a la ausculta-
Comentarios TEST

Pregunta 57.- R: 5 cin. Los brotes de pleuritis suelen durar pocos das o semanas,
Desde el punto de vista radiolgico puede manifestarse como de- remitiendo espontneamente o con tratamiento y tendiendo a la
rrame pleural libre o encapsulado. El derrame pleural encapsulado se recidiva. Suele ser bilateral, y cuando es unilateral, es de localiza-
produce por acmulo de lquido en una zona de la pleura con am- cin izquierda. Es un exudado rico en protenas, con pH alto (>7,35;
plias adherencias entre ambas hojas que impiden su despegamiento. la respuesta 4 es incorrecta), glucosa similar a la srica (>60mg/dl) y
Puede simular una condensacin del parnquima o una tumoracin. recuento celular con moderada leucocitosis de predominio mono-
El derrame pleural libre es el que ocupa una cavidad pleural sin nuclear. Puede ser de carcter neutroflico si el derrame es reciente.
adherencias o tabicaciones y se sita, por tanto, en la zona declive, Podemos encontrar adems ttulos altos de ANA, bajos de comple-
movilizndose con los cambios de posicin. Puede presentarse como mento y clulas con fenmeno LE. El tratamiento es sencillo, con
derrame pleural tpico. Aqu la distribucin del lquido con el paciente buena respuesta y con escasa formacin de paquipleuritis. Se suele
en bipedestacin se acumula entre la superficie inferior del pulmn y el comenzar con AINEs, y si no hay respuesta, se recurre a los
diafragma, sobre todo en las zonas posteriores. Lo primero que se ocu- corticoesteroides. No es rara la asociacin de pleuritis y pericarditis
pa es el seno costofrnico, por ello se borra el seno en la Rx de trax. Es en mujeres jvenes, lo que sugiere en este grupo de enfermos la
el signo ms frecuente (resp. 5 correcta). A medida que aumenta la etiologa lpica de la serositis.
cantidad de lquido, el lmite superior adopta la forma de una curva de
concavidad superior y vrtice externo. Esto se denomina lnea de Pregunta 60.- R: 3
damoisseau, y es el signo ms caracterstico. Otra forma de presenta- Se ha comprobado que aproximadamente el 15% de los trasuda-
cin es el derrame pleural subpulmonar. El acmulo de lquido simula dos y el 40% de los exudados tienen aspecto serosanguinolento. Cuan-
la elevacin del hemidiafragma. En el lado izquierdo se sospecha cuan- do esto ocurre, es importante la realizacin de un hematocrito para
do la distancia entre el borde inferior del pulmn y la cmara gstrica es diferenciar entre las distintas etiologas:
superior a 2cms. Por ltimo, puede presentarse como un derrame atpi- < 1% del hematocrito perifrico: suele estar en relacin con la
co (en las cisuras), o un derrame masivo (opacificacin completa de un realizacin traumtica de la tcnica de toracocentesis. Si esto ocu-
hemitrax). rre, la coloracin no es uniforme a lo largo de la extraccin y los
macrfagos no contienen inclusiones de hemoglobina.

M exico A rgentina CTO Medicina C/ Nez de Balboa, 115 28006 MADRID (Espaa) Tfno.: (91) 782 43 32 / Fax: (91) 782 43 27
C hile U ruguay E-mail: secretaria@ctomedicina.com; iberocto@ctomedicina.com WEB: www.ctomedicina.com; www.iberocto.com
NM Pg. 13
NEUMOLOGA Y CIRUGA TORCICA
Seguimiento a distancia Preparacin Examen de Seleccin 05/06 1 Vuelta
> 50% del hematocrito perifrico: es una entidad especfica de- en el recuento celular aparece, sin embargo, en todas las etiologas
nominada hemotrax. Las causas ms frecuentes son las traumti- descritas en las cuatro primeras respuestas y es poco probable en el
cas. El tratamiento es la colocacin de un tubo de drenaje para empiema, donde lo habitual es el predominio de neutrfilos. Ade-
cuantificar el sangrado. ms, el empiema se define como presencia de pus en el lquido pleural.
1 - 50% del hematocrito perifrico: las causas ms frecuentes en Para ello se necesita que se complique un derrame pleural
este grupo son los derrames pleurales de origen neoplsico, secun- paraneumnico, y esto va definido en funcin del pH. Si est entre 7
darios a un tromboembolismo pulmonar y los traumatismos y 7,2, hay alto riesgo; si es < 7, se define como empiema. Con un pH
torcicos. En esta pregunta no estn las neoplasias, por lo que la de 7,4, no es posible la existencia de un empiema.
respuesta correcta ser el embolismo pulmonar (respuesta 3). Re- Ver tabla en la pgina siguiente.
cuerda que el tromboembolismo pulmonar puede producir de-
rrames pleurales exudados o trasudados, siendo ms frecuentes los Pregunta 62.- R: 1
exudados asociados a pequeos infartos pulmonares. Es un tumor que deriva de las clulas mesoteliales que tapizan la
cavidad pleural. Es un tumor raro (la resp. 1 es incorrecta, son las
Pregunta 61.- R: 5 metstasis) que tiene una clara relacin con la exposicin a asbesto,
Estamos ante un paciente con un derrame pleural con criterios de aunque tambin se ha asociado a otros factores concomitantes como
exudado, ya que el cociente protenas LP/SP es mayor de 0,5. En las radiaciones. El tiempo desde la primera exposicin al asbesto y la
funcin del pH podemos determinar que, al ser superior a 7,2, es presencia del tumor es variable, y puede oscilar entre los 20 a 40
poco probable que ocurra en derrames complicados. Es probable aos. Dos tercios de los casos tienen edad entre 40 y 70 aos. La
que sea un lupus eritematoso y poco probable que sea un cncer, clnica es muy insidiosa, con dolor torcico de caractersticas pleur-
una tuberculosis pleural, un empiema o una artritis reumatoide, don- ticas y disnea. Posteriormente presentan prdida de peso, febrcula y
de habitualmente el pH es menor de 7,20. Por esto la respuesta 3 tos seca. Puede asociar hipoglucemia por consumo excesivo de glu-
queda excluida. Lo mismo ocurre con las cifras de glucosa; son nor- cosa por parte del tumor y osteopata hipertrfica. El hallazgo radiol-
males en el lupus y estn bajas en el resto. El predominio de linfocitos gico ms frecuente es el derrame pleural, y se acompaa en 1% de

Pregunta 61. Caractersticas diferenciales de los derrames pleurales.

Proceso Bioqumica Otras caractersticas Tratamiento


Exudado.
Bact = Pm n.
Etiologa + frecuente: bacteriana
Neumona Virus: m ononucleares. El de la neum ona
(neum ococo, BGN)
Cuanto m s inflam acin LDH y glucosa.
pH<7,2 riesgo evolucin em piem a.
Hem tico, seroso o serohem tico. Es la causa m s frecuente de
Exudado. derram e sanguinolento. Paliativo/sintom tico=
Neoplasia
Hto >1% y <50%. Ca pulm n > m am a >linfom a. evacuacin del derram e
Citologa lq. pleural. Sintom tica.
El del TEP.
Exudado (75%).
Peligro de transform ar el derram e
T EP Trasudado (25%) si existe fracaso hem odinm ico. 25-50% de los TEP
en hem otrax por el tratam iento
Hem tico, Hto >1% y <50%.
anticoagulante.
Exudado. < 35a . Tto TBC.
linfocitos. Prim oinfeccin tuberculosa: Si el paciente:
ADA. m a n to u x - . - < 35 a .
T BC

Comentarios TEST
IFN (lo m s sensible y especfico). Con frecuencia unilateral. - No factores de riesgo de neoplasia.
Glucosa <60 m g/dl. Diagnstico con biopsia pleural - ADA >70.
Ausencia de clulas m esoteliales. ciega. Iniciar tto antituberculoso em prico.
Exudado seroso.
Clulas lupus. Bilat 50%.
Glucosa norm al. Unilat 50%.
L ES Tto del LES (corticoides)
pH >7,2 (norm al). Con m ayor frecuencia el lado
C' . izquierdo.
ANA lquido pleural.
Exudado.
5% casos.
pH <7,2.
U n il a t e r a l .
Glucosa <30 m g/dl (si <15m g/dl
AR M s frecuente en lado derecho. Tratam iento AR
diagnstico confirm acin).
Varn con ndulos subcutneos y
C' .
ttulos altos de FR.
FR en lquido pleural.
Clulas epiteliales, partculas de Tto etiolgico
Exudado.
Rotura esofgica a l im e n t o . y
am ilasa pancretica.
pH cido. Tto de la m ediastinitis
Localizacin.
Exudado.
Pancreatitis Lado izquierdo. Tto etiolgico
am ilasa pancretica.
15-20% de las pancreatitis.

M exico A rgentina CTO Medicina C/ Nez de Balboa, 115 28006 MADRID (Espaa) Tfno.: (91) 782 43 32 / Fax: (91) 782 43 27
Pg. 14 NM C hile U ruguay E-mail: secretaria@ctomedicina.com; iberocto@ctomedicina.com WEB: www.ctomedicina.com; www.iberocto.com
NEUMOLOGA Y CIRUGA TORCICA
Preparacin Examen de Seleccin 05/06 1 Vuelta Seguimiento a distancia
casos de placas pleurales. En el TAC se observa una pleura engrosada Hernia de Bochdalek o lumbocostal: es casi siempre una hernia
e irregular. En el anlisis del lquido pleural encontramos un exudado sin saco herniario. Se produce a travs de un defecto congnito
seroso o serosanguinolento con cifras de pH y glucosa disminuidas. A situado en la zona posterolateral del trax, sobre todo en el lado
mayor disminucin, peor pronstico. Adems suele ser un lquido izquierdo. La forma neonatal es la ms frecuente. Hay un paso del
rico en cido hialurnico. El diagnstico suele requerir toracoscopia contenido abdominal a la caja torcica y al nacimiento el RN tiene
e incluso toracotoma. La biopsia pleural cerrada no puede distinguir un pulmn total o parcialmente colapsado. Segn la gravedad de
entre mesotelioma y metstasis de adenocarcinoma. No es infrecuen- insuficiencia respiratoria siempre tienen indicacin quirrgica, y
te que el diagnstico se haga en la autopsia. El tratamiento de los en el momento actual se realiza incluso tratamiento intratero.
mesoteliomas localizados es la ciruga. En los extensos se puede pro- Hernia de Morgagni: es poco frecuente. La puerta herniaria se
poner la quimioterapia y radioterapia. De todas formas, la superviven- encuentra en la hendidura situada entre la pared esternal y la parte
cia no es muy prolongada (supervivencia media de 7 meses tras el costal de la musculatura diafragmtica. Se localizan en la zona
diagnstico), ya que cualquier forma de tratamiento es paliativo. anterior del trax, fundamentalmente en el lado derecho Ocurren
con mayor frecuencia en adultos obesos (la respuesta 3 es inco-
Pregunta 63.- R: 2 rrecta). Tienen saco herniario y pueden contener grasa y/o vsceras
El neumotrax se define por la presencia de aire en el espacio pleural, abdominales. Son quirrgicas cuando producen sntomas, se han
con la consiguiente prdida de la presin subatmosfrica pleural y estrangulado o hay riesgo de estrangulacin.
colapso pulmonar parcial o total. Habitualmente el neumotrax se
clasifica segn su etiologa. Se distinguen los neumotrax traumticos, Pregunta 66.- R: 3
iatrognicos y espontneos. A su vez, los neumotrax espontneos pue- Las masas mediastnicas se clasifican en funcin de su localizacin
den ser primarios (en ausencia de enfermedad pulmonar previa) o en los distintos compartimientos mediastnicos. Segn donde se en-
secundarios (en el curso de otra enfermedad pulmonar); siempre sin cuentren es ms orientativo de una u otra etiologa.
traumatismo previo (la respuesta 2, por tanto, es incorrecta). Mediastino anterior: aqu podemos encontrar una masa tmica, la
El neumotrax espontneo primario se produce preferentemente ms frecuente del compartimento. Asocia con frecuencia sndro-
en individuos jvenes, talla alta y delgados. No se ha demostrado su mes generales, como el sndrome de Cushing, la miastenia gravis o
relacin con el consumo de tabaco. Alguna teora establece como la aplasia de la serie roja. Otras lesiones son las tiroideas, como el
causa la sobredistensin de los alveolos apicales del pulmn, espe- bocio endotorcico, y los tumores germinales, como el seminoma
cialmente en los pulmones muy alargados, que acaban por romperse o el coriocarcinoma. El teratoma es un tumor de origen dermoide
dando lugar a zonas localizadas de enfisema paraseptal (blebs que afecta con mayor frecuencia a adultos jvenes. Raramente son
subpleurales). La rotura de estos blebs son la causa inmediata del sintomticos y generalmente su hallazgo es casual en un Rx de
neumotrax. trax, donde se presentan como masas bien definidas con calcifi-
El neumotrax traumtico es el que aparece en el contexto de un caciones en su interior. Este es el tumor al que hace referencia la
traumatismo torcico, y el iatrgeno el debido a maniobras diagnsti- pregunta, por lo tanto la respuesta correcta es la 3.
cas o teraputicas. El resto de los tumores de las otras cuatro respuestas se localizan en
mediastino posterior, a excepcin del linfoma, que puede apare-
Pregunta 64.- R: 2 cer en cualquier compartimento mediastnico.
La parlisis diafragmtica es una patologa que cursa con un aspec- Mediastino medio: los tumores ms frecuentes en esta localiza-
to clnico muy amplio. Afecta al principal msculo implicado en la cin son los quistes del desarrollo. Otras masas son derivadas de la
inspiracin, y segn su afectacin, el enfermo puede estar desde com- afectacin ganglionar (metstasis, tuberculosis, linfoma).
pletamente asintomtico hasta en insuficiencia respiratoria franca. Hay Mediastino posterior: el grupo principal de esta regin lo constitu-
dos tipos segn la localizacin de la lesin. yen los tumores neurognicos como el neurofibroma, ganglioneu-
En la parlisis diafragmtica unilateral se afecta slo el nervio frni- roma y paraganglioma. Con menos frecuencia aparecen lesiones
co de un lado. Se sospecha al encontrar un hemidiafragma elevado esofgicas, pseudoquistes pancreticos o hernia de Bochdalek.
en la Rx de trax, ya que habitualmente el enfermo est asintomtico
o paucisintomtico (la respuesta 2, por tanto, es incorrecta). La causa Pregunta 67.- R: 5
ms frecuente es la infiltracin tumoral del nervio frnico (general- El tratamiento del neumotrax viene definido fundamentalmente
mente por extensin local de un carcinoma pulmonar). Otras causas en funcin de su cuantificacin y la presencia de compromiso respi-
Comentarios TEST

son el traumatismo quirrgico y la idioptica. Su diagnstico se confir- ratorio. En pacientes con un neumotrax < 20-30% y sin compromi-
ma por radioscopia dinmica o fluoroscopia. La parlisis diafragmtica so respiratorio, la actitud teraputica es la analgesia para controlar el
bilateral puede ser secundaria a enfermedades neurodegenerativas y intenso dolor y la observacin. En pacientes con compromiso respira-
a traumatismos cervicales o dorsales altos con compromiso medular. torio y en pacientes con neumotrax >20-30%, la actitud teraputica
Estos enfermos desarrollan un problema ventilatorio restrictivo de ori- indicada es la colocacin de un tubo de drenaje endotorcico. Si no
gen extratorcico, comportndose como sndromes de hipoventila- hay reexpansin o a pesar de la reexpansin persiste la fuga area ms
cin graves. La mayora de los enfermos refiere intensa disnea que de 7 das, es necesario recurrir al tratamiento quirrgico, al igual que
empeora con el decbito. El tratamiento de eleccin es la ventilacin en las siguientes situaciones:
mecnica, y si el nervio frnico est intacto se puede emplear marca- Fuga persistente ms de 7-10 das tras la colocacin de tubo de
pasos diafragmtico. drenaje endotorcico.
Segundo episodio de neumotrax.
Pregunta 65.- R: 3 Neumotrax bilateral simultneo.
Con el nombre de hernia diafragmtica se entiende la profusin de Neumotrax contralateral a uno previo.
vsceras abdominales en la cavidad torcia a travs de una abertura Complicaciones del neumotrax como hidroneumotrax, fstula
normal o patolgica del diafragma. Pueden ser de etiologa traumti- broncopleural, empiema, neumotrax a tensin, fracaso respira-
ca o atraumtica. torio o edema de reexpansin.
Las hernias traumticas aparecen en el contexto de traumatismos Enfermedad de base susceptible de tratamiento quirrgico como
con lesin diafragmtica y no son verdaderas hernias, ya que carecen quiste hidatdico, absceso neoplsico o bullas de gran tamao.
de saco herniario. La mayora tienen indicacin quirrgica. Situacin socioprofesional peculiar (pilotos, buceadores).
Las hernias atraumticas son tres: La pleurodesis qumica, generalmente realizada con talco o tetra-
Hernia de hiato esofgico: una parte del estmago pasa a travs ciclinas, se utiliza en enfermos con pulmn ya expandido y en mal
del hiato esofgico del diafragma al trax. La pirosis y el dolor estado, que por algn otro motivo no pueden tolerar la actitud
torcico a nivel del apndice xifoides son los sntomas ms habi- quirrgica.
tuales. Casi siempre responden a tratamiento mdico. Todas las respuestas son indicacin de ciruga, a excepcin del
neumotrax iatrgeno (respuesta 5). Cuando se produce un neumo-

M exico A rgentina CTO Medicina C/ Nez de Balboa, 115 28006 MADRID (Espaa) Tfno.: (91) 782 43 32 / Fax: (91) 782 43 27
C hile U ruguay E-mail: secretaria@ctomedicina.com; iberocto@ctomedicina.com WEB: www.ctomedicina.com; www.iberocto.com
NM Pg. 15
NEUMOLOGA Y CIRUGA TORCICA
Seguimiento a distancia Preparacin Examen de Seleccin 05/06 1 Vuelta
trax en el contexto de la realizacin de una tcnica diagnstica o estar en relacin con la ciruga y se hubieran visto los implantes pleura-
teraputica hay que valorar igualmente la repercusin respiratoria y les durante la ciruga (respuesta 2 incorrecta). Tampoco es probable el
su cuantificacin para decidir si es susceptible de ciruga. pseudoquilotrax porque stos son derrames pleurales de larga evolu-
cin, secundarios fundamentalmente a artritis reumatoide y tuberculo-
Pregunta 68.- R: 2 sis pleural (respuesta 4 incorrecta). Lo ms probable es, como decamos
En esta pregunta se plantea la actitud ante un derrame pleural al principio, el quilotrax, que se caracteriza por ser un lquido lechoso
neoplsico. La causa ms frecuente de derrame pleural maligno con cifras altas de triglicridos y grasas totales.
son las metstasis de tumores extrapleurales. El cncer de pulmn,
de mama y los linfomas suponen el 75% de los casos, y hay un 6%
de casos en los que se desconoce el tumor primitivo. Desde un
punto de vista histolgico las metstasis ms frecuentes son las del
adenocarcinoma de cualquier origen. Es el segundo tipo de exuda-
do ms frecuente (tras los derrames paraneumnicos). La forma de
presentacin ms tpica es un derrame pleural de cuanta impor-
tante con dolor torcico y disnea. El diagnstico se realiza por
citologa (entre el 50-80% de los casos), biopsia pleural ciega o
toracoscopia. El derrame pleural maligno indica enfermedad sist-
mica y se debe valorar la quimioterapia si el tumor primario es
sensible. Si produce sintomatologa respiratoria debe ser evacuado.
Inicialmente se realiza una toracocentesis evacuadora, y si recidiva,
hay que plantearse la pleurodesis qumica tras colocacin de dre-
naje, endotorcico (respuestas 1 y 3 incorrectas). Los tumores no
quimiosensibles slo son subsidiarios de tratamiento paliativo (res-
puesta 5 incorrecta). En los derrames pleurales malignos secunda-
rios a tumor pulmonar, antes de colocar un tubo de drenaje endo-
torcico hay que ver si hay lesin endobronquial, ya que si la hay,
es imposible la reexpansin pulmonar y la resolucin del derrame.
En este caso es un tumor de mama diseminado; es posible tambin
que exista afectacin pulmonar endobronquial por metstasis, por
tanto habr que descartarla antes de colocar el tubo de drenaje endo-
torcico, como dice la respuesta 2.

Pregunta 69.- R: 4
Estamos ante un paciente con factores de riesgo para desarrollar
cncer de pulmn (es >35 aos y fumador) que presenta un sndro-
me constitucional en el contexto de un derrame pleural. Pregunta 70. Conducta a seguir ante un derrame pleural.
Las caractersticas bioqumicas del lquido pleural confirman que
es un exudado porque cumple uno de los criterios de Light (protenas El tratamiento es la nutricin rica en triglicridos de cadena media
LP/SP >0,5). Esto descarta la respuesta 2, que habla de un derrame y pobre en el resto de grasas. Si es secundario a traumatismo, se colo-
pleural tipo trasudado. car un tubo de drenaje endotorcico, y slo si no mejora, se proce-
El aspecto serohemtico nos obliga a hacer el diagnstico diferen- der a la ciruga (respuesta 5 incorrecta).
ciado entre derrame de origen neoplsico, tromboemblico y trau-
mtico fundamentalmente. Lo ms frecuente es el derrame pleural Pregunta 71.- R: 5
maligno, por lo que la respuesta 3 tambin es incorrecta. La hipertensin pulmonar primaria (HPP) es una rara enfermedad
El hallazgo en la biopsia pleural cerrada de ausencia de granulo- que predomina en mujeres entre la 2-4 dcada. Se asume el diag-
mas indica la poca probabilidad de que sea una tuberculosis, pero en nstico cuando el paciente tiene una presin media en la arteria
ningn caso la citologa negativa descarta la existencia de proceso pulmonar mayor de 25mmHg en reposo o mayor de 30mmHg en

Comentarios TEST
tumoral. La respuesta 1 es por tanto incorrecta, ya que no se puede ejercicio, con una presin de enclavamiento normal.
asumir que es infeccioso, tratarlo con antibioterapia y no seguir ade- Para diagnosticar una hipertensin pulmonar como primaria hay
lante en el algoritmo diagnstico. La respuesta 5 tambin es incorrecta que haber excluido otras causas ms frecuentes de hipertensin pul-
porque no concurre ninguna indicacin de tubo de drenaje. No hay monar secundarias como la EPOC, el TEP de repeticin, la infeccin
criterios de empiema (glucosa <50) ni repercusin respiratoria sufi- VIH, la inhalacin de cocana, los frmacos estimuladores del apetito,
ciente para indicar drenaje inmediato. la hipertensin portal.
Estamos, por tanto, ante un derrame pleural exudado donde an Hay varios tipos histolgicos:
no se ha descartado el proceso tumoral. Hay que realizar la siguiente Arteriopata pulmonar plexognica: es la forma ms frecuente.
prueba diagnstica que, como dice la respuesta correcta (la 4), es la Hay hipertrofia de la media, lesiones en la ntima y lesiones plexi-
biopsia guiada por toracoscopia. formes que consisten en destruccin segmentaria de la media con
plexos de microvasos proliferantes. Las lesiones estn en arteriolas
Pregunta 70.- R: 3 y arterias de pequeo calibre.
La presencia de derrame pleural 1 semana despus de que el pa- Arteriopata pulmonar trombtica: los hallazgos son similares a la
ciente sea sometido a una ciruga de cavidad torcica debe hacer anterior con signos de trombos recanalizados.
pensar en rotura iatrgena del conducto torcico, sobre todo si se Enfermedad venoclusiva: representa aproximadamente el 10%
tiene en cuenta la proximidad anatmica del campo quirrgico y de los casos de hipertensin pulmonar primaria. Hay lesin de las
dicha estructura. arteriolas y fundamentalmente de las vnulas donde se produce
Adems el tiempo de evolucin hasta que aparece el derrame es una arterializacin de las venas y vnulas. Predomina en la infan-
amplio (1 semana), lo que hace ms probable que sea secundario al cia, y si aparece en adultos, lo hace con ms frecuencia en varones
acmulo de linfa en el espacio pleural tras la rotura del conducto (respuesta 5 falsa).
torcico, y no a una lesin vascular donde el tiempo de presentacin Hemangiomatosis capilar: es una enfermedad poco frecuente don-
del hemotrax sera menor (respuesta 1 incorrecta). de proliferan microvasos de paredes finas que infiltran el intersticio
Igualmente es poco probable que sea un derrame pleural maligno, y el parnquima.
ya que debera tener un mayor tiempo de evolucin, no tiene por qu

M exico A rgentina CTO Medicina C/ Nez de Balboa, 115 28006 MADRID (Espaa) Tfno.: (91) 782 43 32 / Fax: (91) 782 43 27
Pg. 16 NM C hile U ruguay E-mail: secretaria@ctomedicina.com; iberocto@ctomedicina.com WEB: www.ctomedicina.com; www.iberocto.com
NEUMOLOGA Y CIRUGA TORCICA
Preparacin Examen de Seleccin 05/06 1 Vuelta Seguimiento a distancia
Pregunta 72.- R: 3 mente asociado a una condensacin parenquimatosa triangular peri-
La hipertensin pulmonar primaria es una enfermedad progresiva frica en la zona inferior. Es lo que se denomina "joroba de Hampton"
para la que no hay curacin. La supervivencia media sin tratamiento y aparece cuando hay infarto pulmonar asociado. La resp. 4 es inco-
es inferior a 2,5 aos. rrecta porque, aunque ste es un dato caracterstico, la forma de
presentacin radiolgica ms frecuente es la Rx de trax normal.

Pregunta 74.- R: 3
El diagnstico de TEP es complicado. Se basa en la combinacin
de una anamnesis metdica, una correcta exploracin fsica y una
serie de tcnicas invasivas y no invasivas.
La sospecha clnica es muy importante. Dada la gravedad del pro-
ceso, la presencia de clnica compatible, junto con la presencia de
factores de riesgo, es suficiente para iniciar tratamiento anticoagulante
(respuesta 1 correcta).
Las pruebas complementarias habituales no muestran generalmente
alteraciones. En ocasiones en la Rx de trax podemos encontrar ele-
vacin de un hemidiafragma, oligohemia perifrica , atelectasias
laminares o derrame pleural (respuesta 2 correcta). En el ECG las
alteraciones ms comunes son la taquicardia sinusal y las anomalas
inespecficas del ST. En la GAB suele encontrarse hipoxemia con
hipocapnia.
La gammagrafa de perfusin es el test de screening ms til para
descartar un TEP clnicamente importante, pero en ocasiones es ne-
cesario realizar una angiografa pulmonar. Es la tcnica gold standard,
y puede precisarse cuando la sospecha clnica es muy alta y la gam-
magrafa y el resto de tcnicas diagnsticas no son concluyentes (res-
puesta 5 correcta).
Cuando tenemos sospecha clnica de TEP, si se demuestra la exis-
Pregunta 72. Tratamiento de la hipertensin pulmonar primaria. tencia de TVP se puede asumir el diagnstico de TVP + TEP (enferme-
dad tromboemblica). En el estudio de la TVP se utiliza fundamental-
Hay 3 medidas fundamentales en el tratamiento: mente la ultrasonografa venosa, ya que detecta enfermedad tanto en
Vasodilatadores: en el momento del diagnstico de la hiperten- el sistema venoso profundo proximal como distal (la respuesta 4 es
sin pulmonar primaria se plantea la realizacin de un test de correcta).
vasorreactividad para predecir la respuesta del paciente al trata- Otras tcnicas diagnsticas son la pletismografa de impedancia,
miento con vasodilatadores va oral (antagonistas del calcio). Se que detecta trombos en el sistema venoso proximal (popltea o supe-
realiza con prostaciclina i.v. y se considera que la respuesta es rior) y la flebografa isotpica con fibringeno marcado, que detecta
positiva si la presin media de la arteria pulmonar disminuye un trombos en el sistema venoso distal (pantorrilla). Por tanto, la respuesta
20%. Si esto ocurre, se administrarn vasodilatadores va oral. Estos incorrecta es la 3.
producen una mejora sostenida en un tercio de los pacientes (la
respuesta 3 es incorrecta). En los pacientes en los que no hay res- Pregunta 75.- R: 4
puesta utilizaremos vasodilatadores de accin corta y potente como Una vez diagnosticado el paciente de TEP, el tratamiento de elec-
la prostaciclina i.v.; siempre y cuando tengan un grado de ICC III - cin, si no hay contraindicaciones, es la anticoagulacin. En el mo-
IV de la NYHA. mento inicial, el tratamiento debe realizarse con heparina. Este frma-
Anticoagulantes orales: el tratamiento con estos frmacos aumenta co acelera la accin de la antitrombina III, por lo que adems de
la supervivencia al prevenir la trombosis in situ. Estn indicados en prevenir la formacin de nuevos trombos, permite que la fibrinlisis
todos los pacientes con hipertensin pulmonar primaria. endgena disuelva parcialmente el trombo ya formado.
La heparina sdica intravenosa se utiliza la mayora de las veces
Comentarios TEST

Transplante pulmonar. Est indicado en pacientes con enferme-


dad muy avanzada con insuficiencia cardaca derecha a pesar del como anticoagulante de eleccin (respuesta 1 correcta). Requiere
monitorizacin de los tiempos de coagulacin para controlar su ac-
tratamiento vasodilatador. No se ha descrito recidiva de la enfer-
cin. El efecto deseado se consigue cuando el tiempo parcial de trom-
medad en los rganos trasplantados.
boplastina activada (TTPa) se establece alrededor del doble del tiem-
po control (respuesta 2 correcta).
Pregunta 73.- R: 4 Las heparinas de bajo peso molecular han demostrado ser tan
El sntoma ms frecuente cuando hay un tromboembolismo pul- eficaces y seguras como la heparina intravenosa, por lo que en el
monar (TEP) es la disnea, y el signo ms frecuente la taquipnea. momento actual se utilizan de forma habitual en el tratamiento inicial
Sin embargo hay que realizar una correcta anamnesis y explora- del TEP (respuesta 3 correcta). Se administran por va subcutnea y no
cin fsica para identificar qu sntomas y signos acompaan con requieren monitorizacin del TTPa, ya que su respuesta es predecible
mayor frecuencia a las distintas formas de presentacin del TEP. en funcin de la dosis.
El tromboembolismo de localizacin central en arterias pulmonares Son frmacos bastante seguros, aunque pueden asociar efectos
proximales se caracteriza por producir importantes alteraciones en el secundarios como la osteopenia, plaquetopenia o hemorragia. La
intercambio de gases. Tiene poco riesgo de infarto parenquimatoso, al hemorragia es una complicacin poco frecuente que suele aparecer
ser posible la vascularizacin del parnquima por circulacin colateral. si hay otros factores sobreaadidos como trombopenia o alteraciones
El TEP perifrico, en arterias segmentarias y subsegmentarias, pro- de la coagulacin (respuesta 4 incorrecta).
duce pocas alteraciones del intercambio gaseoso, pero tiene un ma- Se ha puesto de manifiesto que las complicaciones hemorrgicas
yor riesgo de asociar infarto pulmonar. Cuando esto ocurre, aparece aparecen con menos frecuencia en las heparinas de bajo peso mole-
generalmente dolor pleurtico, hemoptisis y febrcula. cular en la heparina sdica intravenosa.
Cuando hay un TEP muy importante pueden existir alteraciones Ver esquema en la pgina siguiente.
hemodinmicas asociadas. A la disnea sbita se suele aadir hipoten-
sin, sncope o cianosis. Pregunta 76.- R: 3
Las pruebas complementarias habitualmente son normales, aun- Estamos ante un paciente que, tras ser sometido a una ciruga de larga
que podemos encontrar en ocasiones datos que ayuden al diagnsti- duracin, comienza con clnica muy sugestiva de embolismo pulmonar
co de TEP. En la Rx de trax puede existir derrame pleural, general- que se confirma en una gammagrafa de ventilacin perfusin.

M exico A rgentina CTO Medicina C/ Nez de Balboa, 115 28006 MADRID (Espaa) Tfno.: (91) 782 43 32 / Fax: (91) 782 43 27
C hile U ruguay E-mail: secretaria@ctomedicina.com; iberocto@ctomedicina.com WEB: www.ctomedicina.com; www.iberocto.com
NM Pg. 17
NEUMOLOGA Y CIRUGA TORCICA
Seguimiento a distancia Preparacin Examen de Seleccin 05/06 1 Vuelta

Comentarios TEST

Pregunta 75. Algoritmo diagnstico- teraputico del TEP

M exico A rgentina CTO Medicina C/ Nez de Balboa, 115 28006 MADRID (Espaa) Tfno.: (91) 782 43 32 / Fax: (91) 782 43 27
Pg. 18 NM C hile U ruguay E-mail: secretaria@ctomedicina.com; iberocto@ctomedicina.com WEB: www.ctomedicina.com; www.iberocto.com
NEUMOLOGA Y CIRUGA TORCICA
Preparacin Examen de Seleccin 05/06 1 Vuelta Seguimiento a distancia
La presencia de factores de riesgo (ciruga prolongada) ms clnica Pregunta 78.- R: 1
sugestiva (disnea brusca con hipoxemia) y el hallazgo de alta probabili- La inhalacin y depsito de polvos inorgnicos provoca reaccio-
dad de TEP nos permite hacer un diagnstico de certeza en este paciente. nes tisulares que con el paso del tiempo conllevan un dao pulmo-
El estudio de las extremidades inferiores para valorar el origen del embolismo nar importante.
pulmonar es recomendable, pero no es necesario realizar tcnicas La exposicin a asbesto puede producir distintas manifestaciones:
invasivas. Es suficiente con un Eco doppler (resp. 1 incorrecta). Asbestosis: produce una fibrosis pulmonar similar a la fibrosis pul-
El tratamiento anticoagulante descrito en la respuesta 2 es correcto, monar idioptica, es decir, una enfermedad intersticial de predomi-
pero en este caso el paciente tiene una contraindicacin de anticoagu- nio bibasal. Suele aparecer por encima de 10 aos de exposicin y
lacin al estar recin intervenido de una tumoracin en el sistema ner- su curso es variable (respuesta 4 correcta). Las placas pleurales en la
vioso central. El tratamiento anticoagulante que aparece en la respuesta Rx trax slo indican exposicin a asbesto, pero no asbestosis. Se
5 es poco probable porque la duracin habitual de la anticoagulacin localizan preferentemente en pleura diafragmtica, y aunque cuan-
suele ser 6 meses. Aunque en este caso da igual, puesto que la do aparecen en el contesto de una enfermedad intersticial son muy
anticoagulacin est contraindicada. Son contraindicaciones absolu- orientativas de asbestosis, no son, como dice la respuesta 1, que es
tas de anticoagulacin las ditesis y procesos hemorrgicos, la hiperten- incorrecta, especficas de enfermedad por asbesto. Pueden apare-
sin arterial severa, los aneurismas y hemorragias intracraneales y las cer en otras patologas como la tuberculosis pleural.
cirugas recientes del sistema nervioso central y ocular. El embarazo es Cncer de pulmn: aumenta el riesgo a partir de 15 aos de expo-
contraindicacin absoluta slo de anticoagulacin oral. sicin. Dicho riesgo aumenta si el paciente es fumador, puesto que
La terapia tromboltica a la que se hace referencia en la respuesta 4 el asbesto y el tabaco son cocarcingenos. La histologa ms fre-
slo est indicada en el TEP y TVP masivos. cuente es el carcinoma epidermoide y el adenocarcinoma.
La mejor alternativa para este paciente es el filtro de cava (resp. 3). Mesotelioma maligno: aumenta el riesgo por encima de 20-30
aos de exposicin.
Pregunta 77.- R: 5 Derrame pleural benigno y atelectasia redonda: son complica-
Segn la intensidad y frecuencia de la exposicin a polvos inorg- ciones que pueden aparecer en cualquier momento de la evolu-
nicos se describen 3 formas clnicas de alveolitis alrgica o neumonitis cin de la exposicin a asbesto por encima de los 10 aos.
por hipersensibilidad. La reaccin inmunolgica ms relevante es el
dao tisular mediado por inmunocomplejos (reaccin tipo III). Pregunta 79.- R: 3
Forma aguda: se produce ante exposiciones breves e intermiten- Se diferencian cuatro formas de silicosis segn la intensidad de la
tes a dosis bajas de antgeno. Su clnica puede confundirse con exposicin, el periodo de latencia y la historia natural. Cada una de
una neumona infecciosa (respuesta 4 correcta). A las pocas ho- ellas tiene un patrn radiolgico caracterstico, que es a lo que hace
ras de exposicin el paciente presenta fiebre, escalofros, disnea, referencia esta pregunta. La silicosis simple o clsica suele aparecer
tos seca, mialgias y malestar general. En la exploracin fsica hay tras 15 aos o ms de la exposicin. Radiolgicamente presenta lesio-
crepitantes en ambos pulmones, taquipnea y cianosis. Los snto- nes micronodulares menores de 1 cm, de predominio en lbulos
mas suelen cesar tras la exposicin en menos de 24 horas, aun- superiores, como dice la respuesta 1. Pueden existir tambin adeno-
que a veces pueden durar 2-3 das. Radiolgicamente se caracte- patas hiliares calcificadas en "cscara de huevo". Este hallazgo es fre-
riza por infiltrados nodulares mal definidos localizados en lbu- cuente y caracterstico, pero no patognomnico de esta enfermedad
los inferiores (respuestas 1, 2, 3, correctas; respuesta 5 incorrecta). (la respuesta 5 es correcta). La silicosis acelerada es similar a la simple,
Forma subaguda: es similar a la anterior, con un comienzo ms pero aparece con exposiciones ms intensas y tiene un tiempo de
insidioso y una sintomatologa menos florida. La Rx trax es igual latencia menor, entre 5 y 10 aos. Radiolgicamente presenta las
que en la forma aguda. mismas alteraciones que la simple; por tanto, la respuesta 3 es la
Forma crnica: aparece ante exposiciones prolongadas a dosis incorrecta, ya que las lesiones predominan en lbulos superiores y no
bajas de antgenos. Presenta durante meses o aos sntomas como en medios como se dice en ella. La silicosis complicada ocurre como
disnea progresiva, tos seca, prdida de peso. No es infrecuente la evolucin de otras formas de silicosis. La imagen radiolgica tpica es
presencia de COR PULMONALE. La clnica es parecida a la bron- la fibrosis masiva progresiva (conglomerados silicticos mayores de
quitis crnica. En la radiografa de trax se encuentran los hallaz- 1cm localizados en lbulos superiores). Hay compromiso clnico y de
gos definidos en la respuesta 5. Es muy raro que revierta. funcin respiratoria importante, y el riesgo de tuberculosis est au-
mentado (respuesta 2 correcta). Por ltimo, la silicosis aguda es conse-
Comentarios TEST

Pregunta 79. Patrones radiolgicos de afectacin en la silicosis.

M exico A rgentina CTO Medicina C/ Nez de Balboa, 115 28006 MADRID (Espaa) Tfno.: (91) 782 43 32 / Fax: (91) 782 43 27
C hile U ruguay E-mail: secretaria@ctomedicina.com; iberocto@ctomedicina.com WEB: www.ctomedicina.com; www.iberocto.com
NM Pg. 19
NEUMOLOGA Y CIRUGA TORCICA
Seguimiento a distancia Preparacin Examen de Seleccin 05/06 1 Vuelta
cuencia de una exposicin muy intensa. La radiografa de trax mues- diolgicamente hay infiltrados bibasales, como dice la respuesta 3.
tra un patrn miliar o patrn alveolar difuso que predomina en lbu- Son de etiologa desconocida la esosinofilia pulmonar simple (a veces
los inferiores. La respuesta 4 tambin es correcta. es secundaria a parsitos y est indicado el tratamiento con mebenda-
zol, la respuesta 4 es correcta), la neumona eosinfila crnica, que es
Pregunta 80.- R: 3 caracterstica de mujeres de mediana edad con historia de rinitis,
La fibrosis pulmonar idioptica (FPI) es una enfermedad intersti- atopia y asma (como dice la respuesta 2), y el sndrome hipereosin-
cial que aparece generalmente entre los 50 y 70 aos, aunque pue- filo, que como veis en la respuesta 5, afecta fundamentalmente a
de aparecer a cualquier edad. Inicialmente el paciente comienza varones de edad media. Hay una infiltracin difusa de rganos por
con clnica tpica de tos seca y disnea al ejercicio. A medida que la eosinfilos y el aparato respiratorio se afecta slo en la mitad de los
enfermedad progresa la disnea se hace ms intensa. En la explora- casos. La enfermedad cardaca es la principal causa de muerte. Sin
cin fsica lo caracterstico es la aparicin de acropaquias y los cre- tratamiento, la supervivencia es menor de un ao, pero si tratamos
pitantes teleinspiratorios bibasales. En fases avanzadas pueden apa- con corticoides en altas dosis, supera los 10 aos.
recer sntomas de hipertensin pulmonar secundaria y cor pulmo-
nale. En el estudio de funcin pulmonar inicialmente no hay hallaz- Pregunta 83.- R: 4
gos patolgicos, pero a medida que progresa la enfermedad aparece La Aspergilosis broncopulmonar alrgica (ABPA) es una eosinofilia
un patrn restrictivo (disminucin de la CPT, la CV, el VR y el FEV1) pulmonar de origen conocido. Est producida por el hongo Aspergi-
con descenso de la DLCO. A veces este parmetro se ve afectado llus fumigatus como consecuencia de una reaccin inmunolgica
antes que los datos de la espirometra. En la Rx de trax aparece, tipo I y tipo III frente a antgenos del Aspergillus. Al depositarse el hongo
como dice la respuesta 3, un patrn reticular o reticulonodular de en el rbol traqueobronquial, se producen IgG e IgE especficos, as
predominio bibasal. Sin embargo no hay, como tambin dice la como un aumento de la IgE total. Estos anticuerpos se unen a los
respuesta 3, placas pleurales. Estas aparecen fundamentalmente con antgenos del hongo y forman inmunocomplejos circulantes que se
la exposicin a asbesto. Por tanto, la respuesta 3 es la incorrecta. depositan en la pared bronquial dandola. Esto conlleva la apari-
Recuerda que la asbestosis es casi igual a la FPI, salvo por los antece- cin de bronquiectasias de localizacin preferentemente central. A
dentes de exposicin a asbesto, la presencia de placas pleurales y el todo esto se aade la presencia de infiltrados pulmonares por el
hallazgo de fibras de asbesto en material histolgico. La TC de alta acmulo de clulas inflamatorias, principalmente eosinfilos (las res-
resolucin es muy til en el diagnstico precoz, aunque el diagns- puestas 1 a 3 son correctas). Clnicamente se presenta de forma tpica
tico definitivo se hace siempre sobre una muestra histolgica (de como asma de difcil control, disnea, sibilancias, febrcula y tos pro-
biopsia transbronquial en el 25% de los pacientes y de biopsia ductiva con tapones marronceos.
toracotmica en el resto). Sin embargo, para el diagnstico existen unos criterios mayores y
menores, que son los siguientes:
Pregunta 81.- R: 1
Las enfermedades del tejido conectivo son un grupo heterogneo Criterios mayores:
de trastornos inflamatorios de origen multifactorial mediados inmu- Asma.
nolgicamente. Los pulmones se afectan frecuentemente, dada la Eosinofilia sangunea (>1000/mm3) (Resp. 5).
abundancia de tejido conectivo. Las manifestaciones pulmonares Precipitinas IgG contra Aspergillus.
pueden, en ocasiones, preceder o anunciar el comienzo de la expre- Test cutneo de hipersensibilidad inmediata positivo para Aspergillus.
sin sistmica de estas enfermedades. Las enfermedades del colgeno IgE total srica elevada.
que hay que conocer son: Infiltrados pulmonares transitorios.
Artritis reumatoide: puede presentar varias manifestaciones. La Bronquiectasias centrales.
afectacin pleuropulmonar ms frecuente es la pleuritis. La afecta- IgG e IgE especficas elevadas.
cin intersticial aparece fundamentalmente en varones con ttulos
altos de factor reumatoide, y a veces precede a la enfermedad Criterios menores:
articular (respuesta 3 correcta). Otras manifestaciones son los n- Expectoracin marroncea.
Aspergillus en esputo (la respuesta 4 no es correcta, porque dice
dulos reumatoideos o la bronquiolitis obliterante.
Lupus eritematoso sistmico: tambin es la afectacin pleural ms que es un criterio mayor).
Reaccin cutnea retardada.
frecuente y aparece aproximadamente en el 50% de los casos. La
afectacin intersticial, por el contrario, es muy poco frecuente y
Pregunta 84.- R: 3

Comentarios TEST
aparece en lupus de larga evolucin (respuesta 1 incorrecta). En
La sarcoidosis es una enfermedad multisistmica, de etiologa des-
ocasiones puede aparecer en cuadro de hemorragia alveolar secun- conocida, que se caracteriza por la presencia de granulomas no
dario al depsito pulmonar de inmunoglobulinas y complemento.
caseificados en los tejidos afectados. Prcticamente, cualquier rgano
Puede alcanzar tasas de mortalidad de hasta el 80%. puede verse involucrado, aunque son el pulmn y los ganglios linfti-
Esclerodermia: la afectacin pulmonar es frecuente (alrededor cos torcicos los ms frecuentemente afectados (hasta el 90% de los
del 40%) y es la principal causa de mortalidad. Es la colagenosis casos). Es rara la afectacin pleural y el derrame es excepcional (res-
que con mayor frecuencia desarrolla enfermedad intersticial (res- puesta 5 incorrecta). Es una enfermedad de distribucin mundial ms
puesta 5 correcta). frecuente en la raza negra. El 80% de los casos ocurre en personas entre
Espondilitis anquilosante: es la nica que presenta afectacin en 20-40 aos de edad y es algo ms habitual en las mujeres. Por tanto, la
vrtices, el resto la tiene en bases. respuesta que buscamos es la 3. La respuesta cuatro es incorrecta, ya
que es ms frecuente en pacientes no fumadores donde no hay afecta-
Pregunta 82.- R: 1 cin de la inmunidad pulmonar y se pueden perpetuar los procesos
La eosinofilia pulmonar en un sndrome clnico que engloba a inflamatorios que ocurren en esta enfermedad. Se describen dos
diversos procesos que cursan con infiltrados pulmonares con eosin- sndromes clnicos que se enuncian en las respuestas 1 y 2 cuyas mani-
filos y eosinofilia perifrica (a veces no aparece). Los infiltrados pul- festaciones no son las descritas en dichas respuestas. El sndrome de
monares suelen manifestarse radiolgicamente en forma de opacida- Lfgren es un cuadro de fiebre, poliartralgias, eritema nodoso y
des bilaterales de distribucin segmentaria, de aspecto no homog- adenopatas hiliares bilaterales y simtricas. El sndrome de Heerfordt
neo y carcter migratorio; salvo la neumona eosinfila crnica don- presenta fiebre, uvetis anterior, parotiditis y parlisis facial.
de los infiltrados no son migratorios, y si recurren, lo hacen en la
misma localizacin (por tanto, la respuesta 1 es incorrecta). Pregunta 85.- R: 1
Se clasifican en funcin de si se conoce o no su etiologa. Son de Las manifestaciones extrapulmonares a las que hace referencia
etiologa conocida la ABPA, las neumonas por parsitos y las secun- esta pregunta son muy variadas y confieren gran pleomorfismo a la
darias a frmacos. Es caracterstica la afectacin por nitrofurantona enfermedad. Destacan por su frecuencia y relevancia:
que presenta en su forma aguda fiebre, escalofros, tos y disnea. Ra-

M exico A rgentina CTO Medicina C/ Nez de Balboa, 115 28006 MADRID (Espaa) Tfno.: (91) 782 43 32 / Fax: (91) 782 43 27
Pg. 20 NM C hile U ruguay E-mail: secretaria@ctomedicina.com; iberocto@ctomedicina.com WEB: www.ctomedicina.com; www.iberocto.com
NEUMOLOGA Y CIRUGA TORCICA
Preparacin Examen de Seleccin 05/06 1 Vuelta Seguimiento a distancia
Cutneas: la ms frecuente es el eritema nodoso y la ms especfi- Pregunta 87.- R: 4
ca el lupus pernio. La histiocitosis X es una enfermedad de etiologa desconocida in-
Oculares: la afectacin ms frecuente es la uvetis anterior, aunque cluida dentro del grupo de desrdenes proliferativos del sistema
se pueden presentar escleritis o queratitis. mononuclear fagoctico. Es una afeccin poco frecuente, aunque no
Neurolgicas: aparecen con mayor frecuencia en las formas cr- excepcional, que incide fundamentalmente en varones fumadores
nicas. Son mononeuritis (parlisis facial o multineuritis). entre 20 y 40 aos, como dice la respuesta 2. Aunque la lesin predo-
Articulares: generalmente son artritis errticas. minante es la pulmonar, como enfermedad intersticial, puede afectar
Hepticas: hay afectacin histolgica en el 60-90% de los pacien- tambin a huesos e hipfisis (respuesta 1 correcta). Se desconoce el
tes, pero slo hay sntomas como hepatomegalia o alteraciones mecanismo patognico mediante el cual se produce una prolifera-
enzimticas en 1/3 de los pacientes. cin y activacin de los macrfagos alveolares y las clulas de
Cardacas: son poco frecuentes y generalmente asintomticas. Langerhans. Estas clulas son clulas muy diferenciadas que pertene-
Renales: son muy raras. cen a la serie monocito-macrfago y presentan de forma caractersti-
Sistema endocrino metablico: el eje hipotlamo-hipofisario es ca grnulos intracitoplasmticos de Birbeck al estudio con microsco-
lo ms afectado (la respuesta 1 es correcta). Clnicamente se mani- pio electrnico. Estas clulas son caractersticas, pero no patognom-
fiesta como una diabetes inspida. nicas de la enfermedad (resp. 4 incorrecta), ya que tambin las en-
Glndulas exocrinas. Es caracterstica la afectacin bilateral de las contramos aumentadas en los pacientes fumadores sin histiocitosis X.
partidas, que slo da clnica en el 10% de los pacientes (respues- La enfermedad intersticial se inicia en los bronquiolos ms pequeos,
tas 4 y 5 incorrectas). donde se asocia un infiltrado inflamatorio. Es de distribucin bronco-
Aparato reproductor. En el varn, lo ms frecuente es el aumento vascular y se afectan tambin arteriolas y vnulas. Al avanzar el proce-
del tamao testicular. Es rara la esterilidad. En la mujer, es caracte- so aparece la fibrosis intersticial y los quistes areos, dato muy carac-
rstica la mejora con el embarazo (respuestas 2 y 3 incorrectas). terstico de estos enfermos.

Pregunta 86.- R: 2 Pregunta 88.- R: 5


Las manifestaciones radiolgicas difieren de unos casos a otros y se La linfangioleiomiomatosis es una rara enfermedad que se caracteri-
han utilizado para establecer distintas situaciones que se correlacio- za por la proliferacin de clulas musculares lisas alrededor de las es-
nan con la severidad, pronstico y evolucin de la enfermedad. A tructuras broncovasculares y el intersticio, unido a dilatacin qustica
esto hacen referencia las cuatro primeras respuestas. Son correctas la de los espacios areos terminales que en el TAC de alta resolucin se
1, 3 y 4, mientras que es incorrecta la respuesta 2. La clasificacin identifican como quistes de pared fina (respuestas 1 y 2 correctas).
radiolgica es la siguiente: Tambin hay afectacin linftica con dilatacin y tortuosidad de los
Estadio 0: la radiologa es normal. Ocurre en menos de un 5% de vasos linfticos, incluyendo el conducto torcico. Los linfticos pueden
las formas subagudas. verse daados tanto a nivel torcico como extratorcico, por lo que
Estadio I: se presenta fundamentalmente en las formas agudas. podemos visualizar adenopatas en distintas regiones (respuesta 3 co-
Ocurre aproximadamente en un 75% de los pacientes. Es la forma rrecta). Los angiomiolipomas renales son otra manifestacin extrator-
ms benigna por su escasa repercusin funcional y su tendencia a cica que hay que estudiar (respuesta 4 correcta). El sntoma ms fre-
la remisin espontnea. Slo se evidencian adenopatas hiliares y cuente es la disnea. El neumotrax espontneo por ruptura de quistes
mediastnicas bilaterales y simtricas. No se pone de manifiesto aparece en el 50% de los casos y el derrame pleural, que es raro al
afectacin pulmonar. diagnstico de la enfermedad, ocurre en 1/3 de los enfermos a lo largo
Estadio II: se manifiesta por la presencia de adenopatas y patrn de su evolucin. Las caractersticas del derrame pleural son de un
intersticial bilateral difuso. Se presenta tanto en las formas agudas quilotrax, ya que es secundario a la lesin del conducto torcico o los
como crnicas. La tendencia a la regresin con tratamiento con vasos linfticos pleurales. Encontraremos, por tanto, ttulos altos de gra-
corticoides es ms lenta y menos constante. sas totales y triglicridos con cifras normales de colesterol. Esto justifica
Estadio III: generalmente aparece en formas crnicas. Se manifies- que la respuesta 5 sea incorrecta.
ta como un patrn intersticial difuso con tendencia a la evolucin
fibrtica. No suelen existir adenopatas. Pregunta 89.- R: 4
Estadio IV: fibrosis pulmonar difusa (pulmn en panal). La granulomatosis de Wegener es una vasculitis necrosante granu-
lomatosa de pequeo vaso que afecta tanto al tracto respiratorio su-
Comentarios TEST

perior como inferior. En ocasiones tambin afecta a arterias y a venas


(respuesta 1 correcta). La sinusitis es el sntoma de presentacin ms
frecuente. La afectacin pulmonar vara desde infiltrados asintomti-
cos o acompaados de tos, disnea, dolor torcico y de forma caracte-
rstica hemoptisis. En la Rx de trax es frecuente el hallazgo de infiltra-
dos nodulares bilaterales y tpico la cavitacin de estos infiltrados.
Podemos encontrar tambin afectacin ocular, articular, cutnea y
con mayor frecuencia renal. Una asociacin frecuente es la
glomerulonefritis rpidamente progresiva. El diagnstico ms rentable
es la biopsia pulmonar, donde en la muestra histolgica encontramos
hallazgos de vasculitis y granulomas intra y extravasculares (respuesta
2 correcta). Sin embargo en la biopsia de otros rganos no encontra-
mos todos los hallazgos histolgicos de esta vasculitis. Como dice la
respuesta 5, en la biopsia de senos paranasales hay granulomas pero
no vasculitis, y en la biopsia renal hay vasculitis, pero no granulomas.
La respuesta 4 es la incorrecta.

Pregunta 90.- R: 3
El sndrome de Goodpasture es un sndrome de etiologa descono-
cida que puede manifestarse a cualquier edad, aunque predomina
Pregunta 86. Estadiaje radiolgico de la sarcoidosis pulmonar.
en varones jvenes. Se caracteriza por la trada de hemorragia alveo-
lar, glomerulonefritis (que puede ser progresiva y conducir a la insufi-
El sndrome de Lfgren es una forma aguda de sarcoidosis que ciencia renal) y anticuerpos circulantes frente a antgenos de la mem-
presenta eritema nodoso, fiebre, adenopatas hiliares bilaterales y si- brana basal. El cuadro clnico inicial suele manifestarse con hemopti-
mtricas y poliartralgias (respuesta 5 correcta). sis (80%), disnea (70%), tos y debilidad. Estos sntomas se acompaan

M exico A rgentina CTO Medicina C/ Nez de Balboa, 115 28006 MADRID (Espaa) Tfno.: (91) 782 43 32 / Fax: (91) 782 43 27
C hile U ruguay E-mail: secretaria@ctomedicina.com; iberocto@ctomedicina.com WEB: www.ctomedicina.com; www.iberocto.com
NM Pg. 21
NEUMOLOGA Y CIRUGA TORCICA
Seguimiento a distancia Preparacin Examen de Seleccin 05/06 1 Vuelta

Comentarios TEST

Pregunta 90. Enfermedades intersticiales con afectacin radiolgica de lbulos superiores e inferiores.

M exico A rgentina CTO Medicina C/ Nez de Balboa, 115 28006 MADRID (Espaa) Tfno.: (91) 782 43 32 / Fax: (91) 782 43 27
Pg. 22 NM C hile U ruguay E-mail: secretaria@ctomedicina.com; iberocto@ctomedicina.com WEB: www.ctomedicina.com; www.iberocto.com
NEUMOLOGA Y CIRUGA TORCICA
Preparacin Examen de Seleccin 05/06 1 Vuelta Seguimiento a distancia
en el 80% de los pacientes de infiltrados alveolares perihiliares. La Pregunta 93.- R: 5
afectacin renal inicial consiste en proteinuria y microhematuria. La La hidatidosis pulmonar es una antropozoonosis producida por el
insuficiencia renal manifiesta aparece posteriormente. Es caractersti- desarrollo, en dicha vscera, de un gusano llamado Echinococcus. El
co que curse en brotes. Es caracterstico encontrar la DLCO aumenta- hombre se contagia cuando ingiere aguas o verduras contaminadas
da cuando hay hemorragia alveolar aguda (respuesta 1) y en la explo- por heces de crnidos que portan los huevecillos de este parsito.
racin funcional el hallazgo ms frecuente es un patrn restrictivo. Inicialmente la infeccin se localiza a nivel heptico, y desde all el
Para el diagnstico, la biopsia renal es el proceder de eleccin, ya que germen migra por las venas suprahepticas hasta la circulacin pul-
desde fases precoces el depsito de inmunoglobulinas es constante, monar. La clnica caracterstica en sus primeros estadios evolutivos es
teniendo adems implicacin pronstica (respuestas 2 y 5 correctas). la que presenta el paciente de nuestra pregunta, es decir, asintomti-
La causa principal de muerte es la hemorragia pulmonar, que se trata co. El diagnstico de sospecha es radiolgico (Rx trax, TC, y RMN) y
con corticoides intravenosos a altas dosis (respuesta 4 correcta). El se confirma mediante serologa. El quiste ir progresando en sus distin-
tratamiento de la enfermedad se hace con inmunosupresores y tos estadios evolutivos, siendo frecuentes las complicaciones. Estas son
plasmafresis, siendo esta ltima la que ha aumentado la tasa de su- ms probables a mayor evolucin de la lesin. Por este motivo, las
pervivencia. respuestas 1 y 2 son incorrectas. El tratamiento farmacolgico se utili-
Ambos tratamientos y la hemodilisis son necesarios para controlar za generalmente asociado a ciruga. Tanto la biopsia mediada por
la afectacin renal (respuesta 3 es incorrecta, porque los corticoides fibrobroncoscopia como la PAAF guiada por TAC estn contraindica-
solos no bastan para controlar la glomerulonefritis). das, ya que el riesgo con estas tcnicas de rotura del quiste es muy alta.
Ver esquema en la pgina 23. Si esto ocurre, puede aparecer un sndrome sptico o una anafilaxia
grave. El tratamiento de eleccin es la exresis mediante toracotoma,
Pregunta 91.- R: 2 como dice la respuesta 5.
Las neumonas se pueden clasificar segn su forma de presenta-
cin clnica en tpicas y atpicas. La neumona tpica se presenta de Pregunta 94.- R: 4
forma brusca con fiebre alta, escalofros, tos con expectoracin puru- El cuadro clnico del paciente es compatible con una neumona
lenta y dolor torcico de caractersticas pleurticas. Es tambin fre- de origen bacteriano, una neumona tpica. El germen ms frecuente-
cuente la asociacin de herpes labial (respuesta 1 correcta). mente implicado es el neumococo. Las respuestas 1 y 3 son incorrec-
En la exploracin, la semiologa es de condensacin (crepitantes, tas porque no es un enfermo que cumpla criterios de gravedad para
aumento de las vibraciones vocales y soplo tubrico) y en la Rx de neumona extrahospitalaria. Los criterios de gravedad son: taquipnea
trax el hallazgo suele ser un patrn alveolar con broncograma. A importante (FR>30), insuficiencia respiratoria, signos de fatiga muscu-
veces se asocia derrame pleural. Los patgenos que de forma caracte- lar, disminucin del nivel de conciencia, afectacin radiolgica bila-
rstica se presentan as son las bacterias como el neumococo, el esta- teral y/o multilobar, shock sptico (TA< 90/60) y oliguria. La respuesta
filococo, el Haemophilus o los gram negativos (respuestas 4 y 5 correc- 2 es incorrecta, porque no nos dan ningn dato que nos oriente a que
tas). el paciente tiene una neumona por Legionella como hiponatremia,
Las neumonas atpicas tienen un curso ms larvado con predomi- diarrea o sndrome confusional. La respuesta 5 tambin es incorrecta
nio de sntomas generales, como dice la respuesta 3. Puede existir porque, como decamos al principio, es una neumona tpica y la C.
febrcula y tos seca. En la exploracin fsica son raros los hallazgos psitacci se pone de manifiesto como una neumona atpica. La res-
patolgicos. La respuesta 2 es incorrecta, ya que describe la semiolo- puesta 4 es, por tanto, la respuesta correcta. Al ser de probable etiolo-
ga de una condensacin. En la Rx de trax el hallazgo ms caracters- ga neumoccica, el tratamiento inicial en un paciente sin factores de
tico es un infiltrado intersticial y es rara la presencia de derrame para- riesgo es la penicilina oral durante 7-10 das. Slo si la clnica no
neumnico asociado (respuesta 5 correcta). mejora o hay datos de progresin radiolgica, valoraremos cambiar el
Patgenos que producen estas manifestaciones son el Mycoplasma, tipo de antibitico. Esto suele ocurrir en pacientes infectados por
la Coxiella y los virus. neumococos resistentes a la penicilina, ms frecuentes en las otitis
peditricas que en las neumonas adquiridas en la comunidad.
Pregunta 92.- R: 4
La Legionella pneumophila es ubicua en ambientes hmedos y c- Pregunta 95.- R: 4
lidos, siendo el agua su hbitat natural. La infeccin de individuos En esta pregunta, lo que necesitamos conocer son los criterios de
debilitados o inmunocomprometidos es consecuencia de la inhala- ingreso en los pacientes con neumona adquirida en la comunidad.
Comentarios TEST

cin de la bacteria en aerosol. Puede originar neumonas espordicas Son los siguientes:
o en brotes epidmicos. La confusin puede ser un rasgo prominente Edad superior a los 65 aos.
y en la mitad de los pacientes aparece cefalea importante, confusin Enfermedades subyacentes crnico-debilitantes, principalmente
y delirio. Entre las manifestaciones extrapulmonares podemos encon- EPOC, diabetes mellitus, insuficiencia cardaca o renal crnicos,
trar pericarditis, miocarditis, afectacin digestiva, afectacin renal y alcoholismo, esplenectoma, malnutricin, cirrosis heptica, de-
encefalitis. Sospechamos Legionella ante todo paciente grave con neu- mencia y sospecha de aspiracin.
mona sin respuesta a betalactmicos, fiebre alta, hiponatremia, mani- Cavitacin o derrame paraneumnico.
festaciones neurolgicas, alteraciones digestivas y LDH >700 u/ml. La Recuento leucocitario <4000 o > 30000/l.
respuesta 1 es, por tanto, correcta. La Chlamydia, tanto la pneumoniae Hematocrito < 30%.
como la psittaci, cursan como neumonas atpicas y pueden manifes- Sospecha de etiologa poco habitual, como S. aureus, bacilos gram
tarse como broncoespasmos con sibilancias en la auscultacin negativos o anaerobios.
pulmonar. En la C. psittaci hay que buscar tambin en la exploracin Complicaciones spticas extrapulmonares, por ejemplo, meningi-
esplenomegalia. Este germen es causante de la psitacosis y se relaciona tis, artritis o endocarditis.
con la exposicin a pjaros (respuestas 2 y 3 correctas). La respuesta 4 Neumona grave (FR >30 rpm, insuficiencia respiratoria, fatiga
es incorrecta porque, aunque es cierto que la Coxiella burnetti es una muscular, deterioro del nivel de conciencia, afectacin radiolgi-
rickettsiosis, es la nica que cursa sin exantema. Es el agente responsa- ca bilateral y/o multilobar, TA < 90/60 y oliguria).
ble de la Fiebre Q, enfermedad febril aguda que se acompaa de Falta de respuesta al tratamiento inicial, imposibilidad de ingesta
neumona en casi la mitad de los pacientes. La transmisin al hombre oral o dificultad para el manejo ambulatorio.
suele ser resultado de la inhalacin de polvo contaminado con
coxiellas procedentes de heces secas, orina, lecho o aerosoles en El paciente de la respuesta 1 tiene una cifra de leucocitos <4000/
mataderos. Como resiste mucho tiempo en el medio ambiente, a ml; el de la respuesta 2 presenta comorbilidad asociada; el de la
veces no se conoce el antecedente epidemiolgico del contacto ani- respuesta 3 est hipotenso (TA < 90/60), y el de la respuesta 5 presenta
mal. Como dice la respuesta 5, la manifestacin extrapulmonar ms un fallo teraputico. El nico paciente sin criterios de ingreso es el
frecuente es la hepatitis. paciente de la respuesta 4.

M exico A rgentina CTO Medicina C/ Nez de Balboa, 115 28006 MADRID (Espaa) Tfno.: (91) 782 43 32 / Fax: (91) 782 43 27
C hile U ruguay E-mail: secretaria@ctomedicina.com; iberocto@ctomedicina.com WEB: www.ctomedicina.com; www.iberocto.com
NM Pg. 23
NEUMOLOGA Y CIRUGA TORCICA
Seguimiento a distancia Preparacin Examen de Seleccin 05/06 1 Vuelta
Pregunta 96.- R: 1 la situacin es sostenida en el tiempo, aparece la fibrosis en 2-3 sema-
La hipoventilacin se produce cuando la PaCO2 aumenta por en- nas. La sospecha clnica se confirma con criterios gasomtricos (PO2
cima de los valores normales, es decir, hay hipercapnia. < 55mmHg con FiO2 >50%; PaO2 <50mmHg con FiO2 > 60%) y
Los sndromes de hipoventilacin crnica pueden producirse por con la presencia de una compliance pulmonar disminuida (<50ml/
3 mecanismos: alteracin del impulso respiratorio, defectos en el sis- cm M2O).
tema neuromuscular o alteracin del aparato ventilatorio. La causa ms frecuente de este cuadro es la sepsis, y el tratamiento
Las alteraciones del impulso respiratorio se localizan en los qui- consiste en solucionar el problema de base, oxigenoterapia y ventila-
miorreceptores centrales y perifricos y en las neuronas respiratorias cin mecnica.
del tronco enceflico. En estos enfermos no hay respuesta de hiper-
ventilacin a los estmulos qumicos y la Po.1 y el EMGdi estn altera-
dos. Son capaces de hiperventilar de forma voluntaria y las presiones
mximas, volmenes, flujos, resistencia y distensibilidad son normales
(respuestas 2 y 3 correctas).
Los defectos en el sistema neuromuscular se localizan en la mdula
y nervios perifricos, y en los msculos respiratorios. La respuesta a los
estmulos qumicos, la Po. 1 y el EMGdi son patolgicos. En estos
enfermos es caracterstica la incapacidad para hiperventilar volunta-
riamente (respuesta 1 incorrecta) y la disminucin de las presiones
mximas (respuesta 4 correcta).
Los defectos en el aparato ventilatorio se localizan en la pared
torcica, en el pulmn y en las vas respiratorias. La respuesta ventila-
toria a los estmulos qumicos est deteriorada, pero la Po.1 y el EMGdi
estn conservados. Lo tpico en estos defectos es la disminucin de los
volmenes y flujos, el aumento de las resistencias, la disminucin de
la distensibilidad y el aumento del gradiente alveolo-arterial de oxge-
no.

Pregunta 97.- R: 5
Una apnea consiste en el cese completo del flujo inspiratorio supe-
rior a 10 segundos de duracin. Hay tres tipos de apnea: la obstructi-
va, la central y la mixta. En la apnea obstructiva hay una oclusin de
la va area superior a nivel de la orofaringe. Como consecuencia de
esto, el flujo cesa mantenindose el esfuerzo muscular ventilatorio, ya
que el impulso respiratorio continua. Son las apneas ms frecuentes.
En la apnea central, el flujo cesa al abolirse transitoriamente el impul-
so a los msculos respiratorios. No hay, por tanto, esfuerzo muscular.
Las apneas mixtas consisten en episodios centrales seguidos de un
componente obstructivo. Las apneas obstructivas casi siempre son
patolgicas, las centrales pueden aparecer de forma aislada en el
sueo REM de gente sana (respuestas 1, 2 y 4 correctas). Pregunta 98. Fisiopatologa del SDRA.
Una hipopnea consiste en el cese del flujo areo de al menos un 50%,
acompaado de una desaturacin de la hemoglobina de al menos 4%.
El ndice de apnea-hipopnea (IAH) es el nmero total de apneas e Pregunta 98. Diagnstico diferencial SDRA vs EAP
hipopneas dividido por los minutos totales de sueo y multiplicado
por 60. Se considera patolgico un IAH >10 (respuesta 3 correcta). 1234 546
El diagnstico definitivo para confirmar el SAOS es la polisomno-
grafa, en la que se evidencian los episodios de cese de flujo areo con 5789
9 12342565789
39 95 54944 25238
3253982

Comentarios TEST
mantenimiento del esfuerzo respiratorio con IAH mayor o igual a 10. 23 92556589

La oximetra domiciliaria se utiliza como prueba de screening (res- 
8 4 925889425
 789 2582595 1
puesta 5 incorrecta). 93
 25259 79892 3982
Pregunta 98.- R: 2
 7 824529 984 
294 "5 7 8245292894
El sndrome de dificultad respiratoria del adulto (SDRA) se sospe-
cha cuando aparece una insuficiencia respiratoria aguda de rpida
3
734459589 5!28 55598!2894
"5128922
evolucin, junto con un patrn radiolgico con infiltrados alveolo-
intersticiales difusos sin cardiomegalia. La ausencia de cardiomega- 66 #82
lia, al igual que la presencia de una presin capilar de enclavamien- 9
8 
to (PCP) normal, son muy importantes, ya que sitan el problema en
el pulmn. Si hay aumento de la PCP o cardiomegalia la afectacin "5$%9  9822
pulmonar es por hipertensin pulmonar postcapilar secundaria a
18898595949
29 2 9 "5&372 59587 2
un problema cardaco en cavidades izquierdas. Por tanto, la res-
$%9  9822 "538492
puesta 2 es incorrecta.  7 879
&5 5945437
9 9' "5&5(*&5+,--'
En el SDRA aparece un edema pulmonar por aumento de la per-
())( 555.5#/
meabilidad de la membrana alveolocapilar cuando es lesionada. Esto 555.5# 88342 
hace que el lquido intraalveolar sea muy rico en protenas e interfiera "5  80
454 4
con el surfactante, lo que conlleva el colapso alveolar. Se forman
entonces membranas hialinas en las zonas ms declives y queda sola-
mente una pequea parte de parnquima sano que debe realizar
todo el trabajo. Estas alteraciones ventilacin/perfusin condicionan Pregunta 99.- R: 2
la aparicin en fases avanzadas de la enfermedad de shunt El secuestro broncopulmonar es una masa de tejido pulmonar
intrapulmonar secundario al colapso y atelectasias que aparecen. Si separada del parnquima sano, sin comunicacin con la va area y

M exico A rgentina CTO Medicina C/ Nez de Balboa, 115 28006 MADRID (Espaa) Tfno.: (91) 782 43 32 / Fax: (91) 782 43 27
Pg. 24 NM C hile U ruguay E-mail: secretaria@ctomedicina.com; iberocto@ctomedicina.com WEB: www.ctomedicina.com; www.iberocto.com
NEUMOLOGA Y CIRUGA TORCICA
Preparacin Examen de Seleccin 05/06 1 Vuelta Seguimiento a distancia
cuyo aporte sanguneo proviene de una arteria sistmica (lo ms fre-
cuente es la aorta torcica o abdominal y en ocasiones una arteria
intercostal). De la definicin de esta malformacin pulmonar extrae-
mos la informacin necesaria para decir que la respuesta falsa es la 2
y que la respuesta 1 es correcta.
Hay 2 tipos de secuestros: intralobar y extralobar. El secuestro
intralobar carece de pleura propia y suele localizarse en el ngulo
costofrnico posterior izquierdo. El secuestro extralobar tiene pleura
propia, est en relacin con el hemidiafragma izquierdo en ms de un
90% de los casos y su drenaje venoso es anmalo a la circulacin
sistmica, creando un cortocircuito izquierda - derecha (respuestas 3
y 4 correctas).
Suelen ser asintomticos hasta que se infectan. En este momento
cursan con clnica de hemoptisis y expectoracin purulenta.
El tratamiento siempre es quirrgico, aun cuando todava no ha-
yan aparecido los sntomas, para evitar las complicaciones. En el estu-
dio preoperatorio es necesario realizar una arteriografa, para ver de
donde proviene el aporte sanguneo, y un esofagograma, para descar-
tar comunicacin con el esfago.

Pregunta 100.- R: 3
Una de las complicaciones del trasplante pulmonar es el rechazo.
Hay dos tipos de rechazo: agudo y crnico. El rechazo agudo lo
presentan tres cuartas partes de los pacientes, generalmente en los dos
o tres primeros meses. Clnicamente se presenta como fiebre, tos, dis-
nea, crepitantes..., simulando un proceso infeccioso. Hay afectacin
de la funcin pulmonar con hipoxemia y disminucin del FEV1. El
diagnstico se realiza con biopsia transbronquial (tiene una alta sensi-
bilidad y especificidad) y la respuesta al tratamiento con corticoides
intravenosos es bueno (respuestas 1, 2, y 4 correctas). El rechazo cr-
nico aparece en el 50% de los pacientes y cursa como una bronquio-
litis obliterante (respuesta 5 correcta). El diagnstico es clnico (infec-
cin respiratoria de curso subagudo en pacientes con antecedentes
de rechazo agudo y cada importante y progresiva del FEV1), ya que la
biopsia transbronquial es poco sensible. El tratamiento consiste en
aumentar los inmunosupresores, aunque no siempre se consigue fre-
nar la enfermedad. Es la principal causa de muerte tarda, pero no la
causa ms frecuente de muerte. La primera causa de mortalidad son
las infecciones (respuesta 3 incorrecta). La neumona ms frecuente es
la neumona por CMV. Aparece tras el primer mes postrasplante y
antes de los 6 meses. Clnicamente es indistinguible del rechazo agu-
do y se trata con ganciclovir. Otras causas de neumonas son el estafi-
lococo, los bacilos gram negativos y el Aspergillus.
Comentarios TEST

M exico A rgentina CTO Medicina C/ Nez de Balboa, 115 28006 MADRID (Espaa) Tfno.: (91) 782 43 32 / Fax: (91) 782 43 27
C hile U ruguay E-mail: secretaria@ctomedicina.com; iberocto@ctomedicina.com WEB: www.ctomedicina.com; www.iberocto.com
NM Pg. 25
NEUROLOGA
Preparacin Examen de Seleccin 05/06 1 Vuelta Seguimiento a distancia

SEMIOLOGA. 1) Sndrome bulbar lateral o de Wallenberg.


2) Siringomielia cervical.
1. Un paciente con lesin en el rea motora puede presentar 3) Tumor apical pulmonar.
todas, EXCEPTO una: 4) Patologa carotdea.
5) Neuroles.
1) Ausencia de reflejos miotticos, en fases muy precoces.
2) Reflejos miotticos exaltados. 7. Paciente con disminucin de fuerza en miembro inferior
3) Ausencia de reflejos cutneo abdominales. derecho, con disminucin de sensibilidad trmica en miem-
4) Respuesta cutneo plantar flexora en fases avanzadas del bro inferior izquierdo y dficit de sensibilidad posicional en
cuadro. miembro inferior derecho. Se trata de:
5) Disminucin de la masa muscular del territorio afectado.
1) Sndrome medular anterior.
2. Un paciente presenta hemiparesia derecha que afecta a cara, 2) Sndrome medular posterolateral.
brazo y pierna. Seale, de entre las siguientes opciones, la 3) Sndrome medular central.
localizacin ms probable de la lesin: 4) Sndrome de hemiseccin medular derecha.
5) Sndrome de hemiseccin medular izquierda.
1) Hemimdula derecha.
2) Bulbo medial derecho. 8. La prdida de sensibilidad trmica y dolorosa bilateral entre los
3) Protuberancia izquierda. niveles D3 y D6, con sensibilidad conservada por encima y
4) Cpsula interna derecha. debajo de la lesin, nos debe hacer sospechar el diagnstico de:
5) Cpsula interna izquierda.
1) Polineuropata de fibras finas.
3. La exploracin de un paciente que consulta por diplopa 2) Polirradiculopata.
revela limitacin en la mirada hacia fuera del ojo izquierdo, 3) Lesin centromedular.
paresia hemifacial izquierda y disminucin de fuerza en 4) Lesin cordonal anterior.
extremidades derechas. La lesin se sita en: 5) Lesin cordonal posterior.

1) Cpsula interna izquierda. 9. Un varn de 60 aos refiere, desde hace seis meses, dficit
2) Mesencfalo izquierdo. motor progresivo en miembros inferiores, acompaado de
3) Mesencfalo derecho. sensacin elctrica cuando flexiona el cuello, que se inicia a
4) Protuberancia izquierda. nivel cervical y se irradia distalmente. A la exploracin se
5) Conducto auditivo interno izquierdo. objetivan amiotrofias en musculatura intrnseca de las ma-
nos, reflejos bicipital y estilorradial normales, hiperreflexia
4. Paciente con hipoestesia hemifacial izquierda y hemicorpo- en miembros inferiores y respuesta cutneo plantar extenso-
ral derecha. Seale la localizacin ms probable de su lesin: ra bilateral. Cul de los siguientes diagnsticos le parece ms
probable?:
1) Corteza parietal izquierda.
2) Tlamo izquierdo. 1) Mielopata cervical.
3) Mesencfalo izquierdo. 2) Neuroles.
4) Bulbo izquierdo. 3) Degeneracin subaguda combinada de la mdula.
5) Ganglio de Gasser izquierdo. 4) Sndrome espinal anterior.
5) Esclerosis lateral amiotrfica.
5. Con respecto a la exploracin de la pupila, seale el enuncia-
do INCORRECTO: 10. Seale la definicin correcta:
Preguntas TEST

1) La lesin del tercer par craneal cursa con pupila midritica 1) Distonas: movimientos continuos, reptantes, que pueden
arreactiva homolateral. afectar a cualquier rea corporal y que suelen aumentar
2) Una lesin de la arteria cartida externa puede cursar con si el paciente intenta realizar un movimiento voluntario o
pupila mitica reactiva homolateral. hablar.
3) La presencia de pupilas puntiformes y reactivas puede 2) Mioclonas: movimientos repetitivos, irregulares y estereo-
deberse a una intoxicacin por opiceos.
tipados que pueden afectar a diversos grupos musculares.
4) La presencia de pupilas arreactivas a la luz, pero que se
3) Corea: movimientos involuntarios de torsin relaciona-
contraen al acomodarse a la visin cercana es sugerente
dos con contracciones musculares mantenidas.
de lesin cordonal posterior.
4) Temblor: movimiento oscilatorio rtmico debido a con-
5) La va aferente del reflejo fotomotor es el nervio ptico,
tracciones de tipo alternante de msculos agonistas y
mientras que la eferente es el tercer par.
antagonistas.
5) Atetosis: movimientos rpidos de amplitud variable debi-
6. Un paciente acude a su mdico por haber observado descen-
dos a contracciones simultneas de msculos agonistas y
so de su prpado derecho con disminucin de la hendidura
antagonistas.
palpebral. La exploracin revela una pupila derecha mitica.
En el diagnstico diferencial incluir todos, EXCEPTO uno de
11. Varn de 72 aos que presenta a la exploracin tono muscu-
los siguientes:
lar aumentado de forma generalizada, con resistencia con-

M exico A rgentina CTO Medicina C/ Nez de Balboa, 115 28006 MADRID (Espaa) Tfno.: (91) 782 43 32 / Fax: (91) 782 43 27
C hile U ruguay E-mail: secretaria@ctomedicina.com; iberocto@ctomedicina.com WEB: www.ctomedicina.com; www.iberocto.com
NR Pg. 1
NEUROLOGA
Seguimiento a distancia Preparacin Examen de Seleccin 05/06 1 Vuelta
tinua a la movilizacin pasiva y reflejos miotticos normales. 2) Temblor intencional.
De todas las siguientes, seale cul le parece la definicin 3) Temblor cintico.
correcta de la alteracin motora que presenta el paciente: 4) Marcha atxica.
5) Hipertona.
1) Discinesia.
2) Distona. 17. Paciente que presenta de forma progresiva bipedestacin y
3) Rigidez. marcha inestable, con pasos desiguales, sin clara lateralizacin.
4) Espasticidad. No presenta dismetras, disdiadococinesias ni oscilaciones en
5) Paresia. extremidades. Seale el diagnstico ms probable:

12. Seale cul es el tratamiento de eleccin en las distonas 1) Temblor esencial.


focales crvico-faciales: 2) Lesin ocupante de espacio en fosa posterior.
3) Atrofia aislada de vermis cerebeloso.
1) Anticolinrgicos. 4) Lesin cordonal posterior.
2) Benzodiacepinas. 5) Lesin mesenceflica.
3) Toxina botulnica.
4) Barbitricos. 18. Seale cul de los siguientes sntomas NO corresponde a una
5) Tenotoma. lesin de la corteza occipital:

13. Un paciente de 57 aos presenta oscilaciones en las manos 1) Prosopagnosia.


cuando realiza actividades como intentar enhebrar una 2) Astereognosia.
aguja o sujetar un vaso mientras bebe. No presenta movi- 3) Simultanagnosia.
mientos involuntarios cuando no realiza actividad con las 4) Hemianopsia homnima con respeto macular.
manos. Seale cul de estos trminos le parece ms correcto 5) Apraxia ptica.
para definir este trastorno:
19. Seale dnde localizara la lesin ante un paciente con
1) Temblor de reposo. cuadrantanopsia homnima superior izquierda:
2) Temblor postural.
3) Temblor cintico. 1) Lbulo temporal izquierdo.
4) Temblor intencional. 2) Lbulo temporal derecho.
5) Temblor rbrico. 3) Lbulo parietal izquierdo.
4) Lbulo parietal derecho.
14. Un movimiento involuntario, caracterizado por sacudidas 5) Cintilla ptica izquierda.
rpidas y arrtmicas secundarias a periodos silentes en la
actividad electromiogrfica, se puede encontrar asociado a 20. Cul de los siguientes datos clnicos NO es caracterstico de
cualquiera de estas encefalopatas, SALVO: la lesin del lbulo parietal?:

1) Hipoglucmica. 1) No reconocimiento de objetos a travs del tacto (astereog-


2) Hipercpnica. nosia).
3) Heptica. 2) Negligencia del hemicuerpo contralateral en lesiones no
4) Urmica. dominantes.
5) Txica por anticomiciales. 3) No discriminacin entre dos estmulos cutneos en hemi-
cuerpo contralateral.
15. Paciente de 55 aos que acude a su consulta porque en los 4) Anestesia dolorosa en el hemicuerpo contralateral.
ltimos meses refiere un temblorcillo en la mano izquierda 5) Cuadrantanopsia homnima inferior contralateral.

Preguntas TEST
cuando tiene la mano apoyada; si realiza cualquier movimiento,
este temblor desaparece. Cul sera su actitud?: 21. Una mujer de 72 aos, hipertensa, presenta bruscamente un
trastorno del lenguaje, con habla incoherente, sustituyendo
1) Por tratarse de un temblor postural, insistira sobre sus una palabra por otra, siendo incapaz de repetir palabras que
antecedentes familiares. se le dicen. Sin embargo, obedece rdenes verbales. La
2) Realizara urgentemente una prueba de imagen para localizacin ms probable de la lesin es:
descartar una lesin cerebelosa hemisfrica.
3) Pensara que probablemente en los prximos aos desa- 1) Lbulo frontal izquierdo.
rrollar una enfermedad de Parkinson. 2) Girus supramarginalis izquierdo.
4) Lo tranquilizara, ya que probablemente sean sus nervios. 3) Lbulo temporal izquierdo.
5) Buscara signos que apunten a una ingesta alcohlica 4) rea prefrontal izquierda.
importante. 5) En torno al rea de Wernicke.

16. De entre los siguientes sntomas y signos, seale cul NO CUADROS CONFUSIONALES. ENCEFALOPATAS METAB-
LICAS. COMA.
corresponde a una lesin cerebelosa:
22. Un paciente de 70 aos es trado a urgencias por sus familia-
1) Disartria.
res por presentar un cuadro de instauracin progresiva en

M exico A rgentina CTO Medicina C/ Nez de Balboa, 115 28006 MADRID (Espaa) Tfno.: (91) 782 43 32 / Fax: (91) 782 43 27
Pg. 2 NR C hile U ruguay E-mail: secretaria@ctomedicina.com; iberocto@ctomedicina.com WEB: www.ctomedicina.com; www.iberocto.com
NEUROLOGA
Preparacin Examen de Seleccin 05/06 1 Vuelta Seguimiento a distancia

el curso de varias horas consistente en inatencin, desorien- 4) Lesin bulbar.


tacin temporoespacial, incapacidad para retener informa- 5) Coma acidtico.
cin, lenguaje incoherente y alucinaciones. Seale la etiolo-
ga ms probable de este proceso: 28. Usted est valorando a un paciente que ingres hace 24 horas
por un cuadro confusional con lenguaje incoherente y ataxia.
1) Hematoma subdural subagudo. A la exploracin se le objetiva estado alerta con incapacidad
2) Tumoracin hemisfrica no dominante. para realizar cualquier movimiento voluntario, salvo parpa-
3) Estatus parcial complejo. deo dbil y movimientos verticales oculares. Su opinin sera:
4) Encefalitis herptica.
5) Trastorno metablico. 1) Debe realizarse TC craneal para descartar lesin dience-
flica ocupante de espacio.
23. NO se considera causa de estado comatoso: 2) Debe realizarse RM cerebral centrada en fosa posterior
por ser sta la prueba de imagen de eleccin.
1) Encefalopata urmica. 3) Debe tratrsele de forma inmediata con vitamina B1
2) Encefalopata heptica. parenteral.
3) Lesin compresiva mesenceflica. 4) Deben revisarse los hallazgos analticos al ingreso y la
4) Lesin vascular protuberancial extensa. sueroterapia pautada.
5) Lesin bulbar. 5) La causa ms probable es coma de origen conversivo.

24. La exploracin de un paciente en coma revela una postura en 29. La encefalopata anoxicoisqumica NO se caracteriza por:
extensin y adduccin de miembros inferiores, con exten-
sin, adduccin y rotacin interna de miembros superiores, 1) Buena regeneracin posterior de las reas afectadas.
que aparece con la estimulacin dolorosa. Cul de los 2) Su origen puede ser un infarto agudo de miocardio.
siguientes hallazgos es ms compatible con la situacin 3) Afectar de forma precoz el crtex cerebeloso, los ganglios
clnica descrita?: basales y el rea hipocampal.
4) Causar daos tisulares cerebrales en 5 minutos tras el inicio
1) Patrn respiratorio rtmico con pausas de apnea y rachas de la anoxia.
de hiperventilacin. 5) Aparicin de sntomas das despus de sufrir el proceso
2) Reflejos oculoceflicos conservados. isqumico.
3) Pupilas de tamao y reactividad normales.
4) Hematoma en fosa posterior. 30. Un paciente EPOC es trado a Urgencias por descompensa-
5) Respuesta a rdenes verbales. cin respiratoria. Seale cul de los sntomas NO esperara
encontrar a la exploracin:
25. Un paciente en coma presenta desviacin tnica de la mirada
hacia la izquierda y disminucin de la movilidad en hemicuer- 1) Asterixis.
po izquierdo ante estmulos dolorosos. Como causa de su 2) Temblor de reposo.
coma sospechar: 3) Edema de papila.
4) Cefalea.
1) Lesin hemisfrica izquierda. 5) Disminucin del nivel de conciencia.
2) Lesin protuberancial derecha.
3) Lesin bulbar derecha. 31. Cul de los siguientes datos NO es caracterstico del coma
4) Hipoglucemia severa. hipoglucmico?:
5) Encefalopata hipxica.
1) Aparicin de los primeros sntomas al cabo de minutos de
26. Ante un paciente en coma que presenta una respiracin hipoglucemia severa mantenida.
Preguntas TEST

taquipneica sin aparentes hallazgos en la auscultacin pul- 2) La ingesta de antidiabticos orales puede ser causa de
monar, pensara en primer lugar: coma hipoglucmico.
3) Puede cursar con hemiparesia, especialmente en ancia-
1) Lesin bulbar. nos.
2) Lesin pontina. 4) Se presenta como una desconexin brusca del medio, sin
3) Lesin mesenceflica. sntomas previos.
4) Lesin dienceflica. 5) La sobredosis de insulina es una causa frecuente.
5) Lesin hemisfrica bilateral.
32. Un paciente es trado a Urgencias por presentar diplopa,
27. Un paciente es trado al hospital en situacin clnica de coma. importante inestabilidad en la marcha y disminucin del nivel
A la exploracin se le objetiva patrn respiratorio de hiperven- de conciencia con lenguaje incoherente. Entre sus antecedentes
tilacin rtmica y pupilas isocricas de tamao medio y reac- destaca alcoholismo. De las siguientes opciones, cul le parece
tividad normal. El diagnstico ms probable es: la medida teraputica inicial ms adecuada?:

1) Lesin hemisfrica profunda. 1) Observacin clnica sin tratamiento.


2) Lesin mesenceflica. 2) Puncin lumbar y tratamiento con aciclovir emprico.
3) Lesin pontina. 3) Administracin de vitamina B 1 parenteral.

M exico A rgentina CTO Medicina C/ Nez de Balboa, 115 28006 MADRID (Espaa) Tfno.: (91) 782 43 32 / Fax: (91) 782 43 27
C hile U ruguay E-mail: secretaria@ctomedicina.com; iberocto@ctomedicina.com WEB: www.ctomedicina.com; www.iberocto.com
NR Pg. 3
NEUROLOGA
Seguimiento a distancia Preparacin Examen de Seleccin 05/06 1 Vuelta
4) Sueroterapia con sueros glucosados abundantes. 5) Factores neurotrpicos.
5) Administracin de neurolpticos parenterales.
38. Un paciente ha sido diagnosticado de demencia cortical.
AMNESIAS. DEMENCIAS. Usted NO esperara encontrar en la exploracin:

33. Sobre los defectos de memoria, seale la opcin FALSA: 1) Deterioro del estado de alerta y atencin.
2) Deterioro de la memoria de fijacin.
1) La memoria inmediata depende fundamentalmente del 3) Apraxias.
nivel de atencin. 4) Agnosias.
2) La amnesia global transitoria afecta bsicamente al recuer- 5) Afasias.
do de datos recientes.
3) El tratamiento de los cuadros de amnesia se basa en el 39. Seale cul de las siguientes enfermedades cursa con demen-
bloqueo de los receptores gabargicos. cia de caractersticas corticales:
4) La amnesia antergrada es uno de los hallazgos caracte-
rsticos del sndrome de Korsakoff. 1) Enfermedad de Pick.
5) Una lesin bilateral extensa de lbulos temporales puede 2) Enfermedad de Huntington.
cursar con un sndrome amnsico. 3) Enfermedad de Friedreich.
4) Enfermedad de Binswanger.
34. Acude a urgencias una paciente de 65 aos que de manera 5) Enfermedad de Parkinson.
brusca presenta prdida de la memoria reciente, preguntan-
do continuamente "dnde estoy?". En la exploracin no se 40. Paciente de 65 aos consulta por un cuadro progresivo de
objetiva ningn signo de focalidad neurolgica. Cul sera olvidos frecuentes, trastorno de la marcha y sacudidas mus-
culares breves y arrtmicas desencadenadas por sobresaltos.
su actitud?:
Seale, en relacin con la patologa que probablemente sufre
el paciente, la respuesta INCORRECTA:
1) Iniciara tratamiento antiepilptico, por tratarse de una
crisis parcial compleja.
1) La mitad de los pacientes afectados muestran signos de
2) Tranquilizara a la familia, porque probablemente en menos
afectacin de primera motoneurona, como hiperreflexia
de 6 horas el cuadro remita sin ningn problema.
e hipertona elstica.
3) Pedira una RM para descartar una encefalitis herptica
2) El deterioro severo se produce en la mayora de los pacien-
temporal.
tes antes de un ao desde el inicio de los sntomas.
4) Recomendara una valoracin psiquitrica. 3) Ms de la mitad de los pacientes suelen presentar algn
5) Iniciara antiagregacin con clopidogrel por tratarse de un signo parkinsoniano.
fenmeno isqumico cerebral. 4) El EEG no es til para el diagnstico.
5) Pueden existir antecedentes familiares en un reducido
35. El deterioro de funciones superiores caracterizado por de- nmero de casos.
terioro de memoria, trastornos de concentracin y enlente-
cimiento en los procesos de pensamiento, con ausencia de 41. Una de estas patologas NO est producida por priones:
francas apraxias, agnosias o afasia, es caracterstico de todas,
MENOS una de estas patologas: 1) Enfermedad de Gerstmann-Straussler-Scheinker.
2) Enfermedad de Creutzfeldt-Jakob.
1) Enfermedad de Binswanger. 3) Enfermedad de Strachan.
2) Enfermedad de Huntington. 4) Insomnio familiar fatal.
3) Enfermedad de Alzheimer.
5) Encefalopata espongiforme.
4) Estadio lacunar.
5) Demencia multiinfarto.
42. Sobre la enfermedad de Binswanger, seale la FALSA:

Preguntas TEST
36. La enfermedad de Alzheimer se caracteriza por todos los
1) Aparece en pacientes hipertensos mal controlados de
siguientes enunciados, EXCEPTO uno:
larga evolucin.
2) Presenta leucoaraiosis en las pruebas de imagen cerebral,
1) Es la causa de demencia ms frecuente en Occidente.
2) La prdida neuronal es difusa, con predominio en lbulos que aunque no es patognomnica, resulta muy caracte-
frontales y temporales. rstica de esta patologa.
3) El diagnstico de certeza es anatomopatolgico. 3) Existen alteraciones en la marcha, con una marcha a
4) Las placas seniles y de degeneracin neurofibrilar son pequeos pasos.
patognomnicas. 4) Cursa con demencia subcortical.
5) Cursa con reduccin de neurotransmisores en SNC. 5) El tratamiento antihipertensivo adecuado puede revertir
los sntomas.
37. Cul de los siguientes es el frmaco ms til para la enferme-
dad de Alzheimer?: EPILEPSIA.

1) Agonistas de receptores dopaminrgicos. 43. La diferencia entre crisis parciales simples y complejas es:
2) Inhibidores de la acetilcolinesterasa cerebral.
3) Inhibidores de la MAO-B. 1) Presencia o no de manifestaciones psquicas.
4) Antioxidantes (idebenone). 2) Duracin de la crisis.

M exico A rgentina CTO Medicina C/ Nez de Balboa, 115 28006 MADRID (Espaa) Tfno.: (91) 782 43 32 / Fax: (91) 782 43 27
Pg. 4 NR C hile U ruguay E-mail: secretaria@ctomedicina.com; iberocto@ctomedicina.com WEB: www.ctomedicina.com; www.iberocto.com
NEUROLOGA
Preparacin Examen de Seleccin 05/06 1 Vuelta Seguimiento a distancia

3) Conservacin de la conciencia y percepcin del ambiente. 48. En el sndrome de West es caracterstico:


4) Foco en lbulo temporal.
5) Etiologa subyacente. 1) Conservacin del desarrollo psicomotor a lo largo de la
enfermedad.
44. Un paciente de 25 aos ha comenzado a presentar episodios 2) Existencia de patologa cerebral subyacente en la mayora
repetidos de prdida de conciencia consistentes en sensa- de los casos.
cin epigstrica ascendente seguida de desconexin del 3) Crisis generalizadas mioclnicas.
medio, mirada fija, automatismos deglutorios y falta de 4) Hipsarritmia durante las crisis.
respuesta a estmulos externos. Se sigue de corta confusin 5) Buen control teraputico con fenitona.
posterior. La duracin de estos episodios es de 2-3 minutos.
Se trata probablemente de: 49. Seale la respuesta correcta con respecto a las crisis febriles
infantiles:
1) Crisis generalizada no convulsiva.
2) Crisis parcial simple. 1) Las tpicas duran escasos segundos.
3) Crisis parcial compleja. 2) Son frecuentes los antecedentes familiares de crisis febriles
4) Crisis de ausencia. o de epilepsia.
5) No presenta semiologa de crisis comicial. 3) El EEG intercrtico muestra descargas de punta-onda lenta
sin repercusin clnica.
45. Un paciente de 70 aos, con antecedente de meningioma 4) Precisan tratamiento anticomicial continuado hasta los 9-
operado hace un ao, refiere desde hace 2 semanas episodios 10 aos, edad a la que las crisis ceden espontneamente.
repetidos consistentes en movimientos clnicos involuntarios 5) Se asocian con bajo rendimiento escolar.
que comienzan en la mano derecha y progresan por miembro
superior hasta afectar a todo el hemicuerpo derecho. Duran 50. Un paciente de 25 aos ha presentado una crisis comicial de
unos 3 minutos y ceden sin secuelas. No hay deterioro de tipo tnico-clnico generalizado, sin claro factor desencade-
conciencia asociado. Podemos afirmar que: nante. La exploracin es normal; el EEG interictal y la RM
cerebral son normales. La actitud teraputica recomendada
1) Se trata de una crisis clnica secundariamente generalizada. sera:
2) Se trata de una crisis simple motora.
3) Es una crisis parcial compleja. 1) Esperar a una segunda crisis para instaurar tratamiento.
4) El foco epilptico parece localizarse en el lbulo parietal 2) Iniciar tratamiento con cido valproico.
izquierdo, junto a la hoz. 3) Iniciar tratamiento con etosuximida.
5) Supone un signo inequvoco de recidiva tumoral. 4) Iniciar tratamiento con vigabatrina.
5) Iniciar tratamiento en caso de presentar antecedentes
46. Un varn de 45 aos comienza de modo brusco con movi- familiares.
mientos tnicos en miembro superior derecho, de pocos
segundos de duracin, que en una ocasin son seguidos de 51. Es correcto, en relacin al tratamiento de la epilepsia:
prdida brusca del nivel de conciencia junto con movimien-
tos tnico-clnicos generalizados. Refiere no haber sufrido 1) El tratamiento en monoterapia consigue el control de las
nunca accidentes o contusiones de importancia. Sobre el crisis en el 20% de los pacientes aproximadamente.
caso, es INCORRECTO: 2) La carbamacepina es el frmaco de segunda eleccin en
las crisis de ausencia tpicas.
1) La causa ms probable del caso es una lesin ocupante de 3) La etosuximida es el tratamiento de eleccin para las crisis
espacio a nivel cortical. parciales complejas.
2) Est indicada la realizacin urgente de una TC craneal en 4) El fenobarbital en nios puede producir un sndrome de
busca del diagnstico. hiperactividad.
5) Es habitual el tratamiento profilctico de las crisis febriles
Preguntas TEST

3) El origen ms probable de la crisis es un traumatismo


craneoenceflico de baja intensidad, que el paciente a largo plazo con bajas dosis de valproico.
haya olvidado.
4) El episodio tnico-clnico generalizado puede deberse a 52. El tratamiento de eleccin en el estatus epilptico es:
una generalizacin a partir del foco hemisfrico izquierdo.
5) En el caso de tratarse de una lesin tumoral, lo ms 1) Primero clonacepam i.v. y si no cede, valproato sdico i.v.
probable es que se trate de una metstasis. 2) Primero diacepam i.v. y si no cede, difenilhidantona i.v.
3) Primero fenobarbital i.v.; si no cede, fenitona i.v.
4) Primero valproato i.v. y si no cede, fenitona i.v.
47. Seale cul de las siguientes es un patrn caracterstico de
5) Fenobarbital i.v. y si no cede, penthotal sdico i.v.
crisis originadas en el lbulo frontal:
53. Una mujer de 30 aos, diagnosticada de epilepsia generalizada
1) Parestesias hemicorporales.
tnico-clnica desde los 20, consulta por estar embarazada de
2) Alucinaciones visuales en forma de luces centelleantes.
4 semanas. Se encuentra en tratamiento con carbamacepina
3) Elevacin del brazo con desviacin ceflica hacia ese
desde los 20 aos y no ha presentado crisis desde los 25 aos
brazo.
de edad. La actitud teraputica recomendada sera:
4) Sensacin de bolo gstrico que asciende seguido de
alteracin del nivel de conciencia.
1) Suspender progresivamente el tratamiento.
5) Parpadeo bilateral con mirada fija.

M exico A rgentina CTO Medicina C/ Nez de Balboa, 115 28006 MADRID (Espaa) Tfno.: (91) 782 43 32 / Fax: (91) 782 43 27
C hile U ruguay E-mail: secretaria@ctomedicina.com; iberocto@ctomedicina.com WEB: www.ctomedicina.com; www.iberocto.com
NR Pg. 5
NEUROLOGA
Seguimiento a distancia Preparacin Examen de Seleccin 05/06 1 Vuelta
2) Mantener el tratamiento a la dosis actual y vigilar los niveles 1) Los corticoides ayudan a mejorar secuelas previas.
plasmticos del frmaco. 2) Los corticoides disminuyen el nmero de brotes.
3) Aadir cido valproico, por ser ste el frmaco de eleccin 3) El uso de interfern no est indicado en el primer brote.
para su tipo de epilepsia. 4) El tratamiento concomitante con corticoides e interfern
4) Aumentar la dosis de carbamacepina, por ser habitual una est contraindicado.
cada de los niveles plasmticos del frmaco durante el 5) No existe tratamiento que modifique el curso clnico de
embarazo. la enfermedad.
5) Se recomienda aborto teraputico.
60. Con respecto al tratamiento para modificar el curso de la
54. Seale cul de los siguientes efectos adversos NO es carac- enfermedad en la esclerosis mltiple, seale la respuesta
terstico de cada frmaco: INCORRECTA:

1) Fenitona: hiperplasia gingival. 1) No hay tratamiento especfico para las formas progresivas
2) Valproato: cada de pelo. primarias.
3) Fenobarbital: sedacin. 2) En las formas remitentes-recurrentes puede llegar a redu-
4) Carbamacepina: reduccin concntrica del campo vi- cirse un 30% de los brotes.
sual. 3) Estn disponibles los interferones beta 1a y beta 1b y el
5) Felbamato: aplasia medular. acetato de glatiramer.
4) El tratamiento con interferon puede ayudar a la recupe-
racin de secuelas de brotes previos.
ENFERMEDADES DESMIELINIZANTES. 5) La administracin de estos frmacos es por va parenteral.
55. Seale cul de los siguientes sntomas es el ms frecuente
61. Es FALSO sobre la encefalomielitis diseminada aguda:
como debut de una esclerosis mltiple:
1) Es una enfermedad de curso monofsico.
1) Hipoestesias.
2) Es habitual que las lesiones desmielinizantes presenten
2) Paresia.
captacin de contraste.
3) Disminucin de la agudeza visual.
3) Clsicamente se asocia con la infeccin por el virus del
4) Disfasia.
sarampin.
5) Disfuncin esfinteriana.
4) En pases desarrollados, la varicela es el agente etiolgico
ms frecuentemente asociado.
56. Una joven con antecedente de trastorno sensitivo hemicor-
poral hace 1 ao, presenta diplopa de 1 semana de evolu- 5) El interfern beta es til para el control sintomtico de la
cin. La exploracin muestra limitacin en la aproximacin enfermedad.
del ojo derecho con nistagmo en ojo izquierdo cuando
intenta la mirada lateral a la izquierda. Se trata de: 62. Usted es consultado para valorar a un nio de 10 aos que
presenta retraso escolar, crisis generalizadas y ataxia progre-
1) Lesin fascicular del tercer par derecho. siva. En las exploraciones complementarias destaca un EEG
2) Lesin nuclear del tercer par derecho. con descargas de ondas lentas de alto voltaje seguidas de
3) Lesin del fascculo longitudinal medial derecho. perodo de aplanamiento de la actividad cerebral. En la RM
4) Lesin del fascculo longitudinal medial izquierdo. cerebral realizada se aprecian mltiples lesiones en sustancia
5) Oftalmoplejia internuclear bilateral. blanca y moderada atrofia cortical. En relacin con la enfer-
medad que probablemente presenta, seale la respuesta
57. Seale cul de los siguientes sntomas es ms frecuente en el correcta:
curso de una esclerosis mltiple:
1) Se trata probablemente de un caso de esclerosis mltiple.

Preguntas TEST
1) Depresin. 2) La causa es una infeccin vrica, la ms probable, la
2) Crisis epilpticas. varicela, por ser sta la enfermedad vrica nativa ms
3) Neuralgia del trigmino. frecuente en pases occidentales.
4) Movimientos coreoatetsicos involuntarios. 3) En el LCR esperara encontrar bandas oligoclonales con
5) Signo de Lhermitte. aumento de las gammaglobulinas.
4) Si se aplica en fases iniciales, el tratamiento especfico es
58. Seale cul de los siguientes hallazgos en las pruebas comple- altamente efectivo.
mentarias es MENOS sugerente de esclerosis mltiple: 5) Es un cuadro frecuente y habitualmente autolimitado.

1) Bandas oligoclonales en el LCR. 63. Un paciente con antecedentes de alcoholismo es trado al


2) Aumento del ndice de IgG en el LCR. hospital tras ser encontrado en la calle con alteracin del
3) Enlentecimiento de los potenciales evocados visuales. nivel de conciencia. En la exploracin presenta apertura
4) RM cerebral normal. ocular espontnea, tetraparesia, ausencia de movimientos
5) Lesiones periventriculares en la RM cerebral. faciales, con conservacin de motilidad ocular vertical vo-
luntaria y parpadeo, que parece hacer de forma voluntaria
59. Sobre el tratamiento de la esclerosis mltiple, seale la en respuesta a orden verbal. Seale cul de entre los siguien-
respuesta correcta: tes le parece el diagnstico ms probable:

M exico A rgentina CTO Medicina C/ Nez de Balboa, 115 28006 MADRID (Espaa) Tfno.: (91) 782 43 32 / Fax: (91) 782 43 27
Pg. 6 NR C hile U ruguay E-mail: secretaria@ctomedicina.com; iberocto@ctomedicina.com WEB: www.ctomedicina.com; www.iberocto.com
NEUROLOGA
Preparacin Examen de Seleccin 05/06 1 Vuelta Seguimiento a distancia

1) Brote agudo de esclerosis mltiple. 3) Enfermedad por cuerpos de Lewy difusos.


2) Mielinolisis central pontina 4) Enfermedad de Creutzfeldt-Jakob.
3) Intoxicacin etlica aguda 5) Enfermedad de Alzheimer.
4) Encefalopata de Wernicke.
5) Psicosis de Korsakoff. 68. Varn de 75 aos, con clnica progresiva de trastornos en la
marcha, cadas frecuentes, rigidez cervical en extensin,
parlisis de la mirada conjugada vertical y demencia. El
PATOLOGA EXTRAPIRAMIDAL. trastorno ms probable es:

64. Un paciente de 47 aos presenta un cuadro progresivo de 1) Enfermedad de Parkinson.


decaimiento, tristeza, trastornos de memoria, y movimientos 2) Sndrome de Steele-Richardson-Olszewski.
continuos involuntarios de forma generalizada. No tiene 3) Atrofia olivopontocerebelosa.
antecedentes personales destacables. Sabe que su abuelo 4) Enfermedad de Shy-Drager.
paterno "perdi la cabeza" y que tena movimientos extraos. 5) Hidrocefalia normotensa.
Su padre falleci joven en un accidente. Seale, sobre el
diagnstico ms probable de este paciente, la respuesta FALSA: 69. Ante un paciente que presenta rigidez, bradicinesia, escaso
temblor y marcha atxica con aumento de base de sustenta-
1) En una TC craneal se podr objetivar atrofia de la cabeza cin, su diagnstico de presuncin ser:
del ncleo caudado.
2) El pronstico de esta enfermedad es de una supervivencia 1) Enfermedad de Parkinson.
de entre 10 y 25 aos. 2) Sndrome de Steele-Richardson-Olszewski.
3) El tratamiento de eleccin es con neurolpticos (blo- 3) Atrofia olivopontocerebelosa.
queantes de receptores dopaminrgicos). 4) Enfermedad de Shy-Drager.
4) Si el padre del paciente hubiera vivido lo suficiente, con 5) Hidrocefalia normotensa.
toda probabilidad habra presentado un cuadro similar.
5) El nico aminocido que se encuentra alterado en el 70. Un varn de 65 aos consulta por dificultad para iniciar la
sistema nervioso central es la dopamina. miccin, incontinencia e impotencia. En la historia clnica
destaca la presencia de sncopes posturales desde hace aos,
65. Es INCORRECTO sobre la enfermedad de Huntington: junto con dificultad para la marcha, rigidez y bradicinesia.
Cul es su sospecha diagnstica?:
1) Asocia movimientos coreicos, demencia subcortical y
trastornos psiquitricos. 1) Enfermedad de Parkinson.
2) El diagnstico puede establecerse con la clnica y los 2) Sndrome de Steele-Richardson-Olszewski.
antecedentes familiares. 3) Atrofia olivopontocerebelosa.
3) Presenta una dilatacin selectiva de las astas frontales de 4) Enfermedad de Shy-Drager.
los ventrculos laterales. 5) Hidrocefalia normotensa.
4) Existe una alteracin gentica en el cromosoma X.
5) No existe medicacin que detenga la progresin de la 71. Cul de entre los siguientes datos clnicos, cuando se presen-
enfermedad, aunque s puede pautarse tratamiento sin- tan desde el inicio del cuadro sindrmico, NO es sugerente
tomtico. de enfermedad de Parkinson idioptica?:

66. Cul de los siguientes frmacos NO es til en el tratamiento 1) Temblor de reposo.


de la enfermedad de Huntington?: 2) Bradicinesia.
3) Reflejos miotticos normales.
1) Fluoxetina. 4) Conservacin de funciones corticales.
2) Reserpina. 5) Cadas frecuentes.
Preguntas TEST

3) Haloperidol.
4) Bromocriptina. 72. Con respecto a la enfermedad de Parkinson idioptica, seale
5) Tetrabenacina. la respuesta INCORRECTA:

67. Paciente de 71 aos remitido a consulta de neurologa por 1) El marcador anatomopatolgico ms caracterstico son
cambios de comportamiento de instauracin progresiva en los cuerpos de Lewy, que se encuentran especialmente en
pocos meses, junto a alucinaciones visuales ocasionales y el tronco del encfalo.
cadas al suelo en varias ocasiones, sin prdida del nivel de 2) El temblor de reposo caracterstico se asocia en numerosas
conciencia. En la exploracin se aprecia hipomimia facial, ocasiones a un temblor postural.
aumento del tono muscular de forma global, con resistencia 3) Los cambios en la personalidad son frecuentes en las fases
continua a los movimientos pasivos, reflejos presentes algo evolucionadas de la enfermedad.
hipoactivos y simtricos. No se objetiva movimientos involun- 4) A la exploracin es frecuente encontrar una hipertona
tarios ni temblor de reposo. Ante estos signos, seale cul de elstica.
los siguientes diagnsticos le parece ms probable: 5) En el tratamiento farmacolgico no se incluyen los anta-
gonistas dopaminrgicos.
1) Enfermedad de Parkinson.
2) Enfermedad de Huntington (variante rgida o enfermedad 73. Seale cul de los siguientes neurotransmisores se encuentra
de Westphal). ms alterado en la enfermedad de Parkinson:
M exico A rgentina CTO Medicina C/ Nez de Balboa, 115 28006 MADRID (Espaa) Tfno.: (91) 782 43 32 / Fax: (91) 782 43 27
C hile U ruguay E-mail: secretaria@ctomedicina.com; iberocto@ctomedicina.com WEB: www.ctomedicina.com; www.iberocto.com
NR Pg. 7
NEUROLOGA
Seguimiento a distancia Preparacin Examen de Seleccin 05/06 1 Vuelta
1) Noradrenalina. 1) Deterioro cognitivo.
2) Dopamina. 2) Miocardiopata.
3) Serotonina. 3) Trastornos de conduccin cardaca.
4) Acetilcolina. 4) Malformaciones esquelticas.
5) Somatostatina. 5) Diabetes mellitus.

74. Seale cul de los siguientes frmacos NO es til en el 79. Sobre la evolucin de la ELA, seale la respuesta FALSA:
tratamiento de la enfermedad de Parkinson:
1) Cursa con afectacin de primera y segunda motoneuro-
1) Amantadina. nas, de forma asimtrica.
2) Lisuride. 2) En fases iniciales no se suele afectar la musculatura extrao-
3) Bromocriptina. cular.
4) Clorpromacina. 3) En fases avanzadas es comn la afectacin intelectual.
5) Selegilina. 4) La causa ms frecuente de muerte est en relacin con la
debilidad de la musculatura respiratoria.
75. Seale la respuesta FALSA con relacin a los efectos secun- 5) La media de supervivencia desde el inicio de los sntomas
darios del tratamiento a largo plazo con L-dopa en la enfer- se sita en torno a los tres aos.
medad de Parkinson:

1) El fenmeno wearing-off es la reaparicin de sntomas PATOLOGA VASCULAR CEREBRAL.


parkinsonianos por descenso de las concentraciones
plasmticas de L-dopa. 80. Seale cul de los siguientes es el factor de riesgo ms
2) El fenmeno on-off son fluctuaciones motoras que no importante de la patologa cerebrovascular isqumica:
tienen relacin con los niveles plasmticos de L-dopa.
3) El tratamiento del fenmeno wearing-off se basa en man- 1) HTA.
tener concentraciones plasmticas estables. 2) Hipercolesterolemia.
4) El tratamiento del fenmeno on-off se basa en aumentar 3) Tratamiento con anticonceptivos orales.
de forma sostenida las concentraciones plasmticas de L- 4) Tabaquismo.
dopa. 5) Diabetes mellitus.
5) Estos fenmenos no suelen aparecer en las primeras fases
de la enfermedad.
81. Un paciente de 56 aos present hace dos meses un IAM
anteroseptal. Es trado hoy a Urgencias por cuadro brusco de
76. Paciente de 65 aos que presenta oscilaciones en las manos
prdida de fuerza en miembros derechos. La exploracin
cuando realiza determinadas tareas como escribir, usar cu-
revela desviacin conjugada de los ojos a la izquierda y
biertos o afeitarse. Mejora con la ingesta moderada de alcohol.
hemianopsia derecha por amenaza. Se trata de:
La exploracin neurolgica es normal, salvo una discreta
rueda dentada en mano derecha. No presenta temblor de
reposo, limitacin en la amplitud o rapidez de los movimientos, 1) Crisis comicial afectando al rea frontal derecha asocian-
ni trastornos de la marcha. Seale la opcin que le parece do parlisis de Todd postcrtica.
INCORRECTA con respecto al caso clnico descrito: 2) Embolismo cardaco sobre la arteria basilar con lesin
isqumica de la hemiprotuberancia derecha.
1) Con frecuencia se encuentran en la historia antecedentes 3) Embolismo sobre territorio de la arteria cerebral media
familiares de sntomas similares. izquierda.
2) Dada la existencia de rueda dentada, lo ms probable es 4) Embolismo sobre el territorio de la arteria cerebral poste-
que desarrolle un parkinsonismo en los prximos aos. rior izquierda.
3) El tratamiento con beta-bloqueantes puede ser una buena 5) Embolismo sobre el territorio de la arteria cerebral poste-

Preguntas TEST
opcin como terapia sintomtica. rior derecha.
4) Si se encuentra temblor cintico en la exploracin, habr
que descartar patologa de fosa posterior. 82. Cul de los siguientes signos y sntomas NO esperara
5) Hay que descartar que el paciente est siguiendo algn encontrar en un infarto de la arteria cerebral anterior?:
tratamiento que pueda justificar la sintomatologa.
1) Hemianopsia homnima contralateral.
77. En relacin con la enfermedad de Friedreich, seale la res- 2) Abulia.
puesta INCORRECTA: 3) Desinhibicin o moria.
4) Hemiparesia contralateral de predominio crural.
1) Es la ataxia hereditaria ms frecuente. 5) Apraxia de la marcha en lesiones bilaterales.
2) Los sntomas comienzan antes de los 25 aos de edad.
3) Suele haber hipo-arreflexia generalizada. 83. Paciente de 58 aos que acude a Urgencias por prdida de
4) Las respuestas cutneo plantares son extensoras. visin en el hemicampo derecho, de instauracin brusca. En
5) La principal causa de muerte son las infecciones respira- la exploracin no se aprecia dficit motor y la comprensin
torias de repeticin. y expresin del lenguaje estn conservadas. Destaca una
hipoestesia hemicorporal derecha que incluye hemicara.
78. Seale cul de las siguientes patologas NO se asocia con la Con respecto al diagnstico ms probable de este enfermo,
enfermedad de Friedreich: seale la respuesta correcta:

M exico A rgentina CTO Medicina C/ Nez de Balboa, 115 28006 MADRID (Espaa) Tfno.: (91) 782 43 32 / Fax: (91) 782 43 27
Pg. 8 NR C hile U ruguay E-mail: secretaria@ctomedicina.com; iberocto@ctomedicina.com WEB: www.ctomedicina.com; www.iberocto.com
NEUROLOGA
Preparacin Examen de Seleccin 05/06 1 Vuelta Seguimiento a distancia

1) Es preciso la realizacin de una RM cerebral ante la 87. La localizacin ms frecuente de los hematomas intraparen-
sospecha de patologa de fosa posterior. quimatosos espontneos de causa hipertensiva es:
2) Presenta probablemente isquemia en territorio de arteria
cerebral media izquierda, de ramas parietales. 1) Putamen.
3) Los datos clnicos sugieren afectacin en el territorio de la 2) Cpsula interna.
arteria cerebral posterior izquierda. 3) Tlamo.
4) Si la prueba de imagen cerebral urgente es normal, lo ms 4) Hemisferios cerebrales.
probable es que se trate de un estatus epilptico parcial 5) Hemisferios cerebelosos.
5) Es urgente descartar patologa ateromatosa carotdea.
88. La TC craneal de un paciente muestra un hematoma talmico
84. Paciente de 76 aos que sufre de forma brusca dificultad para de 1,5 cm de dimetro. Seale la causa ms probable de esta
sostener cosas con la mano izquierda, asociando dificultad patologa:
para hablar. Entre sus antecedentes personales destacan:
hipercolesterolemia leve en tratamiento diettico, HTA de 1) Angiopata amiloide.
difcil control teraputico y sustitucin de la cabeza femoral 2) Malformacin vascular.
izquierda por prtesis hace 5 aos. A la exploracin presenta 3) Rotura de aneurisma sacular.
desviacin de la comisura bucal hacia la derecha con capa- 4) Hipertensin.
cidad conservada para cerrar el ojo izquierdo. La explora- 5) Rotura de aneurisma fusiforme.
cin sensitiva es normal. Seale el diagnstico ms probable:
89. Seale cul es la causa ms probable de isquemia en el
1) Hemorragia intraparenquimatosa corticosubcortical de territorio de la arteria cerebral media:
localizacin temporoparietal derecha.
2) Lesin isqumica en territorio de arteria cerebral posterior 1) Aterotrombosis de grandes vasos intracraneales.
izquierda. 2) Aterotrombosis de grandes vasos extracraneales.
3) Infarto lacunar en ganglios basales derechos. 3) Embolia cardiognica.
4) Infarto lacunar en cpsula interna derecha. 4) Embolia paradjica a travs de un foramen oval permea-
5) Infarto isqumico carotdeo derecho extenso. ble.
5) Trombosis in situ.
85. En cuanto al uso de la tomografa computadorizada para el
diagnstico de la patologa cerebrovascular aguda, seale la
respuesta FALSA: SISTEMA NERVIOSO PERIFRICO.

1) Permite detectar infartos lacunares. 90. En el diagnstico diferencial de las neuropatas, seale el
2) Una TC craneal urgente normal puede aparecer en fases enunciado correcto:
muy precoces de las hemorragias intraparenquimatosas.
3) Algunos infartos lacunares pueden no ser visibles. 1) La disminucin en la amplitud de los potenciales de
4) En fase aguda, la isquemia cerebral puede no ser visible. accin es tpica de la afectacin axonal.
5) reas hipodensas en territorios frontera sugieren origen 2) La dispersin de los potenciales de accin es caracterstica
hemodinmico. de la afectacin axonal.
3) La velocidad de conduccin se encuentra disminuida en
86. Paciente de 58 aos que acude por presentar episodio de las neuropatas axonales y en las desmielinizantes.
prdida de visin en ojo izquierdo, de inicio en campo 4) Los bloqueos de conduccin son sugerentes de lesin
superior, de duracin total de unos dos minutos. Fondo de axonal.
ojo: sin hallazgos. Refiere haber presentado das antes episo- 5) No existe lesin axonal y desmielinizante concomitante-
dio de escasos minutos de duracin de hormigueos en la mente en un mismo nervio.
Preguntas TEST

mano derecha, que le cedieron con un masaje. A la explora-


cin, auscultacin cardaca rtmica sin soplos. Cartidas 91. Seale cul de las siguientes neuropatas perifricas NO se ha
rtmicas y simtricas, sin soplos. Seale la respuesta INCO- relacionado con la infeccin por VIH:
RRECTA:
1) Polineuropata desmielinizante aguda.
1) La primera prueba diagnstica a realizar es un estudio eco- 2) Neuropata sensitiva distal.
Doppler de troncos supraarticos. 3) Mononeuritis mltiple.
2) Si se aprecia estenosis en el eco-Doppler, se debe realizar 4) Neuropata amiotrfica proximal.
angiografa posteriormente. 5) Polirradiculitis.
3) Si en el estudio angiogrfico se aprecia estenosis de menos
del 30% en la cartida sintomtica, el tratamiento de 92. Seale el enunciado INCORRECTO con respecto a las neuro-
eleccin es antiagregacin. patas diabticas:
4) Si se detecta estenosis entre el 30 y el 50% de la cartida
sintomtica, el tratamiento se realiza con anticoagulacin 1) La ms frecuente es la polineuropata distal simtrica.
oral. 2) La mononeuropata craneal ms frecuente es el III par
5) En el caso de encontrarse una estenosis de ms del 70 % incompleto.
en la cartida sintomtica, hay que plantearse la posibi- 3) Sntomas como la hipotensin ortosttica, la impotencia o
lidad quirrgica. las diarreas son sugerentes de neuropata vegetativa.

M exico A rgentina CTO Medicina C/ Nez de Balboa, 115 28006 MADRID (Espaa) Tfno.: (91) 782 43 32 / Fax: (91) 782 43 27
C hile U ruguay E-mail: secretaria@ctomedicina.com; iberocto@ctomedicina.com WEB: www.ctomedicina.com; www.iberocto.com
NR Pg. 9
NEUROLOGA
Seguimiento a distancia Preparacin Examen de Seleccin 05/06 1 Vuelta
4) Son potencialmente reversibles si se tratan de forma agresiva. 5) Asocia cataratas subcapsulares, cardiopata con bloqueos
5) La amiotrofia diabtica es de predominio proximal en AV y deterioro intelectual a lo largo de la evolucin.
cintura plvica.
97. La miastenia gravis se caracteriza por:
93. Sobre el sndrome de Guillain-Barr, NO es cierto:
1) Dficit motor de predominio distal en extremidades.
1) Es la forma ms frecuente de polineuropata desmielini- 2) Alteraciones sensitivas, aunque poco intensas.
zante aguda. 3) Frecuente compromiso de la reactividad pupilar por
2) Puede tener alguna sintomatologa sensitiva. afectacin eferente.
3) Se ha asociado a la infeccin previa por Campylobacter 4) Reflejos de estiramiento muscular conservados.
jejuni. 5) Disfuncin autonmica asociada.
4) El diagnstico puede establecerse por la clnica y el estudio
electrofisiolgico. 98. Seale el enunciado FALSO sobre la miastenia gravis:
5) El hallazgo de un LCR normal en fases iniciales descarta la
enfermedad. 1) La lesin est mediada por anticuerpos.
2) Los anticuerpos actan a nivel presinptico.
94. Un paciente presenta clnica de debilidad en miembros 3) Se produce un aplanamiento de los pliegues postsinpti-
inferiores de instauracin progresiva en el curso de escasos cos.
das tras un proceso gastroentertico. En la exploracin 4) El 10% se asocia a timoma.
destaca la presencia de debilidad en extremidades, ms 5) La musculatura craneofacial es la ms frecuentemente
acusada en miembros inferiores, abolicin generalizada de afectada.
reflejos miotticos y conservacin de la sensibilidad. Con
respecto al tratamiento ms adecuado de la patologa que 99. Es INCORRECTO en relacin con la miastenia gravis:
probablemente presenta este paciente, seale la respuesta
INCORRECTA: 1) La aparicin de ptosis y debilidad fluctuantes a lo largo del
da deben hacer sospechar el diagnstico.
1) El tratamiento con corticoides no est indicado. 2) La clnica y los hallazgos neurofisiolgicos pueden ser
2) La administracin de gammaglobulina policlonal intrave- suficientes para establecer el diagnstico.
nosa es til en fases iniciales. 3) Es preciso la realizacin de una RX de trax para confirmar
3) La plasmafresis es til en fases iniciales. el diagnstico.
4) El suero antitoxina puede bloquear la extensin de la 4) La alteracin tmica ms frecuentemente asociada es la
enfermedad. hiperplasia.
5) Lo que garantiza la supervivencia del paciente es el man- 5) En las formas oculares puras el tratamiento de eleccin son
tenimiento de las funciones vitales a medio plazo. los anticolinestersicos.

95. En la enfermedad de Duchenne, NO es cierto: 100. En relacin con el botulismo, es correcto:

1) Se hereda con carcter recesivo ligado a X. 1) En lactantes se produce tpicamente por la ingesta de la
2) Afecta casi exclusivamente a varones. toxina preformada.
3) El gen defectuoso se encuentra en el brazo corto del 2) En adultos, la ingesta de esporas es la causa ms frecuente.
cromosoma X (Xp21). 3) El debut del cuadro clnico es ms rpido en el causado
4) Aparece en la segunda dcada de la vida. por heridas contaminadas.
5) La miocardiopata se presenta en un alto porcentaje de 4) Pupilas puntiformes reactivas.
pacientes. 5) Es frecuente la presencia de oftalmoplejia.

96. Mujer de 29 aos que presenta de forma progresiva, debili-

Preguntas TEST
dad en la musculatura facial, claudicacin mandibular y
debilidad en ambas manos. No presenta dificultad para la
marcha ni para subir escaleras. La paciente refiere que tras
un movimiento, los msculos se le quedan contrados, tar-
dando varios segundos en relajarse; este fenmeno le des-
aparece tras unos minutos de ejercicio continuado y le
reaparece por las maanas al levantarse. Con respecto al
cuadro que probablemente presenta la paciente, seale la
opcin INCORRECTA:

1) Son tpicos los antecedentes familiares.


2) Una biopsia muscular mostrar atrofia, preferentemente
de las fibras tipo I.
3) El tratamiento de eleccin de las miotonas es la fenitona.
4) El proceso patolgico primario es una alteracin de la
sntesis de los receptores de acetilcolina en la membrana
muscular.

M exico A rgentina CTO Medicina C/ Nez de Balboa, 115 28006 MADRID (Espaa) Tfno.: (91) 782 43 32 / Fax: (91) 782 43 27
Pg. 10 NR C hile U ruguay E-mail: secretaria@ctomedicina.com; iberocto@ctomedicina.com WEB: www.ctomedicina.com; www.iberocto.com
NEUROLOGA
Preparacin Examen de Seleccin 05/06 1 Vuelta Seguimiento a distancia
Pregunta 1.- R: 4 superior al tronco del encfalo, y contralateral, puesto que an no ha
El rea motora ocupa la circunvolucin prerrolndica o rea 4 de tenido lugar ninguna decusacin, y por eso es la 5 la respuesta correcta.
Brodmann en el lbulo frontal. All se encuentran los cuerpos
neuronales de las primeras motoneuronas encargadas de inervar el Pregunta 3.- R: 4
hemicuerpo contralateral de forma voluntaria. Las lesiones de las pri- Aunque el motivo de la consulta es la diplopa, en realidad es un
meras motoneuronas, si se preservan las segundas, dan lugar a una dato que no es imprescindible para responder correctamente, dado
paresia espstica, con reflejos de estiramiento muscular aumentados que el resto de la exploracin que presenta el paciente indica un
(opcin 2), hipertona elstica, clonus y respuesta cutneo-plantar sndrome cruzado, con afectacin facial izquierda y de extremidades
extensora (signo de Babinski), por lo que la opcin incorrecta es la 4. derechas.
En fases muy precoces de lesin de primera motoneurona los refle- Los sndromes cruzados son la presentacin caracterstica de las
jos estn abolidos (opcin 1), sobre todo en lesiones de la va piramidal lesiones del tronco del encfalo, donde se lesiona la va piramidal
(mdula), y tardan das o semanas en exaltarse. antes de decusarse en el bulbo, y los ncleos motores del tronco del
En fases avanzadas puede encontrarse disminucin de la masa encfalo.
muscular (opcin 5), dependiente del grado de prdida de fuerza, Ante la presencia de un sndrome cruzado, la localizacin de la
condicionada por la menor actividad muscular. No sucede igual en lesin viene definida por la afectacin craneofacial, tanto para el
las lesiones de la segunda motoneurona, en las que la amiotrofia es nivel lesional como para la lateralidad. La lesin es homolateral a la
precoz y ms intensa. cara afectada, en este caso izquierda, y el nivel lo indica el par
Los reflejos de estiramiento muscular o miotticos se desencade- craneal afectado, en este caso el VII, que se sita en la protuberan-
nan por una conexin monosinptica (entre una neurona sensitiva y cia, con lo que tenemos suficientes datos para dar con la opcin
una segunda motoneurona), mientras que los reflejos cutneos suelen correcta.
ser polisinpticos, y no se comportan de la misma manera que los Las diplopas que se describen como una limitacin en la motili-
anteriores. Los reflejos cutneos abdominales reflejan la respuesta dad del globo ocular en una direccin concreta apuntan a lesin
medular al estmulo en la piel abdominal, con una contraccin de la neuromuscular de un msculo concreto de los encargados de mover
pared del abdomen; se dividen en superiores (D6-9), medios (D9-11) el globo ocular (lo que se denomina motilidad ocular extrnseca o
e inferiores (D11-L1) y suelen abolirse en lesiones de las primeras extraocular). El III par se encarga de la motilidad vertical superior e
motoneuronas, especialmente en las lesiones medulares. inferior del ojo, y del movimiento de adduccin (hacia la lnea me-
dia); el IV par contribuye a un movimiento similar al de mirar a la
punta de la nariz (infraversin y rotacin interna), y el VI par realiza
Pregunta 1. Diagnstico diferencial de lesin de 1 y 2 motoneurona. una accin de abduccin casi pura. Si el paciente tiene limitacin en
la mirada externa con el ojo izquierdo, el origen ms probable es la
1234565789
57 234565789
57 afectacin del VI par izquierdo, que en el caso concreto de la pregun-
7 4678 574 678683 67 ta, apunta a una lesin nuclear en la protuberancia izquierda, junto al
123456789
36782 7 4
7 9
36782 7 4
7 ncleo del VII par izquierdo y la va corticoespinal.

68 256
65 8684836
87 76 Pregunta 4.- R: 4
1256
65 85236
  Ante la presencia de un sndrome cruzado, sea ste motor o sensi-
53
6 873 75 5  tivo, la lesin estar situada en el tronco del encfalo. En el caso de los
467853
567 4678536 567868 77 sndromes cruzados sensitivos, ser una afectacin baja, ya que la
porcin central del trigmino (el ncleo sensitivo central del trigmi-
73 7 8  634 
73 7 8  634 
84!6
no) se sita en su mayor parte en el bulbo y en la parte ms inferior de
!76
8" #57$5 la protuberancia. Dado que entre las respuestas no est la opcin de
68 7 5 4 567 % 8!575
8 7 5 4 567 la protuberancia, la respuesta no puede ser ms que la 4.
Hay que recordar, con respecto a los sndromes sensitivos, que una
lesin parietal no produce anestesia hemicorporal, a pesar de encon-
Pregunta 2.- R: 5 trarse all la corteza sensitiva primaria, sino trastornos en las sensibili-
Cuando un paciente presenta un dficit motor del que no sabe- dades combinadas, esto es, en el reconocimiento adecuado de los
Comentarios TEST

mos sus caractersticas exploratorias (reflejos, tono, etc.) y debemos estmulos sensitivos (agnosias sensitivas).
basarnos exclusivamente en la distribucin, hay que recordar los dis- Las lesiones talmicas s producen anestesia hemicorporal completa,
tintos patrones: que incluye la hemicara, y que si se establece de forma definitiva,
Lesiones medulares: no afectan a la cara, y el paciente presenta suele acompaarse de episodios lancinantes de dolor espontneo en
una hemiparesia ipsilateral a la lesin. Aunque existe una parte de el hemicuerpo anestesiado (dolor talmico), y de movimientos
axones de primeras motoneuronas que descienden por el cordn pseudoatetoides de la mano afectada, probablemente en relacin
anterior y se decusan en la mdula, la mayor parte de la va corti- con la desaferentizacin sensitiva.
coespinal se decusa en el bulbo y se sita en el cordn lateral Por lo que respecta a las lesiones bulbares, como es el caso de la
ipsilateral al hemicuerpo que va a inervar. pregunta, hay que recordar que en el bulbo la va espinotalmica,
Lesiones troncoenceflicas: tpicamente dan lugar a los sndromes portadora de la informacin del dolor y de la temperatura, discurre
cruzados, en los que aparece sintomatologa en un hemicuerpo y por la porcin lateral, mientras que la informacin posicional y vibra-
en la hemicara opuesta. La afectacin de las extremidades se pro- toria que asciende por los cordones posteriores, se sita medial. Esta
duce por la lesin de la va corticoespinal en su trayecto por el disposicin anatmica hace que en la lesin bulbar lateral se vea
tronco del encfalo, an sin decusar, por lo que dan lugar a defec- afectada la sensibilidad termoalgsica y no la propioceptiva, y en el
tos contralaterales, mientras que la afectacin facial se produce sndrome bulbar medial suceda al contrario.
por lesin de los ncleos motores, es decir, las segundas
motoneuronas, que se sitan en el mismo lado del msculo al que Pregunta 5.- R: 2
inervan, por lo que producen defectos ipsilaterales. En los sndromes El equilibro en el dimetro pupilar viene determinado por la ac-
cruzados, la hemicara indica el lado de la lesin. cin permanente del estmulo parasimptico, que produce constric-
Lesiones por encima del tronco del encfalo (cpsula interna, cor- cin, y el estmulo simptico, que provoca dilatacin. La presencia,
teza frontal): dan lugar a defectos hemicorporales contralaterales por tanto, de unas pupilas medias, es diagnstica de un equilibrio
que incluyen la hemicara y las extremidades. simptico-parasimptico.
La existencia de una pupila midritica indica lesin de la va para-
En el caso de la pregunta, al tener un defecto en la hemicara, brazo simptica con conservacin de la simptica. Hay que pensar en lesio-
y pierna en el mismo hemicuerpo, sabemos que se trata de una lesin nes a nivel del ncleo parasimptico o de Edinger-Westphal (mesence-

M exico A rgentina CTO Medicina C/ Nez de Balboa, 115 28006 MADRID (Espaa) Tfno.: (91) 782 43 32 / Fax: (91) 782 43 27
C hile U ruguay E-mail: secretaria@ctomedicina.com; iberocto@ctomedicina.com WEB: www.ctomedicina.com; www.iberocto.com
NR Pg. 1
NEUROLOGA
Seguimiento a distancia Preparacin Examen de Seleccin 05/06 1 Vuelta
flico posterior), de la porcin mesenceflica del III par, o del trayecto zar el nivel correspondiente de la segunda motoneurona. Su decu-
del III par (seno cavernoso, rbita). Las lesiones del ncleo pueden tener sacin se produce en el bulbo raqudeo, por lo que cada cordn
afectacin exclusivamente pupilar; las otras se acompaarn de difi- lateral lleva las vas motoras para el hemicuerpo ipsilateral.
cultad en los movimientos del globo ocular y de ptosis. Va espinotalmica: las fibras sensitivas del nervio perifrico
El hallazgo de una pupila mitica acompaada de ptosis es un que recogen las seales del dolor y la temperatura se decusan
sndrome de Horner e indica lesin del simptico cervical. Debido a su en la mdula, con un trayecto ascendente, tras encontrar su
especial disposicin anatmica, las lesiones del simptico pueden segunda neurona sensitiva en el asta posterior medular, y se
producirse en distintos niveles: hipotlamo, tronco del encfalo, m- sitan en la porcin anterior-lateral medular. Por lo tanto, la
dula cervical (columna intermediolateral), vrtice torcico (ganglio va espinotalmica en la mdula conduce informacin sensiti-
simptico superior cervical), cartida interna y rbita. No se relaciona va contralateral.
con la cartida externa, y por eso la opcin falsa es la 2.
Cualquier alteracin de la motilidad pupilar puede estar originada Conociendo las principales vas medulares, no es difcil adivinar la
por el uso de sustancias de aplicacin externa (colirios), con resulta- sintomatologa que presentar un paciente del que conozcamos la
dos que dependern de la naturaleza del frmaco. lesin, o interpretar la distribucin de la misma si lo que se nos descri-
La exploracin de la motilidad pupilar se realiza fundamental- be es cada uno de los defectos neurolgicos.
mente con el reflejo fotomotor, por el cual un estmulo luminoso es
conducido por el nervio ptico-quiasma-cintilla ptica hasta el me-
sencfalo, y sin detenerse en el cuerpo geniculado lateral, como hace Pregunta 7. Sndromes medulares.
el resto de la va ptica, conecta con los dos ncleos de Edinger-
Westphal, produciendo miosis bilateral.
El reflejo de acomodacin es ms complejo: la imagen de un objeto Mielopata
acercndose hacia el ojo llega hasta la corteza occipital, y un impulso transversa
descendente llega hasta los ncleos oculomotores mesenceflicos,
originando miosis, convergencia (los dos pares III realizan adduccin)
y acomodacin del cristalino. En la neurosfilis, este reflejo permane-
Sd. Brown-Sequard
ce conservado mientras que el fotomotor est abolido. Por esta razn
(hemiseccin medular)
puede considerarse correcta la opcin 4, ya que la lesin de cordo-
nes posteriores no genera una pupila de Argyll- Robertson, pero la
presencia de esta alteracin de las pupilas sugiere fuertemente que
estamos ante una neurosfilis, que se acompaar de tabes dorsal.
Sd. medular central
Pregunta 6.- R: 5
La presencia de ptosis y miosis es altamente sugerente de sndrome de
Horner por lesin del simptico cervical. La inervacin simptica se en-
carga de la dilatacin pupilar, de la musculatura tarsal (contribuye a la Sd. medular posterolateral
elevacin del prpado) y de las glndulas sudorparas y los vasos sangu-
neos de la hemicara. El sndrome de Horner completo incluira pues:
Disminucin de hendidura palpebral
Miosis
Enoftalmos (aparente para algunos autores, por paresia del mscu- Sd. cordonal posterior
lo orbitario)
Anhidrosis
Vasodilatacin
Si la lesin es distal a la bifurcacin carotdea, la anhidrosis y la
vasodilatacin no estarn presentes, dado que dicha inervacin Sd. medular anterior
discurre por separado de la pupila a partir de ese punto.
Dado el complejo trayecto del simptico hasta alcanzar la pupila,

Comentarios TEST
en el diagnstico diferencial del sndrome de Horner hay que incluir
las lesiones de la cartida interna (diseccin), del vrtice pulmonar Pregunta 8.- R: 3
(tumores pulmonares), de la mdula cervical (lesiones centromedula- La prdida de sensibilidad trmica y dolorosa nos debe hacer sos-
res) y del tronco del encfalo. En el bulbo la va se sita lateral, por lo pechar una lesin de la va espinotalmica, por la que discurre esta
que se ve afectada en el sndrome de Wallenberg y no en el sndrome sensibilidad. Una lesin de esta va a nivel medular cursara con alte-
bulbar medial. raciones sensitivas termoalgsicas que comenzaran dos segmentos
De las opciones de la pregunta, la que no se relaciona con el por debajo del nivel lesional (debido al trayecto ascendente de los
sndrome de Horner es la neuroles; esta patologa asocia otro tipo de axones que acceden a la va en el interior de la mdula) y que afecta-
alteracin pupilar: la pupila de Argyll-Robertson. ran a toda la parte inferior del hemicuerpo desde el nivel inicial. El
hecho de que se trate de un nivel lesional suspendido (con reas
Pregunta 7.- R: 4 ntegras por encima y por debajo de la lesin) descarta que se trate de
Aunque algunos sndromes medulares son muy caractersticos y una lesin funicular (del cordn).
constantes, lo fundamental es recordar las principales vas y su dispo- La sensibilidad termoalgsica tambin puede verse afectada en el
sicin en la mdula, dado que lesiones traumticas o compresivas nervio perifrico, dado que es recogida por unas fibras independientes
pueden dar sndromes aproximados que no encajen por completo del resto de sensibilidad tctil. Son fibras III y IV, finas, pobremente
en lo que conocemos previamente. mielinizadas algunas, que tras entrar por la raz posterior medular, en-
Cordones posteriores: conducen la sensibilidad posicional, vibra- cuentran su segunda neurona sensitiva en el asta posterior medular.
toria, de presin y tacto hasta el tlamo (pasando por unos ncleos El axn de esta segunda neurona sensitiva de la va se decusa con
en el bulbo raqudeo, donde se encuentran las segundas neuronas un trayecto ascendente, pasando por la comisura anterior medular,
sensitivas, y decusndose para alcanzar el tlamo contralateral). muy cerca del centro de la mdula, hasta alcanzar el cordn anterior
Cada cordn posterior contiene la informacin del hemicuerpo y lateral contralaterales.
ipsilateral. La lesin de las neuronas III y IV perifricas dara lugar a un defecto
Va piramidal: los axones de las primeras motoneuronas descien- con patrn polineuroptico, en guante y calcetn, siendo extrao que
den en su mayor parte por el cordn lateral medular hasta alcan- presente un defecto proximal y no distal, como lo sera entre D3 y D6.

M exico A rgentina CTO Medicina C/ Nez de Balboa, 115 28006 MADRID (Espaa) Tfno.: (91) 782 43 32 / Fax: (91) 782 43 27
Pg. 2 NR C hile U ruguay E-mail: secretaria@ctomedicina.com; iberocto@ctomedicina.com WEB: www.ctomedicina.com; www.iberocto.com
NEUROLOGA
Preparacin Examen de Seleccin 05/06 1 Vuelta Seguimiento a distancia
La clnica aislada de anestesia termoalgsica bilateral y simtrica Pregunta 11.- R: 3
debe hacer sospechar una lesin centromedular. La causa ms fre- El tono muscular viene definido por la resistencia del msculo a la
cuente de lesin centromedular es la Siringomielia, con dilatacin del movilizacin pasiva. Los aumentos del tono muscular (hipertonas)
conducto ependimario y lesin de las vas adyacentes. pueden estar en relacin con:
Espasticidad: presente cuando se pierde la funcin inhibitoria que
Pregunta 9.- R: 1 ejerce la va piramidal sobre la segunda neurona motora. Es mayor
Tenemos un paciente con dficit motor en miembros inferiores, al comienzo del movimiento y en movimientos rpidos; si se ejerce
que se acompaa de reflejos vivos y signo de Babinski, lo que indica una fuerza continua, se produce una relajacin del msculo (fe-
lesin de la va piramidal. El hecho de que en miembros superiores no nmeno de navaja). Se acompaa de reflejos miotticos vivos y
haya asimismo hiperreflexia sugiere que la lesin es medular, dado respuesta cutaneoplantar extensora. Si la espasticidad es muy in-
que si fuera por encima de los segmentos medulares cervicales, lo ms tensa y mantenida puede llegar a dar lugar a una contraccin
probable es que cursara con hiperreflexia generalizada. mantenida (distona espstica).
En miembros superiores se objetivan amiotrofias, lo que sugiere Rigidez: existe un aumento de tono muscular mantenido en muscu-
lesin de segunda motoneurona. latura flexora y extensora, con lo que aparece una resistencia conti-
En un paciente con clnica de afectacin de primeras y segundas nua a la movilizacin pasiva en cualquier direccin. Los reflejos de
motoneuronas hay que considerar la posibilidad de una esclerosis estiramiento muscular son normales y la respuesta cutneo-plantar
lateral amiotrfica (ELA): de curso progresivo en la edad adulta, en es flexora, salvo que coexista una lesin de la va piramidal. En el
este caso no es compatible, dado que la ELA debe cursar sin alteracio- parkinsonismo, la rigidez se acompaa de interrupciones rtmicas de
nes cognitivas, de esfnteres ni sensitivas, y el paciente presenta signo la resistencia (fenmeno de rueda dentada).
de Lhermitte, lo que indica afectacin de los cordones posteriores
medulares a nivel cervical. La disminucin del tono muscular (hipotona) es una prdida del
La degeneracin subaguda combinada de la mdula presenta de- tono normal en la que los msculos aparecen flccidos y blandos,
terioro de cordones laterales y posteriores medulares, de curso ascen- con una resistencia inferior a la normal al movimiento pasivo. Las
dente, motivado por un dficit de vitamina B12. No cursa con afecta- causas ms frecuentes son las lesiones de la segunda motoneurona o
cin de segundas motoneuronas, por lo que la amiotrofia en manos las miopatas. Las lesiones cerebelosas pueden cursar con una
no apoya el diagnstico. hipotona algo ms leve que la descrita anteriormente. El desuso tam-
El sndrome espinal anterior se produce en la mayora de las oca- bin produce hipotona.
siones por una lesin isqumica en el territorio de la arteria espinal
anterior, que irriga los dos tercios anteriores medulares. No afecta los Pregunta 12.- R: 3
cordones posteriores y cursa con una plejia bilateral aguda, de mal La distona es un trastorno del movimiento que se define por
pronstico en lo que respecta a su recuperacin. contracciones musculares sostenidas que producen posturas anor-
La mielopata cervical se produce por una disminucin del espa- males, contorsiones y, en ocasiones, movimientos anormales.
cio en el canal medular cervical, segmento en que la mdula alcanza Se han utilizado distintas clasificaciones, siendo las ms tiles las
su mximo grosor, con lo que se produce una compresin de curso que marcan la edad de comienzo (precoces si lo hacen antes de los 25
lentamente progresivo, afectando a las vas ascendentes y descenden- aos y tardas a las que aparecen ms tarde), la distribucin anatmi-
tes medulares (lo que justifica el Lhermitte cordones posteriores- y la ca (focales, segmentarias, hemidistonas, generalizadas), la causa (pri-
espasticidad en miembros inferiores cordones laterales-) y a las marias, hereditarias, secundarias) y, ms recientemente, las basadas en
neuronas motoras presentes a ese nivel, lo que explica la amiotrofia los defectos genticos asociados.
coincidente con la lesin. El tratamiento de las distonas depende de la distribucin y la in-
tensidad, y va encaminado a la mayor recuperacin funcional posi-
Pregunta 10.- R: 4 ble, ms que a una recuperacin completa. Se dispone de tres escalo-
Las descripciones de los trastornos del movimiento que cursan con nes teraputicos:
exceso de movimiento (hipercinesias) no siempre hacen fcilmente Frmacos: agentes dopaminrgicos (agonistas dopaminrgicos,
identificable el movimiento en cuestin. Unas pistas para enfocar es- levodopa), anticolinrgicos, benzodiacepinas, baclofn, litio, anti-
tos trastornos: epilpticos y un largo etctera.
Las contracciones musculares mantenidas tpicas son las distonas: Toxina botulnica: es la mejor opcin para la distona cervical, el
Comentarios TEST

tienen una instauracin lenta y progresiva en el tiempo, hasta que se blefaroespasmo y la distona espstica. Se inyecta directamente en
establecen como fijas, limitando la actividad funcional del sujeto. el msculo seleccionado, obtenindose un beneficio que persiste
Los movimientos repetitivos estereotipados son los tics: predomi- semanas-meses. No es til en las distonas que afectan a muchos
nan en el rea facial, de inicio tpicamente en la edad infantil. La msculos.
inmensa mayora son simples y benignos, y no precisan tratamien- Ciruga: en las distonas que no responden a los tratamientos
to farmacolgico. descritos arriba, pueden ser subsidiarios de procedimientos qui-
Los movimientos continuos son coreoatetsicos: los atetsicos son rrgicos como la lesin o estimulacin cerebral profunda selec-
movimientos continuos, lentos, que predominan distalmente, aun- cionada en determinados ganglios basales, similar a la que se
que pueden afectar a cualquier rea corporal; los movimientos realiza en los casos de enfermedad de Parkinson
coreicos son rpidos, de amplitud variable, que tambin pueden farmacorresistente.
afectar a cualquier parte del cuerpo y que cuando son generaliza-
dos aparecen como un baile (chorea = baile) Pregunta 13.- R: 2
Las sacudidas musculares, arrtmicas, breves y saltatorias son tpicas El temblor es un trastorno del movimiento definido por oscilacio-
de las mioclonas: pueden tener origen cortical, medular o perif- nes rtmicas debidas a la actividad alternante de la musculatura ago-
rico, y son arrtmicas. Todo el mundo tiene mioclonas hipnaggi- nista y antagonista. Es el trastorno del movimiento ms comn. Puede
cas (al coger el sueo) que suelen coincidir con imgenes onricas clasificarse siguiendo varios criterios (ver figura en pgina siguiente); el
de cada, tropezn, etc. ms objetivo es segn la frecuencia del temblor, pero lo ms til es,
Las oscilaciones rtmicas, por actividad alternante agonista-antago- probablemente, englobarlo dentro de un sndrome clnico:
nista, son el temblor. Los temblores se distinguen en distintas clasi- El temblor de reposo es visible cuando la parte afectada est en
ficaciones, segn la forma de presentacin, la frecuencia de las reposo (con apoyo o sometida a efecto de la gravedad) y hay que
oscilaciones, o las patologas de base asociadas. Todo el mundo pensar en un sndrome parkinsoniano.
tiene un cierto grado de temblor (temblor fisiolgico) que se exa- El temblor postural aparece cuando se intenta mantener una pos-
cerba en situaciones de cansancio, estrs, consumo de estimulan- tura contra gravedad. Se presenta tpicamente en el temblor esen-
tes, etc. cial benigno; es la forma ms frecuente de temblor de causa farma-
colgica; puede aparecer en enfermos de Parkinson asociado al

M exico A rgentina CTO Medicina C/ Nez de Balboa, 115 28006 MADRID (Espaa) Tfno.: (91) 782 43 32 / Fax: (91) 782 43 27
C hile U ruguay E-mail: secretaria@ctomedicina.com; iberocto@ctomedicina.com WEB: www.ctomedicina.com; www.iberocto.com
NR Pg. 3
NEUROLOGA
Seguimiento a distancia Preparacin Examen de Seleccin 05/06 1 Vuelta
temblor de reposo tpico, y es el temblor fisiolgico que todo el Pregunta 15.- R: 3
mundo presentamos. La descripcin de un temblor que slo aparece cuando no se
El temblor cintico aparece en los movimientos con amplio reco- ejerce accin muscular en la parte afectada (dejada caer a favor de
rrido, y es tpico de la patologa cerebelosa y mesenceflica (tem- gravedad, o apoyada) es diagnstica de temblor de reposo, y ste a su
blor rbrico), con la peculiaridad de que en los casos cerebelosos vez conlleva inmediatamente la sospecha de un sndrome parkinso-
es aislado, mientras en las lesiones mesenceflicas coexisten tem- niano.
blor de reposo, postural y cintico en el mismo paciente. El tem- De los sndromes que cursan con rigidez, bradicinesia y temblor de
blor intencional es el empeoramiento o la aparicin del temblor al reposo, el ms frecuente es la Enfermedad de Parkinson. Es, por otra
final de un movimiento, y puede ser una primera sintomatologa parte, en el que predomina esta forma de debut, con un temblor de
de un temblor cintico. reposo, de predominio en manos, asimtrico (en fases iniciales slo
tiembla una mano) e inicialmente sin otros signos o sntomas, con lo
En esta pregunta, con las descripciones de situaciones en las que cual, la mera aparicin de un temblor de reposo en una mano levan-
aparece el temblor, cabe la duda de si se trata de un temblor cintico, ta la sospecha de enfermedad de Parkinson.
dado que hay accin, o un temblor postural, dado que las acciones De los distintos tipos de temblor, el ms frecuente es el temblor
son mantenimiento de posturas casi fijas, con mnimo recorrido. Como postural; si el paciente refiere un temblor postural de instauracin
ayuda, recordar que el temblor cintico e intencional se suele describir lentamente progresiva, en efecto hay que preguntar por los antece-
en maniobras de exploracin de ataxia (dedo-nariz, taln-rodilla) mien- dentes familiares, ya que la sospecha sera un temblor esencial, que es
tras que el temblor postural se describe en posiciones fijas (manos ade- en la mayora de los casos familiar, con distintos tipos de herencia (la
lante) o con mnimo recorrido (escribir, usar cubiertos, servir agua). ms frecuente, autosmica dominante con alta penetrancia).
En pacientes con antecedente de alcoholismo se pueden encon-
Pregunta 14.- R: 1 trar temblores de tipo cintico por lesin cerebelosa crnica, aunque
Dentro de los movimientos involuntarios, los que vienen definidos es ms caracterstica la atrofia vermiana, y temblores de carcter postural,
como sacudidas musculares deben hacernos pensar en mioclonas, en fases de deprivacin alcohlica.
ms si se especifica que son rpidas y arrtmicas. Slo el hecho de que Todos los temblores posturales se acentan en situaciones de in-
se especifique que aparecen periodos silentes (o de silencio elctrico, es quietud emocional y mejoran con la tranquilidad, independiente-
decir, que no hay actividad muscular en determinados momentos) en mente de cul sea su origen.
el electromiograma, nos da el diagnstico sindrmico de asterixis.
La asterixis se explora pidiendo al paciente que mantenga una Pregunta 16.- R: 5
posicin fija contra gravedad (generalmente las manos en dorsifle- Las lesiones cerebelosas cursan con ataxia, es decir, mala organiza-
xin), observndose las sacudidas porque es incapaz de mantener la cin de los movimientos. Aparecen errores en la amplitud, la direc-
postura. cin, la fuerza y la frecuencia empleada en los distintos movimientos.
Inicialmente descrita en relacin con la insuficiencia heptica, Entran en accin distintos componentes del movimiento en el mo-
denominndose flapping, en realidad puede aparecer en el contexto mento equivocado, y las correcciones que se hacen suelen ser exce-
clnico de otras encefalopatas txico-metablicas, aunque ocasio- sivas, por lo que el movimiento resultante resulta irregular, pendular y
nalmente puede presentarse en pacientes con lesiones hemisfricas espasmdico (asinergia).
cerebrales. Una lesin cerebelosa puede presentar un temblor que no es de
La encefalopata hipoglucmica caractersticamente tiene una fase reposo (postural o cintico) que caractersticamente se acenta al
inicial, marcada por el aumento de catecolaminas circulantes, con un final (temblor intencional); parece relacionarse con la lesin de vas
cuadro de inquietud psicomotora, taquicardia, sudoracin y midriasis dentatorrubrotalmicas, por lo que la lesin de las mismas a nivel
que precede a una disminucin del nivel de conciencia y entrada mesenceflico, cerca del ncleo rojo, puede cursar con un temblor
progresiva en coma hipoglucmico, si no se corrige a tiempo. muy similar (temblor rbrico) que se diferencia del cerebeloso por

Comentarios TEST

Prenguta 13. Clasificacin y tratamiento del temblor

M exico A rgentina CTO Medicina C/ Nez de Balboa, 115 28006 MADRID (Espaa) Tfno.: (91) 782 43 32 / Fax: (91) 782 43 27
Pg. 4 NR C hile U ruguay E-mail: secretaria@ctomedicina.com; iberocto@ctomedicina.com WEB: www.ctomedicina.com; www.iberocto.com
NEUROLOGA
Preparacin Examen de Seleccin 05/06 1 Vuelta Seguimiento a distancia
asociar un componente de temblor de reposo del que aqul carece. acompaar de vrtigo, con sensacin de giro de objetos. El Romberg
En reposo no se pueden objetivar alteraciones del movimiento; en ser positivo hacia el mismo lado de la lateralizacin de la marcha.
la valoracin del tono muscular pueden no encontrarse signos pato-
lgicos, a lo sumo una disminucin del tono muscular que es leve, Pregunta 18.- R: 2
menos intensa de la que presentan las lesiones de segunda Las lesiones en el lbulo occipital pueden dar lugar a distintos tipos
motoneurona; esto justifica que la respuesta falsa sea la 5. de clnica segn la localizacin exacta de la lesin:
Hay que recordar que la marcha atxica por lesin cerebelosa se Lesiones de corteza visual primaria: son exclusivamente visuales;
caracteriza por una aparente debilidad en miembros inferiores: la pueden cursar con cuadrantanopsias o hemianopsia; si son bilate-
extremidad que avanza lo hace con movimientos errticos y al apoyar rales, con visin en tnel (hemianopsia con respeto macular bila-
puede golpear en el suelo, con lo que puede resultar difcil diferen- teral) o con ceguera cortical, que se acompaa habitualmente de
ciar esta marcha de la que se presenta en el dficit de sensibilidad alucinaciones y de anosognosia (el paciente niega que no ve).
profunda. El paciente con lesin cerebelosa presenta mayor dificul- Lesiones de reas asociativas visuales: incluyen sndrome visuales
tad al intentar caminar con ojos cerrados; esto no es un signo de complejos con alteraciones en la percepcin o nominacin de los
Romberg positivo, ya que slo debe considerarse como tal a la inesta- colores, prosopagnosia con dificultad para reconocer las caras, difi-
bilidad que aparece al cerrar los ojos y desaparece al abrirlos. cultad para mantener la mirada fija en un punto (apraxia ptica),
dificultad para ver objetos de forma simultnea (simultanagnosia).
Pregunta 17.- R: 3 Las lesiones occipitales que afectan en parte a la porcin posterior
Ante la descripcin de un paciente con marcha atxica tenemos del cuerpo calloso, pueden originar sndromes de desconexin: el
que establecer el diagnstico diferencial bsicamente entre unos cua- paciente puede ser capaz de escribir pero no de leer (alexia sin
dros concretos: agrafia, asociada a lesiones occipitales izquierdas), de ver los obje-
Ataxia sensitiva: por prdida de la sensibilidad posicional; tpica- tos pero no de cogerlos con la mano (desconexin visuomanual) o
mente en relacin con lesin de cordones posteriores medulares. incapaz de dar nombre a un objeto o a un color que se le muestra
Considerarla si se refieren trastornos sensitivos asociados (sobre todo de forma visual (por desconexin del rea visual con el rea sensi-
de sensibilidad posicional, tctil profunda o vibratoria), en pacien- tiva del lenguaje).
tes con alteraciones en la vitamina B12 (pensando en una degene- La astereognosia o incapacidad para reconocer objetos por medio
racin subaguda combinada de la mdula: operados de estma- del tacto es tpico de lesiones de la corteza parietal.
go, iletis terminal, anemia macroctica o megaloblstica), antece-
dentes de sfilis (tabes dorsal). Cursa con Romberg positivo, y con Pregunta 19.- R: 2
ojos abiertos la marcha no tiene claras lateralizaciones, sino slo Conviene tener claros algunos conceptos en cuanto al campo vi-
problemas con el final del movimiento: los pies golpean fuerte- sual:
mente en el suelo (marcha taloneante). El caso de la pregunta po- La descripcin del dficit visual siempre hace referencia al rea
dra serlo si no es por la descripcin de bipedestacin inestable que el paciente deja de ver en su campo visual; esto supone que el
(que no hace coincidir con el cierre ocular, para pensar en un defecto que lo origina ser siempre contralateral (lesiones derechas
Romberg) y los pasos desiguales. dficit izquierdo y al revs) e invertido (lesiones inferiores dficit
Ataxia cerebelosa: es una marcha insegura, con pasos desiguales, superior y al revs).
inestable con ojos abiertos y cerrados (aunque algo peor con ojos La homonimia y la heteronimia hacen referencia a si el defecto en el
cerrados). Si se acompaa de signos atxicos de extremidades, pen- campo visual coincide o no en ambos ojos en la misma parte del
saremos en una lesin cerebelosa completa, como podra generar campo; en el caso de la pregunta, describe un paciente que si
una lesin ocupante de espacio en fosa posterior, pero si no se cierra el ojo izquierdo tiene prdida de visin superior izquierda y
encuentran ms signos cerebelosos que en la bipedestacin y en la si cierra el ojo derecho tiene prdida de visin superior izquierda:
marcha, hay que pensar en una atrofia aislada de vermis. defecto homnimo. Si al cerrar un ojo el defecto es superior iz-
Ataxia vestibular: la marcha es inestable, pero aparece lateraliza- quierdo y al cerrar el ojo contralateral, la prdida de visin est en
cin de forma constante hacia el mismo lado. Generalmente se otro cuadrante, es un defecto heternimo.
Comentarios TEST

Pregunta 19. Defectos campimtricos y lesiones de la va ptica paraquiasmstica

M exico A rgentina CTO Medicina C/ Nez de Balboa, 115 28006 MADRID (Espaa) Tfno.: (91) 782 43 32 / Fax: (91) 782 43 27
C hile U ruguay E-mail: secretaria@ctomedicina.com; iberocto@ctomedicina.com WEB: www.ctomedicina.com; www.iberocto.com
NR Pg. 5
NEUROLOGA
Seguimiento a distancia Preparacin Examen de Seleccin 05/06 1 Vuelta
Los defectos heternimos son tpicos de las lesiones quiasmticas pero la comprensin est conservada, lo que no corresponde con
(o de un paciente con dos lesiones distintas) mientras que los dfi- este diagnstico; tienen, sin embargo importantes alteraciones de la
cits homnimos son siempre retroquiasmticos. repeticin: a esta afasia se la denomina de conduccin, la lesin suele
La congruencia e incongruencia tienen valor en los defectos homni- afectar a la circunvolucin supramarginal, al final de la cisura de
mos. Aunque el dficit visual coincida en ambos ojos, no siempre Silvio.
es idntico: si el rea que se pierde en el campo superior izquierdo En las afasias, los dficits en el lenguaje escrito suelen coincidir con
(en el caso de la pregunta) es mayor en un ojo que en otro, tene- los del lenguaje oral, aunque tienden a ser ms graves.
mos una cuadrantanopsia homnima incongruente; si el defecto
es idntico, es congruente. Las alteraciones campimtricas heter- Pregunta 22.- R: 5
nimas no son superponibles, con lo que hablar de congruencia o La descripcin del caso clnico es la caracterstica de un cuadro
incongruencia no tiene sentido. confusional. Lo que determina fundamentalmente la existencia de
La congruencia entre los campos visuales de ambos ojos se va dicho cuadro es la inatencin, la baja interaccin del paciente con el
ganando de forma progresiva a medida que nos acercamos a la entorno. Puede presentar un cuadro de exceso de actividad, con
corteza occipital, de modo que un defecto plenamente congruen- agitacin, o con cierta disminucin del nivel de conciencia.
te sugiere una lesin cortical; las lesiones de las radiaciones pticas El resto de los sntomas en realidad pueden relacionarse directa-
son menos congruentes que las occipitales, pero ms que las de mente con la dificultad de contacto con el explorador, con lo que la
cintillas pticas. desorientacin, el lenguaje incoherente y la incapacidad para retener
nuevos datos no indican que el paciente tenga un deterioro de fun-
Pregunta 20.- R: 4 ciones superiores de base, sino que est alterada la elaboracin de
La corteza sensitiva del lbulo parietal es la encargada de recibir los procesos mentales, incluyendo la memoria, el lenguaje, el clculo y la
estmulos sensitivos combinados; sera la encargada de la percepcin orientacin.
global de la sensibilidad ms all del dato aislado de temperatura o El cuadro confusional es una clnica comn a una infinidad de
dolor o forma. De ah que una exploracin sensitiva completa deba etiologas, descritas en la tabla, siendo la ms frecuente los trastornos
incluir el anlisis de datos concretos (recepcin de estmulo tctil txico-metablicos.
superficial roces-, tctil profundo toques-, dolor, temperatura, vi- Consideracin aparte merece el delirium, cuya definicin corres-
bracin, posicin) y el anlisis de datos combinados (reconocer un ponde a un cuadro confusional con hiperactividad vegetativa
objeto, reconocer un dibujo hecho sobre la piel, reconocer varios (taquicardia, sudoracin, hipertensin arterial) y cuyo representante
estmulos simultneos), siendo estos ltimos los que suponen una ex- tpico es el delirium tremens por deprivacin alcohlica.
ploracin cortical parietal, y los primeros la informacin sobre las vas
sensitivas.
Otras funciones perceptivas del lbulo parietal hacen que sus le- Pregunta 22. Causas de cuadro confusional
siones, ms all del trastorno concreto del reconocimiento de datos
aislados, puedan conllevar trastornos ms amplios de la percepcin, a 123456478749
9  5 89
589529
2459 9
5 9 9
lo que a veces se le denomina negligencia: el paciente puede no 9  89
123456478749
245 89
reconocer el hemicuerpo afectado como suyo, no hacer caso a est- 123456789
26 8567 768 92826 764 2 7882 86 76 9
mulos que vienen del lado afectado e incluso no ser claramente cons- 222 6 6789
cientes del dficit. Estos sntomas son ms frecuentes en las lesiones no 12 789
2 6786  67892 69689278 67869
dominantes (derechas). 12!696
2769692 76 9278" 929 89289 7# 6789
Por el contrario, cualquier referencia a alteraciones en una sensibi- 12$ 6 8 7% 6 69
lidad concreta (tctil superficial, vibratoria, etc; dolorosa en el caso de 12&978 9 764 22' 2 76 26 766 
la pregunta) nos obliga a pensar en lesiones infracorticales: si se habla 12(8 6 927 929'  7 86 29'' 
de anestesia, hay que pensar en el tlamo en los casos de hemianestesia 12) %  82 8 2782 88* 6 26+'68282687766 
completa o en lesiones medulares en casos de anestesias con un nivel
bilateral. En la opcin 4, que habla de anestesia dolorosa podra- 123456478749
9 96 89
mos pensar en una lesin talmica o en una lesin aislada de la
12,969
sensibilidad termoalgsica (va espinotalmica). 12(68 '76 2   768 92687 8# 67 9
12) 8567 768 9

Comentarios TEST
Pregunta 21.- R: 2
La localizacin del lenguaje dentro del hemisferio izquierdo (en el
que se encuentran las funciones del lenguaje en la mayor parte de la Pregunta 23.- R: 5
poblacin) no es de una certeza anatmica indiscutible. Se sita en El coma es una situacin de mala funcin cerebral global. El nivel
torno a la cisura de Silvio: reas temporales posterosuperiores de conciencia depende del normal funcionamiento de la sustancia
perisilvianas seran el rea posterior del habla y zonas posterobasales reticular activadora (SRA) troncoenceflica y su proyeccin difusa a
del lbulo frontal, junto al lbulo parietal y sobre la cisura de Silvio, el toda la corteza cerebral. Cuando encontramos a un paciente en esta-
rea anterior del lenguaje. Estas reas se extienden ms all de las do de coma hay que pensar en un trastorno difuso generalizado o
clsicas reas de Wernicke y de Broca respectivamente. multifocal de la corteza o una lesin directa de la SRA.
En los pacientes en los que est alterada la produccin del lenguaje, En las lesiones difusas corticales, la tnica es la ausencia de focali-
con una emisin de palabras escasa o nula se habla de afasia no dad neurolgica. Excepcin notable es la hipoglucemia severa en
fluente, e indica lesin en reas anteriores del lenguaje; si el mismo ancianos, que puede cursar con hemiparesia. Suele haber una cierta
paciente conserva la capacidad de comprender rdenes que se le hipotona, puede haber movimientos horizontales pendulares de los
dan, siempre que no tenga que responder verbalmente, indica que la ojos, las funciones y reflejos del tronco cerebral estn conservadas, y
lesin es slo de reas anteriores: afasia motora. el patrn respiratorio es muy variable: puede ser normal, de tipo
En los pacientes en los que la principal alteracin es la comprensin Cheyne-Stokes, con hipoventilacin en algunas intoxicaciones (de-
de las rdenes que se les pide verbalmente que ejecuten, se sospecha presores del SNC) o hiperventilatorio (acidosis).
lesin de reas posteriores del lenguaje; si adems se acompaa de un Las lesiones del tronco del encfalo son muy ricas
lenguaje fluente (indicando que no hay afectacin de reas anteriores semiolgicamente: pueden tener alteraciones pupilares, alteracin o
del lenguaje), tenemos una afasia sensitiva. El lenguaje, a pesar de ser abolicin de los reflejos del tronco (corneales, oculoceflicos, oculo-
fluente, ser ininteligible, con importantes alteraciones semnticas y vestibulares), trastornos del ritmo respiratorio y con frecuencia reac-
sintcticas. cin de descerebracin (extensora) a estmulos dolorosos.
Algunos pacientes, como en el caso de la pregunta, presentan un Aunque todas las lesiones del tronco del encfalo, a cualquier
habla fluida con parafasias, como el paciente con afasia sensitiva, nivel, pueden generar una situacin de coma, el trmino estado

M exico A rgentina CTO Medicina C/ Nez de Balboa, 115 28006 MADRID (Espaa) Tfno.: (91) 782 43 32 / Fax: (91) 782 43 27
Pg. 6 NR C hile U ruguay E-mail: secretaria@ctomedicina.com; iberocto@ctomedicina.com WEB: www.ctomedicina.com; www.iberocto.com
NEUROLOGA
Preparacin Examen de Seleccin 05/06 1 Vuelta Seguimiento a distancia
comatoso, que aunque no muy correcto sugiere un paciente estabi- que tenemos como dato en un paciente en coma, hay que interpre-
lizado en situacin de coma, no es muy compatible con una lesin tarlo literalmente: el paciente respira rpido.
bulbar, dado que un dao a este nivel, en el que ya no hay SRA, lo Qu patrones respiratorios conocemos en el coma?:
que produce es una alteracin primaria del ritmo respiratorio, alta- Respiracin bulbar: atxica, irregular, con paradas de apnea. No
mente inestable, que puede ser mortal en pocas horas. Si la lesin no podemos considerarla taquipneica, aunque haga ciertas fases de
afecta a los ncleos respiratorios, el paciente tendr focalidad defici- respiracin rpida y superficial.
taria sin alteracin del nivel de conciencia, como puede suceder en el Respiracin protuberancial: apnustica, con pausas entre inspira-
Sndrome de Wallenberg. cin y espiracin y la siguiente inspiracin; casi justo lo contrario
de taquipneico.
Pregunta 24.- R: 4 Respiracin dienceflica: de Cheyne-Stokes, hemisfrica profun-
Las respuestas motoras anmalas son movimientos reflejos que da, en la que hay fases de hiperventilacin seguidas de pausas de
aparecen en lesiones del SNC. No siempre permiten la localizacin apnea, que se suceden a intervalos regulares. Tpica de las lesiones
exacta de la lesin que las origina, pero s pueden ser tiles en el hemisfricas bilaterales profundas, o en algunos casos de afecta-
abordaje diagnstico. cin encefaloptica difusa.
La descerebracin o postura extensora cursa con extensin y Respiracin mesenceflica: hiperventilacin rtmica, que s consi-
adduccin de las cuatro extremidades, con hiperpronacin de las deramos taquipneica. Probablemente encontraremos en la explo-
superiores. Aparece en lesiones mesenceflicas o protuberanciales racin otros signos de afectacin mesenceflica, porque si no ha-
altas. bra que pensar en la acidosis como causa (en ese caso, a la respi-
La decorticacin cursa con extensin de miembros inferiores y racin rpida y rtmica relacionada con la acidosis, se le denomina
flexin de miembros superiores, con adduccin; algunos autores ha- respiracin de Kussmaul).
blan de hiperpronacin asociada y otros de supinacin, con lo que
asumimos que el dato ms importante es la flexin, que sirve para Pregunta 27.- R: 5
distinguir la decorticacin de la descerebracin, dado que por lo que Ante un paciente comatoso, la exploracin de las pupilas y su
respecta a los miembros inferiores son iguales. reactividad es especialmente til, ya que se alteran en lesiones del
Si el coma es muy profundo, puede no encontrarse ningn tipo de tronco del encfalo y aportan informacin de dnde puede estar
movimiento, existiendo una hipotona generalizada. En casos en los que situada la lesin. La existencia de pupilas isocricas y normorreactivas
no haya respuestas motoras a los estmulos, pero el nivel de conciencia indica que el tronco cerebral del paciente permanece sin lesin.
est relativamente conservado, hay que sospechar lesiones de las vas
motoras (tpicamente protuberanciales: sndrome de cautiverio).
En el caso de la pregunta, en el que se describe una postura de
descerebracin, esperaramos encontrar una situacin de coma bas-
tante profundo (respuesta 5 falsa) con alteraciones de los reflejos y las
funciones del tronco (falsas la 2 y la 3) y alteraciones en el patrn
respiratorio que no corresponderan a un ritmo de Cheyne-Stokes
como est descrito en la opcin 1.
Una lesin ocupante de espacio en la fosa posterior puede causar
una compresin de tronco de forma directa o indirecta (herniacio- .
nes).

Pregunta 25.- R: 2
Las desviaciones forzadas de la mirada son tpicas de dos localiza-
ciones distintas: la frontal y la protuberancial, no resultando difcil
Pregunta 27. Respuestas pupilares caractersticas de las lesiones enceflicas
distinguir entre ambas:
La lesin frontal cursa con una desviacin oculoceflica hacia el
lado del lbulo frontal lesionado (con una lesin frontal izquierda Ante un paciente que presenta una situacin de coma sin afecta-
el paciente viene mirando hacia la izquierda); lo que suele corres- cin del tronco cerebral, nos queda la posibilidad de una lesin
Comentarios TEST

ponder con el lado contrario de la hemiparesia (una lesin frontal hemisfrica profunda o una afectacin difusa de toda la corteza, lo
izquierda cursa con una hemiparesia derecha). En resumen: el que conocemos como encefalopatas.
paciente con lesin frontal viene mirando hacia el hemicuerpo El hecho de que este paciente presente un patrn respiratorio de
que puede mover. hiperventilacin rtmica nos debe hacer pensar en una encefalopata,
La lesin protuberancial cursa con desviacin ocular (ms que ya que el patrn ms frecuentemente asociado a la lesin hemisfrica
oculoceflica) hacia el lado contrario a la lesin (una lesin protu- profunda es la respiracin de Cheyne-Stokes, que cursa con ritmo
berancial derecha presenta desviacin ocular hacia la izquierda), regular de hiperventilacin-apnea, por aumento de sensibilidad de
lo que suele corresponder con el hemicuerpo deficitario (la lesin los centros inspiratorios a la concentracin de CO2.
de la va piramidal a su paso por la protuberancia genera una La hiperventilacin rtmica es un signo clsico del coma acidtico,
hemiparesia contralateral, ya que se decusar en el bulbo raqudeo: denominndose respiracin de Kussmaul; no es exclusiva de la aci-
una lesin protuberancial derecha cursa con una hemiparesia iz- dosis, pero es la acidosis metablica de cualquier origen la que se
quierda). En resumen: el paciente con lesin protuberancial vie- relaciona tpicamente con ella.
ne mirando hacia el hemicuerpo partico.
Pregunta 28.- R: 4
Por supuesto hay otra serie de datos que nos ayudan a diferenciar La descripcin del cuadro clnico es lo que nos debe llevar la
entre ambas: la lesin protuberancial es probable que deje al paciente mxima atencin:
en coma, mientras que la frontal no tiene por qu alterar el nivel de Incapacidad para realizar cualquier movimiento voluntario salvo
conciencia; la distribucin del dficit generalmente es de predominio parpadeo y movimientos oculares verticales: los movimientos
faciobraquial en las lesiones frontales; las lesiones protuberanciales aso- oculares horizontales conjugados tienen su rea de coordina-
ciarn probablemente alteraciones pupilares; las lesiones frontales pue- cin en la protuberancia, y desde all se dirigen las rdenes a los
den asociar otros dficits (afasia motora en hemisferio dominante). III pares para los movimientos de adducin (por los fascculos
longitudinales mediales), mientras que los movimientos oculares
Pregunta 26.- R: 3 verticales se organizan en el mesencfalo, donde se sitan los III
Lo cierto es que definir una respiracin como taquipneica no es pares. A nivel mesenceflico se origina tambin el movimiento de
sugerente de ninguna lesin en ninguna zona concreta, pero si es lo

M exico A rgentina CTO Medicina C/ Nez de Balboa, 115 28006 MADRID (Espaa) Tfno.: (91) 782 43 32 / Fax: (91) 782 43 27
C hile U ruguay E-mail: secretaria@ctomedicina.com; iberocto@ctomedicina.com WEB: www.ctomedicina.com; www.iberocto.com
NR Pg. 7
NEUROLOGA
Seguimiento a distancia Preparacin Examen de Seleccin 05/06 1 Vuelta
convergencia, en el que, a pesar de ser horizontal, slo intervie- Pregunta 31.- R: 4
nen los terceros pares, con adduccin de ambos ojos El coma hipoglucmico es una encefalopata metablica, con afec-
Estado alerta: esto implica que es capaz de interaccionar con el tacin difusa de la corteza. Es exclusiva de diabticos tratados, dado
entorno; probablemente significa que sigue objetos con la mirada que en un organismo sin patologa, la gluconeognesis impide que se
en el plano vertical, que se produce cierre palpebral ante la aproxi- llegue a esa situacin de forma fisiolgica. El cerebro es un rgano
macin de un objeto al campo visual (reflejo de amenaza) o inclu- selectivo y slo utiliza glucosa como fuente de energa, salvo en con-
so que es capaz de mantener una conversacin si se le da la opor- diciones de ayuno prolongado, en el que se producen los cambios
tunidad de contestar s o no con un determinado nmero de metablicos suficientes para permitir que se consuman cuerpos
parpadeos. Este dato induce a pensar en una conservacin de la cetnicos.
sustancia reticular activadora (SRA) troncoenceflica. Tanto los antidiabticos orales como la insulina pueden producir
Cmo un paciente con un dficit motor que incluye la parte alta una hipoglucemia lo suficientemente mantenida como para producir
de la protuberancia como lmite superior puede no tener lesin de una situacin de coma.
la SRA, que est situada en el tronco del encfalo, es la respuesta De forma tpica, por muy rpidamente que se produzca el descen-
que hay que dar: el sndrome de cautiverio. so de los niveles de glucemia, el cuerpo tiene una primera reaccin de
El sndrome de cautiverio est originado por una lesin desmielini- stress que consiste en la liberacin de catecolaminas; esto lleva a un
zante que afecta a las vas motoras descendentes de la protuberancia, cuadro clnico hiperadrenrgico, con sudoracin, taquicardia, boca
sin afectar al nivel de conciencia. Una RM cerebral puede mostrar seca, midriasis y agitacin, que es seguido de la disminucin del nivel
desmielinizacin difusa protuberancial. de conciencia, generalmente en minutos-horas (dependiendo del rit-
Las lesiones que pueden dar esta clnica pueden ser de mltiples mo de accin de la terapia hipoglucemiante).
etiologas, incluyendo la idioptica, pero el antecedente de una pa- Ante un paciente en coma del que no podemos obtener cifras de
ciente que ha presentado antes un cuadro confusional que ha sido glucemia, se trate o no de un diabtico conocido en tratamiento (pero
atendido mdicamente debe hacer pensar en una hiponatremia co- especialmente si se conoce este antecedente) una de las primeras medi-
rregida demasiado rpidamente, que puede provocar una lesin de das a tomar es la administracin de glucosa por va endovenosa.
sustancia blanca protuberancial.
Pregunta 32.- R: 3
Pregunta 29.- R: 1 La trada de oftalmoplejia externa (alteraciones de la motilidad del
El cerebro es el tejido con mayor sensibilidad a la falta de perfu- globo ocular sin afectacin pupilar o de la acomodacin), ataxia y
sin, comenzando a producirse lesiones irreversibles tras escasos mi- cuadro confusional es la tpica de la encefalopata de Wernicke por
nutos de hipoxia, aunque no todas las zonas del encfalo sufren en la dficit de vitamina B1 (tiamina). La teora dice que la trada se instaura
misma medida. Las reas ms sensibles son las descritas en la opcin en ese orden y que tras la terapia de reposicin el orden de recupera-
3, de tal manera que un paciente que se recupere de forma incom- cin de los sntomas tambin es el mismo: primero la oftalmoparesia,
pleta y le quede alguna secuela tras una hipoxia cerebral es ms posteriormente la ataxia y por ltimo el cuadro confusional.
frecuente que presente ataxia, alteraciones de memoria y trastornos En la prctica lo que nos interesa es no dejar pasar ninguna ence-
motores extrapiramidales. falopata de Wernicke sin sospecharla. Habr que estar atentos a la
La hipoxia cerebral puede producirse por cualquier causa que presencia de signos de malnutricin o de alcoholismo previos a la
impida la llegada de flujo sanguneo oxigenado al cerebro, sea por presentacin de la nueva clnica.
parada cardiaca de cualquier origen, asfixia, o compresin vascular. Ante la sospecha de sndrome de Wernicke est indicada la admi-
Es la posibilidad de sufrir un dao cerebral irreversible, lo que obliga nistracin de 100 mg de tiamina por va parenteral; la disponibilidad
a realizar desde el primer momento de una parada cardiaca manio- de la vitamina por va intramuscular es prcticamente inmediata, no
bras de resucitacin, porque si se recupera flujo pasados 10-15 minu- teniendo en principio claras ventajas la intravenosa, pero s la va
tos, el paciente probablemente permanezca en situacin de coma. parenteral sobre la enteral, dado que estamos hablando con un pa-
El cuadro denominado encefalopata hipxica retardada se pre- ciente malnutrido que probablemente asociar problemas de
senta en pacientes que, tras sufrir el proceso de hipoxia aguda, recu- malabsorcin digestiva.
peran aparentemente sin secuelas, y stas aparecen en los das si- Hay que intentar evitar la administracin de glucosa intravenosa en
guientes, debido a una desmielinizacin de las reas afectadas, que se un paciente con sospecha de Wernicke antes de la tiamina, ya que
instaura de forma subaguda. puede desencadenar o empeorar la encefalopata.
Las reas cerebrales lesionadas no tienen una regeneracin en lo Lo ms indicado en un paciente con dficit nutricional combina-

Comentarios TEST
que respecta a tejido neuronal: se produce una cicatriz glial que do es la administracin de un combinado polivitamnico, ya que la
puede dar lugar a crisis epilpticas tardas. carencia de otras vitaminas tambin puede producir sntomas neuro-
lgicos.
Pregunta 30.- R: 2
Un paciente EPOC con descompensacin respiratoria presentar AMNESIAS. DEMENCIAS.
problemas ms frecuentemente por la retencin excesiva de CO2 que
por la hipoxia, debido a los cambios que se producen en los pacien- Pregunta 33.- R: 3
tes con hipoxia crnica, que permiten tolerar presiones bajas de O2. La memoria es la capacidad para almacenar datos (y rescatarlos
La hipercapnia produce una vasodilatacin cerebral con el consi- cuando es preciso). En principio se puede asumir que es una funcin
guiente aumento de presin intracraneal, con lo que la retencin de repartida de forma difusa por todo el cerebro, aunque hay algunas
CO2 cursar con toda la clnica que acompaa a la hipertensin reas (hipocampo-cerebro basal) cuyas lesiones provocan importan-
intracraneal. En la exploracin se puede encontrar edema de papila tes alteraciones de la memoria.
en toda patologa que aumente la presin intracraneal. Aunque hay distintas clasificaciones, es prctico recordar algunos
El paciente puede presentar una cefalea holocraneal, opresiva no datos bsicos:
especfica, pero quiz sea ms sugerente de hipertensin intracraneal Memoria inmediata: se valora pidiendo al paciente que repita lo
la presencia de vmitos en escopetazo. que se le dice; depende sobre todo de la atencin; en pacientes
La asterixis, aunque clsicamente descrita asociada a la encefalo- con alteraciones importantes del resto de funciones mnsicas, la
pata heptica, puede presentarse asimismo en algunas otras encefa- memoria inmediata se conserva si mantienen un buen nivel de
lopatas, entre ellas la hipercpnica. El paciente tendr alteracin del conciencia.
nivel de conciencia. Memoria reciente: se valora pidiendo al paciente que retenga al-
El temblor de reposo es tpico de los sndromes parkinsonianos. Su gn dato durante varios minutos, mientras se le encarga alguna otra
aparicin es de curso lentamente progresivo; es difcil asociarlo a un tarea, y pidindole que lo repita ms tarde.
cuadro brusco y menos a una descompensacin respiratoria, salvo Memoria remota: se le pide al paciente que aporte datos de hace
que se tenga una patologa previa. aos, biogrficos o histricos.

M exico A rgentina CTO Medicina C/ Nez de Balboa, 115 28006 MADRID (Espaa) Tfno.: (91) 782 43 32 / Fax: (91) 782 43 27
Pg. 8 NR C hile U ruguay E-mail: secretaria@ctomedicina.com; iberocto@ctomedicina.com WEB: www.ctomedicina.com; www.iberocto.com
NEUROLOGA
Preparacin Examen de Seleccin 05/06 1 Vuelta Seguimiento a distancia
Otra clasificacin dividira las amnesias en retrgrada, cuando se sencia de claros dficits neurolgicos focales. El prototipo es la
olvidan datos que previamente se conocan, y antergrada, cuando el enfermedad de Alzheimer.
paciente no es capaz de retener nueva informacin. Subcorticales: hay fundamentalmente un enlentecimiento global
Lo cierto es que actualmente no existe un tratamiento para los dficits de los procesos mentales, sin claras alteraciones corticales (afasia,
aislados de memoria, fuera de los tratamientos globales para enfermeda- apraxia, agnosia) y con fallos de memoria inicialmente menos lla-
des degenerativas que se acompaen de amnesias (demencias). mativos.

Pregunta 34.- R: 2 El enunciado de la pregunta describe una demencia subcortical,


La amnesia global transitoria es lo que presenta el paciente. Hay entre las cuales no se encuentra la enfermedad de Alzheimer, que es,
que tener claro que el cuadro tpico tiene las siguientes caractersticas: por el contrario, la demencia cortical por excelencia (y por frecuen-
Aparicin brusca y duracin menor de 24 horas cia).
El paciente suele repetir las mismas preguntas, pero no olvida quin
es, no est afsico ni confuso, puede realizar acciones complejas Pregunta 36.- R: 4
(vestirse, conducir) sin aparente dificultad La enfermedad de Alzheimer es en efecto la causa ms frecuente
Predomina el defecto amnsico antergrado: el paciente no retie- de demencia en el mundo occidental. Su etiologa es hasta hoy des-
ne datos durante ms de 1-2 minutos; puede presentar alguna conocida, habindose postulado numerosos factores que podran
laguna retrgrada, menos llamativa. influir en su aparicin.
Se desconoce la etiologa de la amnesia global transitoria. Se ha Los hallazgos anatomopatolgicos son los que proporcionan el
postulado origen isqumico (pero el paciente no presenta ms riesgo diagnstico de certeza, por lo que en realidad no se alcanza in vivo.
antes ni despus de ictus que la poblacin de su mismo rango de Pruebas complementarias funcionales, tales como el SPECT, que
edad, lo que no se correspondera con ataque isqumico transitorio) muestren una hipoperfusin frontotemporal pueden apoyar el diag-
o epilptico (pero la duracin de las crisis es menor, el EEG realizado nstico clnico.
durante la crisis no muestra alteraciones que la justifiquen y el pacien- Las lesiones histolgicas bsicas son:
te no tiene otro tipo de crisis epilpticas). La degeneracin neurofibrilar: acmulos de neurofilamentos de-
El factor desencadenante del episodio puede ser muy variable: generados; su componente principal son protenas fosforiladas,
esfuerzos fsicos intensos, estrs emocional, coito, ataque de migraa, ubiquitina y amiloide. La intensidad de esta degeneracin es lo
ingesta de frmacos. que ms parece correlacionarse con el grado de demencia.
Tratamiento no tiene, puesto que se desconoce la etiologa. Lo que Las placas neurticas: acmulo de neuritas distrficas con clulas
s se conoce es el buen pronstico: al paciente le queda como secue- microgliales que con el paso del tiempo tienen un depsito central
la una laguna de memoria correspondiente al tiempo de la crisis, pero de amiloide (placas maduras). Se encuentra numerosa protena
la incidencia de recurrencias es baja. tau.
Depsito de amiloide: el amiloide se forma a partir de una protena
Pregunta 35.- R: 3 codificada en el cromosoma 21; se deposita en el centro de las
Hay que recordar que las apraxias, las agnosias y las afasias son placas y en las paredes de los vasos de pequeo calibre cerebrales.
sndromes clnicos que se presentan nicamente en procesos que Las lesiones anatomopatolgicas bsicas no son patognomnicas
afecten a la corteza hemisfrica, a diferencia de otros sndromes (pr- de la Enfermedad de Alzheimer (opcin falsa), y es slo su hallazgo de
dida de fuerza, disartria, anopsias) que pueden ser clnicamente co- forma difusa y la densidad de las lesiones lo que es diagnstico.
munes pese a poder estar causados por lesiones de distintas localiza- En la Enfermedad de Alzheimer se desarrolla un dficit de todos los
ciones. neurotransmisores, si bien los sistemas colinrgicos parecen los ms
afectados, pero en este caso no slo por la reduccin presinptica de
neurotransmisor, sino tambin por la prdida de receptores.
Pregunta 35. Diagnstico diferencial entre
demencia cortical y subcortical Pregunta 37.- R: 2
Hoy existen en el mercado frmacos comercializados para el trata-
1232456789
5 657 1232456785 657 miento de la Enfermedad de Alzheimer. Todos comparten el mecanis-
mo de accin de ser inhibidores de la acetilcolinesterasa a nivel cere-
424967 123245465728979
4 8932
Comentarios TEST

bral, dado que ha sido el nico mecanismo farmacolgico que ha


 529 85 466
425 5869 demostrado eficacia teraputica in vivo.
Estn indicados en las fases leve y moderada de la enfermedad,
16569 87794724626589472 62 9499 949899 dado que en fases avanzadas el deterioro es tan acusado que una
42
965 65 9 54859 9 5 9 potenciacin de la actividad colinrgica no consigue unos beneficios
25667 9947282  282 48222 tangibles frente a los potenciales efectos secundarios.
Obtienen mejora en todos los aspectos de la enfermedad: el dete-
4 379  98894 78562
54 9354249 2 492
rioro de funciones superiores, las alteraciones del comportamiento y
3  29 82685 8 75 95 la capacidad funcional global del sujeto. La dosis debe aumentarse de
forma creciente, dependiendo de la tolerancia de los efectos secun-
296 429 ! 897544 965 "582#949 5939
darios, que generalmente se producen a nivel digestivo.
Otras terapias no han demostrado eficacia, a pesar de que se ha
La definicin de demencia incluye un deterioro global (con afecta- intentado actuar sobre los distintos mecanismos que en principio in-
cin de distintas reas) de las funciones intelectuales previas, con con- fluyen en la enfermedad: formacin de depsitos amiloides, otros
servacin del nivel de conciencia y en ausencia de una lesin orgnica neurotransmisores, fenmenos oxidativos, etc.
cerebral definida.
Se quedan por tanto fuera de esta definicin los trastornos de algu- Pregunta 38.- R: 1
na funcin intelectual aislada (memoria, afasia, acalculia, etc), los El diagnstico de demencia se establece cuando existe afectacin
defectos en la adquisicin de dichas funciones (oligofrenias), los co- de distintas reas de funcionamiento cerebral, de forma progresiva,
mas y los procesos secundarios (encefalopatas). sin que existan enfermedades subyacentes que puedan influir en ese
Una vez diagnosticado el paciente de demencia, tambin existen proceso, y con conservacin del nivel de conciencia. Si no existe un
varios subtipos: la principal diferencia a recordar est entre las de- buen nivel de conciencia no se puede evaluar la afectacin de las
mencias: funciones superiores, y no es posible conocer si el deterioro simple-
Corticales: el sntoma inicial es la prdida de memoria, y se van mente es transitorio por una causa que, una vez corregida, no deje
desarrollando progresivamente afasia, apraxias y agnosias en au- una alteracin establecida.

M exico A rgentina CTO Medicina C/ Nez de Balboa, 115 28006 MADRID (Espaa) Tfno.: (91) 782 43 32 / Fax: (91) 782 43 27
C hile U ruguay E-mail: secretaria@ctomedicina.com; iberocto@ctomedicina.com WEB: www.ctomedicina.com; www.iberocto.com
NR Pg. 9
NEUROLOGA
Seguimiento a distancia Preparacin Examen de Seleccin 05/06 1 Vuelta
Esto es vlido para todos los tipos de demencia, corticales y subcor- Pregunta 41.- R: 3
ticales. Hay que recordar que los priones son partculas de naturaleza
En el caso de las demencias corticales, principalmente tendremos proteica y que se comportan como agentes infecciosos. Todo el mun-
problemas de memoria, en especial y en fases iniciales se altera ms la do posee la protena precursora de la protena prinica (PrP), de la
memoria reciente o de fijacin, mientras que el paciente an es capaz que an no se conoce claramente su funcin. Su cdigo gentico est
de recordar datos biogrficos antiguos. En fases avanzadas incluso esa en el cromosoma 20.
memoria remota se altera y se pierde definitivamente. En algunas familias, una mutacin en el cromosoma 20 codifica de
Las apraxias pueden describirse como que el paciente tiene difi- forma anmala la protena precursora, con lo que se produce la
cultad para vestirse, para realizar acciones simples que antes saba enfermedad, con debut generalmente en la vida adulta (5 dcada):
(abrir una puerta con una llave, manejar un mando a distancia). Las son la enfermedad de Gerstmann-Straussler-Scheinker, el insomnio
agnosias pueden ser evidentes en la incapacidad para reconocer los familiar fatal y alguna variante familiar de la enfermedad de Creutzfeldt-
rostros de los familiares, u objetos cotidianos (un reloj, una llave). La Jakob.
afasia inicialmente es nominal: el paciente tiene un lenguaje espont- Una modificacin en la estructura terciaria de la protena normal
neo fluente, pero cuando se le pide que nombre alguna cosa es inca- (sin cambios en la codificacin gentica) puede hacer que dicha pro-
paz de extraer ese nombre de su memoria, aunque lo haya usado de tena se agregue en forma de un depsito amiloide resistente a la activi-
forma espontnea. dad de las proteasas, generando escasa actividad inflamatoria y una
destruccin de caractersticas espongiformes. Esta modificacin la pue-
Pregunta 39.- R: 1 de conseguir el contacto con la protena prinica; estas son las variantes
Las demencias corticales se diferencian de las subcorticales en el infecciosas: las variantes adquiridas de la enfermedad de C-J clsica, la
contexto clnico por una mayor alteracin de las funciones superio- nueva variante del CJ y el kuru.
res: memoria, clculo, lenguaje, acciones motoras complejas; mien- El mecanismo por el que la protena consigue llegar hasta el sistema
tras que las demencias subcorticales conservan esas funciones, en el nervioso central y al propio interior de las clulas hasta conseguir la
contexto de un funcionamiento globalmente enlentecido. Hay, no transformacin de las protenas precursoras en protenas prinicas
obstante, muchas situaciones clnicas en las que resulta muy difcil permanece desconocido.
diferenciar entre ambas entidades. La enfermedad de Strachan o neuritis jamaicana es una enferme-
Por lo general, los trastornos denominados extrapiramidales, cuan- dad de carcter adquirido, de origen carencial, aunque sin corres-
do asocian demencia, suele ser de caractersticas subcorticales, y lo ponder a ninguno de los sndromes carenciales clsicos por dficit de
mismo sucede con las demencias de origen vascular. una vitamina (beri-beri, pelagra), que presenta polineuropata (con
La enfermedad de Pick es una demencia muy similar clnicamente dolor en extremidades y abolicin de reflejos), ataxia, disminucin de
a la enfermedad de Alzheimer, en la que predominan las alteraciones visin y audicin, y que responde a la terapia polivitamnica.
de caractersticas frontales: desinhibicin, alteracin de la persona-
lidad previa, lenguaje verborreico, incontinencia de esfnteres. En fa- Pregunta 42.- R: 5
ses evolucionadas se produce un deterioro global. La enfermedad de Binswanger en realidad debera diagnosticarse
En las pruebas de imagen aparece una atrofia selectiva frontal y nicamente post mortem, ya que en su diagnstico incluye que exista
temporal inferior, ms selectiva que en el Alzheimer, sin que se sepa la una desmielinizacin difusa subcortical con ausencia de infartos ex-
razn. La anatoma patolgica es la diagnstica, con la aparicin de tensos o de una gran nmero de infartos lacunares, junto con
neuronas degeneradas (clulas de Pick) o con inclusiones intraneuro- lipohialinosis y esclerosis de arteriolas.
nales de filamentos anmalos, similares a los de la degeneracin Se trata de pacientes con factores de riesgo vascular, bsicamente
neurofibrilar del Alzheimer (grnulos de Pick). hipertensin, que presentan en las pruebas de imagen una alteracin
No existe tratamiento para esta enfermedad. en la densidad o la seal de la sustancia blanca que sugiere
desmielinizacin y que se denomina leucoaraiosis.
Pregunta 40.- R: 4 En la prctica, ante un paciente con deterioro cognitivo subcorti-
La primera premisa para responder la pregunta es descifrar el cua- cal y la presencia de una prueba de imagen (TC o RM) sugerente, se
dro clnico: considera que estamos ante el mismo cuadro, se denomine enferme-
Olvidos frecuentes de forma progresiva en paciente mayor: sugiere dad de Binswanger o no.
demencia incipiente (otra cosa sera un olvido global de forma De modo que un paciente que presente:
brusca: amnesia global transitoria). - deterioro de caractersticas subcorticales con parkinsonismo, rigi-

Comentarios TEST
Trastorno de la marcha: aunque no est especificado el tipo de dez, urgencia miccional, trastornos de la marcha;
trastorno, una demencia con trastorno de la marcha de tipo aprxico - factores de riesgo arteriosclertico (hipertensin arterial, diabetes,
ayudara a pensar en una demencia cortical, mientras que una isquemia coronaria)
marcha a pequeos pasos apunta ms a patologa subcortical. - infartos lacunares previos, y
Sacudidas musculares breves y arrtmicas: definicin de mioclon- - leucoaraiosis bilateral extensa en las pruebas de imagen;
as; desencadenadas por sobresaltos? mioclonas reflejas. queda diagnosticado de encefalopata isqumica subcortical.
No existe un tratamiento que revierta los sntomas, ya que cuando
Una demencia progresiva con mioclonas debe hacer pensar auto- estamos en fase sintomtica, indica que el dao cerebral est estable-
mticamente en una enfermedad de Creutzfeldt-Jakob. De hecho, la cido. El control estricto de los factores de riesgo aspira a prevenir una
trada tpica es una demencia con mioclona y un EEG caracterstico progresin ms acusada.
con las denominadas ondas trifsicas, aunque el cuadro suele com- De todos modos, conviene recordar que la forma ms frecuente de
pletarse con signos cerebelosos, piramidales, extrapiramidales, crisis demencia vascular es la que se produce por infartos mltiples, gene-
epilpticas y alteraciones de segunda neurona motora. ralmente de predominio subcortical, y que en ocasiones no es fcil de
El diagnstico de esta enfermedad se basa en el contexto clnico separar de una enfermedad de Alzheimer. Para distinguir entre ambas
coherente, con el EEG compatible; las pruebas complementarias sue- se ha utilizado la escala isqumica de Hachinski, que valora datos
len ser negativas. En el LCR, que puede mostrar un aumento inespec- clnicos que apoyen ms un tipo u otro de demencia.
fico de las protenas, se detecta la protena 14-3-3, que no es espec-
fica ni sensible al 100%, pero que aparece en la mayor parte de los EPILEPSIA.
casos.
El curso clnico es fatal en el transcurso de 6-12 meses, con muerte Pregunta 43.- R: 3
generalmente producida en relacin con complicaciones respirato- Las crisis parciales son crisis focales elctricamente, aunque algu-
rias. nas puedan tener sntomas que obligan al diagnstico diferencial con
El diagnstico de certeza es el anatomopatolgico, con el hallazgo crisis generalizadas; el ms importante es la desconexin del medio,
de una espongiosis con escasa actividad inflamatoria. que por definicin comparten las crisis generalizadas, y que tambin

M exico A rgentina CTO Medicina C/ Nez de Balboa, 115 28006 MADRID (Espaa) Tfno.: (91) 782 43 32 / Fax: (91) 782 43 27
Pg. 10 NR C hile U ruguay E-mail: secretaria@ctomedicina.com; iberocto@ctomedicina.com WEB: www.ctomedicina.com; www.iberocto.com
NEUROLOGA
Preparacin Examen de Seleccin 05/06 1 Vuelta Seguimiento a distancia
pueden presentar algunas crisis focales. El diagnstico ms difcil est cordar que se tratarn de lesiones con captacin de contraste en
entre las crisis de ausencia con las crisis parciales complejas, que anillo, con edema digitiforme o vasognico alrededor, con importan-
pueden ser clnicamente muy similares. te efecto de masa.
Otro lmite especialmente difcil se sita entre las crisis parciales Ante la sospecha de una lesin ocupante de espacio, est indicada
simples con sntomas psquicos y las crisis con alteracin del nivel de la realizacin de una prueba de imagen. No necesariamente tiene
conciencia, dado que en muchas ocasiones los propios sntomas ps- que ser urgente si lo entendemos como sinnimo de inmediata,
quicos pueden conducir a trastornos en el nivel de alerta, o ser ya pero no puede dilatarse mucho, puesto que pensamos en un tumor.
parte de la crisis parcial compleja. En cualquier caso, a efectos de un La prueba de imagen que ms datos aportar ser la RM, pero una TC
caso terico, se debe especificar que hay alteracin del nivel de con- puede ser tambin diagnstica.
ciencia para diagnosticar de crisis parciales complejas y no simples. El sndrome clnico que presenta el paciente parecen ser crisis
La mayora de las crisis parciales complejas tienen foco en el lbulo parciales tnicas (se detalla que hay alteracin del nivel de concien-
temporal, lo que no implica que todas las crisis que tienen su origen en cia, luego son crisis parciales complejas) y el hecho de que en una
dicho lbulo cursen con alteracin del nivel de conciencia. ocasin se presente un episodio tonicoclnico generalizado simple-
De igual modo, la etiologa subyacente no es determinante en el mente indica extensin de la actividad elctrica anmala a lo largo de
curso clnico de los episodios comiciales, ni el tipo exacto de los toda la corteza.
mismos implica mejor o peor pronstico de la patologa de base.
Pregunta 47.- R: 3
Pregunta 44.- R: 3 Los sntomas que presentan las crisis epilpticas dependen, lgica-
El paciente presenta desconexin del medio, lo que inicialmente mente, de la funcin del rea cerebral en el que se asienta la anomala
nos reduce el diagnstico diferencial, de entre las opciones, a la 3 o la elctrica paroxstica. Podemos tambin, conociendo la sintomatologa,
4. Las crisis generalizadas no convulsivas no existen como tales en las suponer a la inversa dnde situar el foco epilptico.
clasificaciones actuales de las crisis epilpticas, salvo que se les quiera En el lbulo frontal se sita la corteza motora primaria, y por eso
aplicar ese nombre a lo que conocemos como ausencias, y las crisis encontraremos movimientos involuntarios en el hemicuerpo contra-
parciales simples no alteran el nivel de conciencia. lateral; asimismo cada lbulo frontal coordina la desviacin oculoce-
No es sencillo distinguir entre crisis de ausencia y las crisis parciales flica hacia el lado contrario, que se produce cuando se estimula la
complejas que cursan casi como ausencias, y bsicamente nos tiene corteza; por eso el paciente mira al hemicuerpo que tiene los movi-
que hacer descartar la ausencia el hecho de que aparezca algn mientos involuntarios.
sntoma incompatible con la misma. Las parestesias hemicorporales paroxsticas sugieren un foco en
De entre los datos del caso clnico, la duracin es poco sugerente corteza parietal contralateral.
de ausencias, que suelen durar segundos, pero no es un signo incom- Las luces centelleantes paroxsticas en un hemicampo sugieren
patible, como lo son la sensacin epigstrica previa a la crisis y la focalidad occipital; hay que establecer el diagnstico diferencial en-
confusin posterior al episodio. La presencia de estos sntomas nos tre crisis y episodios de aura migraosa, que es ms frecuentemente
hacen descartar el diagnstico de ausencias tpicas; tendramos que occipital. Las crisis suelen tener inicio y fin bruscos, y suelen ser de
conformarnos con el de ausencias atpicas, que es un trmino menos corta duracin (segundos, escasos minutos), mientras que el aura vi-
definido en el que se incluyen las crisis que cursan predominante- sual migraosa suele durar varios minutos (menos de media hora) y se
mente con prdida del nivel de conciencia y escasos sntomas moto- siguen de cefalea hemicraneal en el lado contrario a las fotopsias
res. Pero preferiblemente habr que elegir la opcin de las crisis par- (aunque pueden existir auras sin migraa posterior).
ciales complejas por ser mucho ms exacta. La sensacin de bolo epigstrico ascendente puede ser
premonitoria de una crisis generalizada, aunque no es privativa de las
Pregunta 45.- R: 2 mismas, y pueden aparecer en las crisis parciales complejas y en las
Los episodios repetidos de movimientos involuntarios sugieren cri- denominadas ausencias atpicas.
sis motoras. Si no conllevan deterioro del nivel de conciencia, se
tratarn de crisis parciales simples, ya que tanto las crisis generalizadas Pregunta 48.- R: 2
como las crisis parciales complejas asocian alteracin del nivel de Hay que recordar que el sndrome de West debuta ms frecuente-
conciencia. El hecho de que se desplacen o extiendan a lo largo del mente entre los 3 y los 6 meses y siempre en el primer ao. En la mayor
mismo hemicuerpo indica que la actividad elctrica tambin se des- parte de los casos (60%) existe patologa subyacente, y es en estos casos
Comentarios TEST

plaza o extiende, pero a lo largo de la circunvolucin motora prima- en los que el pronstico a medio-largo plazo es peor: el paciente suele
ria del lbulo frontal. A este tipo de crisis, descritas por Jackson, se le quedar con una afectacin psicomotriz importante y suelen ser epilep-
llama jacksoniana, pero no hay generalizacin con afectacin del sias crnicas de difcil tratamiento. En los casos (40%) en los que no se
resto de la corteza. encuentra patologa subyacente el pronstico es mejor.
En cuanto a la causa, dado el antecedente de un meningioma Las crisis de espasmos se presentan generalmente al despertar, sin
intervenido, habr que descartar una recidiva del tumor, pero no es que se necesite un estmulo especial para desencadenarlas, y convie-
imprescindible la existencia de lesin ocupante de espacio. Dado ne no olvidar que la hipsarritmia (el EEG que se considera parte de la
que est intervenido, es posible que haya habido una lesin quirrgi- clnica caracterstica del sndrome) es interictal, es decir, cuando el
ca o por compresin por el tumor previo que puede generar la crisis, paciente no tiene crisis (opcin 4, falsa). Los espasmos que se conside-
sin necesidad de ms tumor. ran ms frecuentes varan segn las series, pero probablemente sean
Por ltimo, dado que los sntomas motores comienzan en la mano los flexores y los mixtos (flexo-extensores).
derecha, localizamos el punto de inicio en el lbulo frontal izquierdo, El diagnstico no es difcil desde un punto de vista sindrmico; hay
en el rea correspondiente a las primeras motoneuronas encargadas de que realizar un amplio despistaje para la patologa de base.
la extremidad superior, que no estn en la hoz (donde se sita la raz del Elegir entre los tratamientos que han demostrado eficacia en el
muslo), sino a un nivel inferior, en la circunvolucin motora. sndrome de West no es fcil, ya que hay varios frmacos tiles: el
clsico es con ACTH, aunque el valproato y el clonacepam tambin
Pregunta 46.- R: 3 puede ser eficaces. La vigabatrina da mejor resultado que los otros en
Las causas de crisis epilpticas de debut en adulto joven son funda- el control de los sndromes de West asociados a esclerosis tuberosa,
mentalmente los traumatismos previos y las lesiones ocupantes de pero su uso generalizado se ha seguido del hallazgo de lesiones en la
espacio, de las cuales la ms frecuente es la metstasis de un tumor retina, que comienzan en la retina perifrica (con la consiguiente
primario en otra localizacin. De stos, se necesita un traumatismo al reduccin perifrica del campo visual) y que obliga a realizar
menos moderado para poder considerarlo causante de un foco epi- campimetras de control a los pacientes tratados, dado que el dao es
leptgeno, mientras que no necesitamos antecedente tumoral para irreversible. En principio, el uso de este frmaco est limitado a casos
encontrar una metstasis; de hecho, no es infrecuente que algunos seleccionados, por periodos concretos de tiempo y con seguimiento
tumores den sus primeros sntomas por metstasis intracraneales. Re- campimtrico.

M exico A rgentina CTO Medicina C/ Nez de Balboa, 115 28006 MADRID (Espaa) Tfno.: (91) 782 43 32 / Fax: (91) 782 43 27
C hile U ruguay E-mail: secretaria@ctomedicina.com; iberocto@ctomedicina.com WEB: www.ctomedicina.com; www.iberocto.com
NR Pg. 11
NEUROLOGA
Seguimiento a distancia Preparacin Examen de Seleccin 05/06 1 Vuelta
Pregunta 49.- R: 2 Pregunta 51.- R: 4
Las crisis febriles tpicas son un fenmeno muy frecuente en la Se trata de una serie de generalidades que hay simplemente que
edad infantil, debido a la especial sensibilidad que tiene el cerebro a recordar:
esas edades para presentar crisis generalizadas en relacin con las 1) El tratamiento en monoterapia (una vez seleccionado un frmaco
subidas trmicas. Con la madurez completa de la corteza, que se va de primera lnea para el tipo de crisis) consigue un buen control de
alcanzando a lo largo de la infancia, aumenta la resistencia a que los las mismas en un alta porcentaje de pacientes (60-80%)
cambios de temperatura produzcan alteraciones elctricas. 2) En las crisis de ausencia tpicas hay dos frmacos que, segn los
Se desconocen por el momento los mecanismos bsicos por los autores, se sitan en primer o segundo lugar: la etosuximida y el
cuales se es ms sensible a los cambios de temperatura, pero se ha cido valproico. La fenitona y la carbamacepina pueden empeo-
visto estadsticamente que se presentan antecedentes familiares en un rar las crisis de ausencia
gran nmero de casos. 3) La etosuximida es un frmaco cuya nica indicacin hoy da son
En las crisis tpicas, tanto el desarrollo, como el comportamiento, las crisis de ausencia tpicas, sin que controle ningn otro tipo. En
rendimiento y funciones cerebrales son normales fuera de los episo- las crisis parciales complejas, el tratamiento (o uno de ellos ) de
dios crticos, as como el EEG. No es preciso el tratamiento de base primera lnea sera la carbamacepina
con anticomiciales, dado que es suficiente con el control de los as- 4) El fenobarbital cuenta entre sus efectos secundarios ms frecuentes
censos trmicos bruscos. la sedacin y la somnolencia; en nios puede producirse el efecto
En casos seleccionados, se puede pautar tratamiento con diace- paradjico de desencadenar un cuadro de hiperactividad. Esto no
pam oral o rectal en los dos primeros das de los procesos febriles es muy distinto de lo que sucede con el sndrome de hiperactivi-
como profilaxis de las crisis, pero no existe un consenso de actuacin dad e hipercinesia primario, en el que parece que la alteracin
para todos los pacientes, por lo que no es un tratamiento que pueda bsica es una tendencia al sueo con dificultad para mantener el
aceptarse como generalizado. nivel de alerta y por eso, para conseguirlo, el paciente tiene la
Las denominadas crisis febriles atpicas sugieren patologa de hiperactividad y es preciso tratarlo con estimulantes para que no
base que tiene una repercusin clnica en el contexto de un sndro- tenga la necesidad de ser hiperactivo
me febril, ms que de una corteza cerebral normal con mayor sen- 5) Las crisis febriles son un proceso frecuente en los primeros aos de
sibilidad a la hipertermia, por lo que hay quien prefiere hablar de vida (1-5 aos) y se relaciona con antecedentes familiares de crisis
crisis con fiebre, lo que significa que el paciente tiene crisis no febriles infantiles en los padres y con algunas formas de epilepsia.
causadas nicamente por la fiebre, sino por la suma de sta a otra No est indicado el tratamiento profilctico con anticomiciales a
patologa. largo plazo, y s el control de los ascensos de temperatura en los
procesos febriles. En algunos casos concretos, se puede administrar
Pregunta 50.- R: 1 diacepam al principio de cada proceso febril, pero no es una
Hay que recordar que epilepsia significa tener crisis epilpticas prctica generalizada.
de forma recurrente. Cualquier cerebro puede tener, bajo determina-
das circunstancias, una actividad elctrica paroxstica que clnica- Pregunta 52.- R: 2
mente conlleve una crisis epilptica, sin que esto signifique mayor Hay distintas pautas de actuacin ante un estatus epilptico, pero
riesgo de presentar crisis ms adelante. generalmente se establece una actuacin por escalones:
La decisin de instaurar un tratamiento anticomicial est basada Benzodiacepina: intravenosa en adultos, rectal en nios. En nues-
en el riesgo estimado de presentar nuevas crisis en el futuro; cuando es tro medio se utiliza habitualmente el diacepam; en otros pases
bajo o no es posible determinarlo, no est indicado iniciar un trata- disponen de loracepam por va intravenosa y se utiliza en lugar del
miento, dado el no desestimable nmero de casos en los que se pre- diacepam. En cualquier caso, hay que tener cuidado con la dosis,
senta una crisis aislada con un estudio complementario normal o dado que puede producir depresin respiratoria. El efecto de las
negativo. benzodiacepinas es de corta duracin, con lo que en muchas
Los sndromes epilpticos merecen una consideracin aparte, ocasiones no se espera a que el paciente tenga una segunda crisis
pero ya estamos en el contexto de una patologa hereditaria de la para pasar al segundo escaln.
que se puede estimar el riesgo de presentar nuevas crisis por los Fenitona (difenilhidantona) intravenosa: de inicio de accin algo
antecedentes familiares y lo que conocemos de la enfermedad, y ms lento, pero de vida media larga, permite que posteriormente se
por eso puede estar indicado iniciar el tratamiento a la primera mantenga el tratamiento a largo plazo con el mismo frmaco. Tam-
crisis. bin est admitido el cido valproico intravenoso, aunque no en

Comentarios TEST
De modo que, ante una primera crisis: NO TRATAR. todos los pases. Si con la administracin de fenitona en dosis mxi-
ma (hasta 25 mg/kg) no ceden las crisis, pasar al siguiente escaln.
Barbitricos: el fenobarbital como primera opcin. Su administra-
Pregunta 50. Indicacin de frmaco antiepilptico cin generalmente ha de ser en la UCI o con posibilidad inmediata
en funcin del tipo de crisis. de intubacin, si es preciso. En algunos casos se precisa otro tipo de
barbitricos, con anestesia general si es preciso.
12345675892
2
27323 284
Pregunta 53.- R: 2
1234526 789
1


 En principio, un paciente epilptico con buen control con un
5446542 326522 1
1
789
8 frmaco durante aos, si est cinco aos sin crisis (como en este caso)
con pruebas complementarias negativas, podra ser candidato a un
452  !542 1
" intento de lenta retirada de medicacin; en este caso, si no apareciera
452 2 !542 1

 el tema del embarazo, la opcin correcta sera la 1.
Un embarazo con tratamiento farmacolgico en general tiene ma-
546542 89#
1 yor riesgo de malformaciones fetales. Estas se producen en la fase de
organognesis, es decir, las primeras semanas, con lo que si queremos
" 3$  % 8
1
89#
7 evitar ese riesgo hay que suspender el frmaco al menos durante la
" 3$  &22 1
 
89# primera fase del embarazo; eso obligara a una suspensin brusca del
tratamiento: si hacemos una lenta retirada, no hemos evitado las mal-
542 1
 
89# formaciones.
1234567897
76 7545  75456 57 8 6523459  76 7 La retirada de un frmaco anticomicial debe hacerse de forma
 47
8 ! 75"45 !7978 75#$45 97
75$% 45 &'(
75#245 9789 71 muy lenta (meses), porque una retirada brusca puede hacer que el
paciente presente de nuevo crisis, incluso de una forma ms agresiva
que las que motivaron el inicio del tratamiento.

M exico A rgentina CTO Medicina C/ Nez de Balboa, 115 28006 MADRID (Espaa) Tfno.: (91) 782 43 32 / Fax: (91) 782 43 27
Pg. 12 NR C hile U ruguay E-mail: secretaria@ctomedicina.com; iberocto@ctomedicina.com WEB: www.ctomedicina.com; www.iberocto.com
NEUROLOGA
Preparacin Examen de Seleccin 05/06 1 Vuelta Seguimiento a distancia
Visto lo anterior, tenemos que decidirnos entre asumir el riesgo de Asimismo, la informacin ascendente por el fascculo longitudinal
malformaciones o el riesgo de crisis durante el embarazo. Esta balanza medial facilita el movimiento de abduccin del ojo contralateral, por lo
se inclina hacia lo primero: es menor el riesgo para el feto de estar que, al producirse la lesin, aparece un nistagmo en el ojo que s abduce.
sometido a un frmaco, en especial si es en monoterapia, que el de En resumen: ante la presencia, como nico defecto de motilidad
que la madre embarazada sufra una crisis. ocular, de alteracin en la mirada horizontal hacia un lado, en el que
Es cierto que puede haber una disminucin de niveles del frmaco aparece un ojo que no adduce (no mira hacia adentro) y un ojo que
durante el embarazo por aumento del metabolismo y del volumen de abduce (mira hacia fuera) pero con nistagmo, podemos situar la lesin
distribucin, pero no se suele precisar un aumento de dosis, y menos en el fascculo longitudinal medial del ojo que no mira hacia adentro,
a priori. o, si no se menciona el fascculo, en la protuberancia de ese lado.

Pregunta 54.- R: 4 Pregunta 57.- R: 1


Entre los sntomas menos frecuentes en el curso de una esclerosis
mltiple se encuentran los fenmenos paroxsticos: dolores o pareste-
Pregunta 54. Efectos secundarios de la medicacin anticomicial. sias lancinantes que recuerdan las neuralgias (como la neuralgia del
trigmino) en principio asumidas como originadas por lesiones y reas
12345678349
6   4679376793 37 6 94 de desaferentizacin sensitiva, aunque a veces no hay forma de justi-
76385 8785 4 87 85 58 7"6 68
ficar la causa. Generalmente responden bien al tratamiento con
7
944
73
54  7"6 68
inmunomoduladores (antiepilpticos, antidepresivos tricclicos, como
7 8 73 9  7"6 68
las neuralgias esenciales).
71
53  7"6 68
La existencia de cuadros de actividad neuronal anmala (como las
7
53 23 3
4 746
75 
8
crisis epilpticas) son infrecuentes en la EM (un 3%) y se producen en
7776869 38
relacin con lesiones activas que se siten prximas a la sustancia gris,
7!3 4
786
63
4 7 5  
postulndose que la irritacin cortical que produce la inflamacin
7"6 68
activa sera la causa de las crisis. Lo mismo sucedera en los movimien-
7"3 56#
388 7"6 68
tos coreoatetsicos, en lesiones prximas a los ganglios basales, bien
7$ 895867%3  8 7
  73
56(

por interrupcin de vas inhibidoras, bien por irritacin.
777 3  5587998&'3
El signo de Lhermitte es un signo clsico de afectacin de cordones
7$3 6
3 7 634  7)  646
posteriores a nivel cervical, con la aparicin de un dolor agudo (tipo
7776 38 7
4 3 558
descarga elctrica) que recorre la columna vertebral en sentido des-
cendente. No es exclusivo de la EM, sino que puede presentarse en
ENFERMEDADES DESMIELINIZANTES. lesiones medulares cervicales de otro origen, y en aquella aparece
cuando hay lesin medular, con lo que puede significar un signo de
Pregunta 55.- R: 1 mal pronstico si aparece en el debut de la enfermedad.
El sntoma de comienzo ms frecuente de la EM es la alteracin de Los trastornos afectivos, especialmente la depresin, son comunes
la sensibilidad, lo que se produce hasta en el 45% de los casos. Puede en la EM. Valorada la incidencia de depresin en otras patologas que
consistir en parestesias (descritas como pinchazos u hormigueo) o producen un grado similar de discapacidad, se objetiva un mayor
hipoestesias (acorchamientos). nmero de casos en los causados por la esclerosis mltiple, y se ha
El dficit motor tambin es frecuente como sntoma de debut, que relacionado con la carga lesional o la existencia de lesiones en reas
puede presentarse como franca paresia focal o como debilidad genera- concretas. Tambin pueden aparecer otros sntomas psiquitricos
lizada, que a la exploracin se acompaa de signos de lesin piramidal. causados por la terapia corticoidea, aunque no se recomienda el
Las alteraciones visuales son caractersticas, aunque sean menos tratamiento preventivo con litio u otros estabilizadores del nimo,
frecuentes al debut: lo ms frecuente es un escotoma central con dado que no es lo ms frecuente.
disminucin de la agudeza visual. El fondo de ojo puede presentar
edema de papila (papilitis) o ser normal (neuritis retrobulbar), siendo Pregunta 58.- R: 4
ms frecuente lo ltimo. Pasadas unas semanas, en el fondo de ojo se Los hallazgos en las pruebas complementarias no son por s mis-
apreciar una atrofia de la papila, con palidez de la misma. mos diagnsticos ni descartan el diagnstico de esclerosis mltiple,
Comentarios TEST

La disfasia aislada no encaja dentro del curso clnico de la esclero- limitndose a apoyar o a alejar el diagnstico.
sis mltiple, salvo que est asociada a deterioro cognitivo por numero- Las prueba ms til para el diagnstico es hoy por hoy la resonan-
sas lesiones diseminadas. cia magntica, puesto que es capaz de distinguir un mayor nmero de
Los trastornos del control de los esfnteres son muy infrecuentes al lesiones e incluso diferenciar entre lesiones desmielinizantes activas o
debut, aunque no sucede lo mismo a lo largo del curso de la enferme- antiguas, bien por el uso de determinadas secuencias especficas o
dad, donde ms de la mitad de los pacientes sufre algn tipo de por la administracin de contraste. De este modo, el hallazgo de una
alteracin en este sentido. RM cerebral normal, ante la presencia de una clnica por lo dems
sugerente de EM, si bien no descarta el diagnstico, puesto que podra
Pregunta 56.- R: 3 ser normal en fases muy precoces, s es la menos sugerente de la
La oftalmoplejia internuclear es muy caracterstica de la esclerosis enfermedad y obliga a plantear antes otros diagnsticos.
mltiple; se dice que si es bilateral y en un paciente joven, puede Los hallazgos del LCR apuntan hacia una actividad inflamatoria
considerarse casi patognomnico. dentro del SNC cuando aparecen anticuerpos que han sido sintetiza-
El cuadro clnico se produce por la interrupcin del fascculo dos dentro del SNC: esto se demuestra mediante dos mediciones:
longitudinal medial, que recorre la protuberancia, poniendo en co- Aumento del ndice de IgG: en el total de protenas de un lquido
municacin la zona de organizacin de la motilidad horizontal con- normal aparecen ms inmunoglobulinas de lo habitual, aunque la
jugada, que se encuentra junto a los sextos pares, con el rea mesen- proteinorraquia total no aumente; esto implica una sntesis excesi-
ceflica prxima a los terceros pares. va de IgG
El fascculo longitudinal medial de cada lado es el que lleva la Bandas oligoclonales: al estudiar las IgG en el LCR y en el suero del
informacin para la adducin del ojo ipsilateral, de tal forma que mismo paciente, se detectan anticuerpos en el LCR que no estn
ante una lesin del fascculo derecho se encuentra una imposibilidad presentes en el suero, lo que indica que se han sintetizado dentro
para la adducin del ojo derecho cuando se intenta la mirada conju- del SNC. Este es el hallazgo que aparece de forma ms constante en
gada. Es un movimiento (el del recto interno) que induce el III par, la enfermedad.
pero no hay que confundir la oftalmoplejia con la lesin del III: el ojo
puede hacer movimientos verticales y no est desviado hacia fuera en Los potenciales evocados se basan en la deteccin de la actividad
reposo, como sucedera con una lesin del par III. cortical que se produce con la estimulacin de distintos sistemas

M exico A rgentina CTO Medicina C/ Nez de Balboa, 115 28006 MADRID (Espaa) Tfno.: (91) 782 43 32 / Fax: (91) 782 43 27
C hile U ruguay E-mail: secretaria@ctomedicina.com; iberocto@ctomedicina.com WEB: www.ctomedicina.com; www.iberocto.com
NR Pg. 13
NEUROLOGA
Seguimiento a distancia Preparacin Examen de Seleccin 05/06 1 Vuelta
perifricos (visual, auditivo, sensitivo, motor). Si se detecta enlenteci- Por lo tanto, ninguna enfermedad monofsica se beneficiar de un
miento en alguno de los sistemas, se diagnostica de forma indirecta de tratamiento de este tipo, sino que ser preciso tratar con corticotera-
desmielinizacin en algn punto de la va. Hoy por hoy, las que se pia i.v. como cualquier otra inflamacin asptica activa del SNC.
consideran ms tiles son los visuales, dado que para el resto de los En el caso de la encefalomielitis diseminada aguda, que se presenta
sistemas la RM es ms sensible, mientras que para la valoracin del entre das y escasas semanas despus de una infeccin o una vacuna-
nervio ptico lo es el estudio del potencial evocado visual. cin, se produce una desmielinizacin difusa de la sustancia blanca,
probablemente por activacin de la inmunidad celular, con una
Pregunta 59.- R: 3 reactividad cruzada con antgenos de la mielina.
El tratamiento de la esclerosis mltiple se basa en dos puntos: Las vacunaciones que se han asociado con este cuadro son las de
La disminucin de la intensidad del brote: desde un punto de vista virus vivos atenuados, y tras la administracin de suero antitetnico
conceptual, si conseguimos disminuir la actividad inflamatoria lo (no del toxoide); la ms frecuente, la vacuna del sarampin. Esto hace
antes posible, conseguiramos un menor dao y una mejor recu- que en pases de baja prevalencia de sarampin, se sustituya la vacu-
peracin. Parece que el tratamiento con corticoterapia intraveno- na de virus atenuados por una sin virus vivos.
sa en altas dosis conseguira este efecto, aunque a largo plazo el Hoy las infecciones espontneas con las que se asocia de forma
grado de recuperacin es el mismo en tratados y no tratados, pero ms frecuente esta patologa son las infecciones respiratorias de etio-
en los tratados esa recuperacin se produce antes. Incluso se ha loga desconocida, seguidas de la varicela. Clsicamente era la infec-
valorado que el tratamiento con corticoides en un primer brote de cin por el virus del sarampin.
neuritis ptica retrasara la aparicin del segundo brote, sin que se La clnica puede variar desde un sndrome meningo-encefalo-
sepa la razn y sin que se hayan repetido los resultados en brotes mieltico grave con afectacin de muchos sistemas (cerebeloso, mo-
de otro tipo tor; crisis epilpticas), hasta un cuadro leve similar a un primer brote
La disminucin del nmero de brotes: en las formas recurrentes- de esclerosis mltiple.
remitentes existen tratamientos capaces de disminuir el nmero de El tratamiento, como se ha dicho, es con dosis altas de corticoides.
brotes, con lo que a largo plazo tendramos una menor carga
lesional y por lo tanto menos secuelas. El interfern beta, el acetato Pregunta 62.- R: 3
de glatiramer y algunos inmunosupresores han obtenido este efec- En un paciente de esta edad que previamente est bien y comienza
to, y hoy por hoy, por menos efectos secundarios relativos, se ad- con un cuadro de deterioro progresivo hay que pensar en enferme-
ministra el interfern. No est indicado el tratamiento con frma- dades degenerativas que, dado que se describe con afectacin de la
cos para disminuir el nmero de brotes cuando an no se sabe el sustancia blanca, debera hacernos pensar en alteraciones degenera-
nmero de brotes que espontneamente presentar el paciente, tivas de la sustancia blanca: las leucodistrofias.
por lo que no se trata a partir de un nico brote. En efecto, las leucodistrofias son cuadros degenerativos propios de la
Un paciente que est sometido a tratamiento para modificar el edad infantil, que, de forma genrica, presentan ceguera, sordera, dete-
curso de la enfermedad, con interfern beta o con otro frmaco, en el rioro mental y tetraplejia de instauracin progresiva, pero no suelen
caso de presentar un nuevo brote tiene indicacin de ser tratado con presentar crisis epilpticas, mioclonas, ni alteraciones en el EEG.
corticoterapia intravenosa, no siendo incompatibles ambos tratamien- En este paciente en el que expresamente se nos describe una alte-
tos. racin caracterstica del EEG hay que pensar en una panencefalitis
La existencia de secuelas establecidas en un paciente con esclero- esclerosante subaguda. Se produce en pacientes que han sufrido el
sis mltiple habla de lesin axonal residual aadida a la desmieliniza- sarampin (tambin es posible en vacunados, pero con una inciden-
cin, y hoy por hoy no hay tratamiento que consiga una regenera- cia muchsimo menor) y cursan de la forma descrita en el caso clnico.
cin neuronal, con lo que no se dispone de ningn tratamiento que Se produce una encefalitis difusa de las sustancias gris y blanca, con
ayude a mejorar secuelas establecidas desde brotes antiguos. pocos signos inflamatorios. En el LCR se detectan ttulos de anticuerpos
antisarampin altos, pero no virus. Tambin hay bandas oligoclonales.
Pregunta 60.- R: 4 No existe ningn tratamiento y la evolucin es mortal en el curso
El tratamiento modificador de la historia natural de la esclerosis de 1-4 aos.
mltiple hoy por hoy est basado en la reduccin del nmero de
brotes. En este objetivo han demostrado eficacia varios frmacos, sien- Pregunta 63.- R: 2
do los ms usados en la actualidad el interfern beta y el acetato de La descripcin clnica de un paciente con aparente buen nivel de
glatiramer, que han demostrado reduccin de un 30% de los brotes conciencia, con el que se puede mantener una conversacin en la

Comentarios TEST
previstos en comparacin con grupos control. que las contestaciones son a base de parpadeos, es la imagen caracte-
Se administran por va subcutnea o intramuscular con distintas rstica de un sndrome de cautiverio.
pautas y dosis, a pacientes con formas recurrentes-remitentes de la Este sndrome se produce por lesiones protuberanciales extensas
enfermedad, que hayan presentado dos brotes en los dos ltimos que respeten la sustancia reticular, con lo que el paciente no est
aos; con menos frecuencia no existe indicacin de tratar. comatoso, como ocurrira de tratarse de una lesin ocupante de es-
En las formas monosintomticas (un nico brote) y las formas pro- pacio, pero no tiene motilidad voluntaria por debajo de ese nivel, lo
gresivas primarias (en las que hay una lesin continua, sin brotes) no que incluye los movimientos oculares horizontales, la musculatura
est indicado el tratamiento, ya que no hay brotes que reducir. craneofacial y el resto del cuerpo.
Las secuelas de brotes previos indican lesiones residuales ms all Las causas son mltiples, y hay que recordar como curiosa la
de la desmielinizacin, por lo que no se recuperan con ningn trata- mielinlisis central pontina yatrognica por una recuperacin dema-
miento. Se ha descrito mejora de las lesiones desmielinizantes en la siado rpida de una hiponatremia, lo que da lugar a una deshidrata-
resonancia magntica tras el tratamiento con interfern, pero sin cam- cin y destruccin de las clulas a dicho nivel.
bios en la situacin funcional del sujeto, con lo que probablemente Puede aparecer una destruccin espontnea de la mielina o aso-
signifique una reduccin de las lesiones aparentes, pero que estaban ciado a otras lesiones desmielinizantes.
funcionando a pesar de la desmielinizacin. No tiene buen tratamiento, salvo que se identifique la patologa de
base, y lo ms importante es distinguirlo de un cuadro compresivo, que
Pregunta 61.- R: 5 puede causar el mismo dficit motor, pero que generalmente ir acom-
Para contestar esta pregunta basta conocer cul es la utilidad del paado de coma, alteraciones pupilares y en los reflejos de tronco.
interfern para dejar reducidas a dos las opciones a pensar.
El interfern beta es un frmaco que se administra en las esclerosis PATOLOGA EXTRAPIRAMIDAL.
mltiples remitentes-recurrentes para disminuir el nmero de brotes.
Es un inmunomodulador, que hace que el paciente tenga menos Pregunta 64.- R: 5
episodios, sin modificar la intensidad de stos, por lo que, en caso de Aunque la presencia de trastornos anmicos y de la memoria pueda
presentar brote, el paciente debe recibir terapia con corticoides. ser patolgica, el hecho de que se describa un paciente con movimien-

M exico A rgentina CTO Medicina C/ Nez de Balboa, 115 28006 MADRID (Espaa) Tfno.: (91) 782 43 32 / Fax: (91) 782 43 27
Pg. 14 NR C hile U ruguay E-mail: secretaria@ctomedicina.com; iberocto@ctomedicina.com WEB: www.ctomedicina.com; www.iberocto.com
NEUROLOGA
Preparacin Examen de Seleccin 05/06 1 Vuelta Seguimiento a distancia

Pregunta 69. Diagnstico diferencial del parkinsonismo

tos continuos involuntarios es el dato ms significativo. Los movimientos Pregunta 65.- R: 4


continuos generalizados sin especificar rea predominante ni veloci- Conviene recordar con respecto a esta enfermedad:
dad de ejecucin los podramos denominar coreoatetsicos. Que es de herencia autosmica dominante, por lo tanto, afecta a
Comentarios TEST

Un paciente adulto en el que debutan movimientos continuos hombres y mujeres.


involuntarios hay que pensar que se trate de movimientos coreicos, ya Que aunque el diagnstico se puede establecer por la clnica y los
que los movimientos atetsicos con mayor frecuencia se asocian a antecedentes familiares compatibles, hoy da el estudio gentico,
patologa congnita y no degenerativa de inicio tardo. en busca de las repeticiones del triplete CAG en el cromosoma 4
Estamos por lo tanto ante un paciente con demencia, trastorno establece el diagnstico de certeza incluso en fase sintomtica; esto
afectivo y movimientos involuntarios: esto es suficiente para el diag- no quiere decir que sea preciso, se puede seguir diagnosticando
nstico de la enfermedad de Huntington, aunque hoy da puede como siempre si no se dispone del estudio gentico. Un mayor
establecerse con certeza en ausencia de antecedentes o incluso de nmero de tripletes parece correlacionarse con el inicio ms pre-
clnica, mediante estudio gentico. coz, pero no con la clnica psiquitrica ni con la cognitiva.
La herencia es autosmica dominante, por lo que las posibilidades Que presenta el fenmeno de anticipacin: en la duplicacin
de que los hijos de un paciente afecto padezcan la enfermedad son el gentica, para la formacin de las clulas sexuales se produce un
50%, pero ante la presencia de antecedentes dos generaciones atrs aumento en el nmero de copias CAG, con lo que, si el descen-
se tiene prcticamente la certeza de que el padre del paciente de este diente toma ese cromosoma, la enfermedad debutar antes en l
caso tambin habra padecido la enfermedad. que en su antecesor.
En esta enfermedad se produce una alteracin de la protena Que la atrofia del ncleo caudado (ms acusada en la cabeza)
huntingtina, de la que an no se conoce su funcin normal, y que hace que ese espacio se ocupe por el ventrculo lateral prximo.
conlleva una destruccin neuronal en distintos sistemas, predomi- Eso en la TC semeja una dilatacin ventricular como las hidrocefa-
nando la atrofia del estriado, pero con afectacin en fases ms avan- lias, pero en este caso es selectiva: slo est aumentada de tamao
zadas de la corteza cerebral, el locus ceruleus, el ncleo subtalmico el asta frontal, que es la que est junto al caudado, mientras que el
y otras reas resto del ventrculo (astas temporales y occipitales) es normal.
Desde el punto de vista bioqumico, el principal neurotransmisor Tener en la memoria tambin dos variantes respecto a la forma tpi-
relacionado es la dopamina, existiendo una hiperfuncin dopami- ca:
nrgica; pero tambin se altera la neurotransmisin gabargica, la 1) variante juvenil (de Westphal): presenta un cuadro rigido-acintico
colinrgica, la somatostatina y la sustancia P. ms llamativo que el coreico, y crisis epilpticas. Son casos hereda-
dos del padre y no de la madre, y tienen peor pronstico.

M exico A rgentina CTO Medicina C/ Nez de Balboa, 115 28006 MADRID (Espaa) Tfno.: (91) 782 43 32 / Fax: (91) 782 43 27
C hile U ruguay E-mail: secretaria@ctomedicina.com; iberocto@ctomedicina.com WEB: www.ctomedicina.com; www.iberocto.com
NR Pg. 15
NEUROLOGA
Seguimiento a distancia Preparacin Examen de Seleccin 05/06 1 Vuelta
2) variante de inicio tardo: la corea como signo predominante, evo- La afectacin tpica de los movimientos oculares supranucleares no
lucin ms benigna. suele presentarse hasta transcurridos cuatro-cinco aos del comienzo
de los sntomas, consistiendo en una limitacin para los movimientos
Pregunta 66.- R: 4 oculares verticales con conservacin de los reflejos oculoceflicos.
El tratamiento de la enfermedad tiene un objetivo meramente sin- Puede haber una rigidez axial intensa, lo que es ms llamativo en la
tomtico, ya que hasta el momento no se dispone de ninguna mol- hiperextensin forzada del cuello que suelen presentar estos pacien-
cula capaz de detener la muerte de las neuronas afectadas, con lo que tes, con un tono relativamente conservado en las extremidades.
ningn tratamiento consigue detener la evolucin de la enfermedad. Existe muy frecuentemente un deterioro cognitivo asociado, que
Los sntomas coreicos se controlan con antagonistas dopaminrgi- no suele ser una demencia completa, pero que conlleva bradipsi-
cos, entre los que se encuentran los neurolpticos clsicos (haloperi- quia, irritabilidad, incluso labilidad emocional.
dol, tiapride, sulpiride) y los depletores presinpticos de dopamina La parlisis supranuclear progresiva tiene un rango de presentacin
(reserpina, tetrabenacina). muy similar al de la enfermedad de Parkinson, ms frecuentemente en
Se ha intentado actuar sobre otros sistemas de neurotransmisin torno a los 50-60 aos, aunque existen casos de debut ms precoz.
afectados (gabargico, serotoninrgico, colinrgico, somatostatina) sin La muerte se produce por complicaciones asociadas (respiratorias,
claros efectos beneficiosos. cadas).
La base neuroqumica de la depresin en la enfermedad de
Huntington se desconoce, ya que los sistemas serotoninrgicos pare- Pregunta 69.- R: 3
cen estar afectados tanto en los pacientes deprimidos como en los La presencia de un cuadro parkinsoniano con rigidez y bradicine-
que no lo estn. En los pacientes que presenten sndrome depresivo se sia nos debe hacer pensar siempre en primer lugar en una enferme-
puede iniciar un tratamiento emprico, con inhibidores selectivos de dad de Parkinson, que es la causa ms frecuente de sndrome
la recaptacin de serotonina (fluoxetina, paroxetina) o con otro tipo parkinsoniano (ver figura en pgina siguiente).
de antidepresivos (tricclicos). Si aparecen signos de afectacin de otros sistemas, hay que pensar
En principio, los agonistas dopaminrgicos no tienen papel alguno en principio que no se trata de una enfermedad de Parkinson.
que jugar en una enfermedad con una hiperactividad dopaminrgi- En el caso de la pregunta, ya se nos presenta la existencia de escaso
ca, salvo para empeorarlos, y por eso se considera correcta la opcin temblor, cuando el temblor de reposo es el signo ms frecuente de
4. Sin embargo, en algunas ocasiones se pueden obtener beneficio inicio de la enf. de Parkinson. Pero la presencia de marcha atxica
con dosis bajas de agonistas dopaminrgicos, por un efecto paradji- automticamente nos lleva a la lista de diagnsticos diferenciales en la
co que se postula se produce al actuar sobre los receptores que no cabe el Parkinson, puesto que no puede presentar sntomas
presinpticos, limitando la liberacin de ms dopamina. de afectacin cerebelosa.
Siempre que aparezca un cuadro parkinsoniano con sntomas in-
Pregunta 67.- R: 3 compatibles, hay con considerar la presencia de una atrofia multisis-
Para no caer en la confusin, separar los datos del caso clnico en tmica: este es un cuadro que engloba la coexistencia de sntomas
las distintas reas de la exploracin: parkinsonianos, vegetativos, cerebelosos y piramidales en distinto gra-
Motor: hipertona plstica (resistencia continua), reflejos presentes, do en el mismo paciente.
algo hipoactivos. Todo ello compatible con enfermedad de En un principio se consideraban distintas enfermedades depen-
Parkinson. Cadas frecuentes que no parecen crisis epilpticas (con- diendo de la sintomatologa preponderante, hasta que se demostr
servacin de la conciencia): no sugerente de enf. de Parkinson. que en realidad la anatoma patolgica era la misma y que se trataban
Extrapiramidal: hipomimia facial, hipertona plstica, sin temblor de de presentaciones distintas de un mismo proceso.
reposo (lo que esperaramos en una enfermedad de Parkinson) ni En el caso de la atrofia multisistmica que presenta de forma pre-
movimientos involuntarios (lo que presentara una variante de dominante sntomas cerebelosos, la denominamos atrofia olivo-pon-
Westphal, adems de ser ms joven que el paciente de la pregunta) to-cerebelosa, por ser stas las localizaciones en las que en un princi-
Funciones corticales: cambios de comportamiento, alucinaciones pio se crea estaba confinada la degeneracin
visuales, instauracin subaguda
Son demasiados los datos que hacen ms que dudoso el diagns- Pregunta 70.- R: 4
tico de enfermedad de Parkinson. Ni el curso clnico ni la edad del Nos estn describiendo un cuadro de afectacin vegetativa, con
paciente acompaan a la variante rgida de la enf. de Huntington sntomas predominantemente genitourinarios: podramos encontrar-
Igualmente son demasiados signos motores extrapiramidales para nos ante un cuadro de disautonoma perifrica o bien de afectacin

Comentarios TEST
una enfermedad de Creutzfeldt-Jakob o de Alzheimer, que cursan de medular con afectacin de segmentos inferiores.
una forma menos llamativa en lo motor (salvo las mioclonas en la La presencia del antecedente de sncopes no acompaa para el
primera). diagnstico de patologa medular, por lo que podra corresponder a
En ocasiones es especialmente difcil el diagnstico diferencial en- un sndrome perifrico si no fuera por los signos parkinsonianos.
tre la demencia por cuerpos de Lewy y un Parkinson-demencia, dado Ante un sndrome parkinsoniano con disautonoma, podra tratarse
que el diagnstico definitivo se establece post-mortem y que incluso de una enfermedad de Parkinson si el cuadro motor hubiera sido com-
entonces, dada la aparicin de datos anatomopatolgicos similares, patible y previo a la aparicin de los sntomas vegetativos. Generalmen-
puede no quedar claro. te los signos disautonmicos no suelen ser muy marcados.
De forma arbitraria se ha establecido que, si los sntomas parkinso- En este paciente, sin embargo, lo que parece menos llamativo es el
nianos preceden a los cognitivos en un ao, estaremos ante un cuadro parkinsoniano, y ms limitante los sncopes de largo tiempo
Parkinson-demencia, mientras que si la demencia precede o coexiste de evolucin. Este es el patrn tpico de la enfermedad de Shy-Drager,
al comienzo con el cuadro motor, lo consideraremos una Enferme- que se encuadra dentro de la atrofia multisistmica, en la que predo-
dad por Cuerpos de Lewy. mina el cuadro vegetativo sobre el motor.
El tratamiento de los sntomas motores, como en el resto de atrofias
Pregunta 68.- R: 2 multisistmicas, es poco satisfactorio, y en este caso es preciso un
Existe una serie de datos clnicos muy sugerentes de la existencia de buen control de los sntomas vegetativos, especialmente de los snco-
una parlisis supranuclear progresiva o enfermedad de Steele- pes, que pueden producir problemas por las cadas. Se recomienda
Richardson, aunque al principio del cuadro pueda confundirse y evitar hipotensiones y situaciones en que stas pueden producirse y
diagnosticarse errneamente de Enfermedad de Parkinson. De he- hay algunos autores que recomiendan tratamiento hipertensor, aun-
cho, un nmero no pequeo de pacientes (hasta un 40-50%) presen- que este extremo no est comnmente aceptado.
tan una cierta mejora con la administracin de levodopa.
Entre el 60 y el 90 % de los pacientes presenta inestabilidad postu- Pregunta 71.- R: 5
ral al inicio, dando lugar a cadas frecuentes, dato poco compatible La presentacin tpica de la enfermedad de Parkinson idioptica es
con el diagnstico de Enfermedad de Parkinson. el temblor de reposo, unilateral, en una mano, que con el tiempo se
extiende a la mano contralateral.
M exico A rgentina CTO Medicina C/ Nez de Balboa, 115 28006 MADRID (Espaa) Tfno.: (91) 782 43 32 / Fax: (91) 782 43 27
Pg. 16 NR C hile U ruguay E-mail: secretaria@ctomedicina.com; iberocto@ctomedicina.com WEB: www.ctomedicina.com; www.iberocto.com
NEUROLOGA
Preparacin Examen de Seleccin 05/06 1 Vuelta Seguimiento a distancia
En fases ms avanzadas es cuando aparece la bradicinesia caracte- Pregunta 74.- R: 4
rstica, junto con el aumento de tono muscular y la inestabilidad Los frmacos tiles en la enfermedad de Parkinson son los que de
postural. forma lgica esperaramos que sustituyan la neurotransmisin defec-
Los reflejos miotticos suelen conservarse normales o mnima- tuosa, ms algunos de accin desconocida o mltiple:
mente disminuidos a lo largo de toda la evolucin de la enferme- Anticolinrgicos: frenan el predominio relativo de la transmisin
dad; unos reflejos vivos o exaltados obligara a replantearse el diag- colinrgica en el estriado que se produce por la disminucin de la
nstico, ya que en la enfermedad de Parkinson idioptica no hay dopamina. Contraindicados en pacientes con glaucoma, prostatis-
afectacin de la va piramidal, salvo que haya adems otras lesiones mo y en personas mayores por el riesgo de alteraciones cognitivas.
asociadas. Tampoco se usan en fases avanzadas de la enfermedad; su indica-
La capacidad cognitiva de los pacientes se conserva tambin duran- cin son pacientes jvenes en los que predomina el temblor
te largo tiempo, siendo en fases avanzadas cuando no es infrecuente Levodopa con inhibidor de descarboxilasa perifrica: es el trata-
que stas se alteren, dando lugar a lo que se denomina parkinson- miento ms eficaz para la enfermedad en todos sus aspectos clni-
demencia, y que obliga al diagnstico diferencial con la enfermedad cos, y la limitacin de su uso depende fundamentalmente de los
por cuerpos de Lewy si la demencia aparece en fases iniciales. efectos secundarios a corto y largo plazo
A pesar de las alteraciones posturales, el paciente generalmente no Agonistas dopaminrgicos: estimulan directamente los receptores;
sufre cadas frecuentes hasta fases avanzadas de la enfermedad, y si inducen con frecuencia a sntomas digestivos, que pueden ser con-
aparecen en fases iniciales hay que considerar otros diagnsticos (pa- trolados con antidopaminrgicos perifricos (antiemticos). Sus
rlisis supranuclear progresiva, enfermedad por cuerpos de Lewy, etc). indicaciones son pacientes en fases iniciales, intentando reservar la
levodopa para ms adelante, y el control de las fluctuaciones mo-
Pregunta 72.- R: 4 toras en fases avanzadas de la enfermedad que se ha tratado con
Hay que recordar que el diagnstico de enfermedad de Parkinson levodopa
idioptica es eminentemente clnico, y que las pruebas complemen- Amantadina: antivrico cuya accin parkinsoniana se descubri
tarias son especialmente tiles para descartar otras patologas, salvo las de modo casual, es de accin leve y se utiliza en fases iniciales o
pruebas de imagen funcionales (PET, SPECT) que detecten alteracio- para el control de las discinesias
nes en la funcin de los ganglios basales. Selegilina: inhibidor de la mono-amino-oxidasa, permite que haya
El estudio anatomopatolgico muestra cambios degenerativos simi-
lares a los que se producen en la enfermedad por cuerpos de Lewy, ms dopamina disponible en la terminal sinptica. Se ha postula-
quedando en discusin si ambas entidades no forman parte de una do su mecanismo neuroprotector para las neuronas dopaminrgi-
misma patologa, con presentaciones diferentes. La diferencia es que los cas, pero en este sentido no hay evidencia completa
datos de afectacin estructural se encuadran en ganglios basales y tron- La clorpromazina es un neurolptico clsico, de efecto antidopa-
co del encfalo en la enfermedad de Parkinson, mientras que en la otra, minrgico, no muy aconsejable en el tratamiento de enfermos con
tanto la corteza como el rea subcortical tiene afectacin. Es la afecta- Parkinson.
cin cortical la que se relaciona con la demencia, con lo cual es esperable
que el Parkinson, que suele conservar las funciones superiores hasta Pregunta 75.- R: 4
fases avanzadas, en las que aparecen cambios de personalidad y A las complicaciones derivadas del tratamiento con levodopa de
cognitivos, no presente degeneracin cortical. forma prolongada algunos autores las denominan sndrome de
El temblor caracterstico de la enf. de Parkinson es el temblor de levodopaterapia crnica, y suele aparecer tras 3-5 aos de trata-
reposo; durante aos puede ser incluso el nico dato. Pero a lo largo miento con este frmaco. Bsicamente aparecen fluctuaciones en la
de la evolucin es frecuente que se asocie con un cierto componente movilidad y discinesias.
de temblor postural, que, si es muy intenso, puede llegar a precisar Las fluctuaciones motoras pueden ser simples, en las que el pa-
tratamiento con betabloqueantes. ciente presenta de nuevo dficit de movilidad a las 3-4 horas de la
El tratamiento busca, de forma directa o indirecta, el aumento de la ltima administracin de levodopa y recupera su situacin funcional
transmisin dopaminrgica, por lo que en efecto no hay que incluir tras la nueva dosis. Fluctuaciones complejas son las que se presentan
antagonistas dopaminrgicos, que empeoraran los sntomas. de modo errtico, sin una clara relacin temporal con la ingesta de
Por ltimo, recordar que los signos de afectacin piramidal (refle- dosis, y pueden deberse a mltiples causas, generalmente combina-
jos vivos, hipertona elstica, respuesta cutneo-plantar extensora) das
obligan a replantear el diagnstico de enfermedad de Parkinson, de- Las discinesias pueden clasificarse en discinesias de beneficio de
Comentarios TEST

biendo considerarse en primer lugar una atrofia multisistema (degene- dosis (aparecen en el pico mximo de levodopa, y generalmente son
racin estro-ngrica) o que haya otras lesiones distintas afectando a las de tipo coreico), discinesias bifsicas (movimientos rtmicos alternantes
primeras motoneuronas (isquemia lacunar?).
de las extremidades inferiores que aparecen al inicio y/o al final de la
Pregunta 73.- R: 2 accin de la levodopa) y distona off (postura distnica que aparece
Existen distintos neurotransmisores alterados en la enfermedad de coincidiendo con el fin del efecto de la levodopa)
Parkinson, y a algunos de ellos se les achaca alguno de los sntomas El manejo de las fluctuaciones motoras simples y de las discinesias
caractersticos de la enfermedad: de beneficio de dosis se basa en la distribucin de la dosis de levodo-
Dopamina: es el ms afectado; disminuye sobre todo en el estria- pa a lo largo del da, asegurarse de que no hay interferencias con la
do, lo que se correlaciona con el cuadro motor (bradicinesia, tem- absorcin, o el uso de medicacin con liberacin lenta, con objeto
blor y rigidez). Tambin parece involucrado en los cuadros de evitar altibajos en los niveles plasmticos del frmaco.
disautonmicos, por deplecin en reas hipotalmicas. El control de las fluctuaciones motoras errticas requiere general-
Noradrenalina: disminuye sobre todo en la corteza hemisfrica, y mente la combinacin de distintos frmacos, con objeto de mantener
se sospecha involucrado en las alteraciones cognitivas. un estmulo dopaminrgico continuo sin fluctuaciones.
Serotonina: su disminucin en la corteza frontal y en el estriado se
ha postulado en relacin con los cuadros depresivos que pueden Pregunta 76.- R: 2
aparecer en el curso de la enfermedad. En ocasiones es difcil determinar desde el punto de vista terico,
Acetilcolina: disminuye en crtex, y est clara su relacin con la ante qu tipo de temblor nos encontramos, y nos tenemos que imagi-
demencia; este hecho hace que el tratamiento con anticolinrgi- nar las situaciones en las que aparece el temblor:
cos en el Parkinson est limitado a pacientes jvenes, con la enfer- El temblor de reposo aparece en apoyo y sin hacer ninguna activi-
medad en fases iniciales, con temblor como nico sntoma o pre- dad.
dominante, dado que en fases avanzadas el deterioro de los siste- El temblor de actitud es visible al mantener una postura o con
mas colinrgicos puede empeorarse con dichos frmacos. mnimos movimientos.
Somatostatina: disminuye en corteza, y su significado patolgico es El temblor cintico aparece al realizar movimientos en los que
incierto interviene toda la extremidad.

M exico A rgentina CTO Medicina C/ Nez de Balboa, 115 28006 MADRID (Espaa) Tfno.: (91) 782 43 32 / Fax: (91) 782 43 27
C hile U ruguay E-mail: secretaria@ctomedicina.com; iberocto@ctomedicina.com WEB: www.ctomedicina.com; www.iberocto.com
NR Pg. 17
NEUROLOGA
Seguimiento a distancia Preparacin Examen de Seleccin 05/06 1 Vuelta
En el caso de la pregunta, aunque cupiera la duda de si se trata de extensora) y en otras lesin de motoneurona inferior (atrofia muscu-
un temblor postural o cintico, la ausencia de alteraciones en la mar- lar marcada, fasciculaciones, reflejos disminuidos).
cha no apunta a un cuadro cerebeloso. En ningn momento de la evolucin presenta alteracin del con-
La presencia de una rueda dentada podra hacer sospechar una trol de esfnteres (esas neuronas, sin que se sepa la razn, no se dege-
enfermedad de Parkinson, pero se especifica la ausencia de temblor neran), ni con menoscabo de las funciones superiores, ni con altera-
de reposo, que es lo caracterstico de este cuadro. ciones sensitivas.
Por lo tanto, estamos ante un temblor de caractersticas posturales, El diagnstico diferencial ms frecuente que hay que hacer es con
con fenmeno de rueda dentada, que mejora con alcohol: son los la mielopata cervical, en la que el paciente tambin tiene signos de
datos caractersticas del temblor esencial. afectacin de primera motoneurona (con hiperreflexia y Babinski en
Esta es una enfermedad de etiologa no determinada, pero con un miembros inferiores) y segunda motoneurona en brazos, con atrofia y
claro componente familiar, con lo que son frecuentes los anteceden- reflejos disminuidos. Esta distribucin caracterstica y la posible afec-
tes familiares en la anamnesis. Antes de dar este diagnstico hay que tacin sensitiva y de esfnteres por compresin de otros cordones
descartar fundamentalmente dos cosas: la ingesta de algn frmaco medulares sirven para el diagnstico.
que pueda causar temblor (el temblor farmacolgico es ms frecuen- En las fases iniciales, la musculatura extraocular est preservada,
temente de tipo postural) y la existencia de focalidad neurolgica. aunque en fases avanzadas tambin se afecta.
Puede presentar rueda dentada (de hecho, la rueda dentada en la La supervivencia espontnea de la enfermedad puede alargarse de
enfermedad de Parkinson se achaca al temblor de reposo, ms que forma discreta con el tratamiento con riluzol.
considerarse un signo independiente).
El tratamiento de esta patologa es meramente sintomtico, y se PATOLOGA VASCULAR CEREBRAL
utilizan los betabloqueantes o la primidona (un precursor del feno-
barbital). El momento de comenzar el tratamiento lo da la limitacin Pregunta 80.- R: 1
que tenga el paciente para su vida diaria. De entre los factores de riesgo del ictus isqumico (ver tabla), el ms
importante, por ser el ms frecuente y por el hecho de ser modifica-
Pregunta 77.- R: 5 ble, es la hipertensin arterial.
La enfermedad de Friedreich es, en efecto, la ataxia hereditaria El riesgo relativo de padecer un ictus isqumico est directamente
ms frecuente, con herencia autosmica recesiva ligada al cromoso- relacionado con las cifras elevadas de tensin arterial, tanto la diast-
ma 9. Los sntomas siempre comienzan en las dos primeras dcadas lica como la sistlica, por lo que los pacientes con hipertensin sistlica
de la vida, tras un desarrollo normal, y una vez que aparecen, se aislada tambin tienen ms riesgo de sufrir un ictus isqumico.
produce un desarrollo completo de la enfermedad. El grado y la progresin de la aterosclerosis carotdea se relaciona
El defecto anatomopatolgico fundamental es la degeneracin de con los niveles de colesterol total y LDL-colesterol, mientras que suce-
los cordones posteriores, aunque posteriormente tambin se afectan de a la inversa con el HDL-colesterol. Debido a esto es especialmente
los cordones laterales, con vas descendentes corticoespinal y importante el control de la hipercolesterolemia, tanto en prevencin
espinocerebelosa ascendente: esto da lugar a la ataxia, aunque el primaria como en secundaria del ictus isqumico.
cerebelo no est afectado desde el punto de vista anatmico. El tratamiento con anticonceptivos orales aumenta el riesgo, sobre
Adems del cuadro clnico ms llamativo, que es la ataxia, en la todo si se asocia con otros factores (HTA, tabaquismo), especialmente
exploracin se aprecian alteraciones de la sensibilidad profunda (la en anticonceptivos con alto contenido de estrgenos. Est en tela de
que depende de cordones posteriores) y arreflexia (a pesar de la dege- juicio si la terapia hormonal sustitutiva en la menopausia incrementa
neracin de la va corticoespinal, que debera cursar con hiperreflexia), o no el riesgo.
ya que la degeneracin de las neuronas sensitivas que forman los cor- El tabaquismo promueve la formacin de placas de ateroma, dis-
dones posteriores interrumpe la va aferente de los reflejos de estira- minuye el flujo sanguneo cerebral y favorece la hiperviscosidad san-
miento muscular. Sin embargo, la respuesta cutneo-plantar es exten- gunea, lo que conlleva a multiplicar por dos el riesgo de ictus isqu-
sora, como corresponde a la degeneracin de la va corticoespinal, ya mico. A los cinco aos de dejar de fumar, el riesgo se iguala con los no
que la recepcin del estmulo superficial cutneo s se conserva. fumadores.
En casi todos los casos hay afectacin cardiaca, que es la que ms La diabetes aumenta el riesgo de padecer infarto principalmente
suele limitar el pronstico vital, aunque tpicamente se describe una de tipo aterotrombtico, asociado a la microangiopata y al aumento
miocardiopata hipertrfica; en numerosas ocasiones se detecta dila- de la aterosclerosis cerebral. Adems, los ictus sufridos por pacientes
tacin, con trombos intracavitarios o anomalas de la conduccin, diabticos tienen mayor morbimortalidad.

Comentarios TEST
con lo que los problemas cardiacos pueden estar asociados a proble-
mas restrictivos, emblicos o por arritmias.
Pregunta 80. Factores de riesgo del ictus isqumico.
Pregunta 78.- R: 1
El curso evolutivo de la Enfermedad de Friedreich es completo una 12 24565789

vez que comienzan los sntomas, llevando al fallecimiento de los pa- 324565789

cientes en la cuarta-quinta dcada, debido a complicaciones cardiacas 1234546 12 

2 12
56 12# 6
(ms frecuente) o respiratorias. 1275856 2225
56 123  6 12#
4546
En prcticamente todos los enfermos hay miocardiopata, que 129
6 125

2
6 12 5
2! 2 12$5%454
puede ser hipertrfica o dilatada, y que con frecuencia asocian tras- 12

56 125  5254556 2225  6 222&56
tornos de la conduccin, detectables en el EKG. Es la causa ms fre- 1255 6 1255
 2"  56 12'5(5
cuente de muerte.
Existen alteraciones esquelticas adquiridas por la disfuncin neu-
rolgica, bsicamente cifoescoliosis y pies cavos, aunque la limitacin Pregunta 81.- R: 3
en la marcha la da la ataxia y no estas anomalas. Una prdida de fuerza de instauracin brusca sugiere un origen
Suele presentarse diabetes, pero prcticamente nunca hay altera- vascular. Las prdidas de fuerza asociadas a crisis epilpticas aparecen
cin de las funciones superiores. tras una crisis motora (parlisis postcrtica o de Todd) o en las crisis
atnicas, que son generalizadas.
Pregunta 79.- R: 3 Cuando sospechamos un cuadro vascular, se trata de establecer el
La esclerosis lateral amiotrfica es la enfermedad degenerativa de rea de lesin dependiendo de los sntomas: un cuadro motor puede
la motoneurona ms frecuente. Hay que sospecharla ante un pacien- aparecer por diversas lesiones. La desviacin conjugada de los ojos se
te con clnica de prdida de fuerza, progresiva, subaguda, y que en la presenta en lesiones frontales y en lesiones protuberanciales, diferen-
exploracin muestre en unas reas corporales signos de prdida de cindose (entre otros datos) en que los ojos miran hacia el hemicuerpo
motoneurona superior (reflejos vivos, respuesta cutneo-plantar partico en el caso de las lesiones protuberanciales y hacia el sano en
las frontales.
M exico A rgentina CTO Medicina C/ Nez de Balboa, 115 28006 MADRID (Espaa) Tfno.: (91) 782 43 32 / Fax: (91) 782 43 27
Pg. 18 NR C hile U ruguay E-mail: secretaria@ctomedicina.com; iberocto@ctomedicina.com WEB: www.ctomedicina.com; www.iberocto.com
NEUROLOGA
Preparacin Examen de Seleccin 05/06 1 Vuelta Seguimiento a distancia
En este caso, la presencia de hemianopsia asociada al defecto motor inters, mutismo acintico) o con desinhibicin (moria; menos fre-
automticamente nos debe hacer pensar en una lesin hemisfrica, y cuentemente).
nos permite descartar un proceso lacunar, que cursara con un defec- Pueden aparecer los reflejos de liberacin frontal (prensin, suc-
to motor aislado. La exploracin del campo visual en un paciente no cin, palmomentoniano).
colaborador no es sencilla; nos servimos del reflejo de amenaza, que La incontinencia de esfnteres es ms frecuente en las lesiones bilate-
consiste en aproximar rpidamente hacia los ojos un objeto por el rales, aunque tambin puede encontrarse en lesiones unilaterales.
campo visual que queremos valorar. Esto provoca un cierre incons- Un infarto frontal bilateral tambin puede producirse dado que la
ciente de los prpados; si no se produce, deducimos que existe una malformacin ms frecuente del polgono de Willis es el origen co-
prdida de visin por esa rea. mn de ambas arterias cerebrales anteriores, faltando en una de ellas
En un paciente del que conocemos como antecedente un IAM el segmento arterial entre la cartida interna y la comunicante ante-
reciente, podemos apostar por una etiologa cardioemblica como rior, lo que cursa de forma asintomtica debido a la suplencia de riego
ms probable si aparecieran en otras opciones infarto en la arteria por esta ltima.
cerebral media por otras causas. Las alteraciones del campo visual de causa vascular pueden estar
Hay que recordar que los mbolos cardiacos tienen como destino originadas por una lesin en territorio de la arteria cerebral media, de
ms frecuente la arteria cerebral media. la coroidea anterior o de la cerebral posterior, pero no de cerebral
anterior.
Pregunta 82.- R: 1 Las lesiones frontales bilaterales pueden cursar con un cuadro cl-
La arteria cerebral anterior irriga la porcin medial de los lbulos nico que semeja el de la hidrocefalia a presin normal, con apraxia
frontal y parietal. Por lo que respecta a las funciones sensitiva y moto- de la marcha, incontinencia urinaria y deterioro de funciones supe-
ra, un infarto en este territorio provoca una afectacin contralateral riores.
de predominio crural; en el caso del dficit sensitivo ser de sensibili-
dades combinadas, como corresponde a un dficit cortical, y no Pregunta 83.- R: 3
cursar con anestesia ni con prdida de sensibilidades aisladas. Ante una hemianopsia de instauracin brusca, con lo cual, de
probable origen vascular, es el resto de sntomas asociados lo que nos
da el diagnstico.
Si la hemianopsia no se acompaa de otros dficits, lo ms proba-
ble es que sea en territorio de la arteria cerebral posterior. En los casos
de lesin del territorio completo de esta arteria, aparece hemianopsia
y hemianestesia, por lesin talmica, como sucede en este caso.
Si la hemianopsia se acompaa de hemiparesia y hemihipoestesia,
podemos estar ante una lesin de arteria cerebral media o de coroidea
anterior. La primera es ms frecuente, presenta un dficit de predomi-
nio faciobraquial y probablemente se acompaar de desviacin
oculoceflica, mientras que la coroidea da una hemiparesia-
hemihipoestesia completas, sin desviacin de la mirada.
En el caso de la pregunta, la opcin de la cerebral media est
prcticamente descartada al conservar el paciente un lenguaje nor-
mal, dado que el dficit visual es derecho, y por lo tanto la lesin es
izquierda: un paciente con lesin de cerebral media que le produce
hemianopsia asociara trastornos del lenguaje.
Dado que nuestra sospecha clnica es isquemia en lbulo occipital y
tlamo, la prueba de imagen de eleccin ser una TAC (la RM es en
efecto para fosa posterior, pero no es este caso) que si aparece normal en
un primer momento, ser porque an no se han producido cambios
estructurales tras la isquemia, lo que sucede pasadas las primeras horas.
El mecanismo ms frecuente de oclusin del flujo sanguneo en el
Comentarios TEST

territorio de la arteria cerebral posterior es el emblico, y aunque en


ocasiones estas arterias pueden originarse a partir de las comunicantes
posteriores, lo ms frecuente es que se originen en la bifurcacin de la
basilar, por lo que la patologa ateromatosa carotdea que pueda estar
asociada no justifica isquemia en este territorio.

Pregunta 84.- R: 4
Atencin a la descripcin clnica del cuadro, porque no es muy
detallada y hay que considerar distintos datos:
Dificultad para sostener cosas con la mano: esto podra correspon-
der a una paresia, o bien a una ataxia, con torpeza por incoordina-
cin, y sta a su vez de etiologa cerebelosa o sensitiva (lesin talmica
o en cordones posteriores).
Dificultad para hablar: esto puede referirse a una afasia motora
(con dificultad para emitir lenguaje) o a una disartria, tambin con
dificultad para la emisin, pero por mala articulacin, sea de causa
cerebelosa o por algn dficit motor. Quiz tambin una disfona
podra describirse de esta manera, pero parece ms apropiado
que para una disfona se refieran alteraciones en la voz.

Pregunta 82. A) Territorios vasculares cerebrales. B) Vascularizacin del tronco Los antecedentes en los que se describen distintos factores de ries-
del encfalo y del polgono de Willis. go cerebrovascular no se corresponden con un cuadro vascular, de
inicio brusco.
Por afectacin de reas prefrontales se puede acompaar de tras- La exploracin describe una desviacin de la comisura hacia la
tornos del comportamiento, bien con inhibicin (abulia, apata, des- derecha, esto es, una paresia facial izquierda. El hecho de encontrar

M exico A rgentina CTO Medicina C/ Nez de Balboa, 115 28006 MADRID (Espaa) Tfno.: (91) 782 43 32 / Fax: (91) 782 43 27
C hile U ruguay E-mail: secretaria@ctomedicina.com; iberocto@ctomedicina.com WEB: www.ctomedicina.com; www.iberocto.com
NR Pg. 19
NEUROLOGA
Seguimiento a distancia Preparacin Examen de Seleccin 05/06 1 Vuelta
dficit de fuerza facial apunta que la dificultad del habla ser una disartria para determinar con la mayor exactitud posible el grado de este-
motivada por esta debilidad. Si es capaz de cerrar el ojo izquierdo es nosis. Probablemente dentro de poco se pueda determinar el gra-
que no es una parlisis completa, es decir, tiene una paresia facial do con ms exactitud en angiografas no convencionales (angio-
izquierda central (o supranuclear), no nuclear ni perifrica. TC, angio-RM) pero de momento la prueba gold standard es la
Una vez etiquetado el dficit en el lenguaje como disartria por arteriografa de troncos supraarticos.
paresia facial, la dificultad con la mano izquierda parece menos pro- Arteriografa: con el grado de estenosis de certeza, caben dos op-
bable que se trate de una ataxia (ya que el habla no es por lesin ciones: por encima del 70% de estenosis hay indicacin de inter-
cerebelosa y se describe una exploracin sensitiva normal), y pensa- vencin (endarterectoma carotdea); con menos estenosis est in-
remos antes en una paresia. dicado mantener la antiagregacin.
Tenemos un cuadro tpico que asocia dificultad para hablar y
dificultad con una mano: la disartria - mano torpe, causada por una En estenosis carotdeas altas pero menores del 70%, sintomticas,
infarto lacunar capsular, en este caso derecho, puesto que el dficit es hay dudas sobre qu hacer, pero la anticoagulacin slo se contem-
izquierdo. pla en cartidas que estn a la espera de ciruga, no como tratamiento
De las otras opciones, quiz merezca la pena recordar que los de mantenimiento a medio plazo.
hematomas pueden cursar clnicamente con un sndrom,e lacunar, La angioplastia carotdea como sustitucin endovascular de la
siempre que tengan la misma localizacin que esperamos del infarto endarterectoma abierta est dando buenos resultados, pero an es
lacunar: un hematoma en cpsula interna derecha podra dar este una tcnica pendiente de consolidarse.
cuadro clnico, pero no el hematoma descrito en la primera opcin.
Pregunta 87.- R: 1
Pregunta 85.- R: 2 La etiologa de los sangrados intracraneales puede sugerirse por la
La TC es la prueba de imagen de eleccin en patologa cerebrovas- localizacin de los mismos:
cular en el momento agudo. Ya sabemos que cuanto ms precoz sea, Los hematomas epidurales se producen por rotura de ramas arte-
hay menos posibilidades de ver reas isqumicas, ya que la riales dependiendo de la arteria menngea media: nunca son es-
hipodensidad del parnquima refleja el grado de edema y destruc- pontneos.
cin del tejido, que no empieza inmediatamente, por lo que es posi- Los hematomas subdurales son por rotura de venas subdurales;
ble encontrar una TC normal cuando en realidad hay infarto. Pero lo generalmente traumticos, pero a veces, pueden aparecer sin ante-
que nos interesa en el momento agudo es descartar que se trate de un cedente claro, sea por el tiempo de latencia desde el traumatismo
proceso hemorrgico o de etiologa distinta de la vascular, y para ver o por la escasa energa que se precisa y que hace que se olvide.
sangre, la TC es la prueba de imagen de eleccin. Las hemorragias subaracnoideas se producen por rotura de aneu-
Limitaciones del TAC: el momento hiperagudo (explicado antes); rismas saculares (ms frecuente) o malformaciones vasculares (ms
la definicin (lesiones de milmetros de dimetro pueden no distin- frecuentes en jvenes de menos de 20 aos de edad).
guirse en algunos TC, dependiendo de la calidad del aparato) y la fosa Los hematomas intraparenquimatosos lobares, en el lmite entre
posterior, porque en esos cortes se incluye la base del crneo y el la sustancia gris y la blanca de los lbulos hemisfricos, se produ-
hecho de que haya mucho hueso empeora la definicin. cen por rotura de malformaciones vasculares en personas jvenes
Por lo tanto, si se especifica cul es la prueba de eleccin para ver y asociado a rotura de vasos con depsitos amiloides (angiopata
infartos lacunares, es la RM, lo que no quiere decir que la mayor parte congfila) en personas mayores.
no sean visibles tambin con la TC, pero alguno se escapara. Los hematomas profundos se producen por rotura de vasos perfo-
Lo que es visible desde el primer momento son las hemorragias, de rantes; estos vasos sufren de forma crnica debido a los factores de
tal forma que, ante la sospecha de patologa hemorrgica (clnica de riesgo vascular, especialmente la hipertensin y la diabetes, y su
HSA, subdural, antecedentes de hematomas espontneos previos) la rotura puede relacionarse con un aumento de presin arterial que
prueba de imagen de eleccin sera la TC. a veces no es excesivo, si no fuera porque se parte de una situacin
Las reas hipodensas (correspondientes a infarto) suelen dibujar previa delicada. Dentro de los territorios profundos, la localizacin
bien el territorio vascular de la arteria obstruida: son infartos en cua, ms frecuente son los ganglios basales (putamen, caudado, tla-
porque generalmente suelen tener ms trozo de corteza que de rea mo), tronco del encfalo y cpsula interna.
subcortical. Hay procesos en que la distribucin del infarto no corres-
ponde con un territorio vascular arterial: Pregunta 88.- R: 4
Los infartos venosos, en los que se produce isquemia por dificultad La angiopata amiloide es un proceso de personas mayores, en las

Comentarios TEST
en el retorno venoso, y las reas hipodensas no tiene un patrn que se forman depsitos de material amiloide en las paredes arteriolares.
comn en todos los pacientes; y Puede asociarse a la Enfermedad de Alzheimer, que tambin presenta
Los infartos hemodinmicos: cuando no hay presin suficiente, se depsitos de amiloide extravasculares. La rotura de estas arteriolas de
produce una isquemia en las reas ms distales de los vasos afecta- pared rgida por estos depsitos origina hematomas lobares, en el lmite
dos, que suelen corresponder a los mrgenes de los territorios vas- cortico-subcortical de los hemisferios. No tiene tratamiento, porque los
culares (denominados territorios frontera). depsitos ya no se movilizan, y su curso es presentar hematomas espon-
tneos de repeticin en esas localizaciones.
Pregunta 86.- R: 4 Las malformaciones vasculares, teniendo en cuenta su distinta natu-
El episodio descrito corresponde a una amaurosis fugaz del ojo raleza, disposicin, tamao y localizacin, pueden ser la causa de san-
izquierdo, lo que apunta directamente a patologa carotdea izquier- grados subaracnoideos o intraparenquimatosos (superficiales o profun-
da. En el fondo de ojo se podran haber apreciado, quiz, cristales de dos). Son ms frecuentes en pacientes jvenes, aunque pueden debu-
colesterol, resto del mbolo fragmentado para acabar de completar la tar a cualquier edad. El pronstico, tanto de recurrencias como de un
definicin. primer sangrado, ante el hallazgo de una malformacin en principio
El episodio de hormigueos en la mano derecha de das antes es asintomtica, debe establecerse de forma individualizada.
muy posible que corresponda a un ataque isqumico transitorio he- Los aneurismas saculares se forman en las bifurcaciones vascula-
misfrico izquierdo, adonde podra ir un fragmento de trombo que se res, ms frecuentemente en arterias del polgono de Willis, y dentro
desprendiera de la cartida izquierda. de ste son ms frecuentes en los vasos que dependen de las car-
Luego estamos ante un paciente con dos AIT que suponemos de tidas (la localizacin ms frecuente: bifurcacin cerebral anterior
origen aterotrombtico carotdeo izquierdo. comunicante anterior). Se denominan congnitos, aunque lo que
El proceso diagnstico a seguir con una cartida sintomtica es el en realidad lo es la debilidad en las paredes arteriales en esas bifur-
descrito: caciones, que con los aos dan lugar a las dilataciones saculares.
Eco-doppler de TSA: si fuera normal o sugerente de estenosis no Distintos son los aneurismas fusiformes: su localizacin tpica es la
significativa, se antiagregara ante el cuadro clnico. Si sugiere este- arteria basilar, y se pueden considerar una dilatacin anmala de la
nosis significativa, estara indicada la realizacin de una angiografa misma, que no se rompe ni sangra, y que puede dar problemas como

M exico A rgentina CTO Medicina C/ Nez de Balboa, 115 28006 MADRID (Espaa) Tfno.: (91) 782 43 32 / Fax: (91) 782 43 27
Pg. 20 NR C hile U ruguay E-mail: secretaria@ctomedicina.com; iberocto@ctomedicina.com WEB: www.ctomedicina.com; www.iberocto.com
NEUROLOGA
Preparacin Examen de Seleccin 05/06 1 Vuelta Seguimiento a distancia
otra lesin ocupante de espacio a ese nivel. La indicacin de interve- gico, con lo que puede responder a terapias en ese sentido
nir depende del tamao. (gammaglobulinas, plasmafresis, corticoides). En este contexto estara
La hipertensin de forma crnica es la causa ms frecuente de la forma ms frecuente de afectacin en el comienzo de la infeccin:
hemorragia intraparenquimatosa, y su localizacin ms habitual es en la polineuropata desmilinizante aguda, que recuerda a un sndrome
territorios vasculares de arterias perforantes, cuyas paredes son las que de Guillain-Barr.
ms sufren la presin arterial elevada. En fases avanzadas predominan las neuropatas carenciales y txi-
cas, y la ms frecuente es la polineuropata sensitiva simtrica distal,
Pregunta 89.- R: 2 con distribucin en guante y calcetn, que probablemente es de
Cuidado con la nomenclatura! Lo importante es tener los con- causa multifactorial.
ceptos claros: En fases avanzadas puede aparecer tambin la polirradiculopata
El mecanismo de gnesis de trombo a nivel vascular es por la agre- que se asocia a la infeccin concomitante por CMV, que se puede
gacin de plaquetas y fibrina sobre una estenosis previa, que gene- demostrar en la biopsia, y que puede responder al ganciclovir. Tam-
ralmente est a su vez condicionada por el depsito de colesterol. bin el VIH de forma directa puede causar una polirradiculitis.
A este fenmeno le podemos denominar atero- (placa de ateroma) La amiotrofia proximal en miembros inferiores es un cuadro tpico
trombosis. Lo ms frecuente es que el trombo formado sobre la dentro de las afectaciones del sistema nervioso perifrico asociada a la
placa de ateroma no obstruya el vaso en ese punto (aunque esto diabetes mellitus, en especial la tipo 2. Cursa con dolor en raz de
puede suceder), sino que el agregado se libere y circule hasta obs- miembros inferiores, seguido de prdida de fuerza y de masa muscu-
truir un vaso distalmente: a este fenmeno lo podemos llamar lar, y puede ser reversible si se trata de forma agresiva.
embolia artero-arterial o aterotromboembolia, pero el mecanismo
de gnesis es la aterotrombosis, y el tratamiento debe ir encamina- Pregunta 92.- R: 4
do a evitar la aterotrombosis. La diabetes mellitus es responsable de un gran nmero de patolo-
El mecanismo de formacin de un cogulo es la activacin de gas en el sistema nervioso perifrico, mediadas por microangiopata y
factores de coagulacin generalmente en una zona vascular de microinfartos en los nervios perifricos.
flujo lento (venas profundas, corazn) y la isquemia cerebral slo Los patrones de afectacin del sistema nervioso perifrico por cul-
puede hacerlo mediante su circulacin y la oclusin distal. pa de la DM son bsicamente de tres tipos:
Mononeuropatas: son caractersticas las mononeuropatas cra-
En la prctica, a la hora de abordar el diagnstico etiolgico de un neales, aunque tambin pueden aparecer en extremidades. La
proceso isqumico cerebral en un paciente, no se suele plantear a ms frecuente es la parlisis incompleta del III par (con conserva-
priori la causa, ya que hay que completar el estudio con pruebas en cin de la actividad pupilar). Generalmente tienen buen pronsti-
ambos sentidos. co y se recuperan sin secuelas.
Ante el hallazgo de patologa cardiaca embolgena, lo ms frecuen- Neuropata motora proximal (denominada amiotrofia diabtica):
te es que el ictus haya tenido un origen emblico cardiaco, y ante la generalmente en DM tipo 2, presentan dolor en miembros inferio-
existencia de lesiones carotdeas estenosantes, lo ms probable es que res, junto con prdida de fuerza y de masa muscular, de predomi-
se trate de un mecanismo aterotrombtico. nio proximal. Estos pacientes suelen responder al tratamiento
Existen ictus indeterminados, cuando el paciente tiene ambas co- insulnico, incluso aunque no tuvieran un mal control con antidia-
sas, o cuando no se encuentra justificacin al proceso. Estadsticamente bticos orales, y recuperan en gran medida los sntomas. Se ha
la mayor parte de los procesos cerebrovasculares isqumicos tienen postulado que esta forma de neuropata tendra un mecanismo
en su gnesis mecanismos aterotrombticos con suelta de trombos a distinto al del resto de las polineuropatas asociadas a la DM
distancia, por eso es correcta la opcin 2. Neuropata autonmica: hipotensin ortosttica, alteraciones de
la sudoracin, vejiga neurgena, diarreas y disautonoma cardiaca,
SISTEMA NERVIOSO PERIFRICO. con trastornos del ritmo y posible muerte sbita.
Polineuropata difusa: existe alteracin de las fibras sensitivas peri-
Pregunta 90.- R: 1 fricas, con alteracin de las fibras finas (dolor; parestesias, quema-
El estudio neurofisiolgico es bsico para determinar las alteracio- zn), las fibras gruesas (alteraciones en la posicin, pie neuroptico
nes estructurales bsicas del nervio perifrico: fundamentalmente por torceduras recurrentes) y las autonmicas (trastornos trficos
encaminado a distinguir las afectaciones axonales de las desmielini- cutneos, lceras). Es la forma ms frecuente y generalmente pre-
Comentarios TEST

zantes. senta una mezcla de estos sntomas en distintos grados, con una
La afectacin axonal produce una disminucin de la amplitud del distribucin en guante y calcetn.
potencial nervioso, que en principio se conducir con una velocidad
normal si se conserva la mielina. Salvo escasas excepciones, el tratamiento intensivo de la diabetes
La afectacin desmielinizante produce un enlentecimiento de las no consigue revertir sintomatologa, y lo que ms puede hacer es
velocidades de conduccin; en ocasiones el enlentecimiento se pro- prevencin.
duce en puntos concretos en los que se detecta una dificultad en la
conduccin desproporcionadamente marcada respecto al resto (blo- Pregunta 93.- R: 5
queo de conduccin). La desmielinizacin produce otro efecto aa- El diagnstico de sndrome de Guillain-Barr es eminentemente
dido: el potencial que conduce el nervio generalmente es nico, clnico, ante la presencia de una debilidad ascendente con abolicin
mientras que si la conduccin se produce por un segmento de los reflejos de estiramiento muscular, incluso en reas corporales
desmielinizado, las velocidades de conduccin son variables, detec- en las que an no hay debilidad significativa.
tndose distintos potenciales en lugar de uno solo tras un estmulo Se produce una desmielinizacin muy proximal en las races ante-
(dispersin). riores medulares (las races motoras), con lo que el cuadro es casi
La mayor parte de las polineuropatas de causa metablica tienen nicamente motor, pero no obstante pueden aparecer sntomas de
afectacin mixta axonal y desmielinizante, con disminucin de las otros sistemas (sensitivo, vegetativo), siempre en menor intensidad que
velocidades de conduccin y de las amplitudes. la paresia.
El curso clsico es ascendente, aunque existe el denominado sn-
Pregunta 91.- R: 4 drome de Miller-Fisher, en el que predomina la debilidad superior
Las manifestaciones neurolgicas asociadas al VIH son mltiples y (craneofacial). En el Guillain-Barr clsico, la musculatura que menos
no pocas corresponden al nervio perifrico. Pueden producirse por se afectar ser la ocular, aunque en los casos ms graves tambin
afectacin directa debida al propio virus, por infecciones oportunis- puede verse afectada.
tas, o por efectos secundarios del tratamiento. La prueba complementaria que establece el diagnstico son los
De forma general, en las fases iniciales de la infeccin, la afectacin hallazgos neurofisiolgicos: el estudio del nervio perifrico no suele
del sistema nervioso perifrico suele tener un mecanismo inmunol- mostrar hallazgos patolgicos, dado que la desmielinizacin es muy

M exico A rgentina CTO Medicina C/ Nez de Balboa, 115 28006 MADRID (Espaa) Tfno.: (91) 782 43 32 / Fax: (91) 782 43 27
C hile U ruguay E-mail: secretaria@ctomedicina.com; iberocto@ctomedicina.com WEB: www.ctomedicina.com; www.iberocto.com
NR Pg. 21
NEUROLOGA
Seguimiento a distancia Preparacin Examen de Seleccin 05/06 1 Vuelta
proximal, pero hay una onda (denominada onda F) que se detecta en muscular es precisamente la miotona de predominio en manos, que
el nervio perifrico tras un estmulo que recorre el nervio hacia atrs y no suele ser incapacitante, que mejora con el calentamiento (tras unas
de nuevo hacia adelante, y que est abolida en el sndrome de Guillain- pocas contracciones voluntarias) y empeora en los das fros.
Barr, debido a la lesin. El estudio neurofisiolgico es, por otra parte, Con la progresin de la enfermedad se afecta la musculatura facial,
el primero en ser positivo. de la masticacin, del cuello, antebrazos y piernas. Pero se trata de
En el LCR se puede encontrar un aumento de protenas, sin que una enfermedad sistmica, con lo que el paciente puede tener disfa-
haya aumento de clulas inflamatorias (disociacin albmino-citolgi- gia, trastornos esfinterianos, hipogonadismo, trastornos de conduc-
ca) porque est producido por la desmielinizacin. Es sugerente del cin cardiaca (en el 70% de los pacientes), alteraciones valvulares
diagnstico, pero si no se encuentra no lo descarta. cardiacas y deterioro cognitivo progresivo.
El aspecto de estos pacientes es caracterstico: ptosis palpebral,
Pregunta 94.- R: 4 sonrisa invertida con boca entreabierta (por debilidad facial), voz
Ante un cuadro clnico tpico de sndrome de Guillain-Barr (in- gangosa, cataratas y calvicie.
feccin previa, arreflexia, debilidad de curso ascendente, ausencia de El diagnstico se establece por la clnica, el estudio electromiogrfico
sntomas no motores) debera establecerse el diagnstico con estudio (que muestra un patrn mioptico con descargas miotnicas), y la biopsia
neurofisiolgico y/o con estudio del LCR. muscular, que muestra en efecto atrofia de fibras tipo I. La CPK est
Establecido el diagnstico, deberamos recordar que se trata de un normal o levemente elevada. El estudio gentico (cromosoma 19) es
proceso autoinmune en el que circulan autoanticuerpos contra la diagnstico.
mielina del sistema nervioso perifrico. Cuando tenemos clnica es que No hay tratamiento salvo el sintomtico, intentando evitar infec-
se ha producido la sntesis de dichos anticuerpos, en el tiempo de ciones intercurrentes; control de trastornos cardiacos, con marcapa-
latencia entre el proceso infeccioso y el debut del cuadro neurolgico. sos si es preciso, y la miotona, que rara vez requiere tratamiento,
Los corticoides y otros inmunosupresores pueden disminuir e in- puede controlarse con fenitona o procainamida.
cluso frenar la sntesis de anticuerpos, pero en el caso del Guillain-
Barr no son eficaces, porque a la aparicin de los sntomas ya se han Pregunta 97.- R: 4
producido los anticuerpos; por tanto, los corticoides no son eficaces. La miastenia gravis se produce por un bloqueo de los receptores de
La medida ms til es retirar de la circulacin los anticuerpos cir- acetilcolina de la placa muscular. Este bloqueo no es completo, en
culantes: hay varios sistemas, siendo el clsico la plasmafresis (sustitu- cuyo caso el paciente no podra realizar ningn movimiento, pero
cin del suero por un compuesto artificial), pero hay otros sistemas impide el normal funcionamiento de la placa neuromuscular, que se
menos agresivos para el paciente e igualmente eficaces (inmunoad- hace ms evidente a medida que se repite la transmisin de potenciales.
sorcin, administracin de gammaglobulina policlonal). Estos mto- Este empeoramiento progresivo relacionado con la llegada repetida de
dos son tanto ms eficaces cuanto ms precoces, ya que estn inten- estmulos tiene su correlato clnico en la fatigabilidad: la fuerza y los
tando impedir la gnesis de ms lesin. En fases avanzadas no consi- reflejos estn inicialmente conservados, pero una repeticin del acto
guen beneficios. motor va empobreciendo la transmisin y aparece la debilidad.
La medida que ms ha influido en la supervivencia de estos enfer- Este mecanismo se objetiva con el estudio neurofisiolgico: el pri-
mos es el soporte vital, dado que, al tratarse de un proceso desmielini- mer potencial de accin es normal (como lo son los reflejos, un solo
zante con poco dao axonal, puede producirse una remielinizacin estmulo) pero la estimulacin repetitiva (como si se hiciera un ejerci-
si el paciente no fallece por complicaciones asociadas (dificultad res- cio) conlleva una prdida de la intensidad de la transmisin (respuesta
piratoria o en la deglucin). decremental).
No hay toxina involucrada en el sndrome de Guillain-Barr; los Dado que la afectacin es exclusivamente de la placa motora, en
sueros antitoxina son tiles en el botulismo o en el ttanos. un paciente miastnico no deben aparecer signos de otros sistemas
afectados: autonmico, sensitivo, cerebelo... Slo un dficit motor
Pregunta 95.- R: 4 con fatigabilidad; si aparecen otros datos en la exploracin, habr
La distrofia muscular de Duchenne es en efecto una enfermedad que replantear el diagnstico.
ligada al X, con un gen alterado en el brazo corto de dicho cromoso- La distribucin anatmica de la afectacin en la miastenia gravis es
ma, que debera codificar una protena presente en la membrana preferentemente ocular, y si hay una generalizacin a tronco y extre-
muscular, denominada distrofina, y que no existe en estos pacientes. midades, la afectacin es predominantemente proximal.
Hay que pensar en esta enfermedad en un paciente que, a los 2-4
aos de edad, tras un desarrollo normal, empieza a presentar dificul- Pregunta 98.- R: 2

Comentarios TEST
tad para correr, levantarse del suelo y subir escaleras. En ese momento Los anticuerpos responsables de la miastenia gravis se sitan en la
ya puede ser evidente a la exploracin la hipertrofia de pantorrillas, hendidura sinptica, en la membrana muscular, sobre los receptores
por sustitucin del msculo por grasa. de acetilcolina.
El paciente tendr una marcha dificultosa, bambolendose hacia Cuando un receptor de acetilcolina es bloqueado por anticuer-
los lados por insuficiencia muscular en cintura plvica, y para levan- pos, queda inutilizado de forma definitiva, con lo que tiene que ser
tarse del suelo lo tiene que hacer apoyndose. En torno a los 10 aos reabsorbido y sustituido. Este recambio se hace por recaptacin de
de edad se es incapaz de caminar. No hay tratamiento; la muerte se segmentos de membrana en los que van incluidos los receptores, por
produce alrededor de los 20 aos por problemas respiratorios; slo lo que, si el recambio es muy intenso porque haya muchos receptores
en un 10 % de los casos es por problemas cardiacos. afectados, la reposicin no se hace con la misma velocidad, y empie-
El cuadro completo de esta enfermedad incluye la miopata de za a faltar membrana en la sinapsis.
msculo estriado, la miocardiopata y un bajo cociente intelectual. La sinapsis habitualmente tiene pliegues complicados, con el fin de
No hay alteraciones sensitivas, y los reflejos estn abolidos. tener ms superficie donde situar receptores de acetilcolina; si se qui-
Es especialmente importante el diagnstico de mujeres portadoras, ta mucha membrana, queda poco espacio para receptores, con lo
dado que la mayor parte son asintomticas (la coincidencia de los dos que el funcionamiento de la sinapsis no vuelve a ser correcto por
cromosomas X anmalos es letal intratero). Un estudio que incluya cambios estructurales y no slo por el bloqueo de los receptores.
EKG, CPK seriadas, biopsia muscular y una buena historia familiar, La miastenia se asocia frecuentemente con alteraciones tmicas,
permite asegurar el diagnstico de portadora al 95%; tambin puede siendo la ms frecuente la hiperplasia, pero en un 10% de los casos se
realizarse el estudio gentico de los cromosomas X. asocia a franca tumoracin de origen tmico, lo que automticamente
convierte la reseccin del timo en una medida indicada. En los otros
Pregunta 96.- R: 4 casos en los que se detecta bien una hiperplasia o bien un timo de
Hay que tener presente el diagnstico de un paciente que debute en apariencia normal (el hecho de que se detecten restos tmicos en una
la edad adulta con fenmeno miotnico y debilidad facial y en manos persona adulta hay quien lo considera patolgico, puesto que debe
de forma progresiva, dado que la distrofia miotnica de Steinert es la atrofiarse antes) la indicacin quirrgica la da el cuadro clnico: de-
distrofia ms frecuente de la edad adulta, y la primera manifestacin but precoz, mala respuesta al tratamiento mdico, evolucin rpida.

M exico A rgentina CTO Medicina C/ Nez de Balboa, 115 28006 MADRID (Espaa) Tfno.: (91) 782 43 32 / Fax: (91) 782 43 27
Pg. 22 NR C hile U ruguay E-mail: secretaria@ctomedicina.com; iberocto@ctomedicina.com WEB: www.ctomedicina.com; www.iberocto.com
NEUROLOGA
Preparacin Examen de Seleccin 05/06 1 Vuelta Seguimiento a distancia
Pregunta 99.- R: 3 curso de semanas. A partir de esa edad es excepcional que haya una
El diagnstico de miastenia gravis debe sospecharse ante una cl- colonizacin por la bacteria, por los cambios del pH gstrico y la flora
nica sugerente (opcin 1). Toda debilidad fluctuante es sugerente de bacteriana, por lo que en nios mayores y adultos el cuadro se pro-
miastenia gravis, como causa ms frecuente de sndrome miastenifor- duce por ingesta de toxina preformada: con una latencia que va de
me. Si el paciente es un varn, mayor, con tumor conocido o sndro- horas a escasos das (media de 48 horas) aparecen los sntomas: pupi-
me constitucional, quiz habr que pensar primero en un sndrome las fijas, boca seca, leo paraltico y debilidad generalizada, de predo-
de Eaton-Lambert. minio en musculatura craneal (oftalmoplejia externa, ptosis, debili-
Ante la sospecha clnica existen una serie de pruebas complemen- dad facial) y bulbar (disfagia, disartria, disfona) pero que puede llegar
tarias que confirman el diagnstico: a ser generalizada.
Test de edrofonio: una recuperacin inmediata y transitoria de La latencia del botulismo adquirido por heridas va de 4 a 51 das,
fuerza tras la administracin de este anticolinestersico de vida ms prolongada que la de la va digestiva.
media corta i.v. apoya la sospecha de que la debilidad la cause un
bloqueo en la placa motora.
Estimulacin repetitiva: el estudio basal es normal, pero puede
apreciarse como los potenciales de accin disminuyen de tamao
cuando se somete al msculo a una salva de estmulos.
Estudio de fibra aislada: con un electrodo especial, se valoran los
tiempos de latencia entre el estmulo y el potencial de accin mus-
cular en una misma fibra muscular; en una persona normal, el
tiempo de latencia permanece constante, mientras que si existe
bloqueo de la placa motora, se detecta amplia variabilidad en esos
intervalos de tiempo.
Deteccin de anticuerpos antirreceptor de acetilcolina: no es es-
pecfico al 100%, dado que pueden aparecer asociados a otros
tipos de miastenia (inducida) o a otras patologas autoinmunes.

Ninguna de estas pruebas complementarias es diagnstica per se, y


el hecho de encontrar alguna de ellas negativa no descarta el diagns-
tico de miastenia gravis.
Comentarios TEST

Pregunta 99. Tratamiento de las distintas formas de miastenia gravis.

Pregunta 100.- R: 5
El botulismo es un cuadro mediado por la toxina botulnica, que
bloquea la liberacin de acetilcolina de las terminales presinpticas.
No atraviesa la barrera hematoenceflica, por lo que la transmisin
colinrgica central no se ve afectada.
Hay dos vas por las que se puede adquirir la toxina: por heridas
contaminadas (no una herida simple, sino profunda, o con muy mal
curso clnico, dado que la produccin de la toxina se produce en
medios anaerobios) o digestiva, con paso de toxina a travs de la
pared del tubo digestivo.
La va digestiva es la ms frecuente. En lactantes (menos de un ao)
que ingieran esporas de C. botulinum, stos pueden proliferar en el
tracto digestivo y producir toxina; el cuadro clnico es progresivo en el

M exico A rgentina CTO Medicina C/ Nez de Balboa, 115 28006 MADRID (Espaa) Tfno.: (91) 782 43 32 / Fax: (91) 782 43 27
C hile U ruguay E-mail: secretaria@ctomedicina.com; iberocto@ctomedicina.com WEB: www.ctomedicina.com; www.iberocto.com
NR Pg. 23
NEUROCIRUGA
Preparacin Examen de Seleccin 05/06 1 Vuelta Seguimiento a distancia

HIPERTENSIN INTRACRANEAL E HIDROCEFALIA 1) Carcinoma microctico de pulmn.


2) Vasculopata hipertensiva.
1. Ante un caso de HTIC en fase avanzada, sera MENOS frecuente 3) Enfermedad de Dandy-Walker.
la aparicin de: 4) Quiste coloide.
5) Tumor de clulas germinales.
1) Alteraciones del patrn respiratorio.
2) Ondas A o "plateau" en el registro de PIC. 7. Un anciano de 70 aos presenta un cuadro consistente en
3) Lesin del III par craneal. trastornos mnsicos, incontinencia urinaria y torpeza al cami-
4) lceras de Cushing. nar que ha ido apareciendo a lo largo de varios meses. El
5) Taquicardia supraventricular. diagnstico ms probable de entre los siguientes es:

2. Un varn de 24 aos acude a Urgencias por crisis epilptica de 1) Encefalopata de Binswanger.


pocos segundos de duracin y cefalea hemicraneal. A su ingreso 2) Hidrocefalia crnica del adulto.
se realiza TC cerebral, que muestra un hematoma en lbulo 3) Enfermedad de Alzheimer.
temporal derecho, con efecto de masa y abierto a ventrculo 4) Carcinoma de prstata con metstasis cerebrales.
lateral. Indique el hallazgo MENOS probable de entre los siguien- 5) Mielopata cervical espondiloartrsica.
tes en relacin con este paciente:
8. Un paciente de 5 aos de edad, con derivacin ventriculope-
1) Anisocoria. ritoneal colocada a los pocos das del nacimiento por hidroce-
2) Debilidad motora en extremidades izquierdas. falia secundaria a estenosis del acueducto de Silvio, ha estado
3) Malformacin arteriovenosa como patologa de base. bien hasta hace 2 das, cuando comenz con un cuadro de
4) Hidrocefalia. cefalea, vmitos y disminucin del nivel de conciencia. Cul de
5) Defecto campimtrico inferior izquierdo. estas medidas NO sera adecuada?:

3. Una paciente de 30 aos acude a urgencias por cuadro de dolor 1) Realizar una TC craneal para descartar hidrocefalia por
occipitocervical y vrtigo de aparicin brusca, acompaado de malfuncin valvular.
vmitos e inestabilidad en la marcha. Tras pocas horas en 2) Estudio del fondo de ojo.
observacin, presenta deterioro progresivo en el nivel de 3) Pedir radiografa del trayecto valvular para observar una
conciencia. Se realiza TAC cerebral que muestra una hipoden- posible rotura.
sidad difusa en hemisferio cerebeloso izquierdo, con efecto de 4) Iniciar tratamiento con dexametasona.
masa y compresin del IV ventrculo, originando herniacin 5) Ingreso hospitalario para observacin.
transtentorial inversa e hidrocefalia obstructiva. Indique el
diagnstico etiolgico ms probable de entre los siguientes:
TUMORES
1) Ateromatosis carotdea.
2) Absceso cerebeloso de origen otolgico. 9. Seala lo FALSO en relacin a las neoplasias cerebrales secun-
3) Enfermedad de Lermitte-Duclos. darias o metastsicas:
4) Astrocitoma piloctico de cerebelo.
5) Diseccin de arteria vertebral. 1) Suponen la neoplasia intracraneal ms frecuente.
2) Son ms frecuentes a nivel supratentorial.
4. Un varn de 35 aos viene sufriendo cefaleas, vmitos, visin 3) El primario sistmico ms frecuente es el carcinoma bronco-
borrosa y diplopa en varias ocasiones desde hace pocas semanas. pulmonar.
La exploracin fsica evidencia lesiones aftosas en mucosa oral y 4) La tendencia al sangrado intralesional es mayor que en los
en glande. Fondo de ojo: edema de papila. TC cerebral sin tumores primarios del SNC.
hallazgos. Que diagnstico le parece ms probable?: 5) El tratamiento es fundamentalmente quirrgico en la mayora
Preguntas TEST

de estas lesiones.
1) Sndrome de Reiter.
2) Enfermedad de Behet. 10. Un varn de 55 aos presenta una clnica de hemiparesia
3) Sfilis terciaria. izquierda y deterioro cognitivo de un mes de evolucin. En el
4) Infeccin por criptococo. estudio RM se observa una lesin expansiva cerebral frontal
5) Dficit del complejo vitamnico B. derecha, subcortical, que alcanza cuerpo calloso, de morfolo-
ga irregular, captacin de contraste en anillo y edema perile-
5. La primera medida teraputica a considerar en una paciente sional. Analtica general y Rx de trax sin hallazgos patolgicos.
diagnosticada de "pseudotumor cerebri" idioptico sera: Diagnstico ms probable:

1) Administracin de corticoides. 1) Linfoma.


2) Fenestracin del nervio ptico. 2) Glioblastoma.
3) Realizacin de punciones lumbares repetidas. 3) Toxoplasmosis.
4) Derivacin lumboperitoneal. 4) Metstasis.
5) Restriccin hidrosalina y diurticos. 5) Absceso.

6. Cul de las siguientes patologas se puede asociar con hidro- 11. Varn de 50 aos con crisis comiciales de 2 aos de evolucin
cefalia comunicante con mayor probabilidad?: que ltimamente no responden adecuadamente a la medica-

M exico A rgentina CTO Medicina C/ Nez de Balboa, 115 28006 MADRID (Espaa) Tfno.: (91) 782 43 32 / Fax: (91) 782 43 27
C hile U ruguay E-mail: secretaria@ctomedicina.com; iberocto@ctomedicina.com WEB: www.ctomedicina.com; www.iberocto.com
NC Pg. 1
NEUROCIRUGA
Seguimiento a distancia Preparacin Examen de Seleccin 05/06 1 Vuelta
cin. En la TC cerebral aparece una lesin frontal derecha con 2) El tratamiento consiste fundamentalmente en la exresis
zonas calcificadas y que no capta contraste. Cul sera el quirrgica del tumor.
diagnstico y actitud teraputica ms aconsejable?: 3) Hasta un 50% de los casos se asocian a enfermedad de Von
Hippel-Lindau.
1) Glioblastoma: ciruga + RT. 4) La imagen radiolgica ms habitual es una lesin qustica con
2) Ependimoma: vigilancia con RM anuales. un ndulo mural.
3) Oligodendroglioma: ciruga. 5) Puede cursar con alteraciones hematolgicas.
4) Meduloblastoma: ciruga + RT craneoespinal.
5) Metstasis: descartar tumor sistmico. 18. Seale las localizaciones ms frecuentes de los neurinomas
intracraneales:
12. Es FALSO, en relacin al ependimoma intracraneal:
1) VIII par y VII par.
1) Puede diseminarse por el espacio subaracnoideo. 2) VIII par y V par.
2) La localizacin ms frecuente es el suelo del cuarto ventrculo. 3) V par y VI par.
3) Suele debutar con hidrocefalia. 4) VI par y VII par.
4) La variante mixopapilar tpicamente aparece a nivel supra- 5) VIII par y IX par.
tentorial.
5) El dato histolgico tpico son las formaciones en roseta. 19. Un varn de doce aos es estudiado en Endocrinologa por
diabetes inspida y cefaleas. La exploracin neurolgica
13. Sobre los tumores enceflicos en nios, es ERRNEO: pone de manifiesto una leve dificultad en la elevacin de la
mirada. Los niveles sricos de HCG y AFP son normales.
1) Suponen la neoplasia ms frecuente despus de las leucemias. Usted realizara una RM cerebral para descartar principal-
2) El tumor supratentorial ms frecuente es el astrocitoma cere- mente uno de los siguientes diagnsticos:
bral.
3) Suelen debutar con clnica de HTIC. 1) Craneofaringioma.
4) Las metstasis son excepcionales. 2) Glioma hipotalmico.
5) El meduloblastoma suele ser de localizacin vermiana. 3) Colesteatoma en cisternas de la base.
4) Germinoma de regin pineal.
14. La existencia de una tumoracin qustica con calcificacin 5) Macroadenoma hipofisario.
marginal, de localizacin supraselar, en un joven de 16 aos que
desde hace 3 meses refiere prdida de visin en ambos campos 20. Indique lo FALSO sobre el quiste coloide cerebral:
temporales, sugiere el diagnstico de:
1) Se origina en la regin anterior del tercer ventrculo.
1) Teratoma. 2) Es un derivado de la parfisis.
2) Quiste dermoide. 3) Presenta una pseudocpsula de tejido glial.
3) Craneofaringioma. 4) Su contenido es PAS +.
4) Adenoma hipofisario. 5) Puede producir hidrocefalia aguda con el cambio de posi-
5) Astrocitoma qustico hipotalmico. cin de la cabeza.

15. Seale lo FALSO en relacin al linfoma del SNC: 21. La asociacin de hamartomas del iris, pseudoartrosis de la tibia
y glioma de vas pticas es propia de uno de los siguientes
1) Es una neoplasia de clulas B de bajo grado. sndromes neurocutneos:
2) Asocia afectacin menngea al inicio o afectacin ocular en
ms de un 10 % de los casos. 1) Von Hippel-Lindau.
3) Aparece con mayor frecuencia en pacientes trasplantados o 2) Sturge-Weber.
con SIDA. 3) Neurofibromatosis tipo I.

Preguntas TEST
4) En inmunodeprimidos, se asocia invariablemente con una 4) Neurofibromatosis tipo II.
infeccin de las clulas tumorales por el virus de Epstein-Barr. 5) Esclerosis tuberosa.
5) La radioterapia es la base del tratamiento.
22. NO incluye tumores cerebrales una de las siguientes entidades:
16. Seale la opcin FALSA sobre los meningiomas:
1) MEN tipo I.
1) Son ms frecuentes en mujeres en la 5 y 6 dcadas de la vida.
2) Neurofibromatosis tipo I.
2) La localizacin ms frecuente es la base craneal.
3) Enfermedad de Bourneville.
3) A veces presentan alteraciones genticas en el cromosoma 22.
4) Enfermedad de Lafora.
4) La radioterapia es eficaz ante restos o recidivas que progresan.
5) Sndrome de Turcot.
5) Son tumores que derivan de la aracnoides.

17. Cul de las siguientes opciones es FALSA respecto al hemangio-


TRAUMATISMOS CRANEOENCEFLICOS (TCE)
blastoma?:

1) Es el tumor primario intraaxial de fosa posterior ms frecuente 23. Paciente con TCE leve con prdida de conocimiento inicial,
del adulto. herida en cuero cabelludo y fractura lineal en la Rx de crneo.
Cuando usted lo ve, est asintomtico y la exploracin neuro-

M exico A rgentina CTO Medicina C/ Nez de Balboa, 115 28006 MADRID (Espaa) Tfno.: (91) 782 43 32 / Fax: (91) 782 43 27
Pg. 2 NC C hile U ruguay E-mail: secretaria@ctomedicina.com; iberocto@ctomedicina.com WEB: www.ctomedicina.com; www.iberocto.com
NEUROCIRUGA
Preparacin Examen de Seleccin 05/06 1 Vuelta Seguimiento a distancia

lgica es normal. Cul de las siguientes actitudes es la ms HEMORRAGIA SUBARACNOIDEA (HSA)


indicada?:
28. En relacin con la hemorragia subaracnoidea, es FALSO de
1) Alta a su domicilio. entre las siguientes opciones:
2) TC craneal y, si es normal, alta.
3) TC craneal, para descartar lesiones asociadas e ingresar 24 1) Su etiologa no traumtica ms frecuente es la rotura de un
horas para observacin. aneurisma cerebral.
4) TC craneal y ciruga de la fractura, realizando fijacin interna. 2) Se detectan aneurismas mltiples en menos de un 5% de los
5) TC craneal y, si es normal, dar antibiticos profilcticos por casos.
el riesgo elevado de infeccin intracraneal. 3) Los aneurismas fusiformes suelen aparecer en circulacin
vertebrobasilar.
24. Seale la FALSA respecto a las fracturas del peasco: 4) La tcnica de eleccin para el diagnstico de HSA es la TC
cerebral.
1) La fractura longitudinal puede afectar al nervio facial en hasta 5) La isquemia subendocrdica puede ser una complicacin
un 20% de los casos. que se vea dentro del contexto clnico de una hemorragia
2) La fractura transversal suele asociar hipoacusia de conduccin. subaracnoidea.
3) Es frecuente la lesin del tmpano en las fracturas longitudi-
nales. 29. El aneurisma mictico cerebral se relaciona con:
4) Las fracturas oblicuas cursan casi invariablemente con par-
lisis facial. 1) Infeccin candidisica en un enfermo VIH +.
5) El signo de Battle sugiere fractura de la base del crneo. 2) Absceso cerebral.
3) Hemorragia subaracnoidea de repeticin.
4) Diseminacin hemtica de una aspergilosis pulmonar.
25. Tras un intervalo asintomtico, un joven de 25 aos que ha
5) Endocarditis bacteriana.
sufrido un accidente de motocicleta comienza a presentar
deterioro progresivo del nivel de conciencia hasta alcanzar un
GCS de 10/15, acompaado de anisocoria y prdida de fuerza 30. Una mujer de 35 aos presenta un cuadro clnico de hemorra-
en hemicuerpo derecho. La proyeccin lateral de una placa de gia subaracnoidea asociado a ptosis palpebral derecha y mi-
crneo evidenci una fractura lineal de escama temporal iz- driasis ipsilateral. Indique la etiologa que le parece ms proba-
quierda. Cul de las siguientes opciones es cierta respecto al ble:
diagnstico que debe sospechar?:
1) Aneurisma de arteria comunicante posterior.
1) El pronstico de recuperacin funcional de este paciente es 2) Aneurisma de arteria oftlmica.
muy malo incluso con ciruga. 3) Malformacin arteriovenosa en mesencfalo.
2) El tratamiento de eleccin es la ciruga mediante realizacin 4) Aneurisma de arteria comunicante anterior.
de trpano. 5) Fstula cartido-cavernosa.
3) Lo ms probable es que se deba a rotura de venas puente
corticales. 31. En un paciente que al sptimo da de sufrir una HSA presenta
4) Hay signos en la exploracin que sugieren herniacin trans- un cuadro de disfasia y deterioro del nivel de conciencia, si
tentorial. descartamos resangrado en una TC cerebral, el tratamiento
5) Existe riesgo alto de infeccin intracraneal asociada a la ms recomendable sera:
fractura.
1) Restriccin hidrosalina.
26. Es correcto, en relacin al dao axonal difuso: 2) Reducir la presin arterial con diurticos y betabloqueantes.
3) Administrar suero hipertnico.
1) Presenta siempre alteraciones en la TC cerebral. 4) Practicar clipaje urgente del aneurisma causante de su hemo-
2) Habitualmente ocurre en TCE con flexoextensin brusca rragia.
Preguntas TEST

cervical. 5) Aumentar dosis de calcioantagonistas, inducir hipotensin


3) El deterioro neurolgico aparece a partir de la primera arterial y realizar la embolizacin urgente del aneurisma.
semana del TCE.
4) El punteado hemorrgico que aparece en la TC suele loca- 32. La asociacin con aneurisma cerebral se ha descrito ms
lizarse en los hemisferios cerebrales y el cuerpo calloso. frecuentemente en una de estas cardiopatas congnitas:
5) Tiene muy buen pronstico.
1) Tetraloga de Fallot.
27. Una mujer de 40 aos que consulta por exoftalmos unilateral 2) Coartacin artica.
pulstil, estrabismo convergente de ojo derecho y sensacin 3) Transposicin de los grandes vasos.
de soplo retroocular, con el antecedente de traumatismo 4) Anomalas de los cojinetes subendocrdicos.
craneal hace 24 horas, llevar a pensar en el diagnstico de: 5) Ductus arterioso persistente.

1) Neumoencfalo a tensin.
2) Aneurisma postraumtico de arteria oftlmica. PATOLOGA RAQUIMEDULAR
3) Meningocele intraorbitario.
4) Trombosis de seno cavernoso. 33. Una mujer de 50 aos acude a la consulta por lumbocitica
5) Fstula cartido-cavernosa. derecha de 2 meses de evolucin. El dolor aumenta con las

M exico A rgentina CTO Medicina C/ Nez de Balboa, 115 28006 MADRID (Espaa) Tfno.: (91) 782 43 32 / Fax: (91) 782 43 27
C hile U ruguay E-mail: secretaria@ctomedicina.com; iberocto@ctomedicina.com WEB: www.ctomedicina.com; www.iberocto.com
NC Pg. 3
NEUROCIRUGA
Seguimiento a distancia Preparacin Examen de Seleccin 05/06 1 Vuelta
maniobras de Valsalva y la flexin anterior de tronco. La 38. En la valoracin inicial de un dolor lumbar agudo no suelen estar
exploracin neurolgica evidencia reflejo patelar homolateral indicados estudios complementarios de laboratorio o radiogr-
dbil, con fuerza y sensibilidad conservadas. Realizara una RM ficos, excepto ante la sospecha de una etiologa grave. Indique
para descartar una hernia discal ms probablemente localizada cul de los siguientes NO se considera factor de riesgo para este
a nivel del espacio: posible origen grave de lumbalgia:

1) L1-L2. 1) Duracin del dolor > 1 mes.


2) L2-L3. 2) Reposo en cama sin alivio.
3) L3-L4. 3) Dficit motor en la pierna.
4) L4-L5. 4) Toma crnica de glucocorticoides.
5) L5-S1. 5) Edad menor de 50 aos.

34. En una hernia discal cervical localizada en el espacio C5-C6, 39. El tumor intrarraqudeo ms frecuente es una metstasis.
sera ms DIFCIL encontrar en la exploracin: Indique, de entre las siguientes neoplasias slidas, cul disemi-
na con MENOS frecuencia al canal vertebral:
1) Hiporreflexia bicipital.
2) Hiporreflexia estilorradial. 1) Mama.
3) Debilidad a la extensin del codo. 2) Pulmn.
4) Hipoestesia en segundo dedo de la mano. 3) Prstata.
5) Aumento de la cervicobraquialgia en la maniobra de Spur- 4) Rin.
ling. 5) Colon.

35. El tratamiento de eleccin para un paciente de 65 aos de edad


que consulta por dolor lumbar irradiado a miembros inferiores, MALFORMACIONES DEL SNC
claudicacin neurgena intermitente y dolor a la extensin del
tronco es: 40 Cul de los siguientes binomios es FALSO?:

1) Discectoma. 1) Escafocefalia - Sutura sagital.


2) Laminectoma. 2) Trigonocefalia - Sutura metpica.
3) Flavectoma. 3) Plagiocefalia - Sutura pterional.
4) Corpectoma. 4) Paquicefalia - Sutura lambdoidea.
5) Artrodesis. 5) Braquicefalia - Sutura coronal.

36. Es FALSO sobre las lesiones medulares traumticas: 41. Con respecto a la malformacin de Chiari es FALSA:

1) En dficits medulares est indicado el tratamiento con metil- 1) El tipo I se asocia con frecuencia a siringomielia y suele
prednisolona en altas dosis en las 8 primeras horas del debutar al nacimiento.
traumatismo. 2) El tipo II se asocia con hidrocefalia y mielomeningocele.
2) La ciruga descompresiva est sobre todo indicada en dficits 3) El tipo III se asocia con meningoencefalocele.
medulares completos que no mejoran en las primeras 24 h. 4) El tipo IV es una hipoplasia de cerebelo.
3) Sndrome de Brown-Squard: dficit motor y sensibilidad 5) El sntoma ms frecuente es la cefalea suboccipital.
propioceptiva ipsilateral + defecto en la sensibilidad termoal-
gsica contralateral por debajo de la lesin. 42. Seale cul de los siguientes defectos del tubo neural es una
4) Sndrome medular central: presencia de paresia de miem- espina bfida manifiesta:
bros superiores de intensidad mayor que en miembros

Preguntas TEST
inferiores. 1) Seno drmico congnito.
5) El tratamiento inicial de una listesis cervical traumtica con 2) Diastematomielia.
dficit neurolgico es la traccin cervical. 3) Quiste perineural de Tarlov.
4) Sndrome de regresin caudal.
37. Seale lo INCORRECTO en relacin con la patologa de colum- 5) Raquisquisis.
na cervical:
43. Seale lo FALSO referente al mielomeningocele:
1) La fractura de Jefferson compromete los arcos anterior y
posterior del atlas. 1) Cerca del 90% de los pacientes desarrollan hidrocefalia.
2) La enfermedad de Morquio predispone a luxacin atloaxoi- 2) La posicin del nio debe ser en decbito prono.
dea por hipoplasia de la apfisis odontoides del axis. 3) Se ha relacionado con el dficit de cido flico.
3) La fractura de pedculos de C3 tambin es conocida como 4) Suelen detectarse niveles anormalmente bajos de AFP en
fractura de Hangman. lquido amnitico.
4) El esguince cervical es la patologa ms frecuente de columna 5) La mayor seguridad diagnstica prenatal la aporta la ecografa.
cervical.
5) La presencia de dficit neurolgico en un traumatismo 44. Una joven de 15 aos presenta fiebre y rigidez de nuca desde
cervical sin lesin sea es indicacin absoluta de estudio hace unas horas. La exploracin fsica evidencia una lesin
mediante RM. angiomatosa con hipertricosis en la vecindad de una pequea

M exico A rgentina CTO Medicina C/ Nez de Balboa, 115 28006 MADRID (Espaa) Tfno.: (91) 782 43 32 / Fax: (91) 782 43 27
Pg. 4 NC C hile U ruguay E-mail: secretaria@ctomedicina.com; iberocto@ctomedicina.com WEB: www.ctomedicina.com; www.iberocto.com
NEUROCIRUGA
Preparacin Examen de Seleccin 05/06 1 Vuelta Seguimiento a distancia

depresin en la piel de la lnea media lumbosacra. Qu diag- 1) La Rx simple de columna cervical es imprescindible en el
nstico le sugiere?: estudio del paciente politraumatizado.
2) La TC tiene un valor insuperable en las urgencias neuroqui-
1) Enfermedad de Mollaret. rrgicas.
2) Quiste aracnoideo. 3) La RM supera a la TC en el estudio de las fracturas basilares.
3) Mielocele. 4) La RM es insustituible en el estudio de los tumores de lnea
4) Seno drmico congnito. media.
5) Infeccin diseminada por criptococo en enferma con SIDA. 5) La RM es de eleccin para el estudio de las lesiones de fosa
posterior.

OTROS TEMAS 50. El sndrome del canal de Guyn se relaciona con la compresin
del nervio:
45. Una paciente de 60 aos presenta desde hace varios meses
episodios de dolor muy intenso, lancinante, de presentacin 1) Cubital.
sbita, de escasa duracin y carcter recidivante, de distribucin 2) Radial.
en mejilla y regin perioral de hemifacies izquierda. En ocasiones 3) Mediano.
se desencadena con la masticacin o el simple bostezo. Instaura- 4) Peroneo lateral.
ra tratamiento a corto plazo con: 5) Interdigital metatarsiano.

1) Fluoxetina.
2) Sumatriptn.
3) Derivados mrficos.
4) Carbamacepina.
5) Corticoides ms indometacina.

46. La radiociruga se ha demostrado til en todas, EXCEPTO una


de las siguientes patologas:

1) Meningioma.
2) Malformaciones arteriovenosas.
3) Aneurismas.
4) Metstasis.
5) Neurinoma del acstico.

47. En cuanto al tratamiento del absceso cerebral, es FALSO:

1) La evacuacin quirrgica constituye el tratamiento de elec-


cin cuando hay signos de HTIC.
2) Las mejores respuestas teraputicas se obtienen en los absce-
sos nicos en fase de encapsulacin.
3) La cobertura antibitica abarca desde el momento de la
sospecha de absceso cerebral hasta 7-10 das tras la ciruga.
4) El absceso cerebeloso de origen otolgico supone una urgen-
cia quirrgica en la mayora de los casos.
5) La combinacin antibitica con metronidazol-cefotaxima es
Preguntas TEST

una de las ms eficaces.

48. La ciruga funcional estereotxica logra buenos resultados en


ciertos trastornos del movimiento. Indique la FALSA entre las
siguientes asociaciones enfermedad - tcnica quirrgica indi-
cada:

1) Enfermedad de Parkinson idioptica estimulacin crnica


del ncleo subtalmico.
2) Discinesias por L-dopa palidotoma.
3) Distona generalizada idioptica estimulacin de globo
plido interno.
4) Parlisis supranuclear progresiva estimulacin crnica de
ncleo ventral intermedio del tlamo.
5) Temblor esencial talamotoma ventrolateral.

49. Con respecto a los estudios de imagen del SNC, es FALSO:

M exico A rgentina CTO Medicina C/ Nez de Balboa, 115 28006 MADRID (Espaa) Tfno.: (91) 782 43 32 / Fax: (91) 782 43 27
C hile U ruguay E-mail: secretaria@ctomedicina.com; iberocto@ctomedicina.com WEB: www.ctomedicina.com; www.iberocto.com
NC Pg. 5
NEUROCIRUGA
Preparacin Examen de Seleccin 05/06 1 Vuelta Seguimiento a distancia
HIPERTENSIN INTRACRANEAL E HIDROCEFALIA. Las lceras de Cushing mencionadas en la opcin 4 son lceras
gstricas que pueden verse en pacientes graves con HTIC.
Pregunta 1.- R: 5
El sndrome de hipertensin intracraneal (HTIC) es muy impor- Pregunta 2.- R: 5
tante en Neurociruga, y es comn a un gran nmero de las patolo- Las lesiones intracraneales producen una clnica comn a todas
gas que vas a estudiar en esta asignatura, por lo que si lo compren- ellas por elevacin de la PIC, el sndrome de hipertensin intracraneal
des bien, entenders mejor la clnica y el tratamiento de todas ellas. (HTIC). Sin embargo otros sntomas difieren segn la localizacin de las
Hay una serie de conceptos importantsimos que debemos recor- lesiones; son los llamados signos neurolgicos focales. Para entender-
dar, aunque resulte un tanto extensa la explicacin de la pregunta: los debes repasar la semiologa de las lesiones lobares cerebrales en el
El cerebro es un rgano que consume grandes cantidades de ox- apartado de Semiologa del captulo del Manual CTO.
geno y glucosa para desarrollar sus funciones. Para que le lleguen En esta pregunta se describe el caso clnico de un paciente de 24
stos es imprescindible que se mantenga constante el flujo sangu- aos con un hematoma en el lbulo temporal derecho que ha cursa-
neo cerebral (FSC), que como sabes depende de la tensin arterial do con crisis epilpticas y cefalea. En el estudio de imagen se aprecia
(TA) y la presin intracraneal (PIC) en funcin de la frmula recogi- que el hematoma tiene volumen suficiente como para provocar efec-
da en la figura a continuacin, de modo que un aumento de esta to de masa, y que est abierto al ventrculo, lo que significa que hay
ltima condiciona un descenso del FSC. Por tanto, en los pacientes sangre en el interior del sistema ventricular.
con sndrome de HTIC est comprometido el aporte de oxgeno y En cualquier caso clnico en el que nos pregunten el hallazgo ms
glucosa al cerebro, y secundariamente su funcin. o menos probable, debemos hacer un anlisis como el que hemos
explicado del caso para saber qu sntomas podramos esperar. En
FSC = PPC / RV = (TAM-PIC) / RV este caso concreto podemos esperar sntomas de HTIC (por el volu-
men de la lesin), sntomas focales (segn la localizacin de la lesin,
Pregunta 1. El flujo sanguneo cerebral (FSC) se calcula como la presin de en este caso temporal) y otros relacionados con caractersticas espe-
perfusin cerebral (PPC = TAM - PIC) dividida por las resisten- ciales de la lesin que nos describen (en nuestro caso se puede espe-
cias vasculares cerebrales. rar una dificultad para la circulacin del LCR porque los ventrculos
contienen sangre en su interior).
Otro concepto importante para entender el sndrome de HTIC es Si acudimos ahora a las opciones, encontramos fcilmente la res-
que el cerebro est incluido dentro de una cavidad rgida, el crneo. puesta errnea. Las opciones 1 y 2 son ciertas porque un hematoma
Dentro del crneo hay fundamentalmente parnquima cerebral, temporal puede provocar fcilmente un desplazamiento del uncus
sangre y lquido cefalorraqudeo (LCR). Puesto que el crneo no se temporal por la hendidura del tentorio, es decir, una herniacin un-
puede distender, cualquier aumento de uno de esos tres compo- cal, que cursar con parlisis del III par (y por tanto con midriasis de
nentes va a producir una elevacin de la PIC, cuya manifestacin ese lado que motivar una anisocoria) y hemiparesia contralateral
clnica ser el sndrome de HTIC. Esto simplifica mucho el estudio de (debilidad motora en miembros izquierdos, contralateral al hemato-
la clnica de las enfermedades neuroquirrgicas que se desarrolla- ma, por compresin del pednculo cerebral mesenceflico, por el
rn posteriormente a lo largo del test, puesto que todos aquellos que discurre la va corticoespinal).
procesos intracraneales en los que aumenta el volumen de alguno La opcin 3 es cierta porque en un paciente joven, sin anteceden-
de estos componentes (tumores, hemorragias, hidrocefalias, absce- te traumtico conocido y con un hematoma intraparenquimatoso
sos cerebrales, etc.) tendrn en comn los sntomas de este sndro- lobar, la causa ms frecuente de la hemorragia es la rotura de una
me, y la clnica variar en funcin de los dficits neurolgicos focales malformacin arteriovenosa. Aprovechamos para recordarte que los
asociados segn la localizacin de la lesin. hematomas intraparenquimatosos pueden ser profundos o superfi-
La clnica asociada con la elevacin de la PIC es cefalea, ms ciales (tambin llamados lobares). Debes conocer que la causa ms
intensa durante la noche, que suele acompaarse de nuseas y/o frecuente de hematomas intraparenquimatosos profundos es la hi-
vmitos de predominio matutino. Estos sntomas son los primeros pertensin arterial, y que en estos casos suelen localizarse en los ganglios
en aparecer. El signo exploratorio caracterstico es el edema de basales y el tlamo. Si la rotura de una malformacin arteriovenosa es
papila, y en ocasiones puede apreciarse una parlisis del VI par la causa ms frecuente de hematomas intraparenquimatosos lobares
craneal que da lugar a diplopa (recuerda que el VI par es el que en jvenes, en los ancianos lo es la angiopata amiloide.
ms frecuentemente se afecta en los pacientes con HTIC). La opcin 4 es tambin posible porque el hematoma que se descri-
Comentarios TEST

Cuando la PIC contina elevndose, los mecanismos de compensa- be en el enunciado est abierto al sistema ventricular, y por tanto puede
cin del cerebro pueden ser insuficientes y puede comprometerse producirse una hidrocefalia por obstruccin del flujo normal del LCR.
el flujo cerebral y por tanto el aporte de oxgeno y glucosa. En estas La opcin menos probable es la 5, porque por el lbulo temporal
circunstancias, a los sntomas anteriores se asocian otros que se tra- discurren las fibras pticas responsables de la informacin de la parte
ducen en mal funcionamiento del cerebro. Se produce entonces superior del campo visual, y por tanto una lesin temporal produce
una disminucin progresiva del nivel de conciencia del paciente una cuadrantanopsia superior (no inferior). La figura de la pgina
que se acompaa de signos de disfuncin del tronco del encfalo. siguiente es un esquema de los defectos campimtricos asociados
As, hay mal control de la respiracin, de la tensin arterial y del con lesiones en las distintas partes de la va ptica, que debes repasar
latido cardaco. Esta trada clnica, consistente en alteraciones del en el apartado de Semiologa del Manual CTO.
patrn respiratorio, bradicardia e hipertensin arterial, se conoce La clave en esta pregunta estaba en que el dficit neurolgico focal
como trada de Cushing y aparece en fases avanzadas de la HTIC. La que se describe no corresponde con la localizacin de la lesin que
respuesta incorrecta es la 5, porque en estas circunstancias hay un se indica en el enunciado.
enlentecimiento de la frecuencia cardaca, no taquicardia. La op-
cin 1 es cierta, porque es uno de los componentes de la trada. Pregunta 3.- R: 5
Si se monitoriza la PIC mediante sensores especiales de presin Vamos a analizar el enunciado de la pregunta. Se describe el caso
colocados por el neurocirujano dentro del crneo, se pueden ver clnico de una paciente joven con clnica cerebelosa y dolor cervi-
alteraciones de la morfologa normal de la curva de PIC. La ms cal que sufre un deterioro progresivo del nivel de conciencia. Se
importante es la onda A o plateau, que se recoge en la opcin 2 realiza una TAC craneal y se aprecia una hipodensidad difusa en el
del enunciado de la pregunta. hemisferio cerebeloso izquierdo con efecto de masa que produce
La opcin 3 es tambin cierta. En fases avanzadas de la HTIC, el herniacin transtentorial, es decir, desplazamiento del cerebelo y
aumento de presin provoca desplazamiento de las estructuras el tronco del encfalo hacia arriba a travs de la hendidura tentorial.
cerebrales, que quedan comprimidas contra otras estructuras in- Lo que se pretende es determinar cul es la causa ms probable de
tracraneales como la duramadre. Estos desplazamientos se cono- este cuadro.
cen como herniaciones cerebrales. La ms frecuente es la her- La opcin 1 no puede ser, porque una ateromatosis de la arteria
niacin uncal, uno de cuyos sntomas es la parlisis del III par. No cartida interna puede provocar una isquemia en el territorio de la
olvides repasar la clnica caracterstica de esta entidad. circulacin anterior (y por tanto supratentorial), mientras que la lesin
M exico A rgentina CTO Medicina C/ Nez de Balboa, 115 28006 MADRID (Espaa) Tfno.: (91) 782 43 32 / Fax: (91) 782 43 27
C hile U ruguay E-mail: secretaria@ctomedicina.com; iberocto@ctomedicina.com WEB: www.ctomedicina.com; www.iberocto.com
NC Pg. 1
NEUROCIRUGA
Seguimiento a distancia Preparacin Examen de Seleccin 05/06 1 Vuelta

Pregunta 2. Defectos campimtricos y lesiones de la va ptica.

que nos estn describiendo en el enunciado est en la fosa posterior ocasiones en pocas semanas). Ante este cuadro clnico debemos sos-
(y por tanto en territorio vertebrobasilar). pechar HTIC, y la prueba diagnstica de eleccin en los pacientes
La opcin 2, el absceso cerebeloso otolgico, podra dar un cua- con HTIC es la TAC craneal. Lo que caracteriza al pseudotumor cerebri
dro clnico y una imagen radiolgica similar. Sin embargo, en contra es que estos pacientes tienen un cuadro clnico de HTIC, pero las
est que no se describe clnica previa de infeccin tica y no explica pruebas de imagen en ellos no muestran ninguna alteracin (TAC
el dolor occipitocervical. Recuerda que la fiebre no siempre est pre- craneal sin hallazgos).
sente en los pacientes con absceso cerebral, y por tanto no es un Ante un paciente con sospecha de sndrome de HTIC benigna hay
criterio de exclusin. que descartar posibles etiologas que se recogen en la tabla a conti-
La enfermedad de Lhermitte-Duclos (opcin 3) es un gangliocito- nuacin. Se ha relacionado con una mayor frecuencia en mujeres
ma difuso de cerebelo, es decir, un tumor, cuya evolucin clnica es obesas, ciertas enfermedades sistmicas y la toma de ciertos frmacos.
ms progresiva. Lo mismo podramos decir del astrocitoma piloctico La descripcin en el enunciado de aftas en mucosa oral y genitales es
de cerebelo (opcin 4), un tumor benigno que habitualmente tiene caracterstica de la enfermedad de Behet (opcin 2), una de las enti-
aspecto qustico en la TAC. dades asociadas con este diagnstico.
La respuesta correcta es la 5. El inicio del cuadro con dolor
occipitocervical y vrtigo de aparicin brusca es muy sugestivo de

Comentarios TEST
diseccin de la arteria vertebral (que tiene un recorrido cervical a Pregunta 4. Etiologa del sndrome de hipertensin
travs de las apfisis transversas de las vrtebras cervicales y despus, intracraneal benigna.
tras rodear por detrs las masas laterales del atlas, se introduce en el
interior del crneo por el foramen occipital para irrigar estructuras Alteraciones del drenaje venoso.
enceflicas de la fosa posterior). El deterioro posterior del nivel de Embarazo y anticonceptivos orales.
conciencia del paciente se debe a que la diseccin de la arteria ha Obesidad.
producido un descenso del flujo sanguneo a travs de la misma y por Hipo e hipertiroidismo.
tanto un infarto isqumico de cerebelo, que es la lesin radiolgica Hipoparatiroidismo.
que se describe en el enunciado. El rea de infarto ha producido Insuficiencia suprarrenal.
efecto de masa y compresin del tronco, lo que explicara la disminu- Sndrome de Cushing.
cin del nivel de conciencia. Recuerda que las lesiones isqumicas Anemia ferropnica severa.
cerebrales aparecen en la TAC como lesiones hipodensas (negras), Sarcoidosis.
mientras que las hemorragias son hiperdensas (blancas). La razn es Lupus eritematoso sistmico.
que la sangre tiene hierro, que se ve blanco en las radiografas). Enfermedad de Behet
Frmacos:
Pregunta 4.- R: 2 - Vitamina A y derivados (retinoides).
El cuadro clnico descrito es muy caracterstico, y muy frecuente- - Tetraciclinas.
mente preguntado en el MIR. Corresponde con un sndrome de - cido nalidxico.
hipertensin intracraneal benigna, tambin conocida como pseu- - Nitrofurantona.
dotumor cerebri. - Sulfamidas.
Habitualmente lo preguntan describiendo un sndrome de hiper- - Litio.
tensin intracraneal tpico (cefalea, vmitos, diplopa por parlisis del - Indometacina.
VI par y visin borrosa por edema de papila), que cursa en brotes - Fenitona.
autolimitados (dicen en el enunciado que le ha pasado en varias

M exico A rgentina CTO Medicina C/ Nez de Balboa, 115 28006 MADRID (Espaa) Tfno.: (91) 782 43 32 / Fax: (91) 782 43 27
Pg. 2 NC C hile U ruguay E-mail: secretaria@ctomedicina.com; iberocto@ctomedicina.com WEB: www.ctomedicina.com; www.iberocto.com
NEUROCIRUGA
Preparacin Examen de Seleccin 05/06 1 Vuelta Seguimiento a distancia
Pregunta 5.- R: 5 LCR se acumula en el interior de los ventrculos cerebrales, dando
El sndrome de hipertensin intracraneal benigna o pseudotu- lugar a una dilatacin de los mismos observable en las pruebas de
mor cerebri se caracteriza por sntomas clnicos de HTIC, con ha- imagen, que se conoce como hidrocefalia (ver figura).
llazgos exploratorios compatibles (edema de papila) en ausencia de Hay dos tipos de hidrocefalia. Por un lado estn aquellas en las que
lesiones objetivables en las pruebas de imagen (TAC craneal normal). el LCR no es capaz de llegar al espacio subaracnoideo porque hay
Se ha asociado con algunas enfermedades sistmicas y la toma de una obstruccin en el trayecto intraventricular; son las hidrocefalias
ciertos frmacos (ver tabla anterior). obstructivas o no comunicantes. Un segundo tipo son las hidrocefa-
En esta pregunta se pretende que reflexionemos acerca del manejo lias comunicantes, en las que el LCR producido en los ventrculos
teraputico de este sndrome. puede llegar a los espacios subaracnoideos, pero no circula por ellos
Dado que en muchas ocasiones el pseudotumor cerebri se asocia hasta las granulaciones de Paccioni, o si lo hace no se reabsorbe en
con la ingesta de frmacos, la primera medida teraputica en estos estas ltimas.
pacientes es suprimir el agente etiolgico probable, y por tanto sus-
pender el frmaco que lo puede estar desencadenando. Sin embargo
esta opcin no est recogida entre las posibles respuestas.
Todas las respuestas que se enumeran en las opciones de esta pre-
gunta son medidas eficaces y tiles en el tratamiento de esta entidad,
pero el manejo debe ser escalonado (ver figura). Por tanto, aunque
todas valdran, la respuesta correcta es la 5, porque la restriccin
hidrosalina y la administracin de diurticos (sobre todo acetazolami-
da) debe ser la primera medida teraputica de entre las citadas.
El resto se iran realizando a medida que fracasan las medidas ante-
riores. Tras la restriccin hidrosalina y los diurticos se pueden adminis-
trar corticoides. Si estas medidas farmacolgicas no son eficaces se rea-
lizan punciones lumbares evacuadores para sacar LCR y reducir la PIC.
Si el paciente precisa mltiples punciones para controlar los sntomas,
se coloca una derivacin lumboperitoneal a fin de sacar LCR de forma
continua y no tener que pinchar al paciente reiteradamente.
Cuando todas estas medidas fracasan hay que recurrir a otras tc-
nicas quirrgicas ms agresivas, como la descompresin quirrgica de
la vaina del nervio ptico o las craniectomas descompresivas, aun-
que rara vez es necesario practicarlas.
No olvides que el objetivo del tratamiento de estos pacientes es
conservar la visin, ya que la HTIC repetida produce edema de papila
bilateral que puede conducir a la ceguera.
Pregunta 6. TAC craneal de un neonato con hidrocefalia. Se observa gran
dilatacin del sistema ventricular.

En esta pregunta nos piden determinar la causa ms probable de


una hidrocefalia comunicante (ver esquema a continuacin). Hay
que eliminar por tanto todas las opciones que supongan una obstruc-
cin a la circulacin del LCR por el sistema ventricular. Esto elimina la
opcin 3 (la enfermedad de Dandy-Walker es una atresia de los agu-
jeros de Luschka y Magendie, y por tanto el LCR queda atrapado
antes de salir al espacio subaracnoideo).
Comentarios TEST

Pregunta 5. Manejo teraputico del Sndrome de Hipertensin Intracraneal


Benigna (Pseudotumor cerebrii).

Pregunta 6.- R: 1
Esta pregunta hace referencia a la etiologa de la hidrocefalia.
La hidrocefalia se produce como consecuencia de un disbalance
entre la formacin y la reabsorcin del LCR. El LCR se forma en el
plexo coroideo de los ventrculos cerebrales y circula a travs de estos
hasta alcanzar el espacio subaracnoideo, para ser reabsorbido en las
granulaciones de Paccioni y pasar al torrente sanguneo. Se producen
diariamente unos 500 cc de LCR, que deben ser reabsorbidos. Si la Pregunta 6. Tipos de hidrocefalia y ejemplos de las causas ms frecuentes.
produccin es superior a la capacidad de reabsorcin, ese exceso de

M exico A rgentina CTO Medicina C/ Nez de Balboa, 115 28006 MADRID (Espaa) Tfno.: (91) 782 43 32 / Fax: (91) 782 43 27
C hile U ruguay E-mail: secretaria@ctomedicina.com; iberocto@ctomedicina.com WEB: www.ctomedicina.com; www.iberocto.com
NC Pg. 3
NEUROCIRUGA
Seguimiento a distancia Preparacin Examen de Seleccin 05/06 1 Vuelta
Las opciones 4 y 5 son tumores que tambin obstruyen las vas de Ante la sospecha clnica se realizar una prueba de imagen cere-
circulacin normal del LCR (el quiste coloide es un tumor tpico del bral, en la que debe observarse una hidrocefalia comunicante, es
tercer ventrculo, y los tumores de clulas germinales se localizan con decir, una dilatacin de todo el sistema ventricular sin que se aprecie
ms frecuencia en la regin pineal y pueden obstruir la circulacin en una obstruccin a la circulacin del LCR.
el tercer ventrculo o en el acueducto de Silvio). Dado que la hidrocefalia crnica del adulto es una causa rever-
La vasculopata hipertensiva (opcin 2) no es una causa habitual sible de demencia, la sospecha clnica exige determinar si el paciente
de hidrocefalia, aunque pueden verse cambios de seal radiolgica es candidato a tratamiento. El tratamiento de eleccin es la deriva-
periventricular en estos pacientes. cin ventriculoperitoneal (vlvula).
La respuesta correcta es la 1, porque el carcinoma microctico de Antes de la intervencin se realizan una serie de pruebas clnicas
pulmn puede metastatizar al sistema nervioso central. Aunque lo para intentar averiguar si el paciente responder o no al tratamiento
ms probable es que produzca metstasis cerebrales (masas tumorales (punciones lumbares evacuadoras, test de infusin, etc.). Todas estas
intracerebrales), puede producir tambin metstasis en las meninges pruebas intentan predecir si el paciente mejorar o no tras la coloca-
(carcinomatosis menngea). La carcinomatosis menngea es una cau- cin de la derivacin. Con las punciones lumbares se pretende sacar
sa frecuente de hidrocefalia comunicante en la que el LCR llega al LCR (15 o 20cc) y ver si mejora la capacidad intelectual del paciente
espacio subaracnoideo, pero no puede circular por l porque est y la marcha, en cuyo caso se colocar la vlvula. Los test de infusin,
ocupado por clulas tumorales. El mismo mecanismo es responsable algo ms complejos, hacen lo contrario; introducen lquido a travs
de la hidrocefalia arreabsortiva asociada con infecciones (meningitis) de una puncin lumbar y miden las variaciones de presin para ver la
o con la hemorragia subaracnoidea. tolerancia del sistema y si es necesaria la vlvula. Si el test de infusin
es positivo, la probabilidad de mejora tras la derivacin es mayor.
Pregunta 7.- R: 2
Este es uno de los cuadros clnicos ms frecuentemente pregunta- Pregunta 8.- R: 4
dos en el examen MIR, y siempre lo preguntan del mismo modo Nos presentan a un paciente de 5 aos de edad, portador de una
(forma de presentacin clnica), por lo que no se puede fallar. Aprove- derivacin ventriculoperitoneal para el tratamiento de una hidroce-
chamos para recordarte que, aunque clsicamente se ha llamado falia secundaria a estenosis de acueducto (hidrocefalia obstructiva)
hidrocefalia normotensiva o a presin normal (y as la han pregun- detectada al nacimiento, que desde hace 2 das ha empeorado y
tado en el examen MIR), un nombre ms utilizado por los presenta una clnica compatible con hipertensin intracraneal. Es
neurocirujanos actualmente es hidrocefalia crnica del adulto, y obligatorio sospechar que existe una malfuncin de la vlvula y rea-
tambin podran preguntarla de esta otra manera. No en una entidad lizar todos los esfuerzos necesarios para diagnosticarla (ver tabla) y
nueva; es exactamente lo mismo. tratarla a tiempo, porque una malfuncin en un paciente con vlvula
La hidrocefalia crnica del adulto se define por una trada clnica puede ser mortal.
(trada de Hakim) que incluye demencia con alteraciones de memo- Las derivaciones de LCR son catteres colocados en el interior del
ria, incontinencia urinaria y alteraciones de la marcha, y es un cuadro sistema ventricular, que llevan un sistema valvular para regular el paso
tpico de ancianos (ver figura). Todas las preguntas que han cado en de LCR y que lo derivan hacia otras cavidades del organismo (habi-
el examen MIR haciendo alusin a esta entidad se referan a la trada tualmente el peritoneo, pero tambin la pleura o las cavidades carda-
tpica, preguntada expresamente o incluida en el enunciado para cas pueden ser utilizadas). Por tanto, pueden malfuncionar por pro-
hacer diagnstico diferencial con otras demencias. blemas en cualquier punto de su recorrido, y es obligatorio estu-
diar todo el trayecto de la vlvula.
La opcin 1 est indicada ante la sospecha de malfuncin para
valorar el tamao del sistema ventricular (si hay una dilatacin respec-
Demencia to a los estudios previos puede indicar que la derivacin no est fun-
cionando, o lo est haciendo mal, y adems permite ver la posicin
del extremo ventricular del catter, que debe estar colocado dentro
del ventrculo).
La opcin 2, el examen del fondo de ojo, debe realizarse en todo
paciente con sospecha de HTIC, porque permite objetivar el edema
de papila caracterstico de estos enfermos.
Es obligatorio tambin realizar un estudio del trayecto valvular (op-

Comentarios TEST
cin 3) mediante Rx simple de todo el recorrido (crneo, cuello, trax
y abdomen), con el fin de comprobar la integridad del sistema de
derivacin (que no hay roturas ni desconexiones entre sus elementos)
y la correcta ubicacin de los extremos ventricular y distal (debe estar
en el interior de la cavidad elegida).
Aunque no est recogida como opcin en la pregunta, otra causa de
malfuncin de la vlvula es la infeccin, por lo que ante una clnica
Incontinencia urinaria sugestiva de mal funcionamiento ms fiebre debe descartarse la infec-
cin mediante estudio bioqumico y microbiolgico del LCR (obtenido
por puncin directa del reservorio valvular o por puncin lumbar).
Si se encuentra la causa del mal funcionamiento de la vlvula (hay
roturas, desconexiones, infecciones o se ha salido el catter del ventrculo
o de la cavidad peritoneal), debe ser revisada quirrgicamente. Ningn
paciente con sospecha de malfuncin valvular puede ser dado de alta;
como mnimo debe ser ingresado para observar la evolucin (opcin 5)
Apraxia de la marcha y debe recibir tratamiento especfico si lo requiere.
No est indicado el tratamiento con dexametasona (opcin 4) ante
la sospecha de malfuncin valvular. La dexametasona es un trata-
miento muy eficaz para la HTIC secundaria a tumores cerebrales con
edema vasognico, pero no sirve para tratar la HTIC secundaria a
hidrocefalia. El tratamiento de eleccin ante una malfuncin valvu-
lar es la revisin quirrgica de la misma.
No obstante, aprovechamos para recordar que, aunque las deriva-
Pregunta 7. Hidrocefalia normotensiva. ciones de LCR son las intervenciones indicadas para el tratamiento

M exico A rgentina CTO Medicina C/ Nez de Balboa, 115 28006 MADRID (Espaa) Tfno.: (91) 782 43 32 / Fax: (91) 782 43 27
Pg. 4 NC C hile U ruguay E-mail: secretaria@ctomedicina.com; iberocto@ctomedicina.com WEB: www.ctomedicina.com; www.iberocto.com
NEUROCIRUGA
Preparacin Examen de Seleccin 05/06 1 Vuelta Seguimiento a distancia
permanente de la hidrocefalia, actualmente el tratamiento de elec- se aquellas nicas y accesibles que producen HTIC o efecto de masa
cin para la hidrocefalia obstructiva secundaria a estenosis del acue- significativo.
ducto de Silvio (el paciente de la pregunta) es la ventriculostoma
endoscpica del suelo del tercer ventrculo, que puede resolver la Pregunta 10.- R: 2
hidrocefalia sin necesidad de derivaciones de LCR y por tanto evita los Se describe el caso clnico de un varn de edad media con clnica
reingresos por malfuncin valvular. Esta tcnica utiliza un endoscopio neurolgica focal (hemiparesia), en el que se encuentra en un estudio
para navegar por el interior del sistema ventricular hasta el suelo del de RM una lesin expansiva cerebral frontal, de morfologa irregular,
tercer ventrculo y crear all una comunicacin artificial entre el sistema que se extiende a travs del cuerpo calloso, y que se realza en anillo
ventricular y el espacio subaracnoideo, sin necesidad de que el pacien- tras administrar medio de contraste. Hay dos datos importantes para el
te sea portador de prtesis, por lo que actualmente se considera que diagnstico, el patrn de captacin de contraste y el hecho de que
debe intentarse antes de la colocacin de una derivacin en todos los sea una lesin infiltrante (una lesin irregular en el lbulo frontal que
pacientes con estenosis de acueducto. Si en el nio del caso clnico se se extiende por la sustancia blanca hasta el cuerpo calloso).
hubiera realizado lo anterior, en lugar de colocar la vlvula, no tendra- Un dato que suele ayudar en el diagnstico diferencial es la ima-
mos que plantearnos ahora todas las medidas del enunciado. gen radiolgica tpica de captacin en anillo (ver tabla). Sin embar-
go no es til en esta pregunta, porque todas las opciones descritas
(linfoma, glioblastoma, toxoplasmosis, metstasis y absceso cerebral)
Pregunta 8. Manejo diagnstico del paciente con sospecha pueden dar esta imagen. Por tanto, debemos llegar a la respuesta
de malfuncin valvular. correcta a travs de otros datos del enunciado.

Se sospecha por historia clnica.


Exploracin fsica: Pregunta 10. Lesiones cerebrales con captacin de contraste
- Nivel de conciencia. en anillo en la TAC craneal.
- Fondo de ojo.
- Exploracin neurolgica. Glioblastoma multiforme.
Metstasis.
- Fiebre y rigidez nucal.
Linfoma cerebral primario.
Toxoplasmosis.
TAC craneal.
Absceso pigeno (bacteriano).
Rx del trayecto valvular.
Anlisis bioqumico y microbiolgico del LCR.
Ingreso para observacin. La opcin ms probable es que se trate de un tumor primario
Revisin quirrgica si lo requiere. maligno, un glioblastoma multiforme (opcin 2). Recuerda que los
gliomas son los tumores cerebrales primarios ms frecuentes en el
adulto. Son tumores intraparenquimatosos, que suelen localizarse a
TUMORES.
nivel subcortical. Se trata de tumores infiltrantes, es decir, que no
tienen un plano de separacin claro con el cerebro normal. Pueden
Pregunta 9.- R: 5
extenderse por la sustancia blanca, y por ello los de localizacin
Esta pregunta hace referencia a las metstasis cerebrales.
frontal alcanzan frecuentemente el cuerpo calloso y pueden llegar
Las respuestas 1 a 4 son correctas. Las metstasis cerebrales supo-
hasta el otro hemisferio (tumores en alas de mariposa). Se clasifican
nen la neoplasia intracraneal ms frecuente en el adulto, y habitual-
segn su agresividad histolgica en gliomas de bajo grado, gliomas
mente son de localizacin supratentorial, a nivel corticosubcortical,
anaplsicos y glioblastoma multiforme (el ms agresivo de todos). Este
aunque tambin pueden producirse en la fosa posterior. Son excep-
ltimo es el que caractersticamente capta en anillo y puede cruzar
cionales en nios.
por el cuerpo calloso al otro lado.
En las lneas siguientes te describimos los conceptos ms importan-
tes que hay que conocer respecto a las metstasis cerebrales para el
examen MIR:
Origen. Si vemos a un paciente con una imagen en la TAC sugestiva
de metstasis cerebral (lesin hipodensa que capta contraste peri-
Comentarios TEST

fricamente o en anillo, rodeada de abundante edema vasogni-


co), el tumor primario sistmico que ms probablemente ser res-
ponsable de la metstasis es el carcinoma de pulmn. Sin embar-
go, si consideramos por separado los tumores primarios, el que
con ms frecuencia metastatiza en el SNC es el melanoma.
Tendencia al sangrado. Aunque hay muchos tumores cerebrales
primarios con tendencia al sangrado (glioblastoma, ependimoma,
oligodendroglioma, adenoma de hipfisis, etc.), las metstasis tie-
nen mayor tendencia a sangrar que los tumores primarios, sobre
todo las procedentes de carcinoma de pulmn, coriocarcinoma,
melanoma, rin y tiroides. Como otras lesiones hemorrgicas,
cuando han sangrado son hiperdensas (blancas) en la TAC.
Indicaciones de tratamiento. La respuesta incorrecta es la 5.
Aunque la ciruga est indicada ante una lesin metastsica nica
con efecto de masa en una regin accesible del cerebro sin riesgo
de secuelas neurolgicas importantes, el tratamiento de eleccin
de las metstasis cerebrales es la radioterapia holocraneal. Hay que
Pregunta 10. Captacin de contraste en anillo. Glioblastoma multiforme
tener en cuenta que, con mucha frecuencia, las metstasis cere-
brales son mltiples, y el hecho de que en la TAC slo veamos una
no quiere decir que no haya ms no visibles, por lo que debe Las metstasis (opcin 4) son muy frecuentes, y tienen una imagen
radiarse todo el cerebro. similar en anillo, pero no suelen ser infiltrantes, y tienen un plano
ms o menos definido de separacin con el cerebro. No dan el pa-
Por tanto, y para concretar, el tratamiento de eleccin de las mets- trn en alas de mariposa. Adems, el hecho de que la Rx de trax y la
tasis cerebrales es la radioterapia holocraneal, aunque deben operar- analtica sean normales, aunque no es definitivo, ayuda a excluir esta
opcin.

M exico A rgentina CTO Medicina C/ Nez de Balboa, 115 28006 MADRID (Espaa) Tfno.: (91) 782 43 32 / Fax: (91) 782 43 27
C hile U ruguay E-mail: secretaria@ctomedicina.com; iberocto@ctomedicina.com WEB: www.ctomedicina.com; www.iberocto.com
NC Pg. 5
NEUROCIRUGA
Seguimiento a distancia Preparacin Examen de Seleccin 05/06 1 Vuelta
Los abscesos cerebrales (opcin 5) son lesiones purulentas de ori- jan por tanto con el enunciado. El glioblastoma y las metstasis s son
gen bacteriano generalmente, que producen clnica de HTIC y focalidad tumores del adulto y supratentoriales, pero tienen una historia natural
neurolgica. No necesariamente cursan con fiebre o alteraciones ana- ms corta (este paciente tiene sntomas de 2 aos de evolucin) y
lticas (leucocitosis), pero suelen dar este dato en las preguntas MIR. captan contraste intravenoso intensamente.
Como son lesiones encapsuladas, tienden a tener un plano de separa-
cin con el cerebro (no se comportan como lesiones infiltrantes).
La toxoplasmosis (opcin 3) da un patrn radiolgico similar en
pacientes inmunocomprometidos. Es la principal causa de lesin ocu-
pante de espacio intracerebral en pacientes infectados por VIH, y el
hallazgo de una lesin cerebral en estos pacientes es indicacin de
tratamiento emprico antitoxoplasma (sulfadiacina y pirimetamina).
Son lesiones localizadas, no infiltrantes.
Los linfomas cerebrales (opcin 1) pueden ser tumores primarios o
secundarios (metstasis neurolgicas de un linfoma sistmico). Estas s
son lesiones infiltrantes, de elevada malignidad, pero se ven habitual-
mente en pacientes inmunodeprimidos (aunque cada vez son ms fre-
cuentemente diagnosticadas en inmunocompetentes). Entre los prima-
rios, los ms frecuentes son los linfomas de alto grado de clulas B. Es
caracterstica una distribucin periventricular, aunque no obligada. Fre-
cuentemente la imagen radiolgica mejora tras la administracin de
corticoides, por lo que se han llamado tumores fantasma. El trata-
miento de eleccin es la radioterapia y quimioterapia con metotrexate.
Por tanto, por lo considerado hasta ahora lo ms probable es que
la lesin descrita sea un glioblastoma multiforme.

Pregunta 11.- R: 3 Pregunta 11. Oligodendroglioma frontal derecho.


Como siempre, hay que analizar el enunciado cuidadosamente.
Nos piden llegar a un diagnstico, y segn ste, indicar el tratamiento Pregunta 12.- R: 4
ms adecuado. Una pregunta directa sobre el ependimoma que requiere conocer
Se trata de un varn de edad media, epilptico, que no responde algunas de sus caractersticas.
bien al tratamiento, y que tiene una imagen expansiva en la TAC Los ependimomas son tumores derivados de clulas ependimarias,
cerebral. Los tumores son una causa relativamente frecuente de epi- y por tanto pueden estar localizados a nivel intracraneal, pero tambin
lepsia farmacorresistente, y en las opciones nos dan a elegir entre 5 a nivel espinal. En concreto son ms frecuentes en esta ltima localiza-
tumores distintos. Habr que fijarse en otros datos del enunciado para cin, y especialmente en el filum terminale, que es el nico sitio en el
llegar a la respuesta. que se da una variante histolgica de ependimoma caracterstica, la
Los otros datos que nos dan son: es un tumor de localizacin variante mixopapilar. Por tanto, la respuesta falsa es la 4, ya que el
frontal, muestra calcificaciones y no capta contraste (lo que traducido ependimoma mixopapilar se localiza caractersticamente en el filum
significa que no parece muy maligno, pues los tumores que captan terminale. Los ependimomas intracraneales no son mixopapilares.
contraste intravenoso son habitualmente los malignos, por rotura de Respecto a los intracraneales, debes recordar que son mucho ms
la barrera hematoenceflica, o los muy vascularizados). Otro dato a frecuentes en la fosa posterior (en el suelo del 4 ventrculo opcin
favor de que no sea un tumor muy maligno es el largo tiempo de 2) que supratentoriales. Los intracraneales son tpicos de nios, mien-
evolucin (2 aos). tras que los espinales afectan ms a adultos jvenes.
Resumiendo, se trata de un varn de edad media con crisis epilp- Son tumores benignos y son caractersticas en el estudio anatomo-
ticas de 2 aos de evolucin y un tumor frontal calcificado, que no patolgico las formaciones en roseta (opcin 5).
parece muy maligno. Entre las opciones destaca una por encima de
las dems como la ms probable: el oligodendroglioma (opcin 3).
Fjate en que el oligodendroglioma encaja con todos los datos del

Comentarios TEST
enunciado. De hecho deberas recordarlo precisamente por ellos. Es
el tumor ms epileptognico de todos los tumores primarios; con
frecuencia se diagnostica en pacientes con crisis rebeldes al trata-
miento (y nuestro paciente tiene crisis de larga evolucin). Su localiza-
cin ms habitual es el lbulo frontal. Caractersticamente tiene ten-
dencia a calcificarse, y con frecuencia muestra reas hemorrgicas.
Habitualmente no capta contraste intravenoso porque es un tumor de
bajo grado de malignidad (aunque hay una variante anaplsica). Re-
cuerda que la caracterstica anatomopatolgica tpica de este tumor
son las clulas en huevo frito (clulas con ncleo hipercromtico
rodeado de un citoplasma de escasa apetencia tintorial). El tratamien-
to de eleccin del oligodendroglioma es la ciruga, como recoge la
opcin 3.
Aprovechamos para recordar otros tumores primarios del SNC que
se parecen al oligodendroglioma en el hecho de que tambin son
muy epileptognicos, y tambin tienden a presentar calcificaciones,
pero cuya localizacin ms tpica es el lbulo temporal. Se trata del
gangliocitoma y ganglioglioma.
Los otros tumores descritos en las opciones son el glioblastoma, las Pregunta 12. Ependimoma del suelo de IV ventrculo.
metstasis, el meduloblastoma y el ependimoma, todos ellos tumores
muy malignos. Aunque no supieras reconocer el oligodendroglioma Puesto que su localizacin intracraneal ms habitual es el suelo del
por las caractersticas descritas en el enunciado, hay datos que permi- 4 ventrculo, no es extrao que produzcan hidrocefalia (opcin 3).
ten excluir los otros 4. El meduloblastoma y el ependimoma son tu- Como ya habrs pensado, se trata de una hidrocefalia obstructiva,
mores ms frecuentes en nios y tpicos de la fosa posterior; no enca- porque bloquean la circulacin normal del LCR.

M exico A rgentina CTO Medicina C/ Nez de Balboa, 115 28006 MADRID (Espaa) Tfno.: (91) 782 43 32 / Fax: (91) 782 43 27
Pg. 6 NC C hile U ruguay E-mail: secretaria@ctomedicina.com; iberocto@ctomedicina.com WEB: www.ctomedicina.com; www.iberocto.com
NEUROCIRUGA
Preparacin Examen de Seleccin 05/06 1 Vuelta Seguimiento a distancia
Por su localizacin intraventricular se puede explicar tambin su Pregunta 14.- R: 3
tendencia a diseminar por el LCR y por tanto por el espacio subarac- Atencin a esta pregunta. Hace referencia a uno de los tumores
noideo (opcin 1). Esta es una caracterstica que comparten con otros cerebrales ms preguntados en el MIR, y por suerte siempre nos lo
tumores muy malignos del SNC (los tumores neuroectodrmicos pri- presentan igual.
mitivos o PNET, y a la cabeza de ellos el meduloblastoma, que se Describen un paciente adolescente con un defecto campimtrico
localiza ms frecuentemente en el techo del 4 ventrculo, por lo que caracterstico (se trata de una prdida de visin en ambos campos
exige diagnstico diferencial con el ependimoma). Recuerda que en temporales, es decir, una hemianopsia bitemporal). Desde un punto
el tratamiento del ependimoma (y tambin del meduloblastoma) de- de vista semiolgico la hemianopsia bitemporal se debe a una com-
ben incluirse una RM espinal y un estudio del LCR (citologa) para presin del quiasma ptico (ver figura de pregunta 2), y efectivamen-
descartar diseminaciones de clulas tumorales por el espacio te, en el enunciado nos dan un dato que apoya esto; el paciente tiene
subaracnoideo, y que cuando se realiza radioterapia, sta debe ser una lesin qustica con calcificaciones en localizacin supraselar (y
craneoespinal por el mismo motivo. por encima de la silla turca tenemos el quiasma ptico). No olvides
repasar la semiologa de los defectos campimtricos, pues los pregun-
tan con mucha frecuencia.
Pregunta 13.- R: 2 No hay ninguna duda posible. El paciente que nos describen tiene
Esta pregunta hace referencia a los tumores enceflicos en nios. probablemente un craneofaringioma (opcin 3).
Como sabes, son tumores muy frecuentes en este grupo de edad. En Siempre que han preguntado el craneofaringioma en el MIR lo
concreto son los tumores slidos ms frecuentes, y slo estn supera- han hecho de esta forma: paciente joven con lesin supraselar qusti-
dos en frecuencia por las leucemias (opcin 1). ca y calcificada que presenta un defecto visual (hemianopsia bitem-
Una caracterstica importante que debe recordarse de los tumores poral) o una disfuncin endocrina (obesidad habitualmente).
cerebrales de los nios es que suelen ser primarios; las metstasis Recuerda que el craneofaringioma es el tumor supratentorial
cerebrales en nios son excepcionales (opcin 4). ms frecuente en los nios y que su localizacin es supraselar. Tiene
Otro dato importante es que, a diferencia de los adultos, en los gran tendencia a las calcificaciones marginales (se describen las calci-
nios los tumores cerebrales son ms frecuentes en la fosa posterior, es ficaciones en parntesis), y por su localizacin supraselar comprime
decir, infratentoriales. el quiasma ptico y provoca disfuncin endocrina (compresin del
Los tumores cerebrales ms habituales en los nios son, por orden tallo hipofisario e hipotlamo). Es tpico que se trate de una lesin con
de frecuencia (ver tabla en la pgina siguiente): los gliomas (en con- quistes en cuyo interior hay una sustancia oleosa que se describe
creto el astrocitoma de cerebelo, que suele ser muy benigno), el como aceite de motor.
meduloblastoma (un tumor muy maligno) y el craneofaringioma.
Fjate que los dos primeros son de fosa posterior, y el tercero supraselar Pregunta 15.- R: 1
(y por tanto supratentorial). Esto se presta a preguntas en las que se Los linfomas en el sistema nervioso central (SNC) pueden ser pri-
utilicen juegos de palabras, por ejemplo: marios o secundarios (metstasis en el sistema nervioso de un linfoma
sistmico). Esta pregunta hace referencia al linfoma cerebral prima-
rio, un tumor que caractersticamente afecta a pacientes muy inmu-
1- El tumor cerebral ms frecuente en los nios es? nodeprimidos. Suele diagnosticarse en pacientes trasplantados o in-
El astrocitoma de cerebelo. fectados por VIH y afectados de SIDA (opcin 3). No obstante est
2- El tumor cerebral maligno ms frecuente en nios es? aumentando la frecuencia de casos diagnosticados en pacientes in-
El meduloblastoma (porque el astrocitoma de cerebelo en nios es munocompetentes, aunque de cara al MIR debes recordarlo como
un tumor muy benigno). tpico de inmunodeprimidos y relacionado con infecciones por el
3- El tumor supratentorial ms frecuente en nios es? virus de Epstein-Barr (opcin 4).
El craneofaringioma (porque los otros dos, que son ms frecuentes, Se trata de un tumor muy maligno, con un mal pronstico. Desde
no se localizan a nivel supratentorial). un punto de vista histolgico suele ser una neoplasia de alto grado
compuesta en su mayora por clulas B. Por tanto, la opcin 1 es
falsa; el linfoma cerebral primario suele ser un linfoma de clulas B,
Fjate en que una sola palabra en el enunciado cambia todo el pero de alto grado.
sentido de la pregunta. Evidentemente la respuesta errnea en esta El tratamiento con corticoides est indicado en todos los pacientes
pregunta es la 2, ya que el tumor supratentorial ms frecuente en los
Comentarios TEST

con tumores cerebrales porque son muy eficaces para reducir la HTIC
nios no es el astrocitoma cerebral, sino el craneofaringioma. secundaria al edema vasognico que estos producen. Pues bien, en el
Como ya has aprendido, los tumores cerebrales en los nios son ms caso del linfoma cerebral primario, la respuesta a los corticoides (dexa-
frecuentes a nivel infratentorial, y por eso suelen debutar con clnica de metasona) es espectacular, hasta el punto de llegar a desaparecer la
HTIC (opcin 3) y no con clnica focal (que es la forma de debut ms imagen radiolgica tras su administracin, lo que no quiere decir que se
frecuente en los adultos, en los que suelen ser supratentoriales). Los curen con corticoides. Se han llamado por esto tumores fantasma.
tumores de fosa posterior (infratentoriales) dan clnica de HTIC rpida- Sin embargo el tratamiento de eleccin del linfoma cerebral primario
mente porque la fosa posterior es muy pequea, y cualquier creci- es la radioterapia (opcin 5) y actualmente suele combinarse con qui-
miento en su interior eleva muy deprisa la presin intracraneal. mioterapia, siendo el agente de eleccin el metotrexate.
Finalmente, la opcin 5 es correcta porque el meduloblastoma, No olvides que el linfoma cerebral es una de las lesiones que capta
un tumor tpico de los nios como ya hemos visto, suele ser un tumor contraste en anillo en la TAC, y que debes incluir en el diagnstico
de lnea media que afecta al vermis y al techo del 4 ventrculo (re- diferencial de las lesiones con este patrn de imagen junto al
cuerda que hay que hacer diagnstico diferencial con el ependimo- glioblastoma multiforme, las metstasis y los abscesos cerebrales (pi-
ma, que se localiza en el suelo del 4 ventrculo). genos o bacterianos y por toxoplasma).

Pregunta 13. Tumores cerebrales ms frecuentes en nios y adultos. Pregunta 16.- R: 2


El meningioma es uno de los tumores intracraneales importantes
12345 6789
45 de cara al MIR. Debes conocer su frecuencia, localizacin ms habi-
tual y tratamiento. Otros datos como la clnica dependen de la loca-
4  123456789
79 44
892 1234767 6 62 lizacin y los podrs deducir.
   2349 679 2 234 9 679 2 Los meningiomas son los terceros tumores intracraneales en fre-
  2348 9 8  9 2 234  9 2 cuencia en el adulto, despus de las metstasis y los gliomas. Sin em-
bargo, como estos dos ltimos son intraparenquimatosos, el menin-
4 8 7798 2 #4 8 7798 2
9 !4"8 7798 2 4"8 7798 2
gioma es el tumor intracraneal extraparenquimatoso ms frecuen-
te.

M exico A rgentina CTO Medicina C/ Nez de Balboa, 115 28006 MADRID (Espaa) Tfno.: (91) 782 43 32 / Fax: (91) 782 43 27
C hile U ruguay E-mail: secretaria@ctomedicina.com; iberocto@ctomedicina.com WEB: www.ctomedicina.com; www.iberocto.com
NC Pg. 7
NEUROCIRUGA
Seguimiento a distancia Preparacin Examen de Seleccin 05/06 1 Vuelta
Debes recordar que el meningioma es un tumor derivado de clu-
las aracnoideas (opcin 5) y por tanto de la aracnoides (leptomenin- 1- Cul es el tumor ms frecuente en la fosa posterior del adulto?
ge), no de la duramadre. Por ello podemos encontrarlo en todas Las metstasis.
aquellas localizaciones craneales y espinales en las que se encuentra 2- Cul es el tumor primario ms frecuente en la fosa posterior del
aracnoides y, aunque tambin puede aparecer en la base craneal, es adulto?
mucho ms frecuente a nivel de la convexidad (particularmente la El hemangioblastoma.
convexidad parasagital), por lo que la opcin falsa es la 2. 3- Cul es el tumor primario intraaxial ms frecuente en la fosa
Aunque pueden verse a distintas edades y en ambos sexos, son ms posterior del adulto?
frecuentes en mujeres que en hombres, y sobre todo en las dcadas El hemangioblastoma.
5-6 de la vida (opcin 1). 4- Cul es el tumor primario extraaxial ms frecuente en la fosa
La respuesta 3 es cierta, y hace referencia a los raros casos en que posterior del adulto?
los meningiomas se asocian con sndromes neurocutneos, tambin El neurinoma del VIII par (despus iran, por orden de frecuencia,
conocidos como facomatosis. En concreto, el meningioma puede el meningioma y el epidermoide).
asociarse con la neurofibromatosis tipo II, trastorno caracterizado 5- Cul es el tumor ms frecuente en la fosa posterior del nio? El
genticamente por una alteracin en el cromosoma 22. En estos casos astrocitoma de cerebelo (recuerda que las metstasis son excep-
los meningiomas son con frecuencia mltiples y aparecen a edades cionales en nios).
ms tempranas. Recuerda sin embargo que el tumor intracraneal tpi- 6- Cul es el tumor primario ms frecuente en la fosa posterior del
camente asociado con la neurofibromatosis tipo II es el neurinoma nio?
del VIII par (habitualmente bilateral en estos pacientes, siendo criterio El astrocitoma de cerebelo.
diagnstico de esta enfermedad). 7- Cul es el tumor primario maligno ms frecuente en la fosa
Algunas preguntas recientes del examen MIR han hecho referencia posterior del nio?
al tratamiento del meningioma. Partimos de la base de que la mayor El meduloblastoma (recuerda que el astrocitoma de cerebelo suele
parte de los meningiomas son tumores benignos, y que por el hecho ser muy benigno).
de estar localizados extraparenquimatosos, el cerebro no est infiltra-
do habitualmente, sino desplazado. Por tanto, si se extirpa el tumor
completamente el paciente est curado. As, el tratamiento de elec- Volvemos ahora al hemangioblastoma. Efectivamente, como ya
cin del meningioma es la extirpacin quirrgica, y generalmente has visto, la opcin 1 es verdadera; es el tumor primario intraparen-
no requiere tratamientos complementarios. El factor ms importante quimatoso de fosa posterior ms frecuente en el adulto. Concreta-
para evitar recidivas es la extensin de la ciruga. Sin embargo, debe- mente se localiza a nivel de hemisferios cerebelosos. Recuerda esta
mos tener en cuenta que, por su localizacin, a veces de difcil abor- caracterstica; el hemangioblastoma (en el adulto) y el astrocitoma (en
daje quirrgico, algunos meningiomas no son extirpables por com- los nios) son tumores cerebelosos pero hemisfricos, mientras que el
pleto. meduloblastoma (tpico de nios) es vermiano o de lnea media.
Hay que tener claro que el meningioma se trata con ciruga. Sin Debes recordar la asociacin entre hemangioblastoma de cerebe-
embargo, en algunos casos puede estar indicada la radioterapia, en lo y enfermedad de von Hippel-Lindau, una facomatosis que carac-
concreto en los casos de restos o recidivas que continan creciendo tersticamente se asocia con este tumor y con la presencia de angiomas
(opcin 4) o cuando la histologa es muy agresiva (meningiomas ma- retinianos. Sin embargo, la respuesta 3 es falsa porque la gran mayo-
lignos). ra de los hemangioblastomas son espordicos (slo un 20% se aso-
cian con von Hippel-Lindau).
La otra caracterstica a recordar de este tumor es su capacidad
para producir eritropoyetina (EPO), por lo que es frecuente que
los pacientes que lo padecen tengan una elevacin del hematocrito,
que es la alteracin hematolgica tpica a la que se refiere la res-
puesta 5, y a la que han hecho referencia todas las preguntas MIR
sobre este tumor.
Ya has visto que el hemangioblastoma se localiza en los hemisferios
cerebelosos. Su imagen tpica en la TAC es una lesin qustica, con un
ndulo mural (respuesta 4). Esto significa que en alguna parte de la

Comentarios TEST
pared del quiste hay un engrosamiento, que es la parte slida del
tumor, que habr que resecar en la ciruga, que es el tratamiento de
eleccin (respuesta 2).
En resumen, debes quedarte con la idea de que el hemangioblas-
toma te lo preguntarn en casos clnicos como un paciente con clni-
ca cerebelosa, que tiene el hematocrito alto y un tumor qustico en un
hemisferio cerebeloso.

Pregunta 18.- R: 2
Los neurinomas son tumores derivados de las clulas de Schwann,
y por eso se llaman tambin schwannomas. A nivel intracraneal los
podemos encontrar como tumores de los nervios craneales, pero son
Pregunta 16. Meningioma de la hoz cerebral. ms frecuentes a nivel espinal como tumores de las races nerviosas
raqudeas.
Pregunta 17.- R: 3 La respuesta correcta es la 2 porque los neurinomas intracranea-
El hemangioblastoma es otro de los tumores intracraneales que les ms frecuentes son los neurinomas del VIII par, seguidos de los del
son preguntados con cierta frecuencia en el examen MIR, y te adelan- V par; el resto son muy raros.
tamos que habitualmente lo preguntan como caso clnico y el dato El neurinoma del VIII par es un tumor importante en el MIR. Habi-
clave suele ser que es un tumor de fosa posterior y se asocia con tualmente se conoce como neurinoma del acstico, aunque esto es
elevacin del hematocrito. incorrecto, porque estos tumores crecen a partir del componente
Vamos a aprovechar la opcin 1 para hacer juegos de palabras vestibular del VIII par, no del acstico. Debes recordar este dato
con los tumores de la fosa posterior. Fjate bien, porque cambiando porque te puede inducir a error.
una sola palabra del enunciado, te estaran preguntando por distintos Del neurinoma del VIII par es importante conocer que se trata del
tumores. Por ejemplo: tumor ms frecuente del ngulo pontocerebeloso (el segundo es el

M exico A rgentina CTO Medicina C/ Nez de Balboa, 115 28006 MADRID (Espaa) Tfno.: (91) 782 43 32 / Fax: (91) 782 43 27
Pg. 8 NC C hile U ruguay E-mail: secretaria@ctomedicina.com; iberocto@ctomedicina.com WEB: www.ctomedicina.com; www.iberocto.com
NEUROCIRUGA
Preparacin Examen de Seleccin 05/06 1 Vuelta Seguimiento a distancia
meningioma y el tercero el epidermoide). Tambin te lo pueden pre- marcadores tumorales. Los ms tpicos son la gonadotropina corini-
guntar como el tumor extraaxial de la fosa posterior ms frecuente. ca en el coriocarcinoma y la alfafetoprotena en el tumor del seno
Clnicamente produce sintomatologa ORL: hipoacusia neurosen- endodrmico. Recuerda que, en el caso del germinoma, los marca-
sorial, acfeno y vrtigo, por lo que suele ser diagnosticado por esos dores tumorales son habitualmente negativos.
especialistas, pero en su crecimiento puede afectar a otros pares Te preguntars el motivo por el que se separa el germinoma de los
craneales (el V y el VII fundamentalmente), e incluso al cerebelo, al llamados tumores germinales no germinomatosos. La razn es que el
tronco del encfalo y pares bajos. germinoma tiene una extraordinaria radiosensibilidad, por lo que su
Debes conocer la asociacin entre neurinoma del VIII par y neuro- tratamiento de eleccin es la radioterapia. Para el resto, que no res-
fibromatosis tipo II (en estos casos suelen ser neurinomas bilaterales). ponden bien a RT, la alternativa es la ciruga. Una vez ms llamamos tu
El tratamiento de eleccin es la ciruga o la radiociruga estereo- atencin sobre los marcadores tumorales (si son negativos, hay que
tctica. sospechar germinoma y puede curarse con radioterapia sin necesi-
dad de ciruga).

Pregunta 20.- R: 3
Esta es una pregunta directa acerca del quiste coloide cerebral.
El quiste coloide es un tumor disembrioplsico, como lo es el
craneofaringioma. El primero se supone derivado de la parfisis, y se
localiza en la porcin anterior del tercer ventrculo (opciones 1 y 2).
El craneofaringioma, mucho ms preguntado en el MIR, se origina a
partir de restos embrionarios de la bolsa de Rathke, y se localiza en la
regin supraselar.
Respecto al quiste coloide, debes recordar que es un tumor tpico
del tercer ventrculo, como ya hemos visto, y por tanto puede produ-
cir obstruccin para la circulacin del LCR, originando una hidroce-
falia obstructiva. Lo que es tpico de este tumor es que, al comportarse
como una lesin qustica dentro del tercer ventrculo, puede despla-
zarse con los movimientos de la cabeza, de modo que la obstruccin
puede variar segn la posicin, y puede ser causa de hidrocefalias
agudas intermitentes por bloqueo de los agujeros de Monro (opcin
5). Ahora es infrecuente porque con la TAC se diagnostica ms
precozmente, pero antes se describa como una causa de muerte
sbita por este motivo.
Pregunta 18. Neurinoma del VIII par.
Por tanto, debes recordar que es un tumor qustico del tercer
ventrculo que puede originar hidrocefalia intermitente. Muy caracte-
Pregunta 19.- R: 4 rstico de este tumor es que en las tinciones histolgicas, como dice la
De nuevo nos enfrentamos a un caso clnico que debemos inter- opcin 4, se aprecia un contenido glucoproteico que se tie con
pretar. Podramos resumirlo como un nio con alteraciones endocri- cido peridico de Schiff (PAS +).
nolgicas (diabetes inspida), alteraciones focales (dificultad para la La respuesta falsa es la 3, porque no se trata de un tumor glial, sino
elevacin de la mirada) y clnica de HTIC (cefalea). que tiene una cpsula verdadera de origen epitelial.
De todos estos datos, las alteraciones endocrinolgicas nos lleva-
ran a pensar en un tumor que est afectando al hipotlamo o alrede-
dores (otras estructuras muy mediales y basales). Todos los tumores
descritos, especialmente 1, 2, 4 y 5 cursan con alteraciones endocri-
nolgicas. Por tanto, este dato no es muy til para hacer el diagnstico
diferencial.
Tampoco lo es la cefalea, sntoma comn a la mayor parte de los
Comentarios TEST

tumores cerebrales.
Sin embargo hay un dato definitivo, en este caso el sntoma focal,
que como siempre tiene un valor localizador. El nio tiene una difi-
cultad para la elevacin de la mirada, lo que nos obliga a pensar en
un sndrome de Parinaud, y dirige las sospechas hacia la regin pineal.
Recuerda que en las lesiones de la regin pineal, y con frecuen-
cia tambin en la hidrocefalia, se puede producir este sndrome,
tambin conocido como sndrome mesenceflico dorsal, que cursa
con parlisis de la elevacin de la mirada con ausencia de reflejo
fotomotor, parlisis de la convergencia y reflejo de acomodacin
conservado.
Por tanto, la respuesta ms probable es la opcin 4: germinoma
de la regin pineal.
De los tumores de regin pineal debes conocer que son ms fre-
cuentes en nios y pacientes jvenes que en adultos. Hay que dife- Pregunta 20. Quiste coloide del III ventrculo.
renciar dos tipos de tumores en esta regin: los tumores derivados
del parnquima pineal (pineocitoma y pineoblastoma), que son ms Pregunta 21.- R: 3
raros, y los tumores derivados de clulas germinales (mucho ms Esta es una pregunta importante, porque en ocasiones han pregun-
frecuentes). tado en el examen por la asociacin entre determinadas enfermeda-
Los tumores de clulas germinales son los ms frecuentes en esta des neurocutneas (tambin conocidas como facomatosis) y tumores
regin, especialmente el germinoma. Hay otros ms raros que reci- del sistema nervioso central.
ben tambin el nombre de tumores germinales no germinomatosos Las ms importantes se recogen en la tabla siguiente, y debes recor-
(es decir, los que no son germinomas). Es importante conocer que en darlas:
estos tumores pueden encontrarse elevados en sangre y LCR ciertos

M exico A rgentina CTO Medicina C/ Nez de Balboa, 115 28006 MADRID (Espaa) Tfno.: (91) 782 43 32 / Fax: (91) 782 43 27
C hile U ruguay E-mail: secretaria@ctomedicina.com; iberocto@ctomedicina.com WEB: www.ctomedicina.com; www.iberocto.com
NC Pg. 9
NEUROCIRUGA
Seguimiento a distancia Preparacin Examen de Seleccin 05/06 1 Vuelta
rio cerebeloso que caractersticamente secreta eritropoyetina, por lo
Pregunta 21. Facomatosis y tumores del Sistema Nervioso Central. que suele elevar el hematocrito) y angiomas retinianos.
Esclerosis tuberosa. Astrocitoma gigantocelular Recuerda que la enfermedad de Sturge-Weber se conoce tam-
subependimario. bin como angiomatosis encfalo-trigeminal porque presenta an-
Neurofibromatosis tipo I. Glioma de vas pticas. giomas (nevus flameus) en el territorio sensitivo del trigmino (primera
Neurofibromatosis tipo II. Neurinoma bilateral del VIII par. rama generalmente) y con frecuencia retinianos, con calcificaciones
Meningiomas. cerebrales y crisis epilpticas.
Sturge-Weber. Angiomas leptomenngeos.
Von Hippel-Lindau. Hemangioblastoma cerebeloso. Pregunta 22.- R: 4
Klippel-Trenaunay. Angioma cavernoso de la Como ya hemos visto, son frecuentes y caractersticas las asociacio-
mdula espinal. nes entre sndromes neurocutneos (facomatosis) y tumores cerebrales.
La neurofibromatosis tipo I (opcin 2) se asocia frecuentemente
con gliomas de las vas visuales. La enfermedad de Bourneville, tam-
Como ya habrs adivinado, la respuesta correcta es la opcin 3: bin conocida como esclerosis tuberosa, se asocia con el astrocitoma
neurofibromatosis tipo I, que caractersticamente asocia tumores de las gigantocelular subependimario, un glioma de bajo grado de maligni-
vas visuales, y en concreto el glioma del nervio ptico. Las malformacio- dad.
nes de los huesos largos (pseudoartrosis de la tibia) y los hamartomas de Pero no slo hay asociaciones con las facomatosis, tambin se
iris (que originan una heterocroma, o diferencia de color entre los dos pueden ver con otras enfermedades sistmicas.
ojos) son otros de los criterios diagnsticos, junto con las manchas color La opcin 1 hace referencia a la neoplasia endocrina mltiple
caf con leche, las eflides (pecas) axilares, los neurofibromas cutneos, tipo I (MEN I), que es una enfermedad sistmica que asocia hiperpa-
etc. Los criterios diagnsticos de neurofibromatosis tipo I se recogen en la ratiroidismo primario familiar, tumores hipofisarios (generalmente
tabla a continuacin (debe reunir 2 o ms de los siguientes): adenomas), tumores pancreticos e hipergastrinemia con enferme-
dad ulcerosa pptica (sndrome de Zollinger-Ellison). No la confun-
das con la neoplasia endocrina mltiple tipo II (MEN II), en la que el
Pregunta 21. Criterios diagnsticos de neurofibromatosis tipo I. hiperparatiroidismo primario se asocia con feocromocitoma y carci-
noma medular de tiroides, pero no con tumores hipofisarios.
1- 6 o ms manchas caf con leche de ms de 5 mm de dimetro
El sndrome de Turcot (opcin 5) es una rara variante de poliposis
(prepuberal) o ms de 15 mm (postpuberal).
colnica familiar que se asocia con tumores cerebrales malignos.
2- 2 o ms neurofibromas de cualquier tipo, o un neurofibroma
La respuesta correcta a esta pregunta es la opcin 4, porque la
plexiforme.
enfermedad de Lafora no incluye tumores cerebrales entre sus hallaz-
3- Eflides axilares o inguinales.
gos. Se trata de una forma hereditaria de epilepsia mioclnica progre-
4- Glioma del nervio ptico.
siva, con herencia autosmica recesiva.
5- Ndulos de Lisch (hamartomas de iris).
6- Anomalas seas como displasia del ala del esfenoides o adelgaza-
TRAUMATISMOS CRANEOENCEFLICOS (TCE).
miento de la cortical de los huesos largos con o sin pseudoartrosis
(tibia y peron).
Pregunta 23.- R: 3
7- Un familiar de primer grado con diagnstico de neurofibromatosis
Nos presentan un caso clnico de un paciente que ha sufrido un
tipo I segn los criterios anteriores.
traumatismo craneoenceflico, y nos piden que decidamos sobre la
actitud ms correcta con este paciente. Ojo! con estas preguntas,
No es imprescindible que memorices todos los criterios (no los han porque se han puesto de moda en los ltimos exmenes MIR y,
preguntado nunca), pero conviene familiarizarse con ellos, sobre todo aunque no suelen ser difciles, requieren analizar con cuidado la
con la relacin con el glioma del nervio ptico. situacin.
Por su parte, la neurofibromatosis tipo II tambin se relaciona Antes de contestar la pregunta debes recordar que, a la hora de
con tumores del SNC, en concreto con el neurinoma bilateral del valorar a un paciente con un traumatismo craneoenceflico, el dato
VIII par (tambin llamado neurinoma del acstico, aunque es un ms importante (el de mayor valor pronstico) es el nivel de concien-
trmino incorrecto porque como sabes se origina en la porcin vesti- cia, y para valorarlo de manera homognea en todos los sitios se
bular del VIII par, no en la acstica). Los criterios diagnsticos de describi una escala aceptada universalmente, que es la Escala de

Comentarios TEST
neurofibromatosis tipo II estn recogidos en la tabla: Coma de Glasgow (GCS) recogida en la tabla a continuacin. Aun-
que no hay ninguna pregunta en este test sobre ella, debes repasarla
porque la han preguntado en alguna ocasin. Lo ms importante es
Pregunta 21. Criterios diagnsticos de neurofibromatosis tipo II. conocer que valora tres parmetros: la apertura ocular, la respuesta
motora y la respuesta verbal, y que punta el nivel de conciencia de
1- Neurinoma bilateral del VIII par o 3 a 15 (siendo 15 la puntuacin mxima).
2- Un familiar de primer grado con diagnstico de neurofibromatosis
tipo II y uno de los siguientes:
a. Neurinoma unilateral del VIII par o Pregunta 23. Escala de Coma de Glasgow.
b. Dos de los siguientes:
Neurofibroma. PUNTUACIN
APERTURA RESPUESTA RESPUESTA
Meningioma. DE OJOS MOTORA VERBAL
Glioma.
6 Obedece rdenes
Schwannoma
Catarata subcapsular juvenil 5 Localiza el dolor Orientado
4 Espontnea Retira al dolor Confuso
Tampoco han preguntado nunca estos criterios, y lo importante es Flexora
que recuerdes la asociacin con el neurinoma bilateral del VIII par. 3 A la voz
(decorticacin)
Inapropiado
Otro glioma que se asocia a las facomatosis es el astrocitoma
gigantocelular subependimario, un tumor de bajo grado de maligni- 2 Al dolor
Extensin
Incomprensible
dad que se ve asociado a la esclerosis tuberosa. (descerebracin)
Finalmente, recuerda la asociacin entre enfermedad de von 1 No No No
Hippel Lindau y hemangioblastoma cerebeloso (tumor de hemisfe-

M exico A rgentina CTO Medicina C/ Nez de Balboa, 115 28006 MADRID (Espaa) Tfno.: (91) 782 43 32 / Fax: (91) 782 43 27
Pg. 10 NC C hile U ruguay E-mail: secretaria@ctomedicina.com; iberocto@ctomedicina.com WEB: www.ctomedicina.com; www.iberocto.com
NEUROCIRUGA
Preparacin Examen de Seleccin 05/06 1 Vuelta Seguimiento a distancia
Volviendo a la pregunta que nos ocupa, describen a un paciente do y para reconstruir la duramadre y reparar las lesiones cerebrales
que ha tenido un golpe en la cabeza, con prdida de conciencia subyacentes. En este caso s se suelen aadir antibiticos para pre-
inicial aunque ahora est consciente, que tiene una exploracin venir infecciones, porque la duramadre suele estar rota.
neurolgica normal, y al que se ha realizado una Rx de crneo que El tercer tipo son las fracturas de base de crneo, a las que se
muestra una fractura lineal. refiere esta pregunta. Las fracturas de la base del crneo se ven con
Lo primero que hay que pensar es que si ha perdido la conciencia mucha dificultad en las Rx simples de crneo, por lo que la prueba
inicialmente, y tiene una fractura en el crneo, el golpe ha sido de de eleccin para diagnosticarlas suele ser la TAC craneal, que ade-
suficiente intensidad como para que el paciente permanezca en ob- ms permite ver las lesiones intracraneales asociadas, de las que
servacin. No sera una buena medida darle de alta a su domicilio, depender el pronstico. Recuerda que hay ciertos signos en la
como dice la opcin 1, sin saber si adems de la fractura tiene alguna exploracin fsica del paciente que nos llevan a sospechar que
otra lesin intracraneal. Por tanto, descartamos esta opcin. puede tener una fractura en la base del crneo: hematoma
Las otras 4 opciones insisten en que debe realizarse una TAC cra- periorbitario en ojos de mapache, hematoma retroauricular (sig-
neal, medida que est justificada para saber si por debajo de la fractu- no de Battle), o salida de LCR o sangre por el odo o la nariz.
ra hay alguna lesin intracraneal asociada. De las fracturas de base de crneo, debes recordar las
Descartamos la opcin 2 porque el hecho de que la TAC craneal sea frontoetmoidales (que suelen asociar rinolicuorrea, es decir, salida
normal inicialmente no asegura que pueda aparecer alguna hemorra- de LCR por las fosas nasales) y las de peasco, a las que se refiere esta
gia intracraneal de forma diferida. Con los hallazgos descritos, y aunque pregunta (que pueden asociar otolicuorrea y otorragia, es decir, salida
la exploracin neurolgica sea normal y la TAC craneal no muestre de LCR o sangre por el CAE).
alteraciones, el paciente debe quedar en observacin unas horas. Las fracturas de peasco se dividen segn la direccin del trazo de
La opcin 4 es tambin incorrecta, porque las fracturas lineales del fractura, visible en la TAC craneal, en: longitudinales, transversales y
crneo no precisan tratamiento quirrgico. Aunque suene informal, oblicuas. Si te preguntan algo acerca de ellas, ser en relacin con el
el crneo no se puede escayolar, y por tanto una fractura lineal no riesgo de otorragia (es decir, de lesin del tmpano), hipoacusia (que
desplazada se trata exclusivamente con analgsicos, dejando que con- puede ser neurosensorial, por lesin del odo interno, o de conduc-
solide espontneamente. Slo se operan las fracturas con hundimien- cin, por lesin del odo medio o del tmpano) y parlisis facial.
to de la bveda craneal, pero no las lineales. En la tabla de la pgina siguiente tienes recogidas las manifestacio-
La opcin 5 tambin es incorrecta. Una fractura lineal no implica nes ms habituales de cada una de ellas:
mayor riesgo de infeccin intracraneal, aunque exista una herida en La manera ms sencilla para recordarlas es imaginarse el trazo de
el cuero cabelludo. Slo est aumentado el riesgo de infeccin si hay fractura:
fuga de lquido cefalorraqudeo (lo que significa que la duramadre Las fracturas longitudinales se llaman tambin timpnicas, por-
est abierta), y en esos casos s que se asocian antibiticos, pero no son que afectan al odo medio tmpano CAE. Por tanto, habitual-
necesarios en el caso descrito. mente muestran desgarro timpnico y otorragia, hipoacusia de
Por tanto, la respuesta correcta es la 3. Realizaremos una TAC cra- conduccin, y es raro que afecten al nervio facial (slo el 20% de
neal para descartar lesiones intracraneales asociadas y dejaremos al ellas asocia parlisis facial).
paciente en observacin durante 24 horas para estar seguros de que no Las fracturas transversales se llaman tambin neurosensoriales,
aparece ninguna complicacin diferida no visible en la primera TAC. porque afectan al odo interno CAI agujero rasgado posterior.
Por cierto, recuerda que la complicacin que deberas esperar en Por tanto habitualmente el tmpano est intacto, aunque pueden
este paciente, aunque no necesariamente tiene que ocurrir, es la apa- mostrar hemotmpano, tienen hipoacusia neurosensorial, y es fre-
ricin de un hematoma epidural por sangrado de la fractura lineal cuente la lesin del nervio facial (hasta en un 50%).
que presenta. Si adems te dijeran que la fractura es de la escama del Las fracturas oblicuas se llaman tambin timpanolaberntica, y
hueso temporal, la sospecha sera mayor, porque debajo de dicha muestran caractersticas mixtas entre las dos anteriores. En ellas es
escama se encuentra la arteria menngea media con sus ramas, cuya casi constante la lesin del nervio facial.
rotura es la principal causa del hematoma epidural. Por tanto, la respuesta falsa es la 2, porque en las fracturas trans-
versales del peasco se afecta el odo interno, y en consecuencia la
Pregunta 24.- R: 2 hipoacusia es neurosensorial, no de conduccin.
Por su localizacin, podemos distinguir distintos tipos de fracturas El resto de opciones, como ya hemos explicado, son ciertas.
craneales, esencialmente las de bveda craneal (lineales o fractura-
Comentarios TEST

hundimiento) y las de base del crneo.


Recientemente han hecho algunas preguntas en el examen MIR Pregunta 24. Fracturas del peasco.
sobre fracturas de crneo. Lo importante es que conozcas las implica-
ciones que tienen, si requieren tratamiento quirrgico o no, y si nece- 12345623789
56
387 87349
sitan o no profilaxis antibitica por riesgo de infeccin intracraneal. 7325 4 34 
722
57 879 97 9
Intentaremos, antes de contestar la pregunta, dar unas pinceladas
sobre los puntos que hemos mencionado. 1234567859
6
Una cosa general a todas las fracturas es que el pronstico no 12 4
62759 6
depende de la lesin sea (si se rompe el hueso, no pasa nada), de- 123456

242
pende de las lesiones intracraneales asociadas. Por tanto, en todo 12
 2
2 4242
2759

paciente con lesiones seas hay que indagar sobre si existen otras 12
 24 2422 26 2
7
2962594 242 6
lesiones, explorando al paciente y haciendo pruebas de imagen (la 2225


TAC es la de eleccin en los traumatismos). 12396

2426 
Las fracturas lineales no precisan tratamiento quirrgico. El pro- 12!
2

2"#$ %2 46
9
&
262
&6 675429


nstico, como en todas las fracturas, depende de las lesiones aso- 12'7664
2 44 74
ciadas. No precisan tratamiento antibitico profilctico porque no 123456237523
2387 87349
hay mayor riesgo de infeccin intracraneal (este riesgo slo est 325 4 34 
722
57 879 97
52
9
aumentado si hay salida de LCR, es decir, si la dura est abierta).
Otra cosa es que presenten heridas anfractuosas en cuero cabellu- 12(859
62 7
7626 2)4567859
6
do, con prdida de sustancia, en cuyo caso se asocian antibiticos, 122 7
76
pero no por la fractura sino por la herida. 12396

2 464 6

Las fracturas hundimiento de la bveda craneal son ms graves 12*+76
que las lineales, porque el fragmento de hueso hundido puede 12,7
562496 7 462)

2426862
6
afectar a estructuras intracraneales. Por ello estas fracturas suelen 12!
2

2-#$ %2
&6 6754262 46754
requerir tratamiento quirrgico para retirar el fragmento deprimi- 12.64
2674
2/424
2962
27659
24 249
 4

M exico A rgentina CTO Medicina C/ Nez de Balboa, 115 28006 MADRID (Espaa) Tfno.: (91) 782 43 32 / Fax: (91) 782 43 27
C hile U ruguay E-mail: secretaria@ctomedicina.com; iberocto@ctomedicina.com WEB: www.ctomedicina.com; www.iberocto.com
NC Pg. 11
NEUROCIRUGA
Seguimiento a distancia Preparacin Examen de Seleccin 05/06 1 Vuelta
Pregunta 25.- R: 4 La opcin 3 no es cierta porque el hematoma epidural, como ya
De nuevo se trata de un caso clnico, y debes fijarte especialmente sabes, se debe a rotura de arterias de la duramadre o arterias menn-
en l porque es una de las preguntas ms habituales de la asignatura geas (generalmente la arteria menngea media). Sin embargo, el hema-
en el examen MIR. toma subdural s que se suele producir por rotura de las pequeas
Se trata de un paciente joven que, tras un accidente de trfico, y venas puente corticales (son venas entre el cerebro y la duramadre, y
con un intervalo asintomtico previo, presenta deterioro del nivel de por tanto su rotura produce sangrado subdural).
conciencia y unas anomalas en la exploracin neurolgica compa- Finalmente la opcin 5 es falsa porque, una vez ms, no existe
tibles con herniacin uncal del lado izquierdo (tiene anisocoria, pro- aumento del riesgo de infeccin intracraneal en las fracturas lineales
bablemente por midriasis izquierda secundaria a compresin del III (solamente en los casos en que est rota la duramadre y sale LCR al
par craneal, y prdida de fuerza en hemicuerpo derecho por com- exterior).
presin del pednculo cerebral izquierdo, que es el lugar por el que En la tabla siguiente se recogen las principales diferencias entre
discurren las fibras motoras en el mesencfalo). En la placa de crneo hematoma epidural y subdural agudo.
tiene una fractura de la escama del hueso temporal izquierda.
En resumen: paciente joven con traumatismo craneoenceflico,
inicialmente asintomtico, que ahora presenta deterioro de concien- Pregunta 25. Diagnstico diferencial entre hematoma epidural
cia secundario a herniacin uncal. Hay que sospechar que el pacien- y subdural agudo.
te tiene un hematoma epidural temporal izquierdo.
Hay varios datos en la historia que orientan a este diagnstico. La
HEMATOMA EPIDURAL HEMATOMA SUBDURAL
localizacin de la fractura es de alto riesgo. Como sabes, las fracturas
lineales no tienen ninguna gravedad por s solas, sino que el pronsti- R otura de venas corticales, y
co depende de las lesiones intracraneales asociadas. Siempre que un puede ser:
paciente tiene una fractura debe realizarse una TAC craneal para Lesin arterial (la ms - Agudo (en la primera semana,
descartar estas lesiones. La complicacin intracraneal ms frecuente- Origen frecuente es la arteria que es el que nos ocupa).
mente asociada con las fracturas lineales es el hematoma epidural, menngea media). - Subagudo (7-10 das despus).
- Crnico (tpico de alcohlicos y
sobre todo si estas afectan a la escama del hueso temporal, porque ancianos).
justo debajo de ella se encuentran la arteria menngea media y sus
ramas. Recuerda que la rotura de la arteria menngea media es la Tpicamente, perodo breve de
causa ms frecuente del hematoma epidural, y esto ha sido frecuente- inconsciencia seguido de
No presenta intervalo lcido.
mente motivo de pregunta MIR. intervalo lcido y,
C lnica Disminucin de conciencia desde
posteriormente, coma de
Otro dato a favor de que se trate de un hematoma epidural es la rpida evolucin (herniacin
el inicio.
evolucin clnica. No olvides que la secuencia clsica es: prdida de uncal).
conciencia inicial con recuperacin posterior, seguida de un intervalo
lcido, con deterioro posterior secundario generalmente a herniacin Imagen hiperdensa en forma Imagen hiperdensa
TAC
del uncus del lbulo temporal (lo que provoca anisocoria por compre- de lente B ICONVEXA. en forma de SEMILUNA.
sin del III par ipsilateral con hemiparesia contralateral). Es la misma Lesin Mayor y desde el inicio,
En general menor y ms tarda
secuencia que describen en el enunciado. Sin embargo, debes saber directa del por estar la sangre en contacto
por compresin.
que no siempre se ve esta secuencia (a veces no hay intervalo lcido), parnquima con el parnquima cerebral.
aunque en las preguntas MIR suelen ponerla siempre as. Menor,con diagnstico y 25-90% segn el grado de
Por tanto, se trata de un hematoma epidural. Si hiciramos una Mortalidad tratamiento precoz casi del afectacin parenquimatosa
TAC craneal, veramos una lesin hiperdensa (blanca) con forma de 0%. directa.
lente biconvexa (a diferencia del hematoma subdural, que tiene for-
ma de semiluna). Tratamiento Craneotoma y evacuacin. Craneotoma y evacuacin.
Recuerda que denominamos hematoma epidural al que se en-
cuentra entre el hueso y la duramadre, y que por tanto no contacta Pregunta 26.- R: 4
con el cerebro, frente al hematoma subdural, que se localiza por El dao o lesin axonal difusa es una lesin postraumtica que, a
debajo de la duramadre, en contacto con el cerebro, y suele acom- diferencia del hematoma subdural, el hematoma epidural o las con-
paarse de lesiones del parnquima cerebral subyacente. tusiones hemorrgicas intraparenquimatosas, no tiene tratamiento

Comentarios TEST
La respuesta cierta es evidentemente la 4. Hay signos de hernia- quirrgico. Como su propio nombre indica, se trata de una lesin de
cin transtentorial, que es otra forma de llamar a la herniacin uncal. los axones de las neuronas, y por tanto no es reparable con ciruga.
La respuesta 1 es errnea porque el hematoma epidural, si se Del nombre puedes deducir tambin que las lesiones se encontrarn
opera a tiempo (se trata de una emergencia neuroquirrgica), tiene un principalmente en los lugares por los que discurren los axones, y en
extraordinario pronstico de recuperacin funcional, con una mor- concreto en la sustancia blanca. Adems, tambin puedes deducir
talidad muy baja (inferior al 10%). Todo lo contrario del hematoma que no es una lesin focal, sino difusa por todo el cerebro.
subdural agudo, que tiene una mortalidad y morbilidad muy altas La primera opcin que podemos descartar por tanto es la opcin
(porque suele acompaarse de lesiones del parnquima cerebral sub- 5. La lesin axonal difusa, que es un dao irreversible de los axones
yacente). de mltiples neuronas y de forma difusa en el cerebro, tiene muy mal
La respuesta 2 tambin es incorrecta. Tanto el hematoma epidural pronstico funcional.
como el subdural son eso.... hematomas. Por tanto son slidos; es No es cierta tampoco la opcin 2. El mecanismo principal de lesin
sangre coagulada. El tratamiento de eleccin de ambos es quirrgico, en estos pacientes son los movimientos de aceleracin-desaceleracin,
pero para quitarlos es necesario hacer una craneotoma (es decir, que cizallan los axones de las neuronas en la sustancia blanca.
levantar una parte del hueso para poder extirpar el hematoma). No es Podemos descartar tambin la opcin 1 si pensamos en lo que
suficiente con la realizacin de un trpano porque esto es un peque- significa lesin axonal difusa. Se trata de una lesin difusa de los axones,
o agujero de menos de 1 cm de dimetro en el hueso, y por ese que son de pequeo tamao, no de una lesin focal, por eso en
agujero no sale un cogulo slido. Sin embargo, el trpano es la tcni- muchos pacientes no se ven alteraciones en la TAC cerebral. No obs-
ca de eleccin para evacuar un hematoma subdural crnico, porque tante, cuando se ven alteraciones en la TAC, lo que vemos es un
estos no son slidos, sino lquidos (se trata de sangre evolucionada), y pequeo punteado hemorrgico en la sustancia blanca de los hemis-
por tanto se pueden aspirar a travs de un pequeo agujero. Para que ferios cerebrales y en el cuerpo calloso (precisamente los sitios por
lo recuerdes nos permitimos un pequeo smil: para comerse el flan donde discurren los axones). Por tanto, la respuesta correcta en esta
(slido) hay que quitar la tapa (craneotoma), pero para tomarse un pregunta es la opcin 4.
zumo (lquido) es suficiente con hacer un pequeo agujero (trpano) Finalmente descartamos tambin la opcin 3. Los pacientes con
y meter una pajita para aspirar (drenaje). lesin axonal difusa presentan un deterioro neurolgico importante

M exico A rgentina CTO Medicina C/ Nez de Balboa, 115 28006 MADRID (Espaa) Tfno.: (91) 782 43 32 / Fax: (91) 782 43 27
Pg. 12 NC C hile U ruguay E-mail: secretaria@ctomedicina.com; iberocto@ctomedicina.com WEB: www.ctomedicina.com; www.iberocto.com
NEUROCIRUGA
Preparacin Examen de Seleccin 05/06 1 Vuelta Seguimiento a distancia
desde el mismo momento del traumatismo, porque la rotura de los Comenzamos explicando el manejo inicial del paciente con sos-
axones se produce justamente en ese momento. De hecho, los pecha de HSA, que es muy importante (ver figura). Sospechamos que
neurocirujanos sospechamos lesin axonal difusa en un paciente con un paciente tiene una HSA cuando refiere la aparicin muy brusca de
mala situacin neurolgica inmediatamente tras un traumatismo en el un terrible dolor de cabeza (dicen los libros que es el peor dolor de
que la TAC no muestra alteraciones o muestra slo las pequeas ano- cabeza que existe) acompaado de cortejo vegetativo (sudoracin,
malas que hemos descrito. En otras palabras, hay que sospechar le- nuseas, vmitos). A la exploracin el paciente presentar signos
sin axonal difusa en un paciente con un traumatismo que est muy menngeos por la existencia de sangre en el espacio subaracnoideo (el
malito desde el principio, con una TAC craneal no tan mala. mismo mecanismo por el que hay signos menngeos en las meningitis
al existir pus en el espacio subaracnoideo). No siempre el paciente
Pregunta 27.- R: 5 tiene prdida de conciencia.
Nos preguntan sobre un cuadro clnico muy caracterstico, que si
cae en el MIR lo har precisamente contando los mismos datos que
en este enunciado.
Se trata de una mujer que ha tenido un golpe en la cabeza hace
unas 24 horas y que ahora consulta porque, de forma brusca, ha
aparecido un exoftalmos pulstil unilateral. Adems resulta que el ojo
derecho est metido hacia adentro, por lo que la seora probable-
mente ver doble, y se escucha un soplo detrs del globo ocular. No
nos lo dicen, pero probablemente el ojo correspondiente le duele
mucho y se encuentra congestivo.
Nos ofrecen cinco diagnsticos, de los cuales el ms probable, por
la exploracin clnica de la paciente y por el antecedente traumtico,
es la fstula cartido-cavernosa (opcin 5), lesin que se ve con ms
frecuencia en pacientes que presentan fracturas de la base del cr-
neo.
La fstula cartido-cavernosa es una comunicacin directa entre la
arteria cartida y el seno cavernoso (que es una estructura venosa). Se
trata por tanto de una fstula arteriovenosa, lo que supone un paso
directo de sangre a gran presin desde una arteria a una vena, y por
tanto la congestin del sistema venoso. Eso explica el motivo por el
que la paciente tiene un soplo (caracterstico de las fstulas arteriove-
nosas, en este caso retroocular, porque ese es el lugar que ocupa el
seno cavernoso). Tambin explica por qu la paciente ha desarrolla-
do bruscamente un exoftalmos en ese lado y tiene el globo ocular y
los prpados congestivos (por sobrecarga del sistema venoso).
Las alteraciones de los pares craneales oculomotores, que produ-
cen diplopa, se deben a compresin de los mismos en el interior del
seno cavernoso. El ms frecuentemente afectado es el VI par, porque
es el nico que discurre junto a la arteria cartida interna en el interior
del seno cavernoso (por eso esta paciente tiene el ojo metido hacia
adentro, por lesin del VI par). El resto de pares craneales oculomotores
(III y IV), que tambin pueden afectarse, se localizan en la pared lateral
del seno cavernoso junto a las ramas primera y segunda del V par.
El diagnstico de sospecha que se establece por la clnica y la Pregunta 28. Manejo diagnstico del paciente con HSA espontnea.
exploracin se confirmar mediante angiografa cerebral. El tratamiento
de eleccin es endovascular en los casos en los que la fstula presenta Si un paciente acude a Urgencias con un cuadro clnico como el
Comentarios TEST

alto flujo. que hemos descrito, ante la sospecha de HSA, la primera prueba
diagnstica que hay que realizar es una TAC craneal sin contraste
HEMORRAGIA SUBARACNOIDEA (HSA). (nunca con contraste, porque el contraste se ve blanco como la san-
gre y nos enmascarara la hemorragia). Por tanto, la opcin 4 es co-
Pregunta 28.- R: 2 rrecta.
Esta es una pregunta directa sobre hemorragia subaracnoidea (HSA), La TAC craneal por s sola diagnostica la mayor parte de las HSA
una patologa con gran importancia en el examen por el nmero de (ver figura a continuacin). Slo en aquellos casos en los que la sos-
preguntas que genera. Intentaremos explicarte el manejo de la HSA pecha clnica es alta pero la TAC es normal, recurriremos a la puncin
paso a paso, aunque la explicacin necesariamente debe ser un po- lumbar, que es una manera ms directa de saber si hay sangre o no en
quito extensa. el LCR. De hecho la puncin lumbar es la tcnica diagnstica ms
El primer concepto importante que hay que conocer para el exa- sensible para HSA, pero no es la primera que hacemos. Esto ha sido
men es el recogido en la opcin 1. La causa ms frecuente de hemo- objeto de pregunta MIR.
rragia subaracnoidea como tal (es decir, de sangrado en el espacio Si por TAC craneal, o por puncin lumbar, se confirma la presen-
subaracnoideo) son los traumatismos. Sin embargo, cuando en el MIR cia de sangre en el espacio subaracnoideo, tenemos diagnosticada la
preguntan por hemorragia subaracnoidea, si no dicen lo contrario, HSA (llammoslo diagnstico sindrmico). Este es el momento de
suelen referirse a la espontnea, y la causa ms frecuente de hemo- comenzar a tratarla. La sangre subaracnoidea no se puede extirpar
rragia subaracnoidea espontnea es la rotura de un aneurisma ce- quirrgicamente (no es un hematoma localizado, sino que se trata de
rebral. Por tanto la opcin 1 es cierta. sangre repartida difusamente por los espacios subaracnoideos cra-
El segundo concepto importante que hay que distinguir es que no neal y espinal. Por tanto, un concepto importante es que la hemorra-
es lo mismo hemorragia subaracnoidea que aneurisma cerebral; la gia subaracnoidea no se opera (otra cosa es que pueda ser necesario
rotura de un aneurisma es una de las causas de HSA, la ms fre- el tratamiento quirrgico de la causa de la HSA). El tratamiento de la
cuente, pero no la nica. HSA es mdico, y consiste en administrar al paciente frmacos que
Por tanto, cuando tenemos sospecha de que un paciente tiene una controlen la sintomatologa (analgsicos y antiemticos), junto con
HSA, hay que estudiarlo para conocer la causa de la misma, y hay que otros frmacos que ayudan a prevenir o evitar un nuevo sangrado
tratar por un lado la HSA y por otro lado la causa. (control de la tensin arterial, sedantes, laxantes, etc.). Adems se ad-

M exico A rgentina CTO Medicina C/ Nez de Balboa, 115 28006 MADRID (Espaa) Tfno.: (91) 782 43 32 / Fax: (91) 782 43 27
C hile U ruguay E-mail: secretaria@ctomedicina.com; iberocto@ctomedicina.com WEB: www.ctomedicina.com; www.iberocto.com
NC Pg. 13
NEUROCIRUGA
Seguimiento a distancia Preparacin Examen de Seleccin 05/06 1 Vuelta
ministra nimodipino, un antagonista del calcio que es til para la en colocar un clip metlico en el cuello del aneurisma para que no
prevencin de una de las complicaciones ms terribles de la HSA, el entre sangre en su interior), o el tratamiento endovascular (tambin
vasoespasmo. Generalmente el tratamiento de la HSA se realiza en llamado embolizacin, que consiste en realizar un cateterismo para
Unidades de Cuidados Intensivos para una mejor monitorizacin del rellenar el aneurisma con unas espirales de platino, de modo que la
paciente. Si el nivel de conciencia del paciente est muy afectado, sangre se trombose en su interior y deje de entrar sangre). Cualquiera
puede ser necesario el soporte respiratorio, pero no es una norma. de los dos es un tratamiento vlido, y segn los casos concretos, se
prefiere uno u otro.

Unin de la a. comunicante
anterior con la
Bifurcacin de la a.
a. cerebral anterior
cerebral media

Origen de la a.
Unin de la a. comunicante
cerebral media
posterior con la
a. cartida interna
Top de la a. basilar

Pregunta 28. Localizaciones ms frecuentes de los aneurismas cerebrales.


Pregunta 28. TAC craneal de hemorragia subaracnoidea (rotura de aneuris- Como ves, no es lo mismo el diagnstico de la HSA (TAC y pun-
ma de arteria cerebral media derecha). cin lumbar) que el de la causa de la HSA (arteriografa), y tampoco
es lo mismo el tratamiento de la HSA (mdico) que el de su causa
Hasta ahora te hemos explicado cules son los pasos que damos (ciruga o embolizacin). Esperamos que estos conceptos te hayan
para diagnosticar una HSA (confirmar que hay sangre en el espacio quedado claros, porque son muy importantes.
subaracnoideo), y una vez diagnosticada, cul es el tratamiento mdi-
co inicial que debe realizarse. Pregunta 29.- R: 5
Sin embargo, el manejo de estos pacientes no termina aqu. Es Nos preguntan ahora sobre un tipo muy especial de aneurismas
necesario llegar a un diagnstico etiolgico (cul es la causa de la cerebrales, los aneurismas micticos.
HSA?). Para ello deben realizarse otras pruebas diagnsticas. Dado Como sabes, los aneurismas cerebrales son dilataciones anormales
que hemos visto que la causa ms frecuente de HSA espontnea es la de la pared de una arteria cerebral, y generalmente son aneurismas
rotura de un aneurisma intracraneal (opcin 1), la primera tcnica saculares de origen congnito. La rotura de un aneurisma cerebral es
que realizaremos para buscar la causa ser una arteriografa cerebral, la causa ms frecuente de HSA espontnea.
con la que pretendemos ver el aneurisma que se ha roto. Sin embargo No todos los aneurismas cerebrales son de origen congnito. Algu-
es importante recordar aqu que en no pocas ocasiones los aneuris- nos de ellos son adquiridos por diversas causas. Los aneurismas
mas cerebrales son mltiples (hasta un 20% de los casos), ms frecuen- micticos son un ejemplo de esta situacin.
temente que lo que dice la opcin 2, que por tanto es la incorrecta Los aneurismas micticos se deben a la llegada de mbolos spti-

Comentarios TEST
en esta pregunta. Esto tiene una implicacin muy importante: el cos a las arterias cerebrales, concretamente mbolos procedentes del
hecho de encontrar un aneurisma en un vaso cerebral no excluye la corazn en un paciente con endocarditis bacteriana. Por tanto, la
presencia de otros y, por tanto, la angiografa cerebral debe incluir respuesta correcta en esta pregunta es la opcin 5.
los 4 vasos (las dos arterias cartidas internas y las dos vertebrales). Contrariamente a lo que su nombre sugiere en un principio, no
Slo de esta manera estaremos seguros del nmero de aneurismas tienen nada que ver con los hongos; son de etiologa bacteriana.
que tiene el paciente. Como habrs estudiado en Neurologa, el lugar ms frecuente de
Es importante saber que la mayor parte de los aneurismas cerebrales, embolias cerebrales es el territorio de la arteria cerebral media, por-
a diferencia de los de la aorta abdominal, son aneurismas saculares. que es la arteria cerebral que recibe un mayor flujo. Por el mismo
Los aneurismas fusiformes son muy raros a nivel cerebral, pero cuando motivo, la localizacin ms frecuente de los aneurismas micticos es
aparecen lo hacen ms frecuentemente a nivel de la circulacin poste- la arteria cerebral media.
rior (vertebrobasilar). Por tanto, la opcin 3 es cierta. Adems, contrariamente a los aneurismas saculares, que suelen
Recuerda que la localizacin ms frecuente de los aneurismas ce- estar en los vasos del polgono de Willis, los aneurismas micticos
rebrales es a nivel del polgono de Willis, y concretamente en la unin suelen tener una localizacin ms distal (su localizacin tpica es la
de la arteria comunicante anterior con la arteria cerebral anterior (ver bifurcacin de la arteria cerebral media).
figura).
Si se confirma la existencia de un aneurisma cerebral responsable Pregunta 30.- R: 1
de la HSA, habr que tratarlo para que no se produzca un resangrado Esta pregunta hace referencia a signos con valor localizador en la
(tratamiento etiolgico). Ya hemos visto que la HSA se trata mdica- hemorragia subaracnoidea (HSA).
mente, pero el tratamiento de su causa ms frecuente, la rotura de un Se trata de una mujer con HSA que, en la exploracin, presenta
aneurisma, es quirrgico. Hay dos tcnicas disponibles para tratar los ptosis palpebral derecha y midriasis del mismo lado. Se trata por tanto
aneurismas cerebrales, y ambas tienen como objetivo la exclusin del de una parlisis del III par craneal derecho, que como sabes es el par
aneurisma de la circulacin general (dicho con otras palabras, que craneal por el que llegan las fibras parasimpticas hasta la pupila, de
deje de entrarle sangre para que no vuelva a romperse). Puede tratarse modo que, si se lesiona, produce una midriasis por aumento del tono
un aneurisma cerebral con ciruga (clipaje quirrgico, que consiste simptico. La ptosis palpebral se debe a hipofuncin del msculo

M exico A rgentina CTO Medicina C/ Nez de Balboa, 115 28006 MADRID (Espaa) Tfno.: (91) 782 43 32 / Fax: (91) 782 43 27
Pg. 14 NC C hile U ruguay E-mail: secretaria@ctomedicina.com; iberocto@ctomedicina.com WEB: www.ctomedicina.com; www.iberocto.com
NEUROCIRUGA
Preparacin Examen de Seleccin 05/06 1 Vuelta Seguimiento a distancia
elevador del prpado superior, tambin inervado por el III par cra- ducirse una hidrocefalia comunicante debido a que el espacio suba-
neal. racnoideo est lleno de sangre y el LCR no puede circular libremente
Como ya te hemos explicado, la rotura de un aneurisma cerebral por l.
es la causa ms frecuente de HSA. Sin embargo, no todos los aneuris- Cuando la hidrocefalia aparece inmediatamente tras la HSA pro-
mas cerebrales se rompen; algunos pueden aumentar progresivamen- ducir un cuadro clnico de hipertensin intracraneal (HTIC), con
te de tamao sin romperse, dando lugar a sntomas compresivos que cefalea, vmitos y deterioro rpido del nivel de conciencia. Si un
dependen de la localizacin del aneurisma. paciente con HSA se deteriora de nivel de conciencia y se confirma la
Por tanto, debes saber que los aneurismas cerebrales pueden origi- dilatacin del sistema ventricular en la TAC, el tratamiento de eleccin
nar sntomas de dos clases: derivados de su rotura (hemorragia es la colocacin de un drenaje ventricular externo (se trata de una
subaracnoidea) o por compresin de estructuras vecinas. En algunos medida temporal, pues cuando se reabsorba la sangre subaracnoidea
casos, como el descrito en la pregunta, pueden producir los dos tipos puede restablecerse la circulacin normal del LCR).
de sntoma a la vez. En otros, la aparicin de estos ltimos se interpreta Sin embargo, en ocasiones el paciente se recupera de la hemorra-
como sntomas centinela que anteceden a la rotura del aneuris- gia, se va de alta, y con el paso del tiempo comienza a presentar
ma. sntomas de hidrocefalia crnica (alteraciones cognitivas, trastorno de
Cuando un aneurisma produce sntomas compresivos, da una la marcha, incontinencia, etc.). Estos sntomas se deben al desarrollo
oportunidad al neurocirujano de sospechar la localizacin del mis- de una hidrocefalia comunicante y, como en estos casos no es previ-
mo sin necesidad de angiografa, aunque evidentemente siempre es sible la reversibilidad de los sntomas con el paso del tiempo, el trata-
necesaria la confirmacin radiolgica. miento de eleccin es la derivacin permanente del LCR (una vlvu-
De estos signos clnicos con valor localizador, el ms importante, la).
y el que debes recordar, es el que sealan en esta pregunta. Cuando
un paciente con HSA tiene una parlisis del III par sugiere fuertemen- RESANGRADO.
te la presencia de un aneurisma de la arteria comunicante posterior, El resangrado es la principal causa de mortalidad en los primeros
pues existe una importante relacin anatmica entre esta arteria y el III das. Evidentemente, cualquier aneurisma que se ha roto una vez
par craneal. puede volver a hacerlo hasta que sea tratado y excluido de la circula-
Evidentemente ese no es el nico signo localizador, pero es el ms cin general. Esta es la razn por la que es importante tratar el aneuris-
caracterstico. Los aneurismas de arteria comunicante anterior, cuan- ma lo antes posible. Si te ponen un caso clnico de resangrado, lo
do alcanzan un tamao suficiente, pueden producir defectos campi- identificars por un rpido deterioro de la situacin clnica del pa-
mtricos por compresin del quiasma. ciente y sntomas sugestivos de una nueva HSA (los mismos que se
No todos los aneurismas producen estos signos localizadores. En produjeron la primera vez). Se trata de un evento muy grave, y la nica
muchas ocasiones no existe esta clnica. Podemos recurrir entonces a forma de tratarlo (si no se trata el aneurisma) es prevenirlo, utilizando
signos radiolgicos con valor localizador, en concreto a la distribu- medidas mdicas que reduzcan los factores que lo pueden provocar
cin de la sangre subaracnoidea. Dicho de otra forma, podemos aven- (control de la tensin arterial, evitar maniobras de Valsalva mediante
turar la localizacin ms probable del aneurisma en funcin de en laxantes, sedantes, etc.).
qu lugar del cerebro encontramos ms sangre subaracnoidea. De
entre ellos, merece la pena destacar la presencia de un hematoma VASOESPASMO.
interhemisfrico, que sugiere la presencia de una aneurisma de la El vasoespasmo es la principal causa de morbimortalidad en los
arteria comunicante anterior o la arteria pericallosa, o la presencia de pacientes que sobreviven a los primeros das de la HSA.
un hematoma intrasilviano (en la cisura de Silvio), que sugiere un No es otra cosa que una contraccin de la musculatura de una
aneurisma de la arteria cerebral media. arteria como consecuencia de la irritacin que produce la sangre a su
De todas formas, no debes olvidar que, aunque por la clnica o alrededor. El resultado es una reduccin del flujo a travs de esa arteria,
por la radiologa podamos sospechar la presencia de un aneurisma y la aparicin de sntomas isqumicos en el territorio dependiente de
en alguna localizacin concreta, siempre es necesaria la confirma- esa arteria. Por tanto, lo identificars en los casos clnicos ante la apari-
cin radiolgica mediante arteriografa, sin olvidar que hasta un cin de nuevos dficits focales unos das despus del sangrado. Es im-
20% de los aneurismas cerebrales son mltiples, y por tanto es impres- portante este ltimo dato; es necesario el paso de ciertos das para la
cindible estudiar los 4 vasos cerebrales (las dos arterias cartidas inter- aparicin de los sntomas, ya que la arteria debe ser irritada, cerrarse y
nas y las dos vertebrales) para identificar otras posibles anomalas la reduccin del flujo producir la isquemia, y esto lleva su tiempo. Por
Comentarios TEST

vasculares asociadas. ello no suele verse antes del 4 da postsangrado.


En resumen, se identifica por la aparicin de nueva focalidad
Pregunta 31.- R: 3 neurolgica en el periodo de riesgo (entre el 4 y 12 da).
Esta pregunta hace referencia a las complicaciones de la hemorra- En el caso que nos ocupa hay que sospechar vaoespasmo porque,
gia subaracnoidea (HSA). Antes de contestar la pregunta, permtenos 7 das despus del sangrado, el paciente ha desarrollado una nueva
explicarte brevemente cules son esas complicaciones y la forma prc- focalidad, la disfasia, que sugiere vasoespasmo de la arteria cerebral
tica de enfocarlas de cara al examen MIR. media izquierda. Como consecuencia de la isquemia de un territorio
En primer lugar te recordaremos que la HSA es una patologa muy tan amplio como el de esta arteria, la disfasia se acompaa de un
grave, con un ndice de mortalidad y complicaciones muy elevado. deterioro del nivel de conciencia del paciente.
Sirva como dato que hasta un 10% de los pacientes que la presentan El tratamiento de eleccin del vasoespasmo consiste en aumentar el
fallecen antes de llegar al hospital, y que la mortalidad dentro del flujo por la arteria espstica. Para ello utilizamos la terapia triple H:
primer mes es prxima al 50%. aumentar la tensin arterial para vencer la resistencia que ofrece el vaso
La HSA puede tener tres complicaciones neurolgicas principal- de menor calibre (hipertensin), diluir la sangre para facilitar su paso
mente: la hidrocefalia, el resangrado y el vasoespasmo. Debes apren- por ese vaso (hemodilucin) y aumentar la volemia para aumentar la
der a identificarlas y conocer su manejo. cantidad de sangre que pasa por esos vasos (hipervolemia). Recuerda
que la profilaxis del vasoespasmo se realiza desde el primer da del
HIDROCEFALIA. ingreso tras el sangrado con antagonistas del calcio (nimodipino).
Como sabes la hidrocefalia es un disbalance entre la produccin y De entre las opciones indicadas, slo una recoge la filosofa del
la absorcin de LCR, y hay dos tipos de hidrocefalia (comunicante y tratamiento del vasoespasmo que te hemos explicado.
no comunicante, en funcin de que el LCR sea capaz de salir del La opcin 1 no es vlida, porque si restringimos el aporte de lqui-
sistema ventricular al espacio subaracnoideo o no). En la HSA pueden dos, reduciremos la volemia. La opcin 2 es incorrecta, porque al
darse los dos tipos de hidrocefalia, y pueden aparecer de forma pre- reducir la tensin arterial se reducir el flujo sanguneo cerebral. No
coz (inmediatamente tras el sangrado) o tarda. vale tampoco la opcin 4, porque clipar el aneurisma sera el trata-
Puede haber hidrocefalia obstructiva o no comunicante si hay miento de la causa de la HSA, y evitara el resangrado, pero no afecta
hemorragia intraventricular asociada a la HSA. Tambin puede pro- en nada al vasoespasmo una vez producido. Tampoco vale la opcin

M exico A rgentina CTO Medicina C/ Nez de Balboa, 115 28006 MADRID (Espaa) Tfno.: (91) 782 43 32 / Fax: (91) 782 43 27
C hile U ruguay E-mail: secretaria@ctomedicina.com; iberocto@ctomedicina.com WEB: www.ctomedicina.com; www.iberocto.com
NC Pg. 15
NEUROCIRUGA
Seguimiento a distancia Preparacin Examen de Seleccin 05/06 1 Vuelta
5, porque aunque la primera parte del enunciado es correcta (au- tualmente preguntan en el MIR, la raz que se afecta es la que lleva el
mentar la dosis de nimodipino), la segunda no lo es, ya que no pode- nombre de la vrtebra de abajo).
mos reducir la tensin arterial.
Por todo ello, y aunque la opcin 3 es incompleta, es la nica que
se ajusta a los principios del tratamiento del vasoespasmo (el salino Pregunta 33. Semiologa de la lesin de las races del plexo lumbosacro.
hipertnico aumentar la volemia y elevar la tensin arterial).
12314 14315 15367
Pregunta 32.- R: 2 89

Esta pregunta no requiere mucha explicacin. Hay algunas patolo- 9 9 12 13 45
gas sistmicas que se asocian a una mayor incidencia de aneurismas 999
cerebrales. En muchos casos se trata de enfermedades del tejido
conectivo en las que existe una debilidad de la pared vascular, pero 8
hay otras patologas en las que tambin est aumentada la incidencia, 99
123456782 9
4 5 2
entre ellas la coartacin de aorta recogida en la opcin 2 de esta
67  9 7
pregunta.
6789
 7 7 77 7
 67 9
Algunas enfermedades en las que est aumentada la incidencia de
 77 77
 77
7 7
aneurismas cerebrales se recogen en la tabla.
77   6789
 7512 7 
 !
77 
Pregunta 32. Enfermedades con aumento de riesgo 67"7! 7
de aneurismas cerebrales. 67"
777   67#$
77

7
 67#$ 77 9
 
777  
Sndrome de Marfan.  77
  67& 7
7'
67 7 7
Sndrome de Ehlers-Danlos (defecto del colgeno tipo III). 67"7! 
77%
75127 
Pseudoxantoma elstico. 77 7 
Displasia fibromuscular.
Poliquistosis renal.
Anemia de clulas falciformes.
Coartacin de aorta. Todas las preguntas de hernias discales lumbares del MIR se han
Sndrome de Rendu-Osler-Weber. podido contestar localizando el reflejo alterado. Ya sabes que el rotu-
liano se afecta en las hernias discales L3-L4 (raz L4) y el aquleo en las
L5-S1 (raz S1). Recuerda que la raz L5 (hernias L4-L5) no tiene
ningn reflejo asociado. Slo queremos llamar tu atencin sobre
PATOLOGA RAQUIMEDULAR. una pregunta MIR que hizo referencia a una lesin de la raz L5 y
deca que el aquleo poda estar abolido, dando esta opcin como
Comenzamos ahora con uno de los bloques ms importantes de cierta. Aunque habitualmente la raz L5 no asocia ningn reflejo (y
esta asignatura en trminos de nmero de preguntas MIR, el bloque eso es lo que tienes que aprenderte), ocasionalmente puede asociarse
de la patologa raquimedular y, en concreto, el de las lumbociatalgias. con el aquleo (este dato est recogido en el Harrison, tratado de
Antes de nada nos permitimos una reflexin sobre la orientacin Medicina Interna, pero se trata de una excepcin y no debes tomarlo
que estas preguntas han tenido en los ltimos exmenes MIR. Hasta como un dato importante). En definitiva, lo que debes recordar es que
hace unos dos aos, todas las preguntas pedan averiguar en qu nivel la raz L5 no asocia ningn reflejo.
de la columna lumbar o cervical se localiza una hernia discal que
produce unos sntomas determinados. Es importantsimo que seas Pregunta 34.- R: 3
capaz de reconocer el nivel de lesin segn los sntomas que te expo- Esta pregunta es el equivalente a la pregunta anterior, pero a nivel
nen en el enunciado. de la columna cervical. Se trata de localizar la sintomatologa que no
Sin embargo, desde hace dos aos muchas preguntas hacen refe- aparecera en una hernia discal cervical C5-C6.
rencia a la actitud ms adecuada ante estas enfermedades. Estas pre- Como ya hemos mencionado, en el caso de las preguntas sobre
guntas son relativamente recientes en la historia del MIR, pero no por hernias discales lumbares, la clave est siempre en mirar el reflejo afec-

Comentarios TEST
ello menos importantes, y en las prximas preguntas te daremos pistas tado. No es as en las hernias discales cervicales, porque los reflejos
para poder contestarlas mejor. osteotendinosos de miembros superiores pueden ser compartidos por
varias races. En el caso de las hernias cervicales, no te recomendamos
Pregunta 33.- R: 3 mirar el reflejo afectado; sin embargo, todas las preguntas del MIR que
Como sta han sido las preguntas que clsicamente ponan en el hacan referencia a ellas se contestaban mirando el territorio sensitivo
examen; hay que averiguar el nivel de la hernia discal en base a los afectado, y ms concretamente, los dedos de la mano en los que el
sntomas que te presentan. paciente presentaba dolor, hipoestesia o parestesias.
Como consejo, te diremos que es necesario que conozcas los sn- Recuerda que las hernias C5-C6 (raz C6) afectan a la sensibilidad
tomas motores, sensitivos y los reflejos afectados con la lesin de cada de 1 y 2 dedos, las hernias C6-C7 (raz C7) al 3 dedo, y las hernias
raz nerviosa lumbar o cervical. Sin embargo, en la prctica, todas las C7-D1 (raz C8) a 4 y 5 dedos de la mano. Con esta informacin se
preguntas que han puesto en el examen, relativas a hernias discales contestaban prcticamente todas las preguntas MIR, aunque no fue-
lumbares, se contestaban sabiendo el reflejo que se altera en cada raz ras capaz de recordar las alteraciones motoras o el reflejo asociado
(incluso si no te acordaras de los sntomas motores o sensitivos, aun- con la lesin de las races cervicales.
que lgicamente conocer stos te ayudar mucho). Por tanto, en to- Lo que te hemos dicho es aplicable a la mayora de las preguntas del
das las preguntas de hernias discales lumbares hay que fijarse en el examen MIR, pero lamentablemente no a la pregunta que nos ocupa,
reflejo afectado. que nos obliga a saber la clnica motora, sensitiva y el reflejo afectado en
En este caso clnico, el paciente, de mediana edad, tiene citica las hernias discales C5-C6 (por tanto, asociados a la lesin de la raz C6).
desde hace dos meses, la exploracin de la sensibilidad y la fuerza en La opcin falsa es la 3, puesto que la debilidad para la extensin
miembros inferiores es normal (datos que por tanto no ayudan a del codo, y en general para todos los movimientos extensores del
localizar el nivel), pero tiene un reflejo patelar o rotuliano disminuido miembro superior, es consecuencia de la lesin de la raz C7 (por
en el lado de la citica. La alteracin del reflejo, como siempre, nos tanto aparece en las hernias C6-C7).
localiza el nivel de la lesin. El reflejo rotuliano se afecta en las lesiones Las opciones 1, 2 y 4 son ciertas porque la lesin de la raz C6
de la raz L4, y sta es la raz que se afecta en las hernias discales L3-L4 produce alteraciones sensitivas en 1 y 2 dedo, alteracin de los
(recuerda que en las hernias discales posterolaterales, las que habi- reflejos bicipital y estilorradial, y debilidad para la flexin del codo.

M exico A rgentina CTO Medicina C/ Nez de Balboa, 115 28006 MADRID (Espaa) Tfno.: (91) 782 43 32 / Fax: (91) 782 43 27
Pg. 16 NC C hile U ruguay E-mail: secretaria@ctomedicina.com; iberocto@ctomedicina.com WEB: www.ctomedicina.com; www.iberocto.com
NEUROCIRUGA
Preparacin Examen de Seleccin 05/06 1 Vuelta Seguimiento a distancia
Respecto a la opcin 5, la maniobra de Spurling es el equivalente
en la exploracin de las hernias cervicales a la maniobra de Lasgue Pregunta 35. Diagnstico diferencial entre claudicacin
en las lumbares. Consiste en presionar en el vrtex craneal con la neurognica y claudicacin vascular.
cabeza extendida y girada hacia el lado de la cervicobraquialgia; si se
reproduce el dolor, esta maniobra es positiva.
Claudicacin Claudicacin
neurognica vascular
Pregunta 34. Semiologa de la lesin de las races del plexo braquial.
D istribucin del Territorio de un nervio Grupo muscular con
dolor (dermatoma) irrigacin comn
C 4-C 5 C 5-C 6 C 6-C 7 C 7-D1
Ejercicio de
Raiz intensidades
habitualmente C5 C6 C7 C8 Ejercicio con
variables.
afectada intensidad constante,
Mantenimiento
Factores menor conforme
Reflejo B icipital prolongado de una
B icipital Tricipital Tricipital desencadenantes progresa la enfermedad.
alterado Estilorradial postura.
Raro en pie sin
Al ponerse en pie,
- Separacin - Extensin - Flexin dedos. caminar.
- Flexin codo. antes de comenzar la
hombro. codo. - Musculatura
Dficit motor - Extensin marcha.
- Flexin - Flexin intrnseca de
mueca.
codo. mueca. la mano. D istancia al
- Hombro. caminar para Variable Constante
- Cara lateral - 4 y 5 dedos. aparicin
Dficit - Cara
del antebrazo. - 3 dedo. - Cara medial
sensitivo lateral del
- 1 y 2 dedos. del antebrazo. Lento.
brazo. Inmediato.
Alivio con el Dependiente de la
No depende de la
reposo postura (mejor en
postura.
flexin de la columna).
Pregunta 35.- R: 2
El diagnstico de sospecha que debes establecer en el paciente Pulsos
Conservados Disminuido o ausentes
descrito es una raquiestenosis lumbar. perifricos
La estenosis del canal vertebral lumbar produce clnica dolorosa
Palidez cutnea
en miembros inferiores, que generalmente se desencadena al cami- al elevar los No Marcada
nar (claudicacin neurognica), cediendo con el reposo. A diferencia MMII
de la hernia discal lumbar, en la que el dolor de espalda es mayor con
la flexin de la columna, en el caso de la estenosis del canal, el dolor Temperatura en
Normal Disminuida
aumenta con la extensin del tronco. Se dice clsicamente que los los MMII
pacientes con raquiestenosis lumbar pueden montar en bicicleta,
porque les duele menos la espalda al estar flexionada.
Es importante realizar diagnstico diferencial entre claudicacin En este sentido, han aparecido preguntas en los ltimos exmenes
neurognica y claudicacin vascular, que se produce por insuficien- MIR que hacen referencia a la actitud ms adecuada ante un paciente
cia del sistema arterial de los miembros inferiores (ver tabla). con lumbocitica o cervicobraquialgia de reciente aparicin, y la
Por tanto, estamos ante un paciente con posible raquiestenosis respuesta correcta en todos los casos ha sido que debe tratarse mdi-
lumbar, y nos preguntan el tratamiento de eleccin. La respuesta camente, sin necesidad de realizar pruebas diagnsticas inicialmente,
correcta es la opcin 2; el tratamiento de eleccin de la estenosis de y solamente en caso de que pasado un mes no haya mejorado, o
canal lumbar es la ampliacin del dimetro del canal medular, que se cuando presenta factores de riesgo, deben realizarse estas pruebas
consigue al extirpar los elementos posteriores de la vrtebra (el arco para descartar una causa que requiera tratamiento quirrgico.
posterior, o lo que es lo mismo las lminas). La extirpacin de la
apfisis espinosa y las lminas de la vrtebra se conoce como lami- Pregunta 35. Indicaciones de ciruga de la hernia discal.
Comentarios TEST

nectoma.
La discectoma, recogida en la opcin 1, consiste en la extirpacin Lesin de la raz que produce una prdida aguda o progresiva de
del disco intervertebral, y e s la tcnica quirrgica de eleccin para el fuerza objetivable clnicamente o por EMG. Es indicacin de ciru-
tratamiento de las hernias discales. A nivel lumbar se realiza general- ga urgente.
mente por va posterior, pero a nivel cervical se hace por va anterior Signos clnicos sugestivos de sndrome de cola de caballo o lesin
(ya que es ms difcil llegar al disco desde atrs, por estar en el trayecto medular (disfuncin de esfnteres, anestesia perineal en silla de
la mdula cervical). Generalmente, para tratar las hernias lumbares se montar, etc.). Es indicacin de ciruga urgente.
realiza una hemilaminectoma y discectoma, mientras que para las Fracaso del tratamiento conservador, es decir, dolor incapacitante
hernias cervicales se suele hacer una discectoma y artrodesis de caractersticas radiculares que no responde a tratamiento mdi-
intersomtica por va anterior. co durante un perodo mnimo de 4 semanas.
Se denomina flavectoma (opcin 3) a la extirpacin de los liga- Incapacidad recidivante a pesar del tratamiento mdico.
mentos amarillos, corpectoma (opcin 4) a la extirpacin de un
cuerpo vertebral y sustitucin del mismo por hueso o injertos metli-
cos. Finalmente artrodesis es una fusin entre elementos de dos vr- Pregunta 36.- R: 2
tebras que se consigue por diversos medios (interponiendo hueso, Esta pregunta hace referencia a las lesiones medulares traumticas.
tornillos y barras, injertos metlicos, alambres, etc.). Desde el punto de vista clnico, la semiologa de las lesiones medu-
Recuerda que el tratamiento inicial de eleccin de la hernia discal, lares traumticas no difiere de la que se puede encontrar en otros
tanto cervical como lumbar, no es la ciruga, sino el tratamiento mdi- tipos de afectacin medular (tumoral, infecciosa, esclerosis mltiple,
co con analgsicos, antiinflamatorios y relajantes musculares. Sola- etc.). El estudio de la clnica de estas lesiones se recoge en el apartado
mente deben ser intervenidos los pacientes en los que fracasa el trata- de Semiologa del Manual CTO, que debes repasar, y puede simplifi-
miento conservador, o en los que aparecen signos de lesin radicular carse mucho si recordamos unas breves nociones de la anatoma
con prdida de fuerza objetivable clnicamente o con EMG, o signos medular y su implicacin clnica:
de compresin de la cola de caballo o mielopata (ver tabla a conti- Como sabes, en la mdula hay sustancia gris (donde estn los
nuacin). cuerpos de las neuronas) y sustancia blanca (por donde discurren

M exico A rgentina CTO Medicina C/ Nez de Balboa, 115 28006 MADRID (Espaa) Tfno.: (91) 782 43 32 / Fax: (91) 782 43 27
C hile U ruguay E-mail: secretaria@ctomedicina.com; iberocto@ctomedicina.com WEB: www.ctomedicina.com; www.iberocto.com
NC Pg. 17
NEUROCIRUGA
Seguimiento a distancia Preparacin Examen de Seleccin 05/06 1 Vuelta
las fibras nerviosas de las vas ascendentes o sensitivas y descenden- Una vez recordados estos conceptos, podemos contestar la pregunta.
tes o motoras). Cualquier lesin medular (tambin las traumticas) La opcin 3 se refiere al sndrome de Brown-Squard, tambin
va a afectar a estas dos estructuras en el nivel de la lesin, pero conocido como sndrome de hemiseccin medular. De su nombre se
tambin va a producir una interrupcin de las vas que ascienden deduce que se lesiona la mitad de la mdula (los cordones posterior,
y descienden, y por lo tanto va a interrumpir la inervacin motora lateral y anterior de un lado), y por tanto las vas lemniscal,
y sensitiva por debajo de la lesin. corticoespinal y espinotalmica respectivamente de un mismo lado,
Por tanto, para simplificar el estudio de la clnica de las lesiones por lo que se producir prdida de la sensibilidad propioceptiva
medulares debes recordar que habr sntomas sensitivos y motores ipsilateral, parlisis ipsilateral y prdida de la sensibilidad termoalgsi-
(de 2 motoneurona) en el territorio correspondiente al nivel de la ca contralateral por debajo de la lesin. Por tanto, esta respuesta es
lesin, y clnica sensitiva y motora (pero de 1 motoneurona) por correcta.
debajo de la lesin. La opcin 4 hace referencia al sndrome medular central. Este es
En el nivel de la lesin encontrars por tanto una anestesia de ese un sndrome muy caracterstico en el que se interrumpen las vas que
territorio y una parlisis flccida con arreflexia e hipotona. Ejem- cruzan por el centro de la mdula, y por tanto las fibras sensitivas
plo: una lesin medular C7 producir una anestesia en dermato- termoalgsicas, por lo que suele producir una alteracin suspendida
ma C7 y parlisis flccida de la musculatura extensora de miem- de la sensibilidad para la temperatura y el dolor en los niveles afecta-
bros superiores. dos. Esta alteracin se conoce tambin como dficit disociado de la
Si embargo, por debajo de la lesin tambin encontraremos una sensibilidad (se altera la del dolor y la temperatura, pero se conserva
hipoestesia, pero el trastorno motor es de 1 motoneurona, y por la propioceptiva), y es tpica de la siringomielia (una enfermedad en la
tanto se objetivar parlisis espstica con hiperreflexia, hipertona, que se crea una cavidad rellena de lquido en el centro de la mdula
clonus y respuesta plantar extensora (signo de Babinski). Ejemplo: que interrumpe las fibras descritas). Este es el rasgo clnico ms tpico
la lesin medular C7 que hemos descrito ms arriba produce par- del sndrome centromedular, pero con l no se contesta la pregunta
lisis flccida de la musculatura extensora de miembros superiores, porque han preguntado algo ms rebuscado. Cuando la lesin
pero tambin habr parlisis de miembros inferiores, aunque en centromedular es grande, puede afectar tambin a otras vas largas de
este caso espstica con la exploracin que te hemos descrito. la mdula, en este caso a la corticoespinal, que lleva fibras motoras, y
Respecto a las vas largas, ascendentes y descendentes, debes re- puesto que en la va corticoespinal las fibras nerviosas correspondien-
cordar que se afecta todo el territorio localizado por debajo de la tes a los miembros superiores van colocadas ms medialmente o cen-
lesin. Para simplificar el estudio te recomendamos que recuerdes tralmente que las de los miembros inferiores, es ms probable que el
slo tres de estas vas, y es importante que sepas dnde se cruzan dficit motor afecte ms a brazos que a piernas. Por tanto, esta opcin
para entender si la clnica es ipsilateral o contralateral: tambin es cierta; en el sndrome centromedular, la paresia es mayor
- La va corticoespinal (descendente), por la que discurren las en miembros superiores que inferiores y hay una alteracin suspendi-
primeras motoneuronas, se cruza a nivel del bulbo, y por tanto da y disociada de la sensibilidad.
ya viene cruzada en la mdula, as que los sntomas sern del La clnica de otras lesiones medulares se deduce conociendo la
mismo lado de la lesin. Se localiza en los cordones laterales de localizacin de la lesin (ver tabla a continuacin).
la mdula. Su afectacin produce parlisis espstica ipsilateral Las otras tres opciones se refieren al tratamiento de las lesiones
por debajo de la lesin, como ya hemos descrito. medulares traumticas.
- La va espinotalmica (ascendente) lleva las fibras correspon- La opcin 5 es cierta; si existe una listesis cervical, es decir, un
dientes a la sensibilidad termoalgsica (dolor y temperatura), y desplazamiento de una vrtebra cervical sobre otra, con el riesgo con-
se cruza a nivel medular (por el centro de la mdula), por lo que siguiente de lesin medular, la primera medida teraputica debe ser
su lesin produce sntomas del lado contrario al de la lesin. Se conseguir una correcta alineacin de la columna para reducir el dao
localiza en los cordones anteriores de la mdula. Su lesin pro- medular, y por tanto el tratamiento inicial es la traccin cervical. Des-
ducir prdida de la sensibilidad a la temperatura y al dolor pus, en funcin del tipo de lesin se recurrir a la ciruga para fijar
contralateral por debajo de la lesin. esa columna. El tratamiento de la posible lesin medular es el mismo
- La va lemniscal (ascendente) discurre por los cordones poste- que en otros tipos de lesin traumtica.
riores de la mdula y lleva fibras correspondientes a la sensibi- La opcin 1 tambin es cierta; en todo paciente con lesin medu-
lidad propioceptiva (posicin y vibracin), y se cruza a nivel del lar traumtica debe iniciarse tratamiento con dosis muy altas de
tronco, por lo que su lesin produce sntomas del mismo lado metilprednisolona en las primeras 8 horas del traumatismo, que se
de la lesin. La clnica, cuando se afecta, es prdida de la sensi- mantienen durante 8 horas. El motivo es que ciertos estudios (NASCIS)

Comentarios TEST
bilidad propioceptiva ipsilateral por debajo de la lesin, lo que demostraron un mejor pronstico funcional de los pacientes tratados
implica la aparicin de una ataxia sensitiva. de este modo comparados con los que no reciban corticoides.

Cordn posterior.
Alt. sensibilidad propioceptiva y ataxia sensitiva ipsilaterales

Va piramidal o corticoespinal.
Lesin de 1 motoneurona (parlisis espstica) ipsilateral

Asta anterior.
Lesin de 2 motoneurona (parlisis flccida) ipsilateral

Va espinotalmica.
Alt. sensibilidad termoalgsica contralateral

Pregunta 36. Vas ms importantes de la mdula espinal y clnica asociada a su lesin.

M exico A rgentina CTO Medicina C/ Nez de Balboa, 115 28006 MADRID (Espaa) Tfno.: (91) 782 43 32 / Fax: (91) 782 43 27
Pg. 18 NC C hile U ruguay E-mail: secretaria@ctomedicina.com; iberocto@ctomedicina.com WEB: www.ctomedicina.com; www.iberocto.com
NEUROCIRUGA
Preparacin Examen de Seleccin 05/06 1 Vuelta Seguimiento a distancia
La respuesta falsa es la 2, porque la persistencia de un dficit Pregunta 38.- R: 5
medular completo ms all de las primeras 24 horas indica que no se Atencin a esta pregunta, porque en los ltimos aos estn pregun-
va a producir recuperacin neurolgica, y por tanto el tratamiento tando sobre los factores de riesgo que pueden indicar una etiologa
descompresivo no ser til. Recuerda que se denomina lesin grave del dolor lumbar. Como ya te hemos explicado, hasta hace
completa a aquella en la que no existe ninguna funcin neurolgica unos dos aos, todas las preguntas sobre lumbociticas hacan refe-
conservada ms de tres segmentos por debajo del nivel de la lesin, y rencia al nivel en el que se encontraba la posible hernia discal en base
que es un signo de mal pronstico, pues menos de un 3% de los a los datos clnicos y a la exploracin del paciente. Sin embargo, en
pacientes con lesiones completas en la primera exploracin presenta- los ltimos exmenes han preguntado sobre la actitud diagnstica y
rn alguna mejora en las primeras 24 horas. teraputica con estos pacientes. Es sencillo contestar estas cuestiones,
pero hay que tener algunos conceptos claros:
El manejo inicial de todo paciente con lumbalgia o lumbocitica de
Pregunta 36. Sndromes medulares. reciente comienzo sin factores de riesgo que sugieran una etiologa
grave del dolor debe ser conservador. Lo que han querido que
contestemos en las preguntas MIR que han aparecido sobre este tema
Mielopata es que a estos pacientes hay que recomendarles tratamiento mdico
transversa con analgsicos, antiinflamatorios no esteroideos y relajantes muscu-
lares, no hacerles ninguna prueba de imagen, y reevaluarles al cabo
de 15 das. Es muy importante que recuerdes esto.
Cundo solicitaremos pruebas diagnsticas?. Como te hemos
Sd. Brown-Sequard
explicado, ante una lumbocitica o lumbalgia de reciente co-
(hemiseccin medular)
mienzo sin factores de riesgo se iniciar tratamiento mdico y se
reevaluar al paciente a los 15 das. Si es necesario, se corregir el
tratamiento en ese momento. Slo si persiste la clnica con un
tratamiento correcto al cabo de un mes, y por supuesto, si hay
Sd. medular central factores de riesgo de etiologa grave del dolor, se solicitarn estu-
dios de imagen u otras pruebas diagnsticas (analticas,
electrofisiologa, etc).
Cules son los factores de riesgo que sugieren una etiologa
Sd. medular posterolateral grave del dolor lumbar o la citica?. Si en la historia clnica o en la
exploracin se recoge algn dato que sugiera que la etiologa del
dolor puede ser infecciosa o tumoral, cuando existe el anteceden-
te de un traumatismo, o en los casos en que la exploracin
neurolgica muestra un dficit neurolgico, es necesario realizar
Sd. cordonal posterior estudios diagnsticos desde un primer momento. La siguiente tabla
recoge los factores de riesgo que debes recordar.

Pregunta 38. Factores de riesgo de etiologa grave del dolor lumbar.


Sd. medular anterior
Edad mayor de 50 aos.
Diagnstico previo de cncer.
Diagnstico previo de enfermedad sistmica grave.
Pregunta 37.- R: 3 Antecedentes de traumatismo espinal.
Esta pregunta hace referencia a lesiones de las primeras vrtebras Historia de ciruga reciente (espinal o no).
cervicales, que encontrars mejor explicadas en Traumatologa. No Infeccin crnica pulmonar, urinaria o cutnea.
obstante, recordaremos aqu las ms frecuentes. Consumo de drogas por va parenteral.
La fractura de Jefferson (opcin 1) es una fractura de los arcos Historia de inmunosupresin (trasplante, VIH, etc).
Comentarios TEST

posterior y anterior del atlas, generalmente inestable, que no suele Tratamiento prolongado con glucocorticoides.
acompaarse de dficit neurolgicos si no hay otras lesiones asocia- Duracin del dolor superior a 1 mes.
das debido a que el canal raqudeo es muy amplio a ese nivel. Ausencia de alivio con reposo en cama.
La fractura de Hangman o fractura del ahorcado, producida por Aparicin de incontinencia urinaria o fecal, o nicturia reciente.
hiperextensin del crneo sobre la columna cervical, es una fractura Hallazgos exploratorios:
bilateral de la pars interarticularis de los pedculos de C2, y por tanto - Fiebre inexplicada.
la respuesta 3 es falsa. Suele acompaarse de subluxacin anterior - Prdida de peso llamativa inexplicada.
de C2 sobre C3. Generalmente son fracturas estables y no requieren - Masa abdominal.
fijacin quirrgica, pero s inmovilizacin externa prolongada. - Alteraciones neurolgicas sugestivas de sndrome de cauda
La luxacin atloaxoidea (opcin 2) suele ser secundaria a traumatis- equina (prdida de fuerza en miembros inferiores, anestesia en
mos o aparece en relacin con enfermedades sistmicas. La asociacin silla de montar, incontinencia urinaria o fecal, etc...).
ms frecuente es con la artritis reumatoide como consecuencia del
pannus inflamatorio que se produce alrededor de la articulacin La respuesta falsa es la 5, porque los individuos jvenes no tienen
odontoidea. Sin embargo, puede aparecer en el contexto de la enfer- mayor riesgo. Sin embargo son ciertas todas las dems. La falta de
medad de Morquio por hipoplasia de la apfisis odontoides. General- alivio del dolor con el reposo en cama, o su persistencia durante ms
mente provoca dolor suboccipital. Si no produce sntomas y la luxa- de un mes, son criterios para profundizar en la etiologa del dolor, al
cin es pequea, puede tratarse con collarn cervical. Si da sntomas igual que la aparicin de defectos neurolgicos. Finalmente, la toma
importantes o hay gran desplazamiento requiere ciruga. crnica de glucocorticoides favorece la infeccin y la osteoporosis,
Finalmente haremos un breve comentario para recordar que la por lo que la aparicin de dolor lumbar en estos pacientes obliga a
ausencia de lesiones seas en un traumatismo cervical no excluye descartar estas causas.
que pueda existir un dao neurolgico asociado. Hay lesiones medu-
lares cervicales sin lesin sea asociada (en ingls se conocen con las Pregunta 39.- R: 5
siglas SCIWORA Spinal Cord Injuries Without Radiographic Vamos a aprovechar esta pregunta para recordarte algunos con-
Abnormality). En estos casos es obligada la realizacin de una RM ceptos bsicos sobre los tumores espinales.
cervical, por lo que es cierta la opcin 5.

M exico A rgentina CTO Medicina C/ Nez de Balboa, 115 28006 MADRID (Espaa) Tfno.: (91) 782 43 32 / Fax: (91) 782 43 27
C hile U ruguay E-mail: secretaria@ctomedicina.com; iberocto@ctomedicina.com WEB: www.ctomedicina.com; www.iberocto.com
NC Pg. 19
NEUROCIRUGA
Seguimiento a distancia Preparacin Examen de Seleccin 05/06 1 Vuelta
La clnica de estas lesiones va a depender del nivel en el que se
encuentren, y se deduce fcilmente si tienes en cuenta los conceptos Pregunta 40. Craneoestenosis y sutura implicada.
que te hemos explicado sobre las lesiones medulares.
Los tumores raquimedulares se clasifican en tres grandes grupos Escafocefalia Sutura sagital
(no estn incluidos los tumores primarios del hueso, que estudiars en Trigonocefalia Sutura metpica
Traumatologa): Plagiocefalia Sutura coronal unilateral
Tumores extradurales o epidurales. En este grupo se recogen to- Braquicefalia Sutura coronal bilateral
dos los tumores que se localizan en el canal raqudeo por fuera de Paquicefalia (plagiocefalia posterior) Sutura lambdoidea
la duramadre. De todos ellos, los ms frecuentes son las metstasis, Turricefalia (oxicefalia) Todas las suturas
aunque la mdula puede comprimirse tambin por tumores verte-
brales que en su crecimiento alcancen el canal raqudeo. Pregunta 41.- R: 1
Las metstasis epidurales espinales pueden tener su origen en diver- El trmino malformacin de Chiari se refiere a un conjunto de entida-
sos tumores primarios, pero lo ms frecuente es que sean de origen des que probablemente se deben a distintos mecanismos patognicos.
hematolgico (linfomas), o tengan su origen en el pulmn, la mama Aunque se numeran del tipo I al tipo IV, no son distintos estadios de una
o la prstata (opciones 1, 2 y 3). Generalmente las metstasis verte- misma enfermedad, es decir, no se pasa de un tipo al otro. No obstante, se
brales son destructivas, pero pueden ser osteoblsticas (forman hue-
cumple que a mayor nmero en el nombre mayor es la gravedad, menor
so) las de mama y las de prstata, y debes recordar este dato. es la edad de aparicin de la sintomatologa y menos frecuentes son.
De entre todos los tumores citados, el que menos frecuentemente Es importante que recuerdes en qu consisten, la edad de apari-
produce metstasis raqudeas es el de colon. Por tanto, la respues- cin y si asocian algunas otras patologas neurolgicas (en concreto,
ta correcta es la 5. cul asocia hidrocefalia, siringomielia o mielomeningocele). Fjate en
Tumores intradurales extramedulares. Los tumores intradurales
estos datos cuando las estudies.
son menos frecuentes que los epidurales. Se subdividen a su vez La ms importante de todas es la malformacin de Chiari tipo I,
segn se localicen dentro o fuera de la mdula. porque es la ms frecuente. Consiste en un descenso de las amgdalas
Entre los tumores intradurales extramedulares hay que destacar cerebelosas por debajo del foramen magno, debido a un menor ta-
dos: los neurinomas (los tumores primarios espinales ms frecuen- mao de la fosa posterior (ver figura). Es la que aparece ms tarde
tes) y los meningiomas. Esto es lgico, porque entre la duramadre cronolgicamente, generalmente en la edad adulta, por lo que la
y la mdula se encuentran la aracnoides (lugar de origen de los respuesta falsa es la 1. Con frecuencia se asocia con siringomielia
meningiomas) y las races nerviosas (origen de los neurinomas).
(cavidades rellenas de lquido en el interior de la mdula espinal), y
Otros tipos histolgicos son mucho menos frecuentes a este nivel, esto ha sido objeto de pregunta MIR. El sntoma ms frecuente del
incluidas las metstasis. Chiari tipo I es la cefalea suboccipital (opcin 5). El tratamiento qui-
Tumores intradurales intramedulares. Debes recordar en este rrgico de esta entidad consiste en aumentar el tamao de la fosa
grupo solamente dos tipos histolgicos: los gliomas (tumores posterior, extirpando el hueso que la recubre y abriendo la durama-
derivados de las clulas gliales) y los ependimomas (derivados
dre que luego se cierra con un injerto de mayor tamao (craniectoma
de tejido ependimario medular). Entre los ependimomas mere- suboccipital y duraplastia).
ce ser recordado el ependimoma del filum terminale, porque
tiene una histologa muy caracterstica que solamente se ve en
los ependimomas de esta localizacin: son ependimomas
mixopapilares.

MALFORMACIONES DEL SNC.

Se trata de un tema poco relevante en el examen MIR, del cual


debes conocer slo conceptos bsicos que se recogen en las siguien-
tes preguntas.

Pregunta 40.- R: 3
Todas las malformaciones recogidas en las opciones de esta pre-

Comentarios TEST
gunta se engloban bajo el trmino comn de craneoestenosis o
craneosinostosis, y se producen como consecuencia del cierre pre-
coz de las suturas craneales recogidas en el segundo componente de
los binomios propuestos.
Respecto a las craneoestenosis, que no han sido preguntadas en el
examen MIR, hay que recordar la patogenia (cierre precoz de la sutu-
ra correspondiente) y, en general, saber que se trata de malformacio-
nes que tienen una importancia esencialmente cosmtica, porque en
general los nios no suelen presentar alteraciones de su capacidad
intelectual.
Por tanto, el tratamiento busca conseguir un resultado esttico ade-
Pregunta 41. Malformacin de Chiari tipo I.
cuado en la mayora de los casos. Para ello recurrimos a reconstruc-
ciones quirrgicas mediante craneotomas, remodelacin del rebor-
de orbitario, etc. Respecto al Chiari tipo II, lo que desciende por debajo del fora-
En la siguiente tabla se recogen las craneoestenosis ms frecuentes men magno no son slo las amgdalas cerebelosas (como en el tipo I).
y la sutura afectada. La respuesta falsa es la 3, porque la plagiocefalia En esta entidad, gran parte del contenido de la fosa posterior se hernia
se produce por el cierre precoz de la sutura coronal unilateral, no de hacia abajo (bulbo, protuberancia, IV ventrculo y vermis). En conse-
la sutura pterional. Aprovechamos para aclarar que, cuando se habla cuencia, cursa con clnica ms grave, y con frecuencia provoca la
de plagiocefalia sin decir nada ms, nos referimos a la plagiocefalia muerte por alteraciones respiratorias (disfuncin troncoenceflica).
anterior, porque existe otra forma de plagiocefalia posterior por cierre Debuta antes que el tipo I (en la infancia) y muy frecuentemente se
precoz de la sutura lambdoidea que tambin se denomina paquicefalia asocia con mielomeningocele e hidrocefalia (opcin 2).
y se recoge en la opcin 4. El cierre precoz de la sutura coronal El Chiari tipo III es incompatible con la vida y se diagnostica al
bilateralmente da lugar a la braquicefalia (opcin 5). nacimiento por la inclusin de todo el contenido de la fosa posterior
en un meningoencefalocele cervical alto (opcin 3).

M exico A rgentina CTO Medicina C/ Nez de Balboa, 115 28006 MADRID (Espaa) Tfno.: (91) 782 43 32 / Fax: (91) 782 43 27
Pg. 20 NC C hile U ruguay E-mail: secretaria@ctomedicina.com; iberocto@ctomedicina.com WEB: www.ctomedicina.com; www.iberocto.com
NEUROCIRUGA
Preparacin Examen de Seleccin 05/06 1 Vuelta Seguimiento a distancia
Finalmente, el Chiari tipo IV es una hipoplasia del cerebelo (op- Pregunta 43.- R: 4
cin 4), pero no hay herniacin de estructuras de fosa posterior por el El mielomeningocele es la forma ms frecuente de espina bfida
foramen magno. manifiesta, y consiste en un defecto de cierre del arco posterior verte-
bral a travs del cual se hernian las meninges y estructuras nerviosas
Pregunta 42.- R: 5 (races nerviosas de la cola de caballo y placa neural), por lo que
Como ya sabes, se denomina espina bfida a una malformacin asocia generalmente disfuncin de estas estructuras.
raqudea consistente en un defecto de cierre de los arcos vertebrales De todas las formas de espina bfida es la ms importante para el
posteriores, a travs del cual pueden quedar expuestos al exterior o examen MIR. Debes recordar algunas cosas sobre esta entidad, reco-
visibles algunas estructuras neurales (espina bfida manifiesta) o no gidas en las opciones de esta pregunta:
(espina bfida oculta). En algunos casos pueden asociarse anomalas Su localizacin ms frecuente es a nivel lumbar.
del tejido nervioso subyacente, pero no es obligado. Es muy frecuente su asociacin con hidrocefalia (opcin 1), y en
La respuesta correcta es la 5. La raquisquisis es un defecto de estos casos requerir tratamiento mediante derivacin ventriculo-
cierre completo de todo el tubo neural, generalmente incompatible peritoneal). Es frecuente tambin su aparicin en los pacientes con
con la vida, en el que todas las estructuras neurales raqudeas quedan Chiari tipo II.
expuestas al exterior. Puede acompaarse tambin de defecto de cie- Al igual que otros defectos de cierre del canal neural, se ha relacio-
rre del crneo, llamndose craneorraquisquisis, y en muchos casos nado con el dficit de cido flico en la madre (opcin 3), por lo
es motivo de aborto espontneo. que se recomienda la administracin de suplementos de esta vita-
El resto de las enfermedades mencionadas en las opciones de esta mina al menos dos meses antes de iniciar la gestacin para prevenir
pregunta son formas de espina bfida oculta. su aparicin. La profilaxis con cido flico ha sido objeto de pre-
El seno drmico congnito (opcin 1) es una de las formas de gunta MIR.
espina bfida oculta en la que hay un tracto fibroepitelial que pone en La opcin falsa es la 4, porque al estar expuestas las estructuras
comunicacin la piel con el tejido celular subcutneo, o en los casos neurales al exterior en el saco amnitico, se detectan niveles anor-
ms graves con el canal medular, debido a un defecto de cierre del malmente elevados de alfafetoprotena en el lquido amnitico y
arco posterior vertebral. Ninguna estructura neural queda expuesta o en el suero materno. Esta determinacin puede utilizarse para ha-
es visible desde el exterior, pero pueden verse un punto deprimido en cer diagnstico prenatal, aunque la ecografa es una tcnica
la piel (como un agujerito) siempre en la lnea media o muy cerca de diagnstica ms segura y especfica (opcin 5).
ella, y en ocasiones signos que se asocian con la espina bfida oculta El tratamiento de esta entidad es quirrgico (cierre del mielome-
(fundamentalmente mechones de pelo, angiomas o lipomas alrede- ningocele) y debe hacerse precozmente. Puesto que la localiza-
dor del punto deprimido). Generalmente no tienen trascendencia cin es a nivel lumbar, la posicin ptima para estos nios es en
clnica a no ser que produzcan un anclaje medular o se infecten, decbito prono (opcin 2).
siendo una causa potencial de meningitis.
La diastematomielia (opcin 2) es una malformacin de la mdula Pregunta 44.- R: 4
espinal que consiste en la presencia de dos hemimdulas, cada una Nos presentan un caso clnico de una paciente de 15 aos de
de ellas en el interior de un tubo dural independiente, separadas por edad, con cuadro clnico de fiebre y rigidez de nuca desde hace unas
un septo osteocartilaginoso. horas, que en ausencia de ms datos nos debe hacer pensar en una
Finalmente, recuerda la mayor frecuencia del sndrome de regre- meningitis infecciosa.
sin caudal (opcin 4), con agenesia del sacro, en los hijos de madres El resto de la exploracin parece normal, con excepcin del hallaz-
diabticas. go de una lesin angiomatosa y un mechn de pelo en la vecindad de
una pequea depresin en la piel de la lnea media lumbosacra.
El primer diagnstico en el que debemos pensar ante los hallazgos
Pregunta 42. Defectos de cierre del tubo neural. descritos es que se trate de una meningitis bacteriana en un paciente
con un seno drmico congnito.
12324356786 Como ya sabes, el seno drmico congnito es una malformacin
928324356786 243567 4373 englobada dentro del grupo de las espinas bfidas ocultas, y consiste
123456547589
76
623 6 8 88 17686484 2286 en un tracto fibroso recubierto de epitelio que pone en comunica-
 678 cin la piel con estructuras profundas (desde el tejido celular subcu-
Comentarios TEST

1234562789
86 82 88 52862
926 62 82
9287 9
tneo a, en los casos ms graves, el interior del canal medular como
consecuencia de un defecto de cierre del arco posterior del tubo
873 neural). La nica anomala que podemos ver desde el exterior es la
32  84 4 228  presencia de esa pequea depresin cutnea que se describe en el
2 334 2 62476  enunciado, que en ocasiones se acompaa de otros estigmas tpicos
8636 83786 de los pacientes con espina bfida oculta (mechones de pelo, angiomas
8 67 64  y lipomas como en el caso clnico que nos plantean). Sin embargo no
 686 67 existe ninguna estructura neural expuesta al exterior, por lo que esta
!3282 437364 malformacin se clasifica como una forma de espina bfida oculta.
826586 476 623 8 382 664  La mayor parte de las formas de espina bfida oculta no tienen
"83#23 32 84 $ trascendencia clnica, pero el seno drmico congnito tiene un riesgo
6 642 83 $3 823 67 potencial de infeccin que se traducira en la aparicin de cuadros
3 5
325 %&6 7 '( de meningitis, por lo que se recomienda la extirpacin quirrgica
  43 8 3  &3 6 664 4 3  profilctica del tracto. En caso de que produzca infeccin primero
2 33 3 38)2 debe tratarse sta, y despus se operar.
4667 6323863764 Recuerda que la meningitis de Mollaret (opcin 1) es una forma
%*8+ 36 386846( de meningitis asptica recidivante que se asocia con la rotura de tu-
!837 3282 4373%, mores dermoides o epidermoides y liberacin del contenido tumoral
5 34323( al espacio subaracnoideo. No es una meningitis infecciosa.
826586 !3282 4373%3+
6285836 Pregunta 45.- R: 4
-6 8 88 837 4373 Nos presentan un caso clnico de dolor facial en una paciente de
%3  2)84 ( 60 aos. La paciente describe el dolor como muy intenso y de carac-
tersticas lancinantes (como un calambre). El dolor aparece sbita-
mente y en ocasiones se desencadena al masticar o bostezar (con los

M exico A rgentina CTO Medicina C/ Nez de Balboa, 115 28006 MADRID (Espaa) Tfno.: (91) 782 43 32 / Fax: (91) 782 43 27
C hile U ruguay E-mail: secretaria@ctomedicina.com; iberocto@ctomedicina.com WEB: www.ctomedicina.com; www.iberocto.com
NC Pg. 21
NEUROCIRUGA
Seguimiento a distancia Preparacin Examen de Seleccin 05/06 1 Vuelta
movimientos de la boca). Aparece y desaparece sin previo aviso, los de menos de 3 cm de dimetro (opcin 2), en reas en las que la
episodios de dolor son de corta duracin, pero es muy recidivante. ciruga no es aconsejable (localizadas en zonas profundas del cerebro
Adems, la paciente informa sobre una distribucin del dolor muy o cerca de reas funcionales elocuentes del cerebro). Una limitacin
caracterstica en la regin perioral y la mejilla del lado izquierdo (pero de la radiociruga es que, una vez administrada, la MAV tarda en
no del lado derecho, slo afecta a una hemicara). obliterarse un periodo de tiempo de entre 1 y 3 aos, durante el cual
El enunciado de la pregunta est describiendo un caso tpico de persiste el riesgo de sangrado de la malformacin.
neuralgia del trigmino, afectando al territorio de la segunda (mejilla) Entre los tumores, se acepta actualmente que la radiociruga puede
y tercera ramas (regin perioral y mandibular) del nervio en el lado ser de utilidad en el tratamiento de determinadas metstasis,
izquierdo. Adems se describen muy bien las caractersticas del dolor: meningiomas (sobre todo del seno cavernoso) y neurinomas del VIII
lancinante, aparece como una corriente elctrica y con frecuencia al par en los que se descarte tratamiento quirrgico (opciones 1, 4 y 5).
hacer ciertas maniobras o tocar en determinado punto de la cara En general, no est indicada para el tratamiento de tumores infiltrantes
(reas gatillo). No se describe que exista alteracin de la sensibilidad como los gliomas.
en las ramas del trigmino afectadas, lo que tambin es tpico de esta En ningn caso se utiliza la radiociruga para el tratamiento de
enfermedad (la sensibilidad suele ser normal). aneurismas cerebrales (la respuesta falsa es la 3). Como ya sabes, el
No hay ninguna duda de que este paciente presenta una neuralgia tratamiento de eleccin de los aneurismas cerebrales es su exclusin
del trigmino, y el tratamiento de eleccin es la carbamacepina. La de la circulacin general mediante clipaje quirrgico o embolizacin
respuesta correcta es la 4. Slo en los casos en los que falla el trata- (tratamiento endovascular). La radiociruga no es una opcin en estos
miento mdico se recurre a tratamiento quirrgico por diversas tcni- pacientes en ningn caso.
cas que est recogidas en el Manual CTO.
En la mayor parte de los pacientes con neuralgia del trigmino no Pregunta 47.- R: 3
se encuentra una anomala estructural que cause el dolor. Se denomi- Esta pregunta hace referencia al tratamiento de los abscesos cere-
nan entonces neuralgias esenciales. brales, que como sabes son colecciones purulentas intracerebrales
Sin embargo, en un pequeo grupo pueden existir causas estructu- que la mayor parte de las veces tienen una etiologa bacteriana (re-
rales que motiven la compresin o la lesin del nervio; se habla enton- cuerda que lo ms frecuente es que se trate de infecciones polimicro-
ces de neuralgias secundarias. La ms frecuente de ellas es que exista bianas, con flora mixta en la que predominan los grmenes aerobios).
una compresin de la raz del nervio por un vaso sanguneo (general- Otro dato que debes recordar es que el sntoma ms frecuente de
mente la arteria cerebelosa superior o una arteria trigeminal persistente), presentacin es la cefalea, y que no siempre asocian fiebre.
pero hay otras causas (tumores, esclerosis mltiple, traumatismos, infartos Los abscesos cerebrales son colecciones de pus intracerebral. El
pontinos que afecten al ncleo del V par, etc.). tratamiento ideal es el que combina la antibioterapia y la ciruga (si
esta es posible, porque se trate de un absceso nico en una localiza-
cin apropiada).
Pregunta 45. Clasificacin etiolgica de la neuralgia del trigmino. Las opciones 3 y 5 se refieren a la utilizacin de antibiticos para el
tratamiento del absceso cerebral. Puesto que los abscesos cerebrales
12345678592
2 856 12345678967
7 925 
492996  649 suelen tener flora mixta con aerobios y anaerobios, la pauta antibitica
988 8 5 12928972 2297 debe incluir frmacos que cubran ambos tipos de grmenes. La pauta
9 5

 12724462 546472 52 45264 8459
 recogida en la respuesta 5 es la ms habitualmente utilizada, e incluye
1272 4 5492 2992 669
 cefotaxima (u otra cefalosporina de 3 generacin) y metronidazol.
Cuando se sospecha que puede haber estafilococos implicados (sobre
124778 4529
92 2
929 2
25 47 todo en los abscesos postquirrgicos) se asocia tambin vancomicina.
222!99282  56 " La opcin 3 es la falsa porque el diagnstico de absceso cerebral obliga
12#87 2! 4 874 $25 45789$28 545 4789" a mantener el tratamiento antibitico durante un mnimo de 4-6 sema-
123456785 129
78945296 47 579 nas, independientemente de que el paciente sea intervenido o no.

2 3 54859 12% 452 84
454 956 2! 
7428&
64
" Las opciones 1, 2 y 4 se refieren al papel de la ciruga.

8  8 5 12 796 9 Como sabes, la formacin del absceso se produce en varias etapas.
12#9896487 En la primera fase, conocida como cerebritis, se organiza la respuesta
12% 452
25&
72
2'292!
475  inflamatoria alrededor del foco necrtico. En una segunda fase se
2227567(
(9 " organiza una cpsula de colgeno y gliosis que limita el absceso. La

Comentarios TEST
evacuacin quirrgica es ms eficaz (tanto por puncin como la ciru-
Pregunta 46.- R: 3 ga abierta con reseccin del absceso y su cpsula) cuando el absceso
Esta pregunta hace referencia a una tcnica neuroquirrgica espec- est encapsulado, y no est indicada en fases de cerebritis. Lgica-
fica, que no ha sido preguntada antes en el examen MIR, pero que mente, si el absceso es nico y est bien delimitado, el xito de la
cuenta con un amplio campo actualmente, en expansin, y con unas ciruga es ms probable (opcin 2).
indicaciones que muy brevemente y de manera simplificada te comen- La respuesta 1 es cierta. Todos los pacientes con absceso cerebral
taremos. Es ms correcto denominarla radiociruga estereotctica. reciben tratamiento antibitico, y la mayora son operados. La ciruga
La radiociruga estereotctica, para explicarlo de manera muy sim- est indicada de eleccin en lesiones que producen clnica y signos
ple, es una tcnica que permite aplicar una dosis muy alta de radia- de hipertensin intracraneal (HTIC), efecto de masa en las pruebas de
cin, generalmente en una nica sesin, de manera muy precisa y imagen, o edema importante, sobre todo si se trata de lesiones nicas
focalizada sobre una lesin cerebral, minimizando la radiacin que y accesibles mediante ciruga.
recibe el tejido cerebral sano perilesional. Para conseguir esa preci- Finalmente, la opcin 4 tambin es cierta. Los abscesos cerebrales
sin se utiliza el marco de estereotaxia (un sistema que permite deter- localizados en el cerebelo suelen producirse por contigidad, siendo
minar con extraordinaria seguridad las coordenadas de cualquier el foco primario ms frecuente una infeccin otolgica. Dado que el
estructura o punto del cerebro). Esta excelente precisin en la distri- cerebelo se localiza en el interior de la fosa posterior, que anatmica-
bucin de la radiacin, junto con la posibilidad de administrar el mente es un espacio ya reducido, la existencia de una coleccin
tratamiento en una nica sesin, son las principales diferencias con la purulenta en el cerebelo incrementa rpidamente la presin en la
radioterapia convencional. fosa posterior, comprimiendo las estructuras troncoenceflicas, por lo
No es necesario que conozcas muchas cosas ms sobre la radioci- que generalmente un absceso cerebral en la fosa posterior es una
ruga, pero debes tener algunas ideas sobre sus indicaciones. En gene- urgencia quirrgica.
ral, y de forma muy simple, la radiociruga es til para tratar lesiones
muy bien circunscritas, de tamao igual o inferior a 3 centmetros. Pregunta 48.- R: 4
Las principal indicacin de la radiociruga en el momento actual Esta pregunta aborda el tema de la ciruga funcional estereotctica
es el tratamiento de malformaciones arteriovenosas (MAVs) con nido aplicada al tratamiento de los trastornos del movimiento. Es un tema

M exico A rgentina CTO Medicina C/ Nez de Balboa, 115 28006 MADRID (Espaa) Tfno.: (91) 782 43 32 / Fax: (91) 782 43 27
Pg. 22 NC C hile U ruguay E-mail: secretaria@ctomedicina.com; iberocto@ctomedicina.com WEB: www.ctomedicina.com; www.iberocto.com
NEUROCIRUGA
Preparacin Examen de Seleccin 05/06 1 Vuelta Seguimiento a distancia
muy especfico de la Neurociruga, que nunca han preguntado en el tante, no debemos olvidar que la Rx simple de la columna cervical es
examen MIR, pero que est en alza, motivo por el que queremos imprescindible en todo paciente politraumatizado para descartar le-
aportarte algunas nociones. siones de la columna cervical (opcin 1). Tiene la ventaja sobre los
Como sabes, la ciruga estereotctica utiliza un sistema de coorde- cortes axiales de la TC de que permite ver la alineacin de los cuerpos
nadas para poder localizar con extraordinaria precisin cualquier vertebrales.
estructura o punto del cerebro, y dirigir los instrumentos quirrgicos a La RM, sin embargo, es superior a la TC en el estudio de ciertas
esa estructura con mnimo margen de error. Con ayuda del marco de patologas. Precisamente porque la TC ve muy bien el hueso, en espa-
estereotaxia podemos, por ejemplo, tomar una biopsia de una lesin cios pequeos rodeados de hueso se crean artefactos que impiden
cerebral de muy pequeo tamao (con precisin prxima al milme- ver con buena resolucin los tejidos blandos. Este es el motivo por el
tro). Del mismo modo podemos localizar y dirigir un electrodo a que la RM es superior a la TAC en el estudio de lesiones de la mdula
cualquier ncleo profundo del cerebro, y esta es precisamente la espinal y de la fosa craneal posterior (cerebelo y tronco del encfa-
finalidad de la ciruga de los trastornos del movimiento. lo), como dice la opcin 5.
Sabemos que los trastornos del movimiento se deben a alteracio- Indudablemente la resolucin de las imgenes del cerebro que se
nes en el balance de los neurotransmisores en los ganglios de la base consigue con la RM es muy superior a la conseguida con la TC, por
(ncleo caudado, putamen, globo plido medial y lateral, ncleo ello, aunque para el diagnstico inicial de los tumores cerebrales
subtalmico, sustancia negra compacta, etc....). Se ha descubierto que suele utilizarse la TC por su mayor disponibilidad, el estudio de la
la lesin de algunos de estos ncleos puede controlar los trastornos morfologa del tumor, sus relaciones anatmicas, distribucin del ede-
del movimiento que se asocian a estas enfermedades. ma perilesional, etc. se realiza mejor con RM, especialmente en los
Existen dos tipos de tcnicas para el tratamiento de los trastornos tumores de lnea media que guardan relaciones muy importantes con
del movimiento. Las tcnicas ablativas realizan una lesin por radio- estructuras anatmicas cruciales como el tronco del encfalo, la
frecuencia que destruye las clulas y fibras nerviosas del ncleo elegi- hipfisis y tallo hipotalmico, el quiasma ptico, el tercer ventrculo,
do como diana. La estimulacin cerebral profunda lo que hace es el tlamo e hipotlamo, la glndula pineal, etc. (opcin 4).
colocar un electrodo permanente en el ncleo y estimularlo de forma
continua a alta frecuencia; se ha visto que esa estimulacin continua Pregunta 50.- R: 1
tiene el mismo efecto que la lesin del ncleo, pero es reversible si se El sndrome del canal de Guyn es el nombre que recibe la com-
retira el electrodo. presin del nervio cubital (opcin 1) a su paso por el carpo en direc-
Los ncleos diana de estas tcnicas dependen del tipo de trastor- cin a la palma de la mano. El nervio cubital queda atrapado en su
no del movimiento. recorrido por este canal anatmico localizado en el borde cubital del
En el tratamiento de la enfermedad de Parkinson, la tcnica de carpo, entre el hueso pisiforme y el gancho del hueso ganchoso. El
eleccin es la estimulacin crnica del ncleo subtalmico, porque suelo del canal de Guyon es el retinculo de los msculos flexores, y
es la tcnica que consigue un mejor control de todos los sntomas el techo son fibras procedentes del ligamento volar transverso del
(opcin 1). carpo. Por el interior del canal de Guyon discurren la arteria cubital
En algunos pacientes en los que el sntoma predominante de la (lateralmente) y el nervio cubital (medialmente).
enfermedad de Parkinson es el temblor, o en los pacientes con tem- La compresin del nervio cubital en el carpo ocasiona una altera-
blor esencial, la diana de eleccin es el tlamo (talamotoma ventro- cin sensitiva (dolor y/o parestesias o hipoestesia en el borde cubital
lateral) como se recoge en la opcin 5. de la palma de la mano, 5 dedo y borde cubital del 4 dedo), pero
En pacientes con distona generalizada, y en pacientes con enfer- no se afecta el dorso de la mano porque la rama sensitiva del nervio
medad de Parkinson en los que existen discinesias invalidantes en cubital para el dorso sale por encima del canal de Guyon (diferencia
relacin con la administracin de L-dopa, se utiliza como diana el con la compresin en el codo). El trastorno sensitivo puede acompa-
globo plido medial (opciones 2 y 3). arse de alteraciones motoras que afectan fundamentalmente a los
La respuesta falsa es la 4. La parlisis supranuclear progresiva (en- msculos de la eminencia hipotenar (oponente, flexor y separador
fermedad de Steele-Richardson-Olszewski) es una enfermedad tpica del 5 dedo), pero tambin al aproximador del pulgar y a los mscu-
de ancianos caracterizada por la presencia de signos extrapiramidales los interseos, todos ellos msculos inervados por el nervio cubital.
en conjuncin con otros sntomas entre los que destacan cadas fre- El tratamiento consiste en la seccin quirrgica del ligamento volar
cuentes, rigidez cervical con distona y una parlisis de la mirada transverso del carpo por encima del nervio cubital para aumentar el
vertical (signo clnico ms caracterstico de la enfermedad), y no se espacio en el interior del canal y liberar el nervio.
Comentarios TEST

beneficia de ninguna de las tcnicas de neurociruga funcional.

Pregunta 49.- R: 3
Han aparecido algunas preguntas en el examen MIR sobre la indica-
cin de las tcnicas de imagen aplicadas al estudio de las patologas
neurolgicas. Por este motivo merece la pena dedicar unas lneas a
aclarar en qu patologas se prefiere la tomografa computerizada cra-
neal (TC), y cundo es mejor la resonancia magntica cerebral (RM).
La respuesta falsa es la 3. La RM es inferior a la TC para el estudio
de las lesiones seas en todos los casos, simplemente porque los rayos
X ven mejor el hueso. La TC craneal es superior para el estudio de
todas las lesiones seas (fracturas craneales y vertebrales) y las lesiones
calcificadas.
Otra ventaja de la TC sobre la RM es su mayor disponibilidad
(accesible en casi todos los hospitales) y rapidez en la exploracin,
que la hacen insustituible en la valoracin de las urgencias neuroqui-
rrgicas (opcin 2). La TC craneal es la prueba diagnstica de elec-
cin en el estudio de los traumatismos craneoenceflicos, muy por
encima de la RM. Igualmente permite diagnosticar con mucha facili-
dad los problemas hemorrgicos (la sangre se ve blanca o hiperdensa
en la TC), mientras que en la RM la sangre se ve con apariencia distinta
segn las secuencias y el tiempo de evolucin.
Por tanto, podemos decir que la TC es superior a la RM para el
estudio del hueso, calcificaciones y urgencias neuroquirrgicas (so-
bre todo traumatismos craneoenceflicos y hemorragias). No obs-

M exico A rgentina CTO Medicina C/ Nez de Balboa, 115 28006 MADRID (Espaa) Tfno.: (91) 782 43 32 / Fax: (91) 782 43 27
C hile U ruguay E-mail: secretaria@ctomedicina.com; iberocto@ctomedicina.com WEB: www.ctomedicina.com; www.iberocto.com
NC Pg. 23
PEDIATRA
Preparacin Examen de Seleccin 05/06 1 Vuelta Seguimiento a distancia

1. Seale la afirmacin correcta respecto al test de Apgar: 4) Generalmente persiste de forma intermitente hasta al-
canzar las treinta y seis semanas de edad postconcepcio-
1) Se debe practicar nicamente a los RN patolgicos o nal.
supuestamente patolgicos. 5) Puede reducirse el nmero de episodios apneicos me-
2) Entre los parmetros que se valoran se encuentra la diante una transfusin de concentrado de hemates o
frecuencia respiratoria. estimulacin fsica externa.
3) Se debe realizar a los 1-5-10 minutos de vida.
4) Un test de Apgar de 4 al minuto de vida implica un alto 6. Ante un neonato a trmino que tras cesrea presenta
riesgo de parlisis cerebral. precozmente taquipnea y cianosis que desaparece con
5) La palidez cutnea generalizada supone una puntua- pequeas cantidades de oxgeno, en el que la auscultacin
cin de 1 en el test de Apgar. pulmonar es normal y en el que la radiografa muestra
marcas vasculares prominentes y lquido en las cisuras sin
2. RN pretrmino que, a las 6 h de vida, inicia cuadro compa- broncograma areo, sospecharemos:
tible con un cuadro de distress respiratorio tipo I, y que a
la exploracin presenta quejido espiratorio a la ausculta- 1) Enfermedad de membrana hialina.
cin, ausencia de retraccin esternal, tiraje intercostal leve 2) Taquipnea transitoria del recin nacido.
con movimientos toracoabdominales sincrnicos y sin ale- 3) Sndrome de aspiracin meconial.
teo nasal. Su puntuacin en el Score de Silverman ser: 4) Persistencia de la circulacin fetal.
5) Sndrome de Wilson-Mikity.
1) 2.
2) 3. 7. RN de 35 semanas de edad gestacional que en las primeras
3) 4. 12 horas de vida, presenta taquipnea, quejido audible y
4) 5. cianosis progresiva, refractaria a la administracin de oxge-
5) 6. no. A la auscultacin encontramos crepitantes, sobre todo en
bases pulmonares. En la gasometra aparece hipoxemia pro-
3. Cul es la etiologa ms frecuente del sndrome febril en el gresiva, aumento de la PCO2 y acidosis metablica. Qu
lactante?: radiografa de trax esperara encontrar en este momento?:

1) Fiebre de sed. 1) Normal.


2) Colagenosis. 2) Infiltrado de patrn reticulogranular con broncograma
3) Neoplsica. areo.
4) Infecciosa. 3) Hiperinsuflacin y aumento de las marcas vasculares, sin
5) Facticia. broncograma.
4) Patrn de esponja.
4. Seale la afirmacin FALSA respecto a las lesiones produci- 5) Borde de ambos pulmones colapsados.
das por traumatismos obsttricos:
8. En el tratamiento de la EMH se incluyen las siguientes
1) El cefalohematoma es una hemorragia subperistica, y medidas, EXCEPTO:
est delimitado por las suturas craneales.
2) La clavcula es el hueso que se fractura con mayor 1) Administracin de surfactante endotraqueal.
frecuencia durante el parto. 2) Antibioterapia.
3) El llamado ndulo de Stroemayer suele requerir trata- 3) Administracin de oxgeno y ventilacin mecnica, si es
miento quirrgico. preciso.
4) La rotura heptica no siempre precisa tratamiento qui- 4) Prostaglandinas.
rrgico. 5) Control del aporte de lquidos.
Preguntas TEST

5) Las fracturas craneales ms frecuentes en el parto son las


lineales. 9. Un nio de 42 semanas de gestacin, arrugado, plido,
hipotnico, apneico y cubierto de lquido amnitico verdo-
5. Recin nacido de cinco das de vida, EG 36 semanas y peso so. Respecto al cuadro que puede padecer este paciente,
2.500 g, es llevado al servicio de Urgencias porque, mientras seale la opcin FALSA:
dorma, sus padres han notado que durante 5 segundos
dejaba de mover el trax, volviendo a respirar de forma muy 1) La primera maniobra en su reanimacin debe ser la
rpida posteriormente. No refieren cambios en la colora- aspiracin de trquea bajo visin laringoscpica.
cin de la piel. Seale la respuesta FALSA: 2) Es un patologa tpica de RNPT y RNT.
3) El E. coli es el germen ms frecuente de infeccin bacte-
1) Es ms frecuente en RNPT. riana en estos pacientes.
2) Estamos ante un patrn de respiracin peridica, que 4) En la Rx de trax es tpico encontrar un patrn de
consiste en pausas apneicas de cinco a diez segundos, atrapamiento areo.
seguidas de tandas de respiracin rpida durante diez o 5) Esta patologa produce una llamativa hipertensin pul-
quince segundos. monar.
3) Lo ms frecuente es que se acompaen de cianosis y
cambios en la FC. 10. Un nio a trmino recibe al nacer asistencia respiratoria
con bolsa y mscara con una presin mxima superior a

M exico A rgentina
C hile U ruguay
CTO Medicina C/ Nez de Balboa, 115 28006 MADRID (Espaa) Tfno.: (91) 782 43 32 / Fax: (91) 782 43 27
E-mail: secretaria@ctomedicina.com; iberocto@ctomedicina.com WEB: www.ctomedicina.com; www.iberocto.com
PD Pg. 1
PEDIATRA
Seguimiento a distancia Preparacin Examen de Seleccin 05/06 1 Vuelta
20 cmH2O por un perodo de 3 minutos. Al llegar a planta, 4) La vitamina C no parece tener relacin con ella.
presenta taquipnea, ronquidos y cianosis con desviacin 5) Hay otros factores cooperantes con la administracin de
a la derecha del choque de la punta. El trax es asimtrico, oxgeno para su desarrollo.
y a la auscultacin se evidencia hipoventilacin en el
hemitrax izquierdo. El diagnstico ms probable es: 14. Un RN pretrmino de 1.800 g de peso, con antecedentes de
hipoxia perinatal, a los 10 das de vida presenta distensin
1) Sepsis por estreptococo del grupo B. abdominal importante, vmitos y deposiciones hemorrgi-
2) Sndrome de corazn izquierdo hipoplsico. cas. En la Rx de abdomen se observa edema de asas intes-
3) Neumomediastino. tinales, con un patrn en "miga de pan" y presencia de gas
4) Neumotrax. en la pared intestinal. Cul sera la opcin teraputica ms
5) Fstula traqueoesofgica. adecuada en este paciente?:

11. Un RN de 32 semanas, con antecedente de enfermedad de 1) Enema de solucin hiperosmolar.


membrana hialina sometido a ventilacin mecnica con 2) Alimentacin enteral a dbito continuo a travs de una
altas concentraciones de oxgeno, tiene actualmente un sonda nasogstrica.
mes de vida y sigue dependiendo del respirador para 3) Actitud expectante y vigilancia cuidadosa, ante la posi-
mantener adecuadas saturaciones de oxgeno. En los lti- bilidad de perforacin intestinal.
mos das presenta adems oliguria con edemas en miem- 4) Se debe suspender la alimentacin enteral y pautar
bros inferiores y se palpa el hgado a 5 cm bajo el reborde fluidoterapia y antibioterapia i.v.
costal derecho. En la Rx de trax se observa una imagen de 5) La presencia de neumatosis intestinal aconseja la inter-
pequeas zonas redondas y claras que alternan con otras vencin quirrgica urgente.
de mayor densidad. En relacin con la patologa que presen-
ta este paciente, seale la respuesta FALSA: 15. RN de 38 semanas de edad gestacional presenta distensin
abdominal y vmitos biliosos durante el primer da de vida.
1) Se consideran factores de riesgo para su desarrollo tener En el examen fsico se palpan cordones duros que siguen el
distress respiratorio severo que precise largos perodos de marco clico. Una radiografa en bipedestacin muestra
administracin de O2. una masa en pompa de jabn en cuadrante inferior dere-
2) Las principales causas de muerte de estos pacientes son cho. El tratamiento inmediato ms apropiado es:
la bronquiolitis necrotizante y fallo cardaco derecho.
3) El tratamiento incluye broncodilatadores, diurticos y 1) Antibioterapia.
dexametasona. 2) Supositorios de glicerina.
4) La nefrolitiasis puede ser una complicacin de estos 3) Enema de solucin hiperosmolar.
pacientes, secundaria al uso de diurticos y a la alimen- 4) Estimulacin rectal.
tacin parenteral. 5) Ciruga.
5) A pesar de que pueda retirarse la administracin de O2
antes de salir de la unidad de cuidados intensivos el 16. Seale la afirmacin FALSA de entre las siguientes respecto
pronstico para la mayora de estos pacientes es malo, a la ictericia fisiolgica:
presentando en la adolescencia obstruccin de las vas
respiratorias e hiperreactividad bronquial. 1) Las cifras mximas de bilirrubina suelen alcanzarse a los
3-4 das.
12. Un varn nacido a trmino de una hora de vida, est 2) La ictericia en los RNPT suele ser de inicio algo ms tardo
ciantico y presenta quejido respiratorio. El examen fsico y ms prolongada que en los RNT.
revela dificultad respiratoria moderada, asimetra torcica 3) Puede comenzar en las primeras 24 horas de vida en el
y abdomen excavado. Los ruidos cardacos se auscultan RNT.
sobre hemitrax derecho y aparece hipoventilacin en 4) Su duracin suele ser inferior a 10-15 das.

Preguntas TEST
hemitrax izquierdo. El diagnstico ms compatible con 5) En los RNPT pueden alcanzarse cifras de hasta 14 mg/dl.
estos hallazgos es:
17. En relacin a la ictericia por lactancia materna o sndrome
1) Eventracin diafragmtica. de Arias es FALSO que:
2) Malformacin adenomatosa del pulmn.
3) Quiste por duplicacin del esfago. 1) Su frecuencia es aproximadamente de 1 por cada 200
4) Hernia diafragmtica. RN a trmino que reciben lactancia materna.
5) Fstula traqueoesofgica. 2) Es una ictericia que aparece a la mitad o final de la
primera semana, alcanzando su mximo en la tercera.
13. Respecto a la retinopata del prematuro, seale la respuesta 3) Se pueden alcanzar cifras de hasta 30 mg/dl, de ah el
FALSA: riesgo de kernicterus en estos neonatos.
4) En la patogenia se implican varias sustancias presentes en
1) La administracin de oxgeno en altas concentraciones la leche materna como pregnanodiol, cidos grasos de
es un factor de claro riesgo. cadena larga y glucuronidasas.
2) El tiempo de administracin tambin influye. 5) Cuando se suspende la lactancia materna durante 2-4
3) En el 90% de los afectados, la enfermedad progresa hacia das se produce una rpida disminucin de la bilirrubina
una fibroplasia retrolental. srica.

M exico A rgentina
Pg. 2 PD C hile U ruguay
CTO Medicina C/ Nez de Balboa, 115 28006 MADRID (Espaa) Tfno.: (91) 782 43 32 / Fax: (91) 782 43 27
E-mail: secretaria@ctomedicina.com; iberocto@ctomedicina.com WEB: www.ctomedicina.com; www.iberocto.com
PEDIATRA
Preparacin Examen de Seleccin 05/06 1 Vuelta Seguimiento a distancia

18. En relacin a la enfermedad hemoltica por isosensibiliza- 5) El inicio de las manifestaciones clnicas es ms tardo.
cin ABO, seale la FALSA:
23. NO es propio del RN hijo de madre diabtica:
1) La madre suele ser O.
2) Los anticuerpos causantes suelen ser de la clase Ig G. 1) Megalencefalia.
3) Pueden resultar afectos los productos del primer embarazo. 2) Sndrome asfctico.
4) El antgeno ms frecuentemente implicado es A1. 3) Hiperbilirrubinemia.
5) La enfermedad se agrava en los embarazos siguientes, si 4) Sndrome de regresin caudal.
no hacemos profilaxis. 5) Cardiomiopata.

19. Un lactante de 1 semana es llevado a Urgencias en estado de 24. En la sepsis del RN, los grmenes ms frecuentes son:
coma con hemorragias en la retina y palidez importante.
Entre sus antecedentes destaca que el parto tuvo lugar en el 1) Klebsiella.
domicilio, no present complicaciones posteriores y est 2) E. coli y estreptococos del grupo A.
recibiendo lactancia materna. Seale lo FALSO respecto a 3) E. coli y estreptococos del grupo B.
este cuadro: 4) Estafilococo.
5) Gardnerella vaginalis.
1) Se debe a un dficit de los factores dependientes de
vitamina K. 25. Qu hara usted ante un RN de madre con hepatitis B activa
2) Suele iniciarse a partir de las primeras 24 horas. en el momento del nacimiento?:
3) Es ms frecuente en hijos de madres que han tomado
fenitona o fenobarbital. 1) Observar la aparicin de los sntomas y luego tratar.
4) Las complicaciones hemorrgicas son menos frecuentes 2) Extraer sangre de cordn para estudiar transaminasas.
en nios que toman LM que en los que reciben bibern. 3) Exanguinotransfusin inmediata, si la madre tiene el
5) Se trata con la administracin de vitamina K y/o plasma antgeno HBe positivo.
fresco congelado. 4) Poner una dosis bimensual de gammaglobulina.
5) Iniciar vacunacin y gammaglobulina en paritorio.
20. Respecto a la anemia en el perodo neonatal, seale cul de
las siguientes afirmaciones es FALSA: 26. Un RN de bajo peso para su edad gestacional presenta un
exantema petequial, hepatoesplenomegalia y coriorretini-
1) La anemia fisiolgica es ms intensa y ms precoz en los tis. En la TC craneal se observan calcificaciones periventri-
RNT que en los RNPT. culares sin otros hallazgos. Con respecto al cuadro que
2) El cuadro de anemia fisiolgica neonatal est en relacin parece presentar este paciente, seale la verdadera:
con la hemlisis de glbulos rojos y con un dficit de
eritropoyetina. 1) Es la segunda causa ms frecuente de infeccin viral
3) En los RNPT est indicada la administracin de suple- congnita.
mentos de hierro a partir de los 2 meses de vida. 2) El 90% de los recin nacidos infectados estarn asinto-
4) La anemia fisiolgica puede precisar transfusin en mticos al nacer.
algunos casos. 3) La manifestacin ms tarda y caracterstica es una
5) La betatalasemia no se manifestar antes de los 6 meses hipoacusia de transmisin.
de vida. 4) La afectacin ocular tpica es una retinitis pigmentosa.
5) Estos nios suelen desarrollar macrocefalia evolutiva-
21. Seale el diagnstico que le parece ms probable para un mente.
RN hijo de madre diabtica, que, tras un parto sin inciden-
cias, present a las 12 horas de vida temblores, letargia y 27. Cul de las siguientes manifestaciones NO forma parte de
Preguntas TEST

rechazo de las tomas as como ligera acrocianosis. A los 2 la ttrada de Sabin?:


das de vida presenta ictericia que requiere fototerapia:
1) Coriorretinitis.
1) Hipoglucemia neonatal. 2) Cataratas.
2) Hipocalcemia neonatal. 3) Crisis convulsivas.
3) Policitemia. 4) Hidrocefalia.
4) Alteracin del sistema nervioso central. 5) Calcificaciones cerebrales.
5) Sndrome de abstinencia a opiceos.
28. Un RN de 38 semanas de edad gestacional y 1.750 g,
22. Comparando los RN de madres adictas a herona y adictas presenta erupcin prpuro-petequial, hepato-esplenome-
a metadona, es cierto que en los hijos de heroinmanas: galia y se ausculta un soplo continuo spero en 2 espacio
intercostal izquierdo. En la Rx de trax se objetiva cardio-
1) La asistencia prenatal es mejor. megalia con aumento de la vascularizacin pulmonar as
2) Es mayor la incidencia de consumo de mltiples drogas como imgenes lineales radiotransparentes con aumento
por la madre. de densidad sea en la zona metafisaria del hmero sin
3) El peso medio de los neonatos al nacimiento es mayor. reaccin peristica. En relacin con el cuadro que presenta
4) La incidencia de convulsiones es menor. este paciente, seale la respuesta FALSA:

M exico A rgentina
C hile U ruguay
CTO Medicina C/ Nez de Balboa, 115 28006 MADRID (Espaa) Tfno.: (91) 782 43 32 / Fax: (91) 782 43 27
E-mail: secretaria@ctomedicina.com; iberocto@ctomedicina.com WEB: www.ctomedicina.com; www.iberocto.com
PD Pg. 3
PEDIATRA
Seguimiento a distancia Preparacin Examen de Seleccin 05/06 1 Vuelta
1) Cuando la madre contrae la infeccin en el tercer sobre base eritematosa en mucosa subyugal. Respecto a la
trimestre aumenta el riesgo de infeccin del feto, pero el enfermedad que padece esta paciente, seale la afirmacin
cuadro clnico es ms leve. FALSA:
2) Si la infeccin se produce despus de la 12 semana de
gestacin, es poco probable la asociacin de una car- 1) El exantema comienza por la cara, tiene evolucin
diopata congnita. descendente, y desaparece en el mismo orden en el que
3) La presencia de IgM especfica anti-rubola refleja la apareci.
produccin en tero de anticuerpos por el feto. 2) La neumona de clulas gigantes de Hecht es menos
4) El lactante puede permanecer con infeccin crnica frecuente que la neumona por sobreinfeccin bacteria-
durante meses despus del nacimiento. na.
5) El virus puede aislarse en sangre. 3) La gravedad de la enfermedad est directamente relacio-
nada con la intensidad y confluencia del exantema.
29. Cul de las siguientes manifestaciones es ms frecuente en 4) El prurito suele ser intenso.
la forma precoz de sfilis congnita?: 5) Entre las posibles complicaciones se encuentra la aner-
gia cutnea y la reactivacin de una TBC preexistente.
1) Hepatoesplenomegalia.
2) Rinitis. 34. Seale cul de los siguientes afirmaciones respecto a la
3) Pnfigo sifiltico. rubola es FALSA:
4) Osteocondritis con reaccin peristica.
5) Queratitis. 1) El signo ms caracterstico del perodo prodrmico son
las adenopatas.
30. Un RN hijo de madre prostituta presenta en el 7 da de vida 2) El exantema suele aclararse al tercer da con mnima
un cuadro de fiebre con letargia y rechazo de las tomas as descamacin.
como convulsiones. A la exploracin destaca hepatoesple- 3) La artritis afecta sobre todo a mujeres y nios preadoles-
nomegalia con fontanela anterior abombada y lesiones centes, principalmente a grandes articulaciones.
vesiculares en calota craneal. Cul sera la causa ms 4) El bazo puede estar aumentado de tamao.
probable de este cuadro?: 5) Las complicaciones neurolgicas en forma de encefalitis
son mucho menos frecuentes que en un caso de saram-
1) Infeccin por VIH. pin.
2) Infeccin por CMV.
3) Infeccin por VHS tipo I. 35. Nos traen a Urgencias a un escolar de 7 aos de edad, que
4) Infeccin por VHS tipo II. estando totalmente bien, presenta fiebre de 40C, acompa-
5) Exantema txico del RN. ada de vmitos, cefaleas, pulso rpido y algo dbil, con
sensacin de enfermedad grave. A la exploracin destacan
31. Un recin nacido de tres semanas de vida que desprendi amgdalas hipertrficas con exudado blanco-amarillento, as
el cordn umbilical a los diecisis das y cuyo ombligo no ha como enantema en paladar blando. Adems presenta adeni-
cicatrizado, presenta una pequea tumoracin sonrosada tis satlite en ngulo mandibular izquierdo. A las 14 horas
con secrecin seromucosa con un pH de 7,4. Cul de los aparece exantema puntiforme a la palpacin que respeta el
siguientes diagnsticos le parece ms probable?: ngulo nasogeniano. A los 3-4 das se inicia descamacin
furfurcea. El tratamiento etiolgico de este proceso es:
1) Onfalocele.
2) Granuloma umbilical. 1) Midecamicina por 5 das.
3) Quiste o persistencia del uraco. 2) Tratamiento sintomtico.
4) Gastrosquisis. 3) Penicilina oral durante 10 das.
5) Ombligo amnitico. 4) Penicilina benzatina en 3 dosis consecutivas.

Preguntas TEST
5) Asociar corticoides al antibitico elegido.
32. En relacin con el desarrollo psicomotor en el nio, seale
la respuesta FALSA: 36. Un nio de 4 aos acude al pediatra por aparicin de
lesiones papulosas rojas, algunas con vesculas blanqueci-
1) La sonrisa referencial aparece ente el 1 y 2 mes. nas no umbilicadas en tronco y mucosa oral, muy prurigi-
2) El sostn ceflico se inicia a los tres meses. nosas. Dos das antes presentaba cuadro catarral con fiebre
3) La sedestacin estable se consigue a los 6-8 meses. moderada. Respecto al cuadro que presenta este nio, todo
4) Emite sus primeros fonemas con 3-4 semanas. lo siguiente es cierto, EXCEPTO:
5) Realiza frases cortas con 2 aos.
1) Est causado por un poxvirus, el virus de la varicela zoster.
33. Nia de 6 aos que presenta desde hace 3 das fiebre de 38 2) Las lesiones costrosas no contienen virus viables.
C que no cede con antitrmicos, tos seca y facies congestiva. 3) La complicacin ms frecuente es la sobreinfeccin de
Hoy acude por aparecer exantema maculopapuloso, rojo las lesiones cutneas.
intenso, no puntiforme, confluente, que no se blanquea a 4) La neumona varicelosa es poco frecuente en la infancia.
la presin en la parte superior del trax, cara y parte 5) La encefalitis postvaricela que se presenta con signos
proximal de EESS. A la exploracin. T 40,3C, adenopatas cerebelosos tiene mejor pronstico que si lo hace con
bilaterales en ngulo mandibular y lesiones blanquecinas signos cerebrales.

M exico A rgentina
Pg. 4 PD C hile U ruguay
CTO Medicina C/ Nez de Balboa, 115 28006 MADRID (Espaa) Tfno.: (91) 782 43 32 / Fax: (91) 782 43 27
E-mail: secretaria@ctomedicina.com; iberocto@ctomedicina.com WEB: www.ctomedicina.com; www.iberocto.com
PEDIATRA
Preparacin Examen de Seleccin 05/06 1 Vuelta Seguimiento a distancia

37. Nio de 7 aos presenta rash eritematoso en ambas mejillas, 1) Reaccin a la penicilina.
tronco y zona proximal de extremidades, con tendencia a 2) Prpura de Schnlein-Henoch.
aclararse en la zona central. Con respecto a esta enferme- 3) Mononucleosis infecciosa.
dad, seale la afirmacin FALSA: 4) Enfermedad de Kawasaki.
5) Prpura trombocitopnica idioptica.
1) Es producida por el parvovirus B19.
2) El perodo de incubacin es de 7 das. 41. Nio de 9 aos que acude a Urgencias por presentar
3) El perodo prodrmico se caracteriza por fiebre alta. tumefaccin cervical bilateral sin fiebre ni otros sntomas.
4) El exantema caracterstico aparece en tres etapas. En la exploracin fsica presenta: BEG, febrcula, palpacin
5) Como complicacin puede producirse una crisis apl- de varias adenopatas dolorosas de 1,5 cm, bilaterales en
sica grave. regin submandibular, y un exudado blanco-amarillento
sobre amgdalas hipertrficas e hipermicas. En el frotis
38. Un lactante de 7 meses presenta sbitamente fiebre de nasofarngeo aparece cultivo positivo para estreptococo
40C. No existe ningn otro sntoma y la exploracin fsica beta hemoltico del grupo A. Se instaura tratamiento con
es normal, salvo el hallazgo de una coriza ligera y adenopa- amoxicilina v.o., a pesar de lo cual la paciente regresa al 5
tas cervicales posteriores. El nio parece encontrarse bas- da por persistencia del cuadro, fatiga intensa, exantema
tante bien. Cul de las siguientes afirmaciones describe maculopapuloso en tronco y extremidades, y fiebre de
mejor los factores diagnsticos que deben considerarse en 39C. Cul es el diagnstico ms probable?:
este caso?:
1) Herpangina.
1) Si apareciera una erupcin cutnea 24 horas despus de 2) Escarlatina.
comenzar la fiebre, sera probable el diagnstico de 3) Sarampin.
rosola o exantema sbito. 4) Rubola.
2) En la rosola infantil, el diagnstico diferencial puede ser 5) Mononucleosis infecciosa.
difcil, puesto que en las primeras 36 horas cursa con
leucocitosis, con predominio de neutrfilos. 42. Respecto a la mononucleosis infecciosa, es FALSO que:
3) La fiebre podra corresponder a los prdromos de una
rubola. 1) Hasta en el 80% hay aumento de transaminasas.
4) Si la fiebre dura 3 das, desaparece luego rpidamente 2) En la analtica aparece leucopenia.
y aparece una erupcin cutnea en ese momento, sera 3) Ttulos elevados de anticuerpos frente al virus de Epstein-
probable el diagnstico de eritema infeccioso. Barr.
5) Es probable una escarlatina, pues es una enfermedad 4) Existe un sndrome linfoproliferativo en varones que han
propia de nios de 6 a 18 meses. sufrido la infeccin por este virus.
5) La complicacin ms temida es la rotura esplnica.
39. Jennifer tiene 16 meses de edad y acude a Urgencias por
presentar desde hace 2 das fiebre de 39C que su pediatra 43. Un beb de 3 meses, que recibe lactancia materna, presenta
atribuy a cuadro respiratorio. A la exploracin presenta una historia de accesos de tos con congestin facial, lagri-
fiebre de 40C y contractura antilgica en flexin de extre- meo y acaba vomitando. Hace 2 semanas present cuadro
midad inferior izquierda. Queda ingresada para estudio, y catarral de vas reas altas. En la Rx trax se aprecian breves
4 das ms tarde persiste el cuadro febril, junto con edema infiltrados perihiliares. Con respecto a este cuadro, seale
indurado en ambos pies, fisuras labiales, conjuntivitis no la respuesta que NO sea cierta:
purulenta y exantema polimorfo en tronco. El diagnstico
ms probable ser: 1) Puede aparecer al principio de la vida por falta de
anticuerpos maternos.
1) Rubola. 2) La fase catarral precede al perodo paroxstico.
Preguntas TEST

2) Mononucleosis infecciosa. 3) La tos paroxstica puede durar de 1 a 4 semanas.


3) Prpura de Schnlein-Henoch. 4) El recuento de leucocitos suele elevarse en forma consi-
4) Enfermedad de Kawasaki. derable con predominio de polimorfonucleares.
5) Eritema infeccioso. 5) El perodo de incubacin oscila entre 5 y 10 das.

40. Paciente varn de 5 aos de edad que acude por presentar 44. Un varn de 4 aos presenta fiebre, tumefaccin parotdea
cuadro febril y odinofagia con exudado blanquecino en y cefalea. Respecto al cuadro que presenta este paciente,
ambas amgdalas que su pediatra trat con penicilina oral. seale la opcin FALSA:
Dos das despus, presenta petequias y prpura palpables,
principalmente en miembros inferiores, as como artral- 1) La tumefaccin parotdea es bilateral, pero asincrnica.
gias en ambos tobillos que le impiden la marcha y dolor 2) La orquitis y la meningoencefalitis pueden aparecer en
abdominal de tipo clico. Los estudios complementarios ausencia de parotiditis.
revelan 13 g/dl de Hb; leucocitos 10.500/mm3 con frmula 3) La inmunidad transplacentaria probablemente explica
normal; plaquetas 485.000/mm3; tiempo de protrombina su ausencia durante los 6 primeros meses de vida.
de 95% y tiempo parcial de tromboplastina de 27 segun- 4) La enfermedad est causada por un virus.
dos (control 25 segundos). La causa ms probable de estos 5) La sordera nerviosa unilateral es una complicacin
hallazgos es: frecuente de esta enfermedad.

M exico A rgentina
C hile U ruguay
CTO Medicina C/ Nez de Balboa, 115 28006 MADRID (Espaa) Tfno.: (91) 782 43 32 / Fax: (91) 782 43 27
E-mail: secretaria@ctomedicina.com; iberocto@ctomedicina.com WEB: www.ctomedicina.com; www.iberocto.com
PD Pg. 5
PEDIATRA
Seguimiento a distancia Preparacin Examen de Seleccin 05/06 1 Vuelta
45. Con respecto al SIDA en la infancia, seala la opcin 4) A los 14 aos es preciso vacunar con Td (ttanos y
correcta: difteria adultos), y no son necesarios nuevas dosis de
recuerdo.
1) El SIDA precoz es menos frecuente que el tardo y la 5) No est contraindicada en pacientes inmuno-deprimi-
clnica predominante es infecciosa. dos.
2) El SIDA tardo es ms frecuente y predomina la clnica
neurolgica. 50. Con respecto a la vacuna contra el Haemophilus influenzae
3) La neumona por MAI es la infeccin oportunista ms tipo B, es FALSO que:
frecuente en el SIDA peditrico.
4) La manifestacin ms frecuente en el SIDA infantil es la 1) Las nuevas vacunas conjugadas son aptas para la inmu-
Hepatitis B. nizacin en lactantes mayores de 2 meses.
5) La parotiditis es ms frecuente en el SIDA infantil que en 2) Es muy inmungena.
el del adulto. 3) Sus efectos adversos son mnimos.
4) Es necesario aplicarla despus de los 5 aos.
46. Sobre el diagnstico de un nio VIH+, es FALSO que: 5) En el calendario vacunal debera administrarse a los 2,
4, 6 y 18 meses.
1) Una IgG+ en un lactante de 10 meses indica infeccin
activa por el VIH. 51. Seale la afirmacin FALSA respecto a la vacuna del VHB:
2) La mayora de los nios infectados presenta hipergam-
maglobulinemia policlonal precoz. 1) La vacuna de VHB est realizada por ingeniera gentica
3) La linfopenia e inversin del cociente CD4/CD8 son con efectos adversos mnimos.
menos llamativas que en el adulto. 2) La pauta de administracin en caso de riesgo inmediato
4) No es til la determinacin de IgA contra el VIH, porque como contacto con una jeringuilla sera 0, 1, 2 meses, y
no suele aparecer hasta el 6 mes de vida. al ao.
5) Los resultados positivos en la PCR han mostrado una 3) La vacuna VHB no est recomendada en mujeres emba-
correlacin positiva con el aislamiento del virus en razadas.
cultivo. 4) No est indicada su administracin en nios mayores de
7 aos.
47. Respecto a la vacuna triple vrica, seale la afirmacin 5) Se puede administrar con la gammaglobulina anti VHB,
FALSA: siempre que se administre en distinto punto.

1) Es una vacuna de microorganismos atenuados. 52. Cul sera la actitud ms adecuada a seguir en los contactos
2) La alergia al huevo ha dejado de ser una contraindica- de un nio con meningitis meningoccica?:
cin absoluta para su administracin.
3) Est contraindicada su utilizacin en embarazadas y 1) Realizar quimioprofilaxis con ceftriaxona durante 2 das.
pacientes VIH+. 2) No existe quimioprofilaxis eficaz.
4) Su administracin en el calendario actual se realiza a los 3) Administracin de la vacuna antimeningoccica en
15 meses y a los 4 aos. nios menores de 5 aos.
5) En zonas de elevada morbimortalidad puede adminis- 4) Administracin de rifampicina durante 2 das.
trarse la primera dosis a los 6 - 12 meses, con una segunda 5) Administracin de rifampicina durante 4 das.
dosis de refuerzo despus del ao de vida.
53. Cul sera la actitud a seguir ante un nio de 4 aos
48. Seale la afirmacin FALSA respecto a la vacuna de la polio: asintomtico con Mantoux negativo, hijo de un tuberculoso
bacilfero?:
1) La vacuna tipo Sabin es de virus vivos atenuados.
1) Observacin y repetir Mantoux en 6 meses.

Preguntas TEST
2) La vacuna oral produce inmunidad local (IgA) y general
(IgG). 2) Realizar quimioprofilaxis primaria durante 2 meses con
INH y repetir el Mantoux posteriormente.
3) En el calendario actual se administra a los 2-4-6-18
3) Realizar quimioprofilaxis primaria durante 6 meses con
meses y entre los 4 y 6 aos.
INH.
4) La vacuna tipo Salk est contraindicada en familiares de
4) Administrar tratamiento completo con tres frmacos
inmunodeprimidos.
durante 6 meses.
5) En caso de diarrea o vmitos sera conveniente retrasar
5) Vacunacin con BCG.
la vacunacin con virus atenuados.
54. Respecto a las diferencias entre la leche materna y la leche
49. Seale la afirmacin cierta respecto a la vacuna DTP:
de vaca, seale la afirmacin correcta:
1) La vacuna de la tos ferina est contraindicada en mayo-
res de 18 meses. 1) La leche de vaca tiene un contenido energtico 2 veces
2) No puede administrarse conjuntamente con otras vacu- superior al de la leche materna.
nas. 2) La leche de vaca contiene una cantidad de calcio
3) La vacuna DTPa (acelular) tiene mayor riesgo de efectos superior a la leche materna.
secundarios que la DTP, por lo que no se utiliza de forma 3) La leche materna contiene mayor cantidad de vitamina
rutinaria. K que la leche de vaca.

M exico A rgentina
Pg. 6 PD C hile U ruguay
CTO Medicina C/ Nez de Balboa, 115 28006 MADRID (Espaa) Tfno.: (91) 782 43 32 / Fax: (91) 782 43 27
E-mail: secretaria@ctomedicina.com; iberocto@ctomedicina.com WEB: www.ctomedicina.com; www.iberocto.com
PEDIATRA
Preparacin Examen de Seleccin 05/06 1 Vuelta Seguimiento a distancia

4) La leche de vaca es ms rica en hidratos de carbono que 5) Indicar a la madre medidas posturales y usar espesante
la leche materna. de la leche.
5) La leche materna tiene una relacin casena/seroprote-
nas de 70/30. 59. Seale la respuesta verdadera respecto a la estenosis hiper-
trfica de ploro:
55. Varn de 13 meses que desde hace 2 das presenta fiebre de
38 C y sntomas catarrales. Hoy acude por presentar 1) El signo radiolgico frecuente es el llamado "signo de la
vmitos no biliosos y 6 deposiciones en las ltimas 2 horas, cuerda".
lquidas, no ftidas, sin moco ni pus ni sangre. Respecto al 2) Es ms frecuente en nias.
cuadro que presenta este paciente, seale cul de las 3) Presenta alcalosis metablica hiperclormica con hipo-
siguientes afirmaciones es FALSA: potasemia.
4) La prueba diagnstica de eleccin es la Rx de abdomen.
1) El agente causal ms frecuente en la infancia es el 5) El tratamiento consiste en la pilorotoma submucosa de
rotavirus. Ramstedt.
2) Suele ser un proceso autolimitado, que cede en 3-10
das. 60. RN con sndrome de Down presenta vmitos biliosos a las
3) Puede causar deshidratacin isotnica con acidosis. 36 horas de vida. A la exploracin destaca abdomen exca-
4) Es frecuente que tienda a la cronificacin. vado. El diagnstico ms probable es:
5) El diagnstico puede hacerse mediante deteccin rpi-
da de antgeno de rotavirus en heces. 1) Estenosis hipertrfica de ploro.
2) Enfermedad de Hirschprung.
56. Lactante de 3 meses de edad, alimentado al pecho hasta hace 3) Divertculo de Meckel.
1 semana en que, debido a una hipogalactia, se decide 4) Atresia duodenal.
suplementar con una frmula artificial. Hoy acude a Urgen- 5) Invaginacin intestinal.
cias por presentar anorexia, vmitos, prdida de peso as
como exantema urticarial en tronco. Entre sus antecedentes 61. Con respecto al divertculo de Meckel, es FALSO que:
destaca RNT, nacido por cesrea y haber iniciado alimenta-
cin oral a las 4 horas de vida con frmula artificial. Con 1) Afecta al 1-2% de la poblacin.
respecto a la patologa actual, es FALSO que: 2) Se localiza en leon a unos 75 cm de la vlvula ileocecal.
3) Se denomina hernia de Littr cuando el divertculo de
1) No suele haber antecedentes familiares de atopia. Meckel se aloja en una hernia inguinal indirecta.
2) La protena comnmente implicada es la betalactoglo- 4) Su manifestacin clnica ms frecuente es una hemorragia
bulina. gastrointestinal acompaada de dolor abdominal.
3) Las pruebas cutneas sern positivas. 5) El mtodo diagnstico ms til es la gammagrafa con
4) El diagnstico definitivo se realiza mediante un test de Tc 99.
supresin-provocacin.
5) Suele ser un proceso transitorio. 62. Un varn de 14 meses, previamente sano, de forma brusca
presenta crisis de llanto con encogimiento de piernas y
57. Debemos sospechar una atresia de esfago en todos los palidez, separados por perodos asintomticos de duracin
siguientes casos, EXCEPTO: variable en los que permanece decado. Ha presentado dos
vmitos y una deposicin semilquida sanguinolenta. Explo-
1) Oligohidramnios materno. racin fsica: afebril; palpacin de una masa alargada y mal
2) La sonda utilizada en el paritorio no se puede introducir definida, localizada en hipocondrio derecho. En la Rx de
en estmago. abdomen aparece silencio areo en hipocondrio derecho.
3) Exceso de secreciones orales. Seale cul de las siguientes afirmaciones es cierta respecto
Preguntas TEST

4) Cuando aparecen tos y cianosis con los intentos de al cuadro que presenta este paciente:
alimentacin.
5) Ante un abdomen lleno de aire, timpanizado. 1) Debe retrasarse el tratamiento durante 4-6 horas debido
a la alta incidencia de reduccin espontnea.
58. Varn de 3 meses que consulta por vmitos desde hace 1 2) La recidiva es menos frecuente despus de la reduccin
mes. Vomita despus de todas las tomas, sin fuerza; come hidrosttica que tras la reduccin quirrgica.
siempre con buen apetito y presenta adecuado desarrollo 3) Si hay signos de obstruccin intestinal de 48 horas de
ponderoestatural, sin prdida de peso. Realiza dos deposi- evolucin, es preferible la reduccin quirrgica.
ciones diarias con heces de consistencia, aspecto y volumen 4) Una invaginacin asociada a divertculo de Meckel
normal. Entre sus antecedentes destaca estar alimentado suele reducirse satisfactoriamente con enema opaco.
con frmula artificial desde el mes de vida. Su actitud sera: 5) La mortalidad es muy elevada, aunque la reduccin
tenga lugar en las primeras 24 horas.
1) Cambiar su frmula por un hidrolizado de protenas de
vaca. 63. Nio de 4 meses es llevado a consulta por estreimiento
2) Hacer endoscopia para descartar esofagitis. crnico desde el nacimiento. Entre sus antecedentes perso-
3) Solicitar ecografa abdominal y/o trnsito digestivo. nales destacan tapn meconial al nacimiento que se resolvi
4) Realizar manometra esofgica. con enemas. Alimentacin materna exclusiva. Talla y peso

M exico A rgentina
C hile U ruguay
CTO Medicina C/ Nez de Balboa, 115 28006 MADRID (Espaa) Tfno.: (91) 782 43 32 / Fax: (91) 782 43 27
E-mail: secretaria@ctomedicina.com; iberocto@ctomedicina.com WEB: www.ctomedicina.com; www.iberocto.com
PD Pg. 7
PEDIATRA
Seguimiento a distancia Preparacin Examen de Seleccin 05/06 1 Vuelta
<P3. Cul de los siguientes datos NO apoyara el diagnos- dos hidroareos y eritema en regin del paal. En el examen
tico?: macroscpico de heces se detecta presencia de azcares
reductores. Este cuadro es:
1) Biopsia rectal patolgica.
2) Heces acintadas. 1) Dficit de sacarasa-isomaltasa.
3) Ampolla sin heces en el tacto rectal. 2) Dficit aislado de isomaltasa.
4) Relajacin del esfnter anal interno. 3) Dficit de lactasa.
5) Permanencia del bario en colon durante ms de 24 4) Dficit de enteroquinasa.
horas al realizar un trnsito digestivo. 5) Gastroenteritis por rotavirus.

64. Paciente de 12 meses que, de forma progresiva, inicia un 68. Nia de 2 aos acude a urgencias por cuadro de disnea,
cuadro de inapetencia, mal carcter, retraso del crecimien- estridor y tos perruna. En la exploracin destaca rinorrea
to ponderoestatural, as como diarrea abundante, ftida y intensa, amgdalas hipertrficas con exudado puntiforme.
de aspecto grasiento. Exploracin fsica: afebril, regular Fiebre de 38.5. A la auscultacin pulmonar se aprecia
estado general, palidez cutaneomucosa, prdida de masa hipoventilacin bilateral y estridor inspiratorio. El diagns-
muscular proximal y abdomen distendido. Seale lo FALSO tico ms probable ser:
respecto a la enfermedad celaca:
1) Traquetis bacteriana.
1) Existe una predisposicin gentica, relacionada con los 2) Laringitis supragltica.
HLA B8, DR7, DR3 y DQ2. 3) Laringitis aguda.
2) Hay una mayor prevalencia de la enfermedad en nios 4) Laringitis estridulosa.
con dficit selectivo de IgA o diabetes mellitus. 5) Epiglotitis.
3) El perodo ms comn de presentacin es entre los 6
meses y los 2 aos de edad. 69. Ante un nio de 2 aos de edad que, estando previamente
4) Los anticuerpos antiendomisio Ig A tienen una alta bien, se despierta bruscamente por la noche con tos perru-
especificidad y sensibilidad. na, estridor y dificultad respiratoria, pulso acelerado y
5) El trigo, arroz, cebada y centeno deben ser excluidos de afebril, pensaremos en:
la dieta para toda la vida.
1) Laringotraquetis aguda.
65. Paciente de 6 aos de edad que hace una semana present 2) Laringitis espasmdica.
cuadro compatible con varicela, acude hoy por presentar 3) Epiglotitis aguda.
vmitos incoercibles junto con clnica neurolgica de ob- 4) Difteria.
nubilacin, hiperventilacin e hiperreflexia. Seale cul de 5) Traquetis bacteriana.
las siguientes manifestaciones NO es compatible con este
cuadro: 70. Varn de 5 aos de edad que, estando previamente bien,
despierta por la noche con fiebre de 39,8C, voz apagada,
1) Aumento del amonio. intenso babeo, estridor inspiratorio con escasa tos. A la
2) Ictericia con hepatomegalia. exploracin destaca mal estado general, estridor inspirato-
3) Hipoglucemia. rio e intenso tiraje supraesternal y subcostal, as como
4) Convulsiones. tendencia a la hiperextensin del cuello. Se realiza radiogra-
5) Elevacin de las enzimas hepticas. fa lateral de faringe, observando ocupacin de espacio
supragltico. En el hemograma destaca una leucocitosis
66. Neonato varn, de tres semanas de vida, acude a su consulta con desviacin izquierda. Qu medida teraputica NO
por ictericia con coluria e hipocolia. A la exploracin se estara aconsejada en este caso?:
palpa hepatomegalia. En la gammagrafia con HIDA, la cap-

Preguntas TEST
tacin es normal, pero la excrecin es nula. Sobre la entidad 1) Aplicar oxgeno e intentar obtener una va area artificial
que sospecha, seale la opcin FALSA: en quirfano.
2) Iniciar tratamiento antibitico con ceftriaxona.
1) Se puede asociar a poliesplenia. 3) Aplicar adrenalina racmica y corticoides.
2) El tratamiento definitivo es la hepatoportoenterostoma. 4) Usar las medidas generales para descender la hiperter-
3) En la biopsia es posible hallar clulas gigantes. mia que sufre el paciente.
4) Es infrecuente la aparicin de varios casos en una misma 5) Mantener tranquilo al paciente.
familia.
5) En el sondaje duodenal se suele observar ausencia de 71. Lactante varn de 3 meses de edad que desde hace dos das
bilis. presenta cuadro de rinorrea serosa con estornudos y tos
seca acompaado de rechazo de las tomas. Hoy acude a
67. Paciente de 5 meses que acude por presentar, desde hace Urgencias por cuadro de dificultad respiratoria con ta-
2 das, hasta 10 deposiciones al da, lquidas sin moco ni quipnea y tos sibilante paroxstica. Exploracin fsica:
sangre, as como dolor medio abdominal sin relacin tem- 37,7C, regular estado general, aciantico, con aleteo
poral. Entre sus antecedentes destaca diarrea sanguinolen- nasal y tiraje sub e intercostal. Auscultacin pulmonar:
ta hace 1 semana. En la exploracin destaca: afebril, buen sibilancias espiratorias difusas con espiracin alargada. Se
estado general, distensin abdominal con abundantes rui- realiza hemograma con 3,5 millones de hemates; Hb de 10

M exico A rgentina
Pg. 8 PD C hile U ruguay
CTO Medicina C/ Nez de Balboa, 115 28006 MADRID (Espaa) Tfno.: (91) 782 43 32 / Fax: (91) 782 43 27
E-mail: secretaria@ctomedicina.com; iberocto@ctomedicina.com WEB: www.ctomedicina.com; www.iberocto.com
PEDIATRA
Preparacin Examen de Seleccin 05/06 1 Vuelta Seguimiento a distancia

g/dl; Hto de 32%; 7.000 leucocitos/mm3 con 50% neutr- 4) Puede ser til hacer profilaxis antibitica si se han
filos, 35% linfocitos y 5% monocitos. En la Rx de trax repetido varias infecciones urinarias.
destaca hiperinsuflacin pulmonar. Seale la FALSA res- 5) Si el reflujo es masivo, puede llevar a IRC en la infancia
pecto a la evolucin clnica de esta enfermedad: y puede requerir ciruga.

1) La fase ms crtica son las primeras 48-72 horas desde el 76. Varn de 2 aos en cuya historia clnica destaca haber
comienzo de la tos y la disnea. padecido un proceso diarreico en los das previos. En la
2) En menores de un mes puede cursar como apnea con exploracin destaca palidez, estupor y hematuria. Ha pre-
escasez de clnica respiratoria. sentado adems una convulsin focal. Respecto al cuadro
3) Despus de la fase crtica mejoran muy rpidamente. que usted sospecha, seale cul de las siguientes opciones
4) La ribavirina podra usarse en lactantes con cardiopatas es FALSA:
congnitas o displasia broncopulmonar.
5) Las complicaciones bacterianas como la bronconeu- 1) Es la causa ms frecuente de IRA en los nios pequeos.
mona u OMA son frecuentes. 2) Existe anemia hemoltica microangioptica y trombope-
nia dentro del cuadro clnico.
72. Paciente de 2 aos de edad consulta por estancamiento 3) La trombosis bilateral de las venas renales puede provo-
ponderoestatural (<P3). Entre los antecedentes personales car un cuadro similar.
destacan en el ltimo ao episodios repetidos de bronquio- 4) El empleo de corticoides supone el tratamiento de
litis con regular recuperacin posterior, persistiendo una eleccin.
tos crnica. Adems presenta diarrea intermitente sin acom- 5) La aparicin de recidivas es muy poco frecuente.
paarse de fiebre ni vmitos. Cul sera el diagnstico ms
probable?: 77. Varn de 5 aos presenta de forma sbita dolor en teste
derecho, sin antecedente de traumatismo previo. Explora-
1) Enfermedad celaca. cin: afebril, teste tumefacto, doloroso a la exploracin con
2) Tuberculosis. ausencia del reflejo cremastrico. Seale la afirmacin
3) Asma. FALSA:
4) Fibrosis qustica.
5) Dficit de alfa 1 antitripsina. 1) En el Eco-Doppler se observara disminucin del flujo
sanguneo en ese testculo.
73. Respecto a la criptorquidia, seale cul de las siguientes 2) El tratamiento consiste en antibiticos, antiinflamatorios
opciones NO es cierta: y tras la fase aguda, ciruga.
3) Es la causa ms frecuente de escroto agudo en los nios
1) La incidencia es inversamente proporcional a la edad menores de 6 aos.
gestacional. 4) En mayores de 13 aos se debe hacer diagnstico
2) La tasa de esterilidad es similar a la de la poblacin general diferencial con una epididimitis.
cuando se trata de una criptorquidia unilateral. 5) En el periodo neonatal generalmente tiene mal prons-
3) En los casos no tratados o tratados ms all de la pubertad, tico.
existe riesgo de desarrollar gonadoblastoma.
4) Se debe realizar un test de HCG en los casos bilaterales 78. Seale la opcin FALSA al hablar de las CIVs:
para descartar una anorquia.
5) El tratamiento hormonal no sustituye al quirrgico. 1) Es la cardiopata congnita ms frecuente en lactantes y
nios.
74. Lactante de 18 meses consulta por cuadro de vmitos y 2) Se suele auscultar un soplo pansistlico desde el naci-
febrcula de 48 horas de evolucin con progresiva prdida miento.
del apetito. En la exploracin no se objetiva foco infeccioso. 3) La Rx de trax puede mostrar signos de pltora pulmonar.
Preguntas TEST

Hemograma con leucocitosis y desviacin a la izquierda. 4) Puede precisar profilaxis antimicrobiana contra la en-
Sistemtico de orina: leucocitos y nitritos positivos. Cul docarditis.
sera la actitud ms adecuada?: 5) En la mayora de los casos se produce un cierre espon-
tneo del defecto.
1) Tratamiento con antitrmicos y ver evolucin.
2) Realizar puncin lumbar. 79. La persistencia del ductus arterioso se asocia a todo lo
3) Diagnstico de infeccin de orina y administrar antibi- siguiente, EXCEPTO a:
ticos v.o.
4) Realizar puncin suprapbica e iniciar antibiticos. 1) Pulsos perifricos saltones.
5) Descartar reflujo gastroesofgico. 2) Mayor frecuencia en prematuros.
3) Soplo continuo en el rea pulmonar.
75. Es FALSO respecto al reflujo vesicoureteral en el nio: 4) Puede ser efectivo el tratamiento con indometacina.
5) Las prostaglandinas son el mejor tratamiento mdico
1) Puede sospecharse intratero al ver dilatacin pieloca- para conseguir su cierre.
licial en la ecografa obsttrica.
2) Suele dar lugar a infecciones urinarias de repeticin. 80. Ante un recin nacido de dos das con cianosis, taquipnea
3) La indicacin quirrgica parte del nmero anual de sin soplo a la auscultacin y que, en la radiografa de trax,
recidivas de infeccin urinaria. muestra ligera cardiomegalia, con un pedculo cardaco

M exico A rgentina
C hile U ruguay
CTO Medicina C/ Nez de Balboa, 115 28006 MADRID (Espaa) Tfno.: (91) 782 43 32 / Fax: (91) 782 43 27
E-mail: secretaria@ctomedicina.com; iberocto@ctomedicina.com WEB: www.ctomedicina.com; www.iberocto.com
PD Pg. 9
PEDIATRA
Seguimiento a distancia Preparacin Examen de Seleccin 05/06 1 Vuelta
estrecho y aumento del flujo sanguneo pulmonar, el diag- 1) Su causa ms frecuente es la disgenesia tiroidea.
nstico ms probable es: 2) Situaciones de dismorfognesis pueden asociarse a sor-
dera neurosensorial.
1) Tetraloga de Fallot. 3) Un tratamiento precoz supone un mejor pronstico
2) Coartacin de aorta. neurolgico.
3) Anomala de Ebstein. 4) Los signos clnicos del hipotiroidismo estn ya presentes
4) Trasposicin simple de las grandes arterias. inmediatamente al nacimiento.
5) Atresia tricuspdea. 5) En situaciones de mal control pueden acabar desenca-
denando una pubertad precoz.
81. Los nios con enfermedad de Ebstein presentan todo lo
siguiente, EXCEPTO: 86. Un nio de 7 das es ingresado para evaluacin de cuadro
de vmitos y deshidratacin. La exploracin fsica es nor-
1) Desplazamiento inferior de la vlvula tricspide anormal. mal. La concentracin de Na y K en plasma es 120 mEq/l y
2) En el electrocardiograma puede aparecer un patrn de 9 mEq/L respectivamente. El diagnstico ms probable ante
Wolff-Parkinson-White. este cuadro es:
3) Soplo sistlico y diastlico.
4) Aurcula derecha pequea y ventrculo derecho muy 1) Estenosis pilrica.
hipertrofiado. 2) Sndrome adrenogenital congnito pierde sal.
5) Valva tricuspdea anterior agrandada y abombada. 3) Hipotiroidismo congnito.
4) Panhipopituitarismo.
82. Con respecto a la coartacin de aorta, seale la FALSA: 5) Hiperaldosteronismo.

1) El tipo ms frecuente es la coartacin de localizacin 87. Paciente que inicia desarrollo de los caracteres sexuales
yuxtaductal. secundarios a una edad inferior a la considerada normal.
2) La mayor parte de los casos se muestran asintomticos. Con respecto a esta entidad, es FALSO:
3) La tensin arterial en los miembros inferiores es mayor
que en los superiores. 1) Si fuera varn, la causa ms probable de pubertad
4) En ocasiones, la tensin arterial en el brazo derecho es precoz central sera un hamartoma hipotalmico.
mayor que en el izquierdo. 2) La aparicin de los caracteres sexuales secundarios, sin
5) En la radiografa de trax, con el tiempo, se visualizan activacin gonadal previa, permite diagnosticar una
escotaduras en los bordes inferiores de las costillas. pubertad precoz perifrica.
3) En mujeres, la causa ms frecuente de pubertad precoz
83. Ante un recin nacido ciantico con pulsos normales, soplo central son las lesiones del SNC.
sistlico en borde esternal izquierdo, radiografa que mues- 4) Si fuera un varn con tamao testicular inferior a 4 cc,
tra un tamao cardaco normal con elevacin de la punta habra que pensar, entre otras causas, en un hepatoblas-
cardaca y disminucin de la vascularizacin pulmonar, y toma.
un electrocardiograma con signos de hipertrofia del ventr- 5) Si fuera una nia con lesiones cutneas hiperpigmenta-
culo derecho, pensara en: das y lesiones seas, habra que pensar en un sndrome
de McCune-Albright.
1) TGA.
2) Fallot. 88. La estatura de un nio de 4 aos est por debajo del P3. Su
3) DAP. velocidad de crecimiento es normal. La edad sea es de 2
4) Sndrome de Eisenmenger. aos. El padre present un retraso en la pubertad, aunque
5) Anomala de Ebstein. actualmente su talla est dentro de los lmites normales. Este
proceso se denomina:

Preguntas TEST
84. La comunicacin interauricular es la cardiopata congnita
ms frecuente a partir de los 15 aos. Sus distintos tipos se 1) Hipopituitarismo primario.
caracterizan por lo siguiente, EXCEPTO: 2) Hipopituitarismo secundario.
3) Retraso constitucional del crecimiento.
1) El agujero oval persistente no es una forma de CIA. 4) Talla baja gentica.
2) La CIA tipo ostium secundum produce un segundo ruido 5) Sndrome de Laron.
ampliamente desdoblado y que se modifica con la
respiracin. 89. Varn de 4 aos, y con antecedentes personales de haber
3) Es frecuente que asocie retraso de la conduccin ventri- nacido con retraso de crecimiento intrauterino, que actual-
cular derecha. mente, y en el contexto de un cuadro diarreico, presenta
4) La endocarditis infecciosa es extremadamente rara, por valores de glucemia plasmtica de 30 mg/dl, con intensa
lo que no es necesario realizar profilaxis. reaccin a cuerpos cetsicos en orina. La respuesta correc-
5) El canal A-V comn suele evolucionar a ICC y requiere ta es:
un tratamiento quirrgico precoz.
1) Por la reaccin de cuerpos cetnicos, el diagnstico ms
85. Recin nacida de 3 semanas de vida, que presenta valores de probable es un hiperinsulinismo.
TSH de 75 mcgr/ml ( 0,3 5 ) en el screening neonatal de 2) El ayuno prolongado puede desencadenar esta situacin.
hipotiroidismo. Con respecto a esta situacin, es FALSO:

M exico A rgentina
Pg. 10 PD C hile U ruguay
CTO Medicina C/ Nez de Balboa, 115 28006 MADRID (Espaa) Tfno.: (91) 782 43 32 / Fax: (91) 782 43 27
E-mail: secretaria@ctomedicina.com; iberocto@ctomedicina.com WEB: www.ctomedicina.com; www.iberocto.com
PEDIATRA
Preparacin Examen de Seleccin 05/06 1 Vuelta Seguimiento a distancia

3) Los valores de alanina en plasma estn elevados. 95. La forma de epilepsia mas frecuente en la infancia es:
4) En la RM pancretica se demostrar la existencia de una
lesin focal. 1) Crisis de ausencia.
5) Este cuadro empeora con la edad. 2) Epilepsia mioclnica juvenil.
3) Epilepsia rolndica.
90. Respecto a las convulsiones neonatales, seale la afirma- 4) Sd. de West.
cin FALSA: 5) Sd. de Lennox- Gastaut.

1) Los RN con convulsiones suelen tener mal pronstico. 96. Todo lo siguiente es cierto al hablar del astrocitoma del
2) El tratamiento consiste en medidas de sostn y fenobar- cerebelo, EXCEPTO:
bital o diacepam.
3) Las crisis tnico-clnicas generalizadas son raras durante 1) Su pico de incidencia est entre los 5 y 8 aos de edad.
el primer mes de vida. 2) Suelen ser tumores slidos con escasa cavitacin.
4) La clnica es variable y el EEG puede ser la nica forma 3) Hay clnica de hipertensin intracraneal.
para su diagnstico. 4) Es el tumor de fosa posterior ms frecuente en la niez.
5) La causa ms frecuente de crisis en el perodo neonatal 5) La supervivencia a los 5 aos es del 90%.
es la encefalopata hipxico-isqumica.
97. Cul es el tipo de cncer ms frecuente en la niez?:
91. Un varn de 7 aos tiene mal rendimiento escolar. Su maestro
comunica que a veces parece preocupado, presenta episo- 1) Tumores del SNC.
dios de parpadeo y, ocasionalmente, tarda en responder o sus 2) Osteosarcoma.
respuestas son incorrectas. El examen fsico es normal. El 3) Leucemias.
estudio que ms ayuda al diagnstico es: 4) Tumor de Wilms.
5) Neuroblastoma.
1) Determinacin del nivel madurativo.
2) EEG. 98. Nia de 2 aos presenta movimientos mioclnicos de
3) Calcemia. extremidades y sacudidas desordenadas de los ojos. En la
4) Glucemia. exploracin se palpa masa abdominal en lnea media y
5) Examen visual. flanco derecho. Se le realiza una TC abdominal, evidencin-
dose una masa de consistencia mixta y calcificaciones en su
92. Seale la respuesta FALSA, de entre las siguientes, con interior. Seale la afirmacin FALSA:
respecto a los espasmos infantiles:
1) Es el tumor extracraneal ms frecuente en la infancia.
1) Se trata de una epilepsia generalizada secundaria, que 2) La edad ms frecuente al diagnstico son los 2 aos.
suele comenzar a los 4 - 8 meses de edad. 3) La presentacin con un sndrome de mioclonus- opso-
2) Suele clasificarse en idiopticos, criptognicos y secun- clonus ensombrece el pronstico.
darios. 4) Las catecolaminas en orina estarn elevadas.
3) En el EEG aparece un patrn de ondas lentas hipervol- 5) Se asocia con delecin del cromosoma 1 y anomalas
tadas, bilaterales y asincrnicas. del 17.
4) El tratamiento de eleccin es ACTH o prednisona.
5) Los casos idiopticos tienen peor pronstico que los 99. Seale la opcin FALSA respecto al tumor de Wilms:
secundarios.
1) Se asocia a delecin del cromosoma 11, ya sea en las
93. Llega a Urgencias un nio de 18 meses con fiebre de 39, que clulas tumorales o en todas las clulas del organismo.
ha presentado en su domicilio un episodio tnico-clnico 2) Se asocia a aniridia y anomalas genitourinarias.
3) La HTA es un hallazgo frecuente.
Preguntas TEST

generalizado de 3 minutos de duracin, con posterior


somnolencia. Lo correcto a su llegada a Urgencias sera: 4) El diagnstico definitivo nos lo da la biopsia.
5) Suele tratarse de una masa abdominal palpable asinto-
1) Administracin de bolo de glucosa. mtica, con o sin metstasis pulmonares.
2) Administracin de antitrmicos y diacepam.
3) Administracin de fenitona. 100. Respecto al retinoblastoma, seale la respuesta FALSA:
4) Realizar puncin lumbar.
5) Administracin de piridoxina. 1) Las metstasis o recidivas pueden asociarse a elevaciones
de AFP y CEA.
94. Seale la opcin FALSA respecto al sndrome de Lennox- 2) Las formas bilaterales suelen diagnosticarse a una edad
Gastaut: ms precoz que las unilaterales.
3) Si el tumor es bilateral debe procederse siempre a la
1) Suele asociar varios tipos de crisis, entre ellos, tnicas, enucleacin.
atnicas y ausencias atpicas. 4) La supervivencia global es de aproximadamente un
2) Son frecuentes los episodios de status epilptico. 90%, sobre todo en las formas intraoculares.
3) Las crisis responden bien al tratamiento farmacolgico. 5) Rara vez existen metstasis en el momento del diagns-
4) En la mayora de los casos es un proceso secundario. tico.
5) Suele asociar retraso psicomotor.

M exico A rgentina
C hile U ruguay
CTO Medicina C/ Nez de Balboa, 115 28006 MADRID (Espaa) Tfno.: (91) 782 43 32 / Fax: (91) 782 43 27
E-mail: secretaria@ctomedicina.com; iberocto@ctomedicina.com WEB: www.ctomedicina.com; www.iberocto.com
PD Pg. 11
PEDIATRA
Preparacin Examen de Seleccin 05/06 1 Vuelta Seguimiento a distancia
Pregunta 1.- R: 3 cir ictericia precoz prolongada, sin otra repercusin clnica. Recordar
El test de Apgar es un test consensuado para documentar el estado para el MIR el cuadro clnico tpico de la hemorragia suprarrenal:
del RN en momentos puntuales. Lo ms frecuente es que en un RN nio grande, parto de nalgas, que en las primeras horas de vida pre-
sano se mida al minuto de vida y a los 5 minutos, pero si a los 5 senta clnica de hipotensin, sangrado y shock.
minutos la puntuacin es inferior a 7 se contina midiendo a los 10- Respuesta 5: las fracturas craneales lineales generalmente no preci-
15 y 20 minutos. Se debe realizar a cualquier RN. san tratamiento quirrgico, salvo que se traten de fracturas deprimidas
Es importante conocer los parmetros que se puntan y su valora- que produzcan lesiones en el parnquima.
cin.
Este test no sirve para valorar el riesgo de mortalidad perinatal ni la
probabilidad de dao neurolgico.

Pregunta 1. Test de Apgar.

3DUiPHWURV   
(VIXHU]R
UHVSLUDWRULR
$XVHQWH 'pELO /ODQWR

)& $XVHQWH OSP !OSP


7RQRPXVFXODU +LSRWRQtD /LJHUDIOH[LyQ 0RYLPLHQWRVDFWLYRV

,UULWDELOLGDGUHIOHMD 1R 0XHFDV 7RV


&RORU 3DOLGH] $FURFLDQRVLV 5RVDGR

Pregunta 2.- R: 1
Para valorar la dificultad respiratoria en los RN se emplea el score
de Silverman.

Pregunta 2. Test de Silverman

9$/25$&,1   
'LVRFLDFLyQ 7yUD[ILMRVHPXHYH 5HVSLUDFLyQHQ
WyUD[ 1RUPDO HODEGRPHQ EDODQFHR
DEGRPHQ

7LUDMH $XVHQWH ,QWHUFRVWDO ,QWHUFRVWDOVXSUDH


LQIUDHVWHUQDO
5HWUDFFLyQ $XVHQWH 'LVFUHWD ,QWHQVD
[LIRLGHD
$OHWHRQDVDO $XVHQWH 'LVFUHWR ,QWHQVR
Comentarios TEST

4XHMLGR
UHVSLUDWRULR $XVHQWH 6HR\HFRQIRQHQGR 6HR\HDGLVWDQFLD
6LQIRQHQGR

Pregunta 3.- R: 4
La causa ms frecuente de sndrome febril en el lactante es infec-
ciosa y fundamentalmente vrica.

Pregunta 4.- R: 3
La respuesta 1 es cierta: es importante para el MIR tener claro las
diferencias entre caput (edema de partes blandas, no limitado por
suturas) y cefalohematoma (hematoma entre hueso y periostio, limita-
da por suturas).
La respuesta 2 es cierta: el traumatismo obsttrico ms frecuente es
la fractura de clavcula. A la exploracin lo ms llamativo es la crepi-
tacin y el reflejo de Moro asimtrico.
La respuesta 3 es falsa: el llamado ndulo de Stromayer es un
hematoma palpable a nivel del ECM. La mayora de las veces se Pregunta 4. Cefalohematoma y caput succedaneum.
reabsorbe espontneamente, pero en ocasiones se puede fibrosar
acortando el msculo. El tratamiento se basa en la fisioterapia y muy Pregunta 5.- R: 3
rara vez requiere ciruga. Estamos ante un cuadro de respiracin peridica del RN. Lo ms
La respuesta 4 es cierta: la vscera que con mayor frecuencia se caracterstico son pequeos episodios de apnea que NO se acompa-
lesiona es el hgado, pero la mayora de las veces son pequeos hema- an de cianosis ni de bradicardia. Esto ltimo es el dato ms impor-
tomas subcapsulares limitados que secundariamente pueden produ- tante para diferenciarlo de las pausas de apnea idiopticas.

M exico A rgentina
C hile U ruguay
CTO Medicina C/ Nez de Balboa, 115 28006 MADRID (Espaa) Tfno.: (91) 782 43 32 / Fax: (91) 782 43 27
E-mail: secretaria@ctomedicina.com; iberocto@ctomedicina.com WEB: www.ctomedicina.com; www.iberocto.com
PD Pg. 1
PEDIATRA
Seguimiento a distancia Preparacin Examen de Seleccin 05/06 1 Vuelta
Generalmente no precisa tratamiento, a diferencia de las apneas Administracin de antibiticos: ampicilina ms gentamicina, por-
idiopticas, que suelen requerir la administracin de cafena o teofili- que la clnica y la Rx pueden ser semejantes en el caso de una
na y el uso de CPAP. sepsis con neumona.
Ambas se solucionan conforme el nio va creciendo y alcanza las
36 semanas de edad postconcepcional. La respuesta incorrecta es la 4. Las prostaglandinas en los neonatos
estn indicadas en caso de cardiopatas ductus dependientes, en las
que es IMPRESCINDIBLE mantener el ductus abierto.
Pregunta 5. Diagnstico diferencial de apneas en el neonato.
Pregunta 9.- R: 2
$SQHDLGLRSiWLFD 5HVSLUDFLyQSHULyGLFD El caso clnico es el de un nio que ha sufrido un SAM. Lo tpico es
que se produzcan en RN postrmino, con un estrs en el momento
(GDG 5137 5137\517 del parto, que estimula el peristaltismo intestinal y se produce la elimi-
$SQHDVVHJ $SQHDVVHJVHJXLGDV nacin de meconio intratero.
GHUHVSLUDFLRQHVUiSLGDV Al nacimiento, el meconio se encuentra en las vas areas altas; si
GXUDQWHVHJ no se elimina a tiempo, este pasar al pulmn con las primeras respi-
&OtQLFD &LDQRVLVEUDGLFDUGLD 12FLDQRVLVQLEUDGLFDUGLD raciones, produciendo un cuadro de alveolitis con riesgo de sobrein-
$XPHQWDGXUDQWHHO $XPHQWDQGXUDQWHODV feccin bacteriana, especialmente por E. coli, atelectasias y atrapa-
VXHxR5(0 IDVHVGHOVXHxR miento areo con riesgo de neumotrax y neumomediastino.
En los casos ms graves se asocia con una alta mortalidad debido a
'HVDSDUHFHDODV 'HVDSDUHFHDODVVHP la hipertensin pulmonar que desarrollan.
3URQyVWLFR VHPSRVWFRQFHSFLRQDO
1RULHVJRGHPXHUWH
SRVWFRQFHSFLRQDO
1RULHVJRGHPXHUWH
El tratamiento consiste en ventilacin mecnica, antibiticos y
medidas para disminuir la hipertensin pulmonar: alcalinizar, tolazo-
V~ELWD V~ELWD lina y oxido ntrico inhalado. Algunos nios se pueden beneficiar de
la ECMO (oxigenacin por membrana extracorprea).
Pregunta 6.- R: 2
El cuadro clnico se refiere a una taquipnea transitoria del RN, Sd Pregunta 10.- R: 4
de Avery, pulmn hmedo o SDR tipo II. Estamos ante un nio con antecedentes de haber estado sometido
La presentacin tpica es un RN nacido por cesrea o por parto a altas presiones respiratorias para ventilarles durante la reanimacin,
vaginal rpido, lo que supone que el trax no ha estado sometido a la producindole una hiperinsuflacin pulmonar. Esto es un factor de
presin positiva que exprime de lquido los alveolos del pulmn. La riesgo claro para el desarrollo de neumotrax. Otras causas que pue-
clnica se inicia en las primeras horas de vida, que cede y mejora con den favorecerlo es patologa pulmonar que disminuya la compliance:
medidas poco agresivas (oxgeno en incubadora). Suelen alcanzar la EMH, SAM, hipoplasia pulmonar,...
resolucin clnica en un plazo mximo de 3 das. La clnica es tpica: disminucin del murmullo vesicular con
Lo que establece el diagnstico, aparte de la evolucin clnica, es hiperresonancia. El diagnstico lo da la Rx de trax donde se aprecia
la Rx de trax, donde se aprecia aumento de marcas vasculares, lqui- el pulmn colapsado.
do en las cisuras, SIN broncograma areo. Esto ltimo es imprescindi- El tratamiento, en caso de que el nio estuviera asintomtico, sera
ble para distinguirlo de la Rx de la EMH (R1). administracin de oxgeno al 100%. Si produjera clnica o estuviera a
R3: El SAM es tpico de RN postrmino, con antecedentes de estrs tensin, habra que evacuarlo mediante puncin.
en el momento del parto. Entre las opciones del caso clnico, ninguna plantea realmente
R4: La PCF suele cursar con clnica marcada de cianosis que no problemas:
responde a la administracin de oxgeno. No hay una Rx de trax R1: la sepsis en un RN tiene una clnica y analtica tpicas, y gene-
caracterstica. ralmente la afectacin pulmonar es bilateral.
R5: el Sd de Wilson-Mikity es a efectos clnicos semejante al trmi- R2: en el Sd del corazn izquierdo hipoplsico, lo tpico es un RN
no de DBP. con distrs severo, auscultacin pulmonar normal y cianosis inten-
sa.
Pregunta 7.- R: 2 R3: el neumomediastino suele dar tonos cardacos apagados y
Estamos ante un cuadro tpico de enfermedad de membranas hia- auscultacin pulmonar normal.

Comentarios TEST
linas: RNPT con dificultad respiratoria intensa que aparece en las pri- R5: en las fstulas T-E, la clnica es dificultad respiratoria intensa al
meras horas de vida, que no mejora tras la administracin de oxgeno. iniciar la alimentacin por episodios de aspiracin masiva.
En la gasometra es llamativa la importante hipoxia. Para el diagnstico
es necesario realizar una Rx de trax, y en ocasiones, la medicin del Pregunta 11.- R: 5
cociente lecitina/ esfingomielina en secreciones bronquiales, que debe Estamos ante un RN con displasia broncopulmonar. La definicin
ser inferior a 2. ya aparece en el caso clnico: necesidad de oxgeno para mantener
No existe una imagen radiolgica patognomnica. Lo ms tpico es adecuadas saturaciones. Dentro de los factores de riesgo para este
encontrar un infiltrado retculo granular con broncograma areo. Pero cuadro figuran: la toxicidad del oxgeno en altas concentraciones,
esto mismo nos lo podemos encontrar en una neumona, por lo que inmadurez pulmonar y barotrauma.
a la hora de instaurar tratamiento debemos cubrir esta posibilidad. El diagnstico se establece por la historia clnica y la Rx de trax,
La R3 corresponde al pulmn hmedo. donde es tpico encontrar el llamado patrn de esponja (aparecen
La R4: El patrn de esponja en la Rx es tpico de la DBP. reas ms claras que alternan con otras de mayor densidad).
La R5: corresponde a un neumotrax bilateral. El tratamiento se basa en el uso de diurticos y broncodilatadores;
el uso de corticoides actualmente est muy debatido.
Pregunta 8.- R: 4 R5: la mayora de los nios suelen tener un curso favorable, alcan-
El tratamiento de la EMH consiste en: zando la normalidad de la funcin pulmonar hacia los 2 aos de
Monitorizacin estrecha, control de lquidos. vida. Hay un pequeo porcentaje que evolucionan hacia un cuadro
Intentar mantener una aceptable oxigenacin (pO2 50-70 mmHg). de hipertensin pulmonar persistente. Las dos causas ms frecuentes
Si no se consigue con oxgeno indirecto, ser necesario intubar al de muerte en estos nios son la ICC derecha y la bronquiolitis necro-
paciente e iniciar ventilacin mecnica. tizante.
Administracin de surfactante intratraqueal en las primeras 24 ho-
ras de vida, pudiendo recibir hasta 4 dosis segn evolucin. Esta Pregunta 12.- R: 4
medida se ha visto que mejora la clnica de la EMH, pero que no El diagnostico ms compatible es la hernia diafragmtica. Los dos
disminuye los casos de DBP. datos claves para el diagnstico son:

M exico A rgentina
Pg. 2 PD C hile U ruguay
CTO Medicina C/ Nez de Balboa, 115 28006 MADRID (Espaa) Tfno.: (91) 782 43 32 / Fax: (91) 782 43 27
E-mail: secretaria@ctomedicina.com; iberocto@ctomedicina.com WEB: www.ctomedicina.com; www.iberocto.com
PEDIATRA
Preparacin Examen de Seleccin 05/06 1 Vuelta Seguimiento a distancia
Dificultad respiratoria inmediata.
Abdomen excavado. Pregunta 16. Causas de ictericia neonatal.

El diagnstico definitivo lo obtendremos mediante la Rx de trax 1as 24 horas Hemlisis.


donde se aprecia en el hemitrax izquierdo imgenes de asas intesti- Infecciones: sepsis, TORCH.
nales con desplazamiento del mediastino hacia el lado contralateral. 2-3er da Fisiolgica.
Lo ms decisivo en el tratamiento de este nio va a ser la estabiliza- Infecciones: sepsis, TORCH.
cin de la funcin respiratoria, ya que en las mayora de las ocasiones Anemias hemolticas.
se asocia con una severa hipoplasia pulmonar. Una vez que el pacien- 4-7 da Sepsis.
te est estabilizado respiratoriamente, se realizar la intervencin qui- TORCH.
rrgica consistente en descender las asas intestinales y poner un par- Obstruccin intestinal.
che en el hemidiafragma. Lactancia materna.
>1 mes Galactosemia, hipotiroidismo, lactancia materna,
Pregunta 13.- R: 3 metabolopatas, ictericia obstructiva, Gilbert,
Dentro de los factores de riesgo para el desarrollo de retinopata en Crigler-Najjar...
los grandes prematuros (peso < 1500 g) figuran: el oxgeno en altas
concentraciones, variaciones en la TA. La vitamina E parece tener un Caractersticas de la ictericia fisiolgica vs no fisiolgica:
factor protector (R4) por su papel antioxidante. - Inicio en las primeras 24 horas: SIEMPRE patolgico.
En las UCIs neonatales se ha establecido de forma rutinaria estu- - Predominio de bilirrubina directa: SIEMPRE patolgico.
dios oftalmolgicos en todos los nios menores de 1500 grs. Si se
aprecian zonas de proliferacin de vasos con riesgo de desprendi- Otros datos de la no fisiolgica:
miento de retina, se realizar laserterapia o crioterapia. - Incremento mayor de 5 mg/24 horas.
El pronstico para la mayora de los casos es bueno (R3 falsa). Las - Bilirrubina en sangre de cordn mayor de 3 mg/dl.
secuelas suelen ser pequeos defectos de la refraccin: miopa o - RNT > 12 y RNPT > 14.
astigmatismo, o en ocasiones nistagmus. La displasia retrolental es una - Duracin mayor de 14 das.
complicacin rara y poco frecuente, pero muy grave, puesto que
puede dejar al nio ciego. Pregunta 17.- R: 3
La ictericia por lactancia materna (LM) es la causa ms frecuente de
Pregunta 14.- R: 4 ictericia tarda. Su frecuencia es de aproximadamente 1/200 (R1).
Estamos ante un RN con un cuadro de enterocolitis necrotizante Se debe a que en la leche materna existen unas sustancias
(NEC). La presentacin tpica es en un RNPT que a la semana inicia (pregnanodiol, cidos grasos de cadena larga) que inhiben la glucoronil
cuadro de vmitos, distensin abdominal y deposiciones con sangre. transferasa, dando como resultado un aumento de la bilirrubina a
El primer diagnstico que nos debemos plantear es una NEC. expensas de fraccin indirecta (R4).
Dentro de los factores de riesgo figuran, aparte de la prematuridad, En ocasiones, cuando existen dudas diagnsticas, se puede interrum-
la administracin precoz y en altas concentraciones de leche y facto- pir la LM transitoriamente durante unos das, observndose un claro
res infecciosos. descenso en la bilirrubina, pero sta no es indicacin de suspender defi-
La prueba diagnstica indicada sera una Rx de abdomen donde nitivamente la LM, puesto que no se han descrito casos de kernicterus
se apreciara el dato ms caracterstico, la neumatosis intestinal. (R3).
El tratamiento se debe instaurar ante la mnima sospecha, puesto
que de eso va a depender el pronstico. Pregunta 18.- R: 5
Si el nio no tiene signos de perforacin intestinal, se dejar a dieta Para el MIR es importante saberse bien las diferencias entre la
absoluta durante semanas, con antibioterapia. Si en algn momento incompatibilidad Rh y ABO:
de la evolucin se produjera una perforacin intestinal o sepsis refrac-
taria a tratamiento antibitico, sera necesario realizar tratamiento
quirrgico. Pregunta 18. Diferencias entre incompatibilidad Rh y AB0.
Las complicaciones a largo plazo son la estenosis intestinales y el
Comentarios TEST

sndrome de intestino corto. 5K $%


Pregunta 15.- R: 3 0DGUH5K +LMR5K 
 

El cuadro clnico se refiere a un leo meconial. La forma de presen- 1HFHVDULDLQPXQL]DFLyQSUHYLD


/DLQFRPSDWLELOLGDGGHJUXSR 0DGUH+LMR$
tacin ms habitual en los neonatos es como un cuadro de obstruc- 3XHGHSURGXFLUVHHQHOSULPHU
cin intestinal. El dato que apoyara este diagnostico sera palpar unos SURWHJHIUHQWHDOD HPEDUD]R
cordones duros que siguen el marco clico, puesto que el lugar don- LQFRPSDWLELOLGDG5K 1RVHDJUDYDFRQORVVLJXLHQWHV
de con mayor frecuencia se impacta el meconio es la zona de leon 6HDJUDYDHQORVVLJXLHQWHV
distal y coln proximal. HPEDUD]RV
Una vez establecido el diagnstico es siempre obligatorio des-
cartar una fibrosis qustica (un 15% debutan como leo meconial) ,FWHULFLD ,FWHULFLD
con la realizacin de un tripsingeno srico, que estar aumenta- $QHPLD $QHPLD
do. +LGURSVIHWDOHQFDVRVJUDYHV 1RKLGURSV
El tratamiento, si el nio no tiene signos de perforacin intestinal, 3UHQDWDO&,DODPDGUHGXUDQWH 3UHQDWDOQRWLHQHQXWLOLGDG
se realiza mediante la administracin de enemas hiperosmolares. Si 7\7 3RVWDQDWDO&' GpELO&, FRQ
 

con esta medida no se soluciona el problema, sera necesario recurrir 3RVWQDWDO&' 
HVIHURFLWRVLV
a la ciruga, realizando una reseccin de la zona de impactacin y
anastomosis termino-terminal. 7UDWDPLHQWRIRWRWHUDSLD ,GHP
H[DQJXLQRWUDQVIXVLyQ
Pregunta 16.- R: 3 3UHYHQFLyQ*DPPDHQ7\HQODV
Respecto a las ictericias neonatales, es importante tener claro dos SULPHUDVKRUDVWUDVHOSDUWRVLOD 1RWLHQH
aspectos. PDGUHWLHQHXQWHVWGH&,QHJDWLYR
Las causas ms frecuentes segn el momento de aparicin apare-
cen reflejadas en la siguiente tabla:

M exico A rgentina
C hile U ruguay
CTO Medicina C/ Nez de Balboa, 115 28006 MADRID (Espaa) Tfno.: (91) 782 43 32 / Fax: (91) 782 43 27
E-mail: secretaria@ctomedicina.com; iberocto@ctomedicina.com WEB: www.ctomedicina.com; www.iberocto.com
PD Pg. 3
PEDIATRA
Seguimiento a distancia Preparacin Examen de Seleccin 05/06 1 Vuelta
Pregunta 19.- R: 4 na beta de la hemoglobina se empieza a sintetizar a partir de los 6
El caso clnico hace referencia a una enfermedad hemorrgica del meses.
RN. La causa es un dficit de vitamina K, debido a que durante el
embarazo el paso transplacentario de vitamina es escaso, la LM es Pregunta 21.- R: 3
pobre en vitamina K (R4) y en el intestino no hay bacterias que sinte- Por la clnica de temblores, letargia y rechazo de las tomas, podra
ticen vitamina. Todas estas circunstancias obligan a administrar vitami- corresponder a cualquiera de las opciones. Pero los datos de acrocia-
na K (1 mg im ) a todos los RN en las primeras 24 horas de vida. En nosis e ictericia a los 2 das hablan a favor de la policitemia (el hijo de
algunas ocasiones va a ser necesario ms de una dosis. madre diabtica tiene riesgo de tener policitemia).
La clnica consiste en sangrado a distintos niveles: umbilical, diges- Se define policitemia como Hto venoso mayor o igual a 65%. El
tivo, nasal,... Hto capilar es por definicin un 5-20% mayor que el venoso.
El tratamiento consiste en la administracin de nuevas dosis de Las consecuencias clnicas derivan de la hiperviscosidad de la san-
vitamina K o plasma fresco congelado. gre: distrs respiratorio, riesgo de NEC, trombosis de la vena renal,
Es importante recordar que los hijos de madres que durante el disminucin de flujo a nivel del SNC,... y tambin pueden presentar
embarazo han tomado fenitona o fenobarbital tienen ms riesgo de hipoglucemia (el sustrato metablico de los glbulos rojos es la gluco-
padecer este trastorno. sa; al haber muchos GR, consumen mucha glucosa).
El tratamiento consiste en la exanguinotrasfusin parcial, que se
Pregunta 20.- R: 1 realiza extrayendo un volumen de sangre y reponindolo con SSF,
La llamada anemia fisiolgica se debe a un dficit transitorio de para que disminuya la viscosidad.
EPO, que unido a la hemlisis de glbulos rojos durante los primeros
meses de vida, dejan al nio anmico. Pregunta 22.- R: 4
Las cifras ms bajas se producen a los 2 meses en los RNPT y a los Recordamos brevemente las diferencias entre los hijos de madre
3 en los RNT (R1). adictas a herona y las adictas a metadona (ver tabla).
El tratamiento consiste en suplementos de hierro a partir de los 2 Estos nios estn sometidos a un estrs crnico intratero, por lo
meses en los RNPT, ya que antes los depsitos estn llenos por la que es menos frecuente la hiperbilirrubinemia y el desarrollo de
hemlisis de los glbulos rojos, y en casos seleccionados sera necesa- EMH.
rio la trasfusin de concentrados de hemates. Durante la reanimacin neonatal estos nios no deben recibir
En los neonatos no se manifiesta la betatalasemia, porque la cade- naloxona por el riesgo de desarrollar un Sd. de abstinencia brusco.

Pregunta 24. Caractersticas de la sepsis neonatal.

Caractersticas. Comienzo precoz. Comienzo tardo. Nosocomial.

Comienzo. Nacimiento; menos de 7 das 8-28 das, en ocasiones hasta De la primera semana al alta
normalmente menos de 3 das. 60 das. Hospitalaria.

Riesgos obsttricos. Colonizacin, amnionitis, Poco frecuentes. Premadurez; ingreso en UCI


premadurez. neonatales, reseccin intestinal.

Presentacin. Dificultad respiratoria, neumona Fiebre, signos


1 nerviosos centrales Apnea, bradicardia, letargia,
Shock. o focales. inestabilidad trmica.

Meningitis. 20%. 75%. 10-20%.

Otros sistemas. Raro. Pielonefritis, osteomielitis, artritis Neumona, pielonefritis, endoftalmitis,


sptica, celulitis. trombos spticos, flebitis, infecciones

Comentarios TEST
cutneas, sepsis de lneas centrales,
enterocolitis necrotizante.

Microorganismos
patgenos. Estreptococo del grupo B tipos Estreptococo del grupo B tipo III, Staphylococcus epidermidis, S. aureus,patgenos.
Ia, Ib, Ia/c, II, III, E. coli, E. coli Ag K L. monocytogenes, Candida albicans, Pseudomonas
Klebsiella, Listeria monocytogenes, herpes simple. aeruginosa, E. coli, herpes simple,
enterococos, Haemophilus Klebsiella, Serratia.
influenzae no tipables,
S. pneumoniae.

Tratamiento. Ampicilina y gentamicina o Ampicilina y gentamicina o Depende de los agentes nosocomiales


cefotaxima. cefotaxima. Presentes en la sala de neonatologa;
cefotaxima, vancomicina y gentamicina.

Medidas de apoyo. Ventilacin mecnica, frmacos Ventilacin mecnica, frmacos Ventilacin mecnica, frmacos
vasoactivos, reposicin vasoactivos, reposicin vasoactivos, reposicin
hidroelectroltica, ECMO. hidroelectroltica. hidroelectroltica.

Mortalidad. 15-70%. 10-20%. 5-10%.

M exico A rgentina
Pg. 4 PD C hile U ruguay
CTO Medicina C/ Nez de Balboa, 115 28006 MADRID (Espaa) Tfno.: (91) 782 43 32 / Fax: (91) 782 43 27
E-mail: secretaria@ctomedicina.com; iberocto@ctomedicina.com WEB: www.ctomedicina.com; www.iberocto.com
PEDIATRA
Preparacin Examen de Seleccin 05/06 1 Vuelta Seguimiento a distancia
Alteraciones metablicas:
Pregunta 22. Diferencias entre hijos de madres adictas a herona y a - Hipoglucemia: ms frecuente en las primeras horas. Es necesa-
metadona. rio un control (venoso o capilar) pues la mayora de las veces es
asintomtico. El tratamiento consiste en el aporte de glucosa iv
+HURtQD 0HWDGRQD mediante perfusin continua, evitando los bolos por el riesgo
de hiperglucemia de rebote.
0iVULHVJRGHDERUWRV\&,5 ,GHP - Hipocalcemia.
1RGHDQRPDOtDVFRQJpQLWDV
Pregunta 24.- R: 3
3HRUFRQWUROGHHPEDUD]R (PEDUD]RPHMRUFRQWURODGR Mediante el esquema de la parte inferior de la pgina recordamos
5LHVJRGHSROLGURJDGLFFLyQ las diferencias entre sepsis precoz y tarda
6tQGURPHGHDEVWLQHQFLD
WHPEORUHVLUUULWDELOLGDGGLDUUHD $SDUHFHPiVWDUGH\GXUDPiVWLHPSR Pregunta 25.- R: 5
YyPLWRVODELOLGDGYDVRPRWRUD 0iVULHVJRGHFRQYXOVLRQHV La madre que puede contagiar una hepatitis B al RN es aquella
que tiene el Ag s +, (hepatitis crnica activa, hepatitis aguda, porta-
)HQREDUELWDO ,GHP dora). Si adems el Age es positivo, el riesgo asciende hasta un 90%.
El momento de mayor riesgo es el momento del parto.
Pregunta 23.- R: 1 La mayora de las veces, la enfermedad en el neonato es asintom-
El embarazo del hijo de madre diabtica es considerado un emba- tica, pero tiene una alta probabilidad de evolucionar hacia una forma
razo de alto riesgo, debido al gran nmero de problemas que puede crnica y sufrir degeneracin maligna. Para evitar este curso es funda-
presentar el RN. Entre ellos figuran: mental realizar profilaxis con vacuna frente a hepatitis B (0,1,6) y
Macrosoma con visceromegalia (pero no megalencefalia), excep- gammaglobulina especfica en el mismo momento. Si la profilaxis se
to las madres con diabetes avanzadas, y vasculopata, que suelen realiza correctamente, podr recibir lactancia materna.
tener nios CIR.
Cardiomiopatas: lo ms frecuente es la hipertrofia asimtrica del Pregunta 26.- R: 2
tabique interventricular, que clnicamente se manifiesta como una El caso clnico nos presenta a un RN con clnica de CMV congni-
estenosis subartica. ta. Los datos claves que nos permiten llegar al diagnstico son: corio-
Sndrome asfctico: son nios ms grandes y con ms complicacio- rretinitis y calcificaciones periventriculares (en el caso de la TXP, estas
nes durante el parto. son difusas). La infeccin congnita por CMV es la infeccin congni-
Riesgo de EMH: la insulina en altas concentraciones inhibe la sn- ta ms frecuente. Lo ms frecuente es que estn asintomticos, pero si
tesis de surfactante. tienen clnica pueden presentar ictericia, hepatoesplenomegalia, cal-
Policitemia, ictericia y trombosis de la vena renal. cificaciones, hepatitis,... La neumonitis es la forma ms caracterstica
Malformaciones: las ms frecuentes malformaciones cardacas. de infeccin postnatal. No existe tratamiento eficaz; en casos de alta
Otras: agenesia lumbosacra y sd. del coln izquierdo hipoplsi- replicacin al nacimiento se puede emplear Ganciclovir. La secuela
co. ms frecuente es la hipoacusia neurosensorial.

Pregunta 27. Diferencias entre toxoplasma y citomegalovirus.

CITOMEGALOVIRUS TOXOPLASMA
Se reactiva asintomtica 1-2%
Infeccin materna Se infecta
Se infecta sintomtica 1-2
Placentaria 1e r trimestre ms frecuente, ms grave Placentario
Transmisin Canal del parto - 1e r trimestre, ms grave
Comentarios TEST

Lactancia - 3e r trimestre: ms frecuente


5-18% secuelas tardas, hipoacusia neurosensorial
Asintomtico ms frecuente Sin tratamiento coriorretinitis
bilateral y severa
SABIN
Coriorretinitis Coriorretinitis
Sntoma Microcefalia Hidrocefalia
Calcificaciones periventriculares Calcificaciones intracraneales difusas
Convulsiones
1 determinacin:
Negativo: vigilar
Screening prenatal Prenatal Ac antiCMV (no se hace prevencin) Positivo: hacer una 2 determinacin:
- Disminuido o igual: infeccin pasada
- Aumentado: tratar
Aislamiento y cultivo en orina "inclusiones en ojo de
Aislamiento en placenta (sangre?)
buho"
Diagnstico IgM
IgM
IgG o estable a las 6 semanas
IgG o estable a las 6 semanas
Siempre:
Sintomtico: pirimetadina + sulfadicina 6 meses
No
Tratamiento pirimetadina + sulfamicina/espiromicina 6
Ganciclovir a veces
meses
Asintomtico. espiromicina y valorar serologa

M exico A rgentina
C hile U ruguay
CTO Medicina C/ Nez de Balboa, 115 28006 MADRID (Espaa) Tfno.: (91) 782 43 32 / Fax: (91) 782 43 27
E-mail: secretaria@ctomedicina.com; iberocto@ctomedicina.com WEB: www.ctomedicina.com; www.iberocto.com
PD Pg. 5
PEDIATRA
Seguimiento a distancia Preparacin Examen de Seleccin 05/06 1 Vuelta
Pregunta 27.- R: 2 El neonato contrae la infeccin cuando pasa por el canal del
Ver tabla en la pgina anteror. parto, por lo tanto lesiones genitales activas en el momento del parto
contraindican un parto vaginal y es indicacin de cesrea.
Pregunta 28.- R: 1 El tratamiento en este nio sera la administracin de aciclovir iv.
Estamos ante un nio RN CIR (edad gestacional de 38 semanas y (Ver tabla).
peso < 2500gr) con clnica de erupcin petequial y hepatoesple-
nomegalia. Esto lo puede presentar cualquier infeccin connatal. Pregunta 31.- R: 2
En relacin al ombligo del RN, es importante recordar que consta
de 2 arterias (en ocasiones una sola arteria se asocia con anomalas
renales y trisoma del 18) y 1 vena.
Pueden existir alteraciones como el onfalocele (dd con gastrosqui-
sis) y hernia umbilical (ver tabla).
Sobre secreciones por el ombligo han preguntando en el MIR:
Persistencia del uraco: sale por el ombligo un lquido amarillento
similar a la orina con un pH cido.
Persistencia del conducto onfalomesentrico (fstula intestinal): sale
un lquido amarillento con un pH alcalino.

El ombligo amnitico se refiere a un ombligo con mucha gelatina.

Pregunta 31. Patologa umbilical.

HERNIA ONFALO-
GASTROSQUISIS
UMBILICAL CELE

DEFECTO DE Paramedio
Umbilical. Umbilical.
CIERRE (lat. dcho. + frec.).
Pregunta 28. Manifestaciones clnicas de la rubola congnita. CUBIERTAS Peritoneo y piel. Peritoneo. Sin peritoneo.

A continuacin nos describen la clnica tpica del DAP: soplo - Anomalas


continuo en 2 espacio intercostal izquierdo con pulsos femorales - Estrangulacin cromosmicas.
saltones. Las lesiones seas son las tpicas de la rubola, las que nos COMPLICACIONES
(rara). - Extrofia vesical.
Atresia intestinal.
encontraramos en la sfilis seran las mismas, pero con reaccin - Reduccin - Sd. Beckwith
peristica. espontnea. (macrosoma e
Aparte de esto, para el MIR debemos recordar: hipoglucemia).
Al contrario que en todas las infecciones connatales, la rubola Quirrgico, si:
tiene ms riesgo de trasmisin en el 1 trimestre. - Estrangulacin.
Correcin
Correcin
La trada de Gregg se caracteriza por: - Crecimiento
quirrgica
quirrgica precoz
- Sordera de percepcin. TRATAMIENTO precoz
progresivo. (buen
- Cardiopata: la ms frecuente el DAP. (peor
- Persiste a los pronstico).
pronstico).
3-5 aos.
- Afectacin ocular: lo ms frecuente son las cataratas.
Un RN infectado por rubola congnita puede eliminar el virus
en las secreciones hasta ao y medio despus del nacimiento, Pregunta 32.- R: 4
por lo que es obligatorio aislarlo de mujeres embarazadas. Recordar los siguientes datos de forma muy simplificada:
1,5 m: sonrisa social.
Pregunta 29.- R: 1 3 m: inicio sostn ceflico.

Comentarios TEST
La clnica de la sfilis congnita se subdivide en 2 grupos: 6 m: inicia sedestacin.
Precoz: los dos primeros aos. 9 m: movimiento de pinza.
Tarda: ms all de los dos aos. 10-12 m. Inicia bipedestacin.
12 m: emite bislabos referenciales.
La sfilis precoz viene a ser parecida a una sfilis 2 del adulto. 15-22 m: realiza torres de cubos de 2 hasta seis.
Recordar que la trasmisin de la sfilis en el RN es por va transplacen- 16-19 m: corre y realiza combinaciones de 2 palabras.
taria, por lo tanto no deberemos encontrar chancro de inoculacin.
La clnica ms frecuente de la sfilis precoz es la hepatoesplenome- Pregunta 33.- R: 4
galia. Otras manifestaciones son: lesiones cutneo-mucosas (pnfigo, Estamos ante un cuadro de sarampin. El caso clnico va relatando
rash, rinorrea intensa), lesiones seas (imgenes radiolucentes en detalladamente las fases de esta enfermedad: Fase prodrmica: cata-
metfisis de huesos largos con reaccin peristica). La sfilis tarda es rro con tos y fiebre, manchas de Koplik (lesiones blanquecinas sobre
semejante a la sfilis terciaria del adulto: lesiones cutneas (rgades base eritematosa en mucosa subyugal), es un dato patognomnico de
sifilticas), lesiones seas (nariz en silla de montar, tibias en sable), lesio- esta enfermedad. Fase exantemtica: inicio del exantema por la cara
nes del SNC (tabes dorsal y paresia). Recordar los elementos de la y descenso. El exantema es maculopapular, no pruriginoso. La tempe-
trada de Hutchinson: sordera, queratitis y anomalas dentarias. ratura sube hasta 39-40. Fase de resolucin :el exantema desapa-
El tratamiento consiste en la administracin de penicilina.
rece en el mismo orden que apareci.
Pregunta 30.- R: 4 Entre las complicaciones figuran:
Todo el caso clnico hace referencia a la infeccin por el virus Cuadros ORL: complicacin ms frecuente.
herpes simple, siendo el ms frecuente el tipo II. Neumonas: en los nios es ms frecuente la sobreinfeccin bacte-
La clnica suele comenzar a la semana con grave afectacin del riana, ms que la neumona de clulas gigantes de Hecht.
estado general, fontanela abombada y vesculas en la zona de pre- Afectacin del SNC: encefalitis aguda y PEES (panencefalitis escle-
sentacin (en la prctica clnica slo aparecen en un 70% de los rosante subaguda): forma de encefalitis por virus lentos, con un
casos). Se asocia con una elevada mortalidad. pronstico psimo, el diagnostico se hace detectando en LCR au-
mento de Acs frente al sarampin.

M exico A rgentina
Pg. 6 PD C hile U ruguay
CTO Medicina C/ Nez de Balboa, 115 28006 MADRID (Espaa) Tfno.: (91) 782 43 32 / Fax: (91) 782 43 27
E-mail: secretaria@ctomedicina.com; iberocto@ctomedicina.com WEB: www.ctomedicina.com; www.iberocto.com
PEDIATRA
Preparacin Examen de Seleccin 05/06 1 Vuelta Seguimiento a distancia

Pregunta 30. Infecciones connatales.

VARICELA SFILIS RUBEOLA HERPES SIMPLE

Infeccin Se reactiva (VHS II)


Se infecta Se infecta Se infecta sintomtica 70%
materna Se infecta: poco frecuente
er
Placentaria: 3 trimestre:
Placentaria: Placentaria
er ms frecuente si madre con Placentaria, 1er trimestre ms frecuente,
Transmisin 1 y 2 trimestre, ms frecuente. Canal del parto
sfilis primaria o secundaria ms grave
Prx imo a parto: ms grave. Postparto
no tratadas
er
Transmisin 1 y 2 trimestre:
Precoz (<2 aos): Transitoria: CIR, hepatoesplenomegalia, Generalizada (1 semana)
atrofia miembros, cicatrices y
- Hepatomegalia. meningitis... sepsis, encefalitis...
malformaciones.
- Pnfigo sifiltico, coriza, Permanente (Gregg): Localizada
5 a 21 das preparto: varicela
neumona alta de Hetcht. - Cataratas (2 y 3 semanas):
Sntomas leve en 4 primeros das
Tarda (>2 aos): - Sordera de percepcin - Queratoconjuntivitis
postparto.
- Sordera de percepcin. - Cardiopata - Vesculas cutneas
5 das preparto a 2 das
- Queratitis intersticial. - Retraso mental, fenmenos - Encefalitis del lbulo
postparto: varicela grave en
- Anomalas dentarias. autoinmunes. temporal
das 5 y 10 postparto.

1er trimestre y antes del


parto.
Screaning
VDRL y FTA-ABS .
prenatal
Si infeccin activa tratar a
la madre.

ADN del virus.


Aislamiento del virus.
IgM en sangre de cordn VDRL.
IgM.
Diagnstico (descenso rpido). FTA-ABS tratar con Cultivo y aislamiento virus
IgG o estable a las 6
Aislamiento del virus en penicilina.
semanas.
el lquido vesicular.

Si infeccin materna entre 5 das


preparto y 2 postparto:
Tratamiento Ig anti VVZ. Penicilina 15 das No Aciclovir
Aislamiento estricto.
Aciclovir si es grave.

Pregunta 34.- R: 3 Pregunta 36.- R: 1


La rubola es una enfermedad banal por su curso, pero de gran El caso que se nos presenta es una varicela. Ya sabemos que el
gravedad si afecta a la mujer embarazada, por las graves secuelas que agente causal es el virus de la varicelazoster, de la familia de los
puede producir en el feto. herpes virus. La fase prodrmica consiste en un catarro de vas altas
Comentarios TEST

Clnicamente es muy parecida al sarampin, pero con un curso con febrcula. Lo caracterstico del exantema es que suele afectar a
ms leve y recortado. Suele cursar con febrcula y el exantema desa- tronco y parte proximal de extremidades, respetando las zonas dista-
parece al 3 da. A la hora de contarnos un caso clnico nos deben les. Las lesiones aparecen en distintos estadios evolutivos (o lesiones en
poner siempre en la fase prodrmica, las adenopatas retroauriculares cielo estrellado); estas son lesiones eritematosas con vesculas y otras
dolorosas a la palpacin que tardan semanas en resolverse. en fase de costra. Las lesiones pican mucho. La complicacin ms
La complicacin ms frecuente es la artritis de pequeas articulacio- frecuente es la sobreinfeccin bacteriana de las lesiones.
nes, ms frecuentes en mujeres jvenes, que se resuelve sin dejar secuelas. Otras complicaciones son:
Neumona varicelosa: en la infancia es ms frecuente la secunda-
Pregunta 35.- R: 3 ria a sobreinfeccin bacteriana, ms que la producida por el pro-
Estamos ante un caso de escarlatina. El agente ms frecuente es el pio virus.
Streptococcus pyogenes (alfa hemoltico). Afecta a nios entre 5-15 Afectacin del SNC: lo ms frecuente es en forma de cerebelitis,
aos de edad. La fase prodrmica suele ser una faringoamigdalitis que evoluciona de forma favorable.
bacteriana aguda (fiebre alta, exudado faringoamigdalar). Posterior-
mente aparece un exantema, de inicio en tronco , que se hace ms La varicela en los nios sanos no precisa tratamiento con aciclovir.
confluente en los pliegues (lneas de Pastia), afectando a la cara y Son indicaciones para su uso: neonatos e inmunodeprimidos.
respetando el tringulo nasogeniano. Afecta a palmas y plantas, y
puede dejar descamacin residual al resolverse. Pregunta 37.- R: 3
El diagnstico de eleccin es un frotis de exudado farngeo. El La pregunta hace referencia al llamado eritema infeccioso,
tratamiento consiste en la administracin de penicilina oral durante megaloeritema o 5 enfermedad. En el MIR lo suelen preguntar para
10 das. En los casos en los que no es posible realizar el tratamiento confundirlo con el exantema sbito, roseola infantil o 6 enfermedad.
oral completo estar indicado la administracin de penicilina benza- El eritema infeccioso tiene como agente causal el Parvovirus B 19.
tina (im) 1 dosis. El cuadro se caracteriza por febrcula y aparicin de un exantema en
Entre las complicaciones ms importantes figuran: encaje o cartogrfico en mejillas y zona proximal de extremidades. Se
Glomerulonefritis postestreptoccica. resuelve espontneamente, pero en ocasiones puede recidivar cuan-
Fiebre reumtica: slo si la puerta de entrada ha sido un foco ORL. do el nio se estresa, tiene fiebre,...

M exico A rgentina
C hile U ruguay
CTO Medicina C/ Nez de Balboa, 115 28006 MADRID (Espaa) Tfno.: (91) 782 43 32 / Fax: (91) 782 43 27
E-mail: secretaria@ctomedicina.com; iberocto@ctomedicina.com WEB: www.ctomedicina.com; www.iberocto.com
PD Pg. 7
PEDIATRA
Seguimiento a distancia Preparacin Examen de Seleccin 05/06 1 Vuelta
Entre las complicaciones figuran la aplasia medular en nios sus- Pregunta 41.- R: 5
ceptibles (con enfermedades hematolgicas). Estamos ante un cuadro de mononucleosis infecciosa, caracteriza-
do por fiebre, adenopatas y faringoamigdalitis.
Pregunta 38.- R: 2 El agente causal ms frecuente es el virus Epstein-Barr. Hasta en un
El cuadro al que se refiere este caso clnico es un exantema sbito, 5% se puede asociar con exantema, pero si le administramos antibi-
roseola infantil o 6 enfermedad. Lo tpico es que se produzca en ticos (amoxicilina) ante la sospecha de una infeccin bacteriana, el
lactantes, cursando con fiebre alta de 39-40 y algn signo catarral, riesgo de aparecer un exantema maculopapular pruriginoso aumenta
pero con un estado general bueno. hasta en un 90% de los casos. En este caso clnico, el S. pyogenes
Con respecto a la R1, es falsa porque lo tpico es que la fiebre alta aislado en el frotis no significa una infeccin activa por ste, si no que
persista durante 3 das, no slo 24 horas. se traduce en un estado de portador crnico.
La respuesta 2 es la cierta: si en las primeras 24 horas le realizra- Dentro de las complicaciones, la ms grave es la rotura esplnica.
mos una analtica, encontraramos una importante leucocitosis con Otras seran la hipertrofia amigdalar, que puede producir obstruccin
neutrofilia. Si esperaramos 36 horas ms, se apreciara un cambio de la va area y hepatitis subclnica con elevacin de las transamina-
significativo en la analtica con neutropenia y linfocitosis. sas en hasta un 70-90% de los casos.
La 3 es falsa, porque la rubola suele cursar con febrcula o inclu- La herpangina es un cuadro caracterizado por fiebre alta, adeno-
so sin fiebre. patas y lceras en los pilares anteriores del paladar. El tratamiento es
La 4 es falsa: OJO no confundir con el eritema infeccioso, sintomtico.
megaloeritema o 5 enfermedad.
La 5 es falsa, porque la escarlatina es una enfermedad tpica de Pregunta 42.- R: 2
nios ms mayores, entre los 5 y 15 aos de edad. Para el diagnstico de mononucleosis infecciosa, aparte de la sos-
pecha clnica, podemos encontrar:
Pregunta 39.- R: 4 Leucocitosis (10000-20000) con ms de un 20-40% de linfocitos
Recordemos los criterios diagnsticos de la enfermedad de Kawasaki atpicos.
o sd. mucocutneo ganglionar. Aumento de las transaminasas.
Acs heterfilos: son especficos frente al VEB. Es til en mayores de
4 aos; por debajo de esta edad, la sensibilidad es < del 20%.
Pregunta 39. Criterios diagnsticos de la enfermedad de Kawasaki. Puede permanecer positivo hasta 9 meses despus, por lo que no
es til para el diagnstico de infeccin activa.
$ 'JFCSFEFBMNFOPTEBTEFEVSBDJO Acs especficos contra el VEB:
- Fase aguda:
% 1SFTFODJBEFEFMPTDJODPTJHVJFOUFTTJHOPT
IgM + - IgG: frente a la cpside viral : CVA
 $POKVOUJWJUJTCJMBUFSBMOPQVSVMFOUB EA + (Ag temprano)
 "MUFSBDJPOFTFOMBNVDPTBEFMBPSPGBSJOHF DPO EL VEB se ha asociado con distintos tumores como el linfoma Burkitt,
JOZFDDJOGBSOHFBMBCJPTTFDPTDPOGJTVSBT carcinoma nasofarngeo, sd. de Duncan y diversos sds. linfoprolifera-
JOZFDUBEPTPBNCPT ZMFOHVBFOGSFTB tivos.
 "MUFSBDJPOFTFOMBT[POBTQFSJGSJDBTEFMBT El tratamiento consiste en reposo; en algunos casos puede ser til la
FYUSFNJEBEFT DPNPFEFNBZFSJUFNBFONBOPTP administracin de corticoides (obstruccin de la va area alta, ane-
QJFT EFTDBNBDJOEFJOJDJPQFSJVOHVFBM mia hemoltica autoinmune, convulsiones,...).
 &YBOUFNB EFJOJDJPFOFMUSPODPQPMJNPSGP OP Pregunta 43.- R: 4
WFTJDVMBS El cuadro del que vamos a hablar a continuacin es de una tos ferina.
 -JOGBEFOPQBUBDFSWJDBMVOJMBUFSBM La edad con ms riesgo es el periodo de lactante (la madre no le da Acs
protectores, porque estos van disminuyendo a lo largo de la vida).
-BFOGFSNFEBEOPTFQVFEFFYQMJDBSQPSOJOHVOBPUSB
& DBVTBDPOPDJEB
La clnica consta de una fase prodrmica: fase catarral, con febr-
cula, rinorrea y tos blanda. Este es el periodo de mxima contagiosi-
dad. La fase paroxstica: consiste en episodios de golpes o accesos

Comentarios TEST
Adems de estos sntomas se puede encontrar patologa muy varia- de tos, seguido en muchas ocasiones de vmitos. En los nios mayo-
da: artritis , pericarditis, iritis,... Es muy llamativa la trombocitosis, espe- res es caracterstico el gallo inspiratorio. Y por ltimo, la fase de
cialmente en la fase subaguda de la enfermedad. convalecencia: esta fase se caracteriza por encontrarse el nio afebril.
El tratamiento consiste en el empleo de AAS y Gammaglobulina en El diagnstico se basa en el cultivo del moco nasal en medio de
la fase aguda, manteniendo el AAS en dosis antiagregantes posterior- Bordet-Gengou.
mente. El empleo de la gammaglobulina en la fase aguda previene el En la analtica es tpico encontrar una linfocitosis absoluta. Para el
desarrollo de aneurismas coronarios en la fase subaguda. tratamiento se utiliza la eritromicina.

Pregunta 40.- R: 2 Pregunta 44.- R: 5


Este paciente presenta una prpura de Schlein-Henoch. Como La parotiditis est producida por un paramixovirus. Tiene mayor
ya recordis, es una vasculitis de pequeo vaso. incidencia entre los 515 aos, provocando ms complicaciones en
La fase prodrmica suele consistir en proceso ORL. Posteriormente los pacientes post-pberes. Los Acs maternos protegen los primeros 6-
puede aparecer, en orden de frecuencia: 8 meses de vida. La clnica consiste en una inflamacin de las glndu-
Afectacin cutnea: prpura palpable, en mmii, que suele cursar las partidas de comienzo brusco, asincrnico, siendo tpico el des-
en brotes. plazamiento del pabelln auricular hacia delante con borramiento
Artritis de rodillas y tobillos. del ngulo mandibular.
Dolor abdominal clico, vmitos y deposiciones con sangre. Entre las complicaciones figuran:
Afectacin renal: hematuria con o sin proteinuria. Este es el factor Meningitis asptica: es la complicacin ms frecuente en la infan-
que marca el pronstico. cia, siendo el 50% subclnica.
Slo en menos de un 1% se produce afectacin del SNC. Orquitis: 25% de varones postpberes, el 90% es unilateral. La
El tratamiento consiste en antiinflamatorios para las molestias arti- esterilidad es una secuela poco frecuente.
culares y en ciclos cortos de corticoides, en caso de dolor abdominal Otros: ooforitis, pancreatitis, sordera neurosensorial, etc.
intenso. El tratamiento consiste en medidas generales y analgsicos.

M exico A rgentina
Pg. 8 PD C hile U ruguay
CTO Medicina C/ Nez de Balboa, 115 28006 MADRID (Espaa) Tfno.: (91) 782 43 32 / Fax: (91) 782 43 27
E-mail: secretaria@ctomedicina.com; iberocto@ctomedicina.com WEB: www.ctomedicina.com; www.iberocto.com
PEDIATRA
Preparacin Examen de Seleccin 05/06 1 Vuelta Seguimiento a distancia
Pregunta 45.- R: 5 7 aos, porque a partir de esa edad hay ms riesgos que bene-
Respecto al SIDA en Pediatra, hay que recordar: ficios.
La principal va de transmisin es vertical, durante el parto. El por- Indicaciones: en el calendario vacunal se administra a los 2,4,6 y
centaje de infectados es aproximadamente del 5-8%. 18 meses y a los 4 aos se puede administrar DT o DTPa , luego
El periodo de latencia es menor que en adulto. Se subdivide en cada 10 aos dT. Las primeras dosis se pueden administrar en un
SIDA precoz (periodo de latencia de pocos meses, clnica antes del mismo preparado con otras vacunas (polio inactivada, Hib, VHB),
ao de vida, predominando las manifestaciones neurolgicas, pro- para minimizar el nmero de pinchazos.
nostico malo) y tardo (ms parecido al del adulto, predominando
la clnica infecciosa). En pacientes inmunodeprimidos se puede administrar sin riesgo.
Las manifestaciones clnicas ms frecuentes en pediatra, compa-
rndolo con los adultos son: cuadros ORL, parotiditis, neumona Pregunta 50.- R: 4
intersticial linfoide y manifestaciones neurolgicas (calcificaciones La vacuna del Hib es una vacuna conjugada, muy segura y eficaz.
de los ganglios basales, atrofia cerebral,...). La causa ms frecuente Protege frente a infecciones invasivas (meningitis y epiglotitis). En el
de muerte es la neumona por P. Carinii. calendario vacunal se incluye a los 2-4-6 y 18 meses. Estara indicada
Son infecciones menos frecuentes en los nios: TBC, hepatitis B, solamente en nios menores de 5 aos.
linfomas y sarcoma de Kaposi.
Pregunta 51.- R: 4
Pregunta 46.- R: 1 La vacuna frente a la hepatitis B est realizada por ingeniera genti-
Es importante realizar quimioprofilaxis con AZT durante el emba- ca. Produce en las personas que la recibe Acs frente al Ag s. En el
razo, el momento del parto y luego al RN. calendario vacunal se puede administrar con diversas pautas: (0-1-6),
El tratamiento se basa en la triple terapia, al igual que en los adultos. (0,2,6) y (2, 4,6) y una cuarta a los 18 meses. Para prevenir la trasmisin
Para evaluar la evolucin, se emplea la carga viral y el cociente de la hepatitis B en el neonato, en el caso que la madre sea portadora,
CD4/CD8, recordando que la cifra de linfocitos hay que extrapolarla se debe asociar con gammaglobulina especifica al nacimiento.
segn la edad del nio. Est indicada a cualquier edad si el nio no ha sido vacunado
previamente. Sus contraindicaciones son muy escasas. Recordar que
el embarazo no es una contraindicacin absoluta, pero se aconseja
Pregunta 46. Diagnstico de SIDA en la infancia. evitar todo tipo de vacunas siempre que sea posible.
GHPHVHV !PHVHV Pregunta 52.- R: 4
De forma simplificada, vamos a exponer la quimioprofilaxis frente
'RVUHVXOWDGRVSRVLWLYRV a los agentes ms frecuentes de meningitis.
3&5
&XOWLYRYLUDO (/,6$\VHFRQILUPDSRU:HVWHUQ%ORW Meningococo: a todos los contactos, independiente de la edad,
$JS 2ORVFULWHULRVGHODSDUWDGRDQWHULRU con rifampicina en dosis de 10 mg/K/dosis cada 12 horas 4 dosis +/
- vacuna en caso del meningococo C.
2FOtQLFDGHO6,'$ Hib: nios menores de 5 aos sin vacunar y adultos que conviven
con menores de 5 aos sin vacunar (stos pueden ser portadores):
rifampicina en dosis de 20 mg/K/dosis cada 24 horas 4 dosis +7-
Pregunta 47.- R: 3 vacuna.
La triple vrica protege contra el virus del sarampin, rubola y Neumococo: no existe quimioprofilaxis activa. Se puede indicar la
parotiditis. Es una vacuna de virus vivos atenuados. vacuna del neumococo de 7 serotipos para prevenir futuros con-
Indicaciones: a los 15 meses y una dosis de recuerdo a los 4 aos, tactos.
a nios y nias.
Efectos adversos: cuadro de fiebre y exantema, ms atenuados que Pregunta 53.- R: 2
las infecciones frente a las que protege (son virus vivos). Vamos a esquematizar a continuacin la actitud que hay que llevar a
Contraindicaciones: cabo con un nio que ha tenido un contacto con un bacilfero positivo.
Inmunodeprimidos: excepto nio HIV+.
Comentarios TEST

Embarazo.
La alergia al huevo ha dejado de ser una contraindicacin absoluta.

Pregunta 48.- R: 4
Existen actualmente dos tipos de vacunas: Polio oral (Sabin) y Polio
im (Salk).
La polio Sabin es una vacuna de virus vivos atenuados que produce
inmunidad local tipo IgA y vacunacin comunitaria por contacto fecal-
oral. Como efecto adverso ms importante destacan algunos casos ais-
lados de polio en nios sanos, lo que ha motivado en los ltimos aos
que se estn potenciando las vacunas de virus inactivados.
Est contraindicada en personas inmunodeprimidas y personas
que conviven con stos.
La vacuna tipo Salk (im) est elaborada con virus inactivados, es
menos eficaz, pero ms segura. Es la que actualmente recomienda la
AEP. La pauta de administracin es a los 2-4-6 18 meses y a los 4 aos.

Pregunta 49.- R: 5
Lo bsico que tenis que recordar de la vacuna DTP es lo siguiente:
Componentes:
- Difteria: toxoide.
- Ttanos: toxoide.
- Pertussis: dos tipos: celular y acelular. Esta ltima con muchos
Pregunta 53. Actitud ante un nio que ha tenido contacto con un paciente
menos efectos adversos a nivel neurolgico. Recomendada por
bacilfero positivo.
la AEP. La Pertussis celular estaba contraindicada en mayores de

M exico A rgentina
C hile U ruguay
CTO Medicina C/ Nez de Balboa, 115 28006 MADRID (Espaa) Tfno.: (91) 782 43 32 / Fax: (91) 782 43 27
E-mail: secretaria@ctomedicina.com; iberocto@ctomedicina.com WEB: www.ctomedicina.com; www.iberocto.com
PD Pg. 9
PEDIATRA
Seguimiento a distancia Preparacin Examen de Seleccin 05/06 1 Vuelta

Pregunta 59. Vmitos del lactante.

Reflujo gastroesofgico Estenosis hipertrfica de ploro Atresia pilrica Atresia duodenal

Momento de aparicin 1 semana 2 a 3 semana

Vmitos Alimentarios sin fuerza Alimentarios a chorro Alimentarios B il io s o s

Sntomas respiratorios. Masa palpable. Abdomen ex cavado.


Otros sntomas
Disminucin del crecimiento. Alcalosis hipoclormica. Asociar al Down.

Clnica + respuesta a tratamiento


Evolucin. Rx: R X:
Diagnstico pHmetra 24h R x : "signo de la cuerda". 1 burbuja gstrica. 1 burbuja gstrica.
1 burbuja gstrica. Ausencia de gas distal. 1 burbuja duodenal.
endoscopia

Medidas posturales. Ciruga:


Tratamiento Cisapride. Piloromiotoma. Ciruga Ciruga
Ciruga (funduplicatura de Nissen). Fredet R amsted.

Pregunta 54.- R: 2 tos, diarrea), cutnea (urticaria) y en ocasiones respiratoria (broncoes-


El calostro es la primera secrecin mamaria, las principales diferen- pasmo, edema de glotis).
cias con la leche materna madura es que contiene ms protenas y El cuadro est mediado por un mecanismo inmunolgico tipo Ig E.
ms minerales que la leche madura. Para el diagnstico se emplean las pruebas cutneas, que sern
positivas.
El tratamiento consiste en la suspensin de la leche de vaca, reem-
Pregunta 54. Diferencias entre la leche materna y la de vaca. plazndola por frmulas especiales (hidrolizado de protenas). Suele
ser un proceso autolimitado, la mayora de los nios pueden tomar
/(&+(+80$1$ /(&+('(9$&$ leche de vaca a partir de los 2 aos.
&DORUtDV NFDOO NFDOO
J J Pregunta 57.- R: 1
3URWHtQDV &DVHtQD &DVHtQD El tipo ms frecuente de fstula es la tipo 3 (atresia esofgica con
6HURSURWHtQDV 6HURSURWHtQDV fstula distal).
+LGUDWRVGH&DUERQR JODFWRVD\RWUDV JODFWRVD A nivel proximal nos encontraremos la siguiente clnica:
J - Polihidramnios: el feto no traga adecuadamente el lquido am-
J nitico.
FJUDVRVHVHQFLDOHV
(VFDVRViFJUDVRV - La sonda se queda enrollada en el bolsn superior.
*UDVDV FJUDVRVFDGHQDODUJD
HVHQFLDOHV
LQVDWXUDGRV
FJUDVRVVDWXUDGRV
- Salivacin excesiva, aspiracin pulmonar masiva (fstula proxi-
&ROHVWHURO mal) o por rebosamiento.
0LQHUDOHV   YHFHV
+LHUUR   A nivel distal:
- Abdomen excavado si la fstula es proximal.
&REUH  
- Abdomen distendido, si la fstula es distal.
)O~RU 

Comentarios TEST
5HODFLyQ
 
&iOFLR)yVIRUR
9LWDPLQD$  
9LWDPLQD%  
9LWDPLQD&  (VFDVD
9LWDPLQD'  (VFDVD
9LWDPLQD(  
9LWDPLQD. (VFDVD 
1LWUyJHQRQRSURWpLFR  

Pregunta 55.- R: 4
La causa ms frecuente de GEA en nuestro medio es el rotavirus. Es Pregunta 57. Tipos de fstulas traqueoesofgicas.
una enfermedad tpica de lactantes que produce una diarrea lquida,
SIN sangre. El diagnstico se basa en la realizacin de un test de ELISA Pregunta 58.- R: 5
en las heces para detectar el Ag. Estamos ante un caso de RGE leve, caracterizado por pequeos vmi-
El tratamiento es sintomtico. tos sin fuerza despus de las tomas, por lo dems el nio se encuentra
Se est estudiando una vacuna (semejante a la polio oral) para asintomtico con peso y talla en percentiles normales. El RGE es muy
intentar evitar esta enfermedad en los lactantes. frecuente en el primer ao de vida, hasta un 85%. Generalmente desapa-
rece hacia los 2 aos, aunque en ocasiones puede persistir hasta los 4.
Pregunta 56.- R: 1 En un 10% de los casos, el RGE se asocia con escasa ganancia
Estamos ante un cuadro de alergia a protenas de leche de vaca. El ponderal, esofagitis, anemia ferropnica y clnica respiratoria.
cuadro tpico consiste en inicio de la clnica tras la ingesta de un El diagnostico se basa en la historia clnica y en ocasiones en la
bibern de leche de vaca. La sintomatologa suele ser digestiva (vmi- realizacin de pruebas complementarias: transito superior con bario

M exico A rgentina
Pg. 10 PD C hile U ruguay
CTO Medicina C/ Nez de Balboa, 115 28006 MADRID (Espaa) Tfno.: (91) 782 43 32 / Fax: (91) 782 43 27
E-mail: secretaria@ctomedicina.com; iberocto@ctomedicina.com WEB: www.ctomedicina.com; www.iberocto.com
PEDIATRA
Preparacin Examen de Seleccin 05/06 1 Vuelta Seguimiento a distancia
y una pHmetra de 24 horas (para cuantificar el grado de reflujo, es la La clnica se caracteriza por un cuadro de malabsorcin, de
prueba ms sensible y especfica). inicio ms frecuente entre los 6 meses y los 2 aos. Suelen presen-
El tratamiento en los casos leves consiste en medidas posturales y tar esteatorrea, estancamiento de la curva ponderal, disminucin
espesantes de las tomas. En casos de reflujos ms graves se administra- de la masa muscular, irritabilidad y distensin abdominal impor-
ran estimulantes del peristaltismo (cisapride, domperidona) o trata- tante.
miento quirrgico (funduplicatura de Nissen). El diagnstico se basa en los test serolgicos (Acs antigliadina, anti-
rreticulina, antiendomisio y antitransglutaminasa, estos ltimos los ms
Pregunta 59.- R: 5 sensibles y especficos de todos, y en la realizacin de biopsias intesti-
La clnica tpica de un cuadro de estenosis hipertrfica de ploro nales.
consiste en vmitos proyectivos tras las tomas, tras el vmito el nio se El tratamiento consiste en la exclusin de por vida del trigo,
queda irritable y hambriento (ver tabla en pgina siguiente). cebada, centeno y +/- avena. Pueden tomar libremente maz y
Debido a los vmitos persistentes, es tpico encontrar una alcalosis arroz.
metablica con normo o hipo K.
La prueba diagnstica a realizar ante la sospecha de este cuadro
sera una ecografa abdominal. En ocasiones se debe recurrir a un
trnsito digestivo, donde aparece de forma tpica la imagen del signo
de la cuerda.
El tratamiento es quirrgico: pilorotoma extramucosa de Ramsted.

Pregunta 60.- R: 4
Lo tpico de la atresia duodenal son los vmitos biliosos en las
primeras 24- 48 horas, con abdomen excavado. Con mayor frecuen-
cia la atresia se localiza a nivel de la 3 porcin del duodeno. En la Rx
de abdomen es tpico encontrar la imagen de doble burbuja.
La R1: lo tpico sera encontrar vmitos a las 2-3 semanas de vida.
R2: lo tpico es estreimiento y en ocasiones vmitos fecaloideos.
R3: sangrado indoloro rectal sin vmitos.
R5: la clnica de la invaginacin intestinal suele comenzar con
episodios de dolor intenso, siendo muy raro en el periodo neonatal.

Pregunta 61.- R: 4
El divertculo de Meckel es un resto del conducto onfalomesent-
rico. Es la malformacin digestiva ms frecuente (R1), localizndose a
unos 50- 70 cm de la vlvula ileocecal.
La clnica ms frecuente suele consistir en una hemorragia rectal
indolora (R4) e intermitente debido a la ulceracin de la mucosa ileal
adyacente al divertculo con mucosa ectpica (gstrica o pancreti-
ca).
La tcnica diagnstica ms sensible es la gammagrafa con Tc 99.
El tratamiento es quirrgico.

Pregunta 62.- R: 3
Estamos ante un cuadro tpico de invaginacin intestinal. Nio que
presenta episodios repetidos de llanto y encogimiento de piernas con
posterior decaimiento. Si la clnica evoluciona hasta un 60%, puede Pregunta 64. Diagnstico de enfermedad celaca.
presentar deposiciones en jalea de grosella, heces con sangre roja
fresca y moco. Pregunta 65.- R: 2
Comentarios TEST

Para el diagnstico inicial se suele realizar una ecografa abdomi- El Sd. de Reye se asocia con una infeccin vrica los das previos.
nal. A la semana inicia un cuadro progresivo de vmitos persistentes con
Una invaginacin intestinal es siempre una urgencia. Si han trascu- deterioro del nivel de conciencia que evoluciona al coma.
rrido < de 48 horas y no hay signos de perforacin, se realiza trata- En el laboratorio aparecen parmetros de fallo heptico agudo:
miento con enemas de bario o con aire, para intentar la reduccin. aumento de GOT/GPT, de LDH, CPK, amonio e hipoglucemia. Es
Un 5% puede recidivar (frecuente en aquellos casos en los que se caracterstico del Sd de Reye que la glutamato DH (enzima
realiza reduccin hidrosttica). mitocondrial) est muy elevada y que la cifra de bilirrubina sea
normal.
Pregunta 63.- R: 4 El tratamiento consiste en tratar la alteracin heptica (glucosa,
La enfermedad de Hirschprung se caracteriza por estreimiento VitK,...) y el edema cerebral para evitar el enclavamiento cerebral.
crnico desde el nacimiento asociado a un estancamiento o retraso El pronstico depende de la evolucin neurolgica.
ponderal (por malabsorcin).
Para el diagnstico es til el tacto rectal, donde se aprecia una Pregunta 66.- R: 2
ampolla rectal vaca de heces. En la manometra se aprecia un au- Estamos ante un cuadro de atresia de vas biliares extrahepticas; lo
mento paradjico del tono del esfnter anal. El diagnstico definitivo tpico es un cuadro de colestasis en un lactante de 2-3 semanas (pre-
lo dar la biopsia: en el segmento afectado se observa ausencia de dominio de bilirrubina directa, coluria, acolia).
clulas ganglionares con aumento de la acetilcolinesterasa y aumen- La prueba diagnostica sera la realizacin de un HIDA, donde se
to de las terminaciones nerviosas. aprecia captacin del contraste por el hgado, pero que no llega al
intestino (ausencia de excrecin).
Pregunta 64.- R: 5 El tratamiento, aparte de las medidas de mantenimiento, consiste en
Los factores que intervienen en la patogenia de la celiaca son: la realizacin de una hepatoportoenterostoma o tcnica de Kasai,
HLA: DR3, DR4, DR7 , DQ y W2. para intentar restablecer el flujo biliar y disminuir el dao heptico. En
Factores inmunolgicos: los linfocitos de la lmina propia se inmu- la mayora de los casos, el tratamiento definitivo es el trasplante hep-
nizan frente a la gliadina. tico.
Factores ambientales.

M exico A rgentina
C hile U ruguay
CTO Medicina C/ Nez de Balboa, 115 28006 MADRID (Espaa) Tfno.: (91) 782 43 32 / Fax: (91) 782 43 27
E-mail: secretaria@ctomedicina.com; iberocto@ctomedicina.com WEB: www.ctomedicina.com; www.iberocto.com
PD Pg. 11
PEDIATRA
Seguimiento a distancia Preparacin Examen de Seleccin 05/06 1 Vuelta
La clnica suele cursar de forma brusca, por la noche, acompan-
dose de intensa dificultad respiratoria y apenas sin tos, con fiebre alta
y aspecto sptico.
En el tratamiento, lo prioritario es asegurarse la va area (mediante
intubacin orotraqueal o traqueostoma) y administracin de anti-
biticos, generalmente cefalosporinas de 3 generacin. Con el trata-
miento adecuado, el cuadro suele remitir en 2448 horas.
En los casos dudosos nunca hay que intentar visualizar la epiglotis,
sino realizar una Rx lateral de faringe donde se apreciar una epiglotis
engrosada.

Pregunta 71.- R: 5
Estamos ante un cuadro de bronquiolitis aguda. El agente ms fre-
cuente es el VRS. Es una enfermedad estacional de los meses de invier-
no y primavera.
Afecta a nios menores de 2 aos, principalmente lactantes. Co-
mienza por un cuadro catarral, con tos blanda, mocos y febrcula, y
en los das siguientes evoluciona hacia un cuadro de obstruccin de
vas areas distales con sibilancias y espiracin alargada. Esta fase cr-
tica suele durar 2- 3 das con posterior mejora.
El tratamiento consiste en fisioterapia y beta 2 agonistas inhalados
(terbutalina, adrenalina,...).
La ribavirina estara indicada en nios con patologa pulmonar o
cardiaca de base; ni los corticoides ni los antibiticos estaran indica-
Pregunta 66. Colestasis neonatal. dos. Actualmente se intenta prevenir su aparicin administrando gam-
maglobulina especfica mensualmente en los meses de riesgo a los
Pregunta 67.- R: 3 lactantes con patologa de base (cardipatas, neumpatas).
Es un cuadro de malabsorcin a hidratos de carbono. Tiene como La secuela a largo plazo ms frecuente es la hiperreactividad bron-
antecedente una GEA que lesiona el borde del enterocito, producien- quial.
do un dficit transitorio de lactasa. La clnica se caracteriza por diarrea,
dolor clico, heces espumosas y cidas (pH< 5.5) que excorian la zona Pregunta 72.- R: 4
del paal. Ante la sospecha se debe realizar un Clinitest en las heces que El cuadro que presenta este nio es claramente una fibrosis qustica.
ser positivo. El tratamiento consiste en la exclusin de la lactosa de la Las manifestaciones clnicas suelen ser:
dieta, que la mayora de las veces es transitoria. Respiratorias: bronquiolitis recurrentes, tos crnica, neumona,
La R1 es falsa porque, aunque la clnica es igual, el Clinitest es plipos nasales, sinusitis,...
negativo. Digestivas: leo meconial, esteatorrea, malabsorcin, prolapso rec-
La R2: no da clnica. tal, cirrosis biliar,...
R4: produce una malabsorcin de protenas con clnica de desnu- Otras: diabetes, azoospermia.
tricin y edemas. Para el diagnstico se requiere al menos dos test positivos del sudor,
R5: la clnica del rotavirus es vmitos y diarrea lquida, no cida. junto con manifestaciones clnicas compatibles y/o presencia de mu-
taciones en el estudio gentico. El estudio gentico no puede despla-
Pregunta 68.- R: 3 zar al test del sudo debido a la gran variedad de mutaciones que
Este es un cuadro de laringitis aguda caracterizado por un antece- existen.
dente previo de CVA con fiebre de 38-38.5 y tos perruna. Para el tratamiento de estos pacientes es fundamental el cuidado
El tratamiento consiste en humedad ambiental, corticoides (en ae- del pulmn. Se debe realizar diariamente fisioterapia respiratoria y
rosol o sistmicos) y adrenalina en aerosol. tratamiento agresivo de las infecciones pulmonares.

Comentarios TEST
Pregunta 69.- R: 2 Pregunta 73.- R: 3
Se resume en el cuadro inferior. Se define criptorquidia como la ausencia del testculo en el escro-
to, situndose en un trayecto fisiolgico del descenso, pudiendo ser o
Pregunta 70.- R: 3 no palpable.
La epiglotitis es una enfermedad tpica de nios menores de 5 aos; La frecuencia es del 4% en el RN, aumentando de forma importan-
el agente causal ms frecuente es el Hib. Actualmente es excepcional te en los RNPT. En la mayora de los casos descienden antes del ao
ver una epiglotitis debido a la vacunacin universal frente al Hib. de edad (rara vez despus del ao descienden fisiolgicamente).

Pregunta 69. Crup infeccioso.

/$5,1*,7,6$*8'$ HVWULGXORVD /$5,1*275$48(7,69,5$/ (3,*/27,7,6$*8'$

(WLRORJtD $OHUJLDSVLFROyJLFR 9LUXVSDUDLQIOXHQ]DH HOPiVIUHFXHQWH +LQIOXHQ]DHE


$QWHFHGHQWHV 1RKD\ &DWDUURYtDVDOWDV SDFLHQWH\RIDPLOLD 

&OtQLFD
(VSDVPRODUtQJHRUHFRUWDGR )LHEUHWRVGLVQHDDOWDHVWULGRULQVSLUDWRULR )LHEUHDOWDEDEHRGLVIDJLD
JHQHUDOPHQWHQRFWXUQR 1RILHEUH GLVQHDFDEH]DH[WHQGLGD
'XUDFLyQ QRFKHV 'tDVVHPDQDV )XOPLQDQWH
$PELHQWHWUDQTXLORKXPLGLILFDU ,QWXEDUR[tJHQR
7UDWDPLHQWR &RUWLFRLGHV &HIDORVSRULQDGHJHQHUDFLyQ
$GUHQDOLQDUDFpPLFDHQDHURVRO 7WRHQ89,

M exico A rgentina
Pg. 12 PD C hile U ruguay
CTO Medicina C/ Nez de Balboa, 115 28006 MADRID (Espaa) Tfno.: (91) 782 43 32 / Fax: (91) 782 43 27
E-mail: secretaria@ctomedicina.com; iberocto@ctomedicina.com WEB: www.ctomedicina.com; www.iberocto.com
PEDIATRA
Preparacin Examen de Seleccin 05/06 1 Vuelta Seguimiento a distancia
Entre las complicaciones figuran: La clnica consiste en un dolor agudo intenso, con tumefaccin
Problemas psicolgicos. escrotal, sin fiebre ni traumatismo previo. El reflejo cremastrico suele
Infertilidad: se producen cambios histolgicos que alteran la estar anulado.
espermiognesis, y estos dependen de la edad. El pronostico de El diagnstico se basa fundamentalmente en la clnica, en caso de
fertilidad es mejor cuanto ms baja sea la localizacin del teste. duda se puede recurrir a la gammagrafa de flujo testicular o a la
Riesgo de degeneracin maligna: el testculo no descendido tiene realizacin de una eco-Doppler.
un riesgo de sufrir degeneracin maligna entre 10 y 40 veces ms, El tratamiento consiste en la reduccin manual o fijacin quirrgi-
siendo el seminoma el tumor ms frecuente en estos casos. ca del teste afecto y del contralateral. El tiempo de viabilidad del
testculo torsionado es de 46 horas, por lo que siempre se debe
Para el diagnstico de testes impalpables, bilateralmente se deter- considerar una urgencia quirrgica.
mina la LH, FSH y testosterona basales, y luego la testosterona despus
de la estimulacin con HCG durante 3 das. Si existe una ausencia en
la elevacin de la testosterona con elevacin de la LH y FSH, estamos Pregunta 77. Patologa testicular aguda.
ante una anorquia (ausencia bilateral de testculos).
Aproximadamente al ao de edad si la criptorquidia persiste se 7RUVLyQ 2UTXLRHSLGLGLPLWLV
debe realizar tratamiento hormonal. El xito se sita en torno al 20%
en nios menores de 6 aos, y si falla se realizar orquidopexia.
0&
'RORUEUXVFR 0iVSURJUHVLYR
LQWHQVR
Pregunta 74.- R: 4
El caso clnico de esta pregunta es una infeccin de orina (ITU). En
los lactantes la clnica de ITU suele ser muy sutil e inespecfica (ano-
(GDG  0D\RUHV
rexia, nuseas, vmitos, febrcula, estancamiento ponderal,...)
El agente ms frecuente es el E. coli. El diagnstico se realiza mediante
$QWFSHUVRQDOHV
(SLVRGLRVSDUHFLGRV 3URVWDWLVPRFLUXJtD
EDMDVRQGDMHV
el cultivo de orina. El mtodo de recogida en ocasiones es problemti- SUHYLRV SDURWLGLWLV
co (bolsa autoadhesiva en rea genital) porque tiende a contaminarse
con las bacterias de la regin anal. Actualmente se prefiere la puncin
suprapbica. En caso de dudas diagnsticas en la fase aguda se puede (VFURWR ,QIODPDGR" ,QIODPDGR
emplear el DMSA (tambin til para ver cicatrices post pielonefritis).
El tratamiento se realiza preferentemente con Gentamicina iv. Es
7HVWH
'RORURVRHOHYDGR\ 'RORURVRPyYLO
necesario en todo nio/a menor de 5 aos descartar la existencia de KRUL]RQWDOL]DGR
un reflujo vesicoureteral.

Pregunta 75.- R: 3
3UHKQ 1R" 6t
Respecto al RVU, hay que saber que es la anomala congnita ms
frecuente de la unin ureterovesical. Se suele diagnosticar al estudiar +LGURFHOH 6t 0iVIUHFXHQWH
las ITUs.
5HIOHMR
Se clasifican segn la intensidad, el grado de dilatacin ureteral y la
deformidad calicial (grados I-V y reflujo intrarrenal). FUHPDVWpULFR
$QXODGR 3HUPDQHFH
La principal complicacin es la nefropata por reflujo intrarrenal,
que es la causa de hasta un 20% de las insuficiencias renales y la 1 (FR'RSSOHU
0HQRVIOXMRTXHHO ,JXDORPD\RUTXHHO
causa de HTA en la infancia FRQWUDODWHUDO FRQWUDODWHUDO
Para el diagnstico se realiza una cistografa miccional. Se deben
descartar malformaciones anatmicas, evaluar el tamao renal y des-
cartar cicatrices con el DMSA. Pregunta 78.- R: 2
El tratamiento quirrgico se establece en funcin del grado de La CIV es la cardiopata congnita ms frecuente en la infancia. La
reflujo. Habitualmente los reflujos grados I y II se resuelven espont- mayora suelen ser pequeas, situadas en la porcin membranosa del
neamente, en el grado III un 50% precisa tratamiento por va endosc- tabique y que con el tiempo sufrirn un cierre espontneo. En cuanto
pica, y los del grado IV y V precisarn tratamiento quirrgico. a la clnica y tratamiento, es necesario diferenciar entre CIVs peque-
Comentarios TEST

as y grandes:
Pregunta 76.- R: 4
El sndrome hemoltico urmico (SHU) suele presentarse en nios Pregunta 78. Diferencias entre CIV pequeas y grandes.
menores de 4 aos. En los das previos el nio tiene una GEA entero-
invasiva (el agente ms frecuente es el E. coli O-156). El patgeno
produce unas toxinas que favorecen por la lesin a nivel endotelial la 3HTXHxD *UDQGH
agregacin plaquetaria y la formacin de trombos y secundariamente
una anemia hemoltica microangioptica.
&OtQLFD
6RSORSDQVLVWyOLFRIXHUWH 6RSORSDVLVWyOLFROHYH
,&&
El rgano ms afectado en este cuadro es el rin produciendo un $VLQWRPiWLFDV
cuadro de IRA. Otros en menor frecuencia son: intestino, SNC,... &OtQLFDUHVSLUDWRULD
El diagnstico se basa en: 5[GHWyUD[FDUGLRPHJDOLD
Anemia hemoltica microangioptica: haptoglobina descendida 5[WyUD[OHYHSOpWRUD \SOpWRUDSXOPRQDU
con esquistocitos en el frotis de sangre perifrica. 'LDJQyVWLFR
(&*1 (&*FUHFLPLHQWRGH
Plaquetopenia. FDYLGDGHVL]TXLHUGD
IRA: por ecografa se debe diferenciar de la trombosis venosa bila-
teral. 6XHOHQFHUUDUVH &LUXJtDDQWHVGHODxRSDUD
7UDWDPLHQWR HVSRQWiQHDPHQWH HYLWDUHOGHVDUUROORGHKWS
El tratamiento consiste en dilisis peritoneal y medidas de sostn. 3URILOD[LVGHHQGRFDUGLWLV 3URILOD[LVGHHQGRFDUGLWLV
En los casos algo ms complicados se puede realizar plasmafresis o
administracin iv de prostaglandinas.
El pronstico es bueno, siendo raras las recidivas, excepto en los Pregunta 79.- R: 5
casos familiares. El ductus arterioso persistente es una cardiopata tpica de RNPT,
en los que la sntesis local de PGE1 mantiene el ductus abierto en la
Pregunta 77.- R: 2 vida extrauterina, unido a la sobrecarga de volumen.
Estamos ante un cuadro de torsin testicular. Se debe realizar diag- Esta cardiopata congnita tambin se asocia a la rubola cong-
nostico diferencial con la orquioepididimitis. nita.

M exico A rgentina
C hile U ruguay
CTO Medicina C/ Nez de Balboa, 115 28006 MADRID (Espaa) Tfno.: (91) 782 43 32 / Fax: (91) 782 43 27
E-mail: secretaria@ctomedicina.com; iberocto@ctomedicina.com WEB: www.ctomedicina.com; www.iberocto.com
PD Pg. 13
PEDIATRA
Seguimiento a distancia Preparacin Examen de Seleccin 05/06 1 Vuelta
La clnica consiste en un soplo continuo, en maquinaria o de El tipo ms frecuente es la CIA ostium 2, que se caracteriza por un
Gibson, ms audible en 2 espacio intercostal izquierdo; los pulsos segundo ruido amplio y fijo que no se modifica con la respiracin. La
femorales son saltones. Rx de trax suele ser normal o con leves alteraciones (pltora, leve
El nio puede presentar clnica de ICC. cardiomegalia). En el ECG suelen aparecer bloqueos de rama dere-
En la Rx de trax se aprecia pltora pulmonar con aumento de cha, con un patrn rSR en precordiales derechas. El tratamiento es
cavidades izquierdas. El diagnstico definitivo lo obtenemos por eco- necesario si el shunt es 2:1 y se prefiere realizar durante la infancia,
grafa. debido al riesgo de desarrollar hipertensin pulmonar en la edad
En los RNPT, si no existen contraindicaciones, se comienza con la adulta. Otro tipo de CIA menos frecuente es la CIA Ostium 1; una
administracin de indometacina, y si no se soluciona, tratamiento variedad es el llamado canal AV: CIA 1 +CIV+ alteraciones de los
quirrgico. En caso de no ser un RNPT, el tratamiento de entrada es cojinetes endocrdicos. Es tpica del sd. de Down.
quirrgico.
Pregunta 85.- R: 4
Pregunta 80.- R: 4 Estamos ante un RN con hipotiroidismo congnito. La causa ms
La TGA es la cardiopata ciangena ms frecuente de inicio en el frecuente es la disgenesia tiroidea. La clnica tpica del hipotiroidismo
RNPT (recordad que la c. ciangena ms frecuente en conjunto es la no est presente al nacimiento, sino que aparece en el transcurso de
T. De Fallot). semanas. Suele presentar, aparte de la sintomatologa comn con el
LA TGA ms frecuente es la que cursa con un tabique interventri- adulto (somnolencia, estreimiento, hipotermia, mixedema ...), her-
cular ntegro. nia umbilical, ictericia prolongada, fontanelas amplias y macroglosia.
La clnica suele consistir en un RN que en las primeras horas de Si no se instaura tratamiento, con el tiempo desarrollan retraso men-
vida presenta taquipnea y cianosis intensa (conforme se va cerrando tal. El tratamiento consiste en la administracin de hormona tiroidea.
el ductus), no tiene soplo y los pulsos son dbiles. En la Rx de trax se
aprecia un pedculo cardaco estrecho con pltora pulmonar. El ECG Pregunta 86.- R: 2
suele ser de caractersticas normales para un RN. El diagnstico se El cuadro clnico es un nio con hiperplasia suprarrenal congnita
confirma mediante la ecografa. por dficit de 21 hidroxilasa forma pierde sal. En este tipo de altera-
El tratamiento consiste en la perfusin de PGE de forma urgente cin se produce una disminucin de la produccin de aldosterona,
para mantener abierto el ductus, y cuanto antes realizar la correccin cursando la analtica con un sodio srico bajo y un potasio alto.
quirrgica. La tcnica de eleccin es el switch arterial o Jatene, que La estenosis hipertrfica de ploro tambin cursa con vmitos y
consiste en seccionar la salida de los grandes vasos e intercambiarlos. puede producir deshidratacin, pero el patrn inico consiste en
una alcalosis metablica con hipocloremia y potasio bajo.
Pregunta 81.- R: 4 El panhipopituitarismo no tiene porqu tener alteraciones inicas,
La anomala de Ebstein consiste en la insercin anmala (ms infe- puesto que la produccin de aldosterona por la suprarrenal no se
rior) de la valva medial de la tricspide, causando una obstruccin a encuentra alterada.
la salida del ventrculo derecho, y produciendo anatmicamente una El hipotiroidismo puede presentar hiponatremia, pero con niveles
AD grande y un VD pequeo. de potasio normales.
La clnica va a consistir en cianosis, cuyo momento de aparicin va En el hiperaldosteronismo, los iones se encuentran alterados en
a depender del grado de obstruccin a la salida del VD. sentido contrario.
Es frecuente la aparicin de arritmias; la ms tpica es un patrn de
Wolf-Parkinson-White. Pregunta 87.- R: 3
En cuanto a la pubertad precoz (inicio de caracteres sexuales en las
Pregunta 82.- R: 3 nias antes de los 8 aos y en los varones antes de los 9), se debe hacer
La CoA es ms frecuente en nias con sd de Turner. El tipo ms diagnstico diferencial entre dos situaciones: pubertad precoz verda-
frecuente es yuxtaductal. Los nios suelen estar asintomticos. En la dera o central (activacin del eje hipotlamo-hipfiso-gonadal) y la
exploracin es tpico encontrar un soplo sistodiastlico interescapu- pseudopubertad precoz o perifrica (no activacin gonadal inicial). La
lar y pulsos femorales dbiles con HTA en mmss. En la Rx de trax causa ms frecuente de la primera en las nias es la forma idioptica, y
aparecen escotaduras costales en los bordes inferiores de las costillas en los varones suelen ser los tumores del SNC. El tratamiento consiste en
y a veces el llamado signo del 3. En el ECG aparecen signos de hiper- anlogos de GnRH para frenar la pubertad. La pubertad precoz verda-
trofia de VI. El tratamiento se realiza mediante arteriografa con dilata- dera tiene una etiologa muy diversa (ver tabla en pgina siguiente).

Comentarios TEST
cin con baln o mediante ciruga colocando un parche.
Pregunta 88.- R: 3
Pregunta 83.- R: 2 La causa ms frecuente de talla baja en el nio son las dos variantes
Se trata de una tetraloga de Fallot (TF). La TF es la cardiopata de la normalidad.
ciangena ms frecuente. Sus componentes son:
Hipertrofia de VD.
CIV grande. Pregunta 88. Tipos de talla baja no patolgica.
Acabalgamiento de la Ao.
Estenosis infundibular de la pulmonar. &DUDFWHUtVWLFDV 7DOODEDMDIDPLOLDU 5HWUDVRFRQVWLWXFLRQDO
La clnica consiste en cianosis que en ocasiones empeora, produ- &DXVDV 7DOODEDMDHQORVSDGUHV $QWHFHGHQWHVIDPLOLDUHV
ciendo crisis de cianosis secundarias a la contraccin del rodete de
msculo cardiaco que produce la estenosis infundibular. La cianosis, 9HORFLGDGFUHFLPLHQWR 9HORFLGDGFUHFLPLHQWR
como en cualquier cardiopata ciangena, produce policitemia y QRUPDO OHQWD
riesgo de formacin de trombos o infartos a nivel del SNC. 'LDJQyVWLFR (2 (&(WDOOD 3XEHUWDGUHWUDVDGD
En la Rx de trax se aprecia un corazn en forma de zueco e 7DOODILQDO WDOODGLDQD (2 (WDOOD(&
isquemia pulmonar. En el ECG aparece signos de hipertrofia de VD. El 7DOODILQDO WDOODGLDQD
tratamiento es quirrgico; si no es posible por la edad y peso del nio, 7UDWDPLHQWR 1RSUHFLVD 1RSUHFLVD
se puede realizar un shunt sistmico-pulmonar en espera de la co-
rreccin completa.
Pregunta 89.- R: 2
Pregunta 84.- R: 2 Estamos ante un cuadro de hipoglucemia cetsica. Es la causa ms
Las CIA es la forma ms frecuente de cardiopata congnita en la frecuente de hipoglucemia en la infancia. La edad de aparicin es
edad adulta y la segunda en la infancia. entre los 18 meses y 5 aos, siendo un proceso autolimitado. Estos

M exico A rgentina
Pg. 14 PD C hile U ruguay
CTO Medicina C/ Nez de Balboa, 115 28006 MADRID (Espaa) Tfno.: (91) 782 43 32 / Fax: (91) 782 43 27
E-mail: secretaria@ctomedicina.com; iberocto@ctomedicina.com WEB: www.ctomedicina.com; www.iberocto.com
PEDIATRA
Preparacin Examen de Seleccin 05/06 1 Vuelta Seguimiento a distancia

Pregunta 87. Diagnstico de la pubertad precoz.

nios sufren hipoglucemia por mala tolerancia al ayuno prolongado. ticas muy variadas con una mala evolucin clnica hacia el retraso
En la analtica encontramos hipoglucemia hipoinsulinmica y alanina psicomotor y trastornos psquicos. El EEG muestra un patrn basal muy
srica disminuida. En orina hay cuerpos cetnicos positivos. El trata- desestructurado, y el tratamiento consiste en el uso de mltiples
miento consiste en comidas frecuentes. anticomiciales con difcil control de las crisis. El pronstico es malo.

Pregunta 90.- R: 1 Pregunta 95.- R: 3


La causa ms frecuente de convulsiones en el RN es la encefalopata La epilepsia rolndica o de paroxismos centrotemporales es el tipo
hipxico-isqumica. Las crisis suelen ser focales, no generalizadas, puesto de crisis ms frecuente en la infancia. La clnica consiste en sntomas
que el SNC del neonato no est completamente mielinizado. El diag- orofarngeos, de predominio nocturno, que en ocasiones se pueden
nstico se basa en el EEG. El fenobarbital es el frmaco de primera generalizar. En el EEG aparece un foco temporal. En la mayora de las
eleccin. El pronstico depender de la causa desencadenante. ocasiones no precisa tratamiento y su pronstico es excelente.

Pregunta 91.- R: 2 Pregunta 96.- R: 2


Estamos ante un cuadro de petit mal o crisis de ausencia. Se inicia El astrocitoma del nio es el tumor de fosa posterior ms frecuente.
alrededor de los seis aos y la clnica consiste en episodios de breve Suelen ser de bajo grado, qustico y tienden a localizarse en hemisfe-
desconexin con el medio de 5 a 20 segundos con algn automatis- rios cerebelosos. La clnica habitual es la hipertensin intracraneal en
mo. En ocasiones se pueden generalizar a crisis de gran mal. En el EEG el momento del diagnstico. El pronstico es bueno, a diferencia del
aparecen punta onda a 3 ciclos/seg de inicio y final brusco. Las crisis meduloblastoma, que suele afectar al vermis cerebeloso y ser de mal
se pueden desencadenar con la hiperventilacin. Para su tratamiento pronstico.
Comentarios TEST

se emplea valproato o etosuximida.


Pregunta 97.- R: 3
Pregunta 92.- R: 5 El tumor ms frecuente en la infancia es la LLA.
Los espasmos infantiles o s. de West suelen aparecer alrededor de Tumor slido ms frecuente: cerebrales. Fosa anterior craneofarin-
los 4 u 8 meses. Consisten en episodios de flexo-extensin de miem- gioma, posterior astrocitoma.
bros y cabeza cuando el nio se est empezando a dormir o a desper- Tumor slido extracraneal ms frecuente: neuroblastoma.
tar. Se acompaa adems de un enlentecimiento del desarrollo Tumor abdominal ms frecuente: Wilms.
psicomotor. En el EEG es tpico el patrn de hipsarritmia. En la etiolo- Tumor de partes blandas ms frecuentes: rabdomiosarcoma.
ga existe un grupo sintomtico, donde hay un dao cerebral previo o Tumor orbitario ms frecuente: rabdomiosarcoma.
una sintomatologa conocida, siendo stos los de peor pronstico. Un Tumor ocular ms frecuente: retinoblastoma.
segundo grupo criptognico donde el nio es normal hasta el inicio
de las crisis. Para el tratamiento se usa ACTH. Pregunta 98.- R: 3
El cuadro que se expone es una nia con un neuroblastoma. La
Pregunta 93.- R: 2 edad de aparicin tpica es en menores de 2 aos. Se suele localizar en
El cuadro clnico hace referencia a un nio con una convulsin el abdomen a nivel de las glndulas suprarrenales y la clnica ms fre-
febril. La clnica tpica consiste en fiebre alta, crisis tnico-clnica cuente es la masa abdominal que cruza lnea media. Entre los sndromes
generalizada de breve duracin con un periodo postcrtico corto. La paraneoplsicos (no modifican el pronstico) figura el opsoclonus-
edad de presentacin es entre los 6 meses y 5 aos. Existen antece- mioclonus (caso clnico) y la diarrea secretora. Para el diagnstico se
dentes familiares hasta en un 30% de los casos. El tratamiento en la utiliza la TC craneal, catecolaminas en orina aumentadas y la
fase aguda consiste en la administracin de diacepam intrarrectal y gammagrafa con MIBG. El tratamiento depende del estadio, realizn-
medidas para disminuir la hipertermia. dose ciruga, quimioterapia y radioterapia. La supervivencia es del 50%.

Pregunta 94.- R: 3 Pregunta 99.- R: 4


Respecto al sndrome de Lennox-Gastaut, hay que decir que tiene un El tumor de Wilms se asocia con anomalas genitourinarias, hemi-
pico de incidencia entre uno y siete aos. Consiste en crisis de caracters- hipertrofia y aniridia. Se han encontrado deleciones en el cromoso-

M exico A rgentina
C hile U ruguay
CTO Medicina C/ Nez de Balboa, 115 28006 MADRID (Espaa) Tfno.: (91) 782 43 32 / Fax: (91) 782 43 27
E-mail: secretaria@ctomedicina.com; iberocto@ctomedicina.com WEB: www.ctomedicina.com; www.iberocto.com
PD Pg. 15
PEDIATRA
Seguimiento a distancia Preparacin Examen de Seleccin 05/06 1 Vuelta
ma 11. La edad al diagnstico es en nias menores de tres aos. La
clnica ms frecuente es la masa abdominal asintomtico. Otros como
HTA, hematuria, etc. La localizacin ms frecuente de las metstasis es
el pulmn. El diagnstico se basa en la TC y la PAAF, no aconsejndo-
se la realizacin de biopsia.

Pregunta 100.- R: 3
El retinoblastoma es un tumor tpico de nios pequeos. La clnica
habitual es la presentacin de leucocoria. Los casos familiares se aso-
cian con afectacin bilateral. El tratamiento en estos casos consiste en
enucleacin de ojo ms afectado y lser o radioterapia local en el
otro. En casos de tumor unilateral, el tratamiento de eleccin es la
enucleacin. Se asocia con el cromosoma 13, y ello confiere la posi-
bilidad de aparicin de otros tumores posteriormente como el
osteosarcoma.

Comentarios TEST

M exico A rgentina
Pg. 16 PD C hile U ruguay
CTO Medicina C/ Nez de Balboa, 115 28006 MADRID (Espaa) Tfno.: (91) 782 43 32 / Fax: (91) 782 43 27
E-mail: secretaria@ctomedicina.com; iberocto@ctomedicina.com WEB: www.ctomedicina.com; www.iberocto.com
PSIQUIATRA
Preparacin Examen de Seleccin 05/06 1 Vuelta Seguimiento a distancia
TRASTORNOS NEURTICOS. 7. Cul de las opciones de tratamiento parece la ms potente para
un paciente dominado por la necesidad de comprobar cada
1. Una paciente presenta desde su adolescencia episodios recu- pequeo acto cotidiano un nmero fijo de veces, hecho ste
rrentes de palpitaciones y mareo con sensacin de prdida de que le provoca gran ansiedad, pero que dice ser incapaz de
estabilidad; ocasionalmente ha sufrido desvanecimientos; tras controlar?:
repetidas exploraciones somticas negativas comienza trata-
miento psiquitrico. Cul de los siguientes cuadros es RARO 1) Psicoanlisis.
que complique el trastorno que sufre la paciente?: 2) Sertralina.
3) Fenelcina.
1) Abuso de sedantes. 4) Loracepam.
2) Trastorno obsesivo. 5) Clomipramina.
3) Depresin.
4) Alcoholismo. 8. Uno de los siguientes sntomas NO forma parte de los criterios
5) Ansiedad crnica. diagnsticos del trastorno por estrs postraumtico:

2. Ante un paciente que est en plena crisis de angustia, una de las 1) Pesadillas recurrentes con el suceso traumtico.
siguientes medidas es INEFICAZ: 2) Amnesia de detalles importantes del acontecimiento.
3) Estado de nimo depresivo.
1) Buspirona. 4) Respuesta de sobresalto exagerada.
2) Loracepam sublingual. 5) Evitacin de los lugares relacionados con el suceso.
3) Control de la respiracin.
4) Diacepam oral. 9. La presencia de mltiples sntomas fsicos para los que no se
5) Explicacin de la benignidad del cuadro. encuentra una causa evidente define al trastorno conocido
como:
3. Cul de las situaciones que se mencionan es MENOS probable
que desencadene miedo en un agorafbico?: 1) Dismorfofobia.
2) Hipocondra.
1) Metro. 3) Sndrome de Briquet.
2) Centros comerciales. 4) Conversin histrica.
3) Hablar en pblico. 5) Trastorno facticio.
4) Ascensores.
10. En la clasificacin DSM, NO se incluye dentro de los trastornos
5) Aglomeraciones.
disociativos:
4. La fobia social es ms DIFCIL de diferenciar de:
1) Amnesia.
2) Fuga.
1) Trastorno esquizoide.
3) Personalidad mltiple.
2) Distimia.
4) Despersonalizacin.
3) Personalidad evitativa.
5) Convulsiones.
4) Agorafobia.
5) Esquizofrenia residual. 11. Las benzodiacepinas ejercen su accin:
5. NO es frecuente que los pacientes obsesivos estn preocupa- 1) Activando el receptor serotoninrgico 5HT1a.
dos con: 2) Modificando la permeabilidad neuronal al calcio.
3) Potenciando la funcin GABArgica.
1) La contaminacin.
Preguntas TEST

4) Interfiriendo con los sistemas de segundo mensajero.


2) La duda. 5) Bloqueando la recaptacin noradrenrgica.
3) La simetra.
4) La soledad. 12. Ante un paciente en el que sospechamos una intoxicacin por
5) La enfermedad. alprazolam, el tratamiento de eleccin es:

6. En la etiologa del trastorno obsesivo NO es cierto: 1) Naloxona.


2) Flumacenil.
1) Existe una asociacin fuerte entre el trastorno de la Tourette 3) N-acetilcistena.
y el TOC. 4) Trihexifenidilo.
2) Algunos casos infantiles se relacionan con infecciones estrep- 5) Fisostigmina.
toccicas.
3) Hay una disfuncin de los circuitos corticosubcorticales 13. En los pacientes con hepatopata, uno de los siguientes ansio-
frontales. lticos est CONTRAINDICADO:
4) El elemento psicopatolgico ms tpico es la sensacin de
duda. 1) Oxacepam.
5) Se ha demostrado una alteracin selectiva de la neurotrans- 2) Loracepam.
misin noradrenrgica. 3) Clormetiazol.

M exico A rgentina CTO Medicina C/ Nez de Balboa, 115 28006 MADRID (Espaa) Tfno.: (91) 782 43 32 / Fax: (91) 782 43 27
C hile U ruguay E-mail: secretaria@ctomedicina.com; iberocto@ctomedicina.com WEB: www.ctomedicina.com; www.iberocto.com
PQ Pg. 1
PSIQUIATRA
Seguimiento a distancia Preparacin Examen de Seleccin 05/06 1 Vuelta
4) Tiapride. 3) Somnolencia excesiva diurna.
5) Cloracepato dipotsico. 4) Frecuentes despertares nocturnos.
5) Sueo no reparador.
14. Una de las benzodiacepinas hipnticas que se relacionan a
continuacin se utiliza exclusivamente para el insomnio de 20. Se describe clsicamente que los pacientes depresivos "end-
conciliacin por su corta vida media. Selela: genos" mejoran:

1) Triazolam. 1) Por la maana.


2) Brotizolam. 2) Con IMAOs ms que con tricclicos.
3) Lormetacepam. 3) En relacin con factores externos.
4) Loprazolam. 4) Al terminar el da.
5) Flunitracepam. 5) En verano.

21. La presencia de un sndrome depresivo leve y autolimitado tras


TRASTORNOS AFECTIVOS, FRMACOS ANTIDEPRESIVOS Y la prdida de un ser querido recibe el nombre de:
ESTABILIZADORES DEL HUMOR.
1) Melancola.
15. La forma ms frecuente de trastorno del estado de nimo es, de 2) Depresin reactiva.
entre las siguientes: 3) Duelo no complicado.
4) Depresin mayor.
1) Enfermedad bipolar. 5) Neurosis depresiva.
2) Distimia.
3) Trastorno ciclotmico. 22. Uno de los siguientes delirios NO se considera congruente con
4) Depresin mayor. las enfermedades afectivas:
5) Melancola.
1) Ruina.
16. El concepto de depresin mayor incluye en el DSM-IV diversos 2) Influencia.
subtipos de depresiones, entre los cuales NO se incluye: 3) Grandeza.
4) Enfermedad.
1) Depresin melanclica. 5) Culpa.
2) Depresin psictica.
3) Depresin atpica. 23. Un paciente de 62 aos lleva un mes prcticamente sin realizar
4) Depresin neurtica. ninguna actividad, no tiene ganas ms que de estar en la cama y
5) Depresin puerperal. dice no tener fuerzas ni para acabar con su vida. Asimismo, est
convencido de estar arruinado a pesar de que, segn su familia,
17. Seale el trastorno afectivo que sufre una paciente de 35 aos mantienen una economa desahogada. El personal de enfermera
que desde su adolescencia experimenta de forma casi constan- ha observado que est algo ms activo por la noche y que coge
te sntomas depresivos de intensidad leve, asociados con una bien el sueo, pero se despierta a las 3 de la madrugada. Tras
sensacin de cansancio enorme y una clara tendencia al sueo, recibir tratamiento antidepresivo con imipramina empieza a
y cuyos sntomas apenas desaparecen unas semanas al ao, estar ms inquieto e irritable, hablar durante horas seguidas y con
empeorando de forma dramtica cuando la paciente sufre un tendencia a la procacidad y a la desinhibicin sexual. Qu
problema sentimental: debemos considerar en primer lugar?:

1) Depresin mayor. 1) Que presenta una reaccin psicolgicamente normal tras la


2) Trastorno mixto ansioso-depresivo. recuperacin de la depresin.
3) Personalidad depresiva. 2) Que padece un trastorno bipolar, desconocido hasta ese

Preguntas TEST
4) Neurastenia. momento.
5) Distimia atpica. 3) Que el tratamiento antidepresivo est induciendo un cuadro
maniforme.
18. Dentro de los cuadros depresivos del anciano, NO es frecuente 4) Que estn apareciendo rasgos patolgicos de personalidad,
que encontremos: ocultos por la depresin.
5) Que est simulando, pues esos sntomas no son congruentes
1) Predominio de los sntomas somticos.
en un trastorno afectivo.
2) Quejas cognitivas.
3) Alto riesgo de suicidio.
24. Dentro de las diferencias entre el trastorno bipolar y la depre-
4) Frecuente ideacin delirante.
sin recurrente, NO encontramos:
5) Mala respuesta al electrochoque.
1) Inicio a una edad ms tarda en el depresivo.
19. De las alteraciones del sueo, cul es la que refieren los
2) Ms antecedentes familiares afectivos en el bipolar.
pacientes con depresin atpica?:
3) Duracin ms larga de las recadas en el depresivo.
1) Insomnio de conciliacin. 4) Alto riesgo de suicidio en el bipolar.
2) Despertar precoz. 5) Equilibrio entre sexos en el depresivo.

M exico A rgentina CTO Medicina C/ Nez de Balboa, 115 28006 MADRID (Espaa) Tfno.: (91) 782 43 32 / Fax: (91) 782 43 27
Pg. 2 PQ C hile U ruguay E-mail: secretaria@ctomedicina.com; iberocto@ctomedicina.com WEB: www.ctomedicina.com; www.iberocto.com
PSIQUIATRA
Preparacin Examen de Seleccin 05/06 1 Vuelta Seguimiento a distancia
25. NO sugiere "bipolaridad" en un paciente depresivo: 3) Mantener al menos 6 meses la misma dosis que produjo la
mejora.
1) Debut en el puerperio. 4) Dejar una dosis mnima y eficaz para prevenir recadas de
2) Antecedentes familiares de mana. forma indefinida.
3) Presencia de ideacin delirante incongruente. 5) Comentar al paciente que tome el frmaco segn se encuentre.
4) Inicio en la adolescencia.
5) Provocacin de hipomanas con antidepresivos. 32. Los antidepresivos tricclicos renen las siguientes caracters-
ticas, EXCEPTO:
26. Se llama trastorno bipolar 2 a la siguiente combinacin de
sndromes afectivos: 1) Control a travs de niveles plasmticos.
2) Bajo precio.
1) Depresin mayor y mana. 3) Importantes efectos anticolinrgicos.
2) Depresin mayor sobre una distimia. 4) Alta letalidad en sobredosis.
3) Depresin mayor e hipomana. 5) Inicio del tratamiento a dosis teraputicas.
4) Mana "unipolar" (sin depresin asociada).
5) Depresin invernal y episodios hipertmicos en verano. 33. Los ISRS son los antidepresivos ms usados en la actualidad, entre
otras razones porque no producen apenas efectos secundarios
27. Cul de los siguientes pacientes presenta un mayor riesgo de graves; sin embargo, muchos pacientes se van a quejar de:
suicidio?:
1) Disfunciones sexuales.
1) Varn, 25 aos, con distimia de inicio precoz, sin anteceden- 2) Somnolencia.
tes de suicidio. 3) Estreimiento.
2) Mujer, 46 aos, con depresin endgena unipolar. 4) Poliuria.
3) Mujer, 85 aos, con depresin reactiva tras su reciente 5) Sequedad de piel y mucosas.
mudanza.
4) Varn, 72 aos, con melancola delirante. 34. Los IMAOs son el grupo de antidepresivos menos utilizado en
5) Mujer, 19 aos, con depresin bipolar y antecedentes la actualidad; sin embargo existen algunas indicaciones para las
familiares de suicidio. que son el frmaco de eleccin; seale una de ellas:

28. Las listas de frmacos capaces de causar depresin son muy 1) Narcolepsia.
largas; sin embargo, uno de los siguientes NO se asocia con 2) Trastorno obsesivo.
depresin sino con mana: 3) Pnico.
4) Depresin atpica.
1) Propranolol. 5) Bulimia.
2) Isoniacida.
3) Neurolpticos. 35. El tratamiento electroconvulsivo est especialmente indicado
4) Reserpina. en uno de los siguientes pacientes:
5) Anticonceptivos hormonales.
1) Distimia, refractaria al tratamiento farmacolgico.
29. En una de las siguientes indicaciones NO se utilizan por el 2) Depresin mayor delirante.
momento como estrategia teraputica los antidepresivos: 3) Trastorno bipolar en fase mixta.
4) Ciclador rpido.
1) Trastorno por angustia. 5) Depresin "doble".
2) Narcolepsia-catapleja.
3) Dolor crnico neuroptico. 36. A la hora de iniciar un tratamiento con litio, una de las pruebas
Preguntas TEST

4) Cefalea tensional. siguientes resulta INNECESARIA:


5) Abstinencia alcohlica.
1) Test de embarazo.
30. A la hora de seleccionar un antidepresivo para el tratamiento de un 2) Sodio y potasio.
paciente, cul termina siendo el principal factor implicado?: 3) Electrocardiograma.
4) Electroencefalograma.
1) Los antecedentes familiares de respuesta a un frmaco. 5) Creatinina y urea.
2) El subtipo sintomtico de depresin.
3) El antecedente de una respuesta personal al frmaco. 37. Una de las siguientes afirmaciones acerca de las cifras de litemia
4) El perfil potencial de efectos adversos. es FALSA:
5) Las preferencias del paciente.
1) Niveles inferiores a 0,4 mEq/L son, en general, ineficaces.
31. Una vez superada la fase aguda de una depresin, se recomienda: 2) Niveles superiores a 1,5 mEq/L se asocian con toxicidad.
3) Durante el perodo de mantenimiento suele bastar con
1) Retirar el tratamiento antidepresivo. litemias inferiores a 1,0 mEq/L.
2) Reducir la dosis a la mitad y mantener el tratamiento 6 4) En las fases agudas manacas se recomiendan litemias supe-
semanas ms. riores a 1,0 mEq/L.

M exico A rgentina CTO Medicina C/ Nez de Balboa, 115 28006 MADRID (Espaa) Tfno.: (91) 782 43 32 / Fax: (91) 782 43 27
C hile U ruguay E-mail: secretaria@ctomedicina.com; iberocto@ctomedicina.com WEB: www.ctomedicina.com; www.iberocto.com
PQ Pg. 3
PSIQUIATRA
Seguimiento a distancia Preparacin Examen de Seleccin 05/06 1 Vuelta
5) En las fases agudas depresivas se recomiendan litemias infe- 4) Ausencia de factores psicosociales precipitantes.
riores a 1,0 mEq/L. 5) Cambios afectivos y reduccin de la sociabilidad.

38. Un paciente bipolar en tratamiento con litio ingresa en Urgen- 43. Cul de los siguientes factores NO se incluye dentro de las
cias por un sndrome confusional; al determinar la litemia se causas de vulnerabilidad a la esquizofrenia?:
obtiene una cifra de 3,1 mEq/l, cuando haca una semana su
litemia era de 1,0 mEq/l; suponiendo que el paciente no haya 1) Antecedentes familiares.
variado la dosis prescrita, cul de las siguientes circunstancias 2) Problemas obsttricos.
explicara esta cifra?: 3) Consumo de txicos.
4) Infecciones virales.
1) Tratamiento concomitante con carbamacepina. 5) Enfermedades infantiles.
2) Uso de nifedipino como antihipertensivo.
3) Sobreingesta hdrica. 44. Un varn de 24 aos es llevado a urgencias por presentar gran
4) Toma de indometacina por un esguince. angustia. Sus padres comentan que llevan notndole raro desde
5) Prescripcin de teofilina para una enfermedad pulmonar hace unos 8 meses; ha dejado de salir con sus amigos y de
obstructiva crnica. estudiar, y parece "encerrado en s mismo". Los dos ltimos meses
tiene una mirada "rara", segn su madre, quien lo ha encontrado
39. Una paciente de 43 aos, diagnosticada desde los 22 aos de hablando slo en numerosas ocasiones. En Urgencias el paciente
trastorno bipolar y sin recadas hasta el ltimo ao tomando te comenta angustiado que est siendo sometido a una vigilancia
exclusivamente litio, durante este ao ha presentado 5 episo- y persecucin asfixiante por todas aquellas personas que van en
dios afectivos que su psiquiatra relaciona con la instauracin los coches con matrcula acabada en M, pues esa letra significa
de paroxetina ante fases depresivas leves y finalmente ha "Morirs pronto". Cmo se denomina este ltimo fenmeno
psicopatolgico?:
diagnosticado a la paciente de cicladora rpida. En estos mo-
mentos est ligeramente deprimida. Cul de las siguientes
1) Bloqueo del pensamiento.
opciones teraputicas te parece ms adecuada?:
2) Descarrilamiento.
3) Pensamiento disgregado.
1) Suspender el litio e instaurar el topiramato como base de su
4) Percepcin delirante.
tratamiento.
5) Idea delirante secundaria.
2) Mantener el litio y aadir carbamacepina o valproico.
3) Quitar el litio y aadir el valproico y un ISRS, pues la paciente
45. Seala cul de los siguientes sntomas NO se incluye entre los
est deprimida. sntomas de primer rango de la esquizofrenia:
4) Mantener el litio y aadir olanzapina.
5) Quitar el litio y realizar terapia de mantenimiento con TEC. 1) Eco del pensamiento.
2) Alucinaciones visuales.
40. Uno de los siguientes pacientes NO presenta una mala respues- 3) Robo del pensamiento.
ta al litio: 4) Vivencias de influencia.
5) Delirios de control.
1) Mana disfrica.
2) Recadas estacionales. 46. Cul de los sntomas siguientes aparece con mayor claridad en
3) Mana secundaria. la fase residual de la esquizofrenia?:
4) Ciclacin rpida.
5) Fase bipolar mixta. 1) Descarrilamientos.
2) Percepciones delirantes.
3) Aplanamiento afectivo.
TRASTORNOS PSICTICOS. FRMACOS ANTIPSICTICOS. 4) Pseudoalucinaciones auditivas.

Preguntas TEST
5) Conducta catatnica.
41. Uno de los siguientes datos acerca de la epidemiologa de la
esquizofrenia es FALSO: 47. El debut precoz de la esquizofrenia se asocia con un peor
pronstico de la enfermedad, como todos los datos siguientes,
1) La prevalencia-vida es del 1%. EXCEPTO uno:
2) En varones se describe un comienzo ms precoz.
3) Hay una mayor frecuencia de enfermedad en clases sociales 1) Sexo masculino.
bajas. 2) Antecedentes familiares de depresin.
4) Existe un exceso de esquizofrnicos que han nacido en los 3) Personalidad previa paranoide.
meses fros. 4) Escasa productividad psictica.
5) La esquizofrenia duplica el riesgo de muerte. 5) Ausencia de desencadenantes.
42. El concepto actual de esquizofrenia se apoya en los siguientes
48. La probabilidad de recada en un paciente esquizofrnico NO
datos SALVO:
est en relacin con:
1) Curso habitualmente crnico y deteriorante.
1) Su conciencia de enfermedad.
2) Presencia de crisis alucinatorias y delirantes.
2) Los efectos secundarios del tratamiento.
3) Duracin superior a 6 meses.

M exico A rgentina CTO Medicina C/ Nez de Balboa, 115 28006 MADRID (Espaa) Tfno.: (91) 782 43 32 / Fax: (91) 782 43 27
Pg. 4 PQ C hile U ruguay E-mail: secretaria@ctomedicina.com; iberocto@ctomedicina.com WEB: www.ctomedicina.com; www.iberocto.com
PSIQUIATRA
Preparacin Examen de Seleccin 05/06 1 Vuelta Seguimiento a distancia
3) El tipo de esquizofrenia. 1) Biperideno.
4) El cumplimiento de la medicacin. 2) Amantadina.
5) La "emocin expresada" en la familia. 3) Dantrolene.
4) Loracepam.
49. A diferencia de la esquizofrenia, la paranoia presenta entre sus 5) Propranolol.
caractersticas:
55. Dentro de las estrategias actuales de tratamiento de la esquizo-
1) Inicio en edad juvenil. frenia, el TEC sigue teniendo un papel en:
2) Escaso deterioro.
3) Tendencia a la cronicidad. 1) Formas hebefrnicas.
4) Delirios de perjuicio. 2) Esquizofrenia catatnica.
5) Alucinaciones. 3) Primeros brotes.
4) Pacientes con graves sntomas extrapiramidales.
50. Un compaero de tu trabajo comienza a sospechar de la 5) Falta de cumplimiento del tratamiento.
fidelidad de su novia, quien trabaja en otro departamento de la
empresa, cercano al tuyo; cada vez que coincids en la cafetera 56. Dentro de las opciones de abordaje de tipo psicosocial de la
y os ve hablando, tiene la certeza de que estaris quedando para esquizofrenia, se considera DESCARTADO en el momento
ms adelante y si no hablis cree que se debe a que os habis actual:
sentido vigilados; cmo se denomina este fenmeno psicopa-
tolgico?: 1) Terapia de modificacin de conducta.
2) Psicoeducacin.
1) Formacin reactiva. 3) Grupos de autoayuda de familias.
2) Anulacin. 4) Psicoterapias dinmicas.
5) Centros de rehabilitacin psicosocial.
3) Interpretacin delirante.
4) Aislamiento del afecto.
5) Fantasa.
TRASTORNOS RELACIONADOS CON SUSTANCIAS.
51. Cul es el mecanismo de accin principal de los antipsicticos
57. Dentro de las complicaciones asociadas al consumo agudo de
"clsicos"?:
alcohol, hay una que est expresada de forma INCORRECTA:
1) Bloqueo serotoninrgico 5HT1a.
1) La intoxicacin idiosincrsica se produce tras un consumo
2) Estmulo GABArgico. mnimo de alcohol.
3) Inhibicin de la COMT. 2) Existe buena correlacin entre la alcoholemia y los efectos
4) Antidopaminrgico D2. neuropsiquitricos.
5) Anticolinrgico muscarnico. 3) El tratamiento de la intoxicacin es sintomtico.
4) La sobredosis de alcohol puede ser letal.
52. Una de las siguientes afirmaciones sobre los antipsicticos es 5) Con frecuencia vemos hiperglucemia como consecuencia
FALSA: de la intoxicacin.

1) Los antipsicticos "tpicos" son todos igual de eficaces. 58. Un paciente ingresado por un traumatismo craneal presenta en
2) La potencia de un antipsictico se refiere a su accin antido- la noche del primer da de ingreso un episodio de agitacin
paminrgica. psicomotriz, encontrndose sudoroso y con temblor grosero;
3) Cuanto ms potente es un antipsictico ms hipotensin dice estar viendo cientos de insectos subiendo por las sbanas
produce. de la cama y en la exploracin se encuentra desorientado en
4) Los efectos extrapiramidales de los antipsicticos de baja tiempo y espacio; uno de los hallazgos siguientes sera INCOM-
Preguntas TEST

potencia son escasos. PATIBLE con el cuadro que padece:


5) La galactorrea se produce por afectacin del sistema tubero-
infundibular. 1) VCM elevado.
2) GGT mayor de 300.
53. El mecanismo de accin que parece aumentar la eficacia de la 3) Descenso de la CDT (desialotransferrina).
risperidona y reducir sus efectos extrapiramidales es: 4) Hipertermia.
5) Convulsiones tnico-clnicas.
1) Bloqueo de la recaptacin de dopamina.
2) Antagonismo del receptor H1. 59. Dentro de las alternativas farmacolgicas para el tratamiento
3) Potenciacin de la funcin GABArgica. de deshabituacin a largo plazo del paciente alcohlico, NO
4) Bloqueo D2 y antagonismo 5HT2a. utilizamos:
5) Estimulacin D1 y D4.
1) Naltrexona.
54. Tras una inyeccin de un psicofrmaco un paciente presenta 2) Diacepam.
un movimiento involuntario de los ojos hacia arriba, que le 3) Tiapride.
resulta doloroso y se mantiene a pesar de sus esfuerzos por 4) Acamprosato.
evitarlo; qu tratamiento precisa?: 5) Cianamida.

M exico A rgentina CTO Medicina C/ Nez de Balboa, 115 28006 MADRID (Espaa) Tfno.: (91) 782 43 32 / Fax: (91) 782 43 27
C hile U ruguay E-mail: secretaria@ctomedicina.com; iberocto@ctomedicina.com WEB: www.ctomedicina.com; www.iberocto.com
PQ Pg. 5
PSIQUIATRA
Seguimiento a distancia Preparacin Examen de Seleccin 05/06 1 Vuelta
60. De las relaciones siguientes entre frmacos y su uso en la 65. En un hospital numerosos enfermos se encuentran en riesgo de
dependencia de opiceos, una es FALSA: sufrir un delirium; en el estudio de este sndrome NO resulta til
una de las siguientes pruebas:
1) Naloxona tratamiento de la intoxicacin (sobredosis).
2) Metadona tratamiento de mantenimiento con opiceos. 1) Hemograma.
3) Naltrexona tratamiento de deshabituacin con antagonistas. 2) TAC craneal.
4) Clonidina tratamiento de la dependencia mixta de herona 3) Test de Rorscharch.
y cocana. 4) Anlisis de orina.
5) Guanfacina tratamiento de la abstinencia (desintoxicacin) 5) ECG.
sin opiceos.
66. Una paciente de 70 aos acude a su consulta refiriendo fre-
61. Un paciente aparece en su consulta demandando tratamiento cuentes prdidas de objetos cotidianos y ocasional desorienta-
pues presenta dolores generalizados, sudoracin, hipertermia, cin en calles poco familiares; qu prueba de entre las siguien-
nuseas y vmitos; a la exploracin detectamos marcas cut- tes NO utilizara en el estudio inicial de este caso de deterioro
neas en forma de ral y cicatrices de abscesos subcutneos en cognitivo?:
brazos y piernas; no parecen existir antecedentes mdicos
relevantes, qu actitud deberemos tomar?: 1) Hemograma y bioqumica plasmtica.
2) Niveles de hormonas tiroideas.
1) Remitir al paciente a la Unidad de Hospitalizacin Psiqui- 3) TC craneal.
trica ms cercana. 4) Polisomnografa (estudio de la latencia REM).
2) Derivar al paciente a un centro especializado en drogode- 5) Mini-Examen Cognoscitivo (MEC) de Lobo.
pendientes.
3) Administrarle midazolam intravenoso como medicacin 67. La prdida de funciones superiores que aparece asociada a la
sedante. enfermedad de Huntington se clasifica como:
4) Ponerle un goteo de naloxona tras dos ampollas i.v. iniciales.
5) Iniciar tratamiento con naltrexona y clonidina. 1) Reversible.
2) Irreversible, pero con tratamiento.
62. Cul de los siguientes frmacos NO se usa para intentar reducir 3) Subcortical.
el riesgo de recadas en consumidores crnicos de cocana?: 4) Cortical.
5) Axial.
1) Bromocriptina.
2) Desipramina. 68. La presencia de demencia, alteraciones de la marcha, parkinso-
3) Propranolol. nismo e incontinencia urinaria orienta hacia el diagnstico de:
4) L-dopa.
5) Amantadina. 1) Enfermedad de Pick.
2) Hidrocefalia normotensiva.
3) Enfermedad de Alzheimer.
TRASTORNOS COGNOSCITIVOS: DELIRIUM, DEMENCIAS Y 4) Demencia vascular.
AMNESIAS. 5) Demencia con cuerpos de Lewy.

63. Un varn de 75 aos es ingresado para una reseccin prosttica 69. En el diagnstico diferencial entre depresin y demencia resulta
transuretral; la primera noche tras la operacin se muestra FALSO que:
inquieto, creyendo reconocer en las enfermeras a los compa-
eros que tuvo durante el Servicio Militar e intentando levan- 1) Las alteraciones de las pruebas de neuroimagen sean alta-
tarse de su cama; al ser retenido por la fuerza comienza a gritar mente inespecficas en ambas enfermedades.

Preguntas TEST
llamando a la Legin pues dice estar secuestrado; qu NO ser 2) Los pacientes depresivos tiendan a mejorar por la noche,
lgico encontrar acompaando a este sndrome?: mientras que los dementes suelen mejorar por la maana.
3) Los pacientes con demencia suelen exagerar sus fallos cog-
1) Empeoramiento nocturno. nitivos y los depresivos minimizarlos.
2) Alucinaciones visuales. 4) Los cambios en las pruebas neuropsicolgicas son incon-
3) Agitacin psicomotriz. gruentes en la depresin y consistentes en las demencias.
4) Preservacin de la capacidad de atencin. 5) La mitad de las llamadas pseudodemencias depresivas desa-
5) Fluctuaciones de la clnica. rrollarn a medio o largo plazo una verdadera demencia.

64. Qu antipsictico de los siguientes se recomienda para el


control de la agitacin psicomotriz en los casos de delirium?: OTROS TRASTORNOS MENTALES: PERSONALIDAD, ALIMEN-
TACIN, SUEO, INFANTILES.
1) Tioridacina.
2) Sulpiride. 70. Una chica de 17 aos es llevada al servicio de urgencias por sus
3) Zuclopentixol. padres porque la han encontrado en su habitacin realizndose
4) Haloperidol. cortes en los antebrazos. La paciente dice hacerse los cortes
5) Clorpromacina. para sentir que existo y expresa sentimientos crnicos de

M exico A rgentina CTO Medicina C/ Nez de Balboa, 115 28006 MADRID (Espaa) Tfno.: (91) 782 43 32 / Fax: (91) 782 43 27
Pg. 6 PQ C hile U ruguay E-mail: secretaria@ctomedicina.com; iberocto@ctomedicina.com WEB: www.ctomedicina.com; www.iberocto.com
PSIQUIATRA
Preparacin Examen de Seleccin 05/06 1 Vuelta Seguimiento a distancia
vaco. Es conocida en el Servicio de Urgencias por los numerosos 1) Betabloqueantes.
intentos autolticos que ha realizado mediante la ingestin de 2) Clonacepam.
pastillas. Refiere tener atracones ocasionalmente y sus padres 3) Carbamacepina y valproico.
estn asustados por su promiscuidad sexual. Durante la entre- 4) Clomipramina.
vista, dice en ocasiones que se quiere ir de vacaciones, para poco 5) Sistemas de ventilacin positiva.
despus ponerse a llorar y expresar deseos de suicidio. Cul es
el trastorno de personalidad que padece?: 76. En uno de los siguientes trastornos de la infancia, el tratamiento
con antipsicticos es de eleccin:
1) Personalidad obsesiva.
2) Personalidad antisocial. 1) Dficit de atencin con hiperactividad.
3) Personalidad pasivo-agresiva. 2) Autismo.
4) Personalidad lmite. 3) Mutismo selectivo.
5) Personalidad depresiva. 4) Sndrome de la Tourette.
5) Enuresis nocturna.
71. Una persona que lleva una vida bastante marginal, con un
repertorio limitado de intereses, una llamativa ausencia de 77. Carlos es un nio de 7 aos que sufre con frecuencia castigos
relaciones sociales y con una respuesta emocional escasa, en clase por no aceptar la disciplina; pasa las horas de colegio
presenta una personalidad: levantndose continuamente del pupitre y molestando a sus
compaeros, que han comenzado a hacerle el vaco pues dicen
1) Esquizoide. que es muy bruto; en casa se comporta de forma parecida,
2) Esquizoafectiva. habiendo destrozado todos sus juguetes y sufriendo con fre-
3) Esquizofrnica. cuencia accidentes, pues no mide correctamente el riesgo; qu
4) Esquizotpica. problema presenta con mayor probabilidad?:
5) Esquizofreniforme.
1) Retraso mental.
72. Una adolescente se desmaya en clase de gimnasia, tras lo que 2) Trastorno disocial de la personalidad.
es llevada al botiqun del instituto; all se detecta una tensin 3) Conducta oposicionista.
arterial de 70/45 mmHg; la chica est muy delgada, detalle que 4) Dficit de atencin con hiperactividad.
no habamos notado hasta la exploracin fsica; confrontada 5) Depresin enmascarada.
con estos datos, cul de las siguientes conductas NO sera
lgica en esta paciente?:

1) Preocupacin por el aspecto fsico y la esttica.


2) Alto rendimiento acadmico.
3) Tendencia a vestirse con ropas ajustadas y ligeras.
4) Uso excesivo de laxantes.
5) Evitacin de las reuniones sociales en donde pueda verse
forzada a comer en pblico.

73. Dentro de las caractersticas del atracn bulmico, NO se


encuentra:

1) Tensin creciente antes del atracn.


2) Relacin desproporcionada ingesta/tiempo.
3) Sensacin de prdida de control.
Preguntas TEST

4) Seleccin de alimentos hipocalricos.


5) Aumento de la frecuencia por la noche.

74. Al revisar la polisomnografa de su paciente encuentra un


perodo con ondas muy lentas en el EEG, asocindose con un
tono muscular bajo y sin movimientos oculares; qu trastorno
del sueo se asocia a esta fase?:

1) Bruxismo.
2) Apnea obstructiva del sueo.
3) Enuresis.
4) Sonambulismo.
5) Pesadillas.

75. La prdida brusca del tono muscular en respuesta a una


emocin es un sntoma casi patognomnico de un trastorno;
cul es el tratamiento del mismo?:

M exico A rgentina CTO Medicina C/ Nez de Balboa, 115 28006 MADRID (Espaa) Tfno.: (91) 782 43 32 / Fax: (91) 782 43 27
C hile U ruguay E-mail: secretaria@ctomedicina.com; iberocto@ctomedicina.com WEB: www.ctomedicina.com; www.iberocto.com
PQ Pg. 7
PSIQUIATRA
Preparacin Examen de Seleccin 05/06 1 Vuelta Seguimiento a distancia

Pregunta 1.-R: 2
Los trastornos primarios por ansiedad se pueden presentar de dos
Pregunta 2. Ventajas y desventajas de los tratamientos
formas: preventivos para el pnico.
1. Continua, como en el trastorno por ansiedad generalizada.
2. Episdica, en forma de crisis de ansiedad; cuando las crisis tienen 12345657 82792345657
lugar siempre al enfrentarse a una determinada situacin decimos
que el paciente sufre una fobia; otros pacientes tienen crisis espon- 12 67 2
2 9
62

94

tneas (llamadas crisis de angustia o ataques de pnico), sin rela- 12342564789



12
 

cin aparente con estmulo alguno; cuando estas crisis espont- 127
5
7
9 
128
7
2564789 628

neas se repiten y repercuten en la vida de las personas hablamos de


34 2 27 97 126  2 254 

12 8
42 9 27
2

trastorno por angustia (o trastorno de pnico). 127


56
  2 49 7 
12 
7 72
5
 62
1274 2  228 6

De una forma algo artificial tambin se van a incluir dentro de los 12
 



trastornos por ansiedad los trastornos obsesivo-compulsivos (en don- 129 27


246
de la ansiedad es secundaria a los fenmenos obsesivos y compul- 126 5 7 27! 
sivos) y los trastornos por estrs agudo y postraumtico (en donde 23 52 357 12"9 42

94
12$
%427
2 842
la ansiedad es uno de los muchos sntomas que aparecen ante una 12 7#
642
7 9 
127
5
7
9 
situacin estresante de intensidad mxima). 126  46 
No es raro que los trastornos por ansiedad se vayan confundiendo
entre s cuando se cronifican o complican; el ejemplo ms claro lo
vemos en las complicaciones que surgen cuando el trastorno por Pregunta 3.-R: 3
angustia se prolonga en el tiempo: La mayora de los pacientes con agorafobia presentan adems crisis
Inicialmente el paciente comienza a preocuparse de forma CON- de angustia espontneas (ataques de pnico); se considera por tanto,
TINUA por la posibilidad de tener nuevas crisis (ANSIEDAD ANTI- que su agorafobia es secundaria. Pero tambin hay formas de agorafobia
CIPATORIA). primaria en las que no detectamos crisis espontneas. El tratamiento de
Luego EVITA determinados lugares en donde no se siente seguro la agorafobia en s no es distinto: tcnicas psicolgicas de modificacin
por el miedo a no recibir ayuda si sufre una crisis (AGORAFOBIA). de conducta (sobre todo la exposicin en vivo). La diferencia vendr
No es raro que desarrolle una intensa PREOCUPACIN por la dada por la necesidad de aadir tratamiento farmacolgico para preve-
posibilidad de tener una ENFERMEDAD fsica que explique sus nir las crisis de angustia en el primer caso. Las tcnicas psicolgicas basa-
sntomas (ANSIEDAD HIPOCONDRACA). das en la modificacin de conducta son el tratamiento de eleccin en
Algunos pacientes recurrirn a sustancias sedantes para aliviar su todas las fobias. Suele realizarse primero un listado de todas las situacio-
ansiedad (ABUSO del ALCOHOL o de BENZODIACEPINAS). nes temidas y una ordenacin en funcin del miedo asociado
Finalmente, no son infrecuentes los sntomas depresivos que se (jerarquizacin); posteriormente se programan ejercicios de exposicin
asocian a un aumento del riesgo de suicidio (DEPRESIN y SUICI- en vivo, comenzando en las situaciones ms fciles (desensibiliza-
DIO). cin sistemtica). Se han desarrollado otras tcnicas ms agresivas que
buscan la exposicin brusca a la situacin fbica en su mxima inten-
En el trastorno obsesivo-compulsivo, los pacientes pueden sufrir sidad (inundacin o implosin), pero suelen dar peores resultados.
crisis de ansiedad (sobre todo cuando se les impide realizar los rituales
compulsivos), pero no vemos lo contrario (un paciente con crisis de Pregunta 4.-R: 3
angustia no desarrolla fenmenos obsesivo-compulsivos). El paciente con fobia social tiene miedo de determinadas situacio-
nes en las que se expone a la opinin de otras personas. Lo que le
Pregunta 2.-R: 1 preocupa no es por tanto el lugar en el que se encuentra, sino la
En el trastorno por angustia (pnico) diferenciamos tres necesida- reaccin de los dems ante su comportamiento. No es raro que estos
des de tratamiento: pacientes limiten sus actividades sociales, pudiendo confundirse con
El control de los sntomas agudos de una crisis de angustia (ataque de otros pacientes que tambin presentan retraccin social (agorafbicos
pnico) se realiza con benzodiacepinas (diacepam, cloracepato, que no salen de sus casas por el miedo a los desplazamientos, esqui-
Comentarios TEST

loracepam o alprazolam son las ms usadas); utilizarlas por va oral zofrnicos o depresivos que restringen sus relaciones sociales por la
o sublingual carece de importancia, pues su farmacocintica no prdida de inters asociada a sus enfermedades, personalidades es-
vara y adems en Espaa NO se comercializa ninguna presenta- quizoides sin inters real por la conducta social). Quizs lo ms com-
cin sublingual de benzodiacepinas. Es igualmente necesario ayu- plicado sea diferenciarles de las personalidades evitativas o fbicas,
dar al paciente a frenar la hiperventilacin y explicarle la ausencia con quienes comparten los mismos errores de pensamiento, si bien se
de consecuencias fsicas de la crisis. La buspirona es un ansioltico supone que la fobia social comienza en un momento dado (en torno
NO benzodiacepnico (agonista parcial serotoninrgico) con un a los veinte aos) y las personas evitativas siempre han sido as.
perfil ms adecuado para el trastorno por ansiedad generalizada; En las fobias sociales los frmacos pueden ayudar a los tratamien-
carece de efectos sedantes (no sirve para cortar una crisis de ansie- tos psicolgicos de dos formas distintas; en las formas de fobia social
dad), no produce dependencia ni abstinencia y no interacciona restringidas a situaciones poco frecuentes (p.ej. miedo a hablar en
con el alcohol; tiene un inicio de accin lento (semanas) y no ha pblico delante de un auditorio) los bloqueantes de los receptores
demostrado efectos preventivos anti-pnico. beta-adrenrgicos (propranolol, atenolol) disminuyen la respuesta
La prevencin de nuevas crisis se realiza con antidepresivos, siendo ansiosa perifrica y pueden permitir un mayor autocontrol de la an-
hoy en da de eleccin los ISRS (paroxetina o citalopram son los siedad; en la forma generalizada de fobia social se propone el trata-
ms utilizados) por su mejor perfil de efectos secundarios; esto no miento con IMAOs como coadyuvante de la psicoterapia. En los
quiere decir que no sean de utilidad los antidepresivos clsicos ltimos aos los ISRS tambin se presentan como alternativa para
(tricclicos o IMAOs). Tambin son eficaces las benzodiacepinas de estos pacientes, habindose convertido quizs en los frmacos de
alta potencia (alprazolam, clonacepam, loracepam), pero el mie- primera opcin, aun cuando los estudios sobre su eficacia son menos
do a la posible dependencia ha hecho que se trate de limitar su uso claros. Los problemas asociados al uso de IMAOs (necesidad de dieta,
a las primeras semanas del tratamiento, mientras el antidepresivo riesgo de interacciones medicamentosas) les han relegado a un segun-
alcanza su mxima eficacia. do plano, a pesar de su indiscutible eficacia.
Finalmente resulta imprescindible evaluar las distintas complicacio-
nes del trastorno por angustia/pnico y ofrecer un tratamiento espe- Pregunta 5.-R: 4
cfico si persisten una vez controladas las crisis. Las dos principales formas de presentacin del trastorno obsesivo-
compulsivo son:

M exico A rgentina CTO Medicina C/ Nez de Balboa, 115 28006 MADRID (Espaa) Tfno.: (91) 782 43 32 / Fax: (91) 782 43 27
C hile U ruguay E-mail: secretaria@ctomedicina.com; iberocto@ctomedicina.com WEB: www.ctomedicina.com; www.iberocto.com
PQ Pg. 1
PSIQUIATRA
Seguimiento a distancia Preparacin Examen de Seleccin 05/06 1 Vuelta
1. La combinacin de ideas obsesivas relacionadas con la posible etc.). En cualquier caso, tanto en las formas idiopticas como en las
contaminacin o contagio de enfermedades y rituales compulsi- formas secundarias se implica a las mismas regiones cerebrales, los
vos de lavado excesivo o de evitacin del contacto con posibles circuitos de conexin entre corteza prefrontal, ganglios basales y
fuentes contaminantes. tlamo. De hecho, la posible utilidad de la psicociruga en algunos
2. La combinacin de ideas obsesivas de duda o incertidumbre y casos seleccionados se basa en la interrupcin de alguna de las vas de
rituales compulsivos de comprobacin. estos circuitos. El neurotransmisor ms relacionado con el trastorno
obsesivo-compulsivo es la serotonina, lo cual va a ser utilizado en el
Menos frecuentes son las obsesiones centradas en la necesidad de tratamiento farmacolgico.
orden y simetra, que dan lugar a compulsiones de colocacin y orde-
nacin. Sin embargo no es raro encontrar pacientes con otros fenme- Pregunta 7.-R: 5
nos ms extraos, destacando las llamadas ideas de contraste (ideas de Uno de los datos ms sorprendentes del trastorno obsesivo-com-
contenidos agresivos o sexuales totalmente inapropiadas para la situa- pulsivo es su respuesta selectiva a medicacin serotoninrgica; el tra-
cin en la que se encuentra el paciente y en desacuerdo con sus deseos tamiento farmacolgico de eleccin son los antidepresivos serotoni-
o creencias) que pueden conducir a la aparicin de fobias de impul- nrgicos, pudiendo en ocasiones potenciarse su efecto con otras sus-
sin (en las que el paciente tiene miedo de llevar a cabo esas ideas tan tancias (litio, buspirona). En los casos refractarios y en aquellos en los
desagradables y evita situaciones potencialmente peligrosas como el que coexistan tics puede ser til aadir un antipsictico (pimocide,
uso de cuchillos, el acercarse a las ventanas o el aproximarse a los haloperidol). Dentro de los antidepresivos el mejor estudiado es el
andenes del tren por el miedo a presentar un impulso frente al que no antidepresivo tricclico clorimipramina (o clomipramina), si bien sus
se pueda resistir). Aunque en muchos casos los fenmenos compulsivos efectos adversos le han relegado a un segundo plano tras la aparicin
guardan una relacin ms o menos lgica con las ideas obsesivas, en de los ISRS. stos se han convertido en la medicacin de primera
otros pacientes veremos rituales totalmente independientes, con una eleccin, teniendo en cuenta adems que en este trastorno suelen
relacin ms bien mgica (por ejemplo, la necesidad de colocar las necesitarse dosis dos o tres veces superiores a las dosis habituales para
cosas en un determinado orden para que no suceda nada malo). la depresin y tiempos de tratamiento muy prolongados, lo que dis-
minuye la efectividad del tratamiento con clorimipramina al aumen-
Pregunta 6.-R: 5 tar los abandonos. Los IMAOs se han usado para pacientes resistentes,
El trastorno obsesivo-compulsivo idioptico suele aparecer al final mientras que las benzodiacepinas carecen de efecto anti-obsesivo
de la adolescencia o principio de la juventud, de una forma insidiosa, especfico. Junto con el tratamiento farmacolgico es necesario un
y evoluciona con carcter crnico y de forma fluctuante, agravndose tratamiento psicolgico basado (como en las fobias) en las tcnicas de
en situaciones de estrs. Es raro que desaparezca por completo, pero modificacin de conducta; en concreto, la tcnica preferida se deno-
tambin es raro que alcance una gravedad invalidante. La mayora de mina exposicin (a la situacin obsesiva temida) con prevencin de
los pacientes acaban por acostumbrarse a los fenmenos obsesivos y respuesta (compulsiva). El psicoanlisis no obtiene resultados en los
compulsivos, lo que puede explicar su escasa presencia en consulta a pacientes con este trastorno. En casos refractarios al tratamiento far-
pesar de su elevada prevalencia en poblacin general (cerca del 2%). macolgico, la psicociruga (cingulotoma, capsulotoma bilateral an-
Cuando debuta en la infancia se puede asociar con los trastornos por terior, tractotoma subcaudada) puede ser eficaz.
tics (tanto los tics motores crnicos como el trastorno de la Tourette).
Recientemente se ha descrito una asociacin entre algunas formas in- Pregunta 8.-R: 3
fantiles de inicio agudo y fenmenos autoinmunes cercanos a la fiebre Los sntomas del trastorno por estrs postraumtico se agrupan en
reumtica (secundarios por tanto a infecciones estreptoccicas cuatro tipos:
[estreptococo beta hemoltico o SGA] farngeas); se ha denominado Reexperimentacin del acontecimiento: pesadillas, imgenes tipo
PANDAS (siglas en ingls del Trastorno Neuropsiquitrico Peditrico de flash-back.
origen Autoinmune Asociado al Estreptococo) y se supone que com- Evitacin voluntaria (tipo fbica) o involuntaria (amnesia psicge-
parte un mismo mecanismo patognico con el corea de Sydenham na) de todo lo relacionado con el acontecimiento.
(lesin mediada por autoanticuerpos frente a los ganglios basales). Estado de hiperalerta: reacciones bruscas ante los ruidos, proble-
mas de concentracin, insomnio.
Cambios emocionales: embotamiento, sensacin de corte vital.

Cuando el trastorno por estrs no ha alcanzado un mes de duracin

Comentarios TEST
se denomina trastorno por estrs agudo, y suelen predominar los snto-
mas emocionales y disociativos. Se diagnostica un trastorno de estrs
postraumtico cuando los sntomas superan el mes de duracin, pu-
diendo verse formas crnicas (de ms de 6 meses de duracin) e incluso
formas de inicio demorado (que comienzan al cabo de 6 meses desde
el acontecimiento). Dentro de las complicaciones de estos trastornos
destacan los sndromes depresivos y el abuso de sustancias.
De las posibles situaciones traumticas, aquellas que se deben a
fenmenos naturales (catstrofes como terremotos o inundaciones)
producen menos problemas que las que se deben a la intervencin
humana; dentro de stas, los accidentes son menos traumticos que
los delitos, en los que una persona provoca voluntariamente un dao
sobre otra; los delitos sexuales y los secuestros producen estrs pos-
traumtico en un nmero muy elevado de casos (ms del 80% de las
vctimas de violacin).
No existe un tratamiento especfico del estrs postraumtico; los
antidepresivos (IMAOs, ISRS) parecen obtener mejores resultados cuan-
do predominan los fenmenos de reexperimentacin y los cambios
emocionales; las conductas de evitacin precisarn de un tratamien-
Pregunta 6. Esquema del sndrome PANDAS. to conductual adecuado; para reducir el nivel de alerta se pueden
usar benzodiacepinas (cuidado con el abuso de sustancias) y tcnicas
Cuando el trastorno obsesivo-compulsivo aparece en adultos o de relajacin. Lo que s parece clara es la necesidad de detectar y
ancianos es necesario buscar problemas neurolgicos como causa tratar precozmente a estos pacientes, pues la cronificacin de los sn-
del mismo (corea de Huntington, accidentes vasculares cerebrales, tomas predice una mala respuesta al tratamiento.

M exico A rgentina CTO Medicina C/ Nez de Balboa, 115 28006 MADRID (Espaa) Tfno.: (91) 782 43 32 / Fax: (91) 782 43 27
Pg. 2 PQ C hile U ruguay E-mail: secretaria@ctomedicina.com; iberocto@ctomedicina.com WEB: www.ctomedicina.com; www.iberocto.com
PSIQUIATRA
Preparacin Examen de Seleccin 05/06 1 Vuelta Seguimiento a distancia
Pregunta 9.-R: 3 luntaria, en donde los factores psicolgicos influyen en su origen o
Dentro de los pacientes que se presentan con sntomas fsicos para evolucin) ni con los pacientes que fingen VOLUNTARIAMENTE tener
los que no se encuentra una explicacin mdica (SOMATOMOR- una enfermedad. Dentro de estos ltimos diferenciamos los simulado-
FOS) tenemos: res, en los que existe una motivacin evidente de tipo econmico o
Somatizadores: se quejan de numerosos sntomas (digestivos, neu- legal, de los trastornos facticios, en los que la motivacin parece ser de
rolgicos, genitales, cardiorrespiratorios); tienden a cambiar de tipo psicolgico, la necesidad de asumir la identidad de enfermo,
mdico cuando se les confronta con la posibilidad de un origen incluso a costa de perjuicios para su salud o su economa. La forma ms
psicolgico de sus molestias; pueden abusar de la espectacular de trastorno facticio es el sndrome de Mnchausen, en el
automedicacin (analgsicos) y corren el riesgo de someterse a que los sntomas inventados y los signos autoprovocados abarcan dife-
pruebas diagnsticas innecesarias y peligrosas; en su mayora rentes rganos y sistemas; la mayora de los facticios, sin embargo, son
son mujeres, que suelen comenzar a tener sntomas en la juven- formas limitadas a sntomas ms concretos, siendo ms difciles de de-
tud, siguiendo un curso crnico y fluctuante; se utiliza a veces en tectar, pues los pacientes se preocupan de no ser descubiertos y de
estos casos el epnimo de sndrome de Briquet. Una variante de parecerse lo ms posible a la enfermedad real. Desde el punto de vista
somatizador ms limitada en su expresin clnica es el paciente legal la simulacin es un delito evidente; los trastornos facticios pueden
que se queja de dolor, siendo ste desproporcionado para los convertirse en un problema legal cuando los pacientes provocan la
hallazgos exploratorios. enfermedad en terceras personas (hijos, esposo) para poder asumir la
Hipocondracos y dismorfofbicos: estn preocupados por pade- identidad de cuidador (trastornos facticios por poderes). En la ma-
cer una enfermedad o sufrir un defecto fsico, respectivamente; yora de estos pacientes encontramos graves alteraciones de la persona-
aunque tambin pueden cambiar de mdico si no obtienen la lidad, siendo refractarios al tratamiento.
atencin que creen necesaria, es ms raro en ellos la tendencia a la
automedicacin o la realizacin de pruebas complementarias de Pregunta 10.-R: 5
riesgo; en el caso de los dismorfofbicos, el peligro fundamental En los trastornos disociativos los pacientes presentan sntomas psi-
son las posibles intervenciones estticas (mdicas o quirrgicas) colgicos (NO fsicos) extraos (cambios de comportamiento, am-
que pueden dejar secuelas; en ambos casos no veremos un predo- nesias, viajes sin sentido) que no encajan en las enfermedades psi-
minio de ningn sexo y el inicio suele ser ms tardo; los pacientes quitricas o neurolgicas habituales y se comportan de forma extra-
oscilan en la intensidad de sus preocupaciones, desde formas cer- a ante los tratamientos. Como en el caso de los pacientes conversivos,
canas a los trastornos obsesivo-compulsivos hasta formas casi deli- es frecuente encontrar un factor estresante relacionado temporal-
rantes; se propone para estos pacientes un tratamiento similar al de mente con el sntoma y una tendencia a la desaparicin espontnea
los trastornos obsesivo-compulsivos (ISRS, clorimipramina), asocia- y a la recurrencia. Los sntomas disociativos pueden aparecer en el
do a veces a antipsicticos (pimocide). contexto de otras enfermedades psiquitricas, destacando su aso-
Conversivos: se suelen presentar en Urgencias con signos neurolgi- ciacin con los trastornos por estrs (agudo y postraumtico) y con
cos incongruentes a la exploracin fsica (convulsiones, parlisis, anes- el trastorno de personalidad lmite. En los ltimos aos se insiste en
tesias, cegueras); casi siempre se identifica un factor estresante relacio- la frecuencia de antecedentes de abuso sexual en la infancia de
nado temporalmente con el inicio de los sntomas; tienden a la recu- estos pacientes. Dependiendo de la cultura del pas encontramos
peracin espontnea pero tambin a la recurrencia; en algunos raros diferentes formas de trastornos disociativos; as, las formas ms fre-
casos veremos formas de evolucin crnica; como en el caso de los cuentes en Espaa parecen ser los episodios de amnesia, siendo
somatizadores, suelen debutar en la juventud y hay un claro predo- excepcionales las fugas y anecdticos los casos de alteraciones diso-
minio de las mujeres; en los pases desarrollados son cada vez menos ciativas de la personalidad (personalidad mltiple); en pases menos
frecuentes; parece que con el desarrollo cultural se produce una desarrollados veremos numerosos casos de trances y posesiones di-
mayor tendencia a la somatizacin como expresin fsica del males- sociativas, y en EE.UU. llama la atencin la elevada frecuencia de
tar psicolgico; ambas, conversin y somatizacin, junto con las va- casos referidos con alteraciones disociativas de la personalidad.
riantes disociativas, componen las distintas formas de presentacin La clasificacin DSM (no la CIE-10) incluye dentro de los trastor-
de la histeria, una enfermedad clsica dentro de la Psiquiatra que nos disociativos a los cuadros de despersonalizacin y desrealizacin
parece traducir a sntomas fsicos o psiquitricos incongruentes la crnicas; no hay que olvidar que estos sntomas de extraeza ante
respuesta de determinadas personas ante el malestar psicolgico. uno mismo o el entorno pueden aparecer en numerosas enfermeda-
des psiquitricas (trastornos por ansiedad, sobre todo en crisis,
Comentarios TEST

sndromes depresivos, psicosis agudas, intoxicaciones por drogas) y en


algunos trastornos neuropsiquitricos, sobre todo los que afectan al
lbulo temporal (epilepsia parcial compleja, tumores, encefalitis); los
casos en donde aparecen de forma aislada y con evolucin crnica
son excepcionales.

Pregunta 11.-R: 3
Las benzodiacepinas (BZD) poseen un sistema de receptores pro-
pios (receptores benzodiacepnicos u omega), con al menos dos sub-
tipos (omega-1 y omega-2); estos receptores estn formados por diver-
sas subunidades proteicas, para las cuales existen numerosas varian-
tes, sin que se conozca todava la importancia funcional de cada una
de ellas. Desde el punto de vista farmacodinmico, todas las BZD
hacen lo mismo: se unen a su receptor y modifican la afinidad del
receptor gabargico de tipo A por el GABA, aumentando su actividad
(modificacin alostrica positiva o potenciacin). Las diferentes BZD
tendrn mayor o menor potencia, pero a las dosis adecuadas todas
comparten los efectos anticonvulsivantes, ansiolticos, hipnticos y
miorrelajantes. Sus principales diferencias son farmacocinticas (velo-
cidad de absorcin, semivida de eliminacin, presencia o no de
Pregunta 9. Caractersticas de los fenmenos histricos. metabolizacin heptica).

Conviene NO confundir a estos pacientes (en donde los sntomas Pregunta 12.-R: 2
son involuntarios y no existe enfermedad fsica real) con los pacientes Una de las ventajas de que las BZD tengan un sistema propio de
PSICOSOMTICOS (que presentan una enfermedad fsica real e invo- receptores ha sido la posibilidad de disear un antagonista especfico

M exico A rgentina CTO Medicina C/ Nez de Balboa, 115 28006 MADRID (Espaa) Tfno.: (91) 782 43 32 / Fax: (91) 782 43 27
C hile U ruguay E-mail: secretaria@ctomedicina.com; iberocto@ctomedicina.com WEB: www.ctomedicina.com; www.iberocto.com
PQ Pg. 3
PSIQUIATRA
Seguimiento a distancia Preparacin Examen de Seleccin 05/06 1 Vuelta
capaz de revertir sus efectos en el caso de sobredosis; el flumacenilo es cedimientos instrumentales potencialmente desagradables (cateterismos,
el tratamiento de eleccin en las intoxicaciones por BZD y se ha colonoscopias) para relajar a los pacientes.
convertido, junto con la naloxona (antagonista de los receptores opioi-
des y tratamiento de eleccin en las intoxicaciones por opiceos), en
una medicacin bsica de los servicios de urgencias. En el caso del Pregunta 14. Alternativas a las benzodiacepinas en sus
resto de psicofrmacos carecemos de antdotos especficos. Las so- diferentes indicaciones.
bredosis de antidepresivos varan en su gravedad en funcin del tipo
de antidepresivo: 4 59256 7
12324567829
8 27 478265
Los ISRS son muy seguros, pues carecen de cardiotoxicidad y sus
efectos sobre el sistema nervioso son escasos. 46 87827486 9724424
Los IMAOs ocupan un lugar intermedio. 46 878  8 27
Los antidepresivos tricclicos son los ms peligrosos, al producir
alteraciones de la conduccin cardaca potencialmente letales. 4!2983
4!28 9265
Los antipsicticos tienen un riesgo cardiolgico considerablemen- 12324 86 8 27 4!59926
te menor, dependiendo sus efectos en sobredosis de otras caractersti- 46 8 87 53
68 27
cas (bloqueo histaminrgico, bloqueo alfa-adrenrgico, efectos anti- 46 878274"456 878  8 274756 7
colinrgicos) que varan enormemente de unos a otros; en el caso de 1232438295#56 7 418 926$5865
presentarse graves efectos extrapiramidales agudos, el tratamiento de
eleccin son los anticolinrgicos (biperideno, trihexifenidilo); en el 4%6258 59
caso de un exceso de accin anticolinrgica se puede recurrir al uso 4%68 2
65
de fisostigmina (anticolinestersico de accin corta). 1232 4& 8245928 2
Dentro de los estabilizadores del humor, el litio es el ms peligroso 56 8 2697856 7 415535 865
4'556 865
en el caso de sobredosis, aunque la principal causa de intoxicacin
por litio no va a ser la toma voluntaria o accidental de un exceso de 4(532 8)865
medicacin, sino las interacciones con frmacos que alteran su elimi- 4*28535 2
nacin renal o la concurrencia de problemas hidroelectrolticos que
tienen el mismo efecto; el tratamiento de eleccin en caso de intoxi- Pregunta 15.-R: 4
cacin grave es la hemodilisis. Clasificamos los sndromes afectivos en funcin de dos criterios: su
gravedad y su duracin. La gravedad se evala con diferentes escalas
Pregunta 13.-R: 5 de sntomas (depresivos o manacos), aunque el principal criterio de
La mayora de las BZD se eliminan por va heptica mediante un gravedad ser la repercusin que el sndrome clnico tenga sobre el
doble mecanismo: metabolizacin y conjugacin; el primero de es- funcionamiento del paciente. Atendiendo a la gravedad, diferencia-
tos procesos se afecta gravemente en el caso de hepatopata, perma- mos los sndromes depresivos o manacos en MAYORES y MENORES.
neciendo el segundo casi inalterado. Por eso las BZD, que slo se Para que un sndrome depresivo mayor sea considerado relevante se
conjugan, sin metabolizarse en el hgado, son las de eleccin en el exige una duracin mnima de 2 semanas (EPISODIO DEPRESIVO
caso de hepatopata (loracepam, oxacepam, temacepam); estas BZD MAYOR), mientras que en el caso de la mana la duracin se reduce a
tienen una semivida de eliminacin corta, cercana a las 8 horas. 1 semana o menos, si precisa hospitalizacin por su intensidad (EPI-
SODIO MANACO). En los sndromes menores se exige una mayor
Pregunta 13. Ventajas e inconvenientes de las benzodiacepinas duracin, debiendo persistir los sntomas depresivos 2 aos para acep-
en funcin de su vida-media. tar el diagnstico de DISTIMIA; en los sndromes manacos s se acepta
la importancia de los cuadros menores de duracin recortada, diag-
nosticndose de HIPOMANA a los sndromes manacos menores que
1234567324 duran al menos 4 das; la alternancia de sntomas hipomanacos y
12342 56372 sntomas depresivos menores durante un perodo de al menos 2 aos
recibe el diagnstico de CICLOTIMIA.
123435678 9
4 8 4 Los cuadros depresivos son mucho ms frecuentes que los mana-

Comentarios TEST
cos, encontrndose en los primeros un claro predominio femenino,
2688
34686 mientras que en los segundos hay una cierta igualdad. De igual forma

836234364
83 2 7 los sndromes episdicos (de semanas o meses de duracin) son ms
 2
16882
48 frecuentes que los sndromes crnicos (de aos de duracin). De
284
8688
34686 ah que el trastorno afectivo ms frecuente sea el episodio depresivo
24
32
42 2
1 2 7 mayor.
6882
48 La melancola (o depresin endgena) es una variante de episodio
depresivo mayor que se identifica por la presencia de unos sntomas
34

48 2 7 caractersticos; por tanto, todos los casos de melancola son episodios
32364
23
332 22 7 12
depresivos mayores, pero tan slo una pequea parte de los episodios
4
4 2!38 8! depresivos mayores presentan sntomas melanclicos.
43"268 8 4

7 2 Pregunta 16.-R: 4
8
48
23 Los episodios depresivos mayores (EDM) reciben diferentes adjeti-
vos calificativos en funcin de distintas caractersticas:
Pregunta 14.-R: 1 Segn la gravedad de los sntomas, podemos ver EDM de intensi-
Existen otras BZD, de alta potencia, que tampoco se metabolizan de dad leve, moderada o severa; las formas ms graves de EDM llegan
forma sustancial en el hgado (alprazolam, triazolam, midazolam) y cuya a presentar sntomas psicticos (delirios sobre todo), por lo que se
semivida de eliminacin es quizs demasiado corta; en el caso de tria- habla de EDM con sntomas psicticos o depresin psictica (o
zolam y midazolam es de menos de 6 horas por lo que no se usan como depresin delirante).
ansiolticos; triazolam se utiliza para tratar el insomnio de conciliacin Segn el patrn de sntomas tenemos EDM con sntomas melanc-
(siempre en tandas cortas de pocas semanas para no producir toleran- licos (o depresin endgena) y EDM con sntomas atpicos; sin
cia y dependencia) y midazolam se usa como preanestsico y en pro- embargo la mayora de los EDM no encajan en ninguno de estos
dos patrones, presentando sntomas inespecficos; parece que con

M exico A rgentina CTO Medicina C/ Nez de Balboa, 115 28006 MADRID (Espaa) Tfno.: (91) 782 43 32 / Fax: (91) 782 43 27
Pg. 4 PQ C hile U ruguay E-mail: secretaria@ctomedicina.com; iberocto@ctomedicina.com WEB: www.ctomedicina.com; www.iberocto.com
PSIQUIATRA
Preparacin Examen de Seleccin 05/06 1 Vuelta Seguimiento a distancia
las recadas sucesivas de la enfermedad los sntomas van adquirien- (EDM con sntomas melanclicos o depresin endgena), sobre todo
do un carcter ms endgeno y pueden alcanzar con mayor si se asocia con una variacin diurna del estado de nimo en la forma
facilidad una gravedad psictica; lo mismo parece ocurrir en los de empeoramiento matutino y mejora vespertina. Los pacientes con
EDM que debutan en la edad avanzada. Tambin podemos ver un sntomas atpicos suelen referir una desagradable necesidad de dor-
EDM con sntomas catatnicos (o depresin estuporosa). mir durante el da, quejndose adems de un cansancio extremo. En
Cuando el inicio del EDM guarda una relacin con el puerperio se lo que hace referencia al apetito y al peso, lo ms frecuente es ver una
habla de EDM de inicio puerperal o depresin puerperal (o depre- disminucin de ambos; en los pacientes atpicos habr aumento del
sin posparto). apetito y del peso, con una tendencia a comer mayores cantidades de
hidratos de carbono (dulces) de lo habitual para el paciente.
La depresin neurtica (trmino clsico) se corresponde ms o
menos con la distimia de la clasificacin actual, no con los EDM. Pregunta 20.-R: 4
Solamente en las depresiones ms graves el estado de nimo per-
Pregunta 17.-R: 5 manece inalterable todo el da, con una ausencia de reactividad ante
Los sntomas depresivos atpicos incluyen tres caractersticas que, en lo que le rodea (anestesia emocional). En los cuadros melanclicos el
teora, predeciran una mala respuesta a medicacin antidepresiva: paciente puede encontrarse algo mejor por la noche (mejora vesper-
1. La presencia de rasgos patolgicos de personalidad, en la lnea tina), pero lo que s va a referir con frecuencia es que est mucho peor
dependiente o histrinica. por la maana, nada ms despertarse (empeoramiento matutino); la
2. La existencia de un factor estresante asociado al inicio de los snto- asociacin entre despertar precoz y empeoramiento matutino es alta-
mas (frecuentemente un problema sentimental). mente especfica de melancola (o depresin endgena), y se ha rela-
3. La ausencia de sntomas melanclicos, observndose una inver- cionado con alteraciones en los marcapasos endgenos pues sigue
sin de los sntomas biolgicos (aparecen somnolencia excesiva y un ritmo muy parecido al ritmo del cortisol (que con frecuencia est
aumento del apetito). alterado en estos pacientes). En los sndromes depresivos menores los
pacientes conservan cierta capacidad de mejorar cuando las circuns-
Sin embargo se obtiene una respuesta razonable al tratamiento con tancias externas les son favorables (preservan la reactividad); as tien-
IMAOs, no con tricclicos. Esta selectividad farmacolgica parece estar den a notarse mejor cuanto ms activos estn, lo que puede dar el
relacionada con una mayor frecuencia de alteraciones serotoninrgi- aspecto de una mejora matutina, opuesta a la melanclica; sin em-
cas en estos pacientes (los IMAOs tienen un efecto ms serotoninrgico bargo en estos casos la variacin del humor NO tiene un carcter
que noradrenrgico). En general, los sntomas atpicos no implican una neurobiolgico, sino psicosocial.
gravedad excesiva; de hecho, podemos ver sntomas atpicos tanto en La respuesta selectiva a IMAOs frente a tricclicos es una de las
los EDM como en las distimias. En cambio los sntomas melanclicos s caractersticas de las depresiones atpicas.
se asocian a una mayor gravedad (y a un mayor riesgo de suicidio, por La asociacin entre depresin y estaciones no es tan frecuente como
tanto); en estos pacientes melanclicos se encuentran ms alteraciones se cree popularmente; tan slo un 15% de los pacientes con enferme-
neurobiolgicas, por lo que su respuesta a los tratamientos biolgicos es dades afectivas recurrentes (depresivas o bipolares) presentan un ritmo
superior (tanto a antidepresivos como a electrochoque). estacional claro, que se repite de ao en ao. Lo habitual en estos casos
es que los EDM aparezcan en otoo y primavera, y los episodios mana-
Pregunta 18.-R: 5 cos en verano. En los pases nrdicos se ha descrito una variacin del
En los ancianos no es raro que se oculte una depresin tras una ritmo estacional en los pacientes bipolares: habra EDM en invierno
serie de quejas fsicas o de memoria, lo cual en parte puede reflejar la (frecuentemente con sntomas atpicos) y episodios manacos en vera-
mayor preocupacin por estos sntomas en este grupo de edad. Tam- no (en realidad son ms frecuentemente hipomanacos, es decir, bipo-
bin es destacable la mayor frecuencia de sntomas psicomotores lares-2). Este cambio se ha puesto en relacin con la duracin del
(inhibicin, agitacin) y de sntomas psicticos; se han relacionado fotoperodo (muy corta en invierno, muy larga en verano), proponin-
ambos sntomas con alteraciones en los ganglios basales, que podran dose el tratamiento con fototerapia con carcter preventivo o curativo.
deberse al deterioro cerebral asociado a la edad o a la presencia de
factores de riesgo cerebrovascular. La presencia de sntomas psicti- Pregunta 21.-R: 2
cos implica la necesidad de modificar el tratamiento, aadiendo an- Uno de los problemas ms frecuentes ante el diagnstico de depre-
tipsicticos a los antidepresivos o recurriendo directamente al electro- sin es la tendencia casi automtica a buscar un factor psicosocial
Comentarios TEST

choque. No hay que olvidar que la edad es por s misma un factor que estresante que consideramos desencadenante. Para empezar, no
aumenta el riesgo de suicidio, lo cual, aadido a la presencia de un hay que olvidar que muchas personas se enfrentan a diario a situacio-
sndrome depresivo y a la gravedad psictica, que con frecuencia nes estresantes muy similares, pero slo una parte de ellas desarrollan
alcanza explica el enorme riesgo de suicidio que tienen las depresio- un sndrome depresivo; hay una serie de factores individuales que
nes graves del anciano. aumentan el riesgo de presentar una depresin, entre los que destaca
En los nios, los sndromes depresivos van a ocultarse tras quejas la presencia de antecedentes familiares, los antecedentes personales
fsicas (los nios pequeos carecen de lenguaje emocional y tien- de depresin y determinados rasgos de personalidad. Los factores
den a expresarse somticamente) o problemas escolares; en los ado- estresantes parece que actan de una forma inespecfica, aumen-
lescentes, los trastornos de conducta, incluido el abuso de sustancias, tando la probabilidad de presentar un sndrome psiquitrico en fun-
pueden enmascarar una depresin. No hay que olvidar que, en nios cin de la predisposicin individual. Adems no existe una buena
y adolescentes el estado de nimo predominante durante una depre- correlacin entre la presencia o ausencia de un factor estresante y la
sin va a ser con frecuencia irritable ms que triste, lo que dificultar duracin, la intensidad o las caractersticas del sndrome depresivo
el diagnstico. asociado; la mitad de los pacientes con EDM no refieren aconteci-
miento estresante alguno en las semanas previas al inicio de su depre-
Pregunta 19.-R: 3 sin, y su evolucin no es diferente de la evolucin de los pacientes
Dentro de los sndromes depresivos, los sntomas fsicos son los ms que s refieren un acontecimiento causal. Hoy en da se reserva el
fciles de diagnosticar, pues tienen un carcter objetivo; sin embargo trmino trastorno adaptativo con sntomas depresivos para aque-
carecen de especificidad diagnstica al presentarse tambin en casi llos pacientes que presentan sntomas depresivos de intensidad me-
todas las enfermedades psiquitricas y mdicas; por esto, en los pa- nor en respuesta a una situacin estresante que produce sntomas
cientes con enfermedades psiquitricas o mdicas coexistentes, habr similares en la mayora de las personas; estos sntomas se resuelven
que recurrir a sntomas ms subjetivos (estado de nimo, pensamien- espontneamente en muchos casos y no suelen precisar tratamiento
tos) para asegurar el diagnstico. La alteracin del sueo ms frecuen- farmacolgico, salvo que su intensidad aumente; correspondera con
te en una depresin es el insomnio de caractersticas inespecficas el concepto de depresin reactiva, aunque podemos ver depresio-
(problemas de conciliacin con despertares nocturnos); cuando apa- nes reactivas (es decir, relacionadas con problemas vitales) de intensi-
rece tendencia al despertar precoz sugiere una depresin melanclica dad tal que permita el diagnstico de EDM.

M exico A rgentina CTO Medicina C/ Nez de Balboa, 115 28006 MADRID (Espaa) Tfno.: (91) 782 43 32 / Fax: (91) 782 43 27
C hile U ruguay E-mail: secretaria@ctomedicina.com; iberocto@ctomedicina.com WEB: www.ctomedicina.com; www.iberocto.com
PQ Pg. 5
PSIQUIATRA
Seguimiento a distancia Preparacin Examen de Seleccin 05/06 1 Vuelta
de fase (a diferencia de los pacientes depresivos, en donde se mantie-
Pregunta 21. Relacin entre los distintos tipos de depresin ne el mismo frmaco 6 meses a la misma dosis). Sin embargo, vemos
y los rasgos de personalidad previos. pacientes que tienen respuestas maniformes al tratamiento con anti-
depresivos sin que hayan presentado nunca episodios manacos es-
123456789
7

213249 6 pontneos; oficialmente no se pueden catalogar como bipolares, pero
debera tenerse en cuenta en ellos la posibilidad de tratamiento de
123456 78696
86 886 56 prevencin de recadas con litio.
4584 4  524 
Pregunta 24.-R: 5
649 6 4  A mediados del siglo XX, se pudo diferenciar la enfermedad bipolar
de la enfermedad depresiva atendiendo a la mayor frecuencia de ante-
66 46 56 524  cedentes familiares afectivos en la primera; de hecho, hay autores que
ante un paciente depresivo con antecedentes familiares de enfermedad
Pregunta 22.-R: 2 bipolar proponen el tratamiento profilctico con litio, entendiendo
En los sndromes afectivos ms graves los pacientes pueden presen- que ser cuestin de tiempo el que presenten un episodio maniforme.
tar sntomas psicticos; en general estos sntomas son ms probables Otras diferencias entre la enfermedad bipolar y la depresiva son de tipo
en los cuadros manacos que en los depresivos; los delirios son mu- epidemiolgico (predominio de mujeres en la enfermedad depresiva,
cho ms frecuentes que las alucinaciones, por lo que usamos los inicio ms temprano en la enfermedad bipolar, mayor riesgo [relativo]
trminos depresin (o mana) psictica o delirante de forma similar. de suicidio en la enfermedad bipolar). Se han realizado numerosas
En la mayora de los pacientes con estos sntomas encontramos una comparaciones entre las recadas depresivas de ambas enfermedades;
relacin lgica entre el contenido de los delirios y alucinaciones y su parece que las depresiones bipolares tienden a ser algo ms cortas que
estado de nimo (sntomas psicticos congruentes, ideas deliroides o las unipolares (6 meses frente a 12 meses), aunque con una mayor
ideas delirantes secundarias); si estn deprimidos, su visin pesimista frecuencia de recidivas a lo largo de la vida (6-9 frente a 2-3). En general,
se extiende hacia el pasado (ideas de culpa), el presente (ideas de los sntomas son muy parecidos, aunque se describen ms sntomas
minusvala y enfermedad) y el futuro (ideas de ruina); si estn mana- atpicos en las formas bipolares (sobre todo en las bipolares-2) y una
cos, su optimismo y autoconfianza puede llegar a los delirios de gran- mayor tendencia a la agitacin psicomotora en las depresiones
deza. En algunos casos los delirios y alucinaciones se alejan de los unipolares. No hay que olvidar que en la enfermedad bipolar pode-
contenidos lgicos (delirios de perjuicio, delirios de control), plan- mos ver cualquier variante de EDM (melanclico, atpico, psictico,
teando el diagnstico diferencial con la esquizofrenia; no hay un catatnico) y diferentes formas de episodios manacos (mana clsica
acuerdo acerca de la importancia de estos sntomas psicticos incon- eufrica, mana atpica disfrica, hipomana, episodios mixtos).
gruentes con el estado de nimo, llegando incluso a proponer la
Clasificacin Internacional de Enfermedades el diagnstico de trastor-
no esquizoafectivo para estos pacientes (la DSM americana slo acep-
ta este diagnstico cuando los pacientes cumplen POR COMPLETO
tanto los criterios de esquizofrenia como los del trastorno afectivo).

Comentarios TEST
Pregunta 24. Espectro bipolar.
Pregunta 22. Escalera de intensidad de la depresin y adaptacin del trata- Pregunta 25.-R: 3
miento. La mayora de los pacientes bipolares debutan con un EDM, sien-
do de especial inters la posibilidad de predecir una evolucin bipolar
Pregunta 23.-R: 3 desde el primer episodio. El principal factor de riesgo para ser bipolar
La respuesta habitual de un paciente deprimido al tratamiento con es tener antecedentes familiares de esa enfermedad. En las mujeres un
antidepresivos es la vuelta a la normalidad. De hecho, si una persona EDM puerperal evolucionar hacia una enfermedad bipolar en cerca
que no est deprimida toma estos medicamentos NO va a notar un del 80% de los casos. El debut en la adolescencia tiende a ser ms
aumento del estado de nimo, sino simplemente efectos secundarios frecuente en las formas bipolares (pero NO es determinante). La res-
(los antidepresivos NO son euforizantes y los euforizantes, como la puesta anormalmente rpida al tratamiento antidepresivo y la apari-
cocana, NO tienen efecto antidepresivo). En algunos casos los pa- cin de cuadros maniformes secundarios al mismo son tambin dos
cientes sufren un cambio radical de su estado de nimo en respuesta caractersticas que sugieren bipolaridad. Aunque algunos trabajos en-
al tratamiento, alcanzando una intensidad hipomanaca o manaca a cuentran una mayor frecuencia de ideacin delirante en las formas
gran velocidad. Esta respuesta tan rpida es tpica de los pacientes bipolares, NO se ha podido comprobar que esa ideacin tenga en las
bipolares, recomendndose en ellos la retirada del frmaco antide- formas bipolares un carcter incongruente con ms frecuencia que
presivo en cuanto se consigue la normalizacin para evitar el cambio en las formas unipolares.

M exico A rgentina CTO Medicina C/ Nez de Balboa, 115 28006 MADRID (Espaa) Tfno.: (91) 782 43 32 / Fax: (91) 782 43 27
Pg. 6 PQ C hile U ruguay E-mail: secretaria@ctomedicina.com; iberocto@ctomedicina.com WEB: www.ctomedicina.com; www.iberocto.com
PSIQUIATRA
Preparacin Examen de Seleccin 05/06 1 Vuelta Seguimiento a distancia
Cuando los pacientes debutan con un episodio manaco, el diag- suicidas y de intento de suicidio para referirse a los pacientes que se
nstico es inmediato; los varones tienen una mayor tendencia a pre- autolesionan por otras razones.
sentar este tipo de comienzo, lo que plantea problemas de diagnsti-
co con la esquizofrenia (varones, jvenes, conducta desorganizada y Pregunta 28.-R: 2
agitada, delirios y alucinaciones) y las conductas disociales (varones, Se llama depresin secundaria o exgena a todo sndrome depresi-
jvenes, conductas arriesgadas, abuso de sustancias). vo para el que se encuentra una causa mdica, txica o farmacolgi-
ca. En la prctica destacaremos 3 causas especialmente frecuentes:
Pregunta 26.-R: 3 Frmacos y drogas: aquellos frmacos que alteran la funcin nor-
Actualmente se aceptan dos variantes mayores de trastorno afecti- adrenrgica central (beta-bloqueantes, reserpina, alfa-metil-DOPA,
vo bipolar (TAB). En el TAB-1, los pacientes tienen episodios manacos clonidina), frmacos con actividad esteroidea (estrgenos, proges-
o mixtos, asociados casi siempre a episodios depresivos (EDM o de- tgenos, corticoides, ACTH), intoxicacin crnica por drogas se-
presiones menores); como se ve, cabe la posibilidad de que un pa- dantes (alcohol, opiceos, benzodiacepinas) y abstinencia de dro-
ciente tenga slo episodios manacos o mixtos, pero es algo excepcio- gas estimulantes (cocana, anfetaminas).
nal y no parece que evolucionen de forma diferente a los pacientes Enfermedades endocrinolgicas: sobre todo el hipotiroidismo, pero
manaco-depresivos. En el TAB-2 veremos EDM junto a episodios hi- tambin el hipertiroidismo (sobre todo en ancianos), el hiperpara-
pomanacos; estas hipomanas son difciles de diagnosticar y no es tiroidismo o los trastornos adrenales (Cushing, Addison).
raro que estos pacientes se clasifiquen errneamente como depresi- Enfermedades neurolgicas: tanto las enfermedades degenerativas
vos, privndoles del tratamiento con estabilizadores, lo que explica que afectan de forma ms o menos difusa a todo el Sistema Nervio-
su peor pronstico a largo plazo (mayores tasas de suicidio, ms ten- so Central (Alzheimer, Parkinson) como algunas enfermedades lo-
dencia a la ciclacin rpida); en los pacientes con TAB-2 vemos un calizadas (tumores, accidentes vasculares, abscesos) que lesionan
ligero predominio femenino, mientras que en los TAB-1 hay un equi- sobre todo las regiones frontales.
librio entre sexos. Se acepta slo una forma menor de trastorno bipolar,
la ciclotimia, definida por la alternancia de sntomas hipomanacos y La isoniacida fue, en realidad, el primer medicamento antidepresi-
sntomas depresivos menores durante un perodo mnimo de 2 aos. vo utilizado en Psiquiatra, a principios de los aos 50, dando origen
La alternancia de episodios depresivos invernales y episodios ma- a los primeros IMAOs; por tanto, comparte con ellos la capacidad de
niformes en verano caracteriza a la forma estacional nrdica del TAB; producir cuadros maniformes.
suelen ser TAB-2 (EDM ms hipomana).
La aparicin de un EDM sobre una distimia previa se conoce a Pregunta 29.-R: 5
veces como depresin doble. Implica una mayor gravedad que los La utilidad de los antidepresivos va mucho ms all de los sndro-
EDM aislados, con menor tendencia a la recuperacin. mes depresivos, de ah que en la actualidad se prefiera denominarles
por su mecanismo de accin (ISRS, IMAOs, inhibidores no selectivos
Pregunta 27.-R: 4 de la recaptacin).
En Psiquiatra diferenciamos dos conceptos referidos al suicidio
que popularmente se mezclan de forma inadecuada. En primer
lugar hablamos de SUICIDIO cuando una persona se autolesiona de Pregunta 29. Otros usos de los antidepresivos.
forma voluntaria y deliberada con la intencin de acabar con su vida.
Por otra parte, hablamos de AUTOLESIN (sin ms) cuando una per- En Psiquiatra:
sona se provoca algn tipo de dao fsico sin que existiera un deseo Trastornos de ansiedad:
explcito de muerte. Estas autolesiones son mucho ms frecuentes que - Pnico: ISRS, ADT, IMAOs.
los verdaderos intentos de suicidio (100 veces o ms) y en general se - TOC: ISRS, ADT (clomipramina).
pueden dividir en aquellas que tienen una finalidad MANIPULADO- - Fobia social: IMAOs, ISRS.
RA (distraer o llamar la atencin del entorno, conseguir algn tipo de - TEPT: ISRS, IMAOs, ADT.
beneficio) y aquellas que tienen un carcter ms IMPULSIVO (como - TAG: IRSN, ISRS, ADT.
las reacciones en cortocircuito de los pacientes con personalidad lmi- Trastornos somatomorfos:
te). Ante cualquier autolesin no accidental debe solicitarse una valo- - Hipocondra / dismorfofobia.
racin psiquitrica que determinar la intencionalidad de la misma. - Somatizacin / dolor.
Trastornos psicticos:
Comentarios TEST

La mayora de las autolesiones no suicidas se presentan en jvenes,


sobre todo mujeres, que se enfrentan a una situacin estresante ms o - Depresin postesquizofrenia.
menos grave, careciendo de otros recursos psicolgicos para abordar- Trastornos por uso de sustancias:
la (con frecuencia hay rasgos patolgicos de personalidad); suelen - Abstinencia de la cocana: ADT (desipramina).
elegirse mtodos de baja letalidad (intoxicacin con medicamentos, - Prevencin de la neurotoxicidad por xtasis (MDMA): ISRS.
cortes con armas blancas) y no se prepara el acto con antelacin, Trastornos infantiles:
siendo relativamente fcil el rescate (a veces se llevan a cabo en pre- - Dficit de atencin: ADT.
sencia de la propia familia); tras su atencin mdica en Urgencias y la - Enuresis: imipramina.
valoracin psiquitrica se les remite a los dispositivos ambulatorios, - Mutismo selectivo: ISRS, IMAOs?
siendo excepcional la necesidad de ingreso. En cambio, la mayora Trastornos del sueo:
de los intentos de suicidio se dan en personas mayores de 50 aos - Insomnio crnico: ADT sedantes, mirtazapina.
(una excepcin notable son los intentos de suicidio que presentan los - Narcolepsia (catapleja): ADT, IMAOs, ISRS?
jvenes con esquizofrenia), varones, que sufren enfermedades psi- Trastornos de la alimentacin:
quitricas bien definidas (depresin, abuso de sustancias, - Bulimia: ISRS en dosis altas, IMAOs?
esquizofrenia), asociadas o no a otros factores adversos de tipo mdi- - Obesidad: sibutramina, ISRS?
co (enfermedades crnicas, dolorosas, incapacitantes) o social (viven Trastornos de la personalidad:
solos, han perdido una relacin significativa recientemente, pasan - Lmite: ISRS.
por dificultades econmicas); suelen utilizarse mtodos ms agresivos
(precipitacin desde gran altura, ahorcamiento, armas de fuego) y se Fuera de la Psiquiatra:
planifica mejor el acto, evitando ser descubiertos; si sobreviven al Dolor crnico neuroptico: ADT (amitriptilina, clomipramina).
intento de suicidio, suelen ingresar y es necesario adoptar una serie de Fibromialgia reumtica: ADT (amitriptilina).
precauciones para que no vuelvan a intentar suicidarse durante el Cefaleas:
ingreso (sedacin, contencin mecnica, vigilancia constante). - Migraas.
Sin embargo, conviene tener en cuenta que algunos libros hablan - Cefaleas tensionales (amitriptilina).
de suicidio consumado para referirse a los pacientes autnticamente

M exico A rgentina CTO Medicina C/ Nez de Balboa, 115 28006 MADRID (Espaa) Tfno.: (91) 782 43 32 / Fax: (91) 782 43 27
C hile U ruguay E-mail: secretaria@ctomedicina.com; iberocto@ctomedicina.com WEB: www.ctomedicina.com; www.iberocto.com
PQ Pg. 7
PSIQUIATRA
Seguimiento a distancia Preparacin Examen de Seleccin 05/06 1 Vuelta
Dentro de los trastornos psiquitricos van a usarse antidepresivos en todo este proceso de tratamiento se simplifica, pues la dosis inicial suele
casi todos los trastornos por ansiedad (en el pnico para prevenir las ser la dosis suficiente (no hay que subir poco a poco, para que el
crisis, en la fobia social grave, en el trastorno obsesivo-compulsivo, en el paciente se acostumbre a los efectos adversos como con los AD tricclicos),
trastorno por estrs postraumtico y, recientemente, en el trastorno por tanto para el tratamiento agudo y de continuacin, como para el trata-
ansiedad generalizada). Tambin los utilizamos para frenar la impulsivi- miento de mantenimiento a largo plazo.
dad en pacientes con trastornos de la conducta alimentaria (atracones Se discute si alguno de los AD ms modernos (venlafaxina o
bulmicos), trastornos del control de los impulsos (juego patolgico o mirtazapina) tendran un efecto algo ms rpido.
ludopata), personalidades tipo lmite o trastornos por abuso de sustan-
cias (alcoholismo). Dentro de estos ltimos, los antidepresivos van a Pregunta 32.-R: 5
tener cierta capacidad de aliviar el sndrome de abstinencia de la coca- Los AD tricclicos (ADT) son los AD ms antiguos y, por tanto, aque-
na y parece que protegen de la neurotoxicidad inducida por xtasis llos de los que se dispone de ms experiencia. De hecho, la imiprami-
y otras anfetaminas de diseo. En algunos trastornos psiquitricos pro- na se considera el AD patrn con el que deben compararse los dems
pios de la infancia (enuresis, trastorno por dficit de atencin) pueden antidepresivos. Los ADT logran su efecto antidepresivo aumentando
utilizarse, aunque no sean el tratamiento de primera eleccin. Por la la disponibilidad de las principales monoaminas (noradrenalina, se-
capacidad sedante de algunos antidepresivos se proponen stos como rotonina y dopamina) al inhibir su recaptacin. Sin embargo no lo
alternativa a las benzodiacepinas en algunos casos de insomnio. hacen de una forma selectiva, sino que tambin bloquean numerosos
Ya en un terreno ms neuropsiquitrico, vemos cmo los antide- receptores, cada uno de los cuales va a producir una serie de efectos
presivos tricclicos se utilizan para la prevencin de la cefalea tensio- secundarios. No todos los ADT son iguales en esta caracterstica; los
nal y el tratamiento de dolores neuropticos crnicos. Tienen adems hay con potentes efectos anticolinrgicos (amitriptilina, clorimipramina)
la capacidad de reducir la frecuencia de episodios de catapleja en la y los hay con menos problemas a ese nivel (nortriptilina, desipramina);
narcolepsia. El efecto antihistamnico de la doxepina supera a la ma- estos efectos muscarnicos son muy mal tolerados, especialmente por
yora de los antihistamnicos oficiales, y no es raro que se use para el pacientes ancianos. Asmismo tenemos ADT muy sedantes por su efecto
tratamiento del prurito idioptico. antihistamnico (amitriptilina, doxepina, clorimipramina) y otros ms
activadores con escasa capacidad sedante (imipramina). Lo que suele
Pregunta 30.-R: 3 ser necesario en todos los ADT es comenzar con dosis muy bajas e ir
A la hora de enfrentarnos al tratamiento de un sndrome depresivo, subiendo poco a poco, para buscar una cierta tolerancia a los efectos
vamos a decidirnos por un tipo de tratamiento u otro en funcin de la adversos. Un problema adicional de los ADT son sus efectos cardio-
intensidad de los sntomas. Cuando un sndrome depresivo se acerca vasculares; por su capacidad de bloqueo adrenrgico pueden causar
a la intensidad de EDM, la probabilidad de respuesta a frmacos hipotensin ortosttica y por su accin parecida a la quinidina sobre
antidepresivos (AD) supera el 60%, siendo mayor an en los casos de la conduccin cardaca cabe la posibilidad de que produzcan arritmias
EDM con sntomas melanclicos. En las formas extremas de depre- potencialmente letales. La intoxicacin con ADT supone un serio
sin (depresin psictica) ser necesario combinar AD con antipsic- problema en los pacientes depresivos con ideas suicidas. Sin embar-
ticos o recurrir al tratamiento con electrochoque. En las formas meno- go, todava vamos a ver utilizarlos atendiendo a tres ventajas:
res de depresin la respuesta a frmacos ronda el 40%, logrndose Su precio (7-10 veces inferior al precio de los nuevos AD).
ms el alivio sintomtico que la curacin completa. La posibilidad de controlar el cumplimiento a travs de los niveles
Una vez decidida la necesidad de poner un AD, la eleccin entre plasmticos (sobre todo con imipramina).
los distintos compuestos depender de: Su posible mayor potencia antidepresiva (que se explicara por el
Los antecedentes de respuesta en episodios previos. hecho de que la mayora de los ADT actan a un doble nivel:
La presencia de otras enfermedades que contraindiquen el uso de noradrenrgico y serotoninrgico).
un AD en concreto (por ejemplo, una cardiopata contraindicara
el uso de AD tricclicos).
La toma de alguna medicacin que interaccione con el AD.
El perfil de efectos adversos del AD que puedan provocar su recha-
zo por el paciente.
Los antecedentes de respuesta a un AD en concreto en un familiar
de primer grado.
El patrn de sntomas dentro de la depresin (por ejemplo, la pre-

Comentarios TEST
sencia de un patrn atpico indicara el uso de IMAOs).
La comodidad de su uso.
El precio del frmaco.

Teniendo en cuenta que la mayora de los pacientes se presentan sin


haber tomado nunca medicacin AD o no la recuerdan, los ISRS renen
las caractersticas idneas para convertirse en los AD de primera eleccin.

Pregunta 31.-R: 3
Los antidepresivos no curan la depresin rpidamente, tienen un
efecto a medio plazo, tardando varias semanas (4-6) en conseguir la
remisin de los sntomas, aunque previamente ya se hayan detectado
signos de respuesta. Una vez alcanzada la remisin, se propone la
necesidad de continuar con el mismo tratamiento (el mismo AD, a la
misma dosis) un mnimo de 6 meses para prevenir la recada del episo- Pregunta 32. Esquema de efectos de los antidepresivos tricclicos y posibles
dio. Pasado ese tiempo (que dependiendo de diversos factores, como la efectos secundarios.
mayor edad, la gravedad del episodio o el nmero de episodios pre-
vios, podr alargarse), se considera que el paciente se ha recuperado Pregunta 33.-R: 1
del episodio depresivo. Entonces se podr retirar el AD (casi siempre de A principios de los 80, los ISRS irrumpen en el mercado psicofar-
forma progresiva, en unas semanas para minimizar posibles sntomas macolgico con tres ventajas evidentes:
abstinenciales), salvo a los pacientes que hayan sufrido numerosos epi- Un perfil de efectos secundarios mucho ms benfico.
sodios, a los que se les propondr tratamiento indefinido de manteni- Una mayor comodidad de uso, al no tener que aumentar lenta-
miento (que suele realizarse en la dosis mnima posible para reducir los mente la dosis, ni fraccionarla en varias tomas diarias.
efectos adversos y mejorar as el cumplimiento). Con los modernos ISRS La casi ausencia de letalidad en el caso de sobredosis.

M exico A rgentina CTO Medicina C/ Nez de Balboa, 115 28006 MADRID (Espaa) Tfno.: (91) 782 43 32 / Fax: (91) 782 43 27
Pg. 8 PQ C hile U ruguay E-mail: secretaria@ctomedicina.com; iberocto@ctomedicina.com WEB: www.ctomedicina.com; www.iberocto.com
PSIQUIATRA
Preparacin Examen de Seleccin 05/06 1 Vuelta Seguimiento a distancia
por razones cientficas, desde finales de los 80 asistimos a un renaci-
Pregunta 33. Ventajas y desventajas de los antidepresivos miento de la TEC. El TEC tiene tres ventajas sobre los frmacos AD:
tricclicos y los ISRS. Es ms potente en su accin antidepresiva, lo que explica su prin-
cipal indicacin, la depresin resistente a AD, y el que, en ocasio-
12345678694
623  9 69
623  9 nes, se utilice como primera opcin en la forma ms grave de
depresin, la depresin psictica o delirante.
123456765727895
572 12 2 4 692 Logra su efecto antidepresivo ms rpido que los frmacos (en 2-3
22
9 7
7 22 68
7 5 semanas cuando se aplica a das alternos), lo que explica su utili-
1 12 65 12 975
7
2 82
5 dad en las depresiones estuporosas o catatnicas y en las depresio-
1289 2
285  2 12726  7 nes con graves ideas suicidas.
22 7956255 7587 2276 8 24 7665875 89 Tiene escasos efectos secundarios fsicos, lo que explica su utilidad
en casos de intolerancia o contraindicacin para el uso de ADT, en
123672 4 692 12 652  7

pacientes polimedicados o con patologa orgnica compleja y en las
22 68
7 5 12!89 76658 2
psicosis graves durante el embarazo.
 1236772 975
7
2 82 2247 76658"9567
22
5 12#3456765727895
572
Slo existe una contraindicacin absoluta para el TEC: la presen-
12
5
7
2
2 22 8 $
cia de hipertensin intracraneal (pues durante cualquier crisis epilp-
tica aumenta la produccin de lquido cefalorraqudeo y cabra la
En apenas 10 aos desplazan casi por completo a los ADT y a los posibilidad de una herniacin cerebral). Otras contraindicaciones
IMAOs en el terreno de la depresin, y poco a poco van avanzando dependen del procedimiento anestsico y cada vez son ms raras.
hacia otros campos (pnico, trastorno obsesivo, bulimia, fobia social). El principal efecto secundario es de tipo neuropsicolgico: la pro-
En la actualidad se han convertido en los AD de primera eleccin en duccin de una amnesia antergrada. Depende del estado cognitivo
la depresin y en la mayora de esas otras indicaciones. Sin embargo, previo (ms frecuente en ancianos), del nmero de sesiones recibidas,
tienen tres aspectos discutibles: de la carga elctrica suministrada y de la disposicin de los electrodos.
Su precio es 7-10 veces superior al precio de los ADT, aunque los Al cabo de unos meses NO existen secuelas del tratamiento (salvo
estudios de coste-efectividad han demostrado que la diferencia de ciertas lagunas de memoria consecuencia del perodo de amnesia) y
precio del frmaco se compensa por otros costes relacionados con el funcionamiento cognitivo de los pacientes es incluso mejor que si
la depresin (das de baja laboral, necesidad de ingreso, necesidad hubieran recibido frmacos, pues la recuperacin de la depresin
de medicacin adicional para tratar efectos adversos). llega a ser ms completa.
Aparentemente tienen una menor potencia antidepresiva (debida La mala fama del TEC procede de la poca inicial, en la que se
quizs a su efecto exclusivamente serotoninrgico), aunque los es- aplicaba sin anestesia (por estar poco desarrollados los procedimien-
tudios sobre efectividad en condiciones reales (no de ensayo clni- tos anestsicos) y en la que se usaba de forma indiscriminada para casi
co) demuestran que los pacientes tienden a abandonar el trata- cualquier tipo de paciente psiquitrico. No hay que olvidar que fuera
miento con ADT o no alcanzan dosis adecuadas por culpa de los de la depresin mayor el TEC apenas tiene indicaciones; se puede
efectos adversos, con lo que la mayor eficacia de los ADT se pierde, usar en los episodios manacos, pero es raro que stos sean resistentes
igualndose su efectividad a la de los ISRS. a los frmacos; tambin se va a usar en episodios esquizofrnicos
Van a presentar numerosas interacciones farmacocinticas en los agudos resistentes a antipsicticos, sobre todo si existen sntomas cata-
citocromos P450 hepticos, a los que inhiben en mayor o menor tnicos o si hay clnica depresiva asociada.
medida (parece que citalopram y sertralina menos que los dems
ISRS) modificando al alza los niveles plasmticos de diversos frma- Pregunta 36.-R: 4
cos, algunos potencialmente peligrosos por sus efectos cardiotxicos El litio es el estabilizador de primera eleccin en la enfermedad
(como los ADT, algunos antihistamnicos modernos como la terbi- bipolar. Su principal beneficio es la reduccin del nmero de reca-
nafina o el astemizol y algn procintico como el cisapride). das (sobre todo manacas); tambin consigue atenuar la intensidad de
las mismas, caso de que se produzcan. Ms discutibles son sus efectos
Pregunta 34.-R: 4 sobre los episodios agudos; como antidepresivo, el litio es poco po-
Los IMAOs tenan un perfil de tolerancia ligeramente mejor que los tente y no es raro que se deban usar AD asociados (con las precaucio-
Comentarios TEST

ADT, pero dos importantes problemas prcticos: nes debidas para no inducir un cambio de fase); como antimanaco,
La necesidad de seguir una dieta para evitar posibles crisis hiper- el litio es muy eficaz pero tiene un efecto retardado (varias semanas)
tensivas secundarias a compuestos vasopresores derivados de las que puede resultar inaceptable para pacientes manacos graves; ade-
aminas (tiramina). ms no se dispone de otra presentacin aparte de la oral, lo que
Algunas interacciones farmacolgicas potencialmente peligrosas por condiciona su uso a la aceptacin del tratamiento por parte del pa-
sus efectos noradrenrgicos (vasoconstrictores nasales) o serotoni- ciente (que no es lo habitual en las formas graves); el litio es especial-
nrgicos (ISRS, meperidina). mente eficaz en los cuadros de mana clsica (eufrica), disminuyen-
do su eficacia en las manas disfricas (irritables). Su efecto en las fases
Aunque la dieta no es complicada (bsicamente se trata de evitar mixtas es tambin bastante discreto, prefirindose en estos casos otros
todos los alimentos que han sufrido procesos de fermentacin, como el frmacos (valproico, carbamacepina, antipsicticos atpicos).
queso, las bebidas alcohlicas fermentadas como el vino o la cerveza, Adems, el litio es un medicamento con numerosos efectos secunda-
los ahumados, las conservas y los salazones) lentamente se han ido rios, hasta tal punto que si un paciente no refiere ningn problema duran-
abandonando; en la actualidad slo nos queda un IMAO clsico en te el tratamiento, probablemente no lo est tomando. Para prevenir los
Espaa (tranilcipromina), aunque en otros pases (Francia, Inglaterra, efectos adversos ms graves se realizan una serie de controles mdicos
EE.UU.) an podemos encontrar otros en uso. Estrictamente hablando, antes y durante el tratamiento con litio; con diferencia, lo ms importante
habra 2 indicaciones oficiales para el uso de IMAOs en primera instan- ser el control de los niveles plasmticos de litio (litemia), pues guardan una
cia: la depresin con sntomas atpicos y la fobia social grave. Sin em- relacin bastante estrecha con la eficacia y la toxicidad. Dado que el litio
bargo, incluso en esos casos es cada vez ms frecuente el uso de ISRS. se elimina por el rin, utilizando los mismos sistemas de transporte que el
sodio o el potasio, ser necesario disponer de parmetros que nos infor-
Pregunta 35.-R: 2 men del estado de la funcin renal (creatinina, BUN) y los iones mencio-
El tratamiento electroconvulsivo (TEC) fue la primera estrategia real- nados. Uno de los efectos ms peligrosos es la alteracin de la conduccin
mente eficaz para el tratamiento de la depresin; se comenz a utilizar cardaca; las alteraciones en el ECG permiten controlar hasta qu punto el
a finales de los aos 30, y hasta la revolucin farmacolgica de los aos litio produce problemas a este nivel (normalmente se van a limitar a cam-
50-60 fue el instrumento preferido para el tratamiento agudo de la bios en la onda T). El litio es un medicamento teratgeno; se ha relaciona-
depresin. Tras unos aos de rechazo ms por motivos polticos que do con malformaciones cardacas (anomala de Ebstein), por lo que antes

M exico A rgentina CTO Medicina C/ Nez de Balboa, 115 28006 MADRID (Espaa) Tfno.: (91) 782 43 32 / Fax: (91) 782 43 27
C hile U ruguay E-mail: secretaria@ctomedicina.com; iberocto@ctomedicina.com WEB: www.ctomedicina.com; www.iberocto.com
PQ Pg. 9
PSIQUIATRA
Seguimiento a distancia Preparacin Examen de Seleccin 05/06 1 Vuelta
de iniciar el tratamiento con litio en una mujer en edad frtil conviene das).
asegurarse de que no est embarazada e informarla de la necesidad de Antagonistas de los canales de calcio, como el verapamil o el nife-
que adopte medidas de anticoncepcin eficaces; ante cualquier sospe- dipino, estudiados en casos de ciclacin rpida.
cha de embarazo habr que repetir la prueba diagnstica de gestacin
pues, de confirmarse, se debera interrumpir el tratamiento (nunca de Pregunta 40.-R: 2
forma brusca por el riesgo de recada). La ltima prueba obligatoria en el El litio es el medicamento de eleccin en la enfermedad bipolar
control rutinario del tratamiento con litio es la determinacin de la TSH pero no es la solucin definitiva. Hay pacientes que no toleran los
para detectar problemas tiroideos (que en ocasiones explican la resistencia efectos adversos, mientras que en otros est contraindicado (insufi-
al tratamiento). Otras determinaciones (glucemia, calcemia, pruebas de ciencia renal, psoriasis, enfermedad del nodo sinusal). Adems hay un
concentracin de la orina) dependern de la evolucin y de la aparicin porcentaje de pacientes (30-40%) que no responden bien al litio:
de efectos secundarios. Las manas atpicas o disfricas (irritables).
El electroencefalograma NO es una prueba necesaria para el con- Los episodios mixtos.
trol del litio ni de ningn otro psicofrmaco; cuando aparecen altera- Los cuadros manacos secundarios a enfermedades mdicas.
ciones en el EEG es que ya se ha provocado neurotoxicidad, que con Los pacientes con ciclacin rpida.
frecuencia ser irreversible.
La ciclacin rpida se ve con ms frecuencia en mujeres, sobre
Pregunta 37.-R: 5 todo con TAB-2. En ocasiones se detectan alteraciones tiroideas que
Los niveles de litio deben estar entre 0,4 y 1,5 mEq /L. Lgicamente necesitan ser corregidas para frenar la ciclacin. En general no res-
en los episodios agudos sern necesarias litemias ms altas (superiores ponden bien a la monoterapia con litio; quizs respondan algo mejor
a 1,0 mEq/L) que en los perodos de estabilidad (inferiores a 1,0 mEq/ al tratamiento con carbamacepina o valproico, pero lo ms habitual
L); la mayora de los pacientes van a estar un poco por arriba o por es utilizar como mnimo dos estabilizadores. Una de las causas ms
debajo de esta cifra (1,0 mEq/L), para no provocar una excesiva toxi- frecuentes de ciclacin rpida es el tratamiento farmacolgico inco-
cidad o perder la eficacia. rrecto, fundamentalmente el abuso de AD y la no utilizacin de esta-
Si el episodio manaco o depresivo no es demasiado grave quizs bilizadores (lo cual es frecuente en los pacientes con TAB-2, por las
baste con una ligera correccin de la dosis del estabilizador, pero en dificultades para el diagnstico de las hipomanas).
los casos graves ser necesario aadir tratamiento especfico; en los
casos de mana grave la falta de colaboracin del paciente suele Pregunta 41.-R: 3
obligar al tratamiento inicial con antipsicticos (intramusculares), La esquizofrenia es una enfermedad frecuente, afectando a cerca del
dejando la adicin del estabilizador para ms adelante. Una vez 1% de la poblacin en algn momento de su vida; teniendo en cuenta
conseguido el control de la recada se retirar el frmaco especfico que tan slo 1/3 de los casos se pueden considerar de buen pronstico,
de la fase en cuanto sea posible, para no inducir un cambio de con una recuperacin ms o menos completa, se calcula que en Espaa
polaridad. hay cerca de 250.000 personas con la enfermedad. Tiende a aparecer
en la adolescencia o juventud (de los 15 a los 35 aos), aunque se han
Pregunta 38.-R: 4 descrito casos de inicio en la infancia y en la vejez. Afecta por igual a
Otro problema adicional del litio es la posibilidad de que los niveles hombres y mujeres, si bien en las mujeres tiene un inicio algo ms tardo
plasmticos se modifiquen por factores que afectan la eliminacin re- (quizs por el efecto protector de los estrgenos contra la actividad
nal, sin que el paciente haya variado la dosis. Dado que el litio utiliza los dopaminrgica excesiva), lo que determina un mejor pronstico. No se
mismos sistemas de transporte renal que el sodio, cualquier circunstan- han descrito variaciones de su incidencia en funcin de la clase social, la
cia que aumente la reabsorcin de este ion (hipovolemia, deshidrata- cultura o la raza; sin embargo la mayora de los pacientes tienen un nivel
cin, hiponatremia) producir un aumento de los niveles de litio, supe- socioeconmico bajo, como consecuencia del deterioro que produce
rando el umbral de toxicidad. Numerosos frmacos van a producir los la enfermedad (hiptesis del descendimiento social); el pronstico pare-
mismos problemas, bien por sus efectos sobre la vascularizacin renal ce ms favorable cuanto menor es el nivel de desarrollo de la regin (p.ej.
(AINEs, inhibidores de la COX-2), bien por sus efectos sobre el tbulo mejor en zonas rurales que en reas urbanas). Estadsticamente los pa-
renal (diurticos, IECAs, ARA-2). En teora podramos ver una disminu- cientes tienen un mayor riesgo de muerte tanto por causas no naturales
cin de la litemia, con la consiguiente prdida de eficacia, en las situa- (suicidio) como por causas naturales.
ciones hidroelectrolticas inversas (hipernatremia, ingesta hdrica exce-
siva) y por algunos frmacos (metilxantinas, diurticos osmticos e inhi- Pregunta 42.-R: 4
En la actualidad, el diagnstico de esquizofrenia se basa en los

Comentarios TEST
bidores de la anhidrasa carbnica), pero son situaciones excepciona-
les. siguientes criterios:
Presencia de unos sntomas caractersticos que se han dividido de
Pregunta 39.-R: 2 una forma algo artificial en positivos (los que predominan en las
La carbamacepina y el cido valproico son las dos grandes alterna- fases agudas o brotes) y negativos (que son ms evidentes en las
tivas al litio (estabilizadores de segunda lnea); ambos tienen una toxici- fases residuales).
dad parecida (hematolgica y heptica), pero la carbamacepina es ms Ausencia de otras enfermedades psiquitricas, enfermedades m-
dicas, frmacos o txicos que los justifiquen.
txica y adems tiene una capacidad notable de induccin enzimti-
Produccin de un deterioro en el funcionamiento habitual del
ca, lo que explica el mayor uso del valproico (en EE.UU. se ha puesto a paciente (acadmico, social, laboral).
la altura del litio, como frmaco de primera lnea en mana aguda). Se Duracin superior a 6 meses, comprendiendo los prdromos, el
controlan de una forma similar a su uso en la epilepsia, aunque no se ha brote psictico (que debe durar como mnimo 1 mes, salvo que se
demostrado que los niveles plasmticos guarden relacin con su efica- trate precozmente) y la fase residual de recuperacin. (Ver tabla en
cia (s con la toxicidad). En general tienen ms efecto como antimanacos, la pgina siguiente).
siendo su efecto antidepresivo mnimo.
Otros estabilizadores, ya de tercera lnea, son:
Algunas benzodiacepinas de alta potencia, como el clonacepam, Pregunta 43.-R: 3
que se usan en casos de ciclacin rpida. Desconocemos cules son las causas precisas de esta enfermedad,
Antipsicticos atpicos, como la clozapina, la olanzapina o la rispe- pero se maneja un modelo terico que incluye factores que aumen-
ridona, que se usan en el control de las fases mixtas, las manas taran el riesgo para padecerla (factores predisponentes o de vulnera-
disfricas y en mana resistente al tratamiento. bilidad) y otros que contribuiran a su aparicin en individuos predis-
Antiepilpticos de ltima generacin, como el topiramato (estudia- puestos (factores precipitantes o desencadenantes).
do en mana aguda), la lamotrigina (estudiado en depresin bipolar Dentro de los primeros tenemos:
y como frmaco auxiliar en la prevencin de recadas) y la gabapen- Presencia de antecedentes familiares de esquizofrenia (quizs el
tina (estudiado como frmaco auxiliar en la prevencin de reca- ms importante).

M exico A rgentina CTO Medicina C/ Nez de Balboa, 115 28006 MADRID (Espaa) Tfno.: (91) 782 43 32 / Fax: (91) 782 43 27
Pg. 10 PQ C hile U ruguay E-mail: secretaria@ctomedicina.com; iberocto@ctomedicina.com WEB: www.ctomedicina.com; www.iberocto.com
PSIQUIATRA
Preparacin Examen de Seleccin 05/06 1 Vuelta Seguimiento a distancia
Haber nacido en los meses fros del ao (se ha relacionado con enfermo y su vida (no, por ejemplo, voces que insultan, que son con
infecciones por el virus de la gripe durante el embarazo). mucho las alucinaciones ms frecuentes de toda la Psiquiatra).
Antecedentes de problemas obsttricos. En los delirios destac tanto su contenido (delirios de control e
Enfermedades graves de la primera infancia (sobre todo las que influencia frente a los mucho ms frecuentes, pero ms inespecficos
afectan directamente al sistema nervioso central, como meningo- delirios de perjuicio) como la forma de aparicin (percepciones deli-
encefalitis, epilepsia grave, traumatismos craneales severos). rantes sin sentido, frente a las mucho ms frecuentes interpretaciones
delirantes, ms o menos conectadas con la realidad).
Los factores desencadenantes NO producen esquizofrenia en per- Ejemplo de percepcin delirante: veo una persona apoyada en una
sonas sin vulnerabilidad; su influencia es ms evidente en las recadas farola de la calle, iluminada por su luz; esa luz quiere decir que soy
de la enfermedad: el nuevo Mesas, y ese es el Demonio que viene a impedir que me
Txicos (sobre todo alucingenos, cannabis y estimulantes como manifieste.
la cocana o las anfetaminas). Ejemplo de interpretacin delirante: veo una persona apoyada en
Estrs social (relacionado sobre todo con el abandono del trata- una farola de la calle, iluminada por su luz; comienzo a pensar si
miento). no lleva mucho tiempo all y si no estar esperando a que salga
Enfermedades graves que afectan al sistema nervioso central. para hacerme dao.

Pregunta 46.-R: 3
Pregunta 42. Sntomas positivos y negativos de la esquizofrenia. En las fases agudas o brotes predominan los sntomas positivos y en
las fases residuales los sntomas negativos. El aplanamiento afectivo es
123454623 789
54623 un sntoma negativo.
Vamos a detenernos un poco en el diagnstico diferencial de los

123456761 845649
3 35 3 4354  
434 5455 sntomas negativos; stos pueden deberse a diferentes causas, cada
393
35456
 9 6 4 56

una de ellas con una solucin especfica:

6 76 841 9  6 18
En ocasiones, los sntomas se deben a que el enfermo est deprimido
3    23 
6 84
3  3
(es frecuente al principio de la enfermedad, cuando tienen todava
1  76 1
conciencia de su cambio), respondiendo al tratamiento antidepresi-
4234
64
373846  4$
56 843  3 435
vo.
4 3 434 35 
4 4443 4 396
Los frmacos antipsicticos tienen la capacidad de producir snto-
4447 6

 42 % 3 4347 6


mas extrapiramidales que mimetizan los sntomas negativos y res-
 4
543
6435
3 42 
395 4 35
3

4443
35 3 435
37 6
3  44435
37 6
3 
ponden a una reduccin o un cambio de la medicacin.
4!33
544 6 4 3 4!33
54 4 6
La repercusin social de la enfermedad (aislamiento, rechazo) con-
4441 9
3
4
561 434"#533
 4419

6 1434 & 3 3

duce a muchos enfermos a una situacin similar a la descrita en los
cuadros deficitarios que mejora radicalmente al comenzar un tra-
8 &!"84"'84 "(2"843  ()""84 "()"
tamiento de tipo rehabilitador.
46
3.4 3 4 35/3
Por ltimo, la persistencia de sntomas positivos intensos puede
4( 5653 
4(9 535 64 3 76 hacer que algunos enfermos rechacen el contacto social o se mues-
423 
6 
4( 6 8 tren retrados como consecuencia del miedo que tienen a ser da-
8  4$  65%

4$65  43
* 4+,-
4445#365 ados por los dems.
42 434  355
42 
354+,-
4(3 6459
6964+,-

Pregunta 44.-R: 4
Los trastornos del pensamiento son quizs (con las alteraciones de
la percepcin) los sntomas ms estudiados en la esquizofrenia; los
podemos dividir en trastornos del contenido del pensamiento (ideas
delirantes) y trastornos del curso o la forma del pensamiento.
Comentarios TEST

Dentro de las ideas delirantes, hablamos de ideas delirantes pri-


Pregunta 46. Historia natural de la esquizofrenia.
marias cuando parecen surgir de forma autnoma, independientes
del resto de fenmenos psicopatolgicos, siendo tpicas de la esqui-
Pregunta 47.-R: 2
zofrenia y la paranoia; las ideas delirantes secundarias (o ideas deli-
Los factores pronsticos de la esquizofrenia han sido preguntados
roides) proceden de fenmenos de tipo afectivo (depresin, mana,
en varias ocasiones; podemos dividirlos de una forma sencilla en:
angustia). Factores previos al inicio de la enfermedad:
Los pacientes tienen problemas para dirigir su pensamiento hacia - Sexo (mujeres mejor que varones).
unas conclusiones lgicas, presentando con frecuencia prdidas de - Antecedentes familiares (mejor si NO hay antecedentes de es-
sentido (descarrilamientos) que acaban por producir una sensacin quizofrenia).
de disgregacin del pensamiento; en otras ocasiones se van a quejar - Nivel de adaptacin previo (mejor cuanto mayor fuera).
de la interrupcin del curso del pensamiento (bloqueo). - Personalidad previa (mejor si era normal).
- Nivel de inteligencia previo (mejor si NO era bajo).
Pregunta 45.-R: 2 Factores relacionados con el debut de la enfermedad:
Schneider se plante cules de todos los sntomas de la esquizofre- - Edad de inicio (uno de los fundamentales; mejor cuanto ms
nia eran ms especficos de la enfermedad, siendo a la vez frecuentes tarda).
y de fcil diagnstico; buscaba lo que ahora llamaramos sntomas de - Presencia de factores precipitantes (mejor si los hay).
alto valor predictivo positivo; aunque se equivoc al creer que eran - Presencia de sntomas depresivos o manacos (mejor si los hay).
frecuentes (aparecen en slo un tercio de los pacientes), s hay que - Presencia de sntomas confusionales (mejor si lo hay).
reconocer que son altamente sugerentes de esquizofrenia, pero NO - Inicio rpido (mejor que si el inicio es insidioso).
patognomnicos. No olvidar que en la esquizofrenia son mucho ms - Patrn de sntomas (mejor si dominan los positivos).
frecuentes los sntomas negativos, que por desgracia son totalmente - Rapidez a la hora de iniciar un tratamiento.
inespecficos. Factores evolutivos:
Dentro de las alucinaciones destac las auditivas, en concreto aque- - Nmero de recadas (fundamentalmente en los 5 10 primeros
llas en las que se escuchan voces humanas que hablan entre s del aos).

M exico A rgentina CTO Medicina C/ Nez de Balboa, 115 28006 MADRID (Espaa) Tfno.: (91) 782 43 32 / Fax: (91) 782 43 27
C hile U ruguay E-mail: secretaria@ctomedicina.com; iberocto@ctomedicina.com WEB: www.ctomedicina.com; www.iberocto.com
PQ Pg. 11
PSIQUIATRA
Seguimiento a distancia Preparacin Examen de Seleccin 05/06 1 Vuelta
- Gravedad de los sntomas negativos (determinantes para el pro- Los ms antiguos tienen importantes efectos antihistamnicos (que
nstico social y laboral). producen somnolencia y aumento del apetito y del peso), efectos
- Resistencia al tratamiento de los sntomas positivos. antiadrenrgicos (con hipotensin ortosttica como consecuen-
cia) y efectos anticolinrgicos (produciendo el tpico sndrome
Pregunta 48.-R: 3 muscarnico: sequedad de boca, estreimiento, visin cercana bo-
En la prevencin de las recadas de la enfermedad esquizofrnica rrosa, retencin urinaria, alteraciones de memoria).
resulta fundamental el mantenimiento de un tratamiento antipsicti-
co a largo plazo; diversos factores influyen en el abandono del mismo Pregunta 52.-R: 3
(que es bastante frecuente y es la principal causa de recada): Dentro de los antipsicticos tpicos o neurolpticos encontramos
El perfil de efectos secundarios del medicamento: para evitar el diferencias en su potencia (capacidad de bloqueo dopaminrgico) que
abandono por esta razn debemos individualizar el tratamiento pueden ser compensadas con un aumento de dosis; la eficacia antipsi-
segn las preferencias del paciente; teniendo en cuenta que los ctica es similar en todos ellos. Al aumentar la dosis van a aparecer ms
efectos extrapiramidales son muy mal tolerados, los antipsicticos efectos sistmicos como consecuencia de los bloqueos colaterales des-
atpicos parecen tener una mejor aceptacin. critos (menor potencia, ms dosis, mayor sedacin, ms hipotensin,
La conciencia de enfermedad: puede mejorarse mediante tcni- ms acciones anticolinrgicas); en los antipsicticos menos potentes la
cas de psicoeducacin en la enfermedad, sobre todo a travs de presencia de un bloqueo anticolinrgico significativo va a disminuir la
terapias grupales. tasa de efectos extrapiramidales (puesto que as en la va nigroestriada
La gravedad del estrs psicosocial sufrido por el paciente, sobre todo los dos neurotransmisores estarn bloqueados).
el debido al ambiente familiar cuantificado por el concepto de la Las consecuencias de la hiperprolactinemia inducida por los anti-
expresin emocional: puede disminuirse mediante tcnicas de inter- psicticos son bien conocidas: en mujeres veremos amenorrea, gine-
vencin familiar, el uso de recursos como centros de da u hospitales comastia, galactorrea y disminucin de la libido; en varones,
de da y, en ltima instancia, mediante alternativas residenciales. disfunciones sexuales y disminucin de la libido sobre todo. Dentro de
los antipsicticos clsicos, destaca la capacidad del sulpiride para indu-
Pregunta 49.-R: 2 cir aumentos enormes de la prolactina, incluso cuando se usa en dosis
La paranoia o trastorno delirante es una enfermedad sorprendente bajas.
en la que los pacientes tienen bsicamente un nico sntoma (un
delirio), con una relativa preservacin del resto del funcionamiento
psicolgico y un escaso deterioro de su personalidad. Podemos ver
cuadros paranoicos de inicio agudo, en respuesta a un estrs identifi-
cable (como los descritos en presos o inmigrantes), pero la paranoia
clsica tiene un inicio insidioso, casi siempre entrelazado con una
personalidad previa peculiar (paranoide, demasiado sensible). Cuan-
do el delirio toma importancia el paciente suele encontrarse en la
edad adulta, sufriendo consecuencias en los terrenos social, familiar o
laboral segn los contenidos del delirio. Tiene un curso crnico, con
una respuesta discreta del delirio al tratamiento; con la medicacin se
va a buscar ms un control conductual que una disolucin del delirio
(encapsulacin).
Las clasificaciones actuales aceptan 5 formas de delirios crnicos
paranoicos:
Delirios de perjuicio (los ms frecuentes, pero muy poco especficos).
Delirios de celos (relacionados en ocasiones con el alcoholismo).
Delirios de enfermedad (hipocondra delirante).
Delirios de grandeza.
Delirios de enamoramiento.

Pregunta 50.-R: 3

Comentarios TEST
El mecanismo por el cual los pacientes paranoicos llegan al delirio
es la interpretacin delirante de la realidad; con frecuencia basan sus
delirios en hechos que han sucedido, pero a los que dan un significa-
do nuevo dentro de lo razonable (a diferencia de los delirios esquizo-
frnicos, que pueden ser increbles). Psicoanalticamente se dice que Pregunta 52. Relacin entre dosis y potencia en los antipsicticos clsicos.
estos pacientes recurren fundamentalmente a la proyeccin como
mecanismo psicolgico de defensa (colocar en el exterior la causa de Pregunta 53.-R: 4
mis problemas, por ejemplo, si no encuentro algo es porque me lo Los modernos antipsicticos atpicos se diferencian tericamente
han quitado). de los antipsicticos clsicos en tres caractersticas:
Clsicamente no se admite la existencia de alucinaciones en los Producen menos efectos extrapiramidales.
trastornos delirantes (s son frecuentes las ilusiones), aunque podemos Tienen un mayor efecto antipsictico que los clsicos (tomando
ver alteraciones de la percepcin (sobre todo tctiles u olfativas) que como referencia el haloperidol).
se encuentren al margen de lo alucinatorio. Tienen cierto efecto sobre los sntomas negativos de la enfermedad
(accin de la que carecen los antipsicticos tpicos).
Pregunta 51.-R: 4
Los antipsicticos clsicos ejercen diferentes acciones sobre los Cmo consiguen esta eficacia diferencial?:
sistemas de neurotransmisin cerebrales: Algunos de ellos tienen la capacidad de bloquear mltiples recep-
Poseen la capacidad de bloquear los receptores dopaminrgicos; el tores (dopaminrgicos, serotoninrgicos, histaminrgicos, adrenr-
bloqueo D2 se ha relacionado tanto con su efecto antipsictico gicos, colinrgicos), sin que se pueda atribuir a alguno de ellos en
como con sus efectos secundarios extrapiramidales (por accin so- concreto su eficacia: clozapina, olanzapina, quetiapina.
bre la va nigroestriada) y endocrinos (hiperprolactinemia por su Otros combinan acciones antidopaminrgicas con acciones
accin sobre la va tuberoinfundibular); adems se relaciona con su antiserotoninrgicas: risperidona, sertindol, ziprasidona.
capacidad para inhibir el vmito y el hipo inducidos por quimiote- Unos pocos buscan el bloqueo dopaminrgico selectivo de las vas
rapia. tericamente implicadas en los sntomas de la enfermedad (vas

M exico A rgentina CTO Medicina C/ Nez de Balboa, 115 28006 MADRID (Espaa) Tfno.: (91) 782 43 32 / Fax: (91) 782 43 27
Pg. 12 PQ C hile U ruguay E-mail: secretaria@ctomedicina.com; iberocto@ctomedicina.com WEB: www.ctomedicina.com; www.iberocto.com
PSIQUIATRA
Preparacin Examen de Seleccin 05/06 1 Vuelta Seguimiento a distancia
mesolmbicas y vas mesocorticales), respetando la va nigroestria- del tronco o las extremidades produciendo una seria limitacin en
da, responsable de los efectos extrapiramidales: amisulpride. Sin el paciente; su respuesta al tratamiento es pobre, pudiendo con-
embargo estos frmacos no estn exentos de problemas. siderarse casi irreversibles.
Carecemos por el momento de presentaciones parenterales, por lo
que cuando el paciente rechaza el tratamiento hemos de recurrir a Cada efecto secundario tiene un tratamiento especfico:
antipsicticos clsicos (tanto en urgencias [haloperidol] como en Anticolinrgicos para las distonas agudas (por va parenteral) y el
tratamiento de mantenimiento [preparados depot]). parkinsonismo (por va oral): biperideno, trihexifenidilo.
Tienen un precio muy superior a los frmacos tradicionales, sin que se Betabloqueantes (propranolol) o benzodiacepinas (loracepam) para
haya podido demostrar en estudios de coste-efectividad que compen- la acatisia.
sen su precio con una reduccin de otros costes de la enfermedad. Depleccionantes dopaminrgicos (tetrabenacina) para las discine-
Algunos de sus efectos secundarios siguen produciendo problemas sias tardas graves.
de tolerancia: Amantadina para el parkinsonismo (no es habitual en Espaa).
- La risperidona produce una tasa de efectos extrapiramidales
importante cuando se usa en dosis altas; adems dispara los Un efecto secundario especialmente grave y con componente extra-
niveles de prolactina incluso en dosis bajas, con las consecuen- piramidal es el sndrome neurolptico maligno; en l se combinan:
cias endocrinas conocidas. Elementos extrapiramidales (rigidez extrema).
- La clozapina, la olanzapina y la quetiapina pueden producir un Hipertermia.
aumento del apetito y del peso muy llamativos, adems de efectos Alteraciones vegetativas (sudoracin, labilidad de la tensin arterial
sobre la tensin arterial y somnolencia; el aumento de peso se ha o la frecuencia cardaca).
relacionado con alteraciones metablicas (diabetes mellitus tipo II, Sntomas confusionales.
dislipemias) y aumento de la morbimortalidad cardiovascular.
Tiene una importante mortalidad cuando se deja sin tratamiento,
fundamentalmente por las repercusiones renales de la necrosis mus-
cular masiva (por la rigidez y la hipertermia), siendo bsico su diag-
nstico precoz (analticamente veremos leucocitosis, aumento de la
CPK y de otros enzimas musculares, hiperfosfatemia, hiperpotase-
mia) y su tratamiento inmediato; se suele indicar el ingreso del pa-
ciente en una UCI para un control mdico estricto, precisando de
una serie de medidas fsicas para frenar la hipertermia y de determi-
nados frmacos (dantroleno [relajante muscular], bromocriptina [ago-
nista dopaminrgico]) para revertir los sntomas principales.

Pregunta 54. Aparicin de los diferentes sntomas


extrapiramidales en funcin del tiempo.

Efectos extrapiramidales
Agudos Subagudos Tardos
Caractersticas Distona Parkinsonismo Acatisia Discinesia
Tras aos de
Inicio 1 semana En 3-4 meses
tratamiento
Temblor
Crisis Movimientos
Acinesia
Clnica oculgiras Inquietud faciobucolinguales
Rigidez
Tortcolis 15% grave
(parkinsonismo)
Mujeres, ancianos
Comentarios TEST

Deterioro previo
Jvenes del SNC
F. R iesgo Varones Sntomas
NL incisivos afectivos
Tto.
anticolinrgicos?
Pregunta 53. Evolucin de los antipsicticos. Clozapina o
Betablo-
Anticolinrgicos depleccionantes de
Tratamiento Anticolinrgicos queantes
Pregunta 54.-R: 1 Amantadina
BZD
dopamina
(tetrabenacina)
De todos los efectos secundarios que pueden presentar los antipsi-
cticos, los ms preguntados son los extrapiramidales; stos van cam-
biando en funcin del tiempo: Pregunta 55.-R: 2
En las primeras horas o das podemos ver distonas agudas; son El tratamiento con TEC fue inicialmente probado en pacientes con
contracciones sostenidas de grupos musculares que pueden resul- esquizofrenia, aunque pronto se comprob su extraordinaria eficacia
tar dolorosas o peligrosas (distonas larngeas); son especialmente en pacientes afectivos (depresin, mana). En la actualidad su uso en
frecuentes en varones, tanto ms cuanto ms jvenes. esquizofrenia es muy limitado:
Al cabo de varios das o semanas aparecen dos tipos de efectos Pacientes con formas catatnicas.
aparentemente opuestos: Pacientes con esquizofrenia y depresin grave asociada o ideas
- Una reduccin de los movimientos en la forma de sndrome graves de suicidio.
parkinsoniano. Pacientes con brotes psicticos resistentes al tratamiento en los que
- Un aumento de la movilidad en la forma de inquietud (objetiva se ha demostrado el cumplimiento del mismo.
y subjetiva) o acatisia.
En general suelen ser necesarias ms sesiones de tratamiento que
Cuando un paciente ha tomado medicacin antipsictica durante en la depresin y los resultados (salvo en las formas catatnicas) no
meses o aos puede presentar movimientos de tipo coreoatetsico son espectaculares.
(los ms frecuentes de la musculatura faciobucolingual) denomi- No hay que olvidar que la nica opcin de tratamiento que, ade-
nados discinesias tardas; en raras ocasiones afectan a msculos ms del TEC, ha demostrado cierta eficacia en los brotes resistentes es

M exico A rgentina CTO Medicina C/ Nez de Balboa, 115 28006 MADRID (Espaa) Tfno.: (91) 782 43 32 / Fax: (91) 782 43 27
C hile U ruguay E-mail: secretaria@ctomedicina.com; iberocto@ctomedicina.com WEB: www.ctomedicina.com; www.iberocto.com
PQ Pg. 13
PSIQUIATRA
Seguimiento a distancia Preparacin Examen de Seleccin 05/06 1 Vuelta
el uso de los antipsicticos atpicos modernos (no son eficaces el uso Pregunta 58.-R: 3
de dosis masivas de neurolpticos, la utilizacin de dosis de carga al Una de las complicaciones ms graves del alcoholismo es el sn-
principio del tratamiento o las asociaciones de neurolpticos). drome de abstinencia severo o delirium tremens. Asocia una impor-
tante mortalidad cuando se deja sin tratamiento, fundamentalmente
Pregunta 56.-R: 4 por las complicaciones hidroelectrolticas secundarias a los vmitos,
Cuando hablamos del tratamiento de la enfermedad no debe- la sudoracin profusa o la diarrea. Para que se produzca un sndrome
mos pensar que todo se reduce al uso de frmacos; en ese caso de abstinencia tiene que existir tolerancia, y sta se adquiere como
bastara con asegurarnos el cumplimiento (con formas intramuscu- consecuencia del consumo crnico de cantidades elevadas de alco-
lares depot) para controlar la enfermedad. Tan importante como el hol; parece que en su fisiopatologa intervienen mecanismos gabar-
tratamiento farmacolgico van a ser todas las medidas psicosociales gicos y glutamatrgicos, lo que quizs explique la elevada incidencia
destinadas a mejorar el cumplimiento (psicoeducacin en la enfer- de convulsiones generalizadas tnico-clnicas.
medad, terapias e intervenciones familiares) y a la rehabilitacin y
reinsercin social de los pacientes (tcnicas de modificacin de con-
ducta, tcnicas de afrontamiento del estrs). La disponibilidad y ges- Pregunta 58. Marcadores del consumo excesivo de alcohol.
tin de los llamados recursos intermedios (hospitales de da, centros
de da, centros de rehabilitacin psicosocial, centros de rehabilita- 123
cin laboral, recursos residenciales) condicionar el pronstico de
345678944
65 298

96 9  965 7967


5  979
8

5
la enfermedad. De aqu que en los ltimos aos se est hablando de 96 154
45 7  965 45
9 9 5
  795657
nuevos problemas en el manejo clnico de este tipo de pacientes,
97
5 45678944
65  49
 995
como el sndrome de la puerta giratoria (reingresos repetidos de los
mismos pacientes por la ausencia de recursos externos al hospital) o 3  965 7967


5 5!5 979
8

5
el problema de los pacientes jvenes crnicos, especialmente grave 55  5
  "5
9 9 5
 9455 # 795657
en las grandes ciudades. 345679
575  5! 49

Las terapias dinmicas (derivadas del psicoanlisis) carecen en la
actualidad de indicacin en este tipo de pacientes, puesto que preci- $1%  5!5 7967


5 9!4 979
8

5
san de una capacidad de introspeccin importante y se enfrentan a $96 147 54  % 96  96 5
54 '( &57
una enfermedad con condicionantes neurobiolgicos evidentes. %9
 9 7 95 &97  5! 49

1
96 9 )3*3  ,
49 545 8

54 65 95

7 5 +
5 -
Pregunta 57.-R: 5 "95
+6 96 49
El alcohol es un sedante del Sistema Nervioso Central y sus efectos 57 45675
65757  . 97 /
 7
6 5 95

7
dependen de la dosis ingerida, la capacidad de metabolizarlo (tole-
rancia farmacocintica) y la adaptacin cerebral al consumo crnico 34
0
14
7 2
 /4

   7967
97    979 &8
7
(tolerancia farmacodinmica). En general encontramos una buena
correlacin entre la alcoholemia y los efectos clnicos, de tal forma .
997 9 '  8+
  %54 5497
6979 &8
7 9 56 4

+6
6 9 5 
7
que si una persona tiene una alcoholemia elevada y no manifiesta
signos de intoxicacin podemos asegurar que tiene una dependencia
del alcohol. De forma inversa vemos pacientes que con alcoholemias
muy bajas sufren graves alteraciones conductuales (agresividad, agita-
cin); son las llamadas intoxicaciones idiosincrsicas o borracheras
patolgicas, y su origen parece ser la mayor sensibilidad de algunas
personas (ancianos, nios, pacientes con dao cerebral) a las sustan-
cias sedantes (alcohol, benzodiacepinas, barbitricos). Cuando se in-
giere una cantidad masiva de alcohol puede producirse la afectacin
del tronco del encfalo, apareciendo somnolencia y llegando en oca-
siones a la depresin cardiorrespiratoria. En general, el tratamiento de
las intoxicaciones alcohlicas es puramente sintomtico, evitando las
complicaciones comunes a cualquier coma (neumonas por aspira-
cin, sndromes compartimentales por decbito prolongado, hipo-

Comentarios TEST
termia por exposicin al aire libre); una caracterstica diferencial del
coma etlico es la aparicin de hipoglucemia como consecuencia de
la inhibicin de la gluconeognesis y la glucogenlisis; por eso es
frecuente que a estos pacientes se les pauten soluciones glucosadas,
con la precaucin de poner previamente tiamina para no precipitar
una encefalopata de Wernicke; en casos excepcionales se puede Pregunta 58. Fisiopatologa de la dependencia y abstinencia del alcohol.
recurrir a la dilisis para eliminar cantidades extremas de alcohol.
Pregunta 59.-R: 2
En la prevencin de recadas del alcoholismo destacamos dos fe-
nmenos psicolgicos que pueden ser parcialmente controlados con
medicacin:
Por un lado, los sntomas de abstinencia condicionados a estmu-
los ambientales: ACAMPROSATO.
Por otro, la prdida de control asociada al consumo de una pe-
quea cantidad del txico: NALTREXONA.

Totalmente distinto es el mecanismo de accin de los interdictores


(disulfiram, cianamida); stos bloquean el metabolismo del alcohol
en uno de sus pasos intermedios (aldehdo deshidrogenasa), provo-
cando la acumulacin de acetaldehdo, el cual desencadena una
reaccin histaminrgica. La base del tratamiento es conductual; se
trata de disminuir una conducta problemtica mediante el miedo a
Pregunta 57. Efectos fisiolgicos de las intoxicaciones y las abstinencias. un castigo potencial; por tanto, es necesario que el paciente est al
tanto de su tratamiento, siendo conveniente que firme un consenti-

M exico A rgentina CTO Medicina C/ Nez de Balboa, 115 28006 MADRID (Espaa) Tfno.: (91) 782 43 32 / Fax: (91) 782 43 27
Pg. 14 PQ C hile U ruguay E-mail: secretaria@ctomedicina.com; iberocto@ctomedicina.com WEB: www.ctomedicina.com; www.iberocto.com
PSIQUIATRA
Preparacin Examen de Seleccin 05/06 1 Vuelta Seguimiento a distancia
Comentarios TEST

Pregunta 64. Caractersticas clnicas del sndrome confusional agudo.

miento informado como precaucin legal, por si decide beber a pe- Pregunta 60.-R: 4
sar de tomarlo. En la dependencia de la herona, el tratamiento farmacolgico es
Otros frmacos como los ISRS o el tiapride parecen jugar un papel ms sencillo al existir frmacos especficos para el sistema opioide
ms limitado. endgeno, tanto agonistas como antagonistas.

M exico A rgentina CTO Medicina C/ Nez de Balboa, 115 28006 MADRID (Espaa) Tfno.: (91) 782 43 32 / Fax: (91) 782 43 27
C hile U ruguay E-mail: secretaria@ctomedicina.com; iberocto@ctomedicina.com WEB: www.ctomedicina.com; www.iberocto.com
PQ Pg. 15
PSIQUIATRA
Seguimiento a distancia Preparacin Examen de Seleccin 05/06 1 Vuelta

Pregunta 60. Fases del tratamiento y los frmacos implicados Pregunta 62.Fases de la dependencia de la cocana y su tratamiento.
en la dependencia de la herona.
12345678479
746445
123456784793
89876 6 84 7
738 7 4567847!"645#
1234564789
2647586 68 1234567847
7
"533657 456


87 12 58 8472 84 123456787692
7465733645 7847365645#
123456787692 12 
4433 2
 
 7
3 74$6834"57 84
128 787
1234564789
2 68  7 6 765645 7443375745744#

1234567847 %&378479


87 72 1
122 8 68 74"457'"3 7
84"4565
64
!68 72
88

124 6
362 2768 %&37847
73'33657 845"(

" 8


362 2768 12868 787 75&67(45 7"6"#
745467"67365(
 3
784793
89876 6 84#

87 72 58 847
128 8 7 4"437847
7)4356457

4 
256 5 8"4

1238 664   27692 7)*565(6 7 , .)
#
  788 7"4)+,7 364743-# 7
6557



87 72 58 847
12$847 242
7 25
  745467"67365(7 3# 7 6(63"#

 64

  27692
 788
12% 84
47264"6  68 72 2
2 
87 72 58 847
Pregunta 64.-R: 4
1256 4 72!8 72 7 8  7 4 

Describimos tres formas de delirium: agitados, estuporosos y mixtos.
En realidad en un mismo enfermo o en un mismo proceso patolgico
Pregunta 61.-R: 2 pueden verse todas las formas. Hay dos aspectos clnicamente impor-
La abstinencia de la herona es un cuadro clnico espectacular, tantes del delirium. En primer lugar, la AGITACIN de algunos delirium
pero sin complicaciones importantes; por tanto, y a diferencia de la puede suponer un riesgo para la salud del paciente y de las personas
abstinencia del alcohol, NO es una urgencia mdica; los pacientes no que le rodean; por esto llaman al psiquiatra, como experto en seda-
deben recibir tratamiento de forma puntual en un servicio de urgen- cin, para elegir el frmaco que controle el comportamiento agresivo,
cias, salvo que deban quedar ingresados por otro motivo; la actitud sin empeorar el estado fsico. Ver figura en la pgina siguiente.
ms correcta es derivarles a centros especializados para que inicien El haloperidol es el frmaco de eleccin para el control de cual-
un tratamiento reglado destinado no slo a la abstinencia sino a la quier cuadro de agitacin, pues carece de efectos cardiovasculares
prevencin de recadas. y respiratorios; su margen de seguridad es elevadsimo, y con esas
Para favorecer el mantenimiento de la abstinencia se han probado dosis curiosamente no se producen los temidos efectos extrapirami-
diferentes frmacos. La naltrexona (antagonista opioide) puede bloquear dales de estos frmacos. Los antipsicticos ms sedantes (clorproma-
los efectos de una dosis puntual de herona; sin embargo un bloqueo cina, levomepromacina, zuclopentixol) tienen el riesgo de producir
puede ser revertido con una dosis suficientemente alta de herona, por lo una hipotensin severa; otros antipsicticos sedantes tienen impor-
que el paciente debe comprometerse a no intentar vencerlo (de nuevo tantes efectos anticolinrgicos (tioridacina) que agravaran la confu-
aparece la necesidad del consentimiento informado), pues se provocara sin mental. Las benzodiacepinas a dosis elevadas pueden producir
una intoxicacin grave al estar desintoxicado (haber perdido la tolerancia); depresin respiratoria, sobre todo si el paciente est tomando otros
por este motivo solamente se propone para pacientes altamente motivados, frmacos con efectos depresores. Sin embargo son el tratamiento de
sin caractersticas impulsivas y sin problemas mdicos o psiquitricos aso- eleccin en los sndromes de abstinencia alcohlica grave (delirium
ciados. La metadona (agonista opioide) consigue disminuir de forma ms tremens), pues corrigen de forma especfica la fisiopatologa del sn-
eficaz las complicaciones legales y mdicas del consumo de herona, per- drome.
mitiendo la progresiva normalizacin del paciente en todas las esferas afec-
Pregunta 65.-R: 3
tadas por la drogodependencia; se plantea como un tratamiento a largo No hay que olvidar que todo delirium tiene una CAUSA que conviene
plazo, pudiendo necesitarse aos hasta que se pueda plantear su retirada; aclarar cuanto antes, pues implica una morbimortalidad notable; por eso

Comentarios TEST
es el tratamiento recomendado para los pacientes refractarios a otros trata- realizamos diferentes pruebas para diagnosticar el origen del mismo (anli-
mientos, los heroinmanos con problemas psiquitricos o mdicos graves, sis de sangre [hemograma, bioqumica completa] y de orina, ECG, pruebas
las embarazadas y los politoxicmanos. de imagen [Rx de trax, TAC], puncin lumbar). El test de Rorscharch (test
de las manchas de tinta) es una prueba proyectiva que explora los aspectos
Pregunta 62.-R: 3 inconscientes de la personalidad; es totalmente absurdo aplicarlo a estos
En el tratamiento de la dependencia de la cocana, las opciones de pacientes, con una alteracin de su nivel de conciencia.
apoyo farmacolgico son mucho ms limitadas; la accin fundamen-
tal de esta sustancia es catecolaminrgica (dopaminrgica sobre todo) Pregunta 66.-R: 4
y todos los frmacos utilizados actan en estos sistemas. (Ver tabla). Cuando la combinacin de fallos de memoria, desorientacin,
disnomia y dispraxias y la demostracin de alteraciones en la exploracin
Pregunta 63.-R: 4 neuropsicolgica elemental (MEC o Mini-mental) hace que sospechemos
Este paciente sufre un delirium o sndrome confusional agudo; esta una posible demencia debemos seguir un sistema de diagnstico diferen-
alteracin del estado mental aparece frecuentemente en enfermos cial para descartar posibles enfermedades que tienen un carcter reversi-
ingresados en hospitales generales, como consecuencia de los proce- ble o al menos algo ms benigno que las demencias degenerativas.
sos mdicos, las intervenciones quirrgicas o los tratamientos farma- El primer paso es descartar que no exista una causa externa que
colgicos; el sustrato fisiopatolgico es una encefalopata aguda cuyo est influyendo en el funcionamiento cerebral hasta el punto de si-
origen pueden ser problemas originados en el cerebro o fuera de l mular una demencia: intoxicaciones crnicas con frmacos sedan-
(procesos metablicos, txicos, frmacos, etc.). El sntoma inicial del tes, abuso de txicos, alteraciones metablicas (insuficiencia renal,
delirium es una alteracin de la atencin y la concentracin, que con hipotiroidismo, encefalopata heptica), anemias severas, lesiones
frecuencia conducen a la desorientacin (primero en el tiempo, ms ocupantes de espacio intracraneal (tumores, quistes, abscesos), enfer-
adelante en el espacio); es frecuente asistir a grandes fluctuaciones de medades infecciosas o inflamatorias con tratamiento eficaz (sfilis, es-
la clnica, en ocasiones relacionadas con factores externos (p.ej. em- clerosis mltiple), dficits vitamnicos, etc.
peoran al llegar la noche, pues la menor iluminacin hace que inter- Por tanto, al paciente con sospecha de demencia deben realizrse-
preten peor el entorno). le pruebas analticas (hemograma, bioqumica plasmtica completa,
serologas [sfilis, VIH], sistemtico de orina, estudio funcional tiroideo,

M exico A rgentina CTO Medicina C/ Nez de Balboa, 115 28006 MADRID (Espaa) Tfno.: (91) 782 43 32 / Fax: (91) 782 43 27
Pg. 16 PQ C hile U ruguay E-mail: secretaria@ctomedicina.com; iberocto@ctomedicina.com WEB: www.ctomedicina.com; www.iberocto.com
PSIQUIATRA
Preparacin Examen de Seleccin 05/06 1 Vuelta Seguimiento a distancia
niveles de B12 y flico), ECG y pruebas de imagen (Rx trax, TAC), Pregunta 68.-R: 2
para descartar las principales causas de demencia reversible. Son frecuentes las preguntas que combinan demencia con dife-
La polisomnografa slo puede ayudar a diferenciar la demencia de rentes sntomas, preguntndonos el diagnstico; repasemos los ms
la depresin, pues en la primera aumenta la latencia REM y en la segun- importantes:
da se acorta; sin embargo es una prueba compleja y con niveles de
sensibilidad y especificidad bajos, por lo que no se suele utilizar.
Pregunta 68. Demencias asociadas a sntomas especiales.
Pregunta 67.-R: 3
111213456789
6 1614818798

7 9684
8
11121 9 5
6 9
817
87
8
7 56
8
1112187



111217 17
86 5617 76

76564528
11121
94 8

111217 16 984 8

11121!6695 1"87968 " !66 9


818 9487

11121#
 1 67 4$
9 1 9846
12324567
11121%7696 56 1988

11121349
89
6 1
846 !66 9
819

1112187

 967 161'6()

11121& 58 19 
846

111218
16167 84
8

11121 958 1 6 


9 5
98
!66 9
8 17 5 2
111213858*18 4
8*1

676 9
8
56 7846 1+
9,
111213456789
6 16148184
6 589
$

1112167 6678 9
8

Pregunta 69.-R: 3

Pregunta 69. Diferencias entre demencia y depresin.

12324567 892
1 12324567
128 296 7
5
6 123454636 78956

Pregunta 67. Esquema de diagnstico de la demencia.


8
2 45

1
8  

Una vez que hemos descartado las demencias reversibles, buscare-  6    342
mos otras demencias cuya causa puede al menos ser controlada,
impidiendo que avance el deterioro, sin que se pueda recuperar lo    24    463

perdido (demencias vasculares, demencias postraumticas, demen- 5 


cia pugilstica, demencias txicas): demencias irreversibles, pero con 7!!"!
Comentarios TEST

tratamiento capaz de detener la progresin. "" ! 2#


Lo que resta son las demencias degenerativas irreversibles o progre-
sivas; algunas de stas son secundarias a enfermedades para las cuales 5!
no tenemos tratamiento en la actualidad (demencia asociada a la "!$"!  6

enfermedad de Creutzfeldt-Jakob, demencia asociada a la infeccin %!  6 9


VIH, demencia asociada a la dilisis crnica), pero la mayora son
demencias degenerativas primarias, es decir, debidas a enfermedades 2!! %!  6

neurodegenerativas cuyo origen puede ser gentico-hereditario


(Huntington) o desconocido (Alzheimer).    346
 4!4254  2

Separamos las demencias degenerativas primarias en dos grupos, 6!& 75 9 56 "#545  6$

en funcin del patrn de sntomas: 2#


Cuando predomina el enlentecimiento tanto motor como ps-
quico, asociado a movimientos anormales y alteraciones emo- 7!$! 7  5 3 %623  5 3

cionales hablamos de un patrn SUBCORTICAL; siguen este pa- !!  76&4 5 3 '4&'
trn las enfermedades degenerativas extrapiramidales (Parkinson,
Huntington). 1&!"  4 2  ' 6 !(!
Cuando predominan los fallos de memoria, la alteracin del len- )&   3 ( 9 2 3

guaje, los problemas en el reconocimiento y las dificultades en la 2*!!  6


realizacin de tareas cotidianas hablamos de un patrn CORTI-
CAL; en stas diferenciamos un patrn FRONTO-TEMPORAL, cuan- !!! %6289 2 ) 4 

do la enfermedad debuta con una transformacin de la personali- !+ 3- 


dad (que refleja el inicio en las regiones anteriores, como en la ", * 6 4 2 6

enfermedad de Pick), y un patrn TEMPORO-PARIETAL, cuando !..! %629342 %6824462 3 5  34 3


dominan las alteraciones afsicas, aprxicas y las agnosias (como
en la enfermedad de Alzheimer).

M exico A rgentina CTO Medicina C/ Nez de Balboa, 115 28006 MADRID (Espaa) Tfno.: (91) 782 43 32 / Fax: (91) 782 43 27
C hile U ruguay E-mail: secretaria@ctomedicina.com; iberocto@ctomedicina.com WEB: www.ctomedicina.com; www.iberocto.com
PQ Pg. 17
PSIQUIATRA
Seguimiento a distancia Preparacin Examen de Seleccin 05/06 1 Vuelta
Pregunta 70.-R: 4 2. Como consecuencia de esa dieta se produce:
El trastorno lmite de la personalidad es probablemente el que ms - Una prdida de peso (se considera importante una prdida del
complicaciones psiquitricas produce; en l se mezclan sntomas de 15% del peso oficialmente aceptado como ideal o un IMC infe-
todas las lneas de la psicopatologa (ansiosos, emocionales, psicti- rior a 17,5).
cos, alimentacin, impulsividad, abuso de sustancias, etc.), en general - Aparece amenorrea.
de una intensidad importante pero de una duracin breve, y, casi 3. Esto es la anorexia nerviosa restrictiva.
siempre, en respuesta a alguna circunstancia vivida como frustrante.
Probablemente por esto se trate del trastorno de la personalidad que
ms puede responder al tratamiento farmacolgico (antidepresivos,
antipsicticos, estabilizadores), teniendo el cuidado de elegir frma-
cos con un elevado nivel de seguridad en el caso de intoxicacin (no
es raro que estos pacientes protagonicen autolesiones de carcter
impulsivo). Los autores norteamericanos han relacionado este trastor-
no con el antecedente de abusos sexuales en la infancia, si bien esta
relacin es muy cuestionada fuera de EE.UU.

Pregunta 71.-R: 1
Los trastornos de personalidad aumentan el riesgo de presentar tras-
tornos mentales, existiendo algunas relaciones especialmente claras:
Los esquizotpicos sufren con frecuencia episodios psicticos bre-
ves; en ellos encontramos una elevada frecuencia de antecedentes
familiares de esquizofrenia; en algunas clasificaciones se conside-
ran formas latentes de esquizofrenia (pues sus sntomas recuerdan
en ocasiones a los de esta enfermedad).
Los esquizoides tambin se relacionan con las enfermedades psi-
cticas, siendo la personalidad anormal que con ms frecuencia
se relaciona con la aparicin de esquizofrenia; de hecho sus rasgos
se solapan con los sntomas negativos de la esquizofrenia en sus
formas simple o residual (pero a diferencia de esos pacientes, los
esquizoides siempre han sido as).
Los paranoides se relacionan muy fuertemente con los trastornos
delirantes, ya sean agudos (psicosis reactivas breves) o crnicos
(paranoia). Pregunta 72. Esquema de los trastornos de la conducta alimentaria.
Pregunta 72.-R: 3
Cuando el trastorno afecta a chicas con un menor autocontrol
En los trastornos alimentarios fundamentales (anorexia y bulimia
nerviosa) el sntoma central es el mismo: una preocupacin PATOL- (con rasgos ms impulsivos) la dieta va a verse interrumpida por:
GICA por la posibilidad de engordar. Este miedo condiciona su rela- 1. Los atracones bulmicos en los que:
cin con la comida, a la que ven ms como una enemiga que como - Se ingiere una enorme cantidad de alimento.
un medio necesario para conseguir una actividad normal. A qu se - En un tiempo llamativamente corto.
debe ese miedo? Muchas veces no llegamos a saberlo, pero subyacen - Con una sensacin de prdida de control.
una serie de pensamientos distorsionados y sobrevalorados (pero NO 2. Para compensar los atracones aparecen una serie de conductas
delirantes) acerca de las consecuencias del aumento de peso en sus destinadas a favorecer la prdida de peso, que van desde las con-
vidas. No es raro encontrar adems distorsiones en la percepcin de ductas purgantes (vmitos autoprovocados, abuso de laxantes, uso
su esquema corporal, pero algunas pacientes, reconociendo que es- de diurticos) a otras menos agresivas (alternancia de ayuno y atra-
tn delgadas, no soportan la idea de recuperar peso. cones, ejercicio compulsivo).
Cuando este trastorno aparece en una chica con una buena capa- 3. Esto es la bulimia nerviosa.
cidad de autocontrol (generalmente con rasgos obsesivos de persona-

Comentarios TEST
lidad) veremos cmo va a ser capaz de: Sin embargo, vemos tambin algunas formas mixtas; lo ms frecuente
1. Seguir una dieta restrictiva, aguantando su deseo de comer (la pa- es que con la cronificacin de una anorexia nerviosa las pacientes
labra anorexia sugiere la falta de apetito, lo cual no es cierto hasta comiencen a presentar conductas purgantes SIN tener atracones, para no
que no se alcanza la emaciacin); absorber lo poco que coman (anorexia restrictiva-purgativa). Adems,

Pregunta 70. Caractersticas bsicas de los trastornos de la personalidad.

1234567 1 
859
2 2345167897923451
1 1 428  1578 42257878
1 8 2 5
1234546789
12759
586
8
12 8487
1268 784 12
87
 12 8787 12458 8 4 12

698
6

12 8789
128 58687 1234 87!4"5
872#$%
12345678965
7 125678965
7 12345678965
7
12&42 7848497 12&42 7848497 12&42 7848497
 1234 994
9459 26
4
  12994
9459 12994
9459
12
4' 497 2
78764
6
( 12
4' 497 2
78764
6
( 12
4' 497 2
78764
6
(
22)57 *286
+5
87 22286
4
229786497 2752
28
97
 12,6
54889492454267 2 88-587 12.54628 88949 123
5 7648949
2/6
584 /
1272
2798)8462762
28
7 123
9
62
48845
2#72
'7545%2762
28
7 120

625768)845

M exico A rgentina CTO Medicina C/ Nez de Balboa, 115 28006 MADRID (Espaa) Tfno.: (91) 782 43 32 / Fax: (91) 782 43 27
Pg. 18 PQ C hile U ruguay E-mail: secretaria@ctomedicina.com; iberocto@ctomedicina.com WEB: www.ctomedicina.com; www.iberocto.com
PSIQUIATRA
Preparacin Examen de Seleccin 05/06 1 Vuelta Seguimiento a distancia
con el paso de los aos, las pacientes con anorexia llegan a perder el
control sobre su alimentacin y comienzan a presentar atracones (lo que En su tratamiento se combinan una serie de medidas higinicas
algunos llaman bulimarexia). Ms raro es que una chica con bulimia que buscan respetar los perodos naturales de sueo e interrumpir la
llegue a enganchar un perodo suficientemente largo de autocontrol como actividad con siestas programadas para disminuir la frecuencia de
para que pierda peso y presente amenorrea prolongada. ataques de sueo con diversos medicamentos destinados a aliviar la
somnolencia (estimulantes anfetamnicos, modafinil) o los fenmenos
Pregunta 73.-R: 4 REM (antidepresivos, modafinil).
Los atracones bulmicos guardan una estrecha relacin con la capaci-
dad de autocontrol y los rasgos impulsivos de personalidad; por eso no es Pregunta 76.-R: 4
raro encontrar en estas pacientes una mayor frecuencia de abuso de El uso de psicofrmacos en los trastornos mentales de la infancia y
drogas, autolesiones, sntomas depresivos, promiscuidad, etc. Suelen ser la adolescencia est muy poco estudiado (por motivos ticos suelen
ms frecuentes por la noche (cuando es menos probable que sean des- hacerse pocos ensayos clnicos) y es muy controvertido (por las hip-
cubiertas) y tras un perodo de ayuno. Los alimentos deben ser fciles de tesis dominantes que atribuyen la mayora de los problemas mentales
tragar y no es raro que se seleccionen sobre todo dulces (la urgencia del de este grupo de edad a conflictos psicolgicos). Sin embargo, en
atracn no hace posible la seleccin de alimentos hipocalricos). algunas entidades est claramente establecida la utilidad de la medi-
De todas las conductas alimentarias anormales, slo los atracones cacin, sin que por ello se descarten los abordajes psicolgicos o
son susceptibles de cierto tratamiento farmacolgico; la medicacin sociales:
de eleccin son los ISRS, utilizndose dosis mucho ms altas que las Trastorno por dficit de atencin con hiperactividad:
habituales para una depresin. - De primera eleccin: estimulantes anfetamnicos (metilfenida-
En general el tratamiento de estas pacientes es multidisciplinar, to, dextroanfetamina, pemolina).
implicndose especialistas en endocrinologa y nutricin (el bajo peso - De segunda eleccin: antidepresivos tricclicos.
es la principal causa de ingreso), en ginecologa (para prevenir las Trastorno de La Tourette:
consecuencias metablicas de la amenorrea), en modificacin de - De primera eleccin: antipsicticos tpicos incisivos (pimocide,
conducta (para ayudar en el autocontrol de los atracones y los vmi- haloperidol).
tos) y en psicoterapia (para explorar los motivos que subyacen a esa - De segunda eleccin: clonidina.
preocupacin por la alimentacin). Enuresis nocturna:
- De primera eleccin: tcnicas de modificacin de conducta
Pregunta 74.-R: 4 (sistemas de alarma).
Ver tabla. - De segunda eleccin: desmopresina nasal, imipramina oral.
Mutismo selectivo:
Pregunta 75.-R: 4 - De primera eleccin: psicoterapias.
La narcolepsia (sndrome de Gelineau) es una enfermedad poco - tiles: ISRS (como en las fobias sociales).
frecuente en la cual se combinan:
Ataques incoercibles del sueo (casi siempre sueo REM). Pregunta 77.-R: 4
Con la aparicin de caractersticas del sueo REM en la vigilia: El trastorno por dficit de atencin con hiperactividad es uno de
- Catapleja (prdida brusca del tono muscular). los diagnsticos ms frecuentes en Psiquiatra Infantil. Estos nios no
- Parlisis del sueo (falta de recuperacin del tono muscular tras consiguen mantener la atencin un tiempo prolongado, siendo fcil-
el despertar). mente distrables por cualquier estmulo y mostrndose sumamente
- Alucinaciones hipnaggicas e hipnopmpicas (aparicin o per- inquietos. Suele debutar en la primera infancia, pero se hace ms
sistencia de las imgenes de los sueos en la transicin entre evidente al iniciar la escolarizacin. De sus tres esferas sintomticas
vigilia y sueo). (dficit de atencin, hiperactividad, impulsividad), las dos primeras

Pregunta 74. Fases del sueo y fenmenos fisiolgicos y patolgicos asociados.



Comentarios TEST

12324256578215 45 572


9

9

6669 
6 27
25
 6
5 28
 75
 9 

Fase 1 (5%) Fase 2 (45%) Fase 3 (12%) Fase 4 (13%)  27


25

Alfa Theta (4-8 Hz) Sueo "delta" (<4Hz) Beta (>12Hz)
Beta (>12Hz)
3 frontal
(8-12Hz)
Complejos K
ondas "en dientes de
occipital <50% >50% sierra"
husos del sueo

3 Activo Descenso Disminuido Atona

Movimientos Movimientos lentos rotatorios



3 rpidos "en balancn"
Ausentes Rpidos, conjugados

 !"#"$" Superficial Media Sueo "profundo" Media


Inestabilidad,
%7&685&6%'()* Descenso, estabilidad
arritmias, apneas
(+ ,$-#.! PRL (+)
/0!$, GH (+), TSH (-), ACTH (-)
%(!.0(!) "Sueos",
1)#-) erecciones peneanas

82 Homeotermia Poiquilotermia

$$)0!#$) Bruxismo Sonambulismo, terror nocturno Pesadilla

M exico A rgentina CTO Medicina C/ Nez de Balboa, 115 28006 MADRID (Espaa) Tfno.: (91) 782 43 32 / Fax: (91) 782 43 27
C hile U ruguay E-mail: secretaria@ctomedicina.com; iberocto@ctomedicina.com WEB: www.ctomedicina.com; www.iberocto.com
PQ Pg. 19
PSIQUIATRA
Seguimiento a distancia Preparacin Examen de Seleccin 05/06 1 Vuelta
suelen mejorar rpidamente con el tratamiento farmacolgico; los
estimulantes anfetamnicos mejoran la capacidad atencional y dismi-
nuyen la distraibilidad, permitiendo que el nio aproveche las horas
escolares y reduzca los comportamientos problemticos. Cuando no
se diagnostica a tiempo o predomina la impulsividad es frecuente que
se vayan sumando conductas antisociales (robos, mentiras, peleas),
encontrndose en los estudios evolutivos una clara tendencia al man-
tenimiento de esos comportamientos y al diagnstico de trastorno
antisocial de la personalidad. Tampoco es raro que se produzca el
abuso de sustancias (a veces como auto-tratamiento) o que persistan
sntomas en la edad adulta.
Conviene tener en cuenta este diagnstico cuando nos enfrenta-
mos a un nio con problemas de comportamiento en el medio esco-
lar, si bien es necesario que encontremos sntomas de las dos esferas
principales (dficit de atencin, hiperactividad), pues existen otras
muchas causas de mal comportamiento en un nio (inicio de un
trastorno antisocial, depresin con predominio de la irritabilidad, pro-
blemas de adaptacin en pacientes con discapacidad intelectual).

Comentarios TEST

M exico A rgentina CTO Medicina C/ Nez de Balboa, 115 28006 MADRID (Espaa) Tfno.: (91) 782 43 32 / Fax: (91) 782 43 27
Pg. 20 PQ C hile U ruguay E-mail: secretaria@ctomedicina.com; iberocto@ctomedicina.com WEB: www.ctomedicina.com; www.iberocto.com
REUMATOLOGA
Preparacin Examen de Seleccin 05/06 1 Vuelta Seguimiento a distancia

1. Una mujer de 44 aos desarrolla a lo largo de dos meses 2) Afectacin de vnulas.


inflamacin articular en ambas rodillas, tobillos, carpos, meta- 3) Respuesta al tratamiento con ciclofosfamida.
carpofalngicas e interfalngicas proximales. Se procede al 4) Afectacin pulmonar.
estudio del lquido sinovial con artrocentesis de una rodilla con 5) Glomerulonefritis.
el siguiente resultado: lquido algo turbio con viscosidad baja,
45.000 clulas/ mm3 (70% neutrfilos), glucosa de 64 mg/dl 7. Mujer de 34 aos que consulta por fiebre y dolor torcico. En
(glucemia en sangre de 112 mg/dl) y protenas de 48 g/l. Con los la radiologa de trax se observan infiltrados pulmonares
datos aportados, el diagnstico ms probable es: bilaterales y a nivel cutneo prpura palpable. En la analtica
destaca la presencia de 1.200 eosinfilos/mm3. Cul es el
1) Artritis psorisica. diagnstico ms probable?:
2) Artritis reactiva.
3) Artritis por cristales de cido rico. 1) Panarteritis nodosa.
4) Lupus eritematoso sistmico. 2) Granulomatosis de Wegener.
5) Artritis reumatoide. 3) Granulomatosis alrgica (enfermedad de Churg-Strauss).
4) Sndrome de poliangetis de superposicin.
2. Una biopsia sinovial informada por el patlogo como sinovitis 5) Vasculitis por hipersensibilidad.
crnica inespecfica es compatible con:
8. Mujer de 59 aos con fiebre, artralgias y deformidad nasal con
1) Artritis reumatoide. obstruccin de vas areas superiores y secrecin hemorrgica.
2) Tuberculosis. Despus de 6 meses de iniciado el proceso se percibe hundimien-
3) Sarcoidosis. to de la pirmide nasal, dificultad para tragar, dolor farngeo y
4) Sinovitis por cuerpo extrao. ronquera. En la exploracin de la faringe aparece enrojecida con
5) Sarcoma sinovial. encas tumefactas. En las fosas nasales se aprecia un tejido
blanquecino que las obstruye parcialmente. Analtica: Hb 12,4
3. En una mujer de 36 aos, con artritis intermitente de articula- g/dl, leucocitos 10.100/mm3 (frmula normal), Cr 2 mg/dl; factor
ciones de manos, rodillas y tobillos se detecta en el laboratorio reumatoide positivo; crioglobulinas negativas. Orina 50-100
la presencia de anticuerpos antinucleares a ttulo 1/640, anti- hemates/campo con cilindros granulosos e hialinos y proteinu-
ADNss, anti-ADNds, y antihistona positivos, el diagnstico ms ria de 850 mg/l. NO espera encontrar en esta enfermedad:
probable es:
1) Ndulos pulmonares.
1) Enfermedad mixta del tejido conectivo. 2) Granulomas larngeos.
2) Sndrome de Sjgren. 3) Destruccin sea orbitaria.
3) Lupus inducido por frmacos. 4) Glomerulonefritis.
4) Sndrome antisintetasa. 5) Proptosis.
5) Lupus eritematoso sistmico.
9. El diagnstico diferencial de la granulomatosis de Wegener NO
4. Una mujer de 34 aos es evaluada en el servicio de ginecologa incluye:
por haber presentado tres abortos en el primer trimestre de
embarazo. Como antecedente present hace 4 aos una trom- 1) Granulomatosis linfomatoide.
boflebitis popltea izquierda. Ante la sospecha de sndrome 2) Sndrome de Goodpasture.
antifosfolpido NO es de utilidad para orientar este diagnstico 3) Granulomatosis alrgica (sndrome de Churg-Strauss).
la determinacin de: 4) Enfermedad de Buerger.
5) Neumona por Legionella.
1) Factor reumatoide.
2) Anticuerpos anticardiolipina IgG. 10. Mujer de 22 aos que ingresa por ictus cerebral. En la explora-
Preguntas TEST

3) Anticuerpos anticardiolipina IgM. cin no se detecta pulso arterial en ambas extremidades supe-
4) VDRL. riores y se ausculta un soplo de insuficiencia artica. Analtica:
5) Test de tromboplastina parcial activado. discreta anemia con elevacin de VSG. Indique el diagnstico
ms probable:
5. Respecto a la enfermedad renal de la panarteritis nodosa
clsica, seale la respuesta FALSA: 1) Panarteritis nodosa.
2) Arteritis de Takayasu.
1) Se produce en la mayora de los pacientes. 3) Lupus eritematoso con anticoagulante lpico.
2) La forma ms habitual de presentacin es la glomerulonefritis. 4) Sndrome antifosfolpido catastrfico
3) Suele producir deterioro de la funcin renal. 5) Vasculitis del sistema nervioso central.
4) En la arteriografa se aprecian microaneurismas.
5) La hipertensin arterial es una manifestacin habitual. 11. En relacin con la arteritis de clulas gigantes, NO es cierto que:

6. Seale qu manifestacin es comn a la panarteritis nodosa 1) La biopsia normal de la arteria temporal descarta el diagns-
clsica y a la panarteritis microscpica: tico.
2) Se acompaa de elevacin marcada de la VSG.
1) Presencia de aneurismas. 3) Afecta a pacientes de edad avanzada.

M exico A rgentina CTO Medicina C/ Nez de Balboa, 115 28006 MADRID (Espaa) Tfno.: (91) 782 43 32 / Fax: (91) 782 43 27
C hile U ruguay E-mail: secretaria@ctomedicina.com; iberocto@ctomedicina.com WEB: www.ctomedicina.com; www.iberocto.com
RM Pg. 1
REUMATOLOGA
Seguimiento a distancia Preparacin Examen de Seleccin 05/06 1 Vuelta
4) A menudo hay sntomas de polimialgia reumtica. 1) Ciclosporina 0,5 mg/kg/semanal.
5) Responde espectacularmente al tratamiento con corticoi- 2) Metotrexate 1 mg/semanal.
des. 3) Prednisona 1 mg/kg/da y ciclofosfamida 2 mg/kg/da.
4) Colchicina 1 mg/da.
12. Seale cul de las siguientes caractersticas de la enfermedad de 5) Prednisona 0,5 mg/kg/da.
SchnleinHenoch NO es comn a las vasculitis predominan-
temente cutneas (de hipersensibilidad): 17. Una mujer de 46 aos desarrolla de forma progresiva en los
ltimos meses parestesias en pie izquierdo, mano derecha y
1) Afectacin de capilares y vnulas. posteriormente izquierda, junto con fiebre, prdida cuantifi-
2) Fenmeno de leucocitoclasia. cada de 10 kilos de peso, dolor abdominal difuso que predo-
3) Prpura palpable como manifestacin ms frecuente. mina despus de comer con rectorragia ocasional y artromi-
4) Buen pronstico. algias. En la exploracin destaca la presencia de hipoestesia
5) Afectacin visceral. del dorso del pie izquierdo, tres primeros dedos de la mano
derecha e imposibilidad para la dorsiflexin de pie izquierdo
13. Respecto a las vasculitis predominantemente cutneas, indique con signos de irritacin peritoneal en la palpacin abdominal.
la respuesta correcta: Hemoglobina 9,6 gr/dl, 13.230 leucocitos/mm3 (frmula nor-
mal) y creatinina de 3,4 mg/dl. Radiografa de trax normal.
1) Se relaciona con infecciones por virus hepatotropos. Los hallazgos referidos son compatibles con el diagnstico de:
2) No se observa relacin entre estas vasculitis y los linfomas.
3) La panarteritis microscpica se incluye en este grupo. 1) Enfermedad de Goodpasture.
4) La prpura palpable desaparece con la vitropresin. 2) Sndrome de Schnlein-Henoch.
5) A excepcin del sndrome de Sjgren, el resto de conectivo- 3) Panarteritis nodosa clsica.
patas se relacionan con estas vasculitis. 4) Enfermedad de Wegener.
5) Sndrome poliangetico de superposicin.
14. Mujer de 43 aos que presenta desde hace tres meses cefalea
pulstil parietotemporal derecha acompaada de nuseas. De 18. En una de las siguientes enfermedades la hiperuricemia NO se
forma brusca presenta un episodio de prdida de fuerza en debe al aumento del catabolismo de las purinas:
hemicuerpo izquierdo que se recupera en 10 minutos y unas
horas despus, prdida de memoria y dificultad para la expre- 1) Policitemia vera.
sin verbal. En el TC no se encuentran alteraciones relevantes 2) Anemia hemoltica.
y en la RM se observan cambios de seal periventriculares 3) Psoriasis extensa.
extensos y mltiples lesiones perifricas corticales y en sustan- 4) Leucemia linftica crnica.
cia blanca. Qu prueba elegira para confirmar el diagnstico?: 5) Enfermedad de Lesch-Nyhan.

1) c-ANCA. 19. Un varn de 76 aos acude al servicio de urgencias por


2) Arteriografa mesentrica. presentar dolor en flanco derecho de instauracin brusca
3) TC toracoabdominal. asociado a nuseas y vmitos. Refiere episodios similares que
4) Biopsia de arteria temporal. han sido etiquetados de clicos nefrticos as como episodios
5) Biopsia cerebral. de podagra de repeticin. En el estudio analtico presenta una
uricemia de 9,3 mg/dl y una uricosuria en 24 horas de 356 mg.
15. Un varn de 23 aos viene presentando lceras orales doloro- Una vez resuelto el episodio agudo, qu actitud teraputica
sas de forma recurrente en los ltimos siete meses. En las ltimas adoptara sobre la hiperuricemia?:
semanas adems presenta lceras escrotales, lesiones de erite-
ma nodoso y dolor, tumefaccin e impotencia funcional en 1) No precisa tratamiento, salvo beber abundante lquido.
rodillas y tobillos. Presenta as mismo elevacin de la VSG. El 2) Alopurinol y colchicina.

Preguntas TEST
factor reumatoide y los ANA son negativos. No presenta snto- 3) Benzobromarona.
mas oculares y la exploracin oftalmolgica es normal. Para el 4) Colchicina aislada.
tratamiento de este paciente, en este momento se podran 5) Sulfinpirazona.
incluir todas, EXCEPTO una de las siguientes medidas:
20. Un paciente de 42 aos, diagnosticado de enfermedad por
1) Colchicina. depsito de cristales de pirofosfato clcico, requiere la realiza-
2) AINEs. cin de las siguientes determinaciones bioqumicas, EXCEPTO:
3) Ciclosporina.
4) Corticoides en dosis bajas. 1) PTH.
5) Corticoides tpicos orales. 2) Magnesio.
3) Ferritina.
16. Varn de 42 aos que hace 7 meses comenz con fatiga, 4) Fosfatasa alcalina.
artralgias y congestin nasal, seguido de un episodio de hemop- 5) ACTH.
tisis. La analtica mostr elevacin de la VSG, leucocitosis,
anemia, alteraciones en el sedimento urinario as como infiltra- 21. La aparicin de artrosis en una de las siguientes articulaciones
dos pulmonares. El ttulo de c-ANCA fue de 1:128. Qu debe hacerle sospechar la presencia de una artropata crnica
tratamiento recomendara en este paciente?: por microcristales:

M exico A rgentina CTO Medicina C/ Nez de Balboa, 115 28006 MADRID (Espaa) Tfno.: (91) 782 43 32 / Fax: (91) 782 43 27
Pg. 2 RM C hile U ruguay E-mail: secretaria@ctomedicina.com; iberocto@ctomedicina.com WEB: www.ctomedicina.com; www.iberocto.com
REUMATOLOGA
Preparacin Examen de Seleccin 05/06 1 Vuelta Seguimiento a distancia

1) Rodilla. 27. Como criterio diagnstico de lupus eritematoso sistmico se


2) Cadera. incluye:
3) Codo.
4) Interfalngica proximal. 1) Anticuerpos anti-ADNss.
5) Interfalngica distal. 2) Anticuerpos anti-RNP.
3) Psicosis lpica.
22. Una paciente de 81 aos presenta un episodio de monoartritis 4) Anemia microangioptica.
aguda en tobillo desde hace 24 h. En los ltimos aos ha 5) Leucopenia farmacolgica.
presentado episodios similares en carpo y rodilla. La radiologa
muestra imgenes compatibles con condrocalcinosis tanto en 28. Cul de los siguientes factores de predisposicin gentica es
la rodilla como en el tobillo y el estudio del lquido sinovial FALSO en el lupus eritematoso sistmico?:
cristales de forma rectangular con estudio microbiolgico
negativo. En el tratamiento de esta paciente estaran indicadas 1) Aumento de concordancia de la enfermedad en gemelos
todas estas medidas, SALVO: monocigticos.
2) Predisposicin familiar a padecer LES.
1) AINEs. 3) Asociacin entre ttulos elevados de anti-ADN IgG y nefritis
2) Reposo de la articulacin. lpica con DR2 y DR3.
3) Drenaje de la articulacin y administracin de corticoides 4) Asociacin de ac anti-Ro y lupus eritematoso cutneo subagu-
intraarticulares. do y DR4.
4) Corticoides sistmicos. 5) Asociacin del anticoagulante lpico con DR4 y DR7.
5) Colchicina.
29. La presencia de anticoagulante lpico significa un mayor riesgo
23. El depsito de hidroxiapatita NO se relaciona con una de las de presentar las siguientes manifestaciones, SALVO:
siguientes manifestaciones clnicas:
1) Pericarditis.
1) Hombro de Milwaukee. 2) Trombocitopenia.
2) Periartritis escapulohumeral. 3) Trombosis venosas.
3) Artrosis. 4) Falsa positividad de la serologa de les.
4) Monoartritis aguda. 5) Livedo reticularis.
5) Poliartritis simtrica.
30. Uno de los hallazgos histolgicos en los pacientes con afecta-
24. El depsito de cristales de oxalato clcico se asocia habitual- cin renal en el lupus eritematoso sistmico es reversible:
mente a:
1) Necrosis glomerular.
1) Oxalosis primaria. 2) Esclerosis glomerular.
2) Insuficiencia renal terminal. 3) Fibrosis intersticial.
3) Insuficiencia cardaca. 4) Atrofia tubular.
4) Hemocromatosis. 5) Semilunas fibrosas.
5) Hiperparatiroidismo.
31. En relacin al LES y el embarazo, seale la respuesta FALSA:
25. Mujer de 76 aos, sin antecedentes reumatolgicos, presenta
3 das despus de una apendicectoma inflamacin articular 1) Los antipaldicos y los inmunosupresores estn contraindi-
en rodilla derecha y fiebre de 38,5 C. El lquido sinovial cados.
muestra 75.000 clulas/mm3, siendo el 98% PMN. El Gram y 2) Los corticoides de eleccin son los de larga vida media.
cultivo son negativos. En el microscopio de luz polarizada se 3) El mtodo anticonceptivo de eleccin son las medidas fsicas.
Preguntas TEST

observan algunos cristales de forma rectangular. Su diagns- 4) La amenorrea es frecuente.


tico es: 5) Son frecuentes los brotes de la enfermedad en el puerperio.

1) Artritis sptica. 32. En relacin a las alteraciones hematolgicas del lupus eritema-
2) Artritis tuberculosa reagudizada. toso sistmico, seale la respuesta correcta:
3) Gota.
4) Pseudogota. 1) La hemlisis es la causa ms frecuente de anemia.
5) Artritis reactiva. 2) La esplenectoma est contraindicada.
3) La trombopenia no suele ser grave.
26. Entre las siguientes manifestaciones clnicas es ms habitual en 4) La leucopenia obliga a administrar cobertura antibitica
el lupus eritematoso sistmico: profilctica.
5) La mayora de los pacientes con test de Coombs directo
1) Artritis. positivo presentan hemlisis.
2) Anemia hemoltica.
3) Sndrome cognitivo leve. 33. Dentro de las manifestaciones neurolgicas en el lupus eritema-
4) Pericarditis. toso NO espera encontrar:
5) Glomerulonefritis proliferativa focal.
1) Anticuerpos antineuronales.

M exico A rgentina CTO Medicina C/ Nez de Balboa, 115 28006 MADRID (Espaa) Tfno.: (91) 782 43 32 / Fax: (91) 782 43 27
C hile U ruguay E-mail: secretaria@ctomedicina.com; iberocto@ctomedicina.com WEB: www.ctomedicina.com; www.iberocto.com
RM Pg. 3
REUMATOLOGA
Seguimiento a distancia Preparacin Examen de Seleccin 05/06 1 Vuelta
2) Anticuerpos antimielina. 3) Suspender los corticoides tpicos.
3) Polineuropata. 4) Carbamacepina.
4) Mielitis transversa. 5) Antidepresivos.
5) Afectacin ms habitual dentro de los pares craneales del
facial. 39. En una paciente de 30 aos con lupus eritematoso sistmico que
presenta trombopenia autoinmune severa con clnica hemo-
34. El anticuerpo ms habitual en los pacientes con lupus eritema- rrgica asociada, NO es de utilidad una de las siguientes
toso es: medidas:

1) Antifosfolpido. 1) Esteroides.
2) Anti-RNP. 2) Gammaglobulina.
3) Antihistona. 3) Ciclofosfamida.
4) Anti-ADN. 4) Penicilamina.
5) Antinucleares. 5) Esplenectoma.

35. En un paciente diagnosticado de lupus inducido por frmacos 40. NO es uno de los criterios diagnsticos de la artritis reumatoide:
espera encontrar:
1) lceras orales.
1) Inicio de los sntomas de forma inmediata al iniciar el trata- 2) Ndulos subcutneos.
miento. 3) Factor reumatoide positivo.
2) Serositis. 4) Osteoporosis yuxtaarticular.
3) Sndrome orgnico cerebral. 5) Artritis simtrica.
4) Anticuerpos anti-Sm.
5) Glomerulonefritis proliferativa focal. 41. Con respecto al factor reumatoide, indique la respuesta INCO-
RRECTA:
36. Una paciente de 23 aos, diagnosticada de LES y actualmente sin
tratamiento, presenta en las ltimas semanas astenia, fiebre 1) Aparece de forma constante en pacientes con ndulos
vespertina, artritis migratoria y dolor de caractersticas pleurti- reumatoides o vasculitis.
cas en la inspiracin profunda. El tratamiento recomendado es: 2) Su frecuencia disminuye con la edad en la poblacin gene-
ral.
1) Corticoides en dosis altas (1 mg/kg/24 h). 3) Se trata de ac contra la fraccin Fc de IgG.
2) Azatioprina. 4) Las pruebas habituales suelen detectar factor reumatoide IgM.
3) Azatioprina y corticoides. 5) Aparece ocasionalmente tras una transfusin o vacunacin.
4) AINEs.
5) Bolos mensuales de ciclofosfamida. 42. Indique cul de los siguientes factores NO indica mal prons-
tico en los pacientes con artritis reumatoide:
37. Mujer de raza negra de 37 aos, con historia de tres meses de
rash persistente que afecta a la cara, parte anterior del trax, 1) Afectacin de las manos.
palmas y plantas de pies. A la exploracin se observan mltiples 2) Ttulos elevados de factor reumatoide.
lesiones hipopigmentadas y atrficas. La biopsia de la piel del 3) Ndulos subcutneos.
trax indica un infiltrado denso en la dermis profunda compa- 4) Elevacin de la VSG.
tible con lupus discoide. Qu tratamiento recomendara?: 5) Sndrome de Felty.

1) AINEs. 43. Una mujer de 62 aos, con artritis reumatoide de larga evolu-
2) Bolos de esteroides i.v. cin, presenta desde hace 72 horas dolor e inflamacin en el

Preguntas TEST
3) Hidroxicloroquina oral. codo derecho con febrcula. La artrocentesis muestra un lqui-
3
4) Ciclofosfamida. do de aspecto purulento con ms de 100.000 leucocitos/mm
5) Sales de oro. y marcado descenso de la glucemia. El germen responsable de
esta situacin con ms frecuencia es:
38. Mujer de 22 aos, diagnosticada de LES con antecedente de rash
facial, poliartritis, pericarditis, y anticuerpos antinucleares y anti- 1) Staphylococcus epidermidis.
ADN. Refiere deterioro de las funciones superiores con labilidad 2) Streptococcus viridans.
emocional, falta de concentracin y de memoria en su trabajo. 3) Staphylococcus aureus.
Tambin refiere cefalea recurrente refractaria al uso de analg- 4) Gonococo.
sicos. Recibe tratamiento con AINEs y corticoides tpicos cut- 5) Mycobacterium tuberculosis.
neos. Analtica: ANA y anti-ADN, anti-Ro y niveles bajos de
complemento. En la RM muestra atrofia ligera, as como reas de 44. La subluxacin atloaxoidea antero-posterior se define por:
captacin de alta intensidad, ms evidentes en regin parieto-
temporal. Recomendara para su tratamiento: 1) Presencia de erosiones en la apfisis odontoides.
2) Aumento de distancia entre la apfisis odontoides y el cuerpo
1) Corticoides orales. del atlas.
2) Suspender los AINEs. 3) Impresin basilar.

M exico A rgentina CTO Medicina C/ Nez de Balboa, 115 28006 MADRID (Espaa) Tfno.: (91) 782 43 32 / Fax: (91) 782 43 27
Pg. 4 RM C hile U ruguay E-mail: secretaria@ctomedicina.com; iberocto@ctomedicina.com WEB: www.ctomedicina.com; www.iberocto.com
REUMATOLOGA
Preparacin Examen de Seleccin 05/06 1 Vuelta Seguimiento a distancia

4) Erosiones en odontoides y en articulaciones interapofisarias junto con rigidez matutina en ambas manos de varias horas de
con movilidad patolgica. evolucin. En las ltimas semanas ha desarrollado lceras dolo-
5) Desplazamiento lateral de la apfisis odontoides respecto al rosas en piernas, as como polineuropata de predominio sensi-
atlas. tivo. Sobre esta situacin, seale la respuesta correcta:

45. La deformidad en "cuello de cisne" que aparece en la artritis 1) El diagnstico de artritis reumatoide se debe descartar por los
reumatoide se caracteriza por: nuevos sntomas aparecidos.
2) El CH50 en suero estar elevado.
1) Hiperextensin de la interfalngica proximal con flexin de 3) El paciente cumple criterios para el diagnstico de sndrome
la interfalngica distal. de Felty.
2) Flexin de la interfalngica proximal con extensin de la 4) La biopsia cutnea mostrar probablemente datos de vascu-
interfalngica distal. litis necrotizante.
3) Desviacin cubital de la mueca. 5) Debemos sospechar el desarrollo de un linfoma.
4) Flexin de las interfalngicas distales.
5) Flexin de interfalngicas proximal y distal. 51. Paciente de 36 aos, diagnosticada hace 6 meses de artritis
reumatoide. Presenta factor reumatoide positivo, anticuerpos
46. Una paciente de 54 aos, diagnosticada de artritis reumatoide antinucleares negativos y HLA DR3. Tras seguir tratamiento con
seronegativa de larga evolucin, ha recibido a lo largo de su sales de oro parenterales desarrolla proteinuria de rango
enfermedad mltiples tratamientos de fondo. Actualmente nefrtico. Tras suspender las sales de oro, NO se debera usar
sigue tratamiento con AINEs y corticoides en dosis bajas. En los como tratamiento en esta paciente:
ltimos 6 meses ha desarrollado proteinuria de intensidad
creciente, sin hematuria, hasta alcanzar el rango nefrtico. En 1) Hidroxicloroquina.
las ltimas determinaciones analticas ha comenzado adems 2) Sulfasalacina.
a presentar deterioro de la funcin renal. La afectacin renal 3) D-penicilamina.
ms probable en esta paciente ser: 4) Metotrexate.
5) Corticoides.
1) Glomerulonefritis membranosa tarda por sales de oro.
2) Nefritis intersticial por AINEs. 52. En qu grupo de pacientes con artritis crnica juvenil aparece
3) Trombosis de la vena renal. con mayor frecuencia la uvetis crnica?:
4) Amiloidosis secundaria.
5) Vasculitis reumatoide. 1) Sistmico.
2) Poliarticular seropositivo.
47. La afectacin ocular ms frecuente en la artritis reumatoide es: 3) Oligoarticular de inicio tardo.
4) Poliarticular seronegativo.
1) Escleritis. 5) Oligoarticular de inicio precoz.
2) Escleromalacia perforante.
3) Queratoconjuntivitis seca. 53. En una nia de 4 aos con inflamacin de la rodilla izquierda
4) Uvetis anterior aguda. y tobillo derecho de 6 meses de evolucin en cuyo lquido
5) Uvetis posterior. sinovial se hallan 34.450 clulas/mm3 con predominio de poli-
morfonucleares, sin microorganismos en el cultivo, que no ha
48. El derrame pleural que se produce en la artritis reumatoide se presentado fiebre ni lesiones cutneas, espera encontrar en el
caracteriza por: estudio analtico:

1) Recuento celular bajo. 1) AAN.


2) Caractersticas de trasudado. 2) Anticardiolipina.
Preguntas TEST

3) Adenosindeaminasa baja. 3) VSG normal.


4) Aparecer siempre acompaado de fibrosis pulmonar inters- 4) DR4.
ticial. 5) B27.
5) Concentracin baja de glucosa.
54. En la artritis crnica juvenil, el tratamiento con esteroides est
49. En la artritis reumatoide, la fibrosis pulmonar intersticial en sus especialmente indicado en:
fases iniciales NO se caracteriza por:
1) Presentacin sistmica.
1) Descenso de la PO2 con el ejercicio. 2) Oligoarticular precoz.
2) Descenso del ndice VEF1/CVF. 3) Oligoarticular tarda.
3) Discreto aumento de la trama reticular en la radiologa de trax. 4) Poliarticular seropositiva.
4) Descenso de la capacidad de difusin pulmonar. 5) Poliarticular seronegativa.
5) Ausencia de cianosis en reposo.
55. Indique la caracterstica que NO suele aparecer en las espon-
50. Un varn de 37 aos presenta desde hace 6 meses inflamacin diloartropatas inflamatorias seronegativas:
articular que incluye articulaciones interfalngicas proximales,
carpos, rodillas y tobillos que cede parcialmente al tomar AINEs 1) Afectacin poliarticular simtrica.

M exico A rgentina CTO Medicina C/ Nez de Balboa, 115 28006 MADRID (Espaa) Tfno.: (91) 782 43 32 / Fax: (91) 782 43 27
C hile U ruguay E-mail: secretaria@ctomedicina.com; iberocto@ctomedicina.com WEB: www.ctomedicina.com; www.iberocto.com
RM Pg. 5
REUMATOLOGA
Seguimiento a distancia Preparacin Examen de Seleccin 05/06 1 Vuelta
2) Curso evolutivo crnico. 2) Pseudomonas.
3) Entesitis. 3) Yersinia.
4) Ausencia de ndulos subcutneos. 4) Shigella.
5) Uvetis anterior aguda. 5) Salmonella.

56. Una mujer de 30 aos, sana hasta hace 6 meses y sin anteceden- 61. Indique la manifestacin cutnea que NO es tpica en pacientes
tes epidemiolgicos relevantes, presenta en este ltimo periodo con artritis reactiva:
dolor e inflamacin en la rodilla derecha y articulacin coxo-
femoral izquierda, as como dolor y rigidez de predominio 1) lceras orales.
nocturno en la zona lumbar. En la radiografa de articulaciones 2) Queratodermia blenorrgica.
perifricas se observa disminucin del espacio articular, que es 3) Balanitis circinada.
ms marcada en la articulacin coxofemoral, mientras que en 4) Oniclisis de las uas.
la radiografa de pelvis se observan erosiones en ambas articu- 5) Pioderma gangrenoso.
laciones sacroilacas con esclerosis marginal del hueso ilaco. En
la exploracin fsica presenta dolor y limitacin a la movilidad 62. Para establecer el diagnstico diferencial entre la artritis reac-
en las articulaciones perifricas afectadas as como maniobras tiva y la artritis gonoccica, es correcto que:
sacroilacas de apertura positivas. No se encuentran lesiones
cutneas ni ungueales y el ojo derecho aparece enrojecido sin 1) En las dos enfermedades hay tenosinovitis.
disminucin de la agudeza visual. Analtica: anemia normocti- 2) El dolor lumbosacro es caracterstico de la artritis reactiva
ca y normocrmica con VSG elevada y factor reumatoide pero no de la gonoccica.
negativo. Con estos datos, el diagnstico ms probable es: 3) El hallazgo de cultivo positivo en crvix de gonococo excluye
el diagnstico de artritis reactiva.
1) Artritis reumatoide. 4) En ambas enfermedades puede haber cultivo positivo del
2) Artritis reactiva. lquido sinovial.
3) Artritis psorisica. 5) En ambos casos la base del tratamiento son los AINEs.
4) Artritis brucelar.
5) Espondilitis anquilosante. 63. Mujer de 54 aos con psoriasis de varios aos de evolucin
tratada tpicamente, presenta desde hace 7 meses inflamacin
57. Seale cul de las siguientes NO es una manifestacin extraar- marcada de las articulaciones interfalngicas proximales y
ticular de la espondilitis anquilosante: distales, carpos, rodilla y cadera izquierda. En la radiografa de
manos se observa destruccin articular marcada de varias
1) Nefropata IgA. articulaciones con tendencia a la luxacin. En la exploracin
2) Fibrosis apical pulmonar bilateral. cutnea se observan placas psorisicas extensas que cubren
3) Uvetis anterior. gran parte del abdomen y de extremidades inferiores. En la
4) Meningoencefalitis. analtica destaca elevacin marcada de VSG y PCR con FR a
5) Insuficiencia artica. ttulos bajos. En este momento, el tratamiento ms adecuado es:

58. En el tratamiento de la espondilitis anquilosante, NO es de 1) Metotrexate.


utilidad: 2) Cloroquina.
3) Sales de oro.
1) AINEs. 4) Penicilamina.
2) Sulfasalazina. 5) Sulfasalazina.
3) Corticoides intralesionales.
4) Rehabilitacin. 64. Respecto a la afectacin articular de la enfermedad inflamatoria
5) Sales de oro. intestinal, seale la respuesta FALSA:

Preguntas TEST
59. Varn de 30 aos que consulta por artritis de rodilla derecha 1) La artritis perifrica es una afectacin ms frecuente que la
de inicio brusco y talalgia bilateral. Las caractersticas del espondilitis.
lquido sinovial son: 40.000 leucocitos/mm3, protenas eleva- 2) El 70% de los pacientes con espondilitis son HLA B27
das y glucosa normal, sin microorganismos en el Gram. Qu positivos.
diagnstico le parece ms probable?: 3) La artritis perifrica puede ser oligoarticular.
4) La artritis suele ser clnicamente de comienzo agudo y no
1) Lupus eritematoso sistmico. destructiva.
2) Artritis reactiva. 5) La mitad de los pacientes con artritis perifrica, sin espondi-
3) Artritis reumatoide. litis, son HLA B27 positivos.
4) Artritis gonoccica.
5) Artritis infecciosa. 65. Un varn de 82 aos consulta por presentar dolor de caracte-
rsticas mecnicas en la cadera derecha. En el estudio radiolgico
60. Uno de los siguientes grmenes NO est implicado en la aparicin se observa un patrn mixto (esclertico y ltico) en pelvis y hueso
de artritis reactiva de origen gastrointestinal. Selelo: femoral; en la analtica presenta una marcada elevacin de la
hidroxiprolinuria y de la fosfatasa alcalina. Como complicacin
1) Campylobacter. de esta enfermedad NO espera encontrar:

M exico A rgentina CTO Medicina C/ Nez de Balboa, 115 28006 MADRID (Espaa) Tfno.: (91) 782 43 32 / Fax: (91) 782 43 27
Pg. 6 RM C hile U ruguay E-mail: secretaria@ctomedicina.com; iberocto@ctomedicina.com WEB: www.ctomedicina.com; www.iberocto.com
REUMATOLOGA
Preparacin Examen de Seleccin 05/06 1 Vuelta Seguimiento a distancia

1) Hipercalciuria. 2) La concentracin circulante media de 1,25-(OH)2-D3 est


2) Artropata coxofemoral. elevada.
3) Fractura patolgica. 3) Se asocia en la clnica con la aparicin de fracturas del cuello
4) Insuficiencia cardaca. femoral.
5) Hipovitaminosis D. 4) Los niveles de PTH tienden a estar elevados.
5) Se detecta por densitometra una disminucin de hueso
66. NO se ha relacionado con la patogenia de la enfermedad de cortical.
Paget:
72. La osteognesis imperfecta tipo I NO incluye:
1) Virus del sarampin.
2) Virus respiratorio sincitial. 1) Sordera.
3) Virus del moquillo canino. 2) Ectopia del cristalino.
4) Agregacin familiar. 3) Esclerticas azules.
5) HLA DR4. 4) Aumento de incidencia de fracturas.
5) Herencia autosmica dominante.
67. La degeneracin sarcomatosa es la complicacin ms grave en
la enfermedad de Paget. Indique la respuesta FALSA respecto a 73. Entre los tratamientos en la osteoporosis postmenopusica NO
la misma: incluimos:

1) Las localizaciones ms habituales son el fmur, hmero, 1) Risedronato.


crneo y pelvis. 2) Alendronato.
2) El pronstico es mejor que en los osteosarcomas primarios. 3) Estrgenos.
3) Son causa habitual de sarcoma seo en los ancianos. 4) Raloxifeno.
4) Los signos sugestivos de su aparicin son la fractura patolgica 5) Mitramicina.
y la tumefaccin de partes blandas.
5) Puede existir un aumento de fosfatasa alcalina sobre la basal. 74. En una anciana ingresada en un hospital de crnicos con inade-
cuada alimentacin y exposicin solar se sospecha que haya
68. Cul NO se considera una indicacin para el tratamiento de la desarrollado una osteomalacia ante la persistencia de dolores
enfermedad de Paget?: seos generalizados y debilidad muscular. En el estudio analtico
NO espera encontrar para confirmar el diagnstico:
1) Dolor persistente en el hueso pagtico.
2) Compresin nerviosa. 1) Hiperfosforemia.
3) Niveles de fosfatasa alcalina superiores al 10% de la norma- 2) Hiperparatiroidismo secundario.
lidad. 3) Hipocalcemia.
4) Hipercalcemia. 4) Aumento de la depuracin renal de fosfato.
5) Preparacin para ciruga ortopdica. 5) Descenso de la 25(OH)D3.

69. La definicin actual de osteoporosis postmenopusica es: 75. En la osteomalacia secundaria a insuficiencia renal crnica, a
diferencia de la causada por dficit nutricional, NO encontra-
1) Reduccin de la masa sea suficiente como para producir remos:
una fractura.
2) Osteopenia radiolgica con o sin fractura asociada. 1) Fracturas de Looser.
3) Descenso de masa sea igual o superior a 2,5 DE respecto a 2) Descenso de 25-(OH)-D3.
la media de las mujeres de la edad de la paciente. 3) Descenso de 1,25-(OH)2-D3.
4) Descenso de masa sea igual o superior a 2,5 DE respecto a 4) Hiperparatiroidismo secundario.
Preguntas TEST

la media de las mujeres jvenes. 5) Normocalcemia o hipocalcemia.


5) Fractura vertebral o de cuello de fmur en mujer postmeno-
pusica independientemente de la densidad sea. 76. Los anticuerpos antitopoisomerasa se asocian en la esclerosis
sistmica a:
70. Una de las siguientes enfermedades NO se asocia a la aparicin
de osteoporosis: 1) Fenmeno de Raynaud intenso.
2) Esclerodactilia.
1) Artrosis. 3) Fibrosis pulmonar.
2) Hipogonadismo. 4) Hipertensin pulmonar.
3) Osteognesis imperfecta. 5) Cirrosis biliar primaria.
4) Hiperparatiroidismo.
5) Hipertiroidismo. 77. Existe relacin entre ciertos factores ambientales y la esclerosis
sistmica desde el punto de vista patognico. Cul de los
71. Respecto a la osteoporosis tipo II o del anciano, seale la siguientes NO se ha relacionado?:
respuesta INCORRECTA:
1) Polvo de slice.
1) Afecta a varones y mujeres con una edad por encima de los 2) Silicona (implantes quirrgicos).
75 aos.

M exico A rgentina CTO Medicina C/ Nez de Balboa, 115 28006 MADRID (Espaa) Tfno.: (91) 782 43 32 / Fax: (91) 782 43 27
C hile U ruguay E-mail: secretaria@ctomedicina.com; iberocto@ctomedicina.com WEB: www.ctomedicina.com; www.iberocto.com
RM Pg. 7
REUMATOLOGA
Seguimiento a distancia Preparacin Examen de Seleccin 05/06 1 Vuelta
3) Cloruro de vinilo. 1) Haemophilus influenzae, doxiciclina.
4) Tricloroetileno. 2) Staphylococcus aureus, cloxacilina.
5) Asbesto. 3) Neisseria gonorrhoeae, ceftriaxona.
4) Pseudomonas aeruginosa, ceftacidima.
78. El estudio bsico de un paciente con fenmeno de Raynaud 5) Streptococcus pneumoniae, ceftriaxona.
incluye las siguientes pruebas complementarias MENOS una,
que es FALSA. Sealar esta: 84. Un paciente de 22 aos presenta de forma aguda una inflama-
cin poliarticular simtrica con preferencia en pequeas arti-
1) Crioglobulinemia. culaciones, as como intensa astenia. Los sntomas desaparecen
2) Anticuerpos anti-SCL 70 y anticentrmero. a los 5 das, apareciendo ictericia conjuntival. La causa ms
3) Capilaroscopia. probable de este cuadro es:
4) Radiografa de trax.
5) Anticuerpos antihistonas. 1) Infeccin por el virus de Epstein-Barr.
2) Artritis reumatoide de inicio.
79. Mujer de 34 aos, diagnosticada hace unos 10 aos de sndro- 3) Poliartritis inespecfica e ictericia por AINEs.
me de CREST. En los ltimos meses presenta disnea progresiva. 4) Rubola.
En la radiografa de trax se observa aumento del cono de la 5) Hepatitis B.
arteria pulmonar y oligohemia perifrica; el ECG muestra
signos de crecimiento de cavidades derechas. Cul le parece 85. Las manifestaciones asociadas a la infeccin por el virus de la
el diagnstico ms probable?: inmunodeficiencia humana NO incluyen:

1) Fibrosis pulmonar. 1) Artralgias.


2) Broncoaspiracin por reflujo gstrico. 2) Artritis reumatoide.
3) Hipertensin pulmonar. 3) Espondiloartropata indiferenciada.
4) Carcinoma de clulas alveolares. 4) Sndrome de Reiter.
5) Tromboembolismo pulmonar. 5) Artritis psorisica.

80. Uno de los siguientes tipos de anemia NO aparece en relacin 86. En el curso clnico de la enfermedad de Lyme NO es caracters-
con la esclerosis sistmica progresiva: tico:

1) Anemia megaloblstica. 1) Respuesta inicial al tratamiento con tetraciclinas.


2) Anemia hemoltica microangioptica. 2) Artritis perifrica de predominio en pequeas articulaciones.
3) Anemia de trastornos crnicos. 3) Neuritis craneal.
4) Anemia refractaria con exceso de blastos. 4) Bloqueo auriculoventricular.
5) Anemia ferropnica. 5) Meningitis linfocitaria.

81. Mujer de 37 aos que consulta por poliartritis de pequeas 87. La prueba ms especfica para el diagnstico de la xerostoma
articulaciones, tumefaccin de manos, palidez ocasional de los en el sndrome de Sjgren es:
pulpejos de los dedos, mialgias y disfagia. En la analtica tiene
elevacin de CK y aldolasa. Cul es el diagnstico ms probable?: 1) Biopsia de glndula salival menor.
2) Sialometra.
1) Enfermedad mixta del tejido conectivo. 3) Gammagrafa.
2) Artritis reumatoide. 4) Sialografa.
3) Esclerosis sistmica. 5) Ecografa de partida.
4) Sndrome mialgia-eosinofilia.

Preguntas TEST
5) Lupus eritematoso sistmico. 88. Uno de los siguientes hallazgos NO es caracterstico del sndro-
me seco que aparece en la infeccin por el virus VIH:
82. Elija la respuesta FALSA en las artritis spticas:
1) Infiltrado linfocitario en glndulas salivares por CD8.
1) Las infecciones por BGN tienen un curso menos agudo que 2) Asociacin con DR5.
las infecciones por CGP. 3) Presencia de ac anti-Ro.
2) La infeccin sobre prtesis articular suele diagnosticarse en 4) Test de Schirmer positivo.
la primera semana despus de la ciruga. 5) Ulceraciones puntiformes en crnea.
3) La infeccin por Pseudomonas afecta con frecuencia a la
articulacin esternoclavicular en los pacientes ADVP. 89. Una mujer de 65 aos presenta desde hace dos meses intensa
4) Los sntomas inflamatorios pueden enmascararse en los debilidad muscular que actualmente le impide levantarse de la
pacientes tratados con corticoides. cama, con dolor muscular de predominio proximal. Ante esta
5) El Haemophilus influenzae produce infecciones frecuentes en sintomatologa, que no ha mejorado con el tratamiento de
los primeros aos de vida. AINEs, se decide su derivacin al servicio de urgencias, donde
se objetiva la prdida de fuerza bilateral y simtrica de predo-
83. Seale la relacin teraputica INADECUADA para el tratamien- minio proximal en extremidades superiores e inferiores, as
to de una artritis sptica: como elevacin de CK muy significativa. En el dorso de las

M exico A rgentina CTO Medicina C/ Nez de Balboa, 115 28006 MADRID (Espaa) Tfno.: (91) 782 43 32 / Fax: (91) 782 43 27
Pg. 8 RM C hile U ruguay E-mail: secretaria@ctomedicina.com; iberocto@ctomedicina.com WEB: www.ctomedicina.com; www.iberocto.com
REUMATOLOGA
Preparacin Examen de Seleccin 05/06 1 Vuelta Seguimiento a distancia

manos presenta lesiones cutneas sobre las articulaciones. 2) Sarcoidosis.


Como estudios complementarios a realizar en esta paciente, 3) Endocarditis bacteriana.
tendr MENOS utilidad: 4) Hemocromatosis.
5) Enfermedad inflamatoria intestinal.
1) Radiografa de trax.
2) TC craneal. 95. Una mujer de 55 aos presenta desde hace aos dolor intermi-
3) Hemograma con frotis. tente en la rodilla derecha que se acenta con la bipedestacin.
4) Ecografa abdominal. El estudio analtico es anodino sin elevacin de VSG. La paciente
5) Mamografa. consulta porque en las ltimas semanas el dolor se ha incremen-
tado notando cierta inflamacin en la articulacin. En la explo-
90. La enfermedad pulmonar intersticial en enfermos con poli- racin fsica la rodilla est aumentada de tamao con rebote
miositis se asocia a la presencia de: positivo de la patela compatible con la presencia de derrame
articular. La radiologa demuestra una disminucin del espacio
1) Anticuerpos anti-ARNt sintetasa. articular con esclerosis marginal. Se realiza una artrocentesis,
2) Presencia de cuerpos de inclusin en la biopsia. obteniendo 35 ml de lquido sinovial de aspecto claro, con 630
3) Anticuerpos anti-Mi2. clulas/mm3 con predominio de mononucleares y sin microcris-
4) Anticuerpos anti-SRP. tales. Seale el tratamiento que NO utilizara:
5) Anticuerpos anti-ADN.
1) Reposo relativo de la articulacin.
91. Seale la respuesta correcta en la polimiositis por cuerpos de 2) Corticoides sistmicos.
inclusin: 3) AINEs sistmicos.
4) cido hialurnico intraarticular.
1) Afecta a personas ancianas, con predominio de varones. 5) Condroitn sulfato oral.
2) El inicio de los sntomas es extremadamente rpido.
3) Se asocia habitualmente a cardiopata. 96. NO es un hallazgo radiolgico propio de la artrosis:
4) Presencia casi constante de autoanticuerpos.
5) La CK siempre est elevada. 1) Esclerosis subcondral.
2) Osteofitos.
92. Un joven de 14 aos, emigrante de origen turco, ha presentado 3) Disminucin del espacio articular.
en los ltimos 15 meses episodios recurrentes de fiebre, dolor 4) Osteoporosis yuxtaarticular.
abdominal, artralgias y dificultad respiratoria. Segn refiere su 5) Geodas.
familia, un hermano mayor presentaba episodios similares
antes de fallecer por insuficiencia renal terminal. Recomenda- 97. La localizacin actualmente ms frecuente de artropata neu-
ra para su tratamiento: roptica es:

1) AINEs. 1) Articulacin glenohumeral.


2) Ciclosporina. 2) Cadera.
3) Colchicina. 3) Rodilla.
4) Corticoides orales. 4) Sacroilacas.
5) Metronidazol. 5) Tarso y metatarsianas.

93. Seale la respuesta FALSA en la amiloidosis: 98. En la policondritis recidivante, aparece en casi todos los enfer-
mos:
1) La disfuncin endocrina es poco frecuente.
2) Puede producir afectacin neurovegetativa con hipotensin 1) Artritis.
Preguntas TEST

postural. 2) Condritis auricular.


3) La artropata amiloide se ve con frecuencia en el mieloma 3) Glomerulonefritis.
mltiple. 4) Conjuntivitis.
4) La amiloidosis heredofamiliar no afecta al sistema nervioso. 5) Insuficiencia artica.
5) La afectacin cardaca cursa con insuficiencia cardaca
congestiva y cardiomegalia. 99. Mujer de 47 aos que refiere en los ltimos meses sensacin de
dolor muscular generalizado que afecta a las extremidades
94. Un varn de 64 aos consulta en el servicio de urgencias por superiores e inferiores y a la musculatura de la espalda. La
inflamacin articular en rodillas y tobillos de carcter simtrico palpacin de la columna vertebral no resulta especialmente
y sensacin de quemazn en la superficie externa de las espinas dolorosa en ninguna localizacin, pero presenta dolor selecti-
tibiales. Desde hace meses refiere deformidad de los dedos de vo a nivel de la insercin de la musculatura suboccipital,
ambas manos en palillo de tambor con engrosamiento y reblan- segmento central del trapecio, junto a ambos epicndilos, cara
decimiento de las bases ungueales. En la radiografa articular no interna de ambas rodillas y en la musculatura adyacente a
se observan hallazgos llamativos y a nivel de la tibia calcificacin ambos trocnteres femorales. No refiere cuadro constitucional
peristica. Dentro de las patologas que hay que descartar en salvo astenia intensa. Seale entre las siguientes caractersticas
este paciente NO se incluye: la que NO espera encontrar en esta paciente:

1) Tumor broncognico. 1) Alteracin del sueo.

M exico A rgentina CTO Medicina C/ Nez de Balboa, 115 28006 MADRID (Espaa) Tfno.: (91) 782 43 32 / Fax: (91) 782 43 27
C hile U ruguay E-mail: secretaria@ctomedicina.com; iberocto@ctomedicina.com WEB: www.ctomedicina.com; www.iberocto.com
RM Pg. 9
REUMATOLOGA
Seguimiento a distancia Preparacin Examen de Seleccin 05/06 1 Vuelta
2) Colon irritable.
3) CPK elevada.
4) Aldolasa normal.
5) VSG normal.

100. Varn de 74 aos que desde hace dos meses presenta gran
impotencia funcional para elevar los brazos, manteniendo
movilidad pasiva normal. En la anamnesis no refiere alteracio-
nes visuales, cefalea ni claudicacin mandibular. La VSG es de
115 mm/1 hora con enzimas musculares normales. La explo-
racin fsica muestra un pulso temporal bilateral normal, sin
dolor a la palpacin ni engrosamiento arterial. En el tratamiento
de este paciente, debemos elegir:

1) Prednisona 1 mg/kg/24 h.
2) Prednisona 10-15 mg/24 h.
3) Paracetamol.
4) Prednisona y azatioprina.
5) No precisa tratamiento, si no existe afectacin histolgica de
la arteria temporal.

Preguntas TEST

M exico A rgentina CTO Medicina C/ Nez de Balboa, 115 28006 MADRID (Espaa) Tfno.: (91) 782 43 32 / Fax: (91) 782 43 27
Pg. 10 RM C hile U ruguay E-mail: secretaria@ctomedicina.com; iberocto@ctomedicina.com WEB: www.ctomedicina.com; www.iberocto.com
REUMATOLOGA
Preparacin Examen de Seleccin 05/06 1 Vuelta Seguimiento a distancia
Preguntas TEST

M exico A rgentina CTO Medicina C/ Nez de Balboa, 115 28006 MADRID (Espaa) Tfno.: (91) 782 43 32 / Fax: (91) 782 43 27
C hile U ruguay E-mail: secretaria@ctomedicina.com; iberocto@ctomedicina.com WEB: www.ctomedicina.com; www.iberocto.com
RM Pg. 11
REUMATOLOGA
Preparacin Examen de Seleccin 05/06 1 Vuelta Seguimiento a distancia
Pregunta 1.- R: 5 to ms elevado sea el denominador, ya que la cifra expresada
Nos encontramos ante una paciente que presenta por un lado un indica la ltima dilucin a la que resultan positivos).
patrn inflamatorio poliarticular (afectacin de ms de 4 articulacio- En el sndrome de Sjgren, los anticuerpos caractersticos son los
nes) y con un estudio de lquido sinovial que muestra datos de lquido anti-Ro (o SS-A) y los anti-La (o SS-B).
inflamatorio: el nmero de clulas se encuentra entre 3.000 y 50.000 En el lupus inducido por frmacos aparecen anticuerpos antinu-
por mm3, la glucosa se encuentra discretamente descendida respecto cleares y anti-histona, pero no aparecen los anticuerpos especfi-
a la encontrada en plasma, y existe un aumento de protenas respecto cos del lupus eritematoso sistmico, como son los anti-DNAds ni
a la normalidad (recuerda que en el lquido sinovial las protenas son los anti-Sm. Pueden aparecer sin embargo anti-DNAss.
parte de las encontradas en el plasma, por lo que se sitan en unos El sndrome antisintetasa es una patologa recientemente descrita
2 gramos/dl, o lo que es lo mismo, 20 gramos/litro). Sin embargo, la que identifica a los pacientes con polimiositis, anticuerpos
presencia de lquido inflamatorio es comn a cualquiera de las res- antisintetasa (anti-Jo1) y enfermedad pulmonar intersticial.
puestas ofrecidas, por lo que la clave para resolver la pregunta se La respuesta correcta es, por lo tanto, el lupus eritematoso sistmi-
encuentra en el patrn articular: co. Debemos recordar que en esta patologa dentro de sus criterios
La artritis psorisica presenta con ms frecuencia un patrn oligo- diagnsticos se incluye la presencia de dos caractersticas que tiene
articular (2-4 articulaciones) asimtrico con frecuente afectacin esta paciente: la presencia de anticuerpos antinucleares y los anti-
de interfalngicas distales. DNAds. Ten en cuenta que en el LES tambin puede haber, como
La artritis reactiva es igualmente otra espondiloartropata con afec- en el lupus inducido por frmacos, anti-histona.
tacin oligoarticular asimtrica, de preferencia en extremidades
inferiores. Pregunta 4.- R: 1
La artritis por cristales de cido rico es habitualmente monoarti- El sndrome antifosfolpido se define por la presencia de criterios
cular. clnicos y analticos:
En el LES suele haber artralgias, con frecuencia migratorias.
La respuesta por lo tanto es la 5, ya que las localizaciones articula- Clnicos:
res incluidas son tpicas de la artritis reumatoide. Morbilidad en el embarazo:
- 3 o ms abortos (< 10 semanas de gestacin) espontneos, con-
Pregunta 2.- R: 1 secutivos y no justificados por otros mecanismos y excluidas
Lo primero que debes conocer es que la biopsia sinovial es un causas cromosmicas maternas o paternas, u hormonales o ana-
procedimiento til para el diagnstico diferencial de enfermedades tmicas maternas.
articulares de etiologa no filiada por la valoracin clnica y otros - Una o ms muertes fetales (>10 semanas de gestacin) inexpli-
mtodos menos invasivos, como el estudio del lquido sinovial. Se cadas con fetos normales.
indica fundamentalmente en los cuadros de monoartritis crnica (su- - Parto prematuro en la semana 34 del embarazo o anteriores por
perior por lo tanto a 6 semanas de evolucin), que no tiene hasta el pre-eclampsia o insuficiencia placentaria.
momento diagnstico preciso, especialmente si se sospecha la pre-
sencia de una infeccin. Podemos encontrar resultados especficos Laboratorio:
en la biopsia de las siguientes enfermedades: Anticuerpos anticardiolipina- IgG (ttulos positivos moderados o
Infecciones: en el caso de las artritis tuberculosas, fngicas o gono- altos).
ccica la sensibilidad del cultivo tisular de la membrana sinovial es Anticuerpos anticardiolipina- IgM (ttulos positivos moderados o
superior al 90%. altos).
Artritis por microcristales: presencia de cristales de urato monos- Anticoagulante lpico positivo. Se realizan a partir de plasma po-
dico o de pirofosfato clcico en la membrana sinovial. bre en plaquetas con pruebas coagulomtricas (tiempo de trom-
Neoplasias: en la reticulocitosis multicntrica, inflamacin sinovial boplastina parcial activado, tiempo de tromboplastina tisular, tiem-
con clulas gigantes e histiocitos espumosos, con citoplasma eosi- po de veneno de vbora de Rusell). En caso de alargamiento de
nfilo y tincin PAS positiva; en la sinovitis villonodular pigmenta- alguna de ellas, se intenta su correccin con plasma normal o con
da, proliferacin de la sinovial, clulas gigantes, histiocitos espu- la adicin de fosfolpidos.
mosos y depsitos de hemosiderina; en tumores primarios y mets-
tasis infiltracin de clulas malignas. Para la clasificacin definitiva se requiere la presencia de un criterio
Comentarios TEST

Enfermedades por depsito como la amiloidosis con depsito de clnico y uno de laboratorio. Se recomienda adems que los anti-
material amiloide que se tie con rojo congo, ocronosis con dep- cuerpos sean positivos en dos ocasiones separadas por un periodo
sito de pigmento ocrontico, hemocromatosis con depsito de mnimo de 6 semanas.
hemosiderina en la sinovial y en los macrfagos, sarcoidosis con La presencia de VDRL falsamente positivo es habitual en estos pacien-
granulomas no caseificantes, artritis por cuerpo extrao con part- tes, ya que el sustrato para la prueba tiene abundantes fosfolpidos,
culas ajenas a la sinovial como espinas vegetales o granulomas por por lo que se da una reaccin cruzada positiva. De esta forma nos
cuerpo extrao. queda como nica respuesta posible el factor reumatoide. En los
ltimos aos se ha descrito un cofactor que incrementa el riesgo de
La respuesta correcta es la artritis reumatoide, ya que en esta enferme- presentar trombosis en los pacientes con sndrome antifosfolpido,
dad encontramos hiperplasia sinovial e hipervascularizacin, infiltra- que son los anticuerpos anti-beta-2-glicoproteina.
dos de clulas inflamatorias crnicas y necrosis. Estos hallazgos son
comunes a otras colagenosis como el lupus eritematoso sistmico y Pregunta 5.- R: 2
otras patologas inflamatorias como las espondiloartropatas. Los La panarteritis nodosa es una vasculitis necrotizante con afectacin
ndulos reumatoides son raramente identificados en la sinovial. de arterias musculares pequeas y medianas que se caracteriza por
afectar a las arterias renales y viscerales. La lesin predominante a
Pregunta 3.- R: 5 nivel renal es una arteritis sin glomerulonefritis, por lo que parece
En esta pregunta la clave para encontrar la respuesta correcta no claro que la respuesta tiene que ser la segunda. Desde el punto de vista
est en el patrn de afectacin articular, ya que la presencia de artritis clnico es caracterstico el desarrollo progresivo de insuficiencia renal
intermitente, aunque caracterstica del LES, puede aparecer en gran e hipertensin arterial (respuestas 3 y 5).
parte de enfermedades inflamatorias articulares y colagenosis, sino en A diferencia de ella, la poliarteritis microscpica es una vasculitis
la combinacin de anticuerpos que tiene la paciente. necrotizante con afectacin de pequeos vasos (capilares, vnulas o
La primera respuesta, la enfermedad mixta del tejido conectivo, arteriolas) que cursa con escasos o nulos depsitos de complejos
presenta de forma caracterstica ttulos elevados de anticuerpos inmunitarios (pauciinmune). Como tambin puede aparecer arteritis
antinucleares que son anti-RNP a titulacin igual o superior a necrosante de arterias de pequeo y mediano tamao, este proceso
1:1.600 (recuerda que la titulacin de anticuerpos es mayor cuan- muestra manifestaciones comunes con la PAN clsica, salvo que la

M exico A rgentina CTO Medicina C/ Nez de Balboa, 115 28006 MADRID (Espaa) Tfno.: (91) 782 43 32 / Fax: (91) 782 43 27
C hile U ruguay E-mail: secretaria@ctomedicina.com; iberocto@ctomedicina.com WEB: www.ctomedicina.com; www.iberocto.com
RM Pg. 1
REUMATOLOGA
Seguimiento a distancia Preparacin Examen de Seleccin 05/06 1 Vuelta
glomerulonefritis es muy frecuente en la poliarteritis microscpica, y a Inflamacin nasal u oral: desarrollo de lceras dolorosas o indolo-
menudo aparece capilaritis pulmonar. ras en la mucosa oral o secrecin nasal purulenta o hemtica.
El diagnstico de la PAN clsica se basa en la presencia de los Alteraciones en la radiologa de trax: ndulos, infiltrados no mi-
hallazgos caractersticos en la biopsia de los tejidos afectados. Si no gratorios o cavitacin.
existe un tejido fcilmente accesible a la biopsia, es suficiente con Alteraciones del sedimento urinario: microhematuria o presencia
demostrar por angiografa la afectacin vascular, especialmente en de cilindros hemticos.
forma de aneurismas de arterias de pequeo y mediano calibre en la Inflamacin granulomatosa en la biopsia: inflamacin granuloma-
circulacin renal, heptica y visceral (respuesta 4). Sin embargo los tosa dentro de la pared vascular o en regin perivascular.
aneurismas no son patognomnicos de la PAN clsica; adems no
siempre tienen que aparecer, quedando reducidos los signos angio- La presencia de al menos dos de estos criterios supone una especifi-
grficos a estenosis y obliteracin de los vasos. cidad/sensibilidad para el diagnstico de la granulomatosis de Wegener.
En este paciente, la determinacin de factor reumatoide positivo
Pregunta 6.- R: 3 no debe hacerte dudar sobre el diagnstico, ya que aunque es carac-
La respuesta 1 se refiere a una caracterstica angiogrfica caracters- terstica su determinacin en la artritis reumatoide, puede aparecer en
tica de la PAN nodosa clsica que no solemos observar en la poliange- otras enfermedades inflamatorias del tejido conectivo como el sndro-
tis microscpica, ya que en esta los vasos afectados suelen ser de me de Sjgren o vasculitis, infecciones y neoplasias.
menor dimetro. La presencia de microaneurismas no es patognom- Respecto a otras manifestaciones clnicas, puede aparecer propto-
nica de la PAN clsica, ya que se han encontrado en otras patologas sis por el desarrollo de granulomas retrooculares, incluyndose como
como la granulomatosis de Wegener, mixoma auricular, endocarditis, otras manifestaciones oculares la conjuntivitis, dacriocistitis y escleri-
lupus eritematoso sistmico o prpura trombtica trombocitopnica. tis. La respuesta que nos queda es la nmero 3, destruccin sea
La respuesta 2 es una caracterstica de la poliangetis microscpica, orbitaria, ya que la granulomatosis de Wegener afecta a tejidos blan-
ya que esta vasculitis afecta a vasos de pequeo calibre. dos, pero no suele producir afectacin sea. Cuando ocurre sta,
La respuesta 4, la afectacin pulmonar, es de nuevo tpica de la debemos pensar en otros procesos destructivos, como el granuloma
panarteritis microscpica en forma de capilaritis pulmonar. Las arte- de la lnea media o tumores.
rias pulmonares no estn afectadas en la PAN clsica, y la afectacin
de las arterias bronquiales es poco habitual. Pregunta 9.- R: 4
La presencia de enfermedad renal es caracterstica de ambas enfer- La granulomatosis de Wegener se considera un sndrome
medades, pero en la PAN clsica se ven afectadas las arterias de me- renopulmonar por la frecuente afectacin que ocurre a estos dos
diano calibre y en la microscpica los capilares, produciendo niveles (junto con la afectacin nasal u oral supone la trada clnica
glomerulonefritis. tpica de la enfermedad). Otras patologas con frecuente afectacin a
La nica respuesta posible, por lo tanto, es la tercera, ya que ambas estos dos niveles se incluyen en su diagnostico diferencial:
enfermedades sin tratamiento tienen un pronstico muy desfavora- La granulomatosis linfomatoide afecta al pulmn, la piel, sistema
ble. El esquema teraputico incluye la combinacin de prednisona 1 nervioso central y el rin, rganos donde se encuentran las clu-
mg/kg/da con ciclofosfamida 2 mg/kg/da. Con este tratamiento se las plasmocitoides y linfocitoides atpicas que infiltran los tejidos
consiguen remisiones en el 90% de los casos. con carcter angioinvasor. En la mitad de los casos se desarrolla un
autntico linfoma maligno.
Pregunta 7.- R: 3 El sndrome de Goodpasture se caracteriza por la presencia de
El planteamiento a la hora de establecer el diagnstico en esta anticuerpos antimembrana basal que producen hemorragias pul-
paciente se orienta por los hallazgos clnicos y analticos, donde des- monares y glomerulonefritis necrosante con depsito lineal de
taca la presencia de eosinofilia perifrica. La respuesta 1 la podemos inmunocomplejos y complemento.
descartar rpidamente, ya que en la PAN en su forma clsica no hay La granulomatosis alrgica es una vasculitis necrotizante caracteri-
afectacin pulmonar y en la microscpica no existe eosinofilia. zada por la presencia de asma, alergia y eosinofilia perifrica en la
La granulomatosis de Wegener (respuesta 2) puede producir infil- mayora de los pacientes. Recuerda que en esta enfermedad los
trados pulmonares bilaterales, pero de nuevo en esta enfermedad no infiltrados pulmonares son migratorios o transitorios.
es caracterstica la presencia de eosinofilia. En algunas neumonas como las producidas por neumococo,
La respuesta nmero 4 habla de un cuadro, la poliangetis de su- Legionella o Mycoplasma se produce glomerulonefritis proliferativa
perposicin, caracterizada por la presencia en un mismo paciente de con depsito granular de inmunocomplejos y complemento.

Comentarios TEST
hallazgos clnicos y/o histolgicos propios de ms de una vasculitis Otros sndromes renopulmonares, que no aparecen dentro de las
incluyendo PAN clsica, granulomatosis alrgica, Wegener, arteritis de respuestas de esta pregunta, son la poliarteritis microscpica, el LES
Takayasu o vasculitis por hipersensibilidad. Sin embargo, todos los o la prpura trombtica trombocitopnica.
hallazgos encontrados en esta paciente, incluyendo la afectacin cu-
tnea en forma de prpura palpable, se describen en el contexto de la Como ves, excepto la respuesta nmero 4 (enfermedad de Buerger),
enfermedad de Churg-Strauss (respuesta correcta 3). Esta enfermedad el resto de las entidades se incluyen en el diagnstico diferencial con
incluye para su diagnstico la presencia de una serie de criterios: la granulomatosis de Wegener. La enfermedad de Buerger o trombo-
Asma. angetis obliterante es un proceso caracterizado por la isquemia de las
Eosinofilia perifrica superior al 10% o mayor de 1.500/mm3. extremidades, en la que la formacin de trombos arteriales y venosos
Historia previa de alergia (excepto medicamentosa). es el mecanismo fundamental. Afecta sobre todo a pacientes jvenes,
Afectacin pulmonar (infiltrados migratorios o transitorios) atribui- varones y fumadores.
bles a vasculitis sistmica.
Mono o polineuropata. Pregunta 10.- R: 2
Afectacin de senos paranasales. Historia de dolor agudo o crni- La paciente de este caso clnico presenta algunas de las caracters-
co paranasal o velamiento radiolgico de senos paranasales. ticas tpicas de la arteritis de Takayasu que se recogen en sus criterios
Eosinfilos extravasculares. Biopsia de arteria, arteriola o vnula diagnsticos:
que muestra acumulacin de eosinfilos extravasculares. Edad inferior a los 40 aos.
Claudicacin de las extremidades (desarrollo o empeoramiento
La presencia de 4 criterios confiere una elevada sensibilidad y de cansancio en los msculos de una o ms extremidades durante
especificidad para el diagnstico de la enfermedad. su uso, especialmente de extremidades superiores).
Diferencia de tensin arterial entre ambos brazos > 10 mmHg (en
Pregunta 8.- R: 3 la tensin sistlica).
Esta paciente presenta las caractersticas tpicas de la granulomatosis Soplos en arterias subclavias y aorta: soplos audibles a la ausculta-
de Wegener para cuyo diagnstico utilizamos los siguientes criterios: cin sobre una o ambas arterias subclavias o en la aorta abdominal.

M exico A rgentina CTO Medicina C/ Nez de Balboa, 115 28006 MADRID (Espaa) Tfno.: (91) 782 43 32 / Fax: (91) 782 43 27
Pg. 2 RM C hile U ruguay E-mail: secretaria@ctomedicina.com; iberocto@ctomedicina.com WEB: www.ctomedicina.com; www.iberocto.com
REUMATOLOGA
Preparacin Examen de Seleccin 05/06 1 Vuelta Seguimiento a distancia
Anomalas en la angiografa: estrechamiento u oclusin de la aorta nando la mayora de los pacientes de forma favorable igual que en las
entera, sus ramas proximales o grandes arterias, en la zona proximal vasculitis cutneas (respuesta 4). La escasa mortalidad suele estar rela-
de las extremidades superiores o inferiores, no debida a cionada con la aparicin de insuficiencia renal.
arteriosclerosis, displasia muscular, o causas similares. Los cambios
habitualmente son focales o segmentarios. Pregunta 13.- R: 1
El trmino de vasculitis predominantemente cutneas se refiere a un
La primera respuesta, panarteritis nodosa, la podemos descartar por cuadro clnico que suele estar dominado por las manifestaciones cut-
varios motivos: la edad de presentacin es demasiado precoz para esta neas, aunque no de forma exclusiva. De hecho, en este tipo de vasculitis
enfermedad y produce con ms frecuencia otra sintomatologa, como cualquier rgano puede estar afectado, aunque la clnica extracutnea
afectacin del sistema nervioso perifrico, gastrointestinal y renal. suele ser mucho menos intensa que la provocada por las vasculitis
La respuesta 3, el LES con anticoagulante lpico, puede producir necrosantes generalizadas. Otros trminos sinnimos son el de vasculitis
ictus cerebral, pero no es tpica la alteracin del pulso en extremida- por hipersensibilidad o el de vasculitis leucocitoclstica.
des superiores. Lo mismo podemos decir de la respuesta 4, el sndro- Las vasculitis predominantemente cutneas pueden dividirse en dos
me antifosfolpido catastrfico. grupos, segn el antgeno responsable de la hipersensibilidad. Dentro
En la vasculitis del sistema nervioso central, respuesta 5, la sintoma- de los antgenos de origen exgeno se incluyen frmacos, infecciones
tologa aparece exclusivamente a este nivel, estando ausente la altera- (respuesta 1) o una protena extraa. La crioglobulinemia mixta esen-
cin de pulsos en extremidades superiores y la insuficiencia artica. cial produce cuadros de vasculitis cutnea y se ha asociado al virus de
la hepatitis C (por lo tanto se confirma que la respuesta correcta es la 1).
Pregunta 11.- R: 1 Dentro de los antgenos endgenos incluimos las situaciones relaciona-
Para el diagnstico de la arteritis de la temporal, igual que como das con enfermedades autoinmunes como el lupus eritematoso
hemos visto en otras vasculitis, nos basamos en la presencia de una sistmico, la artritis reumatoide o el sndrome de Sjgren (respuesta 5), o
serie de manifestaciones clnicas tpicas. En este caso son: algunos tumores como los linfomas (respuesta 2).
Edad igual o superior a los 50 aos (respuesta nmero 3). La respuesta 3 debemos descartarla porque la panarteritis micros-
Cefalea de reciente comienzo. cpica se incluye, al igual que la panarteritis nodosa clsica y la enfer-
Alteraciones de la arteria temporal (dolor a la palpacin, descenso medad de Churg-Strauss, dentro de las vasculitis necrotizantes
del pulso en las arterias temporales no relacionado con arterioscle- sistmicas.
rosis de las arterias carotdeas). La respuesta 4 es incorrecta, ya que es caracterstico que la prpu-
VSG igual o superior a 50 mm/1 hora (respuesta nmero 2). ra de las vasculitis cutneas no desaparezca con la vitropresin.
Biopsia de la arteria temporal anormal (mostrando vasculitis carac-
terizada por un predominio de clulas mononucleares o inflama- Pregunta 14.- R: 5
cin granulomatosa, generalmente con clulas gigantes El cuadro descrito en este caso clnico es una vasculitis aislada del
multinucleadas. sistema nervioso central caracterizada por la afectacin inflamatoria
vascular exclusivamente a este nivel, sin que aparezcan signos de
Desde el punto de vista clnico, en la mitad de los pacientes apare- vasculitis en otras localizaciones. Afecta sobre todo a arteriolas, aun-
cen sntomas de polimialgia reumtica, caracterizada por la presencia que se pueden ver afectados vasos de cualquier tamao. El proceso
de dolor moderado o intenso y rigidez matinal superior a los 30 minu- inflamatorio suele estar integrado por infiltrados de clulas mononu-
tos y de ms de un mes de evolucin al menos en dos de las siguientes cleares, acompaados o no de la formacin de granulomas. En la
tres reas: cuello, cintura escapular y /o cintura pelviana (respuesta mayora de los casos no se encuentra ningn factor patognico
nmero 4). desencadenante, aunque se han visto casos relacionados con infec-
El tratamiento de la enfermedad se basa en la administracin de ciones virales y bacterianas, enfermedad de Hodgkin o consumo de
corticoides en dosis altas (1 mg/kg/da de prednisona) (respuesta 5). anfetaminas.
La nica opcin posible despus de lo que hemos visto es la prime- El motivo de consulta habitual, como aparece descrito en el caso
ra respuesta. Recuerda que la afectacin vascultica suele ser clnico, es la presencia de cefalea, trastornos de las funciones superiores
segmentaria, por lo que la presencia de un resultado normal en la y defectos neurolgicos focales, sin sntomas generales. El diagnstico
biopsia no descarta por completo el diagnstico sospechado. Para generalmente se establece demostrando en la arteriografa las alteracio-
intentar que el resultado no sea negativo se debe hacer una biopsia nes vasculares caractersticas, y se confirma por biopsia del parnquima
Comentarios TEST

amplia de la arteria temporal (unos 2 centmetros), estudiando el cerebral y las leptomeninges (respuesta correcta nmero 5).
patlogo mltiples cortes de la muestra. El pronstico de esta enfermedad es muy desfavorable, aunque
algunos pacientes responden a corticoides solos o en combinacin a
Pregunta 12.- R: 5 ciclofosfamida.
La enfermedad de Schnlein-Henoch es una vasculitis que suele Debes tener en cuenta que la duda que en ocasiones se tiene al
aparecer en la infancia, entre los 4 y 7 aos, caracterizada por la leer el caso clnico con el diagnstico de la arteritis de la temporal,
presencia de prpura palpable (distribuida principalmente en nalgas queda despejada por la edad de la paciente, ya que esta patologa
y en miembros inferiores), artralgias, signos y sntomas gastrointestina- aparece por encima de los 50 aos (respuesta 4).
les y glomerulonefritis. La presencia de estos ltimos sntomas de afec-
tacin visceral no es caracterstica de las vasculitis predominantemen- Pregunta 15.- R: 3
te cutneas, por lo que debemos elegir la respuesta nmero 5. Sin El diagnstico de este paciente se corresponde con un sndrome
embargo, el resto de caractersticas que aparecen en las respuestas son de Behet con datos suficientes como para confirmarlo segn los
comunes a ambas enfermedades. Los vasos predominantemente afec- criterios diagnsticos que se utilizan:
tados son los de menor calibre (capilares y vnulas; respuesta 1). El lceras recidivantes en boca, ms dos de las siguientes manifesta-
fenmeno de leucocitoclasia (respuesta 2) supone la presencia de ciones:
residuos nucleares que quedan de los neutrfilos que han infiltrado lceras genitales recidivantes.
los vasos y sus alrededores durante la fase aguda. La prpura palpable Lesiones oculares (uvetis anterior o posterior o vasculitis retiniana).
es la norma en ambas enfermedades y se debe a la extravasacin de Lesiones cutneas (eritema nodoso, pseudofoliculitis, o lesiones
hemates a nivel cutneo por la rotura de pequeos vasos. papulo-pstulas, o ndulos acneiformes).
En el caso de la vasculitis de Schnlein-Henoch se sospecha la Prueba de patergia positiva (observada por un mdico a las 48
presencia de mltiples antgenos desencadenantes, como infecciones horas).
de vas areas superiores, diversos frmacos, alimentos, picaduras de
insectos o inmunizaciones. Los anticuerpos formados suelen ser IgA, En el caso de esta pregunta, el paciente tiene lceras orales, genita-
cuya presencia se ha demostrado tambin en las biopsias renales de les y eritema nodoso, por lo que cumple el criterio mayor y dos
estos pacientes. La mortalidad de esta vasculitis es escasa, evolucio- menores.

M exico A rgentina CTO Medicina C/ Nez de Balboa, 115 28006 MADRID (Espaa) Tfno.: (91) 782 43 32 / Fax: (91) 782 43 27
C hile U ruguay E-mail: secretaria@ctomedicina.com; iberocto@ctomedicina.com WEB: www.ctomedicina.com; www.iberocto.com
RM Pg. 3
REUMATOLOGA
Seguimiento a distancia Preparacin Examen de Seleccin 05/06 1 Vuelta
El tratamiento del sndrome de Behet es sintomtico y emprico y El sndrome poliangetico de superposicin se describe cuando en
debe estar relacionado con los sntomas que presenta el paciente. Re- un paciente aparecen datos clnicos y/o anatomopatolgicos com-
cuerda que en este caso hay sntomas a nivel cutneo, de mucosas y patibles con ms de una vasculitis. En este caso, los sntomas son
articular. La afectacin de mucosas puede mejorar con la aplicacin todos tpicos de una sola vasculitis, la panarteritis nodosa clsica.
local de corticoides o de forma sistmica en dosis bajas (respuesta 4 y 5).
La colchicina (respuesta 1), los AINEs (respuesta 2) o el interfern alfa Pregunta 18.- R: 5
pueden ser beneficiosos para el tratamiento de la artritis. Las causas de hiperuricemia es una de las cuestiones ms pregun-
En el caso de que el paciente presente uvetis o afectacin del tadas sobre este tema junto con su tratamiento. Dentro de la etiologa
sistema nervioso central, es necesaria la administracin de corticoides de la hiperuricemia solemos distinguir las siguientes posibilidades:
en dosis altas (1 mg/kg/da) y azatioprina (2-3 mg/kg/da) o ciclosporina Hiperuricemia por sntesis de urato aumentada:
(5-10 mg/kg/da). La ausencia de sintomatologa del paciente a este - Por aumento del catabolismo de las purinas: dentro de este grupo
nivel hace innecesaria la administracin de ciclosporina, por lo que se encuentran las enfermedades mielo y linfoproliferativas, mie-
la respuesta es la 3. loma y otros tumores, especialmente si son tratados con quimio-
terapia, por la gran destruccin celular que se deriva. Tambin se
Pregunta 16.- R: 3 puede encontrar en la hemlisis, psoriasis, policitemia vera y en-
La presentacin de este caso clnico es caracterstica de la enferme- fermedad de Paget. La dieta, aunque es una fuente de purinas
dad de Wegener. En esta vasculitis la afectacin clnica ocurre a tres que se pueden metabolizar a cido rico, tiene mucha menor
niveles: importancia. De hecho, la restriccin estricta de purinas reduce
Inflamacin nasal u oral: desarrollo de lceras dolorosas o indolo- la concentracin plasmtica de cido rico en solo 1 mg/dl.
ras en la mucosa oral o secrecin nasal purulenta o hemtica. - Defectos hereditarios del metabolismo: aumento de la activi-
Alteracin en la radiologa de trax: ndulos, cavitacin o infiltra- dad de la PRPP sintetasa y el dficit de HPRT, que cuando es
dos no migratorios ni fugaces en el parnquima pulmonar. parcial, produce el sndrome de Kelley-Seegmiller y cuando es
Alteraciones en el sedimento urinario: microhematuria (> de 5 completo, el sndrome de Lesch-Nyhan. Esta ltima enferme-
hemates por campo) o presencia de cilindros hemticos. dad (respuesta 5) se caracteriza por la presencia de hiperurice-
mia, hiperaciduria, clculos de cido rico y gota por sobre-
Adems se manifiesta la presencia de anticuerpos c-ANCA positi- produccin de uratos. Asocia tambin automutilacin, coreo-
vos, que son muy sensibles y especficos para el diagnstico de la atetosis y otros trastornos neurolgicos.
granulomatosis de Wegener, aunque no patognomnicos, ya que tam-
bin se observan en otras vasculitis necrotizantes como la PAN clsica, Excrecin renal disminuida: este mecanismo supone el 90% de las
la P.A. microscpica, enfermedad de Churg-Strauss y algunas vasculitis hiperuricemias. Est relacionada con mltiples situaciones, entre
por frmacos. las que podemos encontrar la toma de frmacos: diurticos, salici-
Desde el punto de vista del pronstico, este es fatal sin tratamiento. latos en dosis bajas, ciclosporina, tuberculostticos y algunos
Los glucocorticoides solos producan alguna mejora sobre la enfer- antirretrovirales; insuficiencia renal y situaciones que producen
medad, pero no modificaban de forma sustancial el pronstico. Se ha acidosis como la cetoacidosis diabtica y la acidosis lctica.
demostrado claramente que el tratamiento de eleccin es la ciclofos-
famida en dosis de 2 mg/kg/da, por va oral, junto con glucocorticoi- Como puedes comprobar, la respuesta correcta es la enfermedad
des. Durante el tratamiento, hay que vigilar de cerca el contaje de de Lesch-Nyhan, ya que aunque se trata de una patologa que produ-
leucocitos y ajustar la dosis de ciclofosfamida para mantenerlos por ce hiperuricemia por aumento de sntesis de uratos igual que las de-
encima de 3000/ mm3. Al principio del tratamiento hay que incorpo- ms respuestas, no est relacionada como el resto con el aumento del
rar prednisona en dosis de 1 mg/kg/da, con toma diaria durante el catabolismo de las purinas.
primer mes, pasando gradualmente a una pauta de corticoides a das
alternos; por ltimo se disminuyen progresivamente hasta interrum- Pregunta 19.- R: 2
pirlos aproximadamente a los 6 meses. Esta pregunta permite que recuerdes cules son las manifestacio-
nes clnicas y la actitud teraputica que debes adoptar en un paciente
Pregunta 17.- R: 3 con hiperuricemia.
En este caso clnico debes fijarte en las manifestaciones clnicas y Hiperuricemia asintomtica: en aquellos pacientes que presentan
analticas que de forma progresiva te llevan a la respuesta correcta, hiperuricemia sin clnica no est indicado tratarla. La excepcin la

Comentarios TEST
que es la panarteritis nodosa clsica. Se trata de una mujer de media- suponen los pacientes con neoplasias, especialmente los que va-
na edad donde destaca la presencia de afectacin del sistema nervio- yan a recibir quimioterapia, que deben ser adecuadamente hidra-
so perifrico en forma de mononeuritis mltiple confirmada en la tados, alcalinizando la orina para facilitar la excrecin renal del
exploracin fsica, dolor abdominal con signos de irritacin perito- cido rico y administrando alopurinol para disminuir su sntesis.
neal y rectorragia ocasional e insuficiencia renal con elevacin de Artritis aguda por cido rico: debemos tratar al paciente con AINEs
productos nitrogenados en la sangre. y/o colchicina mientras presente los sntomas. Si el paciente no
Desde el punto de vista analtico presenta adems leucocitosis sin reciba previamente tratamiento para la hiperuricemia, no debe
eosinofilia, por lo que la posibilidad de que se trate de una enferme- incorporarse hasta que el episodio agudo ceda.
dad de Churg-Strauss es pequea, mxime si recordamos que no Gota intercrtica: supone la situacin que padece el paciente de
tiene clnica pulmonar ni antecedente de asma o alergia. este caso, es decir, episodios recidivantes de artritis aguda pero sin
Otras respuestas ofrecidas debemos descartarlas por los siguientes clnica entre ellos. En este caso debemos tratar la hiperuricemia y la
motivos: eleccin entre alopurinol o uricosricos depender del resultado
La enfermedad de Goodpasture se caracteriza por la presencia de del balance de cido rico. Si la excrecin renal de cido rico en
anticuerpos anti-membrana basal, apareciendo las manifestaciones 24 horas es superior a 700 mg, administraremos alopurinol, y si es
clnicas a nivel pulmonar (hemorragia alveolar) y renal (glomerulo- menor, uricosricos (benzobromarona, sulfinpirazona o
nefritis), sin presentar afectacin en el sistema nervioso perifrico. probenecid). Sin embargo hay que tener en cuenta que los
El sndrome de Schnlein-Henoch es una vasculitis que suele afec- uricosricos estn contraindicados si el paciente tiene anteceden-
tar a nios entre 4 y 7 aos, aunque en ocasiones aparece tambin tes de nefrolitiasis (como en este paciente) o padece insuficiencia
en adultos. Aunque puede producir dolor abdominal y renal. En estos casos, la alternativa teraputica es la utilizacin de
glomerulonefritis, el cuadro viene dominado por la clnica cut- alopurinol.
nea (prpura palpable) y la artritis. En la gota tofcea crnica, cuando los tofos son de gran tamao se
En la granulomatosis de Wegener, la afectacin preferente ocurre utiliza como frmaco de eleccin el alopurinol, pero si son peque-
en las vas areas superiores, pulmn y rin, estando ausente en os tambin podemos utilizar los uricosricos, como veamos en el
esta paciente las dos primeras localizaciones. apartado anterior.

M exico A rgentina CTO Medicina C/ Nez de Balboa, 115 28006 MADRID (Espaa) Tfno.: (91) 782 43 32 / Fax: (91) 782 43 27
Pg. 4 RM C hile U ruguay E-mail: secretaria@ctomedicina.com; iberocto@ctomedicina.com WEB: www.ctomedicina.com; www.iberocto.com
REUMATOLOGA
Preparacin Examen de Seleccin 05/06 1 Vuelta Seguimiento a distancia
Siempre que utilizamos alopurinol o uricosricos debemos aadir Como puedes comprobar, dentro de las localizaciones habituales
colchicina durante unos meses para prevenir la aparicin de nuevos de la artrosis primaria se encuentran todas las respuestas, excepto la
episodios de artritis aguda al descender los niveles de cido rico. articulacin del codo. En caso de aparecer en esta articulacin artro-
sis deberemos pensar en una causa secundaria, como son las enfer-
medades microcristalinas por depsito de cristales de calcio (pirofos-
fato clcico o hidroxiapatita). Otras posibilidades son los traumatis-
mos, alteraciones congnitas, metablicas (ocronosis, hemocromato-
sis, enfermedad de Wilson), endocrinas (acromegalia, hiperparatiroi-
dismo, hipotiroidismo), enfermedades seas y articulares o artropata
neuroptica.

Pregunta 22.- R: 4
En este caso clnico debemos en primer lugar establecer el diagns-
tico. Se trata de una paciente mayor que presenta un cuadro de mo-
noartritis aguda. Como antecedente relevante nos indican que ya ha
presentado episodios similares en otras articulaciones, siendo el estu-
dio radiolgico caracterstico de condrocalcinosis. Este hallazgo no
asegura el tipo de cristal que tiene la paciente, ya que cualquier
microcristal que tenga calcio (pirofosfato clcico, oxalato clcico, hi-
droxiapatita clcica) puede dar una imagen de condrocalcinosis. Para
establecer el diagnstico de forma definitiva siempre debemos hacer
un estudio de microcristales en el lquido sinovial. En este caso nos
indican que presenta cristales de forma rectangular que son caracters-
ticos de la artropata por pirofosfato clcico. Adems nos descartan,
con el estudio microbiolgico, la presencia de infeccin articular.
Una vez establecido de forma definitiva el diagnstico debemos ele-
gir la actitud teraputica y sealar la alternativa de tratamiento que no se
utiliza.
La respuesta 1 (AINEs) y 5 (colchicina) son los frmacos que habitual-
Pregunta 19. Artritis gotosa. mente utilizamos en la artritis aguda por microcristales. La respuesta 2
(reposo de la articulacin) es una medida correcta en las fases agudas de
Pregunta 20.- R: 5 la inflamacin articular, independientemente de la causa que la produ-
El depsito de cristales de pirofosfato ocurre en la mayora de los ce. Sin embargo, hay que intentar comenzar la movilizacin articular
casos en pacientes mayores de 70 aos y sin ninguna enfermedad en cuanto el dolor lo permita.
metablica que induzca su depsito. Sin embargo, cuando se de- En algunos pacientes puede ser una buena medida la administra-
muestra la presencia de una artropata por estos cristales en una cin intraarticular de corticoides, pero es obligatorio, como se ha reali-
persona por debajo de los 55 aos, es necesario descartar la presen- zado en este caso, confirmar el diagnstico antes de realizar la infiltra-
cia de una serie de patologas que favorecen su depsito como son: cin. Sin embargo, la administracin sistmica de corticoides no es una
Hiperparatiroidismo (respuesta nmero 1, PTH) medida teraputica adecuada, por lo que la respuesta es la 4.
Hipomagnesemia (respuesta nmero 2, magnesio).
Hemocromatosis (respuesta 3, ferritina). Pregunta 23.- R: 5
Hipofosfatasia (respuesta 4, fosfatasa alcalina). La hidroxiapatita clcica es el mineral principal del hueso y de los
dientes. La mayora de las calcificaciones de partes blandas del orga-
Como ves, la respuesta correcta es la 5, ya que no hay relacin nismo son producidas por hidroxiapatita. En la mayora de los casos,
con patologa asociada a la glndulas suprarrenales. La asociacin como ocurre con los cristales de pirofosfato, el depsito es idioptico,
con hipotiroidismo, aunque probable, no est demostrada. pero existen una serie de enfermedades asociadas que debemos re-
Comentarios TEST

Existen tambin formas familiares. Suelen iniciarse entre la 3 y 5 cordar, entre las que incluimos:
dcada de la vida, con afectacin poliarticular severa. El mecanismo Enfermedades del tejido conectivo: LES, dermatomiositis o escle-
hereditario parece ser autosmico dominante, observndose en algu- rodermia.
nas familias defectos genticos de los cromosomas 5p y 8q. Enfermedades metablicas: hiperparatiroidismo, hiperfosfatemia,
intoxicacin por vitamina D, insuficiencia renal crnica, diabetes
Pregunta 21.- R: 3 mellitus.
La artrosis es la patologa articular ms frecuente. En la artrosis prima- Trastornos neurolgicos: ACV y traumas medulares.
ria o idioptica no existen factores predisponentes. En la artrosis secun-
daria el proceso es anatomopatolgicamente indistinguible del prima- Desde el punto de vista clnico los depsitos suelen ser articulares o
rio, pero existe algn factor subyacente que favorece el desarrollo del periarticulares y frecuentemente asintomticos, pero tambin se des-
proceso. En la pregunta que nos realizan existen una serie de localiza- criben los siguientes cuadros sintomticos relacionados:
ciones que son caractersticas de la artrosis primaria, como son: Periartritis calcificante, siendo el hombro la localizacin ms habi-
Manos: nodular (ndulos de Heberden y ndulos de Bouchard), tual (respuesta 2).
artrosis interfalngica erosiva (sin ndulos), de la primera articula- Artrosis: se identifican cristales en el 50% de los cuadros de artrosis
cin carpometacarpiana. (respuesta 3).
Pies: hallux valgus, hallux rigidus, dedos contrados (en martillo), Artritis: inflamacin aguda monoarticular (respuesta 4).
articulacin astragalonavicular. Artropata destructiva: afecta preferentemente a la rodilla y al hom-
Rodilla: compartimento interno, compartimento externo, compar- bro (hombro de Milwaukee, respuesta 1).
timento femororrotuliano.
Cadera: excntrica (superior), concntrica (axial, interna), difusa. La respuesta que nos queda como correcta es la poliartritis simtri-
Columna vertebral: articulaciones apofisarias, articulaciones inter- ca (respuesta 5), que es caracterstica de la artritis reumatoide.
vertebrales (discos), espondilosis (osteofitos), ligamentosa (hiperos-
tosis vertebral anquilosante). Pregunta 24.- R: 2
Otras localizaciones nicas: glenohumeral, acromioclavicular, El oxalato es el producto final del metabolismo del cido ascrbico
tibioastragalina, sacroiliaca, temporomandibular. y de algunos aminocidos. Se alcanzan concentraciones sricas de

M exico A rgentina CTO Medicina C/ Nez de Balboa, 115 28006 MADRID (Espaa) Tfno.: (91) 782 43 32 / Fax: (91) 782 43 27
C hile U ruguay E-mail: secretaria@ctomedicina.com; iberocto@ctomedicina.com WEB: www.ctomedicina.com; www.iberocto.com
RM Pg. 5
REUMATOLOGA
Seguimiento a distancia Preparacin Examen de Seleccin 05/06 1 Vuelta
sobresaturacin como consecuencia de una excrecin renal disminu- Como puedes comprobar, todas las manifestaciones que aparecen
ida y otros factores como la ingesta elevada de vitamina C. Aunque en las respuestas de la pregunta aparecen en el lupus eritematoso
existe una forma de oxalosis primaria (respuesta 1) caracterizada por la sistmico, pero debemos considerar como la ms habitual de ellas la
presencia de hiperoxalemia, nefrolitiasis, insuficiencia renal y muerte aparicin de artritis.
alrededor de los 20 aos, la mayora de los casos se asocian a insufi-
ciencia renal terminal (respuesta correcta 2). Otras enfermedades que Pregunta 27.- R: 3
aparecen en esta pregunta, como la hemocromatosis o el El diagnstico del lupus eritematoso se basa en la utilizacin de 11
hiperparatiroidismo, se relacionan con el depsito de cristales de criterios diagnsticos. Se precisa la presencia de al menos 4 de ellos,
pirofosfato. no existiendo ningn criterio obligatorio. Recientemente, en el ao
Desde el punto de vista clnico, el depsito de cristales de oxalato 1997, se han editado unas pequeas modificaciones respecto a los
afecta con frecuencia a las articulaciones de rodillas y manos, produ- criterios aceptados desde 1982, quedando actualmente de la siguien-
ciendo en la articulacin calcificacin (condrocalcinosis) al igual que te forma:
otros cristales de calcio. Los derrames articulares suelen ser no 1. Eritema malar (eritema fijo, plano o elevado sobre los pmulos).
inflamatorios con un nmero de clulas inferior a las 2000/mm3. El 2. Eritema discoide (placas elevadas de eritema con descamacin
aspecto y la birrefringencia de los cristales de oxalato es variable, queratsica).
siendo frecuentemente de forma bipiramidal y con birrefringencia 3. Fotosensibilidad (la exposicin a rayos ultravioleta causa erupcin
muy positiva. cutnea).
Desde el punto de vista teraputico hay que tener en cuenta que 4. lceras bucales (lceras bucales o nasofarngeas observadas por el
en la oxalosis primaria se indica el trasplante heptico. En los pacien- mdico).
tes con insuficiencia renal se evitan los suplementos con vitamina C. 5. Artritis (artritis erosiva con afectacin de dos o ms articulaciones
Cuando se produce clnica se utilizan la colchicina, los AINEs o los perifricas, que se caracterizan por dolor, tumefaccin o derrame).
corticoides intraarticulares. 6. Serositis (pleuritis o pericarditis).
7. Enfermedad renal (proteinuria mayor de 500 mg/24 horas o cilin-
Pregunta 25.- R: 4 dros celulares).
En este caso clnico debemos plantearnos el diagnstico diferen- 8. Enfermedad neurolgica (convulsiones o psicosis sin ninguna otra
cial en una monoartritis aguda. Las causas ms habituales de esta causa).
sintomatologa son las infecciones articulares, las artritis por microcris- 9. Enfermedad hematolgica (anemia hemoltica, leucopenia, linfo-
tales y tambin se puede considerar la artritis reactiva, aunque en este penia o trombopenia, descartando induccin farmacolgica de la
ltimo caso suelen afectarse nuevas articulaciones de forma aditiva en citopenia).
el curso evolutivo. 10.Trastornos inmunolgicos (anti-DNAds, anti-Sm o anti-fosfolpi-
La paciente de esta pregunta ha sido sometida recientemente a una do).
intervencin quirrgica; esta situacin puede favorecer por un lado la 11.Anticuerpos antinucleares (ttulo de AAN elevado por inmuno-
aparicin de una bacteriemia, y por lo tanto de una artritis sptica, y fluorescencia).
debido a la situacin de stress que supone, favorecer la artritis por
microcristales. Como ves, la respuesta 3 es la nica correcta, al estar incluida la
El estudio del lquido sinovial, tcnica ms importante para poder psicosis lpica en el criterio nmero 8, dentro de la enfermedad
definir el diagnstico, nos muestra un nmero de clulas muy eleva- neurolgica.
do, por encima de las 50.000/mm3, lo cual suele ser tpico de las
infecciones, pero ocasionalmente tambin se observa en procesos Pregunta 28.- R: 4
inflamatorios muy activos. La ausencia de microorganismos en la En esta pregunta se manifiestan algunos de los factores genticos
tincin de GRAM y sobre todo en el cultivo hace poco probable la que se han descrito asociados al lupus eritematoso sistmico.
infeccin, ya que la sensibilidad de esta tcnica es del 90%. Finalmen- La respuesta 1 recuerda que los gemelos monocigticos desarro-
te la presencia positiva de cristales de forma rectangular, caractersti- llan cerca del 50% del lupus cuando uno de ellos est afectado. Esta
cos de la artropata por cristales de pirofosfato, confirma como res- asociacin hereditaria tambin se observa en los familiares de primer
puesta correcta la nmero 4, la artropata aguda por cristales de grado, de los que el 10% desarrollan la enfermedad (respuesta 2).
pirofosfato clcico o pseudogota. A continuacin se describe en varias respuestas la asociacin que
algunos anticuerpos tienen con distintas manifestaciones clnicas de

Comentarios TEST
Pregunta 26.- R: 1 la enfermedad. Por ejemplo, en la respuesta 3 se recuerda que los
El lupus eritematoso sistmico es el prototipo de enfermedad auto- anticuerpos anti-DNA tienen relacin con la aparicin de nefritis
inmune caracterizada por la presencia de clnica inflamatoria en ml- lpica, al demostrarse su depsito formando parte de inmunocom-
tiples rganos y sistemas. Para poder contestar correctamente a esta plejos en los glomrulos afectados. Su aparicin est relacionada con
pregunta debemos tener en cuenta la clasificacin de las manifesta- el HLA-DR2 y DR3. La presencia de anticuerpos anti-Ro tiene una
ciones de la enfermedad y la frecuencia de su aparicin: clara relacin con la aparicin de diversas manifestaciones clnicas
Manifestaciones sistmicas (95%) como fatiga, fiebre, anorexia. como el lupus cutneo subagudo. Sin embargo los anticuerpos anti-
Musculoesquelticas (95%) como artralgias, artritis (60%, respuesta Ro se relacionan con el HLA-DR3, y no DR-4, como aparece en la
correcta 1), mialgias. respuesta 4, por lo que esta es la respuesta que debemos elegir.
Cutneas (80%) como erupcin malar, fotosensibilidad, lceras En la respuesta 5 se recuerda que el anticoagulante lpico se relacio-
orales, erupcin discoide. na con DR4 y DR7. El anticoagulante se define por la presencia de un
Hematolgicas (85%): anemia de trastornos crnicos, leucopenia, alargamiento patolgico del tiempo de tromboplastina parcial activado
linfopenia o trombopenia. La presencia de anemia hemoltica es (TTPA), no corregible con la administracin de plasma fresco, y se debe
poco habitual (10%, respuesta 2). a la presencia de anticuerpos antifosfolpido en sangre.
Neurolgicas (60%): disfuncin cognitiva (50%, respuesta 3), sn-
dromes orgnicos cerebrales. Pregunta 29.- R: 1
Cardiopulmonares (60%): pleuritis (50%) y pericarditis (30%; res- El alargamiento del tiempo de tromboplastina parcial activado
puesta 4). (TTPA) es un fenmeno conocido desde hace muchos aos en los
Renales (50%): proteinuria, cilindros celulares. La forma ms fre- pacientes con lupus eritematoso sistmico y denominado anticoagu-
cuente de afectacin renal activa es la glomerulonefritis proliferati- lante lpico. En los ltimos aos esta alteracin coagulomtrica no
va difusa ms que la focal (respuesta 5). corregible con la administracin de plasma fresco se sabe que es
Gastrointestinales (45%) como dolor, nuseas, alteraciones de las debida a la presencia de anticuerpos antifosfolpido. Estos pacientes
enzimas hepticas. muestran con frecuencia falsa positividad en el test de sfilis (VDRL), al

M exico A rgentina CTO Medicina C/ Nez de Balboa, 115 28006 MADRID (Espaa) Tfno.: (91) 782 43 32 / Fax: (91) 782 43 27
Pg. 6 RM C hile U ruguay E-mail: secretaria@ctomedicina.com; iberocto@ctomedicina.com WEB: www.ctomedicina.com; www.iberocto.com
REUMATOLOGA
Preparacin Examen de Seleccin 05/06 1 Vuelta Seguimiento a distancia
estar formado el sustrato utilizado para esta prueba en gran parte por Cada variable se punta de 0 a 3 (ausente, leve, moderada, grave).
fosfolpidos y producirse una reaccin cruzada al tener el paciente en El valor de las variables necrosis fibrinoide y semilunas celulares se
suero anticuerpos antifosfolpidos (respuesta 4). La determinacin de multiplica por dos.
anticuerpos antifosfolpido se realiza actualmente mediante ELISA di- Como puedes comprobar, la nica lesin aguda (reversible) es la
seado para la determinacin del fosfolpido ms habitual, que es la respuesta 1; las dems corresponden a lesiones crnicas (irreversibles).
cardiolipina (anticuerpos anticardiolipina). Estos anticuerpos pueden
ser de tipo IgG o IgM. Adems se han ido conociendo una serie de Pregunta 31.- R: 2
manifestaciones clnicas asociadas a la presencia de estos anticuer- En esta pregunta se repasan una serie de conceptos importantes del
pos, como son: LES y el embarazo. Hay que recordar que la enfermedad aparece
Morbilidad en el embarazo: abortos, muertes fetales y partos pre- mayoritariamente en mujeres en edad frtil, por lo que es necesario
maturos. conocer las interacciones de la enfermedad con la gestacin.
Trombosis: arteriales, venosas y de pequeo vaso (respuesta 3). La amenorrea es frecuente (respuesta 4), aunque la tasa de fertili-
Trombocitopenia perifrica (respuesta 2). dad en las pacientes con LES es normal. Existe una frecuencia elevada
Lesiones valvulares (endocarditis de Libman-Sachs). de aborto espontneo y muerte intratero (10-30%), especialmente
Livedo reticularis (respuesta 5). en las que tienen anticuerpos antifosfolpido. El tratamiento de elec-
cin de las mujeres con abortos previos y anticuerpos antifosfolpido
Como podemos comprobar, la nica manifestacin no relaciona- es la administracin de heparina de bajo peso molecular.
da es la respuesta 1, la pericarditis. El embarazo produce efectos variables sobre el LES. En ocasiones
la enfermedad se activa, sobre todo durante las 6 semanas posteriores
Pregunta 30.- R: 1 al parto (respuesta 5). Si no se observa enfermedad renal ni cardiaca
La glomerulonefritis se debe a la presencia de depsitos de inmu- grave y la actividad del LES se encuentra bajo control, la mayora de
nocomplejos circulantes o por la formacin de inmunocomplejos in las mujeres tiene embarazos a trmino con recin nacidos normales.
situ en el mesangio y en la membrana basal del glomrulo. La clasifi- Los glucocorticoides (excepto los de vida media larga, dexameta-
cacin de las glomerulonefritis en el LES comprende las siguientes sona y betametasona) son inactivados por enzimas placentarias y no
formas: producen anomalas fetales (respuesta 2). Los inmunosupresores y
I. Glomrulo normal: los antipaldicos no pueden utilizarse durante el embarazo (res-
a) Normal (por todas las tcnicas). puesta 1).
b) Normal por microscopa ptica, pero con depsitos en micros- Queda por lo tanto la respuesta nmero 3 como correcta. Ante la
copa electrnica o inmunofluorescencia. relacin patognica de los estrgenos en el desarrollo del LES se des-
II. Alteraciones mesangiales puras: aconseja su utilizacin como mtodo anticonceptivo, siendo de elec-
a) Ensanchamiento mesangial y/o leve hipercelularidad. cin los mtodos de barrera (preservativos, diafragma).
b) Moderada hipercelularidad.
III. Glomerulonefritis proliferativa focal o segmentaria (asociada a alte- Pregunta 32.- R: 3
raciones mesangiales leves o moderadas): En esta pregunta se repasan algunas de las manifestaciones hema-
a) Lesiones activas necrotizantes. tolgicas que aparecen en el LES.
b) Lesiones activas y esclerosantes. La primera afirmacin es falsa, ya que la causa ms frecuente de
c) Lesiones esclerosantes. anemia es la de trastornos crnicos, siendo sin embargo la anemia
IV. Glomerulonefritis proliferativa difusa: hemoltica menos habitual, pero ms caracterstica (criterio diag-
a) Sin lesiones segmentarias. nstico).
b) Con lesiones activas necrotizantes. La esplenectoma es una medida necesaria en ocasiones cuando
c) Con lesiones activas y esclerosantes. se produce una citopenia (sobre todo trombocitopenia) que no res-
d) Con lesiones esclerosantes. ponde al tratamiento habitual con corticoides en dosis altas. Otras
V. Glomerulonefritis membranosa: medidas que pueden emplearse en caso de trombocitopenia grave
a) Membranosa pura. son la administracin de gammaglobulina intravenosa, los citotxicos
b) Asociada a lesiones de categora II. o el danazol. Sin embargo, la mayora de los pacientes tienen
c) Asociada a lesiones de categora III. trombocitopenia leve (respuesta correcta 3).
Comentarios TEST

d) Asociada a lesiones de categora IV. La leucopenia es otro hallazgo habitual en la enfermedad, pero
VI. Glomerulonefritis esclerosante avanzada. casi nunca se asocia a infecciones recidivantes y no precisa tratamien-
A su vez, se han establecido unos ndices histolgicos de actividad to (respuesta 4).
y de cronicidad: Finalmente se observa que el 50% de los pacientes presentan test
de Coombs directo positivo de tipo complemento (no Ig), que no es
Pregunta 30. Tipos de lesiones en el LES. indicativo de hemlisis (respuesta 5). La anemia hemoltica aparece
en el 10% de los pacientes y est mediada por anticuerpos IgG, que
dan una prueba de Coombs directa positiva para Ig y complemento.
12345673678594
4383 12345673675 2454383 La prueba definitiva es la deteccin de los anticuerpos en el fluido de
4296
8 7 4296
8 7 las clulas sanguneas.

1234567839
6
85593 Pregunta 33.- R: 5
18
346 7636 468 El LES puede afectar a cualquier localizacin neurolgica, inclu-
1865 9
855938 yendo encfalo, mdula, sistema nervioso perifrico y pares cranea-
64267 6  8 6
1344 1
58346 5483593 les. La disfuncin cognitiva leve es la manifestacin mas frecuente. En
6956 499 8 1865 9 76349 esta pregunta se repasan varias de estas manifestaciones, as como
95938 algunas de las relaciones patognicas que se conocen.
1 765394 8 La respuesta 1, la presencia de anticuerpos antineuronales, se des-
58
4
64 cribe hasta en el 60% de los casos, situndose el antgeno en la super-
ficie neuronal y linfocitaria. En algunos estudios, los ttulos elevados
de IgG se relacionan con la afectacin difusa del SNC.
6426 1 765394 8
559 16346 6 836
695 Tambin se han descrito los anticuerpos antimielina que pueden
9 4296 94548 6 4 4 
58938 134769 593 producir dao en la conduccin nerviosa a nivel perifrico (res-
puesta 2).

M exico A rgentina CTO Medicina C/ Nez de Balboa, 115 28006 MADRID (Espaa) Tfno.: (91) 782 43 32 / Fax: (91) 782 43 27
C hile U ruguay E-mail: secretaria@ctomedicina.com; iberocto@ctomedicina.com WEB: www.ctomedicina.com; www.iberocto.com
RM Pg. 7
REUMATOLOGA
Seguimiento a distancia Preparacin Examen de Seleccin 05/06 1 Vuelta
La polineuropata sensitiva o mixta es la manifestacin ms fre- El resto de las alternativas planteadas son la utilizacin de corticoi-
cuente de afectacin del sistema nervioso perifrico (10%) (respuesta des en dosis elevadas e inmunosupresores o la combinacin de am-
3), siendo menos habitual la mononeuritis mltiple o la polineuropa- bos frmacos. Este tipo de tratamiento tiene sentido si las manifestacio-
ta motora en relacin a vasculitis. Los pares craneales afectados mas nes del paciente fueran severas (por ejemplo glomerulonefritis prolife-
frecuentemente son los motores oculares, el nervio ptico (produ- rativa difusa, afectacin del sistema nervioso central, neumonitis lpi-
ciendo oftalmoplejias, ptosis, escotomas, alucinaciones visuales o ce- ca, citopenia severa como anemia hemoltica o trombocitopenia).
guera central) y el trigmino, respetando habitualmente la rama moto- Recientemente se ha incluido dentro del uso de inmunosupreso-
ra (respuesta correcta, la 5). res la administracin de un nuevo frmaco denominado micofeno-
La mielitis trasversa se asocia con frecuencia a la presencia de anti- lato, en dosis de 1 a 2,5 gr/da en varias dosis. La azatioprina se utiliza
cuerpos antifosfolpidos (respuesta 4), por mecanismo isqumico. en dosis de 2-3 mg/kg/da y la ciclofosfamida en bolos mensuales con
dosis de 10 a 15 mg/kg, reduciendo de esta forma su habitual toxici-
Pregunta 34.- R: 5 dad vesical cuando se administra por va oral.
El LES es una enfermedad autoinmune donde se ha descrito la
presencia de mltiples anticuerpos antinucleares. Debes tener en cuen- Pregunta 37.- R: 3
ta que, aunque algunos antgenos no se encuentran en el ncleo, sino En esta paciente se plantean algunas lesiones cutneas que apare-
en el citoplasma, de forma genrica a todos estos anticuerpos se les cen en el lupus eritematoso sistmico y la actitud teraputica que
denomina antinucleares (aunque, como ves, algunos son debemos tomar. De forma esquemtica, lo que podemos recordar es
anticitoplasmticos). Por este motivo, la respuesta correcta es la 5. Los lo siguiente:
AAN se encuentran en el 98% de los pacientes con LES y su presencia Lesiones cutneas en el contexto de manifestaciones sistmicas
se demuestra enfrentando el suero del paciente a clulas de cultivo (anemia, nefropata, serositis): suelen mejorar con el tratamiento
(clulas Hep 2) y detectados mediante inmunofluorescencia. Se con- que se establece para estas ltimas, generalmente corticoides sist-
sideran positivos a titulaciones por encima de 1:40. micos.
Los anticuerpos anti-DNAds (de doble cadena) son relativamente Lesiones cutneas aisladas especficas (lupus discoide): aplicacin
especficos del LES, apareciendo en el 70% de los casos (respuesta 4). de corticoides tpicos (triamcinolona o fluocinolona) dos o tres
Son todava ms especficos los anti-Sm, pero estos se observan solo veces al da. En la cara es aconsejable utilizar preparados de hidro-
en el 30% de los enfermos. cortisona, a fin de evitar las lesiones atrficas residuales, el acn o
Los anticuerpos antihistona aparecen con ms frecuencia en el lu- las telangiectasias. Si la respuesta a las dos semanas es insuficiente,
pus inducido por frmacos (95%) que en el LES (30-70%) (respuesta 3). se asociarn antimalricos (respuesta correcta nmero 3). En caso
Los anticuerpos anti-RNP aparecen en mltiples enfermedades au- de mala respuesta a las pautas anteriores, se puede aadir
toinmunes como polimiositis, esclerodermia, enfermedad mixta del prednisona en dosis de 0,5 mg/kg/da durante 15 das y despus
tejido conectivo y LES (40%) (respuesta 2). Cuando aparecen en el LES reducir progresivamente la dosis.
sin anticuerpos anti-DNA, el riesgo de nefritis es bajo. Lupus cutneo subagudo: en este caso, el tratamiento es con antima-
Por ltimo, los anticuerpos antifosfolpido descritos en la respuesta lrico junto a corticoide tpico. Si a las 4 semanas no hay eviden-
1 se observan en el 20-50% de los casos. cia de mejora, aadir corticoide sistmico (prednisona 0,5 mg/kg/
da). Una alternativa en caso de mala respuesta a la pauta anterior
Pregunta 35.- R: 2 es la talidomida en dosis de 25-50 mg/da, si bien es un frmaco no
El lupus inducido por frmacos es un sndrome similar al LES comercializado en nuestro pas (su indicacin es de uso compasi-
inducido por la toma de distintos medicamentos como la procaina- vo) y sus efectos secundarios ms importantes son la teratogenici-
mida, hidralazina, isoniazida, clorpromazina, D-penicilamina, me- dad y la neuropata perifrica.
tildopa, quinidina o interfern alfa, y posiblemente por hidantonas,
etosuximida y anticonceptivos orales. Este sndrome ocurre con poca El caso planteado es el de un lupus cutneo discoide demostrado
frecuencia entre los frmacos citados, excepto con la procainamida, por la biopsia cutnea, por lo que al no haber dentro de las respuestas
el medicamento ms frecuente, y la hidralazina. Existe una predis- la posibilidad de utilizar corticoides tpicos debemos elegir la
posicin gentica en los sujetos acetiladores lentos hepticos. Los hidroxicloroquina.
sntomas ms frecuentes son los generales (astenia, febrcula) y las
artralgias. La pleuropericarditis aparece en el 25-50% (repuesta co- Pregunta 38.- R: 1
rrecta 2). Los sntomas severos que podemos observar en el LES son En este caso nos preguntan por la actitud teraputica en una pa-

Comentarios TEST
aqu raros, como la afectacin del SNC (respuesta 3) o la renal (res- ciente con lupus eritematoso sistmico con clnica preferente a nivel
puesta 5). del sistema nervioso central, habindose realizado la tcnica de ima-
La aparicin de los sntomas no es inmediata desde el inicio de la gen ms til para su diagnstico, la RNM. En este caso nos muestra
toma del frmaco, pasando habitualmente varios meses hasta que se reas de captacin de alta intensidad compatible con una zona infla-
detecta la aparicin de los anticuerpos antinucleares y el inicio de la matoria mas evidente a nivel parietotemporal. En trminos generales
enfermedad (respuesta 1). las manifestaciones neuropsiquitricas se dividen en dos grupos a la
En la mayora de los pacientes se detectan anticuerpos antinuclea- hora de enfocar el tratamiento:
res tipo anti-histona, pero no se encuentran los anticuerpos especfi- Manifestaciones difusas, como psicosis, depresin profunda, sn-
cos del lupus eritematoso sistmico, como son los anti-DNAds o los drome orgnico cerebral, convulsiones, meningitis, etc, probable-
anti-Sm (respuesta 4). mente mediadas por anticuerpos especficos (antineuronales,
Finalmente el abordaje teraputico en esta enfermedad consiste en antiprotena P ribosmica), inmunocomplejos y/o fenmenos de
la retirada de la medicacin responsable, con lo que la mayora mejora vasculitis: el tratamiento debe individualizarse segn el tipo y grave-
en pocas semanas. Si los sntomas son graves, est indicada la adminis- dad del cuadro clnico. En general, adems de intentar controlar
tracin de corticoides durante poco tiempo (de 2 a 10 semanas). la actividad lpica con dosis altas de corticoides (1-2 mg/kg/da)
(respuesta correcta 1), se deben utilizar frmacos sintomticos como
Pregunta 36.- R: 4 anticonvulsivantes o antipsicticos.
En esta pregunta se plantea el caso de una paciente con LES con los Accidentes isqumicos cerebrales, bien por trombosis secundaria
siguientes sntomas: manifestaciones generales como astenia y febr- a sndrome antifosfolpido, por vasculitis o por combinacin de
cula, artritis y serositis leve, y te plantea distintas posibilidades terapu- ambos factores: el tratamiento fundamental es la anticoagulacin.
ticas. Debes tener en cuenta que se trata de una paciente con snto- Si adems coexisten signos de actividad inflamatoria, aadiremos
mas leves de la enfermedad, por lo que la intensidad del tratamiento corticoides, asociados o no a inmunosupresores.
debe ir en relacin con la gravedad y la respuesta lgica es la utiliza-
cin de AINEs (respuesta correcta nmero 4), y en caso de no respon- En esta paciente hay que incorporar corticoides orales a su trata-
der, utilizar corticoides en dosis bajas. miento, como indica la respuesta correcta. La administracin de car-

M exico A rgentina CTO Medicina C/ Nez de Balboa, 115 28006 MADRID (Espaa) Tfno.: (91) 782 43 32 / Fax: (91) 782 43 27
Pg. 8 RM C hile U ruguay E-mail: secretaria@ctomedicina.com; iberocto@ctomedicina.com WEB: www.ctomedicina.com; www.iberocto.com
REUMATOLOGA
Preparacin Examen de Seleccin 05/06 1 Vuelta Seguimiento a distancia
bamacepina (respuesta 4) tendra sentido si presentara crisis convulsi- La aparicin del factor reumatoide, aunque tpica de la artritis
vas, y la administracin de antidepresivos si presentara depresin, reumatoide, no es exclusiva de esta enfermedad y se ha comprobado
asociados, como comentamos en el punto 1 a la administracin de su presencia en mltiples situaciones como: sndrome de Sjgren y
corticoides. otras enfermedades del tejido conectivo, cirrosis biliar primaria, infec-
ciones, neoplasias, enfermedades pulmonares y otras enfermedades
Pregunta 39.- R: 4 inflamatorias agudas o crnicas. Incluso se demuestra su presencia
De las manifestaciones hematolgicas en el LES, solo requieren trata- despus de vacunaciones o transfusiones (repuesta 5).
miento especfico la anemia hemoltica clnicamente significativa y la Se acepta que entre el 1 y el 8% de los individuos sanos presentan
trombopenia grave. La leucopenia slo precisa tratamiento con predni- factor reumatoide IgM, aunque la frecuencia es mayor por encima de
sona en el caso excepcional de neutropenia autoinmune grave. los 60 aos (respuesta correcta: 2, ya que no disminuye su presencia,
El tratamiento inicial de la anemia hemoltica y la trombopenia grave sino que aumenta con la edad).
es la prednisona en dosis de 1-2 mg/kg/da (respuesta 1). En el caso de la Los pacientes con artritis reumatoide que presentan concentracio-
anemia, si no se controla a las 3 semanas de tratamiento, debe conside- nes importantes de factor reumatoide IgM tienen mayor riesgo de
rarse la esplenectoma o el tratamiento con inmunosupresores. En el presentar manifestaciones extraarticulares, en particular ndulos reu-
caso de la trombopenia, si el paciente presenta hemorragias se debe matoides y vasculitis (respuesta 1), y una enfermedad erosiva grave.
considerar la administracin de gammaglobulina intravenosa (400 mg/
kg/da durante 5 das), que permite una elevacin rpida de plaquetas Pregunta 42.- R: 1
(respuesta 2), aunque suele ser transitoria. Si el control con la predniso- La evolucin de la artritis reumatoide es variable. La mayora
na no es bueno, como alternativa a la esplenectoma pueden utilizarse tienen una actividad mantenida, fluctuante con un grado variable
bolos mensuales de ciclofosfamida (750-1000 mg/m2 de superficie cor- de deformidad articular. Las remisiones son ms probables durante
poral) durante 6 meses (respuesta 3), y si no hay respuesta, pueden el primer ao. La progresin de la enfermedad es mayor durante los
ensayarse antes de la esplenectoma pulsos de dexametasona, eficaces 6 primeros aos y luego tiende a remitir. Tienen peor pronstico los
en la prpura trombtica idioptica. pacientes con ttulos altos de factor reumatoide, elevacin de reac-
Tras la esplenectoma (respuesta 5) hay pacientes que no se contro- tantes de fase aguda como VSG , PCR y haptoglobina, ndulos sub-
lan con una dosis razonable de corticoides. En ellos se puede intentar cutneos, erosiones radiolgicas en la primera valoracin, presen-
vincristina, danazol, gammaglobulina intravenosa o plasmafresis. cia de ms de 20 articulaciones afectadas o nivel socioeconmico
Como puedes comprobar, dentro del arsenal teraputico tiene bajo.
cabida cualquiera de los frmacos sealados excepto la D-penicila- Con lo que acabamos de comentar puedes estar seguro de que las
mina (respuesta 4). Este frmaco lo utilizamos en la artritis reumatoide respuestas 2 (ttulos elevados de factor reumatoide), 3 (ndulos subcu-
en la segunda lnea teraputica y tambin como antifibrtico en la tneos), 4 (elevacin de VSG) son todos ellos parmetros de mal pro-
esclerosis sistmica. nstico en la enfermedad.
La respuesta nmero 5, el sndrome de Felty, es una situacin clni-
Pregunta 40.- R: 1 ca dentro de la artritis reumatoide que se acompaa de esplenomega-
En esta pregunta debes recordar los criterios diagnsticos de la artri- lia y neutropenia. Estos pacientes suelen tener ttulos elevados de fac-
tis reumatoide para encontrar la respuesta adecuada. Ten presente tor reumatoide, que ya hemos visto que por s solo es un dato de mal
que, de los 7 criterios utilizados para el diagnstico de esta enferme- pronstico, pero adems tienen por la neutropenia tendencia a las
dad, el paciente debe presentar al menos 4. Los criterios son: infecciones de repeticin, por lo que el pronstico se agrava.
1) Rigidez matutina: rigidez en y alrededor de las articulaciones que La respuesta correcta es la afectacin de manos. Este hallazgo es
dura una hora antes de que se produzca la mejora funcional prcticamente constante en la artritis reumatoide, y por lo tanto no
mxima. puede discriminar a los pacientes con peor pronstico.
2) Artritis de 3 o ms reas articulares: al menos tres reas observadas
de forma simultnea por el mdico, con tumefaccin de partes Pregunta 43.- R: 3
blandas o derrame articular. Las 14 reas que se pueden afectar de En este caso clnico se plantea la aparicin de un cuadro de mono-
forma bilateral son IFP, MCF, carpo, codo, rodilla, tobillo y MTF. artritis aguda en una paciente con artritis reumatoide como enferme-
3) Artritis de las articulaciones de las manos: incluyendo carpo, MCF dad de base. Siempre que se plantean preguntas en las que la sinto-
e IFP. matologa es la inflamacin monoarticular aguda, la exploracin cla-
Comentarios TEST

4) Artritis simtrica: afectacin simultnea de las mismas reas articu- ve para orientar el diagnstico es la realizacin de la artrocentesis y el
lares en ambos lados del cuerpo. examen del lquido sinovial. En el caso que nos ocupa es lo que se ha
5) Ndulos reumatoides: ndulos subcutneos sobre las prominen- realizado, encontrando un lquido de aspecto purulento (con ms de
cias seas, superficies extensoras o regiones yuxtaarticulares, ob- 50.000 clulas/mm3 y marcado descenso de la glucosa) caracterstico
servados por un mdico. de las infecciones articulares agudas. Respecto a la pregunta que se
6) Factor reumatoide srico: demostrado por cualquier mtodo en realiza, debes realizar el siguiente razonamiento:
que el resultado haya sido positivo en menos de un 5% de la El Staphylococcus epidermidis es el germen que aparece relaciona-
poblacin sana. do con mas frecuencia en las infecciones de prtesis articulares, y
7) Alteraciones radiolgicas: en radiografa de carpos y manos, con despus del aureus en las infecciones articulares secundarias a ino-
erosiones u osteopenia en banda. culacin directa intraaarticular (p.e. artrocentesis).
Como puedes comprobar, la presencia de lceras orales (respuesta Las infecciones por Streptococcus se suelen observan con ms fre-
1, correcta) no se incluye dentro de los criterios diagnsticos en la cuencia en nios, por debajo de los 5 aos.
artritis reumatoide. Ten presente que s son criterio en el LES y en la El Staphylococcus aureus se considera, de forma global, el germen
enfermedad de Behet. que con ms frecuencia produce artritis sptica. Su prevalencia a
partir de los 40 aos es mayor que ningn otro germen. Por este
Pregunta 41.- R: 2 motivo debes considerar esta la respuesta correcta. Hay que recor-
Esta pregunta repasa algunos de los conceptos ms importantes en dar que la artritis reumatoide, por la destruccin articular que pro-
relacin al factor reumatoide. duce, predispone a las infecciones articulares, siendo el aureus el
En primer lugar podemos recordar que el factor reumatoide es una microorganismo ms habitualmente hallado.
inmunoglobulina (habitualmente IgM, pero puede ser de cualquier La infeccin gonoccica ocurre con ms frecuencia en adultos
tipo) dirigida contra la Fc de una IgG. De esta forma podemos compro- jvenes (entre los 15-40 aos), con contactos sexuales mltiples sin
bar cmo las respuestas 3 y 4 son dos afirmaciones correctas. El factor proteccin (preservativo).
reumatoide IgM es detectado por tcnicas de aglutinacin (aglutina- La infeccin tuberculosa produce una artritis lentamente progresi-
cin en ltex: partculas de ltex recubiertas por IgG humana; Waaler- va monoarticular, con lquido sinovial con menor nmero de c-
Rose: hemates de carnero recubiertos por IgG de conejo). lulas que el presentado, unas 20.000 clulas/mm3.

M exico A rgentina CTO Medicina C/ Nez de Balboa, 115 28006 MADRID (Espaa) Tfno.: (91) 782 43 32 / Fax: (91) 782 43 27
C hile U ruguay E-mail: secretaria@ctomedicina.com; iberocto@ctomedicina.com WEB: www.ctomedicina.com; www.iberocto.com
RM Pg. 9
REUMATOLOGA
Seguimiento a distancia Preparacin Examen de Seleccin 05/06 1 Vuelta
Pregunta 44.- R: 2 pender el frmaco. En el caso que discutimos presenta de forma
La artritis reumatoide es una enfermedad inflamatoria articular aislada proteinuria, sin otras alteraciones de la funcin renal, por
que produce afectacin preferente de pequeas articulaciones de lo que esta posibilidad es poco probable.
manos y pies (excepto las interfalngicas distales), aunque se puede La trombosis de la arteria renal ocurre en pacientes con alguna
observar inflamacin en cualquier articulacin sinovial. Respecto a situacin precipitante como traumatismos, compresin extrnseca,
la afectacin del esqueleto axial (columna vertebral y articulaciones deshidratacin, sndrome nefrtico, embarazo o toma de anticon-
sacroiliacas), la enfermedad produce clnica en la columna cervical ceptivos. En los casos agudos se produce prdida repentina de la
en el 40% de los casos, siendo poco habitual la afectacin de la funcin renal, acompaada de fiebre y dolor lumbar. Como ves,
columna dorsal, lumbar o de las articulaciones sacroiliacas. nuestra paciente ni tiene factores precipitantes ni clnica compati-
Dentro de la columna cervical la manifestacin ms caracterstica ble con este diagnstico.
de la enfermedad es la subluxacin atloaxoidea, definida por un La amiloidosis AA complica un 10-20% de los casos con artritis
aumento de distancia patolgico entre la superficie anterior de la reumatoide, pero produce manifestaciones clnicas en el 3-10%
apfisis odontoides de la segunda vrtebra y el atlas (primera vrtebra). de los casos, en forma inicial como sndrome nefrtico y posterior-
Cuando esta distancia supera los 4 milmetros, definimos esta altera- mente como insuficiencia renal. La amiloidosis es ms comn en
cin. Para poder ser visualizada se realiza una radiologa lateral en enfermos con artritis reumatoide de larga evolucin (>10 aos),
flexin del cuello. Esta alteracin radiolgica puede ser asintomtica, con factor reumatoide positivo y artropata destructiva. Como pue-
producir dolor en la zona del occipucio o alteraciones neurolgicas. des comprobar, esta es la respuesta correcta.
En el caso de que produzca manifestaciones leves se puede colocar La vasculitis reumatoide puede afectar a cualquier rgano. En la
collarn cervical durante los periodos sintomticos. Si produce clnica forma ms agresiva (vasculitis necrosante) puede producir poli-
neurolgica, es necesaria la realizacin de reparacin neuroquirrgi- neuropata o mononeuritis mltiple, afectacin cutnea (necro-
ca (fijacin de la columna cervical al crneo). sis y ulceracin) o visceral (infarto visceral). La vasculitis renal es
El resto de las respuestas que aparecen en la pregunta son tambin rara.
manifestaciones que podemos encontrar en la columna cervical en la
artritis reumatoide. Ten en cuenta que las articulaciones interapofisa-
rias son sinoviales y que se pueden inflamar y tener erosiones como
las articulaciones perifricas.

Pregunta 45.- R: 1
La persistencia del proceso inflamatorio en la artritis reumatoide
favorece la aparicin de destruccin articular y deformidades, siendo
la mano una de las localizaciones habituales. En ella se describen las
siguientes deformidades:
Dedo en cuello de cisne: hiperextensin de las interfalngicas proxi-
males con flexin de las interfalngicas distales (respuesta correcta 1).
Dedo en ojal: presentan una alteracin inversa a la descrita en el
cuello de cisne, flexin de interfalngica proximal con extensin
de interfalngica distal (respuesta 2).
Dedo en martillo: flexin de interfalngica distal (respuesta 4).
Deformidad del primer dedo en Z: hiperextensin de la metacar-
pofalngica con flexin de la interfalngica.
Desviacin en rfaga cubital: desviacin de los dedos desde las
articulaciones interfalngicas proximales hacia el lado cubital del
carpo.

A nivel de los pies tambin podemos encontrar distintas deformi-


dades. La lesin mas caracterstica es el hundimiento del antepi,
pero tambin se observa ensanchamiento del metatarso, hallux val-

Comentarios TEST
gus, subluxacin plantar de la cabeza de los metatarsianos, dedos en
martillo con desviacin lateral, de forma que el primero se sita por
debajo del segundo.

Pregunta 46.- R: 4
En esta pregunta se plantean varios mecanismos de dao renal que
pueden aparecer en un paciente con artritis reumatoide.
La primera respuesta habla de la glomerulonefritis membranosa Pregunta 45. Deformacin articular en la artritis reumatoide.
que puede aparecer por la toma de sales de oro. La administra-
cin de este frmaco produce proteinuria en el 5-25% de los Pregunta 47.- R: 3
pacientes. La proteinuria aparece a los 4-6 meses de tratamiento La afectacin ocular ms frecuente en la artritis reumatoide es la
y hasta el 33% desarrollan un sndrome nefrtico florido. La pro- queratoconjuntivitis seca que aparece en el 20% de los pacientes
teinuria cede cuando se suspende el frmaco. Esta posibilidad la (respuesta correcta 3). Debes recordar que la causa secundaria ms
debes descartar, ya que la paciente actualmente no recibe este frecuente de sndrome de Sjgren secundario es la artritis reumatoi-
tratamiento. de.
Los AINEs producen distintos efectos sobre el rin, entre ellos Con mucha menor frecuencia aparece otra manifestacin ocular
insuficiencia renal aguda de causa hemodinmica, retencin de que es la epiescleritis o escleritis, que aparecen en menos del 1% de los
sal y agua, hiponatremia, hiperpotasemia, necrosis papilar, nefritis casos. Suele tratarse de pacientes con artritis reumatoide de larga evolu-
intersticial, sndrome nefrtico. El sndrome nefrtico y la insufi- cin, con ndulos reumatoides. La epiescleritis suele ser leve y transito-
ciencia renal coexisten a menudo por la combinacin de nefritis ria, mientras que la escleritis afecta a capas ms profundas del ojo y
intersticial y lesin glomerular de cambios mnimos. Esta entidad constituye un proceso inflamatorio ms grave. Desde el punto de vista
afecta principalmente a los pacientes que toman derivados del anatomopatolgico, la lesin es similar a un ndulo reumatoide y pue-
cido propinico, ibuprofeno y naproxeno, pero puede ocurrir de causar adelgazamiento y perforacin ocular (escleromalacia
con otros AINEs. La enfermedad renal remite, en general, al sus- perforante).

M exico A rgentina CTO Medicina C/ Nez de Balboa, 115 28006 MADRID (Espaa) Tfno.: (91) 782 43 32 / Fax: (91) 782 43 27
Pg. 10 RM C hile U ruguay E-mail: secretaria@ctomedicina.com; iberocto@ctomedicina.com WEB: www.ctomedicina.com; www.iberocto.com
REUMATOLOGA
Preparacin Examen de Seleccin 05/06 1 Vuelta Seguimiento a distancia
Por ltimo, debes recordar que en la artritis reumatoide, a diferen- y la artritis de articulaciones de las manos. La aparicin de clnica
cia de las espondiloartropatas, no aparece uvetis anterior. La uvetis neurolgica perifrica en forma de polineuropata o mononeuritis
posterior, propia de otras enfermedades como el Behet, tampoco mltiple y lceras cutneas es sugestiva del desarrollo dentro de la
aparece. AR de una vasculitis reumatoide.
Desde el punto de vista teraputico, si el paciente presenta un El CH50 (complemento hemoltico medio) suele estar normal en la
sndrome seco, el tratamiento ser tpico (lgrimas artificiales). Si pre- AR en sangre perifrica, excepto cuando se produce una vasculitis
senta epiescleritis, se recomiendan los AINEs o corticoides tpicos, y si reumatoide donde desciende, al formarse inmunocomplejos cir-
se trata de escleritis necrosante, se administrarn corticoides sistmi- culantes que consumen complemento.
cos en dosis altas (1 mg/kg/da). En el caso de la escleromalacia La presencia de un sndrome de Felty se confirma cuando el pa-
perforante, puede ser necesaria la reparacin quirrgica. ciente tiene la trada clsica: AR, esplenomegalia y neutropenia. Al
no aparecer los tres datos en el paciente, este diagnstico se descar-
Pregunta 48.- R: 5 ta.
La afectacin pleural es la manifestacin pulmonar ms comn en La respuesta correcta es la 4. El caso planteado es caracterstico de
la artritis reumatoide. Se estima que un 20% de los pacientes tienen una vasculitis reumatoide de tipo necrotizante sistmica, por lo
dolor pleurtico, pero solo el 5% presentan derrame. Aparece con que la biopsia de un rgano afectado (piel o sistema nervioso peri-
ms frecuencia en varones y est asociado a la presencia de factor frico), mostrar los hallazgos tpicos histolgicos.
reumatoide. En la artritis reumatoide, la incidencia de linfomas parece algo
El lquido pleural de la artritis reumatoide es un exudado de aspec- superior a la esperada en la poblacin general. Sin embargo, los
to pajizo, verdoso o lechoso, segn su cronicidad. El recuento celular sntomas del paciente son compatibles con una vasculitis necroti-
es inferior a 5000 leucocitos/mm3, con predominio linfocitario. Las zante sistmica.
cifras de glucosa son bajas (respuesta correcta 5), habitualmente me-
nores a 30 mg/dl, el factor reumatoide IgM est elevado (ttulo superior Pregunta 51.- R: 3
a 1/320), la LDH est aumentada y el complemento descendido. La En esta pregunta es necesario recordar la actitud teraputica y
enzima adenosindeaminasa (ADA) est elevada. alguno de los efectos secundarios que pueden aparecer cuando trata-
El estudio histolgico de la pleura parietal muestra ndulos reuma- mos a un paciente con artritis reumatoide (AR).
toides y fibrosis, y en la pleura visceral se ven cambios inflamatorios La primera escala teraputica incluye la administracin de AINEs,
crnicos inespecficos. asociados o no a corticoides en dosis bajas por va oral. Sin embargo,
El patrn citolgico del lquido pleural consiste en clulas gigantes con este tratamiento el proceso inflamatorio se controla en pocos
y un fondo de material necrtico granular. pacientes, por lo que es necesario incorporar un frmaco de segunda
El derrame pleural suele ser bilateral. Se pueden producir empie- lnea en la mayora de los casos. Dentro de este grupo se incluyen una
mas de inicio insidioso, como complicacin del derrame, atribuidos serie de frmacos clsicos (sales de oro, D-penicilamina, sulfasalazina
a diferentes mecanismos, como sobreinfeccin de derrames previos o y antipaldicos), inmunosupresores (metotrexate, ciclosporina),
pequeos neumotrax en pacientes con ndulos subpleurales. inmunomoduladores (leflunomida) y frmacos anti-TNF alfa
(infliximab, etanercep).
Pregunta 49.- R: 2 La administracin de estos frmacos no est exenta de efectos
En esta pregunta debes recordar los conceptos que se explican en secundarios y en la pregunta nos plantean el caso de un paciente
neumologa sobre patrn restrictivo y patrn obstructivo en el estudio que tras recibir sales de oro desarrolla un sndrome nefrtico. Este
espiromtrico. frmaco produce como toxicidad ms habitual lesiones mucocut-
La enfermedad intersticial aparece en la artritis reumatoide con neas, pero algunos pacientes desarrollan una glomerulonefritis mem-
mayor frecuencia en varones, observndose alteraciones radiolgicas branosa con sndrome nefrtico. Se ha comprobado que la apari-
compatibles en un 2% de los casos. Sin embargo, los estudios cin de esta toxicidad es ms habitual en los que presentan HLA-
necrpsicos muestran un porcentaje bastante superior (20%). DR3, como ocurre en nuestro paciente. La aparicin de toxicidad
Los sntomas comunes son la tos no productiva y la disnea progre- importante obliga a la suspensin del tratamiento y a plantearnos
siva, con estertores crepitantes bibasales, pudindose observar acro- otra alternativa dentro de los frmacos de segunda lnea. Sin embar-
paquias e incluso osteoartropata hipertrfica. go, no podemos optar dentro de ellos en este caso a la utilizacin de
El estudio radiolgico puede ser normal en fases iniciales, detec- D-penicilamina, ya que este frmaco tambin produce la misma
Comentarios TEST

tndose la neumopata por estudios funcionales, o presentar el tpico toxicidad renal que las sales de oro, por lo que la respuesta correcta
patrn de infiltrados reticulonodulares basales (respuesta 3) o pulmn es la 3.
en panal en fases avanzadas. El TAC torcico de alta resolucin pre-
senta una mayor sensibilidad que la radiologa convencional en la Pregunta 52.- R: 5
deteccin de enfermedad intersticial. La artritis crnica juvenil, tambin denominada artritis idioptica
Las pruebas de funcin respiratoria muestran un patrn restrictivo, juvenil, se define por la presencia de un cuadro de artritis de curso
por lo que la respuesta correcta es la 2, ya que en este patrn, a diferen- crnico (superior a las 6 semanas), en nios o jvenes menores de 16
cia del obstructivo, no desciende el ndice VEF1/CVF. En las fases iniciales aos, de etiologa autoinmune. Dentro de las respuestas que apare-
de la enfermedad se produce desaturacin y cianosis con el ejercicio cen en esta pregunta se recogen las distintas formas evolutivas de la
(respuestas 1 y 5). El test de difusin mostrar una alteracin intersticial enfermedad:
(respuesta 4), con alteracin de la difusin pulmonar. La forma sistmica aparece por igual en nios y nias, a una edad
El lavado bronquioalveolar es til para detectar la afectacin sub- muy precoz (por debajo de los 5 aos), presentando un cuadro
clnica, as como para cuantificar la actividad inflamatoria. sistmico de fiebre, rash, linfoadenopatas y hepatoesplenomega-
lia acompaando a la artritis.
Pregunta 50.- R: 4 La forma poliarticular se define por la afectacin de ms de 4
En este caso clnico se plantea el caso de un paciente varn con articulaciones, distinguindose la forma seropositiva (con factor
una poliartritis de curso crnico con rigidez matutina prolongada y en reumatoide positivo) y seronegativa (factor reumatoide negativo).
el que se desarrolla un cuadro de polineuropata y lesiones cutneas La forma seropositiva, igual que ocurre en los adultos, es ms grave
en forma de lceras dolorosas. Del anlisis de las respuestas plantea- desde el punto de vista articular y adems puede presentar diversas
das podemos decir: manifestaciones extraarticulares. Sin embargo no aparece uvetis,
La primera respuesta plantea la duda de que se trate de una artritis igual que no apareca uvetis en la artritis reumatoide del adulto. La
reumatoide (AR) por los sntomas no articulares aparecidos. Sin forma poliarticular seronegativa es menos agresiva que la seroposi-
embargo, el diagnstico de AR es muy probable, al cumplir varios tiva, no asocindose a la aparicin de manifestaciones sistmicas.
criterios para este diagnstico, como son la rigidez matutina de ms La forma oligoarticular de inicio tardo es la nica que predomina
de una hora de evolucin, la afectacin de 3 o ms articulaciones en varones y aparece por encima de los 10 aos. Est asociado a

M exico A rgentina CTO Medicina C/ Nez de Balboa, 115 28006 MADRID (Espaa) Tfno.: (91) 782 43 32 / Fax: (91) 782 43 27
C hile U ruguay E-mail: secretaria@ctomedicina.com; iberocto@ctomedicina.com WEB: www.ctomedicina.com; www.iberocto.com
RM Pg. 11
REUMATOLOGA
Seguimiento a distancia Preparacin Examen de Seleccin 05/06 1 Vuelta
HLA-B27 y pueden observarse episodios de uvetis anterior aguda, Las formas oligoarticulares son las de mejor pronstico y menor
y no uvetis crnica, como nos preguntan. agresividad en el dao articular, por lo que la primera escala de
La respuesta correcta es la forma oligoarticular de inicio precoz. tratamiento suele ser suficiente.
Esta es la forma de presentacin ms frecuente de artritis crni-
ca juvenil, y se asocia en la mayora de los casos a la presencia Pregunta 55.- R: 1
de anticuerpos antinucleares. En un 20% de los casos se desa- Las espondiloartropatas son un grupo de enfermedades inflama-
rrolla uvetis de curso crnico como complicacin de la enfer- torias donde se incluyen como entidades ms importantes la espondi-
medad. litis anquilosante, la artritis psorisica, la artritis reactiva y la artritis
asociada a enfermedad inflamatoria intestinal (colitis ulcerosa y enfer-
Pregunta 53.- R: 1 medad de Crohn). Las caractersticas de estas enfermedades se contra-
La presentacin de este caso clnico rene las caractersticas de ponen a las que aparecen en la artritis reumatoide y las podemos
una artritis crnica juvenil: se trata de un cuadro de inflamacin arti- resumir en la tabla a pie de pgina.
cular de curso crnico (superior a las 6 semanas) en el que se descarta Como podemos comprobar, la respuesta nmero 1 es la correcta,
en el cultivo del lquido sinovial la presencia de enfermedad bacteriana ya que la afectacin poliarticular simtrica es caracterstica de la artri-
articular. tis reumatoide y no de las espondiloartropatas, que suelen tener pre-
Dentro de la clasificacin de la artritis crnica juvenil correspon- sentacin oligoarticular asimtrica.
de a la forma de presentacin ms habitual de la enfermedad, la
oligoarticular de inicio precoz: presentacin antes de los 6 aos, Pregunta 56.- R: 5
afectacin de 4 o menos articulaciones (en este caso rodilla y tobi- Los datos caractersticos que presenta esta paciente son compati-
llo). Analizando las respuestas, debemos razonar de la siguiente for- bles con el diagnstico de espondilitis anquilosante (respuesta 5): do-
ma: lor y rigidez de predominio nocturno en la zona lumbar (es decir,
La respuesta 1 es la correcta, ya que en esta forma de presenta- dolor lumbar de caractersticas inflamatorias), radiologa de pelvis don-
cin al menos el 75% de los casos tienen anticuerpos antinucle- de se observa la presencia de erosiones en ambas articulaciones sa-
ares. Recuerda que en esta forma de enfermedad aparece en un croilacas y esclerosis del margen del hueso ilaco (inicialmente en la
20% de los casos uvetis crnica. Sin embargo, el pronstico arti- regin inferior de la articulacin) (sacroiletis grado II). Recuerda que
cular es el ms favorable de todas las formas de artritis crnica para el diagnstico de esta enfermedad nos basamos en la presencia
juvenil. de una serie de criterios diagnsticos:
Los anticuerpos anticardiolipina no aparecen asociados a la artritis Sacroiletis radiolgica y al menos la presencia de un criterio menor:
crnica juvenil.
Dolor inflamatorio en la columna lumbar.
La VSG estar elevada, como en cualquier proceso inflamatorio.
Limitacin en los tres planos en la movilidad de la columna lumbar.
La asociacin de esta forma de enfermedad es con HLA DR5 y
Limitacin de la expansin torcica.
DR8, y no con DR4, que es tpico de los pacientes con forma
poliarticular seropositiva.
El HLA-B27 se asocia a las formas oligoarticulares de inicio tardo. La paciente, como podemos comprobar, cumple el criterio mayor
y uno de los menores, por lo que podemos confirmar el diagnstico
de espondilitis anquilosante. Las otras opciones pueden descartarse
Pregunta 54.- R: 1
por los siguientes motivos:
La actitud teraputica en la artritis crnica juvenil pasa por la iden-
La artritis reumatoide no tiene sacroiletis.
tificacin de su forma de presentacin, ya que cada una de ellas
precisa distintas medidas farmacolgicas. De forma general, en cual- No existen antecedentes epidemiolgicos de artritis reactiva.
quiera de las presentaciones de la enfermedad la primera escala tera- No existe enfermedad cutnea compatible con psoriasis.
putica a ensayar es la administracin de aspirina en dosis altas o La artritis brucelar suele producir una sacroiletis unilateral y no
AINEs. A continuacin se recuerdan los frmacos ensayados segn la bilateral, como en este caso.
forma de presentacin:
En los pacientes con formas sistmicas, la primera escala teraputi- Pregunta 57.- R: 4
ca suele ser insuficiente para el control de la inflamacin articular En la espondilitis anquilosante se describe la presencia de mlti-
y de los sntomas generales, por lo que suele ser preciso aadir ples manifestaciones extraarticulares (ver tabla en la pgina siguien-
corticoides. Por lo tanto, la respuesta correcta es la 1. Dado que los te, donde se compara con la artritis reumatoide), entre las que se

Comentarios TEST
corticoides pueden producir retraso del crecimiento en los nios, encuentran varias de las respuestas de esta pregunta. Las manifesta-
cuando se administran de forma prolongada se debe valorar su ciones ms relevantes son:
combinacin o sustitucin por inmunosupresores. Uvetis anterior aguda (respuesta 3): es una de las manifestaciones
En los pacientes con formas poliarticulares debes plantear el tra- ms habituales. Suele presentarse de forma unilateral y se manifies-
tamiento igual que en la artritis reumatoide del adulto; si no hay ta por dolor, fotofobia, lagrimeo y visin borrosa. Los ataques de
buena respuesta con la primera escala teraputica debemos aa- uvetis no suelen durar ms de 2 3 meses y en la mayora de los
dir un frmaco de segunda lnea como sales de oro, D-penicila- casos curan sin dejar secuelas.
mina, hidroxicloroquina, sulfasalazina, metotrexate o ciclospori- Afectacin cardiovascular: inflamacin de la raz artica produ-
na A. Tambin se utilizan actualmente los frmacos anti-TNF. ciendo insuficiencia artica (respuesta 5) en el 10% de los pacien-

Pregunta 55. Diferencias entre las espondiloartropatas y la AR.

1234567849

439 92

7 729 476
123456789
2 62
8 264
9 6 12(
8 264
9 62&$)*'
123456789
2 627 45264
9 65 12+ 45264
9 652&#)*'
123 68
89729
 984
2
59 98
 12,9
 9 95259 98

12
896 9522
68
8972 6265466 2
9
 12+25
896 9522
68
89728
59268459-
2 62
2847
286-98

12
79 65
8976526 

 984
6528476524884 76
5 12
79 65
8976526 

 984
6527 45254.84 7652/649 95
222679 497
9
5297 65 97
65284
65 222-
5849 95257 62 62(6 26 8
123589
8972872 3!"#$ 123589
8972872 3!012
123 68
8972%95 98
28

8 65 98
267 659 952&97 

8972 6 123 68
8972%95 98
28

8 65 98
23/
774532597-9
2&97 

897
222
297568972 67 975
267262%465' 222 62
26.
7
2597-9
'

M exico A rgentina CTO Medicina C/ Nez de Balboa, 115 28006 MADRID (Espaa) Tfno.: (91) 782 43 32 / Fax: (91) 782 43 27
Pg. 12 RM C hile U ruguay E-mail: secretaria@ctomedicina.com; iberocto@ctomedicina.com WEB: www.ctomedicina.com; www.iberocto.com
REUMATOLOGA
Preparacin Examen de Seleccin 05/06 1 Vuelta Seguimiento a distancia
tes con ms de 30 aos de evolucin de la enfermedad. Tambin
se pueden encontrar trastornos de la conduccin por fibrosis.
Manifestaciones pleuropulmonares: poco frecuentes y de apari-
cin tarda. La ms tpica es la fibrosis de los lbulos pulmonares
superiores, que pude ser colonizada por Aspergillus.
Manifestaciones neurolgicas: fracturas-luxaciones vertebrales cer-
vicales, subluxacin atloaxoidea con clnica cervical o sndrome
de cola de caballo secundario a aracnoiditis crnica. Sin embargo,
no observamos meningoencefalitis, por lo que la respuesta correc-
ta es la 4.
Manifestaciones genitourinarias: la ms frecuente es la prostatitis
crnica y la nefropata IgA (respuesta 1).

Pregunta 57. Manifestaciones extraarticulares de la AR y la EA.

1232434562789
3 4 7 5  44345

84 5
37
1234567879
296 6
385  36 59634 74
  834 6  29 56345 Pregunta 58. Afectacin vertebral en la espondilitis anquilosante.
 8347558 5
34745963 Pregunta 59.- R: 2
344533245 Este cuadro clnico presenta muchas de las caractersticas de la
respuesta que elegimos, la artritis reactiva. La enfermedad aparece
 47 437 95963 con ms frecuencia en varones jvenes (entre los 20-40 aos), en un
782
6689 395 3   47 437 95963 periodo entre 1-4 semanas despus de presentar un cuadro infeccio-
27 279543 397  "46 83 so intestinal o uretral que en ocasiones pasa desapercibido. La clnica
 94733559 articular caracterstica es la de una artritis asimtrica que predomina
479!2 7 6  en extremidades inferiores de inicio brusco, aunque aditiva (se van
sumando articulaciones afectas). Al formar parte de las espondiloar-
 2
 3445334 8#4 87$ %9 2 8 398 5546 85 tropatas es caracterstica la aparicin de entesitis, por este motivo el
34 854 6  paciente tiene talalgia bilateral (inflamacin de los tendones de la
fascia plantar o del aquleo).
& 7 7 
743 824
24 347565'734254 (47 656 6 849 85
347565%'&
Desde el punto de vista del estudio del lquido sinovial, este es de
caractersticas inflamatorias y asptico. Este ltimo dato es imprescin-
74#4587  dible, ya que si hay crecimiento de algn germen a nivel articular, la
%9558 9* 67' 85 artritis sera sptica y no reactiva.
El resto de respuestas las descartamos por los siguientes motivos:
47534 3 58 9339) 5 +9789 85+,  545 El lupus eritematoso sistmico es ms habitual en mujeres jvenes,
*3#6 85  393435 produce artralgias o artritis migratoria sin entesitis.
955674 5 963 6 95 La artritis reumatoide produce un cuadro de poliartritis simtrica
sin entesitis.
Pregunta 58.- R: 5 La artritis gonoccica puede producir una artritis aguda, pero suele
estar precedida de un cuadro febril con bacteriemia, con artritis
Comentarios TEST

Respecto a la actitud teraputica que debemos tomar ante un pa-


migratoria y tenosinovitis.
ciente con espondilitis anquilosante, debemos tener en cuenta las
La artritis infecciosa no produce entesitis, el lquido sinovial es de
siguientes pautas:
caractersticas infecciosas (ms de 50.000 clulas/mm3 y consumo
No existe ningn tratamiento especfico que pueda detener el cur-
de glucosa) y no tiene entesitis. El GRAM es positivo en un nmero
so evolutivo de la enfermedad, por lo que nuestra actitud debe
elevado de pacientes. (Ver tabla en la pgina siguiente)
dirigirse a controlar la inflamacin y prevenir el desarrollo de de-
formidades.
La base del tratamiento mdico es la utilizacin de AINEs. Den- Pregunta 60.- R: 2
tro de este grupo la fenilbutazona es el frmaco ms eficaz para La artritis reactiva o sndrome de Reiter es una espondiloartropata
el control sintomtico, pero su uso est restringido para los bro- que aparece despus de una infeccin uretral o entrica en el plazo
tes de gran inflamacin y durante un tiempo limitado al produ- de 1 a 4 semanas. Todas las bacterias involucradas en la aparicin de
cir en ocasiones agranulocitosis. Por este motivo debemos optar una artritis reactiva son intracelulares y afectan caractersticamente a
para el tratamiento crnico por otros AINEs, como la indometa- mucosas, surgiendo una respuesta autoinmune anmala en stas. Pro-
cina. bablemente el mecanismo es la aparicin de una respuesta antibacte-
Cuando la afectacin perifrica (artritis perifrica) es importante, riana local que posteriormente se reproducira en la sinovial, al existir
podemos utilizar algunos de los tratamientos que se utilizan en la en sta componentes que guardaran similitud morfolgica con algu-
artritis reumatoide, como la sulfasalazina o el metotrexate. Sin em- na estructura bacteriana.
bargo, estos frmacos no modifican el curso de la enfermedad En el momento de la aparicin de la artritis, los pacientes de forma
axial. Otros frmacos de segunda lnea no han mostrado eficacia, mayoritaria han curado de la infeccin inicial. Los cuadros de artritis
como las sales de oro (respuesta correcta 5), D-Penicilamina o los reactiva de origen entrico son ms prevalentes en Europa y los de
antipaldicos. origen uretral en los pases anglosajones (EE.UU. y Reino Unido).
Los corticoides sistmicos no son de utilidad, pero s cuando se Los principales grmenes responsables de la infeccin gastrointes-
utilizan de forma intralesional en la entesopata o en la sinovitis tinal son la Shigella flexneri, Salmonella, Yersinia y Campylobacter, por
persistente que no responde a AINEs. lo que la respuesta a esta pregunta es la 2, Pseudomonas.

M exico A rgentina CTO Medicina C/ Nez de Balboa, 115 28006 MADRID (Espaa) Tfno.: (91) 782 43 32 / Fax: (91) 782 43 27
C hile U ruguay E-mail: secretaria@ctomedicina.com; iberocto@ctomedicina.com WEB: www.ctomedicina.com; www.iberocto.com
RM Pg. 13
REUMATOLOGA
Seguimiento a distancia Preparacin Examen de Seleccin 05/06 1 Vuelta
En las formas genitourinarias el agente desencadenante suele ser la producida por gonococo, una artritis sptica gonoccica (es terica-
Chlamydia trachomatis, y con menor frecuencia, el Ureaplasma mente la causa ms frecuente de infeccin articular en adultos jve-
urealyticum. nes) o una artritis reactiva, si la infeccin ha sido por Chlamydia. Si
analizamos las respuestas que tenemos, analizamos as las diferencias
clnicas:
Pregunta 59. Diagnstico diferencial entre la Enfermedad La tenosinovitis es caracterstica de la artritis gonoccica. En la artri-
de Behet y la artritis reactiva. tis reactiva hay entesitis (inflamacin de la insercin del tendn en
el hueso).
Behet Artritis reactiva El dolor lumbosacro, efectivamente, es caracterstico de la artritis
reactiva y no de la gonoccica, ya que al formar parte del grupo de
Es una las espondiloartropatas puede producirse sacroiletis.
espondiloartropata Si encontramos un cultivo positivo a nivel genital de gonococo, no
Es una vasculitis que podemos estar del todo seguros de que sea el germen productor
s e r o n e g a t iv a , e n
Concepto afecta a vasos de del cuadro articular. En ocasiones hay infecciones mixtas, inclu-
general, secundaria a yendo Chlamydia, y el paciente desarrolla una artritis reactiva. El
pequeo calibre.
una infeccin entrica diagnstico definitivo de artritis gonoccica se establece si el pa-
o g e n it o u r in a r ia . ciente tiene cultivo positivo a nivel del lquido sinovial.
En la artritis reactiva nunca puede haber cultivo positivo a nivel
Poliartritis erosiva y articular, ya que en este caso se tratara de una artritis sptica, y no
m uy dolorosa. de una artritis reactiva.
Oligoartritis. Dactilitis, entesitis En el caso de la artritis gonoccica, la base de su tratamiento, lgi-
Afectacin
N o e r o s iv a . aqulea o fascitis camente, es la administracin de antibiticos (penicilina o mejor
articular cefalosporina de tercera generacin). En el caso de la artritis reac-
No espondilitis. plantar.
Espondilitis en m enos tiva, sern los AINEs.
del 50%. Pregunta 63.- R: 1
Diversas lesiones: En este caso clnico se plantea la actitud teraputica ms ade-
papulopstulas en cara y cuada en un paciente con artritis psorisica con enfermedad cut-
Queratoderm ia nea relevante. El esquema que utilizamos es muy similar al que se
Piel tronco, eritem a nodoso; utiliza en la artritis reumatoide y lo podemos resumir de la siguien-
blenorrgica.
pseudofoliculitis. te forma:
Prueba de patergia +. Si el paciente tiene una forma de artritis leve, utilizamos la primera
escala teraputica, que incluye la administracin de AINEs, y si es
Anejos preciso, aadir corticoides orales en dosis bajas.
No onicopata. Onicopata .
cutneos En el caso de que la enfermedad sea ms severa, aadir frmacos
de segunda lnea para reducir la progresin de la enfermedad.
lceras orales dolorosas. lceras orales
Dentro de estos frmacos es de especial utilidad el metotrexato,
Mucosas lceras genitales indoloras. que es eficaz para la enfermedad articular y cutnea. Los antipal-
dolorosas. Balanitis circinada. dicos como la hidroxicloroquina pueden mejorar los sntomas,
pero no se suelen utilizar porque pueden empeorar las lesiones
Conjuntivitis.
Ojos Uvetis posterior. cutneas. Frmacos de segunda lnea como las sales de oro y la
Uvetis anterior. sulfasalazina pueden resultar igualmente eficaces.
Otro frmaco que se ha ensayado es la ciclosporina A, que tambin
Pregunta 61.- R: 5 puede mejorar la enfermedad reumtica y cutnea. La utilizacin de
Dentro de la artritis reactiva, como ocurre en la mayora de las enfer- retinoides, como el etetrinato, puede tambin ser eficaz pero suele
medades articulares inflamatorias, existen una serie de manifestaciones estar limitado su uso por los efectos secundarios que produce.
extraarticulares, destacando dentro de ellas las mucocutneas.
Como puedes comprobar, el tratamiento que resulta ms eficaz

Comentarios TEST
En la boca hay lceras transitorias, superficiales, y en la mayora de
los casos, asintomticas (respuesta 1). para tratar la enfermedad articular y la enfermedad cutnea es la
La lesin cutnea caracterstica es la queratodermia blenorrgica utilizacin del metotrexate, que es la respuesta que debemos elegir.
(respuesta 2), constituida por vesculas que se vuelven hiperqueratsi-
cas y acaban formando costras antes de desaparecer. Son ms fre- Pregunta 64.- R: 5
cuentes en las palmas de manos y plantas de pies, pero pueden apa- En esta pregunta se plantean dos cuestiones: por un lado, las carac-
recer en cualquier otra parte. En los pacientes infectados por el virus tersticas clnicas de la artritis relacionada con la enfermedad inflama-
VIH, estas lesiones suelen ser muy extensas y sumamente graves, domi- toria intestinal, y por otro, la relacin del HLA-B27 con las distintas
nando el cuadro clnico. espondiloartropatas.
A menudo hay lesiones en el glande, denominadas balanitis circi- La manifestacin reumatolgica ms frecuente es la artritis perifri-
nada (respuesta 3), que consisten en vesculas que rpidamente se ca (respuesta 1), que se encuentra en relacin con la extensin de la
rompen para formar erosiones superficiales indoloras que pueden enfermedad intestinal. El porcentaje de pacientes con EII que la pade-
formar costras parecidas a las de la queratodermia blenorrgica. cen se sita entre el 10-25%. La afectacin axial (sacroiletis y espon-
Tambin son frecuentes las lesiones ungueales en forma de onic- dilitis) vara segn los estudios entre el 1-20%. En general, es ms fre-
lisis (respuesta 4), coloracin amarillenta de la parte distal e hiperque- cuente en la colitis ulcerosa.
ratosis en capas superpuestas. La artritis suele ser mono u oligoarticular asimtrica (respuesta 3),
Como puedes comprobar, la nica manifestacin que no aparece con predominio de grandes articulaciones de miembros inferiores
es el pioderma gangrenoso. Esta lesin cutnea s se puede relacionar (rodillas y tobillos), y en menor medida en miembros superiores
con otras formas de espondiloartropata: la artritis asociada a enfer- (carpos, codos y pequeas articulaciones de manos). El episodio arti-
medad inflamatoria intestinal. cular es de instauracin brusca. Su curso se relaciona con la exten-
sin e intensidad de la enfermedad intestinal. La artritis es autolimita-
Pregunta 62.- R: 2 da y generalmente no erosiva (respuesta 4). Su patrn evolutivo es
Despus de un cuadro de uretritis infecciosa, un paciente puede cclico y recidivante en las mismas articulaciones o migratorio en su-
presentar dos cuadros articulares diferentes: si la infeccin ha sido cesivas articulaciones.

M exico A rgentina CTO Medicina C/ Nez de Balboa, 115 28006 MADRID (Espaa) Tfno.: (91) 782 43 32 / Fax: (91) 782 43 27
Pg. 14 RM C hile U ruguay E-mail: secretaria@ctomedicina.com; iberocto@ctomedicina.com WEB: www.ctomedicina.com; www.iberocto.com
REUMATOLOGA
Preparacin Examen de Seleccin 05/06 1 Vuelta Seguimiento a distancia
Pregunta 67.- R: 2
Pregunta 64. Relacin entre el HLA-27 y las espondiloartropatas. La degeneracin sarcomatosa es la complicacin ms grave que
puede aparecer en la evolucin de la enfermedad de Paget, aunque su
Espondiloartropata Prevalencia de B-27 prevalencia es baja (1%).
Espondilitis anquilosante 90%
Artritis Reactiva 60-80% Pregunta 67. Diferencias entre el osteosarcoma primario
Espondilitis psorisica y espondilitis en 50-70% y el osteosarcoma sobre hueso pagtico.
enfermedad inflamatoria intestinal (EII)
Artritis perifrica psorisica 14% 123452678596 123452678596 25 74
Artritis perifrica en EII 7%
7 967 5 425 63 85
12345676789
6 82  
8 2

Como podemos comprobar, la respuesta es la 5, ya que los pacien- 586 68  3 86 366867388285 

42 889267
tes con artritis perifrica en la enfermedad inflamatoria intestinal no
presentan una prevalencia superior a la de la poblacin general de
3 88
6
HLA-B27 (7%).
6 4 667 8  2 8832
2
 !6 7

Pregunta 65.- R: 5
El paciente que nos plantean en este caso presenta las caractersticas "
6#8#72
radiolgicas clnicas y analticas tpicas de la enfermedad de Paget.  86 "
6#828
6  
8 85
3

Desde el punto de vista radiolgico, la presencia de un patrn 93 # 6
mixto, esclertico y ltico (patrn en mosaico) en localizaciones seas 18  %8
36
tpicas (pelvis y hueso femoral) debemos considerarlo como tpico. La 2
88'(()
afectacin a este nivel puede producir artropata degenerativa, al alte-
2 8273$ %89
2 & 65638*6

6
rar la enfermedad el hueso intraarticular, produciendo dolor de ca-
7523 85 9
8828
289
#736  6&6#8 73
ractersticas mecnicas.
Los pacientes con enfermedad de Paget suelen ser mayores de 50 3
$8 8 23473767
aos, y en la analtica se observa elevacin de los parmetros de 9 
27
formacin sea relacionados con la actividad de los osteoblastos (fos-
fatasa alcalina) y parmetros de resorcin sea relacionados con la
actividad de los osteoclastos (hidroxiprolinuria). Aunque no siempre ocurre, se puede producir elevacin de la fosfa-
Las complicaciones que pueden aparecer en la enfermedad de tasa alcalina respecto a la que el paciente ya tiene elevada por la
Paget son: enfermedad de Paget. Como podemos ver, la respuesta es la 2.
Hipercalcemia e hipercalciuria por aumento del recambio seo
(respuesta 1). Pregunta 68.- R: 3
Artropata coxofemoral por afectacin de esta articulacin (respuesta 2). De forma genrica, debemos recordar que la indicacin para el
Debilitamiento del hueso afecto con fracturas patolgicas (respuesta tratamiento de la enfermedad de Paget se produce en los casos en que
3). la enfermedad es sintomtica. Sin embargo, en algunas ocasiones tam-
Aumento del gasto cardiaco por aumento de vascularizacin de bin puede recomendarse en pacientes asintomticos, como vere-
los huesos afectos por la enfermedad, produciendo en ocasiones mos a continuacin. Los frmacos que vamos a utilizar son aquellos
insuficiencia cardiaca (respuesta 4). que reducen la actividad de los osteoclastos, es decir, la calcitonina y
Como podemos ver, la respuesta es la 5. No tiene por qu presentar sobre todo los bifosfonatos.
hipovitaminosis D, a no ser que tenga asociado un dficit de toma de Las indicaciones absolutas para el tratamiento son:
esta vitamina o de exposicin solar. Dolor seo primario del hueso pagtico.
Deformidad sea rpidamente progresiva.
Pregunta 66.- R: 5 Hipercalcemia e hipercalciuria.
Comentarios TEST

Respecto a la etiopatogenia de la enfermedad de Paget, aunque se Insuficiencia cardiaca.


desconozca la causa exacta que produce la enfermedad, existen una Preparacin para ciruga ortopdica (tratamiento previo para re-
serie de datos que debes conocer: ducir la actividad del hueso, su vascularizacin y hacer que el
En los aos 70 se comunic el hallazgo,mediante microscopa sangrado sea menor).
electrnica, de unas inclusiones que aparecan en el ncleo de los
osteoclastos de pacientes con enfermedad de Paget y que guarda- Podemos considerar como indicaciones relativas el tratamiento de
ban gran similitud con estructuras virales. Ms tarde se consigui las complicaciones evolutivas, como son:
identificar en los osteoclastos protenas del virus del sarampin y Sndromes neurolgicos compresivos (tratar a los pacientes con
del virus respiratorio sincitial, as como del virus distemper (del enfermedad en vrtebras o base del crneo).
moquillo canino). En un principio estas inclusiones se considera- Fisuras y fracturas corticales (tratar a los pacientes con lesiones
ron especficas de la enfermedad de Paget, pero posteriormente se lticas en huesos sustentadores de carga).
han encontrado partculas similares en otras enfermedades (tumor
de clulas gigantes y ostelisis expansiva familiar), as como en la En los pacientes con discreta elevacin de la fosfatasa alcalina sin
picnodisostosis, la osteopetrosis y la oxalosis primaria. Reciente- manifestaciones clnicas asociadas como se indica en la respuesta 3,
mente se ha demostrado que los pacientes con enfermedad de no existe indicacin de tratamiento.
Paget expresan el gen de la IL-6 en los osteoclastos y presentan un
aumento de IL-6 en el plasma. Esta produccin excesiva de IL-6, Pregunta 69.- R: 4
que puede ser inducida por un virus, actuara como estmulo local La definicin actual de la osteoporosis postmenopusica es la si-
del remodelado seo. guiente: densidad mineral sea que disminuye 2,5 desviaciones estn-
Cerca del 40% de los pacientes con enfermedad de Paget tienen dar (DE) por debajo del pico de masa sea que se produce en mujeres
un familiar de primer grado que tambin padece la enfermedad, jvenes (a los 30-35 aos), medido mediante el T-score o puntuacin
por lo que se especula con la presencia de algn factor hereditario T. Las posibilidades que podemos encontrar en una mujer postmeno-
que favorezca el desarrollo de la enfermedad. Sin embargo, el pusica al medir su masa sea son las siguientes:
estudio del HLA no ha arrojado resultados concluyentes (respuesta Densitometra normal: prdida de masa sea hasta 1 DE respecto
correcta, 5). al T-score.

M exico A rgentina CTO Medicina C/ Nez de Balboa, 115 28006 MADRID (Espaa) Tfno.: (91) 782 43 32 / Fax: (91) 782 43 27
C hile U ruguay E-mail: secretaria@ctomedicina.com; iberocto@ctomedicina.com WEB: www.ctomedicina.com; www.iberocto.com
RM Pg. 15
REUMATOLOGA
Seguimiento a distancia Preparacin Examen de Seleccin 05/06 1 Vuelta
Osteopenia: prdida de masa sea entre 1 DE y 2,5 DE respecto absorba menos calcio a nivel intestinal y se estimule la produccin de
al T-score. PTH (hiperparatiroidismo secundario).
Osteoporosis: prdida de masa sea igual o superior a 2,5 DE
respecto al T-score. Pregunta 72.- R: 2
Osteoporosis grave: prdida de masa sea igual o superior a 2,5 Dentro de las causas de osteoporosis de base hereditaria se inclu-
DE respecto al T-score asociada a la presencia de fractura. yen una serie de enfermedades como la osteognesis imperfecta, la
homocistinuria, el sndrome de Ehlers-Danlos y el sndrome de Marfan.
Los valores de la densitometra tambin vers que en ocasiones son De esta ltima enfermedad es tpica la ectopia o luxacin del cristali-
comparados con el denominado Z-score o puntuacin Z. En este no, respuesta que debemos elegir.
caso, la variacin en desviaciones estndar se realiza entre la densito- La osteognesis imperfecta es una enfermedad congnita del teji-
metra de la paciente y la media de masa sea que tienen las mujeres do conectivo, muy heterognea desde el punto de vista clnico, bio-
de su misma edad. Sin embargo, como ya hemos sealado, la defini- qumico y gentico. El defecto bioqumico primario, determinado
cin actual se realiza utilizando el T-score y no el Z-score. genticamente, es el resultado de un dficit cuantitativo y cualitativo
del colgeno tipo I. Por ello, no solo se afecta el esqueleto, sino
Pregunta 70.- R: 1 tambin otros tejidos, como la piel, ligamentos, tendones, dientes,
Las causas de osteoporosis las podemos recordar en la siguiente vasos sanguneos, vlvulas cardiacas y huesos timpnicos. Constitu-
tabla: ye la enfermedad congnita ms comn, que se traduce en
Osteoporosis primaria: osteopenia.
- Postmenopusica. La forma tipo I es la ms benigna y menos deformante, y tambin la
- Senil. que se encuentra con mayor frecuencia en los adultos (60% de los ca-
- Idioptica juvenil y del adulto joven. sos). Normalmente no existen deformaciones, y si existen, son mnimas.
Osteoporosis secundaria: La historia clnica consiste en la existencia de mltiples fracturas, despus
- Enfermedades endocrinas y metablicas: hipogonadismo, hi- de traumatismos mnimos desde el periodo neonatal (respuesta 4). La
perparatiroidismo, hipertiroidismo, hipercorticismo (exgeno o incidencia de fracturas disminuye despus de la pubertad.
endgeno), hipofosfatasia y asociada a otras enfermedades en- Las esclerticas se encuentran azuladas en todas las edades (res-
docrinas como la osteomalacia. puesta 3). Los trastornos auditivos aparecen en el 70% de los casos
- Genticas: osteognesis imperfecta, homocistinuria, sndrome (respuesta 1). Su herencia es autosmica dominante (respuesta 5).
de Ehlers-Danlos, sndrome de Marfan.
- Hematolgicas: mieloma. Pregunta 73.- R: 5
- Frmacos: corticoides, heparina, antiestrgenos. Las posibilidades teraputicas en la osteoporosis tipo I son actual-
- Otras causas: mastocitosis, escorbuto, desnutricin, alcoholis- mente las siguientes:
mo, mastocitosis sistmica. Frmacos moduladores de los receptores estrognicos como el
Raloxifeno. Los efectos de este frmaco ms relevantes son la re-
Podemos comprobar como la artrosis, que es la respuesta que duccin del cncer de mama invasor de un 70% de las pacientes
debemos elegir, no aparece asociada a la aparicin de osteoporo- que lo toman. Al contrario del tamoxifeno, el raloxifeno no se
sis. asocia con un incremento del riesgo de cncer uterino. El raloxifeno
aumenta los sofocos. Reduce el colesterol y el LDL, las lipoprote-
Pregunta 71.- R: 2 nas y el fibringeno, por lo que es posible que reduzca el riesgo
La osteoporosis tipo II o del anciano presenta una serie de caracte- cardiovascular. Aumenta la masa sea y reduce la aparicin de
rsticas analticas y clnicas que podemos recordar precisamente esta- fracturas.
bleciendo el diagnstico diferencial con la osteoporosis tipo I o post- Bifosfonatos, como el alendronato o el risedronato. Ambos han
menopusica. demostrado que aumentan la masa sea y reducen la aparicin de
fracturas.
Estrgenos: los estudios epidemiolgicos indican que las mujeres
Pregunta 71. Diagnstico diferencial entre osteoporosis tipo I y tipo II. que toman estrgenos de sustitucin presentan una reduccin del
50% de las fracturas osteoporticas, incluidas las de cadera. Los
123456575282938659
123456575282938659

efectos beneficiosos son mximos entre las mujeres que empiezan

Comentarios TEST
123456789
456789
pronto con el tratamiento de sustitucin y lo continan de forma
prolongada. Su empleo a largo plazo se puede asociar con un
4 8 2  2 incremento del riesgo de tromboembolia venosa y de cncer de
479574 vescula biliar, uterino y de mama. En los estudios de observacin,
8659 49425 77
776697
los estrgenos se han asociado con una reduccin significativa de
647 8 5 77 la incidencia de infarto de miocardio.
$#7667
76!
76967
Otras posibilidades teraputicas incluyen la calcitonina, la PTH, el
 2 967886  97
76 7!
766%779
9 fluoruro y otros frmacos experimentales.

9"#7

79
Como podemos ver, la mitramicina, frmaco que se utiliz hace
89 49 9 &
! '9 aos en la enfermedad de Paget, no se incluye para el tratamiento de
75 89 4 &
!6
77 &
!677
la osteoporosis.
 !"1#1$
%25789 49 85 Pregunta 74.- R: 1
(7)7 (7)7
834238  La osteomalacia por dficit de aporte de vitamina D es una situa-
cin relativamente frecuente en los ancianos, preferentemente en
&5 8' 89 49 2 *#7
6677+9 *#7
6,7879 aquellos ingresados en centros de crnicos con enfermedades que
(7 37 2 
7 -.6677
les impiden la movilizacin y una adecuada alimentacin. Las dos
fuentes de las que disponemos para la formacin de vitamina D3 son
En la osteoporosis tipo II, como se puede comprobar en la tabla, la exposicin a los rayos ultravioleta con la trasformacin en la piel
hay una reduccin primaria de la produccin de vitamina D3 (res- de la provitamina D3 (7-dihidrocolesterol) en vitamina D3 y la ali-
puesta correcta 2). El motivo es que esta vitamina se produce en el mentacin. Este dficit lleva asociado una reduccin de la produc-
rin, y en los ancianos esta funcin se vuelve ms deficitaria, por lo cin de 25 (OH)D en el hgado, que es el parmetro bioqumico de
que los niveles de esta vitamina suelen estar disminuidos, de ah que se medicin del dficit de vitamina D, y por lo tanto el que utilizamos

M exico A rgentina CTO Medicina C/ Nez de Balboa, 115 28006 MADRID (Espaa) Tfno.: (91) 782 43 32 / Fax: (91) 782 43 27
Pg. 16 RM C hile U ruguay E-mail: secretaria@ctomedicina.com; iberocto@ctomedicina.com WEB: www.ctomedicina.com; www.iberocto.com
REUMATOLOGA
Preparacin Examen de Seleccin 05/06 1 Vuelta Seguimiento a distancia
para realizar el diagnstico de osteomalacia por dficit de aporte de
vitamina D. Pregunta 76. Manifestaciones clnicas de la esclerosis sistmica.
En fase avanzada del dficit, la transformacin de 25(OH)D a
1,25(OH)2 D se ve reducida, y con ello la absorcin de calcio intestinal, Manifestacin clnica Forma difusa (%) Forma limitada (%)
y por lo tanto se produce hipocalcemia. Esta estimula la produccin de
PTH, que aumenta la resorcin sea y corrige la calcemia y aumenta la Raynaud 90-95 95-100
depuracin renal de fosfatos, produciendo hipofosfatemia (respuesta Engrosamiento de piel 100 98
correcta 1). Por lo tanto, el patrn bioqumico resultante ser: descenso Calcinosis subcutnea 10 50
de 25(OH)D y en fases avanzadas de 1, 25(OH)2 D, calcemia descendi- Telangiectasias 40 85
da o normal, hiperparatiroidismo secundario, aumento de la depura- Artritis 70 40
cin renal de fosfato e hipofosforemia. Miopata 50 5
Motilidad esofgica alterada 80 80
Fibrosis pulmonar 40 35
Hipertensin pulmonar aislada <1 < 10
Insuficiencia cardiaca 30 <1
Crisis renales 15 <1

Pregunta 76. Esclerosis sistmica difusa versus


esclerosis sistmica limitada.

123456 728296
6
1234567879
2 5677653 12345677 4
 253
3 96 2 49 6  4
 25375
578 875
5
4

6527 7657595
22 79

75
2 3
664
76572
32377657926 5 3757657632
767 23 75 45
6265 26 353726545537874
4 353
 79
25773453
1265452775353732
9
25
#534
2434256
3 96 225 6 2
323796 5
7
56

2323736

253  $29
43275
4
256
85 66 9 592 6 !534
243425675
"55 96 5
72

323
Pregunta 74. Principales osteopatas metablicas.
2625
79
2 5
25
92 4 5 424 9 23 
5357% 424
 

Pregunta 75.- R: 2 6924  6 5
En el caso de un paciente con insuficiencia renal crnica, a dife-
rencia de la debida al dficit nutricional, no tiene por qu existir
descenso en los niveles de 25 (OH)D, ya que en esta ocasin el trastor- Pregunta 77.- R: 5
no se produce en el paso a nivel renal de la 25 (OH)D a 1,25(OH)2 D. Existen algunos factores ambientales que estn asociados al desa-
Ser esta ltima molcula la que descender de forma significativa al rrollo de la esclerosis sistmica (ES) y de procesos afines a la esclero-
estar enfermo el tejido que la produce. Podemos comprobar que la dermia. La ES parece ms frecuente entre los mineros de carbn y de
respuesta correcta es la 2. oro, especialmente entre los que soportan una exposicin ms inten-
Comentarios TEST

El patrn bioqumico que esperamos encontrar en esta situacin sa, lo que sugiere que el polvo de slice (respuesta 1) quiz sea un
ser el siguiente: 25 (OH) D normal, con descenso marcado de factor predisponente.
1,25(OH)2 D. A partir de este momento el esquema es similar al del Los trabajadores expuestos al cloruro de polivinilo (respuesta 3)
dficit de aporte de la vitamina D, pero con una diferencia final: al pueden presentar fenmeno de Raynaud, acroostelisis, lesiones cu-
reducirse la molcula ms activa de la vitamina D, la 1,25(OH)2 D, la tneas como las de la esclerodermia y alteracin de los capilares.
absorcin de calcio a nivel intestinal se reduce, producindose hipo- Existe tambin una relacin entre la exposicin a resinas epoxi e
calcemia y estimulndose la produccin de PTH (hiperparatiroidis- hidrocarburos aromticos como la bencina, tolueno y tricloroetileno
mo secundario). La PTH aumenta la resorcin de calcio en el hueso y (respuesta 4) y el desarrollo de esclerodermia.
tiende a corregir la calcemia, pero como el rin est enfermo no se En 1981 apareci en Espaa una enfermedad generalizada pareci-
produce el aumento de la depuracin renal de fosfatos, sino disminu- da a la esclerodermia debido al consumo de aceite de colza adultera-
do. Los pacientes presentaron en principio una neumonitis intersticial
cin. Por este motivo, los pacientes con insuficiencia renal crnica
con eosinofilia, artritis y miositis, engrosamiento de la piel, fenmeno de
tienen hiperfosfatemia y no hipofosfatemia.
Raynaud, hipertensin pulmonar, sndrome seco y acroostelisis.
Se han descrito casos de esclerodermia y de otras enfermedades
Pregunta 76.- R: 3
del tejido conectivo en mujeres en las que se han llevado a cabo
Dentro de la esclerosis sistmica se distinguen dos formas de pre-
implantes de silicona (respuesta 2). Alrededor del implante puede
sentacin, que son la esclerosis sistmica difusa y la esclerosis sistmi-
aparecer fibrosis localizada.
ca limitada. Las diferencias fundamentales entre ambas formas de La respuesta correcta es la 5, contacto con asbesto, que produce
presentacin aparecen en la tabla de la pgina siguiente. una enfermedad pulmonar intersticial fibrosante, pero no un sndro-
Podemos ver que los anticuerpos antitopoisomerasa se relacionan me esclerodermiforme.
con la forma ms grave de enfermedad, la esclerosis sistmica difusa, y
en ella aparece con mayor frecuencia la fibrosis pulmonar.
Pregunta 78.- R: 5
La fibrosis pulmonar aparece con mayor frecuencia en la forma
difusa (40%), pero en la limitada tambin puede aparecer (35%), aun- En esta pregunta debemos tener en cuenta las causas que pueden
producir fenmeno de Raynaud y que se pueden clasificar de la
que habitualmente con menor gravedad.
siguiente forma:

M exico A rgentina CTO Medicina C/ Nez de Balboa, 115 28006 MADRID (Espaa) Tfno.: (91) 782 43 32 / Fax: (91) 782 43 27
C hile U ruguay E-mail: secretaria@ctomedicina.com; iberocto@ctomedicina.com WEB: www.ctomedicina.com; www.iberocto.com
RM Pg. 17
REUMATOLOGA
Seguimiento a distancia Preparacin Examen de Seleccin 05/06 1 Vuelta
Fenmeno de Raynaud primario o idioptico: enfermedad de La proliferacin bacteriana excesiva por atona del intestino delga-
Raynaud. do puede causar anemia megaloblstica por dficit de vitamina
Fenmeno de Raynaud secundario: B12, de cido flico, o de ambos nutrientes.
- Colagenosis vasculares: esclerosis sistmica, lupus eritematoso La anemia hemoltica microangioptica es la que con mayor fre-
sistmico, artritis reumatoide, dermatomiositis. cuencia se asocia a enfermedad renal y est causada por el dep-
- Enfermedades arteriales oclusivas: aterosclerosis de las articula- sito intravascular de fibrina en las arteriolas renales.
ciones, tromboangetis obliterante, oclusin arterial aguda, sn-
drome de compresin de la salida del trax. La respuesta que debemos elegir es la 4, ya que la anemia refracta-
- Hipertensin pulmonar. ria por exceso de blastos no tiene relacin con la esclerosis sistmica.
- Alteraciones neurolgicas: trastornos de los discos interverte- Esta forma de anemia se incluye dentro de los sndromes mielodispl-
brales, siringomielia, tumores de la mdula espinal, poliomieli- sicos, encontrndose en la mdula sea un porcentaje de blastos
tis, sndrome del tnel del carpo. entre el 5-20% de la celularidad. La supervivencia de este cuadro se
- Discrasias sanguneas: crioglobulinemias, macroglobulinemia, estima entre 12-15 meses.
enfermedades mieloproliferativas.
- Traumatismos: lesiones por vibraciones, sndrome de la mano Pregunta 81.- R: 1
del martillo, shock elctrico, lesiones por fro, mecanografa, La enfermedad mixta del tejido conectivo (EMTC) rene caracte-
tocar el piano. rsticas de varias conectivopatas. Poliartritis simtrica similar a la en-
- Frmacos: derivados ergotamnicos, metisergida, bloqueantes contrada en la artritis reumatoide. Manifestaciones mucocutneas del
beta-adrenrgicos, bleomicina, vinblastina, cisplatino. lupus eritematoso sistmico como la erupcin malar, fotosensibilidad,
lesiones discoides, alopecia y lceras dolorosas en la boca. Cambios
Una vez que hemos recordado las causas que pueden producir esclerodermiformes en la piel similares a los encontrados en la escle-
fenmeno de Raynaud, debemos elegir la respuesta correcta. rosis sistmica limitada, as como trastornos gastrointestinales caracte-
La crioglobulinemia se incluye dentro de las causas por discrasia rizados por la alteracin de la motilidad esofgica y reflujo
sangunea. gastroesofgico. Algunos pacientes presentan sntomas tpicos de la
La determinacin de anticuerpos anti-SCL 70 es caracterstica de polimiositis con debilidad muscular proximal, alteraciones electro-
la esclerosis sistmica difusa y los anticentrmeros de la esclerosis miogrficas, elevacin de enzimas musculares, y lesiones inflamato-
sistmica limitada. Ambas enfermedades presentan fenmeno de rias en la biopsia muscular. Pocas veces se observa una erupcin erite-
Raynaud en la mayora de los casos. matosa en nudillos, codos y rodillas, o eritema en heliotropo, datos
La capilaroscopia es la tcnica que permite el diagnstico diferen- caractersticos de una dermatopolimiositis.
cial entre el fenmeno de Raynaud primario (donde es normal) del Los pacientes con EMTC presentan varias de las manifestaciones
secundario por colagenosis (donde es patolgica, mostrando dila- referidas como ocurre en nuestro caso. Para completar el diagnstico
tacin e irregularidades del lecho vascular ungueal). es muy til determinar los anticuerpos anti-RNP que se encuentran a
La radiologa de trax es til para diagnosticar los signos de com- ttulos muy elevados en esta enfermedad.
presin torcicos (por ejemplo, la costilla cervical).
La respuesta correcta es la 5, la determinacin de anticuerpos an- Pregunta 82.- R: 2
tihistona. Aunque pueden aparecer en el lupus eritematoso En esta pregunta se repasan varios conceptos importantes referidos
sistmico, no son caractersticos de esta enfermedad y por lo tanto a las artritis spticas.
no orientan para su diagnstico. Se consideran anticuerpos espe- En la primera respuesta podemos recordar que la artritis bacteriana
cficos del lupus los anti-DNAds y los anti-Sm. aguda suele manifestarse como una monoartritis aguda que afecta
con ms frecuencia a las articulaciones que soportan carga, siendo
Pregunta 79.- R: 3 la rodilla la ms afectada seguida de la cadera. La artritis por Gram
Se considera actualmente que la causa ms frecuente de muerte negativos tiene un curso ms indolente, con sntomas generales
en la esclerosis sistmica es la afectacin pulmonar. Adems, dentro ms moderados y molestias articulares menos importantes, que la
de la afectacin visceral, la enfermedad pulmonar es la segunda en infeccin por cocos Gram positivos.
frecuencia despus de la afectacin del esfago. Dentro de la clnica La infeccin de prtesis articulares se produce con ms frecuencia
pulmonar podemos diferenciar dos formas de presentacin: la fibro- por Staphylococcus epidermidis, seguida del Staphylococcus aureus,
sis pulmonar y la hipertensin pulmonar. y suele tardar semanas o meses en poder ser diagnosticada, ya que

Comentarios TEST
En el caso clnico actual, los datos aportados son todos orientativos los sntomas son poco precisos. Por lo tanto, es esta la respuesta que
al diagnstico de hipertensin pulmonar sin fibrosis pulmonar asocia- debemos elegir.
da. Recuerda que esta forma de enfermedad aparece con ms fre- Los ADVP tienen con frecuencia infecciones de la columna verte-
cuencia en la esclerosis sistmica limitada o sndrome de CREST. Se bral, articulaciones sacroiliacas o esternoclaviculares. Suele pre-
caracteriza por la ausencia de fibrosis pulmonar en la radiologa de sentarse como un proceso agudo con sntomas generales, aumen-
trax. De hecho, la hipertensin pulmonar produce un patrn de to de temperatura, inflamacin y limitacin de la movilidad. Los
oligoemia (falta de vasos visibles) en el parnquima pulmonar. La si- grmenes ms frecuentes son el Staphylococcus aureus y la
lueta cardiaca se ve modificada por el incremento de presin pulmo- Pseudomonas.
nar aumentando el tamao de las cavidades derechas, dilatndose el Los pacientes en tratamiento con corticoides pueden tener signos
cono de la arteria pulmonar. inflamatorios menos llamativos, dificultando el diagnstico.
Aunque no coincidan con los datos de este paciente, debes tener El Haemophilus influenzae es el germen que produce con ms fre-
en cuenta que tambin se pueden observar en los pacientes con cuencia artritis sptica en los dos primeros aos de vida, en aque-
esclerosis sistmicas neumonas por aspiracin secundarias a reflujo llas poblaciones no vacunadas. Sin embargo, en la actualidad la
gstrico debido a la atona esofgica inferior. Tambin presentan un inmunizacin se ha introducido en el programa vacunal en la
aumento de riesgo de cncer broncognico y alveolar. mayora de los pases desarrollados, siendo el Staphylococcus au-
reus el germen ms frecuente en estas poblaciones.
Pregunta 80.- R: 4
La mayora de los pacientes con esclerosis sistmica tiene anemia Pregunta 83.- R: 1
que puede deberse a varias causas: La actitud teraputica en los pacientes con artritis sptica se basa en
Anemia hipoproliferativa propia de los procesos inflamatorios cr- la administracin de antibiticos sistmicos y el drenaje de la articula-
nicos; es la causa ms frecuente. cin infectada. Cuando la articulacin tiene un fcil acceso, como
Anemia ferropnica debida a un dficit de hierro por hemorragias ocurre en la rodilla, el drenaje se puede realizar diariamente mediante
digestivas. artrocentesis mientras persista el derrame. En articulaciones de difcil
acceso, como puede ser la cadera, se recomienda el drenaje quirrgi-

M exico A rgentina CTO Medicina C/ Nez de Balboa, 115 28006 MADRID (Espaa) Tfno.: (91) 782 43 32 / Fax: (91) 782 43 27
Pg. 18 RM C hile U ruguay E-mail: secretaria@ctomedicina.com; iberocto@ctomedicina.com WEB: www.ctomedicina.com; www.iberocto.com
REUMATOLOGA
Preparacin Examen de Seleccin 05/06 1 Vuelta Seguimiento a distancia
co, al igual que en los pacientes con cultivo persistentemente positivo Como podemos ver en esta tabla, la respuesta es la nmero 2, la
del lquido sinovial despus de 7 das de tratamiento. artritis reumatoide.
La eleccin del antibitico se basa inicialmente en el resultado de la Debes tener en cuenta que la artritis reactiva en estos pacientes
tincin de Gram del lquido articular, y posteriormente en el resultado suele ser grave y, en general, se inicia en la situacin de inmunodepre-
del cultivo y antibiograma correspondiente. En la pregunta nos plan- sin severa. La afectacin axial es poco frecuente, predominando la
tean el aislamiento definitivo del germen y nos piden que escojamos la oligoartritis de miembros inferiores. Las lesiones cutneas pueden pre-
asociacin de tratamiento incorrecta. La respuesta es la 1, ya que el dominar en la presentacin clnica. El 70% son HLA-B27. La alta
tratamiento de la infeccin por Haemophilus influenzae se realiza con prevalencia de la artritis reactiva est en relacin con las prcticas
cefalosporina de tercera generacin, como la cefotaxima o la ceftria- sexuales de riesgo, ms habituales en estos pacientes.
xona. En las infecciones menos graves por este germen se pueden
utilizar antimicrobianos por va oral. Hay que tener en cuenta que el Pregunta 86.- R: 2
25% de las cepas son productoras de beta-lactamasas y muestran resis- La enfermedad de Lyme, debida a la infeccin por la espiroque-
tencia a la ampicilina, pudiendo ser tratadas con trimetoprim-sulfame- ta Borrelia burgdorfferi, produce artritis en el 70% de las personas
toxazol, amoxicilina/cido clavulnico, fluorquinolonas o claritromi- no tratadas. Das o semanas despus de la inoculacin de la espiro-
cina. queta aparecen artralgias y mialgias intermitentes, pero no una au-
El resto de las respuestas corresponden a los antibiticos de elec- tntica artritis. Ms tarde se configuran tres fases de la enfermedad
cin en los distintos grmenes que producen artritis sptica. articular:
El 50% de las personas no tratadas sufren brotes intermitentes de
Pregunta 84.- R: 5 monoartritis u oligoartritis que afecta a grandes articulaciones como
En esta pregunta se plantean como posibilidades diagnsticas una la rodilla.
serie de infecciones virales que pueden presentar clnica articular, as El 20% de los no tratados presentan artralgias intermitentes.
como otros diagnsticos diferenciales a tener en cuenta. Veamos a El 10% de los pacientes presentan una sinovitis inflamatoria crni-
continuacin cmo debemos analizar las distintas respuestas: ca con lesiones erosivas y destruccin articular.
La infeccin por el virus de Epstein-Barr produce un cuadro de
mononucleosis infecciosa caracterizado por la presencia de fie- Como podemos ver, el patrn articular ms habitual es el primero,
bre, adenopatas y faringitis. En un 5% pueden tener ictericia, con afectacin de grandes articulaciones, por lo que debemos elegir
siendo excepcional la aparicin de manifestaciones articulares. la respuesta 2.
La presentacin clnica no coincide, por lo tanto, con el caso Otras manifestaciones caractersticas de la enfermedad de Lyme no
presentado. tratada es la aparicin de:
En un paciente con artritis reumatoide de inicio no es esperable la Afectacin del sistema nervioso (respuestas 3 y 5): meningitis linfo-
desaparicin espontnea de los sntomas articulares ni la ictericia. citaria, neuritis craneal, radiculoneuropata, encefalitis.
La aparicin de ictericia es poco habitual al administrar AINEs ( lo Afectacin cardiovascular (respuesta 4): bloqueo cardiaco, miocarditis.
ms frecuente es la hipertransaminasemia), por lo que es poco
probable esta posibilidad. En cuanto a la actitud teraputica inicial de la enfermedad, es la
En los tres das de la erupcin por rubola y en las 2 a 6 semanas administracin de doxiciclina (respuesta 1). Sin embargo, cuando
siguientes a la aplicacin de una vacuna con virus atenuados de aparece clnica articular crnica y/o neurolgica y/o cardiovascular
rubola, hasta un 50% de las mujeres presentan artralgias y un 10% se recomienda la utilizacin de ceftriaxona parenteral durante 2-4
artritis persistente. La paciente, sin embargo, no ha presentado erup- semanas.
cin cutnea ni vacunacin reciente, por lo que descartamos esta
posibilidad. Pregunta 87.- R: 1
La respuesta correcta es la infeccin por el virus de la hepatitis B. Como suele ser habitual para la confirmacin de un diagnstico
Hasta un 10% de los pacientes con hepatitis B presentan, en las 2 de sospecha, el estudio histolgico (respuesta 1) es la prueba ms
semanas antes de que aparezca la ictericia, una reaccin mediada especfica para el estudio de la xerostoma en un paciente con sndro-
por anticuerpos que se manifiesta por una erupcin maculopapu- me de Sjgren. Dentro de los criterios para la definicin de la enfer-
losa, urticaria, fiebre y artralgias. Tambin puede aparecer una medad aparece incluida la biopsia caracterstica tomada de una gln-
artritis simtrica de las manos, carpos, codos o tobillos con rigidez dula salivar menor.
Comentarios TEST

matutina similar a la artritis reumatoide. Los sntomas, como en El diagnstico subjetivo de la xerostoma se obtiene por la respues-
nuestro caso, se resuelven cuando aparece la ictericia. Recuerda ta afirmativa a una de estas preguntas:
que la infeccin por el virus de la hepatitis C se asocia a la apari- Tiene sensacin de sequedad de boca durante los ltimos tres me-
cin de crioglobulinemia. ses?
Tiene que beber cada da ms lquidos para poder deglutir ali-
Pregunta 85.- R: 2 mentos secos?
La infeccin por el virus VIH puede asociar la aparicin de una Ha notado hinchazn permanente o recurrente en las parti-
serie de manifestaciones musculoesquelticas que resumimos en la das?
siguiente tabla:
Si el paciente contesta afirmativamente a alguna de las cuestiones
planteadas, la xerostoma se puede objetivar mediante sialografa y
Pregunta 85. Manifestaciones musculoesquelticas en la infeccin VIH. sialometra (medicin de la cantidad de saliva), gammagrafa de par-
tidas, ecografa o resonancia magntica de partida o submaxilares.
Artralgias. Todas estas pruebas tienen escasa especificidad, de ah que sea bsico
Espondiloartropatas: realizar la biopsia de glndula salivar menor.
- Artritis reactiva. Se debe realizar la biopsia glandular en el labio inferior a mitad de
- Artritis psorisica. camino entre la comisura y mitad del labio y se obtendrn 4 o ms
- Espondiloartropata indiferenciada. lbulos para valorar la presencia de infiltrado inflamatorio. El diag-
Artritis asociada a VIH. nstico requiere la presencia de dos o ms focos (presencia de 50 o
Mialgias-Miositis. ms linfocitos en un rea de 4mm2).
Fibromialgia.
Artritis sptica. Pregunta 88.- R: 3
Sndrome de Sjgren.
La infeccin VIH puede presentar distintas manifestaciones muscu-
Vasculitis.
loesquelticas, y entre ellas, la aparicin de un sndrome de Sjgren
secundario.

M exico A rgentina CTO Medicina C/ Nez de Balboa, 115 28006 MADRID (Espaa) Tfno.: (91) 782 43 32 / Fax: (91) 782 43 27
C hile U ruguay E-mail: secretaria@ctomedicina.com; iberocto@ctomedicina.com WEB: www.ctomedicina.com; www.iberocto.com
RM Pg. 19
REUMATOLOGA
Seguimiento a distancia Preparacin Examen de Seleccin 05/06 1 Vuelta
En esta enfermedad no se asocia la presencia de anticuerpos anti-
Pregunta 88. Diagnstico diferencial entre el sndrome de Sjgren nucleares.
primario y el asociado a la infeccin VIH.
Pregunta 91.- R: 1
123455672891
8  2484842 La miositis por cuerpos de inclusin supone la forma de miopata
248 45 66 inflamatoria ms frecuente por encima de los 50 aos, con predomi-
nio de los varones respecto a las mujeres, por lo que la respuesta 1 es
4 123456789
6567 963678686252 la correcta.
62 El comienzo de los sntomas es insidioso (respuesta 2 incorrecta),
2654  7652 4884828 desarrollndose a lo largo de meses o aos. A diferencia de la polimio-
sitis o dermatopolimiositis, se afectan grupos musculares proximales y
345567282 532
5843 532
5843 distales, siendo tpico el desarrollo de debilidad de los flexores del
64 54478 ! 54478"! antebrazo y de la musculatura intrnseca de la mano, y atrofia y adel-
gazamiento de antebrazos y piernas. En la mitad de los casos aparece
 56567284265 #$ %8&8'$( dificultad para la deglucin por compromiso de la musculatura farngea
46891
)47 2 *6+2 2 y esofgica alta. Sin embargo no suele llevar asociada la aparicin de
cardiopata, como algunas formas graves de polimiositis (respuesta 3
incorrecta).
Puedes comprobar que la respuesta 1, infiltrado por linfocitos Las enzimas musculares suelen estar elevadas, pero en algn caso
CD8+, y la respuesta 2, asociacin con HLA DR5, aparecen como pueden ser normales (respuesta 5 incorrecta). A diferencia de lo que
manifestaciones caractersticas en la infeccin VIH. Las respuestas 4 se observa en algunas formas de polimiositis y dermatopolimiositis, no
(test de Schirmer positivo) y 5 (ulceraciones puntiformes en la crnea) se describen anticuerpos antinucleares ni anticitoplasmticos (respuesta
aparecen en el sndrome de Sjgren tanto primario como secundario, 4 incorrecta).
por lo que tambin estarn presentes. El diagnstico es confirmado mediante la biopsia muscular que
Queda como respuesta correcta la 3, ya que los autoanticuerpos muestra infiltrados perivasculares, necrosis y regeneracin de fibras
son caractersticos de la forma primaria. musculares y vacuolas intramiocitarias que contienen material basfi-
lo e inclusiones eosinfilas, y tien con rojo congo.
Pregunta 89.- R: 2 La respuesta a los corticoides e inmunosupresores es deficiente,
El planteamiento de este caso clnico es caracterstico de una der- habindose empleado en algunos casos inmunoglobulinas intrave-
matopolimiositis: debilidad muscular de predominio proximal en las nosas con resultados variables.
extremidades, acompaado de elevacin de las enzimas musculares
(CPK) y de lesiones cutneas tpicas, las manchas de Gottron (lesiones Pregunta 92.- R: 3
eritematosas que predominan en la superficie extensora de pequeas En primer lugar debemos resolver, con los datos aportados en el caso
articulaciones de las manos). clnico actual, el diagnstico del paciente, destacando que las manifes-
La presentacin clnica de un cuadro de dermatopolimiositis en taciones son recurrentes, y el cuadro probablemente hereditario al
un paciente de ms de 55 aos nos obliga a descartar la presencia de presentar un hermano una clnica similar. Los hallazgos que ms sobre-
una neoplasia oculta, que ocurre de forma global en el 15% de los salen son la presencia de fiebre, dolor abdominal con dificultad respi-
pacientes diagnosticados de esta enfermedad. Dentro de los tumores ratoria. El origen del paciente, en la zona mediterrnea, facilita todava
que se asocian con ms frecuencia se encuentran el cncer de ovario, ms el diagnstico: fiebre mediterrnea familiar. En esta enfermedad se
el cncer de mama, el melanoma cutneo y el cncer de colon, producen cuadros recurrentes de fiebre y dolor asociado a la afecta-
aunque otros tumores como el de pulmn o los hematolgicos tam- cin de serosas (pleuritis, pericarditis, o serositis peritoneal).
bin hay que tenerlos en cuenta. Se trata de una enfermedad autosmica recesiva, ligada a un de-
Si analizamos las respuestas de la pregunta actual, vemos que parte fecto en el brazo corto del cromosoma 16. Junto con los cuadros
de las exploraciones planteadas pueden estar justificadas para la bs- recurrentes de fiebre y serositis, los pacientes pueden desarrollar ami-
queda de una neoplasia oculta: radiologa de trax-cncer de pul- loidosis tipo AA, que es la complicacin ms grave de la enfermedad
mn, hemograma con frotis-tumor hematolgico, ecografa abdomi- (que en el caso clnico desarroll el hermano del paciente).
nal-neoplasias abdominales como la de ovario, mamografa-neopla- Desde el punto de vista teraputico, el tratamiento para prevenir la

Comentarios TEST
sia de mama. aparicin de las crisis agudas y el desarrollo de amiloidosis es la colchicina,
La respuesta correcta es la 2, ya que es la exploracin menos til por lo que la respuesta correcta es la 3. La dosis mnima inicial es de 1 mg/
para buscar una neoplasia relacionada con la dematopolimiositis. da y puede incrementarse a 1,5-2 mg/da, si se precisa. Aproximadamente
el 60% responden con completa remisin de los ataques. Un 20-30%
Pregunta 90.- R: 1 experimentan mejora significativa en el nmero y gravedad de los ataques.
En la polimiositis o dermatopolimiositis, como ocurre en todas las El resto permanece sin cambios, incluso con 2 mg o ms de colchicina al
enfermedades del tejido conectivo, se han descrito una serie de anti- da. En estos casos se puede probar la colchicina intravenosa.
cuerpos asociados a la enfermedad, que adems pueden tener rela-
cin con manifestaciones clnicas peculiares. Hasta en un 20% de los Pregunta 93.- R: 4
pacientes con miopata inflamatoria de cualquier tipo se observan an- En esta pregunta se realiza un repaso de algunas de las manifesta-
ticuerpos antinucleares y anticitoplasmticos. Los anticuerpos dirigidos ciones clnicas caractersticas en la amiloidosis. A continuacin repa-
contra la histidin-ARN de transferencia sintetasa, denominados anti-Jo- samos cada una de las respuestas:
1, representan el 75% de los anticuerpos anti-sintetasa y son clnica- Se puede producir infiltracin de material amiloide en las distintas
mente tiles, ya que el 80% de los pacientes con anticuerpos anti-Jo1 glndulas endocrinas, aunque no suelen producirse alteraciones
padece neumonitis intersticial. Algunos pacientes adems presentan funcionales.
fenmeno de Raynaud y artritis no erosiva. Algunos autores hablan de La afectacin del sistema nervioso puede adoptar distintas formas.
sndrome anti-sintetasa cuando el paciente presenta las caractersticas La neuropata perifrica es especialmente frecuente en las formas
comentadas: polimiositis, enfermedad pulmonar intersticial y anticuer- hereditarias (por lo que es evidente que la respuesta correcta es la
pos anti-sintetasa. 4), aunque tambin aparece en el 15% de las formas sistmicas. La
Otros anticuerpos que aparecen en algunas de las respuestas tambin afectacin del sistema nervioso autnomo es tpica, produciendo
se asocian a diferentes manifestaciones clnicas: los anti-SRP se asocian a hipotensin ortosttica, incapacidad para sudar, pupila de Adie o
un cuadro de polimiositis grave de debut agudo, con frecuente afecta- incontinencia esfinteriana. El tnel del carpo aparece especial-
cin cardiaca. Los anticuerpos anti-Mi2 se asocian a los pacientes con mente en la amiloidosis primaria (AL) y en la asociada a hemodilisis
dermatopolimiositis con buena respuesta al tratamiento con corticoides. crnica (beta-2-microglobulina).

M exico A rgentina CTO Medicina C/ Nez de Balboa, 115 28006 MADRID (Espaa) Tfno.: (91) 782 43 32 / Fax: (91) 782 43 27
Pg. 20 RM C hile U ruguay E-mail: secretaria@ctomedicina.com; iberocto@ctomedicina.com WEB: www.ctomedicina.com; www.iberocto.com
REUMATOLOGA
Preparacin Examen de Seleccin 05/06 1 Vuelta Seguimiento a distancia
El material amiloide puede depositarse en la membrana sinovial, y subluxaciones. Aunque se ha considerado el estrechamiento del
preferentemente en las articulaciones de hombros (hombro de espacio de la articulacin femorotibial como un signo indirecto pre-
jugador de rugby), rodillas, carpos, MCFs e Ifs. La mayora de los coz de la artrosis donde no hay cambios evidentes en las radiografas
pacientes con artropata amiloide suelen tener mieloma mltiple. (por ejemplo esclerosis o quistes subcondrales, osteofitos), la estrechez
Las formas heredofamiliares comprenden una serie de sndromes del espacio articular por s sola no indica con seguridad el estado del
neuropticos (de tipo transtirretina, prealbmina, AF), renales o cartlago articular. De igual modo, la presencia exclusiva de osteofitos,
cardiovasculares y la amiloidosis asociada a la fiebre mediterrnea si no hay otros signos radiolgicos de artrosis, puede deberse ms al
familiar. envejecimiento que a la artrosis.
Las manifestaciones cardiacas aparecen habitualmente en la amiloi- Evidentemente la respuesta a esta pregunta es la osteoporosis yux-
dosis primaria en forma de miopata infiltrativa de tipo congestivo. taarticular, hallazgo inverso al que suele aparecer en la artrosis, que es
la esclerosis del hueso subcondral. La osteoporosis es caracterstica
Pregunta 94.- R: 4 radiolgica de la artritis reumatoide.
Los datos ms relevantes en este caso clnico son la presencia de
artritis (que afecta a rodillas y tobillos), periostitis a nivel de las espinas Pregunta 97.- R: 5
tibiales y dedos en palillo de tambor. Esta trada es caracterstica de la La artropata neuroptica o artropata de Charcot es una forma
osteoartropata hipertrfica. Desde el punto de vista etiolgico la ma- agresiva de artropata degenerativa que se produce en las articulacio-
yora de los pacientes suelen ser adultos y presentan una enfermedad nes de miembros que por diferentes causas han perdido la sensibili-
pulmonar. Sin embargo, en esta pregunta debes recordar otras patolo- dad algsica, propioceptiva y los reflejos. Estos dficits permiten que
gas que se asocian a la osteoartropata hipertrfica: las articulaciones estn sometidas a continuos traumatismos, originan-
Forma primaria que puede ser idioptica (raro) o familiar (enfer- do dao progresivo de la articulacin y destruccin sea.
medad de Touraine-Solente-Gol), tambin denominada paqui-
La distribucin de la artropata neuroptica depende del proceso
dermoperiostitis por la tendencia a producir hipertrofia cutnea
neurolgico que la desencadena:
generalizada. Se hereda con patrn autosmico dominante y las
En la tabes dorsal se localiza a nivel de cadera, rodilla y tobillo y era
manifestaciones articulares aparecen aos despus de la afecta-
cin cutnea. causa habitual de artropata neuroptica hace muchos aos.
Formas secundarias asociadas a mltiples patologas como: En los nios, la causa ms habitual es el mielomeningocele, con
- Neoplasias pulmonares y pleurales; inflamacin crnica pul- clnica en extremidades inferiores.
monar. Fibrosis qustica, sarcoidosis y neumonitis intersticial. Actualmente la causa ms frecuente de artropata neuroptica es la
- Cardiopata congnita con cortocircuito derecha-izquierda y polineuropata diabtica, que produce una artropata en tarso y
endocarditis bacteriana. metatarsianos (respuesta correcta).
- Enfermedades inflamatorias intestinales y neoplasias gastrointes- En la siringomielia se afectarn las articulaciones de hombro (gle-
tinales. nohumeral), codo y carpo.
Otras situaciones asociadas son la amiloidosis, la lepra y las inyec-
Como puedes comprobar, cualquiera de las enfermedades, ex- ciones intraarticulares repetidas de forma excesiva. En estas depen-
cepto la que debemos elegir, la hemocromatosis, se pueden asociar a der de la localizacin de la enfermedad.
la aparicin de osteoartropata hipertrfica.
Pregunta 98.- R: 2
Pregunta 95.- R: 2 En esta pregunta debes recordar cules son las manifestaciones ca-
Las caractersticas clnicas, analticas (incluyendo los datos del lqui- ractersticas en la policondritis recidivante. Esta patologa inflamatoria
do sinovial) y radiolgicas son caractersticas de una artrosis de rodilla. crnica, de etiologa desconocida, se caracteriza por la afectacin del
Desde el punto de vista clnico, la paciente presenta dolor que se incre- cartlago, especialmente el auricular, que es el ms frecuentemente afec-
menta con el movimiento (dolor de caractersticas mecnicas). En oca- tado (por lo que la respuesta es la 2), y en ocasiones se acompaa de
siones puede haber cierta inflamacin por la aparicin de derrame manifestaciones viscerales a nivel renal, cardiacas o vasculitis.
articular. El estudio del lquido sinovial muestra caractersticas no infla- La manifestacin ms habitual y precoz suele ser la condritis auri-
matorias: aspecto claro, menos de 3.000 clulas/mm3 con predominio cular (85%). Suele ser bilateral y produce dolor, tumefaccin y erite-
de mononucleares y ausencia de microcristales. Desde el punto de vista ma en la zona cartilaginosa respetando los lbulos.
teraputico es aceptable utilizar en la paciente: La afectacin nasal se produce en el 55% y produce dolor, tume-
Comentarios TEST

Limitacin de la actividad de la articulacin afecta mientras los faccin y eritema en el puente nasal, acompaado de congestin
sntomas sean intensos. Sin embargo, es recomendable la realiza- nasal, epistaxis y rinorrea.
cin de ejercicios isomtricos y sin carga lo antes posible para La mitad de los pacientes desarrollan artritis y en el 35% de los
evitar la atrofia muscular. casos es la manifestacin inicial.
Los corticoides sistmicos no estn indicados en el tratamiento de Las manifestaciones oculares aparecen en el 50% y pueden ser
la artrosis (respuesta correcta 2). Sin embargo su utilizacin intra o mltiples, produciendo conjuntivitis, queratitis, escleritis y epiescleritis.
periarticular puede producir mejora sintomtica. La afectacin cardiaca es poco frecuente (5%), habitualmente
Analgsicos o AINEs. Estos ltimos pueden estar ms indicados cuan- en forma de insuficiencia artica. Tambin es poco habitual la
do hay inflamacin articular, como ocurre en este caso. aparicin de glomerulonefritis necrotizante, con o sin vasculitis aso-
Recientemente se ha autorizado la inyeccin intraarticular de ci- ciada.
do hialurnico para tratar a los pacientes con artrosis de rodilla
que no han mejorado con las medidas analgsicas habituales. Pregunta 99.- R: 3
Tambin se ha incorporado recientemente como posibilidad tera-
putica la administracin de condroitn sulfato oral. Esta molcula El proceso descrito en este caso clnico es compatible con el diag-
forma parte del cartlago condral y tericamente se incorporara al nstico de fibromialgia. Esta enfermedad aparece con ms frecuencia
mismo, reparndolo. en mujeres en torno a los 50 aos (en nuestro caso tiene 47), que
manifiestan dolor muscular generalizado, con sensacin de rigidez y
fatigabilidad.
Pregunta 96.- R: 4
La exploracin fsica no demuestra la presencia de signos inflama-
El diagnstico de la artrosis se basa en los hallazgos clnicos y radio- torios, observndose dolor selectivo a la palpacin (puntos gatillo).
lgicos. En las fases iniciales la radiologa puede ser normal, pero el Se han descrito 19 puntos y debe ser positiva la palpacin en al
estrechamiento del espacio articular se vuelve ms evidente a medida menos 11 de ellos.
que desaparece el cartlago articular. Otros signos radiolgicos carac- Desde el punto de vista clnico es habitual que estas pacientes pre-
tersticos son la esclerosis del hueso subcondral, los quistes subcon- senten asociadas una serie de manifestaciones psicosomticas como
drales y la presencia de osteofitos marginales. Tambin pueden verse alteraciones del sueo que es poco reparador, depresin o colon irrita-
alteraciones en el contorno articular debidas a la remodelacin sea, ble.

M exico A rgentina CTO Medicina C/ Nez de Balboa, 115 28006 MADRID (Espaa) Tfno.: (91) 782 43 32 / Fax: (91) 782 43 27
C hile U ruguay E-mail: secretaria@ctomedicina.com; iberocto@ctomedicina.com WEB: www.ctomedicina.com; www.iberocto.com
RM Pg. 21
REUMATOLOGA
Seguimiento a distancia Preparacin Examen de Seleccin 05/06 1 Vuelta
Al realizar el estudio analtico no encontramos alteraciones. No se
observa elevacin de la VSG, como en la polimialgia reumtica, ni
elevacin de las enzimas musculares, como en la polimiositis. La res-
puesta es la 3, CPK elevada. El resto de exploraciones que muestran
hallazgos patolgicos en la polimiositis, como el electromiograma o la
biopsia muscular, tambin son normales en la fibromialgia.

Pregunta 100.- R: 2
En esta pregunta se nos plantea el caso de un paciente con datos
analticos y clnicos sugestivos de polimialgia reumtica. Para el diag-
nstico de esta enfermedad nos basamos en la presencia de los
siguientes criterios:
Edad igual o superior a 50 aos.
Dolor moderado o intenso y rigidez matinal superior a 30 minutos
de ms de un mes de evolucin en al menos dos de las siguientes
tres reas musculares: cuello, cintura escapular y/o cintura pelviana.
Velocidad de sedimentacin superior a 40 mm/1 hora.
Resolucin rpida de los sntomas con dosis bajas de corticoides.

El tratamiento, como aparece dentro de los criterios, se fundamen-


ta en la utilizacin de corticoides. No existe ninguna pauta unnime-
mente aceptada, pero se considera que se debe administrar una dosis
diaria de 10 mg de prednisona. Posteriormente, la dosis de corticoides
se reduce gradualmente, aunque la mayora de los pacientes requie-
ren mantener el tratamiento al menos dos aos.
Ten en cuenta que en el caso planteado no existen datos de arteritis
de la temporal, que en ocasiones se asocia a la polimialgia reumtica.
Si el paciente hubiera tenido alteracin del pulso de la arteria tempo-
ral o alteraciones visuales, deberamos sospechar la presencia de la
vasculitis y utilizar en este caso corticoides, pero en dosis altas (1 mg/
kg/da).

Comentarios TEST

M exico A rgentina CTO Medicina C/ Nez de Balboa, 115 28006 MADRID (Espaa) Tfno.: (91) 782 43 32 / Fax: (91) 782 43 27
Pg. 22 RM C hile U ruguay E-mail: secretaria@ctomedicina.com; iberocto@ctomedicina.com WEB: www.ctomedicina.com; www.iberocto.com

You might also like